Sie sind auf Seite 1von 924

Separation of Molecules, Macromolecules and Particles

Providing chemical engineering undergraduate and graduate students with a basic understanding of how the separation
of a mixture of molecules, macromolecules or particles is achieved, this textbook is a comprehensive introduction to the
engineering science of separation.

Students learn how to apply their knowledge to determine the separation achieved in a given device or process.
Real-world examples are taken from biotechnology, chemical, food, petrochemical, pharmaceutical and pollution
control industries.
Worked examples, elementary separator designs and chapter-end problems give students a practical understanding of
separation.
The textbook systematically develops different separation processes by considering the forces causing the separation, and
how this separation is influenced by the patterns of bulk flow in the separation device. Readers will be able to take this
knowledge and apply it to their own future studies and research in separation and purification.

Kamalesh K. Sirkar is a Distinguished Professor of Chemical Engineering and the Foundation Professor of Membrane
Separations at New Jersey Institute of Technology (NJIT). His research areas are membranes and novel membrane based
processes.
The first comprehensive book that takes the fundamentals This book provides a unique and in depth coverage of
of separation on a molecular level as the starting point! The separation processes. It is an essential reference for the
benefit of this approach is that it gives you a thorough practicing engineer. Unlike more conventional textbooks
insight in the mechanisms of separation, regardless of that focus on rate and equilibrium based separations, Prof
which separation is considered. This makes it remarkably Sirkar focuses on how a given separation takes place and
easy to understand any separation process, and not only how this is used in practical separation devices. Thus the
the classical ones. This textbook finally brings the walls book is not limited by application e.g. chemical or petro-
down that divide separation processes in classical and chemical separations.
non-classical. As chemical engineering becomes increasingly multi-
Bart Van der Bruggen, University of Leuven, Belgium disciplinary, where the basic principles of separations are
applied to new frontier areas, the book will become an
This strong text organizes separation processes as batch vs essential guide for practitioners as well as students.
continuous and as staged vs differential. It sensibly The unique layout of the text book allows the
includes coupled separation and chemical reaction. Sup- instructor to tailor the content covered to a particular
ported by strong examples and problems, this non-conven- course. Undergraduate courses will benefit from the com-
tional organization reinforces the more conventional prehensive and systematic coverage of the basics of separ-
picture of unit operations. ation processes. Whether the focus of a graduate course is
Ed Cussler, University of Minnesota traditional chemical separations, bioseparations, or separ-
ation processes for production of renewable resources the
This book fills the need by providing a very comprehen- book is an essential text.
sive approach to separation phenomena for both trad- Ranil Wickramasinghe, University of Arkansas
itional and emerging fields. It is effectively organized and
presents separations in a unique manner. This book pre- This advanced textbook provides students and profession-
sents the principles of a wide spectrum of separations from als with a unique and thought-provoking approach to
classical distillation to modern field-induced methods in a learning separation principles and processes. Prof. Sirkar
unifying way. This is an excellent book for academic use has leveraged his years of experience as a separation sci-
and as a professional resource. entist and membrane separation specialist, to provide the
C. Stewart Slater, Rowan University reader with a clearly written textbook full of multiple
examples pulled from all applications of separations,
This book is an excellent resource for the topic of chem- including contemporary bioseparations. Compared to
ical separations. The text starts by using examples to clarify other separations textbooks, Prof. Sirkars textbook is hol-
concepts. Then throughout the text, examples from many istically different in its approach to teaching separations,
different technology areas and separation approaches are yet provides the reader with a rich learning experience.
given. The book is framed around various fundamental Chemical engineering students and practicing profession-
approaches to chemical separations. This allows one to als will find much to learn by reading this textbook.
use this knowledge for both current and future needs. Daniel Lepek, The Cooper Union
Richard D. Noble, University of Colorado
Separation of Molecules,
Macromolecules and Particles
Principles, Phenomena and Processes

Kamalesh K. Sirkar
New Jersey Institute of Technology
University Printing House, Cambridge CB2 8BS, United Kingdom

Published in the United States of America by Cambridge University Press, New York
Cambridge University Press is part of the University of Cambridge.
It furthers the Universitys mission by disseminating knowledge in the pursuit of
education, learning and research at the highest international levels of excellence.

www.cambridge.org
Information on this title: www.cambridge.org/9780521895736
K. Sirkar 2014
This publication is in copyright. Subject to statutory exception
and to the provisions of relevant collective licensing agreements,
no reproduction of any part may take place without the written
permission of Cambridge University Press.
First published 2014
Printed in the United Kingdom by TJ International Ltd. Padstow Cornwall
A catalog record for this publication is available from the British Library
Library of Congress Cataloging-in-Publication Data
Sirkar, Kamalesh K., 1942- author.
Separation of molecules, macromolecules and particles : principles, phenomena and
processes / Kamalesh Sirkar, New Jersey Institute of Technology.
pages cm. (Cambridge series in chemical engineering)
isbn 978-0-521-89573-6 (Hardback)
1. Separation (Technology)Textbooks. 2. MoleculesTextbooks. I. Title.
TP156.S45S57 2013
5410 .22dc23 2012037018

ISBN 978-0-521-89573-6 Hardback


Additional resources for this publication at www.cambridge.org/sirkar
Cambridge University Press has no responsibility for the persistence or accuracy of
URLs for external or third-party internet websites referred to in this publication,
and does not guarantee that any content on such websites is, or will remain,
accurate or appropriate.
Contents

Preface page xi
List of notation xiii

Introduction to the book 1


Introduction to chapters 2
Linked footprints of a separation process/technique 3
Classification of separation processes 6
Additional comments on using the book 7
Textbooks, handbooks and major references on separation processes 7

1 Description of separation in a closed system 19


1.1 Binary separation between two regions in a closed vessel 19
1.2 Multicomponent separation between two regions in a closed vessel 22
1.3 Definitions of composition for a binary system in a closed vessel 24
1.4 Indices of separation for binary systems 25
1.5 Comparison of indices of separation for a closed system 29
1.6 Indices for separation of multicomponent systems between two regions 33
1.7 Some specialized nomenclature 35
Problems 35

2 Description of separation in open separators 39


2.1 Preliminary quantitative considerations 39
2.2 Binary separation in a single-entry separator with or without recycle 42
2.2.1 Examples of separation in single-entry separators 45
2.2.2 Single-entry separator with a product recycle 48
2.2.3 Separative power and value function 49
2.3 Binary separation in a double-entry separator 50
2.3.1 Examples of separation in double-entry separators 51
2.3.2 Separation in a double-entry separator with recycle/reflux 53
2.4 Multicomponent systems 53
2.4.1 Size-distributed particle population 54
2.4.2 Continuous chemical mixtures 62
2.4.3 Multicomponent chemical mixtures 64
2.5 Separation in an output stream with time-varying concentration 66
Problems 70

3 Physicochemical basis for separation 76


3.1 Displacements, driving forces, velocities and fluxes 76
3.1.1 Nature of displacements 76
3.1.2 Forces on particles and molecules 77
3.1.3 Particle velocity, molecular migration velocity and chemical species flux 88
3.1.4 Integrated flux expressions for molecular diffusion and convection: single-phase systems 102
vi Contents

3.1.5 Flux expressions in multicomponent systems 112


3.1.6 Additional topics 117
3.2 Separation development and multicomponent separation capability 118
3.2.1 Separation development in a closed system 119
3.2.2 Multicomponent separation capability 124
3.2.3 Particulate systems 127
3.3 Criteria for equilibrium separation in a closed separator 128
3.3.1 Phase equilibrium with equal pressure in all phases 129
3.3.2 Phase equilibrium where different phases have different pressures 130
3.3.3 Single-phase equilibrium in an external force field 132
3.3.4 Equilibrium between phases with electrical charges 132
3.3.5 Equilibrium between bulk and interfacial phases 133
3.3.6 Curved interfaces 136
3.3.7 Solute distribution between phases at equilibrium: some examples 137
3.3.8 Particle distribution between two immiscible phases 159
3.4 Interphase transport: flux expressions 159
3.4.1 Interphase transport in two-phase systems 160
3.4.2 Interphase transport: membranes 170
3.4.3 Interphase transport in two-phase systems with phase barrier membranes 189
Problems 192
Appendix: Diffusion coefficients in different systems 202

4 Separation in a closed vessel 205


4.1 Equilibrium separation between two phases or two regions in a closed vessel 205
4.1.1 Gasliquid systems 206
4.1.2 Vaporliquid systems 208
4.1.3 Liquidliquid systems 217
4.1.4 Liquidsolid systems 222
4.1.5 Interfacial adsorption systems 223
4.1.6 Liquidion exchanger systems 228
4.1.7 Supercritical fluidbulk solid/liquid phase 231
4.1.8 Bulk fluid phase mesophase systems 231
4.1.9 Partitioning between a bulk fluid phase and an individual molecule/macromolecule
or a collection of molecules for noncovalent solute binding 234
4.1.10 Gassolid particleliquid system in mineral flotation 245
4.2 Equilibrium separation in a single phase in an external force field 245
4.2.1 Centrifugal force field 246
4.2.2 Electrical force field 253
4.2.3 Gravitational force field 256
4.2.4 Particle separation with acoustic forces 260
4.2.5 Externally imposed temperature gradient: thermal diffusion 261
4.3 Equilibrium separation between two regions in a closed vessel separated by a membrane 262
4.3.1 Separation by dialysis using neutral membranes 262
4.3.2 Separation between two counterions in two solutions separated by an ion exchange
membrane: Donnan dialysis 264
4.3.3 Separation of a gas mixture by gas permeation 266
4.3.4 Separation of a pressurized liquid solution through a membrane 272
Problems 273

5 Effect of chemical reactions on separation 280


5.1 Extent of separation in a closed vessel with a chemical reaction 280
5.2 Change in separation equilibria due to chemical reactions 281
5.2.1 Gasliquid and vaporliquid equilibria 281
5.2.2 Liquidliquid equilibrium 289
Contents vii

5.2.3 Stationarymobile phase equilibria 299


5.2.4 Crystallization and precipitation equilibrium 303
5.2.5 Surface adsorption equilibrium 306
5.2.6 Complexation in Donnan dialysis 308
5.2.7 Enzymatic separation of isomers 308
5.3 Rate-controlled equilibrium separation processes: role of chemical reactions 309
5.3.1 Absorption of a gas in a reactive liquid 309
5.3.2 Solvent extraction of a species with chemical reaction 315
5.4 Rate-governed membrane processes: role of chemical reactions 318
5.4.1 Reverse osmosis: solute ionization 318
5.4.2 Ultrafiltration: complexation 319
5.4.3 Dialysis: reaction in dialysate 323
5.4.4 Chemical reactions in liquid membrane permeationseparation 324
5.4.5 Separation through solid nonporous membrane 337
Problems 338

6 Open separators: bulk flow parallel to force and continuous stirred tank separators 346
6.1 Sources and nature of bulk flow 348
6.1.1 Hydrostatic pressure induced bulk flow 348
6.1.2 Gravity induced bulk flow 350
6.1.3 Free convection 350
6.1.4 Bulk motion due to capillarity 352
6.1.5 Electroosmotic flow 353
6.1.6 Centrifugal force driven flow 354
6.1.7 Surface tension gradient based flow 354
6.1.8 Drag flow 355
6.1.9 Feed introduction mode vs. bulk flow 356
6.2 Equations of change 358
6.2.1 Equations of change for species concentration in a mixture 358
6.2.2 Equation of motion of a particle in a fluid: trajectory equation 368
6.2.3 General equation of change for a particle population 368
6.3 Bulk flow parallel to force direction 372
6.3.1 External forces 373
6.3.2 Chemical potential gradient driven phase-equilibrium systems 390
6.3.3 Filtration and membrane separation processes 412
6.4 Continuous stirred tank separators 445
6.4.1 Well-mixed separators CSTSs and batch separators 445
6.4.2 Well-mixed separators membrane based devices 469
Problems 475

7 Separation in bulk flow of feed-containing phase perpendicular to the direction of the force 485
7.1 Chemical potential gradient based force in phase equilibrium: fixed-bed processes 487
7.1.1 Fixed-bed adsorption/desorption processes 487
7.1.2 Pressure-swing adsorption process for gas separation 511
7.1.3 Potential-swing adsorption 519
7.1.4 Parametric pumping 520
7.1.5 Chromatographic processes 527
7.1.6 Expanded bed adsorption (EBA) from a broth/lysate aided by gravitational force 550
7.1.7 Counteracting chromatographic electrophoresis and electrochromatography 551
7.2 Crossflow membrane separations, crossflow filtration and granular filtration 555
7.2.1 Crossflow membrane separations, crossflow filtration 555
7.2.2 Granular filtration of hydrosols (and aerosols) 586
7.3 External force field based separation: bulk flow perpendicular to force 596
7.3.1 Electrical force field 596
viii Contents

7.3.2 Centrifugal force field 618


7.3.3 Gravitational force field 634
7.3.4 Field-flow fractionation for colloids, macromolecules and particles 640
7.3.5 Magnetic force field 648
7.3.6 Radiation pressure optical force 654
Problems 655

8 Bulk flow of two phases/regions perpendicular to the direction(s) of the force(s) 670
8.1 Countercurrent bulk flow of two phases or regions perpendicular to the direction(s) of the force(s)
driving species 670
8.1.1 Development of separation in countercurrent flow systems 670
8.1.2 Gas (vapor) absorption/stripping 683
8.1.3 Distillation 709
8.1.4 Countercurrent solvent extraction 736
8.1.5 Countercurrent melt crystallization in a column 751
8.1.6 Countercurrent adsorption and simulated moving bed system 754
8.1.7 Membrane processes of dialysis and electrodialysis 761
8.1.8 Countercurrent liquid membrane separation 767
8.1.9 Countercurrent gas permeation 771
8.1.10 Countercurrent gas centrifuge 775
8.1.11 Thermal diffusion and mass diffusion 781
8.2 Cocurrent bulk motion of two phases or regions perpendicular to the direction(s) of the force(s)
driving species/particles 782
8.2.1 Cocurrent two-phase flow devices general considerations 783
8.2.2 Chromatographic separations in cocurrent two-phase flow devices 785
8.2.3 Particle separation in cocurrent gasliquid flowVenturi scrubber 788
8.2.4 Cocurrent membrane separators 789
8.3 Crossflow of two bulk phases moving perpendicular to the direction(s) of the driving force(s) 794
8.3.1 Continuous chromatographic separation 794
8.3.2 Crossflow plate in a distillation column 799
Problems 804

9 Cascades 812
9.1 Types of cascades 812
9.1.1 Countercurrent cascades, ideal cascade 812
9.1.2 Other cascade configurations 818
9.2 Cascades for multicomponent mixture separation via distillation 822
9.3 Cascades for multicomponent mixture separation involving other separation processes 824
Problems 824

10 Energy required for separation 827


10.1 Minimum energy required for separation 827
10.1.1 Evaporation of water from sea water 828
10.1.2 Recovery of water by reverse osmosis 828
10.1.3 Net work consumption 830
10.1.4 Minimum energy required for membrane gas permeation, distillation, extraction,
and other separation processes 831
10.2 Reducing energy required for separation 836
10.2.1 Evaporation of water for desalination 836
10.2.2 Distillation 839
10.2.3 Free energy of mixing 843
10.2.4 Dilute solutions 844
Problems 845
Contents ix

11 Common separation sequences 847


11.1 Bioseparations 847
11.2 Separation sequences for water treatment 851
11.2.1 Sea-water and brackish-water desalination 851
11.2.2 Ultrapure water production 852
11.2.3 Pharmaceutical grade water 853
11.3 Chemical and petrochemical industries 853
11.4 Hydrometallurgical processes 855

Postface 856
Appendix A Units, various constants and equivalent values of various quantities in different units 857
Appendix B Constants 858
Appendix C Various quantities expressed in different units 859
References 861
Index 877
Preface

This is an introductory textbook for studying separation. undergraduate levels of knowledge of ordinary differential
Primarily, this book covers the separation of mixtures of equations and elementary partial differential equations.
molecules; in addition, it provides a significant treatment Specific aspects of a given separation process are stud-
of particle separation methods. Separation of macromol- ied in the chapter devoted to those aspects for all separ-
ecules has also received some attention. The treatment and ation processes. To study a particular separation process in
coverage of topics are suitable for chemical engineering great detail, one therefore has to go to different chapters.
students at undergraduate and graduate levels. There is The footprints of a given separation process are provided at
enough material here to cover a variety of introductory the beginning of the book (Tables 17); there are quite a
courses on separation processes at different levels. few tables to cover a variety of separation processes. The
This book is focused on developing a basic under- list of processes is large; however, it is far from being all
standing of how separation takes place, and of how the inclusive. The introductory chapter, which provides add-
resulting separation phenomenon is utilized in a separ- itional details about various chapters, as well as about the
ation device. The role of various forces driving molecules book, is preceded by a notation section. All references
or particles from a feed mixture into separate phases/ appear at the end of the book.
fractions/regions is basic to such an approach to studying The description of the extent of separation achieved in
separation. The separation achieved is then amplified in an a closed vessel for a mixture of molecules is treated in
open separator via different patterns of bulk-phase veloci- Chapter 1. Chapter 2 illustrates how to describe the separ-
ties vis--vis the direction(s) of the force(s). The forces are ation of molecules in open separators under steady and
generated by chemical potential gradient, electrical field, unsteady state operation; a description of separation for a
rotational motion, gravity, magnetic field, etc. The resulting size-distributed system of particles is also included. Chapter 3
separation is studied under three broad categories of sep- introduces various forces developing species-specific
aration processes. velocities, fluxes and mass-transfer coefficients, and illus-
Separation processes driven by a negative chemical trates how the spatial variation of the potential of the force
potential gradient are generally multiphase systems and field can develop multicomponent separation ability. The
are treated under the broad category of phase equilibrium criteria for chemical equilibrium are then specified for
driven processes. External force driven processes populate different types of multiphase separation systems, followed
the second category, and include those operating under an by an illustration of integrated flux expressions for two-
electrical field, rotational motion, magnetic field or gravity; phase and membrane based systems.
thermal diffusion processes are also briefly included here. Chapter 4 develops the extent of separation achieved
The third category of membrane based processes studied is in a closed vessel to a variety of individual processes under
driven generally by a negative chemical potential gradient; each of the three broad categories of separation processes.
however, electrical force is also relevant for some processes. Chapter 5 demonstrates how separation can be consider-
The treatment of any external force driven processes will ably enhanced by chemical reactions in phase equilibrium
cover both separation of molecules and particle separations. based and membrane based processes under both equilib-
These physical separation methods are often reinforced rium- and rate-controlled conditions. For open separators
by chemical reactions, which are usually reversible. An having bulk flow in and out, including continuous stirred
elementary treatment of the role of chemical reactions tank separators (CSTSs), Chapter 6 provides first the equa-
in enhancing separation across a broad spectrum of tions of change for molecular species concentration in
phase equilibrium driven processes and membrane based single-phase and two-phase systems, the trajectory equa-
processes has been included. The level of treatment in tion for a particle in a fluid and the general equation of
this book assumes familiarity with elementary principles change for a particle population. Chapter 6 then treats
of chemical engineering thermodynamics and traditional individual separation processes under each of the three
xii Preface

broad categories of separation processes when the bulk references. A website will provide guidance for computer
flow is parallel to the direction of the force and in CSTS simulations for a few selected problems.
mode. The introductory chapter provides references to art-
Chapter 7 follows this latter approach of treating indi- icles and books which influenced the development of vari-
vidual separation processes under each of the three broad ous aspects of this book. I have benefitted considerably
categories of separation processes when the bulk flow of from the comments on selected chapters of the book by
feed-containing phase is perpendicular to the direction of reviewers, anonymous or otherwise. Comments by Profes-
the force. Chapter 8 follows the same approach when the sors C. Stewart Slater, of Rowan University, Steven Cramer,
bulk flows of two phases/regions in the separator are per- of Rensselaer Polytechnic Institute, and Ranil Wickrama-
pendicular to the direction(s) of the force(s). Chapter 9 singhe, of Colorado State University (now at University of
briefly elaborates on cascades, which were already intro- Arkansas), were particularly useful.
duced in the countercurrent multistaged flow systems Many doctoral students and postdoctoral fellows were
of Chapter 8. Chapter 10 introduces the energy required of invaluable help during the long gestation period of this
for a number of separation processes. Chapter 11 illustrates book, either in formulating solutions of the problems or in
a few common separation sequences in a number of developing illustrative drawings. I want to mention in par-
common industries involved in bioseparations, water treat- ticular Amit Sengupta, Theoharris Papadopoulos, Xiao-Ping
ment, chemical and petrochemical separations and hydro- Dai, Meredith Feins, Dimitrios Zarkadas, Quixi Fan, Praveen
metallurgy. Conversion factors between various systems of Kosaraju, Fei He, Atsawin Thongsukmak, Sagar Roy,
units are provided in an Appendix. Dhananjay Singh, John Tang and John Chau. The first two
Virtually all separation processes taught to chemical students helped me when we were at Stevens Institute of
engineering students in a variety of courses have been Technology. Sarah Matthews of Cambridge University Press
covered via the approach illustrated in Chapters 3, 4, 6, 7 patiently provided manuscript preparation guidelines and
and 8; in addition, many particle separation methods have encouraging comments during an ever-shifting timetable.
been treated. The structural similarity in the separation Irene Pizzie did an extraordinary job as the copy editor.
method between apparently unrelated separation pro- Brenda Arthur of New Jersey Institute of Technology tire-
cesses becomes quite clear. A few basic principles equip lessly typed the draft of the whole manuscript over a consid-
the students with the capability to understand a wide erable length of time, while carrying out many other duties.
variety of separation processes and techniques, including I must also mention at the end my wife, Keka, without
emerging ones. To aid the student, there are 118 worked whose patience, help and understanding this book would
examples, 300 problems, 340 figures, 100 tables and 1011 never have been finished.
Notation

Equation numbers identify where the symbols have been introduced or defined.

The following styles have been adopted. asp pore surface area per unit volume of
the porous medium of porosity
Bold vector quantity
av, avc surface area of a particle per unit
particle volume, value of av in a cake
Overlines (6.3.135j)
quantity averaged over time or a specific a mean electrolyte activity (3.3.119d)
coordinate direction, multicomponent system, A; A1, A2, A3 amplitude (7.3.18), pure-water
Laplace transformed dependent variable permeability constant in reverse osmosis
= averaged quantity and diffusive ultrafiltration; constants
quantity in a mixture, per unit mass of bulk phase in (7.1.90b), three surface areas in
^ control volume of Figure 7.2.6(b)
A(r) cross-sectional area of a cone in
Underlines centrifugal elutriation
_ hypothetical binary system quantity (2.4.23), (2.4.24) Ac cross-sectional area of duct
~, vector quantity, tensor quantity Ahex; Aij; Am; heat exchanger surface area; constant
Atm ; Ai T; Aoi in equations for activity coefficients
(4.1.34d); surface area of membrane;
Brackets total membrane surface area; modified
vtj average value of vtj over surface area Sj equilibrium constant (7.1.66); constant
(7.1.72b)
A1 constant in crystal growth rate
a ellipsoid semiaxis dimension; see also equation (6.4.27)
(3.1.10a); constant in relations (3.1.49), Ap projected area of a particle (3.1.64);
(4.3.7) and (4.3.43a); interfacial area per transport coefficient in solution-
unit volume, defined by (7.2.191), (7.3.25) diffusion-imperfection model (3.4.60a)
a1; a2, a3, constant in (3.3.105), (4.1.42b), (4.3.29) A
p x cumulative crystal surface area
and (5.2.147); constants in (7.2.73), distribution fraction (6.4.17)
(7.3.50) and (7.3.139) AT total particle surface area per unit
a1 constant in (7.2.198a) volume of total mixture (Example
aA stoichiometric coefficient for species A 2.4.2); total crystal surface area per
aH Hamaker constant (3.1.16) unit liquid volume (6.4.16)
ai; ai(T); aij, ail; activity of species i; equilibrium Am, Am, Am three forms of amino acid
0
^a i ; am ; ams constant (7.1.63); atom fraction of ith
isotope of an element in region j b ellipsoid semiaxis dimension;
(1.3.6), value of ai in region j and liquid proportionality constant in osmotic
phase, respectively; amplitude (7.1.72b); pressure relation (3.4.61b); half of
constant in (4.3.43c); membrane surface channel gap; width of region of gas flow
per unit channel length (7.2.70) completely cleaned up by a fiber
ap surface area of a particle (6.3.42a); constant in crystal growth rate
as1, as2; asw activity of solvent s in regions 1 and 2, expression (6.4.35); parameter (7.2.18);
respectively; activity of salt in water liquid envelope radius (7.2.208)
xiv Notation

b1, b2, b3 constants in relations (3.1.143f) Ci; C i ; C ti ; C i ; molar concentration of species i; an


b1 constant in (3.3.105), (5.2.147) C ig ; C ii1 , C ii2 average of the molar concentration of
bf; bi; bif; bi; bi1 membrane feed channel height (Figure species i in the feed and the permeate
7.2.3(a)); constant in equilibrium (5.4.74), (6.3.158b); total molar
relation (3.3.81); constant in (7.1.73), concentration of species i in the
constant in Freundlich isotherm porous medium per unit volume
(3.3.112c); constant in Langmuir (3.1.118b); nondimensional species i
isotherm (3.3.112a); constant in concentration (5.3.35j), hypothetical
equation (4.1.42a) gas-phase species i concentration
0 0
bik , bk proportionality constant in (5.2.154), (8.1.47); initial bed concentration of
equilibrium constant in (5.2.155) solute i in phases 1 and 2
0
bm, bm constants in (4.3.43a,c) C i10 , C i20 values of C i1 , C i2 at z = 0
B magnetic induction vector (3.1.19) C oi liquid-phase concentration of pure
B; Bo; Boi ;Bp constant, density function of the birth solute i at spreading pressure and
rate of new particles (6.2.50g), value of temperature T providing the same
B as rp ! 0; value of Bo for crystal surface phase concentration of i as the
growth rate gri; duct perimeter mixture
(7.3.41) C igel molar species i concentration in gel
Bi Biot number (3.4.35) Cij molar concentration of species i in
B1p, B2p second virial coefficients for region j or location j or stream j; j = b,
interaction between polymers 1 and 2 bulk; j = E, extract; j = f, feed region;
(4.1.34p) j = g, gas phase; j = k, kth phase; j = ,
0
liquid; j = , adhering liquid phase on
c; cC velocity of light; gap between plates at crystal; j = m, membrane; j = o, organic;
entrance (Figure 7.3.10), j = p, permeate, product; j = r, raffinate;
stoichiometric coefficient for species C j = R, ion exchange resin phase,
C clearance of a solute (8.1.390) raffinate; j = s, solution, solid phase or
C1,C2; C2(x) integration constants (3.3.10b); molar pore surface; j = w, water, mol/liter
concentration of species 2 at C tij ; C ijn total molar concentration of species i
location x in region j including complexed or
C2f molar concentration of species 2 in dissociated forms; value of Cij on nth
initial mixture, mol/liter plate/stage
C(2) defined in (6.3.75) C 0ij , C ij , C ij species i concentrations in phase j at
C 0A , C A molar concentration of A at z = 0, locations 0 (or initial concentration),
(3.1.124) and
CAb, CAi molar concentration of A in phase bulk C ib , C ie , C ii molar concentrations of species i in
and phase interface and in liquid at bulk solution, at the end of
gasliquid interface (3.4.1b) concentration process and at the
0
CAwb, CAwi; C Awb molar concentration of A in bulk water beginning of concentration process
and in water at phase interface (6.3.173)
0 0
(3.4.45e); critical value of CAwb for C ig , C igf molar species i concentration per unit
maximum enhancement (5.3.53) gas phase volume in a pore
CBob, CBoi molar concentration of species B in ^ ik ; C it
C ik , C ik , C intrinsic phase average, phase average
organic-phase bulk and interface, and deviation in Cik for species i
respectively concentration in phase k (6.2.24a,b),
0
Cc; C Cb ;CD slip (Cunningham) correction factor (6.2.28); defined by (7.1.94)
p
(3.1.215); critical value of bulk C dim , C H
im ; C im species i concentration in membrane:
concentration of C for maximum for Henrys law and Langmuir species,
enhancement (5.3.29b); particle drag respectively, in dual sorption model
coefficient (3.1.64), (6.3.4) (3.3.81); membrane pore liquid
CFC molar concentration of fixed charges in Cimi,Cimo,Ciob, molar concentrations of species i at
ion exchange resin Ciwi, Ciwb various locations in Figure 3.4.11
0 0
C H1 , C H2 dual mode sorption constants for CpR, Cpw molar protein concentrations in resin
species 1 and 2 (3.3.82a,b) phase and aqueous solution
Notation xv

Csf, Csm, Csp molar solvent concentration in feed, Dim; Dimo; Dip diffusion coefficient of i in membrane
membrane and permeate, (6.3.149); value of Dim for Cim = 0
respectively (3.4.67b), Di in a pore (3.4.89c),
Ct; Ctj; Cvp total molar concentration; Ct in a (6.3.145a); effective Di in the pores of a
mixture in region j; volume of particles particle
per unit fluid volume (7.2.176) Dik , Dim multicomponent diffusion coefficient
of species pair (i, k) and (i, m) in
d; dh; di; d m diameter of tube/pipe/vessel; MaxwellStefan approach
hydraulic diameter (Table 3.1.8); DiK Knudsen diffusion coefficient for
effective diameter of a molecule of gas species i (3.1.115c)
species i (3.3.90a); logarithmic mean DiM effective binary diffusivity of species i
diameter (8.1.417) in a mixture (3.1.184), (3.1.185)
di; dgr; dimp force-type term (3.1.178), (3.1.181); Dis ; D0is ; DN
is binary diffusion coefficient for solute i/
grain diameter; diameter of an solvent s; value of Dis at infinite dilution;
impeller NernstPlanck binary diffusion
dion; dp; dw; dp1, mean diameter of a molecular ion; coefficient for species i/solvent s
dp2 mean diameter of a particle (6.1.4b); D12 binary diffusion coefficient for species
wire diameter (3.1.23); diameters of 1 and 2
particles 1 and 2 De; (De)mv density function of particles which
d32 Sauter mean diameter of a drop or disappear (die) (6.2.50h); see
particle (6.4.88), (6.4.89) (7.2.170a) and (7.2.172)
D diffusion coefficient of species in Df; DF decontamination factor (2.2.1c);
countertransport through liquid dilution factor (6.4.106), (7.2.91c)
membrane
DA, DC; DB diffusion coefficient of species A and C, e charge of an electron, 1.60210  1019
respectively; dialysance in coulomb
hemodialysis (8.1.389) ei constant in adsorption isotherm for
Deff effective diffusion coefficient (5.4.64a) solute i (3.3.113d)
Dgr crystal growth diffusivity (6.4.45) enb,enM,enMN, molecular energy in the bulk, due to
Dp; Dp() diffusion coefficient for particle enMP,enp intramolecular interactions, due to
(3.1.68), (6.2.52); shear-induced intermolecular interactions, due to
particle diffusivity (3.1.74), (7.2.126), interaction between molecules and
(7.2.131a) pores and total energy for molecules in
Dr desalination ratio (1.4.25), (2.2.1a) the pore (3.3.89d)
Di,eff; Di,eff,r, effective diffusion coefficient of i in E; E; E c ; E Di ; E e electrical force field; its magnitude
Di,eff,z; Di,eff,1 liquid (6.2.18), (6.3.16b); value of Di,eff (3.1.8), (6.1.22), (6.3.8f), extraction
in r-and z-directions; value of Di,eff in factor (8.1.281), stage efficiency
phase/region 1 (6.4.72); electrical field strength Ec
Di; eff; k ; D effective diffusion coefficient of i in (7.3.32a); activation energy for
i; eff; k
phase k (6.2.33); dispersion tensor diffusion of species i in polymer
(6.2.31) (4.3.46b); extraction factor for
Dij; Dil; Dis diffusion coefficient of species i in extraction section (8.1.303)
region j; j = l, liquid; j = s, solvent Ei; Eo; EoG; Es; Ey; enrichment of species i by
DTA , DTB ; DTis thermal diffusion coefficient for EBRS, EGrS pervaporation (6.3.193b), particle
species A and B (3.1.44); for species i in collection efficiency (7.2.200b); overall
solution column efficiency (8.1.195); point
DAB; DBR binary diffusion coefficient for mixture efficiency (8.3.13); extraction factor for
of gases A and B; diffusion coefficient the scrubbing section; electrical field
of particles due to Brownian motion strength in y-direction (7.3.48); particle
(7.2.216) collection efficiency (7.2.219), (7.2.214)
DiD, DiH diffusion coefficients in dual sorption EIS; EIS inertial impaction based single fiber
dual transport model (3.4.78) capture efficiency (6.3.42a); particle
Die effective diffusion coefficient of i in a capture efficiency by interception
porous medium (3.1.112d) (7.2.224)
xvi Notation

EME, EMR; EMV; Murphree extract stage efficiency, F; F^ , F^ p ;F force on a particle; value of F per unit
EN Murphree raffinate stage efficiency particle mass; degrees of freedom
(6.4.70), (6.4.71); Murphree vapor (4.1.22)
efficiency (8.1.198); Newton particle F(rp) probability distribution function
separation efficiency (2.4.14a) corresponding to f(rp) (2.4.1c), crystal
ET; E 1T ; E T i total efficiency in solidfluid size distribution function (6.4.11)
separation (2.4.4a), overall filter Fi; F ext
i electrostatic force on 1 gmol of a
efficiency (6.3.45), (7.2.201); reduced charged species in solution (6.3.8a);
efficiency of Kelsall (2.4.16a); ET for ith magnitude of external force on 1 gmol
solidfluid separator (2.4.17c,d) of species i
Facrx acoustic radiation force in x-direction
f friction factor (6.1.3a), fractional (3.1.48)
consumption of chemical adsorbent Frad radiation pressure force (3.1.47),
(5.2.19d) (7.3.267)
f2 fraction of the solute in ionized form F BR force on very small particle due to
(i = 2) in RO (5.4.4) random Brownian motion (3.1.43)
f(r); f(rp); f() molar density function in a F ELK
i electrokinetic force on particle in
continuous/semi-continuous mixture double layer (3.1.17)
with characteristic property r; particle F ELS
i electrostatic force on particle i,
size probability density function Coulombs law (3.1.15)
(2.4.1a), pore size distribution function F Lret
i London attraction force (3.1.16)
in a membrane; defined by (7.2.222a,c) Fmk force on species k in mass flux ji, force
ff(rp), f1(rp), f2(rp) value of f(rp) for feed stream, overflow relation (3.1.202)
and underflow based on particle weight F ext
net net external force; for gravity
fraction in a given size range (2.4.1b) see (3.1.5)
fA, fi; f 0i fugacity of species A, species i; F drag iner ext
p ; F p ; F pz frictional force on a particle; inertial
standard state fugacity of species i force on a particle (6.2.45); external
fg(gr) probability density function of crystal force on a particle in z-direction
growth rate (6.4.41a) F ti , F ext
ti total force and total external force on 1
fil fugacity of pure species i in liquid gmol of species i (3.1.50)
phase F ext ext
tp ; F tpx , total external force on a particle
ext
fm, f0 m quantities characteristic of a F ext
tpy ; F tpz (3.1.59), (6.2.45); components of F ext
tp in
membrane polymer (4.3.46a,d) x-, y-and z-directions
f 0ij , f 0ig , f 0il standard state fugacity of species i in F TA force on 1 gmol of species A due to a
region j; j = g, gas phase; j = , liquid temperature gradient (3.1.44)
phase F Faradays constant, 96 485 coulomb/
f di , f dp frictional coefficient for species i and gm-equivalent
spherical particle
^f , ^f , ^f ; ^f value of fugacity of i in a mixture in gas g; gc; gm acceleration due to gravity; conversion
ig ij il ijpl
phase, phase j and liquid phase, respec- factor; a quantity characteristic of a
tively; value of ^f ij for a planar surface membrane polymer (4.3.46a)
f dio value of f di for a sphere of equivalent g ext ; g ext ext
x , gy , external body force per unit mass; its
volume (3.1.91e) g ext
z components in x-, y-and
f dim , f dsm frictional coefficient for solute i and z-directions
solvent s in a membrane gri intrinsic growth rate of ith crystal
f f M, f l M , value of f(r), where r = M, molecular (6.4.41a)
f v M ; f, f weight, for the feed mixture, liquid G; Gg superficial mass average velocity based
fraction and the vapor fraction, on empty flow cross section, G for gas
respectively; defined by (7.1.59a), phase
(7.2.187), respectively G; Ga, Gb; Gi; Go growth rate of crystal (6.4.25), (6.4.3b);
fM; fQm, fyo fraction of the total metal ion value of G under condition a, condition
concentration in the aqueous phase b; factor representing contribution of
present as Mn (5.2.97); probability species i properties to Qim (4.3.56a);
density functions (7.3.79), (7.3.80) constant
Notation xvii

Gc ; GDr ; G D convective hindrance factor (3.1.113); j unit vector in positive y-direction


drag factor reducing solute diffusion by ji; jix, jiy, jiz mass flux vector of species i, Mi Ji
hindrance (3.1.112e); function of (3.1.98), Tables 3.1.3A, 3.1.3B, (6.2.5n);
particle volume fraction in hindered components of ji in x-, y- and z-directions
settling (4.2.61) JD factor defined by (3.1.143g)
Gi ;Gij partial molar Gibbs free energy of Ji, J i ; J i k ; J 1 , J T1 molar flux vector of species i (3.1.98),
species i, ratio of solute i velocity to the (3.1.99), Tables 3.1.3A, 3.1.3B; value of
averaged pore solvent velocity, Ji in region k; diffusive molar flux
convective hindrance factor (=Gc ) vector of species 1 (4.2.63);
(3.1.113), (3.4.89b); value of Gi in temperature gradient driven molar flux
region j vector of species 1 (4.2.62)
Gr(rp) grade efficiency function (2.4.4b) Jiz, Jsz; J iy z-components of flux vectors Ji and Js;
Gtj; Gcrit total Gibbs free energy of all molecules y-component of flux vector J i
in region j (3.3.1), (4.2.23); defined by J iz , J jz , J sz , J Az z-components of flux vectors J i , J j , J s
(7.1.58e) and J A
Gr; Gr(r, v); Gz Grashof number (3.1.143e); function J t
Ay total molar flux of species A in y-
defined by (7.2.174); Graetz number direction (5.4.51)
(8.1.276) Jvz volume flux through membrane in
z-direction (3.4.60c), (6.3.155a)
h; ho membrane flow channel height,
distance between particle and collector k; k unit vector in positive z-direction;
(3.1.17), constant in (4.1.9a) for region or phase, constant in (2.2.8ac),
Henrys law constant, (1/h) is a (2/) (3.1.48)
characteristic thickness of double layer kB Boltzmanns constant (3.1.72), (3.3.90c)
(6.3.31a), height of liquid in a capillary ka, kd rate constants for adsorption and
at any time t; value of h as t ! (6.1.11) dissociation, respectively (4.1.77a)
h, h, hG contributions of different species to h kb, kf backward and forward reaction rate
(4.1.9c) constants (5.4.42)
hmin minimum value of h kAo,kAw mass-transfer coefficient of species
H; Hf; Hi; plate height, stack height; molar A in organic or water phase
0
H Ci , HCi  0 ; HPi ; enthalpy of feed; value of H for species kc, kg, kxj, ky mass-transfer coefficients for species i
Hlf, Hvf i (6.3.22), Henrys law constants for (3.1.139), (3.4.3)
0 0 0 0
species i in gasliquid equilibrium; k c, k g, k x , k y values of kc, kg, kx and ky for equimolar
(3.3.59) (4.1.7); (3.4.1b), (5.2.6); (5.2.7); counterdiffusion (3.1.124)
(8.1.49), (3.4.1a); molar enthalpy of kd, ks; k , mass-transfer coefficients in
liquid fraction and vapor fraction, crystallization (3.4.23a,b); liquid film
respectively, of the feed mass transfer coefficient (5.3.3)
H o , H oP height of a transfer unit defined by kigc, kigx, kigy mass-transfer coefficients for species i
(8.1.96) in gas phase when the concentration
Hm magnetic field strength vector gradient is expressed in terms of C,
H A , H oA , H cA ; Henrys law constants for species A molar concentration of species i in gas
H1, H2 (3.4.1a,b), (3.4.8); defined by (7.1.20b) phase, x, mole fraction of species i in
HD, HM, HS, HSM components of plate height (7.1.107ei) gas phase and similarly y, mole
Hi partial molar enthalpy of species i fraction in gas phase, respectively
0
HTU height of a transfer unit (6.4.85), kT ; k T thermal diffusion ratio (3.1.45);
(8.1.54b), (8.1.57b), (8.1.65e), thermal diffusion constant (4.2.64)
(8.1.245b), (8.1.247a), (8.1.357b) kgf, kgs gas film mass-transfer coefficient on
feed side and strip side of a liquid
i unit vector in positive x-direction, membrane (5.4.97a), (5.4.99a)
current density (3.1.108c) k f , k s ; k p liquid film mass-transfer coefficient on
I; I; Ij; IC
isbL ionic strength of the solution (3.1.10c), feed side and strip side of a liquid
(4.1.9b); purity index (1.4.3b), current; membrane (5.4.97b), (5.4.99b);
value of purity index for region j particle mass-transfer coefficient
(1.4.3b); integral (7.2.86) (7.2.217b)
xviii Notation

0
k s , k s rate constants for forward and Kijc; kijx; k ijx molar concentration based overall
backward interfacial reactions mass-transfer coefficient for phase j
0
(5.3.40) (8.1.1c); j phase mass-transfer
0
k il ; k_ i1 distribution ratio of species i between coefficient (8.1.60); value of kijx for
regions 1 and 2, also called capacity equimolar counterdiffusion (8.1.62a)
factor (1.4.1); distribution ratio defined Ko, Kw overall mass-transfer coefficient based
by (2.2.19) for species i between on organic or aqueous phase,
streams 1 and 2 ionization product for water (5.4.41c)
kcR, kcE mass-transfer coefficient in the Kis, Kps values of K for ion i/protein (p)salt (s)
continuous phase, raffinate based, exchange on an ion exchange resin
extract based (6.4.97a,b) (7.1.109d), (3.3.122b)
kgr, knu rate constant for crystal growth and KxE, KxR overall mass-transfer coefficient Kx
nucleation, respectively (6.4.51) (3.4.5) based on extract phase and
kil; kil3, kil4, kilo; mass-transfer coefficients for species i raffinate phase, respectively (6.4.77),
kimo in liquid phase; value of kil for (6.4.81)
condition 3, condition 4, channel inlet;
species i mass-transfer coefficient l; length of a device, length of molten
through organic filled membrane pore zone in zone melting (6.3.109b),
k1 first-order reaction rate constant characteristic dimension of the
(5.3.7) separator; constant in (2.2.8ac),
k 1m membrane mass-transfer coefficient length
for species 1 (4.3.1) ik , ki phenomenological coefficients
K equilibrium constant for a chemical (3.1.203)
reaction (3.3.68), or an ion exchange loc characteristic length of a local volume
process (3.3.121i), a constant (6.3.49) corresponding to a point in volume
Kx; K* mole fraction based K for a chemical averaging Section 6.2.1.1
reaction (5.2.35); defined by (5.4.100) x , y , z dimensions of a rectangular separator,
K1, K2, K3 constants in membrane transport Figure 3.2.1
(6.3.155a,b) L; Lf ; L length of a separator, dimension of
KAo, KAw; KAB overall mass-transfer coefficient of length, characteristic crystal size;
species A based on organic or water molar feed flow rate; nondimensional
phase; equilibrium constant (7.1.42c) L (7.2.38)
Kc, Kg, Kx, Ky overall mass-transfer coefficients Lii, Lis, Lss phenomenological coefficients for
(3.4.5), (3.4.6) binary system (i, s) (3.1.208), (3.1.209)
KcE Kc based on extract phase (6.4.80) Lik,Lki,LiT phenomenological coefficients
K AC equilibrium constant for ion exchange (3.1.205)
reaction (5.2.122) Lp ; Lap ; Lbp hydraulic transport parameter in
Kd equilibrium constant for proteinion KedemKatchalsky model (6.3.158a);
exchange resin binding (4.1.77c), value of Lp in perfect region; value of
ionization equilibrium constant Lp in leaky region
(5.2.4) LT; Lmin; LMTZ; separator length (= L); (7.1.60); Figure
Kd1, Kd2 dissociation constant for solutes 1 and LUB 7.1.5(b); (7.1.21g)
2, respectively (5.2.61a), (6.3.29)
Ki; K1, K2, K3; K ai ; equilibrium ratio of species i between m; mB; mi; m1; velocity profile constant (7.3.134);
K i regions 1 and 2 (1.4.1) or (3.3.61); m0i moles of B; moles of species i in
value of Ki for species 1, 2 and 3; values separator; moles of species 1 in
of Ki in terms of activities (4.1.3); separator; total number of moles of
o
il il =P in dilute solution stripping species i in separator
(4.1.19b) mij; moij ; mij n; moles of species i in region j, total
K i ; K overall liquid-phase mass-transfer maij number of moles of i in region j at t = 0;
coefficient for species i (7.1.5a); moles of i in region j after nth contact;
reaction equilibrium constant in the number of atoms of ith isotope of the
liquid phase based on molar element in region j
concentrations (5.2.52a) mi moles of species i in interfacial region
Notation xix

mi,j molality, moles of i per kilogram of nmax; nmed, npar peak capacity (3.2.32) (6.3.26a);
solvent in region j: j = R, resin; j = w, refractive index of medium and
aqueous phase particle
mF,R molality of fixed charges in the resin np, np; npy; nt particle number flux, (3.1.65), (3.1.66),
phase (3.1.68); particle flux in y-direction;
mp; msl mass of particle; solvent moles in number of turns by gas in a cyclone
stationary liquid phase (7.1.104b) (7.3.146b)
mtj total moles of all species in region j N; N(rp); total number of stages in a multistage
(j = f, feed; j = 1, vapor phase; j = 2, Nr min , r p , device or in the enriching section of a
, liquid phase) Nr p max cascade, anionic species in Donnan
m11(t), m21(t) moles of species 1 and 2 in region 1 at dialysis, a metal species, number
time t concentration of molecules; numbers/
mr11 t, mr21 t values of m11(t) and m21(t) in the case cm3, number of particles per unit fluid
of a chemical reaction volume in the size range of rmin to rp;
mag
mop magnetophoretic mobility (7.3.251) value of N(rmin, rp) for r p max
M; Mi, Ms; M sl molecular weight, a metal species, ~ Ni
N; Avogadros number (6.02  1023
number of stages in stripping section molecules/gmol); plate number for i
of a cascade; value of M for species i, (6.3.27a)
for solvent s; M for coating liquid in N A ; N rA , N tAy ; molar flux of species A in a fixed
stationary phase N dil reference frame without and with
Mt average molecular weight of solution reaction; total molar flux of species
(3.1.56) A in facilitated transport or counter-
Mw magnetization of wire (3.1.23) transport or co-transport in y-direction;
Mse seed mass density per unit liquid normality of diluate solution (8.1.404)
volume (6.4.40a) Ni; Nim, Njm; Nix, species i flux; Ni through membrane,
MT; MTa, MTb suspension density of a crystal- Niy, Niz; Nir; Nij; Nj through membrane; components of
containing solution (2.4.2f), (6.4.18); jN iy j Ni in x-, y- and z-directions; radial
value of MT for cases a and b component of Ni; Ni through surface
i
Mo(n); Mof , nth moment of the density function area Sj; magnitude of Niy
i i p
Mo , Mov (2.4.1g); ith moment of molecular N iz z-component of species i flux, Niz,
weight density function of based on unit pore cross-sectional area
feed, liquid and vapor, respectively (3.1.112a)
(6.3.70) Noj; NoP; Np number of transfer units (8.1.92),
j j
Mof r , Mogr jth moment of crystal size density (8.1.96); defined in (8.1.96); number of
functions f(rp) and fg(gr) pores per unit area size rp (6.3.135d)
MWCO molecular weight cut off of a NR; NS flux ratio (3.1.129a); solvent flux
membrane Ni(rp); Nit number density of crystals having a
size less than rp and growth rate gri;
n; nc number of species/components in a total number of crystals per unit
system, number of contacts, stage/ volume having a growth rate of gri
plate number, number of positive (6.4.41c)
charges in a metal ion, number of unit Nt total number of particles per unit
bed elements; number of collectors, volume (6.4.10)
number of channels (7.3.109) NtoE, NtoR number of transfer units based on
nr p ; no ; n
~ r p population density function, particle extract and raffinate phases,
number density function (2.4.2a); respectively (6.4.86a), (6.4.83)
nucleation population density Noi number of particles of size rpi (2.4.2k)
parameter (6.4.7); defined by NTU number of transfer units (8.1.54c),
(6.4.46b) (8.1.57c), (8.1.66b), (8.1.67d), (8.1.338)
ni; nix,niy,niz; nij mass flux vector of species i, MiNi; its
components in x-, y- and z-directions; p dipole moment of a dielectric particle
ni through surface area Sj p stoichiometric coefficient for product
nk outwardly directed unit normal to the P, kinetic order in the dependence of
k-phase surface (6.2.26bd) nucleation rate (6.4.30a)
xx Notation

pA; pB; pi,pj partial pressures of species A, species qfr fraction of light reflected (3.1.47)
B; species i and species j qi(Ci2), qi1(Ci2), moles of species i in solid phase 1 per
pAb; pAi; pB;m value of pA in the gas bulk; value of pA qi1 ; q0i1 , qsi1 unit mass of solid phase, cross-
at gasliquid interface; logarithmic sectional average of qi1, initial value of
mean of pB (3.1.131b) qi1; saturation value of qi1
pif, pij, pip, piv value of pi in feed gas, region/stream j, qiR, qis moles of species i per unit mass of
permeate gas, vapor phase ion exchange resin (R) or solid
pib , pii species i partial pressure in bulk and adsorbent (s)
particle interface, averaged over bed qmaxR maximum molar fixed charge density
cross section, Figures 3.4.4(a), (b) per unit resin mass
pH; pI indicator of hydrogen ion q(r, , ) probability that a molecule having
concentration (5.2.65a); isoelectric configuration (r, , ) does not
point for a protein/amino acid; at pI = intersect pore wall (3.3.89f)
pH, net charge is zero Q volumetric fluid flow rate, product
pKi log10 Kdi, (5.2.65b) for i = 1, (5.2.74b) species in reaction (5.3.5), hydraulic
for i = 2, (6.3.29) permeability in Darcys law (6.1.4g,h),
P; Po;Pc total pressure, system pressure; heat transfer rate (6.4.47a,d)
standard state pressure; critical pressure Qc; Qd electrical charge of a collector; volume
P; P f ; P j , local solute permeability coefficient flow rate of dialysate solution
0
Pp ; Pp ; P (6.3.157b); feed pressure; total Qf; Qg; Qh; Qi; volumetric feed flow rate to separator;
pressure of jth region and permeate, Q, Qo, Qp, QR, Darcy permeability for gas through
respectively; gas pressure (Figure Q1, Q2 packed bed; amount of heat supplied
7.2.1(b)); gas pressure at the end of a at a high temperature; electrical charge
capillary of length l (6.1.5d) on 1 gmol of charged species i; amount
P 0i ; P i equilibrium gas-phase pressure for of heat rejected at a low temperature;
pure i adsorption at spreading volumetric flow rate at membrane
pressure , which is the same for a channel inlet; electrical charge of a
mixture (3.3.111a); pressure at particle; excess particle flux (7.2.123),
crossover point for solute i in volumetric flow rate of product stream
supercritical extraction from separator; heat supplied at the
Patm; Pliq; P1, P2 atmospheric pressure; pressure in the reboiler per mole of feed; volume flow
liquid (6.1.12); purification factors rate of overflow; volumetric rate of
(7.2.97) underflow/concentrate
Pe; Pei ; Pem i ; Pclet number (3.1.143g), (7.3.34d); Pe QAm, QBm, Qim, permeabilities of species A, B, i and j,
Pez; eff ; Pezj number for dispersion of solute i Qjm respectively, through membrane in gas
(6.3.23a); pore Pclet number (6.3.145a); permeation and pervaporation,
(z vz/Di,eff.z) (7.1.18h); j phase Pez (8.1.92) respectively
P sat sat
i , Pj ; vapor pressure of pure i and pure j, Qij permeability of species i through
P sat
iP ; P sat
i, curved respectively, at system temperature; region j (= A, B, C, D, 1, 2) of the
value of P sati on a plane surface; value membrane
of P sat
i on a curved surface Qov
im overall permeability of species in
P sat
M value of P sati for pure species i of membrane pervaporation for a
molecular weight M membrane of thickness m
P0 amplitude of pressure wave (3.1.48) Qcrys, Qsub heat transfer rate of a solution during
PR, Pw pressure of resin phase and external crystallization and subcooling,
aqueous solution, respectively respectively
Po, Pr power number (6.4.976), power (3.1.47) Qim0 value of Qim for Cim = 0
Qsm, Qsc solvent permeability through
q number of variables in a problem, membrane and cake in cake
stoichiometric coefficient for species Q, filtration
heat flux in a heat exchanger attached
to a cooling crystallizer (6.4.47a), the r vector of molecular position, radius
power of MT in expression (6.4.39a) for vector, unit vector in radially outward
B0, factor (8.1.150) direction
Notation xxi

r radial coordinate, any characterizing R1, R2 intrinsic RO rejection of the unionized


property of a continuous mixture species 1 and ionized species 2
r1, r2; r1i, r2o; rc; rf radii of curvature of interface (3.3.47); R (R) ion exchange resin with fixed positive
liquid outlet radii in tubular centrifuge; (negative) charge; cation (anion) of a
critical size of a nucleus (3.3.100b), surface active solute
cyclone radius; free surface radius in Rij;Rim;Ri,reqd permeation resistance of region j (j = A,
tubular centrifuge B, C, D) in the membrane to species i;
rg; rh; ri; rin radius of gyration of a macromolecule; membrane permeation resistance of
hydrodynamic viscosity based radius species i; solute i rejection required in
(3.3.90f); radius of spherical solute a RO membrane
molecule (3.3.90a); radius of liquid Rik, Rki; Rmin phenomenological coefficients
liquid interface in tubular centrifuge (3.1.207); minimum reflux ratio
ro radius of a sphere whose volume is (8.1.170)
equal to that of an ellipsoid (3.1.91g), Robs, Rtrue; Re; observed and true solute rejection in
radial location of the center of solute ReL; Reimp ultrafiltration and RO; Reynolds
peak profile, radius of a cylindrical number; Re for a plate of length L
centrifuge (3.1.143a), Table 3.1.5; Re for an
rp; rp,a; rp1, rp2; radius of particle, pore radius; impeller (Table 3.1.7), (6.4.96)
r pi ; rs; rt; rw analytical cut size (2.4.18); particles of
two different sizes; dimension of s fractional supersaturation (3.3.98b),
particle of a certain size; particle size power of the P dependence of R ^ c
for classification (2.4.8); cyclone exit (6.3.138j), Laplace transform complex
pipe radius; radius of wire variable (5.4.35), solution volume fed
rmax, rmin; rp,50 maximum and minimum radius of to bed per unit empty bed cross
membrane pores or particle sizes; section (7.1.17a), eluent/salt
q equiprobable size (7.1.109b), salinity (10.1.13)
r 2p hydraulic mean pore radius (3.4.87) sm; smp, sms; sp pore surface area/membrane volume
(6.3.135e); value of sm for membrane
r p , r p1; 0 , r pi1; i mean of particle size distribution
pore volume or membrane solids
based on f(rp) and n(rp), respectively
volume (6.3.135f,g); sedimentation
(2.4.1e), (2.4.2g), (2.4.2h)
coefficient (4.2.16b)
r p3; 2 Sauter mean radius of a drop, bubble
s1, s2 two different solvents
or particle (6.4.89)
S; Sc solute transmission/sieving coefficient
ra; re; re|flow rate of arrival of cells; fractional water
(6.3.141e), supersaturation ratio
recovery; value of re in a cell (6.3.172b)
(3.3.98c), stripping factor (8.1.135),
R; R universal gas constant; radius of a tube
(8.1.189b); bed/column cross-sectional
or capillary, reflux ratio (2.3.5), (2.3.7),
area
(8.1.137), solute rejection in reverse
Sf, Sk, SM; Sj; SM, cross-sectional area vector of feed
osmosis, solute retention in
Sdp entrance, kth feed exit and membrane
ultrafiltration
^ c , R
^ cw surface area vector in a separator;
Rc ; R cake resistance; specific cake
cross-sectional area of flow for jth
resistance per unit cake thickness and
stream; selectivities (7.3.219a,b)
unit cake mass, respectively (6.3.135l)
Sij; Sij molar entropy of species i; partial
Rh; Ri; R0i; RL hydraulic radius (6.1.4c); solute
molar entropy of species i (3.3.17b)
rejection/retention by membrane for
Sim; Sim0 solubility coefficient of species i in
species i, retention ratio for species i
membrane; value of Sim for Cim = 0
(7.3.211); fraction of solute i in mobile
S value of S when Pem i >>1 (6.3.145b)
phase (7.1.16c); largest radius of a
S ; S ; S1 , S2 surface area of interfacial region
conical tube
(Figure 3.3.2A); molar surface area in
Rm ; R s ; R membrane resistance; resolution
gassolid adsorption (3.3.107); sieving
between neighboring peaks (2.5.7) in
coefficients for species 1 and 2
chromatography; value of Ri at large
Stj; Smag total entropy for region j (3.3.3);
Pemi
magnetic field force strength
RA, Ri molar rate of production of A per unit
(7.3.251)
volume (5.3.7), (6.2.2d), for species i
xxii Notation

Sobs; Strue observed solute transmission/sieving ui; uk; uij; uil; uio; mass fraction of impurity species i;
coefficient; true value of S uis; uilb mass fraction of species k; mass
Sc; Scc Schmidt number (3.1.143a); Sc for fraction of species i in region j (1.3.5)
continuous phase or jth stream (2.1.20); value of ui in
Sh; Shc, ShD; Shp; Sherwood number (3.1.143a); Sh for melt; value of ui initially in the solid;
Shz continuous phase and dispersed value of ui in the recrystallized solid;
phase, respectively; Sh in a packed bed value of uil in the bulk melt
(7.2.218a); Sh at location z (7.2.64) uo; urc; uro mass fraction of solute in the solution
St Stanton number (3.1.143g), Stokes charged for crystallization; uc/(1 uc);
number (6.3.41) uo/(1 uo) (6.4.47b)
um
i ionic mobility (3.1.108j)
t; tbr; tc; tres; t time; breakthrough time; time for cut uiEn, uiRn mass fraction of species i in extract and
point in the chromatographic raffinate streams, respectively, from
separator output (2.5.1); residence stage n
time; thickness of interfacial region urig, uris defined by (8.1.349) and (8.1.350)
(Figure 3.3.2A) urij weight of solute i per unit weight of
t
i ; t1 , t2 ; nondimensional time variable for phase j (9.1.32)
t; t 1 , t 2 , t 3 species i (6.3.12); value for t
i for U i ; Up migration velocity vector for species i
species 1, 2 (3.2.9), (3.2.20); (3.1.84b); particle velocity vector
breakthrough time (7.1.15c); value of t Ui averaged velocity vector of ith
for species 1, 2 and 3 molecules due to all forces
new
t in 0
i , ti times when species i appears and Ui defined by (3.1.103)
disappears, respectively, from a Uix, Uiy, Uiz components of migration velocity of
chromatographic separator output species i in x-, y- and z-directions,
tim, tis transport number of i in membrane or (3.1.82)
solution (3.1.108d) U ik ; hU ik ik ; U
^ ik value of Ui in the kth phase/region;
t s1 , t s2 time required for solvents, s1, s2 (3.2.24) average of Uik in the kth phase/region;
t Ri ; t RM ; t Ro retention time for species i in capillary defined as a fluctuation by (6.2.28)
electrophoresis (6.3.18a), (7.1.99d); U int int int
p ; U px i ; U pr p internal particle velocity vector
retention time for the mobile phase; (6.2.50d); its component in the
retention time based on vz,avg (7.3.207) direction of the internal coordinate xi;
T; T1; T2; Tc absolute temperature; temperature of internal particle velocity vector for all
cooled plate; temperature of the particles of size rp
heated plate; critical temperature Upt; Upr; Uprt; terminal velocity vector of particle
Tf; Tg; Ti; Tp feed temperature; glass transition Upzt, Upyt (3.1.62); radial particle velocity;
temperature of a glassy polymer; value terminal value of Upr; value of Upt in
of absolute temperature T of region i; z-direction (6.3.1), (7.2.211),
product temperature y-direction (7.3.154)
TC; TH, TL; TR temperature of condenser; two
temperatures in supercritical v, vt ; v , vt ; v , mass averaged velocity vector of a
ref
extraction; reboiler temperature vt ; vtj , vtj fluid (also vt); molar average velocity
Tcf; Tci; Tmi temperature of cooling fluid, critical vector of a fluid (also vt );
temperature of species i; melting nondimensional v (6.3.39); reference
temperature for species i vt; values of vt and vt on surface area Sj
Tsat temperature at which the solution is (2.1.1), (2.1.2)
saturated vi; vij averaged velocity vector of ith
Tsol temperature of the solution due to species; value of vi on surface area Sj
undergo crystallization (2.1.1)
Th dimensionless group (3.1.46b) vk ; vkz ; vint
k velocity of region k; z-component of
velocity vk of region k; mass average
u number of fundamental dimensions velocity of the interface of two
(Section 3.1.4.1) phases
uc mass fraction of solute in the vrp particle diffusion velocity relative to
crystallized solution on a solid-free basis that of the fluid phase (3.1.43)
Notation xxiii

p
vci ; vCi ; vo; vo; average velocity of liquid zone carrying Vk; Vl; V ; Vo; volume of region k; liquid volume;
v
Ci ; vog species i (7.3.208); concentration wave V to ;Vo specific pore volume of a microporous
velocity of species i (7.1.12a); velocity adsorbent (4.1.64a); volume of
of micelles, superficial velocity organic solvent; total amount of Vo
(3.1.176), in a packed bed (6.1.4a); (6.3.99); sample volume
amplitude of square-wave velocity (7.1.101b)
(7.1.72a); nondimensional VM; Vp; Vs; V S ; mobile-phase volume in a column
concentration wave velocity (7.1.105o); Vsp (7.1.99g); volume of a particle,
superficial velocity for gas phase cumulative permeate volume (7.2.87);
vr; vs; vso; vtw; vtz; radial velocity; shell-side velocity stationary adsorbent-phase volume in
vvz (3.1.175), volume flux of solvent/ a column; suspension volume;
permeate (7.2.71); value of vs at stationary liquid-phase volume;
membrane channel inlet; tangential mobile-phase volume present inside
fluid velocity in wall region (7.3.134); the pores of a particle (7.1.110c)
same as vz; vapor velocity in V s; V t ; V w; V partial molar volume of solvent;
z-direction (6.3.47a) averaged partial molar volume
vx, vy; vyw; vywf local convective velocity in x-direction, (3.1.56); volume of water; volume of
y-direction; value of vy at wall; value of interfacial region, Figure 3.3.2A
vyw at feed entrance (7.2.39) Vfi; Vfe initial volume of feed solution; final
vz ; vzmax ; velocity of fluid in z-direction; volume of solution
vz; avg ; vz; avg; f maximum value of vz, averaged value Vf0; VfR; V N i ; V Ri volume of feed solution at time t = 0;
of vz over flow cross section; value of volume of retentate; net retention
vz,avg at feed location (6.1.5f) volume for species i (7.1.99j), retention
v uniform gas velocity far away from object volume for species i (7.1.99e,f),
vf r ; v
i, eff velocity of freezing interface in zone (7.1.99h)
melting (6.3.110c); nondimensional V iex
cap ; V eff ; volumetric ion exchange capacity of
velocity of species i in capillary V channel packed bed; liquid filled centrifuge
electrophoresis (6.3.12) volume; volume of channel
vAA
z ; vzH , vzL velocity of interface AA between V AY partial molar volume of electrolyte AY
suspension and clarified liquid in z- (3.3.119b)
direction (4.2.52); interstitial gas V ic value of V i for a crystal of i
velocity at high-pressure feed step and Vim molar volume of species i at its normal
low-pressure purge step, respectively boiling point
(7.1.53a,b) V ij value of V i in region j
vEOF; vEOF,z electroosmotic velocity (6.1.22); value VCR volume concentration ratio (Vf 0/Vf R)
in z-direction (6.4.98)
vr defined by (7.2.167)
V ; V b; V d ; V f ; volume of a region, volume of w; wcrys ; wi ; wso mass of adsorbent, quantity defined by
^ a ; V_ ; V
V h; V separator, volume of feed solution/ (5.2.54); mass of crystals formed at any
eluent passed, volume of a sphere time, mass of solute i charged; mass of
(3.3.52a), voltage between electrodes; solution charged to the crystallizer
volume of buffer; volume of the wij; wtj; wtz; wstj ; mass flow rate of species i through Sj
dialysate solution; volume of feed wtE, wER (2.1.4); total mass flow rate through Sj
solution; hydrodynamic volume of a (2.1.5); total mass flow rate in z-
macromolecule/protein (3.3.90f); direction; total solids flow rate in jth
adsorbed monolayer phase volume/ stream (2.4.3a); wtj for j = extract, E, wtj
weight of adsorbent (3.3.114b); radial for j = raffinate, R
volume flow rate of solution (Figure wE, wM, wR weight of mixture at E, M and R,
7.1.6); defined by (7.1.18i) respectively
~ c; V i; V i ; V j
V critical molar volume of a species; wl; ws; wsp (rp); mass of stationary liquid phase used as
molar volume of pure species i; partial wa coating; mass of particles per unit of
molar volume of species i; volume of fluid volume (2.4.2e); mass of a solid
region j: j = 1, stationary phase; j = 2, particle of size rp (2.4.2e); waist size of
mobile phase a light beam
xxiv Notation

W, WA width of rectangular channel walls, xs,lm defined by (3.1.137)


molar transfer rate of species A X molar density of fixed charges in
W total reversible work done electrically charged system
(3.1.25a,b) Xij, Xijn mole ratio of species i in region j
Wbi, Wij, Wtj band width of the chromatographic (1.3.2) or stream j (2.2.2d), Xij for
output of species i (Figure 2.5.2), molar state n
flow rate of species i through Sj (2.1.7), X aij abundance ratio of isotope i in region j
total molar flow rate through (1.3.7)
Sj (2.1.8)
WtAm, WtBm molar rate of permeation of species y, yf coordinate direction, normal to gas
A and B through membrane liquid, liquidliquid or membrane
Wtf, Wtl, Wtv molar flow rate of feed, liquid fraction fluid interface, feed gas mole fraction
and vapor fraction, respectively (Figure 7.1.14)
Wtlb, Wtld total molar flow rate of bottoms yi, yA, yH, yL mole fraction of species i and A in
product and distillate product from a vapor/gas phase, respectively, fluid
distillation column mole fraction (7.1.62b), gas-phase
Wt1, Wt2, Wtp total molar flow rate of streams j = 1, j = mole fractions defined by
2 and j = permeate (7.1.54a)
0
W ttjn total molar flow rate of stream j for y A , y  , y im hypothetical gas-phase mole fraction
stage n in enriching section of cascade of species A in equilibrium with xAb,
We Weber number, (c vc2 dp/) (6.4.91) nondimensional y-coordinate (5.3.5j),
y-coordinate of limiting trajectory
x, xA coordinate direction, mole fraction of (7.3.263a)
species A in liquid phase yAb, yAi bulk gas mole fraction of species A,
x A hypothetical liquid phase mole value of yA at a two-phase interface
fraction of species A in equilibrium Yij, YiR, Y2, Y2,f segregation fraction of species i in
with yAb region j (1.3.8a), segregation fraction of
xAb, xAi bulk liquid mole fraction of species A, solute i in retentate in a membrane
value of xA at a two-phase interface process (6.4.107), defined by (7.2.19)
xi, xij, x i , xi,j mole fraction of species i, value of xi (i and (7.2.20)
= A,B, 1, 2, s(solvent), etc.) in region j Y11(t), Y21(t) segregation fraction of species 1 and 2
or stream j (j =1, 2, f(feed), g = gas, l = in region 1 at time t
liquid, p = permeate, R = resin, s = Y aij , Y_ ij; Y_ rj segregation fraction for an isotopic
solution, v = vapor, w = water), xij mixture (1.3.9a), of species i stream j
where j=, the surface adsorbed phase, (2.2.12), of particles of size r in stream j
mole fraction of species i in region j (2.4.5)
when molecular formula is substituted
for species i z, zcr (or zcrit), zH, coordinate direction, critical distance
x iE , x iE , x eiE mole fraction of species i in extract zL, zAA, zBB (7.2.133), characteristic locations at
phase, value of xiE in equilibrium with pressures PH and PL (7.1.55e), vertical
xiR, value of xiE if both streams leave coordinates of interfaces AA and BB,
stage in equilibrium respectively
xilb, xild bulk value of xil, mole fraction of i in z HL , z HS height of the liquid level of a dilute
liquid product from reboiler in a suspension, height of sludge layer
distillation column, xil in liquid z , z
i nondimensional z-coordinate (Figure
product from condenser at the top of a 3.2.2), (6.3.12), value of z for center of
distillation column mass of concentration profile of
x iR , x iR , x eiR mole fraction of species i in raffinate species i (3.2.22a,b)
phase, value of xiR in equilibrium with z , z , z o C i2 z-coordinate locations of regions and
xiE, value of xiR if both streams leave , respectively, defined by (7.1.17e,f)
stage in equilibrium Zi, Zi,eff, Zp electrochemical valence of species i,
0
xjf, xjp, x ip mole fraction of species j in feed and effective charge on ion i due to the
permeate, respectively, mole fraction diffuse double layer (< Zi) (6.3.31a),
(Figure 7.2.1(b)) value of Zi for a protein
Notation xxv

Greek letters 1, 2, 3, 4, 12; factors/parameters in plate height


SL, LG, SG (7.1.107fi); interfacial tension
constant in (3.1.215), separation factor between two bulk phases 1 and 2;
between two species 1 and 2 in interfacial tension between phases S
capillary electrophoresis (6.3.27e) and L, L and G, S and G, respectively
f , l , v parameters for feed, liquid fraction, (S = solid, L = liquid, G = gaseous)
vapor fraction in a continuous _ ; _ w shear rate (3.1.74), (6.1.31); wall shear
chemical mixture described by a rate
distribution function (4.1.33f), il ; il, u infinite dilution activity coefficient for
(6.3.71) species i in the liquid; activity
hf ft
ij ; ij , ij , separation factor between two species i coefficient of species i in liquid phase
ht
ij ; ijn (i = 1, A, s (solvent)) and regions j on a mass fraction basis at infinite
(j = 2, B (species), i), value of ij dilution (6.3.83)
between two streams (2.2.3), (2.2.4),
(2.2.2a); value of ij for nth plate Eis molar excess surface concentration of
in multicomponent separation factor species i/cm2 of pore surface area
between species i and n (1.6.6) (3.1.117a)
0
separation factor of Sandell (1.4.15) i ; Ei surface concentration of species i
AB ; evap
ij ; perm
ij value of AB under ideal condition of (3.3.34); algebraic surface excess of
zero pressure ratio (6.3.198), (8.1.426); species i (3.3.40b), (5.2.145a)
value of ij for evaporation only ; (x), (z), falling film thickness; delta function in
(6.3.180); ideal separation factor in z ; c x-, z- and z-directions (3.2.14b); cake
vapor permeation (6.3.180) layer thickness
g ; i ; ; thickness of gas film; solubility
defined by (2.2.8a,b), exponent in 0
m ; s ; parameter of species i, characteristic
(3.3.89a), indicator of macromolecular thickness of concentration profile
shape for a given rg (3.3.90e), constant (7.3.202); thickness of liquid film;
in (3.1.215), defined by (7.1.52b), membrane thickness, pore length;
(7.2.58), (7.2.138) sorbed surface phase thickness (3.1.118b);
f , , v compressibility of fluid (3.1.48), thickness of liquid film from phase
parameters for feed, liquid fraction and interface to reaction interface (5.3.22)
vapor fraction in a continuous A , B , C membrane thickness in regions A, B, C
chemical mixture (4.1.33f), (6.3.71) of a membrane (Figure 6.3.35(b))
i ; p; A, B; exponent in (3.3.112c), parameter 1 , 2 , n thickness of layer 1, layer 2, layer n in a
; c (7.2.18); compressibility of particle composite membrane
(3.1.48); defined by (7.1.52a); C; E s ; G; molar supersaturation (3.3.98a);
coefficient of volume expansion Gt ; H s ; P; activation energy for crystal growth rate
(6.1.9); a parameter (6.4.129); S; T (3.4.29); Gibbs free energy change, e.g.
coefficient of thermal expansion of for forming a crystal (3.3.100a); change
solution density (7.3.233) in Gibbs free energy for all molecules;
enthalpy of solution of a gas species
; cr ; A ; R ; R2 surface tension, interfacial tension, in a polymer; pressure difference
pressure ratio (6.3.197), (6.4.123), (Pf Pp), etc.; selectivity index (7.2.93);
(8.1.426), parameter (7.1.18f); critical extent of supercooling (3.3.98d),
surface tension for a polymer; temperature difference (T2 T1)
Damkohler number (kb2m =DA ) H crys heat of crystallization
(5.4.100); (DAk1/k 2 ) (5.3.11); defined G0 , H0 , S0 standard Gibbs free energy change,
by (5.3.35j) enthalpy change and entropy change
f , , v parameters for feed, liquid and vapor for a reaction
fraction (4.1.33f), (6.3.69) dp ; r p ; ws particle diameter difference between
i; if, ij; jf activity coefficient for species i, two consecutive sieves; (dp/2) or the
dimensionless steric correction factor; net growth in size of all seed crystals
value of i in feed stream, in phase j; (6.4.40i); mass of crystals retained on a
value of j in feed stream (species j) sieve of given size
xxvi Notation

x, y, z, lengths of a small rectangular volume cut (2.2.10a), stage cut, fraction of


zL , zH element in three coordinate directions, adsorbent sites occupied (3.3.112a),
defined by (7.1.53a,b) contact angle, gasliquidsolid system
0i ; 1 , , 0i1 0i2 , number of water molecules, (Figure 3.3.16), contact angle between
, cations and anions, respectively membrane surface and liquid
released during binding; osmotic (6.3.140)
pressure difference (f p) i component cut for species i
i; i,sat; mass concentration change of species i (2.2.10b), Langmuir isotherm
in crystallizer (6.4.24); change in (7.1.36a), nondimensional
saturation concentration due to T mobile phase concentration
(6.4.51); potential difference between (7.1.18b)
two phases id, kd component cut for species i and k,
R defined by (8.1.328) respectively, in the distillation column
ri del operator in internal coordinates xi, top product (8.1.224)
yi and zi
Ao ; d ;i distribution coefficient of species
void volume fraction in a packed bed, A between aqueous and organic
electrical permittivity of the fluid (= 0d) phases (3.4.16); reciprocal Debye
a; b phase angle; fractional cross-sectional length (3.1.17); distribution coefficient
area of membrane which is defective of species i
d; i; k dielectric constant of a fluid, porosity i1 , N
i1 distribution coefficient of species i
of particle deposit on filter media; between regions 1 and 2 (1.4.1), its
LennardJones force constant for Nernst limit (3.3.78), (3.3.79)
species i; volume fraction of phase k in if ; ij ; im partition coefficient of solute i between
a multiphase system feed solution and feed side of the
m;p;0;0 porosity of membrane, pellet, bead or membrane; partition/distribution
particle; dielectrical constant of a coefficient of solute i between two
particle; electrical permittivity of immiscible phases; ij for feed
vacuum; porosity of spacer in spiral- solution and the membrane (3.3.87),
wound module (3.1.170), porosity of (3.3.89a)
0 00
clean filter ij , ij effective value of ij when there is no
B; C fractional area of defects in glassy skin reaction or there is reaction
region B (Figure 6.3.35(b)); value of io partition coefficient of i between an
fractional membrane cross section in organic and an aqueous phase
region C (6.3.101a)
12 enrichment factor for species 1 and 2 ip im at permeatemembrane interface
(1.4.10), (2.2.2e) (3.4.58)
i; loc r local equilibrium partition constant
zeta (electrokinetic) potential (3.1.11a) (3.3.89g)
0
is , is impurity distribution coefficient
recycle ratio (2.2.22), nondimensional (6.3.109a)
0
variable (3.2.11), (6.3.14b), (6.3.114), is, eff effective partition coefficient in zone
fraction of particles collected (7.3.37), melting for species i (6.3.119a)
(7.3.38), (7.3.46), intrinsic viscosity iE distribution (partition) coefficient for
variable defined by (3.2.17) species i in solvent extraction
0
i ; j; ij; j , variable (2.5.5), for species i (3.2.25); iR distribution coefficient of i in ion
; i ; iF impurity ratio for region j (1.4.3a); exchange system (3.3.115)
impurity ratio for ith species in region j p1 ; 0p1 protein partition coefficient in aqueous
(1.6.4); defined by (1.4.4); intrinsic two-phase extraction; protein
viscosity (3.3.90f); its value for polymer distribution coefficient in the
i (7.1.110f); current utilization factor absence of a charge gradient at
(8.1.406) interface
hi i;h2i i first moment of Ci(i , t i ) (3.2.26); m
salt distribution coefficient of salt, molal
defined by (3.2.29) basis (4.1.34o)
Notation xxvii

Debye length (3.1.10b), mean free path ; constant (3.1416); osmotic pressure,
of a gas molecule (3.1.114), filter spreading pressure (3.3.41a)
coefficient (7.2.187), parameter for a f ; i ; p , w osmotic pressure of feed solution;
dialyzer (8.1.399), parameter for a spreading pressure in adsorption of
distillation plate/stage (8.3.38), latent solute i from a solution (4.1.63a);
heat of vaporization/condensation osmotic pressures of permeate
; x, z; i ; ; molecular conformation coordinate solution and the solution at the
; oi (3.3.89c); electrode spacings (7.3.18); membrane wall, respectively
retention parameter for species i 1 , 2 osmotic pressure of solution in regions
(7.3.213), ionic equivalent 1 and 2
conductance of ion i (3.1.108r); value
of i for a cation; value of i for an ; b ; c ; e ; fluid density; bulk density of the
anion; value of i at infinite dilution f ; f m ; i ; ij packed bed; density of continuous
(Table 3.A.8) phase (also fluid density at critical
1 , 2 defined by (5.4.100) point); electric charge density per unit
equivalent conductance of a salt volume; density of feed fluid; moles/
(an electrolyte) (3.1.108s) fluid volume; mass concentration of
species i (also density of solute i
; 0 ; dr dynamic fluid viscosity; value of at (3.3.90b)); value of i in region j (1.3.4)
inlet; viscosity of drop fluid ; p ; pg ; density of liquid (melt); mass density
C , D fluid viscosity of continuous and s ; t ; tj of particle material; particle mass
dispersed phases, respectively concentration in gas; density of solid
m
EOF electroosmotic mobility (6.3.10c) material; total density of fluid; total
i , 0i , m
i ; species i chemical potential and its mass density of mixture in region j
m ; m
i, eff ion, g standard state value; ionic mobility of (1.3.5)
ion i (3.1.108m); effective value of m i v ; i1 , if density of vapor phase; mass
(6.3.8d,g); ionic mobility of ion in gas concentration of species i in
phase crystallizer outlet solution and feed
ij ; ijn ; e
ij value of i in region j; value of ij for solution, respectively
plate/stage n; value of ij in a system i ; ip ; p; rp mass density of species i in liquid
with electrical charge (3.3.27) (melt); mass density of crystals of
ijPl value of ij for a planar interface species i; mass density contribution of
(3.3.50) particles of size rp to rp drp (6.2.55)
M, v M chemical potential of species of R ; 1 , 2 density of resin particle, reduced
molecular weight M in liquid and density of fluid ( =c ); density of
vapor phases, respectively, in liquids 1 and 2
continuous chemical mixtures dR ; pR fluid resistivity; particle resistivity
m m m
0 , p , s ; magnetic permeabilities of vacuum, avg ; isat average gas density (6.1.5e); mass
particle and solution, respectively; density of solute i in solution at
chemical potential of a crystal saturation
ik , ik intrinsic phase average and phase
; z; 0 kinematic viscosity; at axial location average, respectively, of ik (6.2.24a,b)
z; at inlet ik; avgi ik (6.2.24a)
i stoichiometric coefficient for species i hi i mass density of solute i in solution
in any chemical reaction averaged over flow cross section
A , Y moles of ions A and Y produced by (6.2.16b)
dissociation of 1 mole of electrolyte
AY electrical conductivity of the solution
(3.1.108q), particle sticking probability
extent of separation for a binary system i standard deviation of any profile
(1.4.16), (2.2.11), (6.3.105) (3.2.21a,b), LennardJones parameter,
12 , rp ; r p extent of separation for components 1 ionic equivalent conductance
1 2
and 2 in particle classification (2.4.9), (3.1.108r), solute i reflection coefficient
for particles of size r p1 and r p2 (2.4.6) through membrane (6.3.157a,b)
xxviii Notation

p ; v , iv ; x , y steric factor for protein SMA model ext ext ext


i ; i , i value of or ext for any species i;
(3.3.122d); specific volume based values of ext
i in regions and
deposit in filter (7.2.172), (7.2.189); ^ ig ; sat
; fugacity coefficient (3.3.56); fugacity
i
standard deviations (7.3.173) coefficient of species i in a mixture in
AB ; ix average of A and B (3.1.91b); gas phase; value of ^ ig for pure i at P sat
i
standard deviations (7.3.12a) at system temperature
tot 
 

i ; m nondimensional standard deviation of i ;i ; 0i max ; defined by (3.2.4); defined by (3.2.5b),
a profile (3.2.21a,b); average electrical
i ; i

defined by (3.2.6); defined by (3.2.9)
conductivity of the ionic and electronic and (6.3.14a)
ext ext
X
species in a mixed conducting ti i , summation over different
membrane external forces (3.2.4)
ti , zi ;
zi ; V i standard deviation in the output
profile of species i in t-coordinate and , p , s volume suspectibility of fluid, particle
z-coordinate, respectively (2.5.3), and the solution
(6.3.18c,b); nondimensionalized zi ij , ip Flory interaction parameter between
(6.3.16b); standard deviation in species i and j and species i and
volume flow units (7.1.102b) polymer

; m ; w; wc tortuosity factor; value of for porous ; molecular orientation coordinate


membrane; wall shear stress (7.2.118); vector (3.3.89c); extent of facilitation of
w in cake region (7.2.136a) flux (5.4.58) (5.4.59a), selectivity
yx , yz components of tangential stress y in (7.2.92)
x- and z-directions (6.1.24) k any property or characteristic of the
kth phase
; A, B potential of any external force field s , v shape factor for particle surface and
(also ext), angle in spherical polar volume (Example 2.4.1), (2.4.2e)
coordinate system, enhancement (3.4.26), (6.4.15), (6.4.19)
factor due to reaction, solids volume As association factor (3.1.91a)
fraction profile in suspension e electrical potential in the double layer
boundary layer (7.2.109); enhancement k phase average of in the kth phase
factors for species A and B (6.2.25a) = k,avg
c ; D; s volume fraction of solids in cake; k k intrinsic phase average of in the kth
volume fraction of continuous, phase (6.2.25b) = k,avgi
dispersed phase (Table 3.1.7), (6.4.88);
volume fraction of particles and solids angular velocity, solute permeation
in a suspension parameter (6.3.158b), sign of fixed
j ; m electrical potential of phase j; defined charge in (3.3.30b)
in (7.1.107h)
i ; im ; m ; volume fraction of species i in liquid D; AB quantity in diffusion coefficient
p ; p phase; volume fraction of species i in expression (3.1.91b)
membrane; polymer volume fraction
in membrane; voltage drop over a
Superscripts
cell pair in electrodialysis; volume
fraction of polymer in liquid a; b; Br activity based; bottom/stripping
phase section of a column; Brownian motion
N correction factor for nonequimolar d, drag drag
counterdiffusion (3.1.136b) eff; eph; ext effective; electrophoretic; external
0 arbitrary value of centrifugal potential ft; G between the feed stream and the tails
at r = 0 (3.1.6d) stream; gas phase
w volume fraction of water in resin, hf between the heads stream and the feed
particle volume fraction at wall stream
max maximum enhancement due to an ht between the heads stream and the tails
instantaneous reaction, maximum stream
particle volume fraction i; iner; int species i; inertial; internal
Notation xxix

L liquid phase in; ion at inlet; ionic species


m magnetic, mobility, based on molal j region/phase j, where j = 1, 2, E, f, g, l,
0
quantities, maximum number m, o, p, R, s, t, v, w, T,
N related to NernstPlank equation k species k
0
(3.1.106) l, liq; loc; ; ; L liquid phase; local; adhering liquid
0, o location z = 0, standard state, infinite phase on crystal; low; at end of
dilution, original quantity separator of length L
oo, ov (7.1.101c), overall m; mc; max; min membrane phase; micellar; maximum;
p pore, pure species minimum
P mole fraction based in Henrys law M mixture, molecular weight based,
r quantity in a system with recycle or metallic species
reflux, for the case of a chemical ME, MR Murphree based on extract phase,
reaction raffinate phase
s related to solids only in solidfluid n, (n 1), (n 1) stage number n, (n 1) and (n 1),
separation, at saturation, scrubbing respectively
cascade N; nu metallic species; nucleation
S solid phase o; obs; og; ol; out organic phase; observed; overall gas
t,T total value, top/enriching section of a phase based; overall liquid phase
column, thermal diffusion based; at outlet
v vapor liquid p; pd; P permeate side, product side, particle;
x, mole fraction based, location z = or m dominant particle; planar interface
infinite dilution condition r; r p1 , r p2 ; R radial direction; related to particles of
0
equimolar counterdiffusion case, first size r1 or r2; ion exchange resin phase,
derivative, feed side of membrane resistive, raffinate phase
00
permeate side of membrane s solvent, surface integration step in
nondimensional quantity crystallization, surface adsorption site,
* hypothetical quantity salt/eluent counterion, stripping
section, stage number
s, s; se forward and reverse surface reaction;
Subscripts
seed crystal
atm; b atmospheric; bulk phase value, bottom S location S, stationary phase
product stream from a column, t; tOE; tOR total, top product stream in a column;
backward reaction transfer units (overall) based on extract
A,B,C species A, B and C phase; transfer units (overall) based on
c based on molar concentration, raffinate phase
continuous phase, cake, critical quantity tj total quantity in jth stream, j = f for a
d diffusive, dialysate phase, dissociation, single feed stream, j = f1,f2 for two feed
distillate streams entering separator, j = 1,2 for
D drag related, Henrys law species product streams rich in species 1 and
related, dispersed phase 2, respectively
e enriching section true; T true value; thermal diffusive, total
eff; ex; ext effective value; exit location; external v;w vapor phase, volumetric; aqueous
f; fr feed side, feed based; during formation phase, water
of drop, size based x liquid mole fraction based, coordinate
g; gr gas phase; growth based direction
H; hex high; heat exchanger y gas phase mole fraction based,
i species i, where i = 1, 2, 3, A, B, M, s, coordinate direction
phase interface, ith module/tube/stage Y as in species AY
ij ith species in jth stream, j = f for a z coordinate direction
single feed stream, j = f1, f2 for two feed 1, 2 species 1 and species 2, phase 1 and
streams entering separator, j = 1,2 for phase 2
product streams rich in species 1 and 3, 4 species 3 and species 4, phases 3 and 4
species 2, respectively , phase and phase
imp; iner impeller; inertial , s surface phase
xxx Notation

Abbreviations and acronyms HFCLM hollow fiber contained liquid


membrane
angstrom HGH human growth hormone
AEM anion exchange membrane HGMS high-gradient magnetic separation
atm atmosphere HK heavy key
avg average HPCE high-performance capillary
AVLIS atomic vapor laser isotope separation electrophoresis
BET BrunauerEmmetTeller HPLC high-performance liquid
bar 105 pascal chromatography
barrer unit for permeability coefficient of HPTFF high-performance tangential-flow
gases through membrane, filtration
IEF isoelectric focusing
cm3 STP-cm
1barrer 1010 ILM immobilized liquid membrane
cm2 -s-cmHg
IMAC immobilized metal affinity
Btu British thermal unit chromatography
CAC continuous annular chromatograph ITM ion transport membrane
CACE counteracting chromatographic ITP isotachophoresis
electrophoresis LDF linear driving force approximation
CCEP countercurrent electrophoresis LK light key
CD(s) cyclodextrin(s) LLC liquidliquid chromatography
CDI capacitive deionization LM logarithmic mean
CE capillary electrophoresis LRV log reduction value
CEC capillary electrochromatography LSC liquidsolid adsorption
CEDI continuous electrodeionization chromatography
CEM cation exchange membrane LTU length of transfer unit
CFD computational fluid dynamics mAB monoclonal antibody
CFE continuous free-flow electrophoresis MBE moving boundary electrophoresis
CGC countercurrent gas centrifuge ME multiple effect
CHO Chinese hamster ovary MEKC micellar electrokinetic
CHOPs Chinese hamster ovary cell proteins chromatography
cm Hg pressure indicated in the height of a MEUF micellar enhanced ultrafiltration
column of mercury MSA mass-separating agent
CMC critical micelle concentration MSC molecular sieve carbon
CMS carbon molecular sieve MSF multistage flash
CSC continuous-surface chromatography MSMPR mixed suspension, mixed product
CSTR continuous stirred tank reactor removal
CSTS continuous stirred tank separator MTZ mass-transfer zone
CV control volume NEA nitrogen-enriched air
CZE capillary zone electrophoresis OEA oxygen-enriched air
ED electrodialysis PAC powdered activated carbon
ELM emulsion liquid membrane PBE population balance equation
EOF electroosmotic flow PSA pressure-swing adsorption
ESA energy-separating agent psia pound force per square inch absolute
FACS fluorescence-activated cell sorting psig pound force per square inch gauge
FFE free-flow electrophoresis RO reverse osmosis
FFF field-flow fractionation SCF supercritical fluid
FFM free-flow magnetophoresis SEC size exclusion chromatography
GAC granular activated carbon SLM supported liquid membrane
GLC gasliquid chromatography SMA steric mass action
GPC gel permeation chromatography SMB simulated moving bed
HDPs high-density particles SPs structured packings
HETP height of an equivalent theoretical TDS total dissolved solids
plate TFF tangential-flow filtration
TOC total organic carbon
Notation xxxi

TSA thermal-swing adsorption VOCs volatile organic compounds


UF ultrafiltration WFI water for injection
UPW ultrapure water WPU purified water
VCR volume concentration ratio ZE zone electrophoresis
VLE vaporliquid equilibrium
Introduction to the book

The basic objective of this chapter is to describe the organ- distillation, absorption and extraction, with some attention
ization of this book vis--vis separations from a chemical to adsorption/ion exchange/chromatography. The second
engineering perspective. Separation, sometimes identified wave expanded the treatment of adsorption/ion exchange/
as concentration, enrichment or purification, is employed chromatography and incorporated an introduction to
widely in large industrial-scale as well as small laboratory- membrane processes. The overwhelming emphasis in
scale processes. Here we refer primarily to physical separ- these books was on chemical separations. Simultaneously,
ation methods. However, chemical reactions, especially a series of textbooks emerged focusing on bioseparations.
reversible ones, can enhance separation and have there- In these textbooks, the treatment of particle based separ-
fore received significant attention in this book. Further, we ations appears briefly under mechanical separations, or
have considered not only separation of mixtures of mol- under the equilibrium based process of crystallization, or
ecules, but also mixtures of particles and macromolecules. as special operations under bioseparations.
The number of different separation processes, methods Fundamental principles that facilitate understanding of
and techniques is very large. Further new techniques or a variety of different separations have, however, been
variations of older techniques keep appearing in industries, emerging in the literature for quite some time. It is useful
old and new. The potential for the emergence of new tech- to structure the learning of separation around these basic
niques is very high. Therefore, the approach taken in this principles. Forces present in the separation system act on
book is focused on understanding the basic concepts of molecules, macromolecules or particles and make them
separation. Such an approach is expected not only to help migrate at different velocities and sometimes in different
develop a better understanding of common separation pro- directions. When such velocities/forces interact with the
cesses, but also to lay the foundation for deciphering bulk velocity of the individual phase(s)/region(s) present
emerging separation processes/techniques. The level of in the separation system, molecular species or particles
treatment of an individual separation process is generally follow different trajectories or concentrate in different
elementary. Traditional equilibrium based separation pro- phase(s)/region(s), leading to separation. This book will
cesses have received considerable but not overwhelming systematically develop this overall framework of a few
attention. Many other emerging processes, as well as estab- important configurations of bulk flow direction vis--vis
lished processes dealing with particles and external forces, the direction of the force(s) for open separators. Chemical
are not usually taught to chemical engineering students; thermodynamics provides the local boundaries/limits in
these are integral parts of this book. To facilitate the analysis such configurations for chemical separations. The individ-
of processes over such a broad canvas, a somewhat gener- ual separation processes/techniques will then be illustrated
alized structure has been provided. This includes a core set in each such configuration of bulk flow direction vs. force
of equations of change for species concentration, particle direction in three categories of processes: phase equilib-
population and particle trajectory. These equations are rium driven; external force driven; membrane processes.
expected to be quite useful in general; however, separation These three basic categories of separation processes
systems are quite often very complicated, thereby limiting rely on three different types of separation phenomena.
the direct utilization of such equations. These different types of separation phenomena achieve
Separation and purification are two core activities of different extents of separation when coupled with particu-
chemical engineers. The first wave of textbooks on separ- lar configurations of bulk flow vs. force pattern. The
ation/mass transfer (until the early 1980s) concentrated on description of each process/technique generally includes
2 Introduction to the book

its conventional treatment and often elementary process/ force generating different migration/terminal velocities and
equipment design considerations. fluxes for chemical species and particles. Integrated flux
This illustrative framework is preceded by a few chap- expressions for molecular diffusion and convection for
ters that provide the basic tools for achieving this goal. In single-phase systems, mass-transfer coefficients and empir-
earlier literature, a broad category of separation techniques ical correlations for mass-transfer coefficients are intro-
is identified as a mechanical separation process. These duced. Chapter 3 (Section 3.2) further points out the role of
techniques are invariably restricted to the separation of the spatial profile of the potential attributable to the force in
particles in a fluid or drops in another fluid subjected developing a multicomponent separation capability. The cri-
generally to an external force. In this book, particle separ- teria for achieving equilibrium between different phases and
ations have been studied along with chemical separations regions in the separation system with or without an external
when a particular external force is considered. Therefore force and various types of phase equilibria are discussed in
the structure of this book is somewhat different. The Section 3.3. The presentation of the partitioning of a species
following section provides a brief introduction to each between two phases is at a phenomenological level. The
chapter in the book. molecular basis of this partitioning via intermolecular inter-
actions has not been considered. This is followed by species
flux expressions in interphase transport, including mem-
brane transport (Section 3.4). The notion of an overall
Introduction to chapters
mass-transfer coefficient and its relation to single-phase
What happens to a perfectly mixed binary mixture of two mass-transfer coefficients are introduced here.
species in a closed vessel as separation takes place is intro- Chapter 4 provides a quantitative exposition of how
duced in Chapter 1. How one describes the extent of separ- much separation is achieved at equilibrium in a closed vessel
ation achieved in the closed vessel is illustrated via a few for three broad classes of separation systems: phase equilib-
common separation indices. The separation indices are rium between two phases (Section 4.1); single phase or a
based on the notion of different species-specific regions in particle suspension in an external force field (Section 4.2);
the separation system, and their differing compositions and two regions separated by a membrane (Section 4.3). The
capacities. Double subscript based notation, with the first phase equilibrium systems considered are: gasliquid,
subscript i referring to a component and the second sub- vaporliquid, liquidliquid, liquidsolid, interfacial adsorp-
script j referring to a region/phase/fraction, is introduced tion systems, liquidion exchanger and the supercritical
here. This notation has been used throughout the book as fluidsolid/liquid phase. The external force fields and config-
often as possible. Use of these separation indices is illus- urations studied are: centrifuges, isopycnic sedimentation,
trated for three basic classes of separation systems without isoelectric focusing, gravity (sedimentation, inclined set-
any particles: immiscible phases; membrane-containing tlers), acoustic forces and thermal diffusion. In the case of a
systems; and systems having the same phase throughout membrane based system of dialysis and gas permeation if
the separation system. A description of separation in multi- separation is to be achieved, we come across the need for an
component systems has been included along with the open system. Chapter 5 focuses on the beneficial effects of
notion of a separating agent required for separation. chemical reactions in phase equilibrium and membrane
Chapter 2 presents the description of quantities needed based separation systems. A few common types of reactions,
to quantify separation in open systems with flow(s) in and out such as ionizations, acidbase reactions and different types of
of single-entry and double-entry separators for binary, multi- complexation equilibria, are found to influence strongly the
component and continuous chemical mixtures, as well as a separation achieved across the whole spectrum of separ-
size-distributed particle population. Separation indices ations involving molecules and macromolecules. The phase
useful for describing separation in open systems with or equilibrium systems studied are: gasliquid, vaporliquid,
without recycle or reflux are illustrated for steady state oper- liquidliquid, liquidsolid, surface adsorption and Donnan
ation (Sections 2.2 and 2.3); those for a particle population equilibrium. Reaction based enhancement in the rates of
are provided in Section 2.4. At the end (Section 2.5), indices interphase transport as well as membrane transport has been
for description of separation in time-dependent systems, e.g. illustrated for a variety of systems.
chromatography, have been introduced. Separation is most often implemented in open
The physicochemical basis for separation is the primary systems/devices with bulk flow(s) in and out. The treat-
focus of Chapter 3. Separation happens via species-specific ment of separation achieved in such separators is carried
force driven relative displacement of molecules of one out in Chapters 6, 7 and 8. Chapter 6 begins with the
species in relation to other species into species-specific sources and the nature of bulk flow in separation systems
region in the separation system. Particles of different sizes/ in a multiscale context as well as the feed introduction
properties similarly undergo relative displacements. To mode vis--vis time (Section 6.1). The various equations
develop this perspective, Chapter 3 (Section 3.1) identifies of change for species concentration in a mixture, the equa-
various external forces and chemical potential gradient based tion of motion of a particle in a fluid and the general
Linked footprints of a separation process 3

equation of change for a particle population, including that with a condenser and reboiler, solvent extraction in
in a continuous stirred tank separator, are provided in columns, melt crystallization, adsorption and simulated
Section 6.2. Section 6.3 covers the separation processes/ moving beds, dialysis and electrodialysis, liquid membrane
techniques in which the direction of the bulk flow is paral- separation, gas permeation, gas centrifuge, thermal diffu-
lel to the direction of the force(s). Figure 6.3.1 illustrates sion and mass (sweep) diffusion are studied in Section 8.1.
the widespread use of this flow vs. force configuration for How cocurrent flow of the two phases/flows changes the
three basic classes of separation systems. External force separation achieved is considered vis--vis a few systems
based processes of elutriation, capillary electrophoresis, in Section 8.2. Local multicomponent feed injection in a
centrifugal elutriation, inertial impaction and electrostatic crossflow format in fluidsolid systems leads to the
separation of fine particles are introduced first. Chemical achievement of continuous chromatography. Overall
potential gradient driven processes of flash/vaporization/ crossflow of two phases is exemplified by a crossflow dis-
devolatilization, batch distillation, liquidliquid extraction, tillation plate (Section 8.3).
zone melting, normal freezing and drying are studied next. Although countercurrent multistaged processes of dis-
The membrane based processes covered are: cake filtra- tillation, gas absorption, solvent extraction, etc. have been
tion/microfiltration, ultrafiltration, reverse osmosis, perva- studied in some detail in Chapter 8, the subject of multi-
poration and gas permeation. In the final section (Section staging/cascades is considered briefly in Chapter 9. Ideal
6.4), Chapter 6 considers the continuous stirred tank sep- cascades and constant or variable cross-sectional area are
arator (CSTS) as a special category of bulk flow vs. force introduced, as are cascades of multistage columns for non-
configurations; the separation processes studied are: crys- binary systems. Chapter 10 describes at an elementary
tallization (precipitation), solvent extraction, ultrafiltration level the minimum energy required for separation by dif-
and gas permeation. ferent separation processes. Additional topics discussed in
The nature and extent of separation achieved when the Chapter 10 include the consideration of various concepts
direction of flow of the feed-containing fluid phase is perpen- that reduce the energy required for separation, recovering
dicular to the direction of the force(s) are studied in Chapter the free energy of mixing via a dialytic battery and add-
7. This treatment illustrates the basic separation mechanism itional deliberations for treating dilute solutions in the
clearly, even though the particulars vary widely, as in, for context of bioseparations. In many real-life applications,
example, free-flow electrophoresis, electrostatic precipita- sequences of different separation processes are employed
tors, electrostatic separation of plastic mixtures, laser excita- with and without reaction processes. Chapter 11 illustrates
tion of isotopes and flow cytometry (all of them driven by an such sequences of separation processes for bioseparations,
electrical force field perpendicular to the bulk flow). Figure water treatment, chemical and petrochemical industries
7.0.1 provides this broad perspective across all three classes and hydrometallurgical separations.
of separation processes: phase equilibrium driven, mem- Each of these chapters provides a particular aspect/
brane based, external force driven. This chapter begins perspective of the broad subject of separations. One is
(Section 7.1) with the treatment of fixed-bed adsorption pro- often interested, however, in a particular separation pro-
cesses, pressure-swing adsorption, parametric pumping and cess/technique in all of its aspects, beginning with the
chromatography. Crossflow membrane processes con- basic concept and ending with devices designed to imple-
sidered next (Section 7.2) are: gas permeation, reverse ment the separation. Tables are therefore provided at the
osmosis, ultrafiltration, microfiltration; this has been end of this chapter that identify the essential and important
followed by granular filtration. The external force field based components located in different chapters for a given sep-
processes studied in Section 7.3 involve electrical force (men- aration process. Obviously it is not possible to provide a
tioned earlier), centrifugal force (centrifuges, cyclones), grav- comprehensive treatment of every process, and a few com-
ity (gravity based settlers), magnetic force field (high- monly used separation processes have received much
gradient magnetic separation) and optical force. Field-flow more attention than others. However, the treatment of
fractionation as a special case of a force perpendicular to bulk each such commonly used separation process is at a level
flow interacting with the velocity profile in a novel way has illustrative of the basic principles relevant to the particular
also been treated; a variety of forces may be used. chapter. Furthermore, the treatments are not exhaustive.
Chapter 8 deals with the configuration of bulk flows of Readers interested in greater detail are encouraged to go to
two phases/regions (one of which may be solid) perpen- major texts on such separation processes identified along
dicular to the direction of force(s). The directions of with their treatments.
motion of the two phases may be parallel to each other in
either countercurrent or cocurrent fashion, or they may be
Linked footprints of a separation process/technique
in crossflow. Figures 8.1.18.1.4 illustrate the counter-
current flow vs. force configuration for all three classes of We provide in this section seven tables; they appear at the
separation systems. Conventional countercurrent devices/ end of this chapter Each table has nine columns. The first
processes of gas absorption/stripping, column distillation column identifies the name of a particular separation
4 Introduction to the book

process in a particular row (e.g. Absorption in row 1 of separation achieved in the benzenetoluene system in a
Table 1). The second column focuses on Chapters 1 and 2. closed vessel. Section 1.6 describes multicomponent mix-
Six more columns are identified progressively with each of tures and develops the relations between the compositions
the Chapters 38. The final ninth column covers the much of two phases in equilibrium, a result useful for distillation
smaller Chapters 911. Each row in the tables is dedicated in later chapters.
to a particular separation process. The entry in a box for a Section 2.1 introduces various quantities describing
given row and a given column identifies sections in the flow rates and compositions in an open system; a sieve
chapter where that particular separation process or fund- plate in a distillation column is used as one example,
amental material needed to understand the transport and among others, of a double-entry separator. A flash distill-
thermodynamics for that process has been presented. In ation stage with liquid fraction recycle illustrates recycle in
Chapters 1 and 2 and in Sections 3.1, 3.2, 3.3.13.3.6, 6.1 a single-entry separator (Section 2.2). Section 2.3 for
and 6.2, general features or fundamental relations valid for double-entry separators provides a numerical example of
a variety of separation processes are presented. Therefore, benzenetoluene distillation in a countercurrent column
entries for a given separation process under columns 2, 3 and without a condenser or reboiler. This and other examples
6, specifically Sections 3.1, 3.2, 3.3.13.3.6, 6.1 and 6.2, pro- provide a quantitative background on the separation
viding fundamental information on the description of separ- achieved in a given device without discussing the separ-
ation species/particle transport, thermodynamics relations, ation mechanism. The same strategy of description of sep-
balance/conservation equations and equations of change, aration achieved via reflux to a column is pursued in this
respectively, for species/particles are not tied in general section to demonstrate that a higher reflux ratio leads to
specifically to that separation process; however, any entry higher separation. Sections 2.4.2 and 2.4.3 introduce
will be useful for understanding that separation process. indices to describe continuous chemical mixtures and
Table 1 covers many of the common phase equilibrium multicomponent mixtures vis--vis flash vaporization.
based separation processes. The entries contain a few The introductory Section 3.1.2.5 in Chapter 3 identifies
separation techniques/processes which are not employed the negative chemical potential gradient as the driver of
on a large scale or illustrate important conceptual develop- targeted separation, and the relevant species flux expres-
ments, e.g. cycling zone adsorption, foam fractionation, sion is developed in Section 3.1.3.2 (see Example 3.1.9
parametric pumping. Table 2 includes membrane separ- also). Section 3.1.4 introduces molecular diffusion and
ation processes, where different membrane transport rates convection and basic mass-transfer coefficient based flux
of different species provide the selectivity in open systems. expressions essential to studies of distillation and other
This table also includes membrane contactor based separ- phase equilibrium based separation processes. Section
ation processes, where the basis for separation is the parti- 3.1-5.1 introduces the MaxwellStefan equations forming
tioning equilibrium between two fluid phases contacting the basis of the rate based approach of analyzing distilla-
each other at membrane pore mouths. Tables 3 and 4 tion column operation. After these fundamental transport
identify separation processes driven by centrifugal force considerations (which are also valid for other phase
and electrical force, respectively. Table 5 is devoted to a equilibrium based separation processes), we encounter
few processes driven by magnetic force or gravity. A few Section 3.3.1, where the equality of chemical potential of
separation processes/techniques driven by other forces, a species in all phases at equilibrium is illustrated as the
such as acoustic force, radiation pressure, inertial force thermodynamic basis for phase equilibrium (i.e. iv il).
and thermal gradient driven force, are listed in Table 6. Direct treatment of distillation then begins in Section
Table 7 is devoted to additional separation processes such 3.3.7.1, where Raoults law is introduced. It is followed by
as field-flow fractionation and mass (sweep) diffusion. Section 3.4.1.1, where individual phase based mass-trans-
It is useful now to illustrate how the descriptive treat- fer coefficients are related to an overall mass-transfer
ment of a particular separation process, e.g. distillation, has coefficient based on either the vapor or liquid phase.
been implemented in an evolutionary fashion via the differ- Section 4.1 via Section 4.1.2 formally illustrates vapor
ent chapters as identified in row 7 of Table 1. In Section 1.1, liquid equilibria vis--vis distillation in a closed vessel
Example I of Figure 1.1.2 illustrates the result of heat add- along with bubble-point and dew-point calculations for
ition to an equimolar liquid mixture of benzenetoulene: a multicomponent systems. How vaporliquid equilibrium
benzene-rich vapor phase and a toluene-rich liquid phase. is influenced by chemical reactions in the liquid phase is
Using definitions of compositions etc. introduced in Section treated in Section 5.2.1.2, where two subsections, 5.2.1.2.1
1.3, separation indices such as the separation factor ij and 5.2.1.2.2, deal with reactions influencing vaporliquid
(also the equilibrium ratio Ki) describe the separation equilibria in isotopic systems. We next encounter open
achieved in a closed vessel for the benzenetoluene system systems in Chapter 6. The equations of change for any
and a methanolwater system for various liquid-phase two-phase system (e.g. a vaporliquid system) are pro-
compositions. Section 1.5 illustrates via Example 1.5.1 and vided in Section 6.2.1.1 based on the pseudo-continuum
the values of various separation indices, 12 and , the approach for the dependences of species concentrations
Linked footprints of a separation process 5

on time and the main axial coordinate (i.e. z) direction. covers the minimum energy required for distillation, and
Section 6.3.2.1 starts with the simplest of open systems, a the concepts of net work consumption, multieffect distilla-
flash vaporizer, and illustrates isothermal flash calculations tion and heat pump vis--vis distillation. Chapter 11.3 intro-
under the constraint of phase equilibrium and bulk flow duces very briefly the important role of distillation in the
parallel to (jj) the force direction for multicomponent chemical and petrochemical industries.
systems and continuous chemical mixtures. Batch distilla- If the treatment of distillation in a given section of the
tion without any reflux is then studied as a particular book needs certain building blocks, it is most likely that those
illustration of this flow vs. force configuration for a fixed concepts/methods/building blocks have been introduced in
amount of feed liquid as well as for constant-level batch an earlier chapter or section of the book. Furthermore, in
distillation employed for solvent exchange. Residue curves whichever section distillation appears, it is studied as part of a
are introduced here. specific pattern followed by many other separation processes
Column distillation is the most common form of an based on phase equilibrium. Such patterns have been
open separation system in distillation. Here the two phases emphasized often throughout particular chapters.
have, on an overall basis, bulk motions in parallel flow in A few pointers on phase equilibrium based separation
the countercurrent direction with the forces causing separ- processes are useful. Table 3.3.1 lists possible useful com-
ation being perpendicular () to the directions of bulk binations of two bulk immiscible phases for separation
flows. The general characteristics of such a separation such as gasliquid (vaporliquid included), gassolid,
system are briefly identified in Section 8.1.1, specifically liquidliquid, etc. Quite a few of these combinations form
8.1.1.18.1.1.3. We learn the structural consequences of the basis of existing separation processes. In this book,
this flow vs. force configuration, namely a distillation therefore, each chapter, from Section 3.3 onwards, focus-
column cannot at steady state separate a ternary mixture; ing on a particular aspect of the subject of separation, has
we need two columns for a ternary mixture. Further, the the subject of phase equilibrium driven separation pro-
particular forms of equations of change for the two phases cesses organized along such two immiscible phase com-
are obtained from the more general equations in Section binations. However, all such combinations in practical use
6.2.1.1 (as well as by a control volume analysis). Distillation do not appear in each chapter.
columns with reflux and recycle are studied in detail in The treatment of membrane separation processes in this
various parts of Section 8.1.3. The conventional approach book merits some deliberation. The most commonly used
of assuming ideal equilibrium stages (the stage may have driving force in membrane separation processes is negative
crossflow in an overall countercurrent flow configuration) chemical potential gradient; a few processes also employ
is adopted in the McCabeThiele graphical framework to electrical force. Figure 3.4.5 identifies the variety of feed
study the following: operating lines in both sections of a phasemembrane type combinations with variations due to
column, q-line, total reflux, minimum reflux, partial/total the nature of the permeate phase when negative chemical
reboiler, partial condenser, open steam introduction, potential gradient is imposed across the membrane. Section
Kremser equation and side stream. 3.4.2 illustrates the interphase membrane transport aspects
The deviation from ideal equilibrium stages is studied of many such configurations. The developments in later
next via stage efficiency in Section 8.1.3.4. Vaporliquid chapters follow these feed phasemembrane type permeate
contacting on a plate/tray in a column is considered in phase combinations as often as possible, subject to space
Section 8.1.3.5 vis--vis estimation of column diameter limitations. Electrodialysis as an example of an application of
(with reference to Section 6.3.2.1). Topics such as the rate electrical force appears in Sections 3.4.2 and 8.1.7. Mem-
based approach for modeling distillation and separation of a brane contactors appear with their phase equilibrium pro-
multicomponent mixture in a column are briefly intro- cess counterparts in Sections 8.1.2 and 8.1.4, whereas the
duced, the latter via the Fenske equation, the Underwood basic transport considerations in such membrane devices
equation and the Gilliland correlation. Distillation in a appear much earlier in Sections 3.4.3.1 and 3.4.3.2. A most
packed tower and in a batch vessel with reflux are studied important item in membrane separation processes is that
next. Section 8.2.1 briefly touches on distillation in a cocur- such devices in the absence of external forces achieve separ-
rent two-phase flow device. Section 8.3.2 studies separation ation when operated as an open system Sections 4.3.1 and
in a crossflow distillation plate employing general equations 4.3.3 demonstrate this feature via the processes of dialysis
from Section 6.2.1.1, ultimately yielding the American Insti- and gas permeation.
tute of Chemical Engineers (AIChE) tray efficiency expres- The descriptive treatment of the membrane process of
sion. The total number of worked examples involving reverse osmosis (RO) in the book as identified in Table 2 will
distillation in one form or another in Chapters 18 is 19. be briefly illustrated here. Section 1.1 identifies the basic
Various other aspects of distillation are considered further configuration of RO in Figure 1.1.3. Example 1.5.4 illustrates
in Chapters 911. Chapter 9 (Section 9.2) introduces briefly calculations of separation indices describing separation in
the methodology for multicolumn distillation for separating RO shown in Figure 1.5.1. Sections 2.1 and 2.2 describe
a mixture containing more than two species. Chapter 10 various quantities, as well as the separation indices relevant
6 Introduction to the book

for RO; Example 2.2.1(c) is directly applicable to RO. and continuous free-flow electrophoresis, exploit, among
Sections 3.1.2.5, 3.1.3.2 and 3.1.5.1 provide a general trans- others, electrophoretic transport under the constraints of
port background. Section 3.1.5.2 is directly relevant to an a closed vessel, bulk flow parallel to force and bulk flow
irreversible thermodynamics based solute and solvent perpendicular to force, respectively. Correspondingly,
transport through RO membranes. Section 3.3.7.4 provides in Table 4, isoelectric focusing does not appear in
a membraneliquid equilibrium relation from an osmotic Chapters 68; capillary electrophoresis is absent from
equilibrium point of view. Section 3.4.2.1 formally intro- Chapter 8. However, each such technique benefits from
duces transport rates in RO membranes and flux expres- relevant discussions in earlier chapters, even though
sions, along with issues of concentration polarization in a the technique itself is treated in detail in a later chapter;
feed solution. A closed vessel of Chapter 4 has very limited therefore materials in Chapters 2 and (especially)
relevance for RO (Section 4.3.4). Sections 5.4.1 and 5.4.1.1 Chapter 3 are identified for each of the three techniques.
describe how chemical reactions, such as ionization, in the Capillary electrophoresis appears also in Section 7.1.7.1,
solutions influence separation in RO processes. where it has been coupled with chromatography where
Section 6.3.3.3 studies RO in bulk flow parallel to the the bulk flow is perpendicular to the force.
force configuration and describes various membrane
transport considerations and flux expressions. Practical
RO membranes are employed in devices with bulk feed
Classification of separation processes
flow perpendicular to the force configuration, as illustrated
in Section 7.2.1.2. A simplified solution for a spiral-wound This book has not adopted a comprehensive classification
RO membrane is developed: analytical expressions for the scheme for all separation processes. Readers should go to
water flux as well as for salt rejection are obtained and the references, especially Figure 30 and Table 7 of Lee et al.
illustrated through example problem solving. A total of six (1977a) and Table 1-1 of King (1980), to that end. What has
worked example problems have been provided up to been adopted here is apparent from the titles of Tables 17.
Chapter 7. Chapter 9 (Figure 9.1.5) shows a RO cascade Separation processes are classified into three categories
in a tapered configuration. Section 10.1.2 calculates the based on the three basic types of physiocochemical phe-
minimum energy required in reverse osmosis based desal- nomena: (1) phase equilibrium based separation pro-
ination and compares it with that in evaporation. Section cesses; (2) membrane separation processes; (3) external
11.2 covers the sequence of separation steps in a water force based separation processes. There are a few pro-
treatment process for both desalination and ultrapure cesses where there is an overlap. For example, electro-
water production. The very important role played by RO dialysis is a membrane-separation process driven
in such plants is clearly illustrated. primarily by an external force, the electrical potential gra-
The evolution of separation through different chapters dient; most membrane-separation processes are driven by
due to an external force needs some discussion as well. negative chemical potential gradient. There are a few
Whereas negative chemical potential gradient driven dis- others, e.g. mass diffusion/sweep diffusion, which cannot
tillation is utilized to separate low molecular weight liquids be neatly put into these three categories; they possess
having different volatilities, an external force, such as elec- characteristics of different categories.
trical force arising from a negative gradient of electrical In this framework of three broad categories of separ-
potential, can be used to separate small charged mol- ation processes, further separation development/classi-
ecules, charged macromolecules, charged cells, charged fication comes about due to the nature of the interaction
particles, etc.; the medium may be liquid or gaseous. The between the basic separation phenomena in each category
canvas is large, and the variety of separation processes/ and the directions of bulk flow vis--vis the direction of
techniques driven by electrical force is significant. force(s) responsible for the basic separation mechanism.
Although there is considerable variety also in phase equi- Considerable additional separation development is
librium processes resulting from a variety of two-phase achieved by reflux, recycle, creation of an additional prop-
systems, the separation systems are more often limited to erty gradient in an external force field, mode of feed intro-
smaller molecules. Separation of proteins/macromolecules duction, etc. These aspects have been addressed in the
via chromatography (Section 7.1.5.1) and biphasic/reverse following sections: reflux (Sections 2.3.2, 8.1.1, 8.1.4,
micellar extraction (Sections 4.1.4 and 4.1.8) provide 10.1.4.2, 10.2.2.1); recycle (Sections 2.2.2, 2.4.1, 7.2.1.1,
exceptions; flotation (Section 3.3.8) separates particles with 7.2.4, 8.1.1); development of an additional property gradi-
the helping hand of an external force, gravity, as does a ent in an external force field (Sections 4.2.1.3, 4.2.2.1,
Venturi scrubber (Section 8.2.3) via inertial impaction. 4.2.3.3, 7.1.7); mode of feed introduction (Sections 6.1.9,
Consider the electrophoretic motion of charged mol- 7.1.5, 7.1.6, 8.1.1, 8.2.2.1, 8.2.2.2, 8.3.1).
ecules/macromolecules/proteins in an aqueous solution/ An additional classification approach considers the
buffer subjected to an electrical force. Three separation nature of the mixture to be separated: mixtures of small
techniques, isoelectric focusing, capillary electrophoresis molecules and/or ions in solution or gas phase; mixtures of
Textbooks, handbooks and major references 7

macromolecules in solution; mixtures of particles, Additional comments on using the book


where particles in this book include biological cells
This book has 118 separate numerical examples spread
(Tables 4.2.1, 7.3.1), cell debris, colloidal material and
over Chapters 14 and 69. The numerical examples are
inorganic and organic particles of varying dimensions
not in finer print. Chapter 5 has sometimes employed
(submicron to visible particles, Figure 2.4.1(b)). Of the
numerical calculations to illustrate the effect of chemical
numerous separation techniques involving different types
reactions on separations without formal numerical
of macromolecules, the following have received some
examples. Chapter 10 follows this strategy as well to illus-
attention here: separation of proteins from each other/
trate the amount of energy required for a particular separ-
one another or solvent via isoelectric focusing, etc.
ation. The total number of problems provided at the ends
(Sections 4.2.2.1, 4.2.2.2), ultrafiltration (Sections 6.3.3.2,
of all the chapters is 299. The specific separation process
6.4.2.1, 7.2.1.3), chromatography (Sections 4.1.6, 4.1.9.4,
relevant for the problem is generally obvious from the
7.1.5.1.6, 7.1.5.1.7, 7.1.5.1.8, 7.1.6, 7.1.7), electrophoresis
introductory sentence in the problem. Further, the
(Section 7.3.1.1), field-flow fractionation (Section 7.3.4),
sequence of appearance of a problem on a given separ-
aqueous biphasic extraction (Section 4.1.3) and reverse
ation process reflects/follows the sequences of appearance
micelles (Section 4.1.9); separation of nonbiological
of that separation process in the text.
macromolecules via size exclusion chromatography
Footnotes have been employed occasionally. All refer-
(Section 7.1.5.1.7), flash devolatilization (Section 6.3.2.1),
ences used appear at one location in alphabetical order at
solgel separation (Section 2.4.2); DNA separation via
the end of the book. The symbols and notation employed
isopycnic sedimentation (Section 4.2.1.3).
throughout the book are consistent; any local deviation has
It is useful to provide a list of the basic physical or
been identified. In a few locations, advanced material or
physiochemical properties, each of which could be a basis
additional information has been provided.
for separation; it is also useful to list simultaneously the core
phenomenon exploiting such a physical or physicochemical
property for separation. It is to be noted that this list is not
exhaustive; rather, it contains the more familiar properties. Textbooks, handbooks and major references
Table 8 identifies a variety of these basic properties and lists on separation processes
phenomena employing a particular basic property leading There is an extraordinarily rich literature on separations.
to separation. For each basic property and phenomenon in This book has freely drawn material from this literature
this table, there are three columns corresponding to three consisting of textbooks, monographs or extended chap-
different types of basic separation processes: phase- ters in multiauthor edited volumes apart from numerous
equilibrium-based separation processes; membrane- journal articles. Here we list these books and chapters
separation processes; and external force based separation (but no journal articles) under the following categories:
processes. An entry into these three columns identifies a separations; chemical separations; bioseparations; mem-
separation process or processes where the particular basic brane separations; particle separations; other books.
property is key to separation. References to Tables 17, a Such books and relevant journal articles have been cited
section in the book or a separate reference have been pro- through each section in each chapter. Occasionally some
vided to each entry in these three columns. comments have been attached here to a given reference.
There are some items of interest here. A few basic Books devoted solely to a given separation process/
properties are the basis for separation in two different technique are not, in general, mentioned below. The
types of basic separation processes. For example, conden- following list is given in chronological order. At the
sability of a vapor/gas species is useful for vapor absor- end of each reference, its formal reference has been
ption as well as for membrane gas separation; geometrical identified.
partitioning (or partitioning by other means between a
pore and an external solution) is useful both in adsorp-
Separations
tion/chromatography as well as in the membrane pro-
cesses of dialysis and ultrafiltration, etc. Further, there are (1) Karger, B.L., L.R. Snyder and C. Horvath, An Intro-
many cases where chemical reactions are extraordinarily duction to Separation Science, Wiley, New York
useful for separation; these are not identified here since (1973). Chapter 18 devotes 19 pages to particle
chemical reactions can enhance separation only if the separation; otherwise it covers primarily separations
basic mechanism for separation exists, especially in phase of chemicals and macromolecules. (Karger et al.,
equilibrium based separations. However, there are a few 1973.)
cases where chemical reactions, especially complexations, (2) Lee, H.L., E.N. Lightfoot, J.F.G. Reis and M.D. Waiss-
provide the fundamental basis for separation, as in affinity bluth, The systematic description and development
chromatography, metal extractions and isotope exchange of separation processes, in Recent Developments in
reactions. Separation Science, Vol. III, N.N. Li (ed.), Part A, CRC
8 Introduction to the book

Press, Cleveland, OH (1977), pp.170. An important (8) Humphrey, J.L. and G.E. Keller II, Separation Process
contribution to structuring separations from a mor- Technology, McGraw-Hill, New York (1997). Book
phological perspective with a distinct transport- oriented towards separation technology, useful for
based input. (Lee et al., 1977a.) process design. (Humphrey and Keller, 1997.)
(3) Giddings, J.C., Principles of chemical separations, (9) Seader, J.D. and E.J. Henley, Separation Process
in Treatise on Analytical Chemistry, Part I. Theory Principles, John Wiley, New York (1998). Textbook
and Practice, Vol. 5, P.J. Elving, E. Grushka and I.M. with broad coverage of chemical separation pro-
Kolthoff (eds.), Wiley-Interscience, New York (1982), cesses including adsorption, crystallization and
chap.3. An early and useful contribution toward membrane separations. A second edition was pub-
transport-based understanding of analytical separ- lished in 2006. (Seader and Henley, 1998.)
ations. (Giddings, 1982.) (10) Noble, R.D. and P.A. Terry, Principles of Chemical
(4) Giddings, J.C., Unified Separation Science, John Separations with Environmental Applications, Cam-
Wiley, New York (1991). An important contribution bridge University Press, Cambridge, UK (2004).
to separation science with an emphasis on methods Treats chemical separations in an environmental
used in analytical chemistry, especially chromatog- context. (Noble and Terry, 2004.)
raphy. (Giddings, 1991.) (11) Wankat, P.C., Separation Process Engineering, 2nd
(5) Schweitzer, P.A., Handbook of Separation Techniques edn., Prentice Hall, Upper Saddle River, NJ (2007)
for Chemical Engineers, 3rd edn., McGraw-Hill, New (formerly published as Equilibrium Staged Separ-
York. (Schweitzer, 1997.) ations, Elsevier, New York, 1987). (Wankat, 2007.)
(12) Benitez, J., Principles and Modern Applications of
Mass Transfer Operations, 2nd edn., John Wiley,
Chemical separations
Hoboken, NJ (2009). (Benitez, 2009.)
(1) Benedict, M. and T.H. Pigford, Nuclear Chemical
Engineering, McGraw-Hill, New York (1957). Intro-
duces isotope separations and cascades in a chemical Bioseparations
engineering context for the nuclear industry. The (1) Belter, P.A., E.L. Cussler and W.-S. Hu, Biosepara-
second edition (1981), with added author H.W. Levi, tions: Downstream Processing in Biotechnology,
substantially expands the treatment and coverage. Wiley-Interscience, John Wiley, New York (1988).
(Benedict et al., 1981.) An introduction to bioseparations. (Belter et al.,
(2) Pratt, H.R.C., Countercurrent Separation Processes, 1988.)
Elsevier, Amsterdam (1967). Contains, among others, (2) Garcia, A.A., M.R. Bonen, J. Ramirez-Vick, M. Sadaka
an introduction to cascade analysis for chemical sep- and A. Vuppu, Bioseparation Process Science, Black-
arations and isotope separations. (Pratt, 1967.) well Science, Malden, MA (1999). (Garcia et al., 1999.)
(3) Sherwood, T.K., R.L. Pigford and C.R. Wilke, Mass (3) Ladisch, M.R., Bioseparations Engineering: Principles,
Transfer, McGraw-Hill, New York (1975). (Sherwood Practice and Economics, John Wiley, New York (2001).
et al., 1975.) (Ladisch, 2001.)
(4) King, C.J., Separation Processes, 2nd edn., McGraw- (4) Harrison, R.G., P. Todd, S.R. Rudge and D.P. Petrides,
Hill, New York, (1980). An important textbook which Bioseparations Science and Engineering, Oxford Uni-
analyzes chemical separations in a generalized versity Press, New York (2003). (Harrison et al., 2003.)
framework with considerable emphasis on multi-
stage separation processes. The first edition
appeared in 1970. (King, 1980.) Membrane separations
(5) Treybal, R.E., Mass-transfer Operations, 3rd edn.,
McGraw-Hill, New York (1980). A textbook focusing (1) Hwang, S.T. and K. Kammermeyer, Membranes in
primarily on conventional mass transfer operations. separations, Vol. VII in Techniques of Chemistry,
(Treybal, 1980.) A. Weissberger (ed.), Wiley-Interscience, New York
(6) Hines, A.L. and R.M. Maddox, Mass Transfer: Fun- (1975). Reprinted, Kreiger Publishing, Malabar, FL
damentals and Applications, Prentice-Hall PTR, (1984). (Hwang and Kammermeyer, 1984.)
Upper Saddle River, NJ (1985). (Hines and Maddox, (2) Meares, P. (ed.), Membrane Separation Processes,
1985.) Elsevier Scientific Publishing Co., Amsterdam
(7) Wankat, P.C., Rate-Controlled Separations, Elsevier (1976). (Meares, 1976.)
Applied Science, New York (1990). Textbook providing (3) Belfort, G., Synthetic Membrane Processes, Academic
an extensive treatment of adsorption, chromatography, Press, New York (1984). (Belfort, 1984.)
crystallization, ion exchange and membrane separ- (4) Ho, W.S.W. and N.N. Li, Membrane processes, in
ations. (Wankat, 1990.) Perry's Chemical Engineers' Handbook, R.H. Perry
Textbooks, handbooks and major references 9

and D.W. Green (eds.), 6th edn., McGraw-Hill, New Cliffs, NJ (1988). A useful book written from a funda-
York, (1984), pp. 17.1417.35. (Ho and Li, 1984a.) mental perspective. (Flagan and Seinfeld, 1988.)
(5) Rautenbach, R. and R. Albrecht, Membrane Processes, (6) Randolph, A.D. and M.A. Larson, Theory of Particulate
John Wiley, New York (1989). (Rautenbach and Processes: Analysis and Techniques of Continuous
Albrecht, 1989.) Crystallization, 2nd edn., Academic Press, New York
(6) Mulder, M., Basic Principles of Membrane Technol- (1988). (Randolph and Larson, 1988.)
ogy, 2nd edn., Kluwer Academic Publishers, Dor- (7) Soo, S.L., Particulates and Continuum: Multiphase Fluid
drecht (1991); a second edition followed in 1996. Dynamics, Hemisphere Publishing, New York (1989).
(Mulder, 1991.) (Soo, 1989.)
(7) Ho, W.S.W. and K.K. Sirkar (eds.), Membrane Hand- (8) Tien, C., Granular Filtration of Aerosols and Hydro-
book, Van Nostrand Reinhold (1992). Reprinted, sols, Butterworths, Boston, MA (1989). (Tien, 1989.)
Kluwer Academic Publishers, Boston (2001). (Ho and
Sirkar, 2001.) Other books
(8) Noble, R.D. and S.A. Stern (eds.), Membrane Separ-
(1) Bird, R.B., W.E. Stewart and E.N. Lightfoot, Transport
ations Technology: Principles and Applications,
Phenomena, John Wiley, New York (1960); 2nd edn.,
Elsevier, Amsterdam (1995). (Noble and Stern, 1995.)
John Wiley, New York (2002). A key book for trans-
(9) Baker, R.W., Membrane Technology and Applications,
port phenomena. Bird et al. (2002.)
2nd edn., John Wiley, Hoboken, NJ (2004). (Baker,
(2) Foust, A.S., L.A. Wenzel, C.W. Clump, L. Maus and L.
2004.)
B. Anderson, Principles of Unit Operations, John
Wiley, New York (1960). (Foust et al., 1960.)
(3) McCabe, W.L. and J.C. Smith, Unit Operations of Chem-
Particle separations
ical Engineering, 3rd edn., McGraw-Hill, New York
(1) Wark, K. and D.F. Warner, Air Pollution, Its Origin (1976); 5th edn., (1993), with additional author
and Control, IEP Dun-Donnelley, Harper & Row, P. Harriott. (McCabe et al., 1993.)
New York (1976). (Wark and Warner, 1976.) (4) Perry, R.H. and D.W. Green (eds.), Perry's Chemical
(2) Friedlander, S.K., Smoke, Dust and Haze: Fundamen- Engineers' Handbook, 6th edn., McGraw-Hill, New
tals of Aerosol Behavior, John Wiley, New York (1977). York (1984). (Perry and Green, 1984.)
(3) Svarovsky, L. (ed.), Solid-Liquid Separation, Butter- (5) Cussler, E.L., Diffusion: Mass Transfer in Fluid
worths, London (1977). (Svarovsky, 1977.) Systems, 2nd edn., Cambridge University Press, Cam-
(4) Svarovsky, L., Solid-Gas Separation, Elsevier Scientific bridge, UK (1997); 3rd edn. (2009).
Publishing, Amsterdam (1981). (Svarovsky, 1981.) (6) Geankoplis, C.J., Transport Processes and Separation
(5) Flagan, R.C. and J.H Seinfeld, Fundamentals of Air Process Principles, 4th edn., Prentice-Hall PTR, Upper
Pollution Engineering, Prentice Hall, Englewood Saddle River, NJ (2003). (Geankoplis, 2003.)
Table 1. Relevant sections for each phase equilibrium based separation process

Chapter 6
Chapters 1 & 2 Chapter 4 Chapter 5 Bulk flow jj to Chapter 7 Chapter 8 Chapters 9,
Describe Chapter 3 Separation in a Effect of force(s) and Bulk flow Bulk flow of two 10 & 11
separation Basis for separation closed vessel chemical reaction CSTS to force(s) phases to force(s) Other aspects

Absorption (and stripping) 1.11.4, 2.1, 2.3 3.3.7.1,a 3.4.1.1, 3.4.3.1 4.1, 4.1.1.1 5.1, 5.2.1.1, 5.3.1 6.2.1.1 7.1.2.1 8.1.1, 8.1.1.1
8.1.1.3, 8.1.2,
8.2.1.1, 8.2.2.2
Adsorption (& simulated 1.11.4, 2.1, 2.5 3.1.3.2.3, 3.1.3.2.4,b 3.3.5, 4.1, 4.1.5 5.2.3.2 6.2.1.1 7.1.17.1.7.1 8.1.1, 8.1.1.1 10.1.4.4
moving beds) 3.3.7.4, 3.3.7.6, 8.1.1.3, 8.1.6
3.4.1.4
Chromatographyc 1.11.4, 2.1, 2.5 3.1.3.2.3, 3.1.3.2.4, 3.2.1, 4.1.3, 4.1.5 5.2.3.2 6.2.1.1 7.1.1.1, 7.1.5 8.2.2, 8.3.1 11.1
3.2.2, 3.3.7.4, 3.3.7.6, 4.1.8, 7.1.7.1
3.4.1.4, 3.4.1.5 4.1.9.1
Crystallizationd 1.11.4, 2.1, 3.3.1, 3.3.7.5, 3.4.1.3 4.1.4, 4.1.9.1 5.2.4 6.2.3, 6.4.1.1 8.1.1, 8.1.1.1 9.1.2.2, 9.1.2.3
2.4.1 8.1.1.3, 8.1.5
Cycling zonee adsorption 7.1.4.4
Devolatilization 4.1, 4.1.2 6.3.2.1
Distillation 1.11.7, 3.1.5.1,a 3.3.7.1, 3.4.1.1, 4.1, 4.1.2 5.2.1.2 6.2.1.1, 8.1.1, 8.1.1.1 10.1.1, 10.1.3,f
2.12.3, 3.4.3.1 6.3.2.1 8.1.1.3, 8.1.3, 10.1.4.2,
2.4.2, 2.4.3 8.2.1, 8.3.2 10.2.1.1,
10.2.1.2,
10.2.1.3,
10.2.2
Drying (and freeze-drying) 1.11.4 3.1.4, 3.3.7.5 6.1.4, 6.3.2.4 11.1
Evaporation 1.11.4, 2.2, 10.1.1, 10.2.1
Ex. 2.2.2
Extraction 1.11.6, 2.1, 2.3 3.2.2, 3.3.7.2, 3.3.7.9, 4.1.3, 4.1.7, 4.1.8 5.2.2, 5.3.2 6.2.1.1, 8.1.1, 8.1.1.1 10.1.4.3, 11.4
3.4.1.2, 3.4.3.2 6.3.2.2, 8.1.1.3, 8.1.4
6.4.1.2
Flotation 3.3.8
Foam fractionation Prob. 2.2.4 3.3.5, 3.3.7.6 4.1.5 5.2.5
Ion exchange 1.11.4, 2.1, 2.5 3.1.3.2, 3.3.7.7, 3.4.1.5, 4.1.6 5.2.3.2 7.1.1.4, 8.1.1, 8.1.6 11.2
3.4.2.5 7.1.5.1.6
Leaching 3.3-7.4 4.1.4
Melt crystallization 1.11.4, 2.1, 3.3.7.5 4.1.4 6.2.1, 6.3.2.3 8.1.1, 8.1.5
2.4.1
Normal freezing 1.11.4, 2.1 3.3.7.5 4.1.4 6.3.2.3
Parametric pumping 1.11.4, 2.1, 2.5 3.3.7.4, 3.3.7.6, 3.4.1.4 4.1.5 6.2.1.1 7.1.4, 7.1.4.1,
(see adsorption) 7.1.4.2,
7.1.4.3
Precipitation (see 3.3.7.5 6.2.3, 6.4.1.1
crystallization)
Pressure-swing adsorption 3.1.3.2.3, 3.1.3.2.4, 3.3.5, 4.1, 4.1.5 6.2.1.1 7.1.1, 7.1.2
(see adsorption) 3.3.7.4, 3.3.7.6,
3.4.1.4
Solvent extraction (see 1.11.6, 2.1, 2.3 3.3.7.2, 3.3.7.9, 3.4.1.2, 4.1.3, 4.1.7, 4.1.8 5.2.2, 5.3.2 6.2.1.1, 8.1.1, 8.1.1.1 10.1.4.3, 11.3,
extraction) 3.4.3.2 6.3.2.2, 8.1.1.3, 8.1.4 11.4
6.4.1.2
Supercritical fluid 1.11.6, 2.1, 2.3 3.3.7.2, 3.3.7.9, 3.4.1.2, 4.1.3, 4.1.7 5.2.2 6.2.1.1, 8.1.1, 8.1.4
extraction 3.4.1.6, 3.4.3.2 6.3.2.2,
6.4.1.2
Zone melting (see 1.11.4, 2.1 3.3.7.5 4.1.4, 4.1.9.1 6.2.1, 6.3.2.3 8.1.5
crystallization)
a
See also Sections 3.1.2.5, 3.1.3.2, 3.1.4, 3.3.1 and Example 3.1.9.
b
See also Sections 3.1.2.5, 3.1.4, 3.2.1, 3.2.2, 3.3.1.
c
See also those listed under Adsorption.
d
See also adductive crystallization, clathration, Sections 4.1.9.1.1, 4.1.9.1.2.
e
See also those listed under Adsorption and Parametric pumping.
f
See also Chapter 9 and Sections 10.2.4, 11.3.
Table 2. Relevant sections for each membrane based separation process

Chapter 8
Chapters 1 & 2 Chapter 4 Chapter 5 Chapter 6 Chapter 7 Bulk flow of Chapters 9,
Describe Chapter 3 Separation in a Effect of chemical Bulk flow jj to Bulk flow to two phases 10 & 11
separation Basis for separation closed vessel reaction force(s) and CSTS force(s) to force(s) Other aspects

Cake filtration 2.1, 2.2, 2.4 3.4.2.3 4.2.1.2 6.1.6, 6.3.1.4, 7.2.1.4, 7.2.1.5 11.1, 11.4
6.3.3.1
Dialysis 1.11.4, 2.1, 2.3 3.1.3.2.3, 3.1.4, 4.3.1 5.4.3 8.1.1, 8.1.7, 11.1
3.3.7.4, 3.4.2.3.1 8.2.4.1
Donnan dialysis 2.1, 2.3 3.1.3, 3.3.7.7, 3.4.2.5 4.3.1, 4.3.2 5.2.6 11.4
Electrodialysis 1.11.4, 2.1, 2.3 3.1.3.2, 3.3.7.7, 4.3.2 8.1.1, 8.1.7 10.2.3, 11.2
3.4.2.5
Emulsion liquid 1.11.4, 2.1, 2.3 3.1.4, 3.3.7.2, 3.4.1.2 4.1.3, 4.1.8 5.2.2, 5.3.2, 5.4.4, 6.2.1.1, 6.3.2.2, 8.1.1, 8.1.8
membrane 5.4.4.15.4.4.3, 6.4.2.2
5.4.4.5
Filtration (see cake 2.1, 2.2, 2.4 3.4.2.3 6.1.6, 6.3.1.4, 7.2.1.4, 7.2.1.5 11.2, 11.4
filtration) 6.3.3.1 7.2.2
Gas permeation 1.11.4, 2.1, 2.2 3.1.3.2, 3.3.1, 3.3.7.3, 4.3.3 5.4.4.1, 5.4.4.5, 6.3.3.5, 6.4.2.2 7.2.1.1 8.1.1, 8.1.9, 10.1.4.1
(membrane gas 3.4.2.2 5.4.4.6 8.2.4.2
separation)
Gaseous diffusion 2.1, 2.2 3.1.3.2.4, 3.4.2.4 Prob. 4.3.9 9.1.1
Hollow fiber contained 1.11.4, 2.12.3 3.1.4, 3.3.7.2, 3.3.7.4, 4.1.3, 4.1.8 5.2.2, 5.3.2, 5.4.4, 6.2.1.1, 6.3.3.5, 8.1.8, 8.1.9
liquid membrane 3.4.3 5.4.4.15.4.4.4 6.4.2.2
Immobilized liquid 1.11.4, 2.12.3 3.1.4, 3.3.7.2, 4.1.3, 4.1.8 5.2.2, 5.3.2, 6.2.1.1, 6.3.3.5, 7.2.1.1 8.1.8, 8.1.9
membrane 3.3.7.4, 3.4.3 5.4.4, 5.4.4.1 6.4.2.2
5.4.4.3, 5.4.4.5
Ion transport membrane 5.4-5.2
Liquid membrane 1.11.4, 2.12.3 3.1.4, 3.3.7.2, 3.3.7.4 4.1.3, 4.1.8 5.2.2, 5.3.2, 5.4.4 6.4.1.2 8.1.8, 8.1.9
processes 5.4.4.15.4.4.3,
5.4.4.5
Membrane contactor 1.11.4, 2.12.3 3.1.2.5, 3.1.3.2.3, 4.1.1.2, 4.1.7 5.2.1, 5.3.1 6.2.1.1 8.1.1.1, 8.1.1.2, 11.2
gas absorption/ 3.1.3.2.4, 3.1.4, 8.1.2.1,
stripping 3.3.7.1, 3.4.1.1, 8.1.2.2,
See reverse 3.4.3.1 8.1.2.2.1
Microfiltration 2.1, 2.4 3.1.2, 3.1.3.1, 3.3.6, 4.2.3.14.2.3.3 6.2.2, 6.2.3, 7.2.1.4, 7.2.1.5 11.1, 11.2
3.4.2.3 6.3.1.4,
6.3.3.1
Pervaporation 1.11.4, 2.1, 2.2 3.1.3.2, 3.3.7.3, 4.1.2, 4.3.3 6.3.3.4
3.3.7.4, 3.4.1.1,
3.4.2.1.1
Reverse osmosisa 1.11.4, 1.5, 1.7, 3.1.2.5, 3.1.3.2, 4.3.4 5.4.1, 5.4.1.1 6.3.3.3 7.2.1.2 9.1.2.1, 10.1.2, 11.2
(Hyperfiltration) 2.1, 2.2 3.1.5.1, 3.1.5.2,
3.3.7.4, 3.4.2.1
Ultrafiltration 1.11.4, 1.5, 2.1, 3.1.3.2, 3.1.3.2.3, 4.3.4 5.4.2, 5.4.2.15.4.2.3 6.3.3.2 7.2.1.3 9.1.1, 9.1.2.1,
2.2, 2.4.2, 3.1.5, 3.3.7.4, 11.1, 11.2
2.5 3.4.2.3

a
Forward osmosis, Prob. 4.3.12.
Table 3. Relevant sections for each centrifugal force driven separation process

Chapters 1 & 2 Chapter 4 Chapter 5 Chapter 6 Chapter 7 Chapter 8 Chapters 9,


Describe Chapter 3 Separation in a Effect of chemical Bulk flow jj to Bulk flow Bulk flow of two 10 & 11
separation Basis for separation closed vessel reaction force(s) and CSTS to force(s) phases to force(s) Other aspects

Centrifugal elutriation 2.1, 2.4, 2.5 3.1.2.2, 3.1.2.7, 6.2.2, 6.3.1.3 11.1
3.1.3.1, 3.1.3.2,
3.2.1
Centrifugal separations 2.1, 2.4, 2.5 3.1.2.2, 3.1.2.7, 4.2.1.1, 4.2.1.2, 6.1.3, 6.1.6, 6.2.2, 7.3.2.1 8.2.3 11.1
(centrifugal filtration) 3.1.3.1, 3.1.3.2, 4.2.1.3 6.3.1.3 7.3.2.4
3.2.1
Cyclone dust separator 2.1, 2.4, 2.5 3.1.2.2, 3.1.2.7, 4.2.1.1, 4.2.1.3 6.2.2 7.3.2.3 8.2.3
3.1.3.1, 3.1.3.2,
3.2.1
Gas centrifuge 2.1, 2.2, 2.3 3.1.2.2, 3.1.2.7, 4.2.1.1 6.1.3, 6.2.1 7.3.2.4 8.1.1, 8.1.10 9.1.1
3.1.3.1, 3.1.3.2,
3.2.1, 3.3.3
Hydroclone 2.1, 2.2, 2.4 3.1.2.2, 3.1.2.7, 4.2.1.2 6.2.1, 6.2.2 7.3.2.3.1
3.1.3.1, 3.1.3.2
Isopycnic sedimentation 1.11.4, 2.1, 3.1.2.2, 3.1.2.7, 4.2.1.3
2.2, 2.4 3.1.3.1, 3.1.3.2,
3.2.1, 3.3.3
Separation nozzle 1.11.4, 2.1 3.1.2.2, 3.1.2.7, 4.2.1.1 7.3.2.4
2.2, 2.4 3.1.3.1, 3.1.3.2,
3.2.1
Table 4. Relevant sections for each electrical force driven separation process

Chapter 6 Chapter 8
Chapter 4 Chapter 5 Bulk flow jj to Chapter 7 Bulk flow of Chapters 9, 10
Chapters 1 & 2 Chapter 3 Separation in Effect of chemical force(s) and Bulk flow two phases & 11
Describe separation Basis for separation a closed vessel reaction CSTS to force(s) to force(s) Other aspects

Capillary electrophoresis 1.11.4, 2.1, 2.4.2, 3.1.2.2, 3.1.2.7, 4.2.2.1, 4.2.2.2 5.2.2.1, 5.2.2.2 6.1.5, 6.2.1 7.1.7, 7.1.7.1 8.2.2.1, 8.2.2.3 11.1
2.4.3, 2.5 3.1.3.2, 3.2.1 6.3.1.2
Continuous free-flow 1.11.4, 2.1, 3.1.2.2, 3.1.2.7, 4.2.2.1, 4.2.2.2 5.2.2.1, 5.2.2.2 6.1.1, 6.1.5, 7.3.1.1
electrophoresisa 2.2, 2.4, 2.5 3.1.3.2, 3.2.1 6.2.1, 6.3.1.2
Corona-discharge reactor 3.1.6.1, 3.1.6.2 7.3.1.2
Dielectrophoresis 3.1.2.2, 3.1.2.7 7.3.1.1.1 11.1
Electrochemical membrane 5.4.4.6 8.1.8
gas separation
Electrochromatography 3.1.2.2, 3.1.2.7, 4.2.2.1 7.1.7, 7.1.7.1 8.1.1, 8.1.6 11.2
3.1.3.2, 3.2.1
Electrodialysis see Table 2
Electrosettler 3.1.2.2, 3.1.2.7 4.2.2, 4.2.3.4.1
Electrostatic particle separator 2.1, 2.2, 2.4 3.1.2.2, 3.1.2.7, 3.1.3.1 6.2.2, 6.3.1.5 7.2.2, 7.3.1.4
Electrostatic precipitator 2.1, 2.2, 2.4 3.1.2.2, 3.1.2.7, 3.1.3.1 6.2.2, 6.3.1.5 7.2.2, 7.3.1.3
Flow cytometry 2.1, 2.2, 2.4 3.1.2.2, 3.1.2.7, 3.1.3.1 6.2.2 7.3.1.5
Ion mobility spectrometry 2.1, 2.2, 2.4 3.1.2.2, 3.1.2.7, 7.3.1.2
3.1.6.1, 3.1.6.2
Isoelectric focusing 1.11.4, 2.1, 2.5 3.1.2.2, 3.1.2.7, 4.2.2.1, 4.2.2.2
3.1.3.2, 3.2.1
Laser isotope separation 2.1, 2.2, 2.4 3.1.2.2 7.3.1.2
3.1.2.7, 3.1.6.1/2
Micellar electro-kinetic 1.11.4, 2.1, 2.4.2, 3.1.2.2, 3.1.2.7, 4.2.2.1, 4.2.2.2 5.2.3.2 6.1.5, 6.2.1, 8.2.2.1
chromatography 2.4.3, 2.5 3.1.3.2, 3.2.1 6.3.1.2
Potential swing adsorption 7.1.3
(Electrosorption,
capacitive deionization)
a
Moving boundary electrophoresis, zone electrophoresis: Section 4.2.2.2.
Table 5. Relevant sections for gravitational or magnetic force driven separation processes

Chapters Chapter 4 Chapter 5 Chapter 6 Chapter 8


1&2 Chapter 3 Separation Effect of Bulk flow jj Chapter 7 Bulk flow of Chapters 9,
Describe Basis for in a closed chemical to force(s) Bulk flow two phases 10 & 11
separation separation vessel reaction and CSTS to force(s) to force(s) Other aspects

Gravity driven 3.1.2.1, 4.1.10, 4.2.3, 6.3.1.1, 7.2.2, 7.3.3,


separation 3.1.2.7, 4.2.3.1, 6.3.1.5 7.3.3.1,
(elutriation, 3.1.3.1, 4.2.3.2 7.3.4
jigging) 3.1.3.2, 4.2.3.4
3.2.3
Magnetic force 3.1.2.4, 7.3.4, 7.3.5,
driven 3.1.2.7, 7.3.5.1,
separation 3.1.3.1, 7.3.5.2,
3.1.3.2 7.3.5.2.1
Magnetophoresis 3.1.2.4, 7.3.5, 7.3.5.1
3.1.2.7,
3.1.3.1,
3.1.3.2
High-gradient 3.1.2.4, 7.3.5, 7.3.5.1,
magnetic 3.1.2.7, 7.3.5.2,
separation 3.1.3.1, 7.3.5.2.1
3.1.3.2

Table 6. Relevant sections for each separation process driven by other forces

Chapters Chapter 5 Chapter 6 Chapter 7 Chapter 8


1&2 Chapter 3 Chapter 4 Effect of Bulk flow jj Bulk flow Bulk flow of Chapters 9,
Describe Basis for Separation in chemical to force(s) to two phases 10 & 11
separation separation a closed vessel reaction and CSTS force(s) to force(s) Other aspects

Acoustic forces/waves 3.1.2.6 4.2.4, 4.2.4.1


Inertial 3.1.2.6.1 6.2.2, 7.2.2 11.2
impactiondepth 6.3.1.4
filtration
Radiation pressure 3.1.2.6, 6.2.2 7.3.6
photophoretic Ex. 3.1.4
separation
Thermal diffusion 1.11.5 3.1.2.6 4.2.5 6.2.1.1 8.1.1, 8.1.11
Thermophoresis 3.1.2.6

Table 7. Relevant sections for additional separation processes

Chapters Chapter 5 Chapter 6 Chapter 8 Chapters 9,


1&2 Chapter 3 Chapter 4 Effect of Bulk flow jj Chapter 7 Bulk flow of 10 & 11
Describe Basis for Separation in a chemical to force(s) Bulk flow two phases Other
separation separation closed vessel reaction and CSTS to force(s) to force(s) aspects

Field-flow 1.11.4, 3.1.2.1 6.1.1 7.3.4


fractionation 2.1, 2.5 3.1.2.7,
3.1.3.1,
3.1.3.2
Mass diffusion 1.11.6, 2.1, 3.1.2.5, 8.1.11
(sweep 2.3, 3.1.3.1,
diffusion) 2.4.3 3.1.3.2,
3.1.4,
3.1.5.1
Table 8. Basic properties and phenomena underlying separation processes for molecules, macromolecules and particles

Phase equilibrium based External force based and other


Basic property Phenomenon causing separation separation processes Membrane separation processes separation processes

Charge-to-mass ratio different charge-to-mass ratios of electrostatic separation of particles


different particles/molecules (Section 7.3.1.4), mass
spectrometry (Karger et al., 1973)
Chelation of metal ions selective chelation solvent extraction of metals (Section liquid membrane processes (Table 2,
5.2.2.4, Table 1) Section 8.1.8)
Condensability of a vapor/gas different condensabilities of vapor absorption, vaporliquid membrane gas permeation (Section
species vapors/gases chromatography (Table 1, Section 4.3.3, Figure 4.3.4, Table 2)
4.1.1.1)
Density of liquid heavier liquid is located further centrifugal separation (Section
from the center for an 7.3.2.1.1)
immiscible liquid mixture in a
centrifugal force field
Dielectric constant particles/cells develop a dipole dieelectrophoresis (Table 4,
moment in a nonuniform Section 7.3.1.1.1)
electrical field
Diffusivity different diffusivities for different ultrafiltration, nanofiltration, dialysis, thermal diffusion (Table 6), mass/
species gas permeation, gaseous diffusion sweep diffusion (Table 7,
(Table 2) Section 7.3.4)
Electrical charge on an attraction toward oppositely electrodialysis (Table 2) element, molecule, particle in gas
element, molecule, charged electrode phase: laser isotope separation,
macromolecule, particle corona-discharge reactor, ion
mobility spectrometry,
electrostatic precipitator, flow
cytometry (Table 4)

different ionic/electrophoretic capillary electrophoresis, continuous


mobilities and attraction free-flow electrophoresis
toward oppositely charged
electrode
membrane surface has opposite nanofiltration separation of divalent
charge anions, ultrafiltration of charged
proteins (Table 2)
Geometrical partitioning of a size exclusion for different gel permeation chromatography, size dialysis, ultrafiltration (Table 2,
solute between pore and a size solutes exclusion chromatography Sections 3.3.7.4, 3.4.2.3,
solution (Table 1, Sections 3.3.7.4, 3.4.2.3.1, 4.3.1, 6.3.3.2)
7.1.5.1.7)
Magnetic permeability or different magnetic permeability/ magnetophoresis, high-gradient
susceptibility susceptibility between magnetic separation (Table 5)
particles and the solution
Mass, density of particles different terminal/settling gravitational settlers, jigging,
velocities in gravitational/ elutriational devices, centrifuges,
centrifugal force field cyclone separators (Tables 3, 5)
Molecular mass of species heavier gas/isotope concentrates gas centrifuge, separation nozzle
further from the center in a (Table 3)
centrifugal force field
Particle compressibility positive and negative values of acoustic force based separation
f-factor in an acoustic force (Table 6)
field
pI, isoelectric point of a proteins having different pI values isoelectric focusing (Table 4)
protein equilibrate at different
locations in a pH gradient
Reversible complexation of selective complexation capability affinity chromatography membrane chromatography
solute/macrosolute with a (Section 7.1.5.1.8) (Section 7.1.5.1.8), ultrafiltration
ligand (Section 5.4.2.1)
Reversible electrostatic preferential interaction of a ion exchange, ion exchange electrodialysis, Donnan dialysis
interaction between counterion in solution with chromatography (Table 1, (Table 2, Sections 3.4.2.5, 4.3.2)
counterions in solution porous charged solid Sections 3.3.7.7, 4.1.6)
and porous charged
solids
Solubility in a solvent of a different solubilities for different solvent extraction, liquidliquid liquid membrane processes
solute present in a solutes chromatography, leaching, (Table 2, Section 8.1.8)
solution or a solid supercritical extraction (Table 1,
Sections 4.1.3, 4.1.7, 3.3.7.5)
Solubility in micellar core or selective partitioning into micellar micellar extraction, reverse micellar micellar-enhanced ultrafiltration micellar electrokinetic
selective complexation core or complexation with extraction (Section 4.1.8) (Section 5.4.2.2) chromatography (Section 8.2.2.1)
with micellar headgroups charged headgroups
Solubility of a gas/vapor in a different solubilities of gases/ absorption, gasliquid chromatography membrane contactor (Table 2)
liquid vapors in a liquid (Table 1, Figure 3.3.3, Section
4.1.1.1)
Solubility of a gas/vapor in a different solubilities of gases/ membrane gas permeation,
polymer vapors pervaporation (Table 2)
Species come out of solution phase change leading to crystal crystallization, zone melting (Table 1,
or melt as a crystal formation Section 3.3.7.5)
Surface active nature of solute excess accumulation of a species at foam fractionation (Table 1,
the interface of phase 1 and Sections 3.3.7.6, 4.1.5, 5.2.5)
phase 2: gasliquid,
liquidliquid
Table 8. (cont.)

Phase equilibrium based External force based and other


Basic property Phenomenon causing separation separation processes Membrane separation processes separation processes

Surface adsorption potential excess accumulation of a species at adsorption, chromatography gas separation by surface diffusion
the interface of phase 1 and (gassolid, liquidsolid) (Section 3.4.2.4), preferential
phase 2: gassolid, (Table 1, Sections 3.3.7.6, 4.1.5) sorption and capillary transport in
liquidsolid reverse osmosis (Table 2)
Vapor pressure of a liquid, different volatilities of bulk liquids distillation, stripping (Table 1, pervaporation (Table 2, Section 6.3.3.4)
volatility of a solute and solutes Sections 4.1.1, 4.1.2) (plus membrane permeability)
Water removed by evaporation of water evaporation, drying (Table 1) membrane distillation (Song et al.,
vaporization from a 2008)
solution or moist solid
Water removed from a solid by sublimation of water freeze-drying (Table 1)
sublimation
1

Description of separation in a
closed system

Separation is a major activity of chemical engineers and solutions, the two different chemical species are intimately
chemists. To separate a mixture of two or more substances, mixed at the molecular level. In a mechanical mixture,
various operations called separation processes are utilized. solid particles, comprise aggregates of molecules of species
Before we understand how a mixture can be separated 2, so the two species are not intimately mixed at the
using a given separation process, we should be able to molecular level. Colloidal solutions and macromolecular
describe the amount of separation obtained in any given solutions provide a spectrum of behavior in between these
operation. This chapter and Chapter 2 therefore deal with two limits. In this book we will primarily treat separation
qualitative and quantitative descriptions of separation. of true solutions, the so-called chemical separations.
Chapter 2 covers open systems; this chapter describes Separation of mechanical mixtures, as well as colloidal
separations in a closed system. and macromolecular solutions, will also receive significant
In Section 1.1, we briefly illustrate the meaning of coverage.
separation between two regions for a system of two com-
ponents in a closed vessel. Section 1.2 extends this to a
multicomponent system. In Section 1.3, various definitions
1.1 Binary separation between two regions
of compositions and concentrations are given for a two-
in a closed vessel
component system. In Section 1.4, we are concerned with
describing the various indices of separation and their inter- Consider a closed vessel A containing a true solution of
relationships for a two-region, two-component separation species 1 and 2, as shown in Figure 1.1.1. Let the solution
system. A number of such indices are compared with composition be uniform throughout the vessel. We will
regard to their capacity to describe separation in Section refer to this condition as the perfectly mixed state. The
1.5 for a binary system. Next, Section 1.6 briefly considers process of separation is now initiated somehow, either by
the definitions of compositions and indices of separation the addition or extraction of some amount of energy with-
for the description of separation in a multicomponent out allowing any mass to enter or leave the system. The
system between two regions in a closed vessel. Finally, end result of any such process is shown by the condition
Section 1.7 briefly describes some terms that are frequently existing in another vessel, B. Depending on the solution
encountered. and the process employed, vessel B can have any number
The separation of a mixture of chemical species 1 and 2 of conditions. If the condition is represented by I, we note
may involve separating either a mechanical mixture or a that vessel B has two regions, each region being occupied
true solution of the two species. Examples of mechanical by molecules of only one kind. Thus species 1 and 2 have
mixtures include water containing fine particles forming a been completely separated from each other. If the material
slurry, air containing fine dust particles, etc. Here water or in any one of the two regions is now removed from vessel
air comprise species 1, whereas the chemical substance B, we will have obtained the corresponding pure species.
present in the form of solid particles comprises species 2. Such a condition is called perfect separation. It is the exact
(Although air is a mixture of various species, we consider it opposite of the perfectly mixed state present in vessel
here as species 1.) Examples of true solutions of species 1 A. Further, the objective of all separation would be to
and 2 are: a solution of sugar in water, a liquid solution of achieve condition I in vessel B whatever the initial condi-
benzene and toluene, a gas mixture of nitrogen and carbon tions. Note that, henceforth, we would like to identify
dioxide, a solid solution of silver in gold, etc. In such true region 1 with species 1 and region 2 with species 2. This
20 Description of separation in a closed system

Condition I

Region 1
Perfect
separation
Region 2

Vessel B
Uniform Condition II Condition IV
mixture
Region 1 Region 1

Region 2 Region 2

Vessel B Vessel B
Vessel A
Condition III
Region 1
Species 1 Imperfect
Region 2 separation
Species 2

Vessel B

Figure 1.1.1. Binary separation between two regions in a closed vessel.

means that, for perfect separation, region 1 will have Figure 1.1.1. We have, as before, two regions, region 1 and
only molecules of species 1, and region 2 will have, only region 2. Region 1 has only pure species 1, whereas region
molecules of species 2. 2 has both species 1 and 2. If we consider region 1 in III
The condition represented by II in Figure 1.1.1 is, and region 1 in I and consider only the compositions of this
however, quite different. Vessel B still has two regions. region, we may mistakenly conclude that perfect separ-
But region 1, instead of having only species 1, has some ation has been achieved in both cases, since region 1 has
molecules of species 2, while region 2, instead of having only pure component 1 under both conditions. What we
only species 2, has some molecules of species 1. If the should also consider is how much of species 1 present in
material in any one of the two regions is removed from the original mixture in vessel A has been recovered in
vessel B, we will obtain a solution of two species instead of region 1 of vessel B in condition III. Since not all of the
an amount of pure species. This condition illustrates species 1 has been recovered and put in region 1, condition
imperfect separation. All real separation processes almost III represents another case of imperfect separation. Simi-
always result in an imperfect separation. Although perfect larly, if region 2 had only species 2 and region 1 had a
separation is therefore never attained, the condition of mixture of species 1 and 2, we again have imperfect
perfect separation is a valuable yardstick for measuring separation.
the amount of separation achieved by an actual separation If the previous paragraphs have led to the belief that a
process. If we refer to the separated material in any one of separation process always starts with a uniform mixture, as
the two regions in condition II as a fraction, then the purity in vessel A of Figure 1.1.1, and ends up with any of the
level or the impurity level of the fraction would provide one three types of conditions I, II or III in vessel B, this is not
such yardstick. By the convention adopted earlier, region the case. One can start with a condition shown in II and
1 will have now species 2 as an impurity, whereas region carry out a separation process, perhaps by altering the
2 will have species 1 as an impurity. physical state (e.g. temperature, pressure, etc.), and achieve
Another estimate of the separative efficiency of the greater separation. For example, consider condition IV in
process is provided by how far removed the compositions vessel B achieved by changing condition II. Obviously,
of the separated regions are from the original mixture in region 1 in condition IV is purer in species 1, and similarly
vessel A. Before we develop and use an index of separation, for species 2 in region 2. Thus a separation operation can
which serves as a criterion of merit to indicate the separa- be carried out not only on a uniform mixture, but also on
tive efficiency, we need to examine the quantities that two mixtures of different compositions existing in two
describe the composition of a binary mixture. However, contiguous regions of a vessel or a container.
knowing the composition only can sometimes be mislead- We have used the notion of a region in a separation
ing. For example, see the condition represented by III in system without much explanation; it is worthwhile to
1.1 Binary separation in a closed vessel 21

Feed Separated
mixture system

Example I Benzene
Region 1 Vapor phase Benzene-
Add toluene rich
heat mixture
(Liquid) Region 2 Liquid phase Toluene-
rich

Example II
Saline Region 1 Ice
Extract
water crystals
heat (Liquid) Region 2 Concentrated
Salt-rich
brine
water

Example III Aniline Hexane-rich


Extract hexane Region 1 liquid layer
heat mixture Aniline-rich
(Liquid) Region 2
liquid layer

Example IV Carbon dioxide Carbon dioxide


Extract phthalic anhydride Region 1 (gas) Temperature
mixture less than
heat Phthalic anhydride 126 C
(Gas) Region 2
crystals

Figure 1.1.2. Binary separation systems with immiscible phases or regions.

explain its physical meaning. A region is a certain volume liquid system and example IV has a vaporsolid system1
in space enclosed within given boundaries such that its (Lowenheim and Moran, 1975).
contents have a composition different from that of an In each of these four examples, the top phase, which is the
adjoining region (or those of adjoining regions if the sep- lighter phase, is usually referred to as the light fraction, while
arated system has more than two regions). The words the bottom phase is called the heavy fraction. Remember that
fraction or phase may also be used to denote a region in for separation systems containing immiscible phases, phase
a separation system. When the separation system contains or fraction is more commonly used than region.
two immiscible phases in a vessel, e.g. vapor (or gas) and On the other hand, consider the separation of saline
liquid, solid and liquid, solid and vapor (or gas) or liquid water by a semipermeable membrane, as shown in
and liquid, the use of phase or fraction is common. example I of Figure 1.1.3. The uniform saline solution can
Consider Figure 1.1.2, where we show examples of be separated by the application of pressure energy on the
several practical separation processes. In example I, a feed saline to yield almost pure water on the low-pressure
liquid mixture containing the same number of moles of side of the membrane and concentrated brine on the high-
benzene and toluene is separated by the addition of heat pressure side. The membrane (or the barrier or partition)
into two immiscible phases a vapor phase richer in separates the two fractions in the two regions. However,
benzene and a liquid phase richer in toluene. The space both of these fractions are miscible with each other, unlike
occupied by vapor is region 1, and the liquid phase is the immiscible phases of Figure 1.1.2. The words fraction
region 2. or region are more appropriate here. Whereas in Figure
In example II of Figure 1.1.2, a solution of salt in water 1.1.2, the two regions were separated only by the individual
can be separated by cooling the solution (extraction of phase boundaries, in this case the membrane actually
heat) to form two distinct phases: ice crystals floating at defines the boundaries of the two regions.
the top (since they are significantly lighter than water or Another example of a membrane providing the bound-
brine) and concentrated brine at the bottom. If the ice aries between two regions is obtained in the separation of
crystals are collected together without any brine sticking compressed air by a silicone membrane (example II of
to the ice crystals (something which is almost impossible to
carry out), then the concentrated brine is phase 2 and the 1
In the manufacture of phthalic anhydride by naphthalene
ice crystals make up phase 1. If ice crystals cannot be
oxidation, the gaseous products are CO2, water vapor and
collected together, then each ice crystal is considered to
phthalic anhydride. For illustration here we use a CO2 and
be part of region 1. Similarly, examples III and IV have two phthalic anhydride system, where phthalic anhydride crystals
regions in a separated system; example III has a liquid are formed from the vapor below 126  C.
22 Description of separation in a closed system

Atmospheric
pressure
Example I
50% N250% H2
Purified Example III
mixture
Region 1 water
Saline Semipermeable
water membrane
Region 2 Concentrated
brine Bulb 1 Bulb 2
Pressure High System
energy pressure at 50F

Thermal
Example II

energy
O2 - rich H2 - rich N2 - rich
Region 1 fraction
Air Silicone region 1 region 2
25 C Region 2 membrane
N2 - rich
fraction
Pressure Bulb 1 Bulb 2
energy 500 F 50 F

Figure 1.1.3. Binary separation systems with both regions miscible with each other.

Figure 1.1.3). Although separation in a closed vessel with 1.2 Multicomponent separation between two
membranes is often time-dependent, the conditions at any regions in a closed vessel
instant of time are sufficient for describing separation.
If the separation system contains more than two species,
There can also be a separated system in which the two
strictly we are dealing with multicomponent separation.
regions are not separated from each other by a membrane
Thus, if we have species 1, 2 and 3 or 1, 2, 3 and 4 or 1,
nor are they immiscible with each other. Such a separation
2,. . ., n, etc., the separated system will have, in general,
system is shown in the form of a two-bulb cell in example
different compositions with respect to each species in the
III of Figure 1.1.3. Originally, the two-bulb cell contained a
two regions. All situations described in Figures 1.1.11.1.3
uniform gas mixture of nitrogen and hydrogen (say, 50%
are also valid here. The only difference is that the uniform
N250% H2, mole percent) at, say, 50  F. If now the bulb 1
solution we start with in Figure 1.1.1 is a multicomponent
(region 1) has its temperature raised to and held at, say,
mixture. Further, whereas in Figure 1.1.1 one region has
500  F, while the temperature of bulb 2 (region 2) is main-
only pure component 1 present and the other region has a
tained at 50  F, we will observe that region 1 has become
binary mixture of species 1 and 2, for multicomponent
richer in the light component, hydrogen, whereas region 2
separation, if region 1 has only pure species 1, region 2 will
has become richer in the heavy component, nitrogen, due
have a mixture of all species (with or without species 1).
to the phenomenon of thermal diffusion.
There is an additional point worth emphasizing. If
What is important here is to recognize that the two
there are only two regions in the closed vessel, it is not
regions having different gas compositions at steady state are
possible to have perfect separation since one requires as
not separated by either a membrane or a phase boundary.2 In
many regions as there are numbers of species present in
this case, the composition changes continuously through the
the original mixture. Therefore, perfect separation for a
capillary from one bulb to the other. However, since the
three-component mixture requires three regions (or four
capillary volume is very small compared to the volume of
regions for a four-component mixture). These require-
either of the bulbs, we may neglect it. So we have a separated
ments are illustrated by various alternative separation con-
system with two regions of different compositions such that
ditions in Figure 1.2.1 for a four-component mixture. For
not only are the materials in both regions completely mis-
example, starting with a uniform mixture in vessel A, one
cible, but also there is no barrier separating the two regions.
can conceive of perfect separation in vessel B with four
Although light fraction (hydrogen-rich fraction) or heavy
regions (conditions I or II). On the other hand, if two
fraction (nitrogen-rich fraction) are used to describe such a
regions are available, only an imperfect separation is pos-
separation, the word region is more descriptive.
sible (conditions III or IV).
A few separated systems with continuously varying com-
The case of multicomponent separation between two
positions are described in Problems 1.4.3 and 1.4.4. These
immiscible phases requires further consideration if each
involve systems without any barriers or immiscible phases.
phase is primarily made up of one species and the
remaining components are present in these phases in
2
In any separated system without two immiscible phases or a small amounts. Consider a three-component system con-
barrier separating the two regions, the composition profile will sisting of, say, water, benzene and picric acid, the latter
be continuously varying. being present in very small amounts. Water and benzene
1.2 Multicomponent separation in a closed vessel 23

Perfect separation

Condition I Condition II
Region 1
Region 1 Region 3
Region 3
Region 4 Region 2 Region 4
Region 2
Vessel B Vessel B

Uniform mixture Condition III

Region 1

Region 2

Vessel A Vessel B Imperfect


separation
Condition IV
Species 1 Region 1
Species 2

Species 3 Region 2

Species 4
Vessel B

Figure 1.2.1. Separation of a four-component mixture in a closed vessel.

Add
Uniform mixture benzene
and stir Region 1
(Benzene phase)

Region 2
(Water phase)
Picric acid in
water
Water Picric acid Benzene

Figure 1.2.2. Multicomponent separation between two immiscible liquid phases.

form two immiscible liquid phases, with the lighter ben- which is a benzene phase and the other one is a water
zene layer above the heavier water layer. It is known that phase. Each phase can be described as if it were a ternary
picric acid at low concentrations will be distributed system since benzene and water are immiscible (in reality,
(Hougen et al., 1954) between the two phases such that very small amounts of water are soluble in benzene and
its concentration in the benzene phase is higher than that vice versa).
in the water phase. Let the original mixture to be separated In fact, one can go a step further by identifying the
be picric acid in water. Benzene is added to this mixture, benzene extract phase as region 1 and the water raffinate
shown in Figure 1.2.2, since it is easier to recover picric phase as region 2 and working only with the concentra-
acid from benzene than from its original mixture in water. tions of the two acids in each region. We then, in effect,
Two fractions have been formed: a benzene based fraction, have a binary system of benzoic acid and picric acid
the extract, and the water based residue, the raffinate. between two regions, 1 (benzene layer), and 2 (water
Although this is a three-component system, the description layer), and one could proceed with the description in the
of each phase can be given as if it were a binary system, i.e. manner of Section 1.1. Thus the description of a four-
a solution of picric acid in water or a solution of picric acid component separation system may be reduced to that of
in benzene. Similarly, suppose two solutes, picric acid and a two-component separation system provided each of the
benzoic acid, are distributed between two phases, one of immiscible phases is made up of essentially one species
24 Description of separation in a closed system

only and the two components, in whose separation we are ij and the mass fraction uij of species i in region j are
interested, are present in small amounts. defined by
A variety of conditions are possible with multicom-
ponent systems in general. When one needs to separ- mij M i
ij C ij M i , 1:3:4
ate particles of one size from particles of all other Vj
sizes present in a suspension, or when particles of ij ij
different sizes are to be separated from one another uij 2
, 1:3:5
X tj
as well as from the medium of suspension, a multi- ij
i1
component separation problem exists. If a macromol-
ecular substance or solution is to be fractionated, the where Mi is the molecular weight of the ith species and tj
nature of the description of the separation problem is is the total mass density of the mixture in region j.
similar. Such problems are handled most conveniently In dealing with mixtures of two isotopes (e.g. H2
by dealing with pseudo binary systems. We will come and D2), one comes across instead of mole fraction the
across such cases as we proceed further in this book, atom fraction aij. The atom fraction aij of the ith isotope of
although binary systems will be encountered much an element in region j is obtained as
more frequently.
maij
aij 2
, 1:3:6
maij
X
1.3 Definitions of composition for a binary system i1
in a closed vessel
where maij  ith isotope
is the number of atoms of the  of the
So far we have described separation qualitatively. In order element in the jth region. The sum ma1 j ma2 j repre-
to describe separation quantitatively, first we have to sents the total number of atoms of the element in the jth
define compositions and then use such compositions to region, with i 1, 2 being the only two isotopic forms
define indices of separation. Only with the help of indices under consideration. For example, in a gas mixture of H2
of separation can one describe separation quantitatively. and D2, if there are 0.186 moles of D2 for 0.189 moles of
Refer to Figure 1.1.1 for a physical background behind H2 and D2, the atom fraction of deuterium in that gas
the definitions of compositions. With both vessels A and mixture is (0.186  2/0.189  2) 0.984.
B closed, we assume that the separation process which For binary isotopic mixtures, the quantity similar
resulted in the condition of vessel B from that of vessel to mole ratio Xij is the abundance ratio X aij , which is
A conserved the total number of moles of each of the given by
species 1 and 2. Assume further that within any region
the composition is uniform everywhere (although this is aij aij
X aij ; i 1, 2; k 1 or 2: 1:3:7
not true for some of the representations actually used in akj 1 aij
Figure 1.1.1). Let mij denote the number of moles of the ith
Thus the abundance ratio represents the number of atoms
species in the jth region in vessel B under any particular
of the desired ith isotope per atom of the other isotope,
condition. For a binary system, i 1, 2, since there are two
where a1 j a2 j 1.
regions only, j has values 1 and 2. The mole fraction xij of
The quantities defined so far describe the composition
species i in region j is defined by
of a given region, phase or fraction with respect to how
mij much of one species is present amongst all the species in
x ij 2
; i 1, 2; j 1, 2: 1:3:1
X the given region, phase or fraction. It is also useful to know
mij
i1
what fraction of the total amount of a species present in the
total separation system is present in a given region. Such a
The mole ratio Xij of species i in region j is given by quantity is the segregation fraction Yij :
mij
X ij ; i 1, 2; k 1 or 2: 1:3:2 mij mij
mkj Y ij 0: 1:3:8a
2
X mi
mij
Note that (xij x2j) 1 but (Xij X2j) need not equal 1. If
j1
the volume of the jth region is Vj, then the molar concen-
tration Cij of species i in region j of vessel B is given by Here m0imoles of the ith species are present in
mij separation vessel B having two regions j 1 and 2.
C ij : 1:3:3 Further, Y i1 Y i2 1. For a two-component system
Vj
with j 1, 2, one may define a segregation matrix
It should be noted that C1j C2j Ctj, the total molar [Yij] as well as the corresponding matrix for mole
density of the mixture in region j. The mass concentration numbers [mij]:
1.4 Indices of separation for binary systems 25

A balance of the ith species only yields


   
 Y Y 12     m11 m12 
Y ij  11 m

; ij : 1:3:8b
Y 21 Y 22   m21 m22   0 2
m1 m02 x if
X
mij :

1:3:11
See Problem 1.3.1 for related representations. j1

Following the segregation fraction Yij for nonisotopic Sometimes the initial binary mixture to be separated is not
systems, one can define the segregation fraction Y aij of an a uniform mixture as in vessel A, but instead has two
isotopic mixture as regions whose compositions are characterized as x if 1 and
x if 2 corresponding to j f1, f2. Note that these regions are
maij maij
Y aij a0 : 1:3:9a such that region f1 usually has more of species 1, whereas
2 mi region f2 has more of species 2. The total and ith compon-
maij
X

j1 ent mass balances in such a case lead to


2 2
All three quantities, the mole fraction xij, the mass fraction m01 m02
X X
mif 1 mif 2 ;
uij and the atom fraction aij, vary between the limits of 0 i1 i1
and 1. When their value is 1, we have pure species i in ! !
2 2
region j. The closer their value is to 1, the greater is the
mif 2 x if 2 m0i :
X X
mif 1 x if 1 1:3:12
purity of the region (fraction or phase) in the ith species. i1 i1
On the other hand, the mole ratio Xij and the abun-
dance ratio X aij vary between 0 and , the latter value
1.4 Indices of separation for binary systems
indicating only pure species i in region j. Thus, the upper
limits of the two sets of quantities (xij, uij, aij and Xij, X aij ) A number of indices of separation may now be developed
have radically different values, although all of these quan- using the quantities defined in Section 1.3. A separation
tities indicate the level of purity of the ith species in the index is needed to indicate how much separation is
given jth region in their own ways. obtained in a given separation process. Such a quantity
The remaining quantities Yij and Y aij have limits similar enables one to determine quickly the separation capabilities
to those of xij, uij and aij, namely 0 and 1. But the upper of any particular process with respect to a given separation
limit has a different meaning. For example, Y22 1 signi- task. Since none of the quantities defined in Section 1.3 are
fies that region 2 has all of species 2 present in the separ- restricted to any particular material or separation process
ation system. But this does not mean that region 2 has pure (except aij, X aij and Y aij , which are restricted to isotopic
species 2, since some species 1 may also be present in mixtures), it is expected that the indices of separation given
region 2. Thus if Y22 1, Y12 need not be zero (compare: below for a two-component system will likewise be suffi-
if x22 1, x12 0). Further, Y11 need not be equal to 1. ciently general in their application. We restrict ourselves here
Condition III of Figure 1.1.1 illustrates such a separation. to describing only the separation that has been achieved in
So Yij provides an estimate of the extent of recovery of vessel B (Figure 1.1.1) between regions 1 and 2. Further, the
species i in region j with respect to the total amount of list of indices given below is not exhaustive.
species i present in the whole separation system. The The simplest separation indices are the distribution
goal of perfect separation is to segregate all of each species ratio (or capacity factor) ki10 , the distribution coefficient i1
in its designated region in a pure form. For a separation and the equilibrium ratio Ki:3
system with i 1, 2 and j 1, 2, perfect separation may be
mi1 C i1 x i1
represented as follows: k i10 ; i1 ; Ki : 1:4:1
mi2 C i2 x i2
 
   Y 11 1 0 
Y ij  ; In general,
0 Y 22 1 
h i   a mi1 C i1 V 1 V1
Y 1 0 k i10

,

i1 1:4:2
Y aij  11

a
 :
Y 22 1 
1:3:9b mi2 C i2 V 2 V2
0
so that if V2 V1, k i10 i1 . Note that the three indices in
It should be noted at this stage that some mass balance
equation (1.4.1) are all defined with the light fraction, i.e.
relations are necessary for relating conditions in vessel A to
fraction 1, on the top. In this convention, then, for species 1
those in vessel B due to the law of conservation of mass. If
to concentrate more in region 1 implies that i1 is greater
the ith species mole fraction in the initial uniform mixture
than 1, since Ci1 > Ci2. This does not mean that k il0 > 1
in vessel A is xif , then, in the absence of chemical reactions,
a balance of the total number of moles of both species
leads to
3
In chemical engineering literature k il0 is rarely used. But i1 is
2
2 X often used, especially in liquid extraction where the system, in
mij m01 m02 :
X
1:3:10 general, is a multicomponent system (see Section 1.2). In
i1 j1 distillation and flash separation processes, Ki is used frequently.
26 Description of separation in a closed system

since for that to be true V1  V2. However, the upper and following two relations for the mole fractions of any species
lower limits for all three indices are, respectively, and 0. between the two regions:
Further, an upper limit of for k il0 and i1 as well as for Ki 12 x 12
does not necessarily imply perfect separation. Similarly, x 11 , 1:4:7b
1 x 12 12  1
the lower limit of zero does not necessarily imply either
x 11
zero or perfect separation. x 12 : 1:4:7c
12  x 11 12  1
Before we present some complex indices of separation,
some more indices apparently similar to those of equation The relationship between x11 and x12 is often presented
(1.4.1) require further consideration. These are as follows. graphically. For example, in systems having a vapor phase
Impurity ratios (Gleuckauf, 1955a): ( j 1) and a liquid phase (example I, Figure 1.1.2), x11 and
x12 are plotted as the ordinate and the abscissa, respect-
m21 m12
1 ; 2 : 1:4:3a ively. Figure 1.4.1 displays this for the systems benzene
m11 m22
toluene and methanolwater. The straight line in each
Purity indices (de Clerk and Cloete, 1971): figure represents x11 x12, a condition where no separ-
2
ation is possible, since x11 x12, x21 x22 and 12 1. The
figure for each system has a dashed line representing the
X
I j log10 j ; j 1, 2; I Ij: 1:4:3b
j1 value of 12 as a function of x12. In the benzenetoluene
system, 12 is a constant; in the other system, 12 varies
The subscript on the impurity ratio j refers to the phase
with composition. The nature of this variation can some-
(fraction or region) under consideration. Since phase 1 is
times be very complex.
supposed to contain primarily species 1 as a valid separ-
By using equation (1.4.1) in definition (1.4.5), we get a
ation goal, species 2 in phase 1 is an impurity. Perfect
few relations between 12 and some other indices:4
separation therefore requires 1 0 and 2 0. For imper-
fect separation j > 0. Note, however, that 1 0 does not  0
12 K 1 =K 2 k 11 0
=k 21

: 1:4:8
imply 2 0 or vice versa.
Since perfect separation requires j 0 for both j 1, 2 Similarly, definitions (1.3.1) and (1.4.3a) substituted into
simultaneously, improved separation in a given problem the definition of 12 yield
implies decreasing values of j. If it is desired that the
12 1=1 2 : 1:4:9
separation index value should increase as separation
improves, one can define an enrichment ratio 0j by utilizing Note that if region 1 or region 2 has pure component 1 or 2,
the definition of j (Boyde, 1971): respectively, 12 , a case of infinite separation factor.
However, as indicated in Figure 1.1.1 (conditions I and III),
0j 1 j : 1:4:4
unless both 1 and 2 are zero simultaneously, perfect
For such a definition, the maximum value of 0j 1 implies separation is not achieved. Thus 12 need not imply
no impurity in region j. One could add subscripts and perfect separation, although at least one region has a pure
superscripts to 0j to indicate whether the separation of component only. An example of such a separation is the
the desirable species 1 is being sought from the undesir- evaporative desalination of brine, where the vapor gener-
able species 2 or vice versa (Boyde, 1971). ated is pure water. Thus, with water as component 1 in the
The most commonly used separation index in chem- vapor region designated as 1, salt is absent in the vapor so
ical engineering is the separation factor 12, where the that 1 0 or x21 0 and 12 . However, if during vapor
subscripts refer to the two species 1 and 2. It is defined by generation some brine droplets are entrained by the rising
vapor, x21 6 0 and 12 6 . (See Example 1.4.2 for another
x 11 x 22
12 : 1:4:5 problem of this type.)
x 21 x 12
Often instead of the separation factor 12, one encoun-
For a binary system of species 1 and 2, since (x1j x2j) 1, ters the enrichment factor5 12 defined by
we can express it also as
x 11  x 12
x 11 1  x 12 X 11 12 12  1 : 1:4:10
12 , 1:4:6 x 12 1  x 11
1  x 11 x 12 X 12

where k 2 in (1.3.2) so that

X 11 12 X 12 : 1:4:7a
4
 0 0
The ratio k 11 has been called the separation quotient

=k 21
The mole ratios of any species between two regions are (Rony, 1968a).
therefore related linearly through the separation factor 12. 5
In separation literature, subscripts 1 and 2 are usually dropped
Similarly, using relations (1.4.6) one can easily obtain the and only is used.
1.4 Indices of separation for binary systems 27

1.0 1.0 11

0.8 5 0.8 9

0.6 4 0.6 7
x11 a12 x11 a12
x11 = x12 x11 = x12
0.4 3 0.4 5

0.2 2 0.2 3

0 0 1
0 0.1 0.3 0.5 0.7 0.9 1 0 0.1 0.3 0.5 0.7 0.9 1
x12 x12

i = 1 = benzene, C6 H6 i = 1 = methanol, CH3 OH


j = 2 = toluene, C7H9 j = 2 = water
j = 1, vapor; j = 2, liquid j = 1, vapor; j = 2, liquid

Figure 1.4.1. Relationships between x11 and x12 and 12 and x12 for benzenetoluene and methanolwater mixtures in a vaporliquid
system.

If 12 is nonzero, 12 is greater than 1, indicating that Substituting definition (1.3.8a) of Yij, we can easily show
separation is possible. If 12  1, the situation corresponds that the following relations are valid:
to close separation. In such a case, the following approxi-
abs jY 11 Y 21 j abs j1 Y 12 Y 21 j
mation is valid (Pratt, 1967a, p. 10):
abs jY 11 Y 22 1j; abs jY 22 Y 12 j
12  12 abs jY 11 Y 22 Y 12 Y 21 j jdetY ij j 1:4:17
n12 12  1     12  1 12 :
2
1:4:11 Therefore one may express in general by

Therefore abs jY ij Y kj j, i, k 1, 2; j 1, 2; i 6 k: 1:4:18

12 e 1 2 exp 12 : 1:4:12 Since the largest value of Yij is 1 and the smallest value is
0, varies between 1 and 0, with the absolute sign taking
Further, for close separation, x 11  x 12 12 x 12 1x 11 , a care of any negative sign if it arises. In terms of perfect and
small quantity so that x11 is quite close to x12. Therefore, imperfect separation, since the maximum value of Yij is 1,
often x12 can be interchanged with x11 in the term 1 implies perfect separation (if Yij is 1 and Ykj is zero; if
12 x 12 1x 11 and vice versa, leading to the following two Ykj is 1 and Yij is zero). A uniform composition between the
relations: two regions is indicated by 0, and there is no separ-
x 11 x 12 12 x 12 1  x 12 ; 1:4:13 ation (see equation (1.4.22)).
The relations between and some of the other indices
x 12 x 11  12 x 11 1  x 11 : 1:4:14 are going to have some use later, so we will obtain them
Some of the more recent indices of separation between two now. By definition (1.4.16),
regions for a two-component system in a closed vessel are m
 11 m21 

abs jY 11 Y 21 j abs  0 0 : 1:4:19
as follows. m1 m2

Separation factor 0 (Sandell, 1968): Further, from equations (1.4.1)


Y 11 m0i k i10 1 m0i
0 : 1:4:15 1 k il0 ; , 1:4:20
Y 22 mi2 k i10 mi1
Extent of separation (Rony, 1968a):
yielding
absjY 11 Y 21 j: 1:4:16 
 0
k 11 0
k 21

 0
 k 11
 0
k 11


abs  0 0  abs 
 
0 0
 ,
1 k 11 1 k 21 1 k 11 12 k 11

One should consider the index , the extent of separation,
in some detail due to its versatility (Rony, 1972). 1:4:21
28 Description of separation in a closed system

where for the final relation we have utilized equation applicable to any impurity to be removed from a solvent,
(1.4.8). An alternative relation between and 12 can be especially in the case of a dilute solution. Note that, in a
obtained by expressing the last of the equalities (1.4.17) as similar manner, one can define 12 and 12 between the
 Y 11 Y 22 initial mixture and the separated region of interest.
 

Y 12 Y 21 abs  1 Y 12 Y 21 abs j12 1j:
Y 12 Y 21 Example 1.4.1 Close separation in thermal diffusion of an
1:4:22 isotopic mixture Isotopic mixtures are difficult to separate
since isotopes are very similar to one another. One method
Similarly, the other separation factor 0 is related to k i10 by sometimes adopted is thermal diffusion. Consider a two-bulb
0 0 cell as shown in example III of Figure 1.1.3, with one bulb at
m11 =m01 k11 1 k 21
 
0 Y 11
: 1:4:23 300  C and the other at 23  C. Initially, both bulbs at the same
Y 22 m22 =m02 0
1 k 11
 
temperature, 23  C, contained an equimolar mixture of C12H4
There is no direct relation between and I or Ij, the purity and C13H4. After the temperature of bulb 1 was raised to 300  C,
indices. But both are different functions of two quantities, the mole fractions of C12H4 in bulbs 1 and 2 were found to be
as can be observed from Problem 1.4.1. 0.5006 and 0.4994, respectively. The separation factor 12 for
C12H4 as species 1, with the hot bulb as region 1, is given by
Although indices of separation have been devised to
indicate the nature of separation between two contiguous x 11 x 22 0:5006  0:5006
12 1:00482;
regions in a separated system, sometimes the nature of the x 21 x 12 0:4994  0:4994
composition difference between one of the separated 12 12 1 0:00482:
regions and the initial mixture is of interest. The following The expressions for some other indices of separation in two-
indices, with their names originating from specialized sep- bulb thermal diffusion are given in Example 1.5.3. Such a
aration processes, are of this type: small separation would render the process of thermal diffu-
sion useless unless a way is found to increase 12 .
decontamination factor,
Example 1.4.2 Description of freeze-concentration of fruit
Df j j C if =C ij ; 1:4:24 juices Large quantities of water are removed from freshly
obtained fruit juices to produce fruit juice concentrates. This
desalination ratio,
reduces, among others, the problem of shipping large
amounts of liquid material from production centers to distri-
Dr C if =C i1 , i 2: 1:4:25
bution centers. This concentration may be achieved by
In the above definitions, Cif is the molar concentration of i cooling the fruit juice to between 3  C and 15  C when
highly pure ice crystals are formed as a suspension in the
in the initial uniform mixture. For the desalination ratio, i
concentrated fruit juice (Figure 1.4.2). Next, the slurry can be
is usually 2, corresponding to salt, which concentrates in
separated in a filtering centrifuge or a filter press or wash
region 2, whereas water concentrates in region 1 (ice in column (Thijssen, 1979) to yield two products: almost pure
example II, Figure 1.1.2; purified water in example I, Figure ice and the fruit juice concentrate. Consider water as com-
1.1.3). For the decontamination factor used with radio- ponent 1 and the active constituents of fruit juice as com-
active or surfactant impurities in solvents (or particles in ponent 2. Let the ice phase be region 1 and the concentrate
air filtration), i is the impurity and j is the region for the be region 2. If the ice crystals formed were absolutely devoid
purified solvent. These definitions need not be restricted of fruit juice active constituents, the separation factor 12
to salt or a radioactive or surfactant impurity. They are would be infinity since x21 would be zero. Actually when ice

Use filtering centrifuges etc.

Vessel A Vessel B

Pure Ice (?) Region 1


Fruit Juice

Concentrate Region 2

Ice crystals
Extract in suspension in
heat concentrated fruit
juice

Figure 1.4.2. Freeze-concentration process for fruit juices.


1.5 Compare indices separation for a closed system 29

z=0 z=L Since the impurity concentration is low, the density of the
rod material may be assumed equal everywhere. Therefore
taking two small and equal volumes at the two ends of the
rod (effectively the same amount of material), we have
Molten zone
z
m11 m22 C 11 C 22
12 :
I m12 m21 C 12 C 21
But the impurity concentration being very low, C11 C12.
Unmelted Therefore
Refrozen zone
zone Direction of C 22 C 2f 1 1 21 exp 10 21
heater movement 12
C 21 C 2f 1 1 21
1 0:5 exp 5
1:9932:
0:5
For 21 0.1,
C2f
1 0:9 exp 1
Impurity 12 6:7:
concentration profile 0:1
C2(z) in refrozen solid
Note here that 21 is a distribution coefficient for species 2
during the melting and refreezing processes. Thus, the lower
0 L
z the distribution coefficient of the impurity between the solid
and the melt, the greater is the separation factor. Further,
Figure 1.4.3. Zone refining of a rod. such a separated system exists without a membrane
(Example 1.5.4) or two immiscible phases.

crystals are formed, some fruit juice solids are frozen inside
the crystals as impurities. In addition, some fruit juice con-
centrate will stick to the surfaces of ice crystals in the separ- 1.5 Comparison of indices of separation for a
ator vessel B, which may be any one of the three types of closed system
equipment mentioned earlier. Therefore x21 6 0 and 12 has
a high value, which is, however, less than infinity. There are many desirable properties of a versatile index of
separation. Some of these are listed below (Rony, 1972):
Example 1.4.3 Zone refining of solid materials Consider a
solid rod of silicon of length L cm containing some impuity (1) It should be dimensionless.
(species 2) at a low concentration level of C2f gmol/cm3. By a (2) It should be usable at all levels of concentrations of any
process called zone refining (Pfann, 1966) (see Section 6.3.2.3), species in the mixture as well as with any separation
a portion of the solid rod can be made substantially more pure process.
by slowly moving a heater of length ( L) along the rod from (3) It should be easily calculable.
one end to the other (Figure 1.4.3). The portion of the rod (4) It should preferably be normalized, varying between
directly concentric with the heater remains molten, while that one (indicating perfect separation) and zero (implying
immediately to the left starts solidifying. This solidifying section
no separation at all), with increasing number indicat-
rejects part of its impurity content into the molten zone. As the
ing improved separation.
heater moves to the right, the molten zone also moves to the
right. The impurity follows this movement so that the left end of (5) It should be unaffected if the component subscript
the refrozen rod is substantially purer than the right end. (i 1, 2) and the region subscript ( j 1, 2) are
Consider the whole rod as the separated system. The impurity interchanged for a binary system.
concentration in the solid after the process is over is given as a (6) It should be sensitive.
function of distance z from the left end by (Pfann, 1966)
One can, no doubt, list additional desirable features. In
C 2 z =C 2f 1 1 21 exp 21 z= :
 
Table 1.5.1, we have indicated some of these features for
During the melting and refreezing process, 21 C 2 z = all indices introduced in Section 1.4. Note that indices k 0i1 ,
C 12 z , C 12 z being the impurity concentration in the molten i1, Ki , 1, 2, I1 and I2 are particularly inadequate when it
rod at a distance z from the left end, which is in contact with the comes to describing separation which involves both
refrozen rod at z, having a concentration of C2(z). Determine species and both regions, since each index either incorpor-
the separation factor for this process for 21 0.5, L 10 if ates both regions or both components. It is also apparent
region 1 is z 0 and region 2 is z L. If 21 0.1, what happens
that, of the remaining indices, e.g. I, 12, 12, 0 and , only
to 12?
the extent of separation, , is normalized. Further, except
Solution For 21 0.5, L 10: for the purity index I and , the maximum values of the
x 11 x 22 m11 m22 others, namely 12, 12 and 0 do not represent perfect
12 :
x 21 x 12 m12 m21 separation.
30 Description of separation in a closed system

Table 1.5.1. Properties of various indices of separationa

Minimum Maximum
value value
Index of Applicable to both Applicable to ( (?) zero ( (?) perfect What value implies
separation Dimensionless components both regions separation) separation) perfect separation?
c
k0il yes no yes 0 (?) (?)
c
i1 yes no yes 0 (?) (?)
c
Ki1 yes no yes 0 (?) (?)
b c
1 yes yes no 0 (?)
b c
2 yes yes no 0 (?)
b b c
I1 yes yes no
b b c
I2 yes yes no
b
I1 yes yes yes (yes)
c
12 yes yes yes 1 (yes) (?)
c
12 yes yes yes 0 (yes) (?)
c
0 yes yes yes 0 (?) (no)
yes yes yes 0 (yes) 1 (yes) 1
a
For a binary system in a closed vessel with two regions only.
b
Depends on further conventions about the limits of contents of each region. See Problem 1.4.1.
c
No specific value is capable of describing perfect separation.

Next we give some examples and calculate the values however, correctly demonstrates that condition 3 repre-
of a selected few indices to obtain an estimate of their sents far better separation than conditions 1 or 2. Thus is
capabilities to describe the quality and amount of separ- a much better index of separation. However, with regard to
ation. Consider first the hypothetical cases of a binary indicating how much more improved the separation in
separation given by the following four conditions.6 condition 4 is with respect to that in condition 3, note that
the relative changes in both 12 and I are much more than
(a) Condition 1: k 011 106 , k 021 104 ;
that of . Therefore, as perfect separation is approached, the
(b) condition 2: k 011 104 , k 021 106 ;
sensitivity of becomes limited compared to either I or 12.
(c) Condition 3: k 011 10, k 021 0:1;
(d) Condition 4: k 011 20, k 021 0:05: Example 1.5.1 Consider the binary system of benzene
(1) and toluene (2) distributed between a liquid and a vapor
In Table 1.5.2, we show the calculated values of Yij for all four region within a closed vessel. Given the benzene mole frac-
conditions of separation. In condition 1, most of species 1 as tion in the liquid and the vapor phase to be 0.780 and 0.90,
well as species 2 are located in region 1, whereas in condition respectively, determine the values of 12 and for the
2 they are almost totally located in region 2. Therefore, these following two cases: (a) 1 gmol of liquid, vapor volume
two conditions represent poor separation. On the other hand, 0.293 liters; (b) 1 gmol of liquid, vapor volume 2.93 liters.
in condition 3, species 1 is located mostly in region 1, The total pressure is 1 atmosphere and the temperature is
whereas species 2 is located mostly in region 2. Thus condi- 85  C. The vapor mixture may be assumed to be ideal.
tion 3 represents much better separation than conditions 1 Solution Since benzene (species 1) is present more in the
and 2. Condition 4 represents even better separation than vapor phase, region 1 is vapor phase. Therefore
condition 3 since the amounts of impurities in both regions
are reduced considerably compared to those in condition 3. x 12 0:78, x 22 0:22, x 11 0:90, x 21 0:10:
We can now judge which one of the four indices of
So
separation 12, 0 , or I accurately reflects the conditions
of separation described above. Note that in Table 1.5.2 the x 11 x 22 0:90  0:22
12 2:54:
two indices 12 and I do not discriminate between the three x 12 x 21 0:78  0:1
conditions of separation 1, 2 and 3. The index 0 incorrectly This is valid for both cases (a) and (b). For the calculation
indicates condition 1 as a much better separation than of , the values of four quantities, m11, m12, m22 and m21,
either of conditions 2 or 3. The extent of separation, , have to be determined.

Case (a): 0.293 liters vapor volume. Assuming that the


ideal gas law holds, the total number of gram moles
6
The first three conditions are based on an example provided by present in the vapor is given by
Rony (1972) in his discussion on the desirable properties of
indices of separation. Note, however, the differences in our 0:293 273
m11 m21  0:01 gmol:
definition,k 0i1 , and Ronys ki2. 22:4 358
1.5 Compare indices separation for a closed system 31

Table 1.5.2. Comparative descriptions of separation by indices 12, 0 , and I

Description of separation Condition 1a Condition 2b Condition 3c Condition 4d

Y11 0.999999 0.0001 0.909 0.9523


Yij Y21 0.9999 0.000001 0.091 0.0477
Y12 0.000001 0.9999 0.091 0.0477
Y22 0.0001 0.999999 0.909 0.9523
12 100 100 100 400
0 104 104 1 1
0.000099 0.000099 0.818 0.9046
I 2 2 1.9991 2.601
a
k011 106 , k021 104 :
b
k011 104 , k 021 106 :
c 0
k11 10, k 021 0:1:
d 0
k11 20, k021 0:05:

Therefore m11 0.01  0.9 0.009 gmol; m21 0.001 Solving the four equations (1.5.1)(1.5.4) for m11, m12, m21
gmol; m12 0.78  1 0.78 gmol; m22 0.22 gmol, so and m22, we get m11 42.09, m12 5.91, m21 8.62 and
m22 43.38 gmol. The extent of separation is given by
m11 m21   0:009 0:001 
   

   0:00688,
m11 m21   42:09 8:62 
   
m11 m12 m21 m22   0:789 0:221  
   0:7122:
m11 m12 m21 m22   48 52 
a case of very poor separation.
The separation factor is given by
Case (b): 2.93 liters vapor volume. Therefore m11 m21
0.10 gmol ) m11 0.1  0.9 0.09 gmol; m21 0.01 gmol; x 11 x 22 0:83  0:88
12 35:8:
m12 0.78 gmol; m22 0.22 gmol, and x 12 x 21 0:12  0:17

 0:09 0:01

 Although the value of 12 indicates an extremely effective
  0:0597: separation, the value of indicates that, since the extent
0:09 0:78 0:01 0:22 
of recoveries is not sufficient, the separation is far from
In both cases, indicates poor separation, although case perfect.
(b) represents somewhat better separation than case (a). How-
ever, 12, based on mole fraction only, is insensitive to such Example 1.5.3 Consider the thermal diffusion separation of
changes. Further, 12 2.54 mistakenly indicates reasonable a gas mixture of H2 and N2 initially present with a hydrogen
separation since it is far away from 12 1.0, corresponding to mole fraction x1f in a two-bulb cell. The volumes of the two
zero separation. bulbs are V1 and V2. Let the uniform temperature of the two-
bulb cell be changed so that the bulb of volume V1 now has a
Example 1.5.2 Aniline and hexane are completely miscible temperature T1 while the other one is at T2 (T1 > T2)
with each other at all temperatures higher than 59.6  C, (Figure 1.1.3, example III). Assuming that this is a case of
below which they separate into two immiscible phases. close separation such that x11 x1f, we obtain
A uniform mixture of aniline and hexane containing 52 mole
x 11 x 22
percent aniline is cooled in a closed vessel from 60  C to 12 1 12 : 1:5:5
x 1f 1 x 1f

42  C. At 42  C, the hexane-rich layer has 83 mole percent
hexane while the heavier and immiscible aniline-rich layer Given that (Pratt, 1967b, p. 404)
has 88 mole percent aniline. Determine the values of 12 and
T1

to indicate the separation that has been achieved compared


12 1 12 12 ln , 1:5:6
to the original feed mixture. T2
Solution Basis: 100 gmol of feed mixture. Species 1 hexane; where 12 is a property of the H2N2 system, obtain a
species 2 aniline; j 1, hexane-rich top layer; j 2, aniline- relation between and 12. How can you increase separ-
rich bottom layer. (Note: aniline is heavier than hexane.) ation in such a system for given T1 and T2? Assume the
Given that m11 ideal gas law to be valid. Neglect the volume of connecting
x 11 0:83 , 1:5:1 capillary.
m11 m21
m22 Solution Since the ideal gas law is valid, with the light
x 22 0:88 1:5:2
m22 m12 component hydrogen (species 1) concentrating more in
From equation (1.3.11), region 1 (higher temperature)

100 x 1f 100  0:48 m11 m12 48; 1:5:3 PV 1 PV 2


m11 x 11 ; m12 x 12 ; 1:5:7
RT 1 RT 2
100 x 2 f 52 m21 m22 : 1:5:4
32 Description of separation in a closed system

PV 2 PV 1 Piston
m22 x 22 ; m21 x 21 , 1:5:8
RT 2 RT 1
where P is the total pressure of the gas mixture everywhere
in the cell. Therefore
m11 m21
 
 
abs  
m11 m12 m21 m22 
x 11 V 1 =T 1


abs 
x 11 V 1 =T 1 x 12 V 2 =T 2
x 21 V 1 =T 1


: 1:5:9
x 21 V 1 =T 1 x 22 V 2 =T 2 
Feed
But brine

m11 m12 Semipermeable


x 1f
m11 m21 m12 m22 membrane
Permeated
V 1 x 11 =T 1 V 2 x 12 =T 2 water
: 1:5:10
V 1 =T 1 V 2 =T 2
Figure 1.5.1. Separation of water from a salt solution under
Similarly,
pressure through a membrane.
 V 1 x 21 =T 1 V 2 x 22 =T 2
1 x 1f

: 1:5:11
V 1 =T 1 V 2 =T 2 (a) After operation for some time, 500 cm3 of pure water has
come to the atmospheric air side through the membrane.
Therefore
Determine the separation factor 12 and the extent of
separation if the density of brine in the concentration
 
V1 V2  x x 
 11 12 
 2  abs  1:5:12 ranges encountered is 1 g/cm3.
 x 1f 1x 1f 

V1 V2
T1 T2 T 1T 2 (b) Suppose some salt also leaks through the membrane to
the permeate side so that the salt concentration in the
on introducing (1.5.10) and (1.5.11) in the expression for solution on the atmospheric pressure side is 0.026 gmol/
obtained by simplifying expression (1.5.9). On rearrange- cm3. If 500 cm3 of water has permeated through the
ment, the relation between and 12 is obtained as (Sirkar, membrane as before, what are the values of 12 and ?
1977)
Solution Region 1, permeated solution or water; region 2,
1 high-pressure brine; component 1, water; component 2, salt.
   12 : 1:5:13
V 1 T2
VV 21 TT 12 2
V 2 T1 Part (a): 12 x 11 x 22 =x 12 x 21 . But x11 1 (pure water),
Suppose T1/T2 10. Let us take two cases: x21 0 (no salt in permeate) and x12, x22 6 0. Therefore
a V 1 =V 2 1; b V 1 =V 2 10: 12 , a case of infinite separation factor.
By definition,
12 12
Case (a) ; m11 m21
 
0:1 10 2 12:1
 
 :
m11 m12 m21 m22 
12
Case (b) 10  12 :
Now m21 0, m11 500=V 1 and m12 500=V 1 , where
   
10 0:1  10 2 4
V 1 is the partial molar volume of water in the brine solution,
Obviously case (b) has much better separation since by assumed to be constant in the range of pressures and con-
providing the hotter bulb with a much larger volume, much centrations (unit, cm3/gmol). Therefore
more hydrogen can now be segregated in the hotter bulb
500  1
 
which provides the light fraction. This is not evident from 12

 0 ,
or 12; both are independent of V1 or V2. 500 500 2
definitely not a case of perfect separation, inspite of an
Example 1.5.4 In the vessel shown in Figure 1.5.1, a mem- infinite separation factor.
brane which allows water to pass through easily and pre-
vents salt from going through has on one side 1000 cm3 of Part (b): m21 0.026  500 13 gmol; m22 (1000 
aqueous solution containing 0.1 gmol of salt per cm3 and 0.1) 13 87 gmol;
atmospheric air on the other side. If the saline water is 
 500 13 

pressurized by means of a piston to a constant but high  0:50:13 0:37,
500 500 100 
enough pressure, water permeates through the membrane
but salt does not. so separation is reduced from case (a).
1.6 Separation indices for multicomponent systems 33

500=V 1 m22 500  87


 
m11 m22 1
12  6:7 A
m12 m21 500=V 1 m21 500  13

10 90
MC
20 80
Thus the change in 12 between cases (a) and (b) is much
more dramatic than that in . 30 70
B
C MB
40 60
From the above examples, it is clear that , the extent of wt % 2 M wt % 1
separation, often describes separation much better than 50 50
12, the separation factor. This is due to the ability of to 60 40
take into account the extent of recovery of a given species in E
70 30
a given region. On the other hand, when one deals with two
F MA
regions with small amounts of impurity, any change in the 80 20
impurity levels are better recognized through the indices 90 R 10
indicating the composition of any region, e.g. 12, j, I, Ij, L
etc. Therefore, no single existing index describes all aspects 2
10 20 30 40 A 50 60 70 80 90
3

of separation efficiently (see Problem 1.5.3). wt % 3

Figure 1.6.1. Compositions of a ternary system in equilateral


1.6 Indices for separation of multicomponent
triangular diagram.
systems between two regions
In Section 1.2, we introduced a brief description of separ- (Figure 1.6.1) representing pure components 1, 2 and 3,
ation for multicomponent systems. Although we learnt respectively. From any point M inside the triangle repre-
there that perfect separation in such a system requires as senting a ternary mixture, draw perpendiculars MA, MB
many regions as there are components, we will restrict and MC to the sides 23, 31 and 12.
ourselves here to separation systems with only two regions The distances MA, MB and MC represent weight per-
in a closed vessel. Thus perfect separation is, in general, centages of species 1, 2 and 3, respectively, in the mixture.
ruled out from our considerations. The sum of the distances MA, MB and MC is equal to the
The familiar quantities used to define the composition altitude of the triangle. Since the perpendicular distance
of a mixture in a region have to be redefined since i 1, 2, from any one of the apexes to the opposing base is the
3,. . ., n for the n-component system: same for an equilateral triangle, this distance represents
mij 100 percent of the mixture. Therefore the weight fractions
x ij n i 1, 2, . . . , n; j 1, 2; 1:6:1a of species 1, 2 and 3 in the mixture are given by
X
mij
i1 MA MB MC
u1 j , u2 j , u3 j , 1:6:1e
AL AL AL
mij
X ij k 6 i; k, i 1, 2, . . . , n; j 1, 2:
mkj where AL is the altitude of the triangle. For easy determin-
1:6:1b ation of these fractions, triangle 123 has percentages
marked on each side. Thus point M in Figure 1.6.1 has
Note weight fractions of 0.50 of 1, 0.30 of 2 and 0.20 of 3.
n
X Two different points inside the triangle represent two
x ij 1 for j 1, 2; 1:6:1c
i1 different mixtures. These two mixtures may or may not be
immiscible. If these two mixtures are brought together, the
n maij ij
composition of the overall mixture (subscript j M) will
X
C ij C tj ; aij n ; uij n : 1:6:1d
i1
maij be represented by the point F on the straight line joining
X X
ij
i1 i1 the two points E (extract) and R (raffinate). The location of
point F is determined by the lever rule:
All other basic definitions of concentrations remain the
same, except for i 1, . . . , n. For multicomponent line EF uiE uiM wR
, 1:6:2
systems, it is worthwhile illustrating the value of aij by line RF uiM uiR wE
way of an example. Consider an isotopic mixture of water where wR and wE are the total weights of mixtures at R and
having H2O, D2O and HDO present in the amounts 0.6, 0.1, E, respectively. This relation follows from a total mass
0.3, respectively (in mole fractions, i.e. if H2O is component balance,
1, then x1f 0.6). The atom fraction of hydrogen is then
[(0.6  2 0.3  1)/2] 0.75. wR wE w M , 1:6:3a
A graphical method of describing compositions is often and an ith-component balance,
adopted for three-component systems. Consider an
equilateral triangle with apexes identified as 1, 2 and 3 wR uiR wE uiE wM uim : 1:6:3b
34 Description of separation in a closed system

With regard to the various indices of separation, the defin- But


itions of k 0i1, i1 and Ki remain unaffected, regardless of n
X
whether i 1, 2 or i 1, 2,. . ., n. The impurity ratio j x i1 1:0,
defined earlier according to de Clerk and Cloete (1971) has i1

to be modified for a multicomponent system to the and therefore


impurity ratio for the ith species in the jth region: n
X
K i x i2 1: 1:6:11b
mij i1
ij j 1, 2; i 1, 2, . . . , n, i 6 j: 1:6:4
mjj
If the separation factor in is defined by (1.6.6) with respect
One could define a purity index corresponding to such a to the heavy species n, then
definition of the impurity ratio as follows: n n
X Ki 1 X
x i2 in x i2 : 1:6:11c
2
X n
X i1
Kn Kn i1
I log10 ij , 1:6:5
j1 i1 But
Ki

following the suggestions of de Clerk and Cloete (1971). The


x i1 K i x i2 K n x i2 ,
separation factor 12 for two species 1 and 2, with species 1 Kn
being lighter than species 2, is usually changed for multi- and therefore
component separations to in (Pratt, 1967b, p. 451), in x i2
x i1 n : 1:6:12
x i1 x n2 X
in , 1:6:6 in x i2
x i2 x n1 i1

such that species n is the heaviest (with the highest The inverse relation is
molecular weight in general) and species 1 is the lightest
x i1 =in
with i 1, 2,. . ., n. Such a definition ensures that in is x i2 n : 1:6:13
X
greater than 1 if there is any separation. Species n is often x i1 =in
referred to as the heavy key component. The corresponding i1

index of enrichment factor is defined as None of the definitions (1.6.4) and (1.6.6)(1.6.9) provide a
in in 1: 1:6:7 single number to indicate the quality of separation, as was
the case for a binary system. If a single number is required
It is not necessary, however, to have the separation factor to describe the separation of a particular ith component
of species i always defined with respect to the heaviest from the rest, one can lump all the other species together
species. One could define the separation factor ij as and treat them as the other component of a binary system.
x i1 x j2 With this rearrangement, all the indices of separation
ij , 1:6:8 defined earlier for binary systems become useful. This is
x i2 x j1
especially true for , the extent of separation, since Rony
where the jth species is heavier than the ith species. This (1972) has shown that the automatic extension of
ensures that ij  1 since, by convention, the ith species jdet Y ij j from a binary system to an n-component
 
concentrates more in region 1 and a species heavier than i system distributed between n regions is not of much use:
will concentrate in region 2. The corresponding enrichment   
  Y 11 Y 12 Y 13 . . . Y 1n  
factor is defined as   
  Y 21 Y 22 Y 23 . . . Y 2n  
det  : 1:6:14
ij ij 1 :   Y 31 Y 32 Y 33 . . . Y 3n  
 
1:6:9

  Y n1 Y n2 Y n3 . . . Y nn  
Note that if the separation factor is defined as
Since a determinant is zero if two rows or columns of the
x j1 x i2 determinant are identical element by element, would be
ji , 1:6:10
x j2 x i1 zero, even though (n 2) components (say) are completely
separated and segregated in their respective regions. (See
where the jth component is heavier than the ith compon-
Problem 1.6.1 for other descriptions of multicomponent
ent, ji will have a value less than 1.
systems.)
As in equation (1.4.7b), the relation between the mole
As pointed out in Section 1.2, for a three- or four-
fractions xi1 and xi2 is of interest for the n-component
component system in which two of the components form
system. Now
two immiscible phases while the other components are
x i1 distributed between these phases in small quantities,
Ki , i 1, 2, . . . , n: 1:6:11a
x i2 indices of separation defined for a two-component system
Problems 35

are quite useful. If we have a three-component system with purification, concentration or enrichment are, however,
species 1 distributing itself between phase 1 (essentially separation processes.
species 2, say) and phase 2 (essentially species 3, say) then When salt is removed from sea water by various desal-
the indices k 0i1 , i and Ki are particularly useful. In fact, ination processes, we have purification of water. Germa-
other indices, e.g. j, Ij, I, 12, , and are not useful at all nium is purified to contain only 1 in 1010 electronically
in such a case since i 1 only. On the other hand, with a active impurities by the zone-refining process (Example
four-component system with species 1 and 2 distributing 1.4.3). Cane juice containing 10 to 12 wt% sugar is concen-
themselves between immiscible phases 1 and 2 (phase 1 trated to about 65 wt% sugar solution by multiple-effect
is, say, essentially species 3 and phase 2 is essentially evaporation (King, 1980). Orange juice solids are concen-
species 4), the indices j, Ij, I, 12, , and regain their trated sometimes by the freeze-concentration process
usefulness. Here, the system is to be treated as if two (Example 1.4.2). The amount of U235 isotope in natural
components 1 and 2 are being separated between two uranium compounds (usually around 0.0075 atom frac-
regions. Therefore perfect separation is possible, with tion) are increased (with respect to the U238 isotope) by
species 1 completely segregated in region 1 and species 2 gaseous diffusion (Pratt, 1967b, p. 349), which becomes an
in region 2. enrichment process.
In a five-component system with two immiscible Chemical separation processes are often described
phases, the above argument would suggest that we are broadly by means of the external agents introduced into
effectively dealing with a three-component system. The the feed mixture to effect separation. In Figures 1.1.2 and
indices of separation (1.6.4)(1.6.7) valid for multicompo- 1.1.3, the external agent is energy addition or energy
nent systems are to be used in such a case. extraction. The form of energy is either heat or pressure
There are systems where there may be thousands of energy. Many other forms of energy addition are also
species. Quantitative indices have been developed to practiced. Such separation processes are called energy-
describe composition and separation in such systems. separating-agent (ESA) processes. On the other hand,
Chapter 2 provides an introduction to this topic. consider Figure 1.2.2 where benzene is added from out-
side to the feed mixture to create a separated system.
Such a process is characterized as a mass-separating-agent
1.7 Some specialized nomenclature
(MSA) process; benzene is the mass-separating agent.
Terms such as concentration, enrichment, purification and Different materials and different phases have been used
separation are commonly used in describing chemical in mass-separating-agent processes. A comprehensive
separation processes. Based on the usage pattern for these characterization of different separation processes in terms
terms, Rony (1972) has suggested that a specified range of of the ESA process or the MSA process is available in King
composition should be associated with each term. For (1980).
example, purification processes will increase the mole frac- It is wrong to assume that no energy is required in
tion of a species being purified, with the mole fraction of separation processes using a mass-separating agent. In the
the species remaining above 0.90 always. In concentration example of Figure 1.2.2, the extract phase containing the
processes, the species mole fraction being increased solute picric acid in benzene has to be subjected to an ESA
remains below 0.90. Enrichment processes, traditionally process to separate benzene from picric acid. Only then is
used for isotopic separations, rarely get the mole fraction picric acid recovered in purer form, and the two initial
of the species above 0.10. All processes achieving species, water and picric acid, are separated.

Problems
1.3.1 One can characterize a binary separation system with two regions in terms of a 2  2 matrix of elements
indicating the number of moles of species i in region j (Rony, 1972). Write the matrix S1 for ideal separation.
(a) Obtain the matrix for the actual separation achieved, S.
(b) Determine the matrix Sf for the initial condition before the separation was started in terms of m0i and Y 0j .
Here Y 0j is that fraction of the total number of moles present in the system which is present in region j
before the separation was initiated at time t 0.
2
1.4.1 (a) If the purity index I I j may be expressed as I log10 A log10 B, show that the extent of separation, ,
X

j1

is given by abs [A B] for a binary system distributed between two regions in a closed vessel. Indicate
the expressions for each of A and B.
(b) Determine the minimum value of I corresponding to maximum impurity in both regions. For this purpose,
recognize that since each region is identified with a species, the composition of any region may not be
36 Description of separation in a closed system

such that the region-specific component i has less than half of m0i in its designated region. While
determining the condition for minimum I with respect to a species, note that you have to assume that
the distribution of the other species is unaffected. Avoid differentiation. (Ans. 0.)
1.4.2 Two enantiomers, species 1 and species 2, are present in a solution. By a process called resolution, two fractions
are obtained. The optical purity of one of the fractions ( j 1) highly purified in species 1 is generally expressed
in terms of an enantiomeric excess
C 11 C 21
ee1 :
C 11 C 21

Relate this quantity to an appropriate index identifying the purity of the fraction in terms of species 1.

1.4.3 If a gas or gas mixture is enclosed within a cylindrical vessel of radius r2 and height h, and the closed vessel is
rotated about its axis at an angular velocity of radian/s, then, due to centrifugal forces, the partial pressure
pi(r) of species i at any radial location at a distance r from the center is given by

M i 2 r 2

pi r pi 0 exp ,
2 RT

where Mi is the molecular weight of species i.


(a) Assuming the ideal gas law to be valid, obtain the following expression for the separation factor 12
between gas species 1 and 2 at radial locations r1 and r2, where r1 < r2 and M1 < M2:

M 2 M 1 2 r 22 r 21
 
x 11 x 12

12 exp ,
x 12 x 21 2RT

where region 1 is located at radius r1, etc. Remember that the total pressure varies with r in such a gas centrifuge.
(b) Consider now the regions 1 and 2 to be thin cylindrical shells of height h and thickness dr at radial
locations r1 and r2, respectively. Obtain the following relation:
2 3 2 3
6 1 7 6 1 7 
6
6
2 2
 7
2 7
6
2 2
 7 12 1:
2 7
M r r M r r
 
r1 r2
6
1 1 2 2 2 1
1 exp 1 exp
4 5 4 5
r2 2RT r1 2RT

1.4.4 For the thermal diffusion separation of solute 2 present in solvent 1, located between two flat plates as shown in
Figure 1.P.1, it is known that the solute will concentrate near the cold plate (region 2) while the solution near
the hot plate at the top gets depleted in solute. With y coordinate values of 0 at the top plate and at the bottom
plate, it is known that the mole fraction distribution of the solute x2(y) is given by the equation

dx 2 T 1 T 2

x 2 1 x 2 :
dy T

Initially, the uniform solute mole fraction in the solution was x2f , where the relation


x 2f 1= x 2 dy
0

is valid due to solute conservation.


(a) Show that the solute mole fraction profile is given by (Powers, 1962, p. 29)

x 2 y 1 0 exp 2 1 , where

A T 1 T 2 =2T , y= and
 


exp f2A 1x 2f g1
  
0 :
1exp f2A x 2f g
 
Problems 37

Hot Plate

y=0

Solution

y=

Cold Plate

Figure 1.P.1. Static thermal diffusion cell with two plates for a solution.

(b) Obtain an expression for the separation factor 12, with y 0 being region 1 and y being region 2.
(Ans. 12 exp(2A).)
(c) Obtain the expression for 12 if this is a case of close separation. (Ans. 12 2A.)

1.5.1 (a) In Example 1.5.3, the index was found to depend on V2 and V1, both of which can be varied to achieve
an increased value of . Thus, if (V2/V1) is considered to be a variable, can be maximized with respect to
(V2/V1). In general, the maximum value of , max, with respect to a variable can be obtained for the
separation of a two-component system between two regions from

abs Y 11 Y 21 0,

whose solution will yield a value of , which, when substituted in , will give max. Consider a two-
component system distributed between two immiscible phases such that 12 is constant. Show that for a
value of the variable k 011 12 1=2 ,
 1=2 1 
 
 12
max abs  1=2 :

 1
12

(b) If the above system is such that 12 ! 0 (i.e. close separation), show that max is given by max 12 =4.
(c) Consider relation (1.5.13) between and 12 for binary thermal diffusion separation. Show that, for
constant values of T1 and T2 for a given two-component system (equation (1.5.6)), max 12 =4 and
that the corresponding value of (V2/V1) (T2/T1).

1.5.2 (a) Consider the separation of a hexaneaniline liquid mixture as described in Example 1.5.2. Obtain the
following relation between , 12 and 11:

C 11 C 21 1

 
V 2V 1 abs 12 1:
m01 m02 11

(b) For the separation of picric and benzoic acids between two immiscible layers of benzene and water, show
that 12 11/21 if both liquid phases may be considered as dilute solutions in benzene and water.
(c) For the problem posed in part (b), it is known that both 11 and 21 are substantially independent of
concentrations of either solutes in either phases. Utilizing the result in (a), suggest ways to increase the
separation at constant temperature (this means 11 and 21 are constants).

1.5.3 Consider two different separators indicated by supercripts A and B. Each has a uniform binary mixture to start
with. After separation of each mixture into two different regions, both separators have k 011 106 and k 021 106 .
In separator A, mo1 mo2 1 gmol. But in separator B, mo1 106 gmol and mo2 106 gmol.
(a) Obtain the value of the extent of separation for each vessel and show that they are equal. (Ans. A B
0.9999981.)
38 Description of separation in a closed system

(b) Define an extent of purification p by the absolute value of the determinant of the matrix whose elements
are x ij .
(c) Show that the extent of purification for separator A is much greater than that of separator B.
This problem illustrates the role of initial mole ratio m01 =m02 in developing a comparison between two
 

different separation systems behaving identically with respect to distribution ratios (Rony, 1968b). Conversely,
it illustrates the relative insensitivity of the extent of separation to considerable degrees of purification (as
pointed out in Section 1.5).
1.6.1 For a qualitative description of separation of a single multicomponent feed mixture of n species into k product
regions or fractions, Lee et al. (1977a) have defined a relative molar fraction of species i in product region j by ij
xij/xif , where each of the mole fractions refer to its averaged value in a given region. Further, r j is defined as
mj
rj k
,
mj
X

j1

where mj is the total number of moles in the


jth product or region.

k 
ij r j :

(a) Develop a relation between Yij and ij. Ans:Y ij ij r j =
X

j1
(b) Obtain a matrix of product compositions in terms of relative molar fractions.
(c) What criterion must be satisfied to indicate that there is some separation between two species in one of the
product regions?
2

Description of separation in open


separators

Separations for preparative or analytical purposes are often general) leave through some other defined surface areas of
carried out batchwise in a closed vessel. On the other hand, the separator.
industrial-scale separations are commonly achieved in a The two product streams may be in contact with each
continuous manner with open vessels into which feed other or they may be separated from each other by a
streams enter and from which product streams leave. In barrier. Next we provide some examples of the nature of
this chapter, we consider the available methods of describ- contact between various feed and product streams in open
ing separation in such open separators. The quantities, separators. Figure 2.1.2(a) shows a nitrogen-rich gas frac-
fluxes and mass balances necessary for such descriptions tion leaving the single-entry separator on one side of the
are presented first in Section 2.1. Section 2.2 describes the silicone membrane, whereas the oxygen-rich gas fraction is
available indices of separation and their interrelationships withdrawn from the low-pressure side of the silicone mem-
for binary separation with a single feed stream entering the brane permeator used for separating air. Figure 2.1.2(b)
separator. In Section 2.3, we briefly introduce indices for shows a single plate in a distillation column in which
binary separation with two feed streams entering a separ- bubbles of vapor leave the plate (i.e. a double-entry separ-
ator. The complications encountered in describing multi- ator) after contacting a liquid stream which flows along the
component separations with a single-entry or double-entry plate. Only a small part of the vapor product stream is in
separator are presented in Section 2.4. This section pro- contact with part of the liquid product stream leaving the
vides also an introduction to the description of systems of plate separator. A product stream may also be in contact
continuous chemical mixtures and size-distributed popu- with a feed stream as it leaves the separator. Figure 2.1.2(c)
lation of particles. Separation by any of the separators shows how a gas stream leaves the top of a spray scrubber
considered in these sections presupposes that the output where a feed liquid stream is introduced to absorb an
streams have different compositions. There are separation undesirable species from a gas mixture. Only part of the
processes, e.g. chromatography, in which the separator has cross-sectional area of the tower is utilized by the falling
only one output stream, but that has a time-varying com- drops of liquid feed, with the rest being utilized by the
position. The description of separation in such a separator product gas. While the specification of the flow cross-
with the help of various indices has been considered in sectional area of the separator for a given stream is quite
Section 2.5. Triple-entry separators etc. have not been dealt difficult in this case, it is straightforward in Figure 2.1.2(a).
with here. Further, except for Section 2.5, steady state We now consider these separators to be fixed in space
operation is assumed throughout. and focus attention on a particular surface area Sj of any
separator shown in Figure 2.1.1. Let vij be the local average
velocity vector of species i with respect to coordinate axes
fixed on the surface area Sj. If the mass concentration of
2.1 Preliminary quantitative considerations
species i at this location on such a surface area is indicated
Open separators can be broadly classified into single-entry by ij, with tj being the local value of the total mass
and double-entry separators. A single feed stream enters concentration, we define the local mass average velocity
the single-entry separator. Two feed streams are necessary vector1 vtj by
for a double-entry separator (Figure 2.1.1). A feed stream
enters a separator through a defined fraction of its surface
1
area, just as the product streams (at least two in number in See Bird et al. (2002), pp. 533535 for more details.
40 Description of separation in open separators

S1 S1
Sf 2

Sf Actual
Actual velocity
velocity profile
profile

Sf 1
S2
S2

Single-entry separator Double-entry separator

Figure 2.1.1. Single-entry and double-entry separator. The velocity profiles shown at the inlet(s) and exits of the separators need not be
parabolic.

Silicone membrane

O2-rich fraction

Air N2-rich fraction


Compressor

(a)

Benzene-rich Scrubbed gas Feed liquid


vapor product
Liquid feed

Vapour feed

Taluene-rich
liquid product Gas
(b) (c)

Figure 2.1.2. Some examples of the nature of contact between various feed and product streams. (a) Membrane separates air into two
product streams in a single-entry separator, which is a silicone membrane permeator for air separation. (b) Limited contact between
vapor and liquid products in a sieve plate (the separator) in a distillation column. (c) Complete contact between a feed scrubbing liquid
stream introduced as drops in a spray and the product gas stream after scrubbing.

n
X n
X of a suitable pitot tube. The local molar average velocity
ij vij ij vij vector vtj is defined by
i1 i1
vtj n 2:1:1
X tj n
X n
X
ij C ij vij C ij vij
i1 i1 i1
vtj n , 2:1:2
for a system of n components. The magnitude of this
X C tj
C ij
velocity, vtj, is, for example, measurable for liquids by use i1
2.1 Preliminary quantitative considerations 41

where Cij is the local molar concentration of species i. In For these special cases, we have assumed that vij is always
general, the quantities ij, tj, Cij, Ctj, vij, vtj, vtj , will vary locally perpendicular to Sj. Further, the quantities Sj, vij, vtj
with location on the surface area Sj. The local mass flux and vtj are the magnitudes of the respective vector quan-
vector nij of species i relative to the stationary surface area tities. If now vij and therefore vtj and vtj vary in magnitude
Sj is defined as only along Sj, and vij is locally perpendicular to Sj, the
following average values hviji, hvtji and hvtj i may be defined
nij ij vij : 2:1:3
such that
The mass rate of inflow or outflow of species i through wij ij hvij iSj ; 2:1:13
surface area Sj is given by
wtj tj hvtj iSj ; 2:1:14
wij nij  dSj : 2:1:4
W ij C ij hvij iSj ; 2:1:15
Sj

W tj C tj hvtj iSj : 2:1:16


The rate at which mass is entering or leaving the separator
through Sj is then given by Note that
2 3
Xn n ij =M i C ij : 2:1:17
7 X
wtj 4 nij  dSj 5 wij : 2:1:5
6
i1
Sj
i1 If the concentrations also vary across the surface area Sj,
we can define the various mass and molar flow rates with
We adopt the convention that when material is entering a respect to averaged concentrations and averaged
separator, it is to be considered positive for mass balance velocities.
purposes, whereas when material is leaving the separator, The mole fraction xij of species i in the stream entering
we consider it to be negative. Further, diffusive contri- or leaving the separator through Sj is defined by
butions to the mass rate of inflow or outflow of species
x ij W ij =W tj : 2:1:18
i through Sj are not considered in this chapter.
Quite often the molar rate of inflow or outflow of mass If vij v for the uniform velocity cases, (2.1.11) and
tj
with respect to the surface area Sj of a separator is of (2.1.12), then only
greater interest. In such a case, the molar concentration
Cij is used. The quantities corresponding to nij, wij and wtj x ij C ij =C tj : 2:1:19
are then Nij, Wij and Wtj, the local molar flux vector of
The mass fraction uij of the ith species in the stream
species i, the molar rate of inflow or outflow of species
passing through area Sj is defined for an open separator by
i and the total molar rate of inflow or outflow of all species,
respectively: uij wij =wtj : 2:1:20

N ij C ij vij ; 2:1:6 When uniform velocity profiles exist and vij vtj, this
relation reduces to
W ij N ij  dSj ; 2:1:7
uij ij =tj : 2:1:21
Sj
2 3 An open separator will have at least three streams coming
n n
in and out through three different surface areas Sj (see
X X
W tj W ij 4 N ij  dSj 5: 2:1:8
6 7
i1 i1 Section 2.5 for a different case). We adopt the convention
Sj
that for a single-entry separator the subscript j will have
If the quantities ij, Cij, vij,, vtj and vtj do not vary across the values 1, 2, . . ., k corresponding to k product streams, but
surface area Sj, we obtain the following relations from j f for the single feed stream. For a double-entry separ-
(2.1.4), (2.1.5), (2.1.7) and (2.1.8), respectively: ator, the two feed streams are to be denoted by j f1 and f2,
respectively, while the k product streams will continue to
wij ij vij Sj ; 2:1:9 have j 1, 2, . . ., k.
n
X
! Consider the steady state operation of the single-entry
wtj ij vij Sj tj vtj Sj ; 2:1:10 separator shown in Figure 2.1.1. Since there is no accumu-
i1 lation of mass inside the separator,
W ij C ij vij Sj ; 2:1:11 rate of mass input rate of mass outflow
n k
!
X X
W tj C ij vij Sj C tj vij Sj : 2:1:12 ) wtf wtj : 2:1:22
i1 j1
42 Description of separation in open separators

Assuming further no accumulation of species i in the and


separator and no chemical reaction, we have the following k
mass balance for species i:
X
x if 1 W tf 1 x if 2 W tf 2 x ij W tj , 2:1:33a
j1
k
are valid in the absence of any chemical reaction in the
X
uif wtf uij wtj , 2:1:23a
j1 separator. Further,
where we know that n
X n
X n
X
x if 1 1; x if 2 1; x ij 1: 2:1:33b
n n
X X i1 i1 i1
uij 1, uif 1: 2:1:23b
i1 i1 2.2 Binary separation in a single-entry separator
For a single-entry separator with two output streams,
with or without recycle
(2.1.22) and (2.1.23a) reduce to We consider now the description of binary separation in a
single-entry separator with only two product streams.
wtf wt 1 wt 2 2:1:24
Assume steady state operation without any chemical reac-
and tion. As in Chapter 1 with region 1 and species 1, we assume
uif wtf uil wt1 ui2 wt2 : 2:1:25 here arbitrarily that species 1 is lighter than species 2 and
that j 1 refers to that product stream which is richer in the
The corresponding molar balances in the absence of a lighter species 1. If we had perfect separation, product
chemical reaction are: stream j 1 will have only species 1 and product stream
k
X j 2 will have only species 2. To start with, consider only a
W tf W tj ; 2:1:26 nonrecycle separator. Such a separator is sometimes called
j1 a splitter (Figure 2.2.1a).
k
X If xi1 6 xi2 and both are different from xif, the single-
x if W tf x ij W tj ; 2:1:27
entry separator has achieved some separation. How much
j1
separation has been attained can be estimated by suitable
W tf W t1 W t2 ; 2:1:28 descriptors, indices of separation. The simplest of these
indices bear the name of specific separation processes
x if W tf x i1 W t1 x i2 W t2 : 2:1:29a
where they are extensively used: the desalination ratio Dr
Further, the solute rejection R and the decontamination factor Df.
n n
X X Desalination ratio:
x if 1, x ij 1: 2:1:29b
i1 i1 C 2f
Dr : 2:2:1a
C 21
With identical assumptions, the total mass balance and the
ith species balance equations for a double-entry separator Solute rejection:
(see Figure 2.1.1) are: 
C 21

R 1 : 2:2:1b
k
X C 2f
wtf 1 wtf 2 wtj ; 2:1:30
j1
Decontamination factor:
k
X C 2f
uif 1 wtf 1 uif 2 wtf 2 uij wtj : 2:1:31a Df : 2:2:1c
j1
C 2l

Note that all three definitions involve elimination of solute


Note that in this case
from a solvent (species 1). Further, the solvent is supposed
n
X n
X n
X to concentrate in exiting stream 1 and solute (species 2)
uij 1, uif 1 1, uif 2 1: 2:1:31b in exiting stream 2. If there is separation, both Dr and Df
i1 i1 i1
will have values greater than 1, whereas R  1. It is also
Like (2.1.23b) and (2.1.29b), these equations (2.1.31b) clear that
are needed as such to solve for the variables, the mass
1 1
   
fractions and the mass flow rates. However, only (n 1) of R 1 1 : 2:2:1d
Dr Df
equations (2.1.31a) are to be used.
The corresponding steady state molar balances, For dilute solutions, the following simplifications are valid:
k
x 2f x 21 x 2f
X  
W tf 1 W tf 2 W tj 2:1:32 Dr ; R 1 ; Df : 2:2:1e
j1 x 21 x 2f x 21
2.2 Binary separation in a single-entry separator 43

xi 1 qWtf r
xi1 (1h)qW rtf
hqW rtf
hf ht
12 12
x ri1
qW rtf
Wtf
(1q)Wtf x ri1
xif
xi 2 Wtf W rtf (1q)W rtf
r r
ft
12 xif x if x i2

(a) (b)

Wtf, xif Pressure-reducing x ri1 Vapor fraction


valve
Fresh r
W tf
feed
Flash separator
Butane (1) x rif
Decane (2)
Pump Liquid fraction

r
x i2
Heater

(c)

Figure 2.2.1. (a) Nonrecycle single-entry separator. (b) Single-entry separator with part of light fraction recycled to feed stream. (c) Single-
entry separator with part of heavy fraction recycled to the feed.

All such indices therefore describe separation by indicating


" #
ht x
how different the exit stream 1 composition is from the x 11  12
12
; 2:2:2b
1 x 12 ht12  1

feed stream with respect to the solute species. A somewhat
similar index is the equilibrium ratio of species i (obtained " #
under phase-equilibrium conditions), x 11
x 12 ht : 2:2:2c
12  x 11 ht

12  1

K i x i1 =x i2 , 2:2:1f
which relates the mole fraction of species 1 in product Further, if one defines the mole ratio Xij (xij/(1 xij)) for a
stream 1 to that in product stream 2. The most common binary system, then
index of separation for the single-entry separator with two
product streams is the separation factor ht12 for species 1
X 11 ht
12 X 12 : 2:2:2d
and 2 between the product streams j 1 (the heads or light
The enrichment factor ht
12 , defined by
fraction, indicated by the superscript h) and j 2 (the tails
or heavy fraction, indicated by the superscript t): x 11  x 12
ht ht
12 12  1 , 2:2:2e
x 11 x 22 x 12 1  x 11
ht
12 : 2:2:2a
x 12 x 21
takes on very small values for close separations
It is sometimes called the stage separation factor. (ht
12 <<< 1); the following approximations are then valid:
If ht
12 > 1, species 1 is preferentially present in the
heads stream j 1 and species 2 concentrates in the tails ht
 ht 
12 exp 12 ; 2:2:2f
stream j 2. Commonly conditions are chosen such that
ht
12 > 1. A few common and useful relations involving the x 11 x 12 ht
12 x 12 1  x 12 ; 2:2:2g
separation factor and the compositions of the exiting
x 12 x 11  ht
12 x 11 1  x 11 : 2:2:2h
streams are given below:
44 Description of separation in open separators

For processes where Ki is useful, In the foregoing treatment, separation was synonymous
with the development of a composition difference between
ht
12 K 1 =K 2 : 2:2:2i the two exiting streams. Since perfect separation requires
all of species 1 in exiting stream 1 and all of species 2 in
How far apart the compositions of the two product streams
exiting stream 2, we need also some quantities to indicate
are is indicated by ht
12 . But this index does not indicate how
how much of species 1 in the feed stream is present in the
much richer the exiting stream 1 is in species 1 compared
product stream j 1. The earliest index of this kind is the
to the feed stream. That is expressed by the heads separ-
hf cut, (sometimes called the stage cut):
ation factor 12 (Benedict et al., 1981, chap. 12):
hf x 11 x 2f W t1 W 11 W 21 x 1f x 12
12 , 2:2:3 , 2:2:10a
x 1f x 21 W tf W 1f W 2f x 11 x 12

where the superscript f refers to the feed stream. Similarly, with 0   1. For example, suppose x11 >> x21 but << 1.
hf
the composition difference between the feed stream and Let x1f x2f, then 12 >> 1. But the separation is poor,
the tails stream is characterized by the tails separation since very little of species 1 present in the feed has
ft been recovered in the heads fraction j 1. Most of
factor 12 (Benedict et al., 1981, chap. 12):
species 1 is present along with most of species 2 in the
ft x 1f x 22
12 : 2:2:4 stream j 2. Note, however, that does not directly
x 2f x 12
indicate the extent of recovery in the light fraction of
Note that species 1 present in the feed stream. That is indicated
hf ft by the component cut, i, for the ith species, defined by
ht
12 12 12 : 2:2:5
hf ft W t1 x i1
Thus if 12 > 1 and 12 > 1, the stage separation factor ht
12 i , 2:2:10b
hf ft W tf x if
is greater than both 12 and 12 . This need not be true if
hf ft
either 12 or 12 is less than one. the value of which ranges between 0 and 1.
The single-entry separator is said to be symmetric if The index , the extent of separation, was shown in
hf ft Chapter 1 to reflect both the quality of the separated
12 12 : 2:2:6 regions in terms of composition and the amount of recov-
In such a case ery of a species in its designated region. For a single-entry
 2  2 separator, an appropriate definition of is as follows
hf ft
ht
12 12 12 : 2:2:7 (see Rony (1970) and the efficiency formula 9 in Rietema
(1957)):
When equation (2.2.6) is not valid, we have an asymmetric
hf ft
separator. If is the highest common root of 12 and 12 j 11  21 j j1  2 j, 2:2:11
such that (Wolf et al., 1976)
where the quantities 11 and 21 are obtained from the
hf
12 k and
ft
12 , 2:2:8a following general definition of the segregation fraction ij
of the ith species in the jth stream:
we get
W ij x ij W tj x ij W tj
k ij : 2:2:12
ht
12 , 2:2:8b k
X W if x if W tf
hf k=k ft
W ij
12 ht
12 ; 12 ht
12
=k
: 2:2:8c j1

Although ht12 has been defined in terms of xij (j 1, 2) only, Note that 11 1 and 21 2 , where i is the ith
it may be defined in terms of Wij also. Use (2.1.18) in component cut. For perfect separation in a single-entry
(2.2.2a) to obtain separator and two product streams,
W 11 W 22 x 11 W t1 x 22 W t2
ht
12 : 2:2:9a 11 1 and 22 1: 2:2:13a
W 12 W 21 x 1f W tf x 2f W tf
Similarly, If there is no separation at all (i.e. x1f x11 x12), then
hf W 11 W 2f
12 : 2:2:9b 11 and 22 1, 2:2:13b
W 1f W 21

and since, for a binary system, x22 x2f x21 for no separation.
We will now obtain a more useful form of for a single-
ft W 1f W 22
12 : 2:2:9c entry separator. Substitution of definition (2.2.12) in
W 2f W 12
(2.2.11) leads, after simplification, to (Sirkar, 1977)
2.2 Binary separation in a single-entry separator 45

x 21  x 11 x 2f
W t1 W i1
  
0
ki1

 1 ; : 2:2:19
x 2f  x 21 x 1f
W tf  W i2
W t1 x 11  x 1f x 21 
   
1 ; 2:2:14 Assuming properties as well as velocities to be invariant
W xtf

1f x x  11 2f along the separator surfaces S1 and S2 as well as vij vtj ,
" # we can write (Rony, 1970)
x 21  hf 1
    
 2 hf  1 2 hf ;
  
12  1 12 12 1 1  :
x 2f     hf
12 0 S1 C i1 vi1 S1 x i1 C t1 hvt1 i
ki1 : 2:2:20
2:2:15 S2 C i2 vi2 S2 x i2 C t2 hvt2 i

The separation index thus combines the descriptive fea- By (1.4.1), (Ci1/Ci2) i1, the distribution coefficient. Thus
hf
tures of the heads separation factor 12 and the component 0
hf ki1 i 1  area ratio  velocity ratioi th species : 2:2:21
cut 1 or 2. Note that 12 indicates the degree of enrich-
ment of the light fraction in species 1 with respect to the If one further assumes that vi1 vt1, vi2 vt2 and S1 S2,
feed stream, with 1 providing an estimate of the extent of then the distribution ratio ki1 0
is directly governed by i1 and
recovery of species i if there is separation. Some special the bulk velocity ratio of the two streams leaving the separ-
cases of relation (2.2.15) are given in the following. ator. Obviously ki1 0
can have any value between 0 and due
Example 2.2.1 Obtain simplified expressions of for (a) to the possible values of i1 , even if both the area ratio and
close separation, (b) a close-separation symmetrical separ- the velocity ratio are each equal to 1. For the special case of
0
ator and (c) a dilute solution of species 2 in solvent 1. C t1 C t2 , hvt1 i hvt2 i, and S1 S2 , ki1 K i:

Solution (a) Close separation In such a case, x11 > x1f


and x21 < x2f, but x11 x1f 0(), where is a very 2.2.1 Examples of separation in single-entry separators
small quantity <<1. Similarly, x2f x21 0().
Therefore Example 2.2.2 In the flash expansion desalination process
  (also called the flash vaporization process), cold sea water,
 hf hf heated to a temperature of 121 C at the corresponding
12  1 12 2:2:16

saturation pressure in a preheater, is allowed to enter a
since x 21 =x 2f 1: flash chamber where the temperature is 38 C and the
 

pressure is much lower at the corresponding saturation


(b) Close-separation symmetrical separator Due to
 1=2 pressure. This produces some pure water vapor and a
hf
symmetry, 12 ht 12 ; further, with close concentrated brine. In practical processes, the vapor mass
separation flow rate produced can be at most 20% (Silver, 1966) of
    the feed brine mass flow rate. If the brine feed has 3.5 wt%
hf hf hf
ln 12 ln1 12  1 12  1 ; salt, describe the separation achieved with the separation
factors, extent of separation, distribution ratio and equili-
but brium ratio for two vapor flow rates expressed as a
hf 1 1 1 ht percentage of the feed brine mass flow rate: (a) 20 wt%,
ln 12 ln ht ht
12 ln1 12  1 a  1 (b) 10 wt%.
2 2 2 12
2
since ht12  1 and higher-order terms (similarly for Solution Basis: 100 g/s of feed brine flow rate wtf ; species
hf
12  12 terms, etc.) are negligibly small due to close sepa- 1 is water, species 2 is NaCl, so
ration; in addition, (x21/x2f) 1. Therefore (Sirkar, 1977)
96:5=18

 
 ht x 1f 0:988, x 2f 0:012:
ht 1 : 2:2:17 96:5=18 3:5=58:5
2 12  2 12
(a) wt1 20 g/s, wt2 80 g/s, u1f 0.965, u2f 0.035,
(c) Dilute solution of species 2 in solvent 1: x1f x11, so u21 0, u11 1, x21 0, x11 1 (pure H20). We have
     
 x 21   1   1  wtf u2f wt2 u22 wt1 u21 80u22 0 100  0:035;
1  1  R 1 : 2:2:18
  
 x 2f   Dr   Df  3:5
u22 0:04375:
80
The index , as per expression (2.2.14) is such that it can have
To determine x22 from u22 :
a maximum value of 1 and a minimum value of 0. A variety of
conditions can lead to the zero value; the reader can easily 4:375=58:5 0:0747 0:0747
verify this. An aspect of that should be noted is that it is x 22 :
4:375 95:625 0:0747 5:3125 5:3872
invariant to a permutation in the subscripts i or j in ij :
58:5 18
Since combines the separation factor as well as the
component cut, it is a composite index. Another such com- So x22 0.01386; x12 0.9861. The value of the indices may
posite index would be the distribution ratio ki0 l defined as now be calculated as follows:
46 Description of separation in open separators

Overflow outlet
Oil-rich fraction 1
Feed nozzle

Feed nozzle

Underflow: Water-rich fraction 2 Feed: Oilwater mixture

Figure 2.2.2. Hydrocyclone separating an oilwater mixture.

x 11 x 22 10:01386 molecular level; rather, there are drops of oil dispersed in


ht
12 ;
x 21 x 12 00:9861 water. However, we may treat the problem as if it were a
hf x 11 x 2f 10:012 binary separation problem.
12 ;
x 21 x 1f 00:988
Example 2.2.3 An oil and water mixture may be sepa-
ft x 1f x 22 0:9880:01386 rated by a hydrocyclone (Figure 2.2.2), which is essentially
12 1:1523:
x 2f x 12 0:0120:9861 a truncated hollow cone with a cylindrical section at the top.
The feed mixture enters tangentially in the cylindrical
Obviously, a flash desalinator is an asymmetric single-entry section through the feed nozzle. A water-rich heavy fraction
separator. We have leaves the unit as an underflow at the bottom of the device.
 x 11 x 21   1 0  The oil-rich light fraction leaves the unit at the top center
   
   20=18 opening.

x 1f x 2f  96:5 3:5  0:988 0:012 
  
(a) Show that the conventional efficiency definition for a
18 58:8
  hydrocyclone (Van Ebbenhorst Tengbergen and Rietema,
0:2051:010 0:2075; 1961),
W 11 20=18 W 21    
k 011 0:262; k 021 0; Q x
 1 1 Q1 y1   Q2 x 2 Q2 y 2 
  
W 12 76:5=18 W 22  ,
 Qf x f Qf yf   Qf x f Qf y f 

x 11 1 x 21
K1 1:0141;K 2 0:
x 12 0:9861 x 22 is exactly equal to definition (2.2.11) of . Here Qf, Q1 and
Q2 are the volumetric flow rates of the feed, overflow and
(b) wt1 10 g/s, wt2 90 g/s, u11 1, u21 0, u1f 0.965,
underflow, respectively; x1 and y1 are the volume fractions
3:5 of oil and water in the overflow; xf and yf refer to similar
u2f 0:035, u22 0:0388;
90 quantities in the feed; and x2 and y2 refer to those in the
x 22 0:01226; x 12 0:9877;
underflow.
hf ft 0:9880:0122 (b) Calculate the value of the extent of separation in a
ht
12 infinity 12 ; 12 1:0219;
0:0120:9877 35 mm hydrocyclone fed with a 70 cm3/s of 50% oil
50% water mixture such that x1 0.85 and (Q1/Q2) 0.815.
10=18  x 11 x 21   1 0 
   
0:1025  0:988 0:012 ;

5:41  x 1f x 2f  Solution (a) Q1x1 is the volumetric flow rate of oil only in the
10=18 overflow, whereas Qf xf refers to that in the feed. Therefore,
0:1037; k 011 0:1155; k 021 0;
86:5=18 Q1 x 1 Q x 1 =M oil x W
 1 oil  11 t1 ,
Qf x f Qf x f oil =M oil x 1f W tf
x 11 1
K1 1:011; K 2 0:
x 12 0:987 where i 1 refers to oil, j 1 refers to the overflow in xij,
The two components in binary mixtures dealt with so far Wtj; oil and Moil are, respectively, the density and molecu-
are intimately mixed at the molecular level. Suppose, lar weight of the oil. Similarly,
instead, we have to separate a mixture of water and oil Q1 y 1 Q y =M water x W
such that the oil is made up of only one chemical species.  1 1 water  21 t1 ;
Qf y f Qf y f water =M water x 2f W tf
Water and oil molecules are not intimately mixed at the
2.2 Binary separation in a single-entry separator 47

Permeate Silicone rubber capillaries


Tygon tubing

Reject out
Feed in

Silicone rubber plug Tygon casing

Figure 2.2.3. Gas separation by permeation through silicone rubber capillaries. Reprinted from Chem. Eng. Sci., 30(7), 751 (1975), J.M.
Thorman, S. Rhim and S.T. Hwang, Gas separation by diffusion through silicone rubber capillaries. Copyright (1975), with permission
from Elsevier.

thus introduced at a pressure of 176 cm Hg into the inside of


  silicone rubber capillary tubings (0.245 mm inner diameter
Q x Q y 
1 (i.d.)  0.655 mm outer diameter (o.d)), a bundle of which
jY 11 Y 21 j  1 1 1 :
 
 Qf x f Qf yf  was kept in a shell-and-tube arrangement inside a 12.5 mm i.
d. tygon tubing shown in Figure 2.2.3. The gas that perme-
Note: Since component 1 is oil, perfect separation corres- ated through the walls of the silicone rubber capillary was
ponds to only oil in the overflow (product stream j 1) and withdrawn at atmospheric pressure through a port in the
only water in the underflow (product stream j 2). The other shell. The feed air flow rate is 1.80 std. cm3/s and the perme-
equality follows using the following two mass balances: ated gas flow rate is 0.243 std. cm3/s. The permeated gas
contains 0.274 mole fraction oxygen. Determine the
Qf x f oil Q1 x 1 oil Q2 x 2 oil ; separation factor ht 12 , the extent of separation and the cut.
Qf yf water Q1 y 1 water Q2 y 2 water : (b) If instead of air, a mixture of O2 and CO2 containing 50.2
For part (b), mole percent oxygen is introduced at 176 cm Hg as feed into
the same capillary tubings in the separator at a flow rate of 1.91
Qf Q1 Q2 70 std. cm3/s, the permeated gas containing 0.315 mole fraction
70 oxygen comes out at a rate of 0.97 std. m3/s. Determine the
1:815 Q2 ) Q2 38:5cm3 =s;
1:815 separation factor ht12 , the extent of separation and the cut.

Q1 31:5cm3 =s: Solution (a) The composition and the flow rate of the reject
stream are determined first. Here, i 1 (O2), i 2 (N2), j 1
We have permeated stream, j 2 reject stream, since the permeated
x f 0:5, y f 0:5, y1 0:15; stream is becoming enriched in oxygen. With the molar flow
rate of a gas stream in gmol/s being directly proportional to
its volumetric flow rate in std. cm3/s, we get, using (2.1.29a),
   
Q x
 1 1 Q1 y 1   31:5  0:85 31:5  0:15 
  
   0:63:
 Qf x f Qf y f   70  0:5 70  0:5  x 1f W tf x 11 W t1
x 12 ;
W t2
Note: In Figure 2.2.2 we do not describe particular flow
0:209  1:80 0:274  0:243
patterns or forces causing separation and yielding 0.63. x 12 0:199,
It is sufficient here to recognize that two predominantly 1:557
immiscible phase product streams are obtained from a dis- since x1f 0.209, x11 0.274 and
persed phase feed through nothing but a suitable flow
gmol

arrangement with tangential entry. W t 2 W tf W t1 conv: factor for


std: cm3
Example 2.2.3 is modeled after a practical example treated
std: cm3
by Sheng (1977) on separation of liquids by a conventional 1:80 0:243 :
hydrocyclone, and the value of 0.63 corresponds to the s
actual efficiency E used by Sheng. The above approach is also Therefore,
adaptable to cases where the oil itself is a multicomponent x 11 x 22 0:274  0:801
mixture of various hydrocarbons. ht
12 1:516:
x 21 x 12 0:726  0:199
Example 2.2.4 (a) Thorman et al. (1975) investigated the Further,
separation of oxygen from air by preferential permeation of  0:243  0:726  0:274  0:791 
oxygen through silicone rubber capillaries kept in a perme-

 hf 
 0:0545,
2 12 1 1
 
ator. Air containing 20.9 mole percent of oxygen was 1:80  0:791  0:726  0:209 
48 Description of separation in open separators

and the cut separator is W rtf > W tf . Similarly, the fresh feed compo-
 

sition is given by x1f, whereas the actual feed composition


W t1 0:243
0:135: to the recycle separator is x r1f (>x1f since x11 > x1f for
W tf 1:80 hf
12 > 1). Since we have assumed that the actual values of
hf ft
(b) Here i 1 (CO2), i 2 (O2), j 1 (permeated stream) and , 12 and 12 for the recycle separator are the same as
j 2 (reject stream); the permeated stream is enriched in those of the simple separator, we obtain, by a total molar
CO2 compared to the feed stream. Given x2f 0.502, x21 balance at steady state for a recycle ratio of ,
0.315, and taking the volumetric flow rate of the reject stream W tf
to be 0.94 cm3/s ( 1.91 0.97), we get W rtf : 2:2:22
1
x 2f W tf x 21 W t1 0:502  1:91 0:315  0:97
x 22 0:694: Due to the binary mixture being dilute in species 1, the
W t2 0:94
following relations are valid:
Therefore,
hf x r11 x r2f x r11 hf x 11 x 2f x 11
0:685  0:694 12 ; 12 ; 2:2:23
ht
12 4:83: x r21 x r1f x r1f x 1f x 21 x 1f
0:315  0:306
Further ft x r1f x r22 x r1f ft x 1f x 22 x 1f
12 ; 12 : 2:2:24
W t1 x 21  hf  0:97  0:315  0:685  0:502 
 x r2f x r12 x r12 x 2f x 12 x 12
12 1 1
 
W tf x 2f 1:91  0:502 0:315  0:498

A light component mass balance for the actual feed stream
0:318j2:191j 0:378:
in the recycle separator leads to
The cut Wt1/Wtf 0.506. Note that
ft
12 4:83=2:19 2:20; it is behaving almost as a
W tf x 1f W rtf x r11 x r1f W rtf , 2:2:25
symmetric separator.
which on rearrangement using (2.2.22) and (2.2.23) yields
It would appear from the preceding treatments that for a 1
single-entry separator, the cut is independent of ht
hf
12 , 12 or
x r1f x 1f hf
: 2:2:26
ft 1 12 
12 . For any given separation process, the relation between a
separation factor and the cut, , can be derived only by detailed hf
Since 12 > 1, obviously x r1f > x 1f , and therefore
considerations of the separation mechanism operative in the hf
x r11 12 x r1f is greater than x11. The apparent cut and
separator. In general, the separation factor is likely to depend the apparent heads separation factor for the recycle separ-
on . These dependencies will be considered when indivi- ator are given by
dual separation processes are discussed in later chapters.
W rtf 1 1
app < ; 2:2:27
W tf 1
2.2.2 Single-entry separator with a product recycle
  x r11  x r 
hf hf 1f hf
Sometimes, to achieve a better separation, a fraction of 12 12 > 12 : 2:2:28
app x 1f x 1f
one of the product streams is recycled to the feed end of
the single entry separator. Consider Figure 2.2.1 where we Thus, the apparent values indicate a better quality of
show a nonrecycle traditional separator with an actual cut separation accompanied by a lesser amount of product
hf ft
and actual heads and tails separation factors 12 and 12 . stream 1. For a better picture, let us compute the extent
This figure also shows the schematic of a separator, where of separation with recycle and without recycle:
a fraction of the actual light fraction molar output from  r   x r 1 W r x r 1 W r 
 
r   11 tf 21 tf 
the separator is recycled to the feed stream (Figure 2.2.1 r  11 21   ;

W tf x 1f W tf 1 x 1f 
(b)). Thus the actual molar feed rate to the recycle separ-

ator is higher than that to the nonrecycle separator. Let us  
1 x r11  x r x 1f 
now consider a special case of a binary mixture in which r 1 21 r ; 2:2:29

1 x 1f  x 2f x 11
x1f << x2f and x2f is close to 1. Suppose further that the

actual cut and the actual heads and tails separation 
x 11  x 21 x 1f 

factors of the recycle separator have the same values , jY_ 11 Y_ 21 j 1 : 2:2:30
hf ft
x 1f  x 2f x 11 
12 and 12 . What are the values of the apparent cut and
the apparent heads and tails separation factors for this Therefore,
recycle separator? Can we compare the separating ability  xr21 x1f 
of these two arrangements with suitable descriptors?  1 r 
The molar fresh feed supply rate to both separators is
r

1 x r11

 x2f x11 
 x x : 2:2:31a
Wtf. While this is the actual feed flow rate into the nonre- 1 x 11  1 21 1f 
cycle separator, the actual feed flow rate into the recycle x 2f x 11
2.2 Binary separation in a single-entry separator 49

Since all the mixtures are dilute in species 1, we may where Va(xij), the value function, is the value of one mole
assume that of the jth stream of composition xij. For the separator of cut
, the following alternative expression is more useful:
x r21 x 1f x 1f x 21 x 1f x 1f
; : 2:2:31b
x 2f x r11 x r11 x 2f x 11 x 11 U
V ax 11 1 V ax 12 V a x 1f :
 
2:2:32b
W tf
Utilizing relations (2.2.23) and (2.2.26) in (2.2.31a), we get,
after various manipulations, One demands Va(xij), the value function, to be such that
hf
U is independent of feed and product compositions.
r
= 1 =1 12 1 , 2:2:31c The quantity U then indicates the net increase in the
from which we can conclude that if r > , then the value of products over that of the feed stream, and is
following inequality must hold:2 therefore a measure of the separator capability or separ-
ation achieved.
hf 1 There exists no general solution for Va(xij) to make U
12 > : 2:2:31d
1 independent of feed and product compositions. For a few
For a binary mixture dilute in one species and being separated special cases, solutions for Va(xij) satisfying such a require-
in a single-entry separator, there will therefore be a better ment are available. One such case is close separation, for
hf
separation with recycle of light fraction for constant , 12 and which both xi1 and xi2 are sufficiently close to xif to permit a
ft
12 if condition (2.2.31d) is satisfied. Obviously close-separation Taylor series expansion of Va(xij) around Va(xif):
cases will not be helped by recycle unless is unrealistically dV ax i1 

close to 1. The description of separation in a recycle single-entry V ax i1 V ax if x i1 x if
dx i1 x i1 x if
separator is now less ambiguous compared to that based
d2 V ax i1  x i1 x if 2

on earlier results (2.2.27) and (2.2.28).3 Note that r has been . . . ; 2:2:33
dx i12 2
calculated by using the fresh feed and the net separated output

x i1 x if
streams to estimate the actual separation achieved.
dV ax i2 

A recycle single-entry separator may have part of its V ax i2 V ax if x i2 x if
dx i2 x i2 x if
tails stream recycled to the feed (flash distillation, Figure
2.2.1(c)). Examples of radioactive rare gas separation with d2 V ax i2  x i2 x if 2

2
. . . : 2:2:34
various recycle arrangements are given in Ohno et al. dx i2 
x i2 x if 2
(1977, 1978). See Problem 2.2.7. One aspect to be noted
while comparing separators with or without recycle is that For a continuous and well-behaved function Va(xij),
since is constant, but the actual feed flow rate to the dV ax i1 

dV ax i2 

separator changes with recycle from that without recycle, and
dx i1 x i1 x if dx i2 xi2 xif
the dimensions of the two separators will be different. See,
d2 V ax i1  d2 V ax i2 
 
however, Problem 2.2.8.
: 2:2:35
dx 2 i1 x i1 x if dx 2 i2 x i2 x if

2.2.3 Separative power and value function Recognizing from equations (2.1.29a) and (2.2.10a) that
An additional index, often used for isotope separation x if x i1 1 x i2 , 2:2:36
plants and originally introduced by P.A.M Dirac (Cohen,
1951), is the separative power U of a single-entry separ- we obtain from definition (2.2.32b)
ator. Assuming that the value of a particular process stream U 1 d2 V ax i1 

increases with the increase in mole fraction of a given x i1 x if 2 1 x i2 x if 2 
W tf 2 dx 2i1 x i1 x if
species, the separative power of a given separating unit
could be determined from the net increase in the value of 2:2:37a
the streams coming into and out of the separator. Such an when relations (2.2.33), (2.2.34) and (2.2.35) are used. To
index may be defined, following Dirac, as simplify the above result further, rewrite the ith species
U W t1 V ax 11 W t2 V ax 12 W tf V ax 1f , 2:2:32a balance equation (2.2.36) in the following two forms:

x i1 x if 1 x if x i2
   
2:2:37b

and

x i1 x i2 x if x i 2 :
 
  2:2:37c
2 hf
This analysis is valid only if 1 12 is nonzero and positive.
3
K.K. Sirkar and S. Teslik, Description of separation in sepa- With these two relations, expression (2.2.37a) for (U/Wtf)
rators with or without recycle, unpublished (1982). is simplified:
50 Description of separation in open separators

7 The value function is plotted in Figure 2.2.4 against xif. As


has been pointed out (Pratt, 1967), a zero value for Va(xif
0.5) is reasonable, especially for nonisotopic mixtures. This
6
is because the mixture is in the lowest energy state and will
require the largest energy for separation (see Chapter 10).
5 See Cohen (1951), Benedict et al. (1981, chap. 12) and Pratt
(2xif 1)ln[xif /(1xif)]

(1967) for detailed treatments on separative power, value


functions, etc. for isotope separation plants.
4

3 2.3 Binary separation in a double-entry separator


Unlike for a single-entry separator, two feed streams enter
2 a double-entry separator. The number of product streams
leaving such a separator in general is two. As shown in
1 Figure 2.3.1, the relative orientations of the two feed
streams with respect to each other as well as the separator
may vary, resulting in crosscurrent, cocurrent or
0
0.0 0.2 0.4 0.6 0.8 1.0 countercurrent configurations. Separation with such an
xif arrangement is achieved when xil 6 xi2, and either xif1 6
xi1 or xif 2 6 xi2 or both. The most frequent situation
Mole fraction
encountered is xif 1 6 xi1, xif 2 6 xi2, xil 6 xi2, with all three
inequalities being valid simultaneously. Further, product
Figure 2.2.4. Value function of equation (2.2.42).
stream 1, usually termed the heads fraction, the light frac-
tion or the vapor fraction, will have only species 1 when
pure, whereas product stream 2, usually identified as the
W tf d2 V ax i1 

U 1 x il x i2 2 2 : 2:2:37d tails fraction, the heavy fraction or the liquid fraction, will
2 dx i1 x i1 x if
have only species 2 when pure.
The double-entry separator performance is often char-
For close separations, use relation (2.2.2h) with xil xi2
acterized for a two-component system by the equilibrium
xif on the right-hand side and substitute in (2.2.37d):
ratio Ki (definition (2.2.1f)) as well as by the separation
W tf 2   2 d2 V ax if factor ht
12 (definition (2.2.2a)). Both definitions relate the
1 ht
12 1 x if 1 x if

U :
2 dx 2if two product stream compositions and are related to each
2:2:38 other by (2.2.2i). The compositional differences between
the light fraction and feed stream 1 may be indicated by a
To make U independent of compositions of various heads separation factor
streams, assume x 1l x 2f 1
hf
12 : 2:3:1a
2
d V ax if 1 x 1f 1 x 21
  2 : 2:2:39
dx 2if x if 1 x if Similarly, the tails separation factor between feed stream 2
and the heavy fraction is
Therefore,
ft x 1f 2 x 22
W tf 2 12 : 2:3:1b
1 ht x 2f 2 x 12

U 12 1 2:2:40
2
for close separations. The corresponding value function Unlike that for a single-entry separator, where
hf ft
can be obtained by specifying the following conditions: ht
12 12  12 by definition, here,

hf ft
V a x if 0:5 0 ht
 
2:2:41a 12 6 12  12 2:3:2

and unless the two feed streams have the same composition,
dV ax if 
 i.e. x if 1 x if 2 . However, relations (2.2.2b)(2.2.2d) are
0 2:2:41b valid for a double-entry separator. One may also define
dx if

x if 0:5
an enrichment factor ht 12 in exactly the same way as in
for Va(xif) satisfying (2.2.39). The solution is simply (2.2.2e). In addition, for close separations (ht12 << 1), rela-
tions (2.2.2f)(2.2.2h) also hold for a double-entry
V a x if 2x if 1 ln x if = 1 x if :
     
2:2:42 separator.
2.3 Binary separation in a double-entry separator 51

xi 2 Wi 2 xif1 xi1 xif1 xi1


xif1 xi1
Wif1 Wi1 Wif1 Wi1
Wif1 Wi1 xi 2 xif 2
xif 2 xi 2
Wi 2 Wif 2
xif 2 Wif 2 Wif 2 Wi 2

Crosscurrent (or crossflow) Cocurrent (or parallel flow) Countercurrent

Figure 2.3.1. Double-entry separator: various orientations of feed streams.

The distribution ratio k 0 il of species i between the two 2.3.1 Examples of separation in double-entry
product streams 1 and 2 can be defined here in the same separators
manner as in (2.2.19):
Example 2.3.1 In a tall vertical column packed with Raschig
W i1 x i1 W t1 W t1 rings, the separation of benzene from toluene is being
k 0 i1 Ki : 2:3:3a carried out by distillation (Figure 2.3.2(a)). A vapor mixture
W i2 x i2 W t2 W t2
of benzene and toluene containing 0.51 mole fraction
For binary vaporliquid systems used in distillation (see benzene is introduced at the column bottom at a rate of
Example 2.3.1) if vapor phase j 1 and liquid phase j 2, 20 gmol/s. At the column top, a benzene-rich stream having
then, for species i, k 0 i1 is equal to , a parameter frequently the same molar flow rate (mole fraction of benzene 0.95) is
used in distillation analysis (see Problem 2.2.1). We see introduced as a liquid through a distributor. As the vapor
further from (2.2.2a) that goes up the column with the liquid coming down, toluene
  from the vapor preferentially goes to the liquid stream, and
ht 0 0 the benzene from the liquid stream preferentially comes to
12 k l1 =k 2l : 2:3:3b
the vapor stream. The vapor product stream flow rate at the
In terms of average velocities of species i across the surface top is 20 gmol/s and the benzene mole fraction in it is 0.98.
Obtain the values of ht12 and for this double-entry counter-
areas S1 and S2 and constant properties across S1 and S2
current separator. Determine the value of for the two feed
(see relation (2.2.21))
streams only, and compare it with that determined from the
1l 1l 22 product streams. Comment thereafter on the utility of this
ht
12 : 2:3:3c separation device.
21 12 21

If vil vt1 and vi2 vt2, then only Solution Wtf1 20 gmol/s, Wtf2 20 gmol/s, Wt1 20
gmol/s (j 1, vapor stream). Since Wtf1 Wtf2 Wt1
ht
12 1l = 2l , 2:3:3d Wt2, Wt2 20 gmol/s (j 2 liquid product stream). Making
a component balance on benzene (i 1), Wtf1 x1f1 Wtf2 x1f2
which is also valid for closed separators when V1 V2. Wt1 x11 Wt2 x12. But x1f1 0.51, x1f2 0.95, x11 0.98 and
The index extent of separation for a double-entry sep- x12 ? Therefore x12 [0.51 0.95 0.98] 0.48;
arator may be expressed as (Sirkar, 1977)
0:98  0:52
ht Y_ 12 Y_ 21 ht
 
12 53; 12 1 ;

abs jY_ 11 Y_ 21 j abs jY_ 11 Y_ 22 Y_ 12 Y_ 21 j; 2:3:4a 0:02  0:48
W t2 x 12 W t1 x 21
 
j53 1j
   
 Y_ Y_ x x
W tf 1 x 1f 1 W tf 2 x 1f 2 W tf 1 x 2f 1 W tf 2 x 2f 2
 
 11 22  11 22
Y_ 12 Y_ 21 abs  1; Y_ 12 Y_ 21 abs  1 ;
 
 Y 12 Y 21   x 12 x 21 
0:48 0:02
   
  52 0:632:
Y_ 12 Y_ 21 abs ht 1:46 0:54
 
12 1 : 2:3:4b

Concentrate now only on the two feed streams and assume
The quantity Y_ 21 may be interpreted as the component cut them to be product streams whose extent of separation is to
2 , i.e. that fraction of species 2 entering the separator be determined. Assuming feed stream f2 to be stream j 1
which appears in the light fraction j 1. Similarly, Y_ 12 and feed stream f1 to be stream j 2, we get (with i 1 for
may be interpreted as 1 1 , where 1 is the component benzene)
cut for species 1. Note that both feed streams have to be  
0:51 0:05  0:95  0:49
 
lumped together here for determining 1 and 2 . Thus 1

0:51 0:95 0:05 0:49  0:05  0:51


1 1 2 absjht
12 1j: 2:3:4c 0:35  0:0927  17:2 0:558:
52 Description of separation in open separators

(a) (b)
Vapor product
Condenser
Liquid product
Liquid Vapor
Liquid (instead of
distributor product
feed vapor product)

Column
Packing
Liquid
recycle

Vapor
Vapor feed
feed

Liquid
Liquid product
product

Figure 2.3.2. Packed column distillation of benzenetoluene mixture: (a) double-entry separator; (b) overhead condenser providing
recycle liquid feed.

Obviously, little separation has been accomplished since


this value of is only marginally smaller than the actual
( 0.632) for this separator. In fact, in exchange for a
benzene-rich liquid stream (benzene mole fraction
0.95) introduced as feed at the top, a benzene-rich vapor
stream (benzene mole fraction 0.98) is being obtained,
although the objective is the separation of benzene and O2-enriched
Permeator product
toluene introduced as vapor feed at the column bottom. In
practice, the liquid feed at the top is obtained by condens-
ing the vapor product stream in a partial or total con-
denser. Thus, the net separator setup appears as in Shell side Compressor
Figure 2.3.2(b), which, is in effect, a single-entry separator. Silicone Feed air
Note that the flow rates and compositions of the various capillary
product streams will differ from the values given in the
example if this is the case. N2-enriched
Example 2.3.1 described separation in a double-entry product
separator having two distinct phases a vapor feed and a
liquid feed, with both phases existing throughout the Figure 2.3.3. Double-entry membrane separator: continuous
column. Double-entry separators are used even when two membrane column for O2 separation from air.
product streams and the two feed streams are of the same
However, the overall separation system incorporating the
phase.
compressor has only one feed stream and two product
Example 2.3.2 The permeator described in Example 2.2.4 streams. This arrangement is quite similar to the
has been used by Thorman et al. (1980) in a different way distillation column, and is called the continuous membrane
to obtain a much higher composition change in the separ- column.
ated streams. Consider Figure 2.3.3, where feed air is
The wall of the silicone capillaries separates the two gas
introduced at the bottom of the separator on the shell side
streams in the permeator described above. It is not
and a compressed and O2-rich stream is introduced as feed
inside the tubes at the top. Part of the O2-rich stream at the necessary to have a barrier between two streams within
top is withdrawn as product, while the rest is sent back to a separator. Later in the book we will come across
the permeator to go through the silicone capillaries and double-entry separators in which two gas streams are
finally emerge as another product stream. Thus this separ- in contact with each other and yet separation is taking
ator has two feed streams and two product streams. place.
2.4 Multicomponent systems 53

2.3.2 Separation in a double-entry separator with


recycle/reflux 1 R
W rt1 r
W t1 Wtf2
We now briefly carry out a quantitative description of R+1 R+1
separation in a double-entry separator with recycle.
A perfectly general double-entry separator with recycle xif2
would have two independent feed streams, and a part of
one (or both) of the product streams is recycled to the feed
stream closest to it (Figure 2.3.4). A more commonly W rtf2
encountered scheme would have only one independent
W rt1 Wt1
feed stream entering the separator (Wtf 2 0). For such a
or
case, the total and the ith component molar balances are r x rif2
xi1 xi 1
given by

W tf 1 W rtf 2 W rt1 W rt2 , 2:3:5

W tf 1 x if 1 W rtf 2 x rif 2 W rtl x ri1 W rt2 x ri2 : 2:3:6

In general, x ri1 6 x rif 2 (for example, in Figure 2.3.2(b), where r


Wtf1 Wt 2 W t2
the vapor from the top of the column is only partially
or
condensed, the composition of the product vapor and the xif xi 2 x ri2
1
product liquid will be different; the partial condenser is
also a single-entry separator. However, with a total con-
Figure 2.3.4. Double-entry separator with or without reflux.
denser, all vapor from the column top is condensed to
liquid: x rif 2 x ril ). But with a total condenser on the
column, Wtf2 0, x if 2 0, x rif 2 x ri1 and R is the reflux   Wr    r   
W t1
 t1
 R1 1 x r11 R1 r  
R1 1 x 21 R1 
(instead of the recycle) ratio (so that (R/R 1) fraction of 
 W x 
W rt1 is recycled to the top of the separator). Equations tf 1 1f 1 W tf 1 x 2f 1 
r jRR1 
(2.3.5) and (2.3.6) may now be simplified to  r    r   : 2:3:12
r jRR2  W t1 x r  W t1 r  
 R2 1 11 R2 R2 1 x 21 R2 
W rt1
W rt2 ,
 
W tf 1 2:3:7  W x
tf 1 1f 1 W tf 1 x 2f 1 
R1 

W rt1 r
W tf 1 x if 1 x W rt2 x ri2 : 2:3:8 On simplification, it yields (Sirkar and Teslik, 1982; see
R 1 i1
footnote 3, p. 49) for R1, R2 6 0
To simplify the problem further, let us assume that
W tf 1 W rt1 and W rtf 2 W rt2 (the so-called constant total r jRR1 1 1=R2
: 2:3:13
molar overflow assumption to be encountered in Sections r jRR2 1 1=R1
8.1-3.1/8.1-1.1). Further, let the following relation be valid
regardless of the value of R: Thus r jRR1 > r jRR2 if R1 > R2. That a higher reflux ratio
achieves a better separation (which is a well-known fact, as
x if 1 K i x ri2 : 2:3:9 we shall see in Section 8.1-3.2) can therefore be described
with the help of the separation index .
For given values of R, Ki and Wtf1, the following solutions
are obtained:

R
 2.4 Multicomponent systems
W rt2 W rtf 2 W tf 1 , 2:3:10
R1 Up to this point, we have considered only the description

R

of separation of a binary mixture in a single- or double-
x ri1 R 1x if 1 x if 1 : 2:3:11 entry separator. We now briefly turn to multicomponent
Ki
mixtures.
Could we now describe the relative performance of this In general, a multicomponent mixture will have, say, n
separation system for two nonzero values of reflux ratio R1 different chemical species so that i 1, 2,. . .,n. An analo-
and R2 (R1 > R2)? We opt to determine the relative values gous problem is encountered when solid particles of the
of the extents of separation for the two cases. But we same material having a wide distribution of sizes are to be
recognize that, in effect, the separator of Figure 2.3.4 has separated from one another or from a fluid in which they
one feed stream and two product streams. Therefore we are suspended. If solid particles of a certain size are to be
use defined by (2.2.14) to obtain the following: considered as one species or one component, then such a
54 Description of separation in open separators

mixture, in general, will have an infinite number of species The composition of the particle population is usually indi-
or components since the particle sizes are, in general, cated by the particle size density function f (rp), where rp is the
continuously distributed over a range. Similarly, a macro- characteristic particle dimension of importance. We denote
molecular mixture which is not monodisperse will have a the values of this function for the feed stream and the product
molecular weight distribution and therefore an infinite streams by ff (rp), f1(rp) and f2(rp), respectively. Since the frac-
number of species in general. tion of particles in the size range rp to rp drp is given by4 f (rp)
We should recognize now that whether the system is of drp when the particle size density function is f (rp), such that
n components or an infinite number of components, no
r max
special indices are available to accommodate this complex-

ity. The indices developed for describing binary separation f r p dr p 1, 2:4:1a
r min
have to be adopted for this task. This is done by lumping a
large number of components or a range of particle sizes (or the following relations are also valid:
molecular weights) into one component of a hypothetical
binary system whose other component embraces the r max
r max
r max

remaining components or the remaining range of particle f f r p dr p f 1 r p dr p f 2 r p dr p 1:
sizes (or molecular weights). An alternative approach is to r min r min r min
work with the heavy key component and the light key 2:4:1b
component, as pointed out in Section 1.6.
We will study first the description of separation of a par- Here, rmax and rmin refer to the maximum and minimum
ticle population and then a chemical solution containing an particle sizes in the feed stream. The nature of such a
infinite number of species. At the end of the section, we density function is illustrated in Figure 2.4.3(a). The
describe n-component chemical solutions using the approach particle size distribution function F(rp) is defined by
of a heavy key component and a light key component. rp rp

dF f r p dr p ) f r p dr p dFr p Fr p :
2.4.1 Size-distributed particle population r min r min

2:4:1c
Consider a single-entry separator used either for separat-
ing solid particles from a fluid or separating solid particles The maximum value of F(rp), namely 1, is achieved when
having sizes above a particular value from those having the upper limit of integration is rmax.
sizes below the particular value. Let wstf be the total mass A variety of particle size distributions are used in prac-
flow rate of solids in the feed fluid whose total volumetric tice. The most widely used example is the Normal distri-
flow rate is Qf. In such a separator, there are only two bution (Gaussian distribution),
product streams, the overflow (j 1) and the underflow "  2 #
(j 2). The total mass flow rate of solids in the overflow 1 rp rp
f r p p exp , 2:4:1d
and the underflow are, respectively, wst1 and wst2 . The over- 2 r 2 2r
flow is identified with essentially the carrier fluid and the
where r p is the mean of the distribution defined by
finer particles, whereas the underflow is assumed to have
most of the coarser particles and small amounts of carrier r max

fluid. Figure 2.4.1 illustrates this for a hydrocyclone rp r p f r p dr p 2:4:1e
(Talbot, 1980). Sizes of various natural and industrial par- r min
ticles are shown in Figure 2.4.2.
and 2r , the variance, is defined by
If the objective is to obtain a particle-free fluid, then
perfect separation means no solid particles in the over- r max

flow (equivalent to the heads stream of Section 2.2) and 2r r p r p 2 f r p dr p : 2:4:1f
no carrier fluid in the underflow (the tails stream of r min
Section 2.2). If particle classification is the goal, then
Other distributions employed are the log-normal distribu-
perfect separation requires all particles above a given size
tion, the gamma distribution, the RosinRammler distri-
to be in the underflow and all particles smaller than the
bution, etc. We will encounter some of them later in the
given size in the overflow. In an imperfect separation,
book.
some particles are always present in the overflow (when
the goal is to have a particle-free carrier fluid). Similarly,
due to imperfections in the separator, some particles
coarser than the given size are in the overflow, just as some
finer particles are in the underflow from the separator 4
One can also think of f (rp) drp as the probability of finding a
functioning as a classifier. particle in the size range rp to rp drp.
2.4 Multicomponent systems 55

100 100
% Weight less than diameter

% Weight less than diameter


Slurry
Overflow
feed
80 80

60 60

40 40

20 20

0 0
0.2 0.5 1 2 3 45 10 0.2 0.5 1 2 3 45
Diameter (m) Diameter (m)

100

% Weight less than diameter


100
% Weight less than diameter

flow
Underflow 80
80

Over

Feed
60
60
40

rflow
40

Unde
20
20

0
0 0.2 0.5 1 2 3 45 10
0.2 0.5 1 2 3 45 10
Diameter (m) Diameter (m)

Figure 2.4.1 Typical particle size distribution curves for removal and separation of solids from an aqueous kaolin solution by a
hydrocyclone. (After Talbot (1980).)

Often, moments of distribution/density functions are particle number density function, n(rp), (also called the
used in particulate systems. The nth moment of the par- population density function) by
ticle size density function is defined by
dN r p n r p dr p ,
   
2:4:2a
r max

rp
Mon r np f r p dr p 2:4:1g
r min
nr p dr p Nr min , r p : 2:4:2b
r min
From definition (2.4.1e), we see that the mean of the
particle size distribution,r p , is the first moment of f(rp), The dimensions of N(rmin, rp) are number/volume, i.e.
Mo(1). The zeroth moment of f(rp), Mo(0), is F(rmax) 1. number/[L]3; the dimensions of n(rp) are number /[L]4.
The variance 2r of the Gaussian distribution from (2.4.1f) The total number of particles per unit fluid volume, Nt, is
may be related to obtained from

r max
r max
r max


N t dNrmin , r p dN r p n r p dr p ,
   
2r r 2p f r p dr p 2r p 2
r p f r p dr p r p f r p dr p 2:4:2c
r min r min r min
0 0 0

Mo2 2r p Mo1 r p 2 Mo2 2r 2p r p 2 , by integrating over all possible dimensions of the particles,
Mo2 r 2p 2r : 2:4:1h where we have assumed rmin 0. For such a case, we write N
(rp) instead of N(0, rp). Figure 2.4.3(b) illustrates the popula-
Thus the variance, 2r , is easily related to the second tion density function n(rp) as well as the cumulative number
moment Mo(2) and the first moment Mo(1). of particles N(rmin, rp) per unit volume against the particle
An item of significant interest is the actual number of dimension, rp. See Randolph and Larson (1988) for more
particles N (rmin, rp) in the size range rmin to rp per unit details. Problem 2.4.1 is a useful exercise for determining
fluid volume; the corresponding number is dN(rp) in the n(rp). One can now relate the particle size distribution func-
size range rp to rp drp. This quantity is related to the tion F(rp) to the number density function n(rp) as follows:
56 Description of separation in open separators

103 102 101 dp m 1 10 102

urban aerosols
smoke
smog dust
mist, fog
colloidal
spray
silica
silt
clay sand
nanoparticles
paint pigment
carbon black
pulverized coal

flexible long-chain macromolecule (MW106)

coiled extended

viruses bacteria
synthetic polymer spheres
red blood cells
yeast cells
mfp of air

Figure 2.4.2. Sizes of various natural and industrial particles: dp particle diameter or length; 1 m 104 cm; 1 nm 10 103 m;
mfp mean free path; MW molecular weight. (After Davis (2001).)

(a) (b)
Density Distribution
function Nt
function
1.0 1.0
numbers of particles

n(rp), population density


N(rp), cumulative

0.8 0.8

function of particles
0.6 0.6
f(rp) F(rp)
0.4 0.4

0.2 0.2

rmin rmax
Particle size, rp rmin rp Particle size, rp rmax

Figure 2.4.3. (a) Particle size density function and distribution function. (b) Typical plots of population density function and cumulative
particle numbers against the particle size.

rp
dws f r p dr p ws n r p dr p wsp r p n r p dr p s v r 3p ,
       

Fr p nr p dr p =N t : 2:4:2d 2:4:2e
rmin
where wsp (rp) is the mass of a particle of size rp, s is the
If, in lieu of f(rp) based on mass fraction, we use n(rp), density of the particle material and is a shape factor
where the number of particles present per unit volume (Foust et al., 1960; Svarovsky, 1977) for particle volume
between the limits rp and rp drp is n(rp)drp, then assumed independent of particle size and having the value
the differential change in mass of the particle population of (4/3) for a sphere (for example). The total particle mass
ws per unit fluid volume for a differential change in rp is per unit volume, ws (also referred to as the suspension
given by density, MT) is
2.4 Multicomponent systems 57

rmax rmax
Solution (a)(i) From equation (2.4.2e), for one spherical

s s
dw w M T s v r 3p n r p dr p s v Mo3 :
 
particle (i.e. dN r p n r p dr p 1), the mass of one spher-
  
rmin rmin ical particle is dws s 43 r 3p ; by definition, dws s v r 3p
2:4:2f which implies that v 43 . If v is defined as
dws s v d 3p this implies that v =6:
The number-based mean radius r p1, 0 of the particle popu- (ii) The volume of a rectangular parallelepiped of dimen-
lation is obtained by using the zeroth and the first moment, sions L  W  W is LW2. By definition, dws s v L3
Mo0 and Mo1 , of n(rp): s LW 2 . Since W L/3, we get v W 2 =L2 1=9:
2r (b) From equation (2.4.2e),
max
, rmax

3
dws dN r p s v r 3p ) 4  102 g
 
r p1, 0 r p n r p dr p n r p dr p 5
   
4
rmin rmin 50  2 g=cm3  v  0:043 cm3 ,
so
Mo1
r p n r p dr p =N t ,
 
2:4:2g
Mo0 4  102  106 4  102
0 v 6:25:
100  64 64
where we have replaced the limits rmin and rmax by 0 and , 3
If v were defined via dws dN r p s v d p , then
 

respectively.
4  102  106 4  102
In general, v 0:78:
100  64  8 64  8
rmax , rmax
(c) For a spherical particle,
r pi1, i r i1
p n r p dr p r ip n r p dr p :
   
2:4:2h
surface area 4r 2p
rmin rmin s 2 ) s :
characteristic dimension d 2p
Another commonly used radius is based on the mass of the
For a spherical particle,
particle (which varies with r 3p ; see (2.4.2f)), r p4, 3:
3
, volume =6d p
v 3 =6:
4
r p4, 3 r p n r p dr p r 3p n r p dr p : d3p
   
2:4:2i characteristic dimension
0 0
Therefore
6 v 2=3
2=3
6
  
Example 2.4.1 (a) We are going to consider the issue of 6
1:
volume shape factor v for different types of particles. s = =

(i) What is the value of v for spherical particles of radius rp? Generally, for other particle shapes, 0  < 1:
(ii) Consider crystals shaped as rectangular parallelepipeds
Example 2.4.2 Consider a particle number  density function
with dimensions of L, W, W, where (L/W) 3. What is
n r p having the form n r p n0 exp
  
the value of v if v is defined with respect to L3?  p : 
 ar
Determine expressions for Nt, F r p , f r p , r p1, 0 , d32 , MT
(b) When a mixture of different sized particles was frac- and AT. Here d 32 is the Sauter mean diameter equal to 2r p3, 2 ;
tionated by a sieve having openings of size 0.09 cm, the AT is the total particle surface area per unit volume of the
particles which passed through were fractionated next by a total mixture. Assume that rp varies between 0 and .
sieve of size 0.07 cm. Those particles which were retained by
Solution From definition (2.4.2c),
this sieve may be considered to have a characteristic dimen-
sion (2rp dp) of 0.08 cm. The number of particles retained
on the sieve is 50; their weight is 4  102 g. The particle N t n r p dr p n0 exp ar p dr p n0 =a :
     
density is 2 g/cm3. What is the value of ? If were defined 0 0
with respect to d 3p what would be its value?
(c) A surface shape factor s of particles may be defined as From definition (2.4.2d),
rp
follows:
F rp n r p dr p =N t :
   
surface area
s : r min
characteristic dimension2
Assume now that rmin 0. Then
Show that the sphericity , defined by
rp
exp ar p exp0
 
a

6 v = 2=3 F rp 0 n0 exp ar p dr p a
   

n a a
s =
0 
1 exp ar p 1 1 exp ar p :
  
has the value of 1 for spherical particles when the charac-
teristic dimension for and s is the diameter. By definition (2.4.1c),
58 Description of separation in open separators

X
  dF r p
 
Noi r pi
dF r p f r p dr p ) f r p n r p =N t
     
dr p r p1, 0 i1
X , 2:4:2k
Noi
(from definition (2.4.2d)), so i1

n0 expar p where the denominator represents the total number of


f r p a expar p :
n0 =a particles in the sample.
By definition (2.4.2g), In many particle based separation processes, particles
0 1 break, coalesce or grow (e.g. crystal growth). Common expec-
tations for the mean particle size in small-sized populations

r p1, 0 @ r p n r p dr p =N t A
 
can often be misleading. Consider the example provided by
0
Neumann et al. (2003). Let there be ten particles of volume
n0 a a1 1

0 r p exp ar p dr p ) r p1, 0 2 :
 
equivalent size of 1 unit and one large particle of size 100
n a a
0 units. The number mean size r p1, 0 from definition (2.4.2k)
By definition (2.4.2h), d 32 Sauter mean diameter 2r p3, 2 :
10  1 1  100
r p1, 0 10: 2:4:2l
11
2 r 3p nr p dr p
6=a4 6 If the large particle breaks into two small particles of simi-
d 32 0 2 :
2=a3 a lar shape, then since the particle volume/mass, s v r 3p , is
2
r p nr p dr p conserved, we get two new particles of size rpn:
0
 3 100
By definition (2.4.2f), s v 1003 2s v r pn ; r pn 1=
79:37:
3

2
2:4:2m
M T s v r p n r p dr p s v n r 3p exp ar p dr p
3 0
   

0 0 The new value of r p1, 0 is


4 3!
s v n0 s v n0 4 ; M T 6s v n0 =a4 : 10  1 2  79:37
a4 a r p1, 0 14:06,
From the given definition of AT, if ap is the surface area of a 12
particle of size rp, then, from the previous example, which is larger than before. Since the breakage led to two
ap s r 2p . Therefore smaller particles, one is tempted to think that the mean
particle size will decrease. It did not because the sample
AT ap n r p dr p s n0 r 2p exp ar p dr p
   
size was small. Neumann et al. (2003) have suggested that
0 0 the number mean size will increase due to halving break-
n0 2! 2 n0 age of the larger particles (each particle breaking into two
s 3 s3 :
a a equal fragments) provided the particles broken are more
The treatment of size-distributed particle populations has than 70% larger than the mean particle size of the initial
so far assumed that the number of particles in the popul- distribution. We will now focus on describing separation in
ation is quite large and there is almost a continuous distri- devices used for separation of particles.
bution in particle sizes in the particle population under The fraction of particles in the range rp to rp drp could
consideration. If the number of particles in the population be based on the weight fraction of particles, or the number
is not large, then we have essentially a discrete distribution fraction of particles, etc. Suppose we assume that f (rp) drp
in particle sizes. Suppose the number of particles in the gives the weight fraction (and therefore the mass fraction) of
size range r pi to r pi r pi is Ni. Then the population particles in the size range rp to rp drp. Then a mass balance
density function ni of particles in this size range is of all particles, as well as of particles in this size range, yields
at steady state for the single-entry separator:
ni N i =r pi 2:4:2j
wstf wst1 wst2 ; 2:4:3a
(Note: we are dealing with a discrete function not a contin-
wstf f f r p dr p wst1 f 1 r p dr p wst2 f 2 r p dr p : 2:4:3b
uous function, hence the subscript i.) This approach is
needed to treat experimental information on different par- Current practice utilizes two descriptors for such a sep-
ticle size fractions collected over different size sieves: that is, aration problem, the total efficiency ET and the grade
particles collected on a particular size sieve have sizes larger efficiency Gr for particles of size rp (Svarovsky, 1977,
than this sieve opening but smaller than the opening size of chap. 3):
the sieve immediately above it through which the particles
fell. If we can characterize the number of particles of a certain wst2
ET ; 2:4:4a
size r pi as Noi , then the mean particle size r p is wstf
1, 0
2.4 Multicomponent systems 59

wst2 f 2 r p f r p to be component 2. We can define the segregation fractions


Gr ET 2 : 2:4:4b
wstf f f r p f f r p Y_ 11 Y rp 1 and Y_ 21 Y_ rp 1 as follows:
1 2

ws f 1 r p1 wst1 f 1 r p2
The total efficiency ET is the fraction of particles of the feed Y_ r p 1 t1 , Y_ rp 1 , 2:4:5
which are exiting through the underflow or the tails stream.
1 wstf f f r p1 2 wstf f f r p2
When ET equals 1, the separation between the solid and the where we have assumed that the mass fraction of the
overflow fluid is complete. If the carrier fluid stream is the particles is equal to the mole fraction of the particles.
preferred one containing finer particles, then obviously Borrowing the definition of the extent of separation from
E T 1s , where s is the cut defined by (2.2.10a) in (2.2.11), we have for the separation of particles of two sizes
terms of only the solid particle flow rates. The grade effi- rp1 and rp2
ciency Gr of particles of size rp is the ratio of the mass of
  ws f 1 r p1 wst1 f 1 r p2 
 
particles of size rp in the underflow to that in the feed. It r p rp Y_ r p 1 Y_ rp 1   t1

: 2:4:6
depends on rp and the separator characteristics. If the par- 1 2 1 2 wstf f f r p1 wstf f f r p2 
ticle classifier or the solidliquid separator is of any use,
Utilizing the relations (2.4.3b), (2.4.4a) and (2.4.4b) for two
then at least all particles of size rmax should be in the
particle sizes rp1 and rp2, we can easily show that
underflow. To generalize,Gr jr p 1: As r p ! 0, Gr tends
to a limiting value defined by (Svarovsky, 1977, chap. 3)  f r p f r p 
 
rp rp Gr r p1 Gr r p2  E T  2 1 2 2 :
 

Q2
1 2 f f r p1 f f r p2
Gr jr p 0 , 2:4:4c
Qf 2:4:7

where Q2 is the total volumetric flow rate of the underflow. If r p1 r max and r p2 r min , then rp r p 1 Q2 =Qf , the
1 2
In other words, even if there is no separation per se of the maximum possible value. Since Q2 6 0, rp rp jmax is never
1 2
finest solid by the separation mechanism from the fluid, equal to 1. On the other hand, as r p1 ! r p2 , rp1 rp2 ! 0. The
the finest solids of the feed are split between exit streams 1 familiar quantity grade efficiency may thus be easily
and 2 since some fluid inevitably leaves with the under- related to the index, , the extent of separation.
flow. The behavior of such types of grade efficiency func- If we are interested in describing the separation of all
tions, along with those for which Gr jrp 0 0, are shown in particles above and below a particular size rs in the over-
Figure 2.4.4. Such functions are also known as Tromp flow and the underflow, we may define component 1 to be
curves. all particles with rp < rs and component 2 to be all particles
We now turn our attention to the description of separ- with rp > rs. Continuing with the assumption of the equiva-
ation of particles of only two sizes, rp1 and rp2. Assume lence of the mass fractions of particles with mole fractions
particles of size rp1 to be component 1 and those of size rp2 of particles, we have in such a case

rmax
rmax
rmax

1.0
Grade efficiency
function for
0.8 Gr r 0
p

0.6
Grade
efficiency
0.4 function for
Gr rp = 0
Gr
0.2

0
Particle size, rp

Figure 2.4.4. Nature of various grade efficiency functions.


60 Description of separation in open separators

rs r max
r max

wst1 f 1 r p dr p wst1 f 1 r p dr p wst1 f 1 r p dr p
r min rs r min wst1 wt1 u21
Y_ 11 ; Y_ 21 : Y_ 21 r max : 2:4:13a
rs r max
wstf wtf u2f
wstf f f r p dr p wstf f f r p dr p wstf f f r p dr p
r min rs r min

2:4:8
Similarly,
The extent of separation for components 1 and 2 defined in wt1 u11
Y_ 11 , 2:4:13b
this manner is then wtf u1f
r max
 r 
 s
 
 leading to
 f r
1 p dr p f r
1 p dr p
  
w u
 t1 11 wt1 u21 

wst1  r min

rs ,

12 s  rs r :

2:4:9 12  2:4:13c
wtf  max
  wtf u1f wtf u2f 
f f r p dr p f f r p dr p 
 
where we recall that wtj is the total mass flow rate of both
 

 r min r

s
species 1 and 2 in the jth stream. In solidliquid separation
If we define the particle size distribution function F(rp) practice, one comes across the Newton efficiency and the
(see Figure 2.4.3(a) for illustration) by reduced efficiency. The Newton efficiency has been defined
rp
as follows (Van Ebbenhorst Tengbergen and Rietema,
1961; Svarovsky, 1979):
Fr p f r p dr p , 2:4:10
r min wt2 u22 wt2 u12
EN : 2:4:14a
we may rewrite (2.4.9) as wtf u2f wtf u1f

Using mass balance relations (2.1.24) and (2.1.25), one can


 
 F r 1 F r 
 1 s 1 s 
12 1 E T  2:4:11 easily show that 12 of (2.4.13c) is the same as EN. Note that
 F f r s 1 F f r s 

if the mass fraction of solids recovered in any stream (e.g.
since wt2 u22/wtf u2f) can be replaced by the corresponding mole
fraction (Wt2 x22/Wtf x2f) and similarly for the fluid, we have
wst1 =wstf 1 E T 2:4:12a
W t2 x 22 W t2 x 12
and EN , 2:4:14b
W tf x 2f W tf x 1f
r max

f r p dr p Fr max 1: 2:4:12b which is the extent of separation for components 1 and 2
r min being separated between product streams 1 (overflow) and
2 (underflow). If we rewrite (2.4.14a) as
Note further that ET in such a case is given by

F f r s F 1 r s wt2 u22

u12 u2f

ws
  !
u22 u1f 1
ET 2:4:12c E 1N 1 t2 1
F 2 r s F 1 r s wtf u2f u22 u1f wstf u12 u2f
if one uses equations (2.4.3a) and (2.4.3b) and integrates 2:4:15
over the size range.5
to illustrate the composite nature of EN, we see that wst2 =wstf
Instead of considering only the separation of particles
indicates how much of the solid in the feed is recovered in the
from one another, consider now the separation of all par-
underflow (and therefore is essentially separated from the
ticles (species 2) from the fluid (species 1) feed or the
feed liquid), whereas (u22 u1f/u12 u2f) indicates how free from
elimination of liquid from the solid-rich underflow (region
liquid this solid in the underflow is.
2). Instead of the segregation fractions defined by (2.4.8),
The reduced efficiency of Kelsall (1966) (Svarovsky,
we will have
1977, chap. 3) is given by
!
wst2
5
From (2.4.4b), the grade efficiency may be related to ET in Qf Q2
E T Q2 =Qf wstf
general by (see Prob. 2.4.2b) E 1T : 2:4:16a
1 Q2 =Qf Qf Q2
1
E T Gr f f r p dr p Gr dF f r p : 2:4:12d When wst2 wstf , E 1T 1, the maximum value, since wst2 
0 0 wstf : Further, note that, for dilute slurries or suspensions,
2.4 Multicomponent systems 61

Dust-free air

Feed air
with dust
Cyclone II

Cyclone I

Dust hopper

Dust hopper

Collected dust

Collected dust

Figure 2.4.5. Two dust-collecting cyclones in series.

 
E N E T Q2 =Qf 2:4:16b Solution (a) Use the results of Problem 2.4.2 (b), and
obtain, in general,
since the solid volume is small compared to the liquid
1
volume so that
Gr f f r p dr p E T Gr dF f r p :
EN
E 1T : 2:4:16c 0 0
1 Q2 =Qf
For the first cyclone,
In solidliquid separation practice, sometimes three or
more phases are encountered. Each phase may be made b1
ET 1 1eb1 r p aearp dr p
 
:
up of only one chemical species or a number of chemical a b1
0
species. But the phases are immiscible. Such a system, if
separated into two product streams, may be described by (b) The air that comes out through the top of cyclone 1
enters cyclone 2 as feed. This air has some dust in it.
indices similar to EN; one such is available in Sheng
The amount of dust entering cyclone 2 is the difference
(1977).
between the total quantity of dust entering cyclone 1
Example 2.4.3. Consider the separation of dust particles and the amount of dust collected in cyclone 1. If we
from air by means of two cyclones connected in series adopt the basis that a total of 1 kg of dust is entering
(shown schematically in Figure 2.4.5). The particle size dis- cyclone 1, then
tribution function of the feed to cyclone 1 is given by (Van
1
der Kolk, 1961)
1 dF f 1 r p aearp dr p :
F f 1 r p 1 earp : 2:4:17a 0 0

The types of cyclones being used are such that the grade Further, the quantity of dust collected by cyclone 1 and
efficiency function is given for the ith cyclone by appearing in the underflow stream 2 is then equal to
Gri 1 eb1 rp : 2:4:17b 1
b1
(a) Show that the total efficiency ET1 for the first cyclone is Gr1 dF f 1 r p Gr 1 f f 1 r p dr p E T 1
a b1
given by 0 0

b1 aearp 1eb1 rp dr p :
ET 1 : 2:4:17c
b1 a
0
(b) Similarly, show that the total efficiency ET2 for the second For the second cyclone, the total quantity of dust entering
cyclone, based on the quantity of dust entering cyclone 1, is between size rp and rp drp is (based upon 1 kg of dust
entering cyclone 1)
a a

ET 2 : 2:4:17d
a b1 a b1 b2 1  aearp aearp 1eb1 rp dr p :
 

(c) What is the total efficiency of the two-cyclone system If the grade efficiency function for cyclone 2 is (1 eb2rp), the
based on the material entering the first cyclone? value of total efficiency ET2 in this case is
62 Description of separation in open separators

1 F f r p,a E T :
 
2:4:18
ET 2 1 aearp aearp 1 eb1 r p 1 eb2 rp dr p

0
The analytical cut size rp,a is not equal to the equiprobable

a a
 cut size rp,50. The relation between these two can be derived,
: for example, for a log-normal particle size distribution.
a b1 a b1 b2
(c) The total efficiency of the series connected system is 2.4.2 Continuous chemical mixtures
ET1,2 ET1 ET2, where ET2 is based on the amount of dust
entering cyclone 1. Therefore Coal-derived liquids, heavy petroleum fractions, vegetable
oils and polymers are mixtures that have very large
b1 a a a
E T 1, 2 1 : numbers of components. It is practically impossible to
b1 a b 1 a b1 b 2 a b1 b 2 a
identify each component by ordinary chemical analysis.
One can no longer use mole fractions of individual com-
Obviously ET1,2> ET1. Other types of connections between two
cyclones are also possible. See Problems 2.4.4 and 2.4.7 for ponents. Traditionally, the pseudo-component approach,
two such arrangements. or the key component approach, has been used to handle
such complex mixtures.
Description of separation in solid particlefluid separ- In the pseudo-component approach, the complex
ation devices by means of the grade efficiency Gr or related mixture is represented by a discontinuous distribution of
functions is often avoided in practice. Instead, a single pseudo-components, where the mole fraction xi of pseudo-
number is used to describe the separation characteristics. component i is represented by a bar in Figure 2.4.6(a)
Such a single number is provided by the cut size, which is (which shows ten components). In the approach of con-
the size of the opening of a hypothetical and ideal screen tinuous chemical mixtures, discrete components are not
achieving the same separation as the device under consid- identified. Instead, the mixture is described by a property
eration. An ideal screen will generate a step function for Gr: density function f(r) of a single distribution variable r
all particles of size smaller than the screen opening will (Figure 2.4.6(b)). The variable r can be the normal boiling
appear in the overflow, whereas all particles of size larger point, the molecular weight, the number of carbon atoms,
than the screen opening will appear in the underflow. the degree of polymerization, etc. (Cotterman et al., 1985;
Actual grade efficiency functions for separators in practice Kehlen et al., 1985). The fraction of molecules in the mix-
are hardly of the step function type; therefore a cut size ture characterized by the range r to r dr is given by f(r)dr
approach cannot be a true substitute for Gr. However, since such that
the cut-size concept is used frequently, we will indicate r max
very briefly a few definitions of cut size.

f rdr 1: 2:4:19
One of the most commonly used cut sizes is the
r min
equiprobable size rp,50, for which the value of the grade
efficiency Gr is 0.50 (Figure 2.4.4). A particle having this size Figure 2.4.6(c) shows the cumulative distribution of com-
has an equal probability of appearing in both the overflow, ponents up to any component i. For continuous chemical
as well as the underflow from the separator. A smaller par- mixtures, we have instead a property distribution function.
ticle will most likely be carried away by the fluid in the The property distribution function F(r) is defined by
overflow whereas a larger particle will most likely be separ- (Figure 2.4.6(d))
ated from the fluid and appear in the underflow (Svarovsky,
r
1979). Note that if Gr is plotted against a normalized particle
radius (rp /rmax), the equiprobable size will be independent f rdr Fr: 2:4:20
r min
of the actual values of the particle sizes for a given problem.
Since the grade efficiency function Gr is needed to These quantities are thus analogous to those we have
know the equiprobable size rp,50, and substantial informa- already defined for a size distributed particle population.
tion is necessary before Gr is known (e.g. ET, f2(rp), and ff Instead of particle size, we have a distribution variable r,
(rp) should be available according to equation (2.4.4b)), which is intrinsic to a given chemical species. For example,
two other cut-size definitions are frequently used in indus- for a flash vaporizer single-entry separator (Figure 2.4.7),
try: the analytical cut size rp,a and the cut size by curve the material balance for 1 mole of feed having a molecular
intersection (Svarovsky, 1979). We will only touch upon the weight density function of ff (M) is
analytical cut size here.
f f M dM f v M dM 1 f l M dM, 2:4:21a
The analytical cut size rp,a is defined such that a hypo-
thetical and ideal screen having this size opening will give where is the fraction of feed vaporized, fv(M) is the
from the feed solid mixture the same value of total effi- molecular weight density function of the vapor and fl(M)
ciency ET as the actual separator. In terms of the feed is the molecular weight density function of the liquid prod-
particle size distribution function, Ff (rp), uct (Cotterman and Prausnitz, 1985). Again,
2.4 Multicomponent systems 63

xi = 1.0
i
Finite-component
(discrete) mixture
xi xi all i
0
xi = mole fraction

Component no. i Component no. i


(a) (c)


F (r = ) = f (r ) dr = 1.0
0

Infinite-component f(r) F(r) f (r ) = Density function


(continuous) mixture
r is a characterizing quantity,
e.g., molecular weight.

Distribution variable r Distribution variable r


(b) (d)

Figure 2.4.6. Discrete and continuous composition for a multicomponent mixture. Reprinted, with permission, from Ind. Eng. Chem.
Proc. Des. Dev., 24(1) (1985), 194. Copyright 1985 American Chemical Society.

Moles
Vapor
Moles

M
Feed
Constant T & P
Moles

Liquid

Figure 2.4.7. Flash vaporizer for a continuous chemical mixture. (After Cotterman and Prausnitz, 1985.)

Mmax Mmax n
X
V x i V i T, P, M i : 2:4:22a
f v MdM 1; f l MdM 1: 2:4:21b
i1
M min M min
In a continuous chemical mixture, we have instead
When a particular quantity, such as the pressure or
volume, of a vaporliquid mixture is to be determined for Mmax
a continuous chemical mixture, the method of moments is
V V T, P, Mf 1 MdM: 2:4:22b
to be adopted. Suppose the molar volume V of a pure
M min
chemical liquid species of molecular weight M can be
expressed as a function of temperature T, pressure P and Such a description of continuous chemical mixtures can
M, i.e. V(M,T,P). If this mixture is an ideal mixture and we also be adopted for polymer solutions containing a
are dealing with a simple n-component system, then the molecular weight distribution. Consider a solution of
molar volume V of the solution is given by polydisperse polystyrene in cyclohexane. The mass fraction
64 Description of separation in open separators

8
Mean = 1.25 105

Mass probability
density 106
6
Variance = 4.28 109
Sol 4

8 2

Mean = 2.19 105


Mass probability

6 0
density 106

Variance = 24.6 109 0 1 2 3 4 5 6 7


Feed Molecular weight 105
4 T = 21.6 C

2 8

Mass probability
Mean = 3.13 105

density 106
0 6
0 1 2 3 4 5 6 7 Variance = 27.1 109
Molecular weight 105 Gel 4

0
0 1 2 3 4 5 6 7
Molecular weight 105

Figure 2.4.8. Phase equilibria in the system (polydisperse) polystyrene/cyclohexane. Flory parameter
0.500.60. Reprinted, with
permission, from Ind. Eng. Chem. Proc. Des. Dev., 24(1) (1985), 194. Copyright 1985 American Chemical Society.

of polymers having a molecular weight in the range M to separators, but are equally applicable here if regions 1 and
M dM in the solution is shown in Figure 2.4.8. The 2 are substituted by product streams 1 and 2.
fraction of polymers in the solution is 0.67. When this Current practice regarding the definition of heavy key
solution is chilled to 21.6 C, it forms two phases, a (Section 1.6) and light key is as follows. The component with
polymer-rich phase called the gel and a solvent-rich the highest molecular weight appearing in the light fraction
phase called the sol. Obviously, the density function of in significant amounts is termed the heavy key. The compon-
polymer molecular weight in the gel is different from that ent with the lowest molecular weight appearing in the heavy
in the sol; the gel has much more of the higher mole- fraction in significant amounts is the light key. Instead of the
cular weight species than the sol as shown (Cotterman molecular weight, other criteria regarding the heaviness or
et al., 1985). lightness of a component may also be used.
Semi-continuous mixtures form a category in between Example 2.4.4 illustrates the selection of heavy key and
the continuous chemical mixtures and the ordinary light key components for the distillation process. Note also
multicomponent mixtures. For example, solvents in a poly- that one can analyze the separator as though there are only
mer solution or light hydrocarbons in a gas-condensate two components, the heavy key (i 2) and the light key
system can be described by discrete concentrations or (i 1). Of course, in such a case the total molar feed flow
mole fractions, whereas the continuous components are rate is to be reduced to Wtf [x1f x2f] for a single-entry
described by a density or distribution function approach as separator, where x1f and x2f refer, respectively, to the actual
just outlined. For an introduction to such systems and the mole fractions of species 1 and species 2 in the actual feed.
basics of calculation procedures needed to describe separ- Thus x1f x2f < 1. But in the hypothetical binary system,
ation, consult Cotterman et al. (1985) and Cotterman and define xlf and x2f such that (Hengstebeck, 1961)
Prausnitz (1985).
x 1f x 2f
x 1f and x 2f : 2:4:23
x 1f x 2f x 1f x 2f
2.4.3 Multicomponent chemical mixtures
Furthermore,
For a multicomponent chemical mixture being fed into a
single-entry separator with only two product streams, W tf W tf x lf x 2f  and W tf x if W tf x if ; i 1, 2,
existing methods of description involve amongst others
2:4:24
separation factors for selected components. Some of these
separation factors, namely in (definition (1.6.6)), ij (def- where Wtf is the total molar feed flow rate in this hypothet-
inition (1.6.8)), etc., were introduced earlier with closed ical binary system.
2.4 Multicomponent systems 65

If a multicomponent feed stream is separated by a constituent of the liquid phase, does not appear to a signifi-
single-entry separator into more than two product streams, cant extent in the gas phase. Although there are many other
the description acquires much greater complexity. Very types of multicomponent, multiphase systems, we will
little is available in the literature for such cases. The best describe one of the preceding types of separation now since
the pseudo-binary approach is valid for both cases.
approach appears to be to concentrate on two specific
species and the respective species-specific product Example 2.4.5 Benzene is to be absorbed in a heavy wash
streams. Perfect separation will then consist of each prod- oil from a nitrogen stream which enters a countercurrent
uct stream containing only a specified species. The various absorber at a total molar flow rate of 10 gmol/s. The mole
indices defined for binary systems in this chapter may be fraction of benzene in this stream is 0.020. The heavy wash
used with due provisions for the distribution of each oil entering the absorber at 1.0 gmol/s contains no ben-
species in each product stream. zene or nitrogen. It is known that benzene is distributed
between the gas and the wash oil according to the relation
Example 2.4.4 Consider a paraffinic hydrocarbon liquid (xbenzene)gas 0.125 (xbenzene)oil for the higher concen-
feed having a composition in mole fractions as given in tration levels near the gas inlet, but that nitrogen is not
Table 2.4.1. absorbed at all. Obtain values of the extent of separation
This liquid feed at a pressure of 15 atm and 90 C is and the separation factor. Consult the countercurrent
introduced into a tall distillation column, which produces a schematic in Figure 2.3.1.
light and a heavy fraction having the compositions given in
Table 2.4.2. Solution Product stream j 1 is the nitrogen stream
The light key component in this case is n-C3H8 because stripped of benzene (almost). Product stream j 2 is
C2H6 is absent in the heavy fraction. Since n-hexane does the wash oil stream leaving the absorber after absorbing
not appear in the overhead vapor, n-pentane is the heavy benzene. Component i 1 is nitrogen; component i 2
key. The component n in in defined by (1.6.6) is then is benzene. In a pseudo-binary calculation scheme, we
n-C5H12. In Table 2.4.3 we calculate in values for all the six neglect the wash oil content in j 2. Similarly, feed stream
components; here i 1, 2, 3 and 4 correspond, respectively, f1 is the nitrogen stream, whereas feed stream f2 is the
to CH4, C2H6, n-C3H8 and n-C4H10, whereas i 6 corres- wash oil stream on a wash oil-free basis. The extent of
ponds to n-C6H14. Naturally in > 1 for i 1, 2, 3 and 4, separation is
but in < 1 for i 6.
 
Y_ 11 Y_ 21 
 
Another type of multicomponent separation involves one  
or more solutes distributed between two solvents that are  W t1 x 11 W t1 x 21 
 ;
 
immiscible and which form two different phases. A third type  W tf 1 x 1f 1 W tf 2 x 1f 2 W tf 1 x 2f 1 W tf 2 x 2f 2 
of multicomponent separation involves a gas phase and a
liquid phase such that the solvent, which is the dominant but x1f2 0, x2f2 0, and also x12 0. Furthermore, Wtf1
x1f1 Wt1 x11 Wt2 x12 Wt1 x11. Therefore
Table 2.4.1. Composition in mole fractions for liquid feed in
 
 W x 
t1 21 
Example 2.4.4 1 :

 W tf 1 x 2f 1 
n-C6H14 n-C5H12 n-C4H10 n-C3H8 C2H6 CH4 So, the species 2 balance is given by
 
0.20 0.25 0.31 0.11 0.09 0.04  W x 
 t2 22 
W tf 1 x 2f 1 W t1 x 21 W t 2 x 22 )  :
 W tf 1 x 2f 1 

Since some benzene always escapes with the nitrogen in


Table 2.4.2. Compositions of light and heavy fractions in product stream j 1, Wt2 x22 < Wtf1 x2f, so < 1 unless we
Example 2.4.4 have complete absorption of benzene. Now,
gmol gmol benzene
 
n-C6H14 n-C5H12 n-C4H10 n-C3H8 C2H6 CH4 W tf 1 x 2f 1 10  0:020 0:20 :
s s
Overhead 0.01 0.32 0.33 0.22 0.12
vapor Using the distribution relation at the absorber gas inlet,
Bottoms 0.35 0.362 0.281 0.007 we have
liquid x benzene gas 0:020
x benzene oil 0:16:
0:125 0:125
Therefore
Table 2.4.3. Value of in for the six components in Example 2.4.4
moles benzene
 
W t2 x 22 wash oil flow rate
n-C6H14 n-C5H12 n-C4H10 n-C3H8 C2H6 CH4 moles wash oil
0:16

x benzene oil
in 0 1 41.2 1710 1:0 1:0  0:1905:
1 x benzene oil 0:84
66 Description of separation in open separators

So, species 1 and 2 between the two regions. On the other


 
 0:1905  hand, consider Figure 2.5.1(b), where the output is such


 0:954,
 that t 02 > t in 0 in in 0
2 ; t 1 > t 1 but t 2 < t 1 . In this case, the output
 0:20 
stream between t in 2 and t 0
1 will have both species 1 and 2. If
a reasonably good separation. But 12 x 11 x 22 =x 21 x 12 we now consider the purity of two regions obtained by
x 11 x 22 =x 21 0 since x12 0 (no nitrogen in wash oil product collecting the outputs between, say, t in 1 and tc and tc and
stream (on a wash oil-free basis or otherwise)). Therefore t 02 , it is obvious that we are dealing with an imperfect
12 , which is not helpful in determining the changes in separation. Both the regions now have some impurity.
the performance of the absorber (if any) since nitrogen is It is now apparent that the description of separation
always absent in the j 2 stream. between species 1 and 2 achieved with such an output stream
can be carried out with the indices developed for a closed
separator with two regions. There are also other indices
2.5 Separation in an output stream with which grew out of the practice of processes using such
timevarying concentration separators (e.g. in chromatographic separations). Before we
In some separation processes, only one product stream introduce these indices, consider the variation in concen-
comes out of a single-entry separator. Further, the molar tration Ci(t) of the ith solute species in the output stream as
flow rates of various species in this product stream as a function of time. This is necessary to estimate the total
measured by a detector (Figure 2.5.1) vary with time in a number of moles of the given species in a given region
manner useful for separation. obtained by collecting the output between specified times.
Consider the molar rates at which two chemical Let m0i be the total number of moles of species
species 1 and 2 present in small amounts in a carrier liquid i introduced into the separator by the feed stream. Assume
or gas stream emerge from the separator. Figure 2.5.1 further that all of m0i has come out of the separator in the
shows two general types of output profile. In Figure 2.5.1 output stream between times t in 0
1 and t i seconds. Let the
(a) between t in 0 output volumetric flow rate be constant at Qf cm3/s. It is
1 and t 1 seconds, the exit gas or liquid stream
contains only species 1, whereas between t in 0 also assumed to be equal to the input volumetric flow
2 and t 2
seconds, the exit stream has only species 2 with rate. The number of moles of species i in the output
t 02 > t in 0 in between t in in
1 and tc (for t c > t 1 ) is given by
2 > t 1 > t 1 . (Here the superscripts in and 0 refer,
respectively, to the times when a particular species appears t c
in the output for the first time and when it disappears mii Qf C i tdt 2:5:1a
completely.) Thus, if we assume the volumetric output of
t in
the separator between t in 0 i
1 and t 1 seconds to constitute
region 1 and the volumetric output between t in 0
2 and t 2 to if region i for t c > t in in
1 is t 1  t  t c . On the other hand, if
in 0
constitute region 2, we have a case of perfect separation of region i for t c > t 1 is t i t t c , then

Feed Separator Output stream


Ci (t )

Detector

Species 1 Species 2 Species 1 Species 2


QFCi (t)
QFCi(t)
gmol/s

gmol/s

0 t1
in in
t2
0
t1
0
t2
in 0 in 0 tc
t1 t1 t2 t2
t t
(a) (b)

Figure 2.5.1. Time-varying molar output rate of solute species in the single product stream from a single-entry separator.
2.5 Separation in timevarying composition 67

Inflection point

Inflection point
QF C1 (t)

A A

tR1

t Wb1 = 4st 1

Figure 2.5.2. Molar output rate of species 1 for a Gaussian profile.

0
h p i
t i t Ri t= 2 ti i , 2:5:5
mii Qf C i tdt: 2:5:1b
relation (2.5.4) becomes
tc
2 3

Note that here Ci (t) is the number of moles of species i per 04 1 2i 2m0 2
QF C i tdt mi p e di 5 pi ei di m0i ,
unit volume of solution collected at any time t. Along with
0
definition (2.5.1a), we have
as required by relation (2.5.2).
0
t i A brief look at the profile of the molar solute output
m0i Qf C i tdt, 2:5:2 rate as a function of time will help in specifying the role of
t in
i
tRi and ti (appearing in the Gaussian profile (2.5.3)) in the
which is also valid for definition (2.5.1b). separation system. In Figure 2.5.2, the Gaussian output
Sometimes it is difficult to specify a value of t 0i where flow rate of species 1 has been shown as gram moles of
Ci (t) 0; one then uses t 0i . Similarly, one uses species 1 per unit time ( Qf  Cl(t)) against time. With Qf
t in constant, the concentration Cl(t) quickly rises to a peak
1 . At this time, we require a functional description
of Ci (t) vs. t to make estimates of mii to be used in various value at t tR1 and then decreases. If two tangents are
indices. A Gaussian profile is often close to what is drawn at the two inflection points A and A0 and extended in
observed in many separator outputs (e.g. the so-called both directions, they intersect above the peak at t tR1.
chromatograms, Figure 2.5.2, where tR1 is called the Furthermore, the distance between the intersections of
retention time for species 1): these two lines with QfCl(t) 0 (the base line) is given as
Wb1; for a Gaussian profile, Wb1 4t1, where t1 is the
1 t Ri t 2
"  #
Qf C i t 1 standard deviation of the species 1 profile. When t tR1,
p exp : 2:5:3
m0i ti 2 2 ti the height of the peak is obtained as follows:

m01
Note that with t 0i and t in 1 , such a profile yields Qf C l t R1 p : 2:5:6a
2 8 92 3 t1 2
<t t =
1 1 R One can therefore rewrite the Gaussian profile (2.5.3) as
QF C i tdt m0i p exp4 i 5dt:
ti 2 2 : ti ;
1 t Ri t 2
"  #
Qf C i t Qf C i t C i t
exp :
2:5:4 Qf C i t Ri Qf C i tjmax C i tjmax 2 ti

On defining 2:5:6b
68 Description of separation in open separators

An approximate measure of the time when species 1 t c t R


mii 1 p i 1 2
appears in the output stream is therefore t R1. Similarly, p 2ti exp2i di p exp0i d0i
m0i



for species 2, one obtains t R2. These time estimates would 0 0
be exact only if t1 0 or t2 0. It is also evident from 1 t c t R 1

erf p i , 2:5:13
Figures 2.5.1 and 2.5.2 that if t R1 and t R2 are wide apart, 2 2 ti 2
then, unless t1 and t2 are very large, the two concen-
where the value of the second integral with variable
tration profiles will not overlap. p
0 i i is =2. Note that mii =m0i is simply Yii, the
 
A simple index commonly used for describing separ-
extent of segregation of species i in region i. However, this
ation in such an output stream is the resolution Rs of two
derivation is valid for t c > t in in
i and t c t t i .
adjacent profiles or bands or chromatograms:
For the second type of region i with t c > t in i and
0
2t R2 t R1 t i t t c , we use definition (2.5.1b) to obtain
Rs : 2:5:7
W b1 W b2
1 t Ri t 2
"  #
m0i
It is merely the distance between the two concentration mii p exp dt: 2:5:14
ti 2 2 ti
peaks divided by the arithmetic average value of Wb1 and tc

Wb2, each referred to as a band width. The larger the value On using transformation (2.5.5) and splitting the integral,
of Rs, the greater the separation. Separation is perfect when we get
Rs ! . Further, the smaller the values of t1 and t2 for
given tR1 and tR2, the better the separation, since the overlap 0
mii 1 2 1
region becomes smaller. Usually, when Rs 1.5, the separ- p exp i d p exp2i di 2:5:15
m0i



ation between two chromatographic profiles is quite good, 0 t c t R
p i
2 ti
with only a small section of each region (less than 1%) p
contaminated by the other species. since (t c t Ri = 2 ti ) is negative here. Changing the vari-
00
When, t1 t2 t and both the outputs are Gaussian, ables in the second integral to i i , we get
the definition of resolution is simplified:
mii 1 1 tpRi2t c 1 1 t Ri t c

00 2 00
p ti exp d erf p :
i i
m0i 2 2 2


t R2 t R1 2 ti
Rs : 2:5:8 0
4 t
2:5:16
In general, however, the standard deviations for the two
solutes are not equal. If we can express t1 and t2 thus With region 1 as t c > t in
and t c t
1 we can use t in
1 ,
relation (2.5.13) to determine Y11 and Y21. The index ,
t1 t and t2 t t , 2:5:9 the extent of separation for such a system of two solute
then species 1 and 2 distributed between regions 1 and 2, is
therefore given by (Rony, 1968b)
t R2 t R1
Rs 1 t c t R t c t R
 

i: 2:5:10
absjY 11 Y 21 j abs erf p 1 erf p 2 :
h

4 t 1 2 t 2 2 t1 2 t2
t
Note that if t << 2t, expression (2.5.10) for Rs is reduced 2:5:17
to (2.5.8). For additional considerations, see Vink (1972).
Other measures describing the degree of overlap between Here t R1 and ti are characteristic parameters for the solute
non-Gaussian outputs have been considered by Dose and species i and the separation system. If t R2 t R1 and tc
Guiochon (1990). halfway between t R1 and t R2 , both error functions will
We now propose to calculate the extent of separation, have an upper limit whose magnitude is infinitely large.
, for a typical chromatographic output. We need to calcu- Therefore, from (2.5.17) we have
late Y11 and Y2l. Substitution of profile (2.5.3) in definition t c t R1 t c t R2 3
2
p p
 
(2.5.la) leads to



6 2 2 t1 2 2
 
1 2 t2 2
7
abs6p e d p e d7
6 7
t c 2 4 5
1 t Ri t 2
"  #
m0i 0 0
 
mii p exp dt: 2:5:11
 
ti 2 2 ti 2
3 

1  2 2
 
2 2 
abs4p e d p e d5
On using transformation (2.5.5), we obtain 2
0 0
 

exp2i 
t c t R
2 3
mii

p i
1 2  1
 
p di , 2:5:12
2 ti 02
abs41 p e d0 5 abs1 1 1:

m0i 2
 2
0

which on rearrangement yields 2:5:18
2.5 Separation in timevarying composition 69

The maximum extent of separation of 1 is achieved t t t t


2  3 2  3
1 erf pR22 c 1 erf pc 2R1
even if tc is not halfway between the two tR values 1 4 t2
 2
5 4 t1
5:
t t t t
 
as long as each jt c t Ri j is large. Obviously, if t R1 t R2 1 erf pc 2R 1 1 erf pR22 c
t1 t2
and t1 t2, 0. Thus, there is no separation
when the two concentration profiles have identical shape This leads to
and come out at the same time. Separation under any
tR tc 2

2
tc tR
 

other conditions then depends on the nature of the erf p2 erf p 1 :
two profiles, their locations and the location of tc, the 2 t2 2 t1
cut point.
It is also possible to calculate the value of the separ- Since t1 t2 t (say), this means either
ation factor 12 and some of the other types of indices !
tR tc t c t R1
 
defined in Chapter 1 for such an open system. erf p2 erf p ,
The interrelationships been various separation indices 2 t 2 t
( and Rs, for example) may also be developed under
which means t R2 t c t c t R1, i.e. t c t R1 t R2 =2, or
certain conditions. See Example 2.5.1 and Problems 2.5.1
and 2.5.2. !
tR tc t c t R1
 
For a description of multicomponent separation in erf p2 erf p
:
chromatographic outputs, the semi-quantitative approach 2 t 2 t
by Stewart (1978) involving the concept of a resolution
matrix is likely to be useful to interested readers. Since erf (x) erf (x), this means t c t R2 t c t R1
i.e. t R1 t R2 , which is a trivial result since we know
Example 2.5.1 Consider the time-varying molar outputs of
t R2 6 t R1. Therefore t c t R1 t R2 =2 and
two species 1 and 2 from a chromatographic system (Figure
2.5.1(b)). Assume both outputs, which are overlapping each
t R2 t c t R2 t R1
2   3  
other, to be Gaussian and t1 t2. If the cut point is located 1 erf p  1 erf p
 2 t 5 t R1t R2  2 2 t  :

such that the impurity ratio in each region is the same, and if 1 2 4 t t t
t R2 t R1
it is known that m01 m02 , develop a relation between this 1 erf cp2R 1  c 2 1 erf 2p2
t t

impurity ratio and the resolution. Calculate the values of this


impurity ratio for Rs 0.2, 0.6, 1.0 and 1.4. But the resolution in this case is Rs t R2 t R1 =4 t . Thus
Solution The impurity ratios for regions 1 and 2 are defined p
by relation (1.4.3), where region 1 is from, say, t in
1 to tc and 1 erf 2Rs
1 2 p
region 2 is from tc to t 02 . We have 1 m21 =m11 1 erf 2Rs
and 2 m12 =m22 ,
m21 m21 =m02 m02 m21 =m02 Y 21 is the required relation. This has been indicated by
1 Glueckauf (1955a) as 1:1.
m11 m11 =m01 m01 m11 =m01 Y 11 p
The values of erf ( 2Rs ) for Rs 0.2, 0.6, 1.0 and 1.4
since m02 m01 . Similarly for 2: are, respectively, approximately 0.310, 0.7658, 0.9545 and
m12 m12 =m01 m01 m12 =m01 Y 12 0.9949. The corresponding values of 1:1 are 0.526, 0.133,
2 : 0.0233 and 0.00256. Thus a resolution value of 1.4 implies
m22 m22 =m02 m02 m22 =m02 Y 22
excellent separation with very low impurities in each band.
From (2.5.13), For a graphical relation between 1:1 and Rs over a wider
1 tc tR range of Rs, see Said (1978).
 

Y 11 erf p 1 1 :
2 2 t1 The preceding treatment considered separation in a time-
But varying output from a separator of the chromatographic
1 tc tR type. Similar methods of describing separation, namely
 

Y 12 1 Y 11 1 erf p 1 :
2 2 t1 Rs, , 1, 2, etc., may also be adopted for describing the
separation between two concentration profiles in
This last result could also have been obtained from relation Figure 2.5.1 if the abscissa is a distance coordinate z
(2.5.16). Similarly, instead of a time coordinate t. One then replaces t Ri by zi,
1

tR tc

ti by zi, Ci(t) by Ci(z), tc by zc, etc. The mathematical
Y 22 1 erf p2 and formulae will be identical with the proper substitutions
2 2 t2
made, and we may conclude that no special treatment is
1 tR tc
 

Y 21 1 Y 22 1 erf p2 : needed to describe the separation between two concen-


2 2 t2
tration profiles spatially displaced but having some
Therefore, as required in Example 2.5.1, overlap.
70 Description of separation in open separators

Problems
2.2.1 The absorption factor, A, in the absorption of species i in a flowing gas stream by a flowing solvent stream is
defined by

molar liquid flow rate liquid mole fraction



A  :
molar gas flow rate gas mole fraction i, equilibrium

The stripping factor, S, which describes the stripping of a volatile species i into a flowing gas stream from a
flowing liquid stream is the inverse of A. The extraction factor for any species is defined in solvent extraction by

mole fraction in extract extract molar flow rate



E  :
mole fraction in raffinate i, equilibrium raffinate molar flow rate

Relate A, S and E to k_0 i1 :Ans: S 1=k_0 i1 ;E k_0 i1 :


hf
2.2.2 Obtain the following relation between the separation factor ht
12 , the heads separation factor 12 , the cut and
x11 for a single-entry separator and a binary feed:

hf ht
12 11
12 1 :
1 ht
12 11 x 11

2.2.3 Consider the separation of a binary mixture in two single-entry separators connected together as shown in
Figure 2.P.1. The tails stream from separator 2 is recycled back to the feed stream to separator 1. The mole
fraction of the ith species in the jth stream of separator n is indicated by xij(n) (for n 1,2). If these two
separators are operated such that6

x i2 2 x if 1
hf ft
and if the values of both 12 and 12 are the same for both separators, show that
hf ft 1=2
12 12 ht
12 :
hf
(Hint: Use mole ratios to convert separation factor definitions to X 11 n 12 X 1f n.)

2.2.4 Consider the elimination of a surface active impurity from water by the foam fractionation process as shown
in Figure 2.P.2. An inert gas is bubbled through the impure water generating a foam rich in the surface

xi1(2) Light fraction product

Separator 2

xif (2)

xi1(1) xi2(2)

Separator 1

xi2 (1) Tails product stream


Wtf (1)
Feed
xif (1)

Figure 2.P.1. Two single-entry connected separators with tails recycle.

6
When two streams to be mixed together are such that they have the same composition, we encounter the so-called no-mixing
condition.
Problems 71

Gas out

Foam
breaker

Foam rising in the


column Surfactant-
rich water
Waste water Purified
feed water

Inert gas

Figure 2.P.2. Water purification by foam fractionation removal of detergent impurities in a column.

active impurity. The foam rises up the column and is collapsed to obtain an impurity-rich water phase
and the inert gas, which is recirculated to the column bottom for renewed foam making. The surfactant
concentration in the feed is in the range 106 gmol/cm3. The decontaminated water has a surfactant
concentration 0.01 times that of the feed. The molar water flow rate in the impurity-rich collapsed foam
is 3% of the feed molar flow rate, Wtf.
(a) Analyze this double-entry separator as a single-entry separator by drawing a suitable envelope in the
figure and showing any other necessary arrangements.
(b) What is the value of the extent of separation?
(c) Show that the value obtained in (b) is very close to that obtained if the decontamination factor is 1000.
(d) Although the extent of purification of water in (c) is much larger than that in (b), does not reflect it.
Define impurity ratios for the X flow system as _ 1 W 21 =W l1 and _ 2 W12 =W22 . Is the new impurity
ratio _ j or I j log10 _ j or I I j more sensitive for such a case? (Note: Ij or I is the purity index for the
flow system.) j

2.2.5 Thorman et al. (1975) have used two permeators of the type described in Example 2.2.4 in series to obtain O2-
enriched air containing 0.310 mole fraction O2 from feed air (0.209 mole fraction O2). The feed air flow rate to
the first permeator is 0.665 std. cm3/s. The O2-enriched permeated stream containing 0.250 mole fraction O2
and having a flow rate of 0.365 std. cm3/s is introduced as feed after compression into the second permeator,
which has a permeate composition of 0.310 mole fraction O2. The permeate flow rate from the second
permeator is 0.151 std. cm3/s. The second permeator is somewhat smaller (silicone capillary length 217
cm) than the first permeator (silicone capillary length 691 cm). Obtain an estimate of the overall separation
using ht
12 , and .

2.2.6 To recover as pure a C8 hydrocarbon liquid as possible from a feed containing small amounts of a C5
hydrocarbon, it is proposed to use a flash separator with a fraction of the bottoms liquids recycled to the feed.
(See Figure 2.2.1(c) for a schematic; replace decane by C8 hydrocarbon and butane by C5 hydrocarbon.)
A fraction 0 of the bottoms liquid fraction is compressed and heated before it is mixed with fresh feed. It is
proposed to compare the purification ability of the recycle system with the no-recycle system. Assume that the
equilibrium ratio Ki1 for the flash separator for the ith species is independent of recycle and depends only on
the pressure and temperature of the flash, which do not change much with recycle. Assume further that the cut
for the flash separator is independent of recycle (not strictly correct). Do not introduce any simplification
based on the C5 concentration in the feed.
(a) Develop a relation between xr22 and x2f containing only 0 , and K21. Ans.

x 2f 10 1
x r22 :
K 21 10 1

(b) Show that x r22 is always greater than x22 (corresponding to no recycle).
72 Description of separation in open separators

(c) Show further that the fraction of the fresh feed recovered as the liquid fraction is lower due to the recycle.
Note: C5 hydrocarbon goes much more into the vapor phase than C8 hydrocarbon, which has a very low volatility.

2.2.7 Consider a single-entry separator with a dilute feed solution of heavy species 2. If a fraction 0 of the tails stream
hf ft
is recycled to the feed stream and the separator operates with the same values of , 12 and 12 as does the non-
recycle separator, show that the following are true.
(a) The apparent cut for the recycle separator based on the light fraction product flow rate from the system
and the fresh feed flow rate is greater than , based on the light fraction actually leaving the separator and
the feed rate actually entering the separator.
ft ft
(b) The apparent value of 12 x r22 =x 2f is greater than the actual value of 12 for the separator feed and tails stream.
(c) The condition under which the extent of separation based on the net product flow rates from the recycle
system and the fresh feed is greater than that for a no-recycle system is (Sirkar and Teslik, 1982; see
footnote 3, p. 49)
ft
" #
0 12 1 1
< ft
ft
:
112 12 12

Comment in general on the utility of such systems.


2.2.8 Consider a single-entry separator for the separation of a binary mixture of species 1 and 2 (Figure 2.2.1(a)). We
wish to compare its separation performance with that of a single-entry separator having a fraction of its light
fraction recycled to the feed stream. The operational conditions are as follows:
hf ft
(a) the heads and tails separation factors 12 and 12 have the same value for both the recycle and the non-
recycle separator;
(b) the recycle separator operates such that its apparent cut (definition (2.2.27)) is equal to the cut of the non-
recycle separator.
Show that under such conditions for any feed mixture r > holds and that the recycle separator is therefore a
better separation device (Sirkar and Teslik, 1982; see footnote 3, p. 49).
2.2.9 The separative power U of a single-entry separator for a binary feed stream is defined by Dirac in terms of a
value function Va(xij) of composition xij as

U W t1 V ax i1 W t2 V ax i2 W tf V ax if ,

where the value function Va(xij) indicates the value of one mole of a given fluid stream of composition xij, and it
changes as the composition of the fluid stream changes. Show that the above relation may also be expressed as
follows (Lee et al., 1977a):
ft hf hf ft ft
! ! ! ! ! !
U 12 1 1 12 X 1f 12 X 1f 12 ht
12 12 X 1f X 1f
V a V a V ax 1f ,
W tf ht
12 1 1 X 1f hf
1 12 X 1f 1ht
12
ft ft
12 X 1f 12
ft
12 X 1f
hf ft
where the feed light component mole ratio X 1f is given by X 1f x 1f =1x 1f , and 12 , 12 and ht
 
12 are defined
by (2.2.3)(2.2.5).

2.2.10 Consider a single-entry separator separating two species g and h in the feed stream. The molar flow rates of
species g and h in the feed stream are, respectively, G0 and H0. The corresponding molar flow rates of species g
and h in stream 1 and stream 2 are, respectively, G1, H1 and G2, H2. Stream 1 is the light fraction. The following
quantities are defined as follows:
a mole percent of g in the feed;
b mole percent of g in stream 1;
j amount of stream 1 as a percentage of the feed.
(1) Define the extent of separation in terms of the quantities defined here (G0, H0, G1, H1, G2, H2, a, b, j).
(2) A new separation efficiency has been defined as
ba
100j :
a100 a

Show that 100.


Problems 73

2.4.1 Particle size distribution may be determined in practice for liquid suspensions by sieving, i.e. using screens/sieves
having different openings and determining the weight of the particles retained by the sieve of a particular size. Part
of the following information is adopted from Randolph and Larson (1988):
ws MT 0.15 g/cm3 of particle free liquid volume; s 2 g/cm3;
particle volume shape factor 0.6.
The sieving process yielded the following information for two sizes of particles: the fraction of particles (ws =ws )
around the two sizes, 100120 m (av. 100 m), 200220 m (av. 210 m), are 0.15 and 0.04, respectively.
Calculate the values of the population density n(rp) for these two sizes in units of numbers/cm3 m.
(Hint: Employ (2.4.2e) over the size range.) (Ans. 704 numbers /cm3 m; 27 numbers/cm3 m.)

2.4.2 (a) Particle removal effectiveness of dead-end filters (Chapter 6.3-3.1) is determined by fractional penetration
out
P ,
in
where out and in are the particle volume fractions at the outlet and the inlet of the filter, respectively. An
additional index is the log reduction value (LRV) of the filter:

LRV log10 1=P :

Identify the indices in Chapters 1 and 2 which are closest to these two indices.
(b) Obtain the following relation for the total efficiency ET in a solidliquid separator operating as a classifier
as a function of the grade efficiency Gr of particles of size rp:

1
E T Gr r p dF f r p ,
0

where Ff(rp) is the particle size distribution function of the feed (0  Ff(rp)  1). Develop a relation to predict
f1(rp) from a knowledge of Gr, ET and ff(rp). (Ans. f 1 r p f f r p  1Gr =1E T :
2.4.3 For classification of particles above size rs and below size rs, respectively, into the underflow and the
overflow of a solidliquid classifier, define component 1 to be the undersize particles and component 2
to be the oversize particles. Show that the extent of separation based on mass fractions in such a case is
given by
 
ws wst2  u22 u21 
t1 :
wstf wstf  u1f u2f 


Compare its numerical value with that of another efficiency E, defined in the literature (McCabe and Smith,
1976, p. 920) as

ws wst2 u21 u12


E t1
wstf wstf u1f u2f

for the case where u2f 0.540, u21 0.895 and u22 0.275, obtained for a rs value equal to 0.75 mm.
Assume that mass fractions of particles are equivalent to mole fractions. (Ans. 12 0:61; E 0.63.)

2.4.4 Consider two cyclones in a series for dust cleaning of air, such that the dust collected in the underflow of
cyclone 1 is withdrawn, along with a small amount of air, and introduced as feed to the smaller cyclone 2,
whose overflow gas stream containing some dust is recycled to the fresh feed air stream entering cyclone 1.
Assume that the grade efficiency functions for cyclones 1 and 2 are given, respectively, by (Van der Kolk, 1961)

Gr 1 1eb1 r and Gr 2 1eb2 r :

(a) Show that if the cleaned air from cyclone 1 and the dust from cyclone 2 are the net products, and if r kg of
dust of certain size r is recycled from cyclone 2 for 1 kg of dust of the same size in the fresh air feed to
cyclone 1, then
74 Description of separation in open separators

Gr 1 Gr 1 Gr 2
r :
1 Gr 1 Gr 1 Gr 2

(b) Show that the total efficiency E T 1, 2 . of this arrangement of two cyclones is given for a feed size distribution
function Ff1(r) 1 ear by

aear 1 eb1 r eb2 r eb1 b2 r
 
E T 1, 2 dr:
f1 eb2 r eb1 b2 r g
0

2.4.5 A disk centrifuge has the following grade efficiency function (Svarovsky, 1977, chap. 7)

r 2p
Gr for r p  r max ; Gr 1 for r p > r max:
r max 2

It is known that the feed particle size density function is Gaussian with an average value r p and a standard
deviation of r . (For the limits of integration, a value of infinity may be substituted for rmax. However, in the
integrand, retain rmax.) Obtain an expression for a total efficiency ET as a function of r p , rmax and r :

2.4.6 The feed solids in a slurry to a hydrocyclone obey the following log-normal law:

nr p nr g 2
" #
dFr p 1
p exp ,
dnr p n g 2 2n g 2

where F(rp) is the particle size distribution function, with g being the standard deviation and rg being the mean
particle size. The feed volumetric flow rate is 6.35  104 m3/s. The feed solids concentration is 30 kg/m3. The
underflow volumetric flow rate is 3.5  105 m3/s and the underflow solids concentration is 314.2 kg/m3.
Determine
(1) the total efficiency ET;
(2) the reduced efficiency of Kelsall;
(3) the particle size distribution function F(rp).
2.4.7 Consider the separation of dust particles from air by means of two cyclones connected in the following
fashion. Feed air containing dust enters cyclone 1; the feed particle size distribution is F f 1 r p 1ear p .
The underflow from cyclone 1 is introduced with a small amount of air as feed to a small cyclone 2. The
underflow from cyclone 2 is collected as dust from the system. The overflow air from cyclone 2 is mixed
with the overflow air from cyclone 1 to obtain the cleaned air. The grade efficiency of the ith cyclone is
given by

Gri r p 1ebi r p :

(1) Show by actual calculation the total efficiency of the first cyclone, E T 1.
(2) Based on 1 kg of dust entering cyclone 1, determine step-by-step the total efficiency E T 1, 2 of the two-
cyclone system.
2.4.8 Consider the separation of a ternary gas mixture of species 1, 2 and 3 through a membrane separator with
two different types of membranes M0 and M00 such that species 1 appears preferentially in product stream
j 1 from membrane M0 , product stream j 2 from membrane M00 is enriched in species 2, while the
tails stream j 3 is enriched in species 3. If the gas composition everywhere inside the separator is
indicated by xi3, the mole fraction of species i in tails stream 3, define the following separation factors
(Sirkar, 1980):
(a) 0 1n and 00 1n for species 1 in the product streams 1 and 2, respectively, with respect to all other species
n 2, 3 and the reject stream j 3;
(b) 1n for species 1 and all other species (n 2, 3) with respect to product streams j 1 and j 2. Show that
1n 0 1n =00 1n .
 

2.5.1 It has been pointed out in Section 2.5 that separation in the output stream from a chromatographic separator
depends on the location of tc, the cut point. Assume t1 t2 t and Gaussian output streams for two species
1 and 2. Use the extent of separation to obtain the value of the optimum location of the cut point and the
Problems 75

corresponding optimum value of the extent of separation. (See Problem 1.5.1 prior to solving this problem.)
With this optimum value of , show that (Rony, 1968b)
p
opt absjerf 2Rs j,

where Rs is the resolution for the given problem.

2.5.2 Obtain an expression for the separation factor 12 between two solute species coming out of a chromatographic
separator.
3

Physicochemical basis for separation

The preceding chapters introduced first the notion of sep- 3.1.1 Nature of displacements
aration and then a variety of indices to describe separation.
The separation of a mixture involves the setting apart of the
These indices were used to characterize quantitatively the
mixture components present in a given region. When per-
amount of separation achieved in a closed or an open
fect separation is attained, each component occupies a
separation vessel. The quantitative description included
separate region where no other component is present.
systems at steady or unsteady state involving chemical or
The total volume of all such regions may or may not be
particulate systems. Systems studied were either binary or
equal to the volume of the region originally occupied by
multicomponent or a continuous mixture. Not considered
the mixture. To achieve this separation, each component
in these two chapters was the fundamental physicochem-
must move selectively toward its own designated region.
ical basis for these separations; appropriately, this is the
Therefore, molecules of each component undergo
focus of our attention in this chapter.
displacement toward their own region during a separation
In Section 3.1, we distinguish between bulk and rela-
process.
tive displacements and describe the external and internal
The initial mixture, as well as the final separated state,
forces that cause separation-inducing displacements. This
may consist of either a single phase or a collection of
section then identifies species migration velocities and the
immiscible phases. If separation is desired in a feed con-
resulting fluxes as a function of various potential gradients.
sisting of two immiscible phases, then each component has
Section 3.2 is devoted to a quantitative analysis of separ-
to be selectively displaced toward its designated phase.
ation phenomena and multicomponent separation ability
Such component-specific displacement and the separation
in a closed vessel as influenced by two basic types of forces.
achieved thereby may or may not lead to pure phases. It is,
The criteria for equilibrium separation in a closed separ-
however, a prerequisite to any separation.
ator vessel and individual species equilibrium between
The direction and the rate of displacements of mol-
immiscible phases are covered in Section 3.3. Section 3.4
ecules of a component (or particles of a certain type) will in
treats flux expressions containing mass-transfer coefficients
general depend on the nature of the chemical species (or
in multiphase systems. Flux expressions for transport
particle), the type, magnitude and direction of forces acting
through membranes are also introduced here.
on the chemical species (or particle) and the surrounding
medium. Such species movements take place spontan-
3.1 Displacements, driving forces, velocities eously at the microscopic level in response to conditions
and fluxes imposed on the separation system (Sweed, 1971, p. 175).
When a particular component in a mixture is displaced in a In most separation processes, bulk displacements also
given direction, it moves with a certain velocity. This velocity take place simultaneously. Generally, bulk displacements
leads to a flux of the species, which is the molar rate of move molecules of all components at the same speed and
species movement per unit area in any given frame of refer- in the same direction. On the other hand, the spontaneous
ence. The nature of the displacements and the forces that microscopic species-specific movement in response to
cause the displacements leading to species velocity and flux forces acting on a particular species (mentioned in the
are considered first in this section. Expressions for species preceding paragraph) is identified as relative displacement
velocities and fluxes are then studied to provide the founda- (Giddings, 1978). These two types of displacements may
tions for a quantitative analysis of separation later. occur simultaneously or consecutively (Sweed, 1971).
3.1 Forces, displacements, velocities and fluxes 77

species-specific forces are: chemical potential gradient,


electrostatic potential gradient, centrifugal force field,
gravitational force field, magnetic force field, thermal gra-
dient, etc. Of these, only the chemical potential gradient is
Lighter phase referred to as an internal force; the others are caused by
devices or phenomena external to the separation system.
Vessel A We have described above the notions of bulk displace-
ment and relative displacement primarily for molecules.
They are equally applicable to the separation of macromol-
ecules, colloids or macroscopic particles from the continu-
ous phase (or to fractionation of macromolecules,
Heavier phase particles, etc.); however, the detailed mechanisms and
methods of description may be different. For example, as
Vessel B we will see later in deep bed filtration and aerosol filtration,
particles of different sizes and therefore different masses
Figure 3.1.1. Decantation of upper phase from vessel A to vessel B. may have different inertial forces in an accelerating or
decelerating flow field. With larger particles, certain types
of bulk motion may, therefore, lead to a relative displace-
Bulk displacement may be caused by fluid flow or
ment. Even for gas molecules, the persistence of velocity
direct mechanical conveying of a complete phase or frac-
phenomenon creates different displacements for mol-
tion from one location or vessel to another. Consider, for
ecules of differing masses moving at a given bulk velocity
example, the mechanical decantation of the upper phase
when an impingement on a target gas occurs (Anderson,
from vessel A to vessel B. In vessel B, additional separation
1980).
takes place because of the redistribution of components of
the upper phase in vessel A between itself and the lower
phase in vessel B (Figure 3.1.1). But all the components in 3.1.2 Forces on particles and molecules
the upper phase of vessel A are displaced to the new vessel
A variety of external forces are of use in separation. Of
B at the same rate and in a nonselective fashion. Such a
course, the internal force of chemical potential gradient
procedure is followed in laboratory decantations as well as
has great importance in chemical separations. We will
in industrial decanters.
begin, however, by treating external forces. Of these, the
Bulk displacement by fluid flow is much more
gravitational force due to the earth is perhaps the most
common. When a multicomponent mixture moves in a
familiar one and is therefore appropriate for consideration
vessel or in a region of a vessel at a certain bulk velocity,
now.
the fluid motion carries all the components, in general, at
the same velocity. In the case of a flat velocity profile with
convective transport being dominant over diffusive trans- 3.1.2.1 Gravitational force
port, the fluid flows like a plug (Froment and Bischoff,
In the external gravitational force field, the work required
1979), and every species has the same displacement and
to raise a particle of mass mp from a location of height z1
velocity. If no forces are present to impart different dis-
(where the gravitational potential is 1 gz1 ; is a scalar
placement rates to different components, all components
quantity) to one of height z2 (Figure 3.1.2A) in free space is
will be non-selectively carried by bulk fluid motion and
mp(gz2  gz1) for z2 > z1.2 Here the positive z-coordinate is
there would not be any separation.1
vertically upward. For a differential displacement dz, the
Relative displacement takes place when different forces
vertically downward gravitational force F is related to dW,
acting on molecules of different species (or on different
the differential amount of work done on the particle and
particles) cause molecules of one species to move relative
d, the change in gravitational potential, by a force F in
to those of the other species (similarly for particles). Such
the positive z-direction acting over the distance dz:
motions lead, in general, to separation, whether the mix-
ture as a whole has a bulk velocity or not. Examples of such dW F  dz mp d F dz mp g dz; 3:1:1

where the magnitude of the force per unit mass of the


1
particle, F^ , is
There can be separation processes in which bulk displacement
could vary from location to location due to gradients in bulk
velocity. Additional complexities and sometimes separation can
2
be achieved by a time dependence in the bulk displacements. g is acceleration due to gravity, 980 cm s2. The unit for force is
The effects of spatial and temporal dependence of bulk the newton or kg m/s2. The unit for work is the joule or newton-
displacement will be treated in later chapters. meter (N-m).
78 Physicochemical basis for separation

Figure 3.1.2B. Centrifugal force of magnitude mpr2 ( mpv2/r)


acting radially outward on a particle of mass mp rotating in a
circle of radius r with a tangential velocity v r.

For a particle in the gravitational field, the net external


force, F ext
net , will remain constant if mp, p and are constant
Figure 3.1.2A. Gravitational force of magnitude mpg acting on a in the z-direction. In a separation system, one can create a
particle of mass mp vertically downward. condition such that the surrounding fluid composition,
and therefore the fluid density , change with the height z.
F d
F^ g: 3:1:2 The net external force on the particle will now depend on
mp dz
the vertical location of the particle. A vertical gradient in
In vector notation, the vertically downward gravita- fluid density may then be considered as an additional
tional force per unit mass may be described as source of an external driving force on a particle along with
the gradient of gravitational potential. Clearly, such an
F
F^ r: 3:1:3a additional external force requires the existence of the grav-
mp
ity force. Note that F ext
net will be zero if for a particle or
The relevant z-component in vector form is i for molecules of species i. Further, the net external
force vector per unit particle mass is g(1  /p)k.
F^ z k d=dzk: 3:1:3b
The vector gravitational driving force per unit mass of the
3.1.2.2 Centrifugal force
particle due to the external force field is simply the nega-
tive of the gradient of the scalar potential of that force field. If a particle rotates at an angular velocity of radian/
Here increases with positive z; nature, i.e. gravity, spon- second (rad/s) in a circle of radius r (Figure 3.1.2B), the
taneously drives the particle to a lower . centrifugal force3 on the particle per unit mass in the out-
The movement of the particle of mass mp under the ward radial direction is
action of the gravitational force considered above, how-
F^ F=mp r2 r d=drr: 3:1:6a
ever, assumed free space. If this particle were instead
immersed in a fluid of density , a buoyancy force would Here is the scalar centrifugal force field potential and r is
act on the particle in the vertically upward direction. The the unit vector along the outward radial direction. If a fluid
net external driving force on the particle acting vertically particle of mass density is rotated as a rigid body in a vessel
downward would be such that is constant, the centrifugal driving force on a unit
! ! mass of the fluid particle is also given by the above relation.
ext mp
F net mp g  g mp g 1  ; 3:1:4 The vector centrifugal force acting on one gram mole of
p p
molecules of species i in the radially outward direction is
where p is the mass density of the particle. This force acts F M i r2 r: 3:1:6b
whether the particle moves or not. If, instead of a single
particle, we have one mole of molecules of the ith species, From relation (3.1.6a), we obtain
the total gravitational force on one mole is
   

F ext
net M i g 1  ; F ext
net M i g 1  k; 3:1:5
i i
3
The tangential velocity v of the particle is r. The magnitude of
where Mi is the molecular weight and i is the density of the centrifugal force is m v2 =r m r2 . The Coriolis force is
the ith species. (To develop this expression, follow the also present under these conditions. However, it may be
procedure illustrated later for a centrifuge in (3.1.51).) neglected in comparison to the r2 term.
3.1 Forces, displacements, velocities and fluxes 79

y Electrode with decreasing in the positive z-direction. The electrical


Electrode potential ~ where N
charge on a single ion is (Z i F =N), ~ is Avogadros
potential F2 + - F1 number. (See Section 3.1.6.2 for ions in a gaseous mixture.)
+ -
+ E - F1 < F2 A binary electrolyte AZ YZ when dissociated fully in
+ - a solvent will produce ions of cation AZ (whose
+ -
+ - electrochemical valence is Z) and  ions of anion YZ 
+ - (whose electrochemical valence is Z). For example, for
+ -
Positively + - Negatively Na2SO4 in water, 2; Z 1;  1; Z  2 since
charged + +
- charged the ions are Na and SO 4 . Thus Zi  s are positive for
electrode ++
- electrode
- cations and negative for anions. If there is an electrical field
+ -
+ - in the solution, each species, positive and negative, will
+ - Positively charged experience a force due to it according to (3.1.7).
+ -
+ - particle/ion In any electrolytic solution, any molecular ion of inter-
+ - est having a mean diameter dion and an algebraic valence
+ -
+ - Zion will attract ions of opposite charge, namely
counterions. However, the centers of such counterions
z can approach the center of the ion of interest to a distance
of a only due to short-range repulsive forces. For radial
Figure 3.1.2C. Ion/charged particle placed in an electric force field distances r (> a), the distribution of the net potential net
of constant strength E d=dz. due to the ion of interest and the counterions is given as
(see Newman, 1973)
r2 d=dr: 3:1:6c
 
r  a
Integrating, we can write Z ion e exp 
net r ; 3:1:10a
4r 1 a
0  r 2 2 =2; 3:1:6d
where the potential due to the ion of interest only at loca-
where 0 is an arbitrary value of the centrifugal potential at
tion r is (Z ion e=4r) and e is the electronic charge, 1.6021
r 0. As r increases, decreases.
 1019 coulomb. Thus the net potential net r decays
very rapidly with r. The extent of rapidity in decay is
3.1.2.3 Electrical force determined by the parameter called the Debye length
Consider next an electrical force field of constant strength E  1=2
RT
(volt/m) whose electrostatic scalar potential (volt) is such ; 3:1:10b
F2 I
that the potential difference d over a distance dz in the
direction of the field is E dz (Figure 3.1.2C). If molecular which is influenced by the ionic strength I defined by
ions of the ith species, each having a valence of Zi (the magni-
n
tude varies from 1 to 10), are exposed to this field in an 1X
Z 2 C il : 3:1:10c
aqueous solution in between the electrodes, the force exerted 2 i1 i
on 1 gmol of such charged molecules in the z-direction is
The higher the value of I, the smaller the value of . Here F is
d
F iz Z i F E Z i F 3:1:7 Faradays constant, R is the universal gas constant, T is the
dz absolute temperature and is the electrical permittivity of
or the fluid. This quantity is the product of the relative dielec-
tric constant of the medium d (for water d 78.54 at 25  C)
F i Z i F r and E r: 3:1:8
and the electrical permittivity 0 of vacuum ( 8.8542 
Here F is Faradays constant having the value of 96 485 1014 farad/cm or coulomb/volt-cm 8.854  1012 cou-
coulomb per g-equivalent (C/g-equiv.); the electrical field lomb2/newton-m2, where recall that 1 newton-m 1 volt-
has the unit of newton/coulomb (N/C). The above expres- coulomb 1 joule). In a uni-univalent electrolyte solution
sion for force assumes that the electrical field has not been of 0.1 M strength (of, say, NaCl) the value of at 25  C is 9.6
disturbed by the ions and vice versa. The work done to move  108 cm, i.e. 0.96 nm (Newman, 1973). The ions of oppos-
these charged molecular ions having a total charge of ZiF ite charge shield the charge of the ion of interest, and the
coulomb along the z-coordinate from z1 to z2 is given by effect of the ion of interest decays very rapidly with distance.
So the description of the electrical force on an ion in an
W Z i F Ez2  z1 3:1:9
applied field E by definition (3.1.8) is generally satisfactory.
per gram mole or gram ion obtained from 1 gmol of the The ion of interest, however, has in an aqueous solu-
compound (the unit of work is the joule (volt-coulomb)) tion a solvation shell of water molecules. Similarly, the
80 Physicochemical basis for separation

counterions have water molecules around them. When the with them by the phenomenon of electro-osmosis, the
ion of interest moves in an applied electrical field E (as in macroion motion is retarded by this solvent motion. The
electrophoresis), the water molecules solvating the coun- electrophoretic retarding force is given by
terions move in an opposite direction with the counterions
F ret
p d r p  Qp E: 3:1:11a
whose charge is equal and opposite to the charge of inter-
est. This motion of the solvent molecules located in a shell Here d is the dielectric constant of the medium, rp is
at a distance exerts a retardation force, F ret
i , on the main the effective particle/macroion radius, Qp is its charge
force on the ion of interest, Z i F r. The net driving and is the zeta (or electrokinetic) potential. (This is
force, F net
i , on the ion of interest becomes (Wieme, 1975) the potential at the surface of shear around the particle;
n d o there are solvent molecules tightly bound to the particle
ion
F i F ret
i Z i F r  Z i F r  ; of radius rp up to the radius rp rp defining
2
the hydration shell, and they move with the particle
  defining the shear surface.) Thus the net force on the
d ion
F net
i Z i F r 1  : 3:1:10d particle is
2

F p F ret
p Qp E d r p  Qp E
Often, this correction is ignored for small ionic species 3:1:11b
d r p E:
where the description of the species-specific force (3.1.8)
is reasonably accurate. A more general analysis yields the following results
If the charged species are larger, for example proteins, (Wieme, 1975):
or if we are dealing with colloidal particles in a solution, the
  fr p =
retardation forces are regularly taken into account. Gener- F p F ret
p d r p E ; 3:1:11c
ally, small ions of charge opposite to that of the charged 1 r p =
protein or particle will collect in a diffuse layer next to the where the function f(rp /) tends to 1 when (rp /) >> 1 (rp /
protein or particle. An electrical double layer is created by ! 1000). For proteins the range is 1 < rp / < 300. When
the fixed charges of the protein or particle and the counter- (rp /) ! 0.1, f(rp /) tends to 2/3.
ions collected from the solution (Figure 3.1.2D). The total We assume the electric field strength E to be constant
charge in this double layer is zero. However, there is an here, independent of the z-coordinate. Therefore, the
electrical potential el in this layer which decreases to zero electrical force on 1 gmol of the charged ith species is
with distance from the particle or protein surface. Note that also independent of the z-coordinate location (we can
the counterions are mobile and can be influenced by the generalize this to all three coordinate directions). However,
external electrical field E. if the charge on the ith molecular ion species, Zi, changes
If the macroion, protein or particle moves in a given from one location to another, the electrical force will
direction due to the external field E, resulting in electro- become location dependent. For example, it is known that
phoretic motion, the large number of counterions move in the net charge on protein molecules in a solution depends
an opposite direction. Since these ions carry some solvent on the solution pH. At the isoelectric pH (identified as pI),
Zi 0, but Zi 6 0 for all other pH values (see Figure
Hydration 4.2.5(c)). The net charge is positive at sufficiently low pH
shell and negative at sufficiently high pH. Thus, the electrical
force on protein molecules can vary even if the field
strength E is constant.
It is possible to create a pH gradient in the solution
of concern in the separation system by external means.
At the location where pH has the isoelectric value for the
ith protein species, Zi will be zero, leading to a zero
external electrical force on the ith species. At other loca-
z
tions, the force will be nonzero. The forces on molecules
Particle
or ions in solution due to an externally imposed primary
or
protein rp field (i.e. electrical field) can then be suitably altered by
Drp ye1 the imposition of additional property gradients in the
y
solution by external means. If we have macroscopic par-
0 ticles instead of molecules or macromolecules, the
driving force per particle may be obtained from defin-
Figure 3.1.2D. Charged particle with its hydration shell and ition (3.1.8) simply by replacing ZiF by Qp, the net par-
double-layer potential profile. ticle charge in coulomb.
3.1 Forces, displacements, velocities and fluxes 81

+Q
E is
nonuniform
Electric + F
dipole

r
a
F

-Q

Figure 3.1.2E A dielectric uncharged particle placed in a nonuniform electric field develops an induced dipole of moment p = Qa, where Q
is the magnitude of the charge of each pole separted by a distance a where the vector coordinate of negative charge Q is r.

If the electrical force field E is nonuniform and a the effective polarization of the spherical particle (Jones,
dielectric particle4 is placed in such a field and develops 1995). When the particle is metallic (pR 0), with p ! ,
a dipole moment p, then the net electrical force on the
F 2 r 3p d rE 2 : 3:1:14
particle with the induced dipole is (Figure 3.1.2E)

F p  r E p rE: 3:1:12 Note: r E2 is r (E  E) so that (E  E) is the local electrical


field intensity.
The motion of particles caused by polarization effects in Both of these forces are proportional to the particle
the nonuniform electrical field is identified as volume as well as gradient of the square of the electrical
dielectrophoresis (Pohl, 1978), whereas, as we have seen field (Von Hippel, 1954; Lin and Benguigui, 1977). The
already, the motion of charged particles in a uniform elec- above results can be obtained from (3.1.12) by introducing
tric field is termed electrophoresis. In the force expression an expression for p, the dipole moment, resulting from the
(3.1.12), rE is the gradient of the nonuniform electric field polarization of the particle in the nonuniform electric field
at the particle center and p is the magnitude of the dipole (p (polarizability)  (particle volume)  (E)).
moment given by the product of the charge and the dis- Note that the dielectric force F is directed along the
tance between the charges (in coulomb-meter). The gradi- gradient of the electric field intensity r E2. For the metallic
ent of the electrical field is to be determined at the center particle, the force direction is always toward the direction
of the particle (Halliday and Resnick, 1962). of the largest field. On the other hand, the force on a
If an uncharged particle is placed in a dielectric fluid nonmetallic particle will be toward the direction of the
having a dielectric constant d and an electrical resistivity largest field only if p > d (positive dielectrophoresis); it
dR, the particle experiences a force only if the electrical field will be toward the lowest field if d > p (negative dielec-
E is nonuniform in space. Assume the particle to be spher- trophoresis). If p >> d or d >> p, the magnitude of
ical with radius rp. If the particle is not metallic and the fluid the force is not influenced, but the direction is. Obviously
resistivity dR is very high, the force is given by (Jones, 1995) if there are two particles with p1 < d < p2 , the forces
  experienced by the two particles will be in opposite direc-
p  d
F 2 r 3p d rE 2 ; 3:1:13 tions (Lin and Benguigui, 1977).
p 2 d
The dielectrophoretic force expressions given above
where p is the dielectric constant of the particle having a are proportional to the square of the field strength and
resistivity pR. The quantity within the brackets is called the are independent of the direction of the field. Therefore an
ClausiusMossoti function, and it indicates the strength of alternating current (AC) field can be used here, unlike
electrophoretic motions induced by a DC field (Pohl and
Kaler, 1979). Such a time-varying nonuniform electrical
4
The small dipole is characterized by two equal and opposite field has been used to separate mixtures of whole cells.
charges, +Q and Q, located at a vector distance a apart (Jones, In the cases considered above, there was an applied
1995). Since the electric field is nonuniform, the two charges are electrostatic potential gradient E, uniform or nonuniform,
subjected to different values of the electric field, E(r + a) and E
acting on the particle with or without charge. If the particle
(r); here r is the location of Q. The force F QE(r + a) QE(r).
However, since jaj << jrj, E(r + a) E(r) + a rE(r) by the had a charge, it was assumed (although we did not indicate
Taylor series expansion. Therefore F Qa rE(r), where Qa is it) that the field generated by this charge did not influence
the dipole moment p of the particle. the external electrical force field generated by E. On the
82 Physicochemical basis for separation

other hand, if there is no externally applied field E, and we The upper sign in this equation corresponds to approach at
have charged particles or molecules, there will be coulom- constant charge, while the lower sign corresponds to
bic attraction between oppositely charged particles or cou- approach at constant potential.
lombic repulsion between particles with similar charges. In
aerosol or deep bed filtration, stationary collectors having 3.1.2.4 Magnetic field
charges attract particles with opposite charges. If both the
An ionic species i with a charge Zi moving with a velocity vi
charged stationary collector and the particle (with charge
can essentially be considered to be a current flow with a
Qp) in the fluid may be considered as point charges, the
current of magnitude Z i F jvi j for 1 gmol of ionic species.
magnitude of the electrostatic force exerted on the particle
A conductor carrying current in a magnetic field of constant
by Coulombs law is
strength B experiences a force. Since the motion of ionic
Qp Qc species constitutes a current, the force on 1 gmol of the ith
F ELS : 3:1:15
4 r2 ionic species in the magnetic field is given by
Here Qc is the charge on the collector whose center is located mag
Fi Z i F vi  B: 3:1:18
at a distance r from the particle center and is the electrical
permittivity of the fluid. The electrical permittivity of the The vector magnetic potential for the magnetic field B is
fluid is defined as d 0 (where 0 is the electrical permittivity given by
of vacuum and d is the dielectric constant of the medium).
The value of 1/4 0 commonly used is 9  109 newton-m2/ B r  : 3:1:19
coulomb2. The force is directed along the minimum distance Vector B is also called the magnetic induction. The unit of
r connecting the two charges. The magnitude of the force, B is (newton/coulomb)/(m/s) and is identified usually as
F ELS, on the particle and the collector is the same, but the weber/m2, or tesla.
forces point in opposite directions. The expression above is Consider now particles without charges in the mag-
based on the point charge model, which is valid only at large netic field. Particles can be classified into three general
values of r. For smaller distances, complicated procedures are classes depending on how much magnetization is induced
followed using image methods (Kurrelmeyer and Mais, 1967). in them in a magnetic field. Ferromagnetic materials, such
In aqueous-suspended particles, or colloidal systems, as iron, cobalt and nickel, are strongly magnetized. In
there are two other forces between the particle and the general, the induced magnetism in these materials become
collector. The unretarded London attraction force between relatively independent of the applied magnetic field. The
a particle of radius rp and a collector is given by (Spielman extent of magnetization induced in the second class of
and Fitzpatrick, 1972) materials, paramagnetics, is far weaker than in ferromag-
8 9
 > > netics. However, 55 elements in the periodic table are
2a < 1 =
F Lret 
H
h  i2 ih ; 3:1:16 paramagnetic, and a magnetic field is used to separate
3r p >
: hmin 2 hmin =r p 2 > ; paramagnetic particles. The third class of materials, dia-
rp
magnetics, have even weaker magnetization.
where The magnetic force exerted on a small magneti-
aH Hamaker constant; zable paramagnetic particle of volume (mp/p) in a
hmin the minimum separation between particle and magnetic field of strength Hm (unit, amp/m) and volume
collector; susceptibility p in vacuo is
ih unit vector in the direction hmin outward from the !
collector. 1 m mp
F 0 p rH m  H m ; 3:1:20
2 p
The electrokinetic force in the double layer (Spielman
and Cukor, 1973) is given by where m 0 is the magnetic permeability of vacuum (Watson,
" 1973; Birss and Parker, 1981). Note that the force is propor-
d r p d exp d h tional to the volume of the particle (i.e. proportional to d 3p ).
F ELK 1 2 2
4 1 exp d h For paramagnetic and diamagnetic particles, Hm is linearly
#
exp d h proportional to the magnetic induction vector B. The magni-
 1  2 2 ih ; 3:1:17 tude of this magnetic force in the z-direction (say) is given by
1  exp d h
!
where m mp dH m
F z 0 p H m ; 3:1:21
p dz
h distance between particle and collector;
d dielectric constant for the fluid; so that increasing Hm or (dHm/dz) or both will increase Fz. If
d reciprocal Debye length (= 1/); the particle is in a fluid of volume susceptibility , then the
1,2 zeta potential of collector and particle, respectively. force on the particle is modified to (Birss and Parker, 1981)
3.1 Forces, displacements, velocities and fluxes 83

the suspension medium and is directed along the gradient


2
of the magnetic field intensity rH 0m . The quantity in
Ferromagnetic wire brackets on the right-hand side of (3.1.24) is a Clausius
Mossotti function K of sorts, where has been replaced by
Particle ms. In positive magnetophoresis, K > 0, and particles
H are attracted to magnetic field intensity maxima and are
repelled from the minima. In negative magnetophoresis,
K < 0, and the phenomenon is reversed.

Magnetic
force 3.1.2.5 Chemical potential gradient
A number of external force fields have been described in
z
the preceding subsections. Of these, the gravitational
Figure 3.1.2F. Simplified cross-sectional view of the magnetic
force field exists in nature regardless of our desire to
forces acting on a paramagnetic particle flowing past a magnet- require it or not. Other force fields, like centrifugal,
ized ferromagnetic wire in a background field H. (After Dobby and electrical and magnetic, are created by engineers and
Finch (1977).) scientists to achieve separation. Most of these external
force fields are often described by the negative of the
gradient of their respective scalar potentials, i.e. r
(exceptions are the magnetic and nonuniform electrical
1
F m vp p  rH m  H m : 3:1:22 fields). But all such force fields originate outside the
2 0
separation system.
The variation of Hm in the z-direction could be brought The force of chemical potential gradient, on the other
about in the following way even if a uniform Hm is created hand, originates inside the separation system due to the
by a solenoid as a magnet. If the region where the magnetic escaping tendency of molecules. For example, suppose
field is applied has fine ferritic steel wires, the uniform field we introduce two species into a closed vessel at constant
is grossly distorted near the wire (Figure 3.1.2F) and the temperature (T) and pressure (P) as a vaporliquid
particles are therefore forced toward the wire (Dobby and system which is not in chemical equilibrium. As time
Finch, 1977) due to the positive gradient of the magnetic progresses, we will observe that the species with the
field. The background field Hm magnetizes the lower boiling point will preferentially escape the liquid
ferromagnetic wire of magnetic permeability w. If the phase and accumulate in the vapor phase; simultan-
magnetization of the wire is Mw, the local field gradient eously, the higher boiling species will preferentially
becomes, approximately, escape into the liquid phase. This directionally oriented
escaping tendency disappears when the chemical poten-
dH m H m M w  H m M w amp
; 3:1:23 tial of each species becomes uniform throughout the two
dz dw d w m2
phases and chemical equilibrium is attained (see Section
where dw is the wire diameter. 3.3). No external agency is involved in this separation
The magnetic force on a spherical paramagnetic (lin- phenomenon. We shall soon see that a large number of
early polarizable) particle of radius rp in a paramagnetic separation processes are governed by the gradient of this
solution exposed to a nonuniform magnetic field having a chemical potential, ri , where i is the chemical poten-
local field intensity vector of H m 0 in vacuo is given by tial of the ith species per gmol of the ith species. For
(Jones, 1995) convenience, this chemical potential is sometimes
! referred to as int
i , the internal chemical potential, the
m m
p  s 2
F 2 m
s pr 3
rH m
0 : 3:1:24 superscript indicating its origin inside the separation
m
p 2s
m
system (Giddings, 1982).
It is known from chemical thermodynamics (Guggen-
Here m m
s and p are the magnetic permeabilities of the
heim, 1967) that, at constant T and P, the reversible work W
solution and the particle, respectively. The permeabilities
done to transfer mi moles of species i from a state of partial
may be related to the susceptibilities via m m 0 1;
2 molar Gibbs free energy Gi j1 to Gi j2 > Gi j1 is given by
where m 0 corresponds to that in vacuum. Further rH 0m
is related to the magnetic flux density B0 in the absence W mi Gi j2  Gi j1 : 3:1:25a
2
m 2
of matter via rH m 0 rB0 =0 . This phenomenon of
particle motion is sometimes described as magneto- However, i, the chemical potential, is simply Gi . If states 1
phoresis. Note that the force is proportional to the particle and 2 refer to spatial locations z1 and z2, where z2 > z1, then
volume through r 3p , the magnetic permeability m s of we immediately conclude that
84 Physicochemical basis for separation

W mi i j2  i j1 : 3:1:25b electrical force fields, such representations are not possible


for a system of molecules or ions. For macroscopic par-
This work is needed to overcome a natural force which ticles, where no chemical potential exists, only external
resists taking mi moles from i j1 to a higher value i j2 . This force fields are important.
natural and spontaneous force on species i must then act Just as we have an idea of how to calculate ext and
i
in the opposite direction: therefore rext i , we need to know more details about

W mi i j2  i j1 F total ri . Consider any region in the separation system where


iz z 2  z 1 : 3:1:25c
we assume equilibrium to exist (see Section 3.3). For a
Per gram mole of species i, the force Fiz is binary system of i 1, 2 at constant temperature, if the
pressure P and the mole fraction xi in the region are
F total j  i j 1
F iz iz
 i2 : 3:1:25d changed by differential amounts, the corresponding
mi z2  z1
change in i is given by
Vectorially, we can now describe this force due to this     
 i i
chemical potential per gmol of ith species as 
di  dP dx i : 3:1:30
T P T;x i x i T;P
dGi d
F i Fjgmol of ith species  k  i k: 3:1:26 The partial molar volume V i of the ith species in this
dz dz
region is defined as
In vector notation, the force on 1 gmol of the ith species  
due to the chemical potential gradient is ri. By analogy, i
Vi : 3:1:31
it has often been indicated that all external driving forces P T;xi
per mole of the ith species may be represented as rext i . Furthermore,
The shortcomings of such an approach (from the point of  
view of the fundamental property relation in chemical i d n ai
RT ; 3:1:32
thermodynamics) for external forces have been demon- x i T;P dx i
strated by Martin (1972, 1983). We therefore express the where ai is the activity of species i in this region. One can
total driving force on 1 gmol of the ith species at constant therefore write
temperature as
di jT V i dP RT d n ai : 3:1:33
F ti F t jmole of ith species ri F ext
ti : 3:1:27
Correspondingly, the gradient of i is given by
Of course, when possible, F ext
ti may be represented by
rext ri jT V i rP RT r n ai 3:1:34
i so that

F ti F t jmole of i ri ext


i ri ext
ti ; for a binary system at constant temperature.
3:1:28 The force due to a chemical potential gradient on
species i at constant temperature, ri jT , then arises due
where the summation for the external force potentials
to the existence of a pressure gradient, an activity gradient,
indicates the sum of the available external forces repre-
or both, in the separation system in a given region.
sentable as r.
For liquid phases at not too high a pressure, the activity
The above representation of external forces combined
ai may be related through an activity coefficient i to the
with the chemical potential force for molecules or ions is
mole fraction xi as follows:
quite useful for those external force fields representable by
the negative of the gradient of their scalar potentials. We ai i x i : 3:1:35
indicate in Table 3.1.1 the value of ext
i for 1 gmol of the ith
Thus
species for a few cases. For magnetic and nonuniform
ri jT V i rP  RT r n i x i ;
Table 3.1.1. The value of external force field ext
i for 1 gmol of
species i which can be expressed as

dn i x i
(1) Uniform electrical field of ZiF ri jT V i rP  RT r n x i : 3:1:36
dn x i T
electrostatic potential and
constant Zi   This expression shows that, ultimately, the mole fraction
(2) Gravity; species i in a solution of M i g 1  z
i gradient can be used instead of the activity gradient. For an
density with z-axis vertically
upwards
ideal solution, we have a simpler expression (since i ! 1):
(3) Centrifugal force field
1 ri jT V i rP  RT r n x i : 3:1:37
0  M i 2 r 2 3:1:29
2
For gaseous mixtures, we can use
3.1 Forces, displacements, velocities and fluxes 85

^f of different kinds. In a gas mixture subjected to a tempera-


ig
ai ; 3:1:38 ture gradient, it has been observed that the lighter gas
f 0ig
species concentrates in the hot region and the heavier
where f 0ig is the standard state fugacity of the gas i at system species concentrates in the cold region.5 This phenom-
temperature and specified pressure of 1 atmosphere and enon, known as thermal diffusion, is also observed in liquid
^f is the fugacity of species i at system temperature and mixtures. The force exerted on gas species A in a binary
ig
pressure. For an ideal gas mixture, ^f ig pig , the partial mixture of A and B subject to a temperature gradient is
pressure of species i in the mixture; for such a case, given by6
ri jT V i rP  RT r n pig : 3:1:39 RT DTA
Fjgmol of A  r n T F TA ; 3:1:44
DAB C A M A
For isobaric systems, we find, for ideal solutions,
liquid solutions : ri jT;p RT r n x i ; where DTA is the thermal diffusion coefficient for species
3:1:40 A in a binary mixture of species A and B and DAB is the
gaseous mixtures : ri jT RT r n pig :
ordinary diffusion coefficient. A thermal diffusion ratio kT
Thus, mole fraction gradient, or partial pressure gradient, is has been defined (Bird et al., 1960) as
the basis of the chemical potential gradient in a binary
system at constant T and P. Recognize, however, that there t DTA
kT : 3:1:45
is no specific force leading to a mole fraction or partial C 2t M A M B DAB
pressure gradient (see Section 3.1.3 after equation (3.1.76)).
Furthermore DTA DTB . Thus if species A goes toward the
It is important to qualify these characterizations of ri
hotter region, species B moves to the colder region.
by indicating that they are valid within a phase, liquid,
No phenomenon analogous to thermal diffusion in
gaseous or solid. When the binary separation system has
chemical mixtures is encountered with particles. However,
two phases, the variation of standard state chemical poten-
when placed in a temperature gradient, small particles in a
tial between the two phases also needs to be considered.
stagnant liquid have been found to move in the direction of
Suppose rP0 in the separation system. Integrate relation
lower temperature. This phenomenon has been referred to
(3.1.33) at constant P and obtain, for any phase j,
as thermophoresis (Fuchs, 1964). It is said to arise in a gas
ij 0ij RT j n aij : 3:1:41 because gas molecules originating in the hot regions
impinge on the particles with greater momenta than mol-
Thus
ecules coming from the colder regions. The magnitude of
rij jT;P r0ij  RT j r n aij ; 3:1:42 the force due to the thermophoresis, FTP, for a spherical
particle obeying Stokes law in the z-direction is
where 0ij is the standard state chemical potential of the ith  
species in phase j at the standard state conditions used to 6 2 r p dT
F TPz Th ; 3:1:46a
calculate of aij. (In Section 3.3, Table 3.3.2 identifies stand- C c t T dz
ard states for various conditions.) In a two-phase system,
where Th is a dimensionless group defined for a gas phase
0ij for species i is usually different for the two phases; thus
of viscosity by
r0ij is also a driving force for species movement and
separation. The value of 0ij in any phase may be calculated U pzt t T
Th  : 3:1:46b
from standard thermodynamic relations (Denbigh, 1971). dT=dz
Very small particles in liquid or gas streams have a
The value of Th is said to range between 0.42 and 1.5 from
random Brownian motion due to the thermal energy of
theoretical predictions. Here Upzt is the steady particle
the continuous phase molecules. If there is a concentration
velocity due to thermophoresis (see Section 3.1.3.2 for its
gradient of particles due to a particle sink, then there is a
definition), rp is the particle radius and Cc is a correction
Brownian motion force on these particles:
factor (see Section 3.1.6.1). For additional details, see
F BR 6 vrp r p ; 3:1:43 Talbot et al. (1980). Explanations for the same phenom-
where vrpis the particle diffusion velocity relative to that of enon in a liquid are somewhat uncertain (McNab and
the liquid or gas phase, rp is the particle radius and is the Meisen, 1973). An alternative situation, in which particle
fluid viscosity. motion is generated in a solute gradient and termed
diffusiophoresis, is treated in Anderson et al. (1982).
3.1.2.6 Other forces: thermal gradient, radiation pressure,
acoustic force
5
This is true for gases observing the inverse power law of
Until now, we have considered the temperature to be repulsion: (force) = (constant)  (intermolecular distance) and
uniform in our separation system. The existence of a > 5. See Present (1958).
6
temperature gradient exerts unequal forces on molecules In a frame of reference where vt vt (see Section 3.1.3).
86 Physicochemical basis for separation

Radiation pressure from continuous wavelength (cw) this lift force normal to (i.e. in the y-direction) the flow
visible laser light is known to accelerate freely suspended direction (say, in the z-direction) is given by
particles in the direction of the light. The magnitude of the
radiation pressure force, Frad, has been given as (Ashkin, 1970) F lift a vz dvz =dy1=2 r 2p =1=2 3:1:49

2 qfr Pr for a spherical particle of radius rp in an axial fluid flow


F rad : 3:1:47
c field (i.e. velocity vz) having a simple shear ((dvz/dy) 6 0)
(Saffman, 1965); the constant a has the value 6.46 and
Here qfr is the fraction of light effectively reflected back
(/) for the fluid in which the particle is suspended.
(generally assumed to be ~0.1), Pr is the power of the laser
light and c is the velocity of light. The medium of interest is
a liquid. 3.1.2.7 A generalized expression for all forces
The acoustic radiation (acr) force Facrx in the x-direction
In the preceding discussions, a large number of forces,
due to an ultrasound of wavelength induced on a particle
both external and internal to the separation system, have
of volume Vp, density p and compressibility p suspended
been identified and described briefly. Note that any force
in a fluid medium of density f and compressibility f, is
so identified was, for example, specific to molecules of
given by (Petersson et al., 2005)
the ith species.7 However, it is known that forces specific
!( )
P 20 V p f 5 p  2f p to the jth species can also affect the motion of molecules of
F acrx   sin2kx; the ith species. For the immediate objectives in the para-
2 2p f f
graphs that follow, these effects are ignored by assuming
3:1:48 uncoupled conditions:8 molecules of species i in a stagnant
where k is defined as (2/), x is the distance from the fluid move only due to forces specific to the ith species;
pressure node and P0 is the amplitude of the pressure similarly for the jth species. It is further assumed that the
wave. The direction of the force Facrx is dependent on the conditions are not too far removed from equilibrium (see
sign of the quantity within the brackets { }, which is some- the introduction to Section 3.3 and Sections 3.3.13.3.6
times called the -factor. Those particles which have a for descriptions of equilibrium conditions); therefore
positive value for the -factor will move toward the pres- thermodynamic quantities (defined only for equilibrium
sure node; those with the reverse sign will move toward the conditions) can be used to described nonequilibrium
pressure antinode. For an introduction, see Nyborg (1978), conditions where a net transport of molecules of species i
Ter Haar and Wyard (1978) and Weiser et al. (1984). exist due to external and internal forces. For illustrative
purposes, an expression for the total driving force Fti on
1 gmol of species i or 1 gion of ion i in a solution or mixture
3.1.2.6.1 Inertial force So far, we have presented a var-
due to a variety of forces identified earlier is given below
iety of forces that can come into play and act on molecules
(force is positive in the direction of the positive axis).
of a given species and/or particles. When such forces act,
Obviously, only one or a few of these forces exist at any
the molecules or particles move in a given direction. At the
time in a given separation system:
beginning of this motion, the molecules/particles undergo
acceleration. From Newtons second law, we know that the
molecules/particles are subjected to an inertial force, F iner, F ti ri jT F ext
ti F Ti ;
mag
F ti ri jT  rexti F i F Ti ;
in the same direction during this period of acceleration.
F ti RT r nai V i rP r0i M i 2 r rM i g kZ i F r
The magnitude of this inertial force is simply the product of
the mass of the molecules/species and the magnitude of RT DTi
Z i F vi B rnT: 3:1:50
the acceleration (see the beginning materials in Sections Dij C i M i
3.1.3.1 and 3.1.3.2). In most circumstances encountered in
separations, one can assume that the acceleration ceases to This expression9 does not include the effect of a nonuni-
exist after some time and a steady velocity comes about; form electrical field (see (3.1.12)) and the electrophoretic
therefore the inertial force ceases to exist. However, if the retardation. Note that buoyancy forces do not appear in Fti
fluid flow field is such that the direction and magnitude of as such. Lee et al. (1977a) have tabulated the magnitudes
its velocity continues to change, inertial forces will always of each of these forces, i.e. gradients, which can create a
be present. This is particularly true of particles flowing in a value of rnai jT;P equal to 1 cm1.
medium with many obstacles, or if there is a change in flow A clarification on the forces acting on solute molecules
direction and flow cross-sectional area. in a solvent undergoing rigid body rotation in a centrifuge

3.1.2.6.2 Lift force on a particle in shear flow When a 7


Particles will be considered next.
particle is flowing in a shear flow field, it experiences a lift 8
For an illustration of coupling, see Section 3.1.5.2.
force normal to the fluid flow direction. The magnitude of 9
For alternate representations, see Lee et al. (1977a).
3.1 Forces, displacements, velocities and fluxes 87

is useful here. The (non-Brownian) force exerted on the V t dP M t 2 rdr 0: 3:1:57


solute molecules is due not only to the 2rr term, but also
to the pressure gradient in the solvent developed by rota- With solution density t defined as (M t =V t ), we can now
tion. To determine this pressure gradient, consider only simplify equation (3.1.53):
the rotation of solvent species i s in the centrifuge 


(no solute present). At equilibrium, there is no net force di  M i  t V i 2 r dr 0: 3:1:58
 T; P
acting on solvent molecules in the centrifuge:
dP Note that t and V t will, in general, depend on radial
F ts 0 V s r M s 2 r r: 3:1:51
dr location r since the pressure and composition vary with r.
This provides an expression for (dP/dr) due to rotation. We now focus on macroscopic particles and provide an
Assume now that this pressure gradient generated by solv- expression for the total external force acting on a particle of
ent rotation is unaffected by the presence of solute species mass mp, density p, radius rp, charge Qp, velocity vp and
i in the rotating centrifuge. The net force10 on solute volume (mp/p). We have not included here the Brownian
species i in the radial direction is then obtained as (exclud- motion force F Br, nor any force due to thermophoresis,
ing the concentration gradient contribution) radiation pressure, acoustic force and the electrical force
in a nonuniform electrical field given by (3.1.13). Although
dP not generated by an external force field, coulombic types of
F ti V i r M i 2 r r
dr interactions, London dispersion and electrokinetic forces in
" 0 10 1 #
3:1:52 the double layer are included in the expression given below:
2 Vi Ms 0 1 0 1
M i r 1  @ A@ A r;
Mi Vs ext t 2 t
F tp mp g @1  Ak mp r @1  Ar  Qp r
p p
where V i =M i is the partial specific volume of solute i and
d r p  Qp E
M s =V s is essentially the solvent density in any region. 0 1
The assumption that the solute species will not affect 1 m @mp A
the pressure gradient in a centrifuge is not generally valid. Qp vp  B 0 p r H m  H m
2 p
Consider the total force on any ith species in such a case.
At equilibrium, Fti is zero, leading to F ELS F Lret F ELK : 3:1:59
 ELS
The expressions for F , F Lret
and F ELK
may be obtained,
di 
F ti 0   r M i 2 r r; respectively, from (3.1.15), (3.1.16) and (3.1.17). Note that
dr  T
 the pressure gradient generated in a centrifugal field has
3:1:53
dP di  2
been replaced by means of equation (3.1.58) as a centrifugal
V i   M i r 0:
dr dr  T;P buoyancy term in which V i has been replaced by particle
volume mp =p and Mi by mp to provide 1  t =p .
From the principles of chemical thermodynamics (Gug- Example 3.1.1 Calculate the gravitational force exerted on a
genheim, 1967), at constant T and P the GibbsDuhem particle for the following two cases: (1) particle diameter
equation is 10 m, particle density 2 g/cm3, fluid density 1.3 g/cm3;
X n  (2) particle diameter 2 cm, particle density 2 g/cm3, fluid

x i di  0: 3:1:54 density 1.3 g/cm3.
i1
T;P
Solution (1) From the force expression (3.1.59), the
Multiply equation (3.1.53) by xi, sum over n species
z-direction gravitational force on the particle
and use the GibbsDuhem equation (3.1.54) to obtain
is given by
n
X n
X
 x i V i dP x i M i 2 r dr 0: 3:1:55 4
F tz  r 3p p g1  t =p
i1 i1 3
Define an average partial molar volume V t and an average 45  104 3  2  9801  1:3=2 g cm
molecular weight of the solution Mt as follows: 
3 s2
n
X n
X
xi V i V t and xi M i M t : 3:1:56 4   125  1012  2  980  0:35

i1 i1 3
Rewrite equation (3.1.55) as 4   1:25  2  0:98  0:35  107 g cm

3 s2
10
One can similarly develop (3.1.5) for species i in a solvent g cm
3:59  107 3:59  1012 newton:
under gravity. s2
88 Physicochemical basis for separation

(2) 4 Z i F E 4:5  96 500  30 coulomb-volt=cm


F tz  r 3p p g 1  t =p
3 4:5  96 500  3000 coulomb-volt=m

4 13  2  9801  1:3=2 g cm 1:3  109 coulomb-volt=m:



3 s2
But 1 coulomb-volt 1 joule 1 newton-m. Therefore, ZiF E
8  980  0:35 g cm 1.3  109 newton, a force much stronger than the centri-
2873:5 2
3 s fugal force considered in Example 3.1.2, case (1).

2:873  102 newton: Example 3.1.4 Calculate the magnitude of the force due to
radiation pressure on a lossless dielectric spherical particle of
Example 3.1.2 Calculate the centrifugal force exerted for the radius 0.5145 m and density p 1g/cm3 subjected to a cw
following two cases. argon laser light of power 1 watt at a wavelength 0.5145
Case (1) 1 gmol of ovalbumin molecules (molecular m. Calculate also the instantaneous acceleration experi-
weight 45 000) in an aqueous solution of density enced by the particle.
1 g/cm3 rotating in a centrifuge at 7000 radians/s at a
Solution From equation (3.1.47),
radial coordinate of 1 cm, given ovalbumin density in
solution 1.34 g/cm3. 2 qfr Pr
F rad :
c
Case (2) a particle of diameter 10 m, density 2 g/cm3 in
a solution of density 1.3 g/cm3 at a radial distance of 1 cm. Assume that qfr 0.1; velocity of light, c 2.799  1010 cm/s;
Pr 1 watt. Then
Solution Case (1) From the force expression (3.1.52),
the magnitude of the radial force experienced by 1 gmol of
2  0:1  1 watt 0:2  1010 watt-s
ovalbumin molecules (species i) is given by F rad 10 :
2:799  10 cm=s 2:799 cm
" 0 1 0 1#

 0:2  1010  107 erg 0:2  103 g-cm2
 Vi Ms
F tir  M i 1  @ A @ A 2 r 2:799 cm 2:799 cm s2
mole M Vi s
" 0 #1 g-cm
7:145  105 7:145  105 dyne:
45 000 g @ 1 A 70002 s2
1 1 1 cm
gmol 1:34 s2
If the instantaneous acceleration is d2z/dt2, then Frad mp
g cm6 d2z/dt2. Now, mp 4=3 r 3p p , so
450001  0:74649  10 2
s gmol
4 4
g cm mp 0:5145  104 3  1 g  0:1362  1012
560 070  106 3 3
s2 gmol
0:57  1012 g;
5:60  106 newton=gmol:
d2 z 7:145  105 dyne
Case (2) From the generalized force expression acceleration 1:25  108 cm=s2 :
dt 2 0:57  1012 g
(3.1.59), we get F tr jparticle experienced by the particle
as follows:
The magnitude of this acceleration is quite high much
0 1 0 1 larger than that due to gravity (Ashkin, 1970).

 4
F tr particle mp r2 @1  t A r 3p p r2 @1  t A Note that the following forces acting on the particle are
p 3 p
0 1 proportional to the particle volume (and therefore r 3p ):
4 3 2 1:3 gravity force; centrifugal force; dielectrophoretic force;
 5  104  2  1  7000  @1  A magnetic force on a paramagnetic particle. The electro-
3 2
phoretic retardation force is proportional to the particle
4  125  2  49  0:35  106 radius.
g cm=s2
3

0:179  101 g cm=s2 :


3.1.3 Particle velocity, molecular migration velocity
Example 3.1.3 Calculate the force exerted on ovalbumin
and chemical species flux
molecules exposed to an electrical field of constant strength
30 volts/cm, given Zi 4.5 for ovalbumin at the solution pH. It is now appropriate to calculate the velocities of particles
Neglect the electrical double-layer effect. or molecules resulting from the forces acting on them. For
Solution The electrical force per mole of ovalbumin is clarity, we assume first that velocities created by bulk flow
obtained from (3.1.8) in the absence of other information are zero. Let the particle velocity vector due to external
(e.g. zeta potential, ionic strength, etc.) It is given by forces be represented by Up. Similarly, let U i be the
3.1 Forces, displacements, velocities and fluxes 89

average velocity vector of the ith chemical species in any Upt, for example in the z-direction as Upzt. (See Section
region due to external as well as internal forces. 3.1.6.2 for corresponding quantities for ions in a gas phase.)
Suppose the fluid medium is not stagnant but has a
mass average velocity vt. If Stokes law still determines the
3.1.3.1 Particle velocity and particle flux
resistive force, then the resistive or frictional (or drag) force
We focus on particle motion first. If a macroscopic particle on the spherical particle is to be determined using the
(mass mp) moves in the z-direction (positive z, vertically velocity of the particle relative to the fluid:
upward) under the action of forces acting in the z-direc-
F drag
p 6r p U p  vt : 3:1:63
tion, then, from the principles of mechanics (Newtons
second law), These relations based on Stokes law are valid only for
  small particles whose particle Reynolds number, defined
magnitude of force on the d2 z
mp 2 : 3:1:60 by 2r p U p =, is very small (<0.1). For larger particles
particle in the z-direction dt
and/or higher velocities, the magnitude of the resistive
If the particle was moving in free space, this force would (drag) force is given by
equal the external force or forces we have identified earlier
U 2pz
in magnitude and direction. If, however, the particle moves F drag
pz C D Ap ; 3:1:64
in a gaseous, liquid or solid (rarely) medium, the force on the 2
particle consists of the external force, (or forces, including the where CD is a drag coefficient and Ap is the projected area of
buoyancy force) and a frictional force, F dragp , which opposes the particle measured in a plane perpendicular to the direc-
the particle motion (and thus has a negative sign). This tion of particle motion (Bird et al., 1960, 2002). The drag
frictional force comes into play as soon as the particle moves. force varies with particle radius for fine particles in Stokes
It is known that, at small values of particle velocity Upz law range, whereas it varies with a larger power of particle
(in the direction of the positive z-coordinate), the frictional radius up to r p 2 at higher particle Reynolds numbers.
resistive force is linearly proportional to the magnitude of The external force based particle velocity in a stagnant
the particle velocity. For example, according to Stokes law, fluid medium leads to a number flux, np, of the particles across
the resistive or drag force vector on a spherical particle any given cross-sectional area. If the total number of particles
of radius rp moving at a velocity Upzk through a fluid of per unit volume of the fluid is Nt (see equation (2.4.2c)) and
viscosity is 6 r p U pz k. If this particle is falling under the terminal velocity of the particles is Upt, then the particle
gravity in a fluid of density t and viscosity , then flux np in terms of numbers of particles crossing a surface area
! perpendicular to Upt is given by (see Figure 3.1.3A)
d2 z t dz
mp 2 F ext pz  F drag
pz m p g 1   6 r p ; np N t U pt : 3:1:65
dt p dt
3:1:61 For any other surface area, indicated by A, the particle flux
across the surface area is obtained as
where Upz dz/dt.
Due to a nonzero acceleration (d2z/dt2 6 0), the vel- np N t U pt  A=jAj: 3:1:66
ocity of the particle increases; the frictional resistive force
(the drag force) also increases. In many systems, after If there is a particle size distribution indicated by nr p , the
some time the particle acceleration in the stagnant fluid particle number density function (equation (2.4.2a)), such
becomes zero and the particle velocity becomes constant. that Upt (rp) is the terminal velocity of particles in the size
This velocity is called the terminal velocity. We express the range of rp to rp drp, the particle flux np across a surface
resistive force vector on the particle as f dp U p ; when Up is area perpendicular to Upt (rp) is
the terminal velocity, for the z-component, r max

F ext np U pt r p nr p dr p : 3:1:67
d pz
F ext
pz f p U pz ) U pz : r min
f dp
When the size of the particles becomes quite small,
Thus
Brownian motion becomes important. The flux of particles
F ext due to Brownian motion is generally indicated by (Flagan
p
Up ; 3:1:62 and Seinfeld, 1988)
f dp
np Dp r N t ; 3:1:68
where f dp is the frictional coefficient of the spherical particle
moving slowly in a stagnant medium of viscosity . For a where we have assumed that the particle diffusivity Dp has
particle resistive force described by Stokes law, f dp is equal the same value regardless of the direction of diffusion. In a
to 6r p . Often, this terminal velocity will be identified as given direction (say z), this flux is obtained as
90 Physicochemical basis for separation

kB T
Dp 3:1:72
B B f dp
y where kB is Boltzmanns constant, 1.38066  1023 J/K
A
1 (1.38060  1016 g-cm2/s2-K) and f dp is the frictional coeffi-
A cient for the particle (see relation (3.1.86) for its molecular
equivalent using molar concentrations). For such aerosols
(suspensions of small particles in a gas/air), a more detailed
1 result based on Stokes law and the slip correction factor Cc
C
C
(see expression (3.1.215)) for a particle of radius rp is

kB T C c
D
D Dp ; 3:1:73
Upt 6 r p
where is the gas phase viscosity and T is the absolute
temperature in kelvin. One can use the same equation for
the liquid phase without the correction factor Cc.
z
x If the particle is flowing in a shear flow of a liquid, the
shear-induced diffusivity of a particle in a particle suspen-
sion, where _ is the local shear rate (e.g. (dvz/dy) in relation
(3.1.49)) has been experimentally measured by Eckstein et al.
Figure 3.1.3A. Explanation for particle flux expression (3.1.65). (1977) to be
Consider the rectangular parallelepiped shown. Let the length of
side DD 0 in the positive z-direction be Upt cm, which is the Dp 0:1p r 2p _ for 0 < p < 0:2
distance crossed in one second by a particle having the terminal 3:1:74
0:25r 2p _ for 0:2 < p < 0:5;
velocity Upt of all particles in the positive z-direction. Let the
lengths of A0 B0 (or AB) and C0 D0 (or CD) be 1 cm. The cross- where p is the volume fraction of the particles.
sectional area of ABCD or A0 B0 C0 D0 in the xy-plane is then 1 cm2.
Any particle in the volume ABCDA0 B0 C0 D0 (= Upt cm3) will cross
3.1.3.2 Molecular migration velocity and species flux
area A0 B0 C0 D0 in one second since the particle furthest from
A0 B0 C0 D0 will be at a distance Upt cm; others closer to A0 B0 C0 D0 Instead of macroscopic particles, we consider now the
will surely cross this area. Now the number of particles in this motion of molecules of the ith species in any region having
volume ABCDA0 B0 C0 D0 is particle number density  volume, i.e. no bulk velocity. The governing equations are similar in
Nt (particles/cm3)  Upt cm3, NtUpt. All of these particles will cross principle to equation (3.1.61). From a simplistic point of view,
the unit area of A0 B0 C0 D0 in one second. Therefore the particle flux the major difference is that we will deal with a mole of ith
across A0 B0 C0 D0 is np NtUpt particles/area-time. species instead of one molecule of ith species. Therefore we
will consider the averaged z-directional motion of a large
number of molecules (since individual molecular motion is
N t
npz Dp : 3:1:69 chaotic). For one mole, this motion may be simply modeled
z
in the absence of a temperature gradient as (Giddings, 1982)
When both Brownian motion as well as external forces act
on the particles, the overall flux expression in, say, the z- d2 z dz
Mi F tiz  f di ; 3:1:75
direction is obtained as follows: dt 2 dt
N t where Ftiz is to be determined from equation (3.1.50) without
npz Dp U pzt N t : 3:1:70 Fti. Further, dz=dt is the averaged species i velocity in the
z
z-direction and f di is the frictional coefficient for one mole of
If there is a particle size distribution given by n(rp), we the ith species in the region11 under consideration, the over-
obtain the particle flux as bar indicating an averaged quantity. When the force Ftiz is
r max
r max
applied at time t 0, the molecules experience an acceler-
nr p
npz Dp dr p U pzt r p nr p dr p : ation, i.e. d2 z=dt 2 is nonzero. However, unless we have
z
r min r min vacuum in the region, the resistance encountered by the
3:1:71

The diffusion coefficient of small particles undergoing 11


Sometimes the medium or the region in which species i moves
Brownian motion is of interest. For the diffusion coefficient will be explicitly identified via an additional subscript; the
of droplets/particles of very small dimensions in air, the frictional coefficient of species i through a membrane, for
StokesEinstein relation has been found to be valid: example, will be described by f dim .
3.1 Forces, displacements, velocities and fluxes 91

molecules of the ith species is so large that the period during


which acceleration exists is very small (Tanford, 1961; Gid-
B B
dings, 1982). Thus, for all practical time scales of interest in y
separation, the equation given above is simplified to12
A
A 1
dz
F tiz f di f di U iz : 3:1:76
dt
Here, U iz is the averaged velocity of the ith molecules in 1
C
the z-direction due to all forces acting on the ith species. C
We have assumed that f di has the same value in all direc-
tions (isotropic) and there is no bulk velocity.
D
At the beginning of this chapter, we identified the Viz D
relative displacement of a species as the key to separation.
What is the velocity of species i that would allow calcula-
tion of such a displacement? At first glance, U iz may
appear to be such a quantity. Note that Ftiz includes ordin- z
ary diffusion (molecular diffusion due to rn ai ), which, x
arising from a random Brownian motion of molecules, is
often detrimental to separation (see Section 3.2.1, in par-
ticular the text following equation (3.2.22)). We will separ- Figure 3.1.3B. Explanation for molar flux expression (3.1.77) of
ate its contribution to determine the most useful species i. Replace Upt in Figure 3.1.3A by viz, particles by molecules
expression for displacement or separation velocity due to of species i and particle number density Nt by Ci, the molar density of
forces acting on individual molecules. i in the volume element ABCDA0 B0 C0 D0 . We obtain Niz Civiz , if we
The key factor is consideration of the forces acting on follow the explanation provided in the caption to Figure 3.1.3A.
individual molecules. In simple molecular diffusion,
resulting from Brownian motion, an individual molecule is
viz v
tz U iz ; 3:1:78
not subjected to any specific force, although we often tend so that
to consider the concentration gradient as a force that causes
N iz C i v
tz C i U iz : 3:1:79
the molecules to go from a region of higher concentration to
a region of lower concentration. (Similarly, for gaseous Using (3.1.76), we obtain
phases, we have transport from a higher partial pressure
F tiz
region to a region with lower partial pressure.) That there is N iz C i v
tz d : 3:1:80
fi
a flux of a species from a higher concentration (higher
partial pressure) to a lower concentration (lower partial From relations (3.1.27) and (3.1.42), we obtain at constant
pressure) is simply a result of a larger number of molecules T and P
making random jumps across any surface area in the direc- " (  )#
1 oi ext RTC i n ai
tion of decreasing concentration (partial pressure). N iz C i vtz  d

 F tiz  :
Calculate now the molar flux Niz of the ith species (in a f i z f di z
binary system) in the positive z-direction across a plane 3:1:81
fixed with respect to stationary axes (Bird et al., 1960,
Now define the displacement or migration velocity of the ith
2002). Assume temporarily that there can be bulk flow; the
species as Uiz such that, at constant T and P,
molar average and mass average velocities in the z-direction  
are v tz and vtz, respectively. If the averaged (over one mole, 1 oi
U iz  d  F ext
tiz : 3:1:82
say) z-directional velocity of molecules of the ith species is fi z
viz, then the flux Niz is defined by (see Figure 3.1.3B)
The expression for Niz, (3.1.81), may now be rewritten as
N iz C i viz : 3:1:77 follows:
!
Obviously13 RT n ai
N iz C i v
tz U iz  C i : 3:1:83
f di z
12
See Lightfoot (1974, p. 54) for calculations of time scales for the In vector notation,
transient behavior of red blood cells (about 104 seconds) and !
comments on protein molecules. RT
13 Ni C i v
t U i  Ci rn ai 3:1:84a
The implication of viz vtz U iz or vi vt U i is f di
explored later in this section (see equation (3.1.102)). For an
observed viz , this will lead to a different U iz . and
92 Physicochemical basis for separation

1 D0is
Ui  r0i  F ext
ti : 3:1:84b Ui F ext 0
ti  ri ; 3:1:90b
f di RT
 
The quantity C i r n ai may be written as d n ai
rC i in where we have assumed that D0is is independent of the
d n C i
general. When ai i xi, then direction of diffusion.
  Since Uiz is directly proportional to D0is , the higher the
d n i
C i rn ai C t 1 rx i : 3:1:84c diffusion coefficient, the greater is the displacement or
d n x i T;P
migration velocity. In the appendix to this chapter (see
For nonconstant P, Tables 3.A.13.A.8), we list, the binary diffusion coefficients
for a few gaseous, liquid and solid systems. The diffusion
1
Ui  r0i  F ext
ti V i rP: 3:1:84d coefficients in solid systems are extremely low; those in
f di
liquid systems (D0is ) are a couple of orders of magnitude
We will now place the flux expression (3.1.84a) for Ni larger, whereas the diffusivities DAB for a gaseous system of
against our basic knowledge of mass flux expression in species A and B are almost five orders of magnitude larger
molecular diffusion. We have two objectives: first, to than those in liquid systems. For an introduction to diffu-
develop a simple way of estimating f di , and second to show sion in fluids, see Cussler (1997).
that we are familiar with the flux expression (3.1.84a) Values of diffusion coefficients are frequently needed
under a somewhat simpler situation. Consider an ideal in many separation calculations. Readers should refer to
liquid solution at constant T and P; therefore, ai xi. Reid et al. (1987) and Cussler (1997) for diffusion coeffi-
Assume C i C t x i , with Ct being the constant total molar cients in liquids and gases. For immediate use, the
concentration. Further, let there be no external force in this following correlations may be used.
single-phase liquid solution (i.e. F ext 0
ti 0; ri 0). Then
there is only molecular diffusion, given by Dilute solution of i in solvents The WilkeChang (Wilke
RT and Chang, 1955) correlation is given by
N i C i v
t  d rC i : 3:1:85
fi T cm2
D0is 7:4  108  As M S 1=2 ; 3:1:91a
This equation is simply Ficks first law of diffusion. Note S V 0:6
im s
that Ui 0, but U i is nonzero. Furthermore, for this binary
where
system of solute i in a liquid s, the ideal solution binary
diffusion coefficient D0is (valid at infinite dilution) is T is the absolute temperature in kelvin;
MS is the molecular weight of the solvent;
D0is RT=f di : 3:1:86 S is the solvent viscosity in centipoise;
Using this identification for D0is , the general flux expression Vim is the solute molar volume at boiling point, in
(3.1.83) may be rewritten as cm3 =gmol (estimate from Reid et al. (1987));
  As is the association factor for the solvent: 2.6 for water,
0 d n i x i
N iz C i v
tz U iz  C t Dis 1 ; 1.9 for methanol, 1.5 for ethanol and 1 for benzene,
d n x i T;P z
ether, heptane and other unassociated solvents.
3:1:87
Binary gaseous mixture of species A and B at low density The
or
ChapmanEnskog formula (Bird et al., 2002) is given by
x i r
N iz C i v
tz U iz  C t Dis ; 3:1:88  
z T 3 M1A M1B
cm2
where the diffusion coefficient for a nonideal solution of DAB 0:0018583 ; 3:1:91b
P 2AB D;AB s
solute i in solvent s, Dis, is defined by
  Here
d n i
Dis D0is 1 : 3:1:89
d n x i T;P T is the absolute temperature in kelvin;
P is the pressure in atmospheres;
The flux expression (3.1.88) clearly shows how Niz is AB 12 A B , A and B are LennardJones par-
increased by the presence of Uiz, the displacement or ameters for gases A and B (see Bird et al. (1960, table
migration velocity of species i in the z-direction due to forces B-1, p. 744); When values of are not known, the
that lead to separation. The corresponding expression for following empirical relations (Bird et al., 2002, p. 26)
Uiz in an isothermal isobaric system is given by may be employed using critical point information:
" #
D0is ext 0i ~ c 1=3 ;
0:841 V 2:44 T c =P c 1=3 : 3:1:91c
U iz F tiz  ; 3:1:90a
RT z
Here, the critical molar volume V ~ c has the units of cm3/
or gmol, Tc is in kelvin and Pc is in atmospheres.
3.1 Forces, displacements, velocities and fluxes 93

D ,AB is a dimensionless function of temperature and Table 3.1.2. Estimation of Perrin factor for various proteins
intermolecular potential field (Bird et al., 2002, table E.2).
Molecular r0, radius,
The magnitudes of D0is , as shown in Table 3.A.13.A.8 Protein weight pI (nm) f di =f di0
(see the appendix to this chapter), vary widely. Such a
Bovine serum 66 000 5.74 3.5 1.29
variation has clear implications for the suitability of differ-
albumin
ent phases in a separation system. For example, if the time Ovalbumin 45 000 5.08 2.78 1.16
required to achieve separation is to be determined from -Lactalbumin 14 200 4.57 2.3 1.18
that required to traverse a given distance by a given Myoglobin 16 900 7.1 2.4 1.18
species, then, other quantities remaining constant, this
time for species i will be directly proportional to, say, Uiz.
Solid systems will therefore require an extraordinarily
large time compared to a liquid or gaseous system (see q
Table 3.3.1 for some practical consequences; namely, for a f di 1  b2 =a2
 p ; 3:1:91f
practical two-phase based separation system, one phase f di0 b2=3 1 1b2 =a2
a n b=a
should be a fluid).
Of further interest here is the variation of f di with the where the volume of the prolate spheroid is equal to the
size of solute molecules i in a given system, liquid, gaseous volume of a sphere of radius r0,
or solid. Analogous to the value of f dp 6 r p (obtained
from Stokes law) for a single spherical particle of radius rp 4 4
a b2 r 30 : 3:1:91g
falling in a medium of viscosity , f di from equation (3.1.75) 3 3
may be represented by 6 r i N,~ where N ~ is Avogadros
For oblate ellipsoids (semiaxes a < b), correspondingly
number (6.02  1023/gmol) and ri is the hydrodynamic
r 30 a2 b and
radius of an ith solute molecule. We can therefore write
in general (for spherical molecules of hydrodynamic radius q
ri) from equation (3.1.84d) that f di 1  a2 =b2
" #: 3:1:91h
f di0 a1=3  1=2
1 tan1 1  ab2
2 b
Ui  d r0i  F ext
ti V i rP
b a
fi
3:1:91d
r0i  F ext
ti V i rP For a brief introduction, see Probstein (1989) for references
 :
~
6 ri N to the work by Perrin; for greater details, see Happel and
Brenner (1983). In Table 3.1.2, we list estimates of the
Estimates of ri, the hydrodynamic radius, may be obtained
Perrin factor for several proteins (information from Basak
from expressions (3.3.90ac). (The basic assumption here
and Ladisch (1995)).
is that Stokes law holds reasonably well for the motion of
The increase in the frictional coefficient is natural since
solute molecules also. In fact, it is routinely used for pro-
the surface area for the spheroid is larger than that of a
teins and macromolecules (Tanford, 1961).) Therefore for a
sphere of equal volume. In any given medium, when the
given magnitude of the driving force, as solute size
solute size increases, the frictional coefficient will increase.
increases, the displacement or migration velocity Uiz
For a given solute, as the medium viscosity increases, f di
decreases.
will increase, leading to a decrease in diffusion coefficients
If the solute shape is nonspherical, a relation other
and therefore in the displacement velocity. For linear flex-
than Stokes law will apply. For the determination of the
ible macromolecules, especially charged ones such as
resistance of nonspherical macromolecules, the reader
polyelectrolytes, estimation of the drag force is compli-
may consult pp. 356364 of Tanford (1961). We will pro-
cated; see Viovy (2000) for an introduction. Note that Ui
vide a very brief perspective on this effect here. Some cells,
for a polyelectrolyte that is uniformly charged is likely to be
and especially many proteins, are ellipsoids of revolution.
independent of its length in free solution; the situation is
The drag force encountered by such an ellipsoid of revolu-
different in gels.
tion of species i is described in terms of the drag encoun-
tered by a sphere of equal volume whose radius is r0 via an Example 3.1.5 Case (1) Calculate the displacement or
appropriate correction factor f di =f di0 which is always migration velocity of ovalbumin molecules subjected to the
greater than 1. This factor is called the Perrin factor. The conditions in a centrifuge described in Example 3.1.2,
drag
magnitude of F i is enhanced by this factor, i.e. case (1), where the solution viscosity is 1 cp.
Case (2) Calculate the terminal settling velocity of the
Fi
drag
6 r 0 U i f di =f di0 : 3:1:91e particles in Example 3.1.2, case (2), where the solution vis-
cosity in 1.5 cp. You are given that for ovalbumin
For prolate ellipsoids (semiaxes a > b), r i 2:78 nm and f di =f di0 1:16.
94 Physicochemical basis for separation

Solution Case (1) If we assume Stokes law to be valid, This migration velocity appears to be much larger than
then the radial displacement/migration velocity of ovalbu- that generated in the centrifugal field of Example 3.1.5,
min molecules is given by case (1).

5:60  1011 g-cm=s2 Example 3.1.7 Determine the gravitational terminal settling
U ir F tir=gmol =f dijgmol
~ f di =f di0
6 rp N velocity of particles in the following two systems.

g-cm (1) Particle of diameter 10 m, density 2 g/cm3 in a solution


5:60  1011 2 of density 1.3 g/cm3 and viscosity 1.5 cp.
s
g (2) E. coli cells of diameter 1 m, particle density 1.09 g/cm3,
6  1  102  27:8  108 cm
cm-s solution density 1 g/cm3 and viscosity 1.5 cp.

1:16  6:02  1023 Solution Case (1)


4 3
3 r p p g 1  t =p
11 13
5:60  10  10 cm jF tz j
0:151  104 cm=s: U pzt   g
36:12   32:48 s 6 rp 6  1:5  102  r p cm
cm-s
Check now the validity of Stokes law:
2 r 2p g p 1  1:3=2 cm
U ir r i 
Reynolds no: 9  1:5  102 s

g 2  5  104 2  980  2  0:35
0:151  104 cm 8
s  27:8  10 cm  1 cm3 
13:5  102
1  102 g=cm s
10
4:19  10 <<< 0:1; 100  980  0:35  108  102 cm

assumption okay. 13:5 s
Case (2) Uprt, the terminal settling velocity of the particle,
0:35  0:98  101 cm
is given by  2:54  103 :
13:5 s
F tr jparticle 0:179  103 g-cm=s2
U prt g
6 rp The terminal velocity in the centrifugal field of Example
6  1:5  102  5  104 cm
cm-s 3.1.5, case (2), is much larger than that due to the gravita-
0:179  103 cm tional field.
1:26 cm=s: Case (2)
45 s
2 r 2p g p 1  1=1:09 cm
Check for validity of Stokes law: U pzt 
9  1:5  102 s
U prt r p 1:26  5  104  1
Reynolds no: 0:042 < 0:1; 2  0:5  104 2  980  1:09  1  0:917 cm
1:5  102 
13:5  102 s
Stokes law is valid.
2  0:25  108  980  1:09  0:083
Example 3.1.6 Compute the migration velocity of ovalbu- 
13:5  102
min molecules subjected to an electrical field of uniform
strength described in Example 3.1.3. The solution viscosity 0:5  0:98  1:09  0:83  104 cm
is 1 cp. Other required information is provided in Example 
13:5 s
3.1.5, case (1).
0:32  105 cm=s:
Solution The migration velocity of ovalbumin molecules is
If we had used the centrifugal field of Example 3.1.2, case (2),
F ti jgmol we would have obtained a settling velocity of
Ui
~ f di =f di0
6 r p N
4 r 3p p r 2 1  t =p 2 2
2 r p r p 1  t =p
1:3  109 newton U prt
3  6 rp 9
g
61102  28  108 cm  6:02  1023  1:16
cm-s 2  0:5  104 2  1  70002  1:09  1  0:917

g-cm 9  1:5  102
1:3  109  105 2
s
g 0:5  108  49  106  1:09  0:083
6  28  6:02  1:16  1013
s 13:5  102
1:3  1014  1013 cm 0:5  49  1:09  0:083 cm
0:35  102 cm=s: 0:164 cm=s;
3685 s 13:5 s
3.1 Forces, displacements, velocities and fluxes 95

an orders of magnitude higher velocity than that due to laser power  particle cross-sectional area
Pr
gravity. Gravity is therefore not likely to be used for the laser beam cross-sectional area
separation of E. coli cells from water. 19 mW  r 2p
Example 3.1.8 Suppose you have a solution of a solute of 19 mW  r p =r l 2 :
r 2l
molecular weight 100 such that the concentration of the
solute on two sides of the solution are 10 gmol/cm3 and Introduce it into equation (3.1.47) to obtain
1 gmol/cm3. Calculate the value of Ui from the high-
concentration region to the low-concentration region. 2qf r Pr 2  0:1  19mW  r 2p
F rad 6 r p U pzt
c c r 2l
Solution This is a case of random Brownian diffusion; Ui
cannot be defined for this case. ) U pzt particle terminal velocity
Example 3.1.9 In Example 3.1.8, the phenomenon under 2  0:1  19  103 watt  r 2p
consideration was ordinary molecular diffusion. Ordinary
c  6  1 cp  r p  r 2l
molecular diffusion due to a concentration gradient or partial
pressure gradient (see relations (3.1.40) for example) does g-cm2 1
result in a molecular velocity i (we assume no bulk 2  0:1  1:34  104 cm  19  103 watt  107 2
s -s watt
motion of the gas or liquid phase). Here we would like to cm g
6  2:799  1010   102  6:2  6:2  108 cm2
calculate this velocity for arbitrary values of concentration s cm-s
difference or partial pressure difference over a diffusional 2  0:1  1:34  19
path length. cm=s 25  104 cm=s
6  2:799   6:2  6:2
(1) Ovalbumin molecules in an aqeuous solution (stagnant)
diffuse due to a concentration ratio of 10 over a distance (Ashkin, 1970). Note: This radiation pressure driven motion
of 10 cm. has other complex dimensions. If the laser beam hits the
(2) Same as case (1) except the magnitude of rn ai is 1 cm1. spherical particle off-center, then the particle is drawn also
to the laser beam axis.
Solution
We need to have additional considerations on the ith
Fti RT rn ai ;
species flux expression (3.1.87). The vectorial form of this
Fti RT expression for a liquid solution is good for a binary system of
vi  d rn ai Dis rn ai :
f di fi species i and species s
 
0 d n i
Case (1) N i C i v
t U i  C t D is 1 rx i 3:1:92
d n x i T; P
n ai1 =ai2 cm2 2:303  log10
iz Dis 7  107
10 s 10 Here species i moves only (1) due to concentration gradient
7  2:303  108 cm=s 1:61  107 cm=s: of species i (since C t rx i may be written as rC i ), (2) the
external species i-specific forces and (3) the bulk motion (if
Case (2) any are considered via v
iz 7  107  1 7  107 cm=s: t ). If there is a concentration gradi-
ent of any other species or some external forces specific to
Note: These velocities are orders of magnitude smaller than that other species, they are not supposed to influence Ni as
the values of Uiz for the ovalbumin molecules in Examples written above except through their contribution to the bulk
3.1.5 (centrifugal acceleration) and 3.1.6 (uniform electrical velocity v
t .
field). Therefore diffusion/Brownian motion is a small nuis- Equation (3.1.92) is most useful for liquid-phase systems.
ance in the separation of proteins in these external force For a binary gaseous phase mixture of species A and B, use
fields having appropriate values. equations (3.1.81), (3.1.82), (3.1.86) and (3.1.88) to obtain the
flux of species A as
Example 3.1.10 If, however, we had considered the diffu-
sion of a gas molecule in a gaseous medium, the diffusion   n ^f A
coefficient would be orders of magnitude higher (e.g. N Az C A v
tz U Az  DAB C A ; 3:1:93
z
DHeN 2 0:687 cm2 =s at 25  C); therefore the values of viz
will also be so much larger (for, say, r n ai of 1 cm1). where we have assumed that n f 0A =z 0.
For an ideal gas mixture, ^f A pA and pA CART; the
Example 3.1.11 Determine the terminal velocity achieved by above equation is then reduced to
a spherical particle of radius 1.34 m in water when subjected   DAB dpA
to a cw argon laser of power Pr 19 mW. The laser beam N Az C A v
tz U Az  : 3:1:94
RT dz
radius rl is 6.2 m; its wavelength 0.5145 m. The viscosity
of the liquid medium (water) is 1 cp. Assume qfr 0.1. Vectorially, this equation is written as
  DAB
Solution Since the laser beam cross section corresponds to N A C A v
t UA  rpA : 3:1:95
a diameter r l of 6.2 m, only a small fraction of its total RT
power of 10 mW is falling on the particle of radius 1.34 m; Since DAB DBA, the flux expression for species B is,
therefore the value of power Pr in equation (3.1.47) is correspondingly,
96 Physicochemical basis for separation

Table 3.1.3A. Different mass and molar fluxes of a species and their relationships to various velocitiesa

Frame of reference Mass flux of species i Sum of mass fluxes Molar flux of species i Sum of molar fluxes

Flux in a fixed frame of reference ni i vi n


X N i C i vi n
X
(fixed coordinate axes) ni vt N i C t v t
i1 i1
(a) (b) (c) (d)
Flux in a frame of reference moving j i i vi  vt n
X J i C i vi  vt n
X
with mass average velocity, vt ji 0 J i C t v t  vt
i1 i1
(e) (f) (g) (h)
Flux in a frame of reference moving j i i vi  v t n
X J i C i vi  v t n
X
with molar average velocity, v t j i v  v t J i 0
i1 i1
(i) (j) (k) (l)
a
vi , averaged velocity vector of species i; vt , mass averaged velocity (equation (2.1.1), v t , molar averaged velocity (equation (2.1.2)).

Table 3.1.3B. Important relations between different fluxes

Frame of reference Mass flux of species i Molar flux of species i

Flux in a fixed frame of reference (fixed coordinate axes) ni N i ni =M i


Flux in a frame of reference moving with vt ji J i j i =M i
Flux in a frame of reference moving with v t j i J i j i =M i
n
X
Relation between N i and J i N i C i v t J i a N i xi N i J i b
i1
Xn
Relation between j i and ni ni i vt j i (c); n i i ni j i d
i1

Table 3.1.3C. Relations between important species fluxes, bulk velocities, migration velocities and
components of chemical potential gradients

   
d
N i C i v t U i  C i RT
f di
rn ai C i v t  f1d r0i  F ext
ti V i rP  C i RT=f i rn ai (a)
i
   
d
J i C i U i  C i RT
f di
rn ai C i  f1d r0i  F ext
ti V i rP  C i RT=f i rn ai (b)
    i
J i C i v t  vt U i  C i RTfd
rn ai (c)
i

  DAB  
d n i
N B C B v t U B  rpB : 3:1:96 J
i C i U i  C D 0
t is 1 rx i : 3:1:99
RT d n x i T;P
In simple molecular diffusion in a binary system under iso-
baric condition, Various other mass and molar fluxes and some relations
between the different flux expressions are explored in Section
U A 0 UB ; rpA pB 0: 3.1.3.2.1. The expressions for J i and J i for simple molecular
Thus diffusion when U i is zero are given below:
N A N B v  
t C A C B C t vt :

3:1:97
0 d n i
J i C i v
t  v i  C t D is 1 rx i ; 3:1:100
Expression (3.1.92) describes the species i flux in a binary d n x i T;P
system across a cross section stationary with respect to the
 
fixed coordinate axes. There can be a number of other flux 0 d n i
expressions depending on the chosen nature of the motion of J
i C t D is 1 rx i : 3:1:101
d n x i T;P
this cross section (across which the species flux takes place)
with respect to the fixed coordinate axes. If the cross section Tables 3.1.3AC summarize three items regarding the vari-
moves with the mass average velocity of the mixture vt, the ous fluxes, species velocities and forces: Table 3.1.3A identi-
molar flux identified as Ji is given by fies the definitions of species fluxes with respect to different
 
0 d n i frames of references with respect to the individual species
J i C i fv
t  vt U i g  C t Dis 1 rx i :
d n x i T;P velocities; Table 3.1.3B provides the interrelationships
between the different flux expressions in Table 3.1.3A; Table
3:1:98
3.1.3C illustrates the relation between the important fluxes,
If the cross section moves with the molar average velocity of bulk velocities, migration velocities and components of
the mixture v t , the molar flux identified as J i is expressed as

chemical potential gradients.
3.1 Forces, displacements, velocities and fluxes 97

The definition in (3.1.90b) of the migration velocity Ui is electrical field. The two basic relations needed for this
independent of any of the frames of reference used above to development are given below.
define the fluxes J i and J
i , Ni, etc. However, if we define U i For calculations in electrolytic solutions, which may or
with respect to the observed species velocity vi by may not be subjected to an electrical field, the principle of
new electroneutrality holds (see Chapman (1969) or Newman
vi v t U i ; 3:1:102
(1973, 1991) for an introduction to ionic transport and elec-
instead of the definition (3.1.78), we will get trochemical systems):
new
X
N i C i vt C i U i 3:1:103 Z i C i 0: 3:1:108a
i1
RT C i
C i vt C i U new
i  r n ai : 3:1:104 This condition implies the absence of any net charge any-
f di
where (except in the double layer (Figure 3.1.2D) and elec-
This suggests that the alternately defined migration velocity trode surfaces). In the absence of any electrical current in the
U new
i is related to U i of definition (3.1.90b) by elelctrolytic solution,
X
U i U new
i vt  v
t : 3:1:105 Z i N i 0; 3:1:108b
i1
These two definitions become identical only when vt v t
since the current density i (amp/m2) is defined by
(which is true, for example, for a dilute solution of species i in
X
solvents s). The flux expressions described earlier for differ- iF Zi N i: 3:1:108c
ent frames of reference will also change if U newi is used. i1
It is appropriate to indicate now an approximate form of flux
The current in a solution is carried by all ionic species, i
force relation in a binary isothermal liquid mixture in the
1,. . .,n. The fraction of the total current carried by a particu-
presence of an electrical potential gradient. It is called the
lar ionic species i in a solution is defined as its transport
NernstPlanck relation and is used for systems containing ions:
number,
 
xi Z i F Zi F N i
J i C t DN
is rx i r : 3:1:106 t is : 3:1:108d
RT n
X
Zk F N k
It has often been used for a dilute binary solution of
k1
species i and solvent s under the following conditions: in
the presence of (i) a concentration gradient and (ii) an The transport number of any ion is always positive.
electrical potential gradient; there is no convection; Obviously,
rP 0. Here DN is is the NernstPlanck binary diffusion n
X
coefficient for species i and solvent s. In the absence of any t is 1: 3:1:108e
electrical field r in the system, it reduces to the ordinary i1

binary diffusion coefficient D0is if the solution behaves When electric current flows in an aqueous electrolytic solu-
ideally. Note that it is not necessary for r to be gener- tion, some amount of water is also transferred along with the
ated by an external source; it can be generated within ions in the current direction. The transference number of
the system by diffusion of ionic species. An individual ion water, t w , is defined as the number of water moles transported
has no way of knowing the source of the electrical field with 1 faraday of electricity in the current direction. Substitute
(Helfferich, 1962). w by i, and t is is the transference number of any species i in
In the solution of an electrolyte, the intrinsic migration the solution. It is related to the transport number tis by
velocities of ions (see equations (6.3.8c, e) and Table 6.3.1)
are, in general, different. Thus, the velocity of the cation is t is t is Z i : 3:1:108f
different from that of the anion. In the absence of any exter- Note that, unlike t is , t is may be positive or negative. But t ws is
nal electric field, these differential migration velocities create zero for water always; the transport number for uncharged
a potential gradient, the diffusion potential, which reduces water molecules is always zero.
the speed of the faster ion and increases the speed of the The NernstPlanck equation (3.1.106) is valid in the
slower ion. Ultimately, the rate of diffusion of each ionic absence of any convection. In the presence of convection,
species is equal to that of the binary electrolyte. The one can use the extended NernstPlanck equation:
diffusion coefficient D of a binary electrolyte, e.g. AZ Z
Y   
in a dilute solution, given by (Newman, 1973, 1991) Ci Z i F
N i DNis rC i r C i grad n i C i vt ;
RT
D0 D0 Z  Z 
D ; 3:1:107 3:1:108g
Z D0  Z  D0
where i is the activity coefficient of the ith ionic species.
reflects this phenomenon. Here D0 and D0 are the intrin- The activity coefficient term is often neglected, and the
sic diffusion coefficients of the cation and the anion, mass velocity vt is used instead of molar average velocity.
respectively (see illustrative values in Table 3.A.4). Section At this time, one should ask what is the migration velocity
3.1.3.2.2 is concerned with developing this result for dilute U i of the ith ionic species exposed to an electrical field r
solutions with concentration-independent diffusion coeffi- and what is its contribution to the total ionic flux N i ? From
cients in the absence of any current and externally applied definitions (3.1.82) and (3.1.84b),
98 Physicochemical basis for separation

1 Equation (3.1.108c) has already defined the current density


Ui  r0i  F ext
ti :
f di i (amp/m2) in the presence of the electrical field. If there is no
bulk fluid motion and no concentration gradients, the species
In a solution, r0i is zero since the standard state 0i exists in solution move due to the electrical field only; therefore
everywhere. Per gmol of ith ionic species, the value of F ext
ti is X X
Z i F r . Therefore for an ionic species i in solution, iF Zi N i F 2 Z 2i um
i C i r; 3:1:108o
i1 i1
Z i F r
Ui  3:1:108h which may be written in the manner of Ohms law as
f di
The flux due to this ionic velocity when the molar ion con- i r E; 3:1:108p
centration is Ci is where , defined as
C i Z i F r X
Ci U i  Z i um F2 Z 2i um
i Ci ; 3:1:108q
i F C i r: 3:1:108i
f di i1

Here umi is identified as the ionic mobility of the ith ion and is
is the electrical conductivity of the solution and has units of
the average velocity of the ionic species in a solution when siemens/meter, S/m, where the unit of siemens is amp/volt.
acted on by a force of 1 newton/mol. Therefore The quantity most often reported is the ionic equivalent con-
U i um ductance due to a specific ion i, i; this is obtained by dividing
i Z i F r : 3:1:108j
i, the contribution to by ionic species i, F 2 Z 2i um
i C i , by
The units of the ionic mobility um
i are m/(s-newton/gmol). Ci, the molar concentration of ions i and Zi:
From the extended NernstPlanck equation (3.1.108g), we  
see that i  
i Z i  F 2 um
i : 3:1:108r
Zi Ci
DN
is Z i F
Ci U i Z i um
i F C i r C i r Here, jZ i j is used to indicate that conductance is a positive
RT
quantity. The units of i/Ci are S-m2/gmol, and those of i are
DN S-m2/g-equiv.
) um
i
is
) DN m
is RT ui :
RT (Note: The ohm is the unit for volt/amp; mho is the unit for
amp/volt, therefore mho siemens. Correspondingly, the
3:1:108k units of i are mho-m2/g equiv.)
This result is called the NernstEinstein relation; it relates the Sometimes the equivalent conductance of a salt produ-
diffusivity of the ion to the ionic mobility and is valid only at cing a cation and an anion is used:
infinite dilution. It is clear from relations (3.1.108h) and : 3:1:108s
(3.1.108i) and the definition (3.1.86) of diffusion coefficient
D0is at infinite dilution that Table 3.A.8 lists the values of 0i
for a few common ions
corresponding to infinite dilution. Their diffusion coefficients
1 D0is (based on the NernstEinstein relation, (3.1.108k), namely
um
i ; 3:1:108l
f di RT Dis RT um 2
i RTi =jZ i j F , are listed in Table 3.A.4.
One can now use the definition of ionic mobility um i to
leading to relation (3.1.108k). However, in the conventional redefine the transport number tis in cases where there is no
literature on capillary electrophoresis etc., ionic mobility, concentration gradient in the solution. Use equation
mi , is defined as (3.1.108g) to get
U i m m
i r i E: 3:1:108m
Z 2i F 2 C i um
i r Z 2 C i um
t is X Xi 2 i m 3:1:108t
The units of this ionic mobility m
i are either F2 2 m
Z i ui C i r Z i C i ui
m cm i1 i1
or : P P
s-volt=m s-volt=cm
since vt i1 Z i F C i vt F i1 Z i C i 0 due to the elec-
Comparing this definition of m i in the context of relation troneutrality condition (3.1.108a).
(3.1.108h), we see that So far, the medium in which the species were diffusing was
Zi F either gaseous or liquid. Diffusion of a species i through a
m
i Z i F um
i : 3:1:108n solid with no holes, defects or pores is also encountered in
f di
separation processes. In general, Ficks first law is assumed
Recall that f di here is the frictional coefficient for 1 gmol of to be valid without any convective component:
ions whose total charge is Z i F coulombs; there are 6.022 
1023 ionized molecules in 1 gmol, each having a charge of N i Di rC i : 3:1:109a
Z i e, where e is the charge of an electron, namely 1.602  For one-dimensional transport in a planar geometry (say the
1019 coulomb, leading to a total charge of z-direction),
coulomb dC i
~ Z i  1:602  1019  6:022  1023
Z i eN N iz Di : 3:1:109b
gmol dz
Z i  96500 coulomb=gmol: For radial transport in a cylindrical or spherical geometry,
3.1 Forces, displacements, velocities and fluxes 99

dC i 3.1.3.2.2 Diffusion of a binary electrolyte in a solution


N ir Di : 3:1:109c
dr Consider the diffusion of an electrolyte AZ Z
Y  in a dilute
The diffusion coefficient Di may or may not vary in the solution without any applied electrical field. In order to
direction of diffusion. More on transport of a species through show that the diffusion coefficient expression (3.1.107) is to
a solid material is provided in Section 3.4.2. The solid mater- be used for the electrolyte as a whole or for any of the ions,
ial considered there is in the form of a thin membrane with we start with the flux expressions for the two charged
or without holes, pores or defects. Diffusion in liquid and species in a stagnant medium. Here the unknown diffusion
gaseous phases through a porous solid material/membrane potential is to be eliminated. Define the molar concentra-
is considered in Sections 3.1.3.2.3 and 3.1.3.2.4. Evaporative tions of positively charged and negatively charged ions in
flux of molecules from a free liquid surface under high
solution as C and C  gmol=cm3 , respectively.
vacuum is described in Section 3.1.3.2.5.
From the electroneutrality relation, (3.1.108a),

3.1.3.2.1 Some relations for different flux expressions Z C Z  C  0 Z rC Z  r C  : 3:1:111a


Two aspects are illustrated here. We consider a binary From (3.1.108b,g), for v t 0, we get
system of solute i and solvent s. First, we write down
expressions for mass fluxes ni and ji, since other relevant D0
Z N Z  N  0 Z C Z F rD0 rC
ones have been illustrated already. Second, and more RT

important, the values of quantities like J i J s and D0
Z  C   Z  F rD0 rC  0;
N i N s are determined for two cases: (1) migration vel- RT
ocities Ui and Us are nonzero; (2) U i and U s are both zero.
By definition, ni , the mass flux of species i across a where we have assumed ideal solution behavior. This last
cross section stationary with respect to the fixed coordinate relation may be rearranged to yield
axes, is related to N i (from equation (3.1.92) and Table
F D0  D0 rC
3.1.3B) by r ; 3:1:111b
RT C Z D0  Z  D0
d n i
ni N i M i M i C i v t U i  M i C t D0is 1 rx i : using result (3.1.111a). Now substitute this expression for
d n x i T;P
diffusion potential r into the NernstPlanck expres-
3:1:110a
sion (3.1.106) for
Similarly,
C D0 Z
J N D0 rC F r
d n i RT
j i M i J i M i C i fv t  vt U i g  M i C t D0is 1 rx i ;
d n x i T;P D0 Z D0  D0
3:1:110b D0 rC rC
Z D0  Z  D0
where we have used equation (3.1.98) for Ji. D0 D0 Z  Z 
 rC :
When the migration velocities are nonzero, we obtain, Z D0  Z  D0
from the flux expression (3.1.99) for J i , the following:
3:1:111c
J i J s N i N s  v t C i C s : 3:1:110c
One can obtain the same diffusion coefficient in J  and J AY .
By definition (3.1.79), N i C i v t C i U i and N s
C s v t C s U s . Therefore
3.1.3.2.3 Solute flux through porous liquid-filled
N i N s v t C i C s C i U i C s U s C t v t C i U i C s U s : material We consider here the expression for solute i
flux through a porous liquid-filled pellet/bead/membrane.
But by definition also N i N s C i vi C s vs C t v t .
Under conditions of no convection and solute radius, ri,
Therefore J i J s 0. Also
being at least two orders of magnitude smaller than the
C i U i C s U s 0: 3:1:110d pore radius, rp, (ri <<< rp), the governing equation for
p
solute flux N iz along the pore length (z-coordinate) in the
These relations are also valid when migration velocities are pore fluid is simply Ficks first law (3.1.85) with v t 0:
zero.
p
A similar result can be obtained for j i j s . By definition p d C im
N iz  Dil : 3:1:112a
(see Tables 3.1.3A and B), dz
p
This expression for N iz provides the moles of i being trans-
j i j s M i J i M s Js M i N i  C i vt M s N s  C s vt
ported per unit pore cross-sectional area per unit time.
i vi  i vt s vs This flux expression also assumes that there are no elec-
s vt fi vi s vs g  vt i s 0: 3:1:110e trostatic interactions between the pore material and the
100 Physicochemical basis for separation

solute. Here subscript m refers to the porous pellet/bead/ is available as a function of the radial location, r, then the
membrane. The solute concentration in the liquid within solute flux averaged over the pore cross section, N iz , is
p
the pore, C im , is related to the overall solute concentration   " 1ri =rp    # p
Cim in the pellet/bead/membrane having a porosity/void Dil m r r d C im
N iz  2 GDr r i ;r p ;r d ;
volume fraction m by m 0 rp rp dz

p 3:1:112g
C im m C im : 3:1:112b
p
where the effective diffusion coefficient under no-
The pore concentration of solute i, C im , is related to the convection conditions, Die or Dim may be expressed as
external solution concentration Cil via a partition coeffi-
p
cient im (see Section 3.3.7.4). When there is no partition- d C im
N iz Die ; 3:1:112h
ing effect, im 1. In reality, the pores are tortuous so that dz
solute molecules traverse a distance longer than the pellet/ where
bead/membrane thickness m; this is commonly taken into
  " 1ri =r p    #
account by a correction factor, the tortuosity factor, m. Dil m r r
Die 2 GDr r i ;r p ;r d :
Correspondingly, if the pellet/bead/membrane pore length m 0 rp rp
is to be considered as m, then the solute flux in moles per 3:1:112i
unit pellet/bead/membrane cross-sectional area per unit
time Niz is expressed as When there is convection through the pores along with
p p diffusion, the solute flux Niz has a diffusive as well as a
p Dil m d C im d C im
N iz m N iz  Dim ; 3:1:112c convective component:
m dz dz
Dil m GDr r i ;r p d C pim p
where the effective diffusion coefficient of solute i for the N iz  Gc C im vz m :
m dz
porous pellet/bead/membrane Die or Dim is defined as
diffusive convective
Dil m
Die Dim : 3:1:112d 3:1:113
m
Here, vz is the local fluid velocity in the pore and Gc is a
Here Dil is the conventional diffusion coefficient of species
factor by which the solute velocity is reduced from the fluid
i in the liquid (Tables 3.A.2 and 3.A.3). Such an expression
velocity (Anderson and Quinn, 1974). One can also average
is frequently used for pellets/beads/particles used in vari-
this flux expression over the whole pore cross section in the
ous separation techniques as well as for membranes.
manner of expression (3.1.112g). The quantity Gc is called
(Empirical estimates of m made in a variety of catalyst
the convective hindrance factor; a pore-average value of
particles, adsorbents and membranes are available: silica
this may be developed as in expression (3.1.112i). The
alumina ~2.12.3 (Satterfield et al., 1973); hollow fiber
treatment provided assumes that there are no solutepore
membranes ~214 (Prasad and Sirkar, 1988)).
wall interactions via adsorption or electrical potential
When the solute dimension is no longer orders of
effects.
magnitude smaller than the pore dimensions, the solute
molecules experience an additional transport resistance
due to the proximity of the pore wall. The effective diffu- 3.1.3.2.4 Gas transport through porous beads/pellets/
sion coefficient is reduced further by a hindrance factor/ membranes In gas transport through porous materials/
drag factor GDr r i ;r p . If the solute molecules are assumed membranes, if the matrix openings are large enough to
spherical of radius ri and diffusing through the centerline prevent molecular diffusion encountered in nonporous
of the pore of radius, rp, Faxens expression may be used to materials, gas convection occurs. Three types of gas trans-
estimate GDr for r i ;r p
0:5 (Lane, 1950; Renkin, 1954): port in the gas phase through the porous/mesoporous/
microporous material/membrane openings (pore or holes)
   3  5
ri ri ri have been postulated. The type of gas convection mechan-
GDr r i ;r p 1  2:1044 2:089  0:948 ...
rp rp rp ism is determined by the relative values of the mean free
3:1:112e path of the gas molecules (average distance traversed by
a molecule before it suffers collision with another gas
The solute flux expression (3.1.112c) is now changed to
molecule) at the prevailing pressure and the matrix
Dil m GDr r i ;r p d C pim opening/pore diameter 2rp. When (rp/) < 0.05, Knudsen
N iz  : 3:1:112f flow or gaseous diffusion takes place (Liepmann, 1961).
m dz
When (rp/) > 50, conventional Poiseuille or viscous flow
For solute molecules having ri<<rp, GDr r i ;r p 1. The occurs. In the intermediate range, slip flow is observed.
flux of solute will vary with the radial location in the pore as barrer (1963) has provided a review of the different types of
well (Anderson and Quinn, 1974). If the local value of GDr flow in a microporous medium or membrane.
3.1 Forces, displacements, velocities and fluxes 101

According to the rigid sphere model in the kinetic binary gas mixture and DAK is the Knudsen diffusivity of
theory of gases, the mean free path is given by (Kennard, species A. An alternative way of writing this expression, namely
1938)
r 1 P dx Ag
1:256 RT N Az 1    DAK ; 3:1:115e
: 3:1:114 1
x Ag DAK
1 RT dz
NR DAB
P Mi

As the total pressure decreases, increases. At very low shows that, at low pressures where DAK/DAB <<1, this
pressures, then, Knudsen flow is likely, whereas at high expression reduces to the Knudsen flux expression
pressures, viscous or slip flow will dominate. (3.1.115a); at high pressures, DAK/DAB >> 1 (since (DAK/
The steady state Knudsen diffusion flux of a gas species DAB) is proportional to P (from definition (3.1.115c), DiK is
i of molecular weight Mi through straight circular pores of independent of P and from definition (3.1.91b), DAB is
radius rp and length m under the condition of total pres- proportional to (P)1)) and the expression reduces to that
sure being the same at both ends of the pore is given by due to molecular diffusion.
When the total pressures at the two ends of the pore
p 4r p 2RT 1=2 d C i 4r p 2RT 1=2 1 dpig
N iz   ; are different, there will be bulk flow. If conditions for
3 Mi dz 3 Mi RT dz
Knudsen diffusion exist, then the molar average bulk flow
3:1:115a
velocity (v z ) may be estimated from
This flux expression is based on the pore cross section only;  
for the second expression on the right-hand side, we have 4r p m 2RT 1=2
C tg v z N Az N Bz 
assumed ideal gas behavior. For a pellet/bead/membrane 3 m

of porosity/void volume fraction m and pore tortuosity 1 1 dpAg 1 dpBg


p p ; 3:1:116a
factor of m, the gas species flux based on the total cross RT M A dz M B dz
section of the pellet/bead/membrane is where Ctg CAg CBg for a binary system.
 1=2 When Poiseuille flow conditions exist, we use the
4r p m 2RT 1 dpig
N iz  : 3:1:115b expression
3 m Mi RT dz
dP 8i dpAg dpBg
An appropriate expression for the Knudsen diffusivity Dik  2 vz   3:1:116b
of species i through a straight circular pore is dz rp dz dz
   1=2
4r p 2RT 1=2 T to estimate the mass average velocity z for the binary gas
DiK 9:7  103 r p cm2 =s;
3 Mi Mi mixture flow. The molar flux of a gas species i under such
3:1:115c conditions can be estimated from vz or (v z ) and a mean
where rp is the pore radius in centimeter. Since rp <<< density (Cmean)i of species in the gas from one end of the
in Knudsen flow, gas molecules of species i collide with the pore to the other. Approximate values of the tortuosity
pore wall rather than with other gas molecules of species i factor m for a variety of beads/pellets in gas transport/
or species j. Therefore Knudsen diffusion based flux of any separation processes have been summarized by Yang
species i is merely proportional to M i 1=2 . When the pore (1987): alumina, 26; silica gel, 26; activated carbon,
dimensions are larger and/or the gas pressure is high 565; micropores in zeolites, 1.74.5.
enough so that (rp/) > 50, the equations of molecular In gas transport through the pores of a pellet/bead/
diffusion (3.1.94) should be used for a gas mixture when particle/membrane, the gas molecules may also be
the total pressure is the same at both ends of the pore. adsorbed on the pore wall or the solid surface. This leads
For conditions intermediate between Knudsen flow to an additional type of gas transport, namely surface diffu-
and Poiseuille flow, slip flow or transitional flow takes sion. Normally, the surface adsorbed species are assumed
place. The molar flux of species A through the circular to be in equilibrium with the bulk gas phase being con-
pores under such conditions has been shown for a binary vected through the pores. The pore gas pressure (or partial
gas mixture of species A and B to be (Scott and Dullien, pressure) usually decreases in the direction of transport.
1962; Rothfeld, 1963) Therefore, the concentration of adsorbed gas species in the
2 1 3 pore walls also decreases in the direction of transport,
x Ag leading to a concentration gradient of surface-adsorbed
61 1 7
6 NR 7 P dx Ag molecules which diffuse downhill, generating what is often
N Az 6 7 ; 3:1:115d
4 DAB DAK 5 RT dz called the surface flow. The additional flux due to surface
flow is expressed via a Fickian diffusion-type expression:

where NR NAz/[NAz NBz]) is the flux ratio (see equation d Eis


N iz Dis asp ; 3:1:117a
(3.1.129a)), DAB is the ordinary diffusion coefficient of the dz
102 Physicochemical basis for separation

where Eis is the molar excess surface concentration of and this last ratio is obtainable from adsorption isotherm
species i in moles/cm2 of the pore surface area, Dis is the measurements. Hwang and Kammermeyer (1975) have
surface diffusion coefficient of species i and asp is the pore provided an overview of surface diffusion in microporous
surface area in cm2/unit volume (cm3) of the porous/ membranes. Yang (1987) has summarized surface diffu-
microporous medium of porosity (pore volume/per unit sion in pellets and beads used for adsorption processes.
volume of the porous medium) (Ash et al., 1973a, b).
Alternative approaches employ the molar surface concen- 3.1.3.2.5 Evaporative flux of molecules from a free
tration of i per unit weight of the adsorbent multiplied by liquid surface under high vacuum When molecules
the density of the adsorbent instead of the product is asp. evaporate from a liquid surface, molecules also come back
For dilute adsorbed films, each species diffuses independ- from the vapor phase into the liquid. Therefore, the net
ently of the other (Ash et al., 1973b). rate of evaporation from a liquid is determined by the net
Surface flow exists along with flow along the pore by rate of heat transfer to the liquid. However, if conditions of
Knudsen diffusion, molecular diffusion and/or convection. high vacuum exist on the evaporating liquid surface and
If Knudsen diffusion exists, then the fluxes due to the two there is a cold condensing surface in the immediate vicin-
different mechanisms are ity, the molecules escaping the evaporating surface are
  condensed on the condensing surface; this process, called
 dC ig  dasp Eis
N iz  DiK ; N iz  Dis molecular distillation, is characterized by the following flux
Knudsen dz surface flow dz
expression for a pure liquid (Langmuir, 1916):
dC is
Dis ; 3:1:117b P sat
dz i
N iz p
: 3:1:119a
2 M i RT
where Cis is the molar concentration of the surface
adsorbed species per unit volume, the porous medium. For a liquid mixture having a mole fraction xil of species i,
The ratio of the two fluxes is
P sat
i x il
N iz p
: 3:1:119b
N iz =surface Dis d Eis 2 M i RT
asp : 3:1:117c
N iz =Knudsen DiK d C ig
Here, the vapor pressure units are dyne/cm2 and R is in
The total flux due to the two mechansims is given by erg/gmol-K.

DiK dpig d E
N iz  Dis asp is : 3:1:118a 3.1.4 Integrated flux expressions for molecular
RT dz dz
diffusion and convection: single-phase systems
However, since there is likely to be an equilibrium between pi
The flux expressions we have considered so far for a mixture
in the gas phase and isE in the surface phase, one can reduce
of chemical species often relate the flux to a number of
this expression to a relation between Niz and (dpi/dz).
gradients of potentials. For the case of a gradient in chem-
A more detailed description requires identification of
ical potential in an isobaric single-phase system, we find
the thickness of the sorbed surface phase s and its volume
from expressions (3.1.84a), (3.1.84c), (3.1.94) and (3.1.95)
asp s so that the volume of the gas phase in the porous
that the species i flux is controlled by C t rx i or rC i or rpi .
medium per unit volume of the porous medium is
Gradients of quantities such as Cij and pij in a given phase j
( asp s). Consider a pure gas i in the porous medium.
are not directly available in practical separation systems.
If the total concentration of species i in the porous medium
The value of Cij or pij at the two ends of the diffusion path are
per unit volume, C ti , is considered as a sum of that in the
much more likely to be available. We therefore integrate
gas phase Cig and the adsorbed phase Cis, then
here the flux expressions along the diffusion/transport path,
C ti C ig C is C 0ig  asp s Eis asp ; 3:1:118b if possible, and express the flux in terms of differences in
concentrations (or mole fractions) or partial pressures
where C ig is the number of molecules in the gas phase per existing at the ends of the transport path. In this process,
unit gas phase volume in equilibrium with the surface- proportionality constants called mass-transfer coefficients
adsorbed phase (Ash et al., 1967, 1973a). Rewriting arise: these are very important for a quantitative analysis
(3.1.117c) using these expressions, one obtains of separation achieved in practical separation problems. We
N iz =surface Dis asp d Eis first consider a mixture of gases and then deal with diffusion
: 3:1:118c in liquids (Cussler, 1997; Bird et al., 2002).
N iz =Knusden DiK  asp s d C 0ig
Consider simple molecular diffusion in a binary gas-
Furthermore eous mixture of species A and B under isobaric condi-
tion; so rP 0 and UA 0 UB. If the diffusion takes
d Eis d Eis place in only the z-direction along the capillary length in
RT; 3:1:118d
d C 0ig dpig Figure 3.1.4, then from flux expression (3.1.94)
3.1 Forces, displacements, velocities and fluxes 103

p 0Ag pdBg

P p0Bg Dz pdAg P
pdBg
p 0Ag NA NA z+Dz
z
p 0Bg Capillary pdAg

dg

Flow channel NAz Flow channel


A,B A,B
MIX. MIX.
z=0 z

Figure 3.1.4. Two flow channels connected by a capillary having molecular diffusion; the total pressure in both channels at the capillary
location is P. Diffusion takes place in the capillary.

DAB dpAg DAB k0 k 0xg P


N Az C Ag v tz  : 3:1:120 k 0g c ; k 0xg k 0c k 0c C tg :
RT dz RT g RT P RT
If the conditions are such that v t 0, i.e. NAz NBz 0 (by 3:1:124
relation (3.1.97)), we have simply
Note that the prime indicates equimolar counterdiffusion
DAB dpAg with no bulk velocity in terms of v t .
N Az  : 3:1:121
RT dz
Example 3.1.12 In Figure 3.1.4, on one side of the capillary,
This condition is identified as the equimolar counterdif- a gas containing 90% N210% He is flowing. On the other
fusion (NAz NBz). For each mole of A transferred in side of the capillary, a gas containing 90% He10% N2 is
the positive z-direction per unit area per unit time, one flowing. The capillary length is 30 cm and its diameter is
mole of B is transferred in the opposite direction. 0.1 mm. The pressure is atmospheric on both sides; the
Assume steady state. Since the cross-sectional area system temperature is 25  C. Given DAB 0.687 cm2/s,
through which diffusion occurs is constant, by making determine (1) the rate at which N2 gas is introduced into
a balance on species A over a differential element of the helium-rich gas stream and the value of its k 0g ; (2) the
length z, we get rate at which He gas is introduced into the N2-rich gas
stream.
dN Az
0: 3:1:122a Solution This is a case of equimolar counterdiffusion
dz since the partial pressure difference of both species over
Integrating along the z-direction, we find that the diffusion path is the same. This case of equimolar
counterdiffusion can be treated via the flux expression
DAB dpAg (3.1.123); let species A N2, species B He. Then we
N Az  constant: 3:1:122b
RT dz have
Integrating again from 0 to g we get DAB DAB P 0
N Az p0Ag  pAg x  x Ag
DAB RT g RT g Ag
N Az p0Ag  pAg k 0g p0Ag  pAg ; 3:1:123
RT g N Bz N Hez k 0g p0Ag  pAg
where k 0g DAB =RT g is the mass-transfer coefficient for 0:687cm2 =s 1atm
0:9  0:1
equimolar counterdiffusion in the gas phase. Alternatively, cm3 -atm
82  298 K  30 cm
1=k 0g is the mass-transfer resistance since NAz may be gmol-K
considered the current and the partial pressure difference,
gmol 0:687
p0Ag  pAg , the potential difference (from Ohms law). N N2z ; k 0g
cm2 -s 82  298  30
It is possible to express the preceding result in several
other forms using the difference in concentrations or mole gmol
0:937  106 :
fractions at the ends of the diffusion path instead of a cm2 -s-atm
difference in partial pressures:
(In chemical engineering process equipments, g is
N Az k 0g p0Ag  pAg k 0c C 0Ag  C Ag k 0xg x 0Ag  x Ag ; far smaller, ~ <0.01 cm, so k 0g is much larger.) We also
where have
104 Physicochemical basis for separation

W Az N Az  capillary cross-sectional area From the definitions of k0xg ; k 0c and k 0g in relations (3.1.124),
0 1 0 1
we obtain
0:687  1  0:8 gmol 2
A  @ 10 Acm2
@ 2
82  298  30 cm2 -s 4 k 0xg k 0c  0  k0 
g

N Az x 0Ag  x Ag C Ag  C Ag p0Ag  pAg ;
N N N
0:687  0:8   104 gmol
3:1:127
82  298  30  4 s
where the bulk flow correction factor N is given by
0:687  0:8   1011 gmol gmol
5:88  1011
0:82  0:298  1:20 s s x 0Ag  x Ag
N 2  3:
N Az
into the helium stream:    x Ag
N Az 6 N Az N Bz 7
Since jN Az j jN Bz j jN He j, W He 5:88  1011 gmol=s n 6 4
  7
5
N Az N Bz N Az 0
into the nitrogen stream in the opposite direction.  x Ag
DAB
N Az N Bz
Note: N Bz N Az p  p0Bg : 3:1:128
RTg Bg
In equimolar counterdiffusion, NAz NBz 0 in a binary Defining the flux ratio, (NAz/{NAz NBz}), as NR (Geankoplis,
gas mixture; so is v tz . Under other conditions, the bulk velocity 1972), N may be given a simpler representation as follows:
v tz 6 0; the corresponding expressions of NAz and the various
mass-transfer coefficients will be developed now. The only N R  x Ag log mean N R  x Ag lm
N 3:1:129a
restriction is that the magnitude of the transfer rate of species NR NR
A is low. Consider again isobaric conditions and no external
forces. From flux expression (3.1.94) and the relation (3.1.97), since

DAB dpAg C Ag DAB dpAg N R  x Ag  N R  x 0Ag


N Az C Ag v tz  N Az N Bz  ; N ! : 3:1:129b
RT dz C tg RT dz N R  x Ag
N R n
3:1:125a N R  x 0Ag
where (CAg/Ctg) xAg. Assume now a constant DAB and a
In equimolar counterdiffusion, knowledge of the mass-
value of (NAz NBz) independent of the z-coordinate at transfer coefficient k 0g (or k0c or k 0xg ) and the values of partial
steady state. Rearrange this relation (3.1.125a) as pressures (or concentrations or mole fractions) at the two
pAg g ends of the diffusion path (z 0 and z g) were sufficient to

dpAg N Az N Bz determine N Az (or N Bz ). In all other cases of molecular
 dz;
RT C tg N Az
 C Ag RT C tg DAB diffusion, the flux ratio NR needs to be known; N can then
N Az N Bz
p0Ag 0 be calculated to provide NAz or NBz from definition (3.1.127).
3:1:125b A particular case of considerable use arises when species
A diffuses through nondiffusing or stagnant B. This can
where CAgRT pAg, CtgRT P and the partial pressure of happen, for example, when SO2 (A) from air (B) is absorbed
species A, pAg, has values of p0Ag and pAg at z 0 and g, by an aqueous solution (Figure 3.1.5(a)) or the vapor of a
respectively. Simplifying, we find that volatile liquid, e.g. acetone (A), diffuses through stagnant
p0Ag air (B) (Figure 3.1.5(b)). Since species B is stagnant, NBz0.

dpAg g Therefore,
  N Az N Bz
N Az C tg DAB N Az
pAg P  pAg NR 1: 3:1:130
N Az N Bz N Az N Bz
3:1:125c From definitions (3.1.127) and (3.1.129),
On integration,
2 3 k 0g
N Az

pAg N Az p0Ag  pAg 3:1:131a
 N
1 6 N N P 7 g
6 Az Bz 7 and
n 6  0 7 ;
N Az N Bz 4 N Az p Ag 5 C tg DAB
 1  x Ag  1  x 0Ag pBg  p0Bg p
N Az N Bz P N ! ! B;lm ;
3:1:125d 1  x Ag

pBg P
n P n 0
where we have assumed that NAz/(NAz NBz) does not 1  x 0Ag pBg
vary with z. Therefore 3:1:131b
2  3
N Az where pB, lm represents a logarithmic average of the partial
   x Ag
N Az C tg DAB 6 N Az N Bz 7 pressure PBg over the gas film thickness g, so that
N Az n 6
4 N Az 
7:
5
N Az N Bz g 0
 x Ag k 0g P p0Ag  pAg k 0xg x 0Ag  x Ag k 0c C t C 0A  C A
N Az N Bz N Az ;
pB;lm x B;lm C B;lm
3:1:126
3.1 Forces, displacements, velocities and fluxes 105

(a) Flowing gas stream


contains SO (A) in air (B)
2
(b) Flowing air stream
z=0 contains acetone (A)

SO z = dg
2
flux z
direction

Stagnant

Stagnant
gas
z

gas
Acetone
flux
direction

z=0
z = dg
Acetone
Aqueous liquid
solution

Figure 3.1.5. Diffusion of (a) SO2 (A) through stagnant air (B) prior to absorption in an aqueous solution z g ; (b) acetone (A)
through stagnant air (B) after evaporation from liquid into flowing air stream at the top.

The partial pressure of ethanol at z 0 corresponds to its


where k 0g DAB =RTg ; k 0xg DAB C t =g and vapor pressure, p0Ag 0:075 atm. At the top of the column
(z g), pAg 0 since the steady air flow rate is consider-
x Bg  x 0Bg able. We determine pB;lm , where P 1 atm, as follows:
x B;lm ! 1  x A lm ; 3:1:132
x Bg
n 0 pBg  p0Bg P  1  0:075
x Bg pB;lm  
2:303 log pBg =p0Bg P
2:303 log
where xB, lm represents a logarithmic average of the mole 1  0:075
fraction of species B, xBg, over the gas film thickness g. 0:075 0:075
When neither equimolar counterdiffusion nor diffusion of   0:975atm:;
1 2:303  0:0334
A through a stagnant B describes the situation, other cri- 2:303 log
0:925
teria have to be used to relate NAz to NBz. This may involve
reaction conditions, enthalpy balances, etc. Such cases will cm2 1 atm
0:135
be treated as and when they arise. k 0g s 0:975 atm
DAB P

N RT g pB;lm cm3 -atm
Example 3.1.13 In Figure 3.1.5(b), a layer of ethanol is 82  298 K  30 cm
gmol-K
maintained at the bottom of a glass column 30 cm long and
1.5 cm in diameter at 25  C. Air is blowing over the top of the gmol
0:188 ;
open glass column; it sweeps away any ethanol evaporating cm2 -s-atm
from the bottom of the column. Air pressure, as well as the
pressure in the glass column, may be assumed to be 1 atm. cm2
0:135 1 atm 0:075  0 atm
Determine k0g =N for ethanol and its rate of loss into the s
air stream, given diffusion coefficient in ethanolair system at N ethanol
cm3 -atm
25  C 0.135 cm2/s; vapor pressure of ethanol at 25  C 82  298 K  30  0:975 atm
gmol-K
0.075 atm.
0:135  0:075 gmol

Solution This is a case of ethanol (A) diffusing through a 82  298  30  0:975 cm2 -s
stagnant column of air (B). From equations (3.1.131) and
(3.1.132), the steady state flux of ethanol is 0:135  0:75  107 gmol
N ethanol 0:14  107 :
0:82  0:298  3:0  0:975 cm2 -s
DAB P
N ethanol N Az p0  pAg
RT g pB;lm Ag The rate of loss of ethanol into the air stream
106 Physicochemical basis for separation

W A N ethanol  r 2 0:14  107   0:752 A special case of this arises as before: the diffusion of
0:14   0:562  107 0:247  107 gmol=s: solute i through stagnant solvent s. Therefore put
N sz 0; N R 1 and we get
So far, the exposition has focused on molecular diffusion in  
a gaseous mixture of species A and B. Almost identical Dis 1  x il Dis C tl x 0il  x il
N iz C tl n ;
results may also be used for molecular diffusion in liquids. l 1  x 0il l x s;lm
In gaseous systems, P 0 implied that Ctg was constant
where
along the diffusion path. In a liquid mixture of solute i in
solvent s, P may be zero, but the molar solution density x sl  x 0sl
x s;lm : 3:1:137
Ctl generally varies substantially along the diffusion path. n x sl =x 0ls
Further, the diffusion coefficient Dis also varies with the
solute concentration. The governing diffusion equation for The case of equimolar counterdiffusion can also be
species i in an is system is obtained from equations obtained when N iz N sz , yielding
(3.1.87) and (3.1.89) for diffusion in the z-direction and dx il
Uiz 0 as N iz C tl Dis ; 3:1:138a
dz
dx il where both Ctl and DiS may vary with composition and
N iz C il v tz  C tl Dis : 3:1:133a
dz therefore z-coordinate. Assume an averaged value of Ctl
Now, and Dis, integrate the above equation at steady state from z
  0 to z l , the diffusion path length, to obtain
dx il dx sl
N iz N sz v tz C il C sl  C t l Dis ; C tl Dis 0
dz dz N iz x il  x il k 0xl x 0il  x il k 0cl C 0il  C il ;
l
where we have used Dis for the diffusion of i in solvent s. 3:1:138b
By definition, locally, xil xsl 1; therefore, with Cil
Csl Ctl, where k 0cl and k 0xl have been defined by (3.1.135). Note that
N 1 in this case.
C il dx il
N iz N iz N sz  C tl Dis : 3:1:133b The preceding treatments were limited to simple
C tl dz
molecular diffusion. If the value of vt or v t was nonzero,
Assume that an averaged value may be used for Ctl and Dis. it was directly a result of individual species velocity vi
Then the result (3.1.126) or (3.1.127) with N defined by arising from a concentration gradient or a partial pressure
(3.1.128) may be used. Replacing, however, DAB by Dis and gradient. In practical separation processes, there is fluid
g by l, the diffusion path length in the liquid is given by convection; the flow may be laminar or turbulent. The rate
2  3 of transfer of a species is enhanced considerably by such
N iz fluid motion. The fluid motion may or may not be in the
  x il 
N iz Dis C tl 6 N N sz 7 direction of the intended species transport. More often
N iz n 6
4  iz 7
5:
N iz N sz l N iz than not, the desired direction of species transport is
x 0il 
N iz N sz perpendicular to the direction of bulk motion of the fluid.
3:1:134 It is possible to predict theoretically the mass transfer
rate (or flux Ni) across any surface located in a fluid
Define having laminar flow in many situations by solving the
Dis differential equation (or equations) for mass balance (Bird
k 0cl and k 0xl k 0c C tl : 3:1:135 et al., 1960, 2002; Skelland, 1974; Sherwood et al., 1975).
l
Our capacity to predict the mass transfer rates a priori in
Then turbulent flow from first principles is, however, virtually
nil. In practice, we follow the form of the integrated flux
k 0xl 0 k0
N iz x il  x il c C 0il  C il ; 3:1:136a expressions in molecular diffusion. Thus, the flux of
N N
species i is expressed as the product of a mass-transfer
where coefficient in phase j and a concentration difference in the
forms shown below:
N R  x il  N R  x 0il
N   3:1:136b
N R  x il N iz k cj C 0ij  C ij k xj x 0ij  x ij k g p0ig  pig :
N R n
N R  x 0il 3:1:139
and
However, the values of the mass-transfer coefficients
N iz are obtained from empirical correlations instead of the
NR : 3:1:136c
N iz N sz simple molecular diffusion based expressions (3.1.124) or
3.1 Forces, displacements, velocities and fluxes 107

Table 3.1.4A. Relations between species flux and mass-transfer coefficients for two special cases

Flux expressions in gaseous systems z-direction

Equimolar Diffusion of A through Units of transfer Relation between the two transfer
counterdiffusion stagnant B coefficient coefficients

N Az k 0xg x 0Ag  x Ag N Az k xg x 0Ag  x Ag mol k0xg k 0xg P


cm2 -s-mole fraction kxg
x B, lm pB, lm
N Az k 0yg y 0Ag  y Ag N Az k yg y 0Ag  y Ag mol k 0yg k 0yg P
cm2 -s-mole fraction kyg
y B, lm pB, lm
N Az k 0g p0Ag  pAg N Az k g p0Ag  pAg mol k 0g P
cm2 -s-atm kg
pB, lm
N Az k 0cg C 0Ag  C Ag N Az k cg C 0Ag  C Ag mol k 0cg C t k 0cg P
kcg
cm2 -s-mol=cm3 C B, lm pB, lm

Flux expressions in liquid systems z-direction

Equimolar Diffusion of i through Units of transfer Relation between the two transfer
counterdiffusion stagnant s coefficient coefficients

N iz k 0xl x 0il  x il k 0xl mol k 0xl


N iz x 0  x il k xl
x s, lm il cm2 -s-mole fraction x s, lm
N iz k 0cl C oil  C il N iz kcl C oil  C il mol k0
k cl cl
cm2 -s-mol=cm3 x s, lm

Relations for a gaseous system between kg, kc , kx and ky : kg ky =P k c =RT = kx =P.


Relation for a liquid system between kc and kx : kx kc C t .

(3.1.135). The mass-transfer coefficients are strong func- Some clarifications on the mass-transfer coefficients for
tions of the extent of convection. A few such correlations use in the flux expressions (3.1.139) are also in order. If the
used frequently are given in Section 3.1.4.1. The quantities mass-transfer coefficient under convection was measured
C 0i ;C i ;x 0ij ;x ij ;p0ig and pig are, as before, values at the two or correlated under particular conditions of NR ( {Niz/(Niz
ends of the mass-transfer path. Nsz)} or {NAz/(NAz NBz)}), then either of the basic rela-
There is a need to provide additional clarification tions (3.1.126) or (3.1.134) should be used to determine the
about the above-mentioned quantities used to define flux value of (DABCt/g) or (Dis C t =l ) as the case may be. These
Niz. Consider transfer of a species from the wall of a duct to values are simply k 0xj , the mass-transfer coefficient for equi-
the fluid flowing in the duct. The duct wall is obviously one molar counterdiffusion in the presence of convection. For
end of the diffusion path. It is not clear where the other end any other value of NR, the value of kxj, kcj or kg in equation
is since material from the duct wall is coming to (and being (3.1.139) is obtained by using the bulk flow correction factor
convected away from) at all distances normal to the duct N defined by (3.1.129b) or (3.1.136b). Table 3.1.4A sum-
wall. Common practice, therefore, is to define a bulk con- marizes the flux vs. concentration difference relations in
centration Cijb (the cup-mixing concentration, (Bird et al., convective mass transfer for pij, Cij, xij under two con-
1960, 2002)) by ditions encountered frequently in separation processes
using gases or liquids. Since the mole fraction of i in the
vxj C ij dAc literature is often expressed as xi for a liquid phase and yi for
A a gas phase, we have also incorporated yi in this table.
C ijb c ; 3:1:140
Further, kc and kl are interchangeable for the liquid phase.
vxj dAc
Sometimes subscript l may be replaced by w or o to indicate
Ac
an aqueous or organic phase. For gaseous mixtures, species
where Ac is the cross-sectional area of the duct, vxj is the A and B have been used instead of i and s. Table 3.1.4B
local convective velocity in the mean flow direction (say provides corresponding expressions where species i has
the x-coordinate) along the duct length in phase j and Cij is been identified specifically in the mass-transfer coefficient
the value of the species i concentration at the location of terms as, for example, kijc, for species i in phase j and
velocity vx. This Cijb is then used as the concentration at the concentration units C.
other end of the diffusion path in the flux expressions In separation process devices/equipment having con-
(3.1.139). vection, the flows are often complex. Yet a simplified
108 Physicochemical basis for separation

Table 3.1.4B. Relations between species flux and mass-transfer coefficients for two special cases with species i identified in
mass-transfer coefficients

Flux expressions in gaseous systems z-direction

Equimolar Diffusion of A through stagnant Units of transfer Relation between the two transfer
counterdiffusion B, i = A coefficient coefficients

N iz k 0igx x 0i  x i N Az k Agx x 0A  x A mol k 0ix P


k ix
cm2 -s-mole fraction P  pi lm
N iz k 0igy y 0i  y i N Az k Agy y 0A  y A mol k 0iy P
cm2 -s-mole fraction k iy
P  pi lm
N iz k 0ig p0i  pi N Az k Ag p0A  pA mol k 0ig P
cm2 -s-pressure k ig
P  pi lm
N iz k 0igc C 0ig  C ig N Az k Agc C 0Ag  C Ag mol k 0igc C t k 0igc P
k igc
cm2 -s-mol=cm3 C t  C i lm P  pi lm

Flux expressions in liquid systems z-direction

Equimolar Diffusion of i through Units of transfer Relation between the two transfer
counterdiffusion stagnant s coefficient coefficients

N iz k 0ilx x oil  x il k 0ilx o mol k0ilc


N iz x  x il k ilc
x s, lm il cm2 -s-mole fraction x s, lm
N iz k 0ilc C 0il  C il N iz k ilc C 0il  C il mol k0
k ilc ilc
cm2 -s-mol=cm3 x s, lm

Relations for a gaseous system between kig , kic , kix and kiy: kig kiy =P kic =RT kix =P.
Relation for a liquid system between kilc and k ilx : kilx kilc C tl .

description of the mass-transfer coefficient (instead of a film theory, the complex mass-transfer rate can be deter-
detailed correlation) is desirable in many cases, including mined by postulating that simple molecular diffusion
those having a chemical reaction. A number of theories of through a stagnant film of thickness l next to the phase
increasing complexity, e.g. film theory, penetration theory, interface can describe the mass-transfer coefficient. Thus
surface renewal theory, boundary layer theory (Bird et al.,
DAs 0
1960, 2002; Danckwerts, 1970; Sherwood et al., 1975), have N Az k cl C 0Al  C Alb C Al  C Alb ; 3:1:141
l
been developed to this end to describe the mass-transfer
coefficient and the mass-transfer rate. Of these, film so that
theory is the simplest and will be used in this book
k cl DAs =l ; 3:1:142
often to describe the mass-transfer coefficient and the
mass-transport process at the interface of a gas or liquid where l is the unknown effective film thickness. If kcl is
phase for the following systems: gasliquid; gassolid; known, l can be determined. As a predictive theory, film
liquidsolid; liquidliquid; liquidmembrane; liquidion theory is rather poor, for example, in predicting the
exchange resin. dependence of kcl on DAs in most situations encountered.
In the film theory description of the mass-transfer pro- However, its use is frequent in complex geometries and
cess occurring between two fluid phases or between a solid flow conditions in the presence of chemical reactions.
and a fluid phase, the complex mass-transfer phenomenon Additionally, the notion of a stagnant film has a vestigial
is substituted by the notion of simple molecular diffusion effect; the mass-transfer coefficient of the gas phase or
of the species through a stagnant fluid film of thickness . the liquid phase is routinely identified as the gas film
The actual concentration profiles of species A being trans- coefficient or the liquid film coefficient.
ferred from phase 2 to phase 1 are shown in Figures 3.1.6
(a) and (b) in one phase only for a solidliquid and a gas
3.1.4.1 Empirical correlations of single-phase
liquid system, respectively. The concentration of A in the
mass-transfer coefficients
liquid phase at the solidliquid or the gasliquid interface
is C 0A . Far away from the interface it is reduced to a low The analysis and estimation of separation in laboratory-
value in the liquid phase. In turbulent flow, the curved scale techniques as well as large-scale processes generally
profile of species A shown would correspond to the time- require an estimate of the mass-transfer coefficient in a
averaged value (Bird et al., 1960, 2002). According to the single-phase system which may be part of a multiphase
3.1 Forces, displacements, velocities and fluxes 109

(a) (b)
Phase interface

Phase interface
0
C Al

0
C Al
Solid Liquid
phase 2 phase 1
Gas Liquid
phase 2 phase 1

C Alb
CAlb
dl
dl

z=0 z
z=0 z

Figure 3.1.6. Stagnant film model for mass transfer near a phase interface. (a) Solidliquid system. (b) Gasliquid system.

separation system. Most practical separation systems dv


Reynolds number Re ;
have complex geometries and convective flow patterns14
for which an exact mass-transfer analysis may not be

available. Over the years, numerous experimental inves- Schmidt number Sc
DAB
tigations of many such systems have been carried out,
and the resulting mass-transfer coefficients have been
=Dis for solute i in solvent s: 3:1:143a
correlated with a number of dimensionless numbers/ DAB
groups formed out of the variables that affect the mass
transfer. This correlation activity is guided by These three basic dimensionless groups are related func-
Buckinghams pi theorem: the functional relationship tionally by
between q variables whose units may be described in
Sh f Re; Sc: 3:1:143b
terms of u fundamental units or dimensions may be
written as a function of q  u independent dimension-
Occasionally, an additional geometrical dimension of
less groups (Langhaar, 1951).
length, l, beside a diameter, d, is simultaneously involved.
There are three fundamental units or dimensions,
Then an additional basic dimensionless number is the
mass M, length L and time t in these problems without
(l=d) ratio:
any heat transfer (which will require T, the temperature, as  
the fourth one). The following is a list of the variables of l
Sh f Re; Sc; : 3:1:143c
importance: a characteristic physical dimension of the d
device, say, d, the diameter;15 density, ; velocity, v; viscos-
The functional relation between the Sherwood number
ity, ; diffusion coefficient, for example, DAB (or Dis): a total
and the other numbers has a general form of the following
of six variables (including k). The number of independent
type:
dimensionless groups are 63, or 3. The three basic
dimensionless groups in mass transfer are:   b3
l
Sh a b Reb1 Scb2 ; 3:1:143d
d
kd where the constants a, b, b1, b2 and b3 reflect particular
Sherwood number Sh ;
DAB flow regimes, flow patterns, mass-transfer regimes, con-
centration profile development, etc.
In separation systems where natural convection is
14 involved (see Section 6.1.3), T, the temperature, becomes
See Section 6.1 for the sources and the nature of bulk flow
commonly employed in separations. the fourth fundamental unit (u 4), and the density dif-
15
For systems containing particles or drops, it is dp, the particle/ ference, , due to a temperature difference and the accel-
drop diameter. eration due to gravity, g, become additional variables of
110 Physicochemical basis for separation

Table 3.1.5. Limiting analytical solutions and experimental correlations for convective mass transfer between a fluid and a solid wall

Equation
Configuration Analytical solution/experimental correlation Variable range Reference number(s)
 
 1=3 d W
Laminar flow in a kz d d Re Sc > 400 LVque (1928); (3.1.144),
Shz 1:077 Re Sc1=3 4 z DAB z
tubea,b
DAB z Skelland (1974) (3.1.145)

 1=3
k avg d d
(1) Entrance Shavg 1:615 Re Sc1=3
DAB L
region

(2) Fully Sh kd=DAB 3:656 uniform wall concentration Skelland (1974) (3.1.146)
developed
concentration
and velocity
profiles
Sh kd=DAB 4:36 uniform mass flux Skelland (1974) (3.1.147)
at wall
Laminar flow Sh k2b=DAB 7:6 uniform wall concentration Skelland (1974) (3.1.148)
between flat
parallel plates:
fully developed
velocity and
concentration
profilesc
Sh k2b=DAB 8:23 uniform mass flux at wall Skelland (1974) (3.1.149)

Re > 2100
Turbulent flow Sh 0:023 Re0:83 Sc0:33 Linton and (3.1.150)
0:6 < Sc < 3000
in a tube Sherwood
(1950)
1=2
Laminar boundary J D 0:664 ReL ReL < 15 000 Sherwood et al. (3.1.151)
layer flow over (1975)
a flat
plate: integral
analysisd
1=2
Shavg 0:646 ReL Sc 1=3 ReL < 3  105 Skelland (1974) (3.1.152)
a
z is the distance along the tube from the entrance.
b
L is the length of the pipe; W is the total mass flow rate in the tube; see Skelland (1974) for solutions of the mass-transfer rate in developing
concentration profiles for fully developed velocity profiles (plug or parabolic) in a tube.
c
2b is the gap between the parallel plates.
d
ReL Lv =v, where L is the plate length and v is the free stream velocity outside the boundary layer.

importance: the larger the , the higher the velocity due Sh a b Reb1 Sc b2 Gr b3 3:1:143f
to the natural convective driving force. Correspondingly,
the number of variables is now eight so the number of in cases where only one length variable, d, is important.
independent dimensionless groups is 8 4, or 4. The Often special combinations of these dimensionless
additional dimensionless group is the Grashof number, groups are employed to illustrate the correlations
of mass-transfer coefficients. A few of them are as follows:
d3 2 g Sh k
Gr ; 3:1:143e Stanton number; St ;
2 Re Sc v
dv dv
where is a density difference (1  2 ) in a given phase Pclet number; Pe Re Sc ; 3:1:143g
DAB Dis
between two locations and is an averaged density at the
mean temperature. (Note that may be created by a Sh
JD factor; J D :
concentration difference in a single phase as well without Re Sc1=3
any temperature difference.) Correspondingly, the func- The correlations that have been developed for mass trans-
tional relation between the Sherwood number and the fer are most often for system configurations of the
other dimensionless numbers may be presented as following type.
3.1 Forces, displacements, velocities and fluxes 111

Table 3.1.6. Experimental correlations and limiting analytical solutions for convective mass transfer between a fluid and a solid sphere

Experimental correlation/analytical Equation


Configuration solution Variable range Reference number

kg d p pB, lm
One sphere in a stagnant Sh 2 (analytical solution) stagnant gas around Cornish (1965) (3.1.153)
DAB C t
fluid sphere
Gas flowing over a Sh 2 0:552 Re0:53 Sc1=3 1 < Re < 48 000 Geankoplis (1972) (3.1.154)
single sphere 0.6 < Sc < 2.7
Liquid flowing over a Sh 2 0:95 Re0:50 Sc1=3 2 < Re < 2000 Geankoplis (1972) (3.1.155)
single sphere 788 < Sc < 1680
kc d p
Sh 0:347 Re0:62 Sc1=3 2000 < Re < 16 900 Geankoplis (1972) (3.1.156)
DAB
Gas flowing over a J D 0:4548= Re0:4069 10 < Re < 10 000 Dwivedi and (3.1.157)
packed beda Upadhyay
(1977)
Sh 2 1:1 Sc 1=3 Re0:6 3 < Re < 104 Wakao and (3.1.158)
Funazkri (1978)
0:0016 < Re < 55
Liquid flowing over a J D 1:09= Re2=3 Wilson and (3.1.159)
165 < Sc < 70 600
packed bed Geankoplis (1966)
55 < Re < 1 500
J D 0:25= Re0:31 (3.1.160)
165 < Sc < 10 690

Fluidized bed of spheres J D 0:01 0:863=Re 0:58  0:483 1


Re
2140 Gupta and Thodos (3.1.161)
(gases and liquids) (1962)

is the void volume fraction (porosity) of the packed bed; Re = (dp G/), where G is the superficial mass average velocity in
the empty flow section (before the single sphere or the empty bed without packing); dp is the sphere diameter; for liquids,
Sh = (kc dp/DAB).
a
The equation for Sh was developed by using an effective axial diffusion coefficient Di,eff,z, (equation (7.1.4)) called the axial dispersion
coefficient in fixed-bed adsorbers via the following expression:

Di, eff;z =DAB Di, eff , z =Dis 20 0:5 Re Sc:

(1) Mass transfer between a fluid and a solid wall under regime, laminar or turbulent, as characterized by the
the following conditions: Reynolds number, the extent of development of the con-
(a) fluid is flowing inside a channel/pipe/tube; centration boundary layer (developing or fully developed),
(b) fluid is flowing outside a channel/pipe/plate/tube. and any other range specified by the developer of the
(2) Mass transfer between a fluid and a single solid particle correlations. Sometimes, analytical solutions are available,
or a collection of particles, as in a packed or fluidized and a few will be provided. For a detailed introduction to
bed. mass transfer and mass-transfer coefficients in such
(3) Mass transfer between a fluid and a single immiscible systems, consult Geankoplis (1972, 2003), Skelland
drop or a collection of drops as, for example, in a (1974), Sherwood et al. (1975), Cussler (1997), Middleman
liquidliquid dispersion. (1998) and Bird et al. (2002). Table 3.1.5 summarizes
(4) Mass transfer between a liquid and a gas bubble or limiting analytical solutions and experimental correlations
a collection of gas bubbles or particles in suspension. for convective mass transfer between a fluid and a solid
(5) Mass transfer between a flowing thin liquid layer wall. Table 3.1.6 lists experimental correlations and some
and another immiscible fluid layer flowing in limiting analytical solutions for convective mass transfer
parallel. between a fluid and a solid sphere, as in packed or
fluidized beds. Table 3.1.7 provides the same information
Many of these areas have a vast amount of literature between a fluid and small particles, bubbles or drops.
devoted to them. What will be included here is as follows: Table 3.1.8 illustrates a few experimental correlations
a few important correlations will be provided, if possible, (sometimes with their analytical basis) for convective
in each case, along with a few references. Often the mass transfer between a liquid and a permeable mem-
geometries and the flow configurations are more complex brane (porous or nonporous) having a particular
than the five categories identified above. Further, there geometry, e.g. spiral-wound reverse osmosis unit, hollow
will be additional limitations depending on the flow fiber membrane contactor.
112 Physicochemical basis for separation

Table 3.1.7. Experimental correlations and limiting analytical solutions for convective mass transfer between a fluid and small particles,
bubbles or drops

Equation
Configuration Experimental correlation/Analytical solution Variable range Reference number
!
kc dp d 3p c g
Small particles, gas Shc 2:0 0:31 Sc 1=3 dp < 0.6 mm Geankoplis (2003) (3.1.162)
DAB 2c
bubbles or liquid
drops in
suspension  1=4
kc dT dT
Particles in a Shc 0:267 Sc c 1=4 Reimp 3=4 N p 1=4 particle size Kulov (1983) (3.1.163)
DAB V dimp
turbulent stirred independent
tank !
4 2 1=4
kc dp Pr=V d
p c
Shc 0:13 Sc c 1=3 Calderbank and (3.1.164)
DAB 3c
Moo-Young (1961);
  Blakebrough (1967)
c g 1=3
Large gas bubbles k c 0:42 Sc 0:5 d p > 2:5 mm Geankoplis (2003) (3.1.165)
2c
k c dp
Continuous phase in Shc 0:725 Pe0:57 Sc c 0:15 1  D Treybal (1963, p. 480) (3.1.166)
DAB C
liquidliquid
dispersion
kD dp 0:00375 dp vt =DABD
Dispersed phase in ShD circulating liquid Treybal (1963, p. 187) (3.1.167)
DABD 1 D =c
liquidliquid sphere
dispersion ShD 2 2 =3 rigid sphere Treybal (1963, p. 186) (3.1.168)
0:5
Droplet formation in k Df 2:957=Mavg DABD =t f Based on total drop Skelland (1974) (3.1.169)
liquidliquid kCf 4:6=Mavg DABc =t f 0:5 surface area at
dispersion where detachment
tf is the time of
formation of a
single drop

Shc, Scc, kc, c, c refer to liquid phase for particles, bubbles, etc.
p  c , kc , kD , Shc , ShD , DABc , DABD , c , D refer to continuous phase (c) and dispersed phase (D) in liquidliquid dispersion: vt = drop
terminal velocity; D = dispersed phase volume fraction; dT = vessel diameter; dimp = impeller diameter; V = liquid volume; Np = power number
(= Pr (mixing power, watts)/c d5imp N 3 ), where N is the rotational speed in revolutions per minute, Reimp = N d 2imp =c , c being the kinematic
viscosity of the liquid.

3.1.5 Flux expressions in multicomponent systems x ig C tg RT r Px ig =Px ig rPx ig


Chemical separation problems can involve a binary or a at constant P and T.
multicomponent mixture. The flux expressions commonly Now, these molecules of species i collide with mol-
used for multicomponent mixtures are considered here. ecules of species k, and momentum is transferred/lost by
molecules of species i, resulting essentially in a frictional
3.1.5.1 MaxwellStefan equations force on molecules of species i; this loss of momentum is
proportional to the difference in velocities (vig vkg)
The MaxwellStefan equations for describing the diffusion
between the two types of molecules. Further, the number
of gases in a multicomponent gas mixture have been
of such collisions per unit time per unit volume of the gas
developed from the kinetic theory of gases. A highly sim-
mixture is proportional to xig xkg. At steady state, the force
plified illustration may be pursued as follows. Consider a
on molecules of species i per unit volume of the mixture,
system of a gas mixture of n species at constant T and P.
Ctg RT rxig, should be equal to the frictional force on
Focus first on molecules of species i. The net force exerted
molecules of species i created by the loss of momentum of
on species i in the absence of any external forces is rig /
species i via collisions with species k molecules, namely
gmol of i. The net force exerted on species i per unit
f ikm x ig x kg vig  vkg , where fikm is a frictional coefficient
volume of the mixture is rig x ig C tg , where Ctg is the
of sorts of species i due to species k in the unit volume:
total molar density of the gas mixture; from equation
(3.1.40) and an ideal gas mixture, the net force on species rPx ig P r x ig f ikm x ig x kg vig  vkg : 3:1:177a
i per unit mixture volume is x ig C tg RT r n Px ig , which
is equivalent to This relation is written generally as
3.1 Forces, displacements, velocities and fluxes 113

Table 3.1.8. Experimental correlations and limiting analytical solutions for convective mass transfer between a liquid and
a membrane surface

Experimental correlation/analytical Equation


Configuration solution Variable range Reference number(s)

Spiral-wound membrane kdh Schock and Miquel (3.1.170)


Shc 0:065 Re0:875 Sc0:25
channel in Dil (1987)
reverse osmosisa Re vdh =
Hollow fiber membrane
module: Laminar
tube-side flow
k avg d
(1) Gas absorption into Shavg 1:615d=L1=3 Re Sc1=3 W =DAB z > 400 LVque (1928); (3.1.145),
liquid in hollow fiber DAB Sc = 464; Skelland (1974); (3.1.171)
lumen Shavg 1:64d=L1=3 Re Sc1=3 3 < (vd/DABL) Yang and Cussler
< 500 (1986)
(2) Membrane solvent Shavg 0:5d=L Re Sc 1
1 300 < Sc < 1000 Prasad and Sirkar (3.1.172)
extraction in hollow 0 < Re < 600 (1988)
fiber lumenb
Hollow fiber membrane
module: shell-
side flow
(1) Liquid in parallel flowa  
k dh dh 0:66
(i) in solvent extraction Dil 5:85 1  L Re
Shavg avg Sc0:33 0.08 <
0.4 Prasad and Sirkar (3.1.173)
(1988)
(ii) in gas absorption Shavg 1:25 d h Re=L0:93 Sc0:33
0.26, Gr < 60 Yang and Cussler (3.1.174)
(1986)
(2) Liquid in cross flow in Sh kd=Dil 0:57 Re0:31 Sc 0:33 0.01 < Re < 1; no. Bhaumik et al. (1998) (3.1.175)
gas absorptionc,d of fiber layers
2 to 7
Sh kd=Dil 0:80 Re0:47 Sc 0:33 0.1 < Re < 10; Wickramasinghe et al. (3.1.176)
Sc = 480 (1993)
a
dh = hydraulic diameter = 4 Rh (hydraulic radius) = [20/{(2/b) (10) sv,sp}]: b = feed channel height; 0 = porosity of spacer; sv,sp = specific
surface of the spacer; dh = 4  (shell-side available flow cross-sectional area/wetted perimeter).
b
For details on , see Skelland (1974, eq. (5.145)) and Prasad and Sirkar (1988, 2001, eqs. (41-30a) and (41-49)).
c
Re = d vs/; d = outside diameter of fiber; vs is based on the shell-side cross-sectional area available for flow.
d
Re = d vo/; d = outside diameter of fiber; v0 is based on the superficial velocity across the fibers.

f ikm C ig C kg Xn
x ig N kg  x kg N ig
rx ig x ig x kg vkg  vig 2 vkg  vig ; dig rx ig
P C tg Dik C tg Dik
k1
3:1:177b
Xn
C ig C kg
where the gas pair (i, k) based diffusion coefficient 2 vkg  vig
k1
C tg Dik
 
C tg RT
Dik P=f ikm :
f ikm Xn
x ig x kg
vkg  vig : 3:1:178
When we add up the frictional contributions of n species to k1
Dik
the drag on species i in an n-component mixture, we get
Xn
C ig C kg Here Dik is the multicomponent diffusion coefficient of the
rx ig dig 2 vkg  vig : 3:1:177c
k1
C tg Dik
gas pair (i, k) in any region, and dig is a force-type term for
the ith species. Note that
For multicomponent gas mixtures containing n species
and ordinary diffusion, rigorous results from the kinetic n n
X x ig X
theory of gases provide a set of (n 1) MaxwellStefan Dik 0 ; Dik Dki ; 0; d ig 0:
(MS) equations (Curtiss and Hirschfelder, 1949; Bird i1 D ik i1

et al., 1960, 2002; Taylor and Krishna, 1993): 3:1:179


114 Physicochemical basis for separation

Equation (3.1.178) indicates that the force-type term dig is form of Ficks first law of diffusion (3.1.87) or (3.1.99) for a
linearly proportional to the relative velocity of the ith species binary system. Thus species i is assumed to diffuse through
with respect to every other species in the system. Since Dik is a second species, which is, in effect, the multicomponent
a diffusion coefficient, P= Dik is a frictional coefficient (see mixture. The manner in which this rearrangement is
(3.1.86)) of sorts; therefore each term on the right-hand side achieved is as follows. Rearrange (3.1.180) to obtain
of equation (3.1.178) is a force-type term (see (3.1.76)). Each 2 3
" #
term may be thought of as a frictional drag force experienced 6X n
x kj 7 X n
x kj
6 7 x ij vij d ij x ij v : 3:1:182
by the ith molecules as they move past the molecules of the 4 Dik 5 Dik
kj
k 1 k1
kth type; further, all types of molecules contribute to the net k 6 i
force on the ith species, resulting in a concentration gradient,
Using any arbitrary constant reference velocity vref
t , the
or mole fraction gradient, or partial pressure gradient, of the
above equation can be arranged as follows:
ith species. This equation also indicates that Nig need not be
2 3
zero when rxig is zero. Further, the flux of i is influenced by
6X n
the fluxes of all other species, Nkg, in the system. 6 x kj 7
7 x ij vij  vref d ij x ij
For ordinary diffusion in liquids and dense gases, the 4 D 5 t
k1 ik
k 6 i
same set of equations is often used, although there is no
rigorous theoretical basis. The multicomponent diffusion 2 3
coefficient Dik for the species pair (i, k) has similar proper- n
6X x kj 7
ties, namely it is the inverse of the frictional coefficient f dik in 6
4 vkj  vref 7
t 5 : 3:1:183
Dik
the dense gas or liquid phase. In fact, such formulations are k1
k 6 i
also used in denser phases such as membranes, etc.; Curtiss
and Bird (1999), however, have shown that the Maxwell It may be shown, after some manipulation, that this rela-
Stefan formalism is applicable to dense gases, liquids and tion is equivalent to
" #
polymers provided the strongly concentration-dependent Xn
x kj DiM
diffusivities Dik are not the binary diffusivities. For solutions, x ij vij  vref
tj DiM d ij x ij v kj  v ref
t ;
k1
Dik
an interpretation of dij by Lightfoot (1974) is useful: (dij)RTCt
is the force moving species i relative to the solution per unit 3:1:184
volume of solution in the region under consideration. If we where DiM, an effective binary diffusivity of i through the
estimate it per gmol of species i, i.e. fdij RTC t =C i g, we find mixture, is defined by
it to be equal to fRT rx i =x i g; i:e: RT rn x i (consider 1  x ij
expression (3.1.50) for ordinary diffusion only). DiM 2 3 3:1:185
For cases where external forces are present along with the n 
6X

6 x kj 7
7:
chemical potential, the generalized MaxwellStefan type of 4 Dik 5
k1
equations in any region j for any i are as follows (Hirschfelder k 6 i
et al., 1954; Lightfoot, 1974; Lee et al., 1977a; Bird et al., 2002):
n n
Multiplying (3.1.184) by Ctj will lead to familiar flux expres-
X x ij N kj  x kj N ij X x ij x kj
d ij vkj  vij sions, if we consider the three reference velocities of
C D Dik greatest interest, namely vref
t 0, vtj and vtj. Following Lee
k1 tj ik k1
3:1:180 et al. (1977a), consider a special case of all Dik being equal.
and Then DiM Dik . Suppose vref t 0, which implies that
" #
  n
X
  n
X
ij g ij  ukj g kj N ij C ij vij DiM C tj d ij x ij C kj vkj : 3:1:186
V i C ij  uij rP k1
k1
d ij x ij rn ai 
C tj RT C tj RT Since C kj vkj N kj , we get
Z i F x ij " #
r; 3:1:181 X n
RT j
N ij x ij N kj  DiM C tj d ij : 3:1:187
where uij is the mass fraction of species i and gkj is the body k1
force on species k per unit mass of species k. For example, for When vref
t v tj ,
gravitational forces g kj g g k. Centrifugal force is
another body force, where g kj 2 r r at any radial location. J ij C ij vij  v tj DiM C tj dij : 3:1:188a
We have not included forces due to thermal diffusion, mag- When vref
t v tj ,
netic forces, nonuniform electrical field and electrophoretic
J ij C ij vij  vtj DiM C tj dij C ij v t  vtj : 3:1:188b
retardation in the expression (3.1.181) for dij.
It is obvious that the relations (3.1.178), (3.1.180) We should not fail to notice that, for a binary system, the
are quite complicated. To provide a simpler picture, it is Ficks law flux expressions for J ij and Jij, namely (3.1.101)
customary to reduce these complicated relations to the and (3.1.100), are identical to the above two expressions for
3.1 Forces, displacements, velocities and fluxes 115

ordinary diffusion. The same statement is also valid for the 3.1.5.1.2 Mass diffusion in a ternary system In the
flux expression Nij. We now consider the application of the mass diffusion or sweep diffusion separation process
MS equations for three different systems. for isotopes 1 and 2 (Benedict et al., 1981, chap. 14), an
inert vapor, e.g. steam, diffuses radially outward while
3.1.5.1.1 Diffusion of species i in a multicomponent the mixture of two isotopes diffuses against the vapor.
stagnant gas mixture Consider flux expression (3.1.186) The two isotopes should have different rates of diffusion
obtained under the condition of vref t 0 (subscript g not due to different diffusion coefficients (as well as different
used here): concentration differences). We use MaxwellStefan formal-
" #
n
X ism to obtain the governing equations (subscript g not used
C i vi N i DiM C t di x i C k vk :
here).
k1
Identify two isotopes by subscripts i 1 and 2 and
For species i diffusing in a stagnant multicomponent gas
steam by i s. Use flux expression (3.1.178) for i 1 and 2
mixture, however, vk 0 for all k 6 i. Further, at z 0,
and the radial coordinate r only:
pi p0i and at z g ; pi pi . Therefore
x 1 x 1 N 2r  x 2 N 1r x 1 N sr  x s N 1r
N i C i vi DiM C t d i x i N i : ; 3:1:193
r C t D12 C t D1s
For ordinary diffusion, di r xi. Further, Ct (P/RT) :
x 2 x 2 N 1r  x 1 N 2r x 2 N sr  x s N 2r
DiM rpi : 3:1:194
Ni 3:1:189 r C t D21 C t D2s
RT1  x i
if total pressure P is independent of the coordinate direc- Determine now (x s / r) by using one of the relations
X n
tions. For one-dimensional transport in the z-direction, use (3.1.179), namely, d 0:
i1 i
definition (3.1.185) to obtain x s x 1 x 2
  : 3:1:195
1  x i =1  x i dpi r r r
N iz  2 3
dz Recall that D12 D21 but D1s 6 D2s ; the latter is provid-
n
6X x k 7
6
RT 4 7 ing the primary basis for separation, resulting in different
D 5 ik
k 1 diffusional speeds for species 1 and 2 through steam.
k 6 i
Therefore
dpi " #
dz x s N sr x 1 x2 N 1r x s N 2r x s
 2 3 : 3:1:190  : 3:1:196
r C t D1s D2s C t D1s C t D2s
6Xn
x k =1  x i 7
1  x i 6
4
7 RT
5
k 1
D ik 3.1.5.1.3 Diffusion of two species through a membrane
k 6 i When two species i 1,2 diffuse through a membrane, we
Now note that the [xk/(1  xi)] term for each k 6 i is (moles have a ternary system of i 1,2,m, where m represents the
of k/total moles of all species except i). Since all species membrane material. We will now obtain a Ficks first law
except i are stagnant, then {xk/(1  xi)} is constant along type of expression for Ni to represent the diffusion of 1 and
the z-coordinate. Since Dik is constant for each k, it is 2 through a membrane mechanically restrained and there-
obvious that DiM is constant along z. Rewrite (3.1.190) as fore having a zero velocity (Lightfoot, 1974) using Maxwell
Stefan formulation. Assume no external forces, no pressure
DiM 1 dpi
N iz  gradients, no temperature gradients and ideal solution
RT 1  x i dz
behavior. The two phases on two sides are liquids. The
DiM P dpi subscript m for the membrane phase has not been used
 :
RT P  pi dz with species subscripts.
Here di x i r n ai x i r n x i for ideal solutions.
Integrate from z 0, pi p0i to z g ; pi pi to obtain
From equation (3.1.186),
DiM p0i  pi
N iz ; 3:1:191a N i C i vi DiM C t d i x i C 1 v1 C 2 v2 C m vm :
RT g P  pi LM
where 3:1:197
P  pi  P  p0i But vm 0. Therefore, with i 1,
P  pi LM  
Pp
: 3:1:191b
n Ppoi
i N1 D1M C t x 1 r n x 1 x 1 N 1 N 2 
Alternatively, 3:1:198
x 1 N 1 N 2   D1M C t rx 1 :
N iz k g p0i  pi ; 3:1:192a Similarly,
where
N 2 x 2 N 1 N 2   D2M C t rx 2 : 3:1:199
DiM 1
kg : 3:1:192b
RT g P  pi LM Here
116 Physicochemical basis for separation

2 31 The mathematical representations of the fluxforce rela-


n  
6X xk 1 7 tions in any given phase/region are:
DiM 6
4
7 :
k 1
Dik 1  x i 5
n
X
k 6 i
ji lik F m
k ; 3:1:202
k1
It follows that
1  x1 l ik lki ; 3:1:203
D1M 3:1:200a
x2
D12
Dx m where the lik  s are phenomenological coefficients. Here ji
1m

is the mass flux of species i in a frame of reference moving


and
with the mass average velocity vt and F m k is the force on
1  x2 species k in an isothermal system of n components such
D2M : 3:1:200b
x1
D21
Dx m that the fluxes and forces are proper and form the right
2m

pair. For a noniosthermal system, one of the forces would


Thus D1M, the effective binary diffusivity of species 1
be due to a thermal gradient in addition to the n species-
through the mixture of 1, 2 and m is different from D2M,
specific forces:
the effective binary diffusivity of species 2 through the
n
X
mixture of 1, 2 and m.
ji l ik F m
k l iT rT : 3:1:204
A ternary system of this type may be reduced to two
k1
pseudo-binary systems: solute 1 and, say, a solvent 2m of
species 2 and membrane m; an alternative pseudo-binary Under systems practically in mechanical equilibrium, it has
system would be solute 2 and, say, a solvent 1m of species been shown (de Groot and Mazur, 1962) that molar fluxes
1 and membrane m. The flux expression for species 1 in the in frames of reference other than barycentric (i.e. mass-
first pseudo-binary system is averaged velocity) can also be used. For example,
n
X
N 1 x 1 N 1 N 2m   C t D12m rx 1 : 3:1:201a
J i Lik F k LiT rT: 3:1:205
k1
The flux expression for species 2 in the second pseudo-
binary system is Here Fk is the force specific for species k. Such a force may
N 2 x 2 N 2 N 1m   C t D21m rx 2 : 3:1:201b be obtained from (3.1.50). The phenomenological coeffi-
cients are symmetrical; Lii coefficients are always positive,
Note that D12m need not equal D21m (Spriggs and whereas Lik or LiT may have any sign:
Gainer, 1973).
Lik Lki ; LiT LTi ; Lii 0; Lii Lkk  L2ik 0:
3:1:206
3.1.5.2 Irreversible thermodynamics approach
For isothermal conditions, an inverted representation of
The principles of irreversible thermodynamics (de Groot
fluxforce relations would be
and Mazur, 1962) provide an alternative way of relating the
flux of species i to all the forces that are present in a n
X
multicomponent multiforce system. The basic assump- Fi Rik J k ; Rik Rki : 3:1:207
k1
tions behind this approach are as follows.
(a) The nonequilibrium conditions leading to species But the phenomenological coefficients Lik  s are not
transport depart from the equilibrium conditions by necessarily reciprocals of the phenomenological coeffi-
sufficiently small amounts. Thermodynamic equilib- cients Rik  s. Conceptually, Rik  s are similar to resistances
rium relations, valid locally under thermostatic16 con- and Lik  s are similar to conductances in electrical circuits.
ditions, may be used for nonequilibrium conditions. For a binary system of solute i and solvent s in an
(b) The flux of any species is a function of all forces pre- isothermal system, equation (3.1.205) is reduced to
sent in the system. The flux-driving force relations are
J i Lii F i Lis F s ; 3:1:208
linear as long as condition (a) holds.
(c) The phenomenological coefficients in (b) are symmetric. J s Lsi F i Lss F s : 3:1:209

Here Lsi ( Lis) is the cross coefficient coupling the two


16
fluxes J i and J s ; Lii and Lss are the straight coefficients. In
In conventional thermodynamics studied for developing
most separation problems of interest, the coupling is
equilibrium relations, the conditions are really static, with no
change anywhere, except when going from one state to another;
essentially negligible, i.e. Lsi 0. For example, in ordinary
however, the process of change, or the dynamics of change, is not diffusion with Ui 0, we observe from (3.1.50), (3.1.81),
studied. Therefore it should be considered as thermostatics. (3.1.89) and (3.1.101) that Lis 0 and
3.1 Forces, displacements, velocities and fluxes 117

Dis C i the particle encounters a denser gas where gas molecules


Lii : 3:1:210
RT collide with one another much more often; correspond-
ingly, the particle encounters much more resistance to
This gives us an idea about the nature of the Lii coefficient
motion due to a high density of gas molecules impinging
in terms of other known transport coefficients. (Note that
on it.
irreversible thermodynamics does not provide any such
The drag force 6 r p U pz experienced by a particle
relation for determining Lii they have to be determined
moving with velocity Upz in a fluid according to Stokes
by experiment.) One can similarly show in this case that
law (3.1.61) and (3.1.63) is valid for a gaseous medium
RT when /rp << 1. When we have very small particles and
Rii : 3:1:211
Dis C i a rarefied gas condition ((/rp) >> 1), the drag force
encountered by a small particle is lower than that given
For a two-component system, a very brief excursion into
by Stokes law since there are fewer gas molecules colli-
the role of the coupling of fluxes is illustrative. For the
sions that create the resistive force encountered by the
system of solute i and solvent s and equations (3.1.208)
particle in a gas moving with a velocity vtz:
and (3.1.209), eliminate the force Fs and rearrange the
equations to get (Johnson et al., 1966, 1980; Meares, 1976) 6 r p U pz
F drag
pz for vtz 0; 3:1:214a
" ( )! ( )# Cc
L2is Lis
J i Lii 1  F i J s : 3:1:212 6 r p U pz  vtz
Lii Lis Lii Lss F drag
pz : 3:1:214b
Cc
By (3.1.206)
Here the quantity Cc, which is greater than 1, is called the
 
L2is L2 slip correction factor. It has the general form (Flagan and
Lii Lis  L2is 0 )
1 ) 1  is
Lii Lis Lii Lis Seinfeld, 1988)
3:1:213 "  #

Cc 1 exp  ; 3:1:215
is positive. From the first term within brackets in (3.1.212), rp =r p
we see that only a part of the force Fi is used to moves
species i. The second term indicates the contribution of the where 1.257, 0.4, 1.10. For small particles, the
movement of species s to the flux of species i (due to deviation from Stokes law is substantial. For example, the
coupling). The higher the magnitude of Lis and J s , the values of Cc for atmospheric air at 25  C for a few different
larger the contribution of coupling to the observed flux J i particle sizes are (Flagan and Seinfeld, 1988) in the form of
of species i. (rp, Cc): (0.005 m, 22.7); (0.025 m, 5.06); (0.05 m, 2.91);
(0.5 m, 1.168); (5 m, 1.017).

3.1.6 Additional topics


3.1.6.2 Motion of slow ions in a gas under an electrical field
A few special topics are covered here. First, slip correction
factor for Stokes law in a rarefied gaseous medium for Chemicals (represented by M here) are sometimes separ-
smaller particles is considered. Second, ion transport due ated in the gas phase by separating their ionized forms.
to a uniform electrical field is treated in the atmospheric/ The ionized product may have positive charge due to
rarefied gas phase. proton transfer (MH), dimerization (MH M !
M2H) etc.; or negative product ions are generated by, for
example, electron attachment (M e ! M ) or dissocia-
3.1.6.1 Slip correction factor for Stokes law in the gas phase
tive electron attachment (MX e ! M X), etc. Separ-
We have seen in Section 3.1.3.2.4 that, depending on the ation of the different ions can also allow their
value of (rp / ), where rp is the pore radius and is the characterization (Eiceman and Karpas, 1994). Surrounded
value of the mean free path for the gas molecules, the gas by neutral gas molecules (sometimes called buffer-gas
transport regime in the porous medium changed. When molecules), an ion is acted on by a uniform electric field
is large (for example, at low gas pressures) and rp is small, E (if it exists), which generates motion of the ion. The force
it is much more likely for the gas molecules to hit the on a single ion is eE, where e is the electronic charge. This
pore wall rather than another gas molecule (Knudsen ion motion is resisted by the buffer-gas molecules in the
diffusion). Similarly, in the case of transport of small surrounding gaseous medium. As we have considered
particles of radius rp in a gas where is the mean free path earlier (see equation (3.1.62)), the initial acceleration
for the gas molecules, if is large and rp is small (rp/ << 1 encountered by the ion having a mass of mion due to the
or /rp >> 1, (/rp) is the Knudsen number) the particle electrical field, namely (eE/mion), will soon disappear, and
appears to other gas molecules (which are far apart) as if it a steady velocity, Uion, called the drift velocity (similar to
were a gas molecule. On the other hand, when (/rp) << 1, terminal velocity) is achieved on average by each ion:
118 Physicochemical basis for separation

f dion E E
eE U ion ; 3:1:216 T  1:0354  102 ; 3:1:221
N ~ N P

where the frictional coefficient for 1 gmol of ions is f dion and where P is in torr (Mason and McDaniel, 1988). When the
~ is Avogadros number. Further, by Einsteins relation,
N electrical field is such that (E/N) is less than 10 Td, the
ionic mobility m ion;g is almost independent of the field.
f dion RT=Dion ; 3:1:217
Consider a situation where ions present in a gas are not
where Dion is the diffusion coefficient of the ion. Therefore subjected to any external force field, temperature gradient
the drift velocity of the ionized species is or coulombic repulsion. Ions will be dispersed by the
~ random motion of the ions, leading to a diffusion process.
e Dion N e Dion
U ion E B E; 3:1:218 Ficks first law is used to describe the corresponding diffu-
RT k T
sive flux expression:
where kB is Boltzmanns constant (1.380  1023 joules/K).
Since e is equal to 1.60210  1019 coulomb, we obtain J ion Dion rC ion : 3:1:222
19
coulomb Dion
1:6021  10 The total flux of the ionic species in an electrical field of
U ion E)
joule
3 T uniform strength E is a sum of the two fluxes:
1:380  10
K
  N ion C ion U ion  Dion rC ion : 3:1:223
D ion
U ion 1:16  104 E; m
ion;g E; 3:1:219
T For an introduction to, and a detailed analysis of, motion of
slow ions in gases, Mason and McDaniel (1988) should be
where m ion;g is the ionic mobility in the gas phase having the consulted.
units of cm2/s-volt since 1 joule 1 volt-coulomb. (The elec- In a highly ionized gas, the number density (number
trical field strength used in the separation of ions in ion mobility per unit volume) of electrons is approximately equal to
spectrometry is usually around E 250 volt/cm (Eiceman et al., that of the positively charged gas ions, except very close to
2004). See Section 7.3.1.2 for ion mobility spectrometry.) a boundary. Like the principle of electroneutrality in a
This analysis is useful when the applied uniform elec- solution (see equation (3.1.108a)), there is charge equality
trical field (of strength E) is weak. Further, the concen- in an ionized gas. If there is a charge imbalance, it will
tration of the ions in the gas is low enough so that the produce forces to restore the balance: what it achieves is
forces of repulsion from similar ions due to Coulombs law to slow down the diffusional rate of electrons whose dif-
(3.1.15) may be neglected. Also, the ionic drift velocity is fusion coefficient (D) is much higher than that of the
not in too much excess over their thermal velocities. If the positively charged ions (D) whose diffusional rate is
electric field strength is increased substantially, the ionic enhanced. The effective diffusional velocity of both
mobility m ion;g becomes dependent on the ratio E/Cg, species are the same. This diffusion is called ambipolar
where Cg is the gas concentration. Instead of Cg, the (Schottky, 1924).
number concentration of molecules, N, is used:
E volt=cm E
volt-cm2 :
N numbers=cm3 N 3.2 Separation development and multicomponent
However the actual unit used is Td for townsend, separation capability
where 1 Td 1017 volt-cm2 and the units of (E/N) are We have now developed a quantitative understanding of
1017 volts-cm2. Often instead of (E/N), (E/P) is used, the movement of specific molecular species (or particles)
where the pressure P is expressed in torr (1 mm Hg). The in a given direction at a particular speed under the action
corresponding conversion is obtained as follows. For ideal of a negative chemical potential gradient and/or external
gas behavior valid at low pressures, forces. Suppose a mixture to be separated is located in a
N R closed container identified as the separation system and
P C t RT RT N T N k B T: 3:1:220
~
N ~
N subjected to a particular force (or forces). It should now be
possible to follow the displacement characteristic of each
Therefore
  species in the mixture. For chemical solutions, such dis-
E E B E R E E volt=cm placements are conveniently studied by following the
k T T; TK
N P P N
~ N P torr spatial and temporal variations of concentration of any
82:05  760 cm3 -atm=gmol-K torr species; we seek to find out how Ci varies with time t and

6:022  1023 no:of molecules=gmol atm the spatial coordinates x, y, z. Development of species-
  specific regions necessary for the separation of a mixture
E
T 1:0354  1019 volt-cm2 : can then be observed first by considering the relative
P
spatial locations of concentration maxima of each individ-
If the unit of 1017 volts-cm2 is employed for (E/N), then ual species.
3.2 Separation development and multicomponent separation 119

Location of pulse of solute species i

lx
y

ly
x

ly
lx

z
0 z z

lz

Figure 3.2.1. Rectangular separator vessel.

An associated question of fundamental importance (2) The concentration of each species in the pulse at t 0
(studied next) is how many different chemical species can be is uniform in the xy-plane.
separated in a given separation system under ideal conditions. (3) The forces acting on the species make them move only
This multicomponent separation capability of a given sepa- in the positive z-direction for time t > 0; as molecules of
ration technique could then be used as a criterion, amongst the solute species move in the positive z-direction, the
others, for developing a hierarchy of separation techniques. concentration of the solute remains uniform in the xy-
Concurrently, the crucial role of the nature of the force poten- plane (at all z).
tial profile in developing the multicomponent separation
How the concentration pulse of a given solute species
capability will be illustrated. The corresponding treatments
moves as a function of z and t is best studied using
given below closely follow those by Giddings (1978).
the solution of the species conservation equation to be
developed now. The species conservation equation can
3.2.1 Separation development in a closed system be obtained by a simple mass balance of solute i in a
small volume element of cross-sectional area lxly and
Consider a rectangular separator vessel of dimensions
thickness z in Figure 3.2.1 at any z for 0
z
l z . For
(lx, ly, lz) in the (x,y,z)-coordinate directions, respectively
species i 1,
(Figure 3.2.1). Let the separator contain a single-phase
system, say a solvent. At time t 0, we subject the system
     
to a pulse of solute species 1 and solute species 2, the total rate of Species 1

rate of Species 1

rate of Species 1
number of moles of each species being m1 and m2, respect- accumulation coming in going out
C 1   :
ively. The pulse is located at the vessel side characterized l x ly z N 1z z lx ly  N 1z zz l x l y
t
by x 0, lx; y 0, ly; z 0. Assume the following.
3:2:1
(1) There is no bulk velocity of the solvent in the closed
container; if there is any solvent motion at all, it is rigid Dividing both sides by lxly z and taking the limit z ! 0,
body rotation, as in a centrifuge. we get
120 Physicochemical basis for separation

Uniform continuous
potential profile Discontinuous
potential profile

(fi)max
Uniform continuous

Discontinuous
fi

z
0
lz

Figure 3.2.2. Profiles of i analyzed. (After Giddings (1978).)

C 1 N 1z
 : 3:2:2 We will find out later that the nature of the profile i in the
t z
z-direction along the separator is crucial. It can be either
Assume now that the movement of solute species 1 in the continuous or discontinuous, with all sorts of variation
solvent does not affect the movement of solute species 2 with z in either category.
and vice versa. Following the derivation of the species We consider now a continuous profile of i , specific-
balance equation (3.2.2) for i 1, we may obtain a similar ally a uniformly continuous profile of i (the discontinuous
species conservation equation for i 2: profile is studied in Section 3.2.2). In Figure 3.2.2, the
uniform continuous profile of i is represented as a
C 2 N 2z
 : 3:2:3 straight line varying from i max at z 0 to 0 at z lz.
t z
Two simple examples of such a profile are: a uniform
The solution of these two partial differential equations, electrical field of electrostatic potential; a gravitational
subject to appropriate boundary and initial conditions, will potential along the vertical axis. A nondimensional
yield the concentration profiles for solutes i =1,2 as a representation
function of z and t.
How each of these two concentration profiles develops
i i =i max 3:2:6
along the z-coordinate with time is intimately connected
with the nature of the species-specific forces, since the will have a maximum value of 1 at z 0 and a minimum
latter determine Niz. Recall from expressions (3.1.27) and value of 0 at z lz. Since both
i and i are functions

(3.1.42), at constant temperature, of the z-coordinate only, we may write them as


i z ,

X  where
ext
F ti F ext
ti  r i r i i z z=l z : 3:2:7
rext
ti 0i  RTr n ai rtot
i ;
Substitute now the expression (3.1.81) for N 1z in the differ-
where ential equation (3.2.2), under the conditions of v tz 0 and
an ideal solution, and obtain for species 1 in solvent s the
tot ext 0
i ti i RT n ai 3:2:4
following:
is the total potential acting on species i (note: includes 0i " #
here any pressure gradient effects also; otherwise employ C 1 1 01 ext
t1 RT 2 C 1
d C1 C1 d :
the expression from (3.1.84d). Further, the rext i repre- t f1 z z z f 1 z2
sentation for any external force is insufficient for magnetic
and some other forces). Rewrite toti as
Therefore
" #
tot
i i RTn ai ; 3:2:5a C 1 1 1 RT 2 C 1
d C1 d ; 3:2:8
where
t f 1 z z f 1 z 2
X
i ext 0
ti i ; ext
ti ext
i : 3:2:5b where RT=f d1 D01s .
3.2 Separation development and multicomponent separation 121

Nondimensionalize 1 =z and the time t by i.e. at time t 0, all of the solute species 1 (m1 moles) is
contained in a very thin slab of cross-sectional area lxly at
1 1 max 1 max t 1 max
1
0
1 ; t1 ; z 0. An alternative way of looking at this is as follows:
z lz z lz l 2z f d1

3:2:9
C 1 l x l y dz m1 z dz m1 : 3:2:14c
0
where we note that 1 is a constant for the uniform 0 0

continuous 1 profile (Figure 3.2.2). Equation (3.2.8) may
now be rewritten as A well-known solution of the diffusion equation (Carslaw
and Jaeger, 1959),18
C 1 0 C 1 RT 2 C 1 " #
 1
; 3:2:10 A 2
t 1 z 1 max z2
C 1 z, t C 1 z ;t 1 C 1 ;t 1 p exp  4RT ;
t
1 1 max t 1
where C1 depends on the independent variables z and t 1.
To simplify equation (3.2.10) further, define a new set of 3:2:15
independent variables and t
1 , where is applicable here provided the z -coordinate varies

0 between and  . To determine the constant A, note


z

1 t1 : 3:2:11
that solute species 1 is conserved. Therefore, the total
In these two new independent variables, equation (3.2.10) number of moles of species 1 in a separator of dimensions
is reduced to17 x 0, lx; y 0, ly; z  to must be the sum of the
number of moles in two separators, one from z 0 to
C 1 RT 2 C 1
: 3:2:12 and the other from z  to 0. However, the latter reser-
t 1 max
2
1 voir does not have any solute in this problem. Therefore
This equation is far simpler than the earlier ones; in 2 3
fact, it is essentially the diffusion equation. ly lx 6 7
A 6 2 7
We now need the solution of this equation for the m1 0 m1 p exp 6 7dx dy dz

t1 6 4RT 7
given initial condition and suitable boundary conditions. 4 t 5
 0 0
1 max 1
Although the z-dimension of our separator is finite (= lz), 2 3
we may assume that lz is sufficiently large for the values
6 7
of t in our range of interest. Then the solution for z ! , 1 6 2 7
Al x l y lz p exp 6 7dz :
i.e. ! will be usable for our purpose. Obviously, at
t1 6
4 4RTt 7
1 5
all t
1,

1 max
C 1 C 1 3:2:16
for z or ; C 1 0 and 0 ;
z
3:2:14a Define
,s!
i.e. the value of C1 as well as its gradient in z will be zero far 4RTt 1
away from the source of C1 at z 0. : 3:2:17
1 max
The initial condition is:
Since dz d, we get
at t 0; C1 m1 =l x l y z ; 3:2:14b
r
m1 Al x l y l z 4RT exp  2  d
1 max
17 
For C 1 C 1 t
1 , z C 1 t 1 , ,
r
   
Al x l y l z 4 RT exp  2  d :
C 1 C 1
dC 1 dt
1 d: 3:1:13a 1 max
t 1 t 0
1

Therefore Therefore
         
C 1 C 1 C 1 C 1 0 C 1

1 :
t 1 z t 1 t t 1 t 1 t
1 1
18
3:1:13b The solution of the heat conduction equation given in sect.
10.3.II of this reference for an instantaneous plane source of
Further,
strength Q parallel to the plane z = 0 can be used to solve the
     2   2  particular diffusion problem. The reader can verify it by
C 1 C 1 C1 C1
) : 3:1:13c substituting solution (3.2.15) into equation (3.2.12) and seeing
z
1 t t z 2
t 2 t
1 1 1 1 that it is satisfied.
122 Physicochemical basis for separation

n o1=2
Species 2

i 2RTt i =i max i =lz 3:2:21a
Species 1
z2+ or
n o1=2
0 2
i =l z
i 2Dis t=l z ; 3:2:21b
Ci
since RT=f di D0is . Therefore, the larger the value of D0is
or the longer the time allowed for separation, the higher
will be the value of i as the species i concentration profile
+
moves along the z-axis.
z1 These results have several important implications.
z+ Consider first the instantaneous location of the center
0
point of each of the two concentration profiles where
Figure 3.2.3. Concentration profiles of species 1 and 2 at any C max
i z ; t
i occurs.
time t. Species 1:
 
0 t 1 max l z 1
m1 z1 t 1 1  2 d :
A r 3:2:18 l z f 1 1 max z
2l x l y lz RT

1 max But
p
since the value of the integral is =2. 1 ext 0
t1 1 f d1 U 1z ;
The concentration profile for species 1 is therefore z z z
given by
so that
2 n o2 3
  0 tU 1z
m1 1 6 z 1 t 1 7 z
1 : 3:2:22a
C 1 z ;t s exp 6 7 lz
1
lx ly lz 4 RT
5:
4RTt 1 4 t Species 2: Similarly,
1
1 max 1 max
tU 2z
3:2:19 z
2 : 3:2:22b
lz
The concentration profile for species 2 may be similarly Note that U1z and U2z are the z-components of the dis-
obtained as follows: placement or migration velocities of species 1 and 2,
2 n o2 3 respectively. Thus, as long as U 1z 6 U 2z , the two concen-
  0
z 2 t 2 7
m2 1 6 tration profiles (of 1 and 2) will have different locations of
C 2 z ;t
2 s exp 6
4 RT
5;
7
lx ly lz their centers. Although at t 0, species 1 and 2 were
4RTt 2 4 t
2 max 2 present in a uniform mixture at z 0 (and x 0, lx; y
2 max
0, ly), different values of U1z and U2z have created a
3:2:20 condition whereby species 1 and 2 are now spatially
where separated in the solvent present in the separator.
From (3.2.21a,b) for i, it is clear that, as the diffusion
t 2 max coefficient D0is of solute species i increases, the concen-
t
2 :
l 2z f d2 tration profile becomes broader at any time t. Therefore,
the larger the value of D0is , the greater the extent of overlap
Here
between two contiguous solute profiles. Thus, an
  s

increased value of D0is leads to reduced separation.
m1 1 max
C max
1 z ;t 1 ; Although molecular diffusion due to a concentration
lx ly lz 4 RT t 1
gradient could be interpreted in terms of a concentra-
similarly for C max
2 z ;t 2 : tion driving force that is part of the overall driving force
Figure 3.2.3 shows the instantaneous concentration Fti , here it acts to reduce separation in the presence of
profiles for solutes 1 and 2 at any time t (only the external forces. Therefore it is useful to subtract it from
z-coordinate is shown for simplicity). The concentra- Fti in developing an expression for displacement or
tion profile of species i (i 1,2) is located around a migration velocity Ui of species i (as was done in defin-
0
zi i t i and has its maxima there. The profiles are ition (3.1.82)).
Gaussian with a standard deviation i (nondimensional The effect of time on the separation achieved is
i), where an additional feature that can be investigated. First, the
3.2 Separation development and multicomponent separation 123

z-coordinate of the center of mass of the concentra- i z 0


i i t i : 3:2:25
tion profile of the ith species, z i , increases linearly with
time (see the developments after equation (3.2.25)). The Then
distance between the centers of two contiguous profiles,

i.e. z
1  z 2 , for species 1 and 2 therefore increases as
C i i ;t
i di
tU 1z  U 2z =lz . Second, the standard deviation i of each
0
profile increases as t 1=2 . Thus, as time progresses, any hi i

; 3:2:26
given solute species disperses over a wider region of space C i i ;t
i di
around the center of the profile. This feature is often called
0
band broadening or dispersion. The net effect of time on 0 1
the separation between species 1 and 2 can be determined
by using an index, e.g. the resolution Rs, where i mi =lx l y l z B C 2i
p

expB
@  RT  Adi
C
4RTt i =
i max 4 t
2z
1  z2 2z 1  z2 t 0
i max i
Rs t / 1=2 t 1=2 3:2:23 hi i 0 1 :
4 1

2 4 1 2 t
2
mi =l x ly l z B i C
Thus separation between species 1 and 2 improves with p

expB
@  RT  Adi
C
4RTt =
increasing time, although there is increased dispersion 0
i i max 4 t
i max i
in each profile. With time, then, we have better separ-
ation but increased dilution in each product region. 3:2:27
Obviously, the finite boundaries of the closed separator At any given t
i ,
the numerator has a zero value. Since hi i
impose a limit on the time during which such a behavior should provide the center of mass of the species i concen-
is feasible with a uniform continuous i potential profile. tration profile,
In practice, open separators are used and additional factors
come into play. 0
hi i 0 hz
i i t i i 0 3:2:28a
We have studied here the one-dimensional migration 0
of solutes introduced as a -function at one end of the ) z
i 
i ti : 3:2:28b
vessel. Lee et al. (1977a) have illustrated the three- To determine the standard deviation of the concentration
dimensional migration of a compact solute pulse. profile, we take the second moment of the concentra-
If a certain state of separation of two solutes or the tion profile with respect to the variable i, since
development of an individual solute profile is under con-
2
sideration, the time lapsed, as we have seen above, is quite
i hi  hi i2 i h2i i with hi i 0;
important. This time will depend amongst other things on
the solvent used. For example, to arrive at a given non- 2i C i i ;t
i di

dimensional position in the separator with two different 0 3:2:29


h2i i :

solvents s1 and s2, the times t s1 and t s2 required for a given
0 C i i ;t
i di
i value may be obtained from
0

t s1 l 2z s1 f di s1 i max s2 Now, the integral in the numerator,


t
s1 t s 2 ) 3:2:24
t s2 l 2z s f di s i max s
2 2 1 0 1

for any solute i. Obviously, the frictional coefficient of 2i B 2i C
mi =lx l y l z p expB 
C Adi ;
solute i in the solvent and therefore solvent viscosity will
4RTt i =i max @ RT
0 4 t
be important for estimating the time required for achieving
i max i

a specified state of solute profile development.


The previous conclusions on separation development can be changed to the integral below if we define
were based on two primary characteristics of any solute i
profile: (1) the center point of each concentration profile is
i ( )1=2 :
given by z 0 RTt
i i t i ; (2) the nondimensional standard
i
2
deviation i max
q i of the Gaussian profile is equal to
2D0is t=l 2z . These characteristics of the concentration mi =l x l y l z 4RTt
 
i 2 2
profiles (3.2.19) and (3.2.20) need to be justified. p
i exp i di
i max
To justify our locating of the center point of each con- 0 1
0

p
0
centration profile at z i i t i , take the first moment of mi A 4RTt i
the concentration profile (see the definition in (2.4.1g)) with @ p
lx ly lz i max 4
respect to the variable i defined from (3.2.11) as
124 Physicochemical basis for separation

at any t
i . The value of the integral in the denominator of 1:245  103  28571=2 cm 1:245  53:45
(3.2.29) is
103 cm 0:066 cm:
  !
mi 1 2 3.2.2 Multicomponent separation capability
p exp  RTi di :
lx ly lz 4RTt i =

i max
4 i max t i

0 What kind of separation processes or forces causing
separation has the capacity to achieve multicomponent
This is considered the zeroth moment of the concentration separation in a single separation vessel?
profile Ci. Using
i , it can be simplified to To answer this question, consider the solute concen-
    p tration profiles (3.2.19) and (3.2.20) obtained earlier. If we
mi 1 mi assume that the solute mixture pulse at z 0 and t 0
p exp2
i d
i p :
lx ly lz lx ly lz 2 contained more than two solute species, and if the move-
0
ment of each species does not influence those of any other
Therefore species, then
2 n o2 3

2 mi =l x l y l z fRTt i =i max g 2RTt mi =l x l y l z z
0
t

i h2i i i C i z ;t
6
exp4
i i 7
1 i max i q4RTt n o 5
mi =l x l y l z i 4 RT
t
2

i max


i max
i

2RTt i 2RTt 2D0 t 3:2:31


2 d i max 2is
i max lz f i i max lz for i 1,2. . .,n. Obviously if each solute species i has a
3:2:30 unique value of Uiz, the center point of the instantaneous
concentration profile for each solute will be at different
at any time t. locations in the separator. Further, if the standard deviations
of the profiles are small compared to the distance between
Example 3.2.1 A pulse of ovalbumin molecules is introduced
at the xy-plane at z 0 at t 0 in a uniform electrical field the center points of two neighboring concentration profiles,
(in the z-direction) of strength 30 volt/cm (see Figure 3.2.1). i.e. the z i , we have succeeded in separating a multicompo-
The length of the vessel lz 10 cm; the solution viscosity is 1 cp; nent mixture at any given instant of time. For practical sepa-
the molecular weight of ovalbumin is 45 000; ri 2.78 nm; rations, one has to isolate physically the solvent in each
( fi/fiw) 1.16 (see Examples 3.1.2, 3.1.5 and 3.1.6). Calculate region characteristic of a given species at any instant of time.
Regardless of the practical difficulties in achieving this, it
(1) the time-dependent location of the center point of the
ovalbumin concentration profile as it moves along the can be concluded that uniform continuous profiles of i ,
z-coordinate; where i 1,. . .,n, are inherently capable of separating an
(2) the standard deviation of this profile (D0iw 7:76 n-component solute mixture in a closed system with no
107 cm2 =s; see Table 3.A.5), given T 20  C. convection since each i has a separate migration velocity Ui.
(3) How long would it take for the profile center point to The above analysis was carried out for a constant value
reach the end of the vessel? of 0
i found in systems with a uniform continuous i

0
Solution (1) From the solution of Example 3.1.6, the profile. As long as i varies continuously along the
migration velocity of ovalbumin molecules separator and has a nonzero value, the multicomponent
under the above-mentioned conditions is separation ability is retained. We will learn soon that
0.35  102 cm/s. From (3.2.22a), multicomponent separation ability is absent in separation
0
systems where i 0 and there is no bulk velocity.
U iz t What is the maximum number of components that can
z
i ) zi U iz t 0:35  102 t cm:
lz be separated in a closed separator of length lz having
(2) The dimensional standard deviation of the profile uniform continuous profiles of i ? This number, called
around the center point z
i is equal to (from (3.2.21b)) the peak capacity, is defined by
0 1=2

i  l z 2Dis t =l z   l z 2D0is t1=2 lz
nmax ; 3:2:32
i 2  7:76  10 7 1=2
t 1=2 4
1:245  103 t 1=2 cm: where we have assumed an averaged value of standard
deviations for all profiles and indicated it by . The quantity
(3)
nmax is thus the maximum number of Gaussian solute
103 concentration profiles that can exist in the separator of
z i l z 10 0:35  102 t ) t s;
0:35 length lz with contiguous profiles being separated, as
t 2857 s 47.6 minutes. At this time, the standard shown in Figure 3.2.4, by a distance of 4 (thus Rs 1
deviation will be between each neighboring peak). To calculate , choose a
3.2 Separation development and multicomponent separation 125

4s 4s 4s

Ci

lz z

Figure 3.2.4. Maximum number of species separable in a separator of length lz.

particular ith species and use19 its i max and f di . Remem- In a differential form, this expression may be written as
ber, however, that the expression for i indicated earlier by (Giddings, 1969)
(3.2.21a/b) is nondimensionalized by lz since or z is dt
nondimensionalized with respect to lz. Therefore dn : 3:2:33d
Wb
lz lz Integrating this over the time difference t t 2  t 1 , we get
nmax ( )1=2 :
4 i
2RT t i max
4 lz n
max t 2
i max l2z f di
nmax dn dt=W b : 3:2:33e
0 t1
For an estimate of the characteristic time t, use
t l z =U iz ,where Uiz is the migration velocity of species For a homologous series of compounds, Medina et al.
i and is given by  i =z=f di . With these, and (2001) have proposed that Wb may be related to the time
 i =z i max =l z , we obtain t when the peak appears by
r
1 i max W b a1 a2 t a3 ; 3:2:33f
nmax ( )1=2 3:2:33a
d 32RT
2RT l z f i where a1, a2, a3 are empirical constants. Substituting into
4 2 d 
l z f i i =z (3.2.33e), we get
t 2
for uniform continuous i profiles. dt 1 a2 t 2 a3
nmax n : 3:2:33g
This result based on definition (3.2.32) assumes that an a1 a2 t a3 a1 a2 a2 t 1 a3
t1
average value of 4 may be defined. However, in practical
situations, when long separation length or times are
Example 3.2.2 A pulse of ovalbumin molecules (molecular
involved, 4 will not be a constant. Suppose one is inter-
weight, 45 000) in a mixture of a variety of proteins is exposed to
ested in finding out the maxmum number of components
an aqueous solution (density, 1 g/cm3) subjected to a steady
that can be separated in a given time window t at any electrical field of strength 30 volt/cm. Given Zi 4.5, develop an
given location in a column/vessel. Since the basic migra- estimate of the peak capacity when the electrodes are separated
tion of peaks remains unchanged, nmax may be defined as by a distance of 1 cm and the potential difference is 30 volt;
ovalbumin may be used as the representative species.
t
nmax 3:2:33b
4 Solution From equation (3.2.33a), the expression for peak
for Rs 1. Now 4 is not a constant for different species/ capacity nmax for Rs 1.0 is
peaks. Expressing 4 as the base width of the peak, Wb, we  1=2
can rewrite the expression for nmax: nmax i max =32RT :
t Here,
nmax : 3:2:33c
Wb i max ext
i max Z i max F ;

where max is the electrical potential, 30 volt. Therefore

19
As a representive value for obtaining . Z i F max 4:5  96500  30 volt-coulomb=gmol;
126 Physicochemical basis for separation

R 8.317 joule/gmol-K. Assume the temperature to be


25  C, T 298 K. We have 1
0 11=2
@ joule jouleA
nmax 4:5  96500  30 =32  8:317  298
gmol gmol 2
1641=2 12:8:
+
Ci (z, t i )
About 12 peaks may be separated. Compare this with the
number of peaks in the centrifugal separation of Problem 3
3.2.1(c).

Example 3.2.3 The definition of peak capacity (3.2.32) was


developed for the case of Rs 1 where the two neighboring
peaks were separated by the distance 4 (where is an z
assumed average value of the standard deviations of all
lz
profiles). If poorer separation is acceptable, i.e. Rs < 1, what
is an estimate of the peak capacity in a given system?
Figure 3.2.5A. Concentration profiles of species 1, 2 and 3 at any
Solution From the definition of resolution, time t for a discontinuous i profile in the vessel 0
z
lz.
z
1  z2
Rs :
2 1

2

Assuming
1 2 2 , where is an averaged stand- thin slab of cross-sectional area lxly at z 0 for t 0. The
ard deviation, Rs z
1  z 2 =4 . Therefore for any value of solution is obtained as a special case of solution (3.2.19) as
Rs less than but close to 1, z
1  z 2 Rs 4 . We can then !
write m1 =l x ly lz z 2
C 1 z ;t s exp  RT : 3:2:35
lz 4RTt 4 t 1
nmax 1 1 max

z
1  z 2 l z 1 max
1
) nmax ; The maximum value, as well as the center of mass of this
Rs 4
profile, is located at z0 (try calculating hz 1 i using the
approach of (3.2.26) with z1 instead of 1). The nondimen-
where is nondimensional.
For dimensional standard deviation , sional standard deviation of this profile will be given, as
0 2 1=2
before, by 1 2tD1s =l z . In Figure 3.2.5A, the profile
lz of C1 is shown at any time t.
nmax :
Rs 4
Example 3.2.4 In the configuration shown in Figure 3.2.5A,
We will now study the multicomponent separation a pulse of species i is injected in fluid j in the thin slab at
capability of a discontinuous i profile (shown in Figure x 0, lx; y 0, ly; z 0. Calculate the standard deviation of
3.2.2 by the dashed line). The particular discontinuous the profile as a function of time for the following systems:
profile we have chosen has a maximum value of i max system 1, n-propanol (species i)water (fluid s); system 2,
at all values of 0
z
l z , but at z lz, i abruptly drops to CO2 (species i)N2 (fluid s). The system temperature is 25  C.
the value of zero and stays at that level beyond lz. It is thus Obtain numerical values for t =100 s.
a step function. Such a profile can be obtained in practice
Solution
by having ext ext
ti 0 or F ti 0 and maintaining two differ-
ent phases or solvents in the two regions: 0
z
l z and System 1: Propanolwater, ) D0is 1:1  105 cm2 =s (Table
l z
z
2l z (say). Recognize that, for all z < l z , the value of 3.A.3). From the formula for standard deviation (dimen-
0 sional), i 2  1:1  105 t1=2 4:69  103 t 1=2 cm.
i (equation (3.2.9)) is zero. The governing equation for
System 2: CO2N2 ) D0is 0:165 cm2 =s DCO2 N2 ; 2
the concentration distribution of species 1 is then obtained
0:165  t1=2 0:574  t 1=2 cm.
from equation (3.2.10) as the simpler
For t 100 s we have (1) propanolwater, i 0:0469 cm;
C 1 RT 2 C 1 (2) CO2N2, i 5:74 cm.
: 3:2:34
t
1 i max z
2
Assuming that the diffusion of each of the solute
Note that this is merely a special case of equation (3.2.12) species i 1,2,3,. . .,n contained in the solute mixture pulse
0
obtained when i is zero so that z from definition at z 0 takes place independently of one another, we have
(3.2.11). The initial and boundary conditions of (3.2.14b) plotted the profiles of two more solutes i 2 and 3 in Figure
and (3.2.14a) for solute 1 may still be used here; the initial 3.2.5A. Obviously, at z 0, the solute whose diffusion coef-
condition merely states that all solute is contained in a very ficient is smallest will have the lowest i and the highest peak
3.2 Separation development and multicomponent separation 127

+
manner governed by the criterion for chemical equilib-
Ci (z, t i ) Solute
rium. This aspect is treated in Section 3.3.
introduction slab,
z = lz
It is important to understand why a continuous i
profile can separate a multicomponent mixture when a
1 1
discontinuous profile cannot. In the case of a continuous
2 profile, the major feature is that the concentration profile
3
of any species i appears to be translated bodily in the
z-direction. In reality, the process may be understood
3 2
by considering the Ci profile at any instant of time. There
are two regions in this profile: the trailing edge, whose
z-values are less than z i , the center-of-mass location,
z lz 2lz and where the Ci/z term is positive, and the leading
0
edge, whose z values are more than z i and where the

Figure 3.2.5B. Concentration profiles of species 1, 2 and 3 at Ci/z term is negative. Consider now equation (3.2.10)
any time t for a discontinuous i in the two adjoining vessels, without the diffusion term fRT= 1 max g2 C 1 =z 2 :
0
z
lz and lz
z
2lz, with solute slab introduced at z lz at C 1 0 C 1
time t 0.
1 : 3:2:37
t
1 z
0
The value of
1 is negative for the continuous i profile
for the same mi; suppose D01s < D02s < D03s and mi m1 (a
chosen. Therefore, in the trailing edge of the profile, C1 is
constant) for all i, then
decreasing with time: the solute species 1 is being picked
m1 =l x l y l z m1 =l x l y lz up and taken ahead (to larger z-values) to make room for
C 1 0;t
1 pq > pq
2 another species coming from behind. In the leading edge
0
2 2D1s t=l z 2 2D02s t=l 2z
of the profile, C1 is increasing with time since solute is
C 2 0;t
2 > C 3 0;t 3 : 3:2:36 being transferred from the trailing edge into it (both
0
C 1 =z and 1 negative). Thus, the existence of a non-
Therefore, solute 1 may be recovered at a high concen- 0
zero 1 directly leads to the capacity of continuously
tration around z 0 if D02s and D03s are much larger than
evacuating solute from the trailing edge and bringing it to
D01s . However, this high concentration of species 1 will have
the leading edge to make room for another species profile
impurities of species 2 and 3. On the other hand, solute 3
to move in. Meanwhile, all of the species are moving
will have spread furthest; one can collect the solvent from
forward in the positive z-direction. A discontinuous i
this furthest region and get pure species 3 in the solvent.
profile just does not have this capability.
Obviously it will be a very dilute solution of pure species 3.
We have already observed that there are two compon-
We observe then that no more than two species may be
0 ents in a i profile: ext 0
ti and i . In developing the result
imperfectly separated when i is zero, and that only one
(3.2.35), we had ext ti 0 and nonzero 0i ; but, as is true in
pure species may be obtained with a poor recovery. To
two-phase systems (see Section 3.3), 0i does not vary
determine what will happen in the other solvent in the
within a phase at equilibrium. Therefore 0i =z 0
region z > lz, it is better to start imagining what will happen
within a phase. Thus, it appears that, unless we have
if the initial solute mixture pulse were introduced at z lz 0
external forces, i will be zero and there will be no
(Figure 3.2.5B). If we can assume that a different sequence20
multicomponent separation capability. We will learn in
of profiles of species will be observed in each solvent vessel,
Section 7.1.5 (equation (7.1.100)) that this is not correct;
then one can get at the most two pure species, one in each
the multicomponent separation capability can exist even
vessel from the furthest region from the solute introduction
though external forces are absent, e.g. in chromatographic
point. It is clear that this type of discontinuous potential
processes in the presence of a nonzero bulk velocity.
profile can at the most separate two pure species and thus
inherently lacks multicomponent separation capability.
When the diffusion process ceases in such a system after 3.2.3 Particulate systems
an adequate time has elapsed, the species are going to So far, we have studied the separation of chemical solu-
distribute themselves between the two solvents in a tions including solutions of macromolecules, e.g. proteins.
In macroscopic particulate systems without any Brownian
forces of significance, for particles of size rp, mass mp, etc.,
20
Different solutes will have different solubilities and partitioning F tp F ext
tp 3:2:38
properties in the two solvents; thus the concentration difference
driving the diffusion will be different in the two solvents for any since there is no chemical potential. We know that, in
species. many cases, these external forces may be represented by
128 Physicochemical basis for separation

their scalar potentials. Such scalar potentials are usually in the two phases under mechanical equilibrium will be
continuous functions of the spatial coordinate, quite often different. If the two regions are separated by a semiperme-
uniformly continuous analogous to the cases studied in able membrane, or if there is a swelling pressure in one
Section 3.2.1. Thus, it would appear that the external phase (e.g. ion exchange resin), the constant pressure in
forces acting on particles have a multicomponent separ- each phase or region will also be different under mechan-
ation ability. Just as different chemical species had differ- ical equilibrium.
ent migration velocities Ui, similarly particles with We are more interested in chemical equilibrium,
different sizes or different densities or different charges achieved after transfer of species between two or more
will have different terminal velocities Upt. For example, in phases or regions. The criteria for equilibrium here will
the gravitational force field, the terminal particle velocity directly allow the calculation of different concentrations of
vector is given by a given species in different phases. This calculation pre-
  sumes the existence of thermal and mechanical equilib-
F ext
tp
mp g 1  t k 2
2 rp g rium. If the region is subjected to an external force field, the
p
U pt d   t k
fp 6r p 9 p criterion for equilibrium separation is affected by the
external potential field. This and other related criteria will
3:2:39
be indicated in Section 3.3.1 without extensive and formal
for spherical particles obeying Stokes law. Thus par- derivations (for which the reader should refer to different
ticles of different radii or different densities (or both) thermodynamics texts and references). The development
will have different terminal velocities, providing the pos- of such criteria will be preceded by a brief illustration of the
sibility of separating different particles. However, differ- variety of two-phase systems encountered in separation
ent particle speeds, by themselves, are insufficient for processes. Our emphasis will be on two immiscible phase
separation since all of them are moving in the same systems.
direction. Some other technique or condition has to be Mixtures in a separation system can exist in a
created to collect these particles at different locations of number of different bulk phases. The most common
the separator (as we shall see later in Chapters 6, 7, adjectives used to characterize different phases are: gas-
etc.). However, different values of Upt provide necessary eous, liquid, solid, supercritical fluid, membrane and ion
conditions for separation. Note that identical comments exchange material.21 The gaseous phase includes both
can also be made about different molecular species i in gas and vapor, just as solid includes crystalline as well as
a chemical mixture subjected to a uniform continuous amorphous materials. Some combinations, e.g. gasgas,
potential profile. gassupercritical fluid, are to be eliminated since they do
not behave as two immiscible phases. From diffusional
rate considerations, two-phase combinations of solid,
3.3 Criteria for equilibrium separation in a closed
membrane and ion exchange materials (a total of six,
separator
i.e. solidmembrane, solidion exchange material,
If two or more immiscible phases are kept in a closed solidsolid, membranemembrane, membraneion
container for a sufficient length of time, isolated from exchange material, ion exchange materialion exchange
their surroundings, the phases come to equilibrium with material) are not useful because separation would take
one another. The amount of separation achieved at forever!22 Thus one of the phases in any useful two-
equilibrium is of considerable interest. We need to know phase combination should be a fluid; i.e. gaseous, liquid
the thermodynamic criteria for equilibrium to determine or supercritical fluid. This eliminates a total of eight
this separation. In this section, such criteria are speci- combinations. (Giddings (1982) has suggested that, in
fied for a variety of equilibrium conditions encountered theory, there can be [k(k 1)/2] combinations of two
in separation processes, including those where a single phases for k available phases, i.e. there are 21 possible
phase is exposed to an external force field in a closed combinations of six phases.) The remaining possible
vessel. Chapter 4 covers the extent of separation combinations are identified in Table 3.3.1.
achieved under equilibrium conditions in a closed Of the 13 two-phase combinations identified, two com-
container. binations, gasion exchanger and supercritical fluidion
Thermodynamic equilibrium between two or more
phases or two or more regions requires the existence of
21
thermal, mechanical and chemical equilibrium. The We ignore liquid crystals or mesophases, vesicles, caged
closed separator has thermal equilibrium if all phases and molecules, etc. for the time being. They are, however, studied
in Section 4.1.8. The membrane phase includes gels. Interfacial
regions are at the same temperature T, which is constant.
phases are ignored for now.
Mechanical equilibrium requires equality of pressure P in 22
Echoes of this may have mistakenly led to the formulation
all phases or regions of the separator with plane phase Corpora non agunt nisi fluida sive soluta substances do not
interfaces. If the phase interfaces are curved, the pressures react unless in a liquid or a dissolved state (Helfferich, 1995).
3.3 Equilibrium separation criteria closed vessel 129

Table 3.3.1 Possible useful combinations of two bulk immiscible Gtj =Tp;mij ;all i Stj ;
phasesa Gtj =PT;mij ;all i V j ; 3:3:3
Gtj =mij T;P;mkj ;k6i ij :
Combinations having a
particular phase Possible useful combinations With the closed separator considered as a thermodynamic
X
Combinations having a gasliquid; gassolid; gas system, the species mole numbers, m0i mij , are con-
gaseous phase membrane; gasion j1

exchanger stant in the absence of any chemical reaction. The total


Combinations having a liquidliquid; liquidsolid; liquid Gibbs free energy of the whole separator as a system
liquid phase membrane; liquidion (obtained by summing Gtj over all j) is then only a function
exchanger; liquid of T and P. At constant T and P, the total differential of this
supercritical fluid
total Gibbs free energy is zero:
Combinations having a supercritical fluidsolid;
supercritical fluid supercritical fluidmembrane; 
X2  X2 X n
phase supercritical fluid 
dGtj T;P 0 ) ij dmij 0; 3:3:4
supercritical fluid; j1
 j1 i1
supercritical fluidion
exchanger where we have used (3.3.2) to obtain the second result. For
a two-phase (or two-region) system, dmi1dmi2 due to
a
Table 3.V in Giddings (1982) is based on seven phases, unlike the six
conservation of species i. This simplifies (3.3.4) to
phases used here.
n
X
i1  i2 dmil 0; 3:3:5
exchanger, may not be fruitful since ions are not normally i1
transferred between these phases. The rest are the basis of
which is valid for any arbitrary change dmi1. Therefore
existing equilibrium based separation processes or have a
significant potential for becoming practical techniques. i1 i2 : 3:3:6
These useful two immiscible phase combinations are to
be kept in mind as we consider now the criteria for chem- This criterion for chemical equilibrium in a two-phase
ical equilibrium in such systems, as well as other related system then requires that the chemical potential of any
material. species i should be uniform and constant through the
separator.
If the closed separator has more than two phases or
3.3.1 Phase equilibrium with equal pressure in two regions, a criterion for chemical equilibrium can be
all phases derived by considering any two of the phases or regions,
We choose for this purpose first a two-region (or two- say j 1 and j 2 , at a time. The relation
phase) separator at constant and uniform T and P without ij1 ij2 3:3:7
any chemical reaction. If we focus on a particular region, it
is obvious that it is open since molecules can be trans- can be easily shown to be valid for the total system of
ferred from this to a contiguous region or vice versa. regions j1 and j2 by following the earlier procedure. Since
Impose now the restriction that a region can only have a this result is valid for any (j1, j2) pair of phases or regions,
single phase. For this particular open single-phase region the general criterion for chemical equilibrium in a closed
chosen as a thermodynamic system, the total Gibbs free separator containing a total of k phases or regions is
energy of all molecules in region j, Gtj, is a function of i1 i2    ij    ik j 1;2;. . .;k: 3:3:8
temperature T, pressure P and the mole numbers, mij, of
all chemical species i in region j: A different form of the criterion for chemical equilibrium
(3.3.6) or (3.3.8) is often quite useful since it uses the
Gtj Gtj T;P; m1j ; m2j ; m3j ;. . .; 3:3:1
fugacities of a component i in solution instead of the
for the n-component system i 1, 2. . ., n. From standard chemical potential of species i. Let ^f ij be the fugacity of
thermodynamics texts (Van Ness and Abbott, 1982), we species i in a homogeneous mixture in region or phase j. At
know that the total differential of Gtj following a small a constant temperature, we know from the thermodynamic
change in T,P and mijs is properties of homogeneous mixtures (Van Ness and Abbott,
n
1982) that any small change in the chemical potential of
X
dGtj Stj dT V j dP ij dmij : 3:3:2 species i in region or phase j is related to the change in ^f ij by
i1
dij jT RT d n ^f ij jT : 3:3:9
Here Stj is the total entropy of all molecules in region j, Vj is
the volume of region j, Integration yields
130 Physicochemical basis for separation

h i
3.3.2 Phase equilibrium where different phases have
ij  ij initial RT n ^f ij  RT n ^f ij initial : 3:3:10a
different pressures
We assume that We consider next the requirements for chemical equilib-
rium between two regions or phases when the temperature
ij initial C 1 a constant; ^f ij initial C 2 a constant;
is uniform and constant throughout the separator, but the
3:3:10b pressure has different (but uniform) values in different
where the constants are dependent on the constant T and phases (or regions). Assume the phase interface to be
P of the separator and are assumed independent of j, the planar and assume no chemical reactions. Equation
region or phase. Then, using the general criterion (3.3.8), (3.3.2) is changed now to
we find n
X
dGtj  Stj dT V j dP j ij dmij ; 3:3:14
n ^f i1 =C 2 n ^f i2 =C 2    n ^f ij =C 2    n ^f ik =C 2 ; i1

3:3:10c where

which yields Gtj =TPj ;mij ;all i  Stj ;


Gtj =P j T;mij ;all i V j ; 3:3:15
^f ^f    ^f    ^f for j 1;2 . . .;k; Gtj =mij T;Pj ;mkj ;k6i ij :
i1 i2 ij ik

3:3:11
The total Gibbs free energy of the system consisting of the
as another form of the criterion for chemical equilibrium in a two phases or regions of the separator as a whole depends
chemically nonreactive separator at constant T and P. (Note: only on T, P1 and P2 for j 1,2 and not on the conserved
The ^ over a quantity indicates, among other things, valid for mole number m0i of any species i. At constant, T, P1 and P2,
a mixture; here, the fugacity of species i in a mixture.) the total differential of the Gibbs total free energy of the
Equality of the chemical potential of a component/ two phases should be zero:
species in two contiguous immiscible phases constituting 
X2  X2 X n
two regions rarely implies equality of concentration. In 
dGtj T;P1 ;P2 0 ) ij dmij 0:
fact, the relation between the different concentrations of j1
 j1 i1
different species in two phases at equilibrium in a separ-
Using the familiar argument of dmi1 dmi2 and the fact
ator will be developed for various types of separation phe-
that the magnitude of dmi1 or dmi2 is arbitrary, we obtain
nomena in Section 4.1 using these relations.
The relations given above are valid for any n-compon- i1 i2 : 3:3:16
ent system. As pointed out in Section 2.4, the mixture may
The chemical potential of a species i should then be uni-
be a continuous chemical mixture, where the composition is
form and constant throughout the separator under condi-
described by a molar density function f(r) whose independ-
tions of equilibrium that include different pressures in
ent variable r is some characterizing property, e.g. molecu-
different phases.
lar weight, carbon number, etc. The criterion for chemical
In the osmotic equilibrium of two liquid solutions on
equilibrium in a two-phase (or two-region) system of
either side of a semipermeable membrane (see Example
j 1, 2 is, for all values of r, (Cotterman et al., 1985)
1.5.4), the semipermeable membrane, if perfect, prevents
1 r 2 r; 3:3:12 the exchange of the solute across the region boundary
while the solvent passes through the membrane, the region
where 1 r and 2 r refer, respectively, to the chemical
boundary. In the development of the criteria for chemical
potentials of species of property r in phases 1 and 2,
equilibrium, it was always presumed that any species i
respectively.
could be exchanged between adjoining phases or regions.
If the mixture is semicontinuous, i.e. the concentrations
Therefore, the criteria for equality of chemical potential
of some components (i 1,. . ., n) have discrete values in
apply only to those species which are permeable through
terms of mole fractions, whereas the concentrations of all
the membrane and not to impermeable species.
others are described by a molar density function f(r), there
We will digress now and describe the dependence of
are two criteria for chemical equilibrium (Cotterman and
ij P j ;T ;x ij on Pj, T and xij. A total differential of ij for any
Prausnitz, 1985),
phase j can be written as follows (Denbigh, 1971):
i1 i2 ; 3:3:13a
n1 
X 
ij
for each i 1,. . ., n; for the continuous mixture part, dij  Sij dT V ij dP j dx kj ;
k1
x kj T;Pj ;x l;l 6k
1 r 2 r 3:3:13b 3:3:17a
for all values of r. where
3.3 Equilibrium separation criteria closed vessel 131

!
Sij ij =TPj ;xnj ;all n ; ij =P j T;x nj ;all n V ij ^f
ij
ij P j ;T;x ij  ij P j ;Tjx ij 1 RT n RT n aij ;
3:3:17b f 0ij

and i 1,2,. . ., n. Here, Sij is the partial molar entropy of 3:3:20b


species i in region j. where aij is the activity of species i in region j. Use relation
Note that only n  1 x kj s are independent. At any (3.3.19) for xij 1 to obtain
fixed composition xij,all i, we get
ij P j ;T ;x ij ij P 0 ;Tjx ij 1 V ij P j  P 0 RT n aij :
dij Si dT V ij dPj : 3:3:18a
3:3:20c
Integrating from standard state pressure P0 to the pressure
Pj of region j, we get, at constant T, Usually, ij P 0;T at standard state pressure P0 and
system temperature T for a pure substance i is identified
P
j as the standard state chemical potential 0i P 0;T 0i .
0
ij P j ;Tx ij ij P ;T x ij V ij dPj : 3:3:18b Then
P0
ij P j ;T;x ij 0i V ij P j  P 0 RT n aij 3:3:21
For liquid solutions, V ij is generally a weak function of
pressure. Therefore is the general relation linking ij, Pj, T and xij (through aij).
0
ij P j ;T xij ij P ;Txij V ij P j  P : 0
3:3:19 A special result for a pure substance is

At any fixed pressure Pj, we can also integrate (3.3.17a) ij P j ;T 0i P 0 ;T V ij P j  P 0 : 3:3:22


between the pure ith species and the composition xij to get,
at constant T, One further obtains, at standard state conditions,

 xij d0ij V ij dP 0 3:3:23
 n1 
X 
 ij
ij P j ;T;x ij  ij P j ;Txij 1 dx kj :
 k1
x kj T;Pj ;x l l6k; for a differential change in the standard state pressure P0.
 x ij 1
Since the standard state is important for many calcula-
3:3:20a
tions, Table 3.3.2 summarizes the commonly employed
But we know from relation (3.3.10a) that, for the initial standard states for the more frequently encountered
state being pure i, conditions.

Table 3.3.2. Standard states

State or phase of system and/or species under consideration Standard state defined by pressure, temperature or composition

(1) Gas or vapor pure component gas or vapor behaving ideally at 1 atm and
system temperature
(2) Liquid acting as a solvent pure liquid component at the same temperature and pressure as
the solution
(3) Solid acting as a solvent pure solid component at the same temperature and pressure as
the solid solution
(4) Solute in a liquid or solid solution such that the pure pure liquid or solid component at the same temperature and
component solute is liquid or solid at the temperature and pressure as the solution
pressure of the solution
(5) Pure component solute does not exist in the same phase as Infinite dilution standard state whereby the standard state value
the solution at the temperature and pressure of the solution of fugacity or activity of a component at the temperature
and pressure of the solution is given by the ratio of the
fugacity or activity to the mole fractiona under conditions of
infinite dilution
a
Sometimes instead of the mole fraction xij, the molality, (mi,j, moles of i/kg of solvent in region j) is used. Whether molality or mole fraction is
used, this standard state is intimately connected with Henrys law:

lim f ^ x il f 0 x il H i ,
x il !0 il il

where Hi is Henrys law constant for species i in the solution and is the hypothetical fugacity fil0 of the species at the infinite dilution standard
state (see relation (3.3.59)).
132 Physicochemical basis for separation

3.3.3 Single-phase equilibrium in an external force field total work done in bringing the species i from a vacuum
into phase j (Adamson, 1967). The criterion for equilibrium
We observed in Chapter 1 that separation is possible in a
for all ith species (i 1,. . ., n) is that the electrochemical
closed separator even if it contains only one phase (see
potential of a species must be the same in all phases:
Figure 1.1.3, example III; Examples 1.4.1 and 1.5.3; Prob-
lem 1.4.3; Section 3.2.1 analyses such a system in general). e1 e1
i1 i2 ; j 1;2: 3:3:28
In such cases, there exists usually one (or more) external
force field (or temperature gradient) which creates a dif- Using definition (3.3.27) in this relation leads to
ference in composition at different locations within the i1  i2 Z i F 2  1 ; 3:3:29
separator. These different locations then become different
regions. A phase at equilibrium in our earlier results for where the phase potentials 2 and 1 are independent of
criteria for equilibrium has the same properties every- the ionic species under consideration. This potential dif-
where. The presence of an external force field, however, ference (21) has been identified as the Donnan
imparts different values of the external potential ext to potential.
i
different locations in a single-phase closed separator. Con- An additional relation has to be used in each phase
sequently, no two locations in such a separator are identi- containing charged species/ions. This is the
cal; one could presume the system to be composed of an electroneutrality condition, according to which there
infinite number of phases of differential thickness in the should be no net electrical charge at any bulk location
direction of the external force field. (there may be deviations from it at phase boundaries; see
Assume now two locations and a differential dis- Helfferich (1962, 1995)):
tance apart. At equilibrium, the net total force Fti on X X
Z i C i1  Z k C k1 ; 3:3:30a
species i should be zero. For the case of a single external
force field F ext
i , we get from the general expression (3.1.50)
 where i positive ions, k negative ions, for phase j 1
 without any fixed charges;

ri  F ext
i : 3:3:24 X X
T
Z i C i2  Z k C k2  X ; 3:3:30b
For an external force field describable by rext
i , the one- for phase j 2 with fixed charges, where X is the molar
dimensional representation of the above relation along the
density of fixed charges and is the sign of fixed charges
coordinate direction z is
  ( for positive charges, for negative charges).
di  dext 
 It is useful to consider an explanation regarding elec-
 k i
k ) di   dext
i ; 3:3:25 troneutrality and the potential difference 21 in ion
dz  T dz T
exchange systems provided by Helfferich (1962, 1995).
relating the change in chemical potential of the ith species Ion exchange resin particles have fixed positive or negative
for a differential distance dz to the corresponding external- charges.23 When they are placed in a solution of electro-
force-created potential difference. For a finite distance lytes, counterions (i.e. ions with charges opposite to the
z  z between two regions and , we obtain, on inte- fixed charges) diffuse into the porous ion exchange resin
grating (3.3.25) (Denbigh, 1971), particles and maintain electroneutrality. However, there
   
    then develop large concentration differences between the
  ext  ext 
i z  i z i z  i z ) i  i ext ext
i  i : two phases. For a cation exchange resin system, say (with
   
fixed negative charges), the cations are counterions; their
3:3:26 concentrations are larger in the ion exchange resin particle,
3.3.4 Equilibrium between phases with electrical charges whereas the concentrations of anions (coions) are larger in
the solution. Therefore anions tend to migrate to the resin
In systems using, say, ion exchange resins (see Section particle and cations from the resin tend to migrate to the
3.3.7.7), the resin particles have fixed electrical charges. solution. The migration of a few ions in both directions
Similarly, ion exchange membranes (Section 3.4.2.5) have immediately builds up an electrical potential difference
fixed electrical charges. If there is no externally applied elec- between the two phases. This is the Donnan potential.
trical field, it is useful to enquire what criteria govern chemical The positive charge in the solution prevents cations from
equilibrium in two-phase systems containing such a phase. leaving the resin, whereas the negative charge on the resin
It is common practice to define an electrochemical prevents the migration of anions from the solution to the
potential el ij in the jth region by resin. An equilibrium is established between the repulsive
el
ij ij Z i F j ; 3:3:27

where j is the electrical potential of the jth phase and Zi is 23


See Figures 3.3.9, 3.3.10 and 3.3.12 for schematics of such
the electrochemical valence of species i. It is defined as the particles and their charges.
3.3 Equilibrium separation criteria closed vessel 133

PI
Solute concentration
profile

Cib
Gas phase
Csb

Solution
phase
Solvent
concentration profile
Scales differ
for solute
and solvent
concentration
t1 t2

Interfacial region

Figure 3.3.1. Concentration profiles across a gasliquid interface: PI = phase interface.

forces of similar charges and the so-called driving force Since the interfacial phase is in addition to the two
of a concentration difference. An identical condition is bulk phases, the criteria (3.3.8) for phase equilibrium
obtained with an anion exchange resin, except the should be applicable if there are no external force fields.
Donnan potential has an opposite sign. Therefore
Regardless of the Donnan potential, however, electro-
i1 i2 i ; 3:3:31
neutrality is maintained in the ion exchange resin or the
solution. Helfferich (1962) states that where we have identified the surface phase by the sub-
Migration of just a few ions is sufficient to build up so script . In conventional phase equilibria, such a relation
strong an electric field counteracting any further migra- is generally sufficient to determine the equilibrium con-
tion that deviations from electroneutrality remain far centrations of any species in the two phases at equilib-
from the limit of accuracy of any method except for the rium. But estimation of the surface phase concentration to
measurement of the electric field itself. determine equilibrium separation between a bulk phase
and the surface phase requires consideration of surface
forces.
3.3.5 Equilibrium between bulk and interfacial phases
The molecules of any species i present in the bulk of
In the absence of external force fields, we have so far a liquid phase experience a net force if they are at the
assumed that, in a two-phase or multiphase system, the surface of this phase (i.e. at the interface of two contigu-
intensive properties of any phase such as pressure, tem- ous phases, liquidliquid or liquidair). This force
perature, composition, etc., were constant and uniform creates a tendency for the bulk phase surface to contract
throughout the phase, including and up to the interface and is termed the interfacial tension 12 (unit, dyne/cm),
between the phase under consideration and the adjoining where the superscripts refer to the two bulk phases
phase or phases. However, in many systems, the interfacial 1 and 2.
region demonstrates properties different from those of Consider a system with a planar interface between the
either of the bulk phases. For example, in a dilute aqueous phases. Let the volume, surface area and thickness of such
solution of surfactants exposed to air, the surfactant con- a surface region shown in Figure 3.3.2A between two bulk
centration at the airsolution interface on the solution side phases 1 and 2 with planar interfaces 110 and 220 be,
is greater than that in the solution bulk. Skimming off the respectively, V, S and t (Guggenheim, 1967).
surface layer provides a way of obtaining a more concen- Denote the uniform pressure in both bulk phases by P.
trated surfactant solution, and is therefore a reasonable The force in the direction parallel to planes 110 and 220
basis for a separation process or processes. A two-phase inside the surface phase is, however, given by Pt  12 .
system has thus become a three-phase system: two bulk This force acts on an area of height t and unit width
phases and one interfacial or surface phase. Figure 3.3.1 perpendicular to the plane of the paper in Figure 3.3.2A.
illustrates how the composition of a surface active solute i Following Guggenheim (1967), it may be shown that, if the
and the solvent s varies in the interfacial region of the surface layer values V, S and t are changed by a differen-
surfactant solution (j 1) air (j 2) system. tial amount to V dV, S dS and t dt through a
134 Physicochemical basis for separation

d12 1 d1 s ds 1 d11 s ds1 3:3:36

where one of the bulk phases is j 1. The GibbsDuhem


equation for bulk phase 1 at constant T and P is
2
x 11 d11 x s1 ds1 0: 3:3:37
2 2
Substitution of this relation into (3.3.36) results in
s
ts

 
x 11
1 1 1  s d11 d12 : 3:3:38
x s1
1
The coefficient in brackets on the left-hand side may be
interpreted as follows. Since species 1 is the solute and s is
the solvent, s x 11 =x s1 is the number of moles of solute
1 per unit area of the interface if the solvent and solute
are present in the -phase in the same ratio as in bulk
phase 1. The coefficient in brackets in (3.3.38), if positive,
indicates the excess number of moles of solute 1 per unit
interfacial area at the interface due to a deviation in behav-
ior from the bulk phase. Denote this explicitly by rewriting
Figure 3.3.2A. Schematic of the interfacial region according to (3.3.38) as
Guggenheim (1967).
E1 d11 d12 3:3:39a

or
reversible process, the reversible work done by the inter- d12
E1  3:3:39b
facial phase as a system is P dV  12 dS . This is in con- d11
trast to the reversible work P dV done by a bulk phase
for the two-component (solute i 1) two-bulk-phase
when there is a differential change dV in volume V.
system with an interfacial region of distinct properties.
Incorporating such a departure in a total differential of
(The superscript E denotes a surface excess quantity.)
the internal energy of the surface phase, and using Eulers
Generalizing such a procedure for an n-component solu-
theorem on homogeneous functions, the following relation
tion with a solvent i s, we get, corresponding to (3.3.39a),
can be shown to be valid for the surface phase (Guggen-
heim, 1967): n
X n
X
Ei di1 Ei di d12 ; 3:3:40a
n
X
12 i 1 i1
St dT  V dP S d mi di 0: 3:3:32 i 6 s i 6 s
i1
where
At constant T and P,  
x i1
Xn Ei i  s ; i 1; 2 ;. . .;n 3:3:40b
S d12  mi di : 3:3:33 x s1 i6s
i1
Here Ei is called the surface excess of species i. Note
Define the surface concentration i of species i (in gmol/ that when i s, Es 0, i.e. the surface excess of
cm2) by the solvent is zero. For surface active solutes,
12
mi d12 =dint 12
i1 ai1 =RT d =dai1 is negative since
i : 3:3:34 E
S decreases with increasing solute concentration: thus i is
positive. For electrolytic solutes (e.g. salts) in an aqueous
The isotherm (3.3.33) may be rewritten as
solution at an airwater interface, Ei could be negative since
n
X n
X n
X the electrolytic solute prefers the environment of the water
d12 i di i di1 i di2 molecules in the bulk compared to that of the airwater
i1 i1 i1
interface, raising the value of 12 over that of pure water.
3:3:35
Consider now equation (3.3.39b). It relates the inter-
using relation (3.3.31). This result is, however, not used in facial concentration of solute 1 to the bulk concentration of
this form. solute 1 through the dependence of interfacial tension on
Consider two species i 1, the solute and i s, the the bulk chemical potential of solute 1. Remember, we
solvent. Equation (3.3.35) can be written for i 1, s as need the equilibrium criterion so that we can relate the
3.3 Equilibrium separation criteria closed vessel 135

solute concentrations in the two phases. Both results same thermodynamic equations which led to the Gibbs
(3.3.39a) and (3.3.40a) are known as the Gibbs equation equation. The Gibbs adsorption isotherm in this case has
or Gibbs adsorption isotherm. The procedure normally been shown to be (Hill, 1949)
used to derive the Gibbs equation, however, is different n n
X X
and is based on the assumptions that V 0 and the S d qi di qi dig ; 3:3:43a
solvent surface excess is also zero (Davies and Rideal, i1 i1

1963; Adamson, 1967). There is a sound reason for where qi is the number of moles of gas i adsorbed per unit
adopting the Gibbsian strategy of V 0. The quantities mass of adsorbent, S is the surface area per unit mass of
E1 and Es in equation (3.3.38) are arbitrary since their the adsorbent, and , the spreading pressure, is defined to
values are dependent on the locations of the boundaries be the lowering of surface tension at the gassolid interface
110 and 220 . However, the quantity 1  fx 11 =x s1 g s is due to adsorption. Unlike gasliquid interfaces where the
independent of the volume V and the locations of the interfacial tension 12 is measurable, the spreading pres-
boundaries. Gibbs therefore replaced the real situation sure of a gassolid surface is not measurable. The inter-
with an imaginary one where bulk properties continued facial region is often identified as the adsorbate. Expressing
all the way to a hypothetical phase interface, a two- (3.3.43a) for a binary gas mixture of species 1 and 2 as
dimensional region which had the surface excess or sur-
2
X
face deficiency of the solute only. S d qi dig ; 3:3:43b
For surface-active solutes lowering the value of 12, a i1
film pressure or spreading pressure has been defined, for 2
X q i
example for an airwater system, as d RT d n ^f ig 1 RT d n ^f 1g 2 RT d n ^f 2g ;
i1
S
12 12
solvent  solution : 3:3:41a d
1 d n ^f 1g 2 d n ^f 2g : 3:3:44
This is a sort of two-dimensional pressure for the two- RT
dimensional interfacial region with no volume (Gibbsian
formulation V 0) but a surface area S. Thus instead of For a pure gas species i
pressure, volume and temperature (P, V, T) of a conven- d
tional three-dimensional phase, the hypothetical i d n fig 3:3:45
RT
interfacial phase has , area and temperature (, S, T).
Correspondingly, the ideal gas equation of state for a pure or
gas, PV RT for 1 mole of gas, is replaced by d d n fig
RT i : 3:3:46a
S RT; 3:3:41b d i d i

where S is the surface area per mole of the gas. For qi Since d n fig =d i is obtainable from an isotherm
moles of a gas i in the surface phase per unit adsorbent (experimental or otherwise), d=d i can be determined
mass (see equation (3.3.43a) for a changed definition of S) (Lewis and Randall, 1961). Note that for an ideal gas
behavior with the pure species at pressure P,
S qi RT: 3:3:41c
d d n P
RT i : 3:3:46b
In the application of equation (3.3.40a), all independent d i d i
ionic species are to be included along with nonionic Expressing i as qi / S, we obtain, by integrating from 0
solutes. Further, the electroneutrality condition requiring to P as increases from 0 to ,
the absence of net charge anywhere suggests that the
following should be satisfied: P
1 S qi dP
S d ; 3:3:46c
X X RT RT 0 P
Z i C i1  Z k C k1 ; 3:3:42a 0
ipositive ions knegative ions
an alternative form of (3.3.46a) for a pure ideal gas being
X X adsorbed.
Z i Ei  Z k Ek : 3:3:42b
ipositive ions knegative ions So far, the surface excess of a solute species was con-
sidered on the surface of a bulk phase, e.g. water, air, solid
If an applied electrical field is present, the surface adsorbent, etc. The surfaces of macromolecules, especially
adsorption equilibrium relation (3.3.40a) is modified by proteins, have an interfacial region, sometimes called the
the inclusion of an electrical potential term (Davis and local domain, which can have compositions different from
Rideal, 1963). the bulk domain, namely the bulk of the solution. For
The criteria for equilibrium in gas adsorption on the example, the bulk of an aqueous protein solution may
surface of a solid adsorbent may also be obtained using the have cosolvents (or cosolutes) such as urea, guanidine
136 Physicochemical basis for separation

A question of some importance is: Does the curved


surface of a drop or other geometries affect the chem-
P2
r ical potential (or fugacity) of a species compared with
that on a planar surface? For gasliquid, vaporliquid,
solidliquid or liquidliquid systems, this subject has
P1 been considered by Lewis and Randall (1961), starting
P1 > P2 from the general equation of a differential change in total
Gibbs free energy of a phase j (compare with relation
(3.3.2)):
Figure 3.3.2B. Pressure in two regions (inside and outside) of a X
dGtj Stj dT V j dP j 12 dS ijPl dmij ; 3:3:49
spherical bubble.
where 12 dS appears due to interfacial tension between
hydrochloride, sucrose, etc., added in large amounts (40 phases j 1 and j 2. Note that the quantity ijPl in
50% by weight). As Tang and Bloomfield (2002) state: (3.3.49) refers to any process of transfer of mij without
These cosolute molecules bathe and solvate the macro- any change in S (planar interface, subscript Pl); to indicate
molecular solute as water does. this explicitly,
 
The composition of the local domain is important from the Gtj
ijPl ; 3:3:50
point of view of how the macromolecular solute, for mij T;Pl;S ;mkj ;k6i
example protein, will behave. If the local domain is
depleted in the cosolutes and enriched in water relative where we have included a subscript Pl to refer to a planar
to that in the bulk solution, the macromolecule (protein) is interface. From Lewis and Randall (1961), for any curved
said to undergo preferential hydration. If, however, the interface system, we can express relation (3.3.49) in general
local domain, is depleted in water relative to the bulk, then as
X
either of two things can happen. The cosolutes may be dGtj Stj dT V j dP j ij dmij : 3:3:51
preferentially accumulated in the local domain, or the local
domains of neighboring macromolecules/proteins may For a sphere, any small change in the surface area of the
interact with each other (for example via hydrophobic sphere dS due to a differential change dr in the radius will
interaction; see Section 4.1.9.4) leading to, for example, lead to the following change in volume dV of the sphere:
precipitation (see Section 3.3.7.5).
dV 4r 2 dr; dS 4r dr2  4r 2 8r dr:

3.3.6 Curved interfaces Correspondingly,

The interfaces between two bulk phases considered so far have dV


dS 2 : 3:3:52a
been planar, and the pressures in the two bulk phases under r
equilibrium have been equal for gasliquid and liquidliquid If the volume change is due to the addition of dmi moles of
systems. When the interface has a curvature, mechanical species i for all i, then
equilibrium requires different values of the pressures in the X
two phases. The general relation (the YoungLaplace equa- dV V ij dmi ; 3:3:52b
tion) governing the pressure difference between bulk phases 1
which leads to
and 2 is as follows (Adamson, 1967; Guggenheim, 1967):
  dV 212 X
1 1 12 dS 212 V ij dmij 3:3:52c
P 1  P 2 12 ; 3:3:47 r r i1
r1 r2

where 12 is the interfacial tension and r1 and r2 are the 2V ij 12


) ij  ijPl 3:3:52d
principal radii of curvature of the interfaces. By conven- r
tion, r1 and r2 are positive if they are in phase 1 and P is is a specific example for a spherical droplet of radius r
positive, with the interface convex going from phase 1 to (Figure 3.3.2B). Using the relation (3.3.9) between ij and
phase 2. For a true spherical surface, as in a spherical ^f , we get24
ij
bubble of radius r (Figure 3.3.2B),

P1  P 2 212 =r 3:3:48
since r1 r2 r. For both results, it has been assumed that
12 is not affected by r1, r2 or r. For a planar interface, r1 24
Other considerations may come into play to prevent ^f ij from
r2 and P1 P2 in relation (3.3.47). increasing continually as r is reduced.
3.3 Equilibrium separation criteria closed vessel 137

^f 2Vij 12 systems in Section 3.4, but will also facilitate calculating


ij
n : 3:3:53 the separation achieved in a closed vessel in many systems
^f
ijPl
RTr
considered in Chapter 4. We consider separately equilib-
rium in gasliquid, liquidliquid, gasmembrane, liquid
Here, ^f ij is the fugacity in a system with a spherical membrane/porous sorbent, liquidsolid, interfacial
surface, with the interface convex going from the droplet adsorption systems, ion exchange systems, as well as in
phase to the surroundings, and ^f ijPl corresponds to that supercritical fluidsolid/liquid systems. Only elementary
with a planar surface. For ideal gas behavior, replace ^f ij and essential aspects of each equilibrium solute distribu-
by pij. tion will be our concern. Although we describe the equilib-
This result has implications for droplets, which have rium distribution of a solute25 between two phases in
their surface convex toward the vapor. Consider a pure general, there are cases here, e.g. vaporliquid equilib-
liquid drop of radius r. Then ^f ijPl is equal to the vapor rium, where the solute is a major constituent of the phases
pressure of the pure liquid (on a flat surface), P sat iPl , if the
under consideration. The notion of a solute in a solvent is
fugacity coefficient sat
i under the conditions of P and T is
inappropriate in such cases.
assumed to be unity. However, equation (3.3.53) implies
that the vapor pressure of the pure liquid in the space
3.3.7.1 Gasliquid equilibrium
above the convex curved surface of the drop, Psat i , is higher
than P sat
iPl at equilibrium (Figure 3.3.2B):
We discuss first the distribution of a solute i between a gas
  and a liquid at pressure P. At equilibrium, we obtain from
2 V ij 12 the criterion (3.3.11)
P sat
i P sat
iPl exp : 3:3:54
RTr
^f ^f : 3:3:55
ig il
Further, the smaller the drop diameter, the higher the
observed vapor pressure of the liquid in the vapor space, From standard thermodynamics texts, the fugacity of
leading to a higher rate of evaporation from the drop. species i in each phase may be expressed as
Equation (3.3.54) is identified as the Kelvin equation and
^f x ig
^ ig P; 3:3:56
the phenomenon is called the Kelvin effect. This increased ig
evaporation tendency appears because a molecule near the ^f x il f 0 ;
il il il 3:3:57
droplet surface is attracted to a lesser extent toward the
interior by the surrounding molecules (since there are where ^ ig is the fugacity coefficient of species i in the gas
fewer of these surrounding molecules). However, this phase at P and system temperature T, and fil0 is the stand-
analysis breaks down as the drop size becomes too small ard state fugacity of species i in the liquid phase at a
and the number of molecules become much smaller. If the standard temperature and pressure. Therefore, the mole
drop consists of a solution of a nonvolatile solute, the fraction ratio of i between the two phases in equilibrium is
Kelvin equation applies to the solvent. Note that if the given by
liquid surface is concave toward the vapor, the fugacity
^f at the surface is less than that on a planar surface. See x il ^ ig P
ij : 3:3:58
Section 3.3.7.5 as well as Problem 3.3.2 for the application x ig il f il0
to a solidliquid system.
If the gas phase behaves as an ideal gas and the liquid
phase behaves as an ideal solution, we know that ^ ig 1,

3.3.7 Solute distribution between phases at il 1 and pig x ig P. If the gas phase is distinctly in the
equilibrium: some examples nonvapor region (see item (5), Table 3.3.2), it is known
from thermodynamics that
When two or more immiscible phases are at equilibrium,
species generally distribute themselves between the phases lim ^f il x il f il0 x il H i ; 3:3:59
x il !0
such that the uniform and constant concentration of a
species in one phase is different from those in other where Hi is Henrys law constant. The relation (3.3.58) is
phases. Since there can be a large number of combinations then reduced to either
of two-phase systems in equilibrium (see Table 3.3.1), the
x ig H i =Px il 3:3:60a
variety in distribution coefficient (il) relations is enor-
mous. Our objective here is somewhat limited. We pick a or
few common two-phase systems in equilibrium and illus-
trate the relations between the concentrations of any solute
between the phases at equilibrium. This will not only allow 25
Particle distribution between two immiscible phases has been
us to develop integrated flux expressions in multiphase considered separately at the end of this section.
138 Physicochemical basis for separation

pig, partial pressure of species in gas (mmHg)


2000
T C
SO2water 20
1500 NH3water 20
O2water 25

1000

500

0
0.000 0.100 0.200 0.300
xil, mole fraction of gas i in water

Figure 3.3.3. Solubility behavior of three different gas species in water. The value of the Henrys law constant may be calculated from (pig
/xil) = Hi. For example, for the SO2water system, at a pig = 450 mm Hg, xil 0.017. Therefore Hi = (450/0.017) mm Hg/mole fraction =
2.65  104 mm Hg/mole fraction = 35 atm/mole fraction.

pig H i x il ; 3:3:60b f 0il f il P sat sat


i i ; 3:3:62

which is Henrys law of solubility of gas species i in the where P sat


i is the vapor pressure of pure component i at
liquid under consideration. Figure 3.3.3 illustrates the the system temperature and sat i is the fugacity coefficient
Henrys law behavior of a number of gases in water of pure i at P sat
i at the system temperature. For pressures
according to relation (3.3.60b). These are based on when the vapor phase is an ideal gas, sat i 1. Further,
values of Hi available in the literature for three gases, ^ iv 1. If the liquid phase is an ideal solution, then

NH3, SO2 and O2 (Geankoplis, 1972; Perry et al., 1984). il 1. Thus
Note that the solubility of O2 is so low that its mole
x iv P sat
fraction in the liquid phase does not show up at the scale Ki i ; 3:3:63
x il P
used. Further, the lower the solubility of a gas species,
the higher the value of Hi. The liquid phase is often where the system pressure is P, and both the vapor and the
called absorbent. liquid phase behave ideally. When written as
If the gaseous phase can be considered as a vapor,
x iv P sat
i x il =P; 3:3:64
then the distribution of a species between the vapor and
liquid phase is of interest. Obviously, the species can be we obtain the well-known Raoults law of vaporliquid
a major constituent of each phase here. The equilibrium.
equilibrium ratio Ki of a component i is defined as An alternative representation of Raoults law is useful
(equation (1.4.1)) for the case of continuous chemical mixtures. Rewrite rela-
x i1 tion (3.3.63) as
Ki :
x i2 Px iv piv P sat
i x il ; 3:3:65
Generally, phase 1 is the vapor phase ( j v) and phase 2 is
where piv is the partial pressure of species i in the vapor
the liquid phase ( j l). At equilibrium, use relations
phase behaving ideally. For a continuous chemical mixture
(3.3.56) and (3.3.57); then, of vapor and liquid phases at equilibrium, let fl(M) and
x iv f0 fv(M) be the molecular weight density functions of the
Ki il il ; 3:3:61 liquid and vapor phases, respectively. If the vapor pressure
x il ^ iv P
of a species of molecular weight M and T is indicated by
since ^f il ^f iv . For low to moderate pressures, the standard Psat (T;M), then Raoults law for species in the molecular
state fugacity in the liquid phase is the fugacity of pure weight range M to M dM is
liquid at the temperature and pressure of the system
(Smith and Van Ness, 1975): Pf v M dM P sat T;Mf l M dM;
3.3 Equilibrium separation criteria closed vessel 139

i.e. illustration of each type of system.) In the general case, we


sat use the equilibrium criterion (3.3.6) for solute i between
Pf v M P T;Mf l M: 3:3:66
two immiscible liquid phases j 1,2, namely i1 i2. But
One can use density functions fv and fl as functions of at any P, T,
quantities other than molecular weight, e.g. boiling point,
carbon number. il P 0il P 0 RTn ail V ij P  P 0 3:3:71
In the vaporliquid or gasliquid equilibrium studied
above, the vapor or gas species existed as molecules of from (3.3.20c) and (3.3.21) for ij. Therefore
that species in the liquid. With diatomic gases, e.g. H2, O2,
N2, etc., and liquid metal, it is, however, found that V i1 P  P 0 RTn ai1 0i1 P 0
the gas is dissolved atomically in the molten metal. For
example, in molten iron, the following equilibrium has RT n ai2 0i2 P 0 V i2 P  P 0 : 3:3:72
been suggested for nitrogen gas (Darken and Gurry,
1953): Two cases arise depending on the solute, P and T if we
assume V i1 V i2 .
N2 g 2Nl: 3:3:67
Case 1 If pure solute i exists as a liquid at P and T, then
The equilibrium constant for this chemical reaction may be the standard state is independent of phase 1 or phase 2
written in terms of activities of each species as (see item (4), Table 3.3.2):

0i1 P 0 0i2 P 0 : 3:3:73


aNl 2
K : 3:3:68
aN2 g This implies
For an ideal gas, ai1 =ai2 1; 3:3:74

i.e.
aN2 g x N2 g pN2 g =P: 3:3:69a x i1 i2
: 3:3:75
x i2 i1
For the liquid phase, the activity of the atomic nitrogen
species is related to its mole fraction: In such a case, the mole fraction of solute i in the two
liquid phases 1 and 2 will be different only if the solute
aNl x Nl Nl : 3:3:69b behaves nonideally in both phases and i2 6 i1. Partition-
ing of, say, acetic acid at 25  C between water and isopro-
pyl ether will fall into this category since pure acetic acid is
Substituting these into (3.3.68), we get
a liquid.
 1=2
K pN2 g 1=2 Case 2 Pure solute i does not exist as a liquid at P and
x Nl : 3:3:70
P Nl T; an infinite dilution standard state is necessary (item
(5), Table 3.3.2). In general, such standard state values
Thus, the mole fraction of atomic nitrogen in the liquid 0ij (P0) are dependent on phase j. Therefore, from rela-
metal phase is proportional to the square root of the partial tion (3.3.72),
pressure of molecular nitrogen in the gas phase. This
square-root dependency is known as Sieverts law and is 0 0 0
ai1 P  0i1 P 0 i
found to be valid for diatomic gases like O2, H2, etc. The exp i2 exp ;
ai2 RT RT
effect of a chemical reaction on solute distribution between
two phases in equilibrium has been considered in great
where
detail in Chapter 5.
0i 0i2 P 0  0i1 P 0 : 3:3:76

3.3.7.2 Liquidliquid equilibrium Correspondingly,

Next, we consider the distribution of a solute in liquid  


x i1 i2 0
liquid equilibrium. Liquidliquid systems can be of three exp  i : 3:3:77
x i2 i1 RT
main types: aqueousorganic, nonpolar organicpolar
organic; aqueousaqueous. Both phases in each case must If the solutions in each phase are dilute enough for i2 1
be immiscible with each other. (See Section 4.1.3 for an and i1 1, then (xi1/xi2) is a constant:
140 Physicochemical basis for separation

where Sim is the solubility coefficient of gas species i, Cim is


10.0
9.0
the concentration of species i in the membrane exposed to
25 C
cm3 CH4 STP/cc. Polymer

8.0 a gas, with the species i partial pressure being pig.


35 C
7.0 45 C The commonly used units of the solubility coefficient
6.0 Sim are (cm3(STP))/(cm3 cm Hg), with Cim and pig
5.0
expressed, respectively, in cm3(STP)/cm3 and cm Hg.
4.0
The gas species is often identified as the penetrant.
3.0
2.0
Generally, the more condensible the gas, the higher the
1.0 solubility coefficient. For a useful correlation of solubility
0 coefficient with the normal boiling point, see Stannett
0 2 4 6 8 10 12 14 16 18 20 22 24 26 28 30 32 34 36
Pressure (atm. abs.) et al. (1979). Sometimes, the solubility coefficient is
also reported as the sorption coefficient. Figure 4.3.4
Figure 3.3.4. Solubility of methane in oriented polystyrene. illustrates the values of the solubility coefficient for a
Reprinted from Journal of Membrane Science, Vol. 1, number of gases and vapors in natural rubber mem-
W. R. Vieth, J. M. Howell and J. H. Hsieh, Dual sorption theory, branes. Section 4.3.3, specifically, equations (4.3.43ac)
pp. 177220. Copyright (1976), with permission from Elsevier. and (4.3.44a), illustrate how the solubility coefficient Sim
depends on the polymer, the temperature as well as the
critical temperature Tci of the gas/vapor species for
amorphous polymers.
x i1 =x i2 N
i1 ; 3:3:78 When the organic polymer is glassy (i.e. the system
temperature is lower than Tg, the polymer glass transition
where N 0
i1 can be determined from ij  s. This behavior is temperature), the solubility behavior of a pure gas
identified as the Nernst distribution law. Even if the species in the membrane is different from Henrys law
amount of solute i, and therefore its mole fraction, is varied (Figure 3.3.4):
in one phase, the ratio of the solute mole fractions in the
C 0 Hi bi P
two phases remains constant. An alternative form is used C im Sim P
1 bi P  3:3:81
for dilute solutions:
C dim C H
im :
C i1 =C i2 i1 ; 3:3:79
At very low pressures, where bi P << 1; C im
where kil is the constant distribution coefficient. Sim C 0 Hi bi P, a linear behavior as in Henrys law.
At very high pressures, where biP >> 1, a linear behav-
ior C im Sim P C 0 Hi is again observed. Figure 3.3.4
3.3.7.3 Gasmembrane equilibrium displays how a low-pressure linear region is connected by
The dissolution of a gas in a nonporous organic polymeric a nonlinear curved behavior to the high-pressure linear
membrane is an example of gasmembrane equilibrium. region for dissolution of CH4 in a polystyrene film (Vieth
Organic polymeric membranes can be amorphous, semi- et al., 1976).
crystalline or crystalline. Crystalline regions are virtually The first term in the above isotherm corresponds to
impenetrable. Therefore practical polymer membranes species dissolving according to Henrys law. The second
are either amorphous or semicrystalline. Semicrystalline term is due to species dissolving according to the Langmuir
polymers are a combination of amorphous and crystalline isotherm (see Section 3.3.7.6). Specifically, the Langmuir
regions. The amorphous region/membrane may be rub- species are assumed to be sorbed in microvoids in the
bery or glassy. The gas molecules dissolve in the amorph- membrane (regions having 0.50.6 nm diameter). The
ous region of the membrane as if it were a liquid. For overall behavior is described as the dual sorption26 mode
permanent gases at temperatures T > Tci (= the critical for two different dissolution modes of gas molecules in the
temperature of gas species i) and nonporous rubbery poly- glassy membrane.
meric membranes, this behavior has been observed as long When a binary gas mixture (i 1, 2) is in equilibrium
as there is no interaction with the membrane material. with a glassy polymeric membrane, the following sorption
Further, under these conditions, Henrys law is obeyed behavior has been suggested (Koros, 1980):
for gases such as H2, He, Ne, Ar, O2, N2, CH4 (Stern
and Frisch, 1981). The form of Henrys law used in gas
membrane equilibrium with the gas phase behaving as an 26
It has been suggested that there is no sharp boundary between
ideal gas is absorption and adsorption. The term sorption was introduced
by J. W. McBain in 1909 to describe situations where both are
C im Sim pig ; 3:3:80 important (Bikerman, 1970).
3.3 Equilibrium separation criteria closed vessel 141

C 0 H1 b1 p1g where we have assumed V i1 V i2 . This relation illustrates


C 1m S1m plg h i; 3:3:82a how the pressure difference between two phases under
1 b1 p1g b2 p2g
osmotic equilibrium can be tied to the composition differ-
C 0 H2 b2 p2g ence between the two solutions. For a one soluteone
C 2m S2m p2g h i: 3:3:82b solvent system with a semipermeable membrane impervi-
1 b2 p2g b1 p1g ous to solute i but permeable to solvent s, the above results
can be reexpressed as follows:
Here, S1m and S2m are the solubility coefficients of those
molecules of species 1 and 2 which dissolve according to as2
V s P 1  P 2 RTn V s 1  2 ; 3:3:86a
Henrys law. The utility of such a mixture sorption iso- as1
therm model has been verified (see, for example, Sanders
et al. (1984) for a mixture of CO2 and C2H4 in poly(methyl where 1 and 2 are the osmotic pressures of the solutions
methacrylate)). in region 1 and region 2, respectively. If 1 > 2, aS2 > aS1;
The gasmembrane equilibrium is basically somewhat further, P1 > P2 for osmotic equilibrium. The osmotic
different if the membrane is made of a metal or an alloy pressure of a dilute solution of small molecules of species
and the gas is diatomic. We have already noted that a i is often calculated from the van t Hoff equation
diatomic gas such as N2, H2 or O2 dissolves in a molten
metal according to Sieverts law, i.e. the mole fraction of i C i RT: 3:3:86b
the gas (in the atomic state) in the molten metal is propor-
Consider now the second type of equilibrium when a
tional to the square root of the gas partial pressure. The
solute partitions between the solution and the membrane,
same equilibrium behavior, relation (3.3.70), is also
which is no longer semipermeable to the solute: the solute
observed between a gas and a solid metallic membrane.
distribution is somewhat similar to that in liquidliquid
It is therefore necessary to assume that the gas is present in
equilibrium of the type (3.3.78) or (3.3.79). Solute parti-
the atomic state while developing any relation for mem-
tions into the membrane, which is assumed to be nonpor-
brane transport of diatomic gases.
ous, and is dissolved in it as if the membrane were a liquid.
For low solute concentrations in the membrane and
3.3.7.4 Liquidmembrane equilibrium and liquidporous no swelling, the following distribution equilibria generally
sorbent equilibrium hold:
There are two aspects to liquidmembrane equilibrium. C im =C il im ; i 1; . . . ;n: 3:3:87
The first one is concerned with the osmotic equilibrium
between two solutions on two sides of a semipermeable If the membrane happens to be porous (or microporous)
membrane permeable to the solvent and impermeable to and uncharged, the nature of the liquidmembrane equi-
the solute; the second one covers partitioning of the solute librium will be determined by the relative size of the solute
between the solution and the membrane. Both porous and molecules with respect to the pore dimensions in
nonporous membranes are of interest. The second aspect the absence of any specific solutepore wall interaction.
is also useful for porous sorbent/gel particles. Similar considerations are also valid for liquidporous sor-
In osmotic equilibrium between two regions 1 and 2 bent equilibria. If the solute dimensions are at least two
separated by a semipermeable membrane, the pressures orders of magnitude smaller, then the solute concentration
P1 and P2 of the two regions are usually different (see in the solution in the pore should be essentially equal to
Figure 1.5.1). For the solvent species i transferable between that in the external solution. However, the solute concen-
the two regions, we have, at equilibrium, tration in the porous membrane/porous sorbent/gel will
be less than that in the external solution due to the porosity
i1 P 1 ;T;x i1 i2 P 2 ;T;x i2 3:3:83 effect. Assuming that the solute exists only in the pores of
from relation (3.3.16). Using expression (3.3.21) for ij (Pj,T, the membrane/porous sorbent with a porosity m, the
xij), we get value of kim should be equal to the membrane or sorbent
porosity m if the solute characteristic dimensions are
0i1 P 0 ;T V i1 P 1  P 0 RTn ai1 at least two orders of magnitude smaller than the radius
0i2 P 0 ;T V i2 P 2  P 0 RTn ai2 : 3:3:84 of the pore.
When the solute dimensions are larger and there are
If the pure ith species exists in the same physical state in no specific solutepore wall interactions, the partitioning
both phases at P0 and T, then effect is indicated by a geometrical partitioning factor for a
cylindrical pore as
ai2
0i1 P 0 ;T 0i2 P 0 ;T ) V i1 P 1  P 2 RTn ;  2
ai1 p
C im ri
3:3:85 1 ; 3:3:88a
C il rp
142 Physicochemical basis for separation

Center line of cylindrical pore Liquid-filled membrane pore

2rp

Membrane wall

ri
ri
ri (rp ri)
Molecules of radius ri

Feed liquid

Figure 3.3.5A. Geometrical partitioning effect: only a distance of r p  r i from the pore center is available for locating the center of the
molecules of the solute.

p
where C im is the concentration of solute i in the mem- kim is the ratio of partition functions for molecules within
brane/sorbent pore liquid, rp is the pore radius and ri is the the pores and within the bulk liquid:
radius of the solute molecule (assumed spherical). The  
center of the solute molecule cannot approach the wall exp enp r; ;=kB T dr d d
im   : 3:3:89c
beyond a radius of rp ri (Figure 3.3.5A). Thus only the exp enb =kB T dr d d
volume fraction r p  r i 2 = r 2p of the pore can have solute
Here the coordinates r, and describe the molecular
molecules at the same concentration as the external solu-
position, orientation and conformation, respectively. The
tion, Cil; yet the solvent molecules exist effectively through-
energy enp in the porous configuration consists of the enM
out the pore, making the pore liquid (total volume r 2p  1
due to intramolecular interactions, an energy enMN due to
for unit length) leaner in solute molecules compared to the
intermolecular interactions and an energy enMp due to
liquid phase concentration Cil outside the pores:
interactions between the macromolecule and the pore;
p
C im r 2p  1 C il r p  r i 2  1: 3:3:88b therefore
enp enM enMN enMP : 3:3:89d
This was originally suggested by Ferry (1936) as a reduc-
tion in cross-sectional area for diffusion. Giddings et al. Correspondingly, for the bulk liquid,
(1968) have developed the same result theoretically by
enb enM enMN : 3:3:89e
considering the limitations in orientations and structural
configurations of macromolecules to avoid sterical overlap If we choose the condition of infinite dilution, enMN will
with the pore wall. If the pore has the shape of a slit ( 1), be zero since the macromolecules are far apart. We fur-
cylinder ( 2) or a cone ( 3), the geometrical parti- ther assume that essentially there is only one conform-
tioning factor is given by ation of the molecule/macromolecule (rigid) (alternatively
  all conformations have the same energy); therefore,
p ri
im C im =C il 1  ; 3:3:89a enM 0. Further, we assume that there are no adsorptive
rp
forces between the macromolecule and the pore wall; in
where rp is the characteristic pore dimension. See Colton addition, the pore wall and macromolecule are distinct
et al. (1975) for a brief review of the relevant literature. In and discontinuous: consequently, exp(enMP/kBT) has the
most porous media being modeled as a collection of cylin- value of 1 for molecular configurations free from overlap
drical capillaries, most pores are unlikely to be truly cylin- with the wall and the value of 0 for configurations of
drical. Therefore, rp may be replaced by (2/s), where s is overlap with the wall. In random-pore networks, an
the surface area of the wall of the capillary per unit pore ensemble average of exp(enMP/kBT), q (r, , ), can
volume: replace it:

 r i s 2 qr; ; dr d d
im 1  : 3:3:89b im : 3:3:89f
2 dr d d

We will illustrate very briefly now a fundamental approach This quantity q(r, , ) may be defined in general as the
to calculating the partitioning factor of (3.3.88a) as probability that a molecule having a given configuration
developed by Giddings et al. (1968). In the formalism of does not intersect a pore wall. If we define a local equilib-
statistical mechanics, the equilibrium partition constant rium partition constant ki,loc(r) such that
3.3 Equilibrium separation criteria closed vessel 143

qr; ; d d Since the solute molecule is in a solution, an alterna-


i;loc r ; 3:3:89g tive estimate of ri is provided by the StokesEinstein
d d
equation:
where i,loc(r) depends only on the position coordinate r in
r i kB T=6D0is ; 3:3:90c
the pore, then
B ~ D0is is the dif-
where k is Boltzmanns constant ( R=N),
i;loc r dr fusivity of solute i in the solvent and is the solvent
im : 3:3:89h viscosity. (This ri is the same hydrodynamic radius dis-
dr cussed in Section (3.1.3.2).) This solute radius based on
the StokesEinstein equation and identified therefore as
For rigid macromolecules/molecules, dependence on the hydrodynamic radius has been correlated with the
disappears, and we get molecular weight of globular proteins by (Tanford et al.,
1974)
i;loc r dr
qr; dr d
im : 3:3:89i logr i i log M i  0:147  0:041; 3:3:90d
dr d dr 3
where the recommended value of (1/i) is 0.74 cm /g. For a
macromolecule in general, whether it can enter a given
To illustrate, consider spherical molecules of radius ri and
pore of certain size or not depends on its radius of gyration
pores of radius rp in a rigid medium. For pores of infinite
rg, which depends on the molecular weight Mi and the
length, it is obvious that, for radial locations r of the center
macromolecular shape 1 via
for the molecule,
r g / M i 1 : 3:3:90e
0
r
r p  r i ; im 1; 3:3:89j
For rod-shaped molecules, 1 1, for spheres, 1 (1/3),
r p  r i
r
r p ; im 0: 3:3:89k
whereas for flexible chains, 1 (1/2). The radius of gyr-
Use now equation (3.3.89h); it is now the ratio of two ation is proportional to the hydrodynamic viscosity based
circular areas, one with the diameter (2(rp  ri)) corres- radius rh determined from the intrinsic viscosity of the
ponding to (3.3.89j) in the numerator and the denominator macrosolute via the hydrodynamic volume Vh (Ladisch,
corresponding to the pore area of diameter 2rp: 2001, pp. 557561):
   1=3
4r p  r i 2 =4 ri 2 3Vh  M i
im 1  ; 3:3:89l rh ; Vh : 3:3:90f
4 r 2p =4 rp 4 ~ sh
N

which is the relation (3.3.88a) described earlier. Here [] is the intrinsic viscosity and sh is a shape factor
The radius, ri cm, of the solute molecule (which is having the value of 2.5 for spheres; sh is greater than 2.5
assumed spherical) needs to be known in liquidporous for ellipsoids. For compact globular proteins,
membrane/porous sorbent equilibrium. For relatively
r h 0:718M i 1=3 ; 3:3:90g
small molecular species, the specific molar volume of the
solute will lead to which is very close to the estimate of ri by equation
 1=3 (3.3.90b) for 1=i 0:75 cm3 =g. Hagel (1989) has
3M i
ri ; 3:3:90a indicated that the estimates of macromolecule/protein
~ i
4N dimensions via the hydrodynamic radius, ri, and the
where Mi is the molecular weight, i is the solute density hydrodynamic viscosity based radius, rh, follow the
~ is Avogadros number (6.022  1023). Spriggs and Li
and N relation
(1976) have therefore suggested that, for solids, rh > ri : 3:3:90h
8 1=3
2r i 1:465  10 M i =i ; 3:3:90b The porous medium (sorbent, membrane, gel particle) can
be made of soft spherical particles that swell quite a bit
where i is in g/cm3 and ri is in cm, whereas, for liquids, when immersed in a solvent; they are called gels. If they
swell in water, they are called hydrogels; however, the
2r i 108 V im 1=3 ;
polymers are crosslinked so that they retain their overall
where Vim is the molecular volume at the normal boiling structure. Of course, the polymers are soft and compress
point (MVNBP) in cm3 (Reid et al., 1977). Expression under pressure; a typical example would be gels (which
(3.3.90a) is also used for determining ri values for larger are crosslinked) based on agarose (Figure 3.3.5B). Such
molecules, like globular proteins, where one usually assumes gel particles are widely used in a variety of separation
that 1=i 0:75 cm3 =g is the partial specific volume. techniques for separating macromolecules/proteins, etc.
144 Physicochemical basis for separation

50

40
Liquid

Temperature, C
lin
30 g
zin
r ee
F
e
lin
20 ing
elt
M
Solid
10

0
0 0.2 0.4 0.6 0.8 1.0
Figure 3.3.5B. Microporous polymer in the form of a gel particle
formed by crosslinking linear chains of monomer; crosslinks are x-naphthol
shown by heavy lines.
Figure 3.3.6A. Phase diagram for the system naphthalene/-
naphthol. (J. M. Prausnitz, Molecular Thermodynamics of
Various empirical expressions have been developed that
Fluid-Phase Equilibria, 1st edn. 1969, pp. 404. Reprinted
describe the partition coefficient kim of solute i as a func-
by permission of Pearson Education, Inc., Upper Saddle
tion of the molecular weight Mi or hydrodynamic radius ri. River, NJ.)
A few are illustrated in the following:

im 1=3 a  b M i 1=2 ; im a b log M i ; known as the solidus and the upper curve is known as the
im a br i : 3:3:90i liquidus. The system is completely liquid above the liqui-
dus and is completely solid below the solidus. In between,
(For more details, see Hagel (1989); for an introduction in a both liquid and solid phases exist. At any temperature, the
gel permeation chromatography context, see Ladisch solid and liquid phases at equilibrium have different com-
(2001).) If the molecule cannot enter the pore, kim 0. positions. Not all solid solutions span the whole compos-
From the relation (3.3.90e), for a given molecular weight ition. For example, in ironcopper alloys around 1000  C, a
Mi, the radius of gyration rg is the highest for a rod-shaped homogeneous solid solution exists only below a compos-
molecule and lowest for a sphere. Therefore, for a given ition of 10 wt% copper (Darken and Gurry, 1953).
molecular weight, a rod-shaped molecule has the lowest To determine the equilibrium ratio of species i between
kim and a spherical molecule the highest kim. the melt (j 1 l) and the solid solution ( j 2 s), note
that, at equilibrium,
3.3.7.5 Liquidsolid equilibria: leaching, crystallization, ^f ^f ) ^f ^f
i1 i2 il is 3:3:91
precipitation
But
Liquidsolid equilibria in which a liquid and a solid phase
(or solid phases) are coexisting can be of the following ^f x il f 0 and ^f is is x is f is0 ; 3:3:92
il il il
types:
leading to
(1) a solid solution in equilibrium with a molten mixture;
(2) a solute being leached from a solid mixture by a x il x i1 f0 f0
K i is is0 jideal is0 : 3:3:93
solvent; x is x i2 il f il f il
(3) crystallization equilibria between a crystal and the
The final simplification is valid only for ideal behavior. If
mother liquor.
experimental data on Ki are not available, the ratio f 0is =f 0il
When components 1 and 2 are miscible in the solid phase has to be calculated by procedures that are elaborate and
over the whole composition range between pure 1 and complicated (Prausnitz, 1969). If the behavior is nonideal,
pure 2, we have a solid solution. For example, Figure the liquidus and solidus behavior shown in Figure 3.3.6A is
3.3.6A shows the temperature vs. composition diagram changed to more complicated ones similar to maximum or
for a naphthalene/-naphthol system. The lower curve is minimum boiling azeotropes (see Section 4.1.2).
3.3 Equilibrium separation criteria closed vessel 145

Crystals present So there are three regions of importance in crystalliza-


Metastable tion (Miers, 1927): an unsaturated region (with no crys-
Supersaturated p2 B region
C A tals), a supersaturated region (with crystals) and an
solution
p3 A
Solubility curve
intermediate region, where crystal growth will occur if
A
there are crystals, but there will be no nucleation (i.e.
Concentration

B A
Csat
p1 f crystal formation without any crystals). This intermediate
Unsaturated region is also called the metastable region. A feed solution
solution
represented by f in the unsaturated region can produce
crystals if (1) it is cooled to the supersaturated region p1,
(2) the solvent is evaporated to reach the supersaturated
region p2 at constant T, or (3) a combination of cooling
T2 Tsat T1 and evaporation is implemented to reach p3. (An add-
Temperature
itional method employed to attain supersaturation
involves the addition of an antisolvent). It is now believed
Figure 3.3.6B. Concentration vs. temperature behavior for satur- that there are a number of other factors such as solution
ation, supersaturation and crystallization. cooling rate, mechanical shock or friction which influence
the locations of the curves. True equilibrium is rarely
attained in crystallization processes.
Consider now the leaching of a solid mixture (which Consider point A on the solubility curve (Figure 3.3.6B).
can be treated as a collection of aggregates of pure com- The saturated solution concentration of solute at tempera-
ponents) by a solvent. In the solid phase, each component ture Tsat is Csat. When this solution loses some solvent at
fugacity is then equal to its pure component solid fugacity the same temperature T and reaches the curve for the
f 0is . Let the leaching solvent be insoluble in the solid mix- supersaturated solution (point B), the solution concentra-
ture. At equilibrium between the solid mixture and the tion becomes C > Csat. If the solution were saturated at this
solvent, for species i, higher concentration C, its temperature would have to be
higher, namely T1, corresponding to point A0 on the solu-
f is f is0 ^f il il x il f il0 : 3:3:94 bility curve. At temperature Tsat, the molar supersaturation
C is defined as
So
C C  C sat : 3:3:98a
x il f 0is =il f il0 : 3:3:95
The fractional supersaturation, or relative supersaturation,
If the solution temperature is lower than the melting point s, is defined as
of solute i (which is usually the case), and if vapor pres-
sures could be substituted for f is0 and f il0 , we find C C  C sat C
s  1; 3:3:98b
C sat C sat C sat
P vap
is
x il : 3:3:96 whereas the supersaturation ratio S is defined as
il P vap
il
C
If the system temperature is the normal melting tempera- S 1 s: 3:3:98c
C sat
ture of the solid, then f is0 f il0 and
If instead of removing the solvent at Tsat, we lower the
x il 1=il : 3:3:97 0
solution temperature and reach point B at T2 (<Tsat), the
Refer to Prausnitz et al. (1999) for a more comprehensive saturated solution at A will become supercooled; the solu-
treatment on standard state fugacity calculations in solid tion will also become supersaturated for the temperature
liquid systems. T2. The solution concentration remains the same as that of
At any given temperature and pressure, a solvent has point A, but the supercooling undergone (leading to super-
a certain capacity for a solute. When the limit is reached, saturation) may be identified by
the solution is said to be saturated. This limit generally T T sat  T 2 : 3:3:98d
increases with temperature and provides the solubility
curve (Figure 3.3.6B). Conversely, if a solution is cooled The fractional supercooling may be defined by
to a temperature below the saturation temperature, there
T=T sat 1  T 2 =T sat : 3:3:98e
would be a tendency for the solute to come out of
0
the solution. The solute may crystallize or precipitate. The distance AB (or AB) provides an estimate of the meta-
In reality, crystallization does not occur till the solution stable zone width. Table 3.3.3 illustrates the metastable
gets supersaturated to some extent, as shown in zone width for a number of salts in terms of T values for
Figure 3.3.6B. three different cooling rates (Nvlt et al., 1985).
146 Physicochemical basis for separation

Table 3.3.3. Metastable zone width in crystallization of various A1 A1 , A2


saltsa A2 A1 , A3
3:3:99

Equilibrium Maximum supercooling Am1 A1 , Am
temperature, prior to nucleation
Salt Tsat ( C) The subscript indicates the number of kinetic units in the
2  C/hr 5  C/hr 20  C/hr cluster. With an increase of m, recognizable clusters
become embryos and a few of them grow enough to be a
Ba(NO3)2 30.8 1.65 2.17 3.27
nucleus (for liquid water, the number m is about 76 for a
CuSO4 5H2O 33.6 5.37 6.82 9.77
FeSO47H2O 30.0 0.89 1.21 1.90 nucleus). Although such nuclei are not in a stable equilib-
KBr 61.0 1.69 2.41 4.11 rium (some nuclei become embryos via loss of kinetic
KCl 29.8 1.62 1.86 2.30 units), some are able to gather kinetic units, attain a
NaBr2H2O 30.6 4.6 6.97 13.08 critical size rc and become a crystal which will be in stable
a equilibrium with a saturated solution.
Data from Nyvlt et al. (1985).
To determine this critical size, rc, we have to have a
situation where the Gibbs free energy change, G, as a
00 function of increase of nucleus radius r is negative. The
Focus now on two points A0 and A on the vertical line Gibbs free energy change, G, is needed to bring kinetic
AB (see Figure 3.36B) at temperature Tsat. The solution is units together and expel the solvent; the G change
0 00
obviously less supersaturated at A than at A . If, in fact, we needed to form a nucleus of radius r has two contributions:
0 00
take the solutions at A and A and allow them to sit without (1) the energy needed to form the surface of the nucleus,
doing anything, namely stirring, etc., at temperature Tsat, 4 r2 12, where 12 is an interfacial tension (dyne/cm) or
we will observe that crystals may appear much sooner in interfacial energy/area (ergs/cm2); (2) the energy needed
00
the solution A having a higher fractional supersaturation. to build the crystal of volume (4/3)r3. This second energy
Further, if a crystal is introduced into any of these solutions term consists of the product of the molar chemical poten-
0 00
(e.g. at A , A or B), we will observe that the crystal will tial difference  ij (between the chemical potential of
grow. To understand these phenomena better, consider the crystal in equilibrium with a saturated solution and
the following brief introduction to the thermodynamics of the chemical potential of the solute i in a supersaturated
nucleation and growth processes in crystallization. How- solution) and the number of gmols in the crystal mass of
ever, first, a brief discussion on different mechanisms of volume (4/3)r3, namely 4=3 r 3 =V ic :
nucleation is useful.
 
When nucleation takes place without any crystal sur- G 4 r 2 12  ij  4=3 r 3 =V ic : 3:3:100a
faces, we have primary nucleation. Primary nucleation is
said to occur by homogeneous nucleation when no dis-
When thermodynamic equilibrium is reached and the
solved impurities are present. When primary nucleation
nucleus becomes stable, the nucleus dimension is called
occurs due to the presence of dissolved impurities, we
the critical size, rc.
encounter heterogeneous nucleation. When nuclei are
To obtain this value rc achieved when dG=dr 0,
formed due to the presence of existing macroscopic
we get, after differentiation,
crystals, interaction with the crystallizer wall, rotary
impellers, fluid shear, etc., we have secondary nucle- r c 212 V ic =ij  : 3:3:100b
ation. Mechanisms of secondary nucleation are not suf-
ficiently clear. An introduction to theories on secondary Expressing ij in terms of fugacities, we obtain
nucleation is provided by Myerson (1993). Here we will !
f ij 212 V ic
focus on homogeneous nucleation. Note that homoge- n : 3:3:100c
neous nucleation is rarely achieved or desired in prac- f ijjsat RT r c
tical crystallization (McCabe and Smith, 1976; Myerson,
Assuming that the activity coefficients of a nonelectrolytic
1993).
solute i in the two solutions ( fij, supersaturated; fij|sat, sat-
In the process of homogeneous nucleation taking
urated) are nearly equal, we may replace the fugacity ratio
place without any mechanical shock, friction, rapid
by a concentration ratio, i.e. f ij =f ij jsat C ij =C sat ;
cooling, etc., molecules, atoms or ions join together to
form a crystal nuclei called a kinetic unit (McCabe and n f ij =f ij jsat n C ij =C sat
Smith, 1976). Individual kinetic units whose volumes are
2 12 V ic
of the order of 10 nm3 can come together or collide; n S n 1 s : 3:3:101
RT r c
combinations of a few such kinetic units lead to a cluster
formation in the manner of a chain reaction (an addition Beyond this critical size rc, where G achieves a critical
mechanism): value,
3.3 Equilibrium separation criteria closed vessel 147

Gcrit 4 r 2c 12  8=3 12 r 2c 4=3 r 2c 12 ; 3:3:102 dioxide, etc.) considered for and produced by precipitation
processes, excellent introductions are available in Nyvlt
the value of G starts decreasing as r increases beyond rc. et al. (1985) as well as in Estrin (1993).
Thus the crystal nucleus has become stable and the add- A major aspect of precipitation, however, does not
ition of kinetic units decreases the free energy, making it a involve chemical reactions as such. Instead, they are akin
spontaneous process. These results are useful for under- to the addition of an antisolvent in that somehow the
standing the nucleus formation process. When the super- solubility of the solute is reduced. Specifically, we focus
saturation ratio S increases, the critical size rc decreases, on proteins and macrosolutes, where addition of neutral
and the solution becomes less stable since it can sustain salts or an acid or base to an aqueous solution causes
smaller and smaller nuclei. The result (3.3.101) has precipitation of the proteins or other macrosolutes. The
another dimension. Since fij or Cij refers to the supersatur- reduction in solubility of the protein by the addition of
ated solution, one can argue also that the smaller the neutral salts is generally due to a hydrophobic effect (see
crystal size, the higher its solubility compared to the value Section 4.1.9.4). A protein molecule in an aqueous solution
Csat for a large crystal or a flat surface, an example of the has a hydration layer around it in the local domain (see
Kelvin equation (3.3.53) already studied. This leads to Section 3.3.5). Such a layer prevents the hydrophobic
dissolution of smaller particles and growth of larger par- patches on a protein molecules surface (which consists
ticles, a phenomenon called Ostwald ripening. For an of hydrophilic as well as hydrophobic patches; for example,
introduction to these subjects, see Mullin (1972), McCabe nonpolar amino acids such as alanine, methionine, trypto-
and Smith (1976), Randolph and Larson (1988) and Myer- phan and phenylalanine, if present in a protein, will pro-
son (1993). vide hydrophobic patches with polar amino acids creating
The phenomenon of supersaturation illustrated in hydrophilic regions) from interacting with each other and
Figure 3.3.6B was created either by solvent evaporation or forming aggregates leading to precipitation. Addition of
decreasing the temperature. In general, supersaturation neutral salt molecules into water (for example, (NH4)2SO4)
may be developed via the following routes: (1) solvent leads to the spontaneous withdrawal of water molecules
evaporation; (2) temperature decrease (or increase in from the proteins local domain to hydration of individual
some cases); (3) addition of antisolvent; (4) precipitation salt molecules. This leads to aggregation of protein mol-
via chemical reaction. We have already considered the ecules (via hydrophobic interaction) and their precipita-
results of solvent evaporation and temperature decrease tion (salting-out effect).
in Figure 3.3.6B. For systems where solubility decreases The protein solubility C2 (in g/liter) decreases as a
with decreasing temperature (as in Figure 3.3.6B), logarithmic function of the solutions ionic strength (Cohn
evaporation-led cooling by applying vacuum (pathway and Ferry, 1943):
fp3) is also practiced with or without some heat addition.
In case such steps are not practical, an alternative strategy log C 2 log C 2 j0  b C 3 : 3:3:104
involves the addition of another solvent into the solution,
Here C 2 j0 is the hypothetical/extrapolated solubility at zero
which drastically decreases the solute solubility in the
ionic strength (corresponding to C3 0) and C3 is the ionic
mixed solvent environment. The solvent added is usually P
strength of salt in gmol/liter ( 1=2 Z 2i C 2i I, where Ci
miscible, and this process of miscible nonsolvent addition
includes cationic and anionic species). There are some
is practiced quite often both for organic/inorganic solutes
salts that may somewhat increase the solubility (salting-in
as well as amino acids.
effect). Both of these effects are schematically illustrated in
There are many systems where the solubility
Figure 3.3.6C. The logarithmic decrease in the protein
temperature curve is essentially flat. If one can carry out
solubility as a function of the ionic strength is an important
a chemical reaction between the solute and another added
basis for protein separation via precipitation (for an intro-
agent, one of the products may precipitate. For example,
duction, see Ladisch (2001)).
Na2SO4 in solution reacts with CaCl2 in solution (both
having essentially flat concentration vs. temperature
behavior) to produce insoluble CaSO4 (which has a low 3.3.7.6 Interfacial adsorption systems
solubility and which precipitates) and NaCl (which The previous three types of liquidsolid equilibria con-
remains in solution and which has a flat concentration vs. sidered equilibrium between two bulk phases; we will
temperature profile): now briefly look at two types of interfacial adsorption
systems where the two bulk phases are either fluidfluid
Na2 SO4 solution CaCl2 solution or fluidsolid. Consider first the interfacial equilibrium
relation for a nonelectrolytic surface active solute in an air
, 2NaClsolution CaSO4 precipitate: 3:3:103
water system (a fluidfluid system). If the surface active
For many inorganic compounds (BaSO4H2O; silver solute i is such that the interfacial tension 12 decreases
halides; calcium phosphates; alumina trihydrate; titanium linearly with the surfactant concentration Ci1 as
148 Physicochemical basis for separation

12 a1  b1 C i1 3:3:105 b1 dC 11 b1 C 11
E1 ; 3:3:106
RT d n C 11 RT
(a1 and b1 are constants in this linear relation) below the
critical micelle concentration (CMC), then it is possible to a linear equilibrium relation between the surface excess
get an expression for (d12 =d11 ) in relation (3.3.39b) and concentration of solute 1 and its bulk concentration C11 in
relate it to E1 . In the present case, i 1 is the solute and water. The proportionality constant (b1/RT) can be deter-
d11 RT d n a11 RT d n C 11 . Further, d12 b1 dC 11 . mined from the dependence of 12 on the bulk solute
Therefore concentration C11.
Figure 3.3.7(a) shows the general behavior of 12 with
the surfactant solute concentration C11 in the bulk of water
(j 1). Beyond the CMC, 12 is almost constant with respect
to C11; thus d120. Therefore a change in bulk concen-
tration of species 1 no longer changes the excess concen-
tration of solute 1 in the interface, which is saturated with a
monolayer of surfactant molecules. The relation between
Protein concentration

E1 and C11 over the whole range of C11 is then of the type
Salting-in
shown in Figure 3.3.7(b). It is of the Langmuir adsorption
C2 g/liter

isotherm type (see equations (3.3.112a,b)).


0 We focus now on fluidsolid systems. This type of
interfacial adsorption system is encountered, for example,
log(C2/C20) in gassolid adsorption. The solids on which the gas is
adsorbed is called an adsorbent and the gas species
adsorbed is called the adsorbate. More often than not,
Salting-out the adsorbents used in practice are porous solid particles;
these particle types and their uses are identified in Table
7.1.1. The porosity values of such particles are high, in the
C3 gmol/liter
range 0.40.85. The particle sizes can vary from quite
small (e.g. 0.1 mm) to large (e.g. 2.54 cm). The surface
Figure 3.3.6C. Solubility of a protein as a function of salt concen- area of these pores is very high and can range easily from
tration due to the salting-out effect. (After Ladisch (2001).) 1001000 m2/g of adsorbent. The pore sizes can vary from

(a)
72
Surface tension, dyne/cm

(b)
12

40
g

GE
1s

CMC

C11 0.01 0.02

Concentration gmol/liter C11

Figure 3.3.7. (a) Surface tension vs. concentration of a surfactant, sodium lauryl sulfate, in water. (b) Surface adsorption isotherm in a
gasliquid system.
3.3 Equilibrium separation criteria closed vessel 149

large macropores of diameter > 50 nm to micropores


< 2 nm, to pores of molecular dimensions (0.31 nm and
above) in zeolites, which are crystalline aluminosilicates.

Mole fraction benzene in gas phase


The pores are invariably tortuous. Gas/vapor molecules 0.8
are adsorbed on the pore surfaces generally by physical
adsorption, involving relatively weak intermolecular
forces, and rarely via chemisorption, involving bond for- 0.6
mation. The physical adsorption due to these weak forces
is assumed to take place on sites strewn throughout the
pore surface area. 0.4
To understand the functional nature of gassolid equi-
libria for physical adsorption processes, one may develop a
relation between i and the gas phase composition using 0.2
equation (3.3.43b) and the illustrations thereafter. Two
alternate approaches will be illustrated here using the
same basic equation (3.3.43b) and a binary system of
species i 1,2. Recognize that S is the total interfacial 0 0.2 0.4 0.6 0.8 1
area per unitmass ofadsorbent and therefore may be Mole fraction benzene in adsorbed phase
P2
expressed as i1 qi S , where S is the molar surface
area. Therefore (Van Ness, 1969) Figure 3.3.8A. Vapor-adsorbed phase equilibria of benzene and
X2 X2 cyclohexane on activated charcoal at 30  C. : P = 0.27 kPa; : P =
q
S d Pi d ig x i d ig : 3:3:107 1.33 kPa; o: P = 13.3 kPa. Reprinted with permission from A. L.
qi
i1 i1 Myers et al., AIChE J., 28(1), 97 (1982). Copyright [1982] Ameri-
For ideal gas behavior in the gas phase, can Institute of Chemical Engineers (AIChE).

2
X P
S d x i RT d n x ig P; observed data of 2i1 qi . The experimental and numerical
i1 procedures are described in Myers et al. (1982).
S S d x 1 dx 1g x 2 dx 2g The adsorption behavior of benzene in a benzene
d 2 d n P ; cyclohexane mixture for the adsorbent, activated charcoal,
RT X x 1g x 2g
qi RT is shown for a particular temperature in Figure 3.3.8A.
i1 P
Note that 2i1 qi , i.e. the total number of moles adsorbed
3:3:108 on the solid adsorbent surface per unit adsorbent mass,
since x 1 x 2 1. Further, x 1g x 2g 1. It follows, is obtained from the experimental data of mass of gas
therefore, adsorbed per mass of adsorbent using

x 1  x 1g 2
X
S d total mass of gas adsorbed=mass of adsorbent
d n P dx 1g : 3:3:109 qi :
X2 x 1g 1  x 1g i1
M 1 x 1 M 2 x 2
RT qi
i1 3:3:110c

For an ideal gas, this is a form of Gibbs adsorption iso- For real gas mixtures, Ritter and Yang (1989) have
therm for a binary mixture: it relates the species 1 mole developed simple numerical and graphical procedures to
fraction in gas phase, x1g, to species 1 mole fraction, x1, in determine adsorbed phase compositions at elevated
the adsorbed phase, the adsorbate (Van Ness, 1969). At pressures.
constant total gas pressure, we obtain An alternative procedure suggested by Myers and
2
! Prausnitz (1965) based on the molar Gibbs free energy of
S =RT X x 1  x 1g
qi : 3:3:110a mixing provides
x 1g P i1
x 1g 1  x 1g
Px ig P 0i i x i 3:3:111a
At constant gas phase composition, x1g, we get
as the relation between the gas phase mole fraction, xig,
!
S =RT X2 and the adsorbate phase mole fraction, xi, for a total gas
qi =P: 3:3:110b pressure P. Here P 0i is the equilibrium gas phase pres-
P x 1g i1
sure for pure i adsorption at spreading pressure (which
These two first-order partial differential equations can be should be the same for the mixture). Further, the gas phase
solved numerically to express x1 and S =RT as a func- was assumed to behave ideally. If the adsorbate phase also
tion of total pressure P and gas phase composition x1g for behaves ideally, i 1. Under this condition,
150 Physicochemical basis for separation

P 0i x i
x ig ; 3:3:111b
P Langmuir type

i.e., the equilibrium behavior is similar to that of Raoults law


in vaporliquid equilibrium (equation (3.3.64)) if P 0i can be
thought of as the pure adsorbate vapor pressure for compon- BET type

Amount adsorbed
ent i at temperature T and spreading pressure of the mixture.
The procedures for obtaining the gassolid adsorption Freundlich type
isotherms briefly outlined above are based on a solution
thermodynamic Gibbs approach. There are two other
approaches: the potential theory and the Langmuir
approach. The latter approach is based on a dynamic equi-
librium between the rates of adsorption and desorption of
Species pressure
any species from adsorption sites on the solid surface. Since Saturation pressure
most data correlation in separation processes employs the
Langmuir approach (Yang, 1987), with the adsorbate amount Figure 3.3.8B. Typical gassolid adsorption isotherms.
expressed as a function of species partial pressure, such
isotherm types will be briefly identified here. For a compre- For a mixture of n species,
hensive introduction to adsorption isotherms and phenom- bif pig 1=i
ena, consult Ruthven (1984) and Yang (1987). i
Xn : 3:3:112d
Consider a few types of isotherms observed in prac- 1 bif pig 1=i
tice (Figure 3.3.8B); the first one is the common i1

Langmuir type. For such an isotherm, initially the The similarities between the Langmuir gassolid isotherm
amount adsorbed is linearly proportional to the gas pres- for a pure gas or a mixture and the Langmuir component
sure P. After further increases in gas pressure, the of gasmembrane equilibrium in equation (3.3.81) or
amount adsorbed approaches a limit, which does not (3.3.82a) should be obvious.
change with further increases in gas pressure. Under The BET (BrunauerEmmettTeller)-type isotherm in
these conditions, all adsorption sites become covered Figure 3.3.8B reflects multilayer adsorption of the adsorb-
with gas molecules, the adsorbate, in a monolayer fash- ate. After a monolayer adsorbate coverage is achieved in
ion. The fraction of sites occupied, or the fraction of the the adsorbent pores, additional molecular layers are
maximum gas that can be adsorbed under other condi- formed on top of the adsorbed monolayer by condensation
tions, is given, for a pure gas i, by of vapors. In adsorbents having small-diameter pores, mul-
tilayer condensation of the adsorbate vapor can fill the
qi bil P pore completely with the liquid adsorbate. This phenom-
; 3:3:112a
qi max 1 bil P enon is called capillary condensation (Figure 3.3.8C). Con-
sider the curved interface between the vapor phase and the
where 1=bil is the Langmuir constant for species i. For a condensed phase of species i in the micropore in this
gas mixture of n species, t is the total fraction of sites figure. The vapor pressure of the condensed liquid above
Xn
the concave curved liquid surface in the capillary P sat
i;curved is
covered by all species and is equal to i ; where i is
i1 less than that over a plane condensed liquid surface P sat i;Pl :
the fraction of sites covered by species i:  12 
2 V ij cos
P sat sat
i;curved =P i; Pl exp  3:3:112e
bil pig R Tr
i n : 3:3:112b
X
1 bil pig (see also relations (3.3.50)(3.3.53)). This equation is also
i1 identified as the Kelvin equation (compare (3.3.54)). As a
result, capillary condensation or pore condensation in a
An additional isotherm type is the Freundlich isotherm. For
fine capillary or pore can take place at a lower value
a pure gas (species i), the Freundlich isotherm has a power
of equilibrium vapor pressure than the saturation vapor
law relation
pressure P sat
i;Pl at that temperature (so far we have used

bif P 1=i ; 3:3:112c this quantity as P sati everywhere). The magnitude of this
effect may be illustrated for pore condensation of benzene
where i < 1, 1 or > 1. Figure 3.3.8B illustrates the case (Ruthven, 1984) (where 12 for benzene 29 dyne/cm;
corresponding to i < 1. A log-log plot of the data satisfy- V ij V benzene 89 cm3 =gmol; T 293 K; 0) for two
ing this type of isotherm will yield from the slope 1=i pore sizes, r 5 nm and 60 nm: the (P sat sat
i;curved =P i; Pl ) value is
and bif from the intercept. 0.67 and 0.96, respectively, for 5 and 60 nm pore size. The
3.3 Equilibrium separation criteria closed vessel 151

q Surface
r tension
A p

Monolayer adsorbate Adsorbate in two layers Capillary condensation

Figure 3.3.8C. Capillary condensation of adsorbate in the adsorbent micropore.

Kelvin effect becomes important for fine capillaries/ the sites occupied by the solute molecules. This isotherm
small pores. relating xi1 to ai2 (or xi2) is the equivalent of the Lang-
Another important case of a fluidsolid adsorption muir adsorption isotherm (3.3.112b) for liquidsolid
system is liquidsolid adsorption. The general principles adsorption.
of adsorption equilibrium between a vapor and an adsorb- Often this isotherm is also expressed in terms of the
ent are also applicable to liquidsolid adsorption, except molar solute concentration in the liquid phase, Ci2, and the
the adsorbate concentrations are often quite high for a moles of solute i adsorbed per unit mass of adsorbent, qil :
liquid phase unlike that in gassolid adsorption (unless
ei C i2
trace concentrations of solute are being adsorbed). qi1 qi1 jmax ; 3:3:113d
Adsorption equilibrium may be viewed here in two differ- 1 ei C i2
ent ways. In the more conventional approach, both solute where qi1 jmax is the maximum value of qi1 for complete
in a solution as well as the solvent molecules get adsorbed coverage of all sites by the solute species i and ei is a
on the solid adsorbent. Thus the exchange equilibrium is constant for solute i. Sometimes, the isotherm is similar
between a solute molecule and a solvent molecule in both to the Freundlich isotherm (3.3.112d) and is represented as
phases. An ith species solute molecule in a solution ( j 2)
displaces a solvent molecule (s) adsorbed on any site in the ei C i2 1=i : 3:3:113e
adsorbent ( j 1) and occupies the adsorbent site:
The graphical method for determining parameters of this
i j 2 s j 1 , i j 1 s j 2: 3:3:113a isotherm has been described in the text following
If the equilibrium constant for this exchange is K, and if the (3.3.112c).
activities of solute i and solvent s molecules in the In an alternative approach, the adsorbent surface is
adsorbed phase are assumed ideal (i.e. ail xil and as1 assumed to have a distinct layer of solvent (in a mono-
xs1), then layer); the solute i merely partitions between the solvent in
solution and the solvent in the monolayer on the
ai1 as2 x i1 as2 adsorbent:
K : 3:3:113b
ai2 as1 ai2 x s1
i j 2 , i j 1: 3:3:114a
Further, in the adsorbed phase, x il x s1 1. Therefore
  If the solute i is such that an infinite dilution standard state
K
    ai2 is necessary, then, from relation (3.3.79), we get
K K as2
x i1 ai2 x s1 ai2 1  x i1 ) x i1   :
as2 as2 K C i1
1 ai2 i1 :
as2 C i2
3:3:113c
This relation, where Ci1 and Ci2 have the units of gmol/
Under ideal conditions of almost equal sized solute and cm3 of adsorbent and gmol/cm3 of bulk solution, may be
solvent molecules and fixed surface area per adsorption expressed in terms of qi1, gmol/mass of adsorbent, and
site on the adsorbent, xi1is equivalent to the fraction of Ci2 as
152 Physicochemical basis for separation

qi1 ^ a;
il V 3:3:114b Cation exchangers : - SO -3, -COO -, -PO 32-, -AsO 32-
C i2
where V ^ a is the volume of the adsorbed monolayer phase
per unit weight of adsorbent (cm3/mass of adsorbent). Anion exchangers : - NH +3 , NH +2 , N+, S+
When a hydrophobic porous adsorbent is in contact
with an aqueous solution which does not wet the adsorb-
ent pores, a hybrid system is formed. Inside the pores Figure 3.3.9. Ionic groups in the polymeric matrix of ion exchange
containing air, there are solutes in vapor form which are resins.
adsorbed on the surfaces in the pore of the adsorbent
(Rixey, 1987). Outside the pores, the aqueous solution
exists. Only solutes that are volatile are adsorbed on the are called cation exchange resins; they are acidic. Resins
adsorbent. Solutes that would not be separated by conven- having fixed positive charges, e.g. NH
3 ;  N CH3 3 ,

tional adsorption in the wetted state may be fractionated etc., exchange anions from an external solution. They are
now. A liquidsolid adsorption system becomes a de facto called anion exchange resins; they are basic in nature.
vaporsolid adsorption system. Consult Helfferich (1995) for a detailed account.
The governing criterion for equilibrium distribution of
a solute between an ion exchange resin and an external
3.3.7.7 Ion exchange systems solution is relation (3.3.8) when the solute is a weak elec-
Ion exchangers are porous insoluble solid materials having trolyte or a nonelectrolyte. The nature of the equilibrium is
fixed charges.27 When immersed in an electrolytic solution, similar to that with nonionic adsorbents. When the solute
ions having charges opposite to the fixed charges, is a strong electrolyte, criterion (3.3.29) has to be used due
counterions, will enter the porous structure; the ion to the presence of fixed ionic charges and counterions in
exchanger as a whole is electrically neutral. If such an ion the resin.
exchange material in particulate form is immersed in The distribution equilibrium of a weak electrolyte or a
another solution containing a different set or type of coun- nonelectrolytic solute i between an external solution (w)
terions, there will be an exchange of the counterions (generally an aqueous solution) and the ion exchange resin
already present in the solution in the ion exchangers particles ( j R) is usually described by a distribution
porous structure with those present in the external solu- coefficient:
tion; hence, the characterization of such systems as ion C iR
exchange systems. The process is reversible. Porous nat- iR : 3:3:115
C iw
ural and synthetic insoluble solid materials can act as ion
exchangers. These include natural ion exchange minerals Often, the concentration C iR is described in terms of
(primarily of the alumino-silicate type), synthetic inorganic moles of species i per unit weight of ion exchange resin
ion exchangers (e.g. zeolites, zirconium phosphates, etc.), particle or per unit weight of solvent in the resin particle. In
ion exchange coals having weak carboxylic acids and, most order to make iR dimensionless, C iw should have similar
importantly, ion exchange resins prepared from organic units. Sometimes iR is constant; for example, acetic acid
polymers. has a linear isotherm in styrene-type cation exchangers
Synthetic ion exchange resin particles consist of a (Helfferich, 1962, p. 126). In general, there are a variety of
three-dimensional crosslinked porous polymeric structure interactions between the solute and the resin system,
having a high porosity and pore dimensions exceeding leading to a complex behavior.
10 nm. The basic polymeric material in such resins is Distribution equilibria of strong electrolytes between a
hydrophobic. Hydrophilic ionic groups (see Figure 3.3.9 solution and ion exchange resin will now be considered.
for an illustration of such groups) are present at a high First, however, certain unusual aspects of ion exchange
density throughout this polymeric network. Although the systems have to be illustrated. Specifically, the phenom-
introduction of such ionic groups, e.g. SO enon of swelling pressure in a resin particle is important.
3 H ; makes the
polymer soluble in water, crosslinking of different polymer When water or a polar solvent enters ion exchange resin
chains in the structure makes them insoluble in water. particle pores, it has a tendency to dissolve the resin
A common polymer is linear polystyrene; divinyl benzene material. The resin swells due to the incorporation of solv-
is employed to prepare a crosslinked polymeric matrix. ent molecules. The resin particle is, however, prevented
Resins having fixed negative group, e.g. SO  from dissolution in the solvent due to crosslinks between
3 , COO ,
etc., can exchange cations from an external solution. These the neighboring polymer chains. This essentially implies
the presence of elastic forces of resin matrix the whole
resin phase including the solvent is now at a higher
27
Liquid ion exchangers are not considered here; see Section pressure higher than that of the surrounding solution.
5.2.2.4. The pore liquid pressure PR, the pressure in the resin
3.3 Equilibrium separation criteria closed vessel 153

Counterions Matrix with fixed charges

Co-ions

Figure 3.3.10. Structure of an ion exchange resin (schematic). (After Helfferich (1962, 1995).)

 
phase, can be related to the external solution (say aqueous) 1 aiw
R  w RT n  V i P R  P w ; 3:3:118b
pressure Pw using criterion (3.3.16) for equilibrium. ZiF aiR
Suppose the external solution is just pure solvent (i s).
using the assumption that the two solute partial molal
Then
volumes V iw and V iR are equal to V i . This equilibrium
sR sw ; 3:3:116a relation is valid for any ionic species i; note that (PR Pw)
is the swelling pressure of the resin and (R w) is the
where j R is the resin phase and j w is the external
Donnan potential, independent of the species i in a given
phase. From expression (3.3.21) for the chemical potential
system.
ij P j ;T;x ij , where j R, i s, we have
An important feature of relation (3.3.118b) needs to be
sR P R ;T ;x sR 0s V sR P R  P 0 RT n asR : 3:3:116b reemphasized: it is valid for any ionic species i. The elec-
trolyte or solute i in general is not just one ionic species.
Similarly,
Thus the distribution of a strong electrolytic solute AY
sw P w ;T;x sw 0s V sw P w  P 0 RT n asw : (which produces A and Y ions) between the solution
3:3:116c and the resin needs to be known. Let the fixed charge in
the resin be negative (Figure 3.3.10). Then A is a counter-
0
Assume standard state pressure P and external solution ion. Suppose one mole of electrolyte AY dissociates to give
pressure Pw to be the same. Since there is pure solvent in A moles of ions A and y moles of ions Y  ; we can follow
the external solution, asw 1. Therefore Helfferichs (1962) treatment:

sR P R ;T;x sR 0s V sR P R  P 0 RT nasR sw P 0 ;T;1 Z A A Z Y Y : 3:3:119a


0s V sw  0 0 0s ;

so that The partial molar volume of the electrolyte AY is


0
V sR P R  P RT n asR ; 3:3:117a V AY A V A Y V Y : 3:3:119b
0
where (PR  P ) is the swelling pressure in the resin phase Use relation (3.3.118b) separately for i A and i Y and
and is related to the solvent activity asR in the resin phase. simplify to get
If asw 6 1 and Pw 6 P0, then, for V sw V sR ,    
aAw A aYw Y
V sw P R  P w RT n asR =asw : 3:3:117b RT n PR  P W V AY : 3:3:119c
aAR aYR
It is now possible to develop the distribution equilibrium Replace the single cation and single anion activities by the
relation for a strongly electrolytic solute i in an ion mean electrolyte activity a as follows:
exchange system using equilibrium criterion (3.3.29):
aAA aAY a ;
iw  iR Z i F R  w : 3:3:118a
where A Y .
We know from expression (3.3.21) for ij , Now use result (3.3.117b) to obtain
   V AY =V sw
iw 0i V iw P w  P 0 RT n aiw aR asR
; 3:3:119d
and aw asw

which relates the electrolyte activity in the external solution


iR 0i V iR P R  P 0 RT n aiR :
to that in the resin. Such a relation for the electrolyte
Substitute these into relation (3.3.118a) to get activity in the two phases of an ion exchange system can
154 Physicochemical basis for separation

be reduced to an explicit relation between the molalities of


the co-ion Y between the two phases. CH CH2 CH CH2
Consider a cation exchange resin with fixed negative
charges it prefers to exchange only cations from the
solution. To demonstrate this preference, rewrite relation
(3.3.119d) for the electrolyte AY as
SO3H+ SO3H+
aAR
A Y
aYR A
aAw aYw
Y
asR =asw V AY =V sw : 3:3:120a

Assume A 1; Y 1 (for example, NaCl). Rewriting the CH CH2


activities in terms of molalities and activity coefficients, we
obtain Figure 3.3.11. The basic unit of a cation exchange resin made of
V AY =V sw
crosslinked polystyrene with sulfonic acid groups.
mA;R mY;R mA;w mY;w Aw Yw = AR YR asR =asw :
3:3:120b
illustrate this aspect following Helfferich (1995). Consider
Now employ electroneutrality relation (3.1.108a) in terms
crosslinked polystyrene with sulfonic acid groups, whose
of molalities for the resin phase and the solution phase:
basic unit is identified within the dashed lines in Figure
solution phase : mA;w mY;w ; 3.3.11. The formula weight of the basic unit C8H8SO3 (with-
resin phase : mA;R mY;R mF;R ; 3:3:120c out the counterion H) has the value 184.2; it can carry one
28
counterion of unit charge (here H). Therefore the max-
where mF;R is the molality of the fixed negative charges in imum ion exchange capacity of this resin is qm iR (where
the resin phase. Relation (3.3.120b) can now be rearranged subscript R stands for the resin phase), (1000/184.2)
via the electroneutrality relations to yield 5.43 meq/g. Since the resin is crosslinked using small
amounts of divinyl benzene (which is also sulfonated,
mY;R mY;R mF;R m2Y;w Aw Yw =AR YR asR =asw V AY =V sw ;
|{z} equivalent weight 210.2) and ethylstyrene (equivalent
b
weight 212.2), the ion exchange capacity (maximum theor-
3:3:120d etical) will be somewhat less. If such a resin is present in a
bm2Y;w m2Y;w packed bed, having a void volume fraction of , and the
mY;R b; 3:3:120e water-swollen density of the resin particles is s , having a
mY;R mF;R mF;R
fractional water content of w , then the volumetric ion
where we have assumed that the fixed molal charge exchange capacity of the packed bed, V iex cap in equivalents
density in the resin, mF;R , is much larger than mY ;R ; it per liter of packed bed volume
can be easily in the range of 5 molal. Relation (3.3.120e)
m
suggests that, as long as the external solution is dilute V iex
cap 1  s 1  w qiR : 3:3:120g
mY;w << 1, the co-ion concentration mY;R in the resin
will be very small. For example, assuming b 1, if
Following Helfferich (1995), consider a bed of sulfo-
mY;w 0:001  0:1 molality, then, for mF;R 5 molality,
nated polystyrene resin beads having s 1:25 g=cm3 ,
mY;R ranges between 0.2  106 and 2  103. Therefore,
w 0:468, 0:4. The value of V iex
cap is
for dilute external solutions, it is clear that (Helfferich,
1962, 1995), at equilibrium, eq
V iex
cap 1  0:41:25  1  0:468 5:4
liter of bed
2
molality of Yresin / molality of Yexternal ; 3:3:120f 2:2eq=liter of bed:
solution
When the counterion in the ion exchanger is different from
i.e. the co-ion concentration in the resin will indeed be
the counterion in the external solution, there will be an
very low. This behavior is essential to the success of ion
exchange between the two counterions. For an ion exchan-
exchange processes with dilute external solutions; corres-
ger with negative fixed charges (indicated by R, say,), an
pondingly, the ion exchange process is most often applied
exchange between cation BZB (say) in the external solution
to dilute solutions. Dependence of the proportionality
and cation AZA in the resin (Figure 3.3.12) is indicated by
factors in relation (3.3.120f) to solution concentration,
the following ion exchange process:
however, complicates the above relationship.
What is the maximum extent of ion exchange of a jZB j AZA jZA jR jZA jBZB w ,
counterion with the ion exchange resin? We will briefly jZA j jZB jR BZB jZB jAZA w: 3:3:121a

28
Molality means gmol of solute per 1000 g of solvent. An example of such a cation exchange is
3.3 Equilibrium separation criteria closed vessel 155

B
B
A
B
A A B
A A
A
A A B A
B
A
A

B B
A

Initial state Equilibrium

Matrix with fixed charges A B Counterions Co-ions

Figure 3.3.12. Ion exchange with a solution (schematic). A cation exchanger containing counterions A is placed in a solution
containing counterions B (left). The counterions are redistributed by diffusion until equilibrium is attained (right). (After Helfferich
(1962, 1995).)

2 NaR CaCl2 aq , CaR2 2 NaClaq: 3:3:121b C H R C Na R C FC : 3:3:121g

When instead of CaCl2 in the external solution, there is, say Correspondingly, in the solution,
CaCO3, MgCO3, etc., or corresponding sulfate salts, the
C H w C Na w C tw ; 3:3:121h
exchange reactions are called water-softening exchange
reactions; such ion exchange processes also remove low where Ctw is the total molar concentration of H and Na

levels of iron and manganese sometimes present in the in the aqueous solution. The equilibrium constant for the
domestic water supply. ion exchange process for ideal solution behavior in both
For an ion exchanger with positive fixed charges phases is given by
(R, say), an exchange between anion BZB in the exter-
C Na R C H w
nal solution and anion AZA in the resin is similarly K : 3:3:121i
C Na w C H R
expressed as
Using the three equations (3.3.121gi), we can obtain the
jZB j jZA jR AZA jZA jBZB , jZA j BZB jZB jR jZB jAZA
equilibrium distribution of the counterion Na between
3:3:121c the resin and the external solution as
An example of such an anion exchange is K C FC C Na w
C Na R : 3:3:121j
2RCl Na2 SO4 aq , R2 SO4 2NaClaq: 3:3:121d C tw C Na w K  1

The ion exchange equilibrium constant K for such an ion To convert this relation to the concentration unit of moles
exchange reaction may be written using the law of mass of Na/mass of the solid particle (swollen gel), qNa R , note
action as follows: that, if R is the resin mass/volume,

C Na R qNa R R ; C FC qmax;R R ; 3:3:121k


aNaCl 2 aR2 SO4
K : 3:3:121e
aNa2 SO4 aRCl 2 K C Na w
qNa R qmax;R ; 3:3:121l
C tw C Na w K  1
We now illustrate the nature of the ion exchange isotherm
using the simple cation exchange of Na in solution with where qmax;R is the maximum molar fixed charge density
the resin in H form in molar concentration units: per unit resin mass. Therefore

Na w R H resin H w R Na resin: C H R C Na R C FC qmax;R R : 3:3:121m


3:3:121f
The equilibrium isotherm relation (3.3.121l) is similar to
If the maximum ion exchange capacity of the resin for the Langmuir adsorption isotherm (3.3.112a). In fact, an
monovalent ions is CFC (where the subscript FC refers to alternative representation will make them essentially iden-
fixed charge), tical (see Figure 3.3.13):
156 Physicochemical basis for separation

C Na R K C Na w =C tw Ion exchange processes (3.3.1121a,c) have been writ-


;
C FC 1 C Na w = C tw K  1 ten to resemble chemical reactions, but they are not chem-
qNa R K C Na w =C tw ical reactions; rather, they are redistributions of ions by
: 3:3:121n diffusion within an environment influenced by electrostatic
qmax;R 1 C Na w = C tw K  1
forces. The heat evolved in such ion exchange processes is
An equilibrium is achieved in these exchanges after some also very low when compared with chemical reactions or
time. Generally, one of the counterions AZA or BZB will be even many adsorption processes (Helfferich, 1962, p. 8).
preferred by the ion exchanger. This subject of ion exchan-
ger preference will be treated in Section 4.1.6, where 3.3.7.7.1 Steric mass-action (SMA) ion exchange equi-
expressions for equilibrium separation factor and selectiv- librium for proteins The treatment of ion exchange
ity between two species A and B will be obtained. Some equilibrium so far covered, de facto, smaller molecules.
materials can exchange both cations and anions and are Ion exchangers are also frequently used for adsorption/
therefore called amphoteric ion exchangers. Alternatively a desorption of larger macromolecules, such as proteins, in
mixture of cation exchange resin particles and anion the presence or absence of various salts (see Section 4.1.6,
exchange resin particles may be used in exchangeable H Figure 4.1.16 and Section 4.1.9.4). Our earlier introduction
and OH forms, respectively, where there is no addition to of adsorption (equations (3.3.112a) and (3.3.113a))
the dissolved solids concentration in the solution. (Other- involved an adsorbate molecule with respect to an adsorb-
wise NaCl, for example, will be added via (3.3.121b, d)). ent site. A charged macromolecule, e.g. protein, however,
Such a strategy is used for water demineralization, e.g. in has many charged patches over its surface; therefore,
treating boiler feed water. adsorption/ion exchange of a protein molecule with the
ion exchange resin bead involves multipoint adsorption/
binding. However, a large macromolecule, such as a pro-
tein, will cover many more adsorbent sites than those
dictated by their characteristic charge and the location of
Counterion concentration
in resin, CNa+, R or qNa+, R

the charges (Figure 3.3.14).


Correspondingly, these sites covered by the macromol-
ecule/protein without any charge based interaction will
have counterions from the microsolutes in the system, e.g.
salts. Such salt counterions bound to the resin particle face
steric hindrance and are unavailable for exchange with
other macrosolutes/proteins in free solution. Brooks and
Cramer (1992) have illustrated a steric mass-action (SMA)
ion-exchange equilibrium formalism to describe the equilib-
rium ion exchange behavior of a single protein and
CNa+ w exchangeable salt counterions with respect to an ion
exchange resin. Their model assumes: ideal solution behav-
Figure 3.3.13. Langmuir adsorption isotherm type behavior for ior; monovalent salt counterions; the use of concentration C
Na exchange with a cation exchange resin. instead of activities (in units of mM) and the law of mass

Protein

+ + +
S+ S+ +

+

S S S +

+

S+ S+


Figure 3.3.14. Idealized protein binding on an ion exchange bed. S^ : sterically hindered salt counterions; S : nonsterically hindered salt
counterions. Reprinted, with permission, from C. A. Brooks and S. M. Cramer, AIChE J., 38(12), 1969 (1992). Copyright [1992]
American Institute of Chemical Engineers (AIChE).
3.3 Equilibrium separation criteria closed vessel 157

action-based equilibrium; neglect of any effect of co-ions in lim C pR K ps C FC =C sw jZ p j C pw : 3:3:122g


C pw !0
the ion exchange process; neglect of nonidealities so that
equilibrium parameters are constant and independent of
The slope of this isotherm decreases drastically as the
macrosolute and salt counterion concentrations.
aqueous phase salt concentration, Csw, increases. As Cpw
The stoichiometric exchange of the protein molecule
increases to high values, CsR tends to zero and the isotherm
(i p) and the exchangeable salt counterions (i s) may
approaches the limit from equation (3.3.122e):
be represented as follows:
    C FC
jZ p j jZ p j lim C pR C max
pR : 3:3:122h
C pw C sR , C pR C sw : 3:3:122a C pw ! jZ p j p
jZ s j jZ s j
  This value, C max
pR , represents the saturation capacity of protein
Here jZ p j=jZ s j represents the ratio of the absolute values sorption by ion exchange; it appears to be independent of the
of the characteristic charges of the protein and the coun- concentration of the monovalent counterion salt. In reality,
terion (n-valent salt); therefore it represents the number of such a limit is achieved only at low values of Csw. Unlike other
sites with which a protein molecule interacts. Since we ion exchange systems, where C max for any counterion i is
iR
have monovalent salt counterions here jZ s j 1, Zp, the simply obtained from CFC and jZ i j, for proteins/biomole-
characteristic charge of the protein, also provides the cules, the steric factor p introduces an additional complex-
number of sites jZ p j with which it interacts. The equilib- ity. For illustrations of the isotherm (3.3.122f), see Brooks and
rium constant for this binding in the ion exchange process, Cramer (1992), which gives a number of proteins/biomole-
Kps, may be defined by (since jZ s j 1) cules, e.g. cytochrome-c, -chymotrypsinogen.
  jZ p j
C pR C sw
K ps : 3:3:122b
C pw C sR 3.3.7.8 Distribution equilibrium representation as a
chemical reaction
The total concentration of sterically hindered salt ions
unavailable for exchange with protein molecules in free It is possible to represent the distribution of a species i
solution is given by between phases 2 and 1 by a form usually reserved for
chemical reactions (Rony, 1969a):
^ sp p C pR ;
C 3:3:122c
i phase 2 , i phase 1: 3:3:123
where p is a steric factor for the protein. After protein
adsorption, the total salt concentration in the stationary Since the condition for chemical equilibrium (not chemical
(ion exchange) phase, C tsR is given by reaction equilibrium) in this case is given by i1 i2, the
physical reaction (3.3.123) may also be represented by
C tsR C sR p C pR : 3:3:122d X
ij ij 0; 3:3:124
The condition of electroneutrality in the resin phase requires j1
 
C FC C sR jZ p j p C pR ; 3:3:122e where, by convention, jij j 1 and i1 1, i2 1. The
quantities ij  s are somewhat analogous to stoichiomet-
where CFC represents the concentration of the total fixed ric numbers in chemical reaction equilibria. The equilib-
charge in the ion exchange resin based on monovalent rium constant for this physical reaction, K, is then given by
counterions; it is also the ion exchange capacity (i.e. mF,R X
in molal units). Substituting equation (3.3.122e) into equa- RT n K ij 0ij 0il  0i2 RT n K ai ;
tion (3.3.122b) and rearranging leads to
!jZ p j where
C pR C sw
C pw   : 3:3:122f ai1
K ps C FC  jZ p j p C pR K ai : 3:3:125
ai2
If the aqueous phase protein concentration Cpw is known Such an approach is consistent with what has been already
along with the aqueous phase salt concentration Csw, then presented in relations (3.3.76), (3.3.113a), (3.3.114a) and
this equation defines implicitly the protein concentration, (3.3.121ad).
CpR, in the resin phase. Once CpR is known, one can calcu-
late CsR from equation (3.3.122e) given CFC, jZ p j and p ; 3.3.7.9 Equilibria in a system with supercritical fluid
correspondingly, C tsR becomes available from (3.3.122d).
Some limiting forms of the isotherm (3.3.122f) for a In separation equilibrium involving a supercritical fluid
single protein are useful. Consider linear conditions such (SCF), the two systems commonly used are solidSCF
that as C pw ! 0; C pR ! 0. Then we get, from isotherm and liquidSCF. A solute or solutes are distributed between
(3.3.122f), the two immiscible phases. A supercritical fluid is normally
158 Physicochemical basis for separation

(b)
(a)
0.1000 TR = T/Tc = 0.8
35 C 0.9
Mole fraction of naphthalene

1.0
2.0
0.0100 12 C 1.1

rR = r/rc
1.2

1.0 CP 1 . 55
0.0010

0.0001 0
0 50 100 150 200 250 300 350 0.1 1.0 10.0
Pressure (atm) PR = P/Pc

Figure 3.3.15 (a) Solubility behavior of solid naphthalene in supercritical ethylene. (b) Variation of reduced density (R) of a pure
component near its critical point (CP). These figures were published on pp. 5 and 6 of Supercritical Fluid Extraction: Principles and
Practice, M. A. McHugh and V. J. Krukonis, ButterworthsHeinemann (507 pp.). Copyright Elsevier (1986).

employed to extract a solute from a solid or liquid phase. TR (T/Tc). The 12 C (TR 1.01) isotherm of R vs: P R
The solute is recovered later by converting the supercritical shows a rapid rise in by more than an order of magnitude
fluid into a subcritical fluid (usually a gas), which has much around PR 1.0. The shape of this curve is quite similar to
reduced solute solubility. The subcritical fluid is recom- that of the naphthalene solubility curve, suggesting that
pressed to the SCF state and recycled for solute extraction. the solvent power of ethylene for naphthalene is directly
A SCF is therefore often called a SCF solvent. related to the SCF solvent density near Tc, Pc.
A pure fluid becomes a supercritical fluid when its To develop an estimate of the solute mole fraction x il
pressure and temperature exceed its critical pressure (Pc) in the SCF solvent in equilibrium with a solid mixture, the
and critical temperature (Tc). For example, the value equilibrium relation is
of Pc, Tc for the commonly used SCF CO2 are 72.8 atm
^f ^f ;
and 31.2 C; the corresponding values for ethylene and il is

propane, sometimes used, are 49.7 atm, 9.4 C and where


41.9 atm, 96.8 C, respectively. On the other hand, the
values of Pc and Tc for water are 217.6 atm and 374 C; ^f x il
^ il P and ^f is x is
^ is P: 3:3:126
il
due to such high values of Pc and Tc, water is almost never
The solid mixture is considered to be an agglomeration of
used as a SCF for separation.
pure species, with each component having its pure com-
To use a SCF as a solvent for extracting solutes, the
ponent solid fugacity f is ; i:e: ^f is f is . At low pressures,
solubility of any solute in a SCF must be substantial. It is
this fugacity is almost equal to the sublimation pressure
generally observed that the capacity of a SCF to dissolve a
P sub
i T . However, at the high pressures characteristic of
solute is directly related to the density of the SCF. For
SCF extraction,
example, consider the solubility characteristics of solid
"   #
naphthalene in supercritical ethylene shown in Figure P
Vis
3.3.15(a) (McHugh and Krukonis, 1986). The solubility fis P sub
i T sub
i T; P sub
i exp dP ;
P sub
i
RT
of solid naphthalene increases by more than an order
3:3:127
of magnitude from a very low value when the ethylene
pressure exceeds the Pc 49.7 atm of ethylene (at T > Tc where sub
is the fugacity coefficient at T and P sub
i i , and
9.4 C, of ethylene). The variation of the density of the exponential term is the Poynting pressure correction
ethylene near its critical point may be seen in Figure for the fugacity of pure solid having a molar volume Vis.
3.3.15(b), a plot of reduced density, R =c against Therefore, the mole fraction of solute i in the SCF
reduced pressure PR (P/Pc) and reduced temperature solvent is
3.4 Interphase transport: flux expressions 159

"   #
P Liquid
sub V is
P sub
i T i T;P sub
i exp dP
P sub RT g LG
i
x il : Liquid Gas
^ il P

Bubble
3:3:128

A review of the literature and thermodynamic calculation q


procedures for such systems are available in McHugh and
Krukonis (1986). The same reference may be studied for g SL
thermodynamic equilibrium calculations for the solute dis- g SG Solid
tribution between a liquid and a supercritical fluid solvent.
Figure 3.3.16. Interfacial tensions and contact angle for a gas
bubbleliquidsolid system with a flat solid surface.
3.3.8 Particle distribution between two immiscible
phases
G LG cos  1: 3:3:131
So far, we have been concerned with the distribution of
If the particle is wetted by the liquid, the contact angle
molecules, ions or macromolecules between two immiscible
should be zero since the liquid spreads over the solid
phases. The molecules may have been solutes present in
surface completely. Thus, air or a gas bubble cannot
small quantities or major constituents of either or both
become attached to the particle surface. If, however, is
phases. Classical principles of thermodynamics were used to
finite, G is negative; there is now a gassolid interface,
develop estimates of such solute distributions. When it comes
and bubble attachment to the particle is possible.
to large particles, such as ore fines, cells or other particulate
To create a nonzero contact angle in a gasliquidsolid
matter, classical thermodynamics may not appear to be of any
particle system where the liquid normally wets the particle
use. However, using the phenomenon of wetting based on
(e.g. water and most mineral matter), surface active species
interfacial thermodynamics, particle separation from one
called collectors are dispersed in the liquid for adsorption
phase is achieved by introducing a second immiscible phase.
on particle surfaces. In aqueous systems, addition of
There are two types of systems here depending on
surfactants such as long-chain fatty acids, and their
whether the two immiscible phases are gasliquid or
adsorption on particle surfaces, make the surface hydro-
liquidliquid. Particle separation in a gasliquid system is
phobic enough, leading to a nonzero . Equation (3.3.40b)
much more common and is called flotation. The following
may be utilized here to study how the various interfacial
three paragraphs will focus on the basic thermodynamic
tensions are changed by changes in the bulk concentra-
principles in such a system.
tions of the added surfactants (Fuerstenau and Healy,
Consider two types of mineral particles in an aqueous
1972).
suspension. If air bubbles can be attached to one type of
Instead of a gas or air, it is possible to use a second
particle only, the latter will float to the surface of the
immiscible liquid, usually water or an aqueous solution, to
suspension, due to reduced density, and can be separated
separate particles or cells from the first immiscible liquid.
from the other type of particles. Normally, mineral par-
The second immiscible liquid may serve as the collector
ticles are wetted completely by water so that air bubbles
phase collecting the particles into it (see Raghavan and
cannot attach to them. However, if the particle surface can
Fuerstenau (1975)). Alternatively, the solid particles may
be made sufficiently hydrophobic to prevent wetting, air
be pushed to the liquidliquid interface. Whether the par-
bubble attachment is possible.
ticles are going to be located at the liquidliquid interface
The criterion for air bubble attachment is that the free
or in the bulk of one of the liquid phases can be deter-
energy change should be negative:
mined by relations between the three interfacial tensions
G SG  SL LG 3:3:129 SL ;SL2 ;L1 L2 (see Henry (1984)), where L1 and L2 are the
two immiscible liquid phases.
where SG , SL and LG are the interfacial tensions for the
systems solidgas, solidliquid and liquidgas, respect-
ively. The general relation between these interfacial ten-
sions and a contact angle for a gasliquidsolid system 3.4 Interphase transport: flux expressions
with a flat solid surface (shown in Figure 3.3.16) is given by In Section 3.1.4, we studied how integrated flux expres-
Young-Dupre equation: sions can be used to describe mass transport in single-
phase systems. In separation processes involving two or
SG SL LG cos 3:3:130
more phases, solute and/or solvent is transferred from one
phase to another. In processes using membranes, a species
Substituting this into expression (3.3.129), we get is transferred from the feed to the membrane phase and
160 Physicochemical basis for separation

then from the membrane phase to the permeate (or prod- Gasliquid
uct or receiving) phase; thus three phases are involved in interface
the direction of species transport. It is necessary to have Liquid phase Gas phase
flux expressions, especially integrated ones, for such inter- pAb , xAgb
phase transport. In general, the approach is to use inte-
grated flux expressions for each phase and couple them
together to develop an overall integrated flux expression Increasing Increasing
xAl , CAl
using interphase equilibrium relations at the phase bound- xAg , pA
xAgi, pAi
aries. Another objective is to develop a relation between an
CAli, xAli
overall mass-transfer coefficient for the overall interphase
transfer process and the mass-transfer coefficient for trans-
fer in each phase.
The following is a partial list of two-phase systems
encountered in separation processes involving interphase
xAlb
transport: gasliquid (alternatively vaporliquid), liquid CAlb
liquid, solidliquid, liquidion exchange resin, solid Direction of transport
supercritical fluid, liquidsupercritical fluid, etc. The first of species A
four systems are used much more frequently. Note that the
two phases in each system are immiscible. Figure 3.4.1. Concentration profiles of species A being absorbed
It is possible to have three-phase combinations of the from a gas into a liquid.
above-mentioned two-phase systems: gasliquid (1)liquid
(2) (or vaporliquid (1)liquid (2)), liquidion exchange
resin (1)ion exchange resin (2), etc. We are not going to developed in Section 3.3. Thus pAi or xAgi is related to CAli
consider here such three-phase systems. The three-phase or xAli for an ideal gasliquid system by Henrys law (rela-
(or region) systems of interest here involve a membrane, tion (3.3.60b)):
the feed phase and the permeate phase. Some of the pAi H A x Ali ; x Agi H A =Px Ali H PA x Ali 3:4:1a
common three-phase systems of this type are: liquid
membraneliquid, gasmembranegas, liquidmembrane or
gas, gasmembraneliquid. The two fluid phases on two
pAi H CA C Ali ; H CA H A =C tl : 3:4:1b
sides of the membrane are, in general, miscible (exceptions
include the liquidmembranegas system, etc.). An elemen- For nonideal systems, the equilibrium relations
tary introduction to species transport through membranes
that are either nonporous or porous will be provided here to pAi f eq x Ali ; x Agi f eq x Ali 3:4:2
facilitate development of flux expressions in a few commonly are to be determined from the equilibrium criterion of
used membrane processes. fugacities ^f Ag ^f Al for species A (similarly for all other
species).
3.4.1 Interphase transport in two-phase systems The flux of species A in both gas and liquid phases can
be expressed by either
3.4.1.1 Gasliquid systems
N Az k xl x Ali  x Alb k xg x Agb  x Agi ; 3:4:3
Figure 3.4.1 shows the gas-phase and liquid-phase concen-
tration profiles of species A being transferred from a gas N Az k g pAb  pAi k c C Ali  C Alb ; 3:4:4
phase into a liquid phase. In the gas phase, the bulk
where kxl, kxg, kg and kc are mass-transfer coefficients
composition is indicated either by the partial pressure pAb
defined for a particular phase, either gas or liquid. We
or the gas-phase mole fraction xAgb. This value is reduced
recall from our earlier discussion in Section 3.1.4 that
to pAi or xAgi at the gasliquid interface. In the liquid phase,
integrated flux expressions are useful since only concen-
the corresponding values are CAli or xAli at the gasliquid
trations or mole fractions or partial pressures at the two
interface. These values are reduced to the bulk liquid-
ends of the diffusion/transport path are required instead of
phase values CAlb or xAlb at some distance away from the
their gradients. Here, however, we find that there are
gasliquid interface.
quantities like pAi, xAgi, xAli corresponding to the gasliquid
In general,29 the interfacial compositions in the two
interface in single-phase integrated flux expressions; these
phases are related to each other by equilibrium criteria
quantities are very difficult to obtain. On the other hand,
the bulk concentrations in each phase, xAgb, CAlb, xAlb and
29
At high mass-transfer rates and with a contaminated interface, pAb are much easier to obtain. The answer is provided by
interfacial resistance is possible (see Sherwood et al. (1975)). an overall mass-transfer coefficient K at any location.
3.4 Interphase transport: flux expressions 161

The strategy is to define the overall mass-transfer coef- An important characteristic of the four relations
ficient K with respect to a single phase, either gas or liquid, (3.4.8)(3.4.11) is that the overall mass-transfer resistance
and known bulk concentrations: indicated by the 1/K term is the sum of the individual gas-
and liquid-phase resistances, regardless of the form of the
N Az K xg x Agb  x Ag K xl x Al  x Alb 3:4:5
concentration driving gradients. Such linear additive
or behavior of resistance we will find, again and again, is
the rule in interphase transport, although there are a few
N Az K g pAb  p A K c C Al  C Alb : 3:4:6
exceptions.
Here x Ag is a hypothetical gas-phase mole fraction which is The individual phase transfer coefficients kxl and kxg
in equilibrium with x Alb ; thus x Ag is known. Similarly, x Al is a are often of the same order of magnitude. However, the
hypothetical liquid-phase mole fraction which is in equilib- value of H PA can vary over a wide range depending on the
rium with x Agb ; similarly, p A is in equilibrium with CAlb and nature of species A and the liquid phase. If a gas species is
C Al is in equilibrium with pAb. Now express x Agb  x Ag as highly soluble in the liquid (Figure 3.3.3 illustrates such a
system), H PA has a small value, which makes
x Agb  x Ag x Agb  x Agi x Agi  x Ag 3:4:7
H PA =k xl << 1=k xg term in (3.4.8). It is then possible to
and utilize equilibrium relation (3.4.1a) as well as the replace Kxg by kxg since
definitions of Kxg, kxg, kxl: 1 1
; 3:4:12a
N Az N Az H A N Az H A N Az K xg k xg
x Ali  x Alb ;
K xg k xg P k xg P k xl
that is, the gas-phase resistance is controlling the species
1 1 HP p HA transport. Similarly, if the gas species has a very low solu-
A; HA :
K xg k xg k xl P bility in the liquid, H A has a very high value (e.g.
3:4:8 H A 12:5  104 (atm/mole fraction) for helium in water
at 20 C; H A 4:0  104 (atm/mole fraction) for O2 in
Therefore, the overall mass-transfer coefficient Kxg is water at 20 C (see Figure 3.3.3). Then
known in terms of the individual phase mass-transfer coef-
1 1
ficients kxg and kxl and Henrys law constant H PA . Further, if ; 3:4:12b
K xl k xl
the bulk compositions of the two phases are known, the
flux N Az of species A from gas to the liquid can be pre- that is, the liquid-phase resistance controls the species
dicted. Note that relation (3.4.8) is also valid if the direction transport. Similar relations are easily developed between
of transport of species A was reversed. Kg and kg for gas-phase control and Kc and kc for liquid-
Equation (3.4.8) relates the overall mass-transfer coeffi- phase control.
cient Kxg to the individual mass-transfer coefficients kxg and Although the relations given have been developed for
kxl. Relations between the overall transfer coefficient and the any gasliquid system, they are also valid for any vapor
individual phase transfer coefficients for other types of concen- liquid system. The only difference is that Raoults law is to
tration driving gradients are also of considerable use. Consider be used instead of Henrys law as the equilibrium relation
between the phases. For example, consider transfer of
N Az K xl x Al  x Alb k xg x Agb  x Agi k xl x Ali  x Alb :
species i from vapor phase to liquid phase, x igb to x ilb , with
But the overall mass-transfer coefficient being expressed in
terms of the vapor phase, i.e. Kxg:
x Agb P x Agi P N Az
x Al  x Alb x Al  x Ali x Ali  x Alb 
HA HA k xl x igb  x ig x igb  x igi x igi  x ig :
1 P 1 1 1 1
) : Here, x ig is the vapor-phase mole fraction in equilibrium
K xl H A k xg k xl H PA k xg k xl
with the actual liquid-phase bulk composition (in mole
3:4:9 fraction) xilb and xigi is the vapor-phase mole fraction of
species i at the vaporliquid interface. Using definitions of
Similarly, mass-transfer coefficients
1 1 HC 1 HC
A A 3:4:10 Ni Ni Ni Ni
K g kg kc kg kl K i x ili  K i x ilb Ki
K xg k xg k xg k xl
and 1 1 Ki 3:4:13a
) ;
1 1 1 1 1 K xg k xg k xl
: 3:4:11
K c k c H CA k g k l H CA k g
where Ki is the equilibrium ratio (see equilibrium relations
Note that kl and kc will be used interchangeably. (3.3.61) or (3.3.63)). We may use here subscript v for vapor
162 Physicochemical basis for separation

phase instead of g. Similarly, for the overall transfer coeffi- Aqueousorganic phase
cient expressed in terms of the liquid phase, we have interface
1 1 1 1
: 3:4:13b
K xl k xl K i k xg CAoi

The various results considered above for a gasliquid (or


vaporliquid) system were obtained for absorption of
species A from gas (or vapor) into a liquid. If species A is
CAwb
being stripped from the liquid into gas (or vapor) phase, CAob
only the direction of transport changes; therefore the flux
expressions (3.4.3)(3.4.6) are changed by a minus sign, as
are the concentration profiles in the two phases. However,
the relations between the overall transfer coefficient K and
the individual phase transfer coefficient k remain CAwi
unaffected by the direction of transport.
At the beginning of this section, it was indicated that Aqueous phase
Organic phase
the route of overall mass-transfer coefficients was a con-
venient way of avoiding the difficulties created by the
difficult-to-obtain interfacial concentrations CAli, pAi, xAgi, Figure 3.4.2. Concentration profiles of species A being extracted
xAli, etc. We have found it to be so. It is also possible to from an aqueous to an organic phase.
solve for the interfacial concentration CAli or xAgi, etc. from
either equations (3.4.3) (3.4.4) if kxl, kxg and the equilib-
distribution coefficient Ao is constant over the whole con-
rium relation are known for given bulk concentrations. For
centration range encountered in this case, then
example, from equation (3.4.3),
N Az
k xl x Agb  x Agi C Awb  C Aw C Awb  C Awi C Awi  C Aw
; 3:4:14 K Aw
k xg x Ali  x Alb
N Az 1
where, however, x Agi H PA x Ali , so that xAgi is the only C Aoi  C Aob ;
k Aw Ao
unknown and can be determined.
N Az N Az N Az 1 1 1
) : 3:4:18
3.4.1.2 Liquidliquid systems K Aw k Aw Ao k Ao K Aw k AW Ao k Ao

Figure 3.4.2 illustrates the concentration profile of solute For the relation between KAo, kAw and kAo, we note that
A being extracted from the aqueous phase ( j w) into the
organic phase ( j o). The molar flux of species A may be N Az
C Ao  C Aob C Ao  C Aoi C Aoi  C Aob
expressed by K Ao

N Az k Aw C Awb  C Awi k Ao C Aoi  C Aob ; 3:4:15 N Az 1 Ao 1


Ao C Awb  C Awi )
k Ao K Ao k Aw k Ao :
where kAw and kAo are mass-transfer coefficients for
species A in the aqueous and organic phases, respectively. 3:4:19
In general, the interfacial concentrations in the two phases
Equations (3.4.18) and (3.4.19) provide the desired relations
CAwi and CAoi are related to each other by equilibrium (see
between the individual phase mass-transfer coefficients kAw,
relation (3.3.79)):
kAo and the overall mass-transfer coefficients KAw and KAo.
C Aoi Although these results are derived for transfer from the aque-
Ao : 3:4:16
C Awi ous to the organic phase, they are equally valid for transfer in
the opposite direction. As before, the overall mass-transfer
As indicated before, the interfacial concentrations are dis-
resistance is the sum of the resistances of the individual
pensed with by defining overall mass-transfer coefficients:
phases, the aqueous and the organic. Further, under particu-
N Az K Aw C Awb  C Aw K Ao C Ao  C Aob : 3:4:17 lar conditions, either the aqueous- or the organic-phase
resistance controls. For example, if Ao >> 1 (the solute
Here, C Aw is a hypothetical aqueous-phase concentration prefers the organic phase strongly over the aqueous phase),
in equilibrium with the bulk organic-phase concentration then the aqueous phase resistance is in control:
CAob. Similarly, C Ao is a hypothetical organic-phase con-
centration in equilibrium with the bulk aqueous-phase 1 1
: 3:4:20
concentration CAwb. If one can assume that the equilibrium K Aw k Aw
3.4 Interphase transport: flux expressions 163

! !
On the other hand, if Ao << 1, then the organic-phase 1612 3 V 2ic 1612 3 V 2ic N
e
resistance is dominant: B0 A exp  A exp 
3k B R2 T 3 n s2 3RT n s2
3

1 1 3:4:22b
: 3:4:21
K Ao k Ao
for a nonelectrolytic solute, where N e is Avagadros number.
Both relations are based on the assumption that kAw and The nucleation rate therefore increases with an increase in
kAo are of similar orders of magnitude. the temperature and the supersaturation ratio, whereas an
Example 3.4.1 Calculate the overall mass-transfer coeffi- increase in the interfacial tension decreases it. It has been
cient for the extraction of diethylamine (A) from its dilute
difficult to verify this equation experimentally since homo-
solution in water into toluene. The following data on this
geneous nucleation probably never occurs in practice (see
extraction have been obtained from Treybal (1963, p. 498):
Ao 0.735; kAw = 0.76 lb mol/hr-ft2-(lb mol/ft3); kAo McCabe and Smith (1976, p. 867); see, however, references
0.129 lb mol/hr-ft2-(lb mol/ft3). Calculate both KAW and KAo in chap. 2 of Myerson (1993)), whereas heterogeneous
in units of of cm/s. nucleation is common. The presence of a solid particle
generally reduces the energy barrier Gcrit needed for
Solution Equation (3.4.18) yields nucleation.
1 1 1 When a crystal grows in a supersaturated solution, the

K Aw k Aw Ao k Ao rate of crystal growth or the rate of solute transfer to the
1 1 1 crystal depends on two steps: (1) the rate of diffusion of
 1:315 10:54; the solute from the bulk through a laminar layer near
0:760 0:735 0:129
the crystal; (2) the rate at which solute molecules are
1 lb mol 1 1 30:48 cm integrated with the surface of the parent crystal.
K Aw  
11:855 2 lb mol 11:855 3600 1 s The second step, known as the particle integration rate,
hr-ft -
ft3 involves desolvation of the solute, counterdiffusion of
7:14  104 cm=s; the solvent away from the surface and orientation of the
solute into the crystal lattice. Figure 3.4.3(a) illustrates
1 Ao 1 0:735 1
0:967 7:75; an absorbed layer of solute on the surface of a growing
K Ao k Aw k Ao 0:760 0:129
crystal a sort of a third phase composed of partially
1 lb mol 1 1 30:48 cm ordered solute, perhaps in a partially desolvated lattice
K Ao  
8:717 2 lb mol 8:717 3600 1 s (Randolph and Larson, 1988). If this surface integration
hr-ft -
ft3 rate is assumed to be linear with the driving force, then
9:71  104 cm=s: the concentration profiles for the diffusional step and the
surface integration step are as shown in Figure 3.4.3(b).
The total rate of transfer of solute to the growing crystal
3.4.1.3 Solidliquid system: solution crystallization at any instant of time can be expressed, respectively, for
steps (1) and (2) above by
There are two basic rate phenomena of interest in crys-
tallization: (1) the nucleation rate; (2) the growth rate. N iy ap k d ap C iwb  C iwi ; 3:4:23a
Let us start with the nucleation rate. We have observed N iy ap k s ap C iwi  C is : 3:4:23b
via equations (3.3.99) how the addition mechanism
leads from kinetic units to a cluster, to an embryo, and An overall crystal growth coefficient Ko may be defined
then to a nucleus; finally, a critical size rc is achieved by using an overall concentration difference C iwb  C is :
the nucleus. From the theory of activated process in N iy ap K o ap C iwb  C is : 3:4:24
chemical kinetics, the rate of nucleation in homoge-
neous nucleation is described by an Arrhenius type Therefore
expression 1 1 1 kd ks
  ) Ko : 3:4:25
Gcrit K o kd ks kd ks
B0 A exp  B ; 3:4:22a
k T Often it is useful to express the rate of solute transfer to the
where k B is Boltzmans constant, 1.3806  1023 joules/K growing crystal via the rate of growth of a characteristic
e Gcrit is the critical free energy change for nucle-
( R=N), radius rp of the particle, such that the particle mass mp and
ation and A is a pre-exponential constant. The units of B0 the surface area ap may be expressed, respectively, by
and A are number of nuclei/cm3-s; the values of A sug-
mp p v r 3p and ap s r 2p : 3:4:26
gested are of the order 1025 (La Mer, 1952).
Introducing the value of Gcrit from expression Now, M i N iy ap dmp =dt, where Mi is the molecular
(3.3.102), we obtain weight of species i. Therefore
164 Physicochemical basis for separation

(a) Diffusion

Supersaturated
bulk solution

Parent
crystal

(b)
Solution
Ciwb

Crystal

dl

Ciwi Figure 3.4.3. (a) Absorbed layer of solute on the surface of a


growing crystal. h, AmBn.pH2O; , hydrated An ions; , hydrated
Bm ions. This figure was published on p. 119 of Randolph, A. D.
Cis and M. A. Larson, Theory of Particulate Processes: Analysis and
y Techniques of Continuous Crystallization, 2nd edn., Academic
Press, New York, 369pp. Copyright Elsevier (1988). (b) Solute con-
Distance
centration profile near a growing crystal showing interfacial resist-
ance to growth. (After Randolph and Larson (1988).)

dmp dr 3p dr p integration-controlled if there is no such effect of solution


p v 3p v r 2p K o M i s r 2p C iwb  C is ; velocity. Often, G is used to indicate the growth rate
dt dt dt
dr p =dt. Further, L is also used very frequently instead of rp.
dr p K o M i s
C iwb  C s G: There is no general method of estimating ks. It has
dt 3p v
been suggested (McCabe and Smith, 1976, p. 875) that
3:4:27 the following growth rate expressions are useful:

The growth rate of the crystal, dr p =dt, is said to be diffusion- for hydrated inorganics, G A1 s; 3:4:28a
controlled if an increase in velocity of the supersaturated
solution relative to the crystal surface leads to an increase in  
A3
crystal growth rate. The crystal growth becomes surface for organic crystals, G A2 exp  : 3:4:28b
s
3.4 Interphase transport: flux expressions 165

Solute adsorbed
into solid
Adsorbed layer Fluid phase in
on surfaces pores

Tb
Ts

Tb Ts

Cib , Pib
Cii Cii
Cib , Pib

Desorption
Adsorption
(a) (b)

Figure 3.4.4. Concentration profiles of gas or solute during (a) adsorption and (b) desorption in a particle whose structure is shown in the
middle. (After Vermeulen et al. (1973) and Yang (1987).)

Here, s is the fractional supersaturation and A1, A2 and adsorbent and the fluid. The over bar indicates the average
A3 are constants. The growth rate G of all crystals follows over the flow cross section in the bed. The concentration
an Arrhenius type of temperature dependence, namely decreases further inside the pores of the particle. There are
three basic and consecutive steps in the adsorption pro-
G A4 expE s =RT; 3:4:29
cesses for a species present in a mixture around any
where E s is an activation energy. adsorbent particle.
Inspection of the growth rates in equations (3.4.28a,b)
(1) The species diffuses from the bulk fluid phase to the
suggest that, regardless of the crystal size, the growth rate
external surface of the sorbent particle through a
depends only on the supersaturation present in the
so-called fluid film. This film provides what is called
system/crystallizer; the growth rate G is independent of
the external film resistance.
the crystal size rp (or L). Therefore
(2) Diffusion and transport of the molecules through the
dG dG porous structure of the adsorbent particle lead to the
0 : 3:4:30
dr p dL intraparticle transport resistance.
(3) Actual adsorption or reaction of the molecules occurs
This generalizaton, developed by W.L. Mc Cabe in 1929, is at numerous surface sites within the pores of the
called the L law of crystal growth. Correspondingly, over a adsorbents. This step is assumed to be essentially
time period t, the total increase in the crystal dimension, instantaneous compared to the other two resistances
L or r p , is given by identified above.
L r p Gt: 3:4:31 For desorption processes, the process steps are reversed:
starting with desorption at the many sites, continuing with
transport/diffusion of the desorbed molecules through the
3.4.1.4 Gassolid and liquidsolid based interfacial
porous structure and finally diffusion from the external
adsorption systems
surface of the sorbent particle to the bulk fluid phase
Separation in interfacial adsorption based gassolid or completes the desorption process. The corresponding
liquidsolid systems is generally carried out in stationary species concentration profile is shown in Figure 3.4.4(b).
beds of solid particles, with the fluid phase (gas or liquid) The structure of the porous adsorbent and the different
being mobile. As a result, the concentration profile of any regions in the porous adsorbent (e.g. the fluid phase in the
species in each phase is time-dependent. Figure 3.4.4(a) pores, numerous adsorption sites on the adsorbent solid
illustrates the concentration profile of a solute species surface, the adsorbed layers on pore surfaces) are sche-
being adsorbed from the fluid phase at any instant of time; matically illustrated via a magnified section in Figures 3.4.4
the solute concentration C ib (or partial pressure pib ) in the (a) and (b).
bulk fluid phase decreases to C ii (or partial pressure pii ) at The molar flux of adsorption of species i from the fluid
the end of the fluid film at the interface of the porous bulk to the surfaces of the porous adsorbent particle per
166 Physicochemical basis for separation

unit volume of the particle whose surface area per unit relative contributions of the internal resistance r p =Dip
volume is av is given by and the external film resistance 1=k ig is determined by
the Biot number:
N i av k i av C ib  C ii : 3:4:32a
r p =Dip k ig r p
For spherical particles, av 3=r p 6=d p . In terms of Bi : 3:4:35
1=k ig Dip
particle surface area a per unit volume of adsorbent par-
ticle bed, where a av 1  ( being the void volume In gassolid adsorption processes, generally Bi >> 1
fraction of the bed), (Yang, 1987). Therefore the external film resistance may
be neglected. The internal diffusion process controls the
N i a k i aC ib  C ii : 3:4:32b
rate of species adsorption/desorption.
The value of k i , the species mass-transfer coefficient in the Since intraparticle diffusion/transport controls the
fluid film, may be obtained from a variety of correlations species transport rate, a linear transport rate expression is
for the gas phase as kig and for the liquid phase as kil. The sought to describe it to facilitate solution of the overall
experimentally obtained correlations for ki popularly adsorption process taking place in a packed bed. The
employed for a gas flowing over a sphere or in a packed transient rate of adsorption/desorption of a species i may
bed are: be described by the time rate of change of a particle-
p
averaged species concentration C is defined for a spherical
(1) the j-factor correlation (3.1.157);
particle of radius rp by
(2) the RanzMarshall type correlation (3.1.154) proposed
in Geankoplis (1972);   rp
4 3 p p
(3) the Wakao and Funazkri (1978) correlation (3.1.158a). r p C is C is 4r 2 dr: 3:4:36a
3
(This equation is recommended when axial dispersion 0
is present in the packed bed. See Section 7.1.1.1 for p
Here, C is is obtained from the mass balance equation in a
equations for a packed bed with axial dispersion in the
spherical particle (see (7.1.19a)):
liquid phase; the axial dispersion coefficient is
"  # " #
expressed by a correlation in the footnote to Table p p p p
C is 1 C 2 C is 2 C is
3.1.6. The same axial dispersion coefficient may be Dip 2 r 2 is Dip :
t r r r r 2 r r
used for liquid flow over a packed bed also (Ruthven,
1984).) 3:4:36b
p
The external fluid film resistance (the corresponding mass- The time rate of change of C is
is described by a linear-
transfer coefficient ki from equations (3.4.32a,b)) is in driving force based expression:
series with the intraparticle transport resistance. The flux   p
4 3 C is 15Dip p p
of a species through a porous/mesoporous/microporous r p 2 C is  C is ; 3:4:36c
3 t rp
adsorbent particle consists, in general, of simultaneous
p
contributions from the four transport mechanisms where C is is a hypothetical averaged pore-phase concen-
described earlier for gas transport in Section 3.1.3.2 (for tration in equilibrium with the instantaneous bulk gas-
molecular diffusion, where DAK =DAB 1): phase concentration at the particlefluid boundary








C ii ;C ib (both being essentially equal to each other since
    kig is, on a relative basis, quite high). This approximation,
Ni N i  Knudsen N i  molecular N i  surface N i  Poiseuille=
 diffusion  diffusion  diffusion  viscous flow
identified as the linear driving force (LDF) approximation,
3:1:115b 3:1:115d 3:1:117a 3:1:116a was developed first by Glueckauf and Coates (1947), and is
3:4:33 widely used in particlefluid adsorption/desorption pro-
cesses. For a basis of this approximation, see the develop-
Not all of these contributions are simultaneously present/
ment in Yang (1987, p.127) based on a solution of the
effective. The concentration profile of a gas species inside
governing diffusion equation (equation (3.4.36b)) for diffu-
the particle has to be obtained by solving the mass balance
sion of gases within a spherical particle. Sircar and Hufton
equation within a particle (see equation (7.1.20c)). Such a
(2000) have shown for gassolid systems that any adsorb-
solution yields a gas-phase concentration profile inside the
p able species concentration profile describable by
particle, C is r, which is related to the external resistance
p
via the equality of fluxes at the particlegas surface r rp: C is r;t at bt Fr; 3:4:37a

C is 
p where F(r) is any monotonic and continuous function of r
k ig C ib  C ii Dip rr ; 3:4:34 in the range 0
r
r p satisfying the boundary condition
r  p

dFr 
where Dip is the effective diffusion coefficient obtained r0 0; 3:4:37b
dr 
from the combined processes of equation (3.4.33). The
3.4 Interphase transport: flux expressions 167

will satisfy the LDF approximation of (3.4.36c), where since C is =C mss is essentially equal to , the fraction of the
15Dip =r 2p may be replaced, in general, by a mass-transfer sites occupied.
coefficient k. The restriction for the LDF approximation of Often, instead of a molar concentration for the
Glueckauf and Coates (1947) is Dip t=r 2p > 0:1, which adsorbed species, Cis (mole/volume), moles of species i
often invalidates its use during initial gas sorption periods per unit mass of adsorbent, qis, is used. In such a notation,
in activated carbon and zeolites. qm
ss is the maximum value of the site concentration. The
The linear driving force approximation (3.4.36c) is also corresponding form of the rate of adsorption equation
used commonly for separation/transport between a liquid (3.4.39b) and the Langmuir adsorption isotherm (3.4.39c)
and porous adsorbent particles. In order to account for any are changed, respectively, to
deviation between the actual rate of adsorption and the 
qis qis 
rate obtained by the LDF approximation, a correction k f C if qm
ss  qis  ; 3:4:39d
t K
factor p has also been introduced:
qm
ss K C if
  p q is ; 3:4:39e
4 3 C is 15Dip p p 1 K C if
r p p 2 C is  C is : 3:4:38
3 t rp
where q is is the equilibrium concentration or saturation
On a comparison of the observed value of the rate of capacity of the adsorbent for species i for the fluid phase
uptake of species i with time (as given by the left-hand concentration Cif.
side of (3.4.36c)) and the rate predicted by the right-hand A nondimensional form of equation (3.4.39d) is fre-
side, p may be determined. Some estimates of p are quently used when qis is nondimensionalized with respect
available in Vermuelen et al. (1973). to qm
ss :

An alternative approach for estimating the rate of 0 


q 1K C
 1
qis =qm k K C if is K C if if  qis  qis
adsorption of species i on the adsorbent employs the ss
@
f A
method used in chemical kinetics to determine the rate t K qmss
of reaction as an algebraic sum of the forward and back-  
kf qis qis
ward reaction rates for the second-order reaction (whose 1 K C if m  m : 3:4:39f
K qss qss
reaction equilibrium constant is K k f =k b ) (Thomas,
1944): An alternative form of this equation for the rate of adsorp-
tion is often used. First, rewrite it as
adsorbing species i
( )
kf qis =qm
ss kf q is qis
adsorbent site s adsorbed complex is: 3:4:39a 1 K C if m  m 1 K C if :
kb t K qss qss

If Cif is the concentration of adsorbing species i in the pore 3:4:40a


fluid phase, Cis is the concentration of adsorbed species i Now define C if C ref corresponding to C ref
qref
if x if
and is if
on the adsorbent phase (s) occupying some of the sites of in the adsorption system. Then the Langmuir adsorption
the adsorbent phase, where C m ss is the maximum value of isotherm (3.4.39e) for C ref
if and Cif may be written, respect-
the site concentration (s) (first subscript) in the solid phase, ively, as
then the forward adsorption reaction rate for this second-
order reaction is kf C if C m ss  C is and the backward reaction m
K C ref
if qm ref
ss K C if x if
qref
is qss and q is ; 3:4:40b
rate is k b C is . The net rate of species i adsorption is given by K C ref 1 K C ref
1 if if x if
 
C is C is leading to
k f C if C m
ss  C is  k b C is k f C if C m
ss  C is  :
t K
3:4:39b 1 K C ref
if
q is =qref
is x if : 3:4:40c
1 K C ref
if x if
The quantity within the brackets on the right-hand side
is called the kinetic driving force; kf is the correspond- Temporarily assuming that qm ref
ss equals qis , we obtain, for
ing kinetic coefficient in this reactionkinetic treatment (3.4.40a),
(Vermeulen et al., 1973). When the rate is zero, the equi- ( )
librium concentration, C is , of the adsorbent for species i is qis =qref
is kf ref qis ref
1 K C if x if  ref 1 K C if x if :
obtained for Cif as follows: t K qis
C is C m K C if 3:4:40d
C if C m
ss  C is  0 ) C is ss : 3:4:39c
K 1 K C if
Alternative forms of this equation are used in analyzing the
This is a form of Langmuir adsorption isotherm illustrated rate of adsorption in adsorbent beds using separation
in Figure 3.3.8B and equations (3.3.112a) and (3.3.113c, d), factor relations.
168 Physicochemical basis for separation

3.4.1.5 Ion exchange resinsolution systems Table 3.4.1. A few diffusion coefficients in ion exchange systemsa

Mass transfer in the ion exchange resinsolution system is Ions Na Cs Ba Sr Mn


briefly considered here for the case where counterion AZ A
in the ion exchanger is exchanged with counterion BZ B in Infinitely dilute external 133 205 84 77.8 70.8
the external solution. An exchange of this kind may be solution,
described by five steps: Dil  107 cm2/s
Dowex 50 W-X8 resin,b 20.5 30.0 1.16 1.95 2.22
(1) the diffusion of ion BZ B from the bulk of the external DiR 107 cm2/s
solution to the outside surface of the ion exchange a
Bajpai et al. (1974).
resin particle; b
H of sulfonic acid group is the exchangeable cation.
(2) the diffusion of ion BZ B through the solvent-filled pores
of the ion exchange matrix to a site containing ion AZ A ;
(3) the instantaneous exchange of places between ions Use these three equations to eliminate r from J A . First,
B and A; substitute for J A and J B in (3.4.42) using the NernstPlank
(4) the diffusion of ion A through the solvent-filled pores expression (3.4.41) for both A and B and rearrange:
of the ion exchange matrix to the outside surface of the
resin particle; RT Z A DAR rC AR Z B DBR rC BR
r :
(5) the diffusion of ion A from the particle surface to the F Z 2A DAR C AR Z 2B DBR C BR
bulk of the external solution.
Substitute this into expression (3.4.41) for J A :
Such a scheme was proposed by Boyd et al. (1947).
Since step (3) is instantaneous, we need only consider Z 2A C AR D2AR rC AR Z A Z B C AR DAR DBR rC BR
J A DAR rC AR :
steps (1), (2), (4) and (5). However, there are essentially Z 2A DAR C AR Z 2B DBR C BR
two types of resistances here: steps (1) and (5) account
for diffusion through the liquid film on the outside of Use here Z B rC BR Z A rC AR from equation (3.4.43) and
the resin particle, whereas steps (2) and (4) describe the simplify:
diffusion of counterions through the pores of the ion
Z 2A C AR D2AR  Z 2A C AR DBR DAR
exchange resin particle. When steps (1) and (5) control J A DAR rC AR rC AR
Z 2A DAR C AR Z 2B DBR C BR
the exchange rate, the ion exchange is said to be film
" #
diffusion controlled. If steps (2) and (4) are the rate- DAR DBR Z 2A C AR Z 2B C BR
determining steps, the mechanism is particle diffusion  rC AR DABR rC AR :
Z 2A DAR C AR Z 2B DBR C BR
controlled (Helfferich, 1962).
The diffusional flux expression for particle diffusion 3:4:44
control in the exchange of counterion AZ A in the ion
An unstated assumption in this derivation was that the
exchanger with the counterion BZ B in the external solution
co-ion was excluded completely from the ion exchanger
will now be derived. The NernstPlank flux expression for
by Donnan exclusion. Expression (3.4.44) for the flux of
species A in the resin ( j R) in the absence of any
counterion A in the particle diffusion controlled exchange
pressure gradient is obtained from (3.1.106) as
of counterions A and B is the equivalent of Ficks first law;
C AR Z A F however, the diffusion coefficient, DABR for the exchange
J A DAR rC AR  DAR r: 3:4:41
RT between A and B in the resin particle is, in general, not
Since there is no electric current in the system, obtain from constant and depends on concentrations of counterions
the current density expression (3.1.108c) A and B. For example, if CAR << CBR with Z A  Z B , then
DABR DAR ; conversely, if CBR << CAR, DABR DBR ; i.e.
ZB the concentration dependence of DABR disappears. An idea
Z A N A Z B N B 0 Z A J A Z B J B ) J A  J ;
ZA B of the magnitudes of such diffusion coefficients is provided
3:4:42 in Table 3.4.1. Note that although ion exchange is usually
where N i J i due to the absence of any convection in the applied to dilute external solutions where the solution
pores of the resin particle. Further, electroneutrality must concentration is low (of the order of milliequivalents per
be maintained in the resin with a fixed molar charge dens- liter), the concentration inside the resin is of the order of
ity of X (see relation (3.3.30b)): equivalents per liter.
In film diffusion controlled situations, a co-ion is present
Z A C AR Z B C BR X : in the external solution. Consequently, the NernstPlank
Taking the gradient of this expression yields equation for co-ions needs to be considered along with the
role of co-ions in the electroneutrality and no electric cur-
Z A rC AR Z B rC BR 0: 3:4:43 rent conditions (Helfferich, 1962) in deriving the exact flux
3.4 Interphase transport: flux expressions 169

expressions for film diffusion controlled situations. There is Here, B is the bulk density of the resin particle and av is the
no general solution to this problem. Solutions for some outer particle surface area per unit particle volume. The
limiting conditions are available (Helfferich, 1962, p. 274). LDF approximation based expressions are provided next.
In a separation process, ion exchange resin particles Consider the solute concentration profile in any of the
are generally used in a column. A complex time-dependent two-phase processes considered earlier; specifically, select
differential equation for mass balance in the column has to Figure 3.4.2 with the aqueous phase representing the exter-
be combined with the diffusion flux expression for a resin nal solution and the organic phase representing the aque-
particle, and other appropriate boundary and initial condi- ous solution in the pores of the ion exchange resin particle.
tions, to determine the extent of separation. It is obvious The rate of transfer of an ionic species A from the bulk
from the preceding few paragraphs that the diffusion flux solution ( j w) to the interface between the solution and
expressions are difficult to handle for resin particles. For the resin particle is given by
ion exchange column analysis, practical approaches there-
N Ar av k cw av C Awb  C Awi ; 3:4:45e
fore utilize a linear-driving-force representation of the
mass flux to a resin particle (Helfferich, 1962; Vermeulen where av is the outer particle surface area per unit particle
et al., 1973); this leads to the use of mass-transfer coeffi- volume. For spherical particles of radius rp,
cients in resin particle flux expressions. 3
Before we identify the flux expression in terms of the av : 3:4:45f
rp
linear-driving-force (LDF) approximation (see expression
(3.4.36c) for fluidsolid adsorption), we will briefly identify Further, kcw is the external solution-phase mass-transfer
the flux expression in ion exchange employing the coefficient. At the external solutionresin boundary inter-
reactionkinetic approach already illustrated for fluidsolid face, the resin-phase concentration in equilibrium with
adsorption via (3.4.39a) for, say, a cation, Na, exchanging CAwi is CARi. The resin-phase mass-transfer expression
with a H ion in a cation exchange resin as a counterion: containing the resin-phase transfer coefficient kcR is

kf N Ar av k cR av C ARi  C ARb : 3:4:46a



Na w H R Na R H w
k b Often, concentration units of mole/mass of resin is used by
C Aw Cm
AR  C AR C AR C tAw  C Aw using qARi , qARb in the driving force expression given above.
qm
AR  qAR qAR 3:4:45a It follows that

In this ion exchange reaction, we have identified the N Ar av =B k cR av qARi  qARb ; 3:4:46b
sodium ion concentration in the resin phase on a volumet- where B is the density of the particle in the mass/volume
ric basis, CAR, as well as on a per unit resin weight basis, of the resin.
qAR; correspondingly, the H ion concentrations are For film diffusion control, expression (3.4.45e) is used.
Cm m m m
AR  C AR and qAR  qAR , where C AR and qAR represent For particle diffusion control, expression (3.4.45b) is used. In
the corresponding maximum resin ion exchange capacities general, both expressions given above are functions of time
for Na. The net rate of transfer of Na ions to the resin when used in an actual ion exchange process. For mass-
phase may be written for this second-order (pseudo) reac- transfer conditions where both film diffusion and particle
tion as (see expression (3.4.39b) or (3.4.39d)) diffusion are important, the two expressions (3.4.45e) and
" # (3.4.45b) may be combined to define an overall mass-transfer
C AR 1
k f C Aw C m
AR  C AR  C AR C t
Aw  C Aw ; coefficient Kc (see Vermeulen et al., 1973, eq. (1656)):
t K
" #
3:4:45b
1
where K is the (pseudo) equilibrium constant ( k f =k b ); K c av : 3:4:46c
1 1

alternatively, k cw av k cR av
" #
qAR m 1 t The correlations (3.1.158) by Wakao and Funazkri (1978)
k f qAR  qAR  qAR C Aw  C Aw : 3:4:45c
t K and (3.1.159) and (3.1.160) by Wilson and Geankoplis
(1966) identified in Table 3.1.6 for flow in a packed bed
However, in reality, the rates are always diffusion con- may be utilized to estimate kcw. For kcR, see equation
trolled (Thomas, 1944) and therefore depend on the par- (3.4.46d) given below.
ticle surface area available: In Vermeulen et al. (1973), the flux expression (3.4.46a)
" #
has a correction factor p to account for the deviation of
q 1
B AR k f av C Aw qm  q AR  q C t
 C
Aw : the actual driving force from the linear driving force
t AR
K AR Aw
approximation used above (originally proposed by Glueck-
3:4:45d auf and Coates (1947)). The latter had also suggested that
170 Physicochemical basis for separation

15DAR An additional category studied is a liquidporous charged


k cR av 3:4:46d
r 2p membrane in an electrical field.
We study first liquid separation through practically
for resin particles of effective radius rp and effective diffu- nonporous membranes and then we move on to porous
sion coefficient DAR of species A in resin. membranes. Of the known techniques using nonporous
membranes, (reverse osmosis, dialysis, liquid membrane
3.4.1.6 Two-phase systems with a supercritical fluid permeation and pervaporation), we select the most
common, reverse osmosis, to begin our study of inte-
The two-phase systems of interest here are SCFliquid and grated flux expression development. Pervaporation is
SCFsolid. Lahiere et al. (1987) have studied the transfer of considered next. There is one feature which is, however,
ethanol or propanol from an aqueous solution into super- common to almost all nonporous membrane processes;
critical CO2. The experimental extraction behavior in a i.e. the additional phase, the membrane phase, is sta-
sieve-tray extractor was compared with that predicted from tionary in general (except in cases of rapid transient
a model used for common liquids. This model for subcrit- membrane swelling or emulsion liquid membranes).
ical liquids uses overall mass-transfer coefficients obtained This is in contrast to molecular diffusion processes in a
from the individual film coefficients by the conventional gas or liquid where all species can diffuse (they may or
sum of the two resistances approach. Lahiere et al. (1987) may not).
observed reasonable correspondence between experimen-
tally observed values and the performance predicted from
conventional models. For SCFsolid systems with pure 3.4.2.1 Liquid permeation through nonporous membranes:
solids, Debenedetti and Reid (1986) have observed that reverse osmosis (RO) and pervaporation
the mass-transfer coefficient for the SCF phase is very Liquid permeation through practically nonporous mem-
strongly influenced by natural convection. This happens branes can take place under a variety of conditions. These
because the SCF has very low viscosity and yet has a high conditions are defined by the nature of the phase on the
density, leading to a much more important role of natural downstream side of the membrane (liquid or vapor/gas),
convection than in normal liquids. the nature of the two components in the feed mixture
(volatile or nonvolatile) and the level of pressure on the
feed side and that on the permeate side (especially includ-
3.4.2 Interphase transport: membranes
ing a vacuum). Of the number of combinations possible
There are a variety of membrane structures available. from this set of variables, the following combinations are of
Correspondingly, the variety of transport expressions or principal interest to us here.
transport models is considerable. The nature and magni-
(1) Reverse osmosis: feed liquid at high pressure per-
tude of the driving forces and the frictional resistances can
meate liquid at lower pressure (with or without a non-
vary, leading to enormous variations in individual species
volatile solute species in the feed).
fluxes. It is not possible to cover the whole spectrum of
(2) Pervaporation: feed liquid at atmospheric pressure
such behavior here. Rather we focus on a few cases of con-
permeate in vapor phase at low pressure (feed liquid
siderable use in practical separation techniques/processes
components are volatile).
to illustrate integrated flux expressions for membranes.
(3) Liquid permeation: feed liquid at atmospheric pres-
A membrane separates a feed mixture which can be
sure permeate liquid at atmospheric pressure (feed
either liquid or gaseous. In general, the product phase is
liquid components volatile/nonvolatile).
the same as that of the feed and the concentrate/residue/
raffinate/retentate. There are a number of exceptions. For Detailed considerations on permeation rates for a volatile
example, a separate sweep, extract or purge stream of a feed mixture for a variety of physical states of the permeate
different phase may be introduced on the other side of the with respect to the liquid feed are available in Greenlaw
membrane. Alternatively, the permeated feed may be in a et al. (1977) and Shelden and Thompson (1978).
gaseous/vapor phase from a liquid feed. The membrane We focus first on reverse osmosis. Consider the trans-
can be nonporous or porous from a gross structural point port of a salt and water through a homogeneous nonpor-
of view. Therefore, we can focus on the following four ous membrane (Figure 3.4.6) kept between two well-stirred
broad feed phasemembrane type categories: (1) liquid salt solutions, a dilute product solution of salt concentra-
nonporous; (2) gaseousnonporous; (3) liquidporous; tion Cip at a pressure Pp and a more concentrated feed
(4) gaseousporous. solution of salt concentration Cif at a pressure Pf (>Pp). If
Figure 3.4.5 illustrates schematically the four broad pressure Pf exceeds Pp by an amount larger than the
feed phasemembrane type categories; it includes some osmotic pressure difference of the two salt solutions, water
subcategories based on different permeate phases and will be transported from the high-pressure feed solution
driving forces excluding electrical potential gradient. through the membrane to the low-pressure product
3.4 Interphase transport: flux expressions 171

(a) Feed : liquid Feed : liquid Feed : volatile liquid


Pf , p f P f , Ci Pf
z =0
Transport z z z
direction z = dm
Pp , p p Pp , Cip Pp
Permeate : liquid Permeate : liquid Permeate : vapor
Pp << Pf : p p < p f Pf Pp : Cif > Cip

1 Reverse osmosis 2 Liquid permeation 3 Pervaporation


D P = Pf Pp : Dp = p f p p aip < aif

(D P Dp) > 0

(b) Feed : gaseous Feed : vapor with/without gas


Pf , xif Pf , xif
z =0
Transport z z
direction z = dm
Pp , xip Pp , xip
Permeate : gaseous Permeate : vapor with/without gas
1 Gas permeation 2 Vapor permeation
Pf xif > Pp xip Pf xif > Pp xip

(c) Feed : liquid Feed : liquid

Pf , C 0if Pf , Cif , C 0il


z =0
Transport
z z
direction z = dm
Pp , C dil Pp , Cip , C dil

Pf > Pp Permeate : liquid Permeate : liquid


Cip < Cif : Pf Pp
1 Ultrafiltration/microfiltration 2 Dialysis/diffusion

(d) Feed : gaseous Pf

z Pp Pf

Feed : gaseous Pp
1 Gaseous diffusion/convection

Figure 3.4.5. Schematics of four broad feed phasemembrane type categories including subcategories based on permeate phases and
driving forces (electrical potential gradient excluded); membrane based two-phase contacting is not included here. (a) Liquid feed
nonporous membrane; (b) gaseous feednonporous membrane; (c) liquid feedporous membrane; (d) gaseous feedporous membrane.

solution; the phenomenon is called reverse osmosis (RO) membranes are highly permeable to water and not so
(see also Figure 1.5.1). How much salt will also go to the much to salt. In this example, one species in the feed
low-pressure product solution depends, amongst other (namely salt) is nonvolatile. However, the following treat-
things, on how permeable the membrane is to the salt. ment is also going to be valid for volatile solutes provided
In water desalination using a reverse osmosis process, the permeated phase is liquid.
172 Physicochemical basis for separation

used. Inside the membrane phase, r0i 0. Membrane


Membrane
concentrations are indicated by Csm and Cim for the solvent
and solute, respectively. For the solvent (subscript, s)
   
a dsm C sm V s dP RTC sm d n asm
Feed solution J sz  d  d : 3:4:47
f sm dz f sm dz
Cif
For solute species i,
as0m    
C im V s dP RTC im d n aim
Pp ( < Pf ) J iz  d  d : 3:4:48
Pf f im dz f im dz
Ci0m
In terms of diffusion coefficients, the frictional coefficients
may be replaced by
Increasing Cip
Ci 1 Dsm RT
and Dim : 3:4:49
f dsm RT f dim

We could have also obtained the two flux expressions given


Cidm above by simply considering the uncoupled flux of any
Increasing
solute in the solvent from the irreversible thermodynamic
asm
formulation based expression (3.1.208). But the solvent in
(3.1.208) is the membrane here. Further, both species
being transported, the solute i and the solvent s, are simply
solutes:
z=0 z=d
J i Lii F i Lii ri ; 3:4:50
z
where
Direction of solvent and solute transport through
membrane in reverse osmosis Lii Dim C im =RT: 3:4:51

Figure 3.4.6. Solvent and solute transport through a membrane in Integrate now the flux expression (3.4.47) from z 0 to
reverse osmosis with well-mixed feed and permeate solutions. z m , assuming CsmDsm and V s to be constants, to obtain
at steady state
We ignore the problems, if any, of the transport of  
solute i and solvent s in the feed solution and the product C sm Dsm V s C sm Dsm asm jm
J sz P f  P p  n :
solution for the time being by assuming them to be well- RTm m asm j0
mixed. In the three-component system of solutesolvent 3:4:52
membrane, the membrane is assumed stationary. Both
Now asm jm asp and asm j0 asf , corresponding to the
solvent and solute diffuse through the membrane under
product and feed solutions. However, relation (3.3.86a)
the driving force of a chemical potential gradient across the
for osmotic equilibrium between two solutions of different
membrane. Such a condition is best studied at first by
solvent activities and osmotic pressures provides
using the solution-diffusion theory (Lonsdale et al., 1965).  
Both the solvent and the solute are assumed to be dis- asp
V s f  p RTn : 3:4:53
solved in the membrane at the high-pressure feed asf
solutionmembrane interface where the species concen-
Therefore
trations in the two phases are related by equilibrium. They
then diffuse through the membrane, each under its own C sm Dsm V s
J sz P f  P p  f  p 
chemical potential gradient, and are finally desorbed into RTm
the product solution at the low-pressure solution C sm Dsm V s Q
membrane interface. The system should then be analyzed fP  g sm P  AP  ;
RTm m
as if we have two separate binary systems: solvent
membrane and solutemembrane. The membrane thick- 3:4:54
ness is m in the z-direction, the direction of transport. where
This is a case of diffusion in the membrane. Therefore,
P P f  P p and f  p :
we use J i N i  C i v t and equation (3.1.84a) for N i for
both i and s. Note that no external forces are present. But, This is an integrated flux expression for the solvent
since P is nonzero, expression (3.1.84d) for U i has to be permeating through the reverse osmosis membrane in
3.4 Interphase transport: flux expressions 173

terms of bulk quantities like P and obtainable out- (2) There is equilibrium partitioning of species between
side the membrane with a mass-transfer coefficient-like the solution and the membrane at both interfaces (the
proportionality constant, Qsm =m . Here Qsm is the two partition coefficients were assumed here to be
permeability coefficient of the solvent through the mem- equal; in general, they are different).
brane of thickness m . In reverse osmosis literature, the (3) The feed and the product phases here are miscible,
quantity Qsm =m is often called the pure water permea- unlike conventional interphase transport in two-phase
bility constant A. It is essentially a mass-transfer coefficient. systems (liquidliquid, gasliquid, etc.), which are
Note: For water to permeate through the membrane immiscible.
from feed to the permeate, P > . (4) Species transport in both feed and product phases
In expression (3.4.48) for the solute flux through the have to be considered in general.
reverse osmosis membrane, the first term is usually unim-
Both J sz and J iz are diffusive fluxes through the mem-
portant (Merten, 1966). Therefore, the vectorial expression
brane. If there are large pores in the membrane, there will
for the solute flux is given by
be convection through such pores or defects. The total flux
 
n aim of any species will no longer be completely diffusive and
J i C im Dim rn x im : 3:4:55
n x im T;P therefore should be expressed in terms of N i (Soltanieh
and Gill, 1981). A simple model for the fluxes through a
Assume ideal solution in the membrane and a constant reverse osmosis membrane having large pores (or defects
total molar density Ctm across the membrane. Then or imperfections) has been provided by Sherwood et al.
C im Dim (1967); it is called the solution-diffusion-imperfection
J i  rx im Dim rC im : 3:4:56 model:
x im

For one-dimensional steady state transport in the z-direc- N sz J sz Ap PC sf AP  Ap PC sf ; 3:4:60a


tion, on integrating J iz one obtains where Csf is the solvent concentration on the high-pressure
Dim 0 feedmembrane interface for a well-mixed feed. Thus there
J iz C im  C im ; 3:4:57 is simply Darcy or Poiseuille flow through the defects (see
m
Section 3.4.2.3 for these types of solvent flow), and the
where C 0im and C im are the solute concentrations in the feed solution at the feedmembrane interface moves
membrane at the two solutionmembrane interfaces. unchanged through the defects. Similarly, for the solute,
The solute distribution coefficient i may now be used
to relate the membrane concentration to the external solu- Dim im
N iz J iz Ap PC if C if  C ip Ap PC if ;
tion concentration at the two interfaces because of m
equilibrium: 3:4:60b

C 0im C where Cif is the solute concentration on the high-pressure


if ; ip im : 3:4:58 feedmembrane interface for a well-mixed feed. There are
C if C ip
three transport coefficients in this model, A, Dim im =m
If if ip im , then and Ap; the solution-diffusion model had only two trans-
port coefficients, A and Dim im =m .
Dim im
J iz C if  C ip : 3:4:59 In practical reverse osmosis, molar fluxes are not
m
usually measured; volume fluxes are. The volume flux
The product, Dim im , is a permeability coefficient of solute i (in units of cm3/cm2-s), often denoted by Jvz, is defined by
through the membrane of thickness m . The product
Dim im =m has been called the solute transport para- J vz J sz V s J iz V i 3:4:60c
meter in reverse osmosis literature (Sourirajan, 1970). 30
for conditions when diffusive models hold. When there is
Expression (3.4.59) is an integrated flux expression for some convection through the membrane defects as well,
solute transport through a reverse osmosis membrane in the volume flux measured is to be obtained from the sum
terms of the differences in concentrations of the external of Nsz and Niz. In most cases, Niz << Nsz. Thus Nsz may be
solutions and a mass-transfer coefficient-like quantity used to determine the volume flux. In general, if V s is used
Dim im =m . to indicate the partial molar volume of the permeate, then
A few distinguishing features of transport of any the volume flux through the reverse osmosis membrane
species through a membrane with respect to interphase can be expressed by
transport in two-phase systems are as follows.
(1) There are two immiscible phase interfaces: the feed
membrane interface and the membraneproduct 30
Here V s and V i are the partial molar volumes of the solvent
interface. and the solute, respectively.
174 Physicochemical basis for separation

  17:7
cm3 C if 1  17:7  103  103 ) C if
volume flux N sz N iz V s : 3:4:60d 18:05
cm2 s
17:7  103 17:7 gmol salt

Note that if one were to define a permeation velocity vz 18:05  0:9823 18:05  0:9823 liter
(cm/s) through the membrane, then 58:4 g salt=liter:

vz N sz N iz V s J sz J iz V s : 3:4:60e (2) x ip 0:4  103 C ip =C ip C sp . This permeate is so


dilute that we incur very little error by writing
This relation is to be used only if diffusive fluxes exist. In
1 gmol 0:4 gmol salt
practical reverse osmosis, C ip 0:4  103 C sp 0:4  103  :
18:05 cm3 18:05 liter
vz N sz V s J sz V s : 3:4:60f Now we can obtain the solvent flux:

For solutions that are dilute, the concentration of species i J sz AfP  C if C ip g


( )
may be represented by 7 C ip gmol H2 O
8:03  10 102  45:7 C if
C if cm2 -s
C ij C t x ij ; 3:4:61a
(  )
0:4  18:05  0:9823
where Cij is the molar concentration of species i in region j 8:03  107 102  45:7 1 
18:05  17:7
( j f or p), xij is the corresponding mole fraction and Ct is
the molar density of the solution, assumed constant. The 8:03  107 f102  45:7  0:978g
osmotic pressure of dilute solutions containing a single 8:03  107  102  44:69
solute may also be represented by gmol H2 O 4:6  105  18:05 cm3
4:6  105
C ij bC ij ; 3:4:61b cm2 -s cm2 -s

liter cm2 s
where i refers only to the solute species. Using these 103  104 2  3600  24
cm3 m day
simplifications, it is now possible to rewrite the solution-
diffusion model fluxes for solvent and solute in reverse liter
4:6  105  18:05  10  3600  24
osmosis (expressions (3.4.54) and (3.4.59)) as m2 -day

J sz AP  bC t x if  x ip ; 3:4:62a liter H2 O
717 ;
  m2 -day
Dim im
J iz C t x if  x ip : 3:4:62b 0 1
m
Dim im
J iz @ AC if  C ip
m
0 1
Example 3.4.2 Earlier literature on reverse osmosis desalin- 3
5 @ 17:7  10 0:4  103 A
ation of seawater at a high P 102 atm provided the 1:774  10 
following information for a cellulose acetate membrane: A = 18:05  0:9823 18:05
8.03  107 gmol H2O/cm2-s-atm; C if bC if 45:7 atm; 1:774  105  103 1:774  108 gmol
Dim im =m for salt 1.774 105 cm/s; xif 17.7  103; 18:0  0:4  17:6 2
18:05 18:05 cm  s
xip = 0.4  103. The membrane has a high rejection for the 1:774  17:6 gmol
solute i, which is NaCl. Calculate the values of water flux  108  104  3600  24 2
18:05 m day
and the salt flux through the membrane. Calculate the salt
concentration on two sides of the membrane, assumed to be gmol salt
14:94 2 ;
of the solution-diffusion type. Calculate the salt rejection R. m day
(Note: Current membranes are more productive. Therefore
P is around 54.468 atm (8001000 psi). One should calcu- C ip 0:4  103 =18:05
R1 1
late based on the final concentration of seawater desired.) C if 17:7  103 =18:05  0:9823

Solution Assume that the osmotic pressure of dilute 0:4  0:9823


1 1  0:022 0:978:
solutions can be represented
 by C ij bC ij . Therefore 17:7
C ip bC ip C ip C if =C if . We will need to calculate
For a dilute solution having more than one solute (say two
(1) Cif and (2) Cip to determine C ip .
(1) Now solutes, 2 and 3), the osmotic pressures may be represented by

C if C if f bC 2f C 3f and p bC 2p C 3p : 3:4:62c
x if 17:7  103 1 ;
C if C sf C if 18:05
Using the approximation (3.4.61a) representation for each
where the partial molar volume of pure water is 18.05 cm3/ solute species 2 and 3 (say), one can develop flux expres-
gmol (V s 1=C sf ). We have sions for J sz ; J 2z and J 3z as follows:
3.4 Interphase transport: flux expressions 175

J sz AP  bC t x 2f x 3f bC t x 2p x 3p ; 3:4:63a Membrane
 
D2m 2m
J 2z C t x 2f  x 2p ; 3:4:63b
m Feed solution Ci0l d
a sm Permeate solution
  Cif
D3m 3m
J 3z C t x 3f  x 3p : 3:4:63c Pf
m
as0m Pp (< Pf )
There are a number of other models of transport of solvent
and solute through a reverse osmosis membrane: the Ci0m
KedemKatchalsky model, the SpieglerKedem model,
Increasing Cip
the frictional model, the finely porous model, the prefer- Cil and as
ential sorptioncapillary flow model, etc. Most of these
models have been reviewed and compared in great detail
0
by Soltanieh and Gill (1981). We will restrict ourselves in A Ci l
Cidm
Species i diffusive flux out A
this book to the solution-diffusion and solution-diffusion- Cil Permeating flux out for species i
Species i convective flux in
imperfection flux expressions for a number of reasons. Cif B
B
First, the form of the solution-diffusion equation is most y y=0
d1
commonly used and is also functionally equivalent to
z=0 z = dm
the preferential sorptioncapillary flow model. Secondly,
the solution-diffusion-imperfection model is functionally z
representative of a number of more exact three-transport- Direction of solvent and solute transport through
membrane in reverse osmosis
coefficient models, even though the transport coefficients
in this model are concentration-dependent. Figure 3.4.7. Transport in reverse osmosis with concentration
So far, the integrated flux expressions for the transport polarization.
of solvent and solute through a membrane in reverse
osmosis assumed well-mixed feed and permeate solutions.
Therefore any transport resistances in the two liquid The relationship between C 0il and Cif may be obtained by
phases on two sides of the membrane were eliminated. considering the transport of solute i through the mem-
In practical reverse osmosis, there is, however, significant brane, the diffusion of solute i from the membranefeed
transport resistance for salt on the feed side. This subject solution interface region to the bulk solution and the
will be briefly treated now. volume flux through the membrane. Such a relationship
In practical reverse osmosis with a positive P  , will be derived next. Note: If the mixing in the feed solution
there is considerable flow of solvent from the feed to the is vigorous enough to make C 0il C if , then such a relation
permeate. However, the membrane is designed to reject is no longer needed.
the solute species. Thus, from the feed solution next to the Consider a control volume A0 B0 BA in the feed solution
membrane, solvent is continuously withdrawn through the film region (see Figure 3.4.7). Let the surface A0 B0 be
membrane, whereas the solute species is not. This leads to located at a distance y from the feedmembrane interface.
a build-up of solute concentration near the membrane Then, at steady state, the rate at which solute i enters the
feed solution interface (Figure 3.4.7) in excess of the bulk control volume via A0 B0 must equal the rate at which it
feed solute concentration Cif. This phenomenon is called leaves the control volume surface AB, which coincides with
concentration polarization. The feedmembrane interface the feedmembrane interface. There are three sources of
is now exposed to a solute concentration C 0il instead of solute flux for the control volume:
C if
C 0il . Consequently, the solvent and solute flux
( dC il
)
expressions in the solution-diffusion model for one solute diffusive flux out Dif
in a solvent system are changed to at y y surface A0 B0 dy
:
convective flux in jvy jC il
J sz AP  f0f  p g AP  bC 0il bC ip at y 0 surface AB permeating flux out jvy jC ip
AP  bC t x 0il bC t x ip ; 3:4:65a
3:4:64a
Therefore
 
Dim im Dim im 0 dC il
J iz C t x 0il  x ip C il  C ip : 3:4:64b jvy jC il  jvy jC ip Dif ; 3:4:65b
m m dy
The solvent flux is decreased now and the solute flux is where jvy j is the magnitude of the permeation velocity
increased. It is desirable to know the fluxes under concen- through the membrane. We have assumed it to be
tration polarization for which C 0il has to be determined. equal to the velocity normal to the membrane in the
176 Physicochemical basis for separation

liquid film region. If equation (3.4.65b) is rearranged and A major difference between the solute transfer from
integrated from y 0 to y l , the thickness of feed side feed solution to permeate solution through a membrane in
liquid film, then reverse osmosis (RO) and the interphase solute transfer
shown in Figures 3.4.1 and 3.4.2 is the following: even
Cif l
dC il jvy j though there is some solute transfer through the RO mem-
 dy; brane, the solute concentration builds up at the feed
C il  C ip Dif
C 0il 0 membrane interface. It is possible to have such a low value
  of k il and such a high value of C 0il that P , and the
C 0il  C ip jvy jl jvy j jvy j
n ; reverse osmosis process stops. Such a situation is unlikely
C if  C ip Dif Dif =l k if to be encountered in conventional nonmembrane inter-
where k if Dif =l is the mass-transfer coefficient in the phase transfer processes.
feed solution. So
3.4.2.1.1 Pervaporation and liquid permeation When a
C 0il  C ip C if  C ip expjvy j=k if : 3:4:65c
volatile liquid mixture is imposed on the feed side of a
By relation (3.4.60e), jvy j is obtained from the two fluxes nonporous membrane and the other side of the mem-
J sz and J iz (or N sz and N iz ) expressed by (3.4.64a) and brane has a vapor/gaseous phase, the process is called
(3.4.64b). Thus J sz and J iz may be calculated in terms pervaporation, a combination of permeation and evapor-
of the two bulk solute concentrations Cif and Cip by ation, permeation through the membrane from the feed
solving the three equations simultaneously. If J sz and J iz side and evaporation on the other side (Figure 3.4.5(a)).
are to be expressed only in terms of Cif, an additional (When the phase on the other side of the membrane is a
relation is needed to express Cip in terms of the other liquid, the process is called liquid permeation.) The rate of
quantities. transport of a feed species through a nonporous amorph-
It is obvious that the flux expressions (3.4.64a) and ous polymeric film is apparently a special case of ordinary
(3.4.64b) are of an implicit type; both J sz and J iz depend diffusion through the film, except the species diffusivity is
on C 0il , which in turn depends on J sz and J iz . It is possible, highly concentration-dependent due to the swelling of the
however, to have explicit flux expressions under the polymer by the components of the feed solution (Long,
limiting conditions of C ip  C 0il and jvy j  k if . Then equa- 1965). At steady state, the permeation flux of a species in
tion (3.4.65c) may be reduced to the z-direction across the film thickness is described by
  Ficks first law (as long as the pressure difference between
jvy j the two sides is small):
C 0il C if 1 : 3:4:65d
k if
dC im
Further, in practical reverse osmosis, J iz Dim : 3:4:67a
dz
vy N sz V s J sz V s : The value of Dim depends strongly on the species i concen-
tration in the membrane (acting as a solvent swelling the
Therefore,
membrane polymer). This concentration dependence has
( )!
J sz V s been expressed in a number of ways:
J sz A P  bC if 1 ;
k if Dim Dim0 expi C im ; Dim Dim0 C im ;
leading to Dim Dim0 1 i C nim : 3:4:67b

AP  bC if In the first and third expressions, the value Dim0 of the


J sz ; 3:4:66a
1 AbC if V s =k if diffusivity of the species i in the membrane is obtained
in the limit of zero concentration of species i in the
as demonstrated by Rao and Sirkar (1978) for tubular RO
membrane. The quantity i is essentially a plasticization
desalination and Sirkar et al. (1982) for spiral-wound mem-
constant (Long, 1965); it illustrates the magnitude of the
branes. (See Section 7.2.1.2 for these separation devices.)
effect of solvent concentration on the diffusive mobility of
Similarly,
species i in the membrane. When i is large, a small value
 (  )
Dim im J V s of Cim can cause a large change in Dim in the case of
J iz C if 1 sz : 3:4:66b exponential concentration dependence.
m k if
Integrate now equation (3.4.67a) using the expo-
Prasad and Sirkar (1985) may be consulted for details of nentially concentration-dependent diffusion coefficient
such approximations for multisolute systems as well as for expression from (3.4.67b) between the limits of z 0 (feed
earlier references providing a quantitative basis for these mixture) and z m (permeate/product, gaseous/vapor
approximations in a single-solute system. phase):
3.4 Interphase transport: flux expressions 177

h    i
Cim
im J H2 O 0:002048 exp 11:155H2 O; m 0 C2 H5 OH; m j0  1 :
J iz dz Dim0 expi C im dC im
However, C2 H5 OH;m j0 may be neglected.
0 C 0im
Determine the value of DH2 O m0 and the plasticization con-
Dim0  
stant for water. The membrane thickness is 2:5 m;
) J iz expi C 0im  expi C im ; 3:4:67c
i m V H2 O 18:05 cm3 =gmol.
Dim 0 Solution Flux expression (3.4.67h) may be written in terms
J iz C im  C im ; 3:4:67d
m of volume fraction of water in the membrane H2 Om as
follows (i 1 H2O):
where
  D1m0  
0
J 1z exp1 01m  exp1 1m :

Dimo expi C im  expi C im V 1 1 m
Dim 3:4:67e
i C 0im  C im But exp1 H2 Om 1 since H2 Om ! 0 under high vacuum.
Therefore
is the effective Ficks law diffusivity under the conditions
D1m0  
valid for the system. J H2 Oz J 1z exp1 01m  1 :
V 1 1 m
This analysis (Long, 1965) is considered quite useful
for a single-solvent species (see Example 3.4.3). However, We get 1 11:155: Therefore
when the feed is a mixture, the presence of both solvent D1m0 mol
species should be considered. Use now expression (3.4.48) 0:002048 2
V 1 1m  11:155 m -s
without the pressure gradient term:
gmol cm3 m3
d n aim ) D1mo 0:002048 18:05  106
J iz Dim C im : 3:4:67f m2 -s gmol cm3
dz
11:155  2:5  106 m
For a binary feed liquid mixture (i =1, 2) and the polymer
) D1m0 36:9  1015  11:155  2:5
membrane (i m) constituting a ternary polymeric system,
1:02  1012 m2 =s:
FloryHuggins theory (Flory, 1953) should be used to express
n a1m for i =1 (similarly for i 2) as ( denotes a volume 3.4.2.2 Gas/vapor permeation through nonporous
fraction): membranes

V1 V1 Three types of nonporous membranes are relevant here:


n a1m n 1m 1  1m  2m  m
V2 Vm polymeric, inorganic and liquid. We start this section with
polymeric membranes, which are used most often. Later,
V1
12 1m 1m m  2m m  2m : we briefly treat inorganic and liquid membranes.
V 2 2m m We observed in Section 3.3.7.3 that permanent gases
3:4:67g dissolve in a nonporous polymeric rubbery membrane
according to Henrys law as if the membrane were a liquid.
Here, im is the volume fraction of i in the membrane, m is
The diffusion of dissolved gases in such a membrane is
the polymer volume fraction in membrane and ij is the
described by Ficks first law of diffusion, exactly as if the gas
Flory interaction parameter between species i and j. Fur-
were diffusing in a liquid by ordinary diffusion:
ther, Dim is a function of 1m and 2m . For detailed analysis,
see Neel (1995). If Cim in (3.4.67a) is expressed as im =V i J i Dim rC im : 3:4:68
and Dim in (3.4.67b) is expressed as Dimo exp(i im), then
For one-dimensional transport in the positive z-direction
expression (3.4.67c)) is reduced to
(Figure 3.4.5(b)) through a membrane of thickness m
Dimo exposed to a gas mixture with a partial pressure
J iz expi oim  expi im : 3:4:67h
i V i m pif P f x if of species i at z 0 and to a gas mixture having
a lower partial pressure pip P p x ip of species i at z m ,
Example 3.4.3 Schaetzel et al. (2004) have studied the per- dC im
vaporation of a waterethanol mixture through a J iz Dim : 3:4:69
dz
polyvinylalcohol based membrane as a function of feed water
concentration at 60  C. They have also measured the equi- Integrating at steady state across the membrane thickness
librium volume fraction of water and ethanol in the mem- m , one obtains
brane for such feed mixtures. When there is very high
Dim 0 Dim
vacuum in the permeate side, it is possible to correlate the J iz C im  C im C im ; 3:4:70
m m
water flux J H2 O (mol/m2-s) with the feed side water and
ethanol volume fractions H2 O;m j0 ; C2 H2 OH;m j0 , respectively, where C im is the concentration difference of species i
in the membrane in the following fashion: across the membrane thickness.
178 Physicochemical basis for separation

Using Henrys law (3.3.80), where Sim is constant in the Ideally, it is necessary to use mass-transfer equations
partial pressure range pif to pip pif > pip , in the feed-gas phase and the product-gas phase along
with the permeation equation (3.4.72). However, in gen-
C 0im Sim pif ; C m Sim pip : 3:4:71
eral, the gas permeation rate through a nonporous mem-
Then the flux J iz may be expressed as brane is so slow that mass-transfer equations in the feed-
gas and product-gas phases are not needed. Note that, for
Dim Sim Q Q species i transport through the membrane from the feed to
J iz pif  pip im pif  pip im pi : 3:4:72
m m m the product gas, pif > pip, but Pf need not be greater than Pp,
Here, Qim is the permeability coefficient of species i through although in practice it generally is. In such a case, flux
the membrane and is a product of Dim, the diffusion coef- expression (3.4.72) may also be expressed as
ficient, and Sim, the solubility coefficient, of species i in the Qim Q
membrane. For permanent gases and rubbery nonporous J iz p  pip im P f x if  P p x ip ; 3:4:76
m if m
polymeric membranes, Qim for any species is constant at
where xif is the mole fraction in the feed mixture and xip is
any temperature. The unit commonly used for Qim is
the mole fraction in the product mixture.
cm3(STP)-cm/cm2-s-cm Hg. The unit barrer, often used
If the gas is condensable and/or we have a vapor in the
for Qim, is 1010 cm3(STP)-cm/cm2-s-cm Hg; cm3(STP) is
gas mixture, the solubility of the permeating species in the
sometimes written as scc. The units for m ; pi and J iz are
polymeric membrane is higher. The condensable organic/
cm, cm Hg and cm3(STP)/cm2-s, respectively. In general,
inorganic vapor/gas may plasticize the membrane; the poly-
the gas concentration in the membrane at any interface
meric chains have higher segmental mobility. As a result, the
(z 0 or m ) is related to the gas partial pressure pi by
diffusion coefficient is increased drastically. The diffusion
C im Sim C im pi ; 3:4:73 coefficient Dim can vary with the concentration Cim of species
in the membrane linearly, or even higher, exponentially:
where the solubility coefficient Sim(Cim) is a function of the
concentration Cim. For low levels of Cim, with a number of Dim Dim0 expAi C im : 3:4:77a
permanent gases like H2, He, Ar, O2, N2, CH4, at T > Tci
(critical temperature of gas i), Henrys law is observed Assuming the solubility coefficient of species i to be con-
(Stern and Frisch, 1981) and stant, Sim0, corresponding to Henrys law, the permeability
coefficient Qim may be expressed as
lim Sim C im Sim ; a constant: 3:4:74
C im !0 Qim Sim0 Dim0 expAi C im Qim0 expAi C im ; 3:4:77b

(For deviations from this behavior, see equations (3.3.81) where Qim0 Sim0 Dim0 . This exponential variation of the
and (3.3.82a, b). permeability coefficient is observed at high partial pres-
Similarly, the diffusion coefficient Dim of species i in sures of condensable gas/vapor species i in rubbery
the membrane is also generally dependent on Cim, but amorphous membranes. Introducing such a variation of
Dim with Cim into Ficks first law, we get
lim Di C im Dim ; a constant: 3:4:75a
C im !0
dC im dC im
J iz Dim Dim0 expAi C im : 3:4:77c
When such a limiting behavior does not hold, Qim is dz dz
defined by Integrate it from z 0 to z m to obtain
m Dim0 h i
1 J iz expAi C 0im  expAi C im
Qim  Dim dC im : 3:4:75b A i m
pif  pip
0 Dim0 h i 3:4:77d
expAi Sim0 pif  expAi Sim0 pip :
This process of gas transport through a membrane is called Ai m
permeation, and the mechanism has been identified as
solution-diffusion. Gas species i dissolves at the feed One could rewrite this as
membrane interface (z 0); by molecular diffusion, the dis- Dim0 h i
J iz expAi Sim0 pi  1 expAi Sim0 pip : 3:4:77e
solved gas molecules move through the membrane and are Ai m
finally desorbed into the product gas phase at the product
membrane interface z m . Under the simplest of condi- Taking logarithms of both sides and plotting log J iz
tions, each species in a mixture diffuses independently of the against pip,
" #
others according to flux expression (3.4.72). The nature of the
Dim0
dependence of Dim on the effective diameter of the gas mol- log J iz log fexpAi Sim0 pi  1g Ai Sim0 pip ;
Ai m
ecules, the temperature and the polymer for an activated diffu-
sion is illustrated in Section 4.3.3 for an amorphous polymer. 3:4:77f
3.4 Interphase transport: flux expressions 179

a straight line results; the slope yields Ai Sim0 and the inter- For the permeation of a binary gas mixture through a
cept can be used to determine Dim0. The permeability glassy polymeric membrane, Koros et al. (1981) have
coefficient Qim will therefore be a strong function of pi . developed expressions for the permeability coefficient of
Knowing Sim0, the slope yields Ai. each species (their equilibrium behavior is described by
For gas transport through nonporous polymeric mem- (3.3.82a,b)):
branes of the glassy type (i.e. the temperature of permeation, "   0
T < Tg, the glass transition temperature of the polymer), it D1H C H1 b1 p1f =p1f  p1p
has been postulated that the dissolved gas species exists in D1D S1m
Q1m S1m D1D 1
the membrane in two forms: the Henrys law species and 1 b1 pf b2 p2f
the Langmuir species. The Henrys law species dissolves   0 #
according to the Henrys law and diffuses in the manner D1H C H1 b1 p1p =p1f  p1p
described earlier. The Langmuir species dissolves according D1D S1m
 : 3:4:81b
to the Langmuir isotherm (see equation (3.3.81)); further, it 1 b1 p1p b2 p2p
has a limited mobility at a fraction of that of the Henrys law
species (Paul and Koros, 1976). One-dimensional transport The expression for Q2m is
of gas molecules i through such a nonporous glassy poly- "   0
meric membrane according to this dual sorptiondual D2H C H2 b2 p2f =p2f  p2p
transport model is then described by D2D S2m
Q2m S2m D2D 1
1 b1 p1f b2 p2f
C d C H
J iz DiD im  DiH im N iz ; 3:4:78
z z   0 #
D2H C H2 b2 p2p =p2f  p2p
where DiD is the diffusion coefficient of the Henrys law D2D S2m
species whose concentration in the membrane is C dim ; DiH 
1 b1 p1p b2 p2p
: 3:4:81c
is the diffusion coefficient of the Langmuir species of molar
concentration C H im and DiH =DiD
1.
Paul and Koros (1976) have solved the one- The permeation flux expressions (3.4.76) and (3.4.81a) are
dimensional unsteady state diffusion equation (use equa- valid for membranes whose properties do not vary across
tion (3.2.2); alternatively simplify equation (6.2.3a)), the thickness. Most practical gas separation membranes
have an asymmetric or composite structure, in which the
C im N iz properties vary across the thickness in particular ways.
 ; 3:4:79
t z Asymmetric membranes are made from a given material;
for diffusion of pure species i through the glassy membrane. therefore the properties varying across m are pore sizes,
They assumed that the ratio DiH =DiD was constant; further, porosity and pore tortuosity. Composite membranes are
C im C dim C H d H
im . Additionally, the C im and C im species are in made from at least two different materials, each present in
equilibrium with each other. For the limiting case of C im 0, a separate layer. Not only does the intrinsic Qim of the
i.e. pip= 0, the species profile in the membrane is linear, and material vary from layer to layer, but also the pore sizes,
the permeability of pure species i, Qim, was found to be porosity and pore tortuosity vary across m . At least one
0   1 layer (in composite membranes) or one section of the
DiH C 0Hi bi
membrane (in asymmetric membranes) must be nonpor-
N iz m N iz m B DiD Sim C
Qim Sim DiD B C: ous for efficient gas separation by gas permeation. The flux
p

Pf @ 1 1 bi p A
if if expressions for such structures can be developed only
when the transport through porous membranes has been
3:4:80 studied.
The quantities C0Hi , bi and Sim are available from the Flux expressions (3.4.72), (3.4.76) and (3.4.81a) for a
membranegas equilibrium behavior (3.3.81). diffusing gas species i through a membrane of thickness m
Note the difference between this permeability expres- describe the observed behavior at steady state achieved
sion and that for Qim for rubbery polymeric membranes after an initial unsteady period. The initial unsteady behav-
( Sim DiD with DiD Dim ). As the feed pressure increases, ior begins when the gas containing the permeating species
the permeability coefficient in the glassy membrane i at concentration Cif (partial pressure pif) is introduced at
decreases. The permeability coefficient has the highest time t 0 to the z 0 surface of the membrane, the feed
value in the limit of pif P f ! 0. The integrated flux side. The rate of penetration of the membrane by species i
expression continues to be simply given by is governed by the unsteady state diffusion equation
(3.4.79), where N iz J iz is governed by Ficks first law
Qim (3.4.68). After some time, the species appears through the
N iz p  pip : 3:4:81a
m if membrane into the permeate side, where z m . For a
180 Physicochemical basis for separation

constant value of Dim, the solution of equation (3.4.79) may general, be at least two to three orders of magnitude larger
be obtained for the downstream boundary condition of than that through a rubbery polymeric membrane. Further-
pip 0C ip 0; from this solution one can calculate the more, the magnitude of the solubility coefficient of the gas
moles of the diffusing species, miD jt , that have crossed the in the liquid will be larger. However, the liquid layer thick-
membrane in time t from t 0 to be (Crank and Park, ness will, in general, be considerably higher.
1968)
 
miD jt Dim t 1 2 X
1n Dim n2 2 t 3.4.2.3 Liquid transport through porous membranes
2   2 exp  :
m C if m 6 1 n2 2m The nature of solute transport in a porous membrane with
3:4:82a solvent flow is complex; the regime is influenced strongly
by the ratio of solute diameter to pore diameter at any
As time t increases and tends to infinity, a steady state is
given transport rate of the solvent. The solvent transport
approached where the exponential term is negligibly small;
rate is, however, commonly described by Poiseuille flow
then, a plot of miD vs. t becomes a straight line, i.e.
through the pores. The nonporous section of the mem-
 
Dim C if 2 brane is assumed impermeable. There are two categories
miD jt t m : 3:4:82b
m 6Dim of porous membranes: Ultrafiltration (UF) membranes,
having pore diameters in the range 130 nm;
The intercept of this line on the time axis is 2m =6Dim , microfiltration (MF) membranes, having pore diameters
called the time lag in simple diffusive permeation through between 0.1 and 10 m (10010 000 nm). All pores in
a membrane for constant diffusion coefficient in linear the membrane may have the same pore size, but
diffusion. The value of the time lag provides a good order generally there is a pore size distribution (Zeman and
of magnitude estimate of the time from t 0 after which Zydney, 1996, chap. 4; Cheryan, 1998; Kulkarni et al.,
steady state permeation through the membrane may be 2001). Figure 3.4.5(c) illustrates the basic schematic of such
assumed. Correspondingly, equations (4.3.24) and (4.3.25) processes. According to IUPAC (the International Union of
for gas permeation in a device (see also Chapters 6, 7 and Pure and Applied Chemistry), pores less than 2 nm in
8) are valid only after this initial unsteady state. Experi- a diameter are called micropores, pores between 2 and
mental measurement of the time lag allows one to measure 50 nm are called mesopores and larger ones are known as
the value of Dim. Knowing Qim from steady state experi- macropores. Correspondingly, microporous membranes
ments, one can determine Sim. should have only pores less than 2 nm, etc. The membrane
Gas transport through nonporous inorganic mem- literature does not follow this practice uniformly.
branes falls into two categories. It is known that the con- Consider a membrane where the pores of uniform
ventional solution-diffusion permeation mechanism is diameter31 2r p occupy a fraction m of the membrane area.
valid for nonporous membranes of silica, zeolite and inor- Let the membrane thickness be m and the pore tortuosity
ganic salts. It is no longer so when the membrane is m . If a pressure P is imposed across the membrane from
metallic in nature (Hwang and Kammermeyer, 1975). the feed to the permeate (or the filtrate) side, the volume
Diatomic gases such as O2, H2 and N2 dissolve atomically flux vz through the pores (in units of cm3/s-cm2 of pore
in the metallic membrane (see (3.3.67)). While a conven- area) of a liquid of viscosity , and therefore the volume
tional flux expression is valid for atomic species i dissolved flux m vz through the membrane (in units of cm3/s-cm2 of
in the membrane, i.e. membrane area), is given by the Poiseuille law:
" #
C 0im  C im m r 2p P m r 2p 1 dP
N iz Dim ; 3:4:83 m vz  : 3:4:85
m 8 m m 8 m dz
the flux expression changes to
That Poiseuille flow exists through track-etched mica
Q 1=2 1=2 membranes with a pore radius around 5.6 nm has been
N iz im pif  pip 3:4:84
m verified by Anderson and Quinn (1972).
in terms of the species partial pressures in the gas phases, The membrane may have a pore size distribution f (rp)
in view of the solubility relation (3.3.70). such that the fraction of pores in the size range rp to rp drp
As long as there is no chemical reaction of the dissolved is f (rp)drp, and they contribute a fraction dm to the mem-
gas species with the liquid, the gas species flux expression brane pore area. If N sz V s is the total volume flux through the
through a thin liquid layer acting as a membrane is identical
to that through a rubbery polymeric membrane, as dis-
cussed earlier. The major difference comes about in the
magnitude of the permeability coefficients. The diffusion 31
Cylindrical pore in a membrane is an idealization widely
coefficient of a gas species in a liquid membrane will, in practiced in membrane literature.
3.4 Interphase transport: flux expressions 181

pores of such a membrane, then the contribution of pores of 7:21  103 :


radius rp to rp drp to the volume flux is given by
So only 0.72% of the membrane surface is covered with
dm r 2p P m r 2p f r p dr p P pores, i.e. m 0.0072. We have
dN sz V s :
8 m m 8 m m 7:21  103  175  108 2  106
N sz V s
For a membrane with pores of all radii between rmax and 32  0:2  106
rmin,
3:45  103 cm=s;
r max
cm s liter 1 m
m P N sz V s 3:45  103  3600  1000 3 
N sz V s dN sz V s r 2p f r p dr p s hour m 100 cm
8 m m
membrane
0 1 r min
liter
m r 2pP 124:3 :
@ A: m2 -hr
8 m m
The experimentally observed value of the volume flux at
3:4:86 20  C is 80 liter/m2-hr (lmh).
q
The quantity r 2p is the hydraulic mean pore radius, There are several sources of discrepancy: there is
defined by a lack of information about the tortuosity m > 1; the
r max
deviation of the pores from circularity; the existence of
r 2p r 2p f r p dr p : 3:4:87 dead-end pores, etc. In the calculation, the membrane
r min thickness of 0:2 m refers to a thin skin on top of the
total membrane of substantial thickness (100300 m).
The volume flux (3.4.86) may also be represented by
The rest of the membrane has pores that are orders of
Qsm P magnitude larger (with very little hydraulic resistance),
N sz V s : 3:4:88
m and its porosity is also much higher (as much as
This relation between volume flux, pressure drop, mem- 0.30.6). Such membranes are called asymmetric mem-
brane thickness and the fluid viscosity is commonly used branes when the membrane is prepared from one
for a porous medium and is known as Darcys law. Here, material. Often they are prepared from two different
Qsm is called the solvent permeability. Observe that all materials to yield a composite membrane.
characteristics of the porous medium that is the mem- The volume flux given above is for the pure solvent. If
brane, namely m , r 2p and m , are incorporated in Qsm. there are solute (macrosolute) molecules in the feed solv-
ent, the velocity of the solute molecule in the pore may or
Example 3.4.4 The utility of equation (3.4.86) for the deter- may not equal the local pore velocity of the solvent. The
mination of the solvent flux through a porous membrane latter is especially likely to be true for macrosolutes,
will be briefly illustrated with an example worked out
macromolecules, proteins, etc. In general, the solute/
by Cheryan (1987). For an XM100A ultrafiltration (UF)
macrosolute molecule can also diffuse along the pore,
membrane, the mean pore diameter ( 2  hydraulic mean
pore radius) 17.5 nm; the number of pores/cm2 of the top down its own concentration gradient. The solute flux Niz
membrane surface area (skin) 3  109; m membrane through the membrane is related to the solute flux through
p
thickness 0.2 m (only of the skin (to be explained later)); the pores, N iz , as follows:
viscosity of water (20  C), 1 cp; P 105 Pa (gauge p
N iz m N iz ; 3:4:89a
pressure); V s 18:05 cm 3 =gmol of water. From equation
(3.4.86), obtain the following estimate of the solvent volume where, from flux expression (3.1.113), we can write
flux (here, water flux):
p
  p p dC
cm3 m 175  108 2 N iz C im Gi vz m J iz m C im Gi vz m  Dip m im :
N sz V s cm2 dz
2
cm -s 8 4 convective diffusive
g flux flux
105 Pa  10 2 - Pa 1
 cm-s ; 3:4:89b
2 g 0:2  10 4
cm
1  10 p
cm-s Here, C im is the concentration of solute i averaged
where we have assumed that m 1 (since no information over a pore cross section, Gi is a convective hindrance
is available). The membrane porosity factor, accounting for the ratio between the velocity of
solute molecules and the averaged pore velocity of the
number of pores
m  pore area cm2 solvent vz (usually less than 1), and Dip is the diffusion
cm2
coefficient of solute molecules of species i in the pore
3  109  (Anderson and Quinn, 1974). We have seen earlier from
3  109  2r p 2  1752  1016
4 4 (3.1.113) that
182 Physicochemical basis for separation

Dil GDr r i ;r p velocity in the pore, m m is the effective pore length and
Dip : 3:4:89c Dip is the effective solute diffusivity in the pore. When
m
Pemi 1, the contribution of solute diffusion to solute flux
Depending on the dimensions of the pores and solute mol- is unimportant and
ecules, as well as any possible partitioning between the exter-
nal solution and the pore solution, the nature of the particular N iz Gi vz m if C 0il : 3:4:92a
terms in the above equation will vary. Integration of the flux If there is no solute partitioning, if 1 and Gi 1, then
expression will be carried out here only for a particular case,
p
namely when the pore concentration of solute C im is related N iz m vz C 0il ; 3:4:92b
to the solution concentration external to the membrane by a
a result valid for a membrane with pores very large com-
partition coefficient or a distribution coefficient,
pared to the solute molecules and no solute partitioning.
p
C im =C il i : 3:4:90a The integrated flux expression (3.4.91c) for Niz contains
the permeate solute concentration C il , which is often
Since there is a feed side and a permeate side of the related to the solvent velocity vz in the pore by
membrane, in general, at z 0,
N iz m vz C il ; 3:4:93a
p p0
C im =C il if C im =C 0il ; 3:4:90b
since the effective solvent flux through the membrane is
and, at z m m , mvz. Substitution of this into the integrated flux expression
p p (3.4.91c) provides the following relation between C il and
C im =C il ip C im =C il : 3:4:90c
C 0il (after rearrangement):
Here, the diffusion length in the z-direction is m m , where  
C il Gi if exp Gvz m m =Dip Gi if exp Pem
i
m is the membrane thickness and m is the membrane 0   :
C il Gi ip  1 exp Gvz m m =Dip Gi ip  1 exp Pem i
tortuosity. At steady state, N iz in expression (3.4.89b) is
constant, and 3:4:93b

p
C im In the absence of any concentration polarization, C 0il and
p
mm
dC dz C il are equal to Cif and Cip, respectively. The extent of
 im   : 3:4:90d
p
C im  N iz =Gi vz m Dip =Gi vz concentration polarization and its effects on the solvent
p0
C im 0
flux and solute transport for porous membranes and
macrosolutes/proteins can be quite severe (see Section
This provides
6.3.3). This model is often termed the combined
" N iz
# diffusionviscous flow model (Merten, 1966), and it can
p
C im 
G i v z m Gi vz m m be used in ultrafiltration (see Sections 6.3.3.2 and 7.2.1.3).
n ; 3:4:91a
p0 N iz Dip The relations between this and other models, such as
C im 
G i v z m the finely porous model, are considered in Soltanieh and
" ( )# Gill (1981).
p
p0 C im Gi vz m m
Gi vz m C im 1  p0 exp  The preceding development assumed that solute mol-
C im Dip
N iz " ecules were present in each pore. The pore size distribution

# :
Gi vz m m of porous membrane may be such that the solute molecules
1  exp 
Dip can enter only pores larger than the solute molecule. This
3:4:91b has two effects. First, the development of the solvent flux
expression (3.4.86) assumed no effects due to any solute
Membrane pore concentrations may be changed to exter- molecules; in reality, the solute molecules increase the
nal concentrations using relations (3.4.90b,c): solvent viscosity. Second, the solute flux expressed by
" ( )# (3.4.91c) may have to be corrected if all of the membrane
0 ip C il Gi vz m m pores are not available to solute molecules. Simplified
Gi vz m if C il 1  exp 
if C 0il Dip analysis of such a case has been provided by Harriott (1973).
N iz "
# ;
Gi vz m m
1  exp  3.4.2.3.1 Solute diffusion through porous liquid-filled
Dip
membrane We consider here solute diffusion through a
3:4:91c
porous liquid-filled membrane under the condition of no
where C 0il and C il are the solute concentrations in the feed convection (Figure 3.4.5(c)). Examples of separation pro-
and the permeate (the filtrate) solution, respectively. cesses/techniques where such a situation is encountered
The quantity fGi vz m m =Dip g is called the pore Pclet are: isotonic dialysis, membrane based nondispersive gas
number, Pem i , of solute i since Gi vz is the effective solute absorption/stripping or solvent extraction, supported or
3.4 Interphase transport: flux expressions 183

contained liquid membrane technique. The governing inside the membrane, the partition coefficient being
equation for solute flux Niz of species i per unit membrane given by
cross-sectional area is obtained from Ficks first law p0 p
C im C im
applied to a porous membrane having a porosity of m im : 3:4:96a
C 0il C il
(see equation (3.1.112c)):
p
dC im
N iz Dim ; 3:4:94a The flux expression is given by (see Section 3.4.3.2 for
dz
greater details)
where Dim is the diffusion coefficient of species i in the
p
membrane and C im is the pore fluid concentration of Dil m im 0 p0 p Q p0 p
N iz C il  C il k im C im  C im im C im  C im ;
species i. If the pore radial dimension, rp, is much m m m
larger than the solute dimension, ri (i.e. ri <<< rp) 3:4:96b
and the tortuosity factor is m, then, in the absence of where Qim and kim are still defined, respectively, by
any specific interaction between the solute and the pore (3.4.95d) and (3.4.95c).
wall, The cases considered above were such that ri <<< rp
p and there was no poresolute interaction. If the solute
Dil m dC im
N iz  : 3:4:94b dimension is no longer orders of magnitude smaller than
m dz
the pore dimensions and there are no specific solutepore
Integrate this flux expression across the membrane thick- wall interactions, we may employ flux expression (3.1.112f)
ness m to obtain and integrate (Lane and Riggle, 1959) under the conditions
of hindered diffusion and geometrical partitioning:
Dil m  p0 p

N iz C im  C im : 3:4:94c  
m m Dil m GDr r i ;r p C 0  C il
N iz im il : 3:4:97
m m
Here, Dil is the diffusion coefficient of solute i in the liquid
present in the pore, and may be obtained from the values We can use Faxens expression (3.1.112e) for GDr r i ;r p as
provided in Table 3.A.3. To relate the pore concentration long as r i =r p
0:5. Further, im may be obtained from
p
C im to the external solution concentration Cil present on relation (3.3.88a). The membrane mass-transfer coefficient
two sides of the membrane (z 0, C 0il ; z m ; C il ), con- and the permeability coefficient in the case of hindered
sider two cases. diffusion and geometrical partitioning are defined as follows:
Case (1) The solvent present outside the membrane is Dil m GDr r i; r p im Dil m GDr r i ;r p im
identical to the solvent/liquid inside the membrane k im ; Qim :
m m m
pores. Since there are no solutepore interactions and
3:4:98a
ri <<< rp (excludes the geometrical partitioning effect
(3.3.88a)), the solute concentration in the solvent in Note that
the pores is identical to that immediately outside the
pores: Qim 0
N iz k im C 0il  C il C il  C il : 3:4:98b
m
p0 p
C im C 0il ; C im C il : 3:4:95a In an alternative representation based on the overall mem-
brane phase concentration Cim, the flux equation is simply
Correspondingly,
represented as (Figure 3.4.8)
Dil m 0 Q
N iz C  C il k im C 0il  C il im C 0il  C il ; dC im C 0  C im
m m il m N iz Dim Dim im : 3:4:99
dz m
3:4:95b
Using a partition coefficient im between the membrane
where the masstransfer coefficient kim for the membrane
phase concentration and the external solution concentra-
is defined by
tion (Figure 3.4.8),
Dil m  0 
k im : 3:4:95c C  C il
m m N iz Dim im il : 3:4:100
m
Further, the permeability coefficient Qim is defined by
For swollen polymeric gels, an introduction to a free-
Dil m volume based interpretation of Dim through water-filled
Qim : 3:4:95d
m hydrophilic gels is available in Yasuda et al. (1969). Specific
Case (2) The solvent present outside the membrane is applications of such concepts to cellulosic dialysis mem-
different from (and immiscible with) the solvent present branes are provided in Colton et al. (1971) and Farrel and
184 Physicochemical basis for separation

Babb (1973). Additional considerations on solute transport,


Porous
membrane and subsequently selectivity in hemodialysis, are available
in Section 4.3.1.

Cif Example 3.4.5 Calculate the membrane mass-transfer coef-


Ci0l Cidl ficients and the permeability coefficients of two solutes,
sodium sulfate and sucrose, through a microporous light
denitrated cellulose membrane whose properties, along with
Ci0m Cip
Cidm those of the two solutes, are provided below (Lane and Riggle,
1959). The temperature is 20  C, and dilute aqueous solutions
are under consideration with essentially no convection
through the membrane. For sodium sulfate: Mi 142; diffu-
Pure
sion coefficient in water 7.7  106 cm2/s; density i 2.698
g/cm3. For sucrose: Mi 342; diffusion coefficient in water
Feed solution 4.5  106 cm2/s; density i 1.588 g/cm3. Membrane pore
Permeate solution diameter 3 nm; membrane swollen thickness 9.4  103 cm;
membrane porosity 0.43; membrane tortuosity 2.6.

Solution Estimate the solute diameter 2ri from equation


(3.3.90b). For Na2SO4,

2r i 1:465  108 142=2:6981=3


z=0 z=d 5:479  108 cm 0:5479 nm:
For sucrose
Figure 3.4.8. Solute concentration profile in diffusive transport
through a liquid-filled porous membrane from a feed solution to 2r i 1:465  108 342=1:5881=3 0:879 nm:
a permeate solution. Employ the definition (3.4.98a) for kim for each solute, where
GDr r i ;r p is given by the Faxen relation (3.1.112e) and im is
given by (3.3.88a). For Na2SO4,

 
k im Dil m im GDr r i , r p =m m
0 12
5:48A  
6
7:7  10  0:43  1  @  1  2:1045:48=30 2:095:48=303  0:955:48=305   
30

9:4  103  2:6
3
7:7  0:43  10  0:672 7:7  0:43  103  0:672
1  0:384 0:012  0:006  0:628
9:4  2:6 9:4  2:6
5:69  105 cm=s:

For sucrose,

0 12
8:79A  
4:5  10 6 @
 0:43  1   1  2:1048:79=30 2:098:79=303  0:958:79=305   
30
k im
9:4  103  2:6
4:5  0:43  103  0:5 0:079  103  0:44
1  0:616 0:052  0:002
9:4  2:6 2
1:74  105 cm=s:
3.4 Interphase transport: flux expressions 185

The larger sucrose molecule partitions into the pore and Here,
diffuses more slowly through water and encounters a higher  
drag from the pore walls, resulting in 3.27 times lower mem- 4r p m 2RT 1=2
DiK : 3:4:106
brane transfer coefficient. 3 m M i
The transport of solute from the feed through the For large membrane pores and high gas pressures, leading
membrane to the receiving solution may be influenced to a value of r p = large enough to ensure Poiseuille or
by the solute transport resistance in the feed liquid and viscous flow, the magnitude of the integrated flux expres-
the permeate liquid. In the absence of pore convection, sion for a pure gas i under a positive pressure difference
the value of the membrane transfer coefficient will be between the feed and the permeate side is given, for a
given by expressions (3.4.95c) and (3.4.96b). Referring membrane of pore radius rp and tortuosity m , by
to Figure 3.4.8, the solute flux at steady state can be
  
expressed as m r 2p Pf Pp Pf  Pp
N iz : 3:4:107a
8i m RT 2 m
N iz k lf C if  C 0il im k im C 0il  C il k lp C il  C ip ;
3:4:101 This is obtained simply from the Poiseuille flow relation
(3.1.116b) adapted for a tortuous porous medium,
where klf and klp are film transfer coefficients in the feed
and permeate for solute i. In the case of hindered diffusion P f8i m m vz =r 2p g; 3:4:107b
and geometrical partitioning, use instead of im k im , k im
from (3.4.98a). ideal gas behavior and
An overall solute-transfer coefficient K may be defined N iz vz C mean ; 3:4:107c
by
where Cmean is the mean density of the gas between the
N iz K C if  C ip ; 3:4:102  
pressures Pf and Pp C mean P f P p =2RT . For a gas
so that mixture, a simplistic representation of flux Niz will be
!
1 1 1 1 m r 2p P 2f  P 2p
3:4:103 N iz x if ; 3:4:108
K k lf im k im k lp 8 m RT 2

is the relation between K and the individual solute-transfer where xif is the mole fraction of species i in the feed. In
coefficients. reality, the situation is complicated by molecular diffusion
of the gas species imposed on the Poiseuille flow.
3.4.2.4 Gas transport through porous membranes
For slip flow or transitional flow conditions intermedi-
The flux expressions for gas transport through porous ate between Knudsen flow and Poiseuille flow, the molar
membranes have been considered in Section 3.1.3.2.4. flux of species A through the pores under such conditions is
The steady state Knudsen diffusion flux expression given by expression (3.1.115e) for a binary gas mixture of
(3.1.115a), species A and B (Scott and Dullien, 1962; Rothfeld, 1963):
  " #
p 4r p 2RT 1=2 dC i
N iz  ;
3 M i dz 1 P dx A
N Az  :
1  x A =N R 1 RT dz
of a gas i of molecular weight Mi through straight circular
DAB DAK
pores of radius rp and length m based on the pore cross
section, can be integrated along the z-direction. Assuming On integrating between z 0, x x 0A and z m m ,
ideal gas behavior, we obtain x A x A , we get
    "   #
p 4r p 2RT 1=2 pif  pip 1 DAB
N iz  ; NR 1  x A
3 M i m RT DAB P DAK
  N Az n   : 3:4:109
pif  pip 1 N R RT m m DAB
p
N iz DiK : 3:4:104 NR 1  x 0A
m RT DAK

(Note: Dik is called the Knudsen diffusivity.) If the mem- A similar expression is valid for species B.
brane has a porosity of m and a tortuosity of m, the We have been introduced to expressions for the gas
integrated membrane flux expression is given by flux due to surface diffusion in a porous medium/mem-
    brane: see expression (3.1.117a). Employing relation
4r p m 2RT 1=2 pif  pip 1 (3.1.118d) relating the surface excess concentration of the
N iz : 3:4:105
3 m M i m RT ith species per unit surface area, Eis , and the gas-phase
186 Physicochemical basis for separation

concentration, C 0 ig moles of i per unit gas phase volume in Qis  103 2.9 cm2/s. The values of Henrys law
the pores, we can assume Henrys law to obtain constant i defined as m d Eis =dC ig were, respectively,
3.48  107 cm and 2.79  107cm.
d Eis =dpig RT H ci 1 RT; 3:2:110a
(1) Calculate the values of the surface diffusion coefficient
where Dis for argon through the two different microporous
pig H ci Eis ; 3:3:110b carbon membranes.
(2) Obtain the values of the gas phase permeability Qig for
signifying equilibrium between the bulk gas phase being krypton for the two carbon membranes at the same
convected through the pores and the surface phase. Here, temperature and pressure.
H ci is the Henrys law constant. We can now integrate the Solution (1) The flux expression (3.4.112) obtained by com-
surface diffusion flux of species i in a mixture to obtain bining Knudsen flow and surface diffusion may be rewritten,
   using equation (3.1.118a), as
Dis asp P f x if  P p x ip
J iz : 3:4:111
H ci RT m dC ig d Eis asp dC ig 0 dC ig 0
N iz DiK  Dis DiK m  Dis i asp :
dz dz dz dz
Here, Dis is the surface diffusion coefficient of species i and Integrating across the membrane thickness, we obtain
asp is the pore surface area of the microporous membrane
in cm2 per unit volume (cm3) of the membrane defined as C ig 0 C ig 0
N iz DiK m Dis i asp :
a medium of porosity m. Ash et al. (1973b) have observed m m
that, as long as the adsorbed films are dilute, each species If we rewrite this expression as N iz fDiK m
diffuses independently of the other gas. Dis i asp gC ig 0 =m ; we observe that Qim Qig Qis
Since surface flow rarely exists by itself and is present Dik m Dis i asp . Therefore, Dis Qis = i asp . For the
Graphon membrane,
along with convective and diffusive flow in a pore, it is
useful to identify the total flux of gas species i in a pore. 3:1  103 cm2 cm2
DAr;s 7 4 989  105 ;
For the case of pore diffusion by the Knudsen mechanism, 3:48  10  90  10 s s
the total flux of i is given by for the Black Pearl membrane
  
DiK Dis asp P f x if  P p x ip 2:9  103 cm2 cm2
N iz c : 3:4:112 DAr;s 527  105 :
RT H i RT m 2:79  107  197  104 s s
p
(2) Qig for the Knudsen permeability DiK m . But DiK M i m
The characteristics of the surface flow are influenced by
is a constant for the porous medium. Therefore
the gas pressure, the temperature and the condensability
p
of the gas or vapor. The lower the temperature, the more p p M Ar
DArK M Ar m DKrK M Kr m ) DKrK DArK p :
likely it is that the extent of surface flow is higher. Similarly, MK
the higher the condensability of a gas species, the higher For the Graphon membrane,
the surface flow. The treatments by Ash et al. (1967, 1973b), p
Kammermeyer (1968) and Hwang and Kammermeyer QKr;g QAr;g  M Ar =M Kr
p
(1975) may be consulted for a detailed study of surface flow 4:8  103  39:9=83:8 3:3  103 cm2 =s:
through microporous membranes. Under extreme condi-
For Black Pearl,
tions, surface sorption followed by multilayer condensation
could block gas-phase flow completely. QKr;g 1:8  103  0:69 1:24  103 cm2 =s:
Example 3.4.6 Ash et al. (1967) have studied the combined
3.4.2.5 Transport in a solution with ion exchange
phenomena of Knudsen flow and surface flow in graphitized
carbon membranes prepared from Graphon and Black Pearl membrane
carbon powder compressed to form a plug. The pore surface A number of types of situations are encountered here: (1)
area measured/volume of these plugs were: 90 m2/cm3
transport in the presence of an electrical field and an ion
(Graphon) and 197 m2/cm3(Black Pearl); the corresponding
exchange membrane with no convection through it; (2)
porosities were 0.42 and 0.43. The temperature of measure-
ment was 303 K. They reported the permeability Qim of argon transport with convection and no external electrical field;
as the sum of Qig and Qis (Qig for Knudsen flow and Qis for (3) transport without any convection and without any elec-
surface diffusion), where their gas flux, is given by trical field.
    We are going to focus on the role of transport without
Qim Q C ig mole
C ig 0 im any convection in an ion exchange membrane placed in an
m m cm2 -s electrolytic solution subjected to a constant electrical field.
and m is the microporous carbon membrane porosity. If the aqueous solution of an electrolyte is placed between
Graphon membrane, Qig  103 4:8 cm2 =s, Qis  103 two electrodes connected outside to two terminals of a
3:1 cm2 =s; Black Pearl membrane, Qig  103 1.8 cm2/s, battery, the current passing through the solution and the
3.4 Interphase transport: flux expressions 187

Anode Cathode

Anion exchange Cation exchange Anion exchange


membrane membrane membrane

+
+ +

Cidm

C dsalt Ci0m +

Diluate chamber C 0salt


+ Concentrate chamber

d
dm

Figure 3.4.9. Concentration profiles in electrodialysis (ED) with an ideal cation exchange membrane (profiles around the anion exchange
membrane have not been shown).

membrane will be carried by both cations and anions membrane arrive in the solution on the right-hand side of
formed by the dissociation of the electrolyte (Figure the membrane in the so-called concentrate chamber at a
3.4.9). Consider a uni-univalent electrolyte (e.g. NaCl) that rate It 1m =F . From this solution, Na ions migrate toward
is completely dissociated into Na and Cl. Identify the cathode in the solution at a rate It 1s =F . Since t1s < t1m
the different species i where i =1 for Na, i 2 for Cl 1, the Na ions accumulate in the solution on the right-
and i 3 for water. The fractions of the current carried by hand side of the cation exchange membrane at the rate
Na in the solution and in the cation exchange membrane It 1m  It 1s =F . To maintain electroneutrality in this solu-
are given by its transport numbers t1s in the solution and tion, Cl ions migrate into this solution toward the anode
t1m in the membrane (see definition (3.1.108d)); similarly at an equal rate, It 2s =F .
for the Cl ion, the quantities are t2s and t2m. By definition, The net effect is that the solution on the left-hand side
of the cation exchange membrane is depleted of NaCl,
t 1s t 2s 1 and t 1m t 2m 1: 3:4:113
whereas that on the right-hand side is concentrated in
The cation exchange membrane prefers cations as the NaCl. From a Fickian diffusion point of view, solute NaCl
counterion. In the limit of an ideal cation exchange mem- should be supplied now to the left of the membrane from
brane, all current in the membrane is carried by cations, i.e. the solution to the right of the membrane. Yet the electrical
here t1m 1. Therefore t2m 0. Since neither t1s nor t2s is potential does just the reverse. A very brief analysis of these
zero in the solution, one observes in Figure 3.4.9 that, at factors is provided below.
steady state, the rate at which Na ions leave the solution The molar rate of transfer of cations by the current
from the left-hand side of the membrane into the mem- through a cation exchange membrane per unit membrane
brane is given by It 1m  It 1s =F . Due to electroneutrality, area is it 1m =F , where i is the current density (Figure 3.4.9).
the Cl ions must go toward the anode from this solution at The direction of this flux is from left to right. A diffusive flux
an equal rate: in the opposite direction (from right to left) is simply
Q1m C 1m  C 01m =m . The net flux of cations from left to
It 1m  It 1s It 2s
: 3:4:114 right through the membrane is given by
F F
it 1m Qim  
As a result, the solution in the left-hand side of the mem-  C 1m  C 01m : 3:4:115a
F m
brane in the so-called diluate chamber is depleted of NaCl.
On the other hand, the Na ions transported through the At steady state, the solution to the left of the membrane,
188 Physicochemical basis for separation

depleted of salt to this extent, must be supplied with salt (2) F i may be represented as
from the bulk solution to the same extent by diffusion and RT
the current to maintain the cation flux, i.e. F i Z i F r  rC i 3:4:118
Ci
it 1s by neglecting a derivative of the activity coefficient of
k d C salt  C 0salt : 3:4:115b
F species i, where i 1 for a positive ion, i 2 for a negative
Note that l is the liquid film thickness on the left-hand ion and i =3 for water.
side of the cation exchange membrane and k d Dsalt =l . If one assumes that there is no coupling between dif-
Therefore ferent fluxes, then, from the general formulation (3.1.205),
it 1m Q1m it 1s RT
 C 1m  C 01m k d C salt  C 0salt : 3:4:116 J i Lii F i Lii fZ i F r  rC i g; 3:4:119
F m F Ci
Most ion exchange membranes are such that the Q1m =m which has the form of the NernstPlank equation (3.1.106)
values are very low and could be neglected: since Lii =C i Dis =RT from (3.1.210).
Consider now a negatively charged cation-permeable
F k d C salt  C 0salt F Q1m C salt  C 0salt membrane. If it behaves ideally and allows no anions and
i
t 1m  t 1s m t 1m  t 1s water to go through, by the NernstPlank equation for the
3:4:117 cations (i =1), at the membranesolution interface,
F k d C salt  C 0salt
i :
t 1m  t 1s C1Z1F
J 1 DN
1s frC 1 rg: 3:4:120
RT
This relation illustrates how the salt mass-transfer coefficient Similarly for the anions (i 2),
in the region adjacent to the membrane on the left-hand side
C2 Z2 F
controls the current through the membrane. Since the solu- J 2 DN
2s frC 2 rg 0; 3:4:121
RT
tion in the left-hand side is being depleted of salt and is
becoming more dilute, it is called the diluate (note, kd). The for this ideal cation-permeable membrane. We know from
solution on the right-hand side of the membrane is called the the definition (3.1.108c) of the current density i that
X
concentrate in this electrodialysis (ED) process. iF Z i N i Z 1 J 1 Z 2 J 2 ; 3:4:122
For a given electrolytic solute and values of kd and C salt , i1
the higher the value of i, the lower the value of C 0salt . In the
under conditions of no bulk flow. To obtain a simplified
limit of C 0salt 0, the value of i, ilim , is called the limiting
result, assume a uni-univalent salt with  1. Then
current density; its unit is usually mA=cm2 , and practical
C1 C2 Csalt, suggesting that rC 1 rC 2 rC salt . There-
values are around 20 mA=cm2 . This condition, identified as
fore, from J 2 0, we get an expression for rC salt in terms of
concentration polarization, specifies the useful limit of i. If i
r. Substituting this into the expression for J 1 leads to
is increased beyond ilim , the excess current is not useful for
cation transport through the cation exchange membrane, DN
1s C salt F
J 1  fZ 1  Z 2 gr
and is essentially wasted. RT
It is useful to explore the steps by which one can arrive 3:4:123
Z1
at a relation similar to (3.4.117) from NernstPlank or other DN
1s f1  grC salt ;
Z2
formulations (Helfferich, 1962; Shaffer and Mintz, 1966,
1980). In the electrodialytic separation of a completely dis- where the flux is linearly proportional to the potential
sociated electrolytic solute in water, there are three species: gradient.
positive ions, negative ions and water. Due to current flow, Therefore,
there will also be heat flow due to any temperature gradient. ( )
An irreversible thermodynamic formulation (Hills et al., Z1 DN F 2 Z 1 C salt
i F Z 1 DN 1  rC salt  1s Z 1  Z 2 r
1961) of the flux of any one of the three species, due to the 1s
Z2 RT
chemical potential gradient of each species as well as the
3:4:124
temperature gradient, can be provided following the gen-
eral formulation (3.1.205). Further, there will be four such at the membranesolution interface of an ideal cation
fluxes (including that of heat), making matters quite compli- exchange membrane. The first of these relations is essen-
cated. An abstracted treatment has been provided by Shaf- tially very similar to relation (3.4.117).
fer and Mintz (1966, 1980). They have shown, by using order It is useful to identify some assumptions and consider-
of magnitude estimates for typical ED conditions, that: ations used in the above analysis.

(1) the temperature effects across the membrane have a (1) There was no convection of water through the ion
negligible effect; exchange membrane; the pressures on both sides of
3.4 Interphase transport: flux expressions 189

the membrane must be equal. Further, there was no


electro-osmotic transport of water in the same direc- Pw Pg
tion as the counterion. Pore

(2) The membrane was perfectly selective to cations.


Interface
(3) The pore size of the membrane was very small (2 xAgb
immobilized
3 nm) so that the diffusive permeability of cation was pAb
xAmi
very low. pAmi

(4) The current I or the current density i is determined by Gas phase Aqueous
the voltage applied between the two electrodes and the xAgi phase

ohmic resistances in the current path. pAi

The preceding analysis can be used in analyzing the per- Pressure Pg Pressure Pw

formance of electrodialytic separations. It is also useful in xAli


studying separation in Donnan dialysis, where only a single xAlb

ion exchange membrane (either a cation exchange mem-


brane or an anion exchange membrane) is used without
any externally applied electrical field. It is additionally
useful in battery separator analysis.
No attempt has been made so far to relate the mem-
brane phase concentration of an ion to its external concen-
tration. A relation of the type (3.3.120e, f) may be derived
Microporous hydrophobic membrane
by using the relation (3.3.118b) for Donnan equilibrium as
well as the electroneutrality relations in the membrane Figure 3.4.10. Immobilized gasliquid interfaces in a micropor-
and in the external solution. The ionic flux of any ion ous/porous hydrophobic membrane (as in a membrane
through an ion exchange membrane may then be obtained oxygenator).
from the NernstPlank equation (3.1.106). If r is to be
eliminated from such an equation, then the following pro-
The larger the interfacial area per unit volume of the two-
cedure may be adopted. Write down the flux for J 1 and J 2 .
phase contacting device, the higher the rate of mass trans-
If there is some constraint like J 2 0 (as in (3.4.121)), the
fer. Generation of interfacial area requires energy input.
problem is easily solved. In some cases, J 1 J 2 (for
Further, after mass is transferred, the dispersed phase has
example, in battery separators for alkaline Ni/Zn batteries,
to be coalesced. In liquidliquid systems, coalescence can
KOH is the electrolyte and J K J OH ). This will also allow
be a major problem, especially in systems prone to forming
elimination of r. Then one can express J i in terms of the
stable emulsions. It is possible to have two phases con-
concentrations of two solutions on the two sides of the
tacted, however, without dispersing one phase into the
membrane by integrating across the battery separator
other as drops or bubbles or thin films by using micropor-
membrane at steady state.
ous/porous membranes as phase barrier membranes.
Mass transport in such systems is briefly studied below.
3.4.3 Interphase transport in two-phase systems with
phase barrier membranes
3.4.3.1 Transport in gasliquid contacting via a micro-
In Section 3.4.1, simple quantitative representations of mass
porous/porous membrane
transport from one phase to another immiscible phase were
provided using mass-transfer coefficients. Separation pro- Membrane oxygenators use a porous/microporous hydro-
cesses where such mass transfer takes place generally utilize phobic membrane (Figure 3.4.10): on one side of the
the following configurations in industrial practice. membrane, the blood from the patient flows, while on
the other side air flows. The blood side pressure is main-
Gasliquid system:
tained slightly above the air pressure to prevent air from
(1) gas dispersed as bubbles in liquid; bubbling into the blood (Callahan, 1988; Sirkar, 1992).
(2) liquid sprayed as drops in gas; Since the membrane is hydrophobic, most aqueous solu-
(3) liquid spread as thin films on packings in a continuous tions, including blood, do not wet the pores, which remain
gas phase. filled with air. A gasliquid interface is created at the
mouth of every pore in the porous/microporous
Liquidliquid system: one liquid dispersed as drops in
hydrophobic membrane; the membrane facilitates the
another liquid.
gasliquid contact. Oxygen is absorbed from the gas into
Such dispersion of one phase into another generates a blood and CO2 is stripped from blood into the gas through
large interfacial area through which mass transfer occurs. such an interface. Unless the liquid pressure exceeds a
190 Physicochemical basis for separation

critical pressure called the breakthrough pressure, the gas and conditions may be used (see Tables 3.1.53.1.8). The
liquid interface remains stable. membrane transfer coefficient, kmg, may be estimated at a
To determine the rate of interphase transport of any total gas pressure P from
species being absorbed from the gas into the liquid (or
Di m P
desorbed from the liquid into the gas) through such a kmg : 3:4:129
m m RTx Bg;lm
gasliquid interface in a microporous/porous hydrophobic
membrane, consider the concentration profile of species Here, Di is the diffusion coefficient of species i in the gas
A shown in Figure 3.4.10. The flux of species A being phase, m is the porosity of the membrane having thickness
absorbed at steady state may be written down for the three m and pore tortuosity m and x Bg;lm is the logarithmic
regions (gas film, membrane pore and liquid film) as mean mole fraction difference of the inert gas species
follows: B along the diffusion path (see definition (3.1.132)). Such
an expression for kmg is valid for conditions of ordinary
N Az k xg x Agb  x Ami k mg x Ami  x Agi k xl x Ali  x Alb
diffusion of A through a stagnant gas film of B in the pores
3:4:125
of the membrane. If the mean free path conditions are
In terms of an overall mass-transfer coefficient, K xl or K xg , such that Knudsen diffusion or other convection mechan-
isms are valid, appropriate equations have to be used from
N Az K xg x Agb  x Ag K xl x Al  x Alb Section 3.4.2.4.
as before (expressions (3.4.5)). Further, from Henrys law From definition (3.4.129), it is clear that solute
relations (3.4.1a, b), we know that x Ag H A =P x Alb transport takes place only through the gas-filled pores.
H PA x Alb ; x Agb H PA x Al . Yet the interfacial area to be used for calculating the
Rearranging these equations and noting that total rate of mass transfer is the total membrane area;
the latter is certainly right for kxg and kxl. For reason-
x Agb  x Ag x Agb  x Ami x Ami  x Agi x Agi  x Ag ; ably porous membranes, Keller and Stein (1967) have
3:4:126 suggested that the model of the one-dimensional
series of resistances (3.4.128) may be valid, even
one can write
though there is a cross-sectional reduction at the
N Az N Az N Az N Az H PA membrane pore. Note that the curvature at the phase
3:4:127 interface is usually neglected in the area calculation.
K xg k xg k mg k xl
Further, there is no transport of gases through the
so that nonporous region.
1 1 1 H PA
: 3:4:128
K xg k xg k mg k xl 3.4.3.2 Solute transport via a microporous membrane
|{z} |{z} |{z}

overall gas-phase liquid-phase In Figure 3.4.2, the liquidliquid interface between an


resistance resistance resistance immiscible aqueousorganic system was shown to be
planar. In practice, the interface is usually curved, since
This relation shows that the overall resistance is increased one phase is dispersed as drops in the other phase. This
by the membrane resistance, which is now included in the dispersion can be avoided, and a stable liquidliquid
gas-phase resistance. For gas species (e.g. O2) with low interface created, by using microporous/porous
solubility in the patients blood, it is known from the membranes in a manner somewhat analogous to that
relation (3.4.12b) that the liquid-phase resistance controls shown in Figure 3.4.10. Consider Figure 3.4.11, in which
the mass-transfer rate. Thus, the increased gas-phase a microporous/porous hydrophobic membrane is
resistance due to the extra membrane resistance does shown. An aqueous nonwetting solution containing a
not influence the rate of oxygenation of the patients solute i to be extracted flows on one side of the mem-
blood at all. If, instead of blood, any aqueous or organic brane. An organic solvent flows on the other side of the
solution is used and oxygen has a low solubility in it, the membrane. If the organic solvent wets the membrane
same conclusion holds, as long as the membrane pores material, it will spontaneously fill the membrane pores
are gas-filled. Obviously, it is true for any other sparingly and would have a tendency to disperse as drops in the
soluble gas as well. On the other hand, if the gas species is aqueous phase on the other side. If the aqueous solution
highly soluble in the liquid, it is possible that the gas- pressure is, however, equal to or higher than that of the
phase resistance (which includes that of the membrane) organic-phase pressure, this dispersion can be pre-
may become important. For more details, see Sirkar (2001). vented and aqueousorganic interface immobilized at
To determine the liquid-film and the gas-film mass- each pore mouth (Kiani et al., 1984). Such an interfacial
transfer coefficients k xl and k xg , standard mass transfer configuration can be easily achieved also if both phases
correlations valid for the particular geometry, flow regime are stagnant.
3.4 Interphase transport: flux expressions 191

Ciwb
1 io 1 1
dm : 3:4:134
Aqueous phase Ko kw k mo k o
solute profile |{z} |{z}
Pw Po aqueous-phase organic-phase
Ciwi resistance resistance

Cimi
Pressure Similarly,
Po
Pressure
   
Pw
C imo C imo
C iwb  C iw C iwb  C iwi C iwi   C iw ;
io io
3:4:135

Ciob N iz N iz 1 N iz 1 N iz
:
Pore K w k w io k mo io k o

Therefore
Organic phase solute
profile Cimo  
1 1 1 1 1
: 3:4:136
Aqueousorganic Kw kw io k mo k o
interface immobilized |{z} |{z}
aqueous-phase organic-phase
resistance resistance
Microporous hydrophobic membrane

Figure 3.4.11. Schematic of solute concentration profiles in solvent


extraction with immobilized aqueousorganic interfaces in a It is useful now to obtain some limiting forms of the two
microporous/porous hydrophobic membrane. resistances-in-series relations for io 1 and io  1, pro-
vided one assumes that kw, ko and kmo are of similar orders
of magnitude. The condition io 1 means that the solute
strongly prefers the organic phase, whereas io  1 means
The concentration profile of a solute i being extracted that the solute strongly prefers the aqueous phase:
from the aqueous solution into the organic solvent is
1 1 )
shown in Figure 3.4.11. The flux of solute i may be
K w kw
expressed at steady state in terms of the aqueous film io 1; aqueous-phase control; 3:4:137
1 io
coefficient, the organic film coefficient and a membrane
K o kw
coefficient as follows:

N iz k w C iwb  C iwi k mo C imi  C imo k o C imo  C iob :


1
0
1 @ 1
1
1A
)
3:4:130
K w io k mo k o
The overall mass-transfer coefficient based expressions are io  1; 0 1 organic-phase control:
1 1 1
@ A
N iz K o C io  C iob K w C iwb  C iw : 3:4:131 Ko k mo k o
The solute concentrations in the two phases at the phase 3:4:138
interface (at the pore mouth) are related by the equilib-
rium distribution coefficient io as follows: For io O1, both phases influence the mass transfer.
Note that, for io 1, kmo is not important at all; the
C imi membrane resistance is irrelevant (Prasad and Sirkar,
io : 3:4:132
C iwi 1988).
Assuming that io is constant over the solute concentration Since for io  1 and for io O1 the membrane
range, resistance is important, an estimate of k mo is provided next.
The result (3.4.96b) and (3.4.98a) from Section 3.4.2.3.1 is
C io C iob C imi useful here, namely
io ; io ; io : 3:4:133
C iwb C iw C iwi
Dio m GDr r i ;r p
Now, C io  C iob io C iwb  C iwi C imi  C imo C imo  C iob ; k mo : 3:4:139a
m m
therefore
Here, Dio is the diffusion coefficient of solute i in the
N iz io N iz N iz N iz
; organic solvent. If there is no hindrance to solute diffusion
Ko kw k mo ko in the pore due to r i  r p (at least by two orders of
leading to magnitude),
192 Physicochemical basis for separation

Dio m porous hydrophilic membrane with the aqueous phase


k mo : 3:4:139b
m m present on one side and in the pores and the organic
phase present on the other side at a pressure P o P w
For a detailed introduction to this topic, see Prasad and to stabilize the aqueousorganic interface (Prasad and
Sirkar (2001). An alternative configuration employs a Sirkar, 1987).

Problems
3.1.1 Consider a pure gas in a tall column of height h in the gravitational field. Relate the pressures at the top and the
bottom of the column to the height of the column.
3.1.2 Develop an expression for the net vertically downward gravitational force on solute species i present in a solvent
s of density s. The solute density is i. Assume that the presence of the solute does not influence the force on
the solvent. Ans:  M i g1  s =i k:

3.1.3 In Example 3.1.3, the force calculated for ovalbumin molecules subjected to an electrical field of constant
strength E was based on neglecting the effect of an electrical double layer. In this problem, do not neglect the
effect of the double layer.
(1) Determine an expression for the ovalbumin velocity vector, given that the electrokinetic potential at the
shear surface is . Ans: fd E=6g ff r 0 ==1 r 0 =g:
(2) For motion in the z-direction, estimate the mobility m i of ovalbumin defined as the steady ovalbumin
velocity in the z-direction per unit value of electrical field strength acting on it in the z-direction. Dielectric
constant of the liquid medium d; ovalbumin molecule may be considered a sphere of radius r0; the
liquid medium vicosity is . Ans: fd f r 0 ==6 1 r 0 =g:
(3) Show that the ionic mobility m i of any small ion i in general in a uniform electrical field E is given by
d
Qnet
i =f i ; where Qi
net
is the net charge on the ion at the plane of motion/shear.
3.1.4 (a) Write down the equation of motion of a charged particle of radius rp having a charge Qp in an electrical
field of strength E in the manner of equation (3.1.60); include the relevant forces. Employ vectorial form.
The fluid phase is gaseous. The drag force is given by Stokes law.
(b) If the drag force is given by Stokes law, obtain the steady particle velocity for two cases: (i) gas bulk
velocity is vt; (ii) the gas is stagnant. The gas viscosity is . The particle velocity in the latter case is the
electrical migration velocity.
(c) Obtain the electrical mobility m p of the charged particle defined as the particle velocity per unit value of
the electrical strength when the gas is stagnant.
3.1.5 Consider two flat electrodes at locations z1 and z2 with electric potentials 1 and 2. For a uniform electrical field
E given by (d/dz)k, the electric field at any location z is
1  2
E k:
z 2  z1

Consider a different electrode configuration, namely an annular geometry with the outer electrode at
radius ro having a potential Vo and the inner electrode at radius ri having a potential Vi with an electrolytic
solution in between. Develop an expression for E in this nonuniform electrical field.
Suggestions: Use Ohms law for a unit cross-sectional area of solution. Current density i (amp/cm2) is easily
related to potential gradient d=dr and conductivity mho/cm by i kd=dr. For a cross-sectional area
2rh at radius r, h being the height of the solution, i2rh I, the total current between the electrodes, which is
independent of radial location. Integrate, assuming to be constant, to obtain the result

V i  V 0 1
E r:
n r 0 =r i r

3.1.6 Consider a single yeast cell in water placed between two cylindrical electrodes with the radius vector r in the
vertically upward direction. The density of the yeast cell p is 1.18 g/cm3 (that for water, w, is 1 g/cm3). If the
yeast cell may be considered as a dielectric uncharged particle, and if Brownian motion may be assumed to be
Problems 193

negligible, identify the forces acting on this yeast cell. Develop an estimate of the unknown dielectric properties
of the cell with respect to the medium (i.e. water) if there is no net force acting on the cell at a given radial
location r between the inner and outer cylindrical electrodes (having voltages Vi and Vo, and the corresponding
radii are ri and ro, respectively).

3.1.7 A spherical paramagnetic particle in a paramagnetic solution is subjected to a magnetic force due to a local
magnetic field intensity H m0 (in vacuum). Consider the situation where the magnetic permeabilities of the
particle and the solution are related to the corresponding susceptibilities p and s via

m m m m
p 0 p 1 and s 0 s 1:

Assuming that the only other force in the system is viscous drag from the solution of viscosity , obtain the
following expression for the terminal velocity of the particle:
2  2
2 p  s r p d B0
U pzt ;
9 dz 2m
0

under the condition that both susceptibilities p and s are much smaller than unity. Note: H m m m
0 B0 =0 ;
Stokes law is valid.
3.1.8 Although in a conventional approach (see the result of Problem 3.1.3 part (3)) the ionic mobility of a small ion is
d d
expressed as Qnet
i =f i ; where f i is the frictional coefficient due to hydrodynamic drag (most often expressed via
Stokes law for a spherical molecule), more advanced theories propose an additional dielectric (charge-
induced) frictional drag coefficient,

f dl m net d dl
i Kay; 1991 : i Qi =f i f i : 3:P:1

For a spherical molecule, f di 6 r i and


!
of  f Z i e2 Z i e2
f dl
i H constant  ;
2of r 3i r 3i

where Zi is the algebraic valence of ion i. Although these models assume spherical molecules, in reality other
effective structural patterns are common as the ions migrate. It has been found that the volume of the ion may
be used in general instead of ri. Develop an expression for m i based on (3.P.1), where ri has been replaced
appropriately by Vi, the volume of the ion, assumed spherical.

3.1.9 Consider the particle motion in a fluid where the particle Reynolds number is larger than 1.
(1) Write down in vectorial form the equation of motion of a particle subjected to a total external force of F ext
p .
The velocity of the fluid is vt.
(2) Obtain an expression for the steady particle velocity if the fluid is stagnant.
3.1.10 You are required to make some simple calculations here to estimate how long a nuclear winter would last
(Sutija and Prausnitz, 1990). The large-scale generation of smoke and dust from urban and forest fires following
a nuclear war would lead to a dust cloud in the stratosphere. The dust cloud would block solar radiation and
cause a substantial reduction in temperature (as much as 2040  C) over large parts of the world (Turco et al.,
1990). Assuming that each dust particle diameter is 10 m and that each particle settles independently of the
others, determine the duration of the nuclear winter if the particles have been carried to a height of 50 km by
the nuclear blasts and subsequent fires. The particle density is 2 g/cm3. (Ans. 105 days.)

3.1.11 Determine the force being exerted on two different enzymes diffusing in a slab of an aqueous gel subjected to a
uniform electrical field strength of 3 volt/cm. At the pH conditions of the gel, enzyme 1 has a charge of Z 4
while enzyme 2 has Z 1. Calculate the migration velocity of the two enzyme molecules if their diffusion
coefficients in the gel are: enzyme 1, 2  107 cm2/s; enzyme 2, 4  107 cm2/s. Comment on the relative
magnitudes of the migration velocities.
3.1.12 Consider a separation system containing no external forces, no pressure gradients, no temperature gradients
and for which n ai =n x i  P;T 0: For such a ternary system of solute i, solvent s and membrane m, write
down the MaxwellStefan equations for solute transport and solvent transport. Identify the diffusion coeffi-
cients; note that the membrane velocity is zero.
194 Physicochemical basis for separation

Write down the equations for solute transport and solvent transport for the same solutesolventmembrane
system using the following irreversible thermodynamics formulation:
n
X
Xi L0ik N k ;
k1

where Xi is the force specific to species i, Nk is the flux of the kth species in a stationary frame of reference and
L0ik are coefficients are such that L0ik L0ki . For an ideal system satisfying all the assumptions for the Maxwell
Stefan equations, obtain the various relations between the L0ik and Dik (from the MaxwellStefan formulation).

3.1.13 From the generalized MaxwellStefan type equations (3.1.180) and (3.1.181), write down the specific expres-
sions for the following situations in terms of the force (di) RTCt moving species i relative to the solution per unit
volume of the solution:
(a) only a uniform electrical field is present;
(b) only a centrifugal force field exists.
3.1.14 Employing equation (3.1.222) to determine an effective velocity U for all ions in ambipolar diffusion, develop a
relation between the ambipolar diffusion coefficient D for a highly ionized gas and the diffusion coefficients of
electrons (D) and positively charged ions (D) and any other necessary quantities, e.g. ionic mobilities m ;g
and m;g of positive ions and electrons, respectively, in the gas phase.

3.1.15 In a dilute solution of a binary electrolyte AvZ YZv , a generalization of equation (3.2.2) for three dimensions and
no bulk motion for the solvent v t 0 is
C i
rN i 3:P:2
t
for each ionic species CA and CY. We are interested in deriving the general result (3.1.107),

D D D Z  Z  = Z D  Z  D ;

using the balance equation for each ionic species and the electrolyte as a whole. Note that the molar concen-
tration of the electrolyte C may be related to those of A and Y by
C C
C 3:P:3
v v

There is no applied potential. Write down the balance equation (3.P.2) for C and C using

N i J i C i U i  Di r C i ;
U i Z i Di F r=RT and equation 3:P:3:

Obtain an expression for r  C r. Use it in equation (3.P.2) for C along with equation (3.P.3). Compare it
with equation (3.P.2) for the electrolyte molar concentration C and obtain the result, assuming that the Di are
concentration-independent in the dilute solution.
3.2.1 (a) Develop an expression for the migration velocity of solute species i in a solution rotating in a centrifuge at
radian/s.
(b) The following information is available: centrifuge cell length 1 cm; centrifugal force field 50 000 times
the force of gravity; temperature 75  F; apparent density of water in solution 1 g/cm3; albumin mol.
wt., Mi 45 000; apparent density of albumin in solution M i =V i 1:34 g=cm3 ; diffusion coefficient
of albumin in water 7  107 cm2/s. Calculate the migration velocity of albumin molecules.
(c) How many different species of noninteracting molecules in a solution can be separated in such a
centrifuge assuming a linear force profile and albumin molecule properties as characteristic properties
to be used in calculation?
3.2.2 It is intended to separate different proteins in an electrical field of constant strength 0.025 volt/cm. The
temperature is 75  F and the diffusion coefficients of all proteins may be considered to be essentially equal
to 7  107 cm2/s. Focus on three proteins n 1, 2, 3 and note that the net charge on the protein of each type is
Zn n 1, 2, 3 at the given pH. Develop an exact expression for the variation of resolution between proteins 1
and 2, 2 and 3 and 1 and 3 in terms of time t and a numerical proportionality constant in each case. You are
given F 96 490 coulomb=gram equiv:; R 8:314 joule=mole-K.
Problems 195

3.2.3 In expression (3.2.23) for the resolution between two neighboring peaks being translated in a vessel under
uniform potential gradient, it was shown that resolution improved with time. One would like to know, however,
the ratio of the standard deviation of a species profile with respect to the distance traveled by the center point of
the profile (zi). Does it depend on the diffusion coefficient as the bandwidth of the profile increases with time?
p p
Assume that only external force exists: r0i 0. Ans: 2RT = z i F ext iz :

3.2.4 Consider the concentration profile of a species i being convected down in a uniform continuous potential
profile driven external field (e.g. equation (3.2.19)). Obtain the maximum value of the concentration Ci at
any instant of time. Compare its magnitude with that for a Gaussian profile, expression (2.5.6a).
p
Ans: mi =l x l y 1= 2 i :

3.2.5 A circular tube of length l Z and radius r0 is filled with a dilute inert gas at a total concentration of Ct. At one end
of this tube, called a drift tube, a cloud of different types of ions are introduced uniformly across the tube cross
section in a delta-function fashion. The drift tube is designed to have a uniform electrical field of strength Ez in
the direction of the tube-axis, the z-coordinate. Obtain:
(1) the concentration profile of ionic species i along the drift tube length, z, as a function of time;
(2) the standard deviation of the profile.
You are given that mi is the number of moles of ionic species i introduced at the inlet.
p
Suggestion: Employ a drift velocity formalism for the ions. Ans: C i z;t mi = r 20 1= 4 t Di
2
expfz  U iz t =4 Di tg:
3.3.1 (1) For a continuous chemical mixture use the expression which is the equivalent of Raoults law to predict
the molecular weight density function fv(M) for the vapor phase. You are given the molecular weight
density function f l (M) for the liquid phase as well as Psat(T; M). The total pressure P is unknown. Assume
an ideal liquid solution.
(2) Let the vapor pressure Psat(T; M) be described via Troutons rule and the ClausiusClapeyron equation by
"  #
sat M
P T;M P exp A 1  :
T

Let the liquid-phase function f l M be described by a Gaussian distribution:


" #
1 M  M2
f l M p : exp  :
2 l 2 2l

Obtain an expression for each of P, the total pressure, and f v M.

3.3.2 It is known that the equilibrium solubility of a solute particle in a solvent increases as the particle diameter is
reduced.32 Assume fugacity ratios are to be replaced by the solubility ratios. Show that the solubilities C1 and C2
of two solute particles of radius rp1 and rp2 are related by
!
2V ij 12 1 1
n C 1 =C 2  :
RT r p1 r p2

Consider separately particles of PbI2 (rp 0.4 m, V ij 74:8 cm3 =gmol; 12 130 dyne=cm and BaSO4 (rp
0.1 m, V ij 52:0 cm3 =gmol; 12 1250 dyne=cm in a solvent (see Mullin (1961) for more systems and
additional discussion). Calculate the percent increase in solubility of each kind of particle due to their small
radius at 25  C.

3.3.3 The values of the distribution coefficient i1 of a volatile liquid i between two essentially completely immiscible
liquid phases, an organic liquid ( j 1) and water ( j 2), are provided in Table 3.P.1 for two molar concen-
trations, Ci1, of liquid i in organic liquid phase j 1 at 25  C.

32
This process does not continue indefinitely. See Mullin (1961) for a review of the role of possible surface electrical charge and a
change of 12 with rp.
196 Physicochemical basis for separation

Table 3.P.1.

C i1 gmol=l i1 C i1 =C i2

0.1 29
2.0 35

Determine:
(1) the values of the equilibrium ratio Ki1 of the volatile liquid i for these two concentrations;
(2) the corresponding ratios of the activity coefficient of liquid i between j 2 and j 1, i2/i1. You are given:
M i 160; M organic liquid 155; density i 3 g=cm3 ; organic liquid 1:5 g=cm3 ; water 1:0 g=cm3 . Assume
that if the molar volume of any pure species in liter/gmol is V i , then the volume change on mixing two
pure species 1 and 2 is zero, so that C 1j V 1j C 2j V 2j 1. Note: V i is normally used for partial molar
volume of species i.
3.3.4 The solubility coefficient Sim of a gas in a semicrystalline polymer is likely to be linearly proportional to the
volume fraction of the amorphous phase since the crystalline region is virtually impenetrable (Section 3.3.7.3).
The gas solubility data in Table 3.P.2 were obtained by Michaels and Bixler (1961) in a polyethylene film at 25  C.

Table 3.P.2. Solubilities of various gases in a semicrystalline polyethylene film

Sim , cm3 STP=cm3 of polymer-atm

He CO CO2 C2H6 CH3Cl


0.008 0.041 0.29 0.84 4.35

In a separate investigation, these authors concluded that the solubility of each of these gases in a completely
amorphous polyethylene film having otherwise same properties is as given in Table 3.P.3. Make an estimate of
the volume fraction of the amorphous phase in the polyethylene film.

Table 3.P.3. Solubilities of various gases in an amorphous polyethylene film

25 CSima , cm3 STP=cm3 of polymer-atm

He CO CO2 C2H6 CH3Cl


0.012 0.064 0.451 1.28 6.55

3.3.5 Calculate the osmotic pressure of the following dilute solutions using the Van t Hoff equation: aqueous solution
of sucrose of 0.991 molality (ignore density change due to sucrose) at 30  C; aqueous solution of NaCl,
0.1 Molar, 0.17 Molar and 35 000 ppm. at 25  C. For solutions of NaCl, calculate also the osmotic pressure if
i C i R T, where A B for a binary electrolyte AB: A and B are the moles of ion A and ion B,
respectively, produced from one mole of electrolyte AB. You may assume NaCl to be completely dissociated.
3.3.6 (1) Derive the expression given below for the geometrical partitioning factor im for a spherical rigid solute i
between an external solution and the solution in a pore in the shape of a slit:
  
ri ri 
im 1  1 :
rp b

Here, ri is the radius of the solute molecule, assumed spherical, and rp is a characteristic slit pore dimension
defined as the half width of the gap between two large flat plates. The slit pore may be imagined as the space
bounded by two rectangular plates each of surface area Asurf set apart by a distance 2b.
Problems 197

(2) Derive the expression for the geometrical partitioning factor im for a spherical rigid solute of radius ri
between an external solution and a pore in the shape of a rectangle of sides b1 and b2. The pore is
infinitely long. Rewrite this result in terms of a hydraulic radius rh of the pore. Ans: im 1  2r i =b2
1  2r i =b1 ;
  
ri r i b2 =b1
im 1  1 :
r h 1 b2 =b1 r h 1 b2 =b1

Suggestion: Calculate the partition coefficient of the solute between the pore solution and the external solution
as the ratio of two cross-sectional areas, one in which the center of the solute molecule can move and the other
as the total pore cross-sectional area. Rewrite the result in terms of a hydraulic radius of the pore.
3.3.7 Determine the solubility of lysozyme in the following aqueous salt solutions:
(1) 1M sodium sulfate solution at pH 8;
(2) 1M sodium chloride solution at pH 8.
You are given (1) for sodium sulfate solution; logC 2j0 3:55, where C 2j0 is in mg/ml; constant b 0.712; (2) for
sodium chloride solution: logC 2j0 1:18, where C 2j0 is in mg/ml; constant b 0.480. Note: The pI for lysozyme
is 10.5. Therefore at pH 8, its solubility is not at its lowest point (without any salt). The data were obtained
from a Table quoted in Ladisch (2001).

3.3.8 Employing the Gibbs adsorption isotherm for gas adsorption on the surface of a solid adsorbent and the
equivalent of the ideal gas equation of state for the surface adsorbed phase, obtain Henrys law for pure gas
adsorption in the form of qi =S H i P, where Hi is the Henrys law constant for species i. Henrys law is valid
for dilute/low-pressure adsorption systems.
3.3.9 In the adsorption of anthracene from its solution in cyclohexane on alumina adsorbent particles, Thomas
(1948) found that the following relation described the equilibrium adsorption behavior:
22 C if
q is :
1 375 C if

Here, the units of q is are millimole of anthracene per gram of the adsorbent and the units of Cif are millimole of
anthracene per cm3 of the external cyclohexane solution. Determine the values of the adsorption equilibrium
constant K and the maximum possible value of anthracene concentration in the solid phase qm is for this
Langmuirian behavior.
3.3.10 For Langmuir adsorption of pure CO2 on an adsorbent having an interfacial adsorption surface area S of
4000 m2/g of adsorbent, the following information on CO2 is available: surface area covered by one CO2
molecule is 1.2 x 1019 m2.
(1) What is the value of qi max for CO2?
(2) At a pure low CO2 pressure of 2.5 cm Hg, the equilibrium CO2 adsorption was observed to be 0.017 gmol of
CO2/g of adsorbent. Obtain the Langmuir adsorption isotherm for CO2 and the adsorbent.
Note: qi max may also be expressed as qm m
i or qis .

3.3.11 Gas mixtures often have moisture. When such a gas mixture is contacted with a porous inorganic adsorbent
material or membrane, water vapor may condense in the pores. If gas mixture adsorption or permeation was
the objective, condensed water vapor will be a hindrance. A porous inorganic material has pores of 5 nm radius
and the temperature of contacting is 25  C. Water vapor pressure at a temperature T  F can be determined from
the Antoine equation (Henley and Seader, 1981):

P sat
H2 O A2
n A1 
P H2 O;critical T F A3

where A1 6.532, A2 7173, A3 389 and PH2O, critical 3206 psia. Determine the relative humidity of the gas
mixture pH2 O;v =P sat
H2 O below which one must operate to prevent pore condensation. You are given:
12
water 72 dyne=cm 72 erg=cm2 ; molar volume of water 18 cm3/gmol; contact angle 0 since water
wets inorganic adsorbent materials in general.
3.3.12 Rogers et al. (1926) have provided data on the decolorization of a petroleum fraction by a clay adsorbent. Their
isotherm may be represented in tabular form (see Table 3.P.4).
198 Physicochemical basis for separation

Table 3.P.4.

   
units of color adsorbed from 100 kg oil equilibrium color units
qi1 C i2
kg clay kg solution

11 200
8.8 85
8.0 60
6.8 38
5.0 13

Determine the parameters of the isotherm if a Freundlich isotherm of the type C i2 mqi1 n describes the
behavior of the data. Relate the parameters here to those of (3.3.113e).

3.3.13 In the calculation of the distribution of the co-ion Y from an electrolyte AY between an external aqueous
solution and the ion exchanger in A form (i.e. cation exchanger with fixed negative charges) by equation
(3.3.120d,e), the importance of two terms which contribute to the term "b" need to be determined. It is known
that the term with activity coefficients is important. One would like to know how important is the other term,
asR =asw V AY =V sw . Develop an estimate of the value of this term for two cases, namely the swelling pressure is
0 atm and 500 atm, and comment on its influence on the value of mY,R when mF,R is 5 molal. You are given: V AY
for most electrolytes varies between 15 and 40 cm3/gmol; T 300 K.
3.3.14 (1) Develop an explicit relation between the external solution phase concentration of an ion and its ion
exchange resin phase concentration. Neglect the swelling pressure in the resin.
(2) Use the result (3.3.119d) and other required relations, including the electroneutrality conditions in the
resin and the external solution, to arrive at the type of relation which is the basis for the functional
dependence suggested in (3.3.120f) for the electrolytic solute AY, where A 2, Y 1.

3.3.15 Brooks and Cramer (1992) have determined the following values for various equilibrium parameters of two
proteins, -chymotrypsinogen and cytochrome-c: for -chymotrypsinogen, jZpj 4.8, p 49.2, Kps 9.22  103;
for cytochrome-c, jZpj 6.0, p 53.6, Kps 1.05  102.
The cation exchanger resin has a total ion exchange capacity of 567 mM.
(1) Obtain the SMA based equilibrium adsorption isotherm for each of these proteins.
(2) What protein concentration in the buffer will yield 6 mM protein concentration in the resin for a buffer salt
(Na based) concentration of 75 mM for cytochrome-c?

3.3.16 (1) Consider gassolid equilibrium as encountered in sublimation, in which a species passes from the solid state
to the vapor phase without passing through the liquid phase. In the separation of a solid mixture by
sublimation, this vapor obtained by vaporizing a solid component may be condensed as such or a carrier
gas may be used to entrain the vapor for condensing it later. This is done usually at a low pressure so that ideal
gas behavior is obeyed. Obtain an expression for the gas phase mole fraction of species i, assuming the solid
phase to be an agglomeration of pure species with each component having its pure component solid fugacity.
(2) Suppose now that gas pressure is much higher. Develop an expression for the gas-phase mole fraction of
species i. (Hint: Use equations (3.3.127) and (3.3.128).) Assuming that the gas phase behaves ideally,
calculate the mole fraction of naphthalene in ethylene gas at 12  C, 100 atm pressure, given
3
P sub
i T 0:0000303 atm; Vis 111.9 cm /gmol.
Compare it with the observed mole fraction of around 0.0085 (McHugh and Krukonis, 1986, fig. 1.3) in the
extraction of naphthalene by supercritical ethylene.

3.4.1 Determine the values of Kc and Kxl for the absorption of O2 from air into water at 20  C. Assume N 1 for both
the gas film and the liquid film (see expression (3.1.127)). You have been provided with the following information:
HO2 4  104 atm/mole fraction; P = 1 atm; k 0c for O2 in water = 2  103 cm/s; DO2g = 0.21 cm2/s is obtained from
DAB, where A and B are O2 and N2; g for the gas film is 0.1 cm; to determine Ctl, assume that pure water has a
molar volume of 18.05 cm3/gmol. Comment on the mass-transfer control (if any) by one of the two films.
Absorption of O2 is encountered in the oxygenation of blood. The desorption of O2 from water is very
important in degassing processes employed in ultrapure water production. Comment on which resistance will
be important in both processes. (Ans. Kc 2  103 cm/s; Kxl 1.1  104 gmol/cm2-s-mole fraction.)
Problems 199

3.4.2 Unlike gases like O2, N2, etc., which have very low solubility in water, there are other gases, such as NH3, SO2,
etc., which have high solubility in water. The following information has been abstracted from an example of
absorption of SO2 from air into water in a packed tower by Flagan and Seinfeld (1988):
kgmol kgmol
k xg a 96:8 ; k xl a 2050 ;
m3 -hr-mole fraction m3 -hr-mole fraction
the location where these values are useful has xAgi 0.07 and xAli in equilibrium 0.002. Here, a is the
interfacial area per unit total system volume and it is unknown. Determine the value of Kxga. If the value of a is
100 m2/m3, determine the value of Kxg in units of gmol/cm2-s-mole fraction.

3.4.3 From a detailed packed tower design by Treybal (1980) for the absorption of benzene vapor present in coal gas
by an essentially nonvolatile wash oil, the following information is available:
kgmol kgmol
k xg 1:96  103 ; k xl 4:11  104 ;
m2 -s-mole fraction m2 -s-mole fraction
Ki xiv/xil 0.125 for benzene at 26  C. Calculate the values of Kxg and Kxl. Comment on the relative roles of
the two film resistances.
3.4.4 Calculate the overall mass-transfer coefficient for the extraction of solute A between water and an organic phase
for the following cases.
(1) An organic compound present in low concentration in water is being extracted into toluene. The distribu-
tion coefficient of A between the aqueous phase and toluene is Ao 25. The aqueous-phase mass-transfer
coefficient for solute A is kAw 1.06  102 cm/s. The organic-phase mass-transfer coefficient is of the same
order as kAw. Calculate KAw and KAo. (Ans. KAw = 1.06  102 cm/s; KAo = 4.2  104 cm/s.)
(2) Acetic acid is being extracted from water into methyl isobutyl ketone (MIBK); Ao 0.545. The values of
kAw and kAo are, respectively, 0.678 and 2.03 in units of lbmol/(hr-ft2-CA) where CA is measured in
lbmol/ft3. Calculate KAw in units of cm/s.
(3) Acetone is being extracted from a toluene solution into water: Ao 0.58; kAo 0.0485 lbmol/(hr-ft2-CA);
kAw 0.516 lbmol/(hr-ft2-CA). Calculate KAw and KAo in units of cm/s. These data have been obtained
from Treybal (1963).

3.4.5 In a reverse osmosis experiment using a flat piece of membrane under the condition of no concentration
polarization for a brackish water feed having a salt concentration of 9 kg/m3, the permeation velocity vy was
found to be 1.512 cm/hour (cm3/cm2-hour). The applied pressure difference was 29 atm at a temperature of
25  C. The observed salt rejection was found to be 0.98. Calculate the values of the pure water permeability
constant A and the solute transport parameter (Dim im/m), assuming that the membrane does not have
any defects, imperfections, etc. You are given that b 0.8 atm-m3/kg NaCl. (Ans: A 1.06  106 gmol H2O/
cm2-s-atm; (Dim im/m) 8.5  106 cm/s.)

3.4.6 In a particular example of reverse osmosis desalination, the values of A and (Dim im/m) are, respectively,
2  107 m/(s-atm) and 1.5  107 m/s. Note the units of A given here refer to the volume flux mode,
i.e. N water V w AP  . The feed salt mass concentration is 10 kg/m3; the temperature is 25  C; the applied
feed pressure is 30 atm. The coefficient b in the osmotic-pressure relation b CijMi is provided as 0.77 atm-m3/
kg NaCl; note Cij is in molar units. Calculate the salt rejection. State your assumptions if any. (Ans. R 0.967.)

3.4.7 A rotating hollow cylindrical reverse osmosis (RO) membrane of cellulose acetate was placed in an autoclave
containing a salt solution of concentration 0.165 mgmol/cm3. The pressure of this salt solution with respect
to the hollow core of the cylinder around which the RO membrane was wrapped was 605 psi (40.44 atm). At
250 r.p.m. of this membrane-wrapped cylinder, the measured salt flux and water flux into the hollow core were,
respectively, 0.00252 and 0.605 mg/s-cm2; the permeate salt concentration was 0.071 mgmol/cm3. Calculate the
values of the membrane transport parameters A and Ap (equation (3.4.60a)), assuming that the salt diffusion
parameter (Dim im/m)is negligible. You are given the following: salt diffusion coefficient in feed solution,
Dif 1.48  105 cm2/s; feed solution density 1.006 g/cm3; f kinematic viscosity 0.0090 cm2/s; 680
Cij psi, where Cij is in mgmol/cm3; rotating membrane/cylinder diameter 2.54 cm; membrane surface area
40.5 cm2. The values of the JD factor for the rotating cylinder are as follows: Re 104, JD 4.8  103; Re 7000,
JD 5.4  103; Re 9000, JD 4.95  103; Re 5000, JD 6  103. The Re value is based on the cylinder
diameter and the r.p.m. of the cylinder in saline water.
200 Physicochemical basis for separation

3.4.8 Develop systematically an explicit solvent flux expression in the reverse osmosis separation of two noninteract-
ing solutes i 2, 3 from water in the manner of equation (3.4.66a). Assume that (3.4.65c) is valid for each solute.
Further, jvy j << kif and C ip << C 0il for i 2, 3. Write down the solute flux expressions as well. Use the
solution-diffusion model; assume dilute solution of each solute.
3.4.9 In an experiment on simple diffusive permeation of the permanent gas N2 through a thin film of the glassy
polymer poly(phenolpthalein terephthalate), the time lag determined was 22 seconds. The measured permea-
bility coefficient QN2m was found to be 0.7 barrer. The film thickness was 0.8 mil (1 mil 25  104 cm 25
m). Measurements were made at 35  C. Assume that you have no other information available. Determine
approximate values of (1) the diffusion coefficient of N2 through the film and (2) the solubility coefficient of N2
in the film. (Ans. (1) Dim 3  108 cm2/s; (2) Sim 0.175 cm3 (STP)/cm3 polymer-atm.)
3.4.10 Permanent gases such as nitrogen, oxygen and carbon dioxide have been found to follow the dual sorption
dual transport model for sorption and diffusion through thin films of the glassy polymer, poly (phenolphthalein
terephthalate) (P Pha-tere). Determine: (1) the concentrations of the pure gases CO2 and N2 in the polymer film
in units of cm3(STP)/cm3 of polymer if the pure gas-phase pressure is 5 atm; (2) the permeability coefficient Qim
of each of pure CO2 and N2 present at a feed gas pressure of 5 atm in the unit of barrer (where 1 barrer 1 
1010 cm3 (STP)-cm/(cm2-s-cm Hg).) You are given that the temperature is 35  C. The data in Table 3.P.5 were
obtained by Chern and Brown (1990).

Table 3.P.5.

Sim (cm3 (STP)/cm3 C 0 Hi (cm3 (STP)/ bi DiD  108 DiH  109


polymer-atm) cm3 polymer) (atm1) (cm2/s) (cm2/s)

CO2 1.44 31.2 0.398 10.2 5.85


N2 0.160 3.89 0.062 3.08 0.755

(Ans. (1) CO2, N2: 27.96, 1.72 cm3(STP)/cm3 polymer; (2) CO2, N2: 22.51, 0.66 barrer.)
3.4.11 Thin films of palladium are of interest because of palladiums extremely high selectivity for H2 over other gases
such as He, N2, O2, hydrocarbons, etc. Sieverts law based square root solubility dependency of diatomic gases
(3.3.70) leads to the unusual permeation flux vs. partial pressure relation (3.4.84) in such systems where the
diffusing species is atomic hydrogen in the palladium membrane.
(1) What are the units of the permeability of QH2 m if the partial pressure of H2 is in pascals, the membrane
thickness is in meters and the flux units are gmol/m2-s?
(2) If the reversible dissociative chemisorption of H2 on the palladium surface influences the hydrogen flux,
then hydrogen flux will be proportional to pnH2 f  pnH2 p , where n values higher than 0.5 are observed.
Collins and Way (1993) have correlated their H2 permeation data successfully with n 0.573 for a 17 m
palladium film at two different temperatures (see Table 3.P.6). Determine an average value of the
hydrogen permeability QH2 m through the palladium film at the two temperatures.

Table 3.P.6

873 K 723 K
p0:573 0:573
H2 f  pH2 p Pa
0:573

N H2 m mole=m2 -s N H2 m mole=m2 -s

1200 0.5 0.32


1800 0.75 0.5
2400 1.00 0.66

(3) If the temperature dependence of QH2 m is described by an Arrhenius type of relation,


 
EH
QH2 m Q0H2 m exp  2 ;
RT

where E H2 is the apparent activation energy, determine the values of E H2 and Q0H m . (Ans. (1) gmol-m/m2-s-
2
Pa0.5; (2) 873K, 7.08  109 gmol-m/m2-s-Pa0.573.)
Problems 201

3.4.12 In Example 3.4.5 involving microporous cellulose membranes used in hemodialysis, consider the situation
where there are boundary layer resistances on two sides of the membrane. For the transport of a microsolute
through the membrane from an aqueous solution, Lane and Riggle (1959) have found that, for their membrane,
feed and permeate aqueous solutions, the mass-transfer coefficients on each of the feed and permeate are given
by klf klp 1000 Dil cm/min, where Dil is the diffusion coefficient of solute i in water in cm2/s. Obtain an
estimate of the overall mass-transfer coefficient of Na2SO4 for such a membrane system with boundary layer
resistances. (Ans. K 3  105 cm/s.)

3.4.13 In electrodialysis, the analysis in Section 3.4.2.5 assumed a perfect cation exchange membrane and no
convection of water through an ion exchange membrane placed in an external electrical field. Consider a
pressure difference across this membrane without any applied electric field r such that water flows through
the membrane. This water flow will drag the counterions in the membrane downstream in the membrane. Thus
the high-pressure interface will be depleted of counterions, whereas the low-pressure interface will have an
excess of counterions; this charge separation will create an electrical field. The field created by charge
separation will prevent any further counterion flow in the downstream direction. We are interested in finding
the ion exchange equilibrium under this condition. Suppose, however, that the membrane is imperfect; a few
co-ions enter the membrane (Dresner, 1972). Then a steady flux of counterions can take place as long as there is
a flux of co-ions of equal but opposite charge. This is equivalent to, say, a salt flow. This salt flow is determined
by the extent of coion flow.
Integrate the extended NernstPlanck equation for any counterion i in the membrane in the z-direction
perpendicular to the membrane surface when the membrane excludes co-ions perfectly. Determine the
constant of integration in terms of counterion concentration and resin-phase potential just inside the mem-
brane, CiR (0) and R (0). Use now the condition of thermodynamic equilibrium for the counterion at the
interface (between CiR(0) and Ciw via relation (3.3.118b)) and the assumption that w 0 to relate CiR(z) to
Ciw. Obtain R(z) by using the electroneutrality condition (3.3.30b).

3.4.14 Consider a microporous hydrophilic membrane with an aqueous solution on one side and an organic solvent
on the other side. Let the pores of the hydrophilic membrane contain the aqueous solution. If the organic-
phase pressure is higher than that of the aqueous phase (but does not exceed a critical pressure difference), the
aqueousorganic phase interface will be immobilized at each pore mouth on the organic side of the membrane.
(1) Draw a figure similar to Figure 3.4.11 and show the relevant solute concentration profiles for extraction
from the aqueous to the organic phase.
(2) Develop the relations between Ko and the individual coefficients and Kw and the individual coefficients.
(3) Obtain the simplified forms of these two relationships for the limiting conditions io << 1 and io >> 1.
3.4.15 Consider microporous membrane-based solvent extraction using a hydrophilic porous membrane whose pores
are filled with the organic solvent used to extract a product, species i, from an aqueous feed solution. The
aqueous-phase pressure is maintained equal to or higher than the organic-phase pressure to maintain the
aqueousorganic interface for nondispersive solvent extraction conditions.
(1) Identify the interface location and plot the concentration profile of i.
(2) Write down the flux expressions of species i through the various resistances.
(3) Develop a relation between the overall mass-transfer coefficient based on the organic phase and the
individual mass-transfer coefficients.
(4) Simplify the above result for conditions where the solute distribution coefficient, io , is either very large
(io >> 1) or very small (io << 1).

3.4.16 Hydrogen is to be recovered from a gas mixture by permeation through a thin film (thickness m) of palladium
(Pd) membrane into an aqueous solution prior to chemical reaction in the aqueous solution. The diatomic gas
H2 is present in the feed gas at a partial pressure pH2 gf . The hydrogen concentration in the bulk water is C H2 wb ;
the mass-transfer coefficient of H2 in the aqueous film next to the palladium membrane is k H2 w . The
permeability coefficient of H2 through the Pd membrane via Sieverts law is QH2 m .
(1) Obtain a relation between an overall H2 transfer coefficient K H2 w based on the aqueous phase, (QH2 m =m ,
k H2 w and any other quantities.
(2) Compare such a relation with a relation like (3.4.11) and comment on the complexities encountered.
Suggest a procedure, experimental or otherwise, to overcome the problem. Assume that Henrys law is
valid. Use the form pH2 g H cH2 C H2 w . Also assume that the palladium membrane allows only H2 to pass
through.
Appendix

Diffusion coefficients in different systems

Note: Table 3.A.8 includes equivalent ionic conductances of selected ions at infinite dilution.

Table 3.A.1 Binary gaseous mixtures at 1 atm

System AB Temperature ( C) DAB (cm2/s)

H2N2 0 0.674
HeN2 20 0.705
EthanolH2 0 0.377
H2Oair 0 0.220
Ethanolair 0 0.102
N2CO2 25 0.165
N2O2 0 0.181
EthanolCO2 0 0.0685

Source: Perry et al. (1984, table 3-318).

Table 3.A.2. Diffusion coefficients of solute i in a dilute liquid solution, 1 atm at 25  C

System is Type of solute D0is  105 cm2 =s

H2H2O gaseous 5.85


O2H2O gaseous 2.50
N2H2O gaseous 1.90
CO2H2O gaseous 1.96
SO2H2O gaseous 1.70
H2SH2O gaseous 1.61
CO2ethanol gaseous 4.0
O2glycerol water (106 poise) gaseous 0.24

Source: Perry et al. (1984, table 3-319).


Appendix 203

Table 3.A.3. Diffusion coefficients of solute i in a dilute liquid solution, 1 atm at 25  C

System is Type of solute D0is  105 cm2 =s

Toluenen-hexane liquid 4.21


Toluenedodecane liquid 1.38
BenzeneCCl4 liquid 1.53
Acetic acidtoluene liquid 2.26
Acetic acidethylene glycol liquid 0.13
Acetic acidH2O liquid 1.24
Formic acidethylene glycol liquid 0.094
n-PropanolH2O liquid 1.1
n-ButanolH2O liquid 0.96
UreaH2O liquid 1.37
SucroseH2O solid 0.56
GlucoseH2O solid 0.69

Source: Perry et al. (1984, table 3-319).

Table 3.A.4. Diffusion coefficients of selected ions at infinite dilution in water at 25  C

Ion Zi Diw  105 (cm2/s) Ion Zi Diw  105 (cm2/s)

H 1 9.31 OH 1 5.26


Li 1 1.03 Cl 1 2.03
Na 1 1.33 Br 1 2.08
K 1 1.96 I 1 2.04
NH 4 1 1.95 NO 3 1 1.90
Mg 2 0.706 HCO 3 1 1.10
Ca 2 0.792 SO2
4 2 1.065
Cu 2 0.72 HSO4 
1 1.33
Zn 2 0.71 FeCN36 3 0.896
Co(NH3)
6 3 0.908 FeCN46 4 0.739

Source: Newman (1973, table 75-1).

Table 3.A.5. Diffusion coefficients of proteins and other macromolecules in pure water at 20  C

Solute i Type of solute Mi D0iw  107 cm2 =s

Lysozyme protein 14 400 10.4


Ovalbumin protein 45 000 7.76
Serum albumin protein 65 000 5.94
Catalase protein 250 000 4.1
Ureasea protein 480 000 3.46
Myosina protein 493 000 1.16
Polymethyl methacrylate macromolecule in acetone 106 2.25
DNA macromolecule 6  106 0.13
a
The conformation, especially deviation from sphericity, will lead to different values of D0iw for similar Mi. For an
interpretation of D0iw , see Tanford (1961). The macromolecule DNA (deoxyribonucleic acid) is a linear flexible
macromolecule, which is essentially a polyelectrolyte and is usually double-stranded in nature (see Russel et al. (1989)
and Viovy (2000)).
Source: Tanford (1961, tables 21-1, 21-3).
204 Physicochemical basis for separation

Table 3.A.6. Diffusion coefficient of gases through a polymeric film at 25  C

Gas i Material Di cm2 =s

He polyvinylchloride 2.3  107


O2 N-methyl dithiocarbamate 6  108
Ar 2.3  109
N2 1.7  109
Kr 4  1010

Source: Hwang and Kammermeyer (1975, fig. 11.1).

Table 3.A.7. Diffusion coefficients in miscellaneous solids

Species i Type of solute Solid material Temperature ( C) Di (cm2/s)

He gas SiO2 20 (2.45.5)  1010


H2 gas Ni 85 1.16  108
CO gas Ni 950 4  108
Cd metal Cu 20 2.7  1015

Source: Geankoplis (1972, table 4.1-1).

Table 3.A.8. Equivalent ionic conductances of selected ions at infinite dilution

   
Ion Zi Temp. (K) mho-cm2 Ion Zi Temp. (K) mho-cm2
0i 0i
g equiv: g equiv:

H 1 298 350 OH 1 298 198


308 397
Li 1 298 38.7 Cl 1 298 76.3
308 92.2
Na 1 298 50.1 Br 1 298 78.3
308 61.5
K 1 298 73.52 I 1 298 76.8
NH 4 1 298 73.4 NO 3 1 298 71.44
Mg 2 298 53.06 HCO 3 1 298 41.5
2
Ca 2 298 59.5 SO4 1 298 80
HSO 4 1 298 50

Sources: Newman (1973, table 75-1) and Atkinson (1972, pp. 5249 to 5263).
4

Separation in a closed vessel

In Section 3.3, we illustrated the thermodynamic relations closed vessel into two regions. We study the rate-governed
that govern the conditions of equilibrium distribution of a basis of separation using membranes and identify the need
species between two or more immiscible phases under for bulk flow in and out of the vessel to make such pro-
thermodynamic equilibrium. In Section 4.1, we focus on cesses useful for separation. A single semipermeable ion
the value of the separation factor or other separation exchange membrane, however, allows equilibrium separ-
indices for two or more species present in a variety ation to be achieved in Donnan dialysis; it has also been
of two-phase separation systems under thermodynamic covered in Section 4.3.
equilibrium in a closed vessel. The closed vessels of
Figure 1.1.2 are appropriate for such equilibrium sepa-
ration calculations. There is no bulk or diffusive flow into 4.1 Equilibrium separation between two phases or
or out of the system in the closed vessel. The processes two regions in a closed vessel
achieving such separations are called equilibrium sepa-
The separation achieved between two species distributed
ration processes. Separations based on such phenomena
between two phases or two regions is considered in this
in an open vessel with bulk flow in and out are studied in
section. In Section 3.3.7, the equilibrium distribution of
Chapters 6, 7 and 8. No chemical reactions are considered
one species between two phases was determined in many
here; however, partitioning between a bulk fluid phase
two-phase systems. Those results will be employed here
and an individual molecule/macromolecule or collection
for individual equilibrium separation processes in a closed
of molecules for noncovalent solute binding has been
vessel which becomes an ideal stage. There will be add-
touched upon here. The effects of chemical reactions are
itional considerations here on the distribution of one
treated in Chapter 5. Partitioning of one species between
species between two immiscible phases at equilibrium.
two phases is an important aspect ever present in this
First, however, a few general results are provided for esti-
section.
mating the separation factor 12 between two species 1
The criteria for thermodynamic equilibrium in a
and 2 in a two-phase system at equilibrium. We ignore
single-phase system in a closed vessel subjected to an
chemical reactions for any separation conditions discussed
external force field were also developed in Section 3.3.
here.
Based on these criteria, we develop in Section 4.2 esti-
Consider any species i distributed between two phases
mates of the separation achieved in a single phase in
j 1, 2 at a uniform pressure P and uniform temperature T.
the closed vessel. These estimates are also developed in
The chemical potential of species i in phase/region j is
a closed vessel when an additional property gradient,
given by
e.g. density gradient, pH gradient, etc., exists across the
vessel length. Focusing is the term often used to charac- ij P, T, x ij 0ij P 0 , T V ij P j P 0 RT ln aij
terize the latter separation techniques. In this section,
we cover in addition the extent of separation achieved from relation (3.3.21). Rewrite this as
when a temperature gradient is imposed on a single-phase ij 0ij V ij P j P 0 RT ln aij :
system in a closed vessel not subjected to any external
force field. For species i distributed between phases j 1 and 2, the
We consider in Section 4.3 the time-dependent separ- criterion (3.3.6) for equilibrium, namely i1 i2, leads in
ation in a closed vessel where a membrane partitions the this case to (for P 1 P 2 P P 0 )
206 Separation in a closed vessel

0i1 RT ln ai1 0i2 RT ln ai2 separating agent. If heating is employed to regenerate the
absorbent, heat or thermal energy is the energy-separating
or agent.
ai1

0 0
 
0
 The following treatments cover the calculation of sep-
exp i1 i2 exp i K ai , 4:1:1 aration achieved in a closed vessel after equilibrium has
ai2 RT RT
been attained via absorption or stripping.
where

0i 0i1 0i2 4:1:2 4.1.1.1 Gas/vapor absorption


and K aiis the equilibrium ratio of species i between We consider gas absorption first. From Section 3.3.7.1 for
two phases 1 and 2 in terms of the activity of species i gasliquid equilibrium, since
(instead of mole fraction used in definition (1.4.1)).
The equilibrium ratio K ai of species i depends on 0i1 x il ^ ig P

and 0i2 , the thermodynamic constants for species i for a x ig il f 0il
given combination of i, j (1, 2), P0 and T. One can, in
we obtain, for gaseous species i 1,2 and j l (1) and
principle, calculate 0i1 and 0i2 from the principles of
j g (2),
thermodynamics.
For two species i 1,2 distributed between phases x 1l x 2g x 11 x 22 ^ 12 f 021 21
12 : 4:1:6
j 1,2 at equilibrium, employ (4.1.1) and obtain an x 1g x 2l x 12 x 21 ^ 22 f 011 11
activity based separation factor:
Here the liquid phase is the preferred phase (j 1) for the
a11 a22 01 02 Ka
 
exp 1a : 4:1:3 preferred gas species (i 1) being absorbed. When
a12 a21 RT K2 Henrys law is applicable for the gas species i 1,2, the
following expression is obtained for the separation factor
Since aij ijxij by definition (equation (3.1.35)), where ij
12 between species 1 and 2:
is the activity coefficient of species i in phase j, we obtain
^ 12 2l H 2

x 11 x 22 K 1 21 12 01 02 Ka
 
12 , 4:1:7
12 exp 21 12 1a : ^ 22 1l H 1

x 12 x 21 K 2 11 22 RT 11 22 K 2
4:1:4 where f 0i1 H i , Henrys constant for species i. If both the
liquid phase and the gas phase behave ideally,
For ideal solutions or mixtures, ij 1 for any i,j combina-
tion. A simple relation is obtained for 12: H2
12 : 4:1:8
H1
01 02 K1
 
12 exp 4:1:5
RT K2 We can understand this result from the following simplistic
explanation based on Henrys law (relation (3.3.60b)):
if ideal solution behavior exists in both phases j 1,2.
For many systems, the fugacity based equilibrium pig H i x il :
criterion f^i1 f^i2 is used instead of the chemical potential
Typical units of Hi are atmospheres/mole fraction. If H2 >
based criterion (3.3.6). By expressing f^ij appropriately in
H1, then, for the same partial pressure (p1g p2g) or mole
terms of xij and other thermodynamic quantities, an
fraction in the gas phase, the mole fraction of gas species 2
expression for 12 may be developed for every system.
in the liquid phase is less than that of gas species 1. Species
1 is thus more soluble in the liquid phase and therefore
4.1.1 Gasliquid systems
may be separated from species 2 by absorption in a suit-
Consider a gas mixture or vapors in a gas mixture. If this able liquid. Gas absorption based separation processes
mixture is in contact with a nonvolatile absorbent liquid, utilize this preferential solubility of some gases in selected
which absorbs selectively one of the gases or the vapor liquid absorbents. The values of Henrys constant Hi in
species, the process is generally called absorption and the units of atmospheres/mole fraction for a variety of gases
absorbent liquid acts as a mass-separating agent. In con- in water are provided in the handbook by Perry and Green
tinuous industrial processing studied in Chapter 8, the (1984). For some species, the values are given at a number
spent absorbent liquid has to be regenerated. The regener- of temperatures and values of pig. The latter does indicate a
ation is carried out using a stripping gas or a stripping weak dependence of Hi on pig.
vapor, or by vacuum or heating to remove the absorbed A common example involves the removal of acid gases,
gas species. This process of volatile species removal from a e.g. SO2, H2S, CO2, COS, etc., from a gas stream by absorp-
liquid is identified as stripping. If a stripping gas or vapor tion in a solvent. Table 4.1.1 identifies the values of Hi
(more commonly steam) is used, both act as a mass- for a number of gaseous species in a few absorbents for
4.1 Two-phase equilibruim separation: closed vessel 207

Table 4.1.1. Henrys law constantsa and separation factors for acid gases in absorbents

Solvent (temperature) H2S Hi  104 CO2 Hi  104 CH4/C3H8 Hi  104 H2 SCO2 CO2 CH4 CO2 C3 H8

Waterb (20  C) 0.0483 0.142 3.76 (CH4) 2.94 26.5


n-Methyl-pyrrolidonec (20  C) 0.000476 0.00588 0.0829 (CH4) 12.35 14.11
Methanold (7  C) 0.00405 0.0160 0.115 (CH4) 0.0245 (C3H8) 3.96 7.2 1.53
a
Units of Hi, atm/mole fraction.
b
National Research Council (1929).
c
From Kohl and Riesenfeld (1979, p. 783).
d
From Astarita et al. (1983, table 1.7.2).

removing H2S and CO2 from CH4/C3H8. The separation atmospheric pressure. Equation (4.1.10) may be used to
factors have also been calculated to illustrate which species estimate, in general, the variation of 12 with temperature.
is more selectively absorbed and which absorbent provides Absorption of a condensable vapor species in a nonvo-
a high selectivity. Much more information on a variety of latile absorbing liquid from a mixture of vapors (or from a
other absorbents is provided in Astarita et al. (1983) and mixture of vapors in a gas or a gas mixture) will be briefly
Perry and Green (1984). treated in the following. Using the expression (3.3.61) for
Table 4.1.1 identifies three pure absorbents, one of the equilibrium ratio Ki of a vapor species i between a gas
which is water. We shall see in Section 5.2.1.2 that a common and a liquid phase at equilibrium, we may develop the
absorbent for acid gases is water containing either organic following expression for the separation factor 12 between
ethanolamines or inorganic salts. The value of the Henrys two vapor species i 1,2:
law constant Hi for various gases in such aqueous electrolytic
solutions may be related to that in pure water H 0i by x 1l x 2g K 2 ^ 1g 2l f 02l
12 : 4:1:11
x 1g x 2l K 1 2g 1l f 01l
^
Hi
 
log10 hI, 4:1:9a
H 0i Here, species 1 is preferred for absorption into the pre-
where I, the ionic strength of the solution, is defined as ferred phase, the liquid phase. For low to moderate pres-
follows: sures, we know that

C Z2 f 0il P sat sat


i i ,
X
n il i
I : 4:1:9b
2 where P sat is the vapor pressure of species i at system
i
temperature T. For ideal gas behavior, sat i 1 and
The solution contains n ionic species; species i has a ^ ig 1. Further, for ideal solution behavior in the liquid
valence of Zi at a molar concentration of Cil gmol/cm3. phase, il 1. These simplifications (i.e. Raoults law
Contributions of positive ions, negative ions and dissolved (3.3.64) holds) lead to:
free gas species (h, h and hG, respectively) have been
P sat
2
tabulated by Danckwerts (1970) for calculating the quantity 12 : 4:1:12
P sat
1
h in definition (4.1.9a) as
This result implies that species 1 will be preferentially
h h h h G : 4:1:9c
absorbed if it has a vapor pressure lower than that of
In general, the solubilities of most gases in an absorbent species 2. Thus, heavier species or higher boiling species
decrease with an increase in temperature. This implies, are more selectively absorbed.
from Henrys law, that Hi increases with temperature. In The above treatments primarily consider systems and
fact, Henrys constant for species i changes with tempera- conditions relevant for the large-scale absorptions of
ture in the manner of gases/vapors in absorbent liquids employed in the chem-
ical processing industry. These principles are also relevant
d ln H i H i
, 4:1:10 for gasliquid chromatography (GLC), wherein a mobile
d1=T R
gas phase containing sorbable species flows over a non-
where T is the absolute temperature, R is the universal gas volatile stationary liquid phase existing as a thin coating
constant and Hi is the heat of absorption of species i. Thus, over a porous solid matrix or a capillary tube. The selectiv-
if Hi is negative, i.e. heat evolves on absorption of the gas ity 12 of vapor species 1 over vapor species 2 present in the
species, Hi increases, reducing the amount of gas dissolved. mobile gas phase (see Section 7.1.5) will be obtained from
For many common gases and water, the opposite behavior expression (4.1.11). For low-pressure operations, assume
is observed at temperatures greater than 100  C and above ideal gas behavior ^ ig 1 and sat
i 1. Therefore
208 Separation in a closed vessel

x 1l x 2g 2l P sat
2
Assuming ideal gas and ideal solution behavior,
12 : 4:1:13
x 1g x 2l 1l P sat
1 P sat
1
12 : 4:1:17
P sat
2
Thus, a higher boiling species would prefer the solvent
phase, the stationary phase, and, as we shall see later Species 1, having a higher vapor pressure, is more easily
(Section 7.1.5), it will have a higher retention time unless stripped and accumulates preferentially in the gas phase.
the activity coefficient ratio affects it. The nonvolatile liquid The example given in the following will show that such a
phase, the absorbent, can, however, modify this behavior result may not hold if nonideal behavior exists.
through the activity coefficients. A common example of vapor stripping involves removal
of volatile organic compounds (VOCs) from water either
obtained within a chemical process plant or discharged
4.1.1.2 Gas/vapor stripping
from it. Such a system rarely displays ideal solution behav-
The removal of a volatile species, gas or a vapor, from a ior. The selectivity 12 then becomes
nonvolatile absorbent or solvent by an inert gas or vapor or   sat 
P1
by simple heating is called stripping. When air is used as 12 1l : 4:1:18
2l P sat
2
the inert gas, the process is called air stripping; in the case
of steam, the term steam stripping is used. The word In industrial practice, the vaporliquid equilibrium of
desorption is also employed. such VOCs employs the ideal gas assumption, and relation
For stripping processes, the preferred species i 1 (4.1.14) is reformulated as
(which may be a gas or a vapor) should be preferably
transferred to the preferred gas phase j 1 g. When il f 0il f 0
x ig x il il il x il , 4:1:19a
selectivity between two gas species i 1,2 being stripped P P
from a nonvolatile liquid by steam or air or just heating is where il represents an infinite dilution activity coefficient
under consideration, we employ the gasliquid equilib- since xil tends to be very small. A useful form of this relation is
rium relation from Section 3.3.7.1:
Px ig pig H Ci C il 18  106 P  f 0il il =PC il , 4:1:19b
0
^ ig P f^il f^i2 x il il f 0il ) x ig il f il :
f^ig f^il x ig where P is in atmospheres and Cil is in gmol/m3; the value
x il ig P
^ of (f 0il il =P), represented as K i , an infinite dilution equilib-
4:1:14 rium ratio, has been provided for a large number of VOCs
by Hwang et al. (1992b) at 100  C (for steam stripping) and
Therefore, for i 1,2, 25  C (for air stripping) along with f 0il and il values. Using
this notation,
^ 2g f 0
x 1g x 2l 1l
12 1l
: 4:1:15a x 1g x 2l K 1
^ 1g f 02l
x 1l x 2g 2l 12 , 4:1:19c
x 1l x 2g K 2

For gaseous species satisfying Henrys law for the system where the subscript of K i identifies the species. For
under consideration, f 0il H i . Further, if the liquid phase example, K 1 4.1  102 for toluene (25  C), whereas the
behaves as an ideal solution and the gas phase behaves as corresponding value for acetone is only 2.1 (25  C). Thus,
an ideal gas, toluene appears to be easily strippable by air compared to
acetone, even though acetone has a much lower boiling
x 1g x 2l H 1 point. Toluene is much more hydrophobic than acetone
12 : 4:1:15b and has a very large value of il (~104 compared to 17.2 for
x 1l x 2g H 2
acetone). The more hydrophobic a compound is, the lower
For species 1 and 2, if H1 > H2, then, for equal liquid-phase its aqueous solubility and the higher its strippability.
mole fractions of 1 and 2 (i.e. x1l x2l), the partial pressure An important aspect of VOC stripping concerns the
of species 1 will be higher in the gas phase, i.e. species 1 is stripping temperature and therefore the stripping medium.
more easily stripped. Should one use air stripping (25  C) or steam stripping
When species 1 and 2 are both vapors at the system (100  C)? Analysis based on 12 of (4.1.19c) is not useful.
temperature and pressure in equilibrium with an appropriate Comparison of K i values at two different temperatures is
liquid phase, we employ relation (3.3.61) for i 1 and j g more useful (Hwang et al., 1992a).
1 for the preferred species and phase, respectively, to obtain
4.1.2 Vaporliquid systems
sat
x 1g x 2l K 1 1l P sat
1 1 2g
12 sat : 4:1:16 Vaporliquid systems are encountered frequently in indus-
x 1l x 2g K 2 2l P sat
2 2 1g
trial processing. The composition difference between the
4.1 Two-phase equilibruim separation: closed vessel 209

vapor and the liquid phases made up of volatile species The Gibbs phase rule for a closed nonreactive system
only is the basis for separation by distillation, which of j phases made up of n species states that, at equilibrium,
involves boiling and condensation. In this section, we first the number of degrees of freedom F is given by
determine the separation factor between the vapor and the
j F n 2: 4:1:22
liquid phases in a closed vessel at equilibrium under con-
ditions where Raoults law may or may not be valid. We Once the intensive variables2 F are specified, the system
consider next a variety of vaporliquid systems encoun- is completely defined. For example, in a binary system
tered in practice. Binary systems are frequently employed (n 2) having vaporliquid equilibrium ( j 2), the
here for simplicity and illustration, along with the Gibbs degrees of freedom available are 2 ( F). Once the values
phase rule. Then we study two extreme situations in the of any two intensive variables are fixed, the values of all
closed vessel, when almost all of the material is liquid or other intensive variables become fixed. If values of pres-
almost all of the material is vapor and calculate the sure and temperature are specified, the compositions of
unknown phase composition for estimating the separation. the two phases of the system are determined. Similarly, if
Multicomponent systems are of primary interest in such the pressure and liquid mole fraction are specified, the
calculations. composition of the vapor phase and the value of the tem-
Using the equality of fugacity of a species in vapor perature become fixed. Alternatively, if the temperature
( j v) and liquid phases ( j l ), and the vapor phase mole fraction are given, the pressure
and the liquid phase mole fraction will be fixed automatic-
^ iv P f^il x il il f 0il ,
f^iv x iv
ally from the solution of the governing equations.
and the relation (3.3.62) for f 0il at low to moderate pressures, Before we present briefly the methodology for such
we get: calculations, it is worthwhile illustrating the basic types
of vaporliquid equilibrium (VLE) behavior frequently
x iv il sat
i Pi
sat
: 4:1:20 encountered. The variety in VLE arises primarily from
x il ^ iv P
different types of nonideal behavior, as indicated, for
For two species i 1 and 2, example, by the liquid-phase activity coefficients. Minimum-
 sat   sat  boiling azeotropes and maximum-boiling azeotropes are
x 1v x 2l P1 ^ 2v

1l 1 two such types of nonideal behavior. To become familiar
12 , 4:1:21a
x 1l x 2v P sat
2 2l ^ 1v
sat
2 with such behavior, we start with the ideal solution
behavior.
where the preferred phase for the preferred (lighter)
Consider VLE for a binary system containing benzene
species (i 1) is j v, the vapor phase. If the vapor phase
(species i 1) and toluene (i 2) at a low pressure, as
is an ideal gas and the liquid phase is an ideal solution,
shown in Figure 4.1.1. Figure 4.1.1(a) (Hougen et al., 1959)
Raoults law (relation (3.3.64)) is valid. The expression for
illustrates the partial pressure of each species and the total
12 is simplified to
pressure P of the system at 90  C. Due to ideal gas behavior
P sat
1 (low pressure) and ideal liquid solution behavior, Raoults
12 4:1:21b
P sat
2 law (3.3.64) is valid:
at the system temperature T and pressure P. This ratio is P sat
1 P sat
x 1v x 1l , x 2v 2 x 2l :
also called the relative volatility, the value of 12 under P P
ideal conditions. If only the vapor phase behaves as an
ideal gas, then
Since the temperature is constant at 90  C, the value of the
separation factor between species 1 and 2,
 sat  
P1 1l
12 : 4:1:21c
P sat 2l
2 P sat
1 1000
12  2:5,
The vapor pressure Pisat depends on temperature (e.g. P sat
2 400
Antoine equation).1 The activity coefficient il depends on is also constant with respect to the composition of any
the composition of the liquid phase and temperature. The phase (see Figure 1.4.1). The total pressure P may be
distillation separation process utilizing vaporliquid equi- expressed as the sum of the partial pressures p1v and p2v
librium has to achieve changes from the initial compos- as follows:
ition to the desired composition. A few thermodynamic
principles and features of vaporliquid equilibria are there-
fore quite useful here. We start with Gibbs phase rule.
2
The intensive variables are temperature, pressure and (n1)
mole fractions in each phase present. For a two-phase two-
component system, one mole fraction of each phase is enough,
1 Bi
ln P sat
i Ai TC i ; Ai, Bi and Ci are constants. since x1j x2j 1.
210 Separation in a closed vessel

(a)
1100
1000
900 (b)
P
800 4.0
ure
Pressure (mmHg)
ss

e
re 3.0

en
700 lp
ta

nz
2.0

Activity coefficient, g1l


To

be
600

of
p 1n

re
500 1.0

su
es
400 Par 0.7
pr
ti al
pre 0.5
300 ssu 0.4
l
tia

re o 0.3
f to
r

200
Pa

lue
p 2n ne 0.2
100
0.1
0 0.1 0.2 0.3 0.4 0.5 0.6 0.7 0.8 0.9 1.0 0 0.1 0.2 0.3 0.4 0.5 0.6 0.7 0.8 0.9 1.0
Mole fraction benzene in liquid phase, x1l Mole fraction benzene in liquid phase, x1l
(c)
1.0
0.9 (d)
0.8 120
ether in vapor phase, x1n
Mole fraction isopropyl

0.7
e

1l

Vapor
=x
lin

0.6 110
Temperature (C)
ium

1n
;x

De
0.5
ibr

w- p
ne

oin
uil

li

Va t li
100 ne
45

0.4
Eq

po
Bu A r +
bb liq B
0.3 le- uid
po
in t
0.2 90 lin
e
Liquid
0.1
80
0 0.1 0.2 0.3 0.4 0.5 0.6 0.7 0.8 0.9 1.0 0 0.1 0.2 0.3 0.4 0.5 0.6 0.7 0.8 0.9 1.0
Mole fraction benzene in liquid phase, x1l Mole fraction benzene, x1j

Figure 4.1.1. Ideal solution behavior, benzenetoluene system: (a) partial and total pressures at 90  C; (b) activity coefficients; (c) vapor
liquid equilibrium; (d) phase diagram at 1 atm pressure. (After Hougen et al. (1959).)

P p1v p2v x 1v P x 2v P P sat


1 P sat
x 1v x 1l , x 2v 2 x 2l :
P sat x 2l 12 x 1l ) P P sat x 1l 12 1: 4:1:23 P P
2 2 1

This linear dependence of P on x1l, as well as that of p1v on Divide x2v by x1v, recognize that x2v 1 x1v and simplify
x1l and p2v on x2l, are shown in Figure 4.1.1(a). The ideal to obtain (see equation (1.4.76))
liquid solution behavior of 1l 2l 1 is shown in 12 x 1l
Figure 4.1.1(b). x 1v : 4:1:24
1 x 1l 12 1
Figure 4.1.1(c) illustrates the equilibrium vapor and
liquid compositions for the benzenetoluene system at a Note that different values of x1l (and the corresponding x1v)
constant total pressure of 1 atmosphere. The vapor phase in Figure 4.1.1(c) represent different closed vaporliquid
is enriched in the lighter component, benzene, at all com- equilibrium systems at 1 atmosphere but slightly different
positions, except in the case of pure benzene and pure temperatures. Note further that 12 in equation (4.1.24)
toluene. If such vapor-phase enrichment of benzene did varies slightly with temperature and is not a constant (since
not occur, the straight reference line (x1v x1l) would have both P1sat and P2sat vary with temperature).
represented the VLE. The actual curved equilibrium line The boiling point of a liquid mixture at a given total
relating x1v to x1l may be obtained in the following manner. pressure is shown in Figure 4.1.1(d) as the bubble-point
Consider first Raoults law expressions for i 1 and i 2: line for the benzenetoluene system at 1 atmosphere.
4.1 Two-phase equilibruim separation: closed vessel 211

Oxygennitrogen Figure 4.1.3 (Hougen et al., 1959) illustrates the VLE of


an isopropyl etherisopropyl alcohol system characterized
132.86
by il > 1, i.e. positive deviations from Raoults law. Figure
127.04 4.1.3(a) illustrates the variations of the total pressure and
P = 20 the partial pressures as a function of composition at 70  C.
P = 15 Figure 4.1.3(b) shows the activity coefficients of both
119.67
species varying with the liquid composition. Since il > 1,
115.67
108.93 the partial pressure of any species is larger than a straight-
line behavior exhibited by an ideal solution. Further, the
Temperature (K)

110.53
P = 10
90.16 maximum of the total pressure is greater than either P sat i
P=5
103.94 (recall that there is no such maximum in an ideal solution
P=1 84.01 system). Such behavior leads to the formation of an azeo-
94.23
trope: at a certain composition, both liquid and vapor have
77.33 P = 0.5 the same composition, as shown in Figure 4.1.3(c). Corres-
pondingly, at this composition, the bubble-point line and
71.91 the dew-point line have a common minimum identifying
0 10 20 30 40 50 60 70 80 90 100 the minimum boiling point (Figure 4.1.3(d)). There is no
Mole % of O2 separation achieved between the two phases at the azeo-
trope composition.
Figure 4.1.2. (T, xil ) curves at 0.5, 1, 5, 10, 15 and 20 atm for an Positive deviations from Raoults law arise when the
O2N2 system (Dodge and Dunbar, 1927). Reprinted, with permis- forces between two molecules of two different species are
sion, from J. Am. Chem. Soc. 49, 591 (1927), Figure 6. Copyright weaker than those between two molecules of the same
(1927) American Chemical Society. species. Negative deviations from Raoults law happen
when the intermolecular forces between two different
The composition of the first bubble in equilibrium with the species are stronger than those between two molecules of
liquid mixture at the boiling point temperature is shown by the same species. Figure 4.1.4 shows the VLE of a system of
the upper line. The straight line AB joining these two this type: that of acetone and chloroform (Hougen et al.,
equilibrium compositions is called a tie line. Conversely, 1959).
when a vapor having any composition is cooled, its tem- The negative deviations in the acetonechloroform
perature comes down to the upper line, the dew-point line. system are clearly evident in Figure 4.1.4(a), where the
The first drop of liquid that condenses from this vapor at partial pressure of each species is lower than that indicated
this temperature will have its composition given by the by the straight-line behavior that would be true in a system
bubble-point line at that temperature, the two being con- exhibiting ideal solution behavior (e.g. benzenetoluene).
nected by another horizontal tie line. The escaping tendency of a given species into the vapor
In low-pressure VLE satisfying ideal gas and ideal solu- phase is reduced by attraction from molecules of other
tion behavior, Raoults law can be used very effectively to species. The total pressure over the system has a minimum,
obtain the values of the intensive variables in a closed which leads to a maximum boiling point. The activity coef-
system at equilibrium. Unfortunately, there are many VLE ficient il for each species is less than 1 (Figure 4.1.4(b)).
systems where Raoults law is not obeyed. Specifically, in Such a system is called a maximum boiling point azeotrope.
many cases of low-pressure VLE, the liquid phase behaves At the azeotropic composition, xiv xil (Figure 4.1.4(c)); the
nonideally; il varies with composition and is 61. The system exhibits a maximum boiling point at the same
nature of the deviation of il from 1 often determines the composition (Figure 4.1.4(d)).
nature of the VLE. Systems where il > 1 are said to display For systems having substantial positive deviations from
positive deviations from Raoults law; when il < 1, the Raoults law, one sometimes encounters limited solubility
systems show a negative deviation from Raoults law. in the liquid phase. Two liquid phases, substantially
Before we illustrate the VLE system behavior for il 6 1, immiscible with each other, coexist over a certain compos-
note that deviations from Raoults law occur also due to ition, whereas, at other compositions, a single liquid-phase
nonideal behavior in the gas phase. For example, for an exists. An example is provided by the watern-butanol
O2N2 system at low temperatures, where the gases are system (Hougen et al., 1959).
liquefied, it has been observed that the partial pressure of For a binary system like benzenetoluene, exhibiting
N2 is lower than that predicted by the straight-line behav- ideal solution behavior at low pressures, we have already
ior shown in Figure 4.1.1(a). Yet the Txil behavior shown learned how to calculate xiv from xil using Raoults law
in Figure 4.1.2 for O2N2 appears similar to that in Figure (more specifically, using relation (4.1.24)). The calculation
4.1.1(d); only the N2 boiling point is lower than that of O2 procedures for vaporliquid equilibrium in multicompo-
(Dodge and Dunbar (1927); Ruhemann (1949)). nent systems behaving nonideally will now be treated
212 Separation in a closed vessel

(a) (b)
1000 6.0
900 re 5.0
ssu
800
P pre
tal ther 4.0
To yl e

Activity coefficient, g1l


Pressure (mmHg)
700
p rop
iso
600 of p1n 3.0
ure

l
ho
500 Par t s s Iso
ial re

lco
pro

p
pres py

la
400 sure 2.0 le g2l

py
of is g1l the
al

ro
300 opro r
r ti

op
pyl a
Pa

Is
p2n lco 1.5
200 h

ol
100
0 1.0
0 0.2 0.4 0.6 0.8 1.0 0 0.2 0.4 0.6 0.8 1.0
Mole fraction isopropyl ether in liquid phase, x1l Mole fraction isopropyl ether in liquid phase, x1l
(c) (d)
1.0 100
0.9
0.8 90
Mole fraction benzene

Vapor
in vapor phase, x1n

0.7
Temperature (C )
e
lin
m
0.6 riu 80 De
w-p
lib oi n
0.5 ui x 1l t lin Vapor + liquid
Eq = e
x 1n
0.4 ne; 70 Bu
bbl
li e-p
0.3 45 oint
line

0.2 60
Liquid
0.1
0 50
0 0.2 0.4 0.6 0.8 1.0 0 0.2 0.4 0.6 0.8 1.0
Mole fraction isopropyl ether in liquid phase, x1l Mole fraction isopropyl ether, x1j

Figure 4.1.3. Minimum-boiling azeotrope, isopropyl etherisopropyl alcohol system: (a) partial and total pressures at 70  C; (b) activity
coefficients; (c) vaporliquid equilibria; (d) phase diagram at 1 atm pressure. (After Hougen et al. (1959).)

briefly. Detailed calculation procedures are available in Both (c) and (d) are identified as dew-point calculations;
standard texts on chemical engineering thermodynamics. they involve calculating the composition of the first liquid
There are four types of VLE calculations for a two- drop in equilibrium with the given vapor composition on
phase n-component system: the dew-point line.
Consider the overall composition of a vaporliquid
(a) The values of T and x1l, x2l, . . .,xn1,l are known; the mixture (x1f, x2f, . . .,xn1,f) in equilibrium at a particular
values of P and x1v, x2v, . . .,xn1,v need to be deter- temperature T and pressure P. Sometimes one would like
mined. This is equivalent to calculating the compos- to know the actual liquid composition (x1l, x2l,. . ., xn1,l) as
ition of the first bubble in equilibrium with the well as the actual vapor composition (x1v, x2v, . . .,xn1,v)
specified liquid composition on the bubble-point line along with the relative molar amounts of the liquid
at the given temperature T. phase and the vapor phase under such conditions. The
(b) The values of P and x1l, x2l, . . .,xn1,l are known; we vaporliquid equilibrium calculation procedure adopted
need to determine T and x1v, x2v, . . .,xn1,v. for this fifth case is identified commonly as a flash
Both (a) and (b) are identified as bubble-point calculation. The flow system equivalent of this calculation
calculations. is touched upon in Chapter 6, Section 6.3.2.1, for a flash
separator.
(c) The values of P and x1v, x2v, . . .,xn1,v are known; we Each of the five above-mentioned VLE calculations for
have to calculate T and x1l, x2l, . . .,xn1,l. nonideal multicomponent systems require extensive
(d) The values of T and x1v, x2v, . . .,xn1,v are known; P and numerical calculations in an iterative fashion best carried
x1l, x2l, . . .,xn1,l are to be estimated. out in a computer. The basic governing equations are:
4.1 Two-phase equilibruim separation: closed vessel 213

(a)
1000

900

800

700 eP
Total pressur
Pressure (mmHg)

600 (b)

e
ton
ce
500 1.0

fa
p1v 0.9

eo

Activity coefficient, g1l


Pa

r
su
400 r t ia 0.8
es
l Ch
pr 0.7 lor
ofo

e
pr
300 rm

on
es
0.6
l
tia

g2l

et
su
r

g1l

Ac
Pa

re
of
200 p2v
ch
lo
0.5
ro
fo
rm
100 0.4

0.3
0 0.2 0.4 0.6 0.8 1.0 0 0.2 0.4 0.6 0.8 1.0
Mole fraction acetone in liquid phase, x1l Mole fraction acetone in liquid phase, x1l
(c) (d)
1.0 100
0.9 e 90
lin Vapor
m
0.8 iu 80
Mole fraction acetone

r
lib
in vapor phase, x1r

Temperature (C )
ui

0.7 70 Dew-point line


Eq

Bubble-point line
0.6 60
0.5 50 Vapor + liquid
0.4 40
1l
=x
0.3 x 1r 30 Liquid
e;
lin 20
0.2 45
0.1 10
0 0
0 0.2 0.4 0.6 0.8 1.0 0 0.2 0.4 0.6 0.8 1.0
Mole fraction acetone in liquid phase, x1l Mole fraction acetone, x1l

Figure 4.1.4. Maximum-boiling azeotrope, acetonechloroform system: (a) partial and total pressures at 60  C; (b) activity coefficients;
(c) vaporliquid equilibria; (d) phase diagram at 1 atm pressure. (After Hougen et al. (1959).)

(1) ^ iv depends on the xiv terms, P and T;


f^iv f^il , i 1, 2, . . ., n:
(2) il depends on the xil terms, P and T;
Therefore (3) f 0il depends on P and T.

^ iv P x il il f 0il Further, the functional dependences are complex.


x iv i 1, 2, . . ., n: 4:1:25
For flash calculations, the total number of moles,
Pn
In addition to these equations, we also need i1 mif , in the system is known in addition to xif for all
i 1,. . .,n. If miv and mil represent the number of moles of
n
X n
X species i in the vapor phase and the liquid phase, respect-
x iv 1 and x il 1 4:1:26
i1 i1
ively, then the additional equations required to solve the
problem of determining the xiv and the relative amounts of
for all bubble-point and dew-point calculations. Equations the vapor and the liquid phases are:
(4.1.25) are to be solved simultaneously under the con- n n n
X X X
straints of equations (4.1.26). The complexity is most often mif miv mil ; 4:1:27
due to the fact that i1 i1 i1
214 Separation in a closed vessel

! ! !
n
X n
X n
X il P sat
i
mif x if miv x iv mil x il : 4:1:28 x iv x il : 4:1:31
P
i1 i1 i1
One can now follow steps (1)(6) iteratively, except it is
Simpler expressions for both equations are as follows: ^ iv does not have to be calculated.
much simpler since
mtf mtv mtl ; 4:1:29a Assume T; calculate the il and P sati and then the xiv. If
Pn
i1 x iv 6 1, assume another T and continue till
x if mtf x iv mtv x il mtl : 4:1:29b Pn
i1 x iv 1.
For higher pressures, the assumption that il and f 0il are
The bubble-point calculations, the dew-point calculations independent of pressure breaks down. A number of
and the flash calculations are not carried out in practice for approaches have been developed. One of the earliest was
any set of P, T, xiv and xil. Rather, a subset of conditions proposed by Chao and Seader (1961), wherein equations
which allow certain simplifications are selected and (4.1.25), written as
thermodynamic calculations are carried out. They are:
(a) low to moderate pressures; x iv il f 0il
Ki , 4:1:32a
(b) ideal gas for low to moderate pressures; x il ^ iv P
(c) high pressures: the ChaoSeader method.
were simplified to
For low to moderate pressures, il and f 0il are assumed to be
independent of pressure. Following the resulting simplifi- il 0il
Ki : 4:1:32b
cation (3.3.62), equations (4.1.25) are written as ^ iv

^ iv P x il il P sat
x iv sat
i i ; Thus, the fugacity coefficient 0il of the pure liquid i at
i.e. temperature T and pressure P is calculated and substituted
for ( f 0il =P). Such Ki factors are used frequently in the oil
sat
il P sat
i i industry; nomographs are available for low molecular
x iv x il : 4:1:30
^
iv P weight alkanes and alkenes as a function of temperature
and pressure (see Figures 4.1.5 and 4.1.6 (Dadyburjor
In an iterative computer-based numerical bubble-point
(1978), adapted from DePriester (1953))). Note that lower
calculation of T, x1v, x2v, . . ., xn1,v when P and x1l, x2l, . . .,
molecular weight hydrocarbon species are likely to have
xn1,l are known, the following approach is often adopted.
Ki greater than 1, whereas higher molecular weight
(1) Assume T and ^ iv 1. hydrocarbon species are likely to have Ki less than 1.
(2) Calculate the xiv from equations (4.1.30) by calculating Example 4.1.1 illustrates how Ki values from such plots
the il, Pisat and isat using appropriate thermodynamic may be used to carry out dew-point and bubble-point
formulas for each i. calculations for multicomponent systems at higher pres-
Pn
(3) Calculate i1 x iv . For the first iteration, normalize sures. Prausnitz and Chueh (1968) have developed a
Pn
the assumed xiv by dividing each one by i1 x iv . more complex and exact method of high-pressure VLE
Using these new xiv, calculate new estimates of ^ iv calculations.
(different from 1) using appropriate thermodynamic
formulas. Example 4.1.1
(4) Estimate new values of xiv from (4.1.30) using the new (a) A vapor mixture of 12% ethane, 32% propane and 56%
estimates of ^ iv . n-butane is present at 32  C in a closed vessel. Calculate
P
(5) Sum all xiv to obtain ni1 x iv . If it is different from that the dew-point liquid composition and pressure using the
obtained earlier, continue the iterative calculation of charts provided in Figure 4.1.6.
P (b) A liquid mixture of 12% ethane, 32% propane and 56%
the xiv using steps (3), (4) and (5). If ni1 x iv changes
n-butane is present at 32  C in a closed vessel. Calculate
by less than a small predetermined amount and it is
the bubble-point vapor composition and pressure using
still different from 1, then assume a different T and the charts provided in Figure 4.1.6.
recalculate the il, Pisat, isat; repeat steps (2), (3), etc.,
P
till ni1 x iv 1. Solution (a) This involves trial-and-error calculations. One
P
(6) For the P and T, when ni1 x iv 1, the results, T and has to guess the value of the pressure, draw a line between
xiv are correct for the problem. 32  C and that pressure and find the values of Ki for different
species from the chart. Using these Ki values and the given
For low to moderate pressures, if the gas phase behaves xiv, one can calculate the xil. If the xil are correct, then
Pn
ideally, it is generally observed that sat ^
i iv and/or both i1 x il 1. If not, a different pressure has to be guessed.
P
are close to unity so that relation (4.1.30) becomes the If the first guess leads to ni1 x il > 1, then a lower pressure is
P
simpler to be employed as the second guess. If ni1 x il < 1, then
4.1 Two-phase equilibruim separation: closed vessel 215

101.3
20
110

150 10

200 0

250
300 10

n-Nonane
400
20
500

n-Octane
Pressure (kPa)

600

n-Heptane

Temperature (C)
700
800 30
900

n-Hexane
1000

40
1500

n-Pentane
Isopentane
2000

50
3000
Ethane

Propane
Propylene
Ethylene

n-Butane

4000
Isobutane

5000
60
Methane

6000

Figure 4.1.5. Low-temperature nomograph for Ki factors in light hydrocarbon systems. Reprinted, with permission, from D. Dadyburjor,
Chem. Eng. Prog., 74(4), 85 (1978). Copyright [1978] American Institute of Chemical Engineers (AIChE).

a higher pressure should be used as the second guess. Iden- known molecular weight density function of the liquid
tify the species ethane, propane and n-butane as i 1,2 and phase. If now the temperature is specified, one would like
3. Assume the pressure to be 550 kPa at 32  C. Obtain the Ki to know the molecular weight density function fv(M) of the
values as shown in Table 4.1.2. first vapor bubble formed and the total pressure P of the
For the next guess, take the pressure to be 500 kPa (see
system. It is known for discontinuous mixtures that the first
Table 4.1.3). The dew-point liquid composition is 1.85%
vapor bubble formed has a composition xiv, which is
ethane, 14.5% propane and 84.8% n-butane.
(b) At the bubble-point temperature of 32  C, assume 690 related to xil by (relation (3.3.61))
P
kPa pressure (see Table 4.1.4). The value of xiv is too high.
^ iv P x il il f 0il :
x iv
Increase the pressure to, say, 1050 kPa (Table 4.1.5). This
guess of 1050 kPa is just about right.
At low to moderate pressures, one can easily assume that
The procedure for calculating vaporliquid equilib-
rium for continuous chemical mixtures will be briefly illus- f 0il P sat sat
i i :
trated here for a bubble-point calculation. Let fl(M) be the
216 Separation in a closed vessel

Table 4.1.2. Table 4.1.3.

550 kPa 500 kPa

Species xiv Ki x il x iv =K i Species xiv Ki x il x iv =K i

i1 0.12 6 0.02 i1 0.12 6.5 0.0185


i2 0.32 2 0.16 i2 0.32 2.2 0.145
i3 0.56 0.6 0.93 i3 0.56 0.66 0.848
P P
x il 1:11 x il 1:011 1

Propylene

n-Decane
Methane

Isopentane

n-Nonane
Ethylene

Propane

n-Heptane
n-Pentane

n-Octane
Isobutane

n-Hexane
Ethane

n-Butane
101.3
110
200
190
150 180
170
160
200 150
140
250
130
300 120
110
400
100
500 90
600
Pressure (kPa)

80
700
70
800

Temperature (C)
900
1000 60

50

1500
40
2000
30
n-Decane

2500
3000
3500 20
4000
Propylene
Ethylene

Propane
Ethane

10
5000
Methane

n-Butane
Isobutane

n-Heptane

n-Nonane
Isopentane
n-Pentane

n-Octane

0
n-Hexane

Figure 4.1.6. High-temperature nomograph for Ki factors in light hydrocarbon systems. Reprinted, with permission, from D. Dadyburjor,
Chem. Eng. Prog., 74(4), 85 (1978). Copyright [1978] American Institute of Chemical Engineers (AIChE).
4.1 Two-phase equilibruim separation: closed vessel 217

Table 4.1.4. M
  
Psat sat
M P M, T P exp A 1 , 4:1:33e
T
690 kPa
where P and A are constants. If fl(M) is known, e.g. it is a
Species xil Ki x iv x il K i Gaussian or distribution, then P can be determined from
(4.1.33d) by integration over the molecular weight range of
i1 0.12 5 0.6
the continuous mixture. From (4.1.33c), the molecular weight
i2 0.32 1.6 0.512
i3 0.56 0.5 0.28 density function of the first vapor bubble can now be deter-
mined; it is clear from this Raoults law relation that if fl(M) is,
x iv 1:392 (too high)
P
say, a distribution, then fv(M) will also be a distribution,
with different values of the mean and the standard deviation.
The two molecular weight density functions frequently
Table 4.1.5.
used in such types of calculations are:
1050 kPa distribution function
Species xil Ki x iv x il K i M1 M
  
f M exp : 4:1:33f
G
i1 0.12 3.4 0.408
i2 0.32 1.3 0.416
For a value of M , f(M) is zero. The mean of this
i3 0.56 0.34 0.19
distribution function is and the standard deviation
x iv 1:014 is 2.
P

Gaussian distribution function


Therefore, 1
 
MMmean

f M p exp : 4:1:33g
^ iv P x il il P sat
x iv sat 2 22
i i :

The equivalent relation for a continuous chemical The mean is Mmean and the standard deviation is .
mixture is
    4.1.3 Liquidliquid systems
^ Mv f l MdM Ml P sat
f v MdM P sat
M M , 4:1:33a
If a species i is distributed between two immiscible liquid
where P sat
M is the vapor pressure of the species of molecular phases j 1,2, the separation achieved between the two
weight M at temperature T; other quantities, namely ^ Mv , liquid phases may be determined by the ratio (ai1/ai2)
sat
M and Ml correspond to the similar quantities for the defined by equation (4.1.1). If separation between two
species of molecular weight M for the given phase at the species i 1,2 distributed between two phases j 1,2 is
system condition. desired, the separation factor is given by equation (4.1.4).
For simplicity, assume an ideal gas phase and an ideal The standard state of each solute species i 1,2 has to be
liquid solution: ^ Mv ~ 1, Ml ~ 1, sat
M ~ 1. We obtain, from specified before these equations can be utilized. When
(3.3.66), pure solute at the same temperature and pressure of the
P f v MdM P sat system under consideration is the standard state, then
M f l MdM, 4:1:33b

which can be written as 0i1 0i2 : 4:1:34a

P sat
M f l M
At equilibrium, the distribution of solute i from relation
f v M , 4:1:33c (4.1.1) is given by
P
the expression which represents Raoults law for continu- ai1 x i1
1 i1 ; 4:1:34b
ous chemical mixtures. Here P is unknown and can be ai2 i2 x i2
determined from x i1
K i i2 : 4:1:34c
x i2 i1
P sat
M f l MdM

P P sat
M f l MdM 4:1:33d Nonideal behavior is therefore a prerequisite to separation
f v MdM of solute i between bulk phases 1 and 2.
In the process of solvent extraction of solute i from
over the whole range of molecular weights since fl(M) is phase 2 (feed solution) into the solvent, the extract (phase 1),
known and P satM should be available. For example, it is important that Ki > 1. The activity coefficients i1 and i2
Troutons rule may be used to describe P sat
M for a pure are to be estimated from commonly used standard equa-
species of molecular weight M as follows: tions (Treybal, 1963, pp. 7071) e.g. the two-constant based
218 Separation in a closed vessel

van Laar equation or Margules equation, the three-con- solvent over another solvent. There are many other con-
stant based RedlichKister equation. Although the ij in siderations in the selection of a solvent for extracting a
these equations are complex functions of concentrations solute from a feed solution. These include: insolubility of
of species i, as well as the concentrations of the two liquids the solvent in the feed liquid and vice versa; the solvent
constituting the bulk of the two immiscible liquid phases should be recoverable easily for reuse; the densities of the
j 1,2, simple results are obtained for the ij under certain solvent and the feed solution when in equilibrium with
limiting conditions (Treybal, 1963, pp. 104105). The each other must be different; the interfacial tension of the
limiting conditions are low concentrations of solute i, the liquidliquid system should not be too small to facilitate
two liquids constituting bulk phases 1 and 2 are very insol- coalescence of the dispersed phase, since most commer-
uble in each other so that x32 and x41 (here i 3,4 refer to cial solvent extraction devices disperse the feed or the
the species making up the two bulk phases j 1,2, respect- solvent phase in the other phase as drops to achieve
ively) are essentially zero; the corresponding result is extractive transfer of solute (membrane solvent extractors
log10 K i Ai2 Ai1 , 4:1:34d are an exception).
When extractive separation between two solutes i 1,2
where the Aij are appropriate constants in the two-constant present in a feed solution (phase j 2) by an extraction
or three-constant equations for activity coefficients men- solvent (phase j 1) is desired, the process is called
tioned earlier (Treybal, 1963, pp. 104105). fractional extraction. The system now becomes quaternary;
Application of regular solution theory (Prausnitz,1969) however, the separation between solute species i 1,2 can
leads to the following estimate of ij for a solution of i in be determined using equation (4.1.4) as if it is a binary
phase j containing primarily liquid j: system of species 1 and 2. For the standard state condition
V i i j 2 (4.1.34a), the separation factor 12 is given by
ln ij , 4:1:34e
RT x 11 x 22 21 12 K 1
12 : 4:1:34g
where the i and j are the solubility parameters of liquids i x 12 x 21 11 22 K 2
and j, respectively. Therefore, from (4.1.34c),
Treybal (1963, p. 108) has illustrated ways of estimating
V i i 4 2 i 3 2  12 by estimating the different activity coefficients 21,
ln K i : 4:1:34f
RT 12, 11 and 22: the system consists of alkyl benzene
Estimates of solubility parameters for a variety of liquids (i 1), paraffin (i 2), ethylene glycol or furfural (solvent)
are available in the literature. Table 4.1.6 illustrates the and n-heptane (feed phase). The solubility-parameter-
values of solubility parameters for some common liquids. based approach using relation (4.1.34f) may also be
If the solute prefers the solvent (phase 1, species 3), then i employed.
is closer to 3 than 4, and therefore, Ki > 1. (Note: Gener- The estimation of Ki or 12 for cases where an infinite
ally, the more polar the liquid, the larger the value of the dilution standard state for the solute(s) is employed, can be
solubility parameter; water, which is highly polar, has a i made from equation (4.1.1) as follows:
of 21, whereas n-pentane, which is extremely nonpolar, ai1 i1 x i1 0 0
 
has a i of only 7.1. Further, the higher the required energy exp i1 i2 , 4:1:34h
ai2 i2 x i2 RT
of vaporization, the higher the solubility parameter.)
Result (4.1.34d) allows rapid estimation of whether Ki where 0i1 6 0i2 . It follows therefore that
is much larger than 1 for solute i being extracted into phase x i1 0
 

1 from phase 2. This may be used to select a particular K i i2 exp i : 4:1:34i
x i2 i1 RT

Table 4.1.6. Solubility parameters for some common liquids at For two solutes i 1,2,
25  C
x 11 x 22 01 02
 

12 12 21 exp : 4:1:34j
i i x 12 x 21 11 22 RT
Liquid (cal/cm3)1/2 Liquid (cal/cm3)1/2
If the solutions behave ideally, both i1 and i2 tend to 1,
Perfluoroalkanes 6.0 Acetonitrile 11.8 resulting in
n-Pentane 7.1 Acetic acid 12.4
0
 
n-Hexane 7.3 Dimethylsulfoxide 12.8 K i exp i ; 4:1:34k
n-Octane 7.5 Methanol 12.9 RT
Carbon 8.6 Propylene 13.3
01 02
 
tetrachloride carbonate 12 exp : 4:1:34l
Toluene 8.9 Ethylene glycol 14.7 RT
Benzene 9.2 Formamide 17.9
If the value of the Nernst distribution coefficient N
i1 is
Ethanol 11.2 Water 21.0
known (see (3.3.78)), then, for dilute solutions,
4.1 Two-phase equilibruim separation: closed vessel 219

1
1 (Ethanol) Water as solvent
0.9

Weight fraction ethanol in extracting


Fe 0.8

solvent layer, solvent-free basis


Benzene as solvent
0.7

0.6

0.5

0.4
P
0.3
F
0.2
M
0.1
Fr R E
0
A 0 0.1 0.2 0.3 0.4 0.5 0.6
3 4 (Water)
(Benzene) B Weight fraction ethanol in nonsolvent
layer, solvent-free basis
(a) (b)

Figure 4.1.7. (a) Selective extraction of ethanol (1) from phase 3 (benzene) by means of a solvent phase 4 (water); (b) selectivity plot on
a solvent-free basis for a Type 1 system at 25  C: ethanol (1)benzene (3)water (4); water is the solvent. (After Treybal, 1963, p.124.)

N
11
However, n-heptane and methylcyclohexane are com-
12 : 4:1:34m pletely miscible with each other over the whole compos-
N
21
ition range. Such systems having two pairs of partially
When two solutes 1 and 2 are distributed between two miscible liquids are called Type 2 systems (Treybal, 1963,
immiscible phases j 1,2, we have four species; for the p. 15). Obviously, if there is another solute (i 2) beside
partitioning of one solute between immiscible phases methylcyclohexane (i 1) in a significant amount, the
j 1,2, there are three species. To make mass balance system will become complex if it is partially miscible with
and separation calculations, it is useful to recognize that some of the other species in the system.
often the two so-called immiscible phases may have par- To understand the separation in solvent extraction in
tial miscibility. For example, consider the ternary system either system (Type 1 or Type 2), focus on a solution of
benzene (i 3), water (i 4) and ethanol (i 1, the solute) composition F in Figure 4.1.7(a) for a Type 1 system. If the
(see Figure 1.2.2 and replace picric acid by ethanol) whose solvent is water (i 4) and it is added to the system, the
liquidliquid equilibrium type is shown in the triangular overall two-phase mixture will be located at M (the object
diagram (see Figure 1.6.1) of Figure 4.1.7(a). As shown, at here is to extract ethanol (i 1) from benzene (i 3) into
the given temperature, ethanol and benzene are completely water (i 4)) somewhere along the line joining F and 4;
miscible; so are ethanol and water over the whole compos- M can be located using the lever rule (see equation (1.6.2)).
ition range. However, benzene and water have clearly only After sufficient time has been allowed for the two phases
limited solubility in each other. So, of the three possible (water phase and the benzene phase) to come to equilib-
binary pairs, only one pair (benzenewater) has partial rium, the two phases will be located at, say, E and R along a
miscibility (Type 1 system) (Treybal, 1963). straight line which is called the tie line.
On the other hand, consider a ternary mixture of The locus of these points E and R for various two-
n-heptane (i 3), aniline (i 4) and methylcyclohexane phase mixtures for different feed compositions, ARPEB,
(i 1) (the solute) shown schematically in Figure 4.1.8(a). provides an envelope in the triangular diagram: the area
As long as the temperature is less than 59.6  C, the two inside the envelope represents the two-phase region (the
bulk liquids n-heptane and aniline will form two immis- water phase and the benzene phase), while the area out-
cible layers with partial solubility (see Example 1.5.2). But side represents a single-phase region. Points A and
aniline and methylcyclohexane also form two immiscible B represent, respectively, a benzene phase saturated with
layers having partial solubility of each in the other phase. water and a water phase saturated with benzene without
220 Separation in a closed vessel

1 (Methylcyclohexane) 1

0.9

0.8

Weight fraction 1 in aniline-rich


Fe

phase, aniline free-basis


0.7

0.6

0.5
F
0.4
Fr 0.3
R 0.2
M
E 0.1

E 0
0 0.2 0.4 0.6 0.8 1
Weight fraction 1 in n-heptane rich
3 (n-heptane) 4 (Aniline) phase, aniline-free basis

(a) (b)

Figure 4.1.8. (a) Selective extraction in Type 2 systems; (b) selectivity plot on a solvent-free basis for a Type 2 system at 25  C:
methylcyclohexane (1)n-heptane (3)aniline (4); aniline is the solvent. (After Treybal, 1963, p.125.)

any ethanol. The envelope ARPEB, called the solubility or which is mostly benzene-rich, and the BEP branch, which
binodal curve, represents compositions of individual solu- is mostly water-rich. However, as the curves tend toward P,
tions which are in equilibrium with another immiscible the concentration of the solute (ethanol) increases and the
solution on the same curve. immiscibility of the two phases disappears at point P, the
Focus now on the mixture M which splits into two Plait point. It is not necessarily the point in ARPEB which
immiscible phases R (raffinate) and E (extract) (Figure has the highest amount of ethanol.
4.1.7(a)); the compositions of R and E are the equilibrium The quantitative behavior of the distribution of ethanol
compositions of the two immiscible layers. The water phase (i 1) between the two phases (i 3, benzene; i 4,
representing point E has primarily water, a considerable water) can be obtained from the two branches of the
amount of ethanol and a little bit of benzene. The benzene binodal curve. A more useful approach would be to plot
phase representing point R has primarily benzene, a small the ethanol concentration at point Fe against ethanol con-
amount of ethanol and a very small amount of water. In centration at point Fr, i.e. on a solvent-free basis after
solvent extraction processes, after the extraction is carried removing water, as if the original feed mixture of
out, the solvent is removed from the extract and the raffinate benzeneethanol has been split into two fractions (not
so that the solvent may be reused. If the solvent (here, phases, since the two fractions are miscible). Figure 4.1.7
water) is removed from the extract (point E), then, by the (b) illustrates the weight fraction of ethanol in the water
lever rule, the composition of the water-free organic phase layer on a solvent-free basis vs. the weight fraction of
represented by Fe is obtained by extending line 4E to inter- ethanol in the benzene layer on a solvent-free basis; here
sect with the line 13. Correspondingly, the water-free raffin- water is the solvent. This figure also has a plot for the same
ate composition is obtained by extending line 4R to intersect system, with benzene being considered as a solvent to
with the line 13 at Fr. The net result of the solvent extraction separate ethanol from water.
of the benzeneethanol feed mixture F with water, and the For a Type 2 system, as shown in Figure 4.1.8(a), the
subsequent removal of water from the extract phase and the area in the triangle in between the two curves represents
raffinate phase, are two fractions having composition Fe the two-phase region. Any feed mixture F of n-heptane and
(ethanol-rich) and Fr (benzene-rich). methylcyclohexane when mixed with the solvent aniline
Two immiscible phases having different concentra- will produce a mixture of overall composition M. If we
tions of the solute to be extracted is a prerequisite to the now follow the notation of Figure 4.1.7(a) and the corres-
application of solvent extraction. In Figure 4.1.7(a), the ponding steps, we will end up with two fractions, Fe and Fr,
binodal curve ARPEB has two branches: the ARP branch, of n-heptane and methylcyclohexane, free of the solvent.
4.1 Two-phase equilibruim separation: closed vessel 221

A plot of the methylcyclohexane weight fraction in the (a)


aniline phase vs. that in the n-heptane phase (based on 100
Thiodipropionitrile Oxidipropionitrile
an aniline-free basis) illustrates the separation achieved 80 Ethylene glycol
Sulfolane
(Figure 4.1.8(b)).
60 Ethylene carbonate
The partitioning of a solute i between two immiscible
Diethylene glycol Nitromethane
liquid phases j 1 and 2 and the selectivity between two 40 Dimethyl sulfoxide

Selectivity
solutes in the two-phase system have so far been con- Triethylene glycol
Methylformamide
sidered primarily in the context of liquid extraction/solvent
Propylene
extraction in large-scale operations. Many such aspects 20
Tetraethylene glycol
Carbonate
Ethylene diamine

and some other considerations are also important for the Methylcarbamate Furfural
basic equilibria in liquidliquid chromatography (LLC) (see
Diethylene glycol methylether
Section 7.1.5). In LLC, a mobile liquid phase ( j 2) flows 10
Triethylene tetramine
over a porous, finely divided solid phase whose surface has 8
Propylene glycol

been coated with a liquid, the stationary phase ( j 1). Any n-Methylpyrrolidone
6
solute introduced via the mobile phase is partitioned
between the two liquid phases. For cases where 0i1 0i2 , 4
the regular solution-theory based expression (4.1.34f ) for Capacity
Dipropylene glycol
3
Ki may be written as 0.06 0.08 0.1 0.2 0.4 0.6 0.8 1.0 2.0

V i i 2 2 i 1 2  Figure 4.1.9A. (a). Capacity and selectivity of polar organic solv-


ln K i : 4:1:34n
RT ents for aromatics (Lo et al., 1983). Reprinted, with permission,
To achieve a high value of Ki between the two phases, i from T.C. Lo, M.H.I. Baird and C. Hanson, Handbook of Solvent
should tend to 1, i.e. the solute i should be similar to the Extraction, Figure 2, p. 525, Wiley-Interscience, 1983. Copyright
stationary phase j 1. Further, K11 should be different 1983, John Wiley & Sons.
from K21 for any chromatographic separation between
two solutes 1 and 2. Note that the two liquid phases must top ( j 1); the dextran-rich layer is at the bottom ( j 2),
also be immiscible, for which, generally, 1 2  4 (Kar- with a clear boundary in between. Proteins, cells, etc. are
ger et al., 1973). There are many combinations of highly partitioned between the two layers. The partition coeffi-
polar organic liquids and water where 1 2  4 does cient, p1 , decreases from about 2 to 0.2 as the molecular
not guarantee phase immiscibility. However, the criterion weight increases (Figure 4.1.9B).
of liquid immiscibility is important for solvent selection. The partition coefficient is strongly influenced by the salt
In liquidliquid extraction, generally one of the two (e.g. KCl, K3PO4, K2SO4, etc.) usually present in such
immiscible liquid phases is aqueous and the other systems. The presence of the salt creates an electrostatic
phase is organic, which is nonpolar or mildly polar. There potential gradient across the interface between the two-
are many examples where both immiscible phases are phase system, which leads to preferential partitioning of the
primarily organic: aromatic species such as toluene and protein into one of the phases, depending on the sign and
benzene are extracted from an essentially nonpolar hydro- magnitude of the protein surface charge. Haynes et al. (1991)
carbon feedstock by an immiscible highly polar organic have related the interfacial electrostatic potential difference
solvent. Large-scale applications utilize highly polar between the two aqueous phases j w1 and j w2, namely
organic solvents, e.g. ethylene glycol, dimethylsulfoxide, w2 w1 ,to the salt distribution coefficient m salt in

n-methylpyrrolidone, etc., with or without a small amount terms of the salt molar concentrations for a 1:1 electrolyte:
of water (Lo et al., 1983). Figure 4.1.9A illustrates the RT RT

ms, w1

behavior of many such solvents in terms of their selectivity w2 w1 lnm
salt ln , 4:1:34o
F F ms, w2
for aromatics with respect to nonaromatics vs. their cap-
acity for the aromatics. The solvent power or capacity is where ms,j is the salt molality in phase j (mol/kg). For low
directly proportional to the equilibrium ratio Ki1 with concentrations of electrolytes and proteins, the protein
the extracting solvent being phase 1. An ideal solvent has partition coefficient, p1 , with lighter phase 1 at the top,
a high selectivity as well as high capacity. has been related to that in the absence of any , 0p1 , by a
Extraction of solute species from one liquid to model based on virial expansion (King et al., 1988):
another immiscible liquid is also carried out when both lnC p1 =C p2 ln p1 ln0p1 Z p F =RT
phases are primarily aqueous. Large-scale purification B1p C 12 C 11 B2p C 22 C 21 Z p F =RT:
processes for proteins employ aqueous two-phase 4:1:34p
systems (Albertsson, 1986) containing two water-soluble
but incompatible polymers in water, e.g. polyethylene Here B1p and B2p are the second virial coefficients for
glycol (PEG) and dextran. The PEG-rich layer is at the interaction between polymers 1 and 2, respectively, Cij is
222 Separation in a closed vessel

(b)
3.0
Liquid solution of i
2.0 and j

Ovalbumin

Bovine serum albumin

b-Galactosidase (from E.coli K12 and WL)


a -Amylase

Transferrin (human)
(bacterial)
1.0
Pure solid j Pure solid i

Temperature
Insulin
RNase
Lysozyme (hen and turkey)
Papain
Trypsin
a -Chymotrypsin

and liquid and liquid


TE
kp1 E

Immiscible mixture of pure solid i and


pure solid j

0
xj = 1 xi xi = 1
0.1
Figure 4.1.10. Temperature vs. composition diagram for two
species i and j in solidliquid systems where the two solid phases
0.05 are immiscible.
1 2 3 4 5 6 7 8 50
4
Protein molecular weight (x 10 Da)

Figure 4.1.9B. Protein molecular weight vs. protein partition coef-


x il is f 0is
ficient in a PEG 6000-dextran 500 system with pH at pI for all :
x is il f 0il
proteins (Sasakawa and Walter, 1972). Reprinted, with permis-
sion, from Biochemistry, 11(15), 2760 (1972), Figure 2. Copyright Therefore, for two species i and j,
(1972) American Chemical Society.
0
x il x js is jl f 0is f jl
the molar concentration of polymer i in region j and Zp is ij : 4:1:35
x is x jl js il f 0js f 0il
the algebraic charge number (valence) of the protein.
Haynes et al. (1993) have developed a model to predict For ideal solutions in both phases, il 1 jl and
p1 for a protein in biphasic aqueous systems containing is 1 js , leading to
strong electrolytes amongst other things. 0
f 0is f jl
ij : 4:1:36
f 0js f 0il
4.1.4 Liquidsolid systems

In Section 3.3.7.5, the equilibrium partitioning of a species


At temperature T and pressure P for the system in equilib-
between a liquid phase and a solid phase was briefly
rium, for any species i,
considered for three types of liquidsolid equilibria. The
separation between two species i and j for such liquid f 0is T, P f 0 T, P f 0is T mi , P f 0il T mi , P
0 is   : 4:1:37
solid two-phase systems is briefly considered here. There f il T, P f is T mi , P f 0il T mi , P f 0il T, P
0

are systems where three phases can be present; for


example, two immiscible solid phases and a saturated Here Tmi is the melting point of species i. For a pure
solution, as in the case of solid salt, ice and a saturated species i at T mi , f 0is T mi , P f 0il T mi , P. Exact expres-
salt solution. Figure 4.1.10 shows a temperature vs. com- sions have been developed for the product of the two
position phase diagram where solid phase 1 coexists with remaining ratios. An expression practically useful and
solid phase 2 and a saturated liquid solution at the eutectic based on particular approximations (Smith et al., 2001,
point E. Below the eutectic temperature TE, immiscible pp. 526531) is
pure solid phase 1 and 2 are present together. For these
f 0is T, P H i T T mi
 

and more complex systems, the reader should refer to exp : 4:1:38
f 0il T, P RT mi T
appropriate texts (Darken and Gurry, 1953; DeHoff, 1993).
Separation between species i and j in simpler two-phase Therefore
systems described in Figures 3.3.6A, where the solid phase 8  9
is a homogeneous solution, will be determined now. <H i H j  TT 1 =
mi

For a solid solution in equilibrium with a liquid, rela- ij exp   : 4:1:39


T
T mj 1
: ;
tion (3.3.93) for the equilibrium ratio of species i is
4.1 Two-phase equilibruim separation: closed vessel 223

In the case of leaching of a solid mixture, using expres- 4.1.5 Interfacial adsorption systems
sion (3.3.96), one can develop an appropriate expression
Following Section 3.3.7.4, the separation factor in
for the separation factor, ij, for species i and j being
interfacial adsorption where both bulk phases are fluids is
leached out.
considered first. For airwater systems containing two
Of the three categories of liquidsolid equilibria con-
surface active solutes, i 1, 2 (both being nonelectrolytes),
sidered in Section 3.3.7.5 and briefly considered above, a
the separation factor 12 has been defined as
solid solution in equilibrium with a molten mixture has
special importance in the purification of semiconductor E1 =C 11
12 , 4:1:41
materials such as silicon. Here, the bulk solid phase, as E2 =C 21
well as the molten mixture, consists essentially of silicon;
the concentrations of impurities are at a very low level. where j 1 corresponds to the bulk phase of water. For a
Therefore the area of focus is very close to either end of the nonelectrolytic surface active solute i, whose presence
type of phase diagram in Figures 3.3.6A for a given reduces the interfacial tension linearly in the manner of
impurity-silicon system. Usually the solution being relation (3.3.105), it is known that
extremely dilute, the distribution coefficient is for the
Ei bi
impurity i between the solid phase and the melt (see 1 , 4:1:42a
C i1 RT
Example 1.4.3),
where bi1 comes from the linear relation between the
C is
is , 4:1:40 interfacial tension 12 and the surfactant concentration Ci1
C il
below the critical micelle concentration (CMC):
is a constant. Illustrative values of the distribution coeffi-
cients for a variety of impurities are provided in Table 4.1.7. 12 a1 bi1 C i1 : 4:1:42b
It appears that, except for oxygen, all impurities will parti-
Therefore,
tion more into the molten phase. This is a basis for purifi-
cation of silicon into silicon single crystals/wafers (see 12 b11 =b21 : 4:1:43
Chapter 6 for the zone refining/melting process). Thus an
impure silicon material may have a molten section and a This result indicates that in a closed vessel containing two
frozen section; this will allow redistribution of impurities bulk phases, namely air and water, the selectivity between
between the newly frozen solid phase and the remaining two nonionic surfactants distributed between the bulk
melt in the manner of (4.1.40); the newly frozen solid liquid and the gasliquid interfacial phase will be deter-
phase will be purer. mined by the ratios of the slopes of the decrease of 12 with
This phenomenon will also allow the creation of a solid the concentration of each individual surfactant. Although
phase having a certain desired level of impurity or dopant this ratio can be easily different from 1, it may not be very
once a certain amount of impurity level is introduced first large. When, however, the surfactants are ionic, significant
into the melt. Doped gallium arsenide is an example of selectivity can be achieved. The expressions for this type of
such a material in the semiconductor industry. Table 4.1.7 system are studied in Section 5.2.5.
illustrates some of the dopants and their partition coeffi- Amongst interfacial adsorption systems, where one of
cients between the bulk crystal and the melt. the bulk phases is a solid, gassolid systems are important
for gas mixture separation. Of the three general approaches
to predicting the separation of a gas mixture in gassolid
adsorption, namely the solution thermodynamic Gibbs
Table 4.1.7. Distribution coefficients for a number of impurities in approach, potential-theory methods and the Langmuir-
silicon and dopants in gallium arsenide type equation, the first approach will be briefly presented
now. Myers and Prausnitz (1965) considered the mixed
Silicona Gallium arsenideb
adsorbate as a solution in equilibrium with the gas mixture
Impurity is Dopant is and applied solution thermodynamics with the spreading
pressure and the surface area of the sorbent A (more
Al 0.002 S 0.30.5 practically, S, the surface area per unit adsorbent mass)
As 0.3 Se 0.10.3
replacing, respectively, the pressure P and volume V for
B 0.8 Te 0.059
C 0.07 Be 3 the adsorbate. The following three equations, derived by
Cu 4  104 Mg 0.1 them, allow the prediction of adsorption for a binary
O 1.25 Ge 0.01 mixture:
P 0.35 C 0.20.8
Sb 0.023 Cr 5.7  104 Px ig P 0i i x i
a b
From Pearce (1983); from Milnes (1973). (equation (3.3.111a), repeated here for convenience);
224 Separation in a closed vessel

1 2
X x i RT X2 
ln i
 For 12 > 1, namely the adsorbate phase is enriched in
X2 p x i ; 4:1:44 species 1 compared to species 2, we have P 02 > P 01 .
q i1 qi
S i1 x i
i1 i Obviously,
!
2
P P 01 x 1 P 02 x 2 : 4:1:51
X
qi qi x i : 4:1:45
i1
These results follow directly from ideal adsorbed solution
p
Here, qi is the number of moles of i per unit mass of theory (Myers and Prausnitz, 1965), whose governing equa-
adsorbent when adsorbed from a pure gas i at the same tions are (4.1.46) to (4.1.48). The key quantities are P 01
surface pressure and temperature T as the mixture. and P 02 . One needs pure component adsorption data for
The determination of the activity coefficients i for the each species to determine P 01 and P 02 and to calculate
adsorbed phase requires accurate experimental data at the separation factor 12 for a binary system. The proced-
constant temperature and pressure for the entire range ure suggested for determining the compositions of the gas
of gas-phase compositions (Myers and Prausnitz, 1965). phase (xig) and the adsorbate phase (xi) at any given total
However, if the adsorbed solution phase is considered pressure P is as follows.
ideal, i.e. i 1, then p
(1) Plot graphically qi =P 0i against P 0i at any given T.
Px ig P 0i x i ; 4:1:46 Calculate the area for different integration limits
of P 0i (equation (4.1.49)). This will yield a plot of
2
1 X x i S =RT against pressure P, correspondingly P 0i
X2 p; 4:1:47 p
q i1 qi for each i. If qi can be described analytically as
i1 i
! a function of P 0i , the same integration can also be
2
X carried out analytically to develop the same plot. See
qi qi x i : 4:1:48 Figure 4.1.11.
i1
(2) To determine relevant quantities for the mixture, select
Note that, from the definition of P 0i (Section 3.3.7.6), a total pressure P and a value of for the mixture. In
P01 p P02 p
Figure 4.1.11, draw a line parallel to the abscissa for the
S q1 q2 corresponding and a vertical line at the selected P.
dP dP, 4:1:49
RT 0 P 0 P The point of intersection of these two lines, M, defines
where each integral is to be used for pure gas adsorption the mixture location at a certain total pressure P, mix-
data. For a binary gas mixture, this is the equivalent of a ture spreading pressure and temperature T. Corres-
Raoults law type of situation in vaporliquid equilibria pondingly, the value of P 0i is obtained from the
(see (3.3.64) and (4.1.21b)). The separation factor 12 for abscissa of the intersection of the line parallel to P-axis
species 1 and 2 between the adsorbed phase () and the (for the corresponding ) and the pure component
gas phase (g) is equilibrium curve. The mixture compositions are
obtained via the lever rule (equation (1.6.2)) using
x 1 x 2g P 02 equations (4.1.50) and (4.1.51). For example, equation
12 : 4:1:50
x 1g x 2 P 01 (4.1.51) may be written as

F Pure Component
(1)

B M
A Pure
C Component (2)

pS
RT E

0 P10 P P20

Figure 4.1.11. Graphical calculation of adsorption equilibrium of a gas mixture from pure component spreading pressure information.
(After Myers and Prausnitz (1965).)
4.1 Two-phase equilibruim separation: closed vessel 225

Px 1 Px 2 P 01 x 1 P 02 x 2 , Table 4.1.8.

which may be rearranged to O2 at 0  C CO at 0  C

P P 01 x 1 P 02 Px 2 , Pressure Volume adsorbed, Pressure Volume adsorbed,


(mm Hg) observed (cm3) (mm Hg) observed (cm3)
i.e.
83.0 3.32 95.5 7.09
x 1 P 02 P length of line CM 142.4 5.57 127.4 9.48
:
x 2 P P 01 length of line MB 224.3 8.73 199.7 14.37
329.6 12.68 272.4 19.41
Further 405.1 15.48 367.4 25.54
544.1 20.42 463.7 31.70
length of line CM length of line MB
x 1 and x 2 , 602.5 22.48 549.7 37.01
length of line CB length of line CB 667.5 24.86 647.9 42.65
4:1:52a 760.0 28.03 760.0 48.89

since x1 x2 1 from (4.1.48). Correspondingly, COO2 Mixtures at 0  C

P 02 P P P 01 Volume of oxygen Volume of carbon


x 1 and x 2 :
P 2 P 01
0
P 2 P 01
0 adsorbed (cm3) monoxide adsorbed (cm3)
4:1:52b pO2 pCO Isotherm Obs. Isotherm Obs.

To determine x1g and x2g, we employ (4.1.46) or (4.1.50) 230.2 529.8 8.98 8.31 35.90 35.02
along with the above expressions for x1 and x2 in Figure 391.1 368.9 15.05 14.1 25.82 24.69
4.1.11. From (4.1.56), 585.1 174.9 21.93 21.73 12.8 11.63

P 01 x 1 P 02 x 2
x 1g , x 2g :
P P
generated. In order to generate these plots, the integral
Therefore, P0
i
p
qi =PdP has to be developed for each species. Employ
x 1g P 01 x 1 P 01 P 02 P 0
the data provided in Table 4.1.8 for the adsorption of each
x 2g P 02 x 2 P 02 P P 01
pure species; take the pressure as P 0i and the volume
P S S P adsorbed (in cm3) to number of moles of i per gram of
1
P 02
RT RT P 02 p p
adsorbent as qi . Now relate qi =P0i to P0i .
 4:1:53
P P 01 P P 01
 
S Data from Table 4.1.8 for pure oxygen (observed) at
0 0
P 1 RT P 1 0  C were used to develop the following relation:
PM PE ME p
: qO2
MF MF 4:228  106 P 0O2 0:0339 4:1:55
P 0O2
So and r2 0.967 (Figure 4.1.12).
ME MF Then we have
x 1g and x 2g : 4:1:54
EF EF P0 p
S O2 qO2 1
dP 4:228106 P 0O2 2 0:0339P 0O2 :
RT 0 P 2
Example 4.1.2 Markham and Benton (1931) studied experi- 4:1:56
mentally the adsorption of pure O2, pure CO as well as CO
Data from Table 4.1.8 for pure CO (observed) at 0  C were
O2 mixtures on 19.6 g of silica at 0  C. The data obtained are
provided in Table 4.1.8. The column for the mixtures under used to obtain the following relation:
isotherm contains values obtained from pure component qCO
p
adsorption isotherms via interpolation. 1:508  105 P 0CO 0:0755 4:1:57
P 0CO
Develop a plot of x O2 g vs. x O2 for the O2CO mixture
adsorption on silica at 0  C at a total pressure of 1 atmos- and r2 0.987 (Figure 4.1.13).
phere using the ideal adsorbed solution theory. Plot the three Then we have
experimental points for the mixture provided in Tabel 4.1.8
P0 p
in the same diagram. S CO qCO 1
dP 1:508105 P 0CO 2 0:0755P 0CO :
Solution To develop a plot of xig vs. xi, where i O2, first
RT 0 P 2
two plots of (S/RT) vs. P 0O2 and (S/RT) vs. P0CO have to be 4:1:58
226 Separation in a closed vessel

0.0405 0.076
Relationship between Pi0 and qi p /Pi0 at 0C 0
Relationship between PCO and q pi /Pi0 at 0C
0.0400 i = O2
0.074 i = CO
6
mip /PO20 = 4.228 x 10 PO + 0.0339
0
mip /PCO0 = 1.508 x 105P CO + 0.0755
0
2

0.0395
0.072
q pi /PO02 (cm3/mm Hg)

qi /PCO (cm3/mm Hg)


0.0390
0.070
0.0385

0
0.068
0.0380

p
0.066
0.0375

0.0370 0.064

0.0365 0.062
0 100 200 300 400 500 600 700 800 0 100 200 300 400 500 600 700 800
Pressure,PO20 (mm Hg) Pressure, PCO0 (mm Hg)

Figure 4.1.13. Plot for carbon monoxide.


Figure 4.1.12. Plot for oxygen.

200 1.0
180 CO

160 0.8
140

120 0.6
S /RT

O2,g

100 O2
x

80 0.4
60

40 0.2
20

0
0 500 1000 1500 2000 2500 3000 3500 4000
0.0
0.0 0.2 0.4 0.6 0.8 1.0
Pressure, P (mm Hg)
x
O ,
2

Figure 4.1.14. Calculation for mixture of O2CO adsorption equi-


libia from pure component spreading pressure at 0  C. Figure 4.1.15. Prediction of adsorption of O2CO mixture on silica
at 0  C, total pressure 1 atm.

Equations (4.1.56) and (4.1.58) have been plotted in Figure large-as well as small-scale applications. A particular
4.1.14 which is ready to use. Employ equations (4.1.52b) example is the adsorption of organic pollutants from
and (4.1.54) to develop estimates of x O2 , ,x CO, ,x O2 , g and industrial and municipal waste waters onto activated
xCO,g. The predicted behavior of x O2 , g vs. x O2 , is shown in carbon adsorbents. The equilibrium adsorption behavior
Figure 4.1.15. The experimental data have also been plot- of a waste water containing multiple solutes may be pre-
ted to indicate how well the ideal adsorbed solution theory dicted by the ideal adsorbed phase model developed by
predicts the observed behavior. Radke and Prausnitz (1972) for dilute solutions, provided
Another interfacial adsorption system, where one of the single-solute equilibrium adsorption behavior of each
the bulk phases is a solid, involves adsorption of solutes solute species is available. The approach is very similar to
from a solvent onto the surface of solid adsorbents. Such that of the ideal adsorbed solution theory of Myers and
liquidsolid adsorption systems are frequently used in Prausnitz (1965) for gassolid adsorption.
4.1 Two-phase equilibruim separation: closed vessel 227

Radke and Prausnitz (1972) suggested that when the 1 2 : 4:1:63c


solute species adsorb simultaneously from a dilute solution
onto the adsorbent surface at constant temperature and For a system of two solutes, relation (4.1.60) provides
spreading pressure , the adsorbed phase forms an ideal
C tl x 1l C 01 x 1 ; 4:1:63d
solution; activity coefficients for all species are unity in the
adsorbed phase. The following relations were proposed for C tl x 2l C 02 x 2 , 4:1:63e
a system containing n solutes:
Xn where x 1 x 2 1. Choose any Ctl and x1l (equivalent to
1 x i
n p constant T, ; 4:1:59 choosing the concentration of two solutes, C tl x 1l C 1l and
X q
qi i1 i C tl 1x 1l C tl x 2l C 2l ). Then unknowns C 01 , C 02 , 1, 2
i1 and x1 (or x2 since x1 x2 1) may be determined from
equations (4.1.63ae). One can adopt a graphical proced-
C tl x il C 0i x i constant T, 4:1:60
ure of the type suggested by Myers and Prausnitz (1965)
where and illustrated earlier for gassolid adsorption. If the
n single-solute isotherms illustrated by equations (4.1.63a)
X
qi x i qi : 4:1:61 and (4.1.63b) can be obtained in an integrable analytic
i1 form, the set of equations may be solved by computer
using the NewtonRaphson iteration scheme (Radke and
Here, Ctl is the total concentration of all solutes in the
Prausnitz, 1972). This is especially relevant for a system of
liquid phase, xil is the solvent-free mole fraction of species
solutes numbering more than two.
i in the liquid phase, xi is the surface phase mole fraction
In some liquidsolid adsorption processes, the number
of species i without the solvent, C 0i is the unknown
of different components to be adsorbed is very large. One
liquid-phase concentration of the pure solute i at spread-
can adopt the formalism of continuous chemical mixtures
ing pressure and temperature T, which provides the
and represent the class of similar adsorbable components
same surface phase concentration of i as the mixture, and
p by one variable M; a useful choice may be a parameter of
qi is the number of moles of i per unit mass of adsorbent
the adsorption isotherm, e.g. the slope at infinite dilution
when adsorbed from a pure solution of i at the same and
(Annesini et al., 1988).
T as the mixture.
In Section 3.3.7.6, the distribution of a solute between
The first relation suggests that the total number of
a bulk phase and an interfacially adsorbed phase was
moles of adsorbed solutes is determined by the sum total
considered for a gasliquid system containing surfactants,
of the adsorptions of single solutes at the same values of T
gassolid adsorption systems and liquidsolid adsorption
and . For the second result, relating the equilibrium mole
systems. So far, in this section, the separation factor for
fractions of a given solute in the two phases, C 0i is
two solutes has been determined for an airwater system
unknown. One has to evaluate the spreading pressure
containing surfactants and for gassolid adsorption. We
in the manner of equation (4.1.49) from each pure com-
p could do the same also for the liquidsolid adsorption
ponent adsorption data of qi vs. Ci,
considered in the preceding paragraphs.
C 01 p In liquidsolid adsorption, if microporous adsorbents
S qi C 0i
dC 0i : 4:1:62 are involved, then partitioning of the solute between the
RT 0 C 0i
external solution and the pore solution has to be con-
Then, for a given , C 0i can be determined for each species i sidered to obtain accurately the surface adsorption equi-
(just as P 0i was determined earlier for gas adsorption), libria. If C 0il is the initial bulk liquid concentration of solute
which will yield xi from relation (4.1.60) for known Ctl i in the initial bulk solution volume V 0l , and Cil is the bulk
and xil. liquid concentration of solute i in the final bulk liquid
If experimental liquidsolid adsorption equilibrium volume Vl, after equilibration of the microporous adsorb-
p
data are available for each solute species i at a given ent having a specific pore volume (cm3/g of support) of V l ,
2
temperature, then, to obtain the equilibria in a multisolute mass ws (g) and specific surface area S (m /g), then the
system, employ relation (4.1.62) for each solute species to following solute balance holds:
obtain a relation between i and fi C 0i . For example, for a p p
C 0il V 0l C il Vl C il V l ws i ws S , 4:1:64a
system of two solutes being present (i 1,2), obtain
p
where is the pore liquid concentration of species i and
C il
1 f1 C 01 ; 4:1:63a
i is the concentration of adsorbed species i per unit
2 f2 C 02 : 4:1:63b surface area of the microporous adsorbent. The different
liquid volumes are related by
For any given for the mixture, C 0i is to be defined at the
p
same ; therefore, V 0l Vl V l ws : 4:1:64b
228 Separation in a closed vessel

We can now rewrite (4.1.64a) as If the swelling pressure, PR Pw, is negligible, we obtain
p p p
C 0il V 0l C il V 0l
" 1 1 #
C il V l ws C il V l ws i ws S , aAw Z A aBR Z B
ln 0, 4:1:67
and rearrange to obtain aAR aBw
 p 
C 0il C il V 0l C il i S which leads, after rearrangement, to

p w s 1 : 4:1:65
C il V l C il C il V pl
aAw Z B aBw Z A
   
p : 4:1:68
The quantity C il =C il is the geometrical partitioning factor im aAR aBR
defined earlier (see (3.3.88a) and (3.3.89a)) for the partitioning
of a solute between a solution and a porous membrane. Here Suppose ZA 2, ZB 1 and aAw aBw. Then
the porous membrane is replaced by the microporous adsorb- aAR
 
aAw 1 aAR aBR
ent. The quantity im is less than 1 unless there are specific or 2 ) : 4:1:69
a2BR aBw aBw aBR aBw
nonspecific interactions (electrostatic or van der Waals inter-
actions) between the solute and the pores; it can be quite small This result indicates that, if, for counterion B, the resin
if the radius ri of the solute molecules is of the order of the pore phase is selective over the aqueous phase, i.e. aBR > aBw,
radius rp. For cylindrical pores (see (3.3.88a)), then aAR > aBR: the resin phase is selective toward counter-
ion A having a higher valence or charge number over
ri 2
 
im 1 : counterion B with a lower valence. This result reflects the
rp strong influence of counterion valence on the selectivity:
from a mixture of two counterions, A and B, the counterion
In any equilibrium partitioning experiment using a micro-
having a higher valence, A, is always preferred by the ion
porous adsorbent and an external solution, one varies C 0il
exchange resin. Correspondingly, the separation factor aAB
and ws for a given adsorbent. From equation (4.1.65), a plot
defined using activities, namely
of C 0il vs. ws for a given adsorbent will be linear, the slope
being equal to the quantity in brackets on the right-hand aAR aBw
aAB , 4:1:70
side of equation (4.1.65). Unless rp >>> ri, it is clear that aAw aBR
p
im C il =C il will be less than 1, and the determination of will be greater than 1, for example, when aAw aBw . For a
the pore surface adsorption equilibrium relation (between cation exchange resin, for example, Ca will be preferred
i and Cil) will be influenced by im. This was demon- over Na.
strated clearly in the adsorption of aromatic compounds Helfferich (1962) has provided a detailed analysis of
(for example, napthalene) on microporous silica gel the influence of other factors on ion exchanger selectivity:
adsorbents by Alishusky and Fournier (1990). the ion exchanger tends to prefer (there are exceptions)
(1) the counterion of higher valence,
4.1.6 Liquidion exchanger systems (2) the counterion having the smaller (solvated) equiva-
lent volume,
Ion exchange processes are employed in a variety of situ-
(3) the counterion having greater polarizability,
ations in process-scale as well as preparative separations.
(4) the counterion which interacts more strongly with the
A situation often encountered involves the selectivity of the
fixed ionic groups or the matrix,
ion exchanger for counterion A over counterion B. To
(5) the counterion which has a lower tendency of complex
develop an expression for the selectivity, we employ rela-
formation with the co-ion.
tion (3.3.118b) describing the equilibrium distribution of
ionic species i between an aqueous solution and the ion Generally, the order of selectivity among univalent cations
exchange resin for ionic species A and B: is as follows (Helfferich, 1962):

Ag > Cs > Rb > K > Na > H > Li :


2 0 1 3
1 4 a Aw A V A P R P w 5 R w
RT ln @
ZA F aAR For weak-acid resins, the position of H will depend on the
2 0 1 3 acid strength of the fixed anionic group. The correspond-
1 4 aBw A ing sequence for bivalent cations is:
RT ln @ V B P R P w 5:
ZB F aBR
Ba2 > Pb2 > Sr2 > Ca2 > Ni2 > Cd2 > Cu2
This leads to > Co2 > Zn2 > Mg2 > UO2 2 :
Z1  Z1 # For anions, the selectivity sequence appears to be
"
aAw aBR VA VB
 
A B
RT ln P R P w :
aAR aBw ZA ZB citrate > SO4 2 > oxalate > I > NO3 > CrO4 2 > Br
4:1:66 > SCN > Cl > formate > acetate > OH > F :
4.1 Two-phase equilibruim separation: closed vessel 229

For weak-base resins, the position of OH is variable, (correspondingly NaCl over Na2SO4). See Problem 4.1.14
but is generally further to the left. for a related exercise.
Equation (4.1.70) expresses the selectivity between So far, we have activities of species to arrive at a
two particular counterions A and B by a resin. One number of conclusions. In practice, species concentrations
would also like to know the nature of the corresponding are measured. Therefore, let us consider relation (4.1.68)
relations that exist between a counterion and a co-ion for the selectivity of the ion exchanger for counterion
or two co-ions. Consider the case where there are two A over counterion B:
counterions, A and B, and a co-ion Y. Using relation 0 1Z B 0 1Z A 0 1 0 1Z A =Z B
(3.3.118b), we obtain aAw a Bw a Aw a Bw
A @ A )@ A@
2 0 1 3
@ A
aAR aBR aAR aBR
1 4 a Aw
RT ln @ A V A P R P w 5 0 1Z A =Z B 0 1
ZAF aAR x AR AR @ x BR BR A x AR x Bw A
2 0 1 3 ) ) @
x Aw Aw x Bw Bw x Aw x BR
1 4 a Bw
R w RT ln @ A V B P R P w 5
0 1Z A =Z B 0 1Z A =Z B 1
ZB F aBR
2 0 1 3

AB Aw @ BR A
@ x BR A ;
AR Bw x Bw
1 4 a Yw
RT ln @ A V Y P R P w 5:
1 Z A =Z B 1
ZY F aYR ZA
=Z B
0
AB
Aw BR
ZA
@ x BR A :
A rearrangement leads to
=Z B AR x Bw
Bw
" 1 1# 
aAw Z A aYR ZY VA VY

RT ln P R P w : 4:1:75a
aAR aYw ZA ZY
In a dilute solution of counterions A and B in water, one
4:1:71 may assume Aw 1, Bw 1. Therefore,
Relation (4.1.66) is another one of these relations. Ignoring  ZA =
x BR Z B 1
ZA
BR =Z B

the swelling pressure in (4.1.71) leads to AB :
AR x Bw
" 1 1#
aAw Z A aYR Z Y
ln 0: If ZA 2, ZB 1, then
aAR aYw
2BR x BR
  
Therefore, AB : 4:1:75b
AR x Bw
Z1 Z1 Z1
aAw aYw aBw
  
A

Y

B
, 4:1:72 For very low xBw, AB will have a large value. For larger
aAR aYR aBR xBw, AB will be much smaller. Consider the case of A
where we have employed the result (4.1.67). In fact, this result Ca2 and B Na, as encountered in the removal of
is valid for all other ions in the solution (Helfferich, 1962). hardness (Ca2) from water. For low values of salt con-
Consider another co-ion in the system, say X. It follows centration in water (i.e. low xBw), the selectivity of the ion
from the above that exchanger for Ca2 over Na will have some value. If,
however, xBw is high, AB will be much lower, leading to
Z1 Z1
aYw aXw removal of the Ca2 ions from the ion exchanger. At low
 
Y X
: 4:1:73
aYR aXR values of xBw, the ion exchanger will have many more
Ca2 ions, thus removing the cause of hardness from
Let ZY 1 and ZX 2. For the cation exchange resin- water.
based system, It is useful to consider now another kind of counterion
1 exchange process between the ion exchanger and the
aYw 2
1
aYw aXw 2 aXw
     
) : 4:1:74a counterions in the external solution. The counterions to
aYR aXR aYR aXR
be considered now are, macroions, specifically proteins of
If aYw aXw and aYR < aYw for the cation exchange resin, large molecular weight. Any such protein molecule in
then the separation factor solution has positive as well as negative charges distributed
aYR aXw aYw on its surface. The net charge of the macromolecular pro-
aYX > 1: 4:1:74b tein due to pH-based interactions of various constituent
aYw aXR aYR
groups is positive if the pH is less than the isoelectric point
Therefore, a co-ion of lower valence (Y) is more strongly (I.E.P pI); the net charge is negative if the solution pH >
preferred by a cation exchange resin; for example Cl will pI. Thus, as long as the solution pH is different from the pI,
be preferred over SO42 by a cation exchanger the protein surface has some net positive or net negative
230 Separation in a closed vessel

Ion exchange
polymer chain Protein

Protein Ion
exchange
polymer
chain

Before ion After ion


exchange exchange

Figure 4.1.16. Ion exchange occurring when a negatively charged protein adsorbs to an anion exchanger. Seven positively charged
ions (e.g. HTris) associated with the protein molecule are displaced, together with seven negative ions (Cl ) from the exchanger.
(After Scopes (1987).)

charges. Interactions due to pH based chemical reactions Another form of ion exchange of interest is that
in general are considered in Chapter 5. between two different proteins, i 1 and 2, and an appro-
Due to the principle of electroneutrality, small counter- priate active charged site on the ion exchange resin
ions present in the solution will be distributed around the surface. For any protein i, one can represent this inter-
protein surface to ensure local electroneutrality. Consider action by
now a negatively charged protein molecule (with positively
i j 2 S j 1 iS j 1, 4:1:76
charged microions distributed around it) to be adsorbed
on an anion exchanger resin bead (Figure 4.1.16). The fixed where the resin phase is j 1 and S represents the resin
positive charges on the anion exchange resin have counter- site. The equilibrium constant for this binding is usually
ions (e.g. Cl) present to start with. The protein molecule identified as Kd and represents the processes of adsorption
has to displace these counterions (Cl) near the resin sur- (rate constant is ka) and the dissociation (rate constant is
face in order for it to be ion exchanged. As shown in the kd). At equilibrium, the rates are equal:
figure, after the protein molecule is adsorbed, seven coun-
k a C i2 C S1 k d C iS, 1 : 4:1:77a
terions (Cl) near the ion exchange resin surface and seven
counterions present around the protein in solution (e.g. Now CS1 representing the concentration of active charged
HTris)3 are displaced together into the solution. This sites, can be expressed as the difference between the max-
addition of TrisCl into the solution increases the ionic imum number of charged sites, C m S1 , and the number of
strength of the solution. sites already occupied by the protein, CiS,1:
The electrostatic attraction between the negative
C S1 C m
S1 C iS, 1 : 4:1:77b
charges on the protein surface and the positive fixed
charges on the anion exchange resin can be considerably At equilibrium,
weakened by two methods. If the pH of the buffered solu-
tion is decreased to a lower value, then the electrostatic k d C i2 C m
S1 C iS, 1
Kd : 4:1:77c
binding force is considerably reduced. Alternatively, if the ka C iS, 1
ionic strength of the solution is increased, the interaction Correspondingly, at equilibrium, the protein concentration
between the protein and the ion exchanger is reduced and in the resin phase, CiS,1 will be related to the protein
that between the microions (e.g. Cl) and the ion exchan- concentration in the surrounding fluid phase, Ci2, by
ger is considerably enhanced. This method is generally
preferred if desorption of the adsorbed protein is desired. C i2 C m
S1
C iS, 1 : 4:1:77d
K d C i2

At any temperature, this relation behaves as a Langmuir


adsorption isotherm (see Section 3.3.7.6). The equilibrium
3
Tris stands for tris (hydroxylmethyl) aminomethane. constant, Kd, has a low value for proteins which bind
4.1 Two-phase equilibruim separation: closed vessel 231

strongly to the resin. For any resin, the comparative bind-


ing strengths of different proteins will be indicated by
the respective values of Kd. Patel and Luo (1998) have
indicated that Kd values for many different proteins and TH
a variety of ion exchange resins vary between 105 and
107 M. Experimental protein adsorption data plotted as Component 2
B
(1/Cis,1) vs. (1/Ci2) will yield (1=C mS1 ) as intercept and TL
TL
(K d =C m
S1 ) as the slope: A
1=C is , 1 K d =C m m
S1 1=Ci 2 1=C S1 : 4:1:77e

xil, mole fraction


TH
4.1.7 Supercritical fluidbulk solid/liquid phase
TH
Section 3.3.7.9 describes how a supercritical fluid (SCF),
e.g. supercritical CO2, can extract effectively a solute from a A
liquid or a solid; the solute is recovered later by reducing Component 1
the pressure: the SCF becomes a gas having very little TL
TL B
solubility of the solute, which precipitates and is recovered.
This section briefly considers the systems where two or TH
more solutes are extracted simultaneously. There are two
goals here. First, one would like to increase the solubility
of a particular species. Second, it would be desirable to
enhance the selectivity of the SCF for a particular solute P1* P*2
with respect to the second solute. Pressure
Generally, it has been found that for a pure SCF (e.g.
CO2), the extent of enhancement of the solubility for a Figure 4.1.17. Separable crossover points with two solutes dissolved
particular solute is very similar for other solutes being in a supercritical solvent. (After Chimowitz and Pennisi (1986).)
extracted over the entire range of the density of the SCF.
Thus pure SCFs are not helpful in developing selectivity
unless other entrainers (e.g. methanol) are added in small other hand, the equilibrium solubility of species 1
amounts. (Note: One of the lures of extraction by SCF is the decreases as the process involves change from A to
opportunity to avoid organic extractants, e.g. methanol.) B. Thus pure component 1 can be made to drop out of
However, Chimowitz and Pennisi (1986) have suggested the SCF phase by a change of temperature in the crossover
the use of the crossover region for developing selective SCF regime. Chimowitz and Pennisi (1986) have provided data
separation. for SCF CO2 based extraction from a solid mixture of
Consider Figure 4.1.17 for the crossover regime of a 1,10-decanediol and benzoic acid; in the crossover regime,
SCF extracting two solid solutes, components 1 and 2, from pure decanediol was deposited. For successful use of the
a solid mixture, where the mole fraction of each solute, xil, resulting selectivity, it is useful to have a wider crossover
in the SCF phase has been plotted against the SCF pressure region (Johnston et al., 1987).
for two temperatures TH and TL (<TH). The crossover point
for any solute i is the pressure P i at which x il =TP 0,
4.1.8 Bulk fluid phase mesophase systems
signifying a change in the temperature dependence of the
solubility of species i. Below this pressure, a decrease in the At the beginning of Section 3.3, bulk phases commonly
temperature increases the solute solubility in the SCF, used for developing separation processes/techniques
whereas above this pressure an increase in the tempera- employing two immiscible phases were identified: gaseous,
ture increases the solute solubility. liquid, solid, supercritical fluid, ion exchange material,
When there are two solid solutes (extracted into the membrane. In addition, interfacial phases are capable
SCF) each having a different crossover pressure, we have a of developing separation with bulk phases, as has been
crossover regime (Figure 4.1.17). At an intermediate pres- illustrated in Sections 3.3.5 and 4.1.5. There are other
sure between the two crossover pressures P 1 and P 2 for mesophases, or molecular aggregates, and associated envir-
solutes 1 and 2, considerable selectivity can be developed onments which can participate in selective partitioning of
between the two solutes by changing the temperature. solute species between itself and another bulk liquid
Suppose the SCF is initially at TH and is then cooled to a phase. Micellar systems provide a particular example
temperature TL (<TH). For component 2, the process will of such a mesophase quite useful for separations. Other
involve movement from A to B (Figure 4.1.17), i.e. the so-called mesophases useful for partitioning of solutes
equilibrium solubility of species 2 is increased. On the from a bulk liquid phase are: reversed micelle, vesicle,
232 Separation in a closed vessel

Table 4.1.9. Solubilization coefficient Ki1 for a number of organic


Surrounding Hydrophobic nonpolar
solutes in different micellesa
water hydrocarbon
Surfactant Organic solute Ki1 (M1)

CPCb 1-butanol 6
toluene 121
p-chlorophenol 495
SDSc phenol 15
4-tert-butylphenol 365
hexane 630
Triton X-100d naphthalene 1120
pyrene 74 000
a
Data from Christian and Scamehorn (1989) and Jafvert et al.
(1995).
b
Cetylpyridinium chloride (CMC 0.88 mM).
c
Sodium dodecyl sulfate.
Locus of solubilization of Hydrophilic d
B8A6E9.5; B8 is a branched hydrocarbon chain, A6 is a six-carbon
nonpolar organic compound polar head
aromatic ring, E9.5 is 9.5 repeating CH2CH2O group (mol.wt. 628).

Figure 4.1.18. A spherical micelle can solubilize hydrophobic


solutes in the hydrophobic interior.
coefficient (Christian and Scamehorn, 1989; Jafvert et al.,
1995) or an equilibrium constant4 Ki1 for the solubilization
liposome, etc. A central feature of these three mesophases process described by
is that they are globular dispersions, with water as the
organic solute surfactant micelle organic solute  micelle
inner core into which solute partitioning occurs. Therefore,
one can question from a solute partitioning point of view as
whether we are dealing with a new phase.
solute mole fraction within a micelle x i1
Surface-active agents (surfactants) spontaneously form K i1
concentration of free solute in bulk solution C i2
dynamic aggregates called micelles when their concentra-
4:1:78
tion in water (or other solvents) increases beyond a certain
level, called the critical micelle concentration (CMC). These where region j 2 is the bulk solution and region j 1 is
micellar mesophases, or pseudophases, can have several the micelle phase. Instead of being dimensionless, the
forms, i.e. spherical, cylindrical, disk, etc. When the solvent units of this Ki1 are (mol/liter)1. Values of this solubiliza-
is water or the solution is aqueous, the micellar aggregate tion coefficient for a number of organic solutes in micelles
forms with the polar headgroup of the surfactant sticking prepared from different surfactants are provided in Table
out on the outside surface and the hydrophobic (or lipo- 4.1.9. Compounds which are highly hydrophobic and have
philic) chain on the inside (Figure 4.1.18). For a spherical very low solubilities in water will have high values of Ki1.
micelle, the sphere radius will be about 2 nm for a 2 nm The solutemicelle equilibrium leads to two basic situ-
long surfactant. The interior of the aggregate is hydropho- ations of relevance to separation: the solution surrounding
bic due to the hydrophobic chains of the surfactants. the micelles is stripped of organic solutes; the solutes are
Such a micelle acts like an oil drop in an aqueous transferred to the micellar phase whose dimensions are
solution and allows solubilization of organic compounds an order of magnitude larger. These features have been
present in the aqueous solution into the hydrophobic exploited in a number of different separation techniques
interior. If the surfactant is ionic, i.e. either cationic or at large and small scale. Contaminated water may be
anionic, then oppositely charged ionic species, namely purified by micelles, which remove the organic pollutants
counterions from the solution, will be adsorbed on the into the micellar core; the purified water may then be
surface of the micelle or bind with the charged micellar removed through an ultrafiltration membrane (see
surface. Such surface binding will effectively increase the Sections 3.4.2.3, 4.3.4, 5.4.2, 6.3.3.2 and 6.4.2.1), while the
micelle dimension. A unique number of surfactant mol- pollutant-laden micelles are retained by the membrane
ecules between 50 and 100 aggregate to form a micelle. (as in micellar-enhanced ultrafiltration (MEUF, see Section
The partitioning of a solute between the micelle (inter- 5.4.2)). A micellar solution may be injected subsurface to
ior for uncharged species, exterior for charged species) solubilize and remove hydrophobic pollutants into the
has been described in a number of ways, including the
equilibrium ratio Ki, the distribution coefficient il, etc.
One method of description for uncharged organic solutes 4
It is obviously different from our usual Ki1, an equilibrium ratio,
used by a number of investigators defines a solubilization defined by equation (1.4.1).
4.1 Two-phase equilibruim separation: closed vessel 233

Reverse
micelle

Surfactant

Protein

Oil
Water

Figure 4.1.19. Schematic representation of the reversed micellar extraction of proteins. (After Gklen and Hatton (1985) and
Hatton (1989).)

micellar solution ultimately withdrawn from the ground. In Surfactant


addition, such micellar solutions may aid in chromato-
graphic partitioning by being employed as a mobile
phase in chromatographic processes (see Sections 7.1.5 Liquid crystals
and 8.2).
In the micelle based extraction of organic solutes from Two-phase
an aqueous solution, the mass-separating agent/phase has region Reversed
micelle
been developed by the addition of surfactants to the aque-
solution
ous solution. In most situations, this separating agent
needs to be recovered for reuse; a concentrated stream of
organic solutes should also be obtained from this micellar Aqueous
phase. When the organic solutes are nonvolatile, chemical phase
reactions can be employed to precipitate the surfactants
(Brant et al., 1989). Solvent extraction through a porous
membrane has also been attempted (Kitiyanan et al.,
1999). When the organic solutes are volatile, they may be Water Oil
removed by stripping or pervaporation and the micellar
solution recycled (Abou-Nemeh et al., 1998). Figure 4.1.20. Typical phase diagram for a watersurfactantoil
We now consider another kind of pseudophase. In the system. (After Hatton (1989).)
presence of an organic solvent immiscible with an aqueous
phase containing surfactant molecules, new types of sur-
factant molecular aggregates, called reversed micelles, are The reversed micellar structure in an aqueousorganic
formed. As shown in Figures 4.1.19, two-tailed surfactants system is obtained under practical conditions in a certain
(having a polar water-soluble head group and two hydro- composition range shown in the phase diagram for
carbon tails insoluble in water) form aggregates in the waterisooctaneAOT (Figure 4.1.20). Any mixture of
organic phase that contain an aqueous core and a surfac- waterisooctaneAOT whose composition falls in the two-
tant shell, where the hydrophobic surfactant tails stretch phase region of this phase diagram will spontaneously
out into the organic solvent. Figures 4.1.19 illustrates the split into an aqueous phase and an organic-dominated
extraction of a protein molecule from the aqueous phase reverse micellar solution phase via a tie line (the dashed
into the aqueous core of the reversed micelle structure line). The nanometer-sized surfactant aggregate in the
present in the organic solvent phase. A typical surfactant organic solvent is stable. A typical example of overall phase
which leads to the development of such structures is concentrations in protein extraction from a protein-
sodium di-2-ethylhexyl sulfosuccinate (AOT). containing aqueous buffer solution is: equal volumes
234 Separation in a closed vessel

Ribonuclease A

100

80
% Solubilization

Lysozyme
60

40
Cytochrome C

20

0 2 4 6 8 10 12 14
pH

Figure 4.1.21. The effect of pH on the solubilization of lysozyme,


cytochrome C and ribonuclease A in AOTisooctane solutions. Salt
concentration: 0.1 M KCl. (After Gklen and Hatton (1987).) Carriers Phosphatidylcholine
lipid

Figure 4.1.22. Vesicle structure.


of 250 mM AOT in isooctane in equilibrium with 0.1 M KCl
in 0.01 M acetate buffer, pH 4.0.
The protein so extracted into the reverse micellar (RM) they can be much bigger, incorporate many layers of the
phase may be easily back extracted if the RM phase is lipid bilayer and are called multilamellar vesicles (MLVs),
contacted with an aqueous buffer solution containing a as opposed to those having a single lipid bilayer (Lasic,
higher KCl concentration, e.g. 0.1 M, or if isopropyl alcohol 1992). Both structures are also called liposomes.
is added to the extent of 1015% to the aqueous phase used The 1418 carbon atoms long chain of the lipid in the
for back extraction (Carlson and Nagarajan, 1992). How- lipid bilayer presents a rigid hydrocarbon gel-like structure
ever, the solubilization of the protein in the RM phase or which is essentially impermeable to charged/polar mol-
the back extraction of the protein into an aqueous buffer ecules larger than water. The transfer of polar/charged
from the RM phase depends on a number of complex species from outside to inside of the vesicle requires a
interactions of a variety of factors. These include the pro- carrier; for transferring metal ions, one uses an ionophore
tein pI, the aqueous phase pH, the ionic strength (e.g. KCl as a carrier of the charged/polar species (see Sections
concentration), the type of salt, the type of surfactant and 4.1.9.1.3 and 5.4.4) incorporated into the lipid bilayer
the interaction between the protein and the charged sur- (Walsh and Monobouquette, 1993). The lipid bilayer is a
factant headgroup (Hatton, 1989). Figure 4.1.21 illustrates new material phase that acts as a selective membrane
the percent solubilization of three proteins into the RM between an external aqueous phase and an aqueous phase
phase as a function of aqueous phase pH (Gklen and enclosed by the lipid bilayer, the vesicle. However, these
Hatton, 1987). Often, solubilization into the RM phase noncovalent assemblies of lipids are damaged in the pres-
proceeds if the protein pI > pH; back extraction is accom- ence of detergents, water-soluble alcohols, etc. Covalently
plished if pH > pI. closed vesicles can be formed by using polymerizable
A third category of molecular aggregates are called vesicle-forming surfactants (Shamsai and Monobouquette,
vesicles, liposomes, etc. Like reversed micelles, they enclose 1997).
water inside a spherical shell. However, unlike reversed
micelles, the shell is made of a lipid bilayer structure
4.1.9 Partitioning between a bulk fluid phase and an
present in biological cell membranes. Specifically, the shell
individual molecule/macromolecule or a collection of
has a bilayer structure with the polar hydrophilic head of
molecules for noncovalent solute binding
two layers of lipids forming the two surfaces of a sandwich
of the two layers of lipids, with the long two-tailed non- A variety of chemical interactions are possible between two
polar hydrophobic chains of each lipid in the interior of the molecules of two different species. Those that result in
sandwich (Figure 4.1.22). The specific lipids have a polar chemical reactions are considered in Chapter 5 for the
phosphate head and are therefore usually called phospho- purpose of separation of mixtures. There are a number of
lipids; an example is phosphatidylcholine. The hydrocar- other, weaker, noncovalent interactions where the bond
bon chains in any such lipid are generally 14 to 18 carbon energy is less than 50 kJ/mol. These include chelation,
atoms long, resulting in a typical lipid bilayer thickness of clathration, hydrogen bonding, hydrophobic interactions,
around 3.7 nm. The spherical vesicle dimensions can vary ionic binding, pi bonding, van der Waals interactions, etc.
over a wide range, for example 0.030.1 m. Sometimes These result in weak chemical complexes, which are
4.1 Two-phase equilibruim separation: closed vessel 235

reversible in nature. Further reversing the complexes with- (1) inclusion compounds;
out destroying the complex-forming molecule is possible (2) ionic binding of metals with charged species;
due to weaker interactions. (3) pi bonding/complexation;
King (1980) has enumerated the variety of interactions (4) hydrophobic interaction.
as originally identified by George K. Keller with respect to
The phrase solute partitioning mentioned above should
the bond energy vs. bond type. These interactions lead to
not suggest that the solute must necessarily be partitioned
reversible binding of a solute molecule from a bulk fluid
into a separate phase. There are many examples (to be
phase with a complex-forming individual molecule or col-
discussed in the following) where the solute molecule is
lection of individual molecules introduced from outside.
bound to a different type of molecule in the same phase;
The complexing agent may also be a macromolecule of
such binding facilitates subsequent separation.
biological (e.g. proteins, etc.) or nonbiological (e.g. con-
ventional polymers) origin. The complex so formed may
remain in the original solution phase or may form a second 4.1.9.1 Inclusion compounds: adducts, clathrates, crown
phase, usually a solid. This solid phase may be withdrawn ethers, cyclodextrins, liquid clathrates
and treated to reverse the complexation.
To avoid the need for separation of the solid phase Different types of molecules acting as mass-separating
thus formed, sometimes these molecules/macromolecules agents may be added to a liquid mixture in gaseous, liquid
may be individually bonded to or deposited on a porous or or solid form to form preferentially a solid phase of the
nonporous bead/particle or the pore surfaces of a mem- added external agent molecules and the solvent/one of the
brane at one end of the molecule (for example), while the solute species in the liquid mixture when cooled. The solid
other ends or parts of the molecule are available to bind phase of the solvent/solute and the external added agent is
the solute species from the solution. Species present in the generally called an inclusion compound (Atwood et al.,
gas phase may also bind to such molecules on a bead or 1984). We now consider a variety of inclusion compounds:
particle. When the solute molecule is a protein or a large adducts; clathrates; crown ethers; cyclodextrins; liquid
entity, a large number of binding sites are necessary to clathrates.
hold the solute; thus, a number of individual complexing
molecules bonded to the porous particle at one end will 4.1.9.1.1 Adducts When the external agent added to the
have their other ends bind at different locations of the solution forms the bulk of the new crystalline solid phase,
solute, which is a protein/large entity. A natural extension the process is called adductive crystallization and the com-
of this concept has led to the bonding/coupling of protein pounds so formed are called adducts. The solute molecules
A to agarose beads for extremely selective binding of from the liquid feed will fit into the crystal lattice of this
immunoglobulins from a solution containing immuno- crystalline solid phase, which acts as the host; the solute
globulins and other materials in antibody purification. molecules included in the host crystal are the guest
Separation generally also requires recovery of the (an inverse scenario happens in the case of clathrates, as
solute species after it has been selectively bound to specific we will soon see). The hosts could be either organic or
molecules added to the solution or bound to a porous or inorganic. The slurry that is produced has to be removed
nonporous bead/particle or membrane pores. When the from the raffinate liquid and may be heated to recover
process takes place without any particles, beads or mem- the guest.
branes, the complex produced has to be removed from the An example of this type of process of selective
solution and decomplexation carried out. The steps usually solute partitioning is urea based adductive crystallization.
taken to achieve decomplexation (with or without the use A saturated solution of urea in water at 70  C may be
of beads, particles or membranes), i.e. regeneration of the mixed with a mixture of aromatic and paraffinic hydrocar-
molecules/macromolecules for reuse in binding solute bons present in a solvent at 40  C (the Edeleanu process).
molecules, include: changing the ionic strength of the Under conditions of appropriate refrigeration, lumps of
solution by adding electrolytes; changing the pH; raising urean-paraffin adducts appear as crystals (Findlay,
the temperature; decreasing the pressure, etc. 1962; Fuller, 1972). As shown in Figure 4.1.23(a), the host
Although the binding of the solute molecule with the compound, urea, has crystallized into a form having a
complexing agent is reversible and noncovalent in nature, central tunnel open at both ends; the tunnel accommo-
the mathematical description is generally identical to situ- dates the guest paraffinic hydrocarbon molecule and
ations where chemical reactions take place. Thus, a holds it by van der Waals forces with m urea molecules
number of such reversible low binding energy based sep- forming the tunnel:
arations are described mathematically in Chapter 5. Here
murea paraffin adduct: 4:1:79a
we illustrate the basic nature of the phenomenon of solute
partitioning in such processes in a closed system under the The guest paraffin has significant freedom in the channel,
following categories: akin to adsorption rather than a chemical reaction. The
236 Separation in a closed vessel

(a) (b)

Ni
NH3 2.1

N
C

8.3

5.0

5.1
Cross section of n-paraffin
molecule within urea unit cell

Figure 4.1.23. (a) Representation of a cross section of a urean-paraffin adduct showing the urea molecules forming the walls of a tunnel
in which the n-paraffin is held by attractive forces. (b) Structure of clathrate formed between monoaminenickel cyanide and benzene,
showing the benzene molecule trapped inside a cage formed by the crystal lattice of the former (Findlay, 1962). Reprinted, with
permission, from R.A. Findlay, Adductive crystallization, Chapter 4, Figure 7, p. 283 and Figure 5, p. 275 in Schoen, H.A. (ed.),
New Chemical Engineering Separation Techniques, Interscience Publishers 1962. Copyright 1962, John Wiley & Sons.

moles of urea per mole of the pseudoreactant, paraffin, 80


varies, for example, between 6.1 and 23.3: 6.1 for 60
n-heptane and 23.3 for n-detriacontane (C32H66). For a Hydrate
given paraffin, m is not fixed; the compounds are therefore 40 +
nonstoichiometric. Apparently, urea forms adducts with SO2 (l)
straight-chain paraffinic and olefinic compounds as long A B
as the carbon number is 6 or higher. The heat of this 20
Pressure (psia)

pseudoreaction is approximately 1.6 kcal per carbon atom,


which is less than that for heats of adsorption (see Ruthven Hydrate C
+
(1984) for heats of adsorption). Thiourea has also been
10 SO2 (g)
employed as an adduct-former (Findlay, 1962).
8
4.1.9.1.2 Clathrates Another class of inclusion com- 6
pounds are clathrates, in which, instead of a channel as Liquid water
+
in urea adducts, a clathrate or a cage is formed by the 4 SO2 (g)
host molecules, and the guest molecule is trapped in
this cage (Figure 4.1.23(b)). Generally, the guest molecule
is brought into the liquid phase of the host molecules and
2
the clathrate phase crystallizes out as the temperature is 30 35 40 45 50 55
lowered. When an aqueous solution is under consideration
Temperature (F)
and the guest is a low molecular weight gas/vapor, e.g.
CH4, SO2, C2H4, H2S, C2H5Cl, Cl2, Br2, CO2, C2H4O, etc., the Figure 4.1.24. Phase diagram for the sulfur dioxidewater system.
clathrate is called a gas hydrate. Reprinted, with permission, from G.N. Werezak, in Unusual
The clathrate compound does not have a fixed chem- Methods of Separation, Chem. Eng. Prog. Symp. Ser., No. 91,
ical formula in terms of the number of guest molecules Vol. 65, AIChE, New York, p. 6 (1969). Copyright [1969]
(GM) which can be accommodated in the cage created by American Institute of Chemical Engineers (AIChE).
the host molecules; however, there is a maximum number
of host molecules. Findlay (1962) has listed the following
among others for water as host: for CH4 guest, 46H2O8GM; varying between 780 and 920 pm; the guest molecules,
for propene guest, 136H2O8GM; for SO2 guest, 46H2O8GM. which do not interfere with hydrogen bonding and have
In the gas hydrate, water molecules are linked through diameters in the range of 410580 pm, stabilize the struc-
hydrogen bonding with a cavity diameter inside the cage ture under appropriate temperature and pressure (see
4.1 Two-phase equilibruim separation: closed vessel 237

Table 4.1.10. Inclusion compounds: types and examples

Host Guest (G) Examples

Multiple molecules organic urea n-alkanes, n-alkenes, adductive crystallization; e.g. 6.1 moles
create host molecules straight-chain urea per mole of G
structure: solid acids, straight-
state compounds chain esters
thiourea cyclopentane,
isoparaffins
phenol HCl, HBr, SO2, CS2 clathrate compounds; e.g.,
12C6H5OH  5G
hydroquinone SO2, HCl, HBr, HCN, 3C6H4(OH)2  G
H2S, CO, etc.
inorganic H2O CH4, C3H8, SO2, Cl2, 46H2O  8G
molecules H2S, etc.
monoaminenickel benzene, thiophene,
cyanide aniline, etc.
Ni(4-methyl p-xylene Werner-complex based clathration
pyridine)4
(SCN)2
Single molecule is host organic crown ethers alkali metal ions, Li, extraction of such metal salts into an
molecules K, Cs, Sr2, etc. organic phase from aqueous
phase/bonded phase in
chromatography
cyclodextrins (CD) small nonpolar extraction of such species into
organic aqueous phase containing CD or
molecules; bonded CD phase in
organic isomers; chromatography
p-xylene
Multiple molecules Organic benzene organometallic salt liquid clathrate containing 6 benzene
create liquid host molecules K[Al2(CH3)6N3] to 1 salt molecules
structure

Englezos (1993) for a review of clathrate hydrates). Such number of molecules needed to develop the host struc-
molecules are often called hydrate-formers. ture is generally larger than 1 and can be as high as
The clathrate hydrates are formed only in given ranges 136 (in clathrate hydrates). On the other hand, organic
of temperature and pressure. Figure 4.1.24 illustrates the compounds, such as, crown ethers, cryptands, cyclodex-
pressuretemperature diagram for an SO2water system trins, etc., incorporate a metallic ion or an organic mol-
(Werezak, 1969). Such a diagram will allow determination ecule as guest inside the existing cavity present in the single
of the conditions under which the hydrates are formed, or host molecule.
the hydrate crystals decompose, releasing the trapped mol-
ecule SO2 for recycle and reuse. Such clathrate hydrate 4.1.9.1.3 Crown ethers, cyclodextrins Crown ethers are
formation provides a way of recovering almost pure water cyclic ethers having repeating OCH2CH2 units; therefore,
or concentrating aqueous solutions using an appropriate they are often called macrocyclic polyethers. Figure 4.1.25
hydrate-former. identifies a variety of crown ethers, namely 14-crown-4,
Clathrate compounds are also formed with organic non- benzo-15-crown-5, dicyclohexyl-18-crown-6, dibenzo-21-
aqueous host species, e.g. hydroquinone, phenol, monoami- crown-7. The number 14, 15, 18, 21, etc., in this nomen-
nenickel cyanide, methylnaphthalene, etc. (Findlay, 1962). clature identifies the number of atoms in the polyether
Clathrate compounds formed with monoaminenickel cyan- ring, whereas 4, 5, 6, 7, etc., specify the number of oxygen
ide as host and benzene/thiophene as guest (Figure 4.1.23 atoms or repeating units in the ring (Pedersen, 1967).
(b)) are of interest in petroleum refining; the selectivity of the Table 4.1.11 provides an estimate of the diameter of the
host for benzene is much higher. cavity in the polyether ring in a few crown ethers. This
The nature and types of inclusion compounds are table also provides the diameters of a variety of cations,
more numerous than the two types, namely adducts and primarily alkali metal cations (Pedersen, 1988; Steed and
clathrates, considered so far. A broader list is provided in Atwood, 2000). As long as the cavity diameter is close to
Table 4.1.10. The list is not comprehensive but illustrative that of an alkali metal cation (M), the metal ion is held in
of each category. In both adducts and clathrates, the the cavity as a guest with several oxygen atoms in the ring
238 Separation in a closed vessel

Table 4.1.11. Crown ethers, their properties and cationsa Table 4.1.12. Extraction of alkali metal picrates by different crown
ethers into methylene chloridea
Cavity Cation diameter (nm)
diameter Picrate extracted (%)
Crown ether (nm) Group I Group II
Polyether Li Na K Cs
b 2
All 14-crown-4 0.120.15 Li 0.136 Ca 0.198
All 15-crown-5 0.170.22 Na 0.194 Zn2 0.148 Dicyclohexyl-14-crown-4 1.1 0 0 0
All 18-crown-6 0.260.32 K 0.266 Sr2 0.226 Cyclohexyl-15-crown-5 1.6 19.7 8.7 4.0
Rb 0.294 Cd2 0.196 Dibenzo-18-crown-6 0 1.6 25.2 5.8
All 21-crown 7 0.340.43 Cs 0.266 Ba2 0.268 Dicyclohexyl-18-crown-6 3.3 25.6 77.8 44.2
NH4 0.286c Ra2 0.280 Dicyclohexyl-21-crown-7 3.1 22.6 51.3 49.7
Dicyclohexyl-24-crown-8 2.9 8.9 20.1 18.1
a
Adapted from Pedersen (1988). a
b
All refers to various substitutent groups, e.g. dicyclohexyl, From Table 4, p. 538, of The discovery of crown ethers, Charles
dibenzo, etc. J. Pedersen, Science, Vol. 241, July 1988, pp. 536540. Reprinted with
c
Nonmetallic cations are also amenable to complexation. permission from AAAS.

O O
O O O O
O O O
14 15 O
O O 18 21
O O O O
O O
O O O
O
14-crown-4 Benzo-15-crown-5 Dicyclohexyl-18-crown-6 Dibenzo-21-crown-7

Figure 4.1.25. A few crown ethers.

acting as donors to complex the strongly electropositive (1988). Crown ethers have been incorporated into numer-
alkali metal cations. For example, 18-crown-6 prefers K ous polymers; the resulting polymers show the expected
strongly over Li and Na. If the cation is too small or too order of alkali metal selectivity. Crown ethers have also
large, the resulting complexes are not very stable. How- been bonded to silica. The bonded crown ethers do show
ever, even in such cases, two molecules of crown ethers selectivity for various metallic cations (Alexandratos and
may form a sandwich to hold one cation. Crick, 1996).
Crown ethers have low solubility in water but have In the case of clathrates and adducts described
considerable solubility in organic solvents. Crown ether earlier, the so-called compounds formed are separated
complexes of alkali metal cations are therefore quite by crystallization. Although handling of slurry/solid in an
stable in organic phases. The anions of the alkali metal industrial context is sometimes not desired, the solid
cations present in the aqueous phase, Y, are simultan- phase conclusively demonstrates the nature of the non-
eously extracted into the organic phase; the complex in stoichiometric dissociable hostguest compound formed.
the organic phase may be represented as (crownM)Y. That macrocyclic ethers form reasonably stable com-
Due to the likely absence of solvation of such anions in plexes has also been demonstrated by the isolation of
the organic phase, they are likely to be highly reactive. the complexes as crystals (Pedersen, 1988). That a par-
Nevertheless, crown ethers and similar host compounds ticular crown ether having a certain cavity diameter will
have been successfully incorporated into organic solv- prefer a certain alkali metal cation having a certain
ents to extract alkali metal ions/salts from aqueous diameter is illustrated in Table 4.1.12 for the extraction
solutions. of a particular alkali metal picrate salt from water into
When the organic solvent containing crown ether is methylene chloride containing the crown ether. From
used in the form of a liquid membrane between two the table it appears that potassium picrate is most effi-
aqueous solutions (see Section 5.4.4), an alkali metal salt ciently extracted by 18-crown-6 compared to other
can be selectively transferred through the organic liquid crown ethers, since the cavity size of 18-crown-6 is quite
membrane from one aqueous feed solution to an aqueous close to that of K (see Table 4.1.11).
strip solution. Illustrative treatment of the kinetics and Cyclodextrins (CDs) are cyclic oligosaccharides having
mechanisms of formation and dissociation of the metal the shape of an asymmetrical doughnut (see Figure 4.1.26
complexes with crown ethers is available in Burgess (a)). The three common types of cyclodextrins, -CD, -CD
4.1 Two-phase equilibruim separation: closed vessel 239

(a) (b)

HO O OH CH3
O O
OO
O OH H H
OH OO
O
HO
HO OH
O OH HO
OH
O H H HO O
OO
O O
HO O OH CH3

Figure 4.1.26. (a) Schematic of -cyclodextrin; (b) schematic of p-xylene as a guest in an -CD host.

and -CD, each have a cavity whose diameters are, respect-


ively, 0.57 nm, 0.78 nm and 0.95 nm. The corresponding CH3 CH3
molecular weights and aqueous solubilities are: 972, 1135,
1297 (molecular weights); 14.5, 1.85, 23.2 (solubilities in
g/100 cm3). (See Bender and Komiyama (1978) for an
introduction.) Due to the presence of hydroxyl groups on O Si O Si O
the outside surface, CDs are soluble in water. However, the
interior cavity lined with hydrogen and oxygen atoms is
relatively hydrophobic and provides sites for inclusion CH3 (CH2)8
complex formation with smaller guest organic solutes n
having limited polarity (Figure 4.1.26(b)).
A wide variety of smaller organic guests have been
found to form inclusion complexes with a cyclodextrin (CH3O)6 O
molecule as host. The stability of the inclusion complex
depends on a variety of factors, specifically the geometric
fit with the cavity opening and the nature of the inter-
actions. Such differences are sufficient to create different -CD
complexation tendencies of different isomers with cyclo-
dextrins. There is a considerable literature on the use of
CDs for chiral separations in chromatography; in fact, they
are used in a variety of chromatographic columns in ana- (CH3O)7 (OCH3)7
lytical chemistry. To this end, the CD is bonded to poly-
siloxane (Figure 4.1.27), which can be coated onto Figure 4.1.27. Structure of polysiloxane-bonded permethylated
appropriate silica columns for use in chromatography -cyclodextrin (Chirasil-Dex) (n  60). (After Jung et al. (1994).)
(see Section 7.1.5) (Armstrong et al., 1993; Jung et al.,
1994). The bonding of CD to polysiloxane phase eliminates
the need for any crystallization based phase separation. and aqueous phases using branched -CDs; the branched
Instead, one has to go through the process of complexation CDs have considerably higher aqueous solubility, and
and decomplexation in sequence. p-xylene is strongly preferred (Figure 4.1.26(b)) (Uemasu,
Selective liquidliquid extraction of xylene isomers 1992). An aqueous solution of -CD containing urea and
(m-, o- and p-) has been demonstrated between organic NaOH was employed by Armstrong and Jin (1987) and
240 Separation in a closed vessel

Mandal et al. (1998) as a liquid membrane (see Section molecule in solution or bound to the surface of a bead or
5.4.4) to separate different types of isomeric mixtures a pore in a porous membrane; alternatively, the individual
effectively. charged molecules may be part of a collection of such
molecules, e.g. a surfactant micelle formed from ionic
4.1.9.1.4 Liquid clathrates Unless crown ethers and surfactants.
cyclodextrins are used either in a liquid membrane format, Metals are most often present as cations Mn in a
in a bonded state or in solvent extractionback extraction solution: Cu2, Ca2, Zn2, Na, Pb2, Cd2, etc. Some
methodology, the inclusion compound has to be crystal- metals exist in anionic forms as well: CrO42, HCrO4, etc.
lized out. Crystallization, however, has to be practiced in Consider now a polyelectrolyte, polystyrene sodium sulfon-
the case of adducts and clathrates. The handling of ate (PSS), the sodium salt of the polystyrene sulfonic acid
large-scale solids is difficult in such processes; further- (Figure 4.1.28(a)). If such polyelectrolyte macromolecules
more, in the case of adducts, the solute-bonding are added to water containing hardness-causing metal
capacity is limited due to the large load of the host ions, e.g. Ca2, Mg2, sodium ions will be replaced by
per unit guest molecule. Liquid clathrates have therefore Ca2 or Mg2, which will remain bound to the polymer
been developed so that solvent extraction methodology is via the charged sulfonic acid group. Since a polymer mol-
available and solids handling can be avoided (Atwood ecule is easily removed by filtration, the undesirable ions
et al., 1984). remain bound to the anionic polyelectrolyte filtered and
Liquid clathrates essentially consist of a liquid made concentrated. Each polyelectrolyte molecule (polymer) will
out of guest molecules entrapped in a host species. They have many metallic ions bound to it at every location
are primarily based on low-melting organometallic salts, where there is an oppositely charged ion (Tabatabai
which have a high solubility in aromatic liquids, as well as et al., 1995).
a high selectivity for liquid aromatic hydrocarbons. For Such polyelectrolyte macromolecules may also be
example, bound at one end on the surface of a bead or on the
surface of a pore in a membrane. The whole polyelectrolyte
KAl2 CH3 6 N3 l 6C6 H6 l KAl2 CH3 6 N3   6C6 H6 l,
molecule is essentially available for binding the metal ions
4:1:79b in solution. However, since the polyelectrolyte is bound at
indicating that a liquid of composition 1(salt): 6(aromatic) one end, filtration is no longer necessary to recover it and
has been formed; this liquid is immiscible with and heavier the bound metal ions. An example of a polyelectrolyte
than pure benzene phase. The low melting salt K bound at one end on the pore surface of a microfiltration
[Al2(CH3)6N3] has weak interionic interactions, and the membrane and having various interactions with metal ions
benzene molecules surround the anion with the cation is schematically shown in Figure 4.1.28(b). Here, poly
outside the cage. There is some order, such that the cations (amino acid) molecules (e.g. poly(L-glutamic acid)) are
attached to different adjacent cages cannot interact with bound to the pore surfaces of porous cellulosic mem-
one another. The stripping/back extraction can be easily branes (Hestekin et al., 2001). The COO groups created
implemented, only with particular liquid clathrates, by at higher pHs bind directly with the metal ion (shown as
changing the temperature by only 10  C. Me2) after deprotonation of COOH groups in the poly
(amino acid). In addition, the electrostatic potential
field created by the neighboring COO groups leads to
4.1.9.2 Ionic binding of metals with charged species
loose retention of metallic counterions (counterion
Metal ions are often present in a variety of aqueous solu- condensation).
tions. In hydrometallurgy, the metal ion is to be recovered; The reverse of the metal ion binding process is import-
in environmental separations, the heavy metal ions5 have ant in the process of recovering a concentrated solution of
to be removed from water and the water purified; in pro- metal ions and the polyelectrolytes. The bound heavy
cess streams, it could be either. Weak ionic binding of metal ions in a homogeneous solution of polyelectrolytes
metal ions to a variety of charged species may be utilized may be released by contacting with a concentrated brine.
to achieve such goals. The charged species can be poly- In the case of PSS in solution used to bind Ca2and Mg2,
electrolytes, ionic surfactants, chelating agents, polyamino Na ions from brine will replace Ca2 and Mg2, etc. Such
acids, etc. The charged species may be an individual a solution, when filtered, will yield a filtrate concentrated in
Ca2, Mg2, etc. The concentrate or retentate solution will
contain the polyelectrolyte in the Na form and can be
5
recycled for re-use (Tabatabai et al., 1995). An alternative
There are a variety of definitions for heavy metals. A few useful
strategy of using high acid/low pH will lead to release
definitions are given here: any metal having a specific gravity
greater than 5; any metal generally toxic to biological systems; of the bound metal from the poly(amino acid)s bound
any metal readily precipitated from solution as a sulfide; any to the membrane pore surface (Figure 4.1.28(c)) (Hestekin
metal located in the lower half of the periodic table, etc. et al., 2001).
4.1 Two-phase equilibruim separation: closed vessel 241

(a)

SO3H SO3H

(b) (c)
Polyamino acid
- - - - chains
COO COO COO COO

Me2+ Me2+ Me2+ Me2+ Me2+

+
Electrostatically COO- COO- COO-
bound (counterion Me2+ Me2+
condensation)

COO- COO- COO- COO-

Me2+ Me2+ Me2+ Me2+ Me2+

Electrostatically COO- COO- COO-


bound (counterion Me2+ Me 2+

condensation)

Figure 4.1.28. (a) Polystyrene supported sulfonic acid. (b) Metal binding by two poly(amino acid) chains bound at one end to the pore
surface of a membrane (Hestekin et al., 2001). Reprinted, with permission, from I&EC Research, 40, 2668 (2001), Figure 1(a). Copyright
(2001) American Chemical Society. (c) Poly(amino acid) chains bound on the surface of a pore in a membrane.

Ionic surfactants will form micelles in a solution if their C12 SO4 aq Na ! NaC12 SO4 s 4:1:80a
concentrations are above their critical micelle concentra-
tion (CMC). The ionic surfactant, for example sodium (see Brant et al. (1989)). A multivalent counterion, such as
dodecyl sulfate (SDS), is prepared as the salt in which Ca2, may also be used to precipitate the surfactant:
Na is the counterion and DS, representing dodecyl sul- 2C12 SO4 aq Ca2 ! CaC12 SO4 2 s: 4:1:80b
fate (C12SO4), is the surfactant ion (here an anion).
A micelle of such a surfactant will have numerous ionic Thus, regeneration of metal ions, surfactants, polyelectro-
headgroups, DS, around the periphery of individual lytes, poly(amino acid)s, etc., participating in the ionic
micelles. If there are other metallic ions present in the binding of metal ions in solution will require chemical
system, e.g. Ca2, Cu2, Zn2, Cd2, etc., there will be an reactions. Chemical reactions are considered in Chapter 5
exchange between these ions and Na as the counterions in general for their effects on separations.
for the DS ions. Thus heavy metals, and other metals, Whether the metal-binding agent is a polyelectrolyte or
present in solution will be bound to the headgroups of an ionic surfactant headgroup in a micelle, it is of interest
the micelle formed from the ionic surfactant. By using an to know the extent of binding of a metal ion to such a
appropriate membrane/filter, one can concentrate the species/agent. Based on the work of Oosawa, Scamehorn
ionic micelles and their bound heavy metallic counterions. et al. (1989) have suggested equations that relate the frac-
The surfactants may be recovered from the concen- tion of a metal ion that is bound to the micelle to the free
trated solution by precipitating the surfactant using high (unbound) metal ions. The estimation of these fractions
concentrations of a monovalent counterion (e.g. Na(aq)). requires additional relations, such as the electroneutrality
For example, neutral SDS salt may be precipitated via condition, concentrations of surfactants present as micelles
242 Separation in a closed vessel

(a)

Na+

NaCl Cl-
Cl- Cl- Na+

Cl-
Na+

Na+

Na+
(b)

Cl-
NaCl
Cl-

Na+ Cl-

Na+

Figure 4.1.29. (a) The polyelectrolyte effect; (b) the anti-polyelectrolyte effect (Lowe and McCormick, 2001). Reprinted, with permission,
from Stimuli-Responsive Water Soluble and Amphiphillic Polymers, ACS Symposium Series 780, 2000, p. 352, C.L. McCormick (ed.),
A.B. Lowe and C.L. McCormik (Chapter 1 authors), Figures 1 and 2 on pp. 2 and 3 of Chapter 1. Copyright (2000) American
Chemical Society.

and other parameters involving the CMC value, the added, the individual ionic groups in the polyelectrolyte
charges of various ions, etc. will be shielded by Na and Cl ions; the attractive inter-
The binding or presence of metallic ions or their coun- actions will be reduced, and the polymer will take up a
terions in aqueous solutions of polyelectrolytes considered more expanded configuration (the anti-polyelectrolyte
so far can influence the conformation of free polyelectro- effect) (Lowe and McCormick, 2001) (Figure 4.1.29(b)).
lytes in solution. At high concentrations of salt (NaCl), Such phenomena are the basis of a number of separ-
there can be precipitation of polyelectrolytes (equation ation techniques. For example, if the polyelectrolyte has
(4.1.80a)). A range of behavior is possible, having signifi- a collapsed globular structure, then the diffusional resis-
cant effects on separation. Consider a polyelectrolyte, poly tance of molecules through such a polyelectrolyte based
(methacrylic acid) (PMMA), which will carry negative medium will be drastically increased; larger molecules,
charges from carboxylate groups at high pH. These will e.g. proteins, may not be able to diffuse effectively. On
repulse each other, leading to an extended conformation; the other hand, a collapsed structure may open up large
there will be counterions present near each charged group gaps between the polyelectrolytes and allow easy transport
in the backbone. The end-to-end distance will be large. through the overall medium. If a gel-like structure can be
When the concentration of the polyelectrolyte (in this created using polyelectrolytes, the swelling or contraction
case, polysalt) is increased, or the concentration of the salt of the gel can open up or shrink transport corridors based
(NaCl) is increased, the radius of the ionic atmosphere at on the pH of the solvent system. Sometimes, this is carried
every charge location is reduced; the polymer chains out in the presence of an uncharged polymer or inside the
undergo shrinkage, resulting in a smaller configuration pores of a rigid membrane. As an extreme case, a separate
(Figure 4.1.29(a)) often called the polyelectrolyte effect gel phase can be formed. This gel phase (e.g. of crosslinked,
(Lowe and McCormick, 2001). On the other hand, if we partially hydrolyzed polyacrylamide) acts as a size-
have a polyzwitterion (i.e. the polymer chain has both selective extractive solvent removing water and smaller
anionic and cationic groups) such that the ratio of the solutes from an aqueous solution. At low pH, the gel
anionic to cationic groups is around 1, there will be attract- shrinks and rejects extra water (soaked in earlier) in a
ive electrostatic interactions between such groups, leading separate vessel (Cussler et al., 1984), just as in solvent
to a collapsed globular structure. If, however, salt (NaCl) is extraction and back extraction processes.
4.1 Two-phase equilibruim separation: closed vessel 243

Antibonding
Outer d orbital
(p*) orbital

- +
+

C
Bonding
(p) orbital

- + +
Metal

-
+ -

Outer s orbital

s component of electron donor/acceptor interaction

p component of electron donor/acceptor interaction

Figure 4.1.30. DewarChatt model of -complexation (Safarik and Eldridge, 1998). Reprinted, with permission, from Ind. Eng. Chem.
Res., 37(7), 25722581(1998), Figure 1. Copyright (1998) American Chemical Society.

4.1.9.3 Pi bonding/complexation glycerol solution, Kovvali et al. (2002); polyvinyl-alcohol-


based polymeric membrane, Ho and Dalrymple (1994);
Mixtures of olefins and paraffins having similar or the same
polymeric perfluorosulfonate membrane, Koval et al.
carbon numbers can be successfully separated by what
(1989)). By chemical absorption in a solvent, olefins have
is called pi bonding or -complexation. In such a process,
been separated successfully from paraffins: aqueous
the olefinparaffin mixture in a gaseous or liquid
AgNO3 solution for C2H4 absorption at 240 psia and 30
state is contacted with transition metals like Cu, Ag, etc.,
40  C (Keller et al., 1992); cuprous aluminum tetrachloride
in a solution or dispersed on a solid substrate having a
in an aromatic solvent (commercially used, Gutierrez et al.
high surface area. The liquid solution/solid substrate
(1978)); AgNO3 solution in water in a microporous
will contain Ag ions; in the case of Cu, cuprous salts are
membrane based absorber (Davis et al., 1993); cuprous
used to make Cu ions available. The bonds of an olefin
diketonate in -methylstyrene solvent (Ho et al., 1988).
molecule interact with the d orbitals of such a metal/ion;
Solid SiO2 sorbent on which Ag was spread showed con-
also, the metal ion forms -bonds with the carbon in the
siderable selectivity for butene over n-butane at 1 atm and
olefin. Collectively this type of metal ionolefin coordina-
70  C (Padin et al., 1999).
tion is identified as -complexation (Figure 4.1.30). Such
A number of different silver salts have been used for
complexations are highly selective for olefins; however,
complexation with olefins, e.g. AgNO3, AgBF4, etc. The
they are weak enough to be broken by low pressure or
system that has been studied in great detail is AgNO3 in
high temperature to release the olefin molecule bound to
water. The following complexation reactions have been
the transition metal ion.
suggested (Herberhold, 1974):
Olefins have been separated with high selectivity from
a mixture with paraffins in gas phase by permeation Ag olefin Ag olefin ; 4:1:81a
through a liquid membrane/polymeric membrane con-
2
taining AgNO3 (aqueous solution, Hughes et al. (1986); Ag olefin Ag Ag2 olefin ; 4:1:81b
244 Separation in a closed vessel

Ag olefin olefin Agolefin2  : 4:1:81c of an adsorbent has been analyzed by Perkins et al. (1997),
who have performed a model-independent thermo-
For a review of -complexation for olefinparaffin separ- dynamic analysis employing what is known as the
ations, see Safarik and Eldridge (1998). preferential interaction analysis. A general result of this
analysis relates the change in the capacity factor k0 il of
the protein species i 2 ( j 1, adsorbent), k 0 21 , to the
4.1.9.4 Hydrophobic interaction
variation in the molal salt concentration, m3, w , in the solu-
Hydrophobic interaction is utilized generally in the sepa- tion phase ( j 2):
ration of protein mixtures via hydrophobic interaction
ln k 0 21 n 1
 
chromatography. Protein molecules have a distribution of
m3, w ,
charged hydrophilic regions, as well as hydrophobic ln m3, w T , P, eq: g m1, w g
regions, on their surface. The hydrophobic regions arise 4:1:85
from the side chains of a variety of amino acids making up
the protein. The extent of such regions will be dependent where subscripts i 1, 2, 3 correspond to the solvent (e.g.
on the protein. Such regions will, in aqueous solutions, water), the protein and the salt, respectively; further, for
prefer to interact with other such regions in other protein the salts which are electrolytes, n is the total number of
molecules; alternatively, if there are adsorbent particles anions and cations per formula unit, m1,w is the molal
having hydrophobic chains, short or long, sticking out, the concentration of water (i 1) (55.5), 1 , and

hydrophobic patches on the protein molecule prefer to are, respectively, the number of water molecules, the
interact with such chains. The chains commonly used have number of cations and the number of anions released
a free benzyl group at the end or consist of a (CH2)n chain, during the binding process. The quantity g is defined as
where n varies from 3 to 10; such chains are bonded to the g lnm3, w =ln a T , P . Here a is the mean ionic
adsorbent particle via various chemistries. Interactions activity for electrolytes (see (3.3.119d)) and is given as
between such chains on the adsorbent surface and the n n 1=n
a m3, w n n , 4:1:86
hydrophobic patches on the protein surface lead to binding
of the protein to the hydrophobic adsorbent surface. where is the mean ionic activity coefficient and n and
If water molecules were to solvate the hydrophobic n are the number of cations and anions per unit of salt.
regions of the protein surface, they have to be highly For a few common electrolytes, e.g. NaCl, (NH4)2SO4,
ordered, requiring a strong decrease in entropy for the NaSCN, in hydrophobic interaction based processes, n is,
process represented as follows (Tanford, 1980): respectively, 2, 3 and 2. The constant g can be calculated
for each system and has a range of values (around 1.11.9
protein sH2 O protein  sH2 O; S0 ve: 4:1:82 in Perkins et al. (1997)) for a variety of systems. The values
This process does happen (so G0 is negative, correspond- of 1 can vary over a wide range (5500) depending on
ingly H0 is also negative; neglect here any interaction with the system.
the charged/hydrophilic regions of the protein). If hydro- Perkins et al. (1997) have integrated equation (4.1.85)
phobic chains are available on an adsorbent, the same for the system of ovalbumin and ammonium sulfate to
arguments are valid. If the hydrophobic patches on a pro- obtain
tein molecule react with the hydrophobic chains of the n 1
adsorbent, ln k 0 21 a lnm3, w m3, w :
g m1 , w g
protein  rH2 O adsorbent  sH2 O protein  adsorbent 4:1:87
0 0
r sH2 O; S ve; 4:1:83 From the experimental data for k 21 obtained against
m3, w , one can obtain values of a, =g and
the entropy increases and water molecules are released. If n 1 =m1, w g.
salt ions are available (AY) in solution, released water An alternative scenario is possible in the absence of
molecules are drawn away to hydrate the salt ions: these hydrophobic chains on the adsorbent/any adsorbent,
A Y r sH2 O A  r 1 H2 O Y  s1 H2 O; 4:1:84 namely hydrophobic patches in two neighboring protein
molecules are bound together; this leads to protein preci-
where (r1 s1) (r s). This will facilitate the forward pitation. This is the salting-out phenomenon for protein
reaction (4.1.83), enhancing the binding of the protein with solutions, wherein a high concentration of a salt, typically
the hydrophobic adsorbent. Thus, in hydrophobic inter- ammonium sulfate (1.53M), added to the protein solution,
action chromatography, high concentrations of salt are precipitates the protein molecules from the solution. Gener-
deliberately maintained to promote reaction (4.1.83). ally, the nature of the cation in the salt is not important.
The effect of a salt on the adsorption of hydrophobic However, the anions are all important. The precipitation
regions of a protein molecule on the hydrophobic patches ability of the anions follow in decreasing order:
4.2 Equilibrium separation in an external force field 245

PO34 > SO42 > acetate > Cl > NO3 > SCN :

surfactants. The system of interest contains two minerals,
hematite (Fe2O3) and quartz, which can be separated by
How does one reverse the binding between the hydropho- froth flotation; these two minerals are usual constituents of
bic patches in a protein with the hydrophobic chains on iron ores.
hydrophobic adsorbents? Detergents, organic solvents The primary step involves the use of surface-active
(ethylene glycol 50% v/v, i-propanol 30% v/v, acetonitrile), collectors, which displace water molecules from the sur-
lower temperature, low salt concentration, increased pH, face of the mineral; if the mineral surface is charged,
chaotropic salts, etc., have been used successfully to appropriate long-chain ionic surfactants, which can act as
desorb the proteins from hydrophobic adsorbents. If a counterion to the charged surface, are used. The long
hydrophobic adsorption was facilitated by high concentra- hydrophobic chains of such surfactants present a hydro-
tion of a salt, lower salt concentration may be employed. phobic surface. Surface-active collectors, which are chemi-
Chaotropic salts,6 such as iodide, lithium bromide, thio- sorbed at the mineralwater interface, can also create a
cyanate, at high concentrations, are often successful in hydrophobic surface. An additional approach employs
facilitating desorption since they are less polar and bind substances called depressants, which inhibit the attach-
water loosely. So water molecules hydrate protein surfaces. ment of gas bubbles to the minerals that should not
On the other hand, a kosmotropic salt such as (NH4)2SO4, undergo flotation.
has ions having high polarity, which bind water strongly; At pH 67, quartz particles have a negative charge on
therefore they will induce the exclusion of water molecules their surface, whereas hematite particles essentially have
from the protein surfaces, which will lead to protein pre- no charge. The addition of an alkylamine salt, which is
cipitation or adsorption on hydrophobic adsorbents. adsorbed on the quartz particle surface, leads to the flota-
Scopes (1994) has provided a more detailed account of tion of quartz particles. Commercially employed alterna-
such techniques for eluting proteins in hydrophobic inter- tive separation strategies include the use of starch as a
action chromatography. depressant for hematite; simultaneously, calcium ions
A recent analysis (Marmur, 2000) of the solubility of are used to activate surfaces of quartz particles, which are
nonpolar solutes in water suggests that such solutes prefer then floated using sodium oleate at pH 1112 (Fuerstenau
to aggregate rather than to be dispersed as single mol- and Herrera-Urbina, 1989). Another industrial approach
ecules. Such aggregation leads to a sufficient loss in employs flotation of hematite. At pH 24, the charges on
entropy, as opposed to a corresponding loss of entropy hematite surfaces are positive; an anionic alkyl sulfonate
via ordering of water molecules around hydrophobic collector is employed to float hematite. Since the quartz
patches/chains. This analysis is also valid for the dissol- surface is negatively charged at these pHs, the anionic
ution of water in nonpolar solvents. Therefore, molecular collector does not become adsorbed on the quartz surface.
aggregation is due to the more general solvophobic effect The key to any approach is knowing the electrical
rather than a hydrophobic effect. charge and potential on the surface of the mineral particle
in an aqueous suspension. The following four phenomena
4.1.10 Gassolid particleliquid system in contribute to the development of the surface charge:
mineral flotation specific adsorption of surface-active ions; preferential dis-
solution of lattice ions; dissociative adsorption of water
In Section 3.3.8, the principle of particle separation via molecules; isomorphous substitution of ions comprising
flotation in a gasliquid system was briefly identified. the mineral lattice (Fuerstenau and Herrera-Urbina, 1989).
Those particles whose surfaces are hydrophobic, and to
which gas/air bubbles can be attached, will float to the
surface of an aqueous suspension due to reduced density.
Those particles whose surfaces are hydrophilic and wetted 4.2 Equilibrium separation in a single phase
by the liquid (commonly, it is water) cannot become in an external force field
attached to gas/air bubbles and will not therefore float to Consider a liquid solution or a gaseous mixture in a closed
the surface. Here, following Fuerstenau and Herrera- vessel subjected to an external force field. We would like to
Urbina (1989), we will briefly illustrate systems where such know the change in composition in the solution or in the
separations are achieved by converting particular mineral gaseous mixture in the direction of the force field and
surfaces to a hydrophobic type via the adsorption of calculate the value of the separation achieved between
any two locations in the vessel. The external force fields
illustrated are centrifugal, electrical and gravitational. This
6 section also explores the separation achieved in the closed
Chaotropic anions create an increase in entropy resulting from
the disruption of water structure around these ions in solution. vessel subjected to an external force field when a property
This facilitates solubilization of hydrophobic proteins by water gradient, e.g. density gradient, pH gradient, etc., exists in
since many more water molecules are available. the single-phase system. Density gradient is important
246 Separation in a closed vessel

di jT dext
i : 4:2:1

For the centrifugal field, it is known that


dext
F ext r M i 2 rr i
r:

i
mole of i dr
r
Further, using relation (3.3.9),
r2
di RT dlnf^ij M i 2 r dr, 4:2:2

T

r1 where subscript j refers to the radial location r under con-


sideration. Integrating between two locations, r r1 (j 1)
and r r2 (j 2), where r2 > r1, leads to

2 2 2
ln f^i2 =f^i1 M i =RT r r : 4:2:3
2 2 1

If the gas mixture behaves as an ideal gas, then


w
f^i1 x i1 P 1 and f^i2 x i2 P 2 , 4:2:4
Figure 4.2.1. Closed hollow cylinder rotating at radian/s: closed where P1 and P2 are the total pressures at radial locations r
gas centrifuge. r1 and r r2. Result (4.2.3) can now be expressed as

x i2 P 2 2
 
when a centrifugal or gravitational force field is present; ln M i =RT r 22 r 21 : 4:2:5
a pH gradient is useful in the presence of an electrical force x i1 P 1 2
field. Finally, the role of a thermal gradient in separation is
A general relation between the gas mixture at any location j
briefly identified in the absence of any external force field.
(r r) and that at r r1 is
!
x ij P j rr 2
4.2.1 Centrifugal force field ln M i =RT r 2 r 21 : 4:2:6
x i1 P 1 2
Three types of mixtures/solutions are considered here: a
gaseous mixture, liquid solutions of low molecular weight The equilibrium separation factor 12 between species
species and high molecular weight species. We begin with 1 and 2 in a binary gas mixture between two locations
a gas mixture in a centrifuge; gas centrifuges have been j 1 (r r1) and j 2 (r r2) is easily obtained for ideal
used at large scale for the separation of uranium isotopes gas behavior:
present in a gaseous mixture of U235F6 and U238F6. Macro-
x 11 x 22 M 2 M 1 2 2 2
 
molecules and biological particles in solutions are rou- 12 exp r 2 r 1 : 4:2:7
tinely separated and studied in the laboratory using x 12 x 21 2RT
ultracentrifuges. Centrifuges are also extensively used in Two more results are needed to provide a reasonably
the chemical process industry to remove particulate matter complete picture of separation in a closed gas centrifuge.
from a liquid/gas (see Section 7.3.2). The first provides the mole fraction profile xij(r) of species i
at radius r (location j) along the centrifuge radius; the
4.2.1.1 Gas separation second describes the variation in total pressure. Using
relation (4.2.7) for two locations (r r and r1 0), and
A simple closed gas centrifuge (Figure 4.2.1) is a rotating species i 1,2, one obtains
hollow cylinder containing a gas or a gas mixture intro-
x 1j r x 1j 0 M 2 M 1 2 2
 
duced into the stationary hollow cylinder at time t 0.
exp r : 4:2:8
Within a short time,7 the gas mixture as well as the cylinder x 2j r x 2j 0 2RT
are assumed to start rotating at a constant angular velocity
Algebraic manipulation leads to
radian/s. Assume isothermal conditions. To determine
the conditions existing at equilibrium, employ the criterion x 1j 0expAr 2 =2
x 1j r , 4:2:9
(3.3.25) for equilibrium in the centrifugal field for species i: x 1j 0expAr 2 =2 1 1

where
 
7 A M 1 M 2 2 =RT : 4:2:10
Auvil and Wilkinson (1976) provide estimates of this time.
4.2 Equilibrium separation in an external force field 247

A. Charge composition, xif = 0.002


B. SO2-H2, xij = Mole fraction SO2 r2 w
0.235 r1
0.23 C. SO2-N2, xij = Mole fraction SO2
Mole fraction, xij x102

0.22 D. UF6 (235, 238 isotopes),


xij = Mole fraction U235 90 90
0.21 Cylindrical cell Cylindrical cell
D
0.20
A
0.19 C
0.18 B
0.17
0.0 0.1 0.2 0.3 0.4 0.5 0.6 0.7 0.8 0.9 1.0
Dimensionless radius, r / rwall w

Figure 4.2.2. Mole fraction profiles developed in a simple centri- Figure 4.2.3. Swing-bucket rotor with centrifugal tubes which
fuge with a peripheral speed of 150 m/s for the gas pairs: sulfur swing 90 upward during centrifugation.
dioxidehydrogen, sulfur dioxidenitrogen and UF6 (235, 238 iso-
topes) at 20  C. Reprinted, with permission, from Auvil and Wilk-
inson, AIChE J., 22, 564 (1976). Copyright [1976] American
velocity () of 4000 radian/s at 20  C. The value of 12 for a
Institute of Chemical Engineers (AIChE). mixture of uranium isotopes (U235F6 and U238F6) will be
only 1.0157 in such a centrifuge, whereas that for a SO2N2
mixture is 1.207. However, uranium isotopes are often
From relation (4.2.7), we get, for r1 0 and r2 r,
separated by high-speed centrifuges since other isotope
Ar 2 separation methods yield usually a lower 12. A gas centri-
 
12 exp : 4:2:11 fuge is never used for SO2N2 separation, however, since
2
other methods provide a far larger separation factor. The
Substituting this into the expression (4.2.9) for x1j(r), we mechanical problems encountered at these high speeds
obtain are also not inconsiderable.
x 1j 0 We will now provide the second result, namely the
x 1j r : 4:2:12 profile of the total gas pressure along the centrifuge radius.
x 1j 0 12 1x 1j 0
The partial pressure of any species is indicated by relation
By our convention, species 1 is lighter than species 2, (4.2.6); the total pressure is obtained by summing over all
making (M2 M1) positive. Therefore 12 is greater than 1 species at any location r (region j):
as long as r2 > r1; it also follows that x1j(r) < x1j(0). Lighter n
M i 2 r 2
X  
species concentrate near the center of the centrifuge and P j Pr P0x i1 0exp , 4:2:13

rr 2RT
heavier species concentrate near the wall (r rwall). Fur- i1

ther, the extent of this composition change is related to the where region j 1 corresponds to r 0, there are n
molecular weight difference (M 2 M1) since 12 increases number of species in the gas mixture and
exponentially with (M2 M1). The extent of separation, , n
for such a centrifuge is calculated in Problem 1.4.3, and the
X
x i1 P 1 P 1 Pr 0 P0: 4:2:14
result has been provided there. i1
The behavior of the mole fraction profile in a gas
4.2.1.2 Liquid separation
centrifuge along the centrifuge radius is illustrated in
Figure 4.2.2 for a number of binary gas mixtures (Auvil Liquid solutions subjected to a centrifugal field also
and Wilkinson, 1976). The simple gas centrifuge rotates undergo separation. Such separations, primarily employed
with a peripheral speed ( rwall) of 150 m/s at 20  C; in laboratories, are achieved by placing the solution in a
the gas initially fed into the centrifuge contained a small cylindrical cell mounted at the tip of an extremely
mole fraction of 0.002 of the species shown. Consider high-speed rotor (Figure 4.2.3) in an ultracentrifuge. When
systems B and C, both containing SO2. The mole fraction equilibrium is achieved in the centrifugal force field, cri-
of SO2 (1 x1j(r)) changes much more rapidly with r in terion (3.3.25) can be applied to species i:
the case of system B since (M2 M1) is much larger for a
SO2H2 than for a SO2N2 system (system C). Similarly, the dij T dext 2
ij M i r dr:

mole fraction variation with radius for U235F6 and U238F6 is


much less since (M2 M1) is only 3.
It is also of interest to know the magnitudes of 12 For the solution, dij T is expressed as
achievable in such gas centrifuges. Consider a gas centri-

dij T R T d lnaij V ij dP M i 2 r dr:

fuge of radius 4 cm rotating at an extremely high angular
248 Separation in a closed vessel

Using the assumption that the pressure gradient may be concentration from one radial location to another is much
determined from the rotation of the solvent only, we have smaller due to the small value of Mi, since the magnitude
obtained the following result (see relation (3.1.52)): of f1 V ij =M i M s =V sj g remains comparable. In fact, the
"   # higher the molecular weight, the larger the separation.
V ij Ms Some of the more common applications of ultracentri-
V ij dP M i 2 r dr M i 2 r 1 :
Mi Vs fuges, therefore, involve large macromolecules, viruses,
cell fragments for preparative scale separations, etc. In
Therefore,
" centrifuges used for preparative applications, the liquid
#
V ij Ms solution is mounted in the rotor. After centrifugation, a
 
RT dlnaij M i 1 2 r dr:
Mi Vs pellet is obtained near the radius r rwall, containing the
particles, large macromolecules, cells, etc., separated from
Integrating between two radial locations r1 and r2 (> r1), we the clear supernatant solvent.
obtain We will now focus on the velocity of such types of cells/
( ) particles/macromolecules during centrifugal separation,
ai2 Mi V ij Ms
   
exp 1 2 r 22 r 21 : 4:2:15a the time required to form the pellet and the relative ease
ai1 2RT Mi Vs
with which different biologically relevant macromolecules/
cells, etc., may be pelleted. The analysis of the behavior of
For an ideal solution, aij xij; therefore ai2 =ai1 x i2 =x i1 .
larger centrifuges that are open with flow in and out are
Further, for dilute solutions, x i2 =x i1 ci2 =ci1 . This
considered in Section 7.3.2 as well as in Section 6.3.1.3.
last result allows us to relate the concentration of a species
For a spherical particle of radius rp, density p and mass
at two radial locations at equilibrium:
( ) mp in a liquid medium of density t, the net external force
C i2 Mi V ij Ms acting on the particle (from equation (3.1.59)) in the radial
    
2 2 2
exp 1 r 2 r 1 :
C i1 2RT Mi Vs direction (without any gravitational force in that direction) is
mp r2 1t =p r if the angular velocity of the particles is
4:2:15b
radian/s. The drag force experienced by this particle from
Consider a dilute aqueous solution of ovalbumin (Mi ~ 45 000) the liquid resisting this radial particle motion according to
at 24  C; the density of ovalbumin is 1.34 g/cm3 M i =V ij . Stokes law is 6rpUpr where Upr, is the radially outward
For a dilute solution, M s =V s 1g=cm3 . From the expres- particle velocity. Reformulating the basic equation of motion
sions given above for (ai2/ai1) and (Ci2/Ci1), it is clear that (3.1.60) for the particle in the r-direction, we have
ai2 > ai1, Ci2 > Ci1 and xi2 > xi1. At equilibrium, there
d2 r dU pr
develops a continuous solute concentration profile along mp r mp r mp r 2 1 t =p r 6r p U pr r:
the cell length. Solute concentrates at the largest radial dt 2 dt
4:2:16a
location in the cell at the cell wall, and the solution nearer
the center becomes depleted in ovalbumin. Such a profile At steady state the particle radial acceleration is reduced to zero
does not change with time at equilibrium. At any radial and the terminal radial particle velocity achieved, Uprt, is
location, the concentration gradient and the pressure gra-
dient (pressure increases also as the radius increases in the 3 2 2
dr 4 r p p r 1 t =p 2 r p p t 2

U prt r;
centrifuge) force ovalbumin molecules toward the center dt 3 6r p 9
terminal velocity
(r 0) to balance the centrifugal force driving the molecule
away from the center. There is no net radial force on U prt sp 2 r, 4:2:16b
ovalbumin molecules at any radial location once equilib-
where sp is called the sedimentation coefficient of the par-
rium separation profile is established.
ticle of size rp (correspondingly sp1 and sp2 for particles of
Ultracentrifuges used in laboratories for solutions
sizes rp1 and rp2). If 2 r may be considered as the centrifu-
employ very high speeds (as much as 60 000 rpm). Meas-
gal force per unit effective mass8 (i.e. mp 1t =p 1 of
urement of the concentration profile (e.g. (4.2.15b)) is
the particle under a given centrifuge condition, then sp is
achieved optically; the refractive index gradient is meas-
an effective indicator of the radial sedimentation particle
ured (it being proportional to the concentration gradient).
terminal velocity per unit of centrifugal force. Its units
For an introduction, see Hsu (1981).
are svedbergs, 1 svedberg having the value 1013 second
The difference in concentration between r 0 and r
(Svedberg and Pederson, 1940). The quantity 2 r is often
rwall for species like ovalbumin in an ultracentrifuge is
expressed as so many times (e.g. n times) gravity, ng,
substantial. If, however, we had subjected a solution of
where g is the acceleration due to gravity.
two low molecular weight liquids, e.g. water and ethanol,
to the centrifugal field in an ultracentrifuge, the difference
(Ci2 (r rwall) Ci1(r 0)) would be very small. For the
8
same values of , T, r2, r1, the change in Magnitude of the centrifugal field strength.
4.2 Equilibrium separation in an external force field 249

Ivory et al. (1995) have collated in svedberg units If the value of is known, then U prt may be calculated for
the sedimentation coefficients of a variety of biologically each individual viral location.
relevant particles in aqueous systems: yeast and red (b) To estimate the time needed for the virus to settle, we
blood cells 106 > sp > 105 ; mitochondria and bacteria employ equation (4.2.16c):
105 > sp > 104 ; viruses and phages 103 > sp > 102 ; r r wall
t wall ln :
proteins (e.g. serum albumin) sp < 10. The higher the sp 2 r r start
value of sp, the easier it is for the particles to settle and the Since is not explicitly given here, we will employ 2r
shorter the particle settling time in a given centrifuge for a 26 000 g and r (rwall rstart)/2.
given angular velocity, . Consider now the time required For particles starting at 15 mm from the base, rstart 137
for such a particle to settle in the centrifuge, i.e. reach the 15 122 mm. Therefore r 129.5 mm and
centrifuge wall, rwall. Employ equation (4.2.16b); integrate 12:95 cm 137
between the starting radius rstart anywhere in the centrifuge t wall cm ln 122
2
10  10 13
s  26 000  980 2
and rwall: s
r wall t wall
12:95  104

12:95  1145
dr dr r wall

2  0:1145 s s
s p 2
r ) sp dt sp 2 t wall ln ; 2:6  0:98 2:6  0:98
dt r r start
settling velocity r start o
5734 s 95:5 minutes:
1 r wall r r wall
t wall ln ln : 4:2:16c
sp 2 r start sp 2 r r start For particles starting at 50 mm away from the base of the
tube, rstart 137 50 87 mm. Therefore r 112 mm
Sometimes, this time is replaced by tpellet, the time needed (averaged) and
to form the pellet in an ultracentrifuge. 11:2 cm 137
t wall cm ln 87
Example 4.2.1 Harvesting time for a virus in an ultracentri- 10
11
s  26 000  980 2
s
fuge. Lotfian et al. (2003) have investigated the centrifugal
recovery of a disabled herpes simplex virus type-1 (HSV-1) 11:2  104  2:303
 0:197 s
vector, potentially useful for gene therapy, from a culture 2:6  0:98
medium containing recombinant cells of a suitable kind.
The extracellular virus is obtained first in the supernatant 19941 s 332 minutes:
of the broth centrifuged at a low value of 2r 1600 g. Since the sedimentation coefficient sp for proteins is around
This supernatant is then subjected to ultracentrifugation 110 svedberg, the settling time needed for proteins in
using a swing-bucket rotor and 15 ml volume centrifuge Example 4.2.1 would be an order of magnitude larger. Ultra-
tubes. Consider operation at 2 r 26000g. The radius of centrifuges are therefore not used for protein separation.
the base of the centrifuge tube from the axis of rotation is So far we have focused on liquid separation or particle
13.7 cm. separation where the solvent density was essentially
(a) Calculate the settling velocity of the virus at the following assumed constant throughout the centrifuge. This condition
two locations in the ultracentrifuge tube: 15 mm and 50 did not pose any problem for separating a given cell or
mm away from the base of the tube. macromolecular species from the solvent; the cells or macro-
(b) Estimate the time needed for the virus to deposit if the molecular species are obtained as a pellet at the centrifuge
starting locations are identified in part (a). tube base. A different strategy has to be pursued if it is
necessary to separate different cells or macromolecular
You are given the sp for virus 102 svedberg. species from one another. We focus on that in Section 4.2.1.3.
The mathematical expressions used so far for liquid
Solution (a) The settling velocity of a particle having
systems subjected to a centrifugal field have assumed that
a sedimentation coefficient sp at a radial location r is
the solvent density M s =V sj was essentially constant across
given by
the centrifuge. In general, it will vary in the radial direction
U prt sp 2 r cm=s; due to the variation of pressure which influences V sj . It is
sp 102 svedberg 102  1013 1011 s; particularly true when there is at least one solute species
2 cm present in the system at a substantial concentration. Under
r 26 000 g 26 000  980 2 ;
s such a condition, we use expression (3.1.58) for centrifugal
equilibrium and expression (3.3.17) for dij jT , Pj at any loca-
Therefore, tion. This leads to
cm cm
n1 
U prt 1011 s  26 000  980 2:6  0:98  104 V ij ij
  
2 r dr
X
s2 s dij M i 1 tj dx kj

T , Pj Mi k1
x kj T , Pj , x l , l6k
2:6  0:98  104 m
4:2:17
102 s
when there are a number of solute species present at
2:55  106 m=s: substantial concentrations. Consider a system of solvent
250 Separation in a closed vessel

(i s) and two solutes i 1 and i 2. The particular forms


(a)
of the above relations are as follows.

solution density

2
1j 1j
i1: M 1 tj V 1j r dr dx 1j dx 2j ;
x 1j T ,Pj ,x 2 ,x s x 2j T ,Pj ,x 1 ,x s

CsCl-based


2
2j 1j
i2: M 2 tj V 2j r dr dx 1j dx 2j :
x 1j T ,Pj ,x 2 ,xs x 2j T ,Pj ,x 1 ,x s

4:2:18
r0|species 2
The use of such equations will be illustrated in the following
for a special case.
r0|species 3

4.2.1.3 Isopycnic sedimentation


Just as a particle denser than a liquid falls through the Distance from rotor center
liquid in the direction of the gravitational force, similarly
a particle or species of density i M i =V ij will be thrown
radially outward in the direction of the centrifugal force if Figure 4.2.4A. Equilibrium density gradient in a centrifuge due to
i > t , the solution density. This phenomenon is identi- CsCl solution.
fied in analytical/preparative chemistry as sedimentation.
When t < i , the centrifugal force is radially outward. forms a small band around a center point where relation
However, if t > i , the centrifugal force on species i will (4.2.19) is satisfied. Determinations of the width of such a
be radially inward. When t i , species i or particle i band and the separation between two neighboring bands
does not experience any radial centrifugal force (outward are needed to determine the usefulness of isopycnic
or inward); therefore its radial location does not change sedimentation.
in the centrifuge with time; it no longer undergoes Following Meselson et al. (1957), we consider the
sedimentation. separation of different macromolecules in an aqueous
These phenomena provide the basis for separating a solution of CsCl. For the purpose of analysis, one macro-
multicomponent mixture in a liquid centrifuge provided a molecular species, i 2, only needs to be considered,
radial density gradient is created in the centrifuge to start where i 1 represents CsCl and i s is water. To simplify
with. Consider Figure 4.2.4A illustrating a density gradient matters, the macromolecular species may be represented
established using a low molecular weight salt such as CsCl as a macromolecular electrolyte P (Cs)n, where P is the
in water in the centrifuge. Into such a solution, where t is a macromolecular backbone (e.g. DNA (deoxyribonucleic
function of r, is introduced a small amount of a mixture of acid)). We will soon find out the utility of such an
different macromolecular species. Assume that the small assumption.
amount of sample will not affect the density gradient estab- The centrifugal equilibrium of the macromolecular
lished in the CsCl solution. Each macromolecular species i species i 2 in an aqueous solution of CsCl (i 1) is
will establish its own centrifugal equilibrium and will move governed by equation (4.2.18) for i 2:
to the radial location r, where
2
2j 2j
Mi M 2 tj V 2j r dr dx 1j dx 2j :
tj r ; 4:2:19

x 1j T , Pj , x 2 , x s x 2j T , Pj , x 1 , x s
V ij

i.e. the solution density is equal to the species density The radial dependence of the CsCl solution density around
(which is also the inverse of the partial specific volume of the location r r0, where
species i). As long as the value of (M i =V ij ) is different for tj r 0 M 2 =V 2j , 4:2:20
each ith species, each ith species will be concentrated at
different radial locations provided the range of tj(r) in may be described by a Taylor series expression:
the centrifuge accommodates the range of the effective
macromolecular density (M i =V ij ). dtj
tj r tj r 0 r r 0 : 4:2:21
The separation of different species in a centrifugal dr r0
force field (the imposed primary force field) having a radial
density gradient (secondary physical property based field) There is an implicit assumption here that the macro-
is identified as isopycnic sedimentation or density-gradient molecular band has a narrow width around the location
sedimentation (Giddings and Dahlgren, 1971). Each species r r0. For ideal solutions of species 1 and 2, we have
4.2 Equilibrium separation in an external force field 251

Since the variation of CsCl concentration over the band-



2j
dx 2j RT d ln C 2j : width of polymeric species 2 is small,
x 2j T , Pj , x 1 , xs
dC 1j =drr 0 r r 0
 
Further, we may define a dimensionless factor b, the bind- ln 1 r r 0 dC 1j =drr0 :
C 1j r 0 C 1j r 0
ing coefficient (Hsu, 1981), to be
 
x 1 Further, the bandwidth is small: jrr 0 j << r 0 .
b : 4:2:22 The concentration profile of species 2 may now be
x 2 P, T , 1
expressed as
Additionally, by Maxwells relation for any extensive quan- "
V 2j 2 r 0 dtj 2 br r 0 dC 1j

tity, e.g. Gibbs free energy Gtj (Pitzer and Brewer, 1961), C 2j rC 2j r 0 exp r r 0
2RT dr r 0 C 1j r 0 dr r0


2j 1j Gtj ! #
, 4:2:23 r r 0 tj r 0 V 2j
1 M 2 2 r 0

x 1j T , Pj , x 2 , x s x 2j T , Pj , x 1 , xs x 1j x 2j T , Pj , x s
RT M2
which leads to
or

2j 1j
r r 0 2 Ar r 0 A2
" #
dx 1j dx 1j bRT d ln C 1j A2

x 1j T , Pj , x 2 , x s

x 2j T , Pj , x 1 , xs C 2j r C 2j r 0 exp ,
2 22 22 2 22 2 22
4:2:24
where
for an ideal solution. Here, b is the effective number of
cesium counterions which are to be moved along with the RT M2
22 , V 2j 4:2:28
charged polymer molecule P(Cs)nb to maintain electo- dtj 2j
V 2j 2 r 0
dr r0

neutrality (so far we have gone about this as if we were
dealing with nonelectrolytes). The governing equation now and
becomes, for ideal solutions, !
tj r 0 V 2j
( ) dC 1j M2 1
dtj bRT M2
M 2 tj r 0 V 2j V 2j r r 0 2 r dr dr r0

dr r0 A : 4:2:29
dtj
dtj
bRT d ln C 1j RT d ln C 2j : 4:2:25 V 2j 2 r 0 C 1j r 0 V 2j
dr r0 dr r 0

Just as the radial dependence of the CsCl solution density Choosing A 0, we get a simple Gaussian profile of the
around r r0 was described by a Taylor series expansion macromolecular solute 2:
(4.2.21), similarly the C1j profile around r r0 may be
r r 0 2
" #
described by C 2j r C 2j r 0 exp 4:2:30
2 22
dC 1j
C 1j r C 1j r 0 r r 0 : 4:2:26 in a band (created by diffusion and balanced by centrifugal
dr r0
focusing) centered around r0 and having a standard deviation
Introducing this into equation (4.2.25), we get 2; the standard deviation is inversely proportional to the
square root of the molecular weight of the macromolecule.
h i dtj 2
M 2 tj r 0 V 2j 2 r dr V 2j rr r 0 dr The choice of A 0 implies
dr r0
dC 1j
2 3
" # bRT
dC 1j M2 6 61 dr r 0 7

bRTd ln C 1j r 0 r r 0 RTd ln C 2j : tj r 0 7: 4:2:31
dr r 0 V 2j 4 M 2 2 r 0 C 1j r 0 5

We integrate from r0 to r to obtain


For the system of DNA (M2 14  106) in an aqueous CsCl
solution, it has been suggested that, if the quantity within the
2 3
C 2j r dC 1j =drr0
ln b ln41 r r 0 5 brackets is replaced by 1, the error is less than 10% (Meselson
C 2j r 0 C 1j r 0
et al., 1957); the assumption A 0 therefore does not lead to
h i r 2 r 2 much error in depicting C2j (r) as a Gaussian profile.
M 2 tj r 0 V 2j 2 0
An illustration of such a Gaussian profile is provided in
2RT
Figure 4.2.4B for a CsDNA salt of molecular weight 18  106
12 0
obtained from the DNA of bacteriophage T4r introduced into
0 1
dtj 2 r r 0 r r r

0 0
V 2j @ A @ A: 4:2:27 the centrifuge rotating at 27 690 rpm in a 7.7 molal CsCl
dr r0 RT 3 2
solution at pH 8.4. The mean of the distribution
252 Separation in a closed vessel

optical systems. The cells have quartz windows (e.g.) and


(b)
the schlieren system, UV absorption, etc., are used to
0.5 mm
Scale : measure the profiles (Hsu, 1981).
Calculated
Gaussian If there is another macromolecular species, i 3, present
DNA concentration

in the solution in dilute amounts such that C1j(r) is unaffected,


Observed DNA a similar analysis can be carried out and a Gaussian distribu-
distribution tion of species 3 obtained with the band center located at a
radius r0jspecies3, where the solution density is given by
tj r 0 jspecies 3 M 3 =V 3j : 4:2:32

The corresponding standard deviation, 3, is obtained from


RT
23 : 4:2:33
Distance from rotor center
dtj
V 3j 2 r 0 j

species 3 dr r 0 jspecies 3
Figure 4.2.4B. The equilibrium distribution of DNA from bacterio-
Figure 4.2.4A provides the profiles for species 2 and 3.
phage T4. An aliquot of osmotically shocked T4r containing 3 g of
DNA was centrifuged at 27 690 rpm for 80 hours in 7.7 molal cesium
Similar results may be developed for other macromolecular
chloride at pH 8.4. Evidence for the attainment of equilibrium was species present in small amounts. Multicomponent separa-
provided by the essential identity of the final band shapes, whether the tion will be achieved as long as the partial specific density
DNA was initially distributed uniformly in the cell or in an extremely of each solute i (M i =V ij ) is sufficiently different from those
tight band. At equilibrium the observed DNA distribution does not of the two solutes with the closest molecular weights.
depart appreciably from Gaussian form, indicating essentially uni- Following Meselson and Stahl (1958), we can calculate
form molecular weight and effective density. The mean of the distri- the resolution, Rs, between two different types of DNA in a
bution corresponds to an effective density of 1.70, and the standard CsCl solution being centrifuged using the expression of
deviation corresponds to a molecular weight for the CsDNA salt of 18 developed above and compare it with that measured by
 106. Assuming the base composition for T4 reported by G. R. Wyatt Meselson and Stahl. The two different species are isotopic-
and S. S. Cohen (Nature, 170, 1072 (1952)) and the glucose content ally labeled (N15) and unlabeled DNA (N14) of E. coli; their
reported by R. L. Sinsheimer (Proc. Natl. Acad. Sci., 42, 502 (1956)), density difference is 0.014 g/cm3.
this corresponds to a molecular weight of 14  106 for the sodium The molecular weight of the unlabeled DNA in the
deoxyribonucleate. The density gradient is essentially constant over CsCl solution was estimated to be 9.4  106. The density
the band and is 0.046 g/cm4 The concentration of DNA at the of the unlabeled DNA is 1.71 g/cm3. The value of 2r0 in
maximum is 20 g/ml. Figure 2, Meselson et al. (1957), reprinted, the Meselson and Stahl experiment was 140 000 g-cm/s2
with permission, from Matthew Meselson, December 5, 2010.
(140 000  980 cm/s2), corresponding to 44 770 rpm in
their ultracentrifuge. The density gradient (dtj/dr) was
corresponds to an effective density of 1.7 and is located
estimated to be 0.08 g/cm4. The resolution between two
around 6.6 cm from the rotor center. The density gradient
species located at r0j2 and r0j3 is
over the band was essentially constant at a value of 0.046 g/
cm4. We can calculate the value of 2 from equation (4.2.28)

r 0 2 r 0 3
Rs , 4:2:34
and compare it with the observed 22 0.5 mm for T 298 K: 4

RT M 2 18  106 where is an acceptable average for both bands that can


22 ; V 2j cm3 =gmol; be determined from the expression for one of the species.
V 2j 2 r 0 dtj =drr 0 2j 1:70
The resolution expression may be rewritten using
27 690  2
923 rad=s;
60 r 0 2 r 0 3 r 0 2 3

4:2:35
dtj =dr
9
r 0 6:6cm; RT 25  10 erg=gmol;
as
where 1 erg 1 dyne-cm 1 g-cm2/s2. So we have
0 11=2 0
2
11=2
2 3 @ V 2j r 0 2 A 0:014 g
B C Rs
B 25109 g-cm2 =s2 -gmol C 4 RTdtj =dr 4 cm3
2 B
B18106 cm3 2
C
C
@ 2 rad g A
0
cm3
11=2
923 2
6:6cm0:046 4 9:4  10 6
=1:71  140 000  980 cm=s 2
1:7 gmol s cm B
B gmol C
C
0:03 cm
B C
2
g-cm
B C
9 4
25  10 0:08 g=cm
B C
)2 2 0:6 mm; 
gmol-s2
@ A

which is reasonably close to the observed value. Analytical


0:014
ultracentrifuges, where such studies are made, rely on  610:8 2:13:
4
4.2 Equilibrium separation in an external force field 253

The measured value of Rs reported by Meselson and Stahl are low molecular weight amphoteric substances (e.g. poly-
(1958) is 1.5. aminopolycarboxylic acids, etc.). They have a net positive
The osmotic pressure of a concentrated solution of a charge near the anode (kept in a strong acid) and a
low molecular weight salt like CsCl (or a solute like net negative charge near the cathode (kept in a strong basic
sucrose, etc.) employed to create a density gradient is solution). When a voltage is applied to the cell,
considerable. If isopycnic sedimentation is to be employed each ampholyte species migrates to a location where the
for separating a variety of complex physiological cell mix- isoelectric point of the ampholyte is equal to the local pH.
tures in a centrifuge, density gradients created by CsCl or The pH gradient is established by each ampholyte species
sucrose are unsuitable due to their high osmotic pressure, being at different locations, acidic ones near the anode and
which may kill or deform the cells. To maintain biological basic ones near the cathode. If an ampholyte species is
integrity and viability of the cells under consideration, it is displaced from its IEP, it immediately acquires a charge
necessary that the material used to create the density and is forced back to its position of equilibrium in the
gradient is isosmotic throughout the gradient. There are electrical field. The pH gradient (Figure 4.2.5(a)) is such that
many other requirements (e.g. low viscosity, chemically the anode or the positive electrode has the most acidic
inert, does not dissolve/penetrate biological membranes, solution and the lowest pH, whereas the cathode or the
etc. (see Van Vlasselaer et al. (1998)). Over the years, a negative electrode has the most basic solution and the
number of materials have been used to develop such gra- highest pH. Proteins have a net positive charge at pHs below

dients. These include: Percoll (colloidal silica covalently
coated with polyvinylpyrrolidone); silanized colloidal
their pI and a net negative charge at pHs above their pI
(Figure 4.2.5(c)). If a protein possessing a particular

silica; Ficoll ; dextran; methylcellulose, etc. The reader
should refer to Catsimpoolas (1977) and Recktenwald and
pI is introduced at a location where pH < pI, it will be
positively charged and would therefore migrate toward the
Radbruch (1998) for more details. cathode. If it is introduced at a location where pH > pI, it will
be negatively charged and would therefore migrate toward
the anode. Thus, regardless of the point of introduction in
4.2.2 Electrical force field the pH profile, the proteins are forced to move to the regions
having their own pIs, where they are not subjected to any
4.2.2.1 Isoelectric focusing
force and therefore have zero migration velocity.
If an electrical field exists in a closed vessel containing an Although a particular protein species is thus focused to
aqueous solution having some charged species, then the the location z z0 by the above mechanism, a protein
charged species will move toward the positively or nega- band develops around z z0 due to molecular diffusion
tively charged electrode (as the case may be) until the (Figure 4.2.5(d)). Since this band represents the equilib-
electrode is reached. There is no room for equilibrium rium profile, there is no net force acting on the particular
separation as such. Suppose, however, there exists a pH protein species anywhere in the band. Consider now any
gradient in the solution and the charge on the species to be location z in the band where jzz 0 j  0 for protein species
separated depends on the local pH, as, for example, in i. Using criterion (3.3.25)
proteins. It is known that, at a particular pH, the
d ij T
dij ext
Z i F d:

isoelectric pH, called pI, a particular protein does not have
any net charge. (For example, the pI for ovalbumin (mol. Now,
wt. 45 000) at 25  C is 4.5, that for bovine serum albumin
(mol. wt. 67 000) at 25  C is 4.9, whereas the pI for
d
d ij T RTd ln aij Z i F dz Z i F E dz, 4:2:36
conalbumin (mol. wt. 86 000) at 25 C is 5.9 (Andrews, dz
1981)). Such pI values vary over a wide range as wide as where E is the strength of the uniform electrical force field.
1 (pepsin, mol. wt. 33 000) to 10 (lysozyme, mol. wt. The value of the electrical charge Zi (ve or ve) of a
14 000). During the migration of the charged protein protein molecule of species i depends on its location in
through the pH gradient, the protein will cross the region the band, i.e. z
z0 z0 , the distance from the band center
where the pH is equal to its pI (Figure 4.2.5); the net charge where Zi is zero due to pH pI. Since the band is very thin,
on the protein will become zero at this location and so will one can assume that Zi changes linearly with distance z0
its migration velocity. Proteins having different pIs will be from the band center in the following way (Giddings and
located thereby at different locations having different pHs. Dahlgren, 1971):
These locations will become equilibrium positions for dif-
ferent protein species having different pIs. A mixture of dZ i
Zi z0 4:2:37a
proteins can therefore be separated. This technique is dz zz0
identified as isoelectric focusing (IEF).
The pH gradient is established between two electrodes !
dZ i dpH
 
using an aqueous solution of a mixture of ampholytes z0 : 4:2:37b
dpH dz zz0

having molecular weights between 300 and 600. Ampholytes
254 Separation in a closed vessel

(a)
Concentration

pH
z0|1 z
z0|2
z0|3

(b) anode cathode


+ -
z

(c) +

pI pH
Zi

-
(d)
Diffusive flux Diffusive flux
Electrical force Electrical force

z
pI
z0|1

(e)
z=0 z=0 z=0
+ - + - +
t=0 t = t1 t1 < t 2 = t

Figure 4.2.5. (a) The pH gradient along a cell in isoelectric focusing and the concentration profiles of three proteins having three different
pIs. (b) Locations of the electrodes and the bands in the cell. (c) Variation of the net charge Zi of a given protein with pH. (d) Different
fluxes and forces on protein molecules having a particular pI at any z around z0. (e) Zone electrophoresis in a cell; individual solute zone
locations at two different times, t1 and t2, t2 > t1 > 0.

We can assume both variations in the above expression to


0 10 !
z z0
dZ i A@dpH
E
be linear, independent of z around z0; this is equivalent to a dlnaij @ F z0 dz0;
z0 0 dpH dz zz0 RT
Taylors series expansion
8 9
aij jz < k 0 z 02 = C j
dZ i dZ i ij z
z0 , exp i ,

Z i Z i z0 z
z0 0 4:2:38a
aij jz0 2 ; C ij jz0

dz zz0 dz zz0 :
4:2:37c where
where the first term is zero. !
dZ i dpH E
 
Equation (4.2.36) may now be integrated, after incorp- k 0i F : 4:2:38b
dpH dz zz0 RT

orating the above expression for Zi:
4.2 Equilibrium separation in an external force field 255

The protein concentration profile (Figure 4.2.5(a)) around electrical field strength E is 25 volts/cm, the value of 40
z z0 is Gaussian in nature, having a standard deviation i at 4  C is
defined by
joule pH >1=2
8   9
1:987  4:2  277  0:05
i 1=k 0i 1=2 :
>
gmol cm =
> >
4:2:39a <
4 0 4
coulomb volt g
equiv: >
Knowing that the diffusion coefficient of species i, Di, is :96 485  25 9
>
> >
g
equiv: cm pH
gmol
;
defined as (RT/fid), we can express i as follows:
0 11=2 40:00000521=2 9:12  103 102 pH:

Di f i d =F E Thus, if two neighboring protein pIs differ by 102 pH units,


i 1=k 0i 1=2 B
B C
C : 4:2:39b they may be effectively separated by isoelectric focusing.
@ dZ i dpH A
Isoelectric focusing can be carried out in commercially
dpH dz zz0

available columns having as much as 110 ml volume. For
This result could have been obtained directly by using the analytical applications, they are carried out in capillaries
following balance equation at any location z in the concen- having one dimension as small as 1 mm. To prevent con-
tration profile: vection currents in larger columns, the solution in the

diffusive flux of
 column also has a density gradient developed by having
protein broadening band different sucrose concentrations along the column height,
amongst other things.
flux due to electrical force
 
: 4:2:40
compressing band toward z z 0
4.2.2.2 Other electrophoresis techniques
The extent of separation between two contiguous protein
bands having centers located at z0j1 for species 1 and z0j2 for There are three other electrophoresis techniques carried
species 2 may be determined by using the resolution, Rs: out in a closed device without any bulk flow in or out: zone
electrophoresis (ZE), moving boundary electrophoresis
z 0 1 z0 2 (MBE) and isotachophoresis (ITP). These are not true
Rs : 4:2:41
2 1 2 equilibrium techniques. We will describe very briefly the
basics of the zone electrophoresis technique here; all three
Since the pIs of species 1 and 2 are located, respectively,
techniques are suitable for laboratory and preparative
at z0j1 and z0j2, one may approximate z0j1 z0j2 for two
applications. For isotachophoresis, see Everaerts et al.
contiguous bands as
(1977) and Andrews (1981).
pI pI 2 We consider zone electrophoresis first. In Figure 4.2.5(e),
z 0 1 z 0 2 z 0  1

: 4:2:42
dpH=dz we show the electrodes in a closed cell. A thin band of
various ions present in a solute mixture is introduced at
Further, assuming 1 2 , one may use of any one of the a location z 0 at time t 0. In the presence of the
two species given by (4.2.39b) to obtain electrical field, different charged solute species move at
different rates; the anionic solutes move toward the posi-
11=2
tively charged electrode, the anode, while the cationic

dZ i
0
FE
pIj1 pIj2 B dpH C solutes move toward the negatively charged electrode,
Rs B C : 4:2:43
4 d dpH the cathode. Neutral species do not move. At another time
@ A
Di f i
dz t, different charged species will have moved to different
locations and are therefore separated from one another (as
According to this result, a higher resolution is achieved if Di
shown in Figure 4.2.5(e) for four charged species and one
is low (bandwidth is smaller), the pH gradient is not very
uncharged species). However, unlike that in IEF (isoelec-
steep, the strength of the uniform electrical force field is
tric focusing), the location of each charged solute where a
high and the pIs of the contiguous species are substantially
thin solute band is formed will change with time as they
different. It is now possible to make an estimate of how far
move toward the respective electrode. Therefore any sep-
apart the pIs of two proteins have to be to achieve a reason-
aration, purification or identification has to be carried out
able resolution, say 1: pIj1 pIj2 40 . We can calculate the
on a time-dependent basis in cells that are usually several
value of 0 in pH units from expression (4.2.43):
centimeters in length. We have actually studied such a

RT dpH=dz
1=2 separation in general in Sections 3.2.1 and 3.2.2 without
0 : 4:2:44 being specific about the driving force: we introduced a thin
F E dZ i =dpH
slab of a mixture of species at one location in a closed
Following Giddings and Dahlgren (1971), consider vessel and subjected the mixture to an external force in
ovalbumin, whose pI 4.6. The value of (dZi/d(pH)) is the z-direction. When a very thick sample band is utilized
9 at the pI. If the pH gradient is 0.05 pH/cm and the containing multiple ions of one particular charge sign,
256 Separation in a closed vessel

along with an electrolyte having the same ionic charge but applied to study the isotopic ratios of atmosphere gases
a higher mobility than all the ions to be separated, the trapped in polar ice caps where the heavier isotope are
technique of moving boundary electrophoresis is applied enriched relative to the free atmosphere (Craig et al., 1988;
(Everaerts et al., 1977). Craig and Wiens, 1996). Sage (1965) has provided a more
rigorous procedure for calculating the change of compos-
ition in an isothermal vertical column of gas mixture in a
4.2.3 Gravitational force field
gravitational field. Sage (1965) has also considered a liquid
For separation in a single-phase, the gravitational force mixture of hydrocarbons and illustrated the change in
field is rather weak compared to the strength of a centrifu- composition over a height of 010 000 ft.
gal or electrical force field. Still, some amount of separation
is achieved in the gravitational field. Employing the criter-
ion (3.3.25) for equilibrium for species i, namely 4.2.3.1 Batch sedimentation

d i T dext
Gravitational force is much more often utilized in separat-
i ,
ing a multiphase mixture, in particular a suspension of
where ext
i M i gz is the gravitational potential of one particles or drops in a liquid. Consider a stationary vessel
mole of species i at a height z from the origin, we get containing a suspension of particles in a liquid (Figure
4.2.6(a)) whose top level is at a height z HL from the vessel
d i T M i g dz
bottom. Assume that at time t 0 the particles are uni-
for a gaseous system. Using the relationship between i formly distributed throughout the suspension. However,
and the fugacity of species i, f^ij , in the region j of the for time t > 0, if the particle density p is larger than that
mixture, we get of the liquid f t , the particles will fall toward the
bottom of the vessel. As pointed out in Section 3.1.3.1,
d i T RT d ln f^ij M i g dz:

4:2:45 although there will be particle acceleration initially in the
stagnant liquid, very soon the particle will achieve a con-
Integrating between two heights z z1 (j 1) and z z2 stant downward velocity, called the terminal velocity, U pzt :
(j 2), we get
ext
U pzt F pz = f pd : 4:2:50a
Mig
ln f^i1 =f^i2 z2 z1 , 4:2:46
RT Here, F ext
pz is the gravitational force on the particle, taking
where z2 > z1; therefore f^i1 > f^i2 . For an ideal gas mixture, account of the buoyancy effect, and f dp is the frictional
this relation leads to coefficient experienced by the particle as it falls through
Mig the liquid (here, the suspension).
ln x i1 P 1 =x i2 P 2 z2 z 1 , 4:2:47a For a spherical particle of radius rp, density p and
RT
mass mp , if the particle motion obeys Stokes law, then
where P1 and P2 are the hydrostatic pressures at heights z1 f dp 6r p . From equation (3.1.4), the magnitude of the
and z2, respectively. For species 2 in a binary mixture, vertically downward force F ext on this particle is
pz
M 2g mp g1 t =p . Therefore
ln x 21 P 1 =x 22 P 2 z2 z1 : 4:2:47b
RT mp g1t =p 2 2 1 t = p
U pzt r p p g, 4:2:50b
The separation factor for species i 1 and 2 present in a 6r p 9
vertical column of gas mixture is

since mp 4=3r 3p p . Remember this is the terminal
x 11 x 22 M 1 M 2 gz2 z1 velocity of an isolated spherical particle due to gravita-
12 exp : 4:2:48
x 21 x 12 RT tional force. Now we go back to the settling of the suspen-
sion of spherical particles in the stationary vessel.
If M1 > M2, heavier species9 1 will concentrate more in
(Although the vessel top in the figure is open, it is de facto
region 1 i.e. z z1, since 12 > 1. (For a pure gas behaving
a closed system.)
ideally, relation (4.2.46) yields
What one observes as the particles fall toward the
Mig vessel bottom is that a clear layer of liquid, free of particles,
ln P 1 =P 2 z 2 z1 , 4:2:49
RT emerges near the top of the vessel, whereas near the
indicating that P1 at z z1 is higher than P2 at z z2 (>z1).) bottom of the vessel a sludge layer or a dense layer of
This enrichment of the heavier species at z z1 has been particles appears (Figures 4.2.6(b), (c)). If sufficient time
is allowed for equilibrium to be achieved in the gravita-
tional force field, most of the liquid is clarified and
9
Note that this convention where species i 1 is heavier is becomes free of particles; all particles accumulate at the
different from our usual convention. bottom of the vessel in a sludge layer of thickness zHS
4.2 Equilibrium separation in an external force field 257

Dilute
(a) t = 0 suspension

zHL Gravity

Upzt rp, rp 1
1
z=0 A A

(b) t = t1
Upzt 2 nzAA
Clear liquid
A A rp, rp 2
zHL
z1
B B
z=0
rp, rp 2
Upzt 22
(c) t = t2
B B
Clear max
liquid rp, rp nzBB
A zHL
A
z2 B B z
z=0

(d) t = tM zAA rp, rp= 0

Clear zHL
liquid
zHL
B B zBB max
rp,
zHS rp
zHS
z=0 t
tM
tM > t2 > t1 > 0
Sludge
(concentrated suspension)

Figure 4.2.6. Batch sedimentation of a dilute suspension of particles under gravitational force: (a) dilute suspension in a vessel at time
t 0; (b) condition at time t1 > 0; (c) condition at time t2 > t1 > 0; (d) final equilibrium state.

(Figure 4.2.6(d)). The questions of interest are as follows. Thus the line AA is moving downward at a velocity of
How long (tM) does it take to achieve this equilibrium in magnitude jvZAA j. The net particle flux from the top across
the closed vessel with no flow in or out? What is the height the line AA at any time then (diffusion is neglected for
of the sludge layer z HS ? Let us assume here that all particles larger particles) is p, rp j1 jU pztj1 j jvZAA j. This value must
have a density p and size rp. equal the rate at which particles are removed downward
Consider now the different diagrams shown in Figure from below the line AA: p, rp j2 jU pztj2 j jvZAA j. Here
4.2.6. In part (b) of this figure, corresponding to a time t 1 , jU pztj2 j is the magnitude of the terminal velocity of par-
where t M > t 1 > 0, a small height of the liquid at the top ticles below line AA and jU pztj1 j is that above line AA. For
is clear of any particles. The line AA which separates the a steady downward movement of this line AA,
clear liquid at the top from the suspension below is a
p, r p j1 jU pztj1 j jvZAA j p, rp j2 jU pztj2 j jvZAA j: 4:2:51
discontinuity in terms of the particle mass concentration
in the liquid, Rearranging, we get

grams of particles

p, r p j2 U pztj2 p, rp j1 U pztj1
 
p , r p : above and below: jvZAA j : 4:2:52
cm of liquid suspension
3 p, r p 1 p, r p 2
p, rp j2 p, rp j1

If the top liquid is clear, p, rp j1 0. At any instant of time In the special case of a dilute suspension and the top
t1, the line AA is at location z1; at another instant of time t2 clarified layer particle density being zero (i.e. p, rp j1 0),
> t 1 , the line AA (Figure 4.2.6(c)) is at location z2 (<z1). the value of jvAA
Z j is
258 Separation in a closed vessel

jvAA
Z j jU pztj2 j jU pztj1 j, 4:2:53 separated by gravity into a clear supernatant liquid and a
thin sludge layer of thickness z HS over a time period of t M .
corresponding to the particle settling velocity jU pztj1 j of There are two more aspects of considerable interest.
a very dilute particle suspension of particle density
p, rp p, rp j2 . We may assume it to be given by expression (1) How do the lines AA and BB move with time?
(4.2.50b). (2) How are these movements affected by an increased
On the other hand, the sludge layer builds up with time concentration of particles in the starting suspension?
at the bottom of the vessel. At time tM its maximum height This is the subject of hindered settling.
zHS is reached; above it only clarified liquid exists (Figure The first question has already been answered by expres-
4.2.6(d)). At an earlier time t2, where t M > t 2 > t 1 > 0, sions (4.2.53) and (4.2.55) for jvAA BB
Z j and jvZ j. If the heights
the sludge layer height indicated by the line BB (Figure of the interfaces AA and BB are denoted by z AA and zBB ,
4.2.6(c)) is less than z HS (Figure 4.2.6(d)). Let the magni- respectively, then
tude of the velocity of the line BB in the upward direction
be jvBB jvAA
Z j jdz AA = dtj and jvBB
Z j jdz BB = dtj:
Z j. Further, let the particle mass concentration in the
sludge layer be max p, r p since it will be the highest particle
4:2:59
mass concentration in the system. For a steady upward
In Figures 4.2.6(d), the trajectories (solid lines) of z AA
movement of line BB, the particle flux into the sludge layer
and z BB have been plotted as a function of time. They
from the top through the line BB for a very dilute suspen-
intersect at t tm when equilibrium settling has been
sion, namely p, rp j2 jU pztj2 j jvBB
Z j must equal the particle reached.
flux built up by the movement of line BB in the upward
The nature of these two trajectories changes if batch
direction, namely max BB
p, r p jvZ j: sedimentation involves a more concentrated suspension
max BB BB (for an introduction, see the general treatment in Probstein
p, r p jvZ j p, r p j2 jU pztj2 j jvZ j; 4:2:54
(1989) of the subject). First, the settling velocity of an
jvBB max
Z j p, r p p, r p j2 p, rp j2 jU pztj2 j p, rp jU pztj1 j individual particle is reduced due to the close proximity
p, rp jU pztj1 j of many other particles. This is due to an increased viscos-
jvBB
Z j :
max
p, r p p, r p
ity of the liquid, as well as an increased drag force experi-
enced by the particle due to the other particles. The
4:2:55 effective settling velocity U eff
pzt is less than U pzt for a dilute
When an equilibrium is achieved, line BB will coincide suspension of the same particle size;
with line AA near the bottom of the vessel (Figure 4.2.6 U eff
pzt U pzt GD particle volume fraction, 4:2:60
(d)). If the distance of line AA from the bottom is indicated
by zAA and that of line BB by zBB , then at time tM both must where GD is a function of the particle volume fraction in
have the same value: the suspension and has a value less than 1. The functional
dependence is often empirically expressed as
zAA zHL jU pztj1 jt M zBB jvBB
z jt M ; 4:2:56  n
8 9 GD 1particle volume fraction , 4:2:61
z HL <p, r p p, rp =
max
zHL
tM where the value of n suggested by different workers varies
jvBB
Z j jU pztj1 j jU pztj1 j : maxp, r p ;
8 9 between 4.7 and 5.1. The analysis by Kynch (1952) of
z HL < p, r p = hindered settling based on (4.2.60) is a useful first step in
1 max :
jU pztj1 j : p, r p ; this complex subject.
A qualitative discussion of the change in the nature of
4:2:57
the interface trajectories shown on the right-hand side of
The value of z HS is (from (4.2.56) and (4.2.57)) Figures 4.2.6(d) is also in order for a concentrated suspen-
8 9 sion. The rate at which the interface AA drops, i.e.
zHL < p, r p = jvAA
z j dz AA =dt changes with time. As more and more
z HS zHL jU pztj1 j 1 max clarified liquid appears at the top, the particle concentra-
jU pztj1 j : p, r p ;
8 9 4:2:58 tion below AA increases. This reduces the value of jU pztj2 j
< =
p, r p (equation (4.2.53)), which is no longer equal to jU pztj1 j;
zHL
:max
p, r p ; further, it decreases with increased t, leading to a more
curved surface, shown in Figure 4.2.6(d) by a dashed line.
We could have also obtained this result simply by a particle Correspondingly, the speed of the rise of the interface BB
mass balance between the original dilute suspension of decreases with time.
height z HS and density p, r p and the final sludge density The principle of sedimentation in a closed vessel dis-
of max
p, r p . A very dilute suspension of particles has now been cussed is the primary basis for the large-scale separation of
4.2 Equilibrium separation in an external force field 259

Table 4.2.1. Cell densitiesa reduces nickel leaching to levels without any adverse
effects. The HDPs with Abs on the surface develop
Cell type Density (g/cm3) extremely specific interactions with particular cell types
so that gravitational settling is accompanied by very high
T Cells 1.077
B Cells 1.077
cell type selectivity.
Monocytes 1.064
Granulocytes 1.09
4.2.3.3 Particle fractionation and density gradient based
NK Cells 1.06
Erythrocytes 1.115 separation
Osteoblasts 1.005 In Section 4.2.3.1, we considered the separation of particles
a
From Recktenwald and Radbruch (1998). See also Table 7.3.1. of one size and density from the liquid medium via particle
settling due to gravity. If two different types of particles
having different densities have to be separated by gravity,
rapidly settling particles in open devices called thickeners. the liquid medium selected should have a density in
Usually, the particle density is significantly higher than that between the densities of the two types of particles. The
of water, which is usually the liquid. On the other hand, the particles, having a higher density than the liquid, will settle
settling/separation of cells of biological origin by gravita- as shown in Figure 4.2.6(d); the lighter particles will float to
tional sedimentation from an aqueous medium/blood, the top of the liquid layer in devices called sorting classi-
etc., is very difficult: the cell densities are very close to that fiers. The variation of size for particles having a given
of water, as shown in Table 4.2.1. From equation (4.2.50b), density is of no consequence in a closed system headed
one can estimate the terminal velocities using appropriate for equilibrium. Although the individual settling velocities
cell dimensions, water viscosity and density (~1.0 g/cm3). will vary due to the particle size variation, this variation is
Since these values are quite low, a technology based on inconsequential since different size particles will ultimately
high-density particles (HDPs) has been developed, and it is have to settle to the bottom or rise to the top in a closed
described in Section 4.2.3.2. It is implemented in a closed system slated for equilibrium. For particles of larger sizes
vessel and is useful in biomedical and biotechnology employed in such separations, the liquid density may vary
applications. from 1.3 upward. Liquids of different densities may be
developed by employing varying concentrations of CaCl2
(used for cleaning coal) or fine suspensions of heavy min-
4.2.3.2 Sedimentation using high-density particles
erals, e.g. magnetite (density ~5.1 g/cm3), silicon (density
Kenyon et al. (1998) have described a technology utilizing ~6.5 g/cm3), etc., in water.
high-density particles (HDPs) of nickel approximately If there are particles of different densities in a sample,
10 m in diameter and having a density of 9 g/cm3. There- they may be separated by introducing the sample mixture
fore the value of p t in the terminal velocity equation into a vessel (or column) containing a number of liquid
(4.2.50b) for such particles will be around 8 g/cm3, layers having different densities; alternatively, the liquid
whereas that for the cells in Table 4.2.1 can at the most layer density may vary continuously along with column
be 0.1 g/cm3. The technology involves attaching the cells to height. Whether one uses a number of discrete densities
the HDPs of nickel so that settling is very rapid. These in a multiple-layer column or a continuous gradient of
dense particles have a highly irregular surface, which is density, heavier liquids must be lower in the column com-
easily coated by monoclonal or polyclonal antibodies (Abs) pared to the lighter liquids to prevent mixing via natural
via passive adsorption. When, for example, a sample of convection (see Section 6.1). Heavier particles will settle to
whole blood containing certain types of cells whose surface the layer where their densities match the density of the
antigens (Ags) interact directly with the Abs is introduced layer. Lighter particles will rise to the liquid layer where
into a suspension of HDPs in a test tube in an appropriate their densities match that of the liquid layer, another
buffer and settling allowed to take place, the particles in example of focusing in an external force field.
the tube settle in about 4 minutes. The attached cells may
be removed from the particles by a number of procedures
4.2.3.4 Inclined settler
(Kenyon et al., 1998).
This concept is generally useful in a variety of other If we look at equation (4.2.58), we can calculate the mass of
biologically relevant applications, for example the removal particles settled per unit bottom surface area of the vessel
of proteins by dense particles having surface ion exchange as z HL p, r p (g/cm2). The total mass of particles settled is
groups or other functional groups from a fermentation zHL p, rp  Abottom , where Abottom is the cross-sectional area
broth. In the case of HDPs of nickel, to prevent leaching of the vessel; correspondingly, the volume of classified
of nickel into the medium, the particles are heated for 35 liquid is z HL Abottom . It is clear that the volume of liquid
hours at 250  C; apparently the surface oxidizes, which clarified is directly proportional to Abottom for a given z HL .
260 Separation in a closed vessel

Consider now Figure 4.2.7, where we show a narrow chan- stabilized first in a tetralin suspension by the surfactant
nel inclined to the vertical by an angle . If this channel Aerosol OT, which stands for dioctyl sodium sulfosucci-
is filled with a particle suspension, then the surface area nate. When an electric field is applied in the inclined settler
over which particles can settle by gravity is equal (Figure 4.2.7), in which the top plate has a positive charge
to (L sin 2b cos )  width of the channel plate and the bottom plate a negative charge, particles settle
W ; i:e: it is given by W L sin 2b cos . If this chan- onto the bottom plate, having a particle-free liquid at the
nel were vertical ( 0), the surface area for settling would top and a concentrated suspension over the bottom plate.
have been only 2bW. Therefore, the ratio of the volume of The higher the voltage gradient applied between the two
liquid clarified or the rate of clarification in an inclined settler electrodes, the lower the height of the clear liquid interface
with respect to a vertical settler is (L/2b) sin cos . When from the bottom of the electrosettler.
L/2b is large (thin lamellae-like channel), this ratio can be
very large (when is nonzero and has a significant inclination
to vertical). 4.2.4 Particle separation with acoustic forces
The acoustic radiation force Facrx in the x-direction experi-
4.2.3.4.1 Inclined electrosettler When the particles are enced by particles of densities p suspended in a medium
in a colloidal suspension, the particles do not settle due to of density f has already been expressed by equation
gravity. An examples is as follows: nonaqueous liquids (3.1.48):
containing asphaltenes, which adsorb on fine particles " #( )
present in various liquids encountered in the production P 20 V p f 5p 2f p
F acrx sin 2kx,
of synthetic fuels from coal, oil shale and tar sands; the 2 2p f f
surface charge carried by asphaltenes leads to a nearly
stable colloidal suspension (Gidaspow et al., 1989). An The quantity within the second set of brackets is the
inclined settler whose two walls are turned into two elec- -factor:
trodes (one positive, the other negative), with a voltage 5p 2f p
applied, can produce a clarified liquid at the top and
factor :
2p f f
concentrated slurry at the bottom. For example, Gidaspow
et al. (1989) have illustrated how alumina particles of size The sign of the acoustic contrast factor, or -factor, deter-
15 m and having a positive zeta potential (30 mV) were mines the direction of the force on the particle: a positive
-factor moves those particles to a pressure node, whereas
a negative -factor moves those particles to a pressure
antinode (Petersson et al., 2005). This is illustrated in
2b

Figures 4.2.8(a) and (b). Consider two types of particles


in water in a closed chamber, as shown in Figure 4.2.8(a)
q without any acoustic force field. The darker particles have
positive values of , whereas the lighter particles have nega-
L

tive values of . When the acoustic force is switched on, with


an acoustic standing wave having a pressure node in the
center of the chamber of the closed vessel, the particles are
separated: the particles having positive -values con-
Figure 4.2.7. Batch sedimentation in an inclined settler. centrate at the pressure node, whereas those having

Figure 4.2.8. (a) Schematic illustration of a particle mixture in a closed chamber. No acoustic force field is present in the chamber.
The suspended particles have negative (white) and positive (dark) -factors.
4.2 Equilibrium separation in an external force field 261

Figure 4.2.8 (cont.) (b) Acoustic standing wave (outlined with the pressure node in the center of the chamber) induced between the walls
of the chamber. Under these conditions, the - factors of the particles will determine whether they move toward the pressure node or the
pressure antinodes; particles (white) with negative - factor are at the wall; dark particles with positive - factor are at the pressure node,
the chamber center. (After Petersson et al. (2005).)

negative -values concentrate at the pressure antinodes The temperature of one of the chambers is now raised to T1
near the walls of the closed vessel. An example of such a (>T2) such that the two bulbs are maintained at two differ-
separation is the separation of lipid particles (emboli) from ent temperatures. If the thermal diffusion ratio kT for a
red blood cells (erythrocytes) for an intraoperative blood given species 1 is negative, then, from definition (3.1.45)
wash system. and force relation (3.1.44), we can conclude that species 1
moves due to the temperature gradient from the colder
bulb to the hotter bulb. This builds up its concentration
4.2.4.1 Gas mixture separation via propagation of acoustic
in the hotter bulb, setting up its back diffusion into the
waves
colder bulb. At equilibrium, the temperature-gradient-
Consider two reservoirs connected by a narrow duct with driven flux of species 1 is balanced by that due to the
the whole volume filled with a 5050 mixture of HeAr. If concentration gradient in the opposite direction, since
each reservoir is connected to a bellows-sealed piston, and the two forces in the opposite direction must balance each
the pistons are driven at, say, a low frequency of 10 Hz with other. To exploit this dynamic equilibrium, we need to
independent phase and amplitude control, the oscillating calculate the flux due to the temperature gradient first.
pressure wave creates a composition difference of as much Employ expression (3.1.44) for the force on a gmol of
as 6% between the two chambers at the ends of the narrow species 1 (say) due to the temperature gradient in a
connecting duct (Spoor and Swift, 2000). The periodic mixture of species 1 and 2:
motion of the pistons produces density fluctuations in the
F T1
gas; this sound wave creates the composition difference via C 1 temperature-gradient-driven molar flux of 1 J T1
f d1
a complex interaction of thermal diffusion (compression 0 1
leads to heating), ordinary diffusion, convective motion, RT DT1 C1 DT
etc. (Geller and Swift, 2002a,b). @ A r ln T d 1 r ln T;
D12 C 1 M 1 f1 M1

4:2:62
4.2.5 Externally imposed temperature gradient:
thermal diffusion where f d1 is the frictional coefficient of species 1 ( RT/D12)
and there is no bulk velocity. This flux is opposed by the
We have so far considered primarily three externally
diffusive flux J1 of species 1 from a concentration gradient:
imposed force fields, centrifugal, electrical and gravita-
tional, and their individual effects in developing separation DT1 C 2 M 1 M 2 D12
in a single phase in a closed vessel. Here we consider the J T1 r ln T J 1 t rx 1 : 4:2:63
M1 t M 1
effect of an externally imposed temperature gradient on
the equilibrium separation in a single phase in a closed Use the one-dimensional form of this relation since the
vessel. Although it is not an externally imposed force field, species movement is only in one coordinate direction:
the temperature gradient achieves a net separation effect DT1 t dT dT
similar to that developed in an external force field. dx 1 k T :
D12 C 2t M 1 M 2 T T
Consider a binary gas mixture of uniform composition
in a vessel shown in Example III of Figures 1.1.3. Initially, Often, kT is expressed as k 0T x 1 1x 1 , where k 0T is a
the temperature T2 of the mixture is uniform everywhere. thermal diffusion constant; this leads to
262 Separation in a closed vessel

dT dx 1 (a)
k 0T : 4:2:64
T x 1 1x 1
Feed C1f, Vf
Integrate between region 1 at T1 and region 2 at T2. The
extent of separation generated by thermal diffusion is Membrane
rather low; therefore the product x1 (1 x1) is considered
to be a constant at an averaged value x 1 1x 1 . Integrating Dialysate
C1d, Vd
equation (4.2.64) now yields
T1
k 0T x 1 1 x 1 ln x 1 jT 1 x 1 jT 2 x 11 x 12 : (b)
T2

Since x11 and x12 are quite close, we may substitute Feed
BN
fx 1 1 x 1 g by (x12)(1 x11) (i.e. as a product of two
Cation exchange
boundary values). Therefore, membrane
x 11 x 12 x 11 1 x 12 T1 Strip
1 12 1 12 k 0T ln : AN
x 12 1 x 11 x 12 1 x 11 T2
4:2:65
Figure 4.3.1. (a) Dialyzer with closed inlets and outlets. (b) Closed
This relation provides an expression for the enrichment vessel having two aqueous solutions separated by a cation
factor 12 for two species 1 and 2 between the two regions exchange membrane: Donnan dialysis.
in terms of k 0T and the two temperatures. Consider now an
equimolar mixture of H2 and N2 subjected to a tempera-
ture difference of 260  C in the hot bulb and 15  C in the present in the feed solution are not allowed to pass
cold bulb. In the above result, (x11 x12) may be estimated through it to the dialysate stream. This process is the
if we can estimate k 0T x 12 1 x 11 , which is equal to k T ; basis for blood purification using an artificial kidney
a rough estimate of this value for illustration is 0.066 (Ibbs (or hemodialyzer). The membrane does not have sub-
et al.,1939). The value of (x11 x12) we obtain is 0.0406, stantial convection of the solvent, namely water. Solutes
indicating about a 4% difference in mole fraction for the diffuse through fine pores to the dialysate; metabolic
lighter species H2 between the hot region (T1) and the cold wastes, e.g. urea, uric acid, creatinine, etc., diffuse
region (T2). through the water-filled pores or hydrogels from the
blood to the dialysate, but macrosolutes, e.g. blood
proteins (albumin, etc.), and larger species are essen-
4.3 Equilibrium separation between two regions tially excluded by the membrane. In actual operation,
in a closed vessel separated by a membrane the blood (or feed solution) flows on one side of the
In Sections 4.1 and 4.2, either a two-phase system or a membrane and the dialysate flows on the other side into
single-phase system was introduced into a vessel which the device and out.
was then kept closed with no mass additions or with- If, in such a device, blood is introduced into the
drawals. After allowing the system to reach equilibrium, feed side and the dialysate stream is introduced into the
the separation achieved between the two regions in the receiving side, and the device inlets and exits are closed
closed vessel was calculated. There was always some sepa- (Figure 4.3.1(a)), one would like to know what will happen
ration achieved between the two regions. In this section, as t ! when equilibrium will be achieved. Would the
either we introduce a feed mixture into one region of a two regions show separation with respect to the microso-
membrane device, or we introduce two different mixtures lute as t ! ?
into two regions of the membrane-containing device. We Let the microsolute (e.g. urea) to be removed from
then allow a long time to elapse and calculate the sepa- feed be species 1, the feed solution region j f and the
ration achieved at equilibrium between the two regions. dialysate solution region j d. Let the feed and the
This has been studied here for the membrane processes of dialysate concentrations of the solute 1 at time t 0,
simple dialysis, Donnan dialysis, gas permeation, reverse be C 01f and C 01d 0. We are interested in knowing the
osmosis and ultrafiltration. values of C1f and C1d as t ! . Assume the two solutions
to be well stirred; therefore the two surfaces are exposed
to the bulk solution concentrations C1f and C1d at any
4.3.1 Separation by dialysis using neutral membranes
time. Let the volumes of the feed solution and the dia-
Dialysis is a membrane process in which a microsolute lysate solution be Vf and Vd , respectively. The solute flux
present in a feed solution is removed from the feed to a expression through the membrane of thickness m and
receiving solution of the same phase called the area Am at any time may be written, following relation
dialysate; the membrane is such that the macrosolutes (3.4.99), as
4.3 Equilibrium separation: membrane, closed vessel 263

Q1m
0 1
N 1z k 1m C 1f C 1d C 1f C 1d : 4:3:1 Q Am 1
m C 1d C 01f @ 1m A 0
m V d
10 1
Q A m 1
@ 1m A@ A 1
A total solute balance on the feed side and the permeate
m Vf Vd
side leads to
Vf
dC 1f Q C 01f ) V f V d C 1d V f C 01f :
Vf 1m Am C 1f C 1d ; 4:3:2 Vf Vd
dt m
4:3:9b
dC 1d Q1m
Vd Am C 1f C 1d : 4:3:3 Thus, the total amount of solute V f C 01f
present initially in
dt m
the feed is distributed between the feed and the dialysate.
Laplace transforms of these two coupled ordinary differen- As t ! , C1d has become a constant, dC 1d =dt in equa-
tial equations will convert the t-domain to the s-domain tion (4.3.3) must be zero and C 1f C 1d .
and yield the following two equations: The process of dialysis has reduced the solute concen-
tration in the feed solution in the closed vessel; at equilib-
Q1m Am
sC 1f C 1f t 0 C 1f C 1d ; rium, the value of C1f has, however, become equal to that
m V f
of C1d, the concentration in the dialysate. Thus, if separ-
Q1m Am ation of solute 1 between the two regions is the goal, at
sC 1d C 1d t 0 C 1f C 1d ,
m V d equilibrium, there is no separation. Further, if separation
between two solutes 1 and 2 in the feed is considered and
where C 1f and C 1d are the transformed variables in the both solutes are permeable through the membrane, then,
s-domain. Introducing the values of C1f and C1d at t 0, at equilibrium, each solute will be present in each region of
we get the closed vessel at the same concentration, resulting in no
Q1m Am separation.
sC 1f C 01f C 1d C 1f ; 4:3:4 In practice, separation is achieved in dialysis since
m V f
the device is operated as an open system: fresh dialysate
Q1m Am is introduced and then taken out continuously. Thus
sC 1d C 1f C 1d : 4:3:5
m V d C1d is always maintained lower than C1f; ultimately, the
feed solution concentration of species 1 can be reduced
After eliminating C 1f from these two equations, the
to a very low level. The continuous renewal/removal
following equation is obtained for C 1d :
of the permeate side fluid, in this case the dialysate, is

Q1m Am

Q Am
 
Q1m Am

Q1m Am
 essential to most membrane based processes, which
s s 1m C 1d C 1d are based on different intrinsic rates of transport of
m V f m V d V d m V f m
species 1 and 2 through the membrane. Thus, mem-
Q1m Am 0
 
C 1f ; 4:3:6 brane devices have to be open, in general, to achieve
m V d
separation.
A closed device employing a dialysis membrane as
2 3
Q1m Am 6 1
 
described above can achieve high purification of the
C 1d C 01f
6 7
7:
m V d 4 2 Q1m Am 1 1 5
 
feed solution if special conditions exist. For example,
s s
m Vf Vd if in the dialysate side there is another species, which
4:3:7 reacts chemically with species 1 and produces a product
3 which cannot diffuse through the membrane to the
If we define feed side, then it may be possible to purify the feed

Q1m Am

1 1
 solution. Obviously, the added reactant in the dialysate
a, side should not be able to diffuse through the membrane
m Vf Vd
to the feed solution if the feed solution has to be puri-
a constant, then fied. One such example has been provided in Section
5.4.3 based on the work carried out by Klein et al.
Q1m Am 1
  
C 1d C 01f : 4:3:8 (1972, 1973).
m V d s2 as
The intrinsic separation capability of a dialysis mem-
Taking an inverse transform of this, we get brane located in a closed vessel can be determined by
considering the transport of two solutes i 1 and 2
Q1m Am 1 through the membrane during the initial time period when
  
C 1d C 01f 1 e at : 4:3:9a t ! 0. The separation factor between two solutes, 12,
m V d a
between the two regions ( j 1, dialysate ! j d; j 2,
As t ! , this is reduced to feed ! j f ) for a two-species (i 1,2) system is
264 Separation in a closed vessel

C 1d C 2f
Heparin (12 000)
x 11 x 22 x 1d x 2f C 1d C 2d C 1f C 2f PEG (4000)
12 : 4:3:10
x 12 x 21 x 1f x 2d C 1f C 2d Myoglobin (17 000)
Ovalbumin (45 000)
C 1f C 2f C 1d C 2d 0.1 Albumin (66 000)

Therefore, 12 C 1d C 2f =C 1f C 2d . To determine its value as


t ! 0, we assume that, as t ! 0, Avisco wet gel
Cuprophan PT-150

C 1f C 01f , C 2f C 02f : 4:3:11


0.01
Further,

dC 1d

Qim
C 1d jt!0 C 1d jt0 t    4:3:12
dt t!0 Dil
0.001

(Taylor series expansion around t 0). But C 1d jt0 0.


Dextran (16 000)
From relation (4.3.7), define Vitamin B-12 (1355)
Inulin (5200)
Sucrose (342)
Q1m Am 1 1
  
Uric acid (5mg %)(168)
a1 and 0.0001 Creatinine (113)
m Vf Vd Urea (60)
Sodium chloride (0.15M)(58)
Q2m Am 1 1
  
a2 :
m Vf Vd
0.00001
From relation (4.3.9a), 0 5 10 15 20 25 30 35
r ()
dC 1d Q1m Am a1 t
 
C 01f e ) C 1d jtt
dt m V d Figure 4.3.2. Permeability reduction as a function of characteristic
Q1m Am solute radius (molecular weight in parenthesis) (Colton et al.,
 
t
C 01f : 1973). Reprinted, with permission, from C.K. Colton, K.A. Smith,
m V d 1 a1 t   
E.W. Merrill, P.C. Farrell, J. Biomed. Mater. Res., 5, 459, (1971),
Similarly for C 2d jtt . Therefore Figure 5, p. 483, 1971, John Wiley & Sons.
. Q A 
1m m
C 01f 1 a2 ttC 02f

V approach is likely to be quite fruitful, provided the mem-
12 .  m d  : brane pore size distribution is quite narrow around an
0 Q2m Am
C 1f 1 a1 ttC 02f

m V d average rp. For many commercially available porous mem-
branes, used, for example, in hemodialyzers, the structure is
In the limit of t ! 0, that of a water-swollen gel, where the transport corridors
Q1m and channels have varying dimensions. Therefore, models
12 : 4:3:13 based on cylindrical capillaries of radius rp may not be
Q2m
accurate enough. However, Klein et al. (1979) suggest
This relation identifies the intrinsic separation capability of that their data for Cuprophan 150PM membrane for a var-
the dialysis membrane for two solutes 1 and 2. The t value iety of solutes may be reasonably modeled by an average
must be larger than the time required for the solutes to pore diameter of 3.5 nm; on the other hand, such an average
diffuse through the membrane and appear in the dialysate. pore model could not describe the behavior of a number of
For porous membranes, one can employ the flux other membranes with a wider pore size distribution. Esti-
expression (3.4.97) to estimate Qim: mates of Qim for a variety of solutes and Cuprophan mem-
branes may be obtained from the data of Colton et al. (1971)
Dil GDr r i , r p m im
Qim : 4:3:14 (see Figure 4.3.2) and Farrell and Babb (1973).
m

Therefore, 4.3.2 Separation between two counterions in two


solutions separated by an ion exchange membrane:
D1l GDr r 1 , r p 1m Donnan dialysis
12 jt!0 : 4:3:15
D2l GDr r 2 , r p 2m
Consider two electrolytes AN and BN in two different
An estimate of GDr r i , r p may be obtained from the Faxen aqueous solutions separated by a cation exchange mem-
relation (3.1.112e); the partition coefficient im will be deter- brane (as in Figure 4.3.1(b)). Assume that the anion N is
mined by the geometric partitioning effect. Such an impermeable through the membrane. Suppose, at time
4.3 Equilibrium separation: membrane, closed vessel 265

t 0, the solution on one side of the membrane (the corres- Consider now the case where Z A Z B 1. This
00
ponding quantities are identified by the prime) contains only implies that, at equilibrium, a0Bw =aBw has a large value.
electrolyte AN, whereas the solution on the other side of the Since the initial concentration of BN in chamber 00 was low
membrane (the corresponding quantities are identified by to start with, distribution of B between the two chambers
the double prime) contains only electrolyte BN. After a long will drastically reduce the concentration of B in chamber 00
time, equilibrium is expected to be established. Therefore at equilibrium from its initial concentration. Therefore, if
the expression for Donnan potential (relation (3.3.118b)), cation B is undesirable in the chamber 00 liquid, the con-
centration of B in chamber 00 can be drastically reduced via
1 aiw
   
R w RTln V i P R P w this phenomenon, called Donnan dialysis, using an accept-
Zi F aiR
able cation A introduced via chamber 0 . Further, by redu-
for ionic species i, should be valid for both solutions on cing the volume of chamber 0 to a small value, undesirable
both sides of the membrane at equilibrium. For cation cation B may now be concentrated to a high level in
A and the side of the membrane corresponding to AN, chamber 0 ; the solution in chamber 0 is called the strip
 0  solution, whereas the original solution in chamber 00 con-
1 a
 
0R 0w RT ln Aw V A P 0R P w , 4:3:16a taining BN only is called the feed solution. After equilib-
ZAF 0
aAR rium is reached, the solution in chamber 00 is called the
where 0w is the feed solution potential, if any. For raffinate (in analogy to liquidliquid extraction).
cation A and the side of the membrane corresponding to BN, Wallace (1967) illustrated this technique by concen-
 00  trating
 uranyl ions (UO2) from a 0.01 M uranyl nitrate
1 a Aw
 
00 00
R w RT ln 00
00
V A P R P w : 4:3:16b UO2 NO3 2 feed with a 2 M nitric acid strip using
ZAF a AR a cation exchange membrane (see Section 3.4.2.5). The
concentration of UO2 in the raffinate was reduced to
The solution pressure, Pw, is the same on both sides.
0.67% of the feed solution ( 0.01 M), whereas that in
Assume swelling pressure effects to be such that
00 the strip was raised to 0.148 M. Wallace (1967) used an
P 0R P R 0. Then
open system with feed and the strip solutions flowing
(  1  00 Z1 ) on two sides of the membrane. A closed system will
00 00 RT a0 Z A aAw A
0R 0w R w ln Aw ln 00 : also achieve similar partitioning and separation. Another
F a0AR aAR
application is water softening. Using a strip solution con-
4:3:16c taining the cation Na at large concentrations, divalent
Now there is no potential difference across the membrane, cation Ca present in dilute concentrations in the feed
00
so 0R R . Therefore solution (hard water) on the other side of a cation
exchange membrane can be removed substantially and
(  1)
00
0 RT a0Aw Z A replaced by Na.
w w ln 00 4:3:17a
F aAw The difference between the basic result achieved at
00
equilibrium under Donnan dialysis with that achieved in
since a0AR aAR , both being merely the activity of the conventional dialysis using a neutral membrane (Section
00
counterion A in the membrane. But w 0w is independent 4.3.1) is due to the assumption of perfect rejection of anion
of any ionic species, which suggests that the relation N by the cation exchange membrane in Donnan dialysis.
(  1) Under such a condition, the requirements of the electro-
00 0 RT a0Bw Z B
w w ln 00 4:3:17b neutrality condition ensure that the two solutions will have
F aBw
radically different cation concentrations at equilibrium for
is also valid. Consequently (Helfferich, 1962), any cation since the anion concentrations of the two cham-
bers are so different.
Z1  0 Z1
a0Aw aBw B In practical Donnan dialysis in open systems, with less

A
00 00 a constant: 4:3:18
aAw aBw than perfect rejection of co-ions, the calculation of ion
transport rates through the membrane requires a know-
The implications of this result may be understood if we ledge of the ionic concentrations at the two membrane
know the concentrations of anion N in the two chambers. solution interfaces on the two sides of the ion exchange
Suppose the chamber for species AN (0 ) has a high concen- membrane. These concentrations are to be determined for
tration of AN to start with, whereas the chamber for BN (00 ) the electrolyte AN based on the Donnan potential based
has a low concentration of BN to start with. Due to the equilibrium relation (4.1.73) developed for ion exchange
requirement for electroneutrality and the impermeability resins:
of the membrane to anion N, the concentrations of cations
Z1 Z1
(A and B) in the chamber for BN will always be low. a0 Aw a0 Nw
 
A N
00 : 4:3:19
Therefore the ratio a0Aw =aAw will always be large. a0 Am a0 Nm
266 Separation in a closed vessel

Here, subscript m refers to the membrane, w refers to the enriched in the preferentially permeating species. The
solution and the 0 side of the membrane is under consider- residual feed gas, as well as the permeated gas mixture,
ation. To determine a0 Am or C 0 Am as a function of known are removed continuously from the gas permeation mem-
quantities, we need also the electroneutrality relations for brane device. Consider such a device. At time t 0, a
the solution and the membrane: mixture containing m0Af moles of species A and m0Bf moles
of species B is introduced into the feed side, the permeate
solution Z A C 0Aw Z N C 0Nw 0; 4:3:20
side does not have any gas molecules, and both feed
membrane Z A C 0Am Z N C 0Nm C m 0, 4:3:21 and permeate side are kept closed. One would like to
know the condition at equilibrium as t ! . The volumes
where Cm is the molar concentration of the negatively of the feed and permeate sides are Vf and Vp, respectively;
charged fixed ionic groups in the cation exchange the temperature is T, the membrane area is Am, the mem-
membrane. brane thickness is m and ideal gas behavior may be
Consider a specific electrolyte CaSO4, so ZA 2 and assumed.
ZN 2. It follows from (4.3.19) that Due to ideal gas assumption, the partial pressures
a0Aw a0Nw a0Am a0Nm : 4:3:22a of species A in the feed chamber, pAf, and the perme-
ate chamber, pAp, are related to the corresponding
Using molar concentrations and appropriate activity moles of species A, mAf and mAp, respectively, at any
coefficients, time t by
0 0 0 0
M M M M mAf RT mAp RT
C 0Aw C 0Nw Aw Nw C 0Am C 0Nm Am Nm : 4:3:22b
pAf and pAp : 4:3:23
0 0 0 0
Vf Vf
M M M M
Assume that Aw Nw Am Nm . Then
The rates of change in the number of moles of species A in
C 0Aw C 0Nw C 0Am C 0Nm : 4:3:22c
the two chambers are
From (4.3.20), C 0Aw C 0 Nw , and, from (4.3.21), dmAf Q Am mAf RT mAp RT
 
A ; 4:3:24
C 0Nm Z A C 0Am C m =Z N : 4:3:22d dt m Vf Vp

dmAp QA Am mAf RT mAp RT


 
Therefore, 4:3:25
dt m Vf Vp
Z A 02 C 0Am C m C 0Am C m
C 02
Aw C Am C 02
Am ;
ZN ZN 2 Laplace transforms of these two equations from the t-
domain to the s-domain yield
C 0Am C m
C 02
Am C 02
Aw 0:
2 Q RTAm Vf
 
s mAf m0Af A mAf mAp ; 4:3:26
V f m Vp
4:3:22e
Q RTAm Vf
 
The acceptable solution of this quadratic is s mAp A mAf mAp : 4:3:27
V f m Vp
r
0 Cm Cm2
C Am C 02 Aw : 4:3:22f From these two equations, one can easily obtain the
4 16
following expression for mAp :
The molar concentration of the anion in the membrane at
QA RTAm 1
 
the interface C 0Nm can now be obtained from (4.3.21) since mAp m0Af 4:3:28
V f m s2 a 1 s
C 0Am is known. A similar analysis may be carried out for the
00
side of the membrane. Note that if the AN electrolyte where
concentration in the solution at the membranesolution
QA RTAm QA RTAm
 
interface is C 0sw , then, for CaSO4, C 0Aw C 0sw . a1 : 4:3:29
V p m Vf

On inverting relation (4.3.28), we get


4.3.3 Separation of a gas mixture by gas permeation
QA RTAm 1
  
mAp m0Af 1 e a1 t : 4:3:30
Gas transport through a nonporous polymeric membrane, V f m a1
called gas permeation, has been considered in Section
3.4.2.2. In practical gas separation processes, the feed gas At time t ! ,
mixture is allowed to flow on one side of the membrane;
m0Af QA RTAm Vp
 
one of the species preferentially permeates through the mAp m0Af : 4:3:31
membrane to the other side. The permeated gases are a1 V f m Vf Vp
4.3 Equilibrium separation: membrane, closed vessel 267

Correspondingly, as t ! , Since the general expression for mBp may be written as


Vf QB RTAm 1
  
mAf m0Af mAp jt! m0Af : 4:3:32 mBp m0Bf 1 e a2 t , 4:3:38
Vf Vp V f m a2

Therefore, as t ! , where
QB RTAm QB RTAm
 
m0Af RT m0Af RT a2 ,
pAf and pAp : 4:3:33 V p m V f m
Vf Vp Vf Vp
we get, similarly,
Thus, the partial pressures of species A on the two sides of
QB RTAm a2 t
 
the membrane are equal at equilibrium. The same result mBp jt!0 m0Bf e t: 4:3:39
will be obtained for species B present originally in the feed V f m
reservoir, i.e. The separation factor AB as t ! 0 may be expressed as
m0Bf RT m0Bf RT mAp mBf
  
pBf and pBp 4:3:34
Vf Vp Vf Vp mAp mBp m mBf m m
AB jt!0   Af  Ap Bf :
mAf mBp mBp mAf
at time t ! . Therefore, at equilibrium, t!0
mAf mBf mAp mBp
pAf =pBf m0Af =m0Bf p0Af =p0Bf , 4:3:35 4:3:40

where p0Af and p0Bf are the partial pressures of species A For a system where very little of the feed permeates in the
and B in the feed chamber in the feed gas mixture at time short time interval t near t ! 0, we can assume easily that
t 0. Therefore, no separation has taken place in the mBf =mAf m0Bf =m0Af . Therefore
gas mixture left on the feed side at equilibrium. Similarly, Q ea2 t QA 1 a2 t   
  
at equilibrium, AB jt!0 A a1 t : 4:3:41
QB e QB 1 a1 t   
pAp =pBp m0Af =m0Bf p0Af =p0Bf : 4:3:36 This result identifies (QA/QB) as the initial time separation
factor (only after the time lag period is over); as time
Therefore, the permeate side gas mixture at equilibrium increases, this factor decreases from this high value. As
has a composition equal to that of the original feed gas t ! , there is no separation.
mixture at t 0; no separation has taken place in the In practical gas separation processes in open separators
closed system. This is why gas permeation devices in prac- using gas permeation through nonporous membranes, the
tice are operated as an open system: any gas mixture selectivity (QA/QB) of gas species A over B is of considerable
produced by permeation is immediately withdrawn from importance. From the flux expression (3.4.72), we may write
the device. Since the intrinsic membrane transport rates
Q DAM SAM
  
of the two species for unit partial pressure difference AB jt!0 A : 4:3:42
between the feed and the permeate are different, the gas QB DBM SBM
mixture produced as a permeate has a composition differ- The selectivity, AB, is often broken up into a product of
ent from that of the feed. The permeate mixture withdrawn two factors, the diffusivity selectivity (also known as the
from the device must have a composition different from mobility selectivity), (DAM/DBM), and the solubility selectiv-
that of the feed mixture for separation. See Sections 6.3.3.5, ity, (SAM/SBM). Table 4.3.1 illustrates the contributions of
6.4.2.2, 7.2.1.1 and 8.1.9 for open systems. Continuous
energy input will yield separation in a closed system see Table 4.3.1. Transport, solubility and selectivity properties for
the gas pair helium (A)methane (B) at 25  C for a variety of
Figures 8.1.4(b) for a system operated at total reflux.
polymersa
One can demonstrate such separation capabilities,
even in the closed vessel considered so far, if we focus, Polymer AB DA/DB SA/SB
for example, on the initial time period in the vessel when t
! 0. We can calculate the moles of species A and B in the Silicone rubber 0.38 5 0.075
permeate chamber when t ! 0. For species A, differentiate Natural rubber 1.08 24 0.044
the expression (4.3.30) for mAp with time t to obtain, by Hydropol (hydrogenated 1.21 28 0.043
polybutadiene)
Taylor series expansion around t 0,
Low-density polyethylene, 1.70 35 0.048
0.9137 g/cm3
dmAp
mAp

mAp

t High-density polyethylene, 2.94 55 0.054
t!0 t0 dt t!0 0.964 g/cm3

QA RTAm a1 t
 Poly (vinyl acetate), glassy 355.00 5000 0.071
0 m0Af e t: 4:3:37
V f m a
After Paul (1971).
268 Separation in a closed vessel

Table 4.3.2. Kinetica sieving diameter of a gas/vapor based on the The basis for the wide variation in the mobility selectivity
smallest zeolite window where it can fit between a rubbery and a glassy polymer in Table 4.3.1
should now be clearer.
Molecule di, kinetic sieving diameter (nm) Figure 4.3.4 illustrates the solubility coefficient Sim (also
He 0.26
called the sorption coefficient) of a number of vapors and a
H2 0.289 few gases in a natural rubber membrane against the molar
NO 0.317 volume of the vapor/gas species. Generally, the larger the
CO2 0.33 molecule, the more condensible it is. Correspondingly, its
Ar 0.34 solubility in the membrane material is also higher. There-
O2 0.346 fore, a larger species which has higher condensibility, and
N2 0.364
therefore higher solubility in the membrane, will have
CO 0.376
CH4 0.38 higher solubility selectivity with respect to a smaller species.
C2H4 0.39 The general characteristics of the solubility selectivity
C3H8 0.396 and the mobility selectivity, and therefore the membrane
n-C4 0.43 selectivity, of gas species A over B through a polymer
CF2Cl2 0.43 membrane as described above can be given in a more
C3H6 0.44
quantitative basis as follows (Michaels and Bixler, 1968).
CF4 0.45
i-C4 0.47 First, consider the solubility coefficient Sim of a gas species i
in a polymer membrane. Jolley and Hildebrand (1958)
a
Breck (1974). have shown that the Henrys law constant for a lot of gases
in simple organic liquids at a reference temperature T0 can
mobility and solubility selectivity to the overall selectivity be correlated with the LennardJones force constant i for
AB for the gas mixture of helium (A)methane (B) through the gas i by an equation of the form
a variety of polymeric membranes (Paul, 1971).
ln Sim, T 0 ai =kB ln bm ;
Due to the smaller size of helium compared to 4:3:43a
Sim, T 0 bm expai =kB ,
methane (Table 4.3.2), the diffusivity of helium is higher
than that of methane through all membranes identified in
where the quantity a depends on the temperature, k B is the
Table 4.3.1. Therefore, the diffusivity selectivity (DAM/DBM)
Boltzmann constant and bm is a constant dependent on
is greater than 1 in all cases. On the other hand, we know
the solvent. This equation has been found to describe the
from Section 3.3.7.3, that the more condensable the gas
gas solubility coefficients Sim, T 0 in amorphous polymers
(higher Tc), the higher its solubility coefficient. Since
also, where polymergas interactions may be neglected.
methane is more easily condensable, (SAM/SBM) is less than
Further, for the solubility of such gases in amorphous
1; further, the value of this ratio is independent of the
polymers, the vant Hoff equation,
polymer material, whereas (DAM/DBM) depends very much
on the structure of the polymer. dlnSim H s
, 4:3:43b
Generally, the larger the size or molar volume of the d1=T R
gas/vapor molecule, the more it is impeded in its motion
has been found to be valid, just as in the case of gas
by the polymer chains in a polymer membrane. Thermal
solubilities in organic liquids (see equation (4.1.10)). Here,
and other motions of the chains, creating openings in the
Hs, the enthalpy of the solution of gas i in polymer m, is
membrane through which gas/vapor species can diffuse
related to the LennardJones constant i via
strongly, influence the value of Dim of any species i. For
polymers that are glassy at a given temperature T (<Tg of H s b0m a0m i =k B , 4:3:43c
the polymer), the mobility of the gas/vapor molecules, and 0
where a0 m and b m depend on the polymer. One can now
therefore of Dim, is considerably reduced. An increase in
combine (4.3.43a,b,c) to obtain
the size of the gas/vapor molecule brings about a sharp
drop in Dim. On the other hand, the polymer chains in a b0 a0 i =kB T0
 
rubbery polymer at a given temperature T (> Tg of the lnSim =Sim, T 0 m m 1 : 4:3:44a
R T0 T
polymer) are very flexible and offer much less hindrance to
the permeation of gas/vapor molecules. Therefore, Dim The solubility selectivity of a polymer for two gases A and
values are much larger for any given species in a rubbery B at T0 may be obtained from (4.3.43a) as follows:
polymeric membrane; further, the effect of a change in the 
SAm, T 0

A B

size of the gas/vapor molecule is much less. These types of exp a : 4:3:44b
SBm, T 0 kB
behaviors are illustrated in Figure 4.3.3 for a wide variety of
gases and vapors for a rubbery polymer such as natural Thus, the solubility selectivity as a first approximation
rubber and a glassy polymer such as polyvinyl chloride. depends only on the gas pair, and does not depend on
4.3 Equilibrium separation: membrane, closed vessel 269

104
He
H2
He O2 N2 CO2

106 H2 A C2H4
CH4 nC4 nC5

Infinite dilution diffusion coefficient, D0 (cm2/s)


C2 H6 nC3
O2 SF6 Bz
H2O N2 Natural rubber
108 CO2

A CH4
Kr
1010
MeOH
C2H6
VCM

1012 Polyvinyl chloride


EtOH Acetone

nC3OH nC4
nC5
1014 Bz
nC4OH
nC6

1016
0 20 40 60 80 100 120 140 160 180
Van der Waals volume (cm3/gmol)

Figure 4.3.3. Dependence of diffusivity on gas/vapor molecular size (Chern et al., 1985). Reprinted, with permission, from Material
Science of Synthetic Membranes, ACS Symposium Series 269, 1985, p. 492, D. Lloyd (ed.), R.T. Chern, W.J. Koros, H.B. Hopfenberg and
V.T. Stannett, (Chapter 2 authors), Figure 1 on p. 28 of Chapter 2. Copyright (1985) American Chemical Society.

the polymer membrane. However, if we use relation window through which a gas molecule can fit, is
(4.3.44a), we obtain for solubility selectivity, at any tem- considered to be quite appropriate (Table 4.3.2).
perature T, As in conventional theories of activated-state-
based diffusion in liquids, diffusion of a gas molecule
SAT 0 a m B A =kB
0
SAm T0
     

exp 1 : through an amorphous polymer membrane is assumed


SBm SBT 0 RT 0 T
to be an activated process; it involves the cooperative
4:3:45 movements of the gas molecule and the local polymer
There is a weak dependence on the membrane. chain segments around it. Correspondingly, the tempe-
Consider now the diffusion coefficient of a gas species i rature dependence of Dim depends on an Arrhenius
through a polymer membrane for the purpose of determin- relation,
ing the mobility selectivity. The diffusion coefficient of a dlnDim ED
gas i at a reference temperature T0 has been found to vary i, 4:3:46b
d1=T R
with the effective diameter di of the gas species as
(Michaels and Bixler, 1968) which implies

ED 1 1


DiT 0 g m exp f m dni , 4:3:46a Dim DiT 0 exp i , 4:3:46c
R T T0
where gm and fm are characteristic of the polymer forming
the membrane; n has the value of 2 for rotationally where the activation energy, E Di , has been related to the
hindered stiff-chain macromolecules such as polypropyl- effective diameter di of the gas species i via
ene, poly(ethylene) terephthalate, cellulose, polyimide, E Di E Dm f 0m d ni : 4:3:46d
etc.; n is 1 for flexible-chain polymers such as natural
rubber, polyethylene, etc. The estimate of di based on the Here, EDm and f 0m are constants, which depend on the
kinetic sieving diameter, which is the smallest zeolite polymer. Thus, Dim may be expressed as follows:
270 Separation in a closed vessel

(   
0.4 i
Qim Dim Sim g m bm exp a B f m d ni
k
)
T 0 E Dm f 0m d ni b0m a0m i =k B
 
C5H12 1 : 4:3:49
T RT 0
0.3
For a binary gas pair i A and B, the ideal selectivity
AB j t!0 AB displayed by the membrane is therefore
Sorption Natural
(
QAm A B
coefficient rubber

AB AB jt!0 exp a f m dnA d nB
(cm3(STP)/ 0.2 QBm kB
cm3-cmHg)
 f 0 d n dn a0 A B )
T0 m A B m

1 kB : 4:3:50
C4H10 T RT 0

0.1 A few comments can now be made about the effect of some
Iso-C4H10
C2H2 important quantities on the ideal selectivity between species
CH4
C3 H8 A and B, where dA is, say, smaller than dB and A =k B is
N2
C2H6 larger than, say, B =kB (Michaels and Bixler, 1968):
0 (1) For a smaller gas A (dA < dB), other items remaining
0 40 80 120 160 200
He Van der Waals molar volume favorable, AB > 1; i.e. the smaller molecules of species
H2 O2 (cm3/mol) A are more permeable than the molecules of species
B. The membrane is A-selective.
Figure 4.3.4. Henrys law sorption coefficient of vapors and gases (2) For a more condensable gas, which therefore has a higher
as a function of molar volume for natural rubber membranes. critical temperature (say, TcjA > TcjB), the selectivity
From Figure 3, p. 361, in Membrane Separation of Organic AB > 1; i.e. the membrane is more selective to species
Vapors from Gas Streams, R.W. Baker and J.G. Wijmans, A. This is so since i =k B is roughly proportional to the
Chapter 8 in Polymeric Gas Separation Membranes, D.R. Paul critical temperature (e.g. i =kB 0:77T c j i .
and Y.P. Yampolskii (eds.) 1994, CRC Press; reprinted by permis- (3) At the reference temperature T0, the ideal selectivity
sion of the publisher (Taylor & Francis Group, http://www. AB is simply expressed by
informaworld.com.)
Q

A B
AB jT 0 Am exp a f m dnA
dnB :

QBm B
T
k
0

E Dm f 0m dni T0 4:3:51
 

Dim g m exp f m dni exp 1 :


RT 0 T
At higher temperatures, the ideal selectivity AB j T will
4:3:47 be lower than AB j T 0 .
Consequently, the mobility selectivity of species A (4) Membranes fabricated from stiff-chain polymers,
over B through an amorphous polymer membrane is where n 2, have much higher selectivity AB than
given by membranes prepared from flexible-chain polymers,
where n 1.
f d n d nB
0
DAm T0
 

expf f m dnA dnB g exp m A 1 , Since different polymers have been observed to possess, for
DBm RT 0 T
a gas, permeability coefficients that vary over many orders of
4:3:48
magnitude, a question of general interest is: what happens to
which shows that the mobility selectivity depends almost the corresponding selectivities for a particular gas pair? We
solely on d nA d nB , i.e. on the relative difference between adopt here an approach suggested by Freeman (1999) to
the nth power of the effective diameters of two types of gas illustrate how AB is likely to vary with, say, QAm where A is
molecules, dA and dB, for a particular membrane. Further, the smaller gas molecule. Freeman (1999) may also be con-
it is much larger for stiff-chain polymers (n 2) compared sulted for an overview of the subject by the current author
to flexible-chain polymers (n 1). (following Michaels and Bixler (1968)), albeit with somewhat
One can now develop an illustrative expression for different notation and the literature, and also for numerical
the permeability Qim of species i through a membrane estimates of various quantities.
by combining expression (4.3.47) for Dim and relation First, the ideal selectivity AB may be described via
(4.3.44a) for Sim (Michaels and Bixler, 1968): equations (4.3.42) and (4.3.48) as
4.3 Equilibrium separation: membrane, closed vessel 271

ln AB ln SAm =SBm ln DAm =DBm Table 4.3.3. Permeabilitiesa of carbon dioxide and nitrogen
2 0 13 through various polymers at 30  C
f 0m @ T0
ln SAm =SBm d nA d nB 4 f m 1 A5
RT 0 T Film QCO2  1011 QN2  1011 CO2 N2 QCO2 =QN2
20 1n 3 2 0 13
dB f0 T0 Saran 0.29 0.0094 30.9
ln SAm =SBm 4@ A 15d nA 4 f m m @1 A5:
dA RT 0 T Mylar 1.53 0.05 30.6
Nylon 1.6 0.10 16.0
4:3:52 Pliofilm NO 1.7 0.08 21.2
Hycar OR 15 746 2.35 31.7
From expression (4.3.49) for the permeability coefficient Butyl rubber 518 3.12 17.4
Qim and expression (4.3.47) for Dim , we get Methyl rubber 75 4.8 15.7
Vulcaprene 186 4.9 37.9
ln Qim ln Sim ln Dim Hycar OR 25 186 6.04 30.9
8 0 19 Pliofilm P4 182 6.2 29.4
< E f d n
T =
Dm 0 Perbunan 309 10.6 29.1
ln Sim g m f m d ni m i @
1 A ;
: RT 0 T ; Neoprene 250 11.8 21.1
Polyethylene 352 19 18.5
ln Qim ln Sim g m Buna S 1240 63.5 19.6
0 1 8 0 19 Polybutadiene 1380 64.5 21.4
< = Natural rubber 1310 80.8 16.3
E Dm @ T 0A f T 0
1 d ni f m m @1 A :
RT 0 T : RT 0 T ; a
From Stannett (1968).
Units of Qi are scc-cm/cm2-s-cm Hg.
4:3:53

Introduce i A in this relation and substitute it into rela-


tion (4.3.52) to obtain
n

dB

E Dm

T0


1000 Upper bound


ln AB 1 ln QAm ln SAm g m 1
dA RT 0 T
ln SAm =SBm :

Rearrange it to get 100


H2N2
 n 
dB
ln AB 1 ln QAm
dA
(   n  10
dB
ln SAm =SBm 1
dA
)
E Dm T0
  
 ln g m 1 ln SAm : 4:3:54 1
RT 0 T
102 101 100 101 102 103 104
To understand the implications of this expression for H2 Permeability x 1010
AB , recognize that the solubility selectivity SAm =SBm for [cm3(STP)cm/(cm2 s cmHg)]
the gas pair AB changes very little with a change in the
polymer (see, e.g., Table 4.3.1); further, the variations of Figure 4.3.5. Relationship between hydrogen permeability and
H2/N2 selectivity for rubbery () and glassy () polymers and
the solubility coefficient of gases with different polymers
the empirical upper bound relation of Robeson (1991). (From
are very limited (thus, gm, EDm, etc., may not vary
Freeman (1999).) Reprinted, with permission, from Macro-
much). On the other hand, it is well known that Qim, as
molecules, 32, (1999) 375, Figure 1, Copyright (1999) American
well as AB , varies widely with the polymer. The relation
Chemical Society.
(4.3.54) developed above suggests that a plot of ln AB
against lnQAm  will decrease linearly with a slope of
d B =d A n 1 . In fact, the so-called upper bound line (shown by the solid line) for a H2N2 system for a wide
of Robeson (1991) observed for a variety of gas-pair range of polymers.
systems over a wide variety of polymers is well described Over a smaller range of permeability variation of a gas
by this type of suggested behavior (Freeman, 1999). The with a series of polymers, the data in Figure 4.3.5 appear to
gas pairs useful in this analysis are: HeH2; HeCO2; indicate that the separation factor may be almost constant.
HeN2; HeCH4; H2O2; H2N2; H2CO2; H2CH4; O2N2; If we consider flexible polymers (n 1) and equation
CO2CH4. Figure 4.3.5 illustrates the suggested behavior (4.3.50) at, say, T T0 (for simplicity), then
272 Separation in a closed vessel

A B (b) From equation (4.3.48), at T T0, the mobility selecti-


AB exp a B f m dA dB : 4:3:55 vity for the heliummethane pair for any membrane is
k
DHem =DCH4 m exp f m dnHe dnCH4 : 4:3:57
Since, for flexible polymers, the mobility based selecti-
vity plays less of an important role, it is likely that AB From Table 4.3.2, dHe 2.69 ( 0.269 nm) and
may depend primarily on the gas-pair AB. Table 4.3.3 d CH4 3:87 0:387 nm; further,
illustrates that, for a number of polymers, the selectivity cal 0:36 250  0:36 1
for the CO2N2 system varies by less than a factor of 2, f m 250 ;
2
mol
1:987 cal 1:987  298 2
298 K
whereas the permeability coefficient Qim varies by 5000 mol
K
times (Stannett, 1968). 0 1
A simplistic interpretation is provided by the following B 250  0:36 1 2C
DHem =DCH4 m exp @ 7:23 14:9 A
approach, where the permeability coefficient of species i 1:987  298 2
through a polymeric membrane m, Qim, is assumed to be
exp 1:165 3:205:
described as the product of three quantities:
The result quoted in Table 4.3.1 is 5.
Qim F m polymer Gi gas i, m, 4:3:56a

where i, m accounts for the specific interaction between


the gas and the polymeric membrane; however, Fm 4.3.4 Separation of a pressurized liquid solution
depends only on the polymer and Gi depends only on the through a membrane
gas species i. If i, m 1, then
Consider a liquid solution of a microsolute or a macroso-
Q lute under pressure (e.g. by a piston) on one side (side 1) of
AB Am GA =GB : 4:3:56b
QBm a membrane; let the other side of the membrane (side 2)
be empty at time t 0. As the solvent passes through the
The limited set of data in Table 4.3.3 lends support to this
membrane (porous/nonporous) under pressure, the other
simplistic explanation since the selectivity for the CO2N2
side will fill up, if it has a fixed volume. At this time, and
system does not appear to depend much on the nature of
afterwards, what will happen to the solute with respect to
the polymers listed.
the two chambers? If the membrane is semipermeable, i.e.
Example 4.3.1 it is impermeable to the solute but permeable to the solv-
(a) Determine the solubility selectivity of helium (A) and ent, then we have seen in Section 3.3.7.4, that, due to
methane (B) at T0, corresponding to 25  C for a osmotic equilibrium, the pressures on the two sides
polydimethylsiloxane membrane. Compare with the will be related to the osmotic pressure of the feed solution
value quoted in Table 4.3.1. The value of a in equation (side 1) on the side of the piston by
(4.3.43a) is 0.023 K1 (Freeman, 1999); (/kB) for He
and CH4 are 10.2 and 149 (K), respectively (Freeman, as2
V s P 1 P 2 V s 1 RT ln : 4:3:58
1999). as1
(b) Determine the mobility selectivity of a helium (A)
One side will have pure solvent at activity as2; the other
methane (B) pair at 25  C for a polydimethylsiloxane
membrane. Compare with the value quoted in Table side will have solvent at activity as1 as well as the solute,
4.3.1. Assume T T0, corresponding to 25  C; the and separation will be achieved at equilibrium. Note: P1 >
value of fm in equation (4.3.46a) may be obtained from P2 and as2 > as1.
Freeman (1999) as Consider now a porous membrane for the process of
ultrafiltration separation of a protein solution (Section
1 a cal 1 0:64
fm c 250 2 : 3.4.2.3). Suppose the regions on the two sides of the mem-
RT mol- RT
brane have become filled with liquid (from the feed region
Solution (a) From equation (4.3.44b), the solubility selecti- 1 to product region 2). If the membrane does not reject the
vity for the gas pair helium (He)methane (CH4) is protein (a macrosolute) completely, the solute protein will
SHe, T 0 aHe CH4 slowly diffuse from region 1 to region 2. Ultimately, both
   
exp B exp 0:023 K 1 10:2 149K sides will have the same protein concentration. The separ-
SCH4 , T 0 k
1 1 ation achieved initially will be lost. Therefore, in practice, it
exp 0:023  138:8 0:041:
exp3:192 24:23 is necessary to have an open system so that any permeate
Therefore, the solubility selectivity of a HeCH4 pair at T0 T
appearing from the feed side into the permeate side is imme-
25  C is 0.041. Table 4.3.1 quotes a value of 0.075. For other diately withdrawn. The same argument is equally valid if the
polymers, such as like natural rubber, the values are close to membrane in reverse osmosis (Section 3.4.2.1) has some
0.041. permeability of the microsolute present in the feed side.
Problems 273

Problems
4.1.1 Dilute waste streams from a chemical plant contain low concentrations of a variety of volatile organic
compounds. We would like to know the effectiveness of air stripping of such waste streams at ambient
temperature and pressure. Contaminated groundwater at many sites may also have very low concentrations
of a variety of organic compounds. The groundwater is pumped to an air stripper and then pumped back into
the ground by what is known as pump-and-treat processes. We would like to know the effectiveness of such
processes for removing volatile organic compounds having similar vapor pressures. Consider two sets of such
compounds:
(1) n-hexane and chloroform;
(2) 1-octene and tetrachloroethene.
It is known that, in each set, the first compound is more hydrophobic and will have much higher activity
coefficients. The vapor pressures of n-hexane, 1-octene and chloroform at 25  C are 0.2, 0.029 and 0.258 atm,
respectively. The 25  C activity coefficients for air-stripping conditions are: n-hexane, 5  105; 1-octene,
2.3  106; chloroform, 7.98  102. The following property values are also available: K i for chloroform and
tetrachloroethene are 2.1  102 and 1.5  103, respectively. (Data from Hwang et al. (1992b).) Obtain the
air-stripping separation factors for the two sets of compounds. Comment on the strippability of individual
compounds in each set.

4.1.2 The wastewater stream from a pharmaceutical plant on two different days had two different sets of pollutants:
day (1) benezene and n-hexane;
day (2) chloroform and 2-propenal (acrolein).
They are removed by air stripping at 25  C. For the wastewater of each day:
(a) Which compound on a given day is more easily stripped?
(b) What is the separation factor between the two volatile organic compounds on a given day?
The vapor pressures at 25  C are: benzene 0.125 atm; n-hexane 0.2 atm; chloroform 0.258 atm;
2-propenal 0.361 atm. Also K i at 25  C are, for chloroform, 2.1  102, and, for benzene, 3  102. The activity
coefficients at 25  C under air stripping conditions are, for n-hexane, 5  105, and, for 2-propenal, 2.16  10.

4.1.3 One step in the preparation of ultrapure water needed for various applications involves the removal of dissolved
gaseous species 1 and volatile organic compound 2. This removal is to be implemented using N2 and/or
vacuum to strip species 1 and 2. Develop an expression for the separation factor between species 1 and 2 in
terms of the thermodynamic constants and temperature-dependent physical properties of the two compounds
1 and 2. Assume nonideal behavior in the liquid phase, an infinitely dilute solution of each species and ideal
gas behavior.
4.1.4 For the determination of steam/air stripping based separation of sparingly soluble hydrophobic volatile organic
compounds (VOCs) i 1, 2 from water, K i , an infinite dilution vaporliquid equilibrium ratio for any species i
is quite useful. Consider two such species having nearly the same vapor pressure at the stripping temperature.
x
Show that, in such a case, if the two species are liquids at the stripping temperature, 12 x 2w=solubility
1w=solubility
:
(Hint: To determine il , consider an equilibrium between the pure organic phase of species i and water.)
4.1.5 Consider the simple vaporliquid equilibrium in the following binary systems:
(1) N2O2 at 1 atmosphere and 90 K;
(2) benzenetoluene at 1 atmosphere and 100  C;
(3) ethylbenzenestyrene at 0.133 atmosphere and 80  C.
Calculate the mole fraction of each species in the vapor phase and liquid phase, respectively, as well as 12,
assuming ideal liquid solution and ideal gas behavior. The following information is available: (1) the vapor
pressures of N2 and O2 at 90 K are 3.53 atm and 750 mm Hg, respectively (Ruhemann, 1949); (2) the vapor
pressures of benzene and toluene at 100  C are 1.78 atm and 0.73 atm, respectively; (3) the vapor pressures of
ethylbenzene and styrene are 0.166 atm and 0.119 atm, respectively. State your assumptions.
4.1.6 A vapor mixture of 7% methane, 8% ethane, 20% propane and 65% n-butane is present at 32  C in a closed
vessel.
(a) Determine the dew-point liquid composition and pressure using the Ki-factor charts.
(b) Determine the bubble-point vapor composition and pressure using the Ki-factor charts.
274 Separation in a closed vessel

4.1.7 (a) In a closed vessel, mtf moles of a mixture of n volatile species are present in vaporliquid equilibrium. If mtv
and mtl are the total number of moles present in the vapor and liquid phases, respectively, and if Ki is defined
as K i x iv =x i , obtain the following relations:
n
X
x if K i x if K i
x iv mtv ; mtv 1:
1 K i 1 i1 1 K i 1
mtf mtf

The temperature is given as T  C.


(b) Consider the feed mixture having the following composition in mole %: methane 7%, ethane 8%, propane
20% and n-butane 65%. It is present in a closed vessel at 32  C and 1000 kPa. What fraction of this mixture is
present in the vapor phase? Obtain the vapor phase composition.
4.1.8 An organic feed mixture present at 80  C and 1 atm consists of 40 mol% acetone, 30 mol % acetonitrile and 30
mol % toluene. Assume that the liquid phase is an ideal solution and that the vapor phase behaves as an ideal
gas. Further, the system has both liquid and vapor phases. Caculate the fraction of the feed present in the vapor
phase and the compositions of the vapor phase and the liquid phase. The P sat i values at 80  C for acetone,
acetonitrile and toluene are 196 kPa, 98 kPa and 39 kPa, respectively.
4.1.9 Consider a continuous chemical mixture present in the vapor phase. Its molecular weight density function is
known to be fv(M). Determine the expression of fl(M) for the first liquid drop formed as the vapor is cooled to
temperature T. Make appropriate simplifications for low-pressure gas and ideal liquid solution.
4.1.10 We have observed in Chapter 3 (equation (3.1.25a)) that the reversible work needed to transfer 1 mole of
species i from state 1 to state 2 is equal to the difference between the values of the partial molar free energy at
the two states: Gi j2 Gi j1 i j2 i j1 . Consider the solution of an impurity B in a solvent A. Comment about
the possibility of obtaining a pure solvent with respect to the energy required.
4.1.11 Markham and Benton (1931) studied experimentally the adsorption of pure O2 and pure CO as well as COO2
mixtures on 19.6 g of silica at 100  C. The data obtained are provided in Table 4.P.1.
Develop a plot of x O2 g vs. x O2 for the O2CO mixture adsorption on silica at 100  C at a total pressure of 1
atmosphere using the ideal adsorbed solution theory. Plot the three experimental points for the mixture in the
same figure.
4.1.12 Consider the following cation exchange process:

2NaR CaCl2 aq , CaR2 NaClaq,

where R represents the resin phase. Develop an estimate of the activity-based separation factor between the
sodium ion and the calcium ion in terms of the activity ratio of the resin phase to that of the water phase for one
of the two cations. Neglect swelling effects.

Table 4.P.1.

Pure oxygen at 100  C Pure CO at 100  C COO2 mixtures at 100  C

Volume of oxygen Volume of carbon


Volume adsorbed Volume adsorbed adsorbed monoxide adsorbed

Press. (mm) Obs. (cc) Press. (mm) Obs. (cc) pO2 pCO Isotherma Observed Isotherma Observed

17.9 0.08 26.3 0.26


88.6 0.54 127.9 1.37 210.3 549.7 1.45 1.56 5.50 5.27
133.9 0.90 224.4 2.36 286.4 473.6 1.95 1.96 4.80 4.40
221.9 1.52 321.0 3.37 335.7 424.3 2.30 2.33 4.34 4.05
329.0 2.19 434.5 4.42
417.1 2.87 537.2 5.30
471.4 3.18 639.4 6.33
570.5 3.92 760.0 7.40
653.2 4.38
760.0 5.01
a
Contains values obtained from pure component adsorption isotherms via interpolation.
Problems 275

4.1.13 Consider a cation exchange resin in an aqueous solution containing Na2 SO4 . Obtain the selectivity of the resin
for the sodium cation over the sulfate anion if the activity based distribution coefficient of the anion between
the resin and the external solution is 0.001. (Ans. 31, 630.)
4.1.14 Consider two anions Cl and SO4 and an anion exchange resin. If the activities of both anions in the aqueous
solution are equal, determine which anion will be preferred by the resin phase.
4.1.15 The activity based separation factor aAB between two counterions A and B in an ion exchange resin system has
been defined by (4.1.70). Assume an ideal system.
(a) Show that, for an ideal solution in both phases, the separation factor AB based on mole fractions is equal
to the separation factor m
AB based on molalities.
(b) Redefine the expression (3.3.121e) for the law of mass action based equilibrium constant for an ion
exchange reaction between counterions A and B for ideal solutions as

mA, R jZ B j mB, w jZ A j
Km
AB :
mA, w jZ B j mB, R jZ A j
m jZ A j
Show that K m
AB AB when ZA ZB.
(c) Show that in ideal solutions, where V A V B , the value of K m m
AB 1 when ZA ZB. The quantity K AB has
been defined as the electroselectivity or selectivity coefficient (Helfferich, 1995). Any nonideality and
specific preference for an ion by the ion exchanger leads to ln K mAB > 0 when ZA ZB.

4.1.16 Define the equivalent ionic fraction x ij for i A, B, a system of two counterions A and B in phase j w, R, by
Z A mA, j
x ij :
Z A mA, j Z B mB, j
Develop the following general relation between x ij and K m ij defined in part (b) of Problem 4.1.15:

x AR jZ B j=jZ A j x Aw jZ B j=jZ A j Z A mA, R Z B mB, R


  
1
K m
AB
jZ A j
,
1 x AR 1 x Aw Z A mA, w Z B mB, w

where K m
AB is the selectivity coefficient between counterions A and B.
Rewrite this result in terms of mF , R and i Z i mi, w , where mF , R is the molality of fixed charges in the resin
P

phase.
4.1.17 Calculate the equivalent ionic fraction x iR of sodium and magnesium ions in a cation exchanger whose
selectivity coefficient K m
AB is unity due to ideal behavior. (See problem 4.1.16.)
Assume mF , R 9  103 mequiv:=1000 g H2 O of fixed ionic groups. The total molality of ions in solution is
11 mequiv./1000 g H2O; the salts NaCl and MgCl2 are present in equal equivalent amounts (assume
x Mgw x Naw 0:5 ).
(Hint: Employ the results obtained in Problem 4.1.16.) (Ans. x MgR 0:9756.)

4.1.18 Chimowitz and Pennisi (1986) have measured the supercritical phase ( j ) mole fractions of
1,10-decanediol (species i d) and benzoic acid (species i b) in supercritical CO2 at two temperatures,
318 K and 308 K for a number of pressures. The results from their extensive experiments are summarized
in Table 4.P.2.

Table 4.P.2.

Pressure (bar) Temperature (K) x d  104 (mole fraction) x b  103 (mole fraction)

306.8 318 5.335 4.84


308 3.064 3.874
228.5 318 4.107 3.843
308 2.542 3.246
163.8 318 3.411 2.755
308 1.814 2.338
132.2 318 2.03 1.72
308 1.53 1.79
276 Separation in a closed vessel

TH
TH

C TH TL
TL
TL
TL

TH
TH

TH

Mole fraction
TL
B TL TL

TL TH

TH
TH
TL TL
A TL
TH
TL
TH

TH P1 P2 P3

Pressure

Figure 4.P.1. Crossover behavior for a ternary system.

Determine the solid phase yield (defined as the ratio of moles of solid deposited/mole of CO2 in feed) of
each of the two species, as well as the purity enhancement factor10 of 1,10-decanediol in the deposited solid
phase when the temperature is reduced from 318 K to 308 K at each one of the four pressures identified above.
4.1.19 The separation of a pure solid from a binary solid mixture extracted by supercritical CO2 via cooling of the
mixture in the crossover region has been experimentally demonstrated by Chimowitz and Pennisi (1986).
Suppose there is a ternany solid mixture of three species A, B and C. If their solubility behavior in the crossover
region is as given in Figure 4.P.1, describe the process sequence you will have to follow to get deposits of pure A,
pure B and pure C in different vessels under appropriate conditions.
4.2.1 Calculate the separation factor for each of the following gas mixtures: (1) SO2N2, (2) U235F6U238F6, present in a
gas centrifuge of radius 4 cm rotating at a high angular velocity, 4000 rad/s at 20  C. The two regions are
r2 4 cm and r1 0 cm. (Ans. N2 SO2 1:207; U235 U238 1.0157.)

4.2.2 The mole fraction of species 1 in a binary gas mixture of species 1 and 2 introduced into a gas centrifuge is x1f.
After the gas centrifuge is rotated at an angular velocity of rad/s, a concentration profile is developed for each
species in the radial direction. If the centrifuge radius is r0, indicate a procedure to determine the mole fraction
profile, x1j(r).

4.2.3 Calculate the maximum number of macromolecular species whose peaks may be clearly resolved in isopycnic
sedimentation if the cell length lz is 1.5 cm. For the purpose of calculations, use the data provided in the
example given in the text to calculate the resolution (Rs) for the isotopically labeled and unlabeled DNA
molecules in a CsCl gradient; the density gradient is 0.08 g/cm4. You may use the values of Rs or
developed/used in the text calculations for an estimate of the peak width. How does this number change if
the Rs value is 1.5? (Ans. nmax 18:25; nmax 13.)

10
Purity enhancement factor ratio of mole fraction of decanediol/benzoic acid in the deposited solid phase to that in the feed phase.
Problems 277

4.2.4 A sedimentation cell has a length of 1 cm; it is located 6.0 cm from the rotation axis (r1 6.0, r2 7.0 cm).
A macromolecular solute of molecular weight 105 is rotated at 25  C at an angular speed of rad/s. When the
rotational speed is 5000 rpm, the concentration of the macromolecule in the solution varies from a maximum to
1% of the maximum over the cell length.
(1) Determine the rotational speed (rpm) which will concentrate the macromolecule to the same extent, over
a distance of only 0.01 cm from the tip of the cell (r 7.0 cm).
(2) Determine the ratio of the macromolecular concentration between the two locations r2 7 cm and r1
6.5 cm for the macromolecular solute when the rotational speed is 5000 rpm. Assume R 8.315  107 erg/
gmol-K. (Ans. (1) 48 180 rpm; (2) 2.54.)
4.2.5 The Gaussian concentration profile of protein species i around location z0 in a cell used for isoelectric focusing,
where the pH is equal to the pI of the protein, can also be determined by a balance of the diffusive flux of the
protein, leading to band broadening, and the flux, due to the uniform electrical force field E forcing the protein
to concentrate at z0. Obtain the result (4.2.38a), employing appropriate assumptions.

4.2.6 Develop an expression for the maximum number of proteins whose peaks may be clearly resolved via
isoelectric focusing with a value of resolution Rs 1 in a cell of length z . The values of pH at locations z
and 0 are pH and pH0, respectively. The variation of Zi with pH will be incorporated as (dZi/dpH).
 !
F E dZ i =dpH z pH pH 0 1=2

Ans: nmax :
16 RT

4.2.7 In a natural gas well assumed to consist of methane and butane only, the mole fraction of butane at 3055 m
below the surface is 0.16. Calculate the value of the butane weight fraction in the gas well at the surface level,
assuming ideal gas behavior and isothermal conditions. (Ans. 0.69.)
4.2.8 Measurements of isotopic ratios or ratios of noble gases (84Kr and 36Ar) in Greenland ice show that (Craig and
Wiens, 1996) gravitational separation in the unconsolidated firn layer above the ice is responsible for the
observed enrichments relative to atmosphere ratios. Define R to be the ratio of the mole fraction of noble gas 1
over the noble gas 2 at a height z1, and let R0 be the corresponding ratio in free atmosphere at the surface z2
(>z1). Obtain, as a first order of approximation, the following expression:
" , ! #
R x 11 x 12 M 1 M 2 z2 z1
  
3
1 10 1  103  1:18,
R0 x 21 x 22 T

where region 1 corresponds to z1 (meters) and region 2 corresponds to z2; species 1 is heavier than species 2. If
(z2z1) ~ 70 m, and species 1 and 2 are 84Kr and 36Ar, respectively, obtain an estimate of when the ice
temperature is 20  C. (Ans. 84 Kr=36 Ar 15:6.)

4.3.1 The extent of solubilization of a hydrophobic solute present in water into a micelle is determined by
semiequilibrium dialysis (SED) techniques. This technique employs a dialysis cell (see Figure 4.3.1(a)), where
the pore size of the dialysis membrane completely rejects the spherical micelle. However, the solute must be
able to pass freely through the membrane. The surfactant monomer may also pass freely through the
membrane. On addition of surfactant at a concentration of Csur mol/liter to the chamber on one side of the
membrane to which C0i mol/liter of solute species i has been added, binding of the solute to the micelles will
occur if the surfactant concentration exceeds the CMC for the surfactant.
(a) Identify what will happen after a sufficient time has been allowed to lapse.
(b) After equilibrium has been established, the free solute concentration in the solution chamber on the other
side of the membrane has been determined to be Cip mol/liter The volumes of the two chambers are Vp
and Vf liter, respectively, where subscripts p and f refer to the permeate side and the feed side (where
solute was introduced at concentration C0i ), respectively. Determine the number of moles of solute i
which have been solubilized into the micellar phase.
(c) Assume now that Csur mol/liter >> C0i mol/liter. Further, assume that Csur mol/liter is >> [CMC] and that
the moles of surfactant in micelle >> free surfactant moles. Determine the micellar equilibrium ratio, Kim,
defined as K im x im =x if , where xim and xif are the mole fractions of i in the micellar phase and the feed
chamber, respectively. Assume that Vp Vf 0.05 liter.
278 Separation in a closed vessel

4.3.2 Cuprophan membranes made from regenerated cellulose are frequently used in hemodialysis. Model this
membrane as one consisting of cylindrical capillaries of radius 18 . Determine the separation factors of the
dialysis membrane for two solutes, urea and vitamin B12. The characteristic radii of urea and vitamin B12 are
2.8 and 8.5 , respectively. The diffusion coefficients of urea and vitamin B12 at infinite dilution in isotonic
saline at 37  C are 1.81  105 and 0.38  105 cm2/s, respectively. Compare the result for the given pore size
estimate with that obtained from the data of Colton et al. (1971), namely 16 (based on effective diffusion
coefficients without any consideration of equilibrium partition coefficients in their Figure 6). (Ans. 23.5.)

4.3.3 Through a microporous denitrated cellulose membrane, the separation factor for two solutes, sodium sulfate
and sucrose, has to be determined for the case of simple diffusion with no convection. The properties of the
membrane, the operating conditions and the solute properties are available in Example 3.4.5.

4.3.4 In Donnan dialysis based removal and concentration of Cu2 ions from a wastewater into an acidic strip
solution through a cation exchange membrane, the equilibrium pH of the strip solution is 1 and the equilibrium
pH of the feed solution is 4. Determine the concentration ratio of the Cu2 ions in the two chambers. The
membrane-impermeable anion in both chambers is SO24 . (Ans. Strip solution concentration is 106 larger than
that in the feed.)

4.3.5 In seawater desalination processes by thermal evaporation or a membrane based technique, the hardness
of the seawater in terms of calcium carbonate or sulfate (also MgCO3, MgSO4) is a problem, which leads to
scaling of evaporator surface or membrane fouling. It has been suggested that one could employ Donnan
dialysis using the concentrated seawater from the process as the strip solution to reduce substantially the
Ca/Mg content of the seawater, which is to undergo desalination by a thermal or a membrane process.
If the strip solution Na concentration is twice that of the seawater to be subjected to desalination, identify
the type of membrane and the maximum possible extent of reduction of CaSO4 from that present in the
feed seawater.

4.3.6 Develop an expression (in terms of known quantities) for the molar concentration of ion Na in a cation
exchange membrane at the membranesolution interface, where the salt Na2SO4 is present at a molar concen-
tration of C 0sw in the feed solution. The molar fixed charge density in the membrane is Cm. Make appropriate
assumptions.

4.3.7 A mixture of CO2 and CH4 containing 20 mol% CO2 has been introduced into one side of a synthetic membrane
in a closed vessel; the other side of the membrane is empty to start with. We would like to know the
composition of the gas mixture that appears first on the other side of the membrane. The permeability
coefficients for CO2 and CH4, QCO2 and QCH4 , are provided in units of Barrers as QCO2 15, QCH4 0:5.
(Ans. x CO2 , p 0:882:)

4.3.8 In molecular distillation, a liquid mixture of A and B is distilled at a very low pressure of 0.001 mm to prevent
heat-sensitive materials from spoiling. At such low pressures, if we are dealing with a pure liquid, the flux of the
species escaping the liquid surface is given by equation (3.1.119a):

P sat
A
N A p
,
2RTM A
where P sat
A is the vapor pressure of pure A of molecular weight MA. The escaping vapor molecules condense on
another surface close by and do not come back to the liquid. For a mixture, the vapor pressure is to be replaced
by the corresponding partial pressure. Obtain an expression for the separation factor AB for species A and B in
a liquid mixture subjected to molecular distillation with the product liquid mixture obtained from the
condensing surface, assuming ideal behavior.

4.3.9 A feed gas mixture of species A and B at atmospheric pressure is exposed to a microporous membrane of
thickness m and pore radius rp. The permeate side is under vacuum at time t 0. Determine the value of the
membrane selectivity AB as t ! 0 in a closed vessel, with the feed gas mixture at total pressure P at time t 0.
Assume Knudsen diffusion exists. The membrane porosity is m .

4.3.10 Consider two gases A and B being separated through a microporous carbon membrane. The gas transport may
be described as that by Knudsen diffusion through the microporous membrane as well as by surface diffusion of
the adsorbed gas layer on the pore surface (3.4.112).
(1) Determine the value of AB as t ! 0 in a closed vessel in the manner of Section 4.3.3.
Problems 279

(2) Speculate what will happen if one of the species (between A and B, for example H2 and NH3) is
condensable and forms a multilayer condensate on the pore wall, blocking gas-phase transport.

4.3.11 The solubility Sim of a gas species i in a polymer has been empirically expressed as

ln Sim M 0:016 T ci ,

where M depends on the polymer; the value for M for the polydimethylsiloxane polymer at 30  C is 8.8.
The diffusivity of a gas species i has been correlated as

Dim =V ci ,

where the value of for the polydimethylsiloxane (PDMS) polymer is 1.5 and has the value of 2.6  102.
Here, Tci and Vci are the critical temperature and the critical molar volume of gas species i. Calculate
the solubility selectivity, mobility selectivity and the ideal selectivity for a PDMS polymer membrane
for the two gas pairs O2N2 and CO2N2. Assume Tci (K) 154.6 (O2), 126.2 (N2), 304.2 (CO2).
Ans: SO2 m =SN2 m 1:575; SO 2 m =SN 2 m 17:24; DO 2 m =DN 2 m 1:35;DCO 2 m =DN 2 m 0:929:

4.3.12 You are in a boat on a lake of brackish water. To recover water from the brackish water (salt concentration
~100010 000 ppm) of salt concentration Csalt into a concentrated sugar solution C 0sugar , you have a forward
osmosis device separating the two solutions by a perfect semipermeable membrane. In forward osmosis, water
from the salt solution with an osmotic pressure lower than that of the concentrated sugar solution will permeate
to the sugar solution. The initial volume of the concentrated sugar solution is V 0sugar . We know that the osmotic
pressure of each solution is defined as

sugar bsugar C sugar ;


salt bsalt C salt ,
where bsalt > bsugar. Determine the final volume of the sugar solution, Vsugar, that you can achieve and the
corresponding sugar concentration, Csugar. Assume that the membrane is perfectly semipermeable.
5

Effect of chemical reactions on separation

Chemical reactions occur in many commonly practiced rate-controlled equilibrium processes is treated in Section
separation processes. By chemical reactions, we mean 5.3. Section 5.4 illustrates how chemical reactions affect the
those molecular interactions in which a new species results separation in rate-governed processes using membranes and
(Prausnitz et al., 1986). In a few processes, there will be external field based processes; for example, electrochemical
hardly any separation without a chemical reaction (e.g. gas separation using membranes in an external electric field.
isotope exchange processes). In some other processes,
chemical reactions enhance the extent of separation con-
siderably (e.g. scrubbing of acid gases with alkaline absorb- 5.1 Extent of separation in a closed vessel
ent solutions, solvent extraction with complexing agents). with a chemical reaction
In still others, chemical reactions happen whether
intended or unintended; estimation of the extent of separ- Consider a closed vessel with two regions j 1, 2 and two
ation requires consideration of the reaction. For example, species i 1, 2. The extent of separation at any time t (or at
in solvent extraction of organic acids, the extent of acid equilibrium, as the case may be) can be written as follows:
dissociation in the aqueous phase at a given pH should be  
m11 t m21 t 
taken into account (Treybal, 1963, pp. 3841). Chemical t jY 11 t Y 21 t j  : 5:1:1
m01 m02 
equilibrium has a secondary role here, yet sometimes it is
crucial to separation. Suppose now that region 1 in the separation system has
Familiarity with the role of chemical reactions in sep- another species 3 with which species 1 reacts to produce a
aration processes will be helpful in many ways. This is fourth species 4, i.e.
especially relevant since a few particular types of chem- 1 3 , 4: 5:1:2
ical reactions occur repeatedly in different separation
processes/techniques. These include ionization reac- Then species 1 is present in region 1 not only as species 1,
tions, acidbase reactions and various types of but also as species 4. Assume that species 3 and 4 cannot
complexation reactions. The complexation reactions also go to region 2 and that species 2 does not react with either
include the weaker noncovalent low binding energy based species 3 or 4. If, at time t, the total number of moles of
bonding/interactions identified in Section 4.1.9. A better species 1 present as free species 1 and species 4 in region 1
understanding and quantitative prediction of separation in as a result of reaction is mr11 (t), the extent of separation
a given process is possible, leading to better process and with the reaction is
equipment design. In processes where a chemical agent is 
 mr t

used, different agents can be evaluated systematically. On  m21 t 
r t jY r11 t Y r21 t j  11 0 ; 5:1:3
occasions, it may facilitate the introduction of reactions to  m1 m02 
processes for enhancing separation.
where we assume that Y r21 t Y21(t) since the distribution
The relation between the extent of separation and
of species 2 is unaffected by the reaction. The difference
the extent of reaction is briefly considered in Section 5.1.
between the two extents of separation is given by
How chemical reactions alter the separation equilibria
 
in gasliquid, vaporliquid, liquidliquid, solidliquid, sur-  mr t m t  mr t m t 
 11  11 11
face adsorption equilibria, etc., is described in Section 5.2. r t t  11 0  : 5:1:4
 m1 m01  m01
The role of chemical reactions in altering the separation in
5.2 Chemical reactions change separation equilibria 281

For the reaction under consideration, the stoichiometric 5.2.1.1.1 Solute ionization in aqueous solution The
coefficient i for i 1 is 1. The molar extent of reaction is solubility of a gas in a liquid is often described by Henrys
traditionally defined for any ith species participating in a law (see equation (3.3.60b)). Thus the gas-phase partial
reaction as pressure of SO2, pSO2 , is related to its aqueous phase mole
fraction x SO2 by
the number of moles consumed or produced by the reaction
:
ji j pSO2 H SO2 x SO2 5:2:1
We note that the difference in (5.1.4) of the two extents of for ideal gas-phase behavior, where H SO2 is the Henrys law
separation is merely the molar extent of reaction of the ith constant for SO2 in water. However, it is valid only for
species normalized with respect to the total number of ith molecular SO2 in water. Dissolved SO2 in water ionizes to
species originally present in the system. Thus, the trad- some extent (Danckwerts, 1970):
itional formalism of reaction engineering or reaction kinet-
ics may be used to estimate directly the change in the SO2 H2 O , HSO3 H : 5:2:2
extent of separation due to a chemical reaction. Such a
Thus, SO2 is present in water also as HSO3 ; the total SO2
straightforward result may not be valid if some of the
concentration in water is given by
assumptions made earlier break down.
C tSO2 C SO2 C HSO3 gmol=liter:
 
5:2:3
5.2 Change in separation equilibria due The ionization equilibrium of (5.2.2) is described by1
to chemical reactions
C H C HSO3
The chemical equilibrium in a multiphase system and the Kd : 5:2:4
C SO2
corresponding species concentrations in chemical separ-
ations are altered when chemical reactions take place. The aqueous solution has no net charge anywhere (see the
We will illustrate this now for gasliquid equilibrium electroneutrality condition (3.1.108a)); therefore,
(as in gas absorption), vaporliquid equilibrium (as in
C HSO3 C H K d C SO2 1=2 : 5:2:5
distillation), liquidliquid equilibrium (as in solvent
extraction), stationary phaseliquid equilibrium (as in ion It is convenient to write Henrys law (5.2.1) for free SO2 as
exchange, chromatography and crystallization), surface
adsorption equilibrium (as in foam fractionation) and pSO2 H CSO2 C SO2 : 5:2:6
Donnan equilibrium.
But the relation between pSO2 and C tSO2 is of interest here,
where
5.2.1 Gasliquid and vaporliquid equilibria h i 00
C tSO2 C SO2 K d C SO2 1=2 ; pSO2 H CSO2 C tSO2 :
First we consider gasliquid equilibrium as in gas absorp-
5:2:7
tion/stripping.
We find
h i 00 1
5.2.1.1 Gasliquid equilibrium H CSO2 K
; 5:2:8
1
q
d
H CSO
H CSO P SO2
2 2
Gas mixtures are commonly separated in industry by h i00
absorption in a solution where chemical reactions may where H CSO2 is the pseudo-Henrys law constant to
take place (the solution is regenerated subsequently in a determine the total SO2 concentration in the liquid phase
separate vessel by stripping). The amount of gas absorbed h i 00
for a given pSO2 . As expected, when Kd 0, H CSO2 H CSO2 .
by simple physical absorption is often insufficient; it can be
increased considerably by incorporating in the solution Alternatively, the total molar concentration of SO2 in water
chemicals with which the dissolved gas undergoes a chem- is given by
ical reaction. Removal of acid gases, e.g. CO2, H2S, SO2, !1=2
COS, by alkaline solutions are well-known examples p 2 K d pSO2
C tSO2 SO : 5:2:9
(Danckwerts, 1970; Astarita et al., 1983). H CSO2 H CSO2
Aqueous absorbent solutions conventionally employed
to increase absorption of a species, e.g. CO2 from a gas,
may have a variety of reagents, e.g. K2CO3, Na2CO3,
NaHCO3, NaOH, monoethanolamine, diethanolamine,
etc. Even without a reagent, the absorption equilibrium of 1
We have generally avoided describing equilibrium constants for
a species may be affected by reaction. Absorption of SO2 any reaction using activities; therefore activity coefficients are
into water where SO2 is ionized illustrates such a case. absent in (5.2.4) as well as in most relations in this chapter.
282 Effect of chemical reactions on separation

Now H CSO2 (1/1.63) atm liter/gmol and Kd 1.7  102 may be represented by RNH2 and R2NH, respectively. The
gmol/liter at ordinary temperatures (~25  C) (Danckwerts, respective overall reactions with CO2 in an aqueous amine
1970). Suppose pSO2 0.01 atm, then C SO2 0.0163 gmol/ solution are as follows:
liter but C tSO2 0.033 gmol/liter, showing the considerable
effect of SO2 ionization in water on its total aqueous solu- primary amine CO2 2RNH2 , RNH COO RNH
3;
bility (Prausnitz et al., 1986). 5:2:13
The gas CO2 also ionizes in water in the manner of
(5.2.2), producing HCO3 and H. However, the equilib- secondary amine CO2 2R2 NH , R2 N COO R2 NH
2:
rium constant Kd for such a reaction is a couple of orders 5:2:14
of magnitude smaller than that of SO2. Hence, unless alka-
line conditions are maintained to shift the reaction to the Consider an aqueous solution of a primary amine in contact
right, the effect of CO2 ionization in water has a negligible with a gas containing CO2. The following equilibria are valid:
influence on CO2 solubility in water. The solubility of an
acid gas such as CO2 or H2S in water is, however, strongly CO2 g
affected by ionization if a basic gas like NH3 is present.
K
Ammonia ionizes in water as CO2 w 2RNH2 w , RNH COO w RNH
3 w :
5:2:15
NH3 H2 O , NH
4 OH , 5:2:10
The molar concentration based equilibrium constant K for
whereas CO2 ionizes as this aqueous-phase reaction is given by
C RNHCOO  C RNH3
CO2 H2 O , HCO3 H : 5:2:11 K : 5:2:16
C CO2  C RNH2 2
The H ions and OH ions are participants in the
At 20  C, its value is 1.1  105 gmol/liter (Danckwerts, 1970).
ionization
The concentration of free CO2 in water, C CO2 , is very small
when there is a substantial amount of free amine RNH2
H2 O , H OH , 5:2:12 present in water. Further, since the concentration of OH
is very much less than that of RNHCOO, by the electro-
where the equilibrium lies far to the left, since water has very
neutrality condition, C RNHCOO C RNH3 . The distribution
little tendency to ionize. Thus both reactions (5.2.10) and
coefficient of CO2 (species i) between the aqueous (j w)
(5.2.11) are pushed to the right, resulting in considerable
phase and the gas phase (j g) in the absence of the amine is
amounts of NH
4 and HCO3 . This means that the partial
pressure of CO2 (or NH3) needed for a certain total liquid- C iw
iw : 5:2:17a
phase concentration of CO2 (or NH3) is much reduced from C ig
that when only either CO2 or NH3 is present (Prausnitz et al.,
In the presence of a large amount of amine and the chem-
1986). A detailed analysis of the equilibria in a NH3CO2
ical reaction in water, the effective distribution coefficient
H2O system is available in Edwards et al. (1978). 00
iw based on the total CO2 concentration in water is
Conversely, if one tries to strip NH3 and CO2 from the
water with steam or air, one finds that the efficiency is 00 C iw C RNH COO C tiw
quite low compared to the cases when either CO2 or NH3 iw , 5:2:17b
C ig C ig
only is present. In fact, sour waste streams in refineries,
steel mills and coal conversion wastewaters are typical where Ciw is the molar free CO2 concentration in water.
examples of this difficulty. It has been suggested therefore Using the electroneutrality condition and the equilibrium
00
that one should extract NH3 by solvent extraction and constant relation in the above definition of iw , we find that
remove CO2 (or H2S) by steam stripping (Cahn et al.,
00 C iw K C CO2 1=2 C RNH2
1978; MacKenzie and King, 1985). Such a process has been iw ,
C ig C ig
called extripping (simultaneous extraction by a solvent and
stripping by steam or air). where C CO2 Ciw. Therefore,
" #
5.2.1.1.2 Reaction with an absorbent in solution We 00 K 1=2 C RNH2
iw iw 1 : 5:2:17c
now consider enhancement of gas absorption into a liquid C iw 1=2
by liquid-phase reactions with an absorbent species. One
of the common industrial absorbents for removal of CO2 From Danckwerts (1970, p.198), for example, some typical
from gas streams is an aqueous solution of a primary or a estimates for monoethanolamine at 20 C are C RNH2 1.75
secondary amine. A primary amine and a secondary amine gmol/liter, Ciw 4.1  107 gmol/liter. These values lead to
5.2 Chemical reactions change separation equilibria 283

CH3 H CH3 There is an added advantage to having a hindered


amine. For gas purification by absorption in an aqueous
amine solution in a continuous process with a fixed
H2N C C OH H3C C NH2 absorbent, the amine solution has to be regenerated by
heating and CO2 is stripped from the solution. Reactions
(5.2.13) and (5.2.18b) now go from right to left. In reaction
H CH3
CH3 (5.2.13), all of the absorbed CO2 cannot be stripped
because RNHCOO for an unhindered amine is stable. On
2-amino-2-methyl-1-propanol t-butylamine the other hand, there is no such problem in reaction
(5.2.18b), leading to much better stripping.
Figure 5.2.1. Examples of hindered amines. It is useful to consider reactive absorbents in the aque-
ous solution other than amines, e.g. K2CO3. In the hot
potassium carbonate process widely practiced for CO2 and
" #
103 1:75  107
1=2 H2S removal, the absorbent is simply K2CO3, which ionizes
00
iw iw 1  : 5:2:17d to give K and CO 3 . When CO2 is present in the gas phase,
3 4:11=2
we have dissolved CO2. Of course, HCO3 , OH and H ions
This illustrates the extraordinary increase in the effective are also present:
distribution coefficient of CO2 due to the chemical reaction
with a primary amine added to water. An alternative way to CO2 H2 O CO
3 2HCO3 , 5:2:19a
illustrate the same effect would be to use the partial pres-
sure pig of species i in the gas phase instead of Cig. which follows from the set
An even higher absorption of CO2 is possible if a hindered 
CO

3 H2 O , HCO3 OH

amine is used. Both reactions (5.2.13) and (5.2.14) indicate :
CO2 H2 O , HCO3 H
that two moles of amine are utilized per mole of CO2
absorbed and one mole of an amine carbamate RNHCOO Two moles of HCO3 are obtained per mole of CO2
is produced. This carbamate can hydrolyze as follows: absorbed. Depending upon the conditions, all or some of
the CO 3 introduced into the system may have reacted. In

RNHCOO H2 O RNH2 HCO3 : 5:2:18a the amine absorption example, we observed how a chem-
ical reaction increased CO2 absorption. Here we study how
An overall reaction in the case of carbamate hydrolysis for much of the chemical absorbent is being utilized in the
CO2 absorption may be written, using (5.2.13), as follows: reactive absorption; the fractional consumption f of the
chemical absorbent has been defined as the degree of
CO2 RNH2 H2 O RNH saturation (Astarita et al., 1983).
3 HCO3 : 5:2:18b
The equilibrium constant K for reaction (5.2.19a) is
Thus, for unstable carbamates, only one mole of amine is
C 2HCO3
utilized per mole of CO2 absorbed; more amine is available K : 5:2:19b
C CO2 C CO3
for CO2 absorption.
For conventional amine absorption processes, the ami-
Since 1 mole of CO 3 reacts with 1 mole of CO2, the total
nes used (monoethanolamine, diethanolamine, dimethy-
molar concentration of CO 3 introduced as absorbent,
lamine, etc.) are unhindered; a hindered amine has a bulky
C tCO3 , includes the CO3 concentration present at equilib-
alkyl group attached to the amino group as in Figure 5.2.1
rium, C CO3 , and the CO 3 concentration which has reacted
(Sartori et al., 1987). The carbamates of such hindered
to produce HCO3 , 0:5C HCO3 . Therefore,
amines are much less stable than the carbamates of the
corresponding unhindered amines. For example, the equi-
C tCO3 C CO3 0:5 C HCO3 : 5:2:19c
librium constant for the hydrolysis reaction (5.2.18a),
Further, from the definition of f (f 1 means all of the
K C RNH2 C HCO3 =C RNHCOO , 5:2:18c absorbent has been consumed; f 0 means no consump-
tion at all),
has the value of 0.143 for t-butylamine and 0.06 for the  
unhindered n-butylamine HOCH2CH2NH2 (see Sartori f C tCO3 0:5 C HCO3 : 5:2:19d
et al., 1987). Thus, reaction (5.2.18b) will occur if the alkyl
group in the amine is bulky and the carbamate is unstable. We substitute these relations into (5.2.19b) to get
The net result with a highly hindered amine is that we can
approach the theoretical absorption capacity of one mole 4 t f2
C CO2 C CO3 : 5:2:19e
of CO2 per mole of amine. K 1f
284 Effect of chemical reactions on separation

102
coal gasification streams have H2S and CO2; only H2S
40 wt% K2 CO3 needs to be removed to avoid pollution. Yet there will be
140C absorption of both H2S and CO2 in the absorbents tradi-
130
tionally employed. Selective removal of H2S over CO2 from
120
110 gases with a high CO2/H2S ratio is highly desirable for
101 90 several reasons: increased CO2 absorption increases cost
Equilibrium partial pressure of CO2, psia

70
for absorbent regeneration and recirculation; reduction of
CO2 in the acid gas obtained from the stripper-regenerator
is useful for sulfur recovery by the Claus process (Astarita
et al., 1983; Savage et al., 1986).
100 102 Under conditions of absorption equilibrium, the
thermodynamic selectivity of an absorbent for H2S over
CO2 has been defined as (Astarita et al., 1983)

101 ! !
101 C tH2 S pCO2
H2 SCO2 , 5:2:20
pH2 S C tCO2

where C ti is the total molar concentration of species i in


20 wt% K2CO3
100 the liquid phase whose equilibrium vapor pressure is pi.
130 C
110
90
The liquid-phase concentration C ti includes the free ith
70 species and the chemically combined form of the ith
species.
101 101
Most chemical absorbents are aqueous alkaline solu-
102 101
10 0
101
tions containing a base, B. The reactions of H2S and CO2
f2 are as follows:
1f
K
H2 S B S BH HS ; 5:2:21
Figure 5.2.2. CO2 equilibrium vapor pressures over potassium
carbonate solutions (Astarita et al., 1983.) Reprinted, with permis- K 1
CO 1

sion, from G. Astarita, D.W. Savage, A. Bisio, Gas Treating with


CO2 B H2 O 2 BH HCO3 : 5:2:22
Chemical Solvents, Figure 2.4.1, p. 70, John Wiley & Son, 1983.
There is an additional reaction for H2S, a simple dissoci-
Copyright 1983, John Wiley & Sons.
ation of HS, given by

HS H S : 5:2:23
Using Henrys law for free CO2, pCO2 H CCO2 C CO2 , we The equilibrium constant for this reaction is very small
obtain (in the range of 1014 to 1012 gion/liter between 20 and
100  C) compared to that for reaction (5.2.21), which is in
4 H CCO2 t f2
pCO2 C CO3 , 5:2:19f the range 0.1 to 10 (Astarita et al., 1983). Therefore we can
K 1 f
neglect dissociation of HS. We can conveniently consider
which relates the partial pressure of CO2 in the gas at reactions (5.2.21) and (5.2.22) in terms of
equilibrium with the total molar absorbent concentration
and its fractional consumption (Figure 5.2.2). For a given K
HCO3 H2 S CS H2 O CO2 HS : 5:2:24
C tCO3 , a higher equilibrium CO2 partial pressure implies a
larger value of the fractional absorbent consumption. The The equilibrium constant for this reaction, KCS, in terms
temperature dependence of K is also crucial to the partial of the molar concentration based equilibrium constants,
pressure of CO2 at equilibrium. Further, the amount of CO2 KS and K CO2 1 , is
absorbed is directly proportional to C tCO3 ; therefore, K2CO3
is preferred to Na2CO3, which has a lower solubility. Table KS C CO2 C HS
K CS , 5:2:25
5.2.1 lists common alkaline reagents used for acid gas K 1
CO2 1 C HCO3 C H2 S
scrubbing in aqueous or organic solutions.
where
5.2.1.1.3 Selective absorption in reactive solution Some
industrial gas streams contain two volatile species, only C BH C HS C CO2 C B
KS ; K CO2 1 : 5:2:26
one of which needs to be removed. For example, some C H2 S C B C BH C HCO3
5.2 Chemical reactions change separation equilibria 285

Table 5.2.1. Common alkaline reagents used in chemical solventsa

C C OH

Monoethanolamine (MEA) N H

H
C C OH

Diethanolamine (DEA) N H

C C OH
OH

C C OH

Diisopropanolamine (DIPA) N H

C C C

OH
C C O C C OH

, 0 Hydroxyaminoethylether (DGA) N H

H
Potassium carbonate (with promoters) K2CO3
O

C C

Potassium glycinate N H OK

H
Caustic soda NaOH
a
Reprinted with permission from G. Astarita, D.W. Savage, A. Bisio, Gas Treating with Chemical Solvents,
John Wiley & Sons (1983), Table 1.2.3, p. 9. Copyright 1983, John Wiley & Sons.

Now, C tH2 S C H2 S C HS . Assume that C H2 S << C HS , Now CO2 exists in the liquid as free CO2 and an HCO3 ion,
which is generally valid unless pH2 S is very high. Further, as well as in other forms, e.g. CO3 for an aqueous K2CO3
solution, RNHCOO for a primary amine solution, etc.
pCO2 H CCO2 C CO2 and pH2 S H CH2 S C H2 S : 5:2:27 Thus, C HCO3  C tCO2 , so that

Substitute (5.2.25) and (5.2.27) into the selectivity defin- H2 SCO2  K CS H CCO2 =H CH2 S : 5:2:29
ition (5.2.20) for C H2 S << C HS to obtain
Obviously, conditions that lead to C tCO2 C HCO3 provide
higher selectivity. This means that the carbamate species
H CCO2 C HCO3 C tH2 S H C 2 C HCO3
H2 SCO2 K CS t K CS CO : RNHCOO should be unstable. Savage et al. (1986) have
H CH2 S C HS C CO2
H CH2 S C tCO2
justified the use of methyldiethanolamine on such
5:2:28 grounds.
286 Effect of chemical reactions on separation

The value of KCS is less than 1 for the practical range of is the liquid phase. Similarly define xij xi2 xi as the
temperatures (see Astarita et al. (1983, p. 87)). Therefore liquid-phase mole fraction. Then relation (1.6.12) becomes
we obtain in x i
yi 4
, 5:2:32a
X
H2 SCO2  H CCO2 =H CH2 S 5:2:30 kn x k
k1
with aqueous alkaline absorbents. Note that the ratio
(H CCO2 =H CH2 S ) is the selectivity with simple physical absorp- where
tion (relation (4.1.8)). Therefore the thermodynamic yi x j
ij , 5:2:32b
selectivity between H2S and CO2 with alkaline aqueous x i yj
absorbents is less than that achieved without any chemical
reaction. For pure water, the limiting value (H CCO2 =H CH2 S ) is nn 1: 5:2:32c
around 3.05; it varies slowly with temperature. Since species 3 and 4 are assumed nonvolatile relative to
In reality, the selectivity achieved in mixed H2SCO2 components 1 and 2,
absorption in alkaline solutions is higher due to the
kinetics of the mixed gas absorption process as long as 31 32 41 42 0: 5:2:32d
the mass-transfer coefficient of the absorber is high
Choose n 2. Also, it is a close separation problem, 12
(Danckwerts, 1970). Thus, thermodynamics (or equilib-
1; obviously 22 1. Therefore relation (5.2.32a) for i 1
rium behavior) alone is insufficient. See Section 5.3.1.1.
and 2 becomes
One can also select organic physical solvents with
much higher values of H2 SCO2 . For example, NMP x1
y1 ; 5:2:33
(n-methyl-pyrrolidone) has a value of around 12 and x1 x2
NMC (N-methyl--caprolactam) has a value of around x2
12.5; the H2S solubility in these solvents is also high (Astarita y2 : 5:2:34
x1 x2
et al., 1983).
The mole fraction based equilibrium constant for reaction
(5.2.31) is
5.2.1.2 Vaporliquid equilibrium x4
Kx : 5:2:35
The distillation separation factor for an isotopic mixture or x1 x3
a close boiling mixture is very close to 1. Separation by The liquid-phase mass balance relation is
conventional distillation would be highly energy-intensive
4
X
and capital-intensive. If, however, a reactive but relatively x i 1 ) x 1 x 2 x 3 x 4 1: 5:2:36
nonvolatile species 3 is added to a close-boiling binary i1
mixture of species 1 and 2, such that the lighter species 1
reacts according to Using relation (5.2.35) for x4 in (5.2.36) to obtain an expres-
sion for (xl x2) in terms of K x, xl and x3, we see that, from
13,4 5:2:31 (5.2.33),
in the liquid phase while species 2 is nonreactive, a much x1
y1 : 5:2:37
higher separation can be obtained provided reaction prod- 1 x 3 K x x 3 x 1
uct 4 is relatively nonvolatile. Species 3 is sometimes When x3 0, y1 x1, indicating essentially no separation.
known as a reactive entrainer. However, for nonzero x3, yl > x1. Figure 5.2.3 shows plots
In order to recover 1 and 2 in an unreacted form, it is of yl vs. xl for x3 0.1 and K x 1, 5 and 10. As K x increases,
necessary for reaction (5.2.31) to be reversible. It is also the vapor becomes highly enriched in species 1. But, due
necessary that the volatilities of 3 and 4 should be either to the liquid-phase reaction of species 1 with species 3,
more or less than those of both 1 and 2; this ensures that mole fraction x1 is considerably reduced from the value of
the subsequent separations are easy. Terrill et al. (1985) x1 without a reaction.
(see also Cleary and Doherty (1985)) have discussed a Since we are interested in the separation of species
number of examples of such systems, e.g. the separation 1 and 2, consider y2 and x2. From relation (5.2.37) and
of m-xylene and p-xylene using organometallic com- yl y2 1.0 (remember 3 and 4 are nonvolatile),
pounds. The reader should consult Terrill et al. (1985) for
x2
references on other systems. We follow their treatment y2 : 5:2:38
1 x 3 K x x 3 x 1
below to demonstrate how the vaporliquid equilibrium
is altered by the reaction. Again, when x3 0, y2 x2, indicating essentially no separ-
For simplicity, we define the vapor-phase mole frac- ation. However, for the case of a reaction with nonzero
tion, xij xi1 yi, where j 1 is the vapor phase and j 2 x3, y2 > x2. Further, since x1 is small, y2 can be much larger
5.2 Chemical reactions change separation equilibria 287

1.0 equilibrium enrichment achieved in one stage, a closed


Kx = 10 Kx = 5 Kx = 1 vessel.
When two isotopic compounds, say XAn (i 1) and
0.8 XAn1B (i 2), are present in a mixture containing two
isotopes A and B, the separation factor 12 may be easily
0.6 obtained from (4.1.21b) as 12 P sat sat
1 =P 2 as long as the gas
Kx = 0 phase behaves ideally and the liquid phase is an ideal
y1
and solution. However, in the case of heavy water, there is a
0.4 x3 = 0 third isotopic compound, HDO, present; there is the
following reaction in the liquid phase:
0.2
Kx
H2 O D2 O , 2HDO: 5:2:40
0 We will now obtain a separation factor 12 between H2O
0 0.2 0.4 0.6 0.8 1.0
(i 1) and D2O (i 2) in the presence of HDO (i 3), all
x1
three species being distributed between the liquid phase
and the vapor phase. Assume (Benedict et al., 1981) that
Figure 5.2.3. Effect of reaction equilibrium constant on phase
equilibrium (x3 0.1 unless stated otherwise) (Terrill et al., (1) we have ideal gas behavior and an ideal solution in
1985). Reprinted, with permission, from I & E Chem. Proc. Des. liquid phase;
Dev., 24, 1062 (1985). Figure 2. Copyright (1985) American Chem- (2) the vapor pressure of HDO is a geometric mean of the
ical Society. vapor pressures of H2O and D2O, i.e.
q
P sat
3 P sat
1 P2 ;
sat
5:2:41
than y1. For example, suppose Kx 10, x3 0.1 and x1 0.1.
(3) reaction (5.2.40) is at equilibrium in the liquid phase;
From the above relations, we get y1 0.125, x2 0.7 and
(4) the equilibrium constant K x of reaction (5.2.40) based
y2 0.875. Thus the vapor is highly enriched in species 2 over
on mole fractions is 4.
species 1. Suppose x2 0.7 (x3 0 x4) and there is no
chemical reaction. Then x1 0.3, y1 0.3 and y2 0.7. The separation factor 12 based on H2O (i 1) concen-
Obviously, the chemical reaction has led to a much purer trating in the vapor phase and D2O (i 2) concentrating in
vapor in terms of species 2, and therefore to a better the liquid phase is calculated using the atom fraction of
separation. hydrogen and the atom fraction of deuterium, which in
The converse approach is sometimes practiced to turn are calculated from the mole fractions of individual
improve the conversion from a reaction. By withdrawing compounds (see (1.6.1d)). For example, the atom fraction
selectively one or more products from the reaction mix- of hydrogen in vapor is given by
ture, a reversible reaction can be driven to the right, e.g.
aH2 v 2x H2 O, v x HDO, V 5:2:42a
CH3 COOH C2 H5 OH , CH3 COOC2 H5 H2 O: 5:2:39
and the atom fraction of deuterium in vapor is given by
Removal of water would push the reaction to the right in
this esterification process, an example of separation facili- aD2 v x HDO, v 2x D2 O, v , 5:2:42b
tating the main objective, namely a chemical reaction yielding
(Suzuki et al., 1971).
  
aH2 v aD2 l
5.2.1.2.1 Separation factor in distillation of isotopic 12
aD2 v aH2 l
mixtures Isotopic mixtures are often separated by distil-   
lation. The production of heavy water, D2O, is a prime 2x H2 O, v x HDO, v x HDO, l 2x D2 O, l
: 5:2:43
example. In the distillation of natural water, which con- x HDO, v 2 x D2 O, v 2 x H2 O, l x HDO, l
tains variable amounts of deuterium depending on the
source (see Table 13.1 in Benedict et al. (1981)), deuter- From assumption (1),
ium is invariably concentrated in the liquid phase. Water
in the tropical oceans contains about 156 ppm deuter-
P sat
H2 O x H2 O, l P sat
D2 O x D2 O, l
ium. Multistage distillation (see Section 8.1.3) has to be x H2 O, v ; x D 2 O, v : 5:2:44a
P P
carried out to obtain a liquid fraction substantially
enriched in deuterium. Here we will consider only the From assumptions (2), (3) and (4),
288 Effect of chemical reactions on separation

(see Chapters 8 and 9), it is possible to produce a highly


q
P sat
HDO P sat sat
H2 O P D2 O ; 5:2:44b
enriched C13 fraction containing as much as 6070% C13 as
x 2HDO, l p HCN vapor. One could react this HCN vapor with an
Kx 4 ) x HDO, l 2 x H2 O, l x D2 O, l ; aqueous NaCN to obtain a NaCN solution in which 6070
x H2 O, l x D2 O, l
atom % of C is C13.
5:2:44c
Any separation factor describing such an isotope
enrichment process has to take into account the vapor
q
P sat P sat sat
H2 O P D2 O x H2 O, l x D2 O, l
HDO x HDO, l liquid equilibrium for the vapor species HCN. Consider
x HDO, v 2 :
P P
now the isotope exchange reaction (5.2.49) equilibrium
5:2:44d
along with the vaporliquid equilibrium of HC12N and
Substituting these results into the expression for 12, we HC13N. There are a total of six mole fractions, xij, to deal
get with:
HC12N (v), i 2, j 1: x21;
2 p 3
Psat
H2 O x H2 O, l
2 P sat sat
H2 O P D2 O x H2 O, l x D2 O, l
62 P P HC13N (v), i 1, j1: x11;
12 4 p
7
Psat HC12N (l), i 2, j 2: x22;
sat
5
H2 O P D2 O x H2 O, l x D2 O, l 2 Psat
D2 O x D 2 O , l
2 P P
 p  HC13N (l), i 1, j 2: x12;
2 x H2 O, l x D2 O, l 2x D2 O, l
 p ; 5:2:45 NaC13N (l), i 3, j 2: x32;
2x H2 O, l 2 x H2 O, l x D2 O, l NaC12N (l), i 4, j 2: x42.
2q3 s

p sat First, the mole fraction based separation factor in
x D2 O, l 6 P H2 O x H2 O, l 7 P sat
H2 O
12 p 4q5 : 5:2:46 vaporliquid equilibrium between HC13N and HC12 N is
x H2 O, l P sat x P sat
D2 O
D2 O D2 O , l given by
x 11 x 22
This value varies between 1.12 at 0  C to 1.026 at 100  C vl
12 : 5:2:50
x 22 x 12
(see Table 13.4 of Benedict et al. (1981)).
Apparently, using similar assumptions, this type of Second, the overall isotope reaction (5.2.49) may be
result has been generalized for a mixture of isotopic com- written for the liquid phase as follows:
pounds. Benedict et al. (1981) have suggested that, for a
mixture of isotopic compounds, XAn, XAn1B, XAn2B2, . . .., Kl
HC12 Nl Na C13 N l , HC13 Nl Na C12 N l:
XBn, the separation factor AB for isotopes A and B may be
5:2:51
approximately obtained as
s
sat One can now develop the following relation between the
n P XA n
AB : 5:2:47 various concentrations and the reaction equilibrium con-
P sat
XBn
stant Kl:
For example, in the distillation of ammonia, the separation C 12 C 42
factor between NH3 and ND3 is quite close to the value Kl : 5:2:52a
C 22 C 32
s
sat
3 P NH3 However, this is also equivalent to
NH3 , ND3 : 5:2:48
P sat
ND3 x 12 x 42
Kl : 5:2:52b
x 22 x 32
This separation factor was measured to be around 1.042.
Define the overall separation factor ov
12 between isotopes 1
5.2.1.2.2 Isotope exchange reactions in a vaporliquid and 2 and the two phases j 1, 2 as
system Consider the following overall isotope exchange
x 11 x 22 x 42
reaction between HC12N vapor and an aqueous NaC13N ov
12 : 5:2:53
x 21 x 12 x 32
solution:
Pratt (1967) has defined a quantity w as the mole fraction
HC12 Nv Na C13 N l , HC13 Nv Na C12 N l: of both isotopic forms of the nonvolatile reactant in the
5:2:49 total dissolved reactants as
As a result of this exchange between C12 and C13 isotopes of
x 32 x 42
carbon, the HCN vapor becomes preferentially enriched in w : 5:2:54
x 22 x 12 x 32 x 42
the C13 isotope. Natural carbon contains 1.11% C13 (Bene-
dict et al., 1981). However, the reaction above will lead to a He has then claimed that ov vl
12 is related to Kl, w and 12 by
higher percentage of C13 in the HCN vapor. If such a
behavior could be multiplied in a cascade of many stages ov vl
12 12 1 w wK l  5:2:55
5.2 Chemical reactions change separation equilibria 289

as long as x11 x21 1.0 by neglecting the humidity in the 5.2.2.1 Dissociation of an organic acid or base in water
vapor phase. For the reaction under consideration, vl 12
Organic solutes to be extracted from an aqueous phase into
0.995 and Kl 1.031 at 25  C. It becomes clear that ov
12 > 1
an organic solvent phase, or vice versa, are often weak
as long as w is considerable. For example, if w is 0.8, ov
12
acids or bases. Only unionized solutes are extracted into
1.0188. The experimentally obtained value of ov 12 is 1.013
the organic solvent. But the unionized weak acid solute,
(Pratt, 1967, chap. 6).
HA, will partially dissociate in an aqueous solution2 as
Many other isotopes have been enriched by isotope
follows:
exchange reactions. However, they have primarily
employed gasliquid systems. A few of them are listed HA , H A : 5:2:57
below (for a more detailed introduction, see Pratt (1967,
chap. 6) and Benedict et al. (1981)): Similarly, an unionized weak base B will partially ionize in
the aqueous phase as follows:
(1) CO16
2 g
18
H 2 O l ,
CO16 O18 g H2 O16 l, T 25 C; B H , BH : 5:2:58

Thus, the concentration of free HA or free B in water is


(2) N15 H3 g N14 H
4 NO3 l , different from the concentration without ionization.
N14 H3 g N15 H 
4 NO3 l, T 25 C;
Consider the distribution equilibrium of the weak acid
HA (i 1) between water and a nonionizing organic
34 32
(3) S O2 g Na HS O3 l , solvent:
S32 O2 g Na HS34 O3 l:T 25 C: 5:2:56
HA
|{z} , HA , H A :
|{z} 5:2:59
solvent phase aqueous phase
5.2.2 Liquidliquid equilibrium j 1 o j 2 w

The effective distribution of a solute i between two immis- Now,


cible liquid phases 1 and 2 is often changed if any one, or
some or all of the following happen: species i dissociates in 00 00 C 1o C 1o
11 1o ,
one phase; species i associates in another phase; species i C 1w C 2w conc:ofHA conc:ofA water
or its dissociated form reacts in one of the phases with 5:2:60
another species deliberately added to this phase or delib-
where ionic species A is i 2. Note that the hydrogen ion
erately added to the other phase in another form. This may
concentration, C H w , here is also equal to C2w. For simpli-
be brought about by using acidbase equilibrium, simple
city, we write C H w C H . The dissociation or ionization
complexation, chelation, ion pairing, solvation, etc. In
constant Kd1 of solute 1 is given by
some cases (for example, in metal extraction), partitioning
of the solute species into another phase is generally not C H C 2w C 2w K d1
K d1 ) : 5:2:61a
possible without some kind of chemical reaction. In the C 1w C 1w C H
following, we first consider extraction of nonmetallic
Further, by definition of i10 ,
species in the presence of an acidbase equilibrium, asso-
0
ciation, dissociation and simple complexation. Metal 1o C 1o =C 1w : 5:2:61b
extraction is treated later. 0
Using these relations for Kd1 and 1o in the expression for
In many extraction systems in practice, species i can 00
1o , we get
exist not only as i, but also in other forms. The effective
distribution (or partition) coefficient il00 will then be based 00
0
1o C 1w 0
1o 0
1o
on the total concentration of i in all forms in the two 1o K d1 C 1w
K d1 : 5:2:62
C 1w C 1 C 1 CC 2w
1w
H H
phases. The intrinsic distribution coefficient il of only the
free species i between the two phases in the absence of 00 0
Thus 1o < 1o in general. Note here that
chemical reactions will, in general, be different from il00 .
1o C 1o =C 1w 5:2:63
The intrinsic distribution coefficient il0 of only the free
species i between the two phases in the presence of chem- 0
in the absence of any ionization; 1o may equal 1o ; i.e.
ical reactions may be different from il . If the presence of the distribution relation of the free acid HA without any
other species does not influence the distribution coeffi-
cient, then il0 il . Assume the mutual solubility of water
and the organic solvent to be unaffected by any of these 2
The reaction in water is HA H2O , H3O A. For dilute
reactions. We now study how to determine i100 for several solutions, we will avoid writing down water, whose concentration
different cases. may be assumed to be essentially constant at 55 gmol/liter.
290 Effect of chemical reactions on separation

ionization (i.e. no other solute species) may equal the 00 0 C 1w 0


1o
1o 1o ; 5:2:70a
distribution relation of the free acid HA when ionization C 1w C H C 1w K d1 1 C H K d1
00 0
is present. But 1o 6 1o unless the ionization constant Kd1 0
1o
is vanishingly small. On the other hand, in an alkaline 00
1o pK 1 pH : 5:2:70b
1 10
solution of a weak acid HA (Treybal, 1963) C H  K d1 , and
00 0 The general relations (5.2.62) for a weak organic acid and
1o 1o C H =K d1 , 5:2:64
(5.2.70a) for a weak organic base can also be expressed in
leading to the following forms (Robinson and Cha, 1985):
00 0
log 1o log 1o pH log K d1 , 5:2:65a
0
1o
acid pH pK 1 log 1 ; 5:2:71
where pH log10C
H . Also, since log Kd1 pK1, an alter-
00
1o
native result is
0
1o
log 00
1o 0
log1o pH pK 1 : 5:2:65b base pK 1 pH log 00 1 : 5:2:72
1o
If we now use an alternative form of relation (5.2.61a) using
From these relations, we find that, for a weak organic acid,
pH and pK1, we get 00
as the pH increases, the value of 1o decreases, since most
pHpK 1 logC 2w =C 1w ) C 2w =C 1w 10pHpK 1 : of the acid is ionized and cannot be extracted. This leads to
5:2:65c a poor extraction of the unionized acid into the organic
phase when compared with the total amount of acid avail-
Correspondingly, expression (5.2.62) becomes able in the aqueous phase. Conversely, for an organic base,
00
0
1o as the pH decreases, 1o decreases, reducing the extent of
00
1o : 5:2:65d extraction of the unionized base. Figure 5.2.4(a) illustrates
1 10pHpK 1
these two relations for a weak organic acid (e.g. phenols,
We now consider the case of a weak organic base B (i 1) formic acid, benzoic acid, etc.) and a weak organic base
and the ionization reaction (5.2.58) with species BH being (Robinson and Cha, 1985). For carboxylic acid extraction,
i 2: consult Kertes and King (1986). The extraction of
aqueous phase j 2 w pencillins, which are weak acids, is illustrated in Figure
z}|{ 5.2.4(b) (Souders et al., 1970). For the application of solv-
B H , BH
ent extraction to pharmaceutical manufacturing processes
m
B 5:2:66 for antibiotics and nonantibiotics, see Ridgway and Thrope
|{z} (1983). The values of pK1 of a number of biological solutes,
solvent phase j 1 o e.g. carboxylic acids, amino acids, etc. are provided in
Tables 5.2.2 and 5.2.5.
Here,

00 C 1o C 1o
1o :
C 1w C 2w conc:ofB conc:ofBH water
Table 5.2.2. pK values of selected solutesa
5:2:67

The dissociation constant3K d1 of base 1 is Solute pK1 pK2 pK3

1 C C 1w C 2w Formic acid 3.74


H ) C H K d1 : 5:2:68a Acetic acid 4.76
K d1 C 2w C 1w
Propionic acid 4.87
An alternative form is n-Butyric acid 4.82
Lactic acid 3.73
C 2w
10pK 1 pH : 5:2:68b Tartaric acid 3.03 4.37
C 1w Oxalic acid 1.25 3.67
Now, Citric acid 3.13 4.76 6.40
0 Ammonium (NH 4) 9.25
1o C 1o =C 1w : 5:2:69
Trimethylamine 9.87
Substitute these definitions of Kd1 and 0
1o into the expres- Phosphate H3PO4 2.15
00
sion for 1o to obtain H2 PO4 7.22
HPO2 4 11.50
a
Values obtained from Belter et al. (1988), Garcia et al. (1999) and
3
Note that this definition of Kd1 is inverse of that used by Table 6.3.1 at 25 C.
Robinson and Cha (1985). See Table 5.2.5 for amino acids.
5.2 Chemical reactions change separation equilibria 291

(b)
(a) 100
Scale for equation (5.2.71)
Penicillin F
pH pK1
Penicillin K
3 2 1 0 1 2 3
1.0 Phenylacetic acid

Distribution coefficient, solvent / aqueous


0.8 10
CX 1
CX 2
TX 2
k 1o 0.6
k1o

0.4
1 Penicillin G
TX 1
0.2 Acetic acid

0
3 2 1 0 1 2 3
pK1 pH 0.1
1 2 3 4 5 6 7 8
pH

Figure 5.2.4 (a) Solutions of equations (5.2.71) and (5.2.72), respectively. (After Robinson and Cha (1985).) (b) Distribution coefficient
data for penicillin broth contents for a solvent. Reprinted, with permission, from Souders et al., Chem. Eng. Prog. Symp. Ser., 66(100), 41
(1970), Figure 2. Copyright [1970] American Institute of Chemical Engineers (AIChE).

Some carboxylic acids have two ionizable protons. They Rearrangement leads to
are called dicarboxylic acids, e.g. oxalic acid, HOOC
COOH. We may represent the unionized acid, species i 1, pK 2 pH log10 C A2 =C HA : 5:2:74c
by H2A. The first ionization of this acid, represented by
We may rewrite this as pK 2 pH log10 C A2 =C H2 A 
H2 A ) HA H ,
5:2:73a log10 C H2 A =C HA ,
i 1 i 2

has a dissociation constant Kd1 corresponding to pK1 via pK 2 pK 1 2pH logC A2 =C H2 A , 5:2:74d
C H C HA K d1 C HA
K d1 ) , 5:2:73b by using the result (5.2.73c). We can now define an
C H2 A C H C H2 A
effective partition coefficient for H2A between water and
which leads to an organic solvent in the absence of any complexing
extractant for the dicarboyxlic acid H2A as
pK 1 pH log10 C HA =C H2 A : 5:2:73c
0
00 C 1o 1o
The second ionization, represented by 1o ; 5:2:74e
C 1w C 2w C 3w 1 C CC 3w
C 2w
1w 1w

HA ) A2 H ,
5:2:74a 0
i 2 i 3 00 1o
1o : 5:2:74f
1 10 pK 1 pH
102pHpK 1 pK 2
has a dissociation constant Kd2, corresponding to pK2 via
Note: Here the total concentration of the acidic species in
C C 2 water is contributed by three species: H2A (species 1),
K d2 H A : 5:2:74b
C HA HA(species 2) and A2 (species 3). In the case of
292 Effect of chemical reactions on separation

00 00 00 0 0 0
tricarboxylic acids, which may be represented by H3A, the (Ao =Bo ) AB and Ao =Bo AB . The separation
number of such species will be four: H3A, H2A, HA2, A3. 00
factor with aqueous dissociation, AB , and the separation
0
There are substances that are amphiprotic: at low pH factor AB unaffected by the dissociation are then
values they behave as a base, and at high pH values they related by
act like organic acids. At intermediate pH, the neutral
00
species predominates. An example is 8-quinolinol: 00 Ao 0 1 C H K dB
AB 00 AB 1 C K : 5:2:77
Bo H dA
5 4
6 3 Rewrite this relation as
7 2
00
00

AB K dB AB
N log 0 1 pK A pH log : 5:2:78
AB 0
K dA AB

Only cases where KdA 6 KdB are useful for our purpose
OH 00 0
since AB AB if KdA KdB. Therefore, we consider cases
If we represent it as HQ, then, at lower pH, H2Q, resulting where the dissociation constants of species A and B are
from the protonation of the nitrogen, will be present in different. For given pH, KdA and KdB, one can now solve
 00 
substantial amounts, whereas at high pH, Q will be prepon- for the ratio AB 0
=AB 0
; since AB is assumed known, AB00
 00 
derant. The neutral species HQ is the major species at inter- can be determined. Figure 5.2.5 illustrates how AB 0
=AB
mediate pH. Problem 5.2.10 considers the development of a varies with pKA pH for different positive values of
distribution coefficient for such a substance over the whole pKA pKB (Robinson and Cha, 1985). Here B is a stronger
pH range (Karger et al., 1973). A most important example of base than A (KdB > KdA); it is also more water soluble.
similar behavior is obtained with amino acids. If we represent Thus, when the solution becomes more acidic, species B
an amino acid NH2CHRCOOH as HA, then, at low pH, H2A ionizes more and its water solubility increases further.
dominates; at high pH, A is preponderant. At intermediate Therefore, Bo00
will decrease much more rapidly, leading
pH, the zwitterionic form HA is the major species. See to a high separation factor; species A will be extracted
Section 5.2-3.1-2 for an analysis of their ionization behavior. much more into the solvent compared to species B, leading
to a higher purity of the extract in species A. Of course, the
00
5.2.2.2 Two weak organic acids or bases: value of Ao will be decreased due to an increase in pH.
dissociation extraction Consequently, an optimum between the purity of A and
its extent of recovery in the solvent has to be struck. The
Many compounds used in practice, especially in the case where pKB > pKA (KdA > KdB) has not been con-
pharmaceutical industry, are weak acids or bases. If two sidered here.
compounds are similar, their distribution coefficients A relationship similar to (5.2.77) can be developed
between the aqueous and organic phases would be similar. (Treybal, 1963, p. 48) when both species A and B are weak
Since one of the compounds is preferentially sought for its acids:
particular properties over the other, conditions for prefer-
00
ential extraction of that compound are required. Select two 00 Ao 00 1 K dB =C H
AB 00 AB 1 K =C : 5:2:79
compounds A and B, both being bases (acids are treated Bo dA H
later). Assume for the present that 0 Ao > 0 Bo , i.e. species B
This can be expressed for KdB >> C H and KdA >>C H (i.e.
is more soluble in the aqueous phase than species A.
pH >> pKA and pKB) as
Remember, however, that 0 Ao is quite close to 0 Bo .
Consider pH conditions in the aqueous solution  00 0
AB =AB

K dB =K dA , 5:2:80
(acidic) so that ionization of A and B has taken place
(obviously the bases will hardly ionize at very large pH, reflecting the effect of different dissociation constants.
say pH > 12). Then from (5.2.70a), Thus, if acid B ionizes more, acid A will be extracted more
00
0
Ao into the organic solvent at higher pH values. For lower pH
Ao ; 5:2:75 00 0
1 C H K dA values, if KdB << C H and KdA << C H , then AB AB ,
0 namely the separation is strictly due to preferential extrac-
00 Bo
Bo : 5:2:76 tion of the undissociated species. The separation of two
1 C H K dB
organic acids or two organic bases of similar distribution
00 00
Now, the ratio Ao = Bo C Ao C B C BH w = coefficients by preferential extraction of one of them into a
C A C AH w C Bo . For a dilute solution, we consider solvent is thus possible only if their dissociation constants
the mixture of these two bases on the following basis: are different. By selecting proper pH conditions, the separ-
organic phase on a solvent-free basis; aqueous solution ation can be considerably enhanced. This process is known
on a water-free and acid-free basis. Then it is obvious that as dissociation extraction.
5.2 Chemical reactions change separation equilibria 293

Dissociation extraction can also be utilized to extract alternative technique of fractional neutralization may be
preferentially into an aqueous solution a particular organic adopted. Add a neutralizing alkali corresponding to the
acid or base from a mixture of organic acids or bases in an weaker base A, which will then revert to A from AH (but
organic solvent. For the separation of organic acids, for B is present as BH). This undissociated A can now be
example, a limited amount of alkali in the aqueous phase extracted by an organic solvent. For additional information
is used to create a competition for it by the solutes A and on, and analysis of, dissocation extraction, the work by
B in the organic phase. The stronger acid is transferred to a Wise and Williams (1963), the description by Pratt (1967,
large extent to the alkaline aqueous phase, whereas the pp. 327332) and the papers by Anwar et al. (1971) and
weaker acid remains essentially in the organic phase. Wadekar and Sharma (1981) may be consulted.
For the separation of organic bases A and B present in
an aqueous solution in completely dissociated forms, an
5.2.2.3 Association and/or complexation of a solute
in organic solvent
1000 Solutes to be extracted sometimes associate in the organic
pKA > pKB 3
solvent. This happens especially when the solute is polar,
KdB > KdA e.g. organic acids, while the solvent is quite nonpolar, e.g.
pKA pKB hexane, benzene, toluene, etc. For example, acetic acid
9Ao > 9Bo (i 1) is known to dimerize (through hydrogen bonding)
significantly in benzene. (Similarly, benzoic acid dimerizes
100 2 in benzene.) Thus the equilibrium distribution of free
acetic acid between the aqueous and organic phase will
be different from that of the total amount of acetic acid in
the two phases. With a polar solvent, dimerization of solute
AB
is likely to disappear. The equilibrium distribution will also
AB be influenced if an agent, present in the organic phase,
complexes or reacts with acetic acid. We consider now the
10
1 distribution of acetic acid, for example, under such condi-
tions. For the time being, we neglect the dissociation of the
acid in the aqueous phase. The analysis is useful in general
0.5 for carboxylic acids.
Conventionally, oil-phase (j o) concentration of
acetic acid is not large due to a low value of 1o. To increase
0.1 it, organic amines, for example, are added to the oil phase.
1 The basic amine (i 3) complexes with acetic acid to form
a complex i 4. In the oil phase, acetic acid is present in
4 3 2 1 0 1 2 3
three forms, whose concentrations are: free acetic acid C1o,
pKA pH acetic acid dimer C2o and the complex C4o, where i 2
refers to the dimer. The equilibrium constants for the
Figure 5.2.5. Solution of equation (5.2.78) for two bases. (After various reactions (shown in Figure 5.2.6) are as follows
Robinson and Cha (1985).) (j w for the aqueous phase):

4
K3 Oil phase
3
+
K2 O OH
CH3COOH + CH3COOH CH3 C OH C CH3
O

Kd 1
CH3 COOH H+ + CH3COO

Aqueous phase

Figure 5.2.6. Distribution of acetic acid between an oil phase and an aqueous phase with attendant reactions.
294 Effect of chemical reactions on separation

K2 3
C 1o C 1o , C 2o ; 5:2:81
Equation (5.2.88)

K3 1o = 0.024
C 1o C 3o , C 4o : 5:2:82
K2 = 0

Assume no acetic acid dissociation in the aqueous 2 K3 = 260


0
phase and that 1o is unaffected by these reactions:
Experimental

1o
C 1o


0 data
1o 1o : 5:2:83
C 1w
00
The effective value of 1o , 1o , however, should be based on
t
C 1o , the total acetic acid concentration in the oil phase: 1

00 C t1o C 1o C 4o 2C 2o
1o : 5:2:84
C 1w C 1w

From (5.2.82),
0
K 3 C 4o =C 1o C 3o : 5:2:85
0 0.1 0.2 0.3 0.4 0.5
Similarly,
Ct3o
K 2 C 2o =C 21o : 5:2:86
Figure 5.2.7. Effective distribution coefficient of acetic acid as a
Substituting these relations into (5.2.84), we get
function of C t3o . (After Kuo and Gregor (1983).)
C 1o C 1o C2
00
1o K3 C 3o 2K 2 1o
C 1w C 1w C 1w
This figure shows that, beyond C t3o 0.2 M, relation
1o 1o K 3 C 3o 21o K 2 C 1o : (5.2.88) no longer holds. This is apparently due to the
Using the definition of 1o , we get a simpler result: association or micellization of TOPO at higher concentra-
tions. However, the addition of TOPO has increased the
00
1o 1o 1o K 3 C 3o 221o K 2 C 1w : 5:2:87 extraction of acetic acid into the solvent phase by an extra-
ordinary amount.
The concentration of free amine C3o in the organic phase is
We will briefly identify now the results for extraction of
the only unknown here (assume that 1o , K3 and K2 are
acetic acid into an organic solvent in the absence of any
available from the literature and that C1w is known). If C t3o ,
complexing agent in the organic phase; however, the acid
the total amine present initially, is known, then
can dissociate in the aqueous phase, as shown in Figure
C t3o C 3o C 4o C 3o K 3 C 1o C 3o 5.2.6. If the dissociation or the ionization constant of acetic
C 3o 1 K 3 1o C 1w : acid (i 1) is Kd1 (see relation (5.2.61a) and Table 5.2.2)
and there is dimerization of the acid in the oil phase, then
Substitute into (5.2.87) to get it can be shown that
00
1o 221o K 2 C 1w f1o K 3 C t3o =1 K 3 1o C 1w g:

1o 0
0 1 2K 2 1o C 1w C H

00
1o 1o 5:2:89
5:2:88 C H K d1
Thus, 00
1o ,the net distribution coefficient of acetic acid, is in general. If the acid is the only solute, then C1w can be
considerably increased from 1o . expressed in terms of C H using the electroneutrality
Kuo and Gregor (1983) have studied the applicability condition.
of such a relation for the solvent decahydronaphthalene
(Cl0H18, decalin) and the complexing agent
5.2.2.4 Extraction of metals
trioctylphosphine oxide (TOPO) under conditions where
the extent of dimerization was negligible (K2 0). Figure Metallic compounds generally ionize in aqueous solution;
00 the metal ion, in addition, is hydrated, i.e. surrounded by
5.2.7 shows the distribution coefficient, 1o , of acetic acid
with a fixed initial concentration (0.0893 M) as the TOPO basic molecules of water. To extract the ionic metal species
concentration was varied from 0 to 0.52 M in decalin. They into an immiscible organic solvent, an uncharged metal-
had earlier observed that a value of K3 260 described the containing species which is soluble in a water-immiscible
00
value of 1o well when 1o 0.024 for a C t3o 0.1295 M. organic solvent has to be produced before it is extracted
5.2 Chemical reactions change separation equilibria 295

Table 5.2.3. Structure and properties of various metal extractantsa

Mol. wt. of Active species Specific


Type Name Formula active species (wt%) gravity

1a Acidic di-2-ethylhexyl phosphoric acid (C4H9CH(C2H5)CH2O)2POOH 322 100 0.98


extractants R2

Versatic 10 R1 C COOH 175 99.6 0.91

R3

1b Chelating LIX 65N R3


R1 339 39.2 0.88
R1 =
extractants aromatic -hydroxyoximes R2 = H
C
N OH R3 = C9H19
R2
Kelex 100 oxime derivative R 311 7480 0.99
N R=Dodecyl

OH

2 Anion tertiary amine, alamine 336 R3N (R C8C10) ~392 95 0.81


exchangers secondary amine, LA-2 R2NH (R C12C13) 351393 100 0.83
quaternary ammonium (R3N(CH3)Cl) (R C8-C10) ~442 >88 0.88
compounds, aliquat 336
3 Solvating phosphorus-oxygen-bonded R1R2R3PO 0.97
extractants donors:
tri-n-butyl phosphate R1 R2 R3 C4H9O 226
trioctyl phosphine oxide R1 R2 R3 C8H17 386
methyl isobutyl ketone R1COR2, R1 CH3, R2 100 0.804
(CH3)2CHCH2
a
Prepared from Tables 2, 3, 4 and 5 in Flett et al. (1991).

into the organic solvent. This involves metal charge neu- itself is achieved with many oxygen-containing solv-
tralization as well as replacing some or all waters of hydra- ents like tributyl phosphate. This is identified as
tion by solvent compatible agents. Conventional solvation of the metal ion.
approaches to achieve this goal are the following (Ritcey
Table 5.2.3 identifies a few extracting agents/extractants/
and Ashbrook, 1984a, p. 6).
solvents from each of the above categories. In the second
(1) Compound formation between the metallic ion and an category of ion-pair formation, two types of extractants
extractant is achieved by using acidic extractants or have been identified as anion exchangers: long-chain
chelating extractants. Acidic extractants have a group amines (tertiary, secondary or primary) and quaternary
like COOH, SO3H, whose hydrogen is exchanged for ammonium compounds. Some properties of each extrac-
the metal. A chelating extractant has two or more sites tant have been identified in the table.
to complex with a metal ion into a cyclic compound. We study the first of these three approaches now.
A chelating agent dissolved in the organic solvent Consider the chelation4 of metal ions Mn with an organic
forms an uncharged metal chelate in the aqueous ion K obtained from the chelating agent HK. This agent,
phase for extraction of the chelate into the organic dissolved in the organic phase, partitions into the aqueous
solvent. The acidic extractant acts similarly. phase and then dissociates into H and K:
(2) Ion-pair formation is initiated in the aqueous phase
between the metallic ion and a large counterion con- HK , HK , H K
taining a bulky organic group. This large counterion is |{z}
organic phase
|{z}
aqueous phase
obtained from a complexing agent added to the
j 1 o j 2 w 5:2:90
organic phase; the agent dissociates in the aqueous
phase to provide the counterion. The neutral ion pair
is extracted into the organic phase. 4
Quantitative representations for acidic or chelating extractants
(3) The formation of a loose nonchelating uncharged com- are quite similar. Macrocyclic compounds can complex metal
pound between the metal ion and the organic solvent ions in a more stable fashion (Burgess, 1988).
296 Effect of chemical reactions on separation

The metal ion Mn in the aqueous phase diffuses to the Therefore, log M1 00
varies linearly with pH, provided
interface, reacts with the ion K and forms MKn, a neutral the other quantities are unaffected by the pH variation. It
species which is extractable into the organic phase: is obvious that by choosing the right pH range, metal extrac-
tion into the organic phase can be drastically increased.
If we have to separate two metals M and N having the
Mn nK , MKn , MKn same valence n, at a given pH and a chelate concentration
|{z} |{z}
aqueous phase organic phase 5:2:91 CHK,o, then
j 2 w j 1 o
" #
0
 00 00
 f M K d, MKn MK n, 1
log M1 =N1 log 0 , 5:2:100
Generally, the reaction is thought to occur in the aqueous f N K d, NKn NK n, 1

side of the interfacial region; MKn then partitions into the


organic phase. The overall chemical reaction may be rep- which is likely to be unaffected by pH, at least over a
resented as certain range. To a first approximation, the curves of log
00 00
M1 vs. log pH and log N1 vs. log pH are therefore parallel,
Mn aq nHKo , MKn o nH aq: 5:2:92 with slope n. Figure 5.2.8(a) shows such a plot for metals
00
M and N. Recognize that if there is a pH at which log M1
To determine the net distribution coefficient of the metal is 2 and log N100
is 2, the two metals are almost completely
00
between the two phases, M1 , defined by separated. Here the difference in the two metal dissoci-
00 C MKn , o ation or hydrolysis constants K d, MKn , K d, NKn , are crucial.
M1 5:2:93
C MKn , w C Mn , w For the extraction of a given metal, depending on the
we proceed as follows. Define the following two distribu- purity and the recovery desired, the pH can be chosen.
tion coefficients of species MKn and HK between the Figure 5.2.8(b) illustrates the extraction of different metals
organic and the water phase: by a particular extractant as a function of pH (Cox and
C MKn , o C HK, o Flett, 1991). Note, however, that, as pH increases, both
0 0
MK n, 1
; HK ,1 : 5:2:94 log M100
and log N100
flatten out due to the pH variation of
C MKn , w C HK, w
fM and fN.
The equilibrium constants for the two aqueous-phase reac-
If the pH of the aqueous solution is increased to a
tions (5.2.90) and (5.2.91) are
high value, at which the metal hydrolyses, the solvent
C H w C K w C MKn , w extractability will drop drastically. The flat behavior we
K d, HK ; K d, MKn :
C HK, w C Mn , w C K w n observed in Figures 5.2.8(a) and (b) will change into one
00
5:2:95 where M1 will decrease rapidly with pH. At very low
pH, M1 also has a very low value since high H concen-
00
00
Using these relations, we can express M1 as tration prevents ionization of the chelating agent HK (see
0 n , 1 K d, HK n K d, MKn C HK, o n equation (5.2.90)) and thus hinders formation of the
00
M1  nMK
 0 n : complex MKn.
C H , w HK ,1 K d, MKn K d, HK n C HK, o n
Some examples of acidic and chelating agents
5:2:96 (Karger et al. (1973), Ritcey and Ashbrook (1984a,
Thus, as the concentration of the chelating agent HK in the p. 24) and Lo et al. (1983) provide more detailed lists)
organic phase, CHK,o, increases, M1 00
increases. Similarly as are provided in Table 5.2.3. Additional examples are
the pH increases (C H , w decreases), M1 00
increases. given in Figure 5.2.9.
Define fM as Note than an acidic extractant, such as di-2-ethylhexyl
phosphoric acid (DEHPA), may have a more complex
C Mn , w
fM , 5:2:97 chemistry than a chelating agent, HK, due to its dimeriza-
C Mn , w C MKn , w
tion or self-association in the organic phase (Ritcey and
i.e. the fraction of the total metal ion concentration in the Ashbrook, 1984a, p. 28). We have observed such behavior
aqueous phase present as Mn. Then, using (5.2.94) and earlier with carboxylic acids in nonpolar organic solvents.
00
(5.2.95), we can rewrite M1 as follows: In the extraction of metals by chelation, the neutral
0
MK K d, HK n K d, MKn C HK, o n f M product MKn is a chelated compound. Sometimes, the neu-
00 n, 1
M1  n  0  n , 5:2:98 tral species is simply a pair of oppositely charged ions which
C H , w HK ,1
is soluble in the organic phase. Basic extractants, like pri-
whereby mary, secondary or tertiary organic amines, are often used
00
logM1 npH nlogC HK, o in solvent extraction. A tertiary amine R3N (where R is usu-
ally an aliphatic long-chain group) in the presence of an
K d, HK n K d, MKn
 0 
f M MK
log n, 1
: 5:2:99 acid HA in the aqueous phase will undergo the following
0 HK, 1 n reactions and produce an ion pair R3NHA:
5.2 Chemical reactions change separation equilibria 297

(b)

100
(a)
3

2 Fe(III) Zn Cd Cu Mg Co Ni

80
M N
log k M1 or log k N1

Extraction (%)
60
0 n=2 n=2

40
1

20
3
1 2 3 4 5 6 7 8 9 10 11 12

0
Equilibrium pH 0 1 2 3 4 5
Equilibrium pH

Figure 5.2.8. (a) Equilibrium distribution of a metal species M or N as a function of pH for Mn or Nn. (b) Extraction of some metals by
DEHPA from a sulfate solution (Cox and Flett, 1991). Reprinted, with permission, from M. Cox and D.S. Flett, Metal extraction
chemistry, in T.C. Lo, M.H.I. Baird and C. Hanson, Handbook of Solvent Extraction, Wiley-Interscience (1983), Figure 4, p. 59.
Copyright 1983, John Wiley & Sons.

8-Quinolinol (HOx)
producesan anion
forcom plexation

N N


OH O

1,3-diketonethenoyltrifluoracetone

HC CH

HC C C CH C CF3

S
O OH

Figure 5.2.9. Examples of acidic and chelating extractants, e.g. both HOx and M(Ox)2 with 8-quinolinol.

R3 Norg HAaq , R3 NH A org; In addition, we have the distribution equilibrium of


R3NH A between the two phases:
i 1 i 2
5:2:101
R3 NH A aq , R3 NH aq A aq:

00 C 3o
31 : 5:2:102
i 3 i 4 i 5 C 3w
298 Effect of chemical reactions on separation

1 n)
{(R3NH+)m nMA(m
m }2 {(R3NH+)m nMA(m n)
}
m
2
1
[R3NH+A]2 R3NH+A
2

R3N

Organic

R3N + H+ + A R3NH+ + A R3NH+A

HA
n)
(m n)R3NH+ + MA(m
m {(R3NH+)m nMA(m n)
}
m

Aqueous

Figure 5.2.10. Ion-pair extraction of a metal ion.

In the aqueous solution, the metal ion Mn in the presence One can also argue that, prior to the ion-pair formation
an acid HA exists as various complexes (charged or of (R3NH)mn with MAmmn to enable metal extraction,
uncharged) MAm mn
. Typical examples for HA will be there was an ion pair R3NHA whose anion A was
HCl, HI, etc. Now, the bulky counterion R3NH in the exchanged with another anion MAmmn to effect metal
aqueous phase will form an ion pair with a charged metal- extraction. The tertiary amine compound R3N had to be
mn
lic species in aqueous solution, say, MAm according to protonated before anion exchange is possible. Secondary
or primary amines also require protonation before anion
m nR3 NH aq MAmmn aq , R3 NH mn MAmmn aq: exchange. On the other hand, quaternary ammonium com-
i 4 i 7 i 6 pounds, used as an extraction agent and representable as
5:2:103 R4NA (e.g. R3N(CH3)Cl, where R may be C8, C9 or C10),
can readily undergo anion exchange with other anions in
The ion pair (R3NH)mn MAm mn
with no charge will the system:
partition into the organic phase
R4 N A org

mn aq , R NH mn 5:2:106
R3 NH mn MAm 3 mn MAm org
i 6 i 6 R4 N A aq , R4 N aq A aq
1
R3 NH mn MAmn
m org,
2 mn R4 N aq MAmmn aq , R4 N mn MAmmn aq
i 8
R4 N mn MAmmn org:
5:2:104
5:2:107
and may even dimerize or polymerize. The various chem-
ical reactions and partition equilibria are schematically Micellization of R4NA is also a problem in the organic
illustrated in Figure 5.2.10, where we have included the phase. Such anion exchange is also a powerful technique
dimerization of R3NHA in the organic phase. Note that used in the solvent extraction of amino acids under
the anionic metal complex MAmmn is formed with the appropriate pH conditions so that the amino acid (Am) is
anion A from the acid according to in a form (Am) (see Section 5.2.3.1.2) ready for anion
exchange, for example using a quaternary ammonium
Mn aq mA aq , MAmmn aq: 5:2:105 compound:

Further, there can be a number of such complexes at any R4 N A org Am aq , R4 N Am org A aq:
time, depending on the value of m. 5:2:108
5.2 Chemical reactions change separation equilibria 299

The third approach to metal extraction uses oxygen- 5.2.3.1 Ion exchange resinliquid solution equilibrium
containing solvents like TBP (tributyl phosphate,
There is an extensive literature in this area. Helfferich
(BuO)3PO). Widely adopted in the extraction of many
(1962, 1995) has provided a comprehensive description of
radioactive metals in the nuclear industry, this technique
the effect of chemical reactions on ion exchange equilibria.
may be illustrated for uranium extraction in the presence
Three types of chemical reactions of interest are: complex
of nitrate ions in the aqueous phase by
formation in solution; complex formation with the ion
exchanger; ionization/dissociation in the solution.
UO2
2 aq2NO3 aq 2TBPorg , UO2 NO3 2 2TBPorg:
5:2:109 5.2.3.1.1 Complex formation The complex or com-
plexes formed in solution or in the ion exchanger can
To shift the equilibrium to the right, the NO3 ion concen-
strongly affect ion exchange equilibria. Consider first the
tration is increased in the aqueous phase by adding, say,
equilibria with cation exchangers in the presence of a com-
HNO3 and the level of free TBP in the organic phase is
plexing anion (say, Cl). Let the two competing cations
raised. Under such conditions, nitric acid is also extracted
(counterions) A and B be Zn2 and H. In the presence
into the organic phase by
of a free anion (Y), Cl, obtained from, say, ZnCl2, and
HCl, the cation Zn2 will form a variety of complexes
H aq NO3 aq TBPorg , HNO3 TBPorg: (Helfferich, 1962):
5:2:110
K1 C ZnCl
Detailed considerations of equilibria in such systems for Zn2 Cl ! ZnCl : K 1 ;
C Zn2 C Cl
the extraction of radioactive metals are available in Bene-
dict et al. (1981). Normally water molecules form a shell K2 C ZnCl2
Zn2 2Cl ! ZnCl2 : K 2 ;
around the metal atom, creating the hydration sphere. TBP C Zn2 C 2Cl
has a highly polar group
P O, which can replace water K3 C ZnCl3 5:2:111
Zn2 3Cl ! ZnCl3 : K 3 ;
around the metal. Other extractants, like ethers, alcohols C Zn2 C 3Cl
and ketones, have also been used to the same end, except C ZnCl2
K4
water molecules are not completely excluded. Table 5.2.4 Zn2 4Cl ! ZnCl2
4 : K4
4
,
C Zn2 C 4Cl
illustrates the variety of solvent extraction agents used
commercially for extracting metals in hydrometallurgy.
where K1, K2, K3 and K4 are the stability constants for the
Extracting agents, such as TPB, are rarely used by
complexes formed. Although the overall concentration of
themselves. The extracting agent is normally used along
zinc in the solution is now given by
with a diluent solvent, e.g. kerosene. The purposes behind
the use of a diluent are several. First, the viscosity of the
extractant is considerably reduced by the diluent. Second, C tZn C Zn2 C ZnCl C ZnCl2 C ZnCl3 C ZnCl2 4
;
the inventory of costly extractant is decreased. Diluents, in 
C tZn C Zn2 1 K 1 C Cl K 2 C 2Cl K 3 C 3Cl K 4 C 4Cl ,
addition, control the surface-active tendencies of most
extractants, leading to improved dispersion-coalescence 5:2:112
behavior. A detailed discussion on diluents is available in
the concentration of the free cation Zn2 is now substan-
Ritcey and Ashbrook (1984a, chap. 4). In addition, some-
tially lower. The extent of reduction depends on how
times modifiers like TBP are added to the organic phase to
strong the complexes are. Further, some of the divalent
avoid what is called the third-phase separation (Flett et al.,
cation Zn2 is now either a monovalent cationic species or
1991), namely to prevent the splitting of the metal
even an anionic species; therefore the preference of the
complex-rich organic phase into a metal complex-rich
cation exchanger for Zn is now substantially reduced. On
phase at the aqueous interface and a diluent-rich phase
the other hand, the selectivity of the competing cation
above.
B (H) is increased substantially in the ion exchange reac-
tions with the cation exchanger (R) having fixed negative
5.2.3 Stationarymobile phase equilibria charges since ZnCl3 , ZnCl2
4 do not participate in the cation
exchange process:
Here we consider briefly the effect of chemical reactions on
two types of phase equilibrium: ion exchange resinliquid
solution equilibrium; stationary mediummobile liquid Zn2 aq 2HR , ZnR2 2H aq,
5:2:113
solution equilibrium, as in chromatography (Section ZnCl HR , ZnClR H aq:
7.1.5). For the first case, the resin phase is generally sta-
tionary in a fixed bed; however, the resin phase may also The expressions for the selectivity of A over B are available
be mobile. in Helfferich (1962). The net result in this case is that
300 Effect of chemical reactions on separation

Table 5.2.4. Solvent extraction reagents for hydrometallurgya

Class Type Examples Manufacturer Commercial use

Acidic carboxylic acids naphthenic acids Shell Chemical Co. coppernickel separation
extractants
alkyl phosphoric di-2-ethylhexyl Union Carbide yttrium recovery, europium extraction, nickel
acids phosphoric acid Mobile Oil Co. cobalt separation
(D2EHPA) uranium extraction
octylphenylphosphoric
acid (OPPA)
aryl sulfonic acids SYNEX 1051 King Industries, Inc. magnesium extraction
Acid hydroxyoximes LIX63, LIX64N, LIX65N, Henkel Corporation copper and nickel extraction
chelating LIX 70
extractants
SME 529 Shell Chemical Co. copper extraction
P5000 series Acorga Ltd. copper extraction
oxime derivatives Kelex 100 Sherex Chemical proposed for copper extraction
Cob.
-diketones Hostarex DK16 Farbwerke Hoechst proposed for copper extraction from ammoniacal
AG solution
LIX54 Henkel Corporation copper extraction from ammoniacal solution
X151 Henkel Corporation proposed for cobalt extraction from ammoniacal
solution
alkarylsulfonamide LIX34 Henkel Corporation proposed for copper extraction from acidic leach
polyols Dow Chemical Co. liquors
boron extraction
Anion primary amines Primene JMT Rohm and Haas no known commercial use
exchangers
secondary amines LA-2 Rohm and Haas zinc and uranium extraction
Adogen 283 Sherex Chemical zinc and tungsten extraction
b
Co.
tertiary amines various alamines; in Henkel Corporation widley used; cobalt, tungsten, vanadium, uranium
particular, Alamine extractions, etc.
336
various adogens; in Sherex Chemical cobalt, vanadium and uranium extractions
particular, Adogen Co.b
364, Adogen 381,
Adogen 382
quaternary amines Aliquat 336 Henkel Corporation vanadium extraction; other possible uses are
chromium, tungsten and uranium extraction
Adogen 464 Sherex Chemical similar to Aliquat 336
b
Co.
Solvating phosphoric, tributylphosphate Union Carbide, nuclear fuel reprocessing, U3O8 refining, iron
extractants phosphonic and (TBP) Albright and extraction, zirconiumhafnium separation,
phosphinic acid Wilson niobiumtantalum separation, rare earth
esters separations, acid extraction
phosphonic acid esters Farbwerke Hoechst no known commercial use
trioctylphophine oxide AG, Henkel recovery of uranium from wet process
(TOPO) Corporation, phosphoric acid liquors (with D2EHPA)
Cyanamid
various alcohols, butanolpentanol various phosphoric acid extraction
ethers, ketones diisopropyl ether various phosphoric acid extraction
methyl isobutyl ketone various niobiumtantalum separation, zirconium
(MIBK) hafnium separation
alkyl sulfides Di-n-hexyl sulfide palladium extraction
a
Reprinted with permission from D.S. Flett, J. Melling and M. Cox, Commercial solvent systems for inorganic processes, in T.C. Lo,
M.H.I. Baird and C. Hanson, Handbook of Solvent Extraction, Table 1, p. 631, Wiley-Interscience (1983). Copyright 1983, John Wiley & Sons.
b
Previously Ashland Chemical Co.
5.2 Chemical reactions change separation equilibria 301

Table 5.2.5. A few amino acids: their structures and dissociation constantsa

Name R pK1 COOH pI pK2NH


3 pK3 R-group

Glycine H 2.34 5.97 9.6


Alanine CH3 2.31 6.02 9.70
Valine (CH3)2CH 2.33 5.97 9.76
Leucine (CH3)2CHCH2 2.27 5.97 9.57
Phenylalanine C6H5CH2 2.17 5.48 9.11
Glutamic acid HOOCCH2CH2 2.18 3.22 9.59 10.68
Lysine H2NCH2CH2CH2CH2 2.19 9.74 9.12 12.48
a
General formula:

NH3+ CH COO

See Bailey (1990); Martell and Smith (1974, 1982); Lehninger (1982); Saunders et al. (1989).

uptake of the metal zinc from the solution is reduced by highly specific interaction, high selectivities can be
complexation of the metal with a complexing anion (Cl) in obtained. It depends on the coordination valence of the
the solution. Additional examples of excellent complexing ligands and the solution concentrations (Helfferich, 1962).
agents are anions of weak acids, such as citric acid, ethy-
lenediaminetetraacetic acid (EDTA), etc. (Helfferich, 1962). 5.2.3.1.2 Ionization/dissociation of amino acids Large-
The situation regarding cation A uptake by an anion scale production of amino acids frequently use ion
exchange resin is different if there is a complexing anion exchange processes. Amino acids are amphoteric mol-
(e.g. Cl) present. For the same cation Zn2, which would ecules that can exist as anions and cations depending upon
not be preferred by an anion exchanger, we now have the pH of the solution. There are three types of amino acids
anionic complexes ZnCl3 , ZnCl24 . These can now success- depending on the nature of the side chain R if we represent
fully participate in the ion exchange process and lead to the amino acid as NH2CHRCOOH: the R-group is not
substantially increased metal uptake when very little would ionizable; the R-group is negatively charged at pH 7; the
have been possible in the absence of the complexation R-group is positively charged at pH 7. Consider now the
with the anion Cl. first case with a nonionizable R-group (examples are
The above two examples considered the role of com- alanine, leucine, glycine, valine, etc.) (see Table 5.2.5).
plexation reactions taking place in the solution on the ion The following forms of dissociation equilibria exist in
exchange equilibria. The effect of complex-forming cations solution:
in the ion exchangers on the partitioning of anionic and
nonionic ligands from the solution is also of interest. K1

NH
3 CHCOOH , NH3 CHCOO H
Examples of complexes in the cation exchange resin (fixed
j j 5:2:115a
charge group, R) formed by cations like Cu2 (and Ni2, R
  R 
Ag, etc.) are (R)2 CuNH3 2 2
4 , (R )2CuH2 O4 . Poten- Am Am
tial ligands from the solution are ammonia, aliphatic
amine, polyhydric alcohols, etc. Consider the following K2
NH
3 CHCOO , NH2 CHCOO H
ligand exchange between ammonia present in the resin j j 5:2:115b
as a ligand with ethylene diamine (NH2C2H4NH2) (EDA):
 R  R
Am Am

R 2 CuNH3 2
4 2NH2 C2 H4 NH2 aq ,
 Here Am is the zwitterion form of the amino acid, whereas
R 2 CuNH2 C2 H4 NH2 2

2 4NH3 aq: 5:2:114 Am is the cationic form and Am is the anionic form of
the amino acid.5 From equation (5.2.115a), the dissociation
Ammonia present in the solution had previously formed a equilibrium constant K1 is
complex with the metal ion in the ion exchanger, which
acts as a solid carrier for the complexing metal ion. Add-
ition of EDA in the solution allows for an exchange 5
The forms should be more appropriately written as HAm for
between EDA and ammonia, the two ligands in this case. the zwitterion form, H2Am for the cationic form and Am for
Since complex formation of a ligand with a metal ion is a the anionic form, with HAm representing the amino acid.
302 Effect of chemical reactions on separation

C Am w C H w R H  NH
3 CHCOOHaq , R NH3 CHCOOH H aq

K1 : 5:2:116a j j
C Am w
R R
Correspondingly, 5:2:119
C Am w C H w
K2 : 5:2:116b Further, if there are other cations in the system, there will
C Am w
be additional ion exchange reactions. Illustrations of the
If we have a cation exchange resin, we should be interested detailed behaviors of the equilibrium sorption of a number
in the concentration of C Am w , especially as a fraction of of amino acids by a cation exchange resin are provided in
the total amino acid concentration, C tAmw : Saunders et al. (1989) and Dye et al. (1990).
C tAmw C Am w C Am w C Am w : 5:2:117
5.2.3.2 Stationarymobile phase equilibria in
Employing the two dissociation equilibrium relations chromatography6
(5.2.116a,b), we obtain In chromatography, the equilibrium distribution of a
K 2 C Am w K 1 C Am w species between the mobile phase and a stationary phase
C tAmw C Am w :
C H w C H w is crucial. To separate two species, it is essential that the
0
distribution ratio k i1 for two species i A and B is signifi-
A repeat application of (5.2.116a) leads to cantly different. Note that (from equation (1.4.2))
C tAmw mi1 C i1 V 1 V1
C Am w  : 5:2:118a 0
k i1 i1 ,
1 CK1 CK21 K 2 mi2 C i2 V 2 V2
H w H w
where V1 is the volume of the stationary phase (j 1) and
Correspondingly, the concentration of C Am is obtained as V2 is the volume of the mobile phase (j 2). For a mobile
a fraction of C tAmw as liquid phase, the stationary phase may be a solid with
bonded liquid ion exchange resin or a liquid phase. The
C tAmw
C Am w h C2
i: 5:2:118b chemical reactions may be acidbase equilibrium, com-
C H w
1 K2 KH1 Kw2 plexation, ion pairing, etc. First, we consider acidbase
equilibrium for two weak acids HA and HB distributed
To get an idea of which charged form of the amino acid between the mobile liquid phase and a stationary solid
dominates under what pH condition, consider the amino phase, with or without any bonded liquid phase. Assume
acid valine, where R (CH3)2CH, K1 102.33 and 0
that their distribution coefficients A1 0
and B1 are almost
K2 109.76. Further, let the pH be 1, i.e. the solution is identical but their dissociation constants KDA, KDB are
highly acidic. Then quite different. Assume further that only the form HA or
C tAmw C tAmw HB partitions; A and B remain in the mobile phase.
C Am w h 2:339:76
ih i, Using relation (5.2.62), write the effective distribution
1 10101 10 1 1011:33 1010:09
1
2:33
102 coefficients for A and B as
whereas 00 0 C H 00 0 C H
A1 A1 ; B1 B1 :
C H K dA C H K dB
C tAmw C tAmw
C Am w h i 8:76 : 5:2:120
1 10109:76 1010
1 2
1 10 1010:09 
12:09

For species A, there are two limits of the distribution ratio


Thus, the cationic form Am dominates at low pH and is 0
k A1 corresponding to the two values of A1 00 0
, i.e. A1 for
very close to C tAmw . At very high pH, the anionic form Am 0
K dA  C H and A1 C H =K dA for K dA  C H since
dominates. In addition, one can conclude from relation (V1/V2) is fixed for a system. Similarly for species B. It
(5.2.116a) that when pH pK1, C Am w C Am w . Similarly, is obvious that if KdA and KdB (therefore pKA and pKB)
from (5.2.116b) when pH pK2, C Am w C Am w . Table are substantially different, then choosing a pH at an inter-
5.2.5 provides the isoelectric point of the amino acid, pI, mediate value anywhere between pKA and pKB will lead
where the concentrations of the positively charged species to substantially different k A1 0
and k B10
, even though
balance those of the negatively charged amino acids, 0
A1 0
B1 . This is vitally important for an effective separ-
leading to no charge: C Am w C Am w . ation of the peaks in chromatography. (Note: The detector
Although the above conclusions are generally useful, records both the undissociated and the dissociated species
there are other considerations. Consider a low pH where of, say, HA as one species and therefore one peak.)
the Am form dominates and is expected to have successful
exchange with a cation exchanger; however, there is a com-
peting ion exchange reaction with the H ion: 6
To be read along with Section 7.1.5.
5.2 Chemical reactions change separation equilibria 303

Next, we briefly examine the role of weak acid (or weak phase and its retention will be altered. For example, tetra-
base) dissociation imposed on ion exchange equilibrium methylammonium chloride ((CH3)4NCl) can be used
between the mobile phase and the stationary ion exchange for pairing with simple anions; sodium dodecyl sulfate
resin (having, say, fixed positive charge, R). The solute of (CH3(CH2)11OSO3 Na) can be utilized for simple cations.
interest is a weak acid HA with a dissociation constant KdA. Still another method is to use a solutemicelle interaction:
The mobile phase has a monovalent anion C, the counter-
ion in the ion exchange resin, which exchanges with A S M , SM, 5:2:126
obtained from the dissociation of HA:
where S is a solute and M represents a micelle (see

R C resin A , R A resin C :
5:2:121 Figure 4.1.18), which is a spontaneous aggregate of surfac-
tants. The solute distributes itself between the eluent and
We assume that the undissociated acid HA does not partition the micelle; the latter has a very limited affinity for the
into the resin. The equilibrium constant for this reaction is stationary phase. Thus, effectively, the retention of the
solute is decreased. Different solutemicelle distribution
C A R C C
K AC , 5:2:122 equilibria will then help in separating different solutes.
C C R C A
The solutes of interest are ionic or ionizable.
where C A R and C C R refer to the ion exchange phase and The following aspect of some common mobile phases
C C and C A are for the aqueous eluent phase. The value of used in liquid chromatography is important. Polar mobile
the distribution coefficient for species A between the aque- phases are rarely aqueous; generally they are partially
ous phase and the ion exchange resin phase (j 1) is aqueous, e.g. methanolwater or acetonitrilewater, etc.
 A  For water, neutral pH is defined as C H C OH 107
00 C A R K C C C R C A gmol/liter at 25  C, but what is the neutral pH in a water
A1 : 5:2:123
C A C HA C C C A C HA methanol system? For water, the ion product or the
00
Now, KdA C H C A =C HA ; use it to rearrange A1 to obtain autopyrolysis constant Kwater is (using concentrations
 A  instead of activities) Kw (C H ) (C OH ) 1014. Corres-
00 K C C C R K dA pondingly, for methanol, which produces H and CH3O,
A1 : 5:2:124
C C K dA C H the autopyrolysis constant is Kmethanol C H C CH3 O
00 1016.7 (see Bates (1964)). Thus in methanolwater mix-
By choosing the pH properly, A1 can be varied over a wide
00 tures, with the autopyrolysis constant varying between 14
limit. Since KdA << 1 for weak acids, if C H >> KdA, A1 is
0 and 16.7, neutral pH may be said to vary between 7 and
small; therefore k A1 is also small. When, however, KdA >>
8.35, depending on the mixture composition. For water
C H , the acid is fully dissociated, leading to a much higher
00 0 0 acetonitrile mixtures, the range will be even broader since
A1 and k A1 . Note that the value of k A1 for a stationary
Kacetonitrile is 26.5. The subject is quite complex; refer to
liquid or solid (uncharged) phase under this condition is
Bates (1964) to develop a more correct picture of the state
the lowest since A cannot partition into it.
of the protons and anions in such mixed polar solvents.
Complexing agents can be added to the mobile phase
to alter the equilibria between the stationary and mobile
phases. This is especially useful for separating metal ions 5.2.4 Crystallization and precipitation equilibrium
using ion exchange resins. For example, suppose the
We will deliberate first on the role of pH, dissociation, etc.,
mobile phase has ligands L and that a metal ion can
on various crystallization/precipitation equilibria. Con-
complex with it as Mn nL , MLn or as
sider aqueous solutions of amino acids. We have seen in
Section 5.2.3.1.2 that amphoteric molecules of amino acids
Mn mL , MLmnm 5:2:125
can exist in three forms in solution: a cationic form Am at
in general. Now different metals will have different extents low pH; an anionic form Am at very high pH; and at
of binding with the ligand L. Therefore the partitioning intermediate pH, the zwitterionic form, Am, is important.
between the mobile phase and the stationary phase will be The relative ratio of Am or Am to Am depends on the
different for different metals (Karger et al., 1973). On the pH with respect to pK1 or pK2. Only the Am form exists at
other hand, when various ligands have to be separated, one the pI of the amino acid. On the other hand, it is the
can introduce metal ions in the eluent for complexation, as neutral form Am which forms the crystal (j s):
in (5.2.125). Chelating agents like 8-hydroxyquinoline are iw 
examples of ligands used for these purposes.
 5:2:127
NH
3 RCHCOO s , NH3 RCHCOO w ,
Many ionic solutes are poorly retained by nonpolar station-
ary phases. However, if a counterion is introduced into an where iw is the equilibrium solubility parameter (
eluent such that it can form a neutral ion pair with the solute, (NH
3 RCHCOO )w/(NH3 RCHCOO )s) which depends on
then the ion pair can partition into the nonpolar stationary the solute, temperature and solute concentration.
304 Effect of chemical reactions on separation

(a) (b)
1.0
)
35 0.6
total Ile in solution
100g solution

0.4
30
g Ile

25 0.2

Solubility, M
20 0.1
(

15 0.06
Solubility

0.04
10
0.02
5

0 0.01
0 2 4 6 0 5 10 15 20 25 30

)
0.05
pH
HCI concentration ( g HCI
100g solution
2 3 4
pH
5 6 7 8

Figure 5.2.11. (a) The solubility of L-isoleucine as a function of pH and HCl concentration at 25 C (Zumstein and Rousseau, 1989).
Reprinted, with permission, from Ind. Eng. Chem. Res., 28, (1989), 1226, Figure 4. Copyright (1989) American Chemical Society.
(b) SolubilitypH profiles of amino penicillin at 37  C. The points are experimental values. The solid curves were generated from
equation (5.2.130) and other parameters. Key: , cyclacillin anhydrate; , ampicillin anhydrate; , ampicillin trihydrate; , amoxicillin
trihydrate; and , epicillin anhydrate (Tsuji et al., 1978). Reprinted, with permission, from A. Tsuji, E. Nakashima, S. Hamano,
T. Yamana, J Pharmaceut. Sci., 67(8), 1059 (1978), 1978, John Wiley & Sons.

For amino acids there is a band of about 23 pH units increase in C Cl , w will reduce C Am w when their product
around the pI where the net charge is zero. This is identified has already reached Ksp. This decreases C Am w and there-
as the isoelectric band; a pH value beyond this band on fore the total amino acid solubility ( C Am w C Am w )
either side increases the amino acid solubility. For example, from the ionization equilibrium (5.2.116a). One therefore
from equation (5.2.118a), we find that as C H w increases, observes a maximum and then a signifcant drop off in the
C Am w increases; therefore the amino acid solubility total solubility of the amino acid L-isoleucine on increasing
increases since C Am w increases. Zumstein and Rousseau the amount of added HCl and a corresponding decrease in
(1989) have illustrated that, for the amino acid L-isoleucine, pH (Figure 5.2.11(a)).
the C Am w species dominates above pH 2 leading to the If the complication of the salt solubility limit being
lowest solubility. As the pH decreases, the C Am w species exceeded is not present, then the total concentration of
forms more and more and the amino acid solubility the amino acid in solution in terms of the concentration
increases sharply. At pH 1, it reaches a maximum, then of the zwitterionic species C Am , w may be obtained from
it decreases sharply at lower pH (Figure 5.2.11a). equations (5.2.116a, b) as
As the pH is decreased below 2 by the addition of HCl
acid, more and more of the acid form of the amino acid is C tAm w C Am w C Am w C Am w
present in solution. However, there is a solubility limit of
this form of amino acid which is subject to an equilibrium C H K2
with the hydrochloric acid salt (Cl NH3RCOOH H2O)s C Am w C Am w C Am w
K1 C H
in the solid phase via
2
3
Cl N H3 RCOOH H2 Os , N H3 RCOOHw Cl w H2 O: C K 2
C Am w 41 H 5: 5:2:130
5:2:128 K1 C H

There is a solubility product Ksp of the two ions in the While the crystals are formed from the uncharged Am
aqueous solution: species, the solubility of the amino acid in the aqueous
solution is obtained from (5.2.130). This solubility decreases
K sp C N H3 RCOOH, w  C Cl , w : 5:2:129 at first at a low pH as the pH is increased, then it has a
minimum around the pI band since both C Am w and C Am w
As HCl is added, initially C Am w will increase, but so does are zero at pI. Then, as the pH is increased further beyond
C Cl , w . Correspondingly, at some point of acid addition, the pK2, the total amino acid solubility increases again.
5.2 Chemical reactions change separation equilibria 305

This type of behavior has been observed also for of the amino acid are much greater than those of the free
proteins. The solubility of a protein built out of amino acids form. Thus, the mixture of the crystals of L-DOPA and D-
is minimum when pH pI, the isoelectric point. The DOPA 0.5 HCL can be easily separated by sieving (Asai,
minimum is a point not a band. As the pH moves away 1985).
from the isoelectric point, the protein solubility increases. A comprehensive account of the effect of chemical
The presence of a net charge on the protein molecule reactions on crystallization in enantiomeric systems has
increases its solubilization due to its interaction with been provided by Jacques et al. (1981). In the resolution
dipolar water molecules (H as well as OH). Kirwan and of a racemic mixture of acid AH (particular forms are
Orella (1993) have illustrated broadly a similar solubility identified as D-AH and L-AH) by an optically active base
behavior for other biological molecules of smaller mole- B, the following dissociations/ionizations, acidbase reac-
cular weight which produce zwitterions. Observe the solu- tions (leading to the salt, e.g. AHB (undissociated form))
bility variation with pH of amphiprotic solutes such as the and partitioning equilibria are relevant:
-lactam antibiotics, ampicillin, amoxicillin, etc., as deter-
Kd C A C H
mined by Tsuji et al. (1978) (Figure 5.2.11(b)). If the species AH ! A H ; Kd : 5:2:133
C AH
is susceptible to hydrolysis at high pH, then pH effects are
observed only at lower pH in the range of pH < pK1  pI. For both D- and L- forms,
Optical isomers in solution can be separated by a
C DA C H C LA C H
number of techniques, e.g. enzymatic methods, mechanical K dAH ; K dAH ; 5:2:134
resolution, etc. In mechanical resolution of racemic mixtures C DAH C LAH
of optical isomers, a supersaturated solution of a racemic K dB C BH
mixture is seeded with the pure crystal of one of the isomers. B H ! BH ; K dB ; 5:2:135
C B C H
This crystal grows and one of the isomers is separated from
K C AHB
the solution. However, the solution remains supersaturated AH B ! AHB; K , 5:2:136
C AH C B
in the other isomer, which tends to precipitate, resulting in
poor separation of the isomers. for both D-AHB (KD) and L-AHB (KL). Also,
If the racemic mixture consists of ionizable species, the
iw
addition of acid or base to the solution has been found to AHBsolid ! AHBsolution : 5:2:137
stabilize the solution (Asai and Ikegami, 1982). This has
Here, Kd represents the dissociation constant of the enan-
been found to be true for ionizable amino acids, e.g. the
tiomeric acid, L- or D-form; KdB represents the dissociation
L and D forms of glutamic acid, the L and D forms of DOPA
constant of the optically active base B, and K represents the
(3,4-dihydroxy--phenylalanine), etc. (Asai, 1985). Here
acidbase reaction equilibrium constant, KD for the D-form
both the free and the salt forms of the acid are racemic
and KL for the L-form. Further, iw represents the equilib-
mixtures. But the salt forms are much more soluble. The
rium solubility parameter of the salt i. However, the pro-
reaction that takes place, for example, in the presence of
cess is useful because iw varies with the enantiomeric salt
NaOH is as follows:
species i, i.e. whether it is D-AHB or L-AHB. We identify
L A D S , L S D A, 5:2:131 this via Dw and Lw.
To start with, we have a certain total concentration of
where A refers to the free form of glutamic acid, S refers to
the racemic mixture C tAH : it consists of C tDAH ( 0.5 C tAH )
the sodium salt of glutamic acid and L and D refer to the
and C tLAH ( 0.5 C tAH ). However, this total initial concen-
two different isomers. When the DA is seeded, the LA
tration after the process is over will consist of, for example,
form of the acid (not seeded) will be pushed to the LS
form since DA is disappearing by crystallization on the C tDAH C DAH C DA C DAHB C Dp , 5:2:138a
growing crystal. Remember, the LS form, being more
soluble, does not precipitate. where C Dp is the concentration of the precipitate or crystals
An alternative strategy relies on differing crystal sizes of of the D-form of AHB (CLp for L-form). Correspondingly,
the two reaction products, which therefore can be success- C tLAH C LAH C LA C LAHB C Lp : 5:2:138b
fully separated. For example, in the resolution of DL-DOPA
and its hemihydrochloride DL-DOPA 0.5 HCl, the From (5.2.134),
0 1
following reaction occurs:
K dAH A
C LAH C LA C LAH @1 ;
C H
L  DOPA 0:5 HCl D  DOPA ,

L  DOPA D  DOPA 0:5 HCl: 5:2:132 C LAHB Lw , C DAHB Dw ; 5:2:139


0 1
Not only are the solubilities of the salt form much greater K dAH
than the free form, but also the crystal sizes of the salt form C tLAH Lw C Lp C LAH @1 A:
C H
306 Effect of chemical reactions on separation

Correspondingly, adsorbed on a solute (collector) which is surface active. We



treat first the ionic surface-active solute distribution
K dAH
C tDAH Dw C Dp C DAH 1 : 5:2:140 between a bulk liquid phase j l and the interfacial phase
C H
j for a gasliquid system.
If the total initial concentration of the base is C tB , then

C tB Lw Dw C Dp C Lp C B 1 K dB C H : 5:2:141 5.2.5.1 Ionic surface-active solute R X in solution

Now, from (5.2.136), (5.2.140) and (5.2.141), we get, for the An ionic surface-active solute RX in a dilute solution may
L-form, or may not have an excess of another electrolyte present
simultaneously in the bulk solution. Further, this electro-
 t  
C LAH Lw C Lp C tB Lw Dw C Dp C Lp lyte may or may not have a common ion (RY). Select first

an ionic surface-active solute RX without any other elec-
Lw K dAH
1 K dB C
H K dAH K dB : 5:2:142 trolyte. From the Gibbs isotherm (3.3.40a),
KL C H
d12 jT ER RTdlnC R 1 jT EX RTdlnC X 1 jT 5:2:145a
Correspondingly, for the D-form, we obtain
 t 
C DAH Dw C Dp C tB Lw Dw C Dp C Lp
 by considering the R and X ions as separate species. If

the solute RX is completely dissociated in the bulk solu-
Dw K dAH
1 K dB C
H K dAH K dB : 5:2:143 tion (j 1), then
KD C H
C R 1 C X 1 C RX, 1 , 5:2:145b
Knowing C tDAH C tLAH 0.5 C tAH , we divide the expression
for the L-form by that for the D-form to obtain where CRX,1 is the bulk concentration of the surfactant
 t 
solute to start with. The equilibrium relation is now sim-
C Lw C Lp Lw K D
 tLAH  , plified to (since ER EX RX; by (3.3.42b))
C DAH Dw C Dp Dw K L
d12 jT 2 ERX, RTdlnC RX, 1 ;
which may be rearranged to give the following:  
1 d12

Lw K D


Lw K D
 ERX, jT : 5:2:146
C Lp C Dp C Dp 1 C tDAH 1 2RT dlnC RX, 1
Dw K L Dw K L

If 12 a1 b1C RX, 1 , then
KD
Lw 1 : 5:2:144
KL b1 C RX, 1
ERX, : 5:2:147
2RT
Here we have arbitrarily assumed that the D-form of the
salt is more soluble so that CLp CDp > 0. The result Thus, ionization of the solute has reduced its surface
above is pH independent, even though the individual excess by a factor of two (compare relation (3.3.106)) from
value of CLp or CDp depends on pH. Consult Jacques that for a nonionic solute.
et al. (1981, sect. 5.1) for quantitative estimates of various When there is also a large excess of an electrolyte with
constants for an enantiomeric system. Knowing Lw , Dw , a common ion (RY) present in the liquid, the Gibbs
K D and K L for a system, and the value of C tDAH , one can isotherm becomes
find out the difference in the amount of crystals of the two d12 jT ER dR 1 EX dX 1 EY dY 1 : 5:2:148a
enantiomers.
Suppose the concentration of the surface active solute
RX is changed in the bulk solution; the concentration
5.2.5 Surface adsorption equilibrium of the anion Y is essentially unchanged, i.e. dY l 0. For
We have considered the equilibrium distribution of a a dilute solution, we can then write
nonelectrolytic surfactant solute between the bulk liquid

d12 jT ER dlnC R 1 EX dlnC X 1 RT: 5:2:148b
phase and the interfacial phase in a gasliquid system via
relation (3.3.106). We have thereby illustrated the applica- But, due to any addition of RX, dC R 1 dC X 1 . Further,
tion of the Gibbs adsorption isotherm (3.3.40a) to a single C R 1 >> C X 1 due to a large excess of RY. Then, assum-
nonionic surface-active solute. Chemical reactions can ing ER and EX to be of the same order of magnitude,
influence such adsorption isotherms in a number of ways. dC R 1 dC X 1
If the surface-active solutes are ionic, the adsorption equi- ER << EX ,
C R 1 C X 1
libria are affected. In other cases, the solute to be removed
(the colligend) is not surface active but it reacts with or is which leads to
5.2 Chemical reactions change separation equilibria 307

d12 jT RT EX dlnC X 1 RT Ei dln C il : 5:2:149 E1, =C 1, 1


12 , 5:2:153
E2, =C 2, 1
Thus, the equilibrium relation becomes exactly similar to
that for a nonionic surface-active solute. where species 1 is CO3 and species 2 is Br.
We can now calculate the separation factor between A different aspect of collectorcolligend equili-
two surface-active solutes using the above results. We need brium will be considered now. The concentration of
to assume that the distribution of one does not affect that the surfactant (collector) w.r.t. the concentration of the
of the other. For example, for one nonionic solute i 1 and colligend in the bulk solution is important if the
an ionic solute i 2 in the absence of any other electrolyte, process is described by (5.2.152) followed by surface
the separation factor is adsorption of the complex. Assume the colligend ion to
be i and the free surfactant counterion to be k; the
E1, =C 1, 1 2b11 collectorcolligend complex is ik. Only species k and
12 2 , 5:2:150
E2, =C 2, 2 b1 ik can adsorb at the interface. Assume that the adsorp-
tion of the complex ik at the interface is proportional to
where the superscript for bl refers to the solute. If both the adsorption of the surfactant species k (Rubin and
surface-active solutes are ionic, the distribution equilibria Gaden, 1962):
of the two solutes are coupled in general. The require-
ments of electroneutrality also have to be used. See the Eik bik
0
Ek : 5:2:154
treatment for two surface-active ionic solutes NaX and
NaY, where EX is a function of EY and vice versa, in But Ek bk0 C k, 1 . Therefore
Rubin and Jorne (1969). An extensive treatment of surface
Eik bik
0
bk0 C k, 1 : 5:2:155
adsorption equilibrium for surface-active solutes is avail-
able in Davies and Rideal (1963).
The effective equilibrium ratio for the colligend is

Eik b 0 b 0 C k, 1
5.2.5.2 Collectorcolligend equilibrium ik k : 5:2:156
C ik, 1 C i, 1 C ik, 1 C i, 1
For solutes that are not surface active, surface adsorption is
possible if a surfactant (i.e. the collector) reacts with the If free surfactant concentration in the bulk solution, Ck,1, is
solute (the colligend). The product may be an ion pair zero, then there is no separation of the colligend between
or a chelate. For an example of the former, consider the bulk phase and the interfacial phase. Such a condition
the following (Grieves, 1982). In a solution of the salt is obtained when the colligend bulk solution concentration
NaCO3, the surfactant added is a quaternary ammonium is greater than that of the collector. On the other hand, if
compound, ethylhexadecyldimethylammonium bromide the surfactant species is present in significant excess in the
(EHDA-Br). The following exchange reaction takes place bulk, there will be no free i, i.e. Ci,1 0. Further, the
between the colligend (CO3 )1 and the collector (EHDA, surface excess of the complex is considerable, especially if
the exchanger): Ck,l Cik,l.
An altogether different aspect of collectorcolligend
EHDA-Br CO3 l , EHDA-CO3 Br 1 ,
    equilibria arises if the surfactant concentration exceeds
the critical micelle concentration (c.m.c.). Beyond this
5:2:151
concentration, the surfactant solution becomes colloidal
in nature, with individual surfactant molecules coming
where subscripts and l refer to the surface and the bulk
together and forming aggregates called micelles (Section
liquid phases, respectively. The above represents a surface
4.1.8). Any surfactant added is used for such aggregate
exchange reaction between ClO3 ion in the bulk being
formation; the gasliquid interfacial layer is saturated with
exchanged with the Br ion in the surfactant molecule
surfactant molecules. The adsorption capacity of a col-
adsorbed at the surface.
lector molecule present in a micelle for a colligend is
If there is no surface exchange, ion-pair formation can
different from that of a collector molecule present in the
take place in the bulk solution,
gasliquid interface. Lemlich (1968) has derived an expres-
sion for the difference between the colligend bulk concen-
EHDA-Br1 CO3 1 , EHDA-CO3 1 Br 1 ,
   
tration below the c.m.c. C 1i and the c.m.c. C 2i in terms
   

5:2:152 of the collector surfactant concentration in excess of the


c.m.c. level C ck , i.e.
and then surface adsorption of the (EHDA-CO3 ) species
occurs. The selectivity of the process (5.2.151) may be C 2k C ck E2
C 2k C ck : E2 2
i bi C i ; C 1i C 2i i
 
: 5:2:157
expressed by E E2
k
308 Effect of chemical reactions on separation

Here the surfactant molecules in the interfacial layer (B) when EDTA is the complexing agent (Wallace, 1967).
are E times as effective in absorbing a colligend A selectivity AB may be defined by
molecule as a surfactant molecule in a micelle. One
possible value of E is 1. C tA1 C tB2 C A1 C B2 1 K A C C1 1 K B C C2
AB :
C tB1 C tA2 C B1 C A2 1 K A C C2 1 K B C C1
5.2.6 Complexation in Donnan dialysis 5:2:163
We know already from the Donnan equilibrium condition
Now, if CC2 is 0, KB << KA and ZA ZB (say), then
(Section 4.3.2) that two electrolytes AN and BN separated
by a cation exchange membrane in two solutions 1 and 2 C A1 C B2
will distribute themselves such that AB 1 K A C C1 : 5:2:164
C B1 C A2
1=Z A
C B1 1=Z B
  
C A1
: 5:2:158a It is known that, without the complexation reaction, there
C A2 C B2 is no selectivity between A and B. It is now clear that the
(This equation is obtained from equation (4.3.18), where complexation provides considerable selectivity in favor of
we have replaced the ionic activity by concentration; fur- species A. Most of the above analysis is based on the study
ther, the superscript primes and double primes have been by Wallace (1967), with one basic underlying assumption,
replaced by subscripts 1 and 2, respectively.) Here the namely that the complexing agent C does not go through
membrane has the same charge as the anion N, so that the ion exchange membrane.
only the cations are transported through the cation
exchange membrane. Now, solutions 1 and 2 can contain 5.2.7 Enzymatic separation of isomers
the same cation complexing agent C or different cation
complexing agents. Assuming that the same agent C is Enzymatic reactions can be stereospecific. Thus enzymes
present in both solutions, we can write the complexation can distinguish between two enantiomers and react with
reaction as only one of the enantiomers (Jones, 1976):

AZ A C ! AC, 5:2:158b enzyme


X a b c d 
! X a b c e d fast reaction; 5:2:165
whose equilibrium constant KA is enantiomer I
C AC
KA 5:2:159 enzyme
X d c b a 
! very very slow reaction:
CA CC 5:2:166
enantiomer II
for each solution. The total concentration of species A in
each solution is the sum of free ions AZ A and the complex The enzymatic catalysis has converted the group d to e
AC: with enantiomer I, which acted as a good substrate. Enan-
tiomer II is, however, a very poor substrate, so it is essen-
C tA1 C A1 C AC1 C A1 K A C A1 C C1 5:2:160a
tially unchanged. Such an approach has been used to
and separate (resolve) racemic mixtures of amino acids. For
example, the N-acetyl L-tyrosine ester is available as a
C tA2 C A2 C AC2 C A2 K A C A2 C C2 : 5:2:160b racemic mixture of the L-form and the D-form. If the
enzyme chymotrypsin (CT) is used to catalyze the hydroly-
The ratio of the total concentrations of species in the two
sis of this ester in two forms at pH 8, 25  C and 18%
solutions is obtained by using (5.2.159):
CH3OH solution, only the L-form is hydrolyzed to the acid
C tA1 C A1 C AC1 C A1 1 K A C C1 (Jones and Beck, 1976) (see Figure 5.2.12).
: 5:2:161
C tA2 C A2 C AC2 C A2 1 K A C C2 We have seen in Section 5.2.4 that the different forms
of the racemic species (free and salt forms) have different
For species B, we obtain similarly tendencies, for example for solubility. In the case of the
C tB1 C B1 1 K B C C1 reaction shown in Figure 5.2.12, the L-acid, being highly
, 5:2:162 ionizable compared to the D-ester, can be separated by
C tB2 C B2 1 K B C C2
using an oil membrane (Matson and Quinn, 1986), which
which assumes that the concentration of the complexing allows the ester to pass through it but not the ionized
agent C is sufficiently large such that complexation with L-acid. Large-scale processes are in operation in Japan
A still leaves a very large amount of C for complexation where the enzyme aminoacylase is used to separate a
with B. In reality, one of the ions should have a high value racemic mixture of D and L isomers of amino acids con-
of K, while that for the other should be low. For example, taining an acetyl group by selective removal of the acetyl
KA >> KB, which is true for, say, copper (A) against silver group from the L-isomer.
5.3 Rate-controlled equilibrium separation: reactions 309

CT
HO CH2 CHCOOR HO CH2 CHCOO + H+

NHCOCH3 NHCOCH3
R=C2H5, D, L-Ester L-Acid

+ D-Ester + C2H5OH

Figure 5.2.12. Enzymatic hydrolysis of the L-form of an ester.

5.3 Rate-controlled equilibrium separation pAi H CA C Ai , 5:3:2


processes: role of chemical reactions
to be valid, with no interfacial resistance. At steady state,
In Section 5.2, we considered the change in separation the flux of species A from the gas phase must equal the flux
equilibrium due to chemical reactions. Many such separ- into the liquid phase at the gasliquid interface (through
ation processes in practice do not have phases in equilib- the liquid film):
rium. This may have come about, for example, due to
inadequate contact time between the phases in the device N A k g pAb pAi k C Ai C Ab , 5:3:3
being used. The extent of separation achieved will be con-
which can be rearranged to yield
trolled by the extent of transfer of the species between the
p H CA C Ab
 
phases. Other conditions remaining constant, the higher
N A Ab : 5:3:4
the rate of transfer, the larger the extent of separation 1 H CA
kg k
achieved. The species transport rate from one phase to
another then controls the actual separation achieved. If a liquid-phase reaction is taking place between the gas
Chemical reactions can influence this interphase species species A being absorbed and a species C in the liquid
transport rate. The role of chemical reactions in the separ- phase,
ation achieved in rate-controlled equilibrium separation
processes is, therefore, the subject of this section. Mass aA A cC C ! pP qQ, 5:3:5
transfer and separation in gasliquid systems are covered
first, followed by liquidliquid systems. then the first and the obvious effect is that the liquid-phase
bulk concentration CAb is reduced. This therefore increases
5.3.1 Absorption of a gas in a reactive liquid the flux NA for the absorption of gas species A by increasing
the concentration difference driving the mass transfer.
In industrial processes, a particular gas from a gas mixture Such an increase in flux of A (and therefore separation) is
is absorbed in a liquid under conditions where both phases valid for almost all separation processes where a chemical
are flowing, or are agitated in general. During this process, reaction removes free species A from the phase into which
the gas species A (say) has to diffuse through the gas-phase species A is transferred (i.e. the receiving phase). If this
film to the gasliquid interface, where it is absorbed, and reaction rate is slow enough not to alter the gradient of
then it diffuses through the liquid film to its bulk or reacts species A profile at y 0, the gasliquid phase interface,
with a species C in the absorbent liquid (Figure 5.3.1(a)). the increase in NA due to a reduction in CAb is all that can
Assume that there is no interfacial resistance to gas absorp- be achieved. In Section 5.4, we will find a similar increase
tion and that the interfacial region is planar with no thick- in separation due to chemical reaction in the receiving
ness. Assume further that we have steady state so that the phase in rate-governed separation processes utilizing
concentration at any location does not change with time. membranes.
Consider first Figure 5.3.1(a). Gas species A diffuses A chemical reaction of this nature in the receiving
through the gas film as its partial pressure is reduced from phase is helpful from another point of view. The receiving
its bulk value pAb to the value pAi at the gasliquid inter- phase volume or flow rate needed to maintain a certain
face. The molar flux and the transfer rate (in an unit of level of CAb is now reduced. Even though species A is
absorber volume) of species A are, respectively, coming to the receiving phase, its concentration cannot
N A k g pAb pAi and W A k g apAb pAi , build up due to the reaction with species C.
If the chemical reaction in the liquid phase takes place at
5:3:1
a substantial rate in the liquid film, it has an additional effect
where a is the gasliquid interfacial area for absorption per on the species A flux that is fundamentally more complex.
unit absorber volume. At the interface, assume Henrys law, Yet it is quite beneficial to separation in general. Chemical
310 Effect of chemical reactions on separation

Gas film Liquid film


(b)
dg d dg d
Bulk gas Bulk liquid
pAb (a)
Bulk gas pAi CAi CCb
Bulk liquid without
reaction
pAi No or very slow
Fast reaction
CAi reaction CAb 0
CAb 0
y
Gas-liquid
phase with
CAb interface reaction
Gas-liquid y
phase y y + y
interface

Gas film Liquid film

dg d
Bulk gas

pAi CAi (c)

Bulk liquid

Instantaneous
CA Cc
reaction
Gas-liquid
phase CAb = 0
interface

Reaction
d interface

Figure 5.3.1. Different mass-transfer regimes in absorption of gas A in a liquid containing reactant C.

reaction essentially enhances the mass-transfer coefficient k Rearrange this linear second-order homogeneous ordinary
of species A in the liquid phase over the value obtained differential equation to obtain
without any reaction other conditions remaining the same.
d2 C A k 1
We understand such behavior first by using a system where C A 0: 5:3:10
dy2 DA
gas A dissolved in the liquid reacts irreversibly in the liquid
film such that the reaction is first order or pseudo-first order The solution satisfying the boundary conditions is (Hatta,
(Danckwerts, 1970; Froment and Bischoff, 1979). 1928, 1932)
At steady state, a mass balance of species A over an " s s#
element of thickness y in the liquid film, with diffusion 1 k1 k1
only in the y-direction, leads to (Figure 5.3.1(b)) CA p C Ab sinhy C Ai sinh y ,
sinh R DA DA
dC A  dC A  5:3:11
DA  DA  RA y 0, 5:3:6
dy y dy yy
where R DAkl/k 2 .
Note that, in film theory descriptions
where RA is the molar rate of production of A per unit of mass transfer (equations (3.1.141) and (3.1.142)), k
volume. For a first-order irreversible reaction in A, DA = when there is no reaction.
RA k 1 C A , 5:3:7 The mass flux of species A at the gasliquid interface
(y 0) for this reaction-enhanced gas absorption is
where kl is the first-order reaction rate constant. Taking the obtained by differentiating (5.3.11):
limit as y ! 0, we get  p  
dC A  DA R C Ab
N rA jy0 DA  p C Ai p ,
d2 C A dy y0 tanh R cosh R
DA k 1 C A 0, 5:3:8
dy2 5:3:12
where, at
where the superscript r indicates reaction. If there was no
y 0, C A C Ai ; y , C A C Ab : 5:3:9 reaction between species A and B in the liquid film (but
5.3 Rate-controlled equilibrium separation: reactions 311

there will be a reaction in the liquid bulk), then the gas An additional feature worthy of note is that, since
absorption flux is equal to that observed in physical N rA jy0 > N A jy0 , the concentration gradient of species
absorption (Figure 5.3.1(a)), A in the liquid at the gasliquid interface in the presence
of a chemical reaction (see the solid line in Figure 5.3.1(b))
N A jy0 DA = C Ai C Ab : 5:3:13
is sharper than that without any chemical reaction (see the
Consider now the limiting case of rapid reactions when dashed line), other quantities like diffusion coefficients,
p p etc., remaining unchanged:
R >> 1. First, CAi >> (CAb/cosh R ). Often CAb is
assumed to be zero under these conditions. Then assume jdC A =dyjry0 > jdC A =dyjy0 :
that CAb<< CAi for the no reaction case and get
p In fact, the faster the reaction, the sharper the profile. Gas
N rA jy0 R
p 5:3:14 molecules of species A dissolved at the gasliquid interface
N A jy0 tanh R
can diffuse through the film or react. If the reaction rate is
(Danckwerts, 1970). For values of R < 1, the enhancement very high, molecules of A do not get much of a chance
factor , by which the chemical reaction enhances the flux of to diffuse far from the interface. Often, molecules of
absorption of species A over that in the absence of a reac- A are completely consumed before the bulk liquid is
tion, is only marginally greater than 1. For R > 1, however, approached. Such a profile of species A is shown in Figure
the value of increases sharply and can reach values of the 5.3.1(c). This behavior is characteristic of instantaneous
order of 101000. Other conditions remaining constant, R reactions.
increases as the reaction rate constant k1 increases. In fact, The profile of species C with which species A reacts
p p
for large values of R, tanh R ! 1 so that ! R . Such (equation (5.3.5)) is also of interest. Concentration of C
reactions are characterized as fast reactions. decreases from the bulk of the liquid phase to the phase
Consider now a special case where CAb 0. Then interface to the extent it is consumed by the reaction. In the
DA extreme case of an instantaneous reaction, molecules of
N A jy0 C Ai k C Ai 5:3:15 A and C cannot coexist. There develops a reaction plane in

the liquid film at a distance 0 from the gasliquid inter-
and
face, where the concentrations of both species A and C are
p
DA R r zero (Figure 5.3.1(c)). Such a condition is sustained only if
N rA jy0 p C Ai k C Ai , 5:3:16
tanh R the magnitude of the flux of species C toward this reaction
interface is enough to react stoichiometrically with that of
where k r is the mass-transfer coefficient with reaction. The species A toward the same plane:
enhancement factor can be related to the two mass-
cC r
transfer coefficients, k and k r , by N j 0 N rC jy 0 : 5:3:21a
p aA A y
N rA jy0 k r R
p : 5:3:17 For linear concentration profiles for both species,
N A jy0 k tanh R

Therefore, the chemical reaction has increased the liquid- c C DA C Ai DC C Cb


5:3:21b
phase mass-transfer coefficient to k r from k. For high R aA 0 0
values, we find
since
DA p
N rA jy0 R C Ai k r C Ai : 5:3:18 C A jy 0 0 C C jy 0 : 5:3:21c

This implies This provides



k r 0  : 5:3:22
p
DA k 1 ; 5:3:19
1 DDCACCCbAi caCA
that is, the mass-transfer coefficient does not depend on
the liquid film thickness for a fast reaction. However, the Therefore the flux of species A being absorbed under
gas film thickness will still influence the gas-phase film chemical reaction is
coefficient kg, which does affect N rA jy0 . For CAb 0, we

can describe this dependence by combining (5.3.12) and C Ai 0 DA C Ai DC C Cb aA
N rA jy0 DA 0 1 : 5:3:23
(5.3.3) into DA C Ai c C

pAb The enhancement factor is given by


N rA jy0 h HC
i, 5:3:20
1
kg kA
N rA jy0


DC C Cb aA
p 1 : 5:3:24
where, for high R, k k r DA k 1 . N A jy0 DA C Ai cC
312 Effect of chemical reactions on separation

Correspondingly, if CAb is zero for the nonreaction case, concentration gradient is sharper than that without any
reaction at the gasliquid interface. However, the concen-
N rA jy0 k r C Ai ; N A jy0 k C Ai , 5:3:25
tration gradient at the liquid filmliquid bulk boundary is
and smaller:
 dC A r  dC A r
     
N rA jy0

 dC A 
k r DC C Cb aA   >   >   :
1 5:3:26  dz  dz y0
 dz y
N A jy0 k DA C Ai cC y0

yields the enhancement factor for such an instantaneous Therefore, if there is reaction in the film, the flux of gas
reaction. absorption, N rA jy0 , is larger than the flux of A into the bulk
This result indicates that, as CCb is increased, liquid, N rA jy , i.e. N rA jy0 > N rA jy .
increases. From relation (5.3.22), we find that 0 decreases When the reaction is instantaneous, CAb 0 and
as CCb increases. Although it would appear that, as CCb is N rA jy 0. On the other hand, when the reaction is slow,
increased, increases linearly, this is not so. There is an it is possible to have N rA jy0 N rA jy ; there is no reaction
upper limit set by the rate at which gas species A can in the liquid film (if there is substantial reaction, it takes
dissolve into the liquid through the gas film. As CCb place in the liquid bulk). In general, then, for reactions that
increases, 0 decreases with the reaction interface ulti- are not instantaneous, we should allow for a significant
mately coming to the gasliquid interface when amount of flux of a gas species A into the bulk liquid whose
concentration of A will change with distance or time as the
DC C Cb c C r case may be. More on this will be provided in Chapter 8
N rC jy0 N A jy0 : 5:3:27
aA when reactive gas absorption is analyzed in a countercur-
We identify the value of CCb at which this is possible as rent contacting device.
The presentation so far has been carried out in the
r
c C r cC DA N A jy0 context of a film theory of mass transfer (see Section
C 0 Cb N j : 5:3:28
aA DC A y0 aA DC k 3.1.4) and steady state conditions. There is considerable
literature on other models of mass transfer, e.g. surface
Because the reaction interface is at the gasliquid inter-
renewal theory. Further, unsteady state analyses exist for
face, CAi and CCi are zero. The flux of A through the gas
a number of cases. Detailed treatments are available in
film is
Danckwerts (1970) and Sherwood et al. (1975).
aA DC
N A k g pAb 0 N rA jy0 k C 0 Cb : 5:3:29a
c C DA 5.3.1.1 Absorption of two gases in a reactive liquid
Therefore,
There are a number of cases of industrial gas cleanups
C 0

cC =aA DA =DC k g pAb =k :

5:3:29b where two (A and B) or more gases are being simultan-
Cb
eously absorbed in the liquid. The nature of mass trans-
Increasing CCb to a value higher than this will not lead to port, and therefore separation, will depend on how fast
any further increase in the flux of species A. each species reacts with the nonvolatile reactive species
For instantaneous reactions in general, the reaction C (often the species C has some volatility, e.g. monoetha-
interface will not always necessarily be at the gasliquid nolamine). Each of gas species A and B can either react
interface. For such cases, gas film resistance, as well as the slowly or have a fast or instantaneous reaction with C.
liquid film resistance, can be used to describe N rA jy0 in Thus, there can be a number of combinations,7 many of
terms of pAb: which are treated in Astarita et al. (1983). We
pAb consider here a particular case of considerable industrial
N rA jy0 HC
: 5:3:30a importance, namely simultaneous absorption of H2S
1
kg kA
and CO2 into a reactive aqueous solution containing, say,
Alternatively, methyldiethanolamine (MDEA). We want to show how the
different rates of absorption lead to a high selectivity of H2S
pAb H CA D aA
DA C Cb c C
C
over CO2. The specific reactions are (Haimour et al., 1987)
N rA jy0 H CA
: 5:3:30b
1
kg k
H2 S R2 NCH3 ! R2 NH CH3 HS , 5:3:31
So far we have considered cases where there is consider- CO2 H2 O R2 NCH3 ! R2 NH CH3
HCO3 : 5:3:32
able reaction in the liquid film and, as a result, there is
enhancement in gas absorption. The role of reaction in the
liquid bulk has not been pointed out. For this purpose, 7
Fast (A) fast (B); instantaneous (A) fast (B); fast (A)slow (B);
observe in Figure 5.3.1(b) the nature of the concentration instantaneous (A) slow (B); instantaneous (A) instantaneous
profile of dissolved gas species A in the liquid film. The (B); slow (A) slow (B).
5.3 Rate-controlled equilibrium separation: reactions 313

Gas film At y 0,
Liquid film
Bulk gas Bulk liquid C A C Ai , C B C Bi : 5:3:35c
Region Region
I II At y 0 ,
C A C 0 A , C B 0: 5:3:35d
For region II:
(CO2)
pAb d2 C A
CCb DA k 2 C C C A 0; 5:3:35e
CAi (MDEA) dy2
pBb CA
d2 C C
Concentration

(H2S) CBi DC k 2 C C C A 0: 5:3:35f


dy 2
CB
At y 0 ,
0
C A C A , C C 0: 5:3:35g
di di
y=0 y At y ,

Figure 5.3.2. Concentration profiles for simultaneous absorption of


C A 0, C C C Cb : 5:3:35h
H2S and CO2 in an aqueous solution of MDEA. (After Haimour The film thickness is related to k , CO2 as before by
et al. (1987).)
k , CO2 DA = : 5:3:35i

The first reaction involves only the transfer of a proton; it Define the following nondimensional quantities:
is therefore modeled as instantaneous. The hydration of C A C A =C Ai ; C B C B =C Bi ; C C C C =C Cb ;
CO2 to HCO3 is considered to be a fast reaction. The k 2 C Cb DA
concentration profiles of CO2, H2S and MDEA in the liquid y y= ; y 0 0 = ; R 2 2 :
k , CO2
film should therefore be as shown in Figure 5.3.2. Identify-
ing species CO2 as A and H2S as B, the reactions may be 5:3:35j
rewritten as follows: Equations (5.3.35a) to (5.3.35h) can now be nondimensio-
nalized to equations (5.3.36a) to (5.3.36h). For region I:
k2
A C ! products; 5:3:33a d2 C A
0; 5:3:36a
dy 2
B C ! products: 5:3:33b
d2 C B
0: 5:3:36b
In Figure 5.3.2, we observe two distinct regions in the dy 2
liquid film: region I, where there is no C due to
At y* 0,
instantaneous reaction of H2S, and region II, where
C reacts only with CO2 species A (species B, H2S, being C C
A 1, B 1: 5:3:36c
nonexistent). The governing diffusion equations in region
I will not therefore have the homogeneous reaction term At y y 0 ,
present in equation (5.3.8). On the other hand, the
0 0
governing diffusion equations for CO2 and MDEA in region A C A =C Ai C A y ,
C C
B 0: 5:3:36d
II will have the homogeneous reaction term. For the
For region II:
second-order reaction between CO2 and MDEA, the molar
rate of production of CO2 can be described as d2 C A
R2 C A C C ; 5:3:36e
RA k 2 C C C A : 5:3:34 dy 2



We now identify the governing equations and the d2 C C C Ai DA
R2 C A C C : 5:3:36f
boundary conditions in dimensional form in the two dy 2 C Cb DC
regions. For region I:
At y y 0 ,
d2 C A
DA 0; 5:3:35a C A C A y 0 , C C 0: 5:3:36g
dy2

At y 1,
d2 C B
DB 0: 5:3:35b C A 0, C C 1: 5:3:36h
dy 2
314 Effect of chemical reactions on separation

The problem is not completely specified unless y 0 is (a)


known. An additional relation needs to be developed at
y 0 based on the instantaneous reaction between H2S and
600
MDEA (corresponding to equation (5.3.31)):
CAi
 
dC B  r
 CCb
DB N
dy  y B y 0
0 = 0.01

2SCO2

  400
 dC 
C
N rC  DC 0 ,

aH
dy  y
0.1
y 0

which reduces to
CBi


 200 = 0.001
dC C  C Bi DB dC B  CCb 1.0
: 5:3:36i
dy  0 y C D dy  0
Cb C y
0.01 0.1 gR 1 10
2

Equation (5.3.36a), with the appropriate boundary condi-


tions at y* 0 and y y 0 , yields
(b)
C y 0 1 0
C A A 0 y 1: 5:3:36j
y
60
CAi
*
Equation (5.3.36b) and the boundary conditions at y 0 CCb
and y y 0 generate = 0.01

y
2SCO2

40
C B 0 1: 5:3:36k 0.1
y
aH

Equation (5.3.36f) and the boundary conditions at y y 0 1.0

and y* 1 lead to CBi


20 = 0.001
CCb
h i
DC


C Ai dC A


DC dC C DA CCCb
Ai
C A y 0
: 5:3:36l 0.01 0.1 gR 1 10
C Cb dy DA dy 1 y 0 2

 
Consider now the location y* only. Obtain dC A =dy from Figure 5.3.3. (a) H2SCO2 selectivity as a function of R2 and
   
(5.3.36j). Express dC C =dy in terms of dC B =dy y 0 using different concentration ratios. (b) Selectivity as a function of R2
(5.3.36i) and then use (5.3.36k). Simplify and obtain the and different concentration ratios. (After Haimour et al. (1987).)
following relation:
DB
CCCb
Ai DB C Bi
D
! 1 C A y 0
C Bi DB DA A C Ci ; 5:3:37a
C A y 0 1 y 0 : 5:3:36m y 0
C Ai DA C Ai =C Cb

dC B 1
This is the additional relation needed: unknown y*0 is B j : 5:3:37b
dy y 0 y 0
related to C A (y*0 ). By assuming either y*0 or C A (y*0 ), one
can now integrate the governing equations numerically till The selectivity of the absorption process for H2S over CO2
the boundary conditions (5.3.36h) are matched. Goettler can be defined for this rate-controlled process (as opposed
and Pigford (1971) developed relation (5.3.36m). Numer- to the thermodynamic selectivity of (5.2.20)) as
ical approaches have been studied by Aiken (1982) and
B 1
Haimour et al. (1987). H2 SCO2 : 5:3:37c
A 1 C
A y
0
The enhancement factors A and B for species A and
B (definition (5.3.14)) may be written as Haimour et al. (1987) have calculated values of H2 SCO2 for
different values of the parameters (CBi/CCb) and (CAi/CCb)
dC A r dC A r
 

 against R2. A set of such results is shown in Figures 5.3.3(a)
N rA  DA
dy y0 dy y0  and (b); they demonstrate how the differences in the rates
dC A 
A y0  of absorption can lead to a very high selectivity in chem-
 dC A  C Ai dy y 0
N A  DA ically reactive systems even though equilibrium conditions
y0 dy y0

do not provide that high a selectivity (see (5.2.30)).
5.3 Rate-controlled equilibrium separation: reactions 315

The results of calculations shown in these figures fur- Reaction interface


ther indicate that selectivity decreases when either the Phase interface
H2S content, the CO2 content or both increase for a given
level of MDEA in the solution. In fact, results not shown
do
here indicate that H2 SCO2 can easily come to around 3,
the thermodynamic limit (see (5.2.30)), for high values of CBob
dw
R2 and high levels of H2S and CO2 vis--vis MDEA in the
solution. High selectivity for H2S over CO2 is desirable8 in
CBoi
industrial gas cleanup processes so that gas obtained from
regenerating the spent absorbent solution can be used for
producing sulfur by the Clauss process.
CCoi
CAwb
5.3.2 Solvent extraction of a species with CCob
chemical reaction CAwi
We have considered in Section 5.2.2 the nature of equilib-
rium relationships in solvent extraction of a species in the CDwi
presence of a chemical reaction in either phase or at the
interface.
CDwb
The rate at which such processes occur depends on a
variety of factors. These include: the diffusional rates of the
Aqueous Organic
relevant species in the respective phases, the rates of bulk film film
reactions, and the rates of interfacial reactions, if any.
Consider the following reaction, where we assume that
the reaction takes place on the aqueous side:
Figure 5.3.4. Concentration profiles in aqueousorganic extraction
with an interfacial reaction Aaq Borg Corg Daq.
Aaq Borg , Corg Daq : 5:3:38

An example would be metal extraction by chelation


We first consider the rates of transport of a solute
(see (5.2.92)) with perhaps an interfacial reaction. On the
species being extracted in the presence of a chemical reac-
other hand, the reaction
tion in the interfacial region, the overall reaction being
Aaq Borg , Corg , 5:3:39 (5.3.38). A schematic is shown in Figure 5.3.4, where the
phase interface, shown as a line, actually has a finite thick-
where we assume that the reaction taking place in the ness. The solute A from aqueous phase bulk (say, Mn(aq))
organic phase is more typical of complexation reactions diffuses to the interfacial region, reacts with species B from
of carboxylic acids in the organic phase (see (5.2.82)). In the organic phase, and the products of the reaction, Corg
each case, one of the reactants, whose characteristic bulk and Daq, diffuse, respectively, into the organic and aqueous
phase is not the phase where the reaction takes place, has phases. At steady state,
to diffuse to the aqueousorganic interface and partition
before reaction can occur. N A k Aw C Awb C Awi k s C Awi C Boi k s C Coi C Dwi
k Bo C Bob C Boi k Co C Coi C Cob ,
However, reaction can occur in either phase, contrary
to gas absorption processes where reaction almost always 5:3:40
takes place only in the liquid phase. Further, in metal where the subscripts o and w refer to the organic and
extraction processes, interfacial reactions are common aqueous phase, b and i refer to the bulk and the interface,
and they may proceed at a slow rate, unlike instantaneous k s and k s are the rate constants for the forward and
gas absorption reaction at the gasliquid interface at high backward interfacial reactions, and the diffusional resist-
chemical reactant concentrations (C 0 Cb ; see (5.3.28)). ance of species D in aqueous phase (say, the H ion) is
neglected. For irreversible reactions, the term k s (CCoi
8
CDwi) can also be neglected.
Hindered amines (Sartori et al., 1987) are particularly useful
If we ignore the diffusion of reaction products, there
here. The reaction rate of CO2 decreases with an increase in the
are three resistances of importance: the aqueous-phase
steric hindrance of amines; k2 for an unhindered amine,
dimethylamine, is 7.9  105 (liter/m-s), whereas k2 for a severely diffusional resistance of species A, the organic-phase diffu-
hindered amine is ~1.3  102. But, unlike CO2, H2S reacts with an sional resistance of species B, and the resistance of the
amine at a rate unaffected by steric hindrance since the reaction is interfacial reaction (Baird, 1980). If the interfacial reaction
a simple protonation. rate is extremely slow, then CAwb CAwi, CBoi CBob and
316 Effect of chemical reactions on separation

Phase
Organic interface
Aqueous
phase Film phase
reaction
plane
bulk film bulk

NaPr
Na+
Ci
CAwb
HPr OH
CBob CBwi
Pr
HPr
CBoi CBwb
dlo
o
y dlw dlw

Figure 5.3.5. Extraction and instantaneous reaction of an organic acid B (HPr) in aqueous phase with a base A: film theory
representation. (After Grosjeans and Sawistowki (1980).)

NA ks CAwb CBob; the interfacial reaction controls the The back extraction of propionic acid (C2H5COOH,
extraction rate. If the interfacial reaction is extremely fast, represented here as HPr) present in toluene into an aque-
the diffusion rates become controlling. For example, if the ous solution containing NaOH will serve to illustrate
diffusional rate of A is slow, then CAwi 0 and NA kA liquidliquid extraction with chemical reaction of the type
CAwb (assuming kBo(CBob CBoi) > kAw (CAwb CAwi)). (5.3.41) with two differences. First,
Conversely, if the diffusion of B is quite slow, CBoi 0,
leading to NA kBo CBob. C2 H5 COOHorgNaOHaq ! C2 H5 COONaaqH2 Oaq
We next examine cases where the reaction takes place 5:3:42
in a region away from the interface, e.g. in the liquid film or
the bulk liquid. For this purpose, the overall reaction can is irreversible; second, the aqueous phase contains, in
be (5.3.39) with the reaction taking place in the organic addition, ions Na, OH, Pr, as shown in Figure 5.3.5.
phase after species A has been extracted into the organic Propionic acid partitions into the aqueous phase and dif-
phase. A number of situations are possible. fuses to the reaction interface at a distance 0 w from the
phase interface, where its concentration is zero due to the
(1) No reaction in the organic film; slow reaction in the
instantaneous reaction with the OH ion. The latter dif-
organic bulk.
fuses from the bulk aqueous phase to the interface. Note,
(2) Slow or fast reaction in the organic film.
however, that not only in the bulk aqueous phase, but also
(3) Instantaneous reaction in the organic film, which,
in the aqueous film region w 0 w ), electroneutrality has
under extreme conditions, can push the reaction to
to be maintained between the three ions.
the interface.
Figure 5.3.5 shows a profile (see the dashed line) of
Each case can be studied in a fashion almost identical to propionic acid (species B) in the aqueous-phase film in the
those for gasliquid absorption (see Section 5.3.1). For absence of any reaction. The overall mass-transfer coeffi-
further details, consult Astarita (1967), Lo et al. (1983), cient Ko based on the organic phase in such a case is
Baird (1980) and Sarkar et al. (1980). Similar consider-
N B K o C Bob C B k o C Bob C Boi k w C Bwi C Bwb
 
ations can be applied if the reaction is
and
Aaq Borg , Caq Daq 5:3:41 1 00
1 Bo
, 5:3:43
K o ko kw
and it takes place in the aqueous phase; consider, for
example, removal of COS from liquefied C3C4 fractions where the distribution coefficient for species B, 00 Bo , is
by treatment with aqueous NaOH or alkanolamine solu- defined by
tions (Dewitt, 1980), or transfer of propionic acid from
toluene into aqueous NaOH (Grosjean and Sawistowski, 00 C B C Boi
Bo : 5:3:44
1980). C Bwb C Bwi
5.3 Rate-controlled equilibrium separation: reactions 317




Note that the film transfer coefficients ko and kw are ko DBw
C 0 Awb C Bob : 5:3:53
defined by kw DAw
DBo DBw
ko ; kw : 5:3:45 The enhancement factor under this condition is
o w
The profile of propionic acid in the case of an 
k o 00

instantaneous reaction with caustic soda (species A) is max 1 Bo : 5:3:54
kw
shown by the straight line in the aqueous phase. The
reaction plane is located at a distance 0 w from the phase Grosjean and Sawistowski (1980) have obtained a max
interface. At this location, value of up to 3.17 for propionic acid extraction from
N rB jyw0 N rA jy 0 w : 5:3:46 toluene and its neutralization by NaOH.
It may appear from relation (5.3.51) that increasing the
For linear concentration profiles in each phase, concentration of caustic soda, species A, is uniformly bene-
ficial to the rate of extraction. Consider the extractive
DBW C Bwi DAw C Awb
, 5:3:47 hydrolytic removal of COS from liquid hydrocarbons (e.g.
0 w w 0 w
propylene at a high pressure and ambient temperature) by
since C Bw jy 0 w 0 C Aw jy 0 w . This leads to contacting with a caustic soda solution (Dewitt, 1980). The
rate of hydrolysis of COS by caustic soda in an aqueous
w
0 w h i: 5:3:48 solution by the reaction
1DAw C Awb
DBw C Bwi

Therefore, COS OH ! HCSO2 5:3:55


2 3
has been modeled as an irreversible second-order reaction:
DBw DBw 4 DAw
N rB jy0 N rB jy 0 w
0 C Bwi C Bwi C Awb 5
w w DBw RCOS k 2 C COS, w C OH , w , 5:3:56
2 3
D Aw
k w 4C Bwi C Awb 5: where k2 is the second-order reaction rate constant. The
DBw
overall reaction can be written as
5:3:49a
COS 4NaOH ! Na2 CO3 Na2 S 2H2 O: 5:3:57
However, in the organic phase,
From the hydrocarbon phase distributed in the aqueous
N rB jy0 k o C Bob C Boi : 5:3:49b
phase as droplets, COS will be partitioned into the aqueous
These two expressions, along with (5.3.43) and (5.3.44) phase, and then a diffusion limited fast reaction will take
lead to the following flux expression for the propionic acid place in the aqueous boundary layer. Since the rate of
being extracted: hydrolysis of COS in the aqueous phase by caustic soda
  depends also on the aqueous phase COS concentration,
DAw 00 CCOS,w, increasing C OH , w by increasing caustic soda con-
N rB jy0 K o C Bob Bo C Awb , 5:3:50
DBw centration can lead to a reduction in RCOS by reducing
where Ko is the overall mass-transfer coefficient (5.3.43) for CCOS,w. An increase in the electrolyte (NaOH) concentra-
the no reaction case. The extraction enhancement factor tion will reduce the COS solubility according to the Setch-
for CBwb 0 is enow (1892) relation. Further, COS diffusivity will also be
h i reduced as the caustic soda concentration is increased.
DAw 00
N rB jy0 K o C Bob DBw Bo C Awb Dewitt (1980) has thereby demonstrated experimentally
and theoretically that the COS extraction rate is maximum
N B jy0 K o C Bob
  at an intermediate NaOH concentration.
DAw 00 C Awb
1 Bo : 5:3:51 Since the solubility of COS into an aqueous alkaline
DBw C Bob
solution has some importance here, and the reaction takes
As CAwb increases, increases, reaching a critical place only in the aqueous phase, Bhave and Sharma (1983)
CAwb,C 0 Awb ; the maximum enhancement is then obtained have introduced species such as tricapryl methyl ammo-
as the reaction interface coincides with the phase interface: nium chloride (Aliquat 336) into the organic phase. This
species acts as a phase-transfer catalyst, transferring OH
DAw C 0 Awb
N rB jy0 N rA jy0 k o C Bob 5:3:52 ions into the organic phase near the interface for reaction
w
with COS. Thus COS reacts both in the organic and the
since CBoi and CAwi are zero. This critical concentration, aqueous phase, and the rate of COS extraction is increased
C 0 Awb , is given by considerably.
318 Effect of chemical reactions on separation

Conventionally, solvent extraction with chemical reac- 5.4 Rate-governed membrane processes: role
tion is implemented in a dispersive system where one of chemical reactions
liquid phase is dispersed as drops in the other immiscible
Separation by preferential transport of one species over
liquid phase. Such a separation is also implemented using
another through a membrane can be influenced by chem-
a porous membrane as a phase barrier (see Figure 3.4.11);
ical reactions. The reaction may take place in any one of
the immiscible phase interface is immobilized at the pore
the following locations: membranefeed fluid interface,
mouth of, for example, a solvent-resistant hydrophobic
membraneproduct fluid interface, membrane bulk, bulk
microporous/porous membrane. Solvent extraction or
feed fluid and/or bulk product fluid. Reactions may take
back extraction through such an interface is easily imple-
place simultaneously at more than one of the above loca-
mented without dispersing one phase in the other. Appli-
tions. The reaction may be instantaneous, irreversible,
cations of solvent extraction with chemical reaction in such
reversible, fast, slow, ion pairing, chelating, solutemicelle,
a nondispersive format have been studied for a variety of
etc. We consider various membrane processes in the
systems.
following order: reverse osmosis, ultrafiltration, dialysis,
Back extraction of phenol from methyl isobutyl ketone
liquid membrane separation of gases or liquid solution.
into an aqueous caustic solution has been studied using a
At the end, we introduce electrochemical cell based reac-
porous membrane, which can be either hydrophobic (with
tions at electrodes and selective transport through ion
the organic phase in pores) or hydrophilic (with the aque-
exchange membranes for the purpose of gas separation;
ous caustic phase in pores) (Basu et al., 1990). Copper
we also briefly indicate the role of chemical reactions
extraction by a chelating extractant (HK) in an organic
in separation through a mixed conducting ceramic
diluent via
membrane.

Cu2 aq 2HKo , CuK2 o 2H aq 5:3:58a


5.4.1 Reverse osmosis: solute ionization
was experimentally studied and modeled by Yun et al.
(1993). These investigators also demonstrated extraction The pH of the aqueous solution will determine the extent
of Cr6 present as an anion HCrO4 in an aqueous solution of ionization of solutes like phenol, 2-chlorophenol or
(pH 2.5) by a long-chain alkyl amine (R3N) via ion-pair acetic acid in the solution. If such a solute is ionized, the
formation through the porous membrane: separation of such a solute from water by reverse osmosis
through a membrane will be changed since the ionized
HCrO4 aq H aq nR3 No , complex o: solute and the nonionized solute have different solute
5:3:58b rejections through the membrane. Consider an ionizable
acidic solute species 1; ionization leads to a negatively
As the extraction of a metallic cation (e.g. Cu2, Ni2, Zn2, charged species 2 (see equation (5.2.57)). The total feed
etc.) proceeds via reactions like (5.3.58a), the protons solute concentration, C t1f , is then given by
released into the aqueous solution will decrease the pH.
From Figure 5.2.8(b), it is obvious that the distribution C t1f C lf C 2f , 5:4:1
coefficient 00 mo for the metallic cation decreases sharply
as the pH decreases. Consider the situation encountered where the subscript f refers to the feed solution. Similarly,
in, say, Cr6 extraction via (5.3.58b); as the pH decreases, the total solute concentration in the permeate, C t1p , is
the extraction ability improves. If, therefore, both processes given by
of (5.3.58a) and (5.3.58b) can be implemented simultan- C t1p C 1p C 2p , 5:4:2
eously in the system, the pH of the aqueous phase may be
controlled better. This was implemented in one device in with subscript p referring to the permeate.
Yang et al. (1996a) by using two separate porous hollow We assume now that, other conditions remaining con-
fiber membranes containing two different extractants, one stant, the intrinsic solute rejections of the ionized and
containing a chelating extractant for copper and the other unionized species remain independent of pH. Let the
containing a long-chain alkylamine for chromium. The solute rejection of the unionized solute 1 be R1 and that
aqueous solution contained both copper and chromium; of the solute 2 be R2. Such values for phenol can be
such situations are encountered often in many aqueous determined, for example, by making two measurements
waste streams. Two different extract streams were with a reverse osmosis membrane (say, the FT-30 mem-
obtained, one containing copper and the other containing brane manufactured by Filmtec Inc., Minnetonka, MN); at
chromium. It is possible to have an organic diluent con- a low pH, phenol is undissociated and so R1 is obtained,
taining both a chelating extractant as well as a long-chain while, at a very high pH, all phenol is dissociated and
alkyl amine and extract both cation and anion species present as C6H5O (or, say, as sodium phenolate in caustic
simultaneously into the diluent (Yang et al., 1996b). solution), yielding R2.
5.4 Rate-governed membrane processes: chemical reactions 319

From the ionization constant Kd1 of solute 1 (see 1.00

(5.2.61a))
0.90
C 2f K d1 C 1f =C H f : 5:4:3

Solute rejection, R
0.80
The fraction of this solute present in the feed in the ionized
form is given by
0.70

C 2f 1
f2 : 5:4:4
C 2f C 1f 1 C H f =K d1
 0.60
Phenol
Using the definition of R2 (1 [C2p/C2f]), we obtain 0.50 DP = 2.08 MPa 2-Chlorophenol
Feed: 22 mg/a of each solute 2, 4-Dichlorophenol
1 R2 Calculated rejections 2, 4, 6-Trichlorophenol
C 2p C t1f f 2 1 R2 C t1f : 5:4:5 0.40
1 fC H f =K d1 g 4 5 6 7 8 9 10 11 12 13
Solution pH
Similarly, the permeate concentration of the unionized
solute Clp can be obtained using the definition of intrinsic Figure 5.4.1. Comparison of predicted and actual values of solute
rejection R1 of the unionized species 1, rejection as a function of pH for a solution containing 22 mg/liter
  of chlorophenols. (After Bhattacharyya et al. (1987).)
R1 1 C 1p =C 1f , 5:4:6
as
C H f =K d1 reverse osmosis permeate (Pusch et al., 1976). However, at
C 1p C t1f 1 f 2 1R1 C t1f 1 R1 :
1 fC H f =K d1 g high pH (if the membrane can withstand it; the FT-30
5:4:7 membrane can, whereas a cellulose acetate membrane
cannot), phenol is ionized, and the reverse osmosis mem-
The solute rejection under partially ionized condition, brane rejection is determined by solutemembrane charge
! repulsion.
C 2p C 1p 1 R 2
R1 1 nC o
C t1f 1 H f

K d1
5.4.1.1 Reverse osmosis: solute complexation
C
1 R1 KHd1f R2 R1
nC o R1 , 5:4:8 The rejection behavior of a solute through a reverse
1 KHd1f
1 fC H f =K d1 g osmosis membrane could be significantly altered if the
solute species in feed solution forms a chelate or a complex
where the feed solution pH log10 C H f . When with an added species and the complex dimensions and/or
C H f  K dl at very high feed pH, and all solute is present charge are quite different from those of the original solute.
as ionized species, we see that R R2. When C H f K dl at Lee et al. (1977b) added EDTA (ethylenediaminetetracetic
a low pH, and there is virtually no ionization, R R1. acid) to a feed solution containing MgCl2 and CaCl2 and
Bhattacharyya et al. (1987) have provided such an analysis studied their separation with somewhat open cellulose
and have demonstrated that expression (5.4.8) describes acetate reverse osmosis membranes. They observed that
quite well the observed rejection R at any pH between the rejection of the MgEDTA complex was around 90%,
completely ionized and completely unionized conditions whereas, without EDTA, the Mg2 rejection was around
for phenol and various chlorophenols (CP) for an FT-30 50%; this was clearly attributable to a much larger size of
membrane used in reverse osmosis (Figure 5.4.1). Obvi- the complex. When there was competition for EDTA
ously, the transition from one regime to the other takes between magnesium and calcium present in the ratio of
place around pH pKphenol. 1:1:1, Lee et al. (1977b) concluded that almost all of the
The intrinsic solute rejection value for a particular calcium was present as the complex CaEDTA, whereas
polymersolute system can sometimes be qualitatively very little of magnesium was complexed (pH 6). Thus the
judged by the solubility parameter sp. The solubility par- rejection of Mg2 was much lower than that of MgEDTA
ameter has contributions from dispersion forces, dipole observed without CaCl2 being present.
dipole forces and hydrogen bonding forces (Hansen, 1969;
Hansen and Beerbower, 1971). For many small molecules
5.4.2 Ultrafiltration: complexation
that are highly polar, hydrogen bonding forces dominate.
Phenol is one such species. Figure 5.4.1 shows that, at low Although solute rejection in reverse osmosis can be influ-
pH, the phenol rejection by the FT-30 membrane is low, enced by the size and shape of the solute, high rejection
around 55% due to hydrogen bonding. In fact, in cellulose reverse osmosis depends primarily on membranesolute
acetate membranes, phenol is known to be enriched in the interactions in the presence of a preferential solvent
320 Effect of chemical reactions on separation

interaction with the membrane. On the contrary, solute (a)


retention in ultrafiltration (UF) is very largely determined CH2 CH2 CH2
by the dimensions of the solute, especially the macroso-
lute, with respect to the membrane pore dimensions; COOH
sometimes the solute charge vis--vis the membrane N
charge is quite important. However, for a given membrane,
if the solute size can be increased, membrane solute reten- COOH
tion can be increased. Alternatively, those solutes which
are not normally retained by UF membranes can now be iminoacetic acid
held back by the membrane pores if the solutes are
(b)
attached to other solutes, especially macrosolutes that are
retained by the membrane. This phenomenon is illustrated CH2 CH CH2
for the following types of systems:
(1) complexation of species to be removed with soluble
macroligands rejected by the UF membrane; C
(2) complexation of species to be removed with compon-
ents of a micelle retained by an appropriate UF
NH OH
membrane;
(3) selective binding of a species with soluble proteins
retained by the UF membrane. N

8-hydroxy quinoline
5.4.2.1 Complexation/binding with soluble macroligands
Figure 5.4.2. Examples of complexing agents used to bind metals
Two different types of systems are briefly illustrated here:
(shown attached to polymers).
complexation of metal ions in solution with soluble
macroligands; affinity binding of solutes with water-
soluble macroligands. We consider first the example of
ultrafiltration of water containing metal ions which have R membrane retention or rejection of metal ion
been selectively bound to a water-soluble macromolecu- C n , w C Mn =permeate
1 M 1 : 5:4:9
lar complex. Ordinarily, heavy metals like Cu, Cd, C tMn , w C tMn , w
Hg will not be rejected/retained by an UF membrane.
But if they are complexed with a macromolecule, the The metal complex MLn is formed with n negatively
macromolecular complex will have a high retention in charged ligands L:
the UF membrane (Michaels, 1968a; Strathmann, 1980).
The macromolecule should preferably have a high Mn nL , MLn : 5:4:10
selectivity and a high binding capacity. Such a macro- The aqueous-phase complexation equilibrium constant
molecule suitable for binding the metal ions is obtained K d, MLn for this complexation reaction is
by binding chelating or complexing agents having
molecular weights that are not too high with water- C MLn , w
K d, MLn : 5:4:11
soluble high molecular weight polymers with reactive C Mn , w C L , w n
groups. Examples of polymers are polyethylenimine, The total metal ion concentration C tMn , w present in the
polyacrylic acid, polyvinyl alcohol, etc.; the chelating feed solution before complexation is
agents are illustrated, for example, in Figure 5.4.2.
We will now develop a simple mathematical descrip- C tMn , w C Mn , w C MLn , w : 5:4:12
tion of how the rejection of the metal ion Mn through the
UF membrane is influenced by complexation with a poly- Using this relation and the definition (5.4.9), we get
mer ligand L present as an acid HL form of a polymer. We
C MLn , w
assume that the free metal ion Mn is not at all rejected by R : 5:4:13
C tMn , w
the membrane. Therefore, the concentration of free Mn in
the feed solution, C Mn, w , is the same as in the permeate
From the equilibrium relation (5.4.11), we get
solution, whereas the effective total concentration of the
metal ion (present in free and complexed form) is C tMn, w .
Further, the metal complex MLn is completely rejected by RC tMn , w R
K d, MLn : 5:4:14
the membrane. Therefore C Mn , w C L , w n 1 RC L , w n
5.4 Rate-governed membrane processes: chemical reactions 321

The ligand L, introduced as an acid HL, is obtained via the membrane of polysulfone. The nature of the depend-
following dissociation: ence of the rejection of the metal ion by the UF mem-
brane on the solution pH exhibits some similarities to
HLw , H w L w : 5:4:15
the solvent extraction of a metal ion by acidic extrac-
The dissociation constant of this acid, Kd,HL, is tants shown in Figure 5.2.8(a). If the values of K d, MLn are
sufficiently different for two metals, selective complex-
C H , w C L , w ation of one of the metal species at a lower pH is also
K d, HL : 5:4:16 possible.
C HL, w
The preceding analysis was based on the following
The sum of the free acid and the dissociated polymer acid assumptions: (1) there is only one type of complex MLn
is in the solution; (2) there were no metal hydroxides present
  in the system (it is certainly correct as long as pH < 7). In
C
C THL, w C HL, w C L , w C L , w 1 H , w : 5:4:17 general, however, water-soluble chelating polymers like
K d, HL
polyacrylic acid (HL) etc. can form a number of complexes
However, the total concentration of the polymer acid with metal ions. For example, for divalent metal ions such
ligand introduced into the system, C tHL, w , is as Cu2, Ni2, Co2, Zn2, etc., the following types of
equilibria have been considered:
C tHL, w C HL, w C L , w nC MLn , w : 5:4:18
K1
Rearrange this to obtain M2 HL ! LM H ; 5:4:22
h i
! H ,w C
nC MLn , w C THL, w C L , w 1 K d, HL K2
LM HL ! ML2 H : 5:4:23
1 : 5:4:19
C tHL, w C tHL, w C tHL, w
The complexation stability constants K1 and K2 are
Introduce from here the expression for C L , w into the rela-
tion (5.4.14) to obtain C LM C H C ML2 C H
K1 ; K2 : 5:4:24
h
C , w n
i C M2 C HL C LM C HL
R 1 KHd, HL
K d, MLn  n  n : 5:4:20 In addition, the following, and other types of, complexes
nC n , w
1 R C tHL, w 1 C ML
t have been found to form (Tomida et al., 2001):
HL, w

 n
Rewrite the expression 1nC MLn , w =C tHL, w in the denom- K3
inator as ML2 nHL ! L2 MHLn : 5:4:25
0  1n 0 1n If one defines the coordination number as the number of
n C tMn , w C Mn , w nC t n nC t n
A @1 M , w M , w C M , w A
n
@1 ligands L bonding to the central metal atom, then an
C tHL, w C tHL, w C tHL, w C tMn , w average coordination number navg for the three complex-
0 0 11n 0 1n ation reactions identified above will be (for n 2)
nC tMn , w C t n
@1 C M , w AA @1 n M , w RA :
n
@1
C tHL, w C tMn , w C tHL, w C LM 2C ML2 4C L2 MHL2
navg : 5:4:26
C LM C ML2 C L2 MHL2
Substitute this result into (5.4.20) to obtain
K d, MLn R 1 Another application of soluble macroligands via complex-
h in
n : 5:4:21 ation is in the application of the removal of smaller bio-
1
C H , w 1 R   n C t n
K d, HL C tHL, w 1n CMt , w R molecules. For example, Lee (1995) has illustrated how the
HL, w
antibiotic vancomycin (mol. wt. ~1500) binds tightly to a
Thus, knowing the two equilibrium constants, K d, MLn peptide ligand containing D-alanyl-D-alanine: this ligand
and Kd, HL, the pH and the total amounts of the metal was coupled to 110 -carbonyldiimidazole activated dextran
ion, C tMn , w , and the polymer acid ligand, C tHL, w , added, of 500 000 mol. wt., and the complex was rejected by a
one can predict the overall effective rejection R of the 20 000 molecular weight cut off (MWCO) UF membrane.
metal ion by an UF membrane. For a low pH, where The rejection was enhanced when there was substantial
C H , w >> Kd,HL, R ! 0. For a high pH, where C H , w << concentration polarization (see Section 6.3.3.2) of the dex-
Kd,HL, R tends to a high but constant value. Rumeau tran on the membrane. Earlier studies illustrated affinity
et al. (1992) have demonstrated the validity of this rela- binding of alcohol dehydrogenase (ADH) enzyme to the
tion for Ag2, Cu2, Ni2 and water-soluble polymeric dye Cibacorn blue, which was bound to starch granules
ligands (50 000 and 100 000 molecular weight) of poly- retained by an UF unit having a 500 000 MWCO mem-
acrylic acid using a 10 000 molecular weight cut off UF brane. This enabled purification of ADH from a crude yeast
322 Effect of chemical reactions on separation

(a)

Unsolubilized
Counterions organic (b)
Solubilized
organic Retentate
D
Multivalent ion L
L D D
L
Micelle D
L
Surfactant D
Membrane
monomer
D
Ultrafiltration
Retentate Permeate L
membrane
side

Permeate
side

Figure 5.4.3. (a) Micellar-enhanced ultrafiltration (MEUF) for the removal of a target multivalent cationic metal in an aqueous solution
which also contains an organic solute that is removed. (After Roberts et al. (2000).) (b) Enantiomer separation at the UF membrane by
an enantioselective micelle. (After Overdevest et al. (2000).)

extract (Mattiasson and Ling, 1986). This example illus- micelles (Leung, 1979). In effect, a very high retention is
trates the purification of an enzyme, a larger molecule, achieved for multivalent cations (e.g. Cu2) (Scamehorn
via affinity binding from a solution containing large impur- et al., 1989; Roberts et al., 2000) (Figure 5.4.3(a)), as well as
ity molecules through a larger-pore UF membrane. for small organic molecules in what is called micellar-
enhanced ultrafiltration (MEUF) (Leung, 1979; Dunn et al.,
1985, 1987). Multivalent anions, such as chromate ions
5.4.2.2 Complexation/binding via a micelle
(CrO24 ), can also be removed by using a cationic surfactant,
Three types of systems will be briefly illustrated here: e.g. cetylpyridinium chloride (Christian et al. (1988)).
An alternative strategy for removing metal ions by
(1) binding of multivalent ionic species in water to oppos-
complexation from an aqueous solution involves solubiliz-
itely charged headgroups of ionic surfactants forming
ing lipophilic metal complexing agents in the hydrophobic
a micelle rejected by an UF membrane;
core of micelles (Tondre, 2000). A number of different
(2) complexation of metal ions in water with lipophilic
extractants, solubilized in the micelle, for example 8-
complexing agents solubilized in the hydrophobic core
hydroxyquinoline, Kelex 100, etc. (see Section 5.2.2.4 for
of a micelle;
these extractants) were used to extract Cu2 successfully.
(3) enantioselective micelle preferentially forming a com-
The micelles were retained by an UF membrane. Tondre
plex with one of the enantiomers.
(2000) has also identified other surfactant based colloidal
Micellar systems (introduced in Section 4.1.8) may employ particles, including vesicles (Section 4.1.8), polymerized
ionic surfactants (Section 4.1.9.2) having an ionic headgroup, micelles, etc., used successfully to the same end.
for example sodium dodecyl sulfate, where Na is the coun- The two technqiues mentioned above were directed
terion and there are numerous ionic headgroups DS (repre- toward removing metallic ions from an aqueous solution
senting dodecyl sulfate (C12 SO4 )). Multivalent cations in by complexation with the constituents of a micelle. The
water, e.g. Cu2, will bind very strongly with these ionic next technique involves resolution of enantiomers using
headgroups. These micelles are spheroidal colloidal aggre- enantioselective micelles, as long as the micelles are
gates containing as many as 50100 surfactant molecules. retained by an appropriate UF membrane.
One can select an UF membrane which will retain the micelle For example, Overdevest et al. (2000) used a regener-
and, therefore, the multivalent cations which are bound to ated cellulose membrane (YM3) having a MWCO of 3000
the charged headgroups of the ionic surfactants. Simultan- to hold back micelles of the nonionic surfactant, nonyl-
eously, if there are small organic species dissolved in water, phenylpolyoxyethylene [E10] ether. Chiral selector mol-
they will become solubilized in the hydrophobic core of the ecules of cholesteryl-L-glutamate are anchored in the
5.4 Rate-governed membrane processes: chemical reactions 323

micelle and complex preferentially with the D-form of concentration difference and the solute transfer rate can
the enantiomeric mixture of D-phenylalanine and be increased considerably. For example, phenol from a
L-phenylalanine present in the feed mixture (schematically dilute aqueous waste stream is transferred through a
illustrated in Figure 5.4.3(b)). It appears that the binding of permselective hydrophobic (or oleophilic) membrane to
either enantiomer with the chiral selector could be the dialyzate containing caustic soda, where the reaction
described by means of a Langmuir iostherm. One can then
describe the enantioselectivity between the D and L K
C6 H5 OH NaOH C6 H5 O Na H2 O 5:4:30
species via (see equation (3.3.113d)) as follows:
takes place. The phenolate ion cannot diffuse back to the
C Dm =C Lm feed side through the membrane (Klein et al., 1973).
DL , 5:4:27
C Dw =C Lw Consider species 1 (e.g. phenol), species 2 (e.g. NaOH)
where CDw and CLw are the corresponding molar concen- and species 3 (e.g. sodium phenolate) in a batch dialysis
trations in the aqueous feed solution, and CDm and CLm are system having a feed solution volume Vf and dialyzate
the molar concentrations in the micellar phase. solution volume Vd. If the membrane area separating the
two solutions is Am, the molar rate of change of solute 1 in
the feed chamber is
5.4.2.3 Chiral separation by selective binding
dC 1f Q
Vf 1m Am C 1f C 1d ,
 
with a soluble protein 5:4:31
dt m
Higuchi et al. (1993) initated the exploration of the protein where Q1m is the permeability of solute 1 through the
bovine serum albumin (BSA) acting as a chirally selective membrane of thickness m. The molar rate of change of
complexing agent for the amino acid tryptophan in an solute 1 in the dialyzate chamber is due to permeation
ultrafiltration system. Romero and Zydney (2001) have from the feed chamber and its disappearance by the reac-
studied in detail the stereoselective separation of a racemic tion (5.4.30):
mixture of D- and L-tryptophan in a solution containing
BSA via ultrafiltration. The polyethersulfone UF membrane dC 1d Q1m Am  dC 3d
Vd C 1f C 1d V d

: 5:4:32
had a MWCO of 50 000 (sufficient to hold back BSA com- dt m dt
pletely). L-tryptophan preferentially binds with BSA, so less The membrane is such, that for all t 0,
L-tryptophan is present in the filtrate. The observed UF
rejection, Ri, of the tryptophan enantiomer i molecule C 2f 0 C 3f : 5:4:33a
(mol. wt. 240) as such is essentially zero. However, the
Further, at t 0, C1f C 01f ,
feed concentration, Cif, of the tryptophan enantiomer i is
related to total feed concentration, C tif , by C 1d 0 C 3d : 5:4:33b

C if C tif 1 f r i , 5:4:28 Usually, species 2 is present in the dialyzate in large excess;


one can assume reaction (5.4.30) to be in equilibrium:
where fri is the fraction of the particular enantiomer i which
is bound to BSA and therefore is not available. So the K C 3d =C 1d C 2d , 5:4:34
permeate concentration of the particular tryptophan, Cip, is with C2d being essentially constant. The two first-order
C ip 1 Ri C if 1 Ri C tif 1 f r i : 5:4:29 ordinary differential equations can be converted, using
the Laplace transform and equation (5.4.34), to
The selectivity between L-tryptophan (i 1) and
Q Am 
D-tryptophan (i 2) will be primarily determined by the sC 1f C 01f 1m C 1d C 1f ; 5:4:35
m V f
relative binding tendencies of L- and D-enantiomers with
BSA. For the L-enantiomer, fri will be considerably larger Q1m Am 
1 K C 2d sC 1d C 1f C 1d , 5:4:36
than that for the D-enantiomer. m V d

where the bar below a quantity indicates a transformed


5.4.3 Dialysis: reaction in dialysate one. Elimination of C 1f from these two equations yields

In dialysis (see Section 4.3.1), the rate at which a solute is C 01f Am Q1m  
V d m1
removed from a feed to the dialyzate through the dialysis C 1d , 5:4:37a
membrane is proportional to the concentration difference 1 K C 2d  s2 Ms
on two sides of the membrane. If a chemical reaction can where
be carried out in the dialyzate side between the solute and h
Am Q1m
i
V d m AVmfQm1m 1 K C 2d
an added agent such that neither the products nor the M : 5:4:37b
agent can diffuse to the feed side, then the solute 1 K C 2d 
324 Effect of chemical reactions on separation

Taking the inverse Laplace transform, we obtain 5.4.4 Chemical reactions in liquid membrane
 permeationseparation
Am Q1m  n
V 1 o
d m
C 1d C 01f 1 expMt : 5:4:38 Figure 5.4.4 illustrates a number of ways by which chem-
1 K C 2d  M
ical reactions can influence the transport and separation of
The total amount of phenol transferred from the feed to the a species A by a liquid membrane of thickness m. Figure
dialysate chamber at any time t from t 0, with reaction, is 5.4.4(a) illustrates the simple permeation of species
h ir A through the liquid membrane. The bulk phases on both
V f C 01f C 1f t . Using equations (5.4.32) and (5.4.34),
sides of the liquid membrane could be either gaseous or
observe that liquid. Ignore the resistances to transport of A in these two
dC 1d

Q1m Am

Q1m Am
bulk phases for the time being. Of course, the bulk liquid
1 K C 2d C 1d C 1f : phases should be immiscible with the liquid membrane
dt V d m V d m
phase. Figure 5.4.4(b) indicates that if there is a liquid
Use (5.4.38) to obtain C1f(t) C1d(t) C 01f exp(Mt). phase on the downstream side, the driving concentration
Therefore, difference between the two sides of the liquid membrane
h h i could be increased by reacting permeating species A with
V f C 01f C 1f tr V C 01f C 1d C 01f expMt an agent E. If the reaction is of the type (5.4.30) and the
2 3 product AE cannot enter the membrane, then the extent of
0 4 1 expMt 5 transport enhancement for a batch permeation process can
V f C 1f V : 5:4:39
1 V d 1Kf C 2d be estimated from relation (5.4.39).
An example is provided by Cahn and Li (1974), where
Consider the value of the same quantity, without reaction, phenol is removed from a wastewater feed into a receiving
Vf [C 01f C1f(t)]. The ratio of the values with and without phase containing NaOH through an oily liquid membrane.
reaction, for a very long time, where exp(Mt) ! 0 is given The sodium phenolate is rejected completely by the non-
by polar oily membrane. When the reaction is instantaneous
h ir and the concentration of A is essentially zero at the inter-
V f C 01f C 1f t face y m, the flux of species A is simply proportional to
 
1 K C 2d V f V d
lim h i   : 5:4:40 the feed concentration of species A, CAf, i.e.
t!
V f C 0 C 1f t V f V d f1 K C 2d g
1f
N Ay QAm =m C Af J AZ :
If Vd << Vf, this ratio becomes essentially (1 KC2d). For
the phenolcaustic system, Klein et al. (1973) have calcu- Two questions are of interest here. First, phenol (species 1)
lated K to be around 1.30  104. With a caustic solution is the only species permeating through the liquid mem-
concentration around, say, 1M, the value of this ratio is brane. Is the concentration of the phenol essentially zero in
large compared to 1. Thus the amount of phenol trans- the caustic-containing receiving phase? Second, if this con-
ferred to the dialyzate in the presence of the reaction is centration of phenol is finite (but very small), what is this
much more than that without any reaction. Klein et al. value so that, when the feed phase has this concentration,
(1973) have recommended several types of membranes permeation will stop? These questions are answered here
made of (for example) silane or ethyl cellulose for this using calculations available in Cahn and Li (1974).
problem. Han et al. (2001) have studied such a process Phenol is a weak acid and it dissociates in feed water
using silicone rubber capillary as a polymeric phenol- (j fw) or strip water (j sw) as follows:
selective membrane between a complex wastewater and a
strongly basic solution. Klein et al. (1972) have also illus- C6 H5 OH C6 H5 O H ,
5:4:41a
trated the utility of a similar concept for the removal of a i 1 i 2
basic solute like aniline from a feed solution by dialysis
the dissociation constant being (at 25  C)
through a hydrophobic membrane; high concentration of
sulfuric acid is used in the dialyzate side to maintain a high K d1 C2 C H =C 1 1:28  1010 : 5:4:41b
value of concentration difference of free aniline between
The ionization product of water, Kw, is
the feed and the dialyzate side.
In hemodialysis, urea and other metabolic wastes are K w C H C OH 1014 : 5:4:41c
transferred from blood to the dialyzate through the dialysis
membrane. By incorporating an enzyme, urease, on the Suppose the feed wastewater pH is 7 and the receiving
dialysate side, it is possible to convert urea to ammonia. phase contains 10 wt% NaOH (2.773 M).
However, it is necessary that the membrane surface facing Electroneutrality in the caustic phase requires
the dialyzate has a positive charge, so that the toxic NH 4
ion cannot diffuse back to the blood. C Na , sw C OH , sw C 2, sw 2:773, 5:4:41d
5.4 Rate-governed membrane processes: chemical reactions 325

M Liquid Membrane

(a) (b) (c)

Feed M Receiving Feed M Receiving M Receiving


Feed
Phase Phase Phase Phase Phase Phase

A A A
A A A A
A E A
AB
AE B

y=0 y = dm y=0 y = dm y=0 y = dm

(d) (e) (f)


Feed M Receiving Feed M Receiving M
Feed Receiving
Phase Phase Phase Phase
Phase Phase

A
A E
A A AB A
AB A A
E C ABC
B BC C C A
AE
C
B
EC

y=0 y = dm y=0 y = dm y=0 y = dm

Figure 5.4.4. Various liquid membrane permeation mechanisms. (After Marr and Kopp (1982).) (a) Simple permeation of species A;
(b) simple permeation enhanced by reaction of A with an agent E in permeate; (c) facilitated transport with a reversible complexing
agent B in the membrane; (d) facilitated transport in the presence of a reactive agent E in permeate; (e) countertransport; (f) cotransport.

the reaction being This shows that the total phenol concentration in the
caustic phase,
C6 H5 OH Na OH C6 H5 O Na H2 O: 5:4:41e
C t1, sw C 1, sw C 2, sw C 2, sw , 5:4:41i
Therefore
is essentially equal to the phenolate ion concentration in
the caustic phase; the free phenol concentration is
C OH , sw 2:773 C 2, sw : 5:4:41f
negligible.
In the wastewater phase ( j fw), at pH 7 (i.e. C H , f w
Substitute this into the ionization product relation (5.4.41c) 107), obtain from (5.4.41b)
on the strip water side:
C 1, f w
C H , sw 1014 =2:773 C 2, sw : 5:4:41g C 2, f w 1:28  1010 1:28  103 C 1, f w : 5:4:41j
C H , f w
To obtain the free phenol concentration in the caustic
Therefore, almost all of phenol in the feed phase is present
solution, use (5.4.41b) to obtain
as undissociated phenol:
C 2, sw C H , sw C 2, sw 104
C 1, sw : 5:4:41h
K d1 1:282:773 C 2, sw C t1, f w C 1, f w C 2, f w C 1, f w : 5:4:41k
326 Effect of chemical reactions on separation

Permeation will stop when Smith and Quinn (1980) observed that the CO flux was
increased by more than 100 times from a 5% CO (rest N2)
C 1, f w C 1, sw : 5:4:41l
feed gas when 0.2M cuprous chloride was present in 1M
Use (5.4.41k), (5.4.41h) and (5.4.41i), respectively, to KCl. Additional reactions were also postulated.
obtain, from (5.4.41l) In the above examples, both the feed and the permeate
phases were gaseous and aqueous liquid membranes were
C t1, sw  104 used. Cahn and Li (1976) have demonstrated facilitated
C t1, f w : 5:4:41m
1:28 2:773 C t1, sw

transport separation of the olefin 1-hexene from n-heptane
through an aqueous membrane containing cuprous
This relation shows that permeation stops when the total
ammonium acetate; the permeate phase was made up of
phenol concentration in the caustic phase is about 10 000
n-octane. Kuo and Gregor (1983) have illustrated facili-
times that in the feed wastewater. It would be quite useful
tated transport of acetic acid from an aqueous solution
to calculate how low the phenol concentration in the feed
through an organic membrane of decalin containing
water will become in such a treatment. Cahn and Li (1974)
trioctylphosphine oxide (TOPO: (CH3(CH2)7)3PO), which
have indicated that a 200 ppm phenol feed will be reduced
complexed with acetic acid.
to 7.4 ppm when 50% of the caustic soda is used up.
We will now provide a simplified analysis to determine
the rate at which species A is transported through the
liquid membrane of thickness m when a complexing agent
5.4.4.1 Facilitated transport separation
B present in the membrane facilitates its transport via
Figure 5.4.4(c) considers a case where the liquid mem- reaction (5.4.42). The governing differential equation for
brane contains a species B which forms a reversible com- the species A concentration in the membrane is obtained
plex AB with species A in the liquid membrane: easily from equation (5.3.6) as
kf
A B , AB: 5:4:42 d2 C Am
kb DAm RA 0, 5:4:44
dy2
Now, not only free species A diffuses through the film
(membrane), but also the species AB diffuses from y 0 where RA, the molar rate of production of A per unit of
to y m. If species AB can exist only within the mem- membrane volume, is
brane, then species AB must dissociate at y m and
release A. Thus the total flux of species A through the RA k f C Am C Bm k b C ABm : 5:4:45
membrane is the sum of the flux of free species A and that Therefore
due to the flux of AB. The flux of species A is facilitated by
the presence of a complexing agent B (sometimes called d2 C Am
DAm k f C Am C Bm k b C ABm : 5:4:46
the carrier species). If other species present in the feed dy2
stream do not complex with agent B or AB, their fluxes
Similarly,
are not facilitated or increased. This leads to a better
separation between species A and other species in the feed d2 C Bm
stream. Note: Species B is chosen such that it does not DBm k f C Am C Bm k b C ABm 5:4:47
dy 2
partition into the feed phase or the receiving phase.
Particular examples of such facilitation are given and
below. When a gas mixture of an olefin (e.g. ethylene) d2 C ABm
and paraffin (e.g. ethane) is exposed to an aqueous mem- DABm k f C Am C Bm k b C ABm : 5:4:48
dy 2
brane containing silver nitrate, ethylene complexes with
the silver ion (see Section 4.1.9.3): The boundary conditions are given by
dC ABm dC Bm
C2 H4 Ag NO3 , AgC2 H4 NO3 , 5:4:43a y 0, C Am C 0Am , 0;
dy dy
while ethane does not. This facilitates the transport of 5:4:49a
ethylene through the liquid membrane enormously dC ABm dC Bm
(Hughes et al., 1986). When a mixture of N2 and CO is y m , C Am C Am , 0:
dy dy
exposed to a membrane of an aqueous KCl solution con- 5:4:49b
taining cuprous chloride, CO binds reversibly with the
ligand complex CuCl2
3 formed in the KCl solution:
The preceding two derivatives imply that neither species
B nor the complex AB can leave the system. Further, the
total amount of B present in the system is constant at a
CO CuCl2 2
3 , CuCOCl3 : 5:4:43b total concentration of C tB :
5.4 Rate-governed membrane processes: chemical reactions 327

!
m N tAy DABm K C tB
C Bm C ABm dy C tB m : 5:4:50 eq DA
  1   :
m C 0Am C Am DAm 1 K C 0Am 1 K C Am
0
5:4:58
Of the four boundary conditions involving gradients, only
The ideal separation factor between species A and any
three are really independent. The total flux of species A at
other species in the feed in the equilibrium limit can also
any location in the membrane is given by
be obtained easily. Equation (5.4.58) is the so-called thick
dC Am dC ABm film result.
J t
Ay DAm DABm N tAy : 5:4:51
dy dy An additional simple approximate analytica1 solution
is available in the so-called thin film theory near the
There is no general closed form solution for the concen-
diffusion limit when the Damkohler number A
tration profiles of species A, AB and B. Various approxi-
k b 2m =DA representing the ratio of diffusion time to reac-
mate solutions exist. One limiting solution by Ward (1970)
tion time approaches zero. Smith et al. (1973) have
is of interest, namely the reaction equilibrium limit achiev-
obtained the following estimate of the facilitation ratio by
able either because of very fast reactions or very thick
a power series expansion:
membranes. Under these conditions, equilibrium exists
everywhere in the membrane for reaction (5.4.42): A C B K  
1 O 2A , 5:4:59a
12
K C ABm =C Am C Bm : 5:4:52
where
Now add equations (5.4.47) and (5.4.48) and obtain, after
two integrations, with a1 and a2 being integration C tBm
C B ; 5:4:59b
constants, 1 K C Am
 
DBm C Bm DABm C ABm a1 y a2 : 5:4:53 C Am C 0Am C Am =2: 5:4:59c

Assume DBm DABm since B is usually a bulky molecule. We will now identify some other facilitated transport
Then, applying the derivatives boundary conditions at systems of considerable interest. Separation of CO2 from
either y 0 or y m, we see that a1 0. Further inert gases like O2, N2, CH4 has been achieved by using
substitution in (5.4.50) leads to alkaline-concentrated HCO3 =CO3 solutions:
a2 CO2 H2 O , H HCO3 ;
C tBm C Bm C ABm C tB : 5:4:54 5:4:60a
DBm CO2 OH , HCO3 :
Therefore, Permeability of CO2 is enhanced by an additional gradient
C ABm

K C Am C tBm C ABm
 of HCO3 across the film. Simultaneously, permeabilities of
K C Am C tBm O2, N2 and CH4 are reduced by the salting out effect at high
) C ABm : 5:4:55
1 K C Am concentrations of the added reagent B, say Cs2CO3, K2CO3,
etc. (Ward and Robb, 1967; Bhave and Sirkar, 1987).
We therefore obtain the total flux of species A through the Aqueous liquid membranes for gas separation are
membrane as unstable unless both feed and sweep gas streams are com-
pletely humidified. One can, however, use a glycerol based
dC Am DABm K C tBm dC Am =1 K C Am  liquid membrane and achieve facilitated transport of CO2 via
N tAy DAm
dy 1 dy reactions (5.4.60a) as long as there is some moisture in the
5:4:56 feed stream. Chen et al. (1999a) have illustrated consider-
  able facilitation of CO2 transport and high selectivity for CO2
DAm  0  DABm K C tBm C 0Am C Am
N tAy C Am C Am    : over N2 using different concentrations of Na2CO3 in glycerol;
0
m m 1 K C Am 1 K C Am
{z} |
| the sweep gas flowing was dry; vacuum may be used.
{z}
flux of free extra flux due to We have seen in Section 5.2.1.1.2 how various amines
A complexation in water can react with CO2 and enhance CO2 absorption.
5:4:57 Sodium glycinate has been dissolved in a solution of gly-
cerol to prepare a stable liquid membrane which facilitates
Obviously, if species A is more permeable than other the transport of CO2 via the following reactions in the
species in the feed through the membrane to start with, presence of some moisture in the feed gas stream (Chen
the addition of species B at a concentration of C tB will et al., 2000):
increase its selectivity, as well as the flux, considerably.
The extent of facilitation is expressed by the ratio eq CO2 RNH2 , RNHCOO H ;
5:4:60b
( N tAy =N Ay ): RNH2 H , RNH
3:
328 Effect of chemical reactions on separation

Here, the glycinate ion, designated as RNH2, is obtained


from the dissociation of sodium glycinate:

H2 NCH2 COONa , H2 NCH2 COO Na : 5:4:60c

The transport of H2S in a K2CO3 solution is facilitated by


the following reaction (Matson et al., 1977):

H2 S CO3 , HS HCO3 : 5:4:61

In fact, H2S is selectively transported over CO2 at low


partial pressures of CO2 and H2S. The transport of SO2 is
considerably facilitated by an alkaline or neutral solution of
sodium salts (NaCl, NaOH, NaHSO3 or Na2SO3) (Roberts Figure 5.4.5. Polyamidoamine dendrimer of generation 0.
and Friedlander, 1980a,b).
In all such cases of facilitated transport, the extra flux
of species A comes about from the complexation of
species A with an added complexing agent B in the solu- By the electroneutrality condition, the concentration of the
tion. Examples will be given now where considerable two ions HSO3 and H should be the same everywhere in
facilitation takes place because of solute ionization in the membrane. But the diffusivity of H in water is six
solvent (without any added reagent B) or solute reaction times that of HSO3 in water. Hence a potential gradient
with the solvent. develops, slowing down the mobility of H and increasing
that of HSO3 (see definition (3.1.107)); the value of the
effective diffusivity suggested by Roberts and Freidlander
5.4.4.1.1 Facilitated transport solvent as a complexing (1980a) for such a case is
agent We know from reaction (5.2.2) that SO2 ionizes
significantly in water: 0 0 2DH DHSO3
Deff D HSO D H , 5:4:64a
3
DH DHSO3
SO2 H2 O , HSO3 H :

Through a water film, SO2 diffuses by itself; it is also where DH and DHSO3 are the corresponding true diffusiv-
transferred by a gradient of the HSO3 ion in the same ities in water.
direction. Although water may be considered as the com- If CO2 is present in the feed gas containing SO2 (e.g.
plexing agent B here, there is a major difference: we can flue gas), CO2 is also transferred through the liquid mem-
ignore any gradient of B across the film. For a thick mem- brane of water. However, the hydration reaction of CO2
brane (equivalent to assuming equilibrium everywhere) given by (5.4.60a), CO2 H2O ! HCO3 H, is negligible
and using activities in the equilibrium constant Kd, we get since Kd for CO2 is a couple of orders of magnitude smaller
than that for SO2. Consequently, the flux of CO2 is purely
aHSO3 aH HSO3 H C HSO3 C H 2 C HSO3 C H diffusive, with essentially no contribution from HCO3 .
Kd :
aSO2 l SO2 l C SO2 SO2 C SO2 Sengupta et al. (1990) have shown theoretically and experi-
mentally that, at 25  C, the separation factor SO2 CO2
The total flux of SO2 is given by (where the subscript m
through a liquid membrane of pure water is around 90;
indicates quantities in the membrane)
however, if no dissociation of SO2 in water is assumed,
then the theoretical value is around 30. This dramatically
dC SO2 , m dC HSO3 , m
J tSO2 DSO2 , m DHSO3 , m : 5:4:62 illustrates the role of solute ionization in facilitated trans-
dy dy
port; conversely, it demonstrates how the solvent itself can
From local electroneutrality everywhere, we get act as a complexing agent.
Another example of the solvent acting as the complex-
SO2 K d C SO2
p
ing agent is the dendrimer polyamidoamine of generation
C H , m C HSO3 , m ) C HSO3 , m :
0, molecular weight 517, which is a nonvolatile liquid
having four terminal primary amine groups and two ter-
Integrate the expression for J tSO2 at steady state to obtain
tiary amines (see Figure 5.4.5).
DSO2 , m  0
Kovvali et al. (2000) and Kovvali and Sirkar (2001) have
J tSO2 C SO2 C SO2
m shown that, in the presence of moisture in the feed gas, this

q pure liquid membrane selectively transports CO2 extremely
DHSO3 , m q efficiently vis--vis gases like O2, N2, etc., at low CO2 feed
SO2 K d C 0SO2 SO2 K d C SO2 : 5:4:63
m partial pressures. They postulate that reactions (5.4.60b) of
5.4 Rate-governed membrane processes: chemical reactions 329

CO2 with the primary amine groups, and the reaction with A against its own concentration gradient. An example from
the tertiary amine group, Lee et al. (1978) is helpful for the special case where there
is no free A in the membrane.
CO2 R3 N H2 O , HCO3 R3 NH , 5:4:64b A basic solution of copper (pH ~ 11) containing 165
ppm of copper is passed on one side of a liquid membrane
are likely to be responsible for the selectivity. The very high M (supported in a porous filter, Figure 5.4.6(a)). On the
charge densities in the pure liquid dendrimer ionized in other side of the membrane, 1N H2SO4 solution flows. The
the presence of moisture will reduce the permeation of N2, organic liquid membrane contains a complexing agent
O2 strongly. LIX65N,9 which transfers copper to the acidic side. Figure
The literature on facilitated transport through liquid 5.4.6(a) shows that the copper concentration in the acidic
membranes is considerable. Interested readers will find stream goes as high as 700 ppm (from 0), whereas the
additional material and references in Schultz et al. (1974), copper concentration on the basic side drops to about 50
Smith et al. (1977), Marr and Kopp (1982), Way et al. ppm. Further, the pH rises on the acidic side and drops on
(1982), Sengupta and Sirkar (1986), Noble and Way the basic side. The detailed mechanism is also illustrated in
(1987), Ho and Li (2001) and Way and Noble (2001). Figure 5.4.6(b).
Figure 5.4.4(d) considers a case of facilitated transport We adopt the simplified analysis of Cussler (1971),
where the receiving phase contains an agent E which reacts wherein everywhere inside the membrane the two com-
with the species A, as illustrated in Figure 5.4.4(b). This will plexation reactions
reduce the concentration of A in the receiving phase and
therefore lead to a higher flux of species A. A B , AB, 5:4:65b

C B , BC, 5:4:65c
5.4.4.2 Separation by countertransport
are assumed to be in equilibrium. Therefore
In this mechanism, solute A from feed solution (see Figure
5.4.4(e)) complexes with complexing agent B in the mem- C ABm C BCm
brane. The complex AB diffuses down its own concentra- KA ; KC : 5:4:66
C Am C Bm C Cm C Bm
tion gradient to the receiving side where it dissociates. The
complexing agent (the carrier) B then complexes with If the solutes are ionic, there can be charge gradients in the
species C from the receiving solution; the complex BC membrane. The following simplifications are made: there
diffuses down its own concentration gradient to the feed is no charge gradient in the membrane and the diffusion
side, where it dissociates, releasing B for complexation coefficients of all species are equal to D. The governing
with species A again. Species C is transferred to the feed differential equations for steady state diffusion of the two
solution, whereas species A is transferred to the receiving solutes A and C, the two complexes AB and BC and the
solution. An example has been provided by Choy et al. complexing agent (the carrier) B are as follows:
(1974). A flux of Na (species A) is created from NaOH
solution to a NaClHCl solution by a flux of protons in the d2 C im
D Ri 00, i A, C; 5:4:67
opposite direction through the liquid membrane contain- dy2
ing the macrocyclic carrier monesin (B), the complexation d2 C iBm
reaction being D RiB 0, i A, C; 5:4:68
dy2

Na RCOOH , RCOO Na H : 5:4:65a d2 C Bm


D RAB RBC 0: 5:4:69
dy2
In the absence of the carrier, the flux of sodium is orders of
Here RA, RC, and RAB and RBC (or RCB), are the molar rates
magnitude smaller.
of production of species A, C, and complexes AB and BC, in
Consider the following: complex AB will be transported
the liquid membrane per unit volume. The molar concen-
down its own concentration gradient, even though the
tration C tBm of carrier B originally incorporated in the
receiving phase may have a concentration of A higher than
membrane can be accounted for, during separation, in
that of the feed phase. Thus species A can be pumped
terms of the concentrations of free species B, complex AB
uphill against its own concentration gradient. The condi-
and complex BC as follows:
tions are such that complex AB preferentially dissociates
and complex BC is preferentially formed at the receiving
interface; the reverse happens at the feed interface. Species 9
Trade name of extracting agent from Henkel Co. Ltd. Contains
C concentration in the receiving side is higher than that in 2-hydroxy-5-nonylbenzophenone oxime and 40 volume percent
the feed side. The transfer of C from the receiving to the inert aromatic diluent (Ritcey and Ashbrook, 1984a) (Table
feed side provides the energy for transfer of species 5.2.4).
330 Effect of chemical reactions on separation

(a) (b)
800
Cu(NH3) 4
++
HON CR2
Step 1 L1X65N
complexes copper
HON CR2
Acid with Copper releasing protons
Acid Concentrate 2
Copper Concentration (ppm)

CR
600 N
O Step 2 The complex diffuses
Copper Flux Cu across the membrane

O
N
Base with Base with

C
ca 50 ppm ca165 ppm

R2
Copper Copper

Dilute Base or Acid

Concentrated Acid
400

R2
C
H+
Step 3 Reaction with protons

N
O
Cu Cu++ releases the copper
NO H+
C
2
200 Initial Concentration R
in Basic side Step 4 The L1X65 diffuses back
HON CR2 across the membrane,
carrying protons
Final Concentration Step 5 Little back diffusion
in Basic side Cu++
because uncomplexed
copper is insoluable in
0 10 20 30
Cu++ the membrane
Space Time of Acid (min) H+
Copper is moved by a
Result
proton flux in the
opposite direction

Figure 5.4.6. (a) Copper stripping with a supported liquid membrane. Copper is selectively concentrated by diffusion across a liquid
membrane supported by a porous polymer film. (b) How the membranes work. Copper is concentrated by means of the steps shown.
(a) and (b) reprinted, with permission, from Lee et al., AIChE J., 24(5), 860 (1978). Copyright [1978] American Institute of Chemical
Engineers (AIChE).

 
m d2 C Bm d2 C ABm d2 C BCm
1 D 0: 5:4:72b
C Bm C ABm C BCm dy C tBm : 5:4:70 dy2 dy 2 dy 2
m
0
Integrate it once and use equation (5.4.71) to obtain
Further, neither species B, nor species AB or BC, can leave
dC Bm dC ABm dC BCm
the membrane at either end: at 0: 5:4:72c
dy dy dy
dC Bm dC ABm dC BCm
y 0, y m , 0 : 5:4:71 Integrate once more and put into equation (5.4.70) to yield
dy dy dy

At both ends of the membrane, the membrane concen- C tBm C Bm C ABm C BCm : 5:4:72d
tration of species A and C are related to the feed and In this relation, substitute equilibrium relations (5.4.66) to
receiving phase concentrations by the equilibrium distri- obtain
bution coefficient i0 :
C tBm
 0 C Bm : 5:4:73
y 0, C =C if if0 , i A, C;

 im 1 K A C Am K C C Cm
0 5:4:72a
y m , C imm =C ip ip , i A, C:

The total flux of solute A can now be obtained as the sum
Assume for mathematical simplicity that if0 ip0
i0 . (In of the fluxes of free species A and the complex AB:
0 0
practice, if 6 ip .) It is now possible to determine the dC Am dC ABm
concentration profile of each species in the membrane; the N tAy N Ay N ABy D D
dy dy
flux of species A and C will follow. The first step is to dC Am dK A C Am C Bm
determine the concentration of free species B. To this D D :
dy dy
end, add equation (5.4.68) for solutes A and C and add it
to equation (5.4.69) to obtain At steady state, integrate across y 0 to y m:
5.4 Rate-governed membrane processes: chemical reactions 331

C 0Am C Am from Section 5.2.2.4 that there is no free metal species in


 
D 
N tAy D K A C 0Am C 0Bm K A C Am
m m
C Bm : the organic 1iquid membrane. The metal A exists only in
m m
the complex form AB in the liquid membrane. Species C is
Rewrite this using (5.4.72a) and (5.4.73) to obtain usually a H ion. Thus the feed solution is less acidic than
D 0 A   D 0 A C tBm the receiving solution. A flux of H ions occurs from the
N tAy C Af C Ap receiving solution to the feed solution, which pumps the
m m
  metal from the feed to the receiving solution. There is no
C Af C Ap
: reaction inside the membrane. Instead the following reac-
1 K A 0 A C Af K C 0 C C Cf 1 K A 0 A C Ap K C 0 C C Cp
tions take place at the feedmembrane and the strip
The result can also be expressed as follows (Cussler, 1971): membrane interfaces, with, for example, an acidic
extracting agent HK in the membrane:
D 0 A  
N tAy C Af C Ap K
m feed

M aqHKmembrane ,
receiving
MKmembraneH aq:
D 0 A    
K A R 1 0 C K C C C C Af C Ap
m 5:4:77
D 0 A 0   Here K is the equilibrium constant for the reactions at y 0
R C K C C A C Cf C Cp : 5:4:74
m (feedmembrane interface) and y m (stripmembrane
interface).
Here The flux of the metal at the feedmembrane interface
C if C ip could be expressed in terms of the forward and backward
Ci , i A, C, interfacial reactions (following (5.3.40)):
2
and N M y k 0s C 0M i k 0s C 0MKo i : 5:4:78

K A C tBm Here k 0s
and k 0s
are pseudo-first-order rate constants
R  :
1K A 0 A C Af K C 0 C C Cf 1K A 0 A C Ap K C 0 C C Cp (otherwise see (5.3.40)). For very fast interfacial reactions,
5:4:75 local equilibrium may be assumed and

If species C does not exist, then the above expression for   C0 oi


K k 0s =k 0s MK 00 Mo , 5:4:79
N tAy reduces to expression (5.4.57) for facilitated trans- C 0M i
port of A in the presence of the complexing agent or
00 Mo being the effective distribution coefficient for the
carrier B.
metal ion. The flux of the metal M across the membrane
Consider now the situation where CAf CAp C A , i.e.
is merely the flux of the complex MK in the membrane:
there is no gradient of free solute A concentration across
the membrane. Will there be a flux of A? The above expres- DMK  0
N M y C MKo C MKo : 5:4:80
sion for N tAy is simplified to m
D 0 A 0   In general, there is also a diffusional step in each aqueous
N tAy R C K C C A C Cp C Cf : 5:4:76
m boundary layer (neglected here).
When there are two different metals M
1 and M2 in the
Since CCp > CCf, species A is indeed being transferred from
feed solution, the selectivity of the membrane is of interest.
the feed solution to the receiving solution due to a gradient
For this purpose assume that C MKo  C 0MKo . Then
in concentration of C in the opposite direction. Obviously,
the concentration of A in the receiving solution can be built N M1 y DM1 K C 0M1 Ko DM1 K M
00 C 0M i
1o 1
up over that in the feed solution. An estimate of the highest : 5:4:81
N M2 y DM2 K C M2 Ko DM2 K M2 o C 0M i
0 00
value of CAp can be obtained by assuming CCf 0 and 2

N tAy 0 for a case where the diffusional rate of the free Assume DM1 K DM2 K . Further, with the crossflow
solute species is negligible (Cussler, 1984). assumption,10
Most examples of countertransport involve a liquid
0
feed and a liquid receiving solution. Smith et al. (1977) N M1 y C M2 i C M i C 0M i
1 2 00 00
M1 M2 M1 o = M2 o : 5:4:82
have studied countertransport when the feed and receiving N M2 y C 0M i C M i C 0M i
1 2 1
phases are gaseous; a gradient of CO2 in one direction
creates transport of H2S in the other direction.
When one considers separating a metallic species from
an aqueous feed solution through an organic liquid mem- 10
Local concentration ratio at the permeatemembrane interface
brane into an aqueous receiving solution, it is apparent is given by the flux ratio (equation (7.2.2a)).
332 Effect of chemical reactions on separation

The separation of the two metals will be primarily deter- Further, such an agent in countertransport with the H ion
mined by the ratio of the two distribution coefficients, is an acid or is acidic.
00 00
M 1o
= M2 o , exactly as in solvent extraction. Separation of
copper from nickel using LIX65N in the liquid membrane
using these principles has been studied by Lee et al. (1978). 5.4.4.3 Separation by cotransport
A general review of metal extraction using liquid mem- In the case shown in Figure 5.4.4(f), solute A, along with
branes supported in microporous polymeric membranes species C in the feed solution, jointly form a complex ABC
is provided by Danesi (198485). with the carrier B in the membrane at the feedmembrane
Separation of cobalt from nickel using the extractant di interface. Complex ABC diffuses through the membrane
(2,4,4-trimethylpentyl)-phosphinic acid, H(DTMPP) or and dissociates at the receiving phasemembrane inter-
CYANEX 272 in an organic liquid membrane has been face. Carrier B, released at y m, diffuses back to y 0,
studied by Danesi et al. (1984). It has been found that whereas species A and C are deposited to the receiving
H(DTMPP), represented here as HK, exists essentially as solution. In order to transport A, simultaneous transport of
a dimer, H2K2, in aromatic solvents (e.g. toluene): C in the same direction is needed. A few examples will be
provided in the following.
Co2 aq 2H2 K2 org , CoHK2 2 org 2H aq; Babcock et a1. (1980) have shown that uranium
5:4:83a metal in the form of uranyl sulfate anions, UO2 SO4 3 , is
transported through a solvent membrane containing a
Ni2 aq 3H2 K2 org , NiHK2 2 H2 K2 org 2H aq: tertiary amine carrier R3N by cotransport in the following
manner:
5:4:83b

The equilibrium constant for each reaction can be written 4R3 Norg 4H aq UO2 SO4 4
3 aq
as
feedside
2 , R3 NH4 UO2 SO4 3 org 5:4:87
C CoHK2 2 , o C H , w

stripside
K Co 2 ; 5:4:84a
C Co2 , w C H2 K2, o

when the feed solution is highly acidic (e.g. pH 1.0) and
2 the receiving solution is less so (pH 4.5). Thus both
C NiHK2 2 H2 K2 , o C H , w

K Ni : 5:4:84b hydrogen ions and uranyl sulfate ions are transported in
C Ni2 , w C H2 K2 , o 3 the same direction through the membrane as the complex
formed with the carrier amine. Danesi et al. (1983) have
The effective distribution coefficient of each metal species demonstrated cotransport of Am3 and NO3 through an
at the feed solutionorganic membrane interface is organic membrane containing the complexing carrier
00
Co , o C CoHK2 2 , o =C Co2 , w ;
 
5:4:85a CMPO (n-octyl (phenyl)-N, N-diisobutylcarbamoyl methyl
phosphine oxide) from a feed containing LiNO3 to a strip
00
, o C NiHK2 2 H2 K2 , o =C Ni2 , w :
solution containing formic acid. Hochhauser and Cussler
 
Ni 5:4:85b
(1975) have experimentally demonstrated cotransport of
Therefore, chromium present as (say) HCr2 O7 and H ions in an
00 acidic feed aqueous solution through an organic liquid
Co ,o K Co =K Ni
: 5:4:86 membrane containing tridodecylamine, [CH3(CH2)11]3N,
00
Ni ,o C H2 K2 , o
as a carrier to a basic receiving aqueous solution. The
This suggests that, at low concentrations of free HK in the amine freed at the receiving solutionmembrane interface
organic phase (present as H2K2), C H2 K2, o , the value of shuttled back to the feed side.
00 00
Co , o = Ni, o , the separation factor in solvent extraction, A detailed analysis of chromium transport, assuming
can be very high, much higher than (KCo/KNi), which was that HCr2 O7 (A) and H (C) forms an ion pair H2CrO7 (AC)
found to be 600 from solvent extraction. Danesi et al. (1984) and then a complex (ABC) with the amine (B) in the
have indicated this to be as high as 104 when free HK in the organic membrane, is available in Hochhauser and Cussler
organic phase is around 103 M. From relation (5.4.82), the (1975). The total flux of chromium in the membrane is the
separation factor for the liquid membrane should also be sum of the flux of the ion pair H2Cr2O7 (AC) and the flux of
00 00
Co , o = Ni, o . However, the boundary layer resistances on the complex (ABC):
the two sides of the organic liquid membrane were found to
DAC 0  DABC 0 
reduce the separation factor from Co 00 00
, o = Ni, o at all except flux A C AC C AC C ABC C ABC : 5:4:88
m m
very low concentrations of C H2 K2, o .
By now, it should be clear that the complexing agent A general expression for the flux of A is available in Cussler
HK acts as a chelating agent as studied in Section 5.2.2.4. (1984).
5.4 Rate-governed membrane processes: chemical reactions 333

Conventionally in cotransport analysis for metal N Ay C 0Bi C Ai C 0Bi 00


DAm Ao jf
species (A), however, the dominant species is ABC, and 0 0 AB :
N By C Ai C Bi C Ai DBm Bo jf
00
only the flux of ABC is analyzed (Danesi, 198485) and
used for determining the metal transport rate. Note further Therefore
that, for metal extraction in cotransport, the carrier is
0

DAm Ao jf K dB
neutral or basic (e.g. long-chain alkylamine). AB : 5:4:95
DBm 0Bo jf C H f
This result is based on B being highly dissociated in the
5.4.4.4 Separation based on dissociation
feed solution and species A not so highly dissociated. If,
Consider a feed aqueous solution containing two weak however, both species are dissociated significantly, but to
organic acids A and B on one side of an organic liquid different degrees, then
membrane without any carrier species. Then, at the feed
0

DAm Ao jf K dB
membrane interface, the following distribution equilibria AB : 5:4:96a
DBm Bo
0 j K dA
hold (equation (5.2.62)): f

0
Ao jf 0
Bo jf A more general expression would be
00 00
Ao jf ; Bo jf :
1 K dA =C H 1 K dB =C H
0
DAm Ao jf 1 K dB =C H f
5:4:89 AB : 5:4:96b
DBm Bo0 j 1 K =C
f dA H f
0
Suppose KdA and KdB are sufficiently apart even if Ao jf is If we discount the role of (DAm/DBm) in these three expres-
0
close to Bo jf . If the pH is moderately high, such that the sions for AB , the separation factors are what would
more acidic species B is highly dissociated, but the species have been achieved in dissociation extraction from an
A is not so highly dissociated, then aqueous solution to an organic solvent. If, however, species
00
Bo 0
jf Bo jf C H =K dB f and 00
Ao 0
jf Ao jf , A were to be reextracted back into an aqueous solution for
recovery, then the liquid membrane step achieves the
5:4:90
same goal using very little solvent and only one device
with K dB C H and K dA  C H . Therefore 00Ao jf >> (instead of an extractor and a back extractor). Such a
0
Bo jf and species A is extracted much more at the feed technique is likely to be highly useful in the pharmaceut-
interface. Now, if the strip solution is highly alkaline, it is ical industry.
obvious that species A is going to be pushed to the strip In this example, the liquid membrane did not have any
aqueous solution since complexing carrier species. Such a carrier species, incorp-
00 0
orated in the liquid membrane, will enhance the flux of
Ao js Ao js C H =K dA s 5:4:91
species A. There is, however, the distinct possibility also for
and K dA C H . For lack of specifics, the distribution equi- complexation of the carrier with the limited amount of
librium for species B at the organic liquid membrane and B that partitions into the membrane.
the strip aqueous solution interface is
0 5.4.4.5 Diffusional film resistances outside the
00 Bo js
Bo js : 5:4:92
1 K dB =C H s liquid membrane
00
It is very likely, however, that Bo 0
js Bo js C H =K dB s . The diffusional film resistances on the two sides of the
The separation factor AB for species A and B between liquid membrane, the feed side and the receiving (or strip)
the feed aqueous and the strip aqueous solution can be side, have been ignored in the preceding treatment. They
determined in the usual fashion. The flux of each species can be quite important when the membrane transport
through the membrane is given by rates and/or the interfacial reaction rates are high. Two
examples where external mass-transfer resistances are
DAm  0
N Ay C Am C Am ; important are briefly considered below.

5:4:93a
m

DBm  0 5.4.4.5.1 External film resistances in facilitated trans-


N By C Bm C Bm :

5:4:93b
m port Facilitated transport separation for the reaction
Since C H js << KdA and KdB, C Am << C 0Am , C Bm << C 0Bm , kf
A B , AB

kb
00
N Ay DAm C 0Am DAm Ao jf C 0Ai
: 5:4:94
N By DBm C 0Bm 00 j C 0
DBm Bo f Bi
has been discussed in Section 5.4.4.1. If there is a mass-
transfer resistance (l/kgf) or (l/klf) in the feed phase (gas or
Using the crossflow assumption, liquid) to the transport of species A to the feedmembrane
334 Effect of chemical reactions on separation

y=0 y = dm

Feed Receiving
solution solution
0
C Am
0
C Awb

d
0
C Awi C Am

d
C Awi
d
Feed side C Awb
B layer Liquid
membrane Receiving side
B layer

Figure 5.4.7. Concentration profile for species A being transported through liquid membrane with boundary layer resistances.

interface (y 0), then the following boundary condition is These can be rewritten as before using H CA and Ao 00
to relate

needed (Figure 5.4.7): pAi or C Awi to C Am , etc.

Noble et al. (1986) have obtained a solution for the


dC Am  facilitated transport of species A through the liquid mem-
k gf p0Ab p0Ai
 
DAm  5:4:97a
dy y 0 brane for such a case when CBm was constant throughout
the film. This is equivalent to assuming a very large excess
(gaseous feed) or
of the complexing species B in the membrane. Further,
dC Am  there is reaction equilibrium everywhere in the membrane
k f C 0Awb C 0Awi :
 
DAm  5:4:97b
dy y 0 and C Awb or pAb (as the case may be) is zero. The extent of
facilitation eq (see (5.4.58)) was obtained as
(aqueous feed). Use Henrys law constant H CA for species
 
A in a gaseous feed or the distribution coefficient 00 Ao for 1 K
1 1K 1 Sh1 f Sh1 s

an aqueous feed. These relations can be rewritten for eq   , 5:4:100
1 K tanh2 1 K 1 1
gaseous feed as 1 1K 2 1 1K Shf Shs

dC Am 

0
p where
DAm  k gf H CA Ab C 0
Am 5:4:98a
dy y 0 H CA 0 11=2
DAB C tB H CA K p0 1 @1 1 1K A
and for aqueous feed with an organic membrane as l ; K CAb ;

2 ;
DA p0Ab HA 2 1 K =A
dC Am    0 
DAm  00
k f =Ao C Aob C 0Am : 5:4:98b Shf k gf H CA m =DA ; Shs k gs H CA m =DA ; A k b 2m =DA :
dy y 0

Note that C 0Am is the membrane-phase concentration


5.4.4.5.2 External film resistances in countertransport
of A at the feed interface and that C 0
Aob is the hypo- separation with interfacial reaction Consider the
thetical membrane-phase concentration which would be
following type of interfacial reaction for a metal being
in equilibrium with the aqueous feed bulk concentration
transported from an aqueous feed solution through an
C 0Awb .
organic liquid membrane to an aqueous strip solution by
A similar boundary condition on the receiving (or
countertransport of H ions (as in (5.4.77)):
strip) side of the membrane will be
feed
M aq HKorg , MKorg H aq,
dC Am    strip
DAm  k gs pAi pAb 5:4:99a
dy
y m where HK is the complexing carrier in the liquid mem-
brane. At steady state, the metal flux through the aqueous
(gaseous feed) or
boundary layer on the feed side, the metal flux at the feed
membrane interface, the metal flux through the mem-
dC Am   
DAm  k s C Awi C Awb : 5:4:99b brane, the metal flux at the membranestrip interface and
dy
y m the metal flux through the strip side boundary layer are all
5.4 Rate-governed membrane processes: chemical reactions 335

Voltage source
+

2e 2e

Air O2

Profile for OH ions in the
membrane
O2

N2
N2 O2

+
Porous anode
Porous cathode
Membrane containing the
electrolyte (porous or
ion exchange membrane)

Figure 5.4.8. Electrochemical membrane gas separator for separating oxygen from air.

equal. Represent this flux as N M z . Then, for the feed 5.4.4.6 Electrochemical membrane gas separation
boundary layer,
Consider an electrochemical cell having two electrodes, a
 cathode and an anode, separated by an electrolyte. Let one
N M z k f C M b C 0M i : 5:4:101 side of each electrode face the electrolyte while the other
side is open to the atmosphere or any gas mixture (Figure
Assume psuedo-first-order rate constants k0s 1 and k 0s 1 to
5.4.8). Let the negatively charged cathode be made out of a
define (see (5.4.78))
nickel screen or a highly porous gas diffusion electrode
N M z k 0s 1 C 0M i k 0s 1 C 0MKo i : containing a catalyst like nickel dispersed well. If a small
voltage of 0.5 volts is applied across the electrodes (the
Through the membrane (see (5.4.80)), other electrode is similarly prepared) when air is present


DMK  0 around the cathode, one finds that virtually pure oxygen is
N M z C MKo i C MKo : generated/liberated at the positively charged electrode, the
m
anode, in the presence of aqueous KOH solution between the
Since strip solutions effectively reduce C MKo to essentially a electrodes. At the cathode of this electrochemical gas separ-
value of zero, we ignore the strip side interfacial reaction as ator, oxygen from air is reduced at room temperature via
well as the boundary layer equations and write C MKo 0.
Combining the three equations, we obtain 1
O2 H2 O 2e ! 2OH : 5:4:103a
C M b 2
N M z  
1 k0 1 =k0s 1 At the anode, the OH ions arrive in the presence of the
k 0s 1
k1f Ds MK =m
applied voltage difference between the electrodes and are
 
k 0s 1 =k 0s 1 C M b converted to oxygen:
  5:4:102
k0s 1 =k0s 1
m
DMK k f k01
s 1 1
2OH ! O2 H2 O 2e: 5:4:103b
2
for the general case. For any particular surface reaction, the
exact rate equation needs to be considered instead of the This electrochemical cell has thus produced pure oxygen
pseudo-first-order reaction considered above. gas at the anode from oxygen in the air at the cathode
336 Effect of chemical reactions on separation


Current
Air feed plate collector Current Current
plate collector collector
End plate Packing Packing screen Packing plate

Current
collector Ion exchange
membrane Packing Water End
screen
feed plate
with electrode
plate

Figure 5.4.9. Exploded view of the electrochemical oxygen separator (100 cm2). Reprinted, with kind permission from Springer Science
Business Media, from the Journal of Applied Electrochemistry: Y. Fujita, H. Nakamura and T. Muto, An electrochemical oxygen
separator using an ion exchange membrane as the electrolyte, J. Appl. Electrochem. 16 935, (1986), Figure 2.

via electrochemical reactions at the two electrodes CO2 2OH ! CO2


3 H2 O: 5:4:104
(Langer and Haldeman, 1964). Other gases at the cathode
do not undergo any such reaction at the applied voltage This will reduce the efficiency of the alkaline membrane
difference. with time. One can avoid this problem via an alternative
The aqueous electrolytic solution can easily leak out or strategy which uses a proton exchange membrane, e.g.
flow into the gas streams at either electrode, creating a Nafion, and electrodes containing a noble metal platinum
number of problems. To avoid this, one can employ a as the catalyst. The electrode reactions then are as follows:
porous membrane or a filter paper soaked in KOH as the
1
electrolytic medium. We have now a supported liquid cathode O2 2H 2e ! H2 O;
2
membrane (SLM) through which the OH ion is selectively 5:4:105
1
transported; we expect the flux of any dissolved O2 through anode H2 O ! O2 2H 2e:
this SLM to be extremely small, since O2 solubility in this 2
concentrated electrolytic solution is likely to be very low.
Fujita et al. (1986) have demonstrated this by circulating
Further, the thickness of the SLM is considerable. How-
water on the anode side and recovering 98.4% O2 from the
ever, there is also a possibility of a change in the concen-
water when 1.4 volts was applied between the two elec-
tration of the electrolyte due to the evaporation or
trodes (Figure 5.4.9).
absorption of water in response to humidity fluctuations
The general principle illustrated by equations
in the ambient conditions at the electrodes. A polymeric
(5.4.103a) and (5.4.103b) has been utilized to separate a
ion exchange membrane would undergo much more
number of gas species from their mixtures with other
limited change due to environmental humidity fluctuations
species. For example, Robinson et al. (1998) have demon-
(Maget, 1970). This is the genesis of the electrochemical
strated that H2S can be selectively removed from a sour
membrane gas separation employing an ion exchange
process gas (coal gas containing H2S) at the cathode, where
membrane between the electrodes.
H2S is reduced via
There still remains a problem created by any atmos-
pheric CO2, which will react with the OH to form
carbonate: H2 S 2e ! H2 S2 , 5:4:106a
5.4 Rate-governed membrane processes: chemical reactions 337

whereas, at the anode, the reduced sulfide ion is oxidized constant. For considerations on the voltage needed to run
to condense as elemental sulfur, such a separation cell, consult Robinson et al. (1998).

S2 ! S2 2e , 5:4:106b
5.4.5 Separation through solid nonporous membrane
which is swept away by a hot inert sweep gas (e.g. N2). This
process operates at a high temperature (~650 C) with a Gaseous mixtures or liquid solutions containing small
molten carbonate (Li2CO3 (62 mol%)K2CO3 (38 mol%)) solute molecules may be separated by transport through
electrolyte located in the pores of a porous ceramic sup- a solid nonporous membrane containing reactive groups
port membrane. The molten carbonate reacts with H2S to or sites. This is different from dialysis in the sense that
provide sulfide ions in a molten state: dialysis is generally defined to separate crystalloids from
colloids by diffusion through a membrane which is gener-
Li0:62 K0:38 2 CO3 H2 S ! Li0:62 K0:38 2 S CO2 H2 O: ally microporous. Although Section 5.4.3 considered an
example of the transport of small molecules through a
5:4:106c
nonporous polymeric membrane, dialysis, certainly hemo-
Complications are encountered since CO2 and H2O vapor dialysis, employs generally microporous polymeric mem-
also participate a little in the cathodic reduction process: branes; no reactions are involved in the membrane.
Here we touch very briefly on examples of gas separation
through nonporous membranes containing reactive
CO2 H2 O 2e ! H2 CO2
3 : 5:4:106d
groups/agents.
An additional example has been provided by Wauters and
Winnick (1998), wherein Br2 gas is obtained from a waste 5.4.5.1 Fixed carrier membranes
gas stream containing HBr vapor through a molten salt
saturated membrane. At the cathode, HBr vapor is Solid nonporous polymeric membranes/films have been
reduced: developed that contain reactive groups or complexing
agents. These agents/groups are not mobile, unlike the
2HBr 2e ! H2 2Br : 5:4:107a carrier species in liquid membranes studied in Section
5.4.4. Instead, the permeating solute molecule complexes
At the anode, the Br ions are oxidized to Br2 vapor: with such a group/agent molecule and then dissociates/
jumps to bind with the next group/agent molecule down
2Br ! Br2 2e : 5:4:107b the permeation pathway. The permeating solute molecule
is assumed to follow two simultaneous pathways. One
The electrochemical cell was operated at 300 C.
consists of regular solution diffusion through the polymeric
In all such processes, the difference in the
membrane. The second consists of a parallel pathway
electrochemical potential (el ij , definition (3.3.27)) driving involving complexation/binding and decomplexation/
the charged species through the membrane is given by
jumping assisted by the reactive groups/complexing agents
 
  dispersed through the polymer. A range of behavior is
e
ij e 
ij  cathode e 
ij  anode possible, depending on the nature of the complexation,

  the density/concentration of the reactive groups/agents
 
ij  ij  Z i F : 5:4:108a and the behavior of the rest of the polymeric matrix.
cathode anode
A brief review of the examples and models is available in
Since the membrane essentially has no free species, the Way and Noble (2001).
process is controlled by the potential difference, ,
driving the negatively charged species like OH, S2, Br,
5.4.5.2 Oxygen separation through thick mixed-conducting
etc. The flux of the ionic species may be obtained from the
solid oxide membrane
NernstPlanck relation (3.1.106) for a membrane of thick-
ness y in an integrated form as In the examples considered in Section 5.4.4.6, the electrode
C i Di Z i F C i (cathode) converted one of the gas species, e.g. O2, from a
J iy Di : 5:4:108b gas mixture to, say, OH in an electrochemical cell in the
RT y y
presence of an applied voltage. The hydroxyl ion so pro-
The current density is obtained from (3.1.108c) as duced was transported through the SLM/ion exchange
membrane to the other electrode to regenerate O2,
C i Di Z 2i F 2 C i
i F Z i Di , 5:4:108c which was evolved at the electrode as a pure gas; the
RT z z
electron generated in the reaction was supplied to the
assuming for the time being that no other species contrib- external electrical circuit. No electron transport was
ute to the current and J i N i . Here F is Faradays allowed through the SLM/ion exchange membrane.
338 Effect of chemical reactions on separation

Mixed-conducting dense (nonporous) solid oxide mem- across the membrane between the two membrane sur-
branes prepared out of appropriate perovskite ceramic faces. Therefore there is no current in the system; however,
materials are such that both ion conduction and a net transport of O2 takes place, as was first observed by
electron transport can occur through the membrane at Teraoka et al. (1985).
high temperatures; the material is otherwise impermeable Quantitative descriptions of these transport processes
to gases. No voltage is applied across the membrane. are available in Heyne (1977) and Gellings and Bouwmee-
The stoichiometry of a perovskite oxide ideally is ABO3, ster (1992). The net result of such analysis is an expression
where A and B are trivalent cations located at two different for oxygen flux through a membrane of thickness m sub-
crystallographic sites. Consider the B cation, which has a jected to a feed side oxygen partial pressure of pO2 f and a
fixed valence. If there is a partial substitution at the A site permeate side partial pressure of pO2 p (<pO2 f ):
(typically a rare earth) with a lower valence cation, then an
oxygen vacancy V ::O is formed to compensate for
  !
RT m pO2 f
the reduced charge of the cations. The composition of J O2 n : 5:4:111
16F 2 m pO2 p
an oxide with A-site substitution, for example, is
La0.6A0.4Co0.8Fe0.2O3- for cases where A Sr, Ba, Ca; at Here m is an average conductivity of the ionic and
elevated temperatures and reduced oxygen partial pres- electronic species in the mixed-conducting ceramic mem-
sures, it becomes highly defective with high oxygen defi- brane over the oxygen partial pressure range across the
ciency ( is the nonstoichiometry of oxygen in the mole membrane. This approach, based on a relatively thick
formula). In such an oxide film, a partial pressure gradient membrane (> 500 m), has been modified by Lin et al.
of oxygen from the feed (pO2 f ) to the permeate side (pO2 p ) (1994) for thin mixed-conducting oxide membranes by
will create an inverse gradient of oxygen vacancies in considering, in addition, surface reactions of the adsorbed
the film (high concentration of oxygen vacancies on the oxygen species on the two membrane surfaces. Due to the
permeate side, Cv,p compared to those on the feed side, Cv,f absence of the necessary fundamentals of simultaneous
(< Cv,p)): electronic transport and ionic transport in this book, no
fundamental analysis leading up to (5.4.111) is provided
1 here.
feed side O2 V ::O !OxO 2h 5:4:109
2 In common parlance, technologies employing such
where OxO could be considered a lattice oxygen and h concepts are often identified as ion transport membrane
represents an electron hole. Consequently, V ::O , the oxygen (ITM) technology for oxygen generation/separation/pro-
vacancy sites, diffuse from the side of pO2 p to the side of duction. Dyer et al. (2000) provide details of the various
pO2 f ; correspondingly, electron holes move in the opposite related technologies. For example, air containing CO2, H2,
direction, from pO2 f to pO2 p : etc., at 800900  C and 100300 psia is passed over the ITM
membrane. A low-pressure oxygen permeate stream (i.e.
1 pure O2) is obtained at a fraction of an atmosphere. The
permeate side OxO 2h !V ::O O2 : 5:4:110 ITM is thin and is supported on layers with larger pore
2
dimensions, so that the mechanical load on the membrane
Thus there are two mobile charged species in the mem- is reasonable. The support layer is also made of the same
brane: the oxygen vacancy and the electron hole. The material as the ITM membrane to minimize differential
membrane is chargewise neutral everywhere; there is no thermal expansions and chemical interaction between the
macroscopic electrical field or potential gradient r membrane and the support.

Problems
5.2.1 When chlorine is absorbed from an inert gas into water, the following reversible reaction takes place in water:
Cl2 aq H2 Oaq !HOClaq H aq Cl aq.
Develop a relation between the chlorine partial pressure in the gas phase and the total molar concentration of
chlorine in the liquid phase (the equilibrium constant for the reaction is K). Assume ideal behavior.

 1=3 
p 1=3
Ans: C tCl2 HClC 2 K =H CCl2 pCl2 :
Cl2

5.2.2 (a) Earlier work in Germany in 1953, quoted in Lightfoot et al. (1962), suggested the dissolution of NH3 from
an inert gas into water to be taking place in the following fashion:

NH3 g

K1 K2
NH3 aq H2 O ! NH4 OHaq ! NH
4 aq OH aq:
Problems 339

If Henrys law constant for NH3 absorption is described via pNH3 H CNH3 C NH3 , and water concentration is
not ignored in the first reaction step (involving K1), show that the total ammonia concentration in the
liquid phase is given by
!1=2
pNH pNH3
C tNH3 C 3 1 K 1 C H2 O K 1 K 2 C H2 O 1=2 :
H NH3 H CNH3

(b) Modern spectroscopic methods doubt the existence of NH4OH:

NH3 g

Kb
NH3 aq H2 O ! NH
4 aq OH aq,

where water concentration is ignored and Kb 1.86  105 gmol/liter at 25  C. Show that
!1=2
p 3 pNH3 1=2
C tNH3 NH Kb :
H CNH3 H CNH3

Given H CNH3 103 =2:6mmHg-liter=g mol, develop a relation between C tNH3 and pNH3 .

5.2.3 Consider pure water as an absorbent for SO2 over CO2 from an inert gas mixture. Obtain an expression for the
separation factor of SO2 over CO2 defined by

C tSO2 pCO2
SO2 CO2
pSO2 C tCO2

at 25  C in terms of Henrys law constants and any other relevant quantities. Use estimates of thermodynamic
quantities like Kdi to make necessary assumptions and obtain simplified results. You are given K d, SO2 1.7 
102 gmol/liter at room temperature and K d, CO2 4.16  107 gion/liter at 20  C.
0 8 91
>
<H C 1=2 C >
K H CO2
=
CO2
 d, SO2 1=2
B
@Ans: SO2 CO2
C
C
:A
: H SO2
>
pSO2 H CSO2
>
;

5.2.4 Develop a relation between the equilibrium partial pressure of H2S in the gas phase, pH2 S , and the fractional
consumption f of a base B which absorbs H2S in an aqueous solution by the reaction (5.2.21). You are given
that C tB is the total molar base concentration to start with; K CS is the equilibrium constant for the given
reaction in terms of molar concentrations; H CH2 S is Henrys law constant for H2S in the solution.


HC f2
Ans: pH2 S KHC2 S C tB 1f :
S

5.2.5 (a) Develop the equilibrium relation between the mole fraction of CO2 in the gas phase of an absorber, at a
pressure of 2.2 MPa and a temperature of 110  C, and the fractional consumption, f, of a 3-molar aqueous
solution of K2CO3. The following information has been summarized by Rousseau and Stanton (1988) for
this system:
(1) reaction equilibrium constant: K 165 exp (1312/T), where T is in kelvin;
(2) Henrys law constant H for CO2 at 110  C in m gmol/liter solution of potassium carbonate is given by
log10(H/H0) 0.125m, where H0 is Henrys law constant in water;
(3) the density, , of aqueous K2CO3 solution at T K is

1:1464 0:0006T 1:11uK2 CO3 ,

where uK2 CO3 is the weight fraction of K2CO3 is solution;


(4) the value of H for a 20 wt% solution of K2CO3 at 110  C is 51 MPa-liter/gmol.
(b) What would be the corresponding relation for a stripper being operated at a pressure of 0.2 MPa?
 
(Answer: (a) x CO2 , g 0:0806 f 2 =1 f ;
 2 
(b) x CO2 , g 0:887 f =1 f .)
340 Effect of chemical reactions on separation

5.2.6 The absorption of nitrogen oxides (NO, NO2, N2O4, N2O5, N2O, etc.) into water is employed to control
air pollution by nitrogen oxides as well as to produce nitric acid. Of these oxides, NO, NO2 and N2O4
are much more important than the others. In the gas phase, the equilibrium constant Kp for the gas phase
reaction
Kp
2NO2 g ! N2 O4 g 5:P:1

has been described by


2993
log K p  9:23: 5:P:2
T K

The complex absorption/reaction can be represented by two reactions (Sherwood et al., 1975):

2NO2 N2 O4 g H2 Ol !HNO3 l HNO2 l; 5:P:3


3HNO2 l !HNO3 l H2 Ol 2NOg: 5:P:4

These two reactions are equivalent to the following overall reaction, where Kc incorporates the HNO3 concen-
tration dependence:
Kc
1:5N2 O4 g H2 Ol ! 2HNO3 l NOg: 5:P:5

Sherwood et al. (1975) have identified HNO3-concentration-dependent Kc from the literature to be

log K c 7:41 20:3uHNO3 32:5u2HNO3 30:9u3HNO3 , 5:P:6

where uHNO3 is the mass fraction of HNO3 in the liquid phase. For atmospheric pressure absorption at 25  C
from a gas containing NO (pNO 0.12 atm) and combined NO2 and N2O4 (pNO2 N2 O4 0:27 atm), determine
the equilibrium HNO3 mass fraction in the aqueous phase. (Ans. 0.61.)

5.2.7 In liquid-phase polycondensation reactions to produce a polymer P1, say, Nylon 6,6, the following system is
separated by distillation (Grosser et al., 1987): nonvolatile adipic acid (A), volatile hexamethylene diamine (B),
nonvolatile salt (C) and volatile water (W) (Jacobs and Zimmerman, 1977). The reactions are

A B ! C 1; A B ! P1 W 2:

Assume:
(a) the vaporliquid system to consist only of species A, B, C and W;
(b) only reaction (1) to take place, equilibrium constant is Kx;
(c) the value of WB is constant and WB > 1.
Express the vapor-phase mole fraction of B, yB, in terms of the liquid-phase mole fractions of A, B (i.e. xA, xB),
WB and Kx. Ans: yB fx B =x B WB 1 x A x B K x x A x B g:

5.2.8 MacKenzie and King (1985) have studied the extraction of ammonia from sour wastewater into solvents
containing acidic cation exchangers. At the pH of the wastewaters, ammonia is present primarily as an NH 4
ion. An organic acid, e.g. D2EHPA (bis(2-ethylhexyl) phosphate) in a solvent (e.g. toluene) complexes with a
cation M as follows:

M aq HAorg M A org H aq:

After the extraction of ammonia, the solvent has to be regenerated by stripping. Stripping is possible when the
ion pair MA in the organic phase dissociates.
(a) Write down the reactions for this dissociation and stripping.
(b) Obtain an expression for the partial pressure of NH3 in the gas phase in terms of the equilibrium constant
K for the reaction in part (a) and various other concentrations.
(c) Suggest conditions that will increase the NH3 partial pressure in the gas phase.

5.2.9 Biodegradation of organic pollutants in aqueous waste streams is often hindered if the aqueous solution has
extreme pH and/or high salt concentration. One can, however, extract the pollutant into an organic solvent and
then back extract the pollutant into an aqueous stream amenable to biodegradation. p-Nitrophenol (PNP) is
Problems 341

one such pollutant. PNP, a weak acid, reacts with aqueous alkalis and forms the phenolate ion, PNP
(Tompkins et al., 1992):
PNPaq OH aq ! PNP aq H2 O:

Usually the ion PNP has very low solubility in the organic phases employed for extraction. Show that the
00 0
effective partition coefficient for PNP in all forms, io , is related to the partition coefficients PNP , o for PNP and
0
PNP , o for PNP by

00 0PNP, o 0PNP , o K d1 =C H
io ,
1 K d1 =C H

where Kd1 is the ionization constant of the acid, PNP.


0
Calculate the value of 00 io as a function of C H . You are given PNP 0
, o 89; PNP , o 0:14; pK1 of PNP 7.1.
00
Ans: io 89 0:14107:1 =C H =1 107:1 =C H :
5.2.10 The species 8-quinolinol (HQ) is amphiprotic. At high pH, it preferentially ionizes in the aqueous phase:
K dl
HQ ! HQ H :

At low pH it behaves as a base in the aqueous medium:


K d3
H2 Q ! HQ H :

The dissociation constants for the two reactions are Kd1 and Kd3, respectively. Species HQ, Q and H2Q are,
respectively, species 1, 2 and 3. Determine the overall distribution coefficient of 8-quinolinol between the
aqueous and an organic phase which has only the species HQ. Show that it may be simplified as follows:

00 0 C H K d3
1o 1o ,
K d1 K d3 K d3 C H C 2H
0
where 1o C 1o =C 1w .
5.2.11 Robinson and Cha (1985) have provided the following example for controlled pH extraction. Flurbiprofen is a
nonsteroidal anti-inflammatory agent. In a synthesis of this compound, a carboxylic acid, a small amount of a
dimeric acid (DA) is formed (Figure 5.P.1). It was found that simple crystallization does not completely
eliminate DA from the product. Separation of these two species is possible by high vacuum distillation. It
was found to be impractical because of a combination of high boiling and melting points which resulted in
sublimation. Although both are carboxylic acids, the measured pK values are quite different: 6.3 (flurbiprofen)
and 11.4 (DA). It should be noted that the pK for DA is a psuedo-pK value as it was measured in a 2-phase

Flurbiprofen
''Dimeric Acid'' (DA)
mol.wt. = 244.25
mol.wt. = 442.6
b.p. = 2.00 C
b.p. = ?
at 2 mmHg
m.p. = ?
m.p. = 115 C
pK = 11.4 (pseudo)
pK = 6.3
(methylene chloride/H2O)

Figure 5.P.1. Properties and structure of Flurbiprofen and Dimeric Acid (DA). Reprinted, with permission, from Robinson and Cha,
Biotechnol. Progr., 1(1), 18 (1985). Copyright [1985] American Institute of Chemical Engineers (AIChE).
342 Effect of chemical reactions on separation

(methylene chloride/H2O) system because of low water solubility. Therefore the extraction of Flurbiprofen from
the methylene chloride layer into water should be an efficient way of separating the compounds. Determine the
00 0
separation factor at pH 8.3. Ans: DA Flurbiprofen DA Flurbiprofen  101:
 

5.2.12 Develop the relation (5.2.89) for the acetic acid distribution coefficient between a nonpolar organic solvent and
00
water. Identify the condition so that 1o is independent of the pH, given Kd1 1.5  105. Identify the aqueous
00
acetic acid concentration beyond which 1o changes by more than 10% from the value valid for negligible
dimerization.
5.2.13 Kuo and Gregor (1983) have measured the distribution coefficient of acetic acid between water and pure

Table 5.P.1

00
Initial acid concentration (M) 1o

0.4 0.0236
0.133 0.0207
0.0866 0.0264
0.0674 0.0271

decalin at 27  C at different initial acid concentrations (see Table 5.P.1).


If the volumes of the two phases were the same, speculate first about the equilibrium acetic acid concentration
in the aqueous phase for each of the four cases. Suggest a way of finding out whether there was any
dimerization in the organic phase. Develop an estimate of the dimerization equilibrium constant K2.
(Ans. K2 0.)

5.2.14 The distribution coefficient of penicillin G between a solvent (isoamyl acetate or methyl isobutyl ketone
(MIBK)) and an aqueous solution is shown in Figure 5.2.4(b) as a function of aqueous solution pH. In the
initial aqueous clarified fermentation broth, there are other impurities from which penicillin has to be
separated. Suggest a pH of the aqueous broth at which solvent extraction should take place. Suggest also the
pH of an aqueous solution into which the penicillin is to be back extracted from the solvent extract to purify it
further. Assume that the impurity distribution coefficient is independent of pH.
5.2.15 Conventionally, penicillin G, a weak monocarboxylic acid, is extracted into polar solvents like n-butyl acetate
rapidly at a low temperature to reduce its loss due to instability. Centrifugal extractors are frequently used.
Reschke and Schgerl (1984) employed reactive extraction of penicillin G (HP) by a secondary amine,
R1
Amberlite LA-2, N H, and suggested the following extraction mechanism:
R2

K
H w P w Ao ! AHPo,

where, in the aqueous phase, the penicillin G dissociation takes place via

HPw H w P w , pK 2:75:

Tamada et al. (1990) have suggested a general reaction mechanism of carboxylic acids with amine extractants:
K
mAo nHPw ! Am HPn o:

Spectroscopic studies indicate that, for a secondary amine Amberlite LA-2 present in a nonpolar solvent like
kerosene (no extraction of HP as such) and HP in water, the reactive extraction mechanism is
K
2Ao 2HPw ! A2 HP2 o
.
(a) Obtain a quartic algebraic equation for the equilibrium concentration of HP in the aqueous phase as a
function of pK, pH, K and the total concentrations of penicillin G in water (C tHP ) and amine LA-2 in organic
phase (C tA ) added initially.
(b) How will you obtain the extent of penicillin G extracted?
Problems 343

5.2.16 It is proposed to separate a mixture of monoacidic bases A and B present in an organic solvent at a total
concentration of C to C Ao C Bo via dissociation extraction into an immiscible aqueous phase containing a
strong extracting agent present at a concentration of C tw , which is stoichiometrically deficient to protonate C to .
The stronger base, species B, will be protonated more than species A; it will therefore be extracted more into the
aqueous phase, provided that 0Ao is quite close to 0Bo . Assume, however, that Ao 0 0
> Bo . Given KdA and KdB for
00 00 00 0
the two bases, and unity for the activity coefficients, develop an expression for AB Ao = Bo in terms of Ao ,
0 n   o 1
K 0
t
 0 C w 0 Bo C Bo K dA Bo C
0 Ao Ao
, C t , KdA, KdB, CAo and CBo. @Ans: AB
dB
0
Bo w
00
Ao0 n Bo
   o :A
K dA K dA 0 Bo
C tw 0 Bo K dB C Bo K dB 0 Ao
C Ao

5.2.17 Many amino acids are produced by microbial fermentation processes. Recovery and purification of the amino
acid (Am) from the fermentation broth is quite costly. Organic liquid membranes or solvents containing
quaternary ammonium chloride (R4NCl) have been employed to extract the amino acid in the negatively
charged state (Am) in exchange for the chloride ion:
Ke
R4 N Cl org Am aq ! R4 N Am org Cl aq:

Knowing the dissociation equilibrium constants K1 and K2 for the amino acid in water, i.e.
K1 K2
Am ! Am H ; Am ! Am H ,
 0 
and the partition coefficient, 0 Cl , o , of the chloride ion Cl
, o C R4 N Cl , o =C Cl , w between the aqueous
and the organic phases, determine the effective partition coefficient, 00 Am, o , of the amino acid
between the broth and the liquid membrane/organic solvent containing the quaternary compound. You
are given that C tAm, w is the total concentration of the amino acid in the broth in equilibrium with the
solvent.
5.2.18 Amino acids are frequently separated by ion exchange based chromatography. Consider a particular amino acid
present in a salt solution at a total concentration of C tAmw .
(a) Identify the equations whose simultaneous solution is needed to determine the concentration of the
amino acid in a particular form, for example C Am w , C Am w as a function of the pH, C tAmw and salt
concentration.
(b) Develop an analytical solution for C Am w , for example at a very low pH (well below pK1). Note:
C Cl C Na C o , a quantity used in the expression for the analytical solution.
  q
 2
(Ans. (b) C Am w 0:5f K 1 C o C tAmw K 1 C o C tAmw 4C o C tAmw g.)

5.2.19 In chromatographic separation of two weak acids, the detector records both the dissociated and undissociated
species. For an acid species HA, develop an expression for the distribution ratio at any pH in terms of the two
0
limiting distribution ratios, one corresponding to fully dissociated acid, k A1 jA , and the other corresponding to
0
no dissociation at all, k A1 jHA . Use linear addition for the detector response, with f being the fraction present in
the dissociated form in the eluent. Assume only that the undissociated species partitions between the mobile
and the stationary phase.

5.3.1 Solvent extraction of p-nitrophenol (PNP) from a high-salt aqueous solution into 1-octanol acting as solvent
was considered in Problem 5.2.9. After the extraction of the pollutant PNP into 1-octanol, the solvent is
regenerated by contacting the solvent containing PNP with another aqueous solution; the solution may be
basic or is amenable to biodegradation of PNP (Tompkins et al., 1992). Microporous hydrophobic polypropyl-
ene membranes (Figure 3.4.10) were used by these authors to back extract PNP from 1-octanol into the
aqueous back extraction phase.
(a) Plot the concentration profile of PNP from the organic phase through the organic-filled membrane pores
to the aqueous back extraction phase.
(b) Identify the individual mass transport coefficients or resistances for each region for PNP transport and
relate them to an overall PNP mass-transfer coefficient. Specify the role of solute ionization in developing
these expressions. Assume that the aqueous-phase mass-transfer coefficients for PNP and PNP are
essentially similar.
(c) Obtain the limiting expressions for the overall mass transport coefficient for PNP for two conditions in the
aqueous back extraction phase: low pH and high pH.
344 Effect of chemical reactions on separation

5.4.1 In pervaporation separation of volatile organics from an aqueous solution through a polymeric membrane
(Section 3.4.2.1.1), Bddeker et al. (1988) have shown that, for small values of organic concentration in feed
aqueous solution, the mass flux of the volatile organic species i (whose feed water concentration is Cif) is
approximately given by
h i
ni  nwater P sat
i if C if =P s :

Here, Ps is the total pressure on the permeate side, nwater is the water mass flux through the membrane, P sat i is
the vapor pressure of species i whose activity coefficient in the feed solution is if at a molar concentration C if .
Assume that P sat
i , if and nwater are unaffected by pH levels. For the solute phenol, indicate how ni will change as
the feed water pH is changed. Given pK phenol 10:4 at 50  C, show that the fractional reduction in phenol flux
with pH is given by 1=f1 10pK phenol pH g.

5.4.2 To utilize poly(acrylic acid) (represented as HL) as a chelating polymer in polymer-assisted ultrafiltration based
removal of a divalent heavy metal (Mn, n 2) present in wastewater, it is useful to consider the following
polymer ligandmetal complexation equilibria:

K1
M2 HL ! LM H ; 5:P:7

K2
LM HL ! ML2 H ; 5:P:8

K3
ML2 2HL ! L2 M HL2 : 5:P:9

When one carries out batch ultrafiltration, by measuring the concentration of free (uncomplexed) M2, one can
find out q, the amount of metal ions bound to polymer (mol/mol of polymer). One can also determine navg, the
average coordination number. Develop expressions for (a) q and (b) navg in terms of K1, K2, K3, KdHL, C H and
C M2 for a divalent metallic cation. You are given that the total poly(acrylic acid) concentration is C tHL .

5.4.3 In Section 5.4.4, we studied how phenol can be removed very efficiently from wastewater through an oily liquid
membrane into a caustic-containing receiving phase.
(a) Suppose the wastewater contains bases like NH3 and amines (R-NH2, say). These are quite soluble in the
oily liquid membrane. What would be the effect of these species on phenol removal from wastewater?
(b) If the wastewater contains only bases like NH3 and amines, what should the receiving aqueous phase
contain to achieve wastewater purification similar to that for phenol?
(c) If you have any oily membrane and the feed wastewater contains a very strong acid, e.g. HCl, would you
use the technique explained in Section 5.4.4?
(d) Wastewater contains phenol and HCl, which are both acidic, and the receiving phase contains caustic
soda. Describe the separation that can be achieved.

5.4.4 Consider an oily membrane without any acidic cation exchanger (see Problem 5.2.8). Through such a
membrane, which may be a liquid or a polymeric membrane, it is possible to remove ammonia NH3 (i 1),
but not the ammonium ion NH 4 i 2. In a process somewhat analogous to that described for phenol removal
in Sections 5.4.3 and 5.4.4, the feed solution is basic (pH > 10); small amounts of ammonia in the feed (j f)
permeate through the membrane to the other side (j p), where there is a highly acidic solution to react with
the ammonia.
(a) Formulate the governing mass balance equations and associated equations for chemical reaction equilib-
rium for the transfer of NH3 from a basic solution in the feed vessel (volume Vf) to the permeate/receiving
vessel (volume Vp) containing an acid (sulfuric or phosphoric acid) at a high concentration.
(b) Write down the boundary conditions.
(c) Develop a solution for the problem in terms of the total ammonia concentration in the feed solution as a
function of time.
(d) Estimate the total ammonia concentration in the feed as t ! for a feed pH 10 and permeate pH 2.
You are given
K dl 
(i) NH3 H2 O ! NH 5
4 OH ; K d1 1:86  10 25 C without water concentration;
K 9
(ii) NH3 H ! NH 4 ; K 1:86  10 25 
C .
Problems 345

5.4.5 The facilitated transport separation of CO2 from a gas mixture through an aqueous liquid membrane of
diethanolamine (DEA) occurs via the following reactions:
k Am
CO2 R2 NH R NCOO2

H 1; R2 NH H !R2 NH
2: 2
k Am

Reaction (2) is considerably faster than reaction (1) (Danckwerts and Sharma, 1966), and is therefore assumed
to be in equilibrium (the equilibrium constant for this protonation reaction is Kp, whereas that for (1) is KAm).
Assume a negligible H contribution to the electroneutrality relation. Develop two governing ordinary differen-
tial equations (one for CO2 and the other for the free amine R2NH) describing the facilitated transport of CO2
and write the boundary conditions. You are given that the total initial amine concentration in the aqueous
liquid membrane is C tR2 NH . The governing ordinary differential equations should contain only the concen-
trations of free CO2 and free amine and C tR2 NH .

5.4.6 Consider the facilitated transport based separation of species A from C in a feed gas mixture, through a liquid
membrane that contains a nonvolatile carrier species B, with which A reacts reversibly via A B !AB. The
equilibrium constant K for this reaction is 12 (M1). The molar concentration of species B added to the liquid is
6 M. The feed molar concentration of species A is 0.1 M. Assume DABM DAM and a thick membrane. Calculate
the ratio of the increased flux of species A with respect to the nonfacilitated flux of species A. The permeate
molar concentration of species may be neglected with respect to the feed concentration. State your
assumptions.

5.4.7 Platinum chloride anions PtCl2 from an acidic solution are being extracted through a liquid membrane into
 
6
a basic solution. The organic liquid membrane in which platinum chloride anions are not soluble contains
trioctylamine (TOA), with which the following complexation occurs by an interfacial reaction:

2R3 Norg 2H PtCl


6 !R3 NH2 PtCl6 org:

Assume a fast interfacial reaction. There are boundary layer resistances to PtCl
6 diffusion on each side of the
membrane.
(a) Draw the concentration profiles of various species (H, R3N (org), PtCl 6 and the complex) in various
regions of the system.
(b) Identify which out of facilitated transport, countertransport and cotransport is operative here.
(c) Write down the governing metal flux equations (flux coefficient  concentration difference) for each
region (the boundary layers and the membrane).
(d) What is the maximum value of the metal flux in each region?
5.4.8 Copper is extracted from an acidic solution into an organic phase containing 2-hydroxy 5-nonylbenzophenone
oxime (RH) by an interfacial reaction

Cu2 2RH !CuR 2 2H ,

where the overbar represents an organic-phase species. The interfacial reaction rate was found by Komasawa
and Otake (1983) to be of first order in oxime concentration for concentrations more than 12 gmol/m3 and of
first order in Cu2 concentration. Without neglecting the backward interfacial reaction rate, write down the flux
expressions for each of the three resistances leading to the reaction, as well as for the diffusion of products.
Assume the diffusion of protons to be extremely rapid. Develop an expression for copper extraction flux in
terms of bulk concentrations, mass-transfer coefficients and rate constants. If the oxime concentration is very
high, develop the simplified expression for NA, where A represents the divalent metal cation Cu2.
6

Open separators: bulk flow parallel to force


and continuous stirred tank separators

Chapter 4 described the extent of separation that can be stirred tank separator (CSTS). None of the three bulk
achieved in a closed vessel under three basic categories of flowforce combinations (a), (b) or (c) can describe the
separation: phase equilibrium based separations; external separation in a CSTS; it provides a separate bulk flowforce
force based separations; membrane based separations. combination.
Beginning with Chapter 6, we focus on separation achieved In this chapter we illustrate separation achieved in
in an open vessel: fluid streams and/or solid streams may the bulk flow of phase(s) parallel to the force direction.
flow into and/or out of the vessel. Thus, we have bulk flow/s We will also briefly study CSTSs at the end of this
in and/or out of this device. A broad variety of bulk flow chapter. Batch well-stirred tank based separators with-
patterns can exist in a separation vessel. We will, however, out any continuous feed in or product out will also be
mostly study separations under three general categories of studied.
bulk flow configurations defined with respect to the direction Chapter 7 will consider separations achieved under the
of the force which is the source of the basic separation bulk flowforce combination of (b). Separation systems
phenomenon. The three general categories of bulk flow utilizing the configurations of (c) are treated in Chapter 8.
force combinations are: (There will be occasional examples of two combinations of
bulk flow and force directions.) Chapters 6, 7 and 8 will
(a) bulk flow of phase(s) parallel to the force direction;
generally employ one separator vessel. Reactive separ-
(b) bulk flow of feed-containing fluid phase/region per-
ations will be treated immediately alongside non-reactive
pendicular to the force direction;
separations as often as possible. Different feed introduc-
(c) bulk flow of two fluid phases/fractions/regions perpen-
tion modes will be considered as required in all three
dicular to the force direction.
configurations, (a), (b) and (c). Multistage separation
In the bulk flowforce combination of (c), there can be schemes, widely used in the processes of gas absorption,
cases where, instead of two fluid phases, one can have distillation, solvent extraction, etc., are studied in Chapter
one fluid phase and another solid phase. Categories (b) 8 when only one vessel is used. When multiple devices are
and (c) provide a broader and more useful framework used to form a separation cascade, an introductory treat-
than the category of bulk flow perpendicular to the force ment is provided in Chapter 9.
direction illustrated by Giddings (1991) using a few Section 6.1 begins with a description of mechanisms/
examples. driving forces/sources that cause bulk flow to take place
Bulk flow(s) in one or more directions in relation to the in the separation device. Following a brief description of
force direction creates conditions leading to the develop- these sources of flow into and out of the separation device,
ment of difference in concentrations of species along the as well as inside the device, we provide a brief illustration
directions of bulk flow(s) and/or force. However, in one of a variety of ways in which the feed is introduced into the
type of open separator, where the whole vessel may be separator. Section 6.2 will identify the general equations of
considered perfectly mixed, there are no spatial gradients change in an open separator.
in species concentration anywhere in the vessel. Yet the First, we consider the variation of concentration of a
feed is introduced continuously into, and the product species in a solution/mixture with space and time inside
stream(s) is (are) withdrawn continuously from, the vessel. a separator. An equation that considers such variations in a
Such open separators are analogous to a continuous stirred phase/region will be illustrated. Since such a concentration
tank reactor (CSTR) and may be identified as a continuous distribution in time and separator location is influenced
Open separators: bulk flow parallel to force and CSTSs 347

strongly by the flow field in the separation device, the descriptions of separation achieved rather than a rigorous
equation of continuity and the equations of motion which theory to describe separation development in every case.
generally govern the flow field in the separator will be This will involve, on the one hand, using approximate/
identified. If the flow field is known, it will be introduced known velocity fields. On the other hand, and more often,
into the equation for species concentration change in a we will assume, in the case of equilibrium separation
given problem to be studied in Chapters 6, 7 and 8. If the processes, the equilibrium limit: we will assume that the
flow field is not known, the equation of continuity and immiscible phases in contact and in motion in the separ-
the equations of motion have to be solved to determine ator are at thermodynamic equilibrium with respect to
the flow field (sometimes along with the equation for con- each other/one another (for three or more phases). The
centration change). The solutions of such combinations of actual performance may then be determined by means of
equations are, in general, quite demanding and will be estimates of stage efficiency, which quantifies the deviation
avoided in general. from thermodynamic equilibrium.
Such equations of change to be presented in Section 6.2 The second broad area in Section 6.2 is concerned with
for spatio-temporal variations of species concentrations particles. For the separation of particles from a fluid or
employ the continuum level description. In this approach, fractionation of particles, one can adopt an Eulerian
the equations of change are developed over a volume elem- approach to determine the particle concentration variation
ent of microscopic dimensions, such that the fluctuations as observed by an observer located at a fixed coordinate
due to individual molecular motions are averaged out yet (x,y,z). In such an approach, the fluid velocity is also what
the volume element is tiny, of the order of micrometers (say). is determined by an observer at (x,y,z) as a function of
To reduce the level of complexity in single-phase as well as time. However, an alternative approach, the Lagrangian
multiphase systems further, volume-averaging techniques approach, is frequently preferred and will be adopted
are adopted. These techniques allow one to replace the local often. The Lagrangian description of particle motion is
velocities and concentrations in the separator by values aver- obtained by an observer who rides on the particle. The
aged over a unit cell, one of whose dimensions is small geometrical coordinates (x,y,z) of the particle/observer
compared to the separator dimensions (Lee et al., 1977a). change with time as the particle changes its location in
The resulting equations of change of species concentrations the device in response to fluid motion and other forces,
are then said to be considered at what is known as the external and/or diffusive, acting on the particle. In such an
pseudo-continuum level (Lee et al., 1977a). approach, the coordinates (x,y,z) of a particle are depend-
This approach is almost always adopted when dealing ent variables whose values as a function of time in the
with separation in packed beds, or with a porous medium separation device are of interest. These equations, called
in general. In such an approach, one averages out the trajectory equations, are also provided in Section 6.2.
species concentration, velocities, etc., in one or two phys- In separation processes involving crystallization, hydro-
ical dimensions; the concentration, velocity, etc., then sols, aerosols and liquidliquid dispersions, we encounter
remain a function of, say, one remaining physical dimen- a size-distributed population of particles/drops which may
sion and time. The usual physical dimension along which be undergoing change in the separation device due to
variations are retained is the direction of main flow, processes of crystal growth, drop/particle breakage, drop/
whereas the directions perpendicular to this main flow particle coalescence, etc. For such systems, a population
are averaged out. For flow in a tubular packed bed, this balance equation in the form of a general dynamic equa-
implies averaging in the radial and circumferential direc- tion for changes in the particle/drop size distribution
tion of, say, concentrations and velocities, which can now function is developed in Section 6.2. An integrated form
vary only along the direction of mean flow. of this equation has also been provided for a CSTS.
Volume-averaging techniques are useful in obtaining Section 6.3 covers three basic categories of separation
the equation of concentration change of a species in a when the bulk flow is parallel to the direction of the
given phase in a multiphase system. At any given cross force. Section 6.3.1 describes how separation is achieved
section of a device, there will be two such equations if there when either the gravitational force, or an electrical force,
are two phases/regions. These two equations will be centrifugal force or inertial force, acts on ions/molecules/
coupled through the boundary conditions at the interfaces particles present in a fluid flowing in a direction parallel
of the phases/regions. The solution of each equation in to the direction of the force. Gravity-driven elutriation,
each region will provide the concentration of the species in capillary electrophoresis, countercurrent electrophoresis,
that phase/region. This task can be quite complex, espe- centrifugal elutriation and inertial impaction are specific
cially if the flow field in the device, packed bed, distillation techniques considered here. The equilibrium separation
plate, etc., is complex. processes of flash vaporization, devolatilization, batch dis-
To develop easier routes to the solutions, we will tillation for vaporliquid systems, liquidliquid extraction
sometimes adopt simpler approaches in the following in the differential extraction mode, zone melting/normal
chapters. The objective is to develop reasonable freezing for solid/meltliquid systems and drying for
348 Open separators: bulk flow parallel to force and CSTSs

solidvapor systems are studied in Section 6.3.2. Section L


6.3.3 illustrates the operation of membrane separation
processes of cake filtration/microfiltration, ultrafiltration, z v 2R (a)
reverse osmosis, pervaporation and gas permeation when
the bulk fluid flow is parallel to the force acting perpen- Parabolic axial velocity profile
Pump
dicular to the membrane. The important role played by in a pipe/tube of radius R
the magnitude of the bulk velocity has been highlighted as
often as possible.
(b)
Section 6.4 covers continuous stirred tank separators.
Section 6.4.1 studies equilibrium separation processes;
most of this section is devoted to crystallization, with add- Pump
itional coverage of liquid extraction. Membrane separation
processes/devices are sometimes modeled as CSTRs. Figure 6.1.1. (a) Pressure-driven laminar flow in a tube/capillary/
Section 6.4.2 touches upon a few of these examples, pipe; (b) pressure-driven flow in a tube/capillary/pipe filled with
encountered, for example, in ultrafiltration and gas perme- packings, a packed bed.
ation. There are brief treatments of batch systems that are
well-stirred in Sections 6.4.1 and 6.4.2 for both equilibrium
based and membrane separation processes. length L in the mean flow direction (z-coordinate). The
following relations are quite useful in a variety of sepa-
ration devices and processes. Consider first flow in a
circular tube of radius R (Figure 6.1.1(a)). Assume steady
6.1 Sources and nature of bulk flow incompressible flow.
The bulk flow of fluids, with or without solid particles, can
be achieved by a variety of forces: hydrostatic pressure 6.1.1.1 Laminar flow in a straight circular tube of radius R
difference, gravity, free convection, capillarity, electrical
force causing electroosmotic flow, centrifugal force, surface Pressure dropflow rate relation:
tension gradient and drag force. For a given force driving 8L
the fluid into bulk motion, the nature of the velocity profile P Q HagenPoiseuille equation: 6:1:1a
R4
will depend on the flow channel geometry and flow
1
obstructions in the channel. Elementary identification of Parabolic axial velocity profile:
each of the above types of sources of bulk flow will be   r 2 
P 2
made in Sections 6.1.1 to 6.1.8, under the assumption that vz r R 1 : 6:1:1b
4L R
the flow of the liquid/gas is viscous. Bulk flow in separation
devices is most often intimately connected with the entry Maximum velocity vz,max at the pipe center:
and exit of the feed stream. The various forces identified P 2
above have been employed to move feed fluid into and out vz, max R 2vz,avg , 6:1:1c
4L
of separation devices. Such feed fluid flow may be conti-
nuous or discontinuous. An elementary illustration of the where vz,avg is the average velocity, R2vz,avg Q.
various types of feed introduction in open systems will be These relations are valid for laminar flow of the liquid.
provided in Section 6.1.9. Laminar flow continues until around a Reynolds number
(Re vz,avg2R/) of 2100. For a given Q, P increases
linearly with L; it increases drastically as R is reduced.
6.1.1 Hydrostatic pressure induced bulk flow
Consider a separation device, tubular or otherwise, filled 6.1.1.2 Laminar flow in a rectangular channel of gap 2b
with a liquid; assume also that the inlet pipe filled with
the liquid is connected to a pump (Figure 6.1.1) at the The following relations are based on the assumption that
inlet of the separation device, which is assumed to be the width W of the channel of length L is large compared to
horizontal. As the pump runs, it introduces mechanical the gap between the two parallel walls, 2b; correspond-
energy into the liquid, which is driven into the device ingly, the viscous effect of the two edges of the channel
against whatever flow resistance is offered. As the liquid distance W apart may be neglected.
moves through the device, its hydrostatic pressure is
reduced due to frictional losses. The relation between
the hydrostatic pressure drop P encountered by the
liquid and its volumetric flow rate Q depends on the flow 1
From now on, the mass average velocity of fluid, vtz, will be
regime, liquid viscosity , the flow channel geometry and replaced by vz for simplicity.
6.1 Sources and nature of bulk flow 349

Pressure dropflow rate relation: v0 v: 6:1:4a


3L The mean particle diameter dp is defined as
P Q: 6:1:2a
2b3 W 6
2
dp , 6:1:4b
Axial velocity profile: av
 y 2  as if the bed consisted of spherical particles of diameter dp
Pb2
vz y 1 : 6:1:2b only. The hydraulic radius Rh is defined as
2L b
available flow cross-sectional area
Maximum velocity vz,max at the center of the channel: Rh :
perimeter of flow channel in contact with the fluid
Pb2 3
vz, max vz,avg : 6:1:2c For a circular capillary of diameter 2R, 4Rh 2R. One can
2L 2
rewrite the hydraulic radius definition as follows:

void volume available for flow=bed volume


Rh ) Rh : 6:1:4c
particle surface area of bed in contact with fluid=bed volume av 1 

6.1.1.3 Turbulent flow in a straight tube of radius R For laminar flow of a fluid of viscosity through such
cylindrical capillaries of radius 2Rh, from equations
As the Reynolds number of the flow in a tube or a channel is
(6.1.1ac),
increased to beyond a certain value (e.g. 2100 for a tube),
turbulent flow sets in. For turbulent flow in a horizontal tube P 2 P 2 v0
vz,avg R 4R v : 6:1:4d
of radius R, length L and average axial velocity3hvz i, a dimen- 8L 8L h
sionless quantity f, the Fanning friction factor, is defined as
  Substituting (6.1.4b) and (6.1.4c) into the above, one obtains
1 R P
f , 6:1:3a 2
2 L hvz i2 =2 P 3 dp
v0 2 : 6:1:4e
L 1  72
where is the fluid density. The Fanning friction factor is a
function of the tube Reynolds number in the following In reality, the length of the flow path in the hypothetical
manner4 capillaries in the bed is larger than L due to the path tortuosity.
0:0791 Experimental measurements indicate that the following
f 6:1:3b
Re1=4 equation (the BlakeKozeny equation) is instead more accur-
over a Re range of 2100 < Re < 105 for fully developed ate as long as < 0.5 and (dp v0/ (1  )) < 10:
turbulent flow.
P 3 d2p
v0 2 : 6:1:4f
6.1.1.4 Flow in a packed bed L 1  150

The flow in a packed bed, where the packing may be spher- Flow in a porous medium and porous membranes is some-
ical, cylindrical, etc., is quite complex (Figure 6.1.1(b)). How- times described by Darcys law, an empirical relation charac-
ever, it is often modeled as a collection of cylindrical terized by a hydraulic permeability, Q (see equation (3.4.88)):
capillaries of hydraulic radius Rh and length L, which is the
packed bed length. Let be the fractional void volume of the QP
volume flux v0 : 6:1:4g
bed, av be the total particle surface area per particle volume, v L
be the actual interstitial velocity in the void volume between From (6.1.4f), one can express the hydraulic permeability
particles and dp ( 2rp) be the mean particle diameter. Then Q as follows:
the superficial velocity v0 based on the empty cross section of
the packed bed is defined as 3 d 2p
Q 2 : 6:1:4h
1  150
2
The y-coordinate is normal to the parallel plates and is at the Such expressions are used also to describe the volumetric
center of the channel gap; thus, the plates have the y-coordinates
flow rate through deposits of particles, macromolecules,
of y b.
3
This is the time-averaged axial velocity averaged over the tube proteins, etc., on top of filters, membranes, etc. Often,
radius. a tortuosity factor, , is used along with L in the defini-
4
Blasiuss law. tion (6.1.4g) of hydraulic permeability to accommodate
350 Open separators: bulk flow parallel to force and CSTSs

empirically the longer effective flow path length (compared the gas velocity, gas pressure and the axial location z,
to L, the thickness of the deposit). we note from (6.1.5c) that
vz,avg Pf
: 6:1:5f
6.1.1.5 Compressible flow of gas in a capillary/pore vz,avg, f P

Under conventional conditions of bulk flow in separation From expression (6.1.5d), if, instead of L, any location z is
devices, liquids may be considered incompressible: there is used, we have
essentially no variation in density even though pressure
can vary considerably along the liquid flow direction. In R4 f 2 R4 f 2
wtz P f  P 2 P  P 2l :
many conventional separation devices, the gas streams 16 z P f 16 L P f f
may also be considered to be undergoing incompressible Thus
flow. However, in the case of some membrane gas sepa-
z 2
ration devices, gas chromatography and flow through P 2f  P 2 P  P 2l : 6:1:5g
porous membranes, variation in gas density, and the L f
consequent change in gas velocity due to a change in the Therefore
gas pressure along the gas flow path, is considerable.
vz,avg Pf
The basic governing equation for compressible flow of h i1=2 : 6:1:5h
vz,avg, f
gas in a capillary/pore of radius R will be the Hagen P 2f  z 2
L P f  P 2l
Poiseuille equation5 applied over a differential length dz
in the bulk flow direction: As (Pf/Pl) increases for any given (z/L), the velocity at
  z increases substantially over that at the feed location.
R4 dP At z L, for example, if Pf 5Pl, vz,avg becomes 5vz,avg,f.
wtz  Qz : 6:1:5a
8 dz

Assume also the ideal gas law to be valid. Two forms of 6.1.2 Gravity induced bulk flow
ideal gas law are useful. One form relates the gas pressure
to the gas density: Wetted-wall columns (Figure 6.1.2(a)) and falling-film
! devices employ gravitational force to allow a thin liquid
 
P Pf film to flow down a vertical wall or an inclined wall, often
, 6:1:5b
z f with a gas stream flowing countercurrently upwards. For the
coordinate direction of flow being z along a plane surface
where f refers to the feed gas and z refers to any axial inclined at an angle to the vertical (Figure 6.1.2(a), 0;
location along the flow direction. The second form relates Figure 6.1.2(b), 6 0), the volumetric flow rate Q of a liquid
the gas pressure to the average gas velocity, vz,avg: film of thickness , density and viscosity is given by
Pvz,avg z P f vz,avg, f : 6:1:5c gW 3 cos
Q , 6:1:6
Assume isothermal flow and not enough change in pres- 3
sure along the flow path of length L so that can be where W is the width of the plate in the x-coordinate
assumed to be constant. From (6.1.5a) and (6.1.5b), since perpendicular to the paper. The parabolic velocity profile
the total mass flow rate of the gas wtz is constant, integra- is given by
tion from z 0 (feed) to z L yields 
g2 cos y2 
R4 f 2 vz y 1 : 6:1:7
wtz P  P 2l : 6:1:5d 2
16L P f f
The average velocity vz,avg over the film thickness is
Here the gas pressure at the capillary exit is Pl. If the
average gas density avg can be determined at the average g2 cos 2
vz, avg vz, max : 6:1:8
gas pressure in the capillary, [(Pf Pl)/2], then 3 3

R4 There are no ripples over the falling-film surface when the


wtz P f  P l , 6:1:5e Reynolds number Re (4vz,avg/) is less than 20 (Bird
8L avg
et al., 2002). The flow becomes turbulent when Re > 1500.
which relates the mass flow rate directly to the gas flow
pressure drop P Pf Pl. To obtain a relation between
6.1.3 Free convection
Bulk motion of a fluid due to a density difference between
5
Equation (6.1.5a) is also the basis of the result (6.1.1a). different fluid elements is called free convection or natural
However, here varies with z; so does Q. convection. The density difference may come about due to
6.1 Sources and nature of bulk flow 351

(a) (b)

Liquid reservoir

Pipe wall Velocity distribution


Liquid film vz (y) z
y
Free surface
d
y
Wall
d
Liquid L
film q
vz
z Direction of gravity
Calming
Liquid section
out Gas

Figure 6.1.2. (a) Wetted-wall column; (b) gravity induced flow in a falling film down an inclined plane.

closed vessel, where it turns around and flows over to the


Temperature T2
colder plate; the fluid closer to the colder plate (at T1)
distribution
meanwhile descends since it is heavier due to a higher
T(z ) T
density at a lower temperature. At the bottom of the closed
vessel, this colder fluid will turn around and rise up near
Velocity
the plate at T2 since it becomes lighter.
distribution T1
vz (y) Let the rectangular channel gap (Figure 6.1.3) between
Heated plate

Cooled plate

the two vertical plates be 2b. Assume the fluid viscosity to


be essentially constant, even though it is dependent on
temperature. The buoyancy forces on the lighter fluid
elements surrounded by heavier fluid elements (see
(3.1.5)) cause the lighter fluid elements to rise and, corres-
pondingly, the heavier fluid elements to sink. The profile of
the vertical velocity profile is given by (Bird et al., 2002)
 
b2 g y 3 y 
z vz  , 6:1:9
12 b b
y
where T T2 T1, is the fluid density at the average
b b
temperature 1 2 =2 and is the coefficient of
 
Figure 6.1.3. Bulk flow due to natural convection in a closed volume expansion d=dTT = evaluated at T. The
rectangular channel with two vertical plates at different average velocity of the lighter stream flowing upward is
temperatures. given by

g b2
temperature or concentration differences. Consider the vz,avg : 6:1:10
48
closed rectangular vessel shown in Figure 6.1.3. The verti-
cal plate on the left is maintained at T2 and the vertical Such bulk motion is primarily used in separation by
plate on the right is kept at T1 (<T2). The fluid (gas or thermal diffusion (see Section 8.1). Countercurrent bulk
liquid) close to the plate at T2 is at a temperature higher motion of this type has also been employed in a gas
than the fluid close to the plate at T1. The fluid close to the centrifuge to separate isotopic mixtures (Bramley, 1940;
plate at T2 is hotter and lighter, and rises to the top of the Martin and Kuhn, 1941). Bulk motion of this type is also
352 Open separators: bulk flow parallel to force and CSTSs

developed if the densities of the solutions at the two 212 cos


P gh0 P atm  P liq : 6:1:12
plates (Figure 6.1.3) are different due to concentration R
differences, which can be created in the process of
This quantity is the capillary pressure. If the contact angle
electro-gravitation (Shaffer and Mintz, 1980) using an elec-
> 90 , then the liquid height is depressed into the tube
trodialysis cell having an ion exchange membrane and a
from the free surface, as is observed, for example, in the
neutral membrane.
case of a glass tube and mercury.
For cases where the contact angle < 90 (especially
6.1.4 Bulk motion due to capillarity for completely wetted surfaces), the final magnitude of the
liquid height h, namely h0, is attained after a sufficiently
In Section 3.3.8, we observed that, in a gasliquidsolid
long time. This value of h0 can be significant for fine
system with a flat solid surface, there is either zero or a
capillaries having small values of R. The spontaneous entry
finite contact angle (see Figure 3.3.16); the liquid attains
of the liquid into the capillary occurs at a certain rate; this
this contact angle spontaneously without any other exter-
bulk motion, regardless of whether the capillary, the mem-
nal force being present. If the contact angle is zero, the
brane or the porous medium/paper is vertical or horizon-
liquid spreads over the solid surface, displacing the gas/air;
tal, is of interest. The fine dimensions of the capillary/
the solid surface is said to be wetted completely by the
porous medium suggest Poiseuille flow. For a vertical
wetting liquid. This phenomenon is routinely encountered
capillary, Levich (1962) has therefore postulated that the
and exploited when a porous medium, such as a porous
rate of rise of the capillary height, dh/dt, is the average
membrane or porous bed or paper, comes into contact
velocity in Poiseuille flow (equation (6.1.1c)):
with a liquid which can wet the material of the porous
medium. dh P 2
vz,avg R , 6:1:13
The pores in a porous membrane or paper and the dt 8h
channels in a porous bed may be idealized as capillaries of
radius R. If we immerse a capillary of radius R made of a where P is the pressure drop needed for the liquid to
particular material into a liquid, we observe either of the flow. At any given time, when the capillary height is h, if
following phenomena. If the material has a finite contact = 0, the driving pressure difference, P, is equal to the
angle < 90 (for example, for water and a glass capillary), difference between the capillary pressure (definition
the liquid rises into the capillary to a finite height h0 (6.1.12)) and gh:
(Figure 6.1.4), given by  
dh R2 212
 gh , 6:1:14
212 cos dt 8h R
gh0 , 6:1:11
R which leads after integration to
where gh0 is the pressure difference P between the    
8h0 1 h
atmospheric pressure, Patm, and the pressure in the liquid t 2 ln  : 6:1:15
R g 1  h=h0 h0
at the curved gasliquid interface, Pliq (see equation
(3.3.48) for a spherical bubble of radius R): When h ! h0, t ! , as it should be.
If equation (6.1.14) is rewritten as,
 
Patm dh 12 2R gR2
 12 , 6:1:16
dt 8 h

Pliq then clearly the rate of rise is fast if the second term is
q small; conversely, the rate of rise is slow if it is large. This
h0 term is defined as the Bond number and is the ratio of the
2R
gravitational force over the force due to surface tension
(12/R) if R is a characteristic dimension for the system:
Liquid
Air gR
Bond number Bo : 6:1:17
12 =R

h Patm If the capillary is horizontal, then the length, l, of the


capillary wetted by the wetting liquid and the average
velocity of liquid motion, vz,avg, in the capillary are
related by

Figure 6.1.4. Rise of a liquid in a vertical capillary wetted by dl P 2


vz,avg R , 6:1:18
the liquid. dt 8l
6.1 Sources and nature of bulk flow 353

where P is given by equation (6.1.12). Therefore, where is the fluid viscosity, is the zeta potential of the
double layer (see equation (3.1.11a)) and d is the dielectric
dl 212 R cos constant of the solution ( relative dielectric constant of
,
dt 8l the solution  0, where 0 is the permittivity of vacuum).
which, upon integration and using the initial condition of Further, this velocity is essentially constant along the
l 0 for t 0, leads to capillary cross section; the velocity at the wall of the insu-
lating capillary will be zero (see Figure 6.1.5). For steady
2l 2 laminar flow, Bird et al. (2002) have provided the following
t 6:1:19
R12 cos expression for the velocity profile in a capillary:
and    
sr d
vEOF E 1  expy= , 6:1:22
1 R12 cos 1
vz,avg : 6:1:20
2 2 t
where y is the distance from the wall and is the Debye
length of the electrolyte solution (definition (3.1.10b)),
6.1.5 Electroosmotic flow which is typically around 1 nm for a 0.1 N electrolytic solu-
tion. For capillaries having dimensions of 50 m, one can
Electroosmotic flow of a liquid is achieved in the direc-
neglect exp(y/) with respect to 1, essentially throughout
tion of an applied electrical voltage due to the phenom-
the whole capillary, leading to a flat velocity profile.
enon of an electrical double layer; the liquid is generally
Newman (1991) has described the fundamental basis
an aqueous electrolytic solution. Consider a capillary of
for the derivation of the above equation. Newman (1973)
an insulating material, e.g. silica filled with an aqueous
has estimated the value of vEOF for an electric field E of
electrolytic solution. If the pH of the solution is greater
10 volt/cm and a zeta potential of 0.1 volt to be 7.8  103
than 3, the surface silanol groups (SiOH) get deproto-
cm/s for aqueous systems. Such low values are of limited
nated and become negatively charged. Cations from
practical use; electroosmotic flows are therefore generally
the solution (counterions) are attracted to the negative
neglected. However, in the recent applications of capillary
surface charges and form an electrical double layer
electrophoresis (CE), capillary zone electrophoresis (CZE),
(in a manner similar to that for a charged particle as
etc., where silica capillaries of diameter around 30200 m
shown in Figure 3.1.2D) to achieve electroneutrality
and length 2550 cm are used, with the voltage difference
on an overall basis (Figure 6.1.5). These counterions,
between the two electrodes at the two ends of the capillary
however, have water molecules around them, providing
being in the range of 900020 000 volts, the electric field
solvation shells.
strength, E, has a much higher value. Values of around
If now an electrical field of magnitude E is applied
250400 volts/cm are quite common. The value of vEOF
along the capillary length, the cations in the electrical
can, under such conditions, be much higher, in the range
double layer will move toward the cathode; the solvent
of 0.2 cm/s. Electroosmotic flow has thus become singu-
molecules in the solvation shell of the counterion will be
larly important in developing the small capillary based
dragged along with the cations toward the cathode. This
techniques of CE and CZE which are being increasingly
solvent flow is termed the electroosmotic flow (EOF); the
employed for the separation of proteins and other biomo-
magnitude of the solvent velocity thus generated is given
lecules (see Section 6.3.1.2).
by (Levich, 1962; Newman, 1973)
Analytical results for the velocity distribution, mass
d flow rate, pressure gradient, wall shear stress, etc., in mixed
vEOF E, 6:1:21
electroosmotic/pressure driven flows are available for
two-dimensional straight-channel geometry in Dutta and
Beskok (2001). The magnitude and direction of the electro-
Capillary wall having negative surface charge
osmotic flow inside a microfabricated fluid channel (25 m
high by 100 m wide) can be controlled by a perpendicular
electrical field of 15 megavolts/cm generated by a voltage
+ Counterions Velocity - of only 50 volts (Schasfoort et al., 1999).
from solution profile
A somewhat different mechanism of water flow due to
for water
the motion of cations having a hydration layer (the solv-
ation shell) has been postulated. It employs porous/ion
Capillary wall having negative surface charge exchange membranes whose pore diameters are in the
range of 15 nm. When such a membrane is placed
Figure 6.1.5. Electroosmotic flow of water in a capillary with between two electrodes containing an aqueous salt solu-
negative surface charge due to an applied electrical potential. tion, and electrolysis takes place on the application of
354 Open separators: bulk flow parallel to force and CSTSs

a voltage, two ions are generated at the electrodes. For centrifugal motion. The solids are removed from the screen
example, through a cation exchange membrane such as by scraping, etc., and discharged from the bottom.
NafionW, cations generated at the anode will be driven
to the cathode through the membrane pore to maintain
6.1.7 Surface tension gradient based flow
electroneutrality. The water molecules dragged by these
cations through the membrane contribute to an electro- Surface tension on the surface of a liquid at gasliquid
chemically driven flow (EDF) (Norman et al., 2005). or vaporliquid interfaces can vary due to a variation in
The magnitude of such a velocity is small; the flow rates temperature or species concentration. The components
generated are of the order of 115 microliter/min. of the tangential stress,6y, yx and yz, are related to the
corresponding gradients in interfacial tension, 12, between
the liquid phase, j 1, and the gas/vapor phase, j 2 by
6.1.6 Centrifugal force driven flow
(Bird et al., 2002):
We have discussed in Section 3.1.2.7 (equation (3.1.51))
12 12
that when a solvent/liquid rotates in a centrifugal field with yx  ; yz  : 6:1:24
x z
an angular velocity (radian/s), the hydrostatic pressure P
in the liquid increases radially outward: Assume that there are no variations of temperature or
composition in the x-direction; these vary only in the
dP M s 2
r: 6:1:23 z-direction. For example,
dr Vs
 12 
T
At the radial boundary of the centrifuge (see Figure 4.2.3 or yz  6:1:25
T z
Figure 4.2.1), the liquid pressure will have the highest
value (see equation (4.2.13) for the corresponding result in the case of temperature variation in the z-direction;
for a gas mixture). If the centrifuge boundary is porous correspondingly,
or has openings or nozzles, this high liquid pressure will  12 
i
drive the liquid through such openings as if we have yz  6:1:26
i z
hydrostatic pressure induced bulk flow, as described in
Section 6.1.1. in the case of variation of surface concentration
This principle is employed in centrifugal filters. Figure (mol/cm2) of species i, i (see definition (3.3.34)) in the
6.1.6 illustrates a basket-type centrifuge. Solid liquid sludge z-direction.
is fed into the device. The particulate phase is radially Bulk motion created by the variation in surface
thrown onto the screen of the basket, through which the tension resulting from a variation in liquid tempera-
liquid flows out, driven by the pressure generated by the ture is called thermocapillary flow (Levich, 1962). In a
shallow pan of depth h (shallow in the y-direction but
deep in the x-direction perpendicular to the plane of the
paper), the two walls of the pan (in the z-direction) are at
temperatures T1 and T2 (>T1) (Figure 6.1.7). Therefore,
Screen
the surface tension 12 varies with the distance z as
(Figure 6.1.7(a))
Feed  12 
T2  T1
12 12 T 1 z: 6:1:27
T l

Note that surface tension decreases with an increase


in temperature. The height of the liquid, h, will depend
on the axial location in the pan, i.e. h(z). At the top surface
of the liquid, the maximum flow velocity toward the
cold surface,
Filtrate
h d12
vz,max , 6:1:28
4 dz

Solids discharge
6
Force per unit area in the tangential direction on a surface
Figure 6.1.6. Centrifugally generated hydrostatic pressure driven whose normal is in the y-direction; the two components of this
flow of liquid through the screen on the basket. stress, namely yx and yz, are relevant here.
6.1 Sources and nature of bulk flow 355

T1

12 T2 > T1
g
z
l
(a)
vz y
T1
vz
T2 > T1
h (z)

z Wall of
(b)
shallow pan

Figure 6.1.7. (a) Surface tension profile in the liquid in a shallow pan whose two ends are at different temperatures, T1 and T2 (>T1);
(b) velocity profiles in the liquid in the shallow pan.

is achieved for a liquid of viscosity , the velocity profile consult Probstein (1989) for a more extended introduction
being given by to surface tension gradient based flow. Generally such
  12 motions are identified as Marangoni effects (see Sternling
y 3y d
vz 1 : 6:1:29 and Scriven, 1959).
2 2 h dz

The profile is shown in Figure 6.1.7(b). The direction of


6.1.8 Drag flow
motion of the z-directional velocity, vz, is reversed at y
(2/3)h, and the flow is toward the hot surface. The basis of When the viscosity of the feed fluid/material is quite high
this analysis has been provided by Levich (1962) and Yih and/or the material is thermally sensitive, separation of
(1968). volatiles by heat, or other phase changes like crystalli-
The surface tension gradient driven flow system in zation, often requires mechanical rotors/screws which
Figure 6.1.7(b) has different heights of the liquid film, rotate in the separator. As they rotate, they drag the highly
h(z), at different locations along the z-coordinate. Gravita- viscous material, which is pushed to one end of the
tional force will tend to eliminate it. The relative influence separator due to helical or other screw/rotor geometries.
of the two forces is indicated by the Bond number, Bo, The bulk motion of the fluid/solid is caused by the drag
defined earlier by equation (6.1.17). The characteristic force induced by the moving solid surface, where the
length dimension of the system to be used in the definition fluid/feed may be assumed to have the same velocity as
of Bo is any particular value of h(z): that of the solid rotor surface. Such fluid motion, however,
is reduced to zero at the other stationary solid surface,
gh2
Bo : 6:1:30 unless there is slip. This is called drag flow (Figure 6.1.8).
12
The figure illustrates the velocity profile created when
A separation process, where the spatial variation of surface the top plate moves with a velocity v and the bottom
tension due to composition is encountered, is distillation. plate is stationary. For a fluid having a constant viscosity,

If, along the length direction of flow of a binary liquid , the constant shear rate and the velocity profile are
mixture, the more volatile species is removed preferen- defined by
tially, one can have two possibilities: a positive system,
vz
where the surface tension increases due to removal of the , vz y y: 6:1:31
y
more volatile species; and a negative system, where the
surface tension decreases due to removal of the more An externally imposed pressure gradient may also be pre-
volatile species. Zuiderweg and Harmens (1958) have con- sent along with the drag flow, as in devolatilizing screw
sidered the effect of such motion on the performance of extruders for polymers.
small distillation columns. Such motions are important in There is an additional source of bulk flow within a
general when there are thin liquid layers (compare the separator vessel, namely a stirrer. It is an integral part of
shallow pan of Figure 6.1.7 as a basis). The reader may what are considered well-stirred vessels. This flow can take
356 Open separators: bulk flow parallel to force and CSTSs

place inside the vessel without any bulk flow coming in or flowing in the separator and their flow directions in rela-
going out. The energy for liquid motion is provided by the tion to the direction of the force. The feed may be intro-
rotating stirrer. duced continuously or discontinuously into the separator.
In many separation processes, the feed is introduced
continuously at a fixed rate into one end, or in the middle,
6.1.9 Feed introduction mode vs. bulk flow
or other location, of the separator (Figure 6.1.9(a)); the
The feed mixture may be introduced into a separation direction, the flow rate and the location of feed introduc-
device via a number of forces identified above. What is, tion are invariant with time. Devices where two phases,
however, much more important is the dependence of the regions or fractions flow perpendicular to the direction of
feed introduction on the time and spatial coordinates in the force are of prime interest (Chapter 8) in this mode of
the separator. We will progressively learn in Sections 6.3, feed introduction. Although there is only one feed stream
6.4 and Chapters 7 and 8 that this feed introduction mode coming in, there will be a total of two/three streams, which
is intimately connected with the number of phases/regions will be entering the separator in such a case. The presence
of a second mobile stream/phase/fraction within the sep-
arator allows species introduced via the feed to be removed
y
continuously as separation is being implemented locally
Top plate everywhere in the separator.
z The same time-invariant mode of feed stream intro-
duction and feed-phase flow perpendicular to the force
direction is also utilized in separations where any one
.
vz (y) = g y
Liquid stream enters the separator, provided at least two product
streams are continuously withdrawn from the separator.
Crossflow membrane separators and external force based
separators studied in Sections 7.2 and 7.3 are particular
Bottom plate examples. Of the at least two product streams, one is the
depleted feed stream. The other product stream continu-
Figure 6.1.8. Simple drag flow of liquid due to the motion of the ously removes selected species from the feed stream
top plate at a velocity v while the bottom plate is stationary. (Figure 6.1.9(b)).

(a)

Feed flow
rate

time

Feed Feed

Feed

(b)

Feed flow
rate

time

Feed
Feed Feed
Membrane

Figure 6.1.9. Constant feed flow rate into a separator. (a) Two/three streams entering/leaving the separator; (b) one stream entering and
at least two streams exiting the separator.
6.1 Sources and nature of bulk flow 357

Sometimes, in external force based systems, the feed where one feed stream enters the bed periodically and the
stream is merely a part of one stream continuously entering purified feed stream is withdrawn in a corresponding
the separator. In such a case, there may be more than two manner, are typical examples. The periodic feed is necessi-
product streams leaving the separator (see Section 7.3.1.1); tated by the absence of another mobile stream, phase or
further it is difficult to identify a depleted feed stream fraction removing species from the feed in a continuous
(Figure 6.1.9(b)). Crossflow fluidsolid adsorption pro- fashion. During the period of feed stoppage, arrangements
cesses employ this mode as well (see Section 8.3.1). such as the introduction of a purge stream, stripping stream
A time-invariant mode of feed stream introduction is also or a regenerating solution, are made to remove the species
practiced with a single-entry separator when the bulk flow is from the feed stream deposited into the separation medium.
parallel to the force, as in flash based distillation/devolatili- A variant of this second mode of feed introduction
zation processes having two steady product streams (see involves a pulse of feed mixture introduced into the separ-
Section 6.3.2.1). In all cases covered by Figures 6.1.9(a),(b), ator at a particular location periodically, as in the chroma-
a variation of this general mode of feed introduction tographic processes discussed in Section 7.1.5 (Figure
employs continuous feed introduction with some time- 6.1.10(b)). During the rest of the period, another non-feed
dependent variation of the feed flow rate magnitude. stream is most likely to be introduced into the separator to
Finally, in this mode of feed introduction (Figure 6.1.9(b)), continue separation and regenerate the separator. The last
in continuous stirred tank separators (see Section 6.4), one mode of feed introduction is simply an amount of fluid/
can have only one product stream, which, however, is a solid introduced into the separator in a batch fashion. The
multiphase stream (a configuration not shown here). product withdrawal may be batchwise or semi-continuous
A second mode of feed introduction involves a constant (stopping when there is no feed).
flow rate of feed into the separator at a fixed separator location A third mode of feed introduction employs a constant
for a fixed period of time, after which feed flow is stopped for a flow rate of feed into the separator at one end for a fixed
certain amount of time; feed flow is restarted after this pause. period of time; then the same feed flow pattern is imple-
The startstop is carried out cyclically for as many cycles as mented at the other end of the separator such that the feed
is feasible/desirable (Figure 6.1.10(a)). Fixed-bed processes in flow direction in the separator is reversed (Figure 6.1.10
a single-entry separator described in Sections 7.1.17.1.3, (c)). Such a feed flow pattern is observed in continuous

(a)

Feed flow
rate

time

(b)

Feed flow
rate

time
(c)

Feed flow
rate time

Figurer 6.1.10. Cyclic feed introduction. (a) Constant feed flow rate for part of the cycle in fixed-bed processes; (b) pulse input of feed in a
periodic fashion in chromatographic processes; (c) cyclic feed introduction from opposite ends of the separator, as in parametric pumping
processes. (After Lee et al., 1977a.)
358 Open separators: bulk flow parallel to force and CSTSs

parametric pumping based processes (see Section 7.1.4). concentration Ci of species i developed there depended
Lee et al. (1977a) have provided a comprehensive descrip- only on the z-coordinate and time, t. Here, we will first
tion of various patterns of feed introduction in relation to develop the corresponding equation for Ci, which depends
the motion/trajectory of solute(s) in separation devices. We on all three spatial coordinates x, y, z, time t and the fluid
note here that product withdrawal modes can also have a molar average velocity vector7v*. A similar equation of
variety of patterns, just as in feed introduction. change for mass concentration i of species i will also be
obtained; it will use the mass average velocity vector v.
These equations will have a term containing the rate of
6.2 Equations of change production of species i by chemical reactions, if any.
Figure 6.2.1 shows a small volume element of dimen-
The equation of change of concentration of a species i sions x, y, z in the x, y and z directions in the flow field
with the spatial coordinates of the separator and time of the separator at the location (x, y, z). For species i, we
may be obtained by considering the motions of individ- will employ the following basic principle of conservation of
ual molecules present in the fluid mixture. However, this mass for the small volume element of volume xyz:
strategy generally leads to extraordinary complexity, which
rate at
is not useful for us. Rather, we adopt the continuum rate of
which
rate at which rate of production
accumulation  species i of species i by
hypothesis. Although the fluid consists of billions of mol- of species i
species i
leaves chemical reaction:
ecules, which leads to tremendous fluctuations in every comes in

quantity of interest at any given point, we replace this fluid 6:2:1


with a hypothetical continuous phase, such that all phys-
We will now consider each of the above terms in some
ical, chemical and thermodynamic quantities of interest,
detail using molar rates:
e.g. density, temperature, composition, energy, are con-
stant over a very small volume, which can be identified as rate of accumulation C i
a point in the fluid for our purposes. Although there can of species i in the xyz; 6:2:2a
t
and will be variations in various quantities from one such volume element
point to the next, one can, at any given point, identify a rate at which species i enters the
particular value of any of these quantities mentioned volume through the face perpendicular
above. Such a point volume is large enough to average all N ix jx yz;
to the x-coordinate and having
fluctuations due to the individual and random motion of a corner coordinate x, y, z
molecules; yet it is small enough to allow for the identifi- 6:2:2b
cation of the actual changes taking place in the fluid during
rate at which species i leaves the
fluid motion and/or separation. After the appropriate dif-
volume through the face perpendicular
ferential equations of change are identified, we provide the N ix jxx yz;
to the x-coordinate and having
volume-averaged equations employed for separation in a corner coordinate x x, y, z
multiphase systems, where only variations in one direction,
6:2:2c
the main flow direction, are generally of interest in separ-
ator analysis. Subsequently, we provide the equations for a molar rate of production of
continuous stirred tank separator (CSTS). species i by chemical reaction Ri xyz: 6:2:2d
To provide an elementary background on separation in the volume element
systems where particles are present in a fluid and particle Additional terms for the molar rate at which species i
motion is important for separation, the equations of motion enters the volume element and leaves the volume element
of a particle in a fluid are provided next; these are also called through the faces perpendicular to the y-axis and the z-axis
trajectory equations. These equations have been followed can be easily developed following (6.2.2b) and (6.2.2c).
by a general equation of change for a particle population, the When all such terms are introduced into the balance equa-
population balance equation. Analysis of a CSTS for parti- tion (6.2.1), we get
culate systems is considered at the end of this section.
C i
xyz yzN ix jx  N ix jxx
t
6.2.1 Equations of change for species concentration in xzN iy jy  N iy jyy
a mixture
xyN iz jz  N iz jzz xyzRi :
In Section 3.2.1, a rectangular separator vessel was 6:2:2e
considered in which there was no bulk velocity of
the solvent/fluid in any direction; further, the solute
species i was subjected to a force in one coordinate 7
We drop the subscript t from vt and vt used in Chapters 2 and 3
direction (z-coordinate). The balance equation for molar from now on.
6.2 Equations of change 359

(x,y,z)
z

y
x

Figure 6.2.1. A small rectangular volume element in the separator having a corner point with coordinates x, y, z.

Dividing by (xyz) and allowing each of x, y and z to Here the migration velocity Ui of species i is defined by
go to zero, we obtain in the limit (3.1.84b) or (3.1.84d). Employing expression (3.1.99) for
the molar flux Ji*, equation (6.2.5a) becomes
C i N ix N iy N iz
   Ri : 6:2:3a
t x y z C i
rC i v  rJ i  Ri
t
Here i 1, 2, . . ., n, if the total number of species in the
system is n. This is the general differential equation of 1 2 3 4 6:2:5b
balance for species i in molar units in a system in the
This is the equation of change for molar concentration of
presence of any chemical reactions involving species i. An
species i. The different terms in this equation correspond to
alternative form in vector notation is
different aspects of the changing molar concentration at
C i any time and at any point in the separator:
rN i Ri : 6:2:3b
t (1) rate of increase of molar concentration of species i per
The corresponding differential equations in mass units for unit volume;
species i are obtained by multiplying the above equations (2) net molar rate of addition of species i by fluid motion
for species i by the molecular weight of species i, Mi: per unit volume;
(3) net molar rate of addition of species i by migration and
i nix niy niz diffusion per unit volume;
   ri ; 6:2:4a
t x y z (4) net molar rate production of species i by chemical
i reaction per unit volume.
rni r i : 6:2:4b
t Equation (6.2.5b) is often rewritten as
The nature of the equations of change to be employed for C i
C i rv  v rC i  rJ i  Ri : 6:2:5c
solving a particular separation problem depends on the t
expressions for molar fluxes Ni or mass fluxes ni. When an appropriate expression of Ji* is introduced in this
Employing expressions (3.1.84a), (3.1.84b) and (3.1.84d), equation, it becomes quite useful in analyzing the perform-
we may rewrite the general balance equation (6.2.3b) in an ance of a separator. Table 6.2.1 illustrates this equation
essentially equivalent but more detailed form for molar using scalar quantities for different coordinate systems,
concentration Ci as all of which employ a simplification developed next to
! express rv . Table 6.2.1 also illustrates the same equation
C i RT
r  C i v  r  C i U i r  C i d rlnai Ri : in Cartesian coordinates for the case where Ji* is expressed
t fi
via equations (3.1.99) and (3.1.89), and Ct and Dis are
6:2:5a constants.
360 Open separators: bulk flow parallel to force and CSTSs

Table 6.2.1. Equation of change for molar concentration of species i (equation (6.2.5g))
h i h  J  i n
X
C i J ix J iz
Cartesian coordinates t  vx C
x
i
v  C i
y y v  C i
z z  x iy
y z Ri  x i Ri
(x, y, z) i1

h i h i n
X
C i v C i 1

1 J i J iz
Cylindrical coordinates t  vr C i  C i 
r r vz z  r r rJ ir r z Ri  x i Ri
(r, , z) i1

h v C i
i h  i n
X
C i v C i 1 1 1 J i
Spherical coordinates t  vr C i  
r r r sin  r 2 r rJ ir r sin J i sin r sin Ri  x i Ri
(r, , ) i1

h i h i
C i 2 C i 2 C i 2 C i
Cartesian coordinates t vx U ix C i  C i  C i
x vy U iy y vz U iz z Dis x 2 Dis y 2 Dis z 2
(x, y, z)a n
X
Ri  x i Ri
i1

Ct is a constant throughout the table; Ri is the molar rate of production of species i by chemical reaction per unit volume.
a
Dis is a constant for this equation only.

If we sum equations (6.2.5c) for all species i 1, 2, . . ., n


X n
X
n together, we get r  ni r  ni r  t v: 6:2:5i
i1 i1
Xn Xn
C t
C t r  v  v  rC t  r  Ji Ri : This allows equation (6.2.5h) to be simplified to
t i1 i1
t
6:2:5d r  t v 0: 6:2:5j
t
By definition, This is called the equation of continuity for the whole
n
X X X  X mixture. If the mixture has constant mass density or mass
J i N i  C i v ) J i Ni  Ci v N i  C t v : concentration t, it is simplified to
i1

X r  v 0: 6:2:5k
From relations (2.1.2) and (3.1.77), C t v C i vi In the cartesian coordinate system, this equation is written as
X Xn
N i so that J  0. Therefore,
i1 i vx vy vz
0: 6:2:5l
C t Xn x y z
C t r  v v  rC t Ri : 6:2:5e
t i1
This equation is routinely used in the case of
incompressible flow.
If the total molar density Ct is constant, we obtain
n
Equation (6.2.5j) describes how the total density of the
X
C t rv Ri : 6:2:5f mixture changes with time t and the spatial coordinates,
i1 whereas equation (6.2.4b) describes how the mass concen-
When this result is introduced into equation (6.2.5c), we get tration of species i changes with time t and the spatial
coordinates. It is useful to transform the latter equation
Xn
C i in the same manner as equation (6.2.3b) for the molar
v  rC i r  J i  Ri  x i Ri : 6:2:5g
t i1 concentration of species i was changed to equation
X (6.2.5c). To that end, recognize that, from (3.1.84a) and
Note: Ri need not equal zero, since moles are not the definition of ni, namely ni NiMi,
necessarily conserved in a chemical reaction. 0 8 0 1 91
i < RT =
The procedure which led to equation (6.2.5e) from the @r  i v U i  i @ d Ar ln ai A r i
summation of equation (6.2.5b) for each individual species t : fi ;
yields a more important result, the equation of continuity 8 9
< RT =
for the mixture, if equation (6.2.4b) for the mass density or  i
r  i v  r  i U i r  d
r ln ai r i :
mass concentration i of each species i is employed: : fi ;
X n n
i t X X
6:2:5m
r  ni ri : 6:2:5h
i
t t i1 i1
n
X Consider now expression (3.1.99), where, instead of
Since mass is conserved in chemical reactions, r i 0. the simplification leading to the rx i term, we use the
Further, by definitions (2.1.1) and (2.1.3), i1 rlnai term; next use the definition ji JiMi. After a little
6.2 Equations of change 361

Table 6.2.2. Equation of change for mass concentration of species i (equation (6.2.5p))
h i h jiy
i
i j ix j iz
Cartesian coordinates (x, y, z) t  vx
x
i
v y
i
y v z
i
z  x y z r i
h i h i
i v i 1 j i j iz
Cylindrical coordinates (r, , z) t  vr
r
i
r v z
i
z  1
r r rj ir r z r i
h v
i h i
i v i 1 1 1 j i
Spherical coordinates (r, , ) t  vr
r
i
r r sin
i
 2
r r rj ir
r sin ij sin r sin r i

Ct is a constant; ri is the mass rate of production of species i by chemical reaction per unit volume.

Table 6.2.3. NavierStokes equation in Cartesian coordinates


  h2 i
vx 2 vx 2 vx ext
x-component t vtx t vx v
x
x
v vx
y y v vx
z z  P
x x 2 y2 z 2 t g x
  h2 i
v v v v vy 2 v y 2 vy ext
y-component t ty t vx xy vy yy vz zy  P y x 2 y 2 z 2 t g y
  h2 i
vz 2 v z 2 vz ext
z-component t vtz t vx v vz vz
x vy y vz z  z x 2 y2 z 2 t g z
z P

Newtonian fluid, constant values of t and .

rearrangement, we will obtain the following equation from Standard textbooks on transport phenomena (Bird et al.,
equation (6.2.5m): 2002) and fluid mechanics provide the derivation of this
equation and a detailed interpretation of each term. Certain
i
r  i v  r  j i r i : 6:2:5n types of conditions are commonly encountered during bulk
t
motion of fluids in separators; these in turn allow some
This is the equation of change for the mass concentration of assumptions to be made that simplify the equation of
species i. Each term in this equation represents a contribu- motion. The three most common assumptions are:
tion similar to that by the corresponding term in equation
(1) it is a Newtonian fluid;
(6.2.5b), except the molar concentration of species i has to
(2) it has constant fluid density(t) and viscosity();
be replaced by the mass concentration of species i. An
(3) fluid acceleration terms are neglected.
alternative form of equation (6.2.5n) is given by
Incorporation of the first two assumptions leads to the
i
i r  v  v  ri  r  j i r i : 6:2:5o following simplified form of the equation of motion, called
t
the NavierStokes equation:
If the mixture has constant total mass density, using
v
(6.2.5k) we get v  rv rP r2 v t g ext : 6:2:6b
t
i
v  ri  r  j i r i : 6:2:5p This vectorial equation has three component equations in
t
three coordinates. Table 6.2.3 provides three equations in
Table 6.2.2 illustrates this equation using scalar quantities. the Cartesian coordinates x, y and z. When the viscous
In all equations of change obtained so far, as well as in terms are neglected (i.e. r2 v 0) due to ! 0, the
the equation of continuity, the fluid velocity appears via v equations describe motion of an ideal fluid or inviscid fluid.
or v*. If the migration velocities of different species do not The fluid acceleration terms in the NavierStokes equation
influence the fluid flow field or the bulk fluid motion, then are (v/t) and (vrv). When these two terms are neglected
the fluid velocity field may be obtained from the solution of (usually for flows that are quite viscous), we obtain
what is known as the equation of motion:8
rP r2 v t g ext 0: 6:2:6c

v r  t vv  rP  r t g ext : 6:2:6a This differential equation is identified often as the equation
t t
for creeping flow or Stokes flow. The three components of
this equation in the Cartesian coordinate system are easily
8
Here t g ext represents all external forces per unit fluid volume obtained from Table 6.2.3 by equating the left-hand side of
acting on the fluid, including gravity, and is the stress tensor. each equation to zero.
362 Open separators: bulk flow parallel to force and CSTSs

Separation of molecular species i from species j in an For an incompressible flow, r  i v v  ri Further,


open separator having bulk flow requires a knowledge of from Ficks first law in a binary system of species i and j,
how Ci and Cj are distributed along the separator length. we obtain
This knowledge is acquired from a solution of equation
j i t Dij rui : 6:2:11
(6.2.5g) for each of species i and j. Similarly, solution
of equation (6.2.5m) for species i and j will provide Substituting these two relations into the equation of
the profiles of i and j in the separator. Tables 6.2.1 and change (6.2.10) for i, we get, for an incompressible flow,
6.2.2 illustrate these equations in terms of molar fluxes
i
and mass fluxes, respectively. It is useful, however, to v  ri t Dij r2 ui 6:2:12
t
consider such equations in terms of various constituent
terms of a flux expression. In Section 6.2.1.1 certain for the case of constant Dij and t. For constant t in the
special expressions will be used for the diffusive term in system, we can also write this as
multiphase systems; it is necessary to provide a limited
i
fundamental background here to approaches and treat- v  ri Dij r2 i : 6:2:13
t
ments that will be routinely employed in that section and
in Chapters 7 and 8. Dividing by Mi, we get
Consider equation (6.2.5g) for species i when there is C i
no chemical reaction (i.e. Ri 0): v  rC i Dij r2 C i , 6:2:14
t
Ci which becomes identical to equation (6.2.9) for an incom-
v  rC i r  J i : 6:2:7
t pressible system, constant diffusivity and dilute i (v v ).
Table 6.2.4 provides detailed expressions for equations
Further, focus on a liquid solution of species i in a solvent
(6.2.9) and 6.2.12) in cartesian and cylindrical polar coord-
where the migration velocity U i is zero; the relevant flux
inate systems for an incompressible fluid, constant diffu-
expression for J i is (3.1.101):
sivity Dij and dilute solution of i.
0 1
Consider now the laminar flow of a liquid in a straight
 0 @ d n i A
J i C t Dis 1 rx i circular tube of radius R and length L (Figure 6.1.1(a)). Let
d n x i 6:2:8
T,P a solute pulse of species i having mi moles or miMi grams
J i Dis rC i , be introduced across the whole cross-sectional area at the
location z 0 of such a straight circular tube shown in
where Dis is the molecular diffusivity of species i in the
Figure 6.2.2(a). As the liquid, having a parabolic velocity
solvent s. On substituting this flux expression in equation of
profile, flows, the question of interest is as follows. What
change (6.2.7) for Ci, we get, for an assumed constant Dis,
would this solute pulse look like way down the length of
Ci the tube?
v  rC i Dis r2 C i : 6:2:9
t We have considered an apparently related problem in
Section 3.2.1 in a stagnant system. We observed, via solu-
Similarly, consider equation (6.2.5n) for species i in the
tions of concentration profiles (3.2.15), (3.2.19) and (3.2.20)
absence of a chemical reaction (i.e. ri 0):
illustrated in Figure 3.2.3, that the species i concentration
i pulse introduced at z 0 was convected down the vessel
r  i v r  j i : 6:2:10
t (z > 0) at a speed equal to the migration velocity Uiz of the

Table 6.2.4. Equations of change for mass concentrationa and molar concentrationb of species i for constant t and Dij in cartesian and
cylindrical polar coordinates in nonreactive systems
 
i i 2 i
2 2
Mass concentration i, cartesian t vx x vy
y vz
i i
z Dij y2i z2i
x2
coordinates (x,y,z)
   2

i i v i i 1 i 1 i 2 i
Mass concentration i, cylindrical t vr r r vz z Dij r r r r r 2 2 z 2
polar coordinates (r, , z)
2 
Ci  Ci  Ci  Ci Ci 2 C i 2 C i
Molar concentration Ci, cartesian t vx x v y y v z z D ij x 2 y 2 z 2
coordinates (x,y,z)



Ci  Ci v C i  Ci 1 C i 1 2 C i 2 C i
Molar concentration Ci, cylindrical t vr r r v z z D ij r r r r r 2
2 z 2
polar coordinates (r,,z)
a
Equations (6.2.12), (6.2.13).
b
Equation (6.2.9); Ui is zero.
6.2 Equations of change 363

(a)
z =0 z Solute
Pulse of solute i R concentration
injected R measured
here
L
Cross-section L
averaged t=
t=0 vz,avg
concentration <ri >
of solute i
t Wb,i
(b)
Solute z =0
z Solute
concentration R concentration
front R measured
introduced
L here
Cross-section
rio
averaged
<ri >
concentration
of solute i
0 t

Figure 6.2.2. Axial dispersion of (a) a solute concentration pulse and (b) a solute concentration front introduced at the inlet of a straight
circular tube having a laminar liquid flow and a parabolic velocity profile.

solute i; furthermore, the narrow solute pulse was becoming diffusion, however, will counteract this axial dispersion in
a broader band due to molecular diffusion as z increased. the following manner. Regions in the band having a higher
This phenomenon was identified as band broadening or concentration at any z vis--vis a lower concentration at a
dispersion; note that this phenomenon was strictly due to larger radial location at the same z location will allow radial
molecular diffusion. diffusion from the higher concentration region to the lower
In the present problem (illustrated in Figure 6.2.2(a)), concentration region.
we will also find that the solute pulse introduced at z 0 For a cylindrical polar coordinate system needed to
will show up (on a radially averaged basis) as a concen- describe convective dispersion of the solute pulse in a
tration peak with a broadened base as z becomes large. straight circular tube (Figure 6.2.2(a)), we can discount
However, this broadening of the solute profile in the any -dependence due to essential circular symmetry.
z-direction is not due to the molecular diffusion coefficient The governing equation from Table 6.2.4 for species i in
Dis of species i in the solvent. Rather, it arises primarily due mass concentration terms is given by (Bird et al., 2002)
to the radially nonuniform axial velocity profile (6.1.1b, c)   r 2     
i i 1 2
of flow in a tube. It is identified as an axial dispersion or vz;max 1  Dis r i 2i :
t R z r r r z
convective dispersion. This phenomenon was first studied
by Taylor (1953, 1954), and is often described also as 6:2:15
Taylor dispersion. There is no exact analytical solution for this equation.
What happens is as follows. Let us ignore axial However, if one neglects the axial diffusion term,
solute diffusion altogether to start with. Since there is a Dis 2 i =z 2 , and solves the equation subject to the
parabolic velocity profile, solute molecules introduced via following boundary conditions:
the pulse further from the center line at z 0 are con-
at r 0, i =r 0; at r R, i =r 0,
vected downstream at a lower velocity compared to the
solute molecules introduced via the pulse near the center 6:2:16a
line at z 0. It is very clear that a band will develop around one can get a solution for i. Taylors solution was obtained
a mean position: sections of the band are at a larger z since by developing hii, which is the value of i when averaged
they were located at higher velocity regions to start with; over the radial cross section of the tube:
conversely, sections of the band are at a smaller z with 2 R
respect to the mean band position since they were located i r dr d
at lower velocity regions to start with. Radial velocity 2 R
hi i 0 2 0 R 2 r dr 6:2:16b
variation in the tube then creates an axial dispersion of R 0 i
r dr d
the (radially averaged) solute concentration profile. Radial 0 0
364 Open separators: bulk flow parallel to force and CSTSs


2 ! (as we will see in modeling adsorbers, chromatographic
mi M i z  vz,avg t
hi i p exp : 6:2:17 columns, etc., in Chapter 7), a dispersed gas phase in a
2R2 Di,eff t 4 Di,eff t continuous liquid phase or vice versa (in absorption/strip-
ping processes, distillation plates, etc., in Chapter 8), a
Here, 2vz,avg vz,max, and Di,eff is an axial dispersion coef-
dispersed liquid phase in another liquid phase (in extrac-
ficient such that a solution of
tion processes in Chapter 8) and other separation pro-
2 cesses/techniques where flow situations are complex.
h i vz,avg h i Di,eff 2 hi i 6:2:18
t i z i z Axial dispersion is also of increasing importance in
analytical-scale activities involving microfluidics, capillary
will yield the profile of the pulse in terms of the radially
electrophoresis, electrokinetic flow, etc. (Kirchner and
averaged i value, hii, as shown in Figure 6.2.2(a). The
Hasselbrink, 2005). In Section 6.2.1.1, where governing
expression for Di,eff is
equations are introduced for multiphase systems, the
1 effective axial dispersion coefficient, Di,eff, will be utilized
Di,eff Dis Pe2i , 6:2:19
48 in equations governing species concentration variables
where averaged over a cross section perpendicular to the mean

flow direction. Turbulence, if present, will introduce add-
Pei R vz,avg =Dis : 6:2:20 itional complexity. Note: The shortcomings of Taylor dis-
persion analysis are identified in pp. 225297 of an article
Solution (6.2.17), developed by neglecting the axial diffu-

in Marin (2005).
sion term Dis 2 i =z2 , has been demonstrated to be valid
when Pei
70 and the base width of the pulse ( W bi ) is
170R. 6.2.1.1 Pseudo-continuum approach for multiphase
Note: If the flow in the tube may be such that it can be systems
hypothesized as a plug flow,9 there is no axial velocity
Many separation systems have two phases, e.g. gasliquid,
variation in the radial direction; therefore the value of
vaporliquid, liquidliquid, liquidsolid, gassolid, vapor
Di,eff 0. Consequently, the solute pulse will appear
solid, supercritical fluidsolid, supercritical fluidliquid,
down the tube length again as a pulse without any
etc. Such systems do not possess the geometrical simplicity
broadening. The value of Pei,eff, based on Di,eff,
of, for example, Poiseuille flow in a round tube or the
Pei,eff R vz =Di,eff , 6:2:21 simple drag flow between two flat plates (Figure 6.1.8).
Packed beds, multiphase countercurrent or cocurrent con-
will then tend toward as Di,eff ! 0. Aris (1956) has tactors, where two immiscible phases flow past each other,
provided a more rigorous analysis, which includes axial etc., have very complex flow patterns. Yet there are two
diffusion, and has suggested the following expression for basic requirements: (1) to account for the species transport
Di,eff for long times: between the phases at the phase interfaces and (2) to
  determine the species concentrations and phase flow rate
R2 v2z,avg Pe2
Di,eff Dis Dis 1 i : 6:2:22 variations along the separator length/bulk flow direction.
48 Dis 48
An appropriate volume-averaging procedure is adopted to
If Pei is defined as (2Rvz,avg/Dis), the factor 48 in the above this end, and the resulting approach is identified as a
equation will be changed to 192. If, instead of a solute pseudo-continuum approach (Lee et al., 1977a).
pulse, a solute front is introduced at z 0 (Figure 6.2.2 Volume averaging means determining the average
(b)), then the solute concentration front down the end of value of a quantity over a certain localized volume; this
the tube will be substantially broadened due to the radial localized volume should be connected with a point in the
variation of the axial velocity in the tube. flow system. The averaging has to be carried out over three
The basic conclusions of Taylor dispersion (and Ariss characteristic lengths of the localized volume, lloc, which is
results) have been supported by solutions of the complete much smaller than the characteristic dimension l of the
equations by a number of investigators. For a brief intro- separator,10 but is much larger than the length, lvar, over
duction to these references and literature, consult Froment which the quantity under consideration, say Ci, fluctuates
and Bischoff (1979, p. 621) and Bird et al. (2002, p. 646). significantly: lvar << lloc << l. However, if we are studying
What is more important from our perspective is that such a the variations of the species concentrations and phase flow
model of axial dispersion (namely equation (6.2.18), can be rates along the separator length/bulk flow direction (say l),
effectively used for complex flow situations in separators. then the volume element over which the averaging is
The complexity may arise due to a packed bed of particles carried out spans the flow cross-sectional area of the

9 10
See Figure 6.1.5 for electroosmotic flow in a capillary. For an introduction to multiscale analysis, see Marin (2005).
6.2 Equations of change 365

separator and an appropriate dimension over the mean it is called an intrinsic phase average for that phase or
flow direction over which local fluctuations are evened region. The intrinsic phase averages ik,avgi ik and
out, and yet the averaged quantities change in the mean C ik,avgi C ik of ik and Cik, respectively, over the
flow direction. In what follows, we utilize the approaches of volume of phase or region k, Vk, are defined by
Slattery (1972), Whitaker (1973), Gray (1975) and Soo
1
(1989), and identify the averaged equations without going ik ik,avgi ik dV ;
Vk
through detailed derivations. Vk
6:2:24a
Consider k phases or regions present in the separation 1
C ik C ik,avgi C ik dV :
system. Most often, k 1,2, corresponding to phases 1 and Vk
Vk
2 or regions 1 and 2. Correspondingly, the equations for
the molar concentration Cik of phase k may be written from The phase averages ik,avg and Cik,avg of ik and Cik, respect-
(6.2.3b) using the relation ively, over the whole volume consisting of all of Vk, corres-
ponding to different values of k, are defined by
N i k J i k C ik vk
1
ik ik,avg ik dV ;
as V1 V2
V 1 V 2
 
C ik 1
 r  J i k  r  C ik vk Rik : 6:2:23a C ik C ik,avg C ik dV , 6:2:24b
t V1 V2
V 1 V 2
If we extract the species-specific migration velocity
where we have assumed that the system has only two
Uik-containing term out of the expression for (Ji)k (see
phases or regions, 1 and 2. If k 
is theX
volume
 fraction of
expression (3.1.98)), we obtain
phase k in the system, i.e. k V k = V k , then, from
C ik   the above definitions,
r  C ik vk r  C ik U ik  r  J i k Rik :
t ik ik,avg k ik,avgi k ik ;
6:2:23b
C ik C ik,avg k C ik,avgi k C ik : 6:2:24c
Note that the J i k term no longer has any contribution
from Uik. The two types of averages of any quantity are generally
Figure 6.2.3 illustrates two phases k 1,2 in the identified as follows. The phase average of in the k-phase
system. One can define two types of volume averages in is given by
such a system. When a quantity is averaged over the
1
volumes of the two phases (k 1,2) in the control volume, h k i X X k dV k,avg 6:2:25a
Vk V
k k
we have what is known as the phase average. When a k
quantity is averaged only over a specific phase or region,
and the intrinsic phase average of in the k-phase is
given by

Surface area A12(t ) 1
Phase k = 2 h k ik dV k,avg i : 6:2:25b
V k Vk

The phase average of, say, the differential equation


(6.2.23b) of transport of Cik is given by
Phase k = 1 * +
C ik
Phase hrC ik vk i hrC ik U ik i hrJ i k i hRik i:
k=2 t
V1 v int
k 6:2:26a

V2 A number of results from the general transport theorem and


nk the volume-averaging techniques identified in Gray (1975),
Soo (1989) and others identified earlier are useful here:
* +
k 1
h k i  k vint
k  n k dA; 6:2:26b
t t V
A12 t

Control volume 1
hr k i rh k i k n k dA; 6:2:26c
V
Figure 6.2.3. Configuration of a two-phase system. A12 t
366 Open separators: bulk flow parallel to force and CSTSs

C ik
1 k r  hvk iC ik r  hJ i k i  r  C ^ ik v^ k
hr  k i r  h k i k  n k dA: 6:2:26d t
V
A12 t 1
X J i k  nk dA: 6:2:30
Vk
These relations relate, for example, the average of the A12 t
k
derivative to the derivative of the average (e.g. (6.2.26c)).
Here, A12(t) is the area of the interface between phases 1 In this equation, the first term on the right-hand side
and 2, where k 1,2, vintk is the mass average velocity of
involves molecular diffusion. Usually, the second term on
this interface and nk is the outwardly directed unit normal the right-hand side is combined with the first term to
on the k-phase surface (Figure 6.2.3). Employing these represent the complex phenomenon of solute dispersion:
relations, one can rewrite the phase-averaged equation ^ ik v^ ik k Dik rC ik C
^ ik v^ k k D
hJ i k i C rC ik ,
(6.2.26a) following Gray (1975) as follows: i,eff , k

6:2:31
C ik k ^ ik v^ k r  C ik hU ik i
k C ik r  C ik hvk i r  C
t t where D is the dispersion tensor (Gray, 1975). The
i,eff ,k
second term on the left-hand side, r  hvk iC ik , is the
r  C ^ ik X1
^ ik U C ik vk U ik  vint k  nk dA
contribution of the convective fluid motion to the transport
k
V k of species i. For incompressible flow, this term is reduced
A12 t
to C ik r  hvk i hvk i  rC ik hvk i  rC ik , since r  hvk i is
1
r  hJ i k i  X J i k  nk dA k hRik ik : zero (see equation (6.2.5k)). The last term on the right-
Vk
A12 t hand side of (6.2.30) represents the rate of interphase
k
transport of species i across the phase interface A12(t); it
6:2:27
may be represented by an overall mass-transfer coefficient
This derivation employed a particular approach of Gray (1975) Kik multiplied by an appropriate surface area and concen-

wherein C^ ik , v^ ik and U
^ ik are defined via the relations tration difference: K ik aC ik  C ik . These simplifications
transform equation (6.2.30) as follows:
^ ik ;
C ik C ik C vk hvk ik v^ k ; U ik hU ik ik U
^ ik ,
6:2:28 C ik 
k hvk i  rC ik k r D rC ik  K ik aC ik  C ik :
t i,eff ,k
such that hC ^ ik ik , hC v k ik , h^
^ ik i, h^ ^ ik ik and hU
v k i, hU ^ ik i are 6:2:32
zero. This assumes that the averages of C ik , vk and U ik 
are well-behaved. Gray (1975) had assumed as such Here, C ik is a hypothetical concentration of i in the phase k
for C ik , vk ; we have added U ik to the list. which would be in equilibrium with the concentration of
We will now make a number of practical assumptions species i in the other phase of the interphase transport
valid for many two-phase separation systems and simplify system (see equation (3.4.6)); a is the interfacial area of
equation (6.2.27) by assuming that. transport per unit control/system volume. In a two-phase
system, where k 1, 2, the equation for C i1 will have a sign
(1) k is constant, i.e. (k/t) 0; for this term which will be opposite to that in the equation
(2) species i is not being produced by a chemical reaction, for C i2 since species i comes into one phase while it goes
i.e. Rik 0; out of the other phase.
(3) no phase change occurs, i.e. When the volume averaging primarily involves an
average over the flow cross section with dependent vari-
C ik vk  vint
k  nk dA 0:
ables and parameters11 varying only along the mean flow
A12 t direction along the separator axis, z, we can simplify the
equation (6.2.32) as follows:
These assumptions lead to
C ik C ik 2 C ik 
k hvkz i k Di, eff;k  K ik aC ik  C ik :
C ik 1 t z z2
k r  hvk iC ik r  hU ik iC ik X C ik U ik  nk dA
t Vk 6:2:33
k A12 t

An overall balance of species i in both phases k 1,2 in
^ ik v^ k  r  C
r  hJ i k i  r  C ^ ik  X1
^ ik U J i k  nk dA
V this context can be obtained by adding the individual
k k A12 t
equations for k 1 and 2 (note that 1 2 1):
6:2:29

For systems where U ik is zero and molecular diffusion and


partitioning between two phases is the primary mechanism
for separation, we can rewrite equation (6.2.29) as follows: 11
Di, eff , k is assumed not to vary in the z-direction.
6.2 Equations of change 367

C i1 C i2 C i1 C i2 ^ ik v^ k C
hJ i k i C ^ ik U ^ ik v^ k C
^ ik k Dik rC ik C ^ ik U
^ ik
1 2 hv1z i hv2z i k D rC ik :
t t z z i,eff ,k
2 C i1 2 C i2 6:2:39a
1 Di,eff , 1 2 Di,eff , 2 : 6:2:34
z 2 z2
The second term on the right-hand side of (6.2.38) repre-
If phase 1 happens to be solid and the stationary phase
sents interphase transport across the phase interface A12(t)
(as in a fixed-bed process, see Section 7.1), hv1z i 0 and
due to diffusion and any species-specific migration velocity
Di,eff ,1 0, and we obtain
U ik . The second term on the left-hand side of (6.2.38) will
C i1 C i2 C i2 2 C i2 be reduced to hvk i  rC ik in the case of incompressible
1  hv2z i Di,eff , 2 6:2:35
t t z z2 flow. The third term on the left-hand side may be
expressed as follows:
for a fixed-bed process with species i exchanging between
the fluid phase 2 and the solid and stationary phase 1, the r  hU ik iC ik C ik r  hU ik i hU ik i  rC ik : 6:2:39b
void volume of the bed being . An alternative form of this
equation is used more often: In the case of some external force, e.g. electrical force in a
uniform electrical field, r  hU ik i is zero.12 In other cases,
C i2 1  C i1 hv2z i C i2 2 C i2 it may not be zero if U ik varies with the location in the
Di,eff ,2 : 6:2:36
t t z z 2 separator.
Here, hv2z i= is the interstitial velocity of the fluid phase. In the context of these considerations, equation
In the case of two fluid phases (gasliquid, liquid (6.2.38) may be written for incompressible flow as
liquid, etc.) flowing simultaneously in the separation follows:
device, we can write down the equations for k 1 and 2 C ik
following equation (6.2.33): k C ik r  hU ik i fhvik i hU ik ig  rC ik
t

C i1 C i1 2 C i1  1
1 hv1z i 1 Di,eff , 1  K i1 aC i1  C i1 ; k D r2 C ik  X fC ik U ik J i k g  nk dA:
t z z2 i,eff ,k V
k k A12 t
6:2:37a
6:2:40
C i2 C i2 2 C i2 
2 hv2z i 2 Di,eff , 2  K i2 aC i2  C i2 : 6.2.1.2 Continuous stirred tank separator
t z z 2
6:2:37b In a continuous stirred tank separator (CSTS), fluid streams
More common forms of these two equations are: (single or multiphase, with or without solids) enter and
leave, and the contents are kept well stirred or well mixed.
C i1 hv1z i C i1 2 C i1 a  There is no spatial dependence of species concentrations
Di,eff , 1  K i1 C i1  C i1 ;
t 1 z z2 1 or any other quantity (such as temperature, pressure,
6:2:37c etc.) inside the separator vessel. The species concentra-
tions inside the vessel are at a steady state, sometimes
C i2 hv2z i C i2 2 C i2 a 
Di,eff , 2  K i2 C i2  C i2 : they may change with time; however, the outlet stream
t 2 z z 2 2
concentration is different from the inlet stream concen-
6:2:37d
tration, and is equal to that inside the CSTS (Figure
Generally, when both phases flow in a device, the separa- 6.2.4). Meanwhile, whatever separation mechanism is
tion takes place under steady state; therefore the unsteady employed is operative inside the vessel. (A batch stirred
state terms in these two equations will disappear. tank separator operates such that there is no spatial
If the species migration velocity U ik is nonzero, we will gradients of concentration, temperature or pressure
have to go back to equation (6.2.29) and rewrite it as inside the vessel. Except when the batch is introduced
or withdrawn from the vessel, no fluid/solid streams
C ik
k r  hvk iC ik r  hU ik iC ik enter or leave the vessel. The conditions inside the vessel
t
may change with time. Generally there is vigorous bulk
^ ik v^ k C
r  fhJ i k i C ^ ik U
^ ik g motion in the vessel.)
If the volume of a separator, which has, say, one or two
1
X fC ik U ik J i k g  nk dA: 6:2:38 feed streams coming in (j f or f1 and f2) and k product
V
k k A12 t streams leaving the separator, is V then, for species i, mass
balance in the separator, at any time t, is given by
We focus on the first term on the right-hand side of this
equation and, in the manner of definition (6.2.31), suggest
a dispersion tensor D such that 12
Here Uik ( imE) is independent of the spatial coordinate.
i,eff , k
368 Open separators: bulk flow parallel to force and CSTSs

by focusing on a particle, its coordinates as they change


with time, its velocity and acceleration and the forces it is
subjected to. Ignoring any random Brownian motion of
the particle, the inertial force F iner
p on a spherical particle
Cif, vf, Sf SM (of radius rp, mass mp and volume (mp/p)), subjected to
an external force F ext
tp and a drag force F p
drag
from the
surrounding fluid, satisfies the following basic equation of
particle motion, where the particle velocity is U p :
Cik
dU p
Cik, vk, Sk F iner
p mp F ext drag
tp  F p : 6:2:45
dt
Here, F ext
tp may be obtained from expression (3.1.59). Note
that we have neglected the following forces:
13
Figure 6.2.4. Continuous stirred tank separator (CSTS) with one the virtual mass force ;
14
incoming stream and one outgoing stream. the Basset force on the particle due to an unsteady flow
field (see Landau and Lifshitz (1959), Friedlander (1977)

dC i X and Soo (1989) for their expressions and physical basis);
V N if 1  dSf 1 N if 2  dSf 2  N ik  dSk N iM  dSM Ri V ,
dt
Sf 1 Sf 2
k
M
any force on the particle

created by an externally imposed

pressure gradient vp rP (note that expression (3.1.59)


6:2:41
includes any P created by centrifugal forces).
where we have assumed that there is a surface (SM ) in the
Now the particle velocity vector U p may be repre-
vessel (with or without a membrane M) through which
sented through the rate of change of its position vector in
species i is being introduced into the separator vessel in a
Cartesian coordinates as
manner quite different from that by the feed streams; there
could be more than one such surface. If, at the separator dx dy dz
Up i j k: 6:2:46
vessel inlets and outlets, diffusion can be neglected and the dt dt dt
averaged stream velocities and concentrations may be util- Therefore the governing equation of particle motion
ized, we obtain (6.2.45) may be reduced to the following equations for
dC i X three coordinates, x, y, z of particle motion:
V hvf 1 ihC if 1 iSf 1 hvf 2 ihC if 2 iSf 2  hvk ihC ik iSk
dt k d2 x
mp F ext drag
tpx  F px ; 6:2:47a
dt 2
N iM  dSM Ri V : 6:2:42
M d2 y
mp F ext drag
tpy  F py ; 6:2:47b
dt 2
Generally, a CSTS has one inlet, one outlet and one add-
itional surface that may act like a membrane. Then, by d2 z
mp F ext drag
tpz  F pz : 6:2:47c
definition of a CSTS, equation (6.2.42) is simplified to dt 2
Corresponding equations may be developed in other
dC i1
V hvf 1 ihC if 1 iSf 1  hv1 ihC i1 iS1 N iM  dSM R i C i1 V :
dt coordinate systems. Expressions for the drag force are
M
available in (3.1.63) for particles and flows which obey
6:2:43 Stokes law and in (3.1.64) for larger particles and/or
For steady state nonreactive systems having one mem- higher velocities.
brane surface, the relation is simplified to
6.2.3 General equation of change for
hvf 1 ihC if 1 iSf 1 N iM  dSM hv1 ihC i1 iS1 : 6:2:44 a particle population
M
Particle separation devices, aerosol/hydrosol separation in
6.2.2 Equation of motion of a particle in a fluid: granular media, crystallization and precipitation devices
trajectory equation generally involve a size-distributed particle population.
The problem of a general equation of motion of a particle
suspended in a flowing gas stream or liquid stream is
complex. For our purposes, we consider only a relatively 13

1=2mp d v  U p =dt.
dilute suspension with no particle-to-particle interaction. 14
May be neglected for spherical particles of unit density in air
Further, to start with, we employ a Lagrangian description (Friedlander, 1977).
6.2 Equations of change 369

The notion of a size-distributed particle population was xi, yi, zi, and the corresponding gradient operator as ri. We
introduced in Section 2.4 via a particle size density func- will assume from now on that all particles having a given
tion f (rp): the quantity f (rp)drp represents the fraction of size rp and located at the same x, y, z will also have the
particles in the size range of rp to rp drp in a unit fluid same U int pr p . This last assumption is equivalent to assuming
volume. It is also the probability of finding a particle having no growth rate dispersion, although it is known now that
a size in the size range rp to rp drp in a unit fluid volume. crystals may display different growth rates in crystallization
We are interested here in the actual number of par- even though they have the same size and exist in the same
ticles dN(rp) in the size range of rp to rp drp per unit environment (Randolph and Larson, 1988). For particles/
volume of the fluid. This quantity is related to a particle crystals whose growth etc., can be characterized by
number density function n(rp) (the population density change in one dimension only, we will use L or rp inter-
function, see Figure 2.4.3(b) and definition (2.4.2a)) by changeably; but rp will be preferred.15
We will now derive an equation of change for the
dNr p nr p dr p ; 6:2:48a
particle number density function n(rp) by developing a
rp particle population balance16 in a small control volume of
nr p dr p Nr min , r p , 6:2:48b dimensions x, y and z (Figure 6.2.1):
r min rate of rate at which rate at which net rate at
accumulation particles of  particles of which particles
where N(rmin, rp) is the number of particles between rmin of particles size r p come in size r p leave of size r p
and rp in a unit fluid volume. The dimensions of N(rp) are of size r p are generated:
number/L3, i.e. numbers per unit volume (L3); the dimen- 6:2:49
sions of n(rp) are numbers per length per unit volume, i.e.
We will determine the contribution of each of these catego-
number/L4. Instead of numbers of particles, we can also
ries now to the overall balance in the number of particles:
consider the mass of particles in the size range rp to rp
drp. The relationship between the corresponding probabil- rate of Nr p
xyz
ity density functions is illustrated in equation (2.4.2e). accumulation t
When particles coalesce or break up, the total mass, as well of particles 6:2:50a
nr p
as the volume, are usually conserved. In this context, the of size r p in the xyz dr p ;
volume element t
number of particles dN(Vp) in the particle volume range of
Vp to Vp dVp is related to the corresponding particle
number density function n(Vp) by
rate at which
dNV p nV p dV p : 6:2:48c particles of size
r p enter the volume  
The changes in the particle number density function n(rp) element through the U px dNr p x yz j
of particles of size rp with the space coordinates (x, y, z) face perpendicular  
and time t are due to the following: convection in the to the x  coordinate j
U px nr p x yz dr p ;
and having a corner
flowing fluid, having velocity v; diffusion in the flowing
coordinate x, y, z
fluid (if relevant); particle growth from the surrounding due to fluid flow
medium by molecular level processes; particle birth: an
6:2:50b
original particle of size rp undergoes breakage to produce
new particles of smaller rp or coalescence with another rate at which
particle to create a new particle of larger rp; particle death particles of size
via breakage or coalescence, resulting in the disappearance r p leave the volume
of the original particle of size rp. When the particles undergo element through the  
convection, the particle velocity Up may or may not be
face perpendicular U px nr p jxx yz drp ;
to the x  coordinate
different from the fluid velocity v varying with the coordin-
and having a corner
ate location in the separator, namely x, y, z and time t. coordinate x x, y, z
There is another kind of particle velocity, Upint, the due to fluid flow
internal particle velocity (Randolph and Larson, 1988), 6:2:50c
which describes the physical growth of the particle due to
molecular level processes going on between the particle
and the surrounding fluid; for example, crystal growth in a 15
If L is used instead of dp, then L 2rp.
given dimension. If we identify such growth through an 16
See Friedlander (1977), Randolph and Larson (1988) and
internal property coordinate, say crystal length rp, then the Ramkrishna (2000) and for more extensive treatments of
internal particle velocity, U int
pr p , in the rp direction is drp/dt. population balance related equations (initiated by Hulbert and
We will designate such internal coordinate directions as Katz (1964)).
370 Open separators: bulk flow parallel to force and CSTSs

    
rate17 at which nr p
particles of size xyz yz U px nr p  U px nr p
t x xx
r p undergoing    
  
physical growth at U int
px i nr p xz U py nr p  U py nr p
growth velocity of xyz dr p ; y yy
U int x i  
px i accumulate in
  
the control volume xy U pz nr p  U pz nr p
z zz
at an internal particle 2      3
coordinate of x i U int
px i nr p U int
py i nr p U int
pz i nr p
6:2:50d xyz4 5
x i yi zi
rate at which 2 #


particles of size  yz4Dp
nr p  Dp nr p
r p to r p dr p x x x xx
enter the volume nr p 2
Dp yz dr p ; 6:2:50e #
element face of x
x 4 nr p nr p
relation 6:2:50b  xz Dp  Dp
y y y yy
by diffusion=
2 #
Brownian motion

 xy 4Dp
nr p  Dp nr p
z z z zz
rate at which
particles of size xyz B  xyz De:

r p to r p dr p nr p 6:2:50i
leave the face of Dp yz dr p ; 6:2:50f
x Dividing by (xyz) and allowing the individual dimen-
relation 6:2:50c xx
by diffusion= sions x, y and z to shrink to zero in the limit, we get the
Brownian motion population balance equation (PBE) in cartesian coordinates:
     
nr p U px nr p U py nr p U pz nr p
rate at which   
particles of size t x y z
r p to r p dr p      
B xyz dr p ; 6:2:50g U int U int U int
are born per unit px i nr p py i nr p pzi nr p
  
volume in the x i yi z i
control volume 0 1 0 1
nr p A @ nr p A
@D p Dp
x x y y
rate at which 0 1
particles of size nr p A
r p to r p dr p @Dp B  De: 6:2:51a
De xyz dr p : 6:2:50h z z
disappear per unit
volume in the In vectorial notation, the population balance equation is
control volume


Here B represents the density function of new particles n r p


 

r  U p n r p ri  U int
p n rp r  Dp rn r p B  De:
t
which are born/created in the size range rp, rp drp; De
6:2:51b
represents the density function of existing particles which
disappear in the same size range. Additional terms due to Here, the ri operator corresponds to the three particle
external particle velocities in the y- and z-directions and growth coordinate directions xi, yi, zi, even though we are
internal particle velocities in the yi- and zi-directions identifying the particle by just one radial dimension, rp. In
should be considered along with the above terms to obtain most practical situations, one particle/crystal dimension is
from equation (6.2.49) the following equation: often sufficient. This equation is a very general equation
which describes the change in n(rp) with x, y, z, t as well as
xi, yi and zi.

 In processes involving, for example, aerosols in a
17
The expression U int p xi n r p =x i can be obtained by a
process analogous to that carried out in equations (6.2.50b,c),
gas stream, one can replace Up in the convective term
 
here in a control volume of dimension x, y, z: consider a r  U p nr p by the fluid velocity vector v, namely,
control volume of internal particle coordinate dimensions xi,  
yi, zi located in the internal particle coordinate space. r  vnr p . However, if there are external forces creating
6.2 Equations of change 371

a particle velocity Up, then there will be an additional term, The fluid velocity field v is quite important in convective
 
r  U p nr p : diffusion of particles occurring in filters and scrubbers
used for gas cleaning. Knowledge of v in the separator is
nr p       essential in predicting particle separation.
r  vnr p r  U p nr p ri  U int
p nr p
t Since there are no birth and death processes here, the

 only independent variables are x, y, z and t. Thus, for
r  Dp rn r p B  De: spherical particles, we may multiply each term in (6.2.54)
3
6:2:51c by p 4=3r p dr p to obtain the equation in terms of
the dependent variable, p 4=3r 3p dNr p , where p is
Such an equation has been identified as a general dynamic the mass density of the particle material. However, this
equation (Friedlander, 1977)) for n(rp), which, to be exact, quantity is simply the mass density of particles of size r
p
should be represented as n(rp, v, Upint, t); namely, it to r dr in the flowing phase j; we identify it as
p p
depends on particle size, fluid velocity, internal particle
p,rp whose governing equation is now given by
velocity and time. This equation does not include one
term, namely a diffusion term on the right-hand side, p, rp
2 v  rp, rp r  U p p, rp Dp r2 p, rp : 6:2:55
Di ri nr p , which arises from random fluctuations in crys- t
tal growth rate for which the diffusion coefficient is Di and
Note that the particle diffusion coefficient, Dp, actually
the coordinate dimensions are xi, yi and zi.
depends on rp.
The trajectory equations for the motion of a particle in
Section 6.2.2 did not include any random Brownian motion
of the particle. Both equations (6.2.51b) and (6.2.51c), on 6.2.3.2 Continuous stirred tank separator in particulate
the other hand, include a diffusion coefficient term on the systems
right-hand side for random Brownian motion of the collec- In a continuous stirred tank separator (CSTS) employed in
tion of particles. When the particle diameter is less than particulate systems, there are no spatial gradients inside
1 m, the effect of random Brownian motion becomes the separator: therefore we need to integrate either equa-
important. In the case of larger particles, the particle tion (6.2.51b) or equation (6.2.51c) over the total volume V
motion is generally deterministic and predictable of the separator. To that end, we may employ the type of
(Friedlander, 1977); equations (6.2.47ac) should be used. relations (6.2.4bd) utilized in volume-averaging proce-
However, turbulent flows may introduce random effects.  
dures. For example, the second term, r  U p nr p , on
the left-hand side of (6.2.51b) will lead to
6.2.3.1 Diffusional transport of particles in flowing
2 3
fluidsconvective diffusion    
V V
r  U p nr p dV r  4 U p nr p dV 5
For particle diameters less than 1 m, the diffusional trans- V V
V V
port of particles in a flowing fluid is identified as convective  
V
diffusion (Friedlander, 1977). The role of Brownian motion U p nr p  nk dA12 :
is considerable at smaller particle sizes. For cases where V
A12 t
there are no birth and death processes (B De 0) and
6:2:56
there is no particle growth/decay, equation (6.2.51c) may
be simplified as follows: The first term in this expression is zero since there are no
    spatial gradients inside the separator. In the second term,
nr p
r  vnr p r  U p nr p Dp r2 nr p : only the particle velocities normal to the surface A12(t)
t
matter; further, in this case, A12(t) represents one or more
6:2:52
interfaces at the system boundaries. For the system bound-
The second term on the left-hand side may be simplified aries with two feed streams (volumetric flow rates Qf1, Qf2)
for incompressible flow via coming in and two product streams (Q1, Q2) leaving (the
      outward-directed normal to A12(t) is positive, therefore Qf1,
r  vnr p nr p r  v v  r nr p v  r nr p : Qf2 are taken negative as coming into V),
6:2:53   h
U p nr p  nk dA12  Qf nf 1 r p Qf nf 2 r p
1 2
Therefore, the governing equation for convective diffusion A12 t
of particle density function n(rp) of size rp becomes i
Q1 n1 r p  Q2 n2 r p : 6:2:57
nr p    
v  r nr p r  U p nr p Dp r2 nr p :
t Although we have assumed the existence of two incoming
6:2:54 streams and two product streams, there may be only one of
372 Open separators: bulk flow parallel to force and CSTSs

  
each kind, as in a crystallizer. For the system boundary, nr p
r  U p nr p
where there is a free interface which can accumulate par- t
V
ticles due to a change in the system volume, 
  ri  U int 2
dV p nr p   Dp r nr p  B De dV 0:
U p nr p  nk dA12 hnr p i : 6:2:58
dt
A12 t 6:2:62

Here we have followed the approach of Randolph and As we have a batch vessel with no flows in and out, and
Larson (1988). since the diffusional term has no contributions at the
The diffusional term (first term on the right-hand side boundaries of the vessel, we end up with (in view of
of (6.2.51b)) has generally no contribution at the boundar- equation (6.2.61))
ies of the vessel. The contributions of the terms vis--vis  
nr p  
the internal coordinates, as well as B and De, may be easily G nr p  B De dV 0: 6:2:63
t r p
replaced by the corresponding volume-averaged values. V

Therefore integration of equation (6.2.51b) over the


Exchanging the differentiation and integration signs, we get
volume V of the CSTS leads to
2 3

  n r p dV G n r p dV De  BdV 0:
4nr p r U p nr p ri  U int nr p   Dp r2 nr p  B De5dV 0 t r p
p V V V
t
V 2 6:2:64
hnr p i 1
) V4  fQf 1 nf 1 r p Qf 2 nf 2 r p  Q1 n1 r p  Q2 n2 r p g
t V If there is no spatial dependence in the tank of n(rp)
# (well-stirred vessel), we get
hnr p i dV  i 

r  U int
p nr p  B De 0: 6:2:59


V dt
n rp V n r p GV De  BdV 0:
t r p
V
We rearrange this as follows:
6:2:65
hnr p i  i  dlnV
r U int
p nr p hnr p i 6.3 Bulk flow parallel to force direction
t dt
1 We begin now the study of separation concepts/devices/
BDe fQf 1 nf 1 r p Qf 2 nf 2 r p Q1 n1 r p Q2 n2 r p g:
V processes/techniques when there is bulk flow of one or more
phases. Specifically, in this section, we consider those separ-
6:2:60
ation configurations where the bulk flow of one phase is
This macroscopic particle population balance equation is parallel to the direction of the force driving the separation.
general and used most often. But the form in which the Generally the magnitude and direction of both the bulk flow
change with respect to the particle growth coordinate dir- as well as the force will be time-invariant: the direction of the
  bulk flow may or may not be opposite to the direction of the
ections is utilized is simpler than ri  U int p nr p . Usually,
force. In rare cases, the magnitude and direction of the bulk
only one particle growth coordinate18rp is employed flow and the force may oscillate with time. Our goal in this
(instead of the three xi, yi, zi): section is to illustrate important examples of how bulk flow
      parallel to the direction of the force achieves separation
dr p
ri  U int
p nr p U int
pr p nr p nr p G nr p ,
r p r p dt r p beyond that which can be achieved in a stagnant system, to
6:2:61 provide some details about the individual separation tech-
nique/process/device and to point out the limits of the mag-
where U int pr p , or G, is the linear crystal growth rate, drp/dt. nitudes of the bulk velocity.
In Section 6.3.1, we cover external forces, specifically
6.2.3.3 Batch stirred tank separator in particulate systems gravitational, electrical and centrifugal forces; inertial force
is also included here. In Section 6.3.2, chemical potential
For this case, we can start with the general population gradient driven equilibrium separation processes involving
balance equation (6.2.51b) integrated over the volume of vaporliquid, liquidliquid, solidmelt and solidvapor
the vessel: systems are considered; the processes are flash vaporiza-
tion, flash devolatilization, batch distillation, liquidliquid
extraction, zone melting, normal freezing and drying.
Section 6.3.3 illustrates a number of membrane separation
18
In the crystallization literature, L is used, so that the linear processes in the so-called dead-end filtration mode
crystal growth rate G (dL/dt); see Randolph and Larson (1988). achieved when the feed bulk flow is parallel to the
6.3 Bulk flow parallel to force direction 373


Bulk Anode Capillary Cathode Centrifugal
flow force Gas flow

Particle Liquid Liquid


flow flow Gas flow
Liquid velocity Bulk flow-based Particle (inertial force)
Gravity profile drag force direction
(a) Elutriation (b) Capillary (d) Inertial impaction:
electrophoresis (c) Centifugal elutriation
particle filtration

Heater
motion
Vapor Heater
Force

Feed Force Organic Force Direction of


phase
in Aqueous rejecting Molten motion of rod
impurity Molten rod
phase zone
Recrystallized
Rod being Heater solid
liquid out purified motion Heater
(e) Distillation (f) Liquid extraction (g) Zone melting (h) Normal freezing
(separating funnel)

Flow

Flow
Flow

Force

Force
Force

Force

Feed
Feed solution of Feed Feed
slurry proteins brine gas
Cake Membrane Membrane
Filter cloth
Filtrate Membrane Potable Permeated
Permeate
water gas

(i) Cake filtration ( j) Ultrafiltration (k) Reverse osmosis (l) Gas permeation

Figure 6.3.1. Separation systems where the bulk flow is parallel to the direction of force: (a) elutriation (particle separation in liquids);
(b) capillary electrophoresis; (c) centrifugal elutriation; (d) inertial impaction in particle filtration; (e) distillation (flash/batch);
(f) liquid extraction (separating funnel); (g) zone melting; (h) normal freezing; (i) cake filtration; (j) ultrafiltration separation of proteins
(dead-end); (k) batch cell reverse osmosis separation of brine; (l) gas permeation.

chemical potential gradient driven force perpendicular to the melt of the impurity rejected at the solidmelt inter-
the membrane. The processes considered are: cake filtra- face. Similarly, in dead-end membrane processes, we will
tion/microfiltration, ultrafiltration, reverse osmosis, perva- observe that separation is reduced unless the bulk flow
poration and gas permeation. Figure 6.3.1 summarizes velocity (whose direction and magnitude coincide with
different systems and representative configurations studied the force induced velocity through the membrane for
in this section where the bulk flow is parallel to the direc- incompressible fluids) is controlled to take into account
tion of the force. the velocity with which the species rejected at the
As mentioned at the end of the first paragraph, the membranefeed interface is transferred back to the bulk
magnitude and direction of the bulk velocity is important of the feed fluid.
in relation to the magnitude and direction of the species/
particle velocity due to the force under consideration. For
6.3.1 External forces
external forces, such as gravitational, electrical and centri-
fugal forces, we will find that the magnitude of the bulk We consider separations achieved due to external forces in
flow velocity is strongly influenced by the magnitudes of the following order: gravity; electrical force; centrifugal
the external force induced individual species/particle vel- force; inertial force (considered at the end).
ocities if one wants to fractionate a mixture of species/
particles. In equilibrium driven dispersive two-phase pro-
6.3.1.1 Gravity: elutriation, hydraulic jigging
cesses, the bulk flow velocity and the dispersed-phase
setting velocity are linked by the need to separate the two Consider the gravitational force acting on a spherical par-
phases successfully. In equilibrium driven solidmelt ticle of mass mp, radius rp, density p falling vertically
systems, the bulk flow velocity will be limited by the downward in a stagnant liquid of density t (< p). (All of
achievable magnitude of the species diffusion velocity in our considerations are also valid if the medium is gaseous
374 Open separators: bulk flow parallel to force and CSTSs

(a) (b) (c) Z


Z

Constant
vz cross vz vz
section Vessel
vessel
Smaller vz
rising vz > Upzt
particle
Upzt
Focused Focused
location location
Larger of of
Upzt > vz
falling vz smaller larger
particle Upzt particles particles
z

vz Liquid
Liquid velocity vz velocity vz

Figure 6.3.2. Gravitational separation of particles in a vertical column of liquid flowing vertically upward: (a) constant column cross
section; (b) column cross section increasing upward; (c) column cross section decreasing upward.

instead of water; further, the particle may be replaced by a down against the liquid motion at a velocity (Upzt vz).
drop.) We pointed out in Section 3.1.3.1 that the particle The particle whose terminal velocity Upzt is equal to the
velocity becomes constant very soon after a very brief upward fluid velocity vz is said to have the critical settling
period of initial acceleration; the constant velocity Upzt velocity U cpzt ; it will not go up or down. Particles whose
of the particle is called the terminal velocity (equation velocity Upzt is less than U cpzt will be carried up by the
(3.1.62)): liquid; particles whose Upzt is greater than U cpzt will fall
vertically downwards against the liquid velocity (Figure
d
U pzt F ext
pz =f p : 6.3.2(a)). If all particles have the same density p, then
the critical size rpc of the particle whose Upzt is U cpzt is
If Stokes law is valid, then f dp 6r  p . Since the  value of obtained from
F ext
pz for an isolated particle is m p g 1  t = p (equation  
(3.1.4)) in a fluid of density t, 2 !1=2
2 r pc p 1  t =p g 9vz
    vz U cpzt ; r pc
9 2p  t g
mp g 1  t =p 4=3r 3p p g 1  t =p
U pzt 6:3:2
6r p 6r p
  Particles whose radius is larger than rpc will fall down;
2
2 r p p 1  t =p smaller particles will be swept upwards by the liquid, gen-
g:
9 erating two particle fractions (Figure 6.3.2(a)). Particles of
size rpc will remain stationary at the location of their intro-
6:3:1
duction into the column.
Of the number of particle and fluid properties affecting If now the columnar vessel, in which the particle popu-
this terminal velocity, particle radius rp and density p are lation is being fractionated, does not have a uniform cross
particularly important. Particles will have different terminal section in the vertical direction, then the liquid velocity vz
velocities if their sizes are different (but densities are will change with the vertical location in the vessel. Corres-
same), or their densities are different (but sizes are equal) pondingly, the value of rpc will now depend on the vertical
or both are different. location in the vessel. Suppose the vessel cross section
To explore how such different vertically downward increases vertically upwards: vz decreases with increasing
particle terminal velocities may be utilized to separate z. Particles of a given size rp will now accumulate in a band
different particles in a mixture, let there be bulk flow of around a z-coordinate whose rp rpc corresponding to the
the liquid vertically upward at a velocity vz. The vertically vz at that cross section (Figure 6.3.2(b)). Particles having a
upward velocity of the particle will be (vz  Upzt) if given size are being focused to a given height in the vessel
vz > Upzt. If Upzt is larger than vz, the particle will go corresponding to a given vz (equation (6.3.2)); if this
6.3 Bulk flow parallel to force direction 375

particle were located at a lower height, vz at that location 2 < Rep < 500, intermediate region, C D 18:5=Rep 0:6 ;
will be larger than Upzt of this particle, which will drag 500 < Rep < 2  105 , Newtons law range, C D 0:44:
the particle up to its appropriate location. Similarly, if a 6:3:4
larger particle were located at a height appropriate for the
terminal velocity of a smaller particle, its Upzt will be larger The corresponding estimates of the particle terminal velo-
than the local vz, and therefore it will fall down and ultim- cities are obtained as follows.
ately become stabilized around its own vz (equation (a) Intermediate range
(6.3.2)). All situations described in Figure 6.3.2 illustrate
the technique of elutriation; the column of Figure 6.3.2(a) 4 3  
r g 1  t =p C D t U 2pzt =2r 2p ;
is called an elutriator. 3 p p
In Section 4.2, we observed the development of focus- 0:339r 1:14 0:71 0:71
g
8 p  t p p  t
ing in the presence of an external force, e.g. an electrical or U 2pzt r p g ) U pzt :
3 C D t 0:42 0:29
t
centrifugal force, etc. It required the creation of a property
gradient such as a pH or density gradient, etc., in the closed 6:3:5a
separator. The development of a liquid velocity gradient
parallel to the direction of the external force of gravity in an (b) Newtons law range
open separator appears to be equally capable of multi- 4 3
component separation. Whereas in Figure 6.3.2(b) the r g  t C D t U 2pzt =2r 2p ;
3 p p
column cross-sectional area increases vertically upwards, 0 1
leading to focusing of smaller particles at higher locations, 8g p  t r p gp  t
U 2pzt rp ) U pzt 2:46@ A:
an inverse column cross-sectional profile (i.e. the lower 3  0:44 t t
section has a larger flow cross-sectional area) will reverse
6:3:5b
the profile of the focused location of the smaller particles
(Figure 6.3.2(c)). The lower sections, having smaller liquid Using these expressions for U pzt for the intermediate
velocities, will stabilize particles of a smaller size. region and the Newtons law range, one can easily develop
If, instead of a system of particles of different sizes but expressions for the particle size corresponding to the crit-
having the same density, the system consisted of uniformly ical settling velocity, U cpzt vz , as well as the case of equal-
sized particles having different densities, a similar separ- settling particles. Considerable complexity, however, will
ation strategy may be adopted. If the system had variable be encountered in determining U pzt in concentrated
particle sizes and densities, separation into many fractions suspensions.
is still feasible. However, there is now a possibility that a There is an upper limit to the value of the upward
given terminal velocity may come about due to particular liquid velocity, beyond which there is no steady state sep-
combinations of rp and p: thus, at a given column height, aration in a system containing a distribution of particle
particles of different sizes may be located, but they will sizes and different densities. This value of vz should equal
have different densities. Such particles are called equal- or exceed the value of U pzt corresponding to r pmax and
settling particles: pmax , causing particles to be carried upwards by the liquid
   
2 2 stream.
2 r p1 p1 g 1  t =p1 2 r p2 p2 g 1  t =p2
U pzt jrp
1 9
; U pzt jrp
2 9
: The separation capabilities considered above are sig-
nificantly reduced in practice due to the actual velocity
6:3:3a
profiles in the vessels of Figure 6.3.2. Regardless of the flow
From these we obtain regime in the vessel, the liquid velocity near the wall is
  ! 1=2 much smaller than in the rest of the cross section at any
r p1 p2  t
, 6:3:3b height. The wall region will therefore accommodate much
r p2 p1  t smaller particles than the rest of the cross section. This will
considerably reduce the separation which could have been
a relation that is valid when Stokes law holds.
achieved otherwise.
For larger particles and/or higher velocities, leading
The general principle of elutriation ought to be
to a value of the particle Reynolds number Re
valid for a suspension of liquid drops in an immis-
2r p U pzt t = > 0:1, the drag force on a particle (relation
cible liquid. For a small spherical drop of radius
(3.1.64)) is described using a drag coefficient CD:
r p of a liquid of viscosity dr , the terminal velocity
F drag 2
pz C D U pzt =2Ap , 6:3:3c is obtained from the Rybczy nskiHadamard formula
(Levich, 1962) as
where AP r 2p .
Approximate estimates of CD for two ranges of higher 2 dr  t dr 2
U pzt r g: 6:3:6
particle Reynolds number are (Bird et al., 1960): 3 2 3dr p
376 Open separators: bulk flow parallel to force and CSTSs

(a) Piston (b) Piston


going going
down up

Screen Screen

Water
input
during
upstroke

Figure 6.3.3. Hydrualic jig operation schematic: (a) downstroke of piston; (b) upstroke of piston.

When the drop viscosity is very large compared to the containing a screen, through which the smallest particles
surrounding liquid viscosity (i.e. dr >> ), the above for- of the heavy material can pass; (3) a tapered-bottom tank
mula is reduced to (6.3.1) for a solid particle. This is valid which connects sections 1 and 2. The mixed particulate
in the viscous Stokes law regime. When the drop fluid suspension is introduced into section 2 on top of the
density dr < t , the drops will rise up to the top and form screen. During a very rapid downstroke of the piston
a separate fluid layer, if the drops can coalesce. In spray (Figure 6.3.3(a)), water/solution is rapidly brought up into
towers, employed in the separation process of liquidliquid section 2, which momentarily pushes the whole suspen-
extraction, the heavier phase is often dispersed as drops; sion up on top of the screen (transient bulk velocity in the
the lighter phase is the continuous phase. The lighter z-direction). Immediately afterwards, a rapid upstroke of
phase moves up and the heavier drop phase falls down the piston allows the particles to settle in the suspension
through the column. The physical separation involved here (Figure 6.3.3(b)) over the screen. The piston cycle time is
is not fractionation of drops of different size. It involves so small that the particles never have time to attain their
bulk separation of the two phases, the dispersed phase and terminal velocity. Gravity becomes the primary driving
the continuous phase, as molecular separation occurs force (equation (3.1.61)):
between the two phases via solvent extraction. !
The bulk liquid flow considered so far has been at d2 z dU pz p  t
mp 2 mp mp g: 6:3:7
steady state. The liquid was flowing either up or down dt dt p
the device at a constant rate. Further, the particles or drops
were assumed to be falling with a steady terminal velocity; The heavier particles fall much further than the lighter
the initial transient acceleration period was not considered particles.
relevant. Correspondingly, the d2 z=dt 2 term in equation Three layers of particles develop on top of the screen.
(3.1.61) was neglected. In the industrial operation of a The layer closest to the screen contains the largest particles
hydraulic jig, the initial transient is quite important. The of the denser material. The next layer has smaller
principle of operation of such a device (shown in Figure particles of the denser material and the largest particles
6.3.3) is as follows. of the less dense material. The top layer has the smallest
The device has three basic sections: (1) a chamber particles of the less dense material. The screen aperture is
open at the bottom on the top left side with a plunger chosen such that the smallest particles of the denser
which goes up or down rapidly (this part has attached to material fall through and are collected at the bottom of
it a source of water or a solution of interest which is the tank (section 3). All four fractions are rapidly removed
opened very briefly to introduce water during the upstroke physically from the tank bottom and from the top of the
of the piston); (2) a chamber on the top right side screen every so often.
6.3 Bulk flow parallel to force direction 377

The hydraulic jig is used to fractionate mineral/par- m d


i,eff Z i,eff F =f i : 6:3:8g
ticulate suspensions (especially coal suspensions) larger
than 1020 mesh (0.1650.083 cm), where the particle/ To separate a mixture of two ions i and j having different
mineral density is substantially larger than that of water. ionic mobilities m m
i 6 j and moving in one direction
The jig described above is of the fixed-screen type with (toward an electrode), the solvent may be made to flow at
water forced up through the screen (Perry and Green, a velocity vz countercurrent to the direction of movement
1984); hydraulic jigs can also be of the moving-screen type. of the two ions. The magnitude of the solvent velocity, vz,
The bulk liquid velocity, parallel to the direction of the should, however, be intermediate between the velocities of
force, is then introduced into the system by the moving the two ions. If m m
i > j , the slower-moving ion j will be
screen. flushed out in the direction of solvent flow, whereas the
faster-moving ion i will migrate against the solvent motion
toward the electrode. This technique has been identified
6.3.1.2 Electrical force: capillary electrophoresis
as countercurrent electrophoresis (CCEP) (Wagener et al.,
The separation of ions due to a uniform electrical field 1971):
applied to a liquid solution having a flow parallel to the
jU iz j jm
i Ej > jvz j; 6:3:9a
direction of the electrical field will be studied here. The
electrostatic force Fi on 1 gmol of a charged species i due to jU jz j jm
j Ej < jvz j: 6:3:9b
a uniform electrical field E ( r) is (see (3.1.8))
The exact physical configuration of such a separator has to
F i Qi E, 6:3:8a
satisfy a number of constraints. If a solution containing,
where Qi is the charge of 1 gmol of charged species (Qi say, a mixture of cations is to be separated by what is
Z iF , where F is Faradays constant of 96 485 coulomb/ known as the self-stabilizing CCEP, then the feed solution
gram equiv. and Zi is the algebraic valence of the molecular may be introduced at a particular location (A) in separating
ion). We may also use this relation for a particular ion column 6 (see Figure 6.3.4) as a salt solution. The separat-
instead of 1 gmol of a charged species: Qi is then the charge ing column, as shown, is a trough divided into compart-
on an ion which experiences a force Fi. Such an ion will ments by porous diaphragms. The flow rate of the
experience fluid friction as it moves toward an electrode electrolyte solution introduced into vessel 1 at a constant
having the opposite charge. Using Stokes law to estimate rate, the electric field strength applied between the two
the drag force experienced by the ion (assumed to be electrodes (platinum gauzes) at the two ends of column 6
spherical, diameter 2ri), the terminal velocity Ui (see and the solution temperature are preselected. This solution
(3.1.8) and (3.1.62)) of such an ion is to be obtained from passes through the separating column 6 into a collector
vessel 8 through an overflow weir 7 (Wagener et al., 1971).
F i Qi E 6r i U i ; 6:3:8b
If, for the different cationic species of interest,
Qi E QE m E > jvz j, then these cations will migrate toward the
Ui i , 6:3:8c i
6r i f di =N
~ cathode. However, different cations will have different
speeds, so the solution near the cathode will now have a
where (f di =N)
~ is the frictional coefficient experienced by

cation mixture composition different from that in the feed
one ion of species i. The quantity Qi = f di =N
~ is identified as
2
solution introduced at A. This assumes that we prevent
the ionic mobility mi (in units of (cm/s)/(volt/cm) or cm / deposition of the cations at the cathode via the use of
s-volt) of ionic species i (see an alternative definition
protective electrolytes. As time passes, the cathode solu-
(3.1.108j)):
tion will be progressively enriched in the faster cation.
Qi The applied voltage level employed is around 1000 volts.
m
i Z i F =f di : 6:3:8d
f di =N~ Such a process is necessarily a batch process. If, however,
a binary cationic mixture separation is desired using
Therefore jm m
i Ej > jvz j > jj Ej, then the process can be run continu-

U i m 6:3:8e ously (Wagener et al., 1971), with the slower species being
i E:
flushed out at the opposite end.
For an electrical field of magnitude E applied in the There are a number of general issues requiring atten-
z-direction, the migration velocity Uiz of the ith ionic tion in any such separator. First, the solvent flow velocity,
species in the z-direction is vz, has to be uniform over the flow channel. However,
through each hole in a diaphragm, there will be a parabolic
U iz m
i E: 6:3:8f
flow profile. Second, the high electrical field required to
If there is a diffuse double layer present around the generate practically useful ionic migration velocities leads
charged species (see Figure 3.1.2D), the effective charge, to considerable joule heating, which has to be dissipated
Zi,eff, will be less than Zi, leading to effectively. Otherwise, the solvent viscosity , and therefore
378 Open separators: bulk flow parallel to force and CSTSs

2 3

h
4 A

5
7

6
8

Figure 6.3.4. Construction of an electrolyte separation plant for the self-stabilizing CCEP process (schematic). Explanations are given in
the text. From Figure 2, p. 487 of Countercurrent Electrophoresis, K. Wagener, H.D. Freyer, B.A. Billal, Separ. Sci., 6(4), 483 (1971);
reprinted by permission of the publisher, Taylor & Francis Group.

the ionic mobility m i , will be affected, leading to consider- other topics, see the articles in Guzman (1993), specifically
able dispersion and a consequent reduction in the quality Karger and Foret (1993).
of separation. Conventional CE employs a fused-silica buffer-filled
Both issues have been successfully addressed in the capillary having an internal diameter between 20 and
widely used analytical technique of capillary electrophor- 200 m, a wall thickness between 50 and 150 m and a length
esis (CE) to be considered now. This technique,19 success- in the range 20200 cm. The two ends of the capillary are
fully introduced by Jorgenson and Lukacs (1981), and immersed in two buffer solution reservoirs containing two
variations thereof, has been widely adopted for analyzing platinum electrodes connected to a high-voltage power
small samples injected into a glass capillary subjected to an supply (Figure 6.3.5(a)). The applied voltage can be as high
electrical field along the capillary length, generally in the as 2530 kV; therefore the axial voltage gradient may easily be
presence of an electroosmotically driven bulk flow along as high as 400500 volts/cm. The current level varies between
the capillary length. Additional identifications of this tech- 20 and 100 A. The sample to be analyzed is injected near
nique are capillary zone electrophoresis (CZE) and high- the anode into the capillary, either by hydrodynamic flow
performance capillary electrophoresis (HPCE). via pressure or by electromigration. A detector is generally
We will consider this technique below. First, we will placed at the cathode end to detect the sample peaks
describe the basic technique. Next, we will introduce the traveling toward the cathode. Figure 6.3.5(b) illustrates
coupling of the ionic migration velocity and the electro- the separation achieved in one of the earliest investigations.
osmotic velocity in the capillary, in the presence of the We have already learned about the presence of an
electrical field. The migration of a sample pulse introduced electroosmotic flow (EOF) in a silica capillary filled with
at one end of the capillary will then be developed in an electrolytic solution and subjected to a voltage differ-
the capillary flow field parallel to the electrical force field. ence at the two ends (Section 6.1.5). This flow is such that
This will lead to expressions for resolution and other quan- the velocity profile, vEOF, is essentially flat, and the liquid
tities for two neighboring peaks. A number of subjects motion is toward the cathode (equation (6.1.22)). If the
will be considered briefly thereafter: partial ionization of silica capillary tube is open (and not a packed bed of very
a solute and its effective electrophoretic mobility; the role small particles), and the surface of the capillary has not
of a gel in the capillary; enhancement of throughput in CE; been coated in any fashion, capillary electrophoresis
separation of uncharged compounds by micellar electro- occurs in the presence of vEOF. Therefore, the net velocity
kinetic chromatography. For an introduction to these and of any ionic species i averaged over the capillary cross
section, hviz i, is given by

19
First attempted in a TeflonW tube by Mikkers et al. (1979). hviz i hvEOF, z i U iz , 6:3:10a
6.3 Bulk flow parallel to force direction 379

(a)
Sample

Detector
Capillary
High-voltage;
power supply

Inlet Outlet
buffer buffer
reservoir reservoir
(b)

E,F
H L

A,B C D

M
N

O
J K

0 5 10 15 20 25
Time (min)
(c)

mm
i E m EmOFE

m EmOFE

m m
i E m EmOFE

Figure 6.3.5. (a) Basic schematic for a capillary electrophoresis system. (b) Capillary electrophoresis separation of dansyl amino acids:
A, unknown impurity; B, -labeled lysine; C, dilabeled lysine; D, asparagine; E, isoleucine; F, methionine; G, serine; H, alanine; I, glycine;
J and K, unknown impurities; L, dilabeled cysteine; M, glutamic acid; N, aspartic acid; O, cysteic acid. The concentration of each
derivative is approximately 5  104 M, dissolved in operating buffer (Jorgenson and Lukacs, 1981). Reprinted, with permission, from
Anal. Chem., 53, 1298, (1981), Figure 1, p. 1300. Copyright (1981) American Chemical Society. (c) Effective velocities of cationic and
anionic species in the presence of electroosmotic flow in a capillary.
380 Open separators: bulk flow parallel to force and CSTSs

where hvEOF, z i is obtained by averaging the expression tV hvEOF, z i U iz


t
i ; z z=LT ; v
i,eff LT f di ,
for vEOF in (6.1.22) over the capillary cross section and L2T f di V
Uiz is the migration velocity defined by equation (6.3.8f). 6:3:12
However, in general, for cations moving toward the
cathode, the effective velocity of the cations will be to obtain

d E C i C i RT 2 C i
hviz i m
i E; 6:3:10b v
i,eff : 6:3:13
t
i z V z2

hviz i m m m m
EOF E i E EOF i E: 6:3:10c This equation is essentially identical to equation (3.2.10) if
we assume the following equalities:
Here, m
EOF is often called the electroosmotic mobility.
0 
On the other hand, anions having a tendency to move v
i,eff 1 ; V 1 max : 6:3:14a
toward the anode will have an effective velocity of
The solution of equation (6.3.13) using the new set of
hviz i m m m m
EOF E  i E EOF  i E: 6:3:10d independent variables

As long as m m z  v
i,eff t i and t
i , 6:3:14b
EOF > i in (6.3.10d), anionic species will
also move toward the cathode. Cationic species will, corresponding to equation (3.2.11), and a pulse input of mi
however, move much faster toward the cathode, at a moles of species i (3.2.14c) in the sample, can then be
speed higher than the bulk velocity arising from the obtained from the solution, (3.2.19):
electroosmotic flow. These behaviors are schematically " #
2
illustrated in Figure 6.3.5(c). Obviously, individual catio- mi 1 fz  v
i,eff t i g
C i z , t
i q
exp  t i :
nic species will move at different net velocities, hviz i. R2 LT 4RTt i 4 RT
V
V
The detector near the cathode will be detecting the
appearance of each such ionic species, cationic or 6:3:15
anionic, at different times. If the sample to be analyzed The center point of each such concentration profile for any
and injected near the anode into the capillary contains species i corresponds to the following value of z
i :
different species, one would like to know how well these
species will be separated, what would be the value of hvEOF, z i U iz
z
i vi,eff t i t: 6:3:16a
the resolution, what is the maximum number of species LT
that can be separated, etc. The nondimensional standard deviation zi for the
To provide approximate answers to these questions, Gaussian profile (6.3.15) is, correspondingly,
recall the analysis of separation development carried out in
( )1=2
Section 3.2.1, where a pulse of a solute mixture was intro- RT t 1=2
1=2
duced into the solvent at time t 0 at one end of the
zi f2RTt i =V g 2 f2Di,eff t=L2T g
f di L2T
separator liquid, which was stagnant and subjected to an
external force field in the z-direction, there being no vari- f2Di,eff tg1=2 zi
, 6:3:16b
ations in the x- and y-directions. The only differences here LT LT
are as follows: (1) there is a convective motion of the buffer where RT=f di may be defined as the effective dispersion/
solution in the capillary in the z-direction and (2) since we diffusion coefficient, Di,eff, for species i.
have a cylindrical capillary of radius R, we will assume The resolution Rs between two neighboring peaks for
no variations in the - and r-directions (instead of the cationic species 1 and 2 is defined by (see definitions
x- and y-directions). Consider the general equation (3.2.23) and (2.5.8))
(6.2.5a) for a molar concentration Ci of species i. Neglect
- and r-dependencies and assume v v hvEOF, z ik t R2  t R1 z  z
Rs 1 2 , 6:3:17
for the electroosmotic flow. Assume hvEOF, z i and Uiz to be 4 ti 4 zi
constant. We get
for an averaged value of . (Now, the peaks are detected in
C i C i C i RT 2 C i a CE at a z less than LT. We ignore this aspect and take z
hvEOF , z i U iz d ; 6:3:11a
t z z f i z2 LT for the detector location.) Consider the first expression
for Rs:
C i C i RT 2 C i
hvEOF , z i U iz d : 6:3:11b 2 3
t z f i z 2 z
4 1 LT 5 LT
t R1 ;
We nondimensionalize the independent variables in the hvEOF, z i U 1z hvEOF, z i U 1z
6:3:18a
following fashion, using the applied voltage V used in the LT
t R2 :
capillary of length LTjEj V =LT : hvEOF, z i U 2z
6.3 Bulk flow parallel to force direction 381

To determine an averaged value of the profile standard the value of Hi, the smaller the standard deviation of the
deviations in time units, consider an averaged value of species profile and the narrower the profile base.
the profile standard deviations in length unit zi: The Pclet number, Pei, for the solute dispersion may
also be defined as (see alternative definitions (6.2.21))
zi 2Di,eff t1=2 : 6:3:18b
2Lhviz i 2LhvEOF, z i U iz
Pei , 6:3:23a
Using an averaged velocity hvEOF, z i U iz between Di,eff Di,eff
species 1 and 2, an averaged
which leads to
zi
ti : 6:3:18c
hvEOF, z i U iz 2zi 4L2 =Pe, 6:3:23b

Therefore indicating that the larger the Pclet number, the smaller
the bandwidth and the lower the dispersion in the species
U 1z  U 2z LT hvEOF, z i U iz concentration profile migrating through the capillary.
Rs
4hvEOF, z i U 1z hvEOF, z i U 2z 2Di,eff t1=2 A second quantity, the plate number, N or Ni, can now be
U 1z  U 2z LT defined as follows:

4hvEOF, z i U iz 2Di,eff t1=2
LT L2T m mi ELT
N Ni 2 EOF : 6:3:24
m m
1  2 LT H i zi 2Di,eff
1=2
,
m
4EOF m
iz 2Di,eff t
To develop an order of magnitude estimate of Ni, consider
6:3:19 an electrophoresis situation for ovalbumin: let the total
migration distance be 30 cm in one hour. Assume that Di,eff
where we have employed expression (6.3.10c) relating the
is essentially Di (see Table 3.A.5: 7.76  107 cm2/s). We
species velocities to various ionic mobilities, and defined
can calculate:
an averaged net species velocity between species 1 and 2,
hvEOF, z i U iz , as the square root of the product of the 30  30
Ni 1:61  105 ;
individual net species velocities. The above expression for 2  7:76  107  3600
resolution then is related to different ionic mobilities in the zi 0:075 cm; H i 1:85  104 cm:
applied field, m m m
1 and 2 , electroosmotic mobility EOF , the
The larger the value of Ni, the better the separation cap-
capillary length, the effective dispersion coefficient Di,eff
ability of the device. Expression (6.3.19) for the resolution
and time (thereby voltage, etc.).
Rs may now be rewritten:
We will now introduce two other quantities frequently
used in the literature to analyze this separator (we will see m m
1  2
Rs N i 1=2 ; 6:3:25
in Section 7.1 that they are also used in analyzing chroma- m
4EOF, z m
iz
tographic and related processes, especially in analytical
and preparative chemistry): the plate height Hi or H and indicating that the larger the number of plates, the higher
the plate number Ni or N, which are related to the separ- the resolution. The peak capacity nmax, (see definition
ator length LT by (3.2.32)), for this system is

LT H i N i HN: 6:3:20 LT LT =4 LT =4
nmax 0 11=2
4 zi 2Di,eff t1=2
To determine the plate height, note that the nondimen- LT
@2Di,eff A
sional standard deviation
zi (since z was nondimensiona- vEOF, z U iz
lized using LT) of definition (6.3.16b) or (3.2.21) may be
0 11=2
written in dimensional form, for any time t/separator
LT A m 1=2 1=2
length L, as follows: @ m
EOF i E 6:3:26a
32Di,eff
2Di,eff L 2Di,eff L
2zi 2Di,eff t : 6:3:21 m 1=2
hvEOF, z i U iz m m
EOF i E V fm
EOF i g =32Di,eff 1=2 : 6:3:26b

Now, as L or t increases, 2zi increases: Note that the peak capacity, nmax, may now be related to
the plate number Ni by
2zi 2Di,eff
2zi H i L; Hi m : 6:3:22
L EOF m i E N i 16n2max : 6:3:27a

The smaller the value of Hi, the lower the extent of the Alternatively, nmax is expressed as
band broadening (see (3.2.23)). Over a certain separator
length L or time t from injection of the sample, the smaller nmax N i 1=2 =4: 6:3:27b
382 Open separators: bulk flow parallel to force and CSTSs

At infinite dilution, a few of these results may be simpli- have an ionic mobility, mA . The effective mobility of the
fied/reexpressed in the absence of any kind of dispersion, species related to the acid HA in an electrical field may
except that due to simple molecular diffusion: be defined as
RT RTm m m m
HA;eff A x A HA x HA

Di,eff D0i i
: 6:3:27c m 6:3:28a
f di ZiF A x A :


Ignoring electroosmotic flow, from (6.3.24), at 25  C, A more general relation between the effective ionic mobil-
ity of a substance A and all other species i which are
m
i ELT Z i F ZiV F
Ni ; derived from A is (Karger and Foret, 1993)
2RTmi 2RT
coulomb m m
A;eff i x i : 6:3:28b
Z i  96 485  V volt i
gequiv
Ni 19:49Z i V From equation (5.2.61a) for the dissociation of the acid,
cal joule
2  1:987  4:18  298 K C H C A w
gmol  K cal K d1 ;
C HAw
6:3:27d
log10 K d1 pK 1 log10 C H log10 C A w  log10 C HAw
(where, remember, 1 volt 1 joule/coulomb) (Giddings, C A w
1991). If Zi is anywhere between 1 and 10, and the pH log10
C HAw
applied voltage V in capillary electrophoresis is around C A w C A w C HAw
)10pHpK 1 ; 10pHpK 1 1
25 000 volt, the plate number can be very high. Further, C HAw C HAw
the peak capacity, nmax, can be enormous, as much as CA  w 10pHpK 1
) x A
(20 ZiV)1/2/4. CHAw C A w 10pHpK 1 1
In the capillary electrophoresis literature, the ratio of 10 pK 1
) x A pK 1 ; 6:3:29
the two ionic mobilities is sometimes identified as the 10 10pH
separation factor :
 
m 10pK 1
1
: 6:3:27e m m
HA;eff A pK 1 pH : 6:3:30a
m 10 10
2

The expression (6.3.25) for the resolution Rs may be rewrit- Karger and Foret (1993) have illustrated this behavior
ten using for ionic species 1 and 2 as graphically: we show it in Figure 6.3.6. When pH < (pK1 2),
m m
HA;eff e0:01A : the ionic mobility is essentially zero since the
 1
Rs   N 1=2
i , 6:3:27f ionization of the acid is negligible. When pH > (pK1 2),
m
4 EOF
m m m
HA;eff A since essentially almost all acid molecules are
2
ionized at a high pH. One-half of the acid molecules are
m m
where we have assumed m m
EOF, z iz 1z EOF, z . The ionized when pH pK1. A not-as-illustrative expression
values of may be close to 1, say 1.1 or 1.01, etc. It is still equivalent to (6.3.30a) is
possible to achieve reasonable resolution between two
species having similar ionic mobilities since the value of K d1 =C H
m m
HA;eff A : 6:3:30b
1=2
Ni, and therefore N i , is very high in capillary electrophor- 1 K d1 =C H
esis. A comprehensive theoretical analysis of the perform-
ance of a buffer-filled capillary electrophoresis with The effective mobility of a weak base is considered in
reference to such relations is available in Datta (1990). Problem 6.3.4. The ionic mobility of a few smaller species
The approach employs an electrokinetic dispersion coeffi- and their pK values (primarily pK1) are provided in
cient, which has contributions from the diffusion coeffi- Table 6.3.1. For a protein, the value of the solution pH in
cient of species i, and the different bulk velocities relation to its pI is important. Since the net charge (Zi) on a
(electroosmotic flow and Poiseuille flow if any) present in protein is positive at pH < pI (see Figure 4.2.5(c)),
the system. the direction of a proteins ionic mobility will correspond
Many solutes present in the sample to be analyzed may to that of a cation. When pH > pI, its mobility direction will
be only partially ionized at the pH of the buffered solution. be reversed.
Consider an acid HA undergoing partial ionization (see For compounds which are quite similar (e.g. enantio-
(5.2.59)) mers), inclusion complexes formed with cyclodextrin (CD)
molecules can lead to selective separation. Cyclodextrin
HA H A :
molecules are uncharged and are therefore moved by the
The neutral solute HA will not have any ionic mobility, electroosmotic flow at a velocity hvEOF, z i. Any racemic
m 
HA 0. Only the fraction present as the anion A will mixture which exists as ions in the solution may be
6.3 Bulk flow parallel to force direction 383

Table 6.3.1. Ionic mobilities of selected ionsa

m
i  10
5
mA
Ions, i cm2/V-s pK

Ammoniumb 76.2 m HA,eff


Hydrogenb 362
Lithiumb 40 mA
Octadecyl tributylammoniumb 17.2
Potassiumb 76 2
Sodiumb 53
Triethylammoniumb 35.2
TRIS+c 29.5 8.3
Acetate 42.4 4.74
Chloride 79.1 0 pH =pK1 pH
Hydroxyl 206
Phosphate 34.1 2.15
58.3 7.22 Figure 6.3.6. Dependence of the effective mobility of a weak acid
71.5 11.50 on pH. (After Karger and Foret (1993).)
ACESd 31.3 6.84
BESe 24 7.16
HEPESf 21.8 7.51
MESg 26.8 6.13 be spherical. Therefore its effective mobility, m
i,eff ,
a
(Giddings, 1991),
Data from Janini and Issaq (1993), Karger and Foret (1993) and Fu
and Lucy (1998). Qi,eff Z i,eff F Z i F 1=h
b
Cations; mi chosen positive.
m
i,eff
c
tris (hydroxylmethyl) aminomethane.
f di =N
~ N ~ 6r i N~ 6r 2i
d 6:3:32
N-2-acetamide-2-aminoethanesulfonic acid. 4r 2i F 1=h
e
N,N-bis (2-hydroxylethyl)-2-aminoethanesulfonic acid. / 6 f r i ,
f 6r 2i N~
N-2-hydroxyethylpiperazine-N-2-ethanesulfonic acid.
g
2-(N-morpholino) ethanesulfonic acid.
is no longer a function of the size of the particle. For colloidal
particles, ionic moblility is therefore not influenced much by
separated if one of the compounds preferentially forms an the size of the particle in free solution. Consequently, capillary
inclusion complex with the CD (see Figure 4.1.26) and has electrophoresis is no longer useful. The capillary has to have a
its charge sticking out of the hydrophobic CD cavity. The porous gel/matrix which will allow smaller colloids to pass
compound forming the inclusion complex will now move through with less resistance in the applied electrical field.
at a velocity different from that of the compound that does The profile development analysis of an injected sample
not. See Fanali (1993) for details of CD based facilitation of carried out earlier led to the concentration profile (6.3.15)
separation in CE. based on a number of assumptions, which included a
It has been already pointed out in expression (6.3.8g) constant Uiz and therefore a constant m i . Constancy of
that the diffuse double layer around an ion/charged par- mi along the capillary length ensures that the peak disper-
ticle will reduce the effective charge on an ion to Zi,eff from sion will not have any contribution from variations in m i .
the true charge Zi. The two are approximately related as Further, constancy of vEOF, z requires constancy of the zeta
follows (Newman, 1973; Wieme, 1975): potential of the silica surface, , which, in turn, requires
1=h constancy of pH. Therefore, the pH has to be constant and
Z i,eff =Z i , 6:3:31a uniform along the capillary, especially for acidic/basic
1=h r i
species, suggesting that the solution has to be buffered.
where (1/h) is a characteristic thickness of the double layer The buffer solution usually employed in CE/CZE is 0.01
(equivalent to the Debye length in equation (6.1.22)) 0.05 M phosphate (0.2 M solution of monobasic sodium
around the ion/charged particle of radius ri. From phosphate and 0.2 M solution of dibasic sodium phosphate
DebyeHckel theory, h is proportional to the square root in different ratios). For solutes that are bases, the same
of the solution ionic strength. Thus, for larger charged buffer is recommended at a low pH. For acidic solutes, a
particles (larger ri), at high ionic strengths, Zi,eff will be borate buffer is recommended at a high pH. Sometimes a
related to Zi by mixed phosphateborate buffer may be used.
Z i,eff =Z i 1=h=r i : 6:3:31b For the analysis of DNA samples, the capillary is filled
with a gel, generally of crosslinked polyacrylamide. Pro-
Now, Zi, for a large ion/charged particle, is proportional to teins and oligonucleotides have also been separated in
the surface area of the particle/ion, 4r 2i , if we assume it to such capillaries. The gel provides a sieving matrix, having
384 Open separators: bulk flow parallel to force and CSTSs

pores through which smaller biomacromolecules pass to migrate toward the positive electrode in the electric
much faster. The gel also eliminates electroosmotic flow field. However, it is observed that, at pH > 5, the value of
and substantially decreases the solute diffusion coefficient; vEOF,z toward the cathode is higher than the electrophor-
thus band broadening is reduced. In fact, for DNA sample etic migration velocity of the negatively charged micelles
analysis, CE employing a gel has become highly successful. toward the anode, vEPMC, z ; the net effective direction of
DNA sequencing is an area where gel-filled CE has been movement of the micelles is therefore toward the cathode:
critically important to develop the speed and selectivity
vMC, z vEOF, z  vEPMC, z , 6:3:33
needed to sequence the entire human genome (Zubritsky,
2002). To solve the problem of gel stability, fresh polymer where vMC, z is the net velocity of the negatively charged
solution was introduced for each sample analysis (Karger micelles toward the cathode.
and Foret, 1993) and polymerization carried out. After each Uncharged compounds in such an environment will
run, the polymer solution may be blown out and the capil- partition into the hydrophobic core of the charged micelles.
lary reloaded with fresh solution. A run may last for a Some will partition much more, the more polar compounds
maximum of 80 minutes. much less, as we have seen in the results illustrated in Table
The sample injection volume in conventional CE/CZE is 4.1.9. Therefore, the net velocity of each such compound
around 110 nanoliter. The sample amount obtained at the toward the cathode will be quite different. This is the basis of
exit in one run is often insufficient for spectral measure- micellar electrokinetic chromatography (MEKC), first
ment/structure determination. If a semipreparative scale of developed by Terabe et al. (1984). However, this technique
operation is employed, then the sample amount at the exit will be covered in Chapter 8, since the partitioning of an
would be significant. Since the sample-loading capacity is uncharged compound from a mobile phase into a micelle,
likely to be proportional to the capillary/column cross- both of which have a bulk motion perpendicular to the force
sectional area, use of larger capillaries up to 180 m diam- direction, causing the compound partitioning into the
eter (including those of rectangular cross section (Tsuda, micelle, is more appropriate for Section 8.2.2.1.
1993)), sometimes packed with octadecyl silica particles In CE using silica capillaries having a charged surface,
(Chen et al. (2001) employed capillaries with diameters oppositely charged solutes may undergo electrostatic

550 m containing 1.5 m octadecyl silica particles), will adsorption on the capillary surface. Biopolymers are espe-
allow larger sample injection volumes up to 1 microliter. cially susceptible to adsorption. The capillary surface is there-
Alternatively, a bundle of multiple capillaries (up to five) fore often coated with a neutral hydrophilic polymer (e.g.
may be introduced into one column to increase the methylcellulose) in a thin layer. A thick neutral coating can
sample-loading capacity (Tsuda, 1993). eliminate the electroosmotic flow. For these and related
This last concept has been extended to what is known issues, the reader should consult a number of relevant chap-
as capillary array electrophoresis (CAE) (Huang et al., 1992; ters in Guzman (1993). Additional details on the experimen-
Kheterpal and Mathies, 1999), wherein there may be as tally observed effects of wall adsorption of polycations in
many as 96 capillaries in parallel in instruments currently particular (Towns and Regnier, 1992), and the theoretically
available (Zubritsky, 2002). This increases the throughput predicted consequences of adsorption in general (Schure
drastically. Such devices were the workhorse of the human and Lenhoff, 1993), provide a useful perspective.
genome analysis. Scaling up to 1000 capillaries has also
been reported (Kheterpal and Mathies, 1999). Capillary
6.3.1.3 Centrifugal elutriation
array electrophoresis enables easy and parallel loading of
multiple samples as well as rapid and parallel separation In Section 6.3.1.1, we studied elutriation based separation of
and detection using appropriate detection techniques. a mixture of particles in the gravitational field in the presence
An uncharged compound, if present in the injected of a bulk flow of the liquid in the vertical direction. To
sample, will move in CE with the bulk liquid at the electro- separate a mixture of different particles (different sizes and/
osmotic velocity vEOF, z toward the cathode. Different or densities), the liquid flow in the vertical direction occurred
uncharged compounds would have the same velocity, in a vessel with changing flow cross-sectional area. Different
namely vEOF, z . On the other hand, if a charged phase could sized particles (or particles of different densities) found equi-
be created into which different uncharged compounds librium positions at different heights where the liquid vel-
would partition in a varying fashion, the situation is ocity changing with height equaled the critical terminal
changed. This is achieved in CE by having an appropriate velocity of the particle. Gravitational force is not strong
solution of an ionic surfactant, for example sodium dodecyl enough to fractionate microscopic particles, such as bacterial
sulfate (SDS), as the medium in CE. cells, etc., in the size range of 150 m. However, as we have
If the SDS concentration is higher than the critical seen earlier, in Section 4.2.1.2, centrifugal force in the radially
micelle concentration (CMC), then the excess surfactants outward direction can.
will form spherical micelles (see Section 4.1.8) with nega- In centrifugal elutriation, a mixture of particles is intro-
tively charged headgroups SO2 4 . These micelles will tend
duced into a rotating chamber through which the liquid
6.3 Bulk flow parallel to force direction 385

(a) (b)

Bulk liquid
flow direction
Centrifugal Centrifugal force
force direction
Liquid Liquid
q

2R
flow flow

2RL
r
rL
L Bulk flow based
Axis of drag force direction
centrifuge
rotation

Figure 6.3.7. (a) Schematic of a truncated cone as an elutriation chamber, with bulk flow toward the axis of rotation of the centrifuge.
(b) Mixture of cells separated according to their size in centrifugal elutriation. (After Figdor et al. (1998).)

flows in a direction opposite to that of the centrifugal force. the conical tube shown in Figure 6.3.7(a), where RL is the
The rotating chamber is essentially a conical tube with its largest radius of the conical tube, L is the distance of this
base near the center of rotation and its apex far away radius from the apex of the conical tube, rL is the radial
(Figure 6.3.7(a)). If the liquid in which the cells/particles distance of the apex of the conical tube from the center of
are suspended is made to flow in this conical tube in a rotation and r is the radial distance from the center of
direction from the apex to the base, then the liquid velocity rotation of the cross-sectional area of radius R under con-
increases as the distance r from the center of rotation sideration. The expression for A(r) is then
increases. Correspondingly, at steady state, the drag force  2
on a cell/particle (assumed Stokesian) of radius rp moving R2 tan  r L  r RL =L2 r L  r2 :
with the liquid velocity vr at radial location r is 6r p vr ; the 6:3:34c
magnitude of this force increases as r increases. This force
is, however, opposed by the  radially outward
 centrifugal Correspondingly, equation (6.3.34b) may be reduced to the
force on the particle, mp r2 1  t =p , which increases following expression for rp:
linearly with r. Under appropriate conditions, there will be s
a radial location r where the two forces will balance each 3 Qf
rp : 6:3:34d
other and the cell/particle of specific size/density will con- r L  rRL =L 2p  t r
centrate at the radial location:
  From this equation, we can infer that the smaller the particle/
6r p vr mp r2 1  t =p : 6:3:34a cell size rp, the closer to the center of rotation the equilibrium
location of the particles, as illustrated schematically in Figure
6.3.7(b). The withdrawal/separation of each size group of cell
The inwardly radial medium velocity vr is related to the population may be accomplished by either a stepwise
medium volume flow rate Qf and the local cross-sectional increasing of the flow velocity enough or a stepwise decreas-
area A(r), which depends on the radial location r. ing of the centrifugal rotational speed, so that the smallest
Replacing the cell mass mp by 4=3 r 3p p , we get particles in the conical rotating tube come out with the bulk
9 Qf liquid flow. As a practical design, there is a counter-taper at
r 2p : 6:3:34b this exit section (closest to the center of rotation), as shown by
2 2 p  t rAr
the dashed-dotted line in Figure 6.3.7(b).
This equation provides a relation between the radial loca- This concept of centrifugal elutriation was developed
tion and the dimension rp of the cell population (assuming first by Lindahl (1948, 1956). A recent review of the exten-
they are of the same density) which accumulates at r. sive literature in the biological field on various types of cell
(Generally, p  t in equation (6.3.34b) is very small; separation is available in Figdor et al. (1998). Convention-
therefore this technique is not used to separate particles ally, the rotating separation chamber volume useful for
of the same rp but differing only slightly in p .) The exact separation is around 34 milliliter. Larger separation
relation will depend on how A(r) varies with r. Consider chambers with volumes up to 40 milliliter have been
386 Open separators: bulk flow parallel to force and CSTSs

Feed Particles to be
Feed Particles filtered

Vessel
wall

Packed
bed of
particles/
Filter fibers
Membrane
cloth
Vessel Filtrate
wall Filtrate

Principle of surface Principle of depth


filtration using a filtration using a packed
membrane or filter cloth bed of particles / fibers

Figure 6.3.8. Principles of surface filtration and depth filtration for removing particles from a fluid.

developed. The highest rotational speed used is 5000 rpm. deposition and Brownian diffusion. Of these, only inertial
Each sample fraction that is obtained has a relatively large impaction involves bulk flow of the gas parallel to the
volume. The volume fraction sizes may vary from 25 to direction of the force on the particle and will be considered
150 milliliter (Figdor et al., 1998). Pumps of various types here. The rest of the mechanisms are mentioned in Section
are used to generate the flow, whose rate may be around 7.2.2 since bulk gas flow will generally be perpendicular to
1518 milliliter/min. the forces involved in the other mechanisms. The same is
true for the capture of hydrosols21 from an aqueous solu-
tion in a depth filter/granular filter, except additional
6.3.1.4 Inertial deposition of particles on a filter/collector
forces are involved.
in depth filtration
It is useful to illustrate briefly how the mechanism of
Small particles in air or water are conventionally removed by inertial impaction leads to the capture of the particles from
passing the air/gas/water through a filter. Particles in the size a gas stream onto filter elements. Figure 6.3.9A illustrates
range of 0.02 to 10 m are removed by microfiltration as well the streamlines of gas flow in the depth filter around a
as granular filtration. The latter is also called depth filtration. cylindrical fiber. Far away from the fiber, the gas velocity
Microfiltration is a general term which describes filtration is uniform: v. For small particles, the velocity is equal to
processes used to remove micron and submicron particles that of the fluid, and the particle flow path follows the
from air/gas/water/solvent. There are two basic mechan- streamline. However, near the fiber, the fluid streamline
isms: surface filtration and depth filtration (Figure 6.3.8). In is changed as the fluid goes around the fiber. Due to the
surface filtration, the fluid passes through the pores of a inertia of the particle, especially those of diameter larger
relatively thin membrane/filter/cloth, whereas the particles than 1m, the particle trajectory does not coincide with the
are retained on top of the membrane/filter/cloth whose fluid streamline near the fiber. The particle continues on a
pores/openings are smaller than the particles. This filtration path (dashed line in Figure 6.3.9A) which takes it straight
technique is covered in Section 6.3.3.1. to the fiber, and a collision takes place. If the adhesion
In depth filtration, taking place in a granular/porous/ forces between the particle and the fiber are strong enough
fibrous medium having a considerably larger thickness in for the particle to remain stuck to the fiber surface, the
the flow direction, the interstitial openings are usually particle is captured.
larger than the particles to be removed. The particles are This situation describes the fate of those particles pre-
carried into the interior of the filter by the flowing fluid; the sent in an envelope bounded by distant streamlines called
particles are deposited on the surface or collectors of the the limiting streamlines. The dimension of this envelope is
filter medium via a number of different mechanisms. of the order of the fiber diameter and has a width of 2b; the
For aerosols20 in a gas, the mechanisms are: inertial gas flow streamline at a distance b from the centerline is
impaction, interception, gravitational settling, electrostatic the limiting streamline, and the particle trajectory that

20 21
A dispersion of tiny particles of solid or liquid in a gas. A dispersion of tiny solid particles or liquid droplets in water.
6.3 Bulk flow parallel to force direction 387

y For spherical particles of size r p , density p and mass mp , if


Particle rp we nondimensionalize equation (6.3.36) using the
rp following nondimensional variables:
Gas streamline
Up tv v
U
p , t , v , 6:3:39
rf v v
vz = v 2b z where is a characteristic length of the filter collector, fiber
etc. (a particular value of for a fiber of diameter 2r f is
Gas streamline r f r p ), we get
!
Solid fiber 2p v r 2p dU p dU
p
Limiting particle trajectory St U
p  v : 6:3:40
9 dt dt

Figure 6.3.9A. Gas flow streamlines around a solid fiber of radius The dimensionless quantity St is called the Stokes number
rf in a filter bed and the trajectory of a particle of radius rp due to and represents the distance such a particle will travel,
inertial force near the fiber. starting with a velocity v , before stopping if the fluid is
stagnant, i.e. v 0. Alternatively, it is the ratio of twice
coincides with this streamline far away from the fiber is the kinetic energy of a particle moving at a velocity v and
called the limiting trajectory. Far away from the fiber, par- the work done against the drag force experienced by the
ticles present in the streamlines at distance greater than b particle moving at a velocity v through the fluid over the
from the centerline are not captured. characteristic distance b (Tien, 1989):
As the streamline changes near the cylindrical fiber in
the filter, the fluid accelerates since both axial and normal mp v2 4=3r 3p p v2 p 2r p 2 v
St : 6:3:41
fluid velocities change rapidly. Heavier aerosol particles in 6r p v 6r p v 18
air, larger than 1 m in size, are unable to follow the
If the particle size is of the order of the mean free path of
accelerating gas; the effect increases as the free stream
the gas, there may be slip at the particle surface between
velocity v and the particle mass increases. In the absence
the fluid and the particle. To account for this effect, St is
of any external forces, equation (6.2.45) may be written as
multiplied by a quantity C C , called the Cunningham cor-
dU p rection factor. For particles of 1 m diameter at normal
F iner
p mp F drag
p : 6:3:35
dt temperature and pressure, C C is around 1.16 (Tien, 1989).
Any solution of equation (6.3.36) or (6.3.37) requires
Depending on the Reynolds number, Re 2r p v = (based
detailed information about the flow field around the filter
on a fixed particle), the drag force may or may not be
bed collector/fiber. The flow field may be available via
described by Stokes law. When Stokes law (3.1.63) is valid
the three velocities vx , vy , vz (or vr , v , vz , etc.) or via the
(Re << 1)22,
stream function , if it can be assumed that the particle
dU p motion does not affect the flow field. The solution of such a
mp 6r p U p  v: 6:3:36
dt problem generally requires a numerical solution of the
governing equations (e.g. equation (6.2.6b)) for the chosen
When Re
1, the equation for particle motion may be
velocity field around the fiber in the depth filter.
written as (Friedlander, 1977)
The quantity of interest from a utilitarian point of view is
dU p C D Re the extent of removal of particles from the air stream in the
mp  6r p U p  v: 6:3:37
dt 24 depth filter. A predictive approach to this usually involves a
where C D is the drag coefficient for the particle (see number of steps. The first step is to calculate the particle
(3.1.64)) and may be described by capture efficiency of a single fiber element in a filter via a
given mechanism, say E IS due to inertial impaction. In the
24 second step, add up appropriately the corresponding
CD 1 0:158Re2=3 6:3:38
Re single-fiber capture efficiencies due to the different mech-
as long as Re < 1000. Obviously, C D is a function of the anisms to obtain E TS , the total efficiency for a single fiber,
local Reynolds number of the particle: and apply the result to the whole filter bed/depth filter.
An alternative approach calculates the particle capture
Re 2r p jU p  vj=: 6:3:38
efficiency of a single fiber element due to the simultaneous
action of different capture mechanisms to obtain E TS and
then applies such a result to the whole bed. We will con-
sider these approaches in Section 7.2.2 along with the other
22
The term 6r p is often corrected by a correction factor, for capture mechanisms primarily for hydrosol removal by
which see equation (6.3.41) and the associated text. granular filters.
388 Open separators: bulk flow parallel to force and CSTSs

L Theextentof change in the particle number concentration,


v d nr p dr p jz , as a result of this in the gas flowing into
the inlet of the filter at a velocity v   1 is given by
Cleaned gas     2b1  dz

v  d nr p dr p jz 2 v  nr p dr p :
r f z
Gas flow
z 6:3:42c
into the dz Fibers in the filter bed
fiber filter
However, if we use the single-fiber efficiency E IS due to
inertial impaction, then, from definition (6.3.42a),
Figure 6.3.9B. Fibrous filter bed of length L containing a collection
2b E IS 2r f :
of single fibers of radius rf used to clean a flowing gas.
dnr p =nr p jz 2E IS 1  =r f dz: 6:3:42d
We will employ here the definition of the inertial Integrate from z 0 to z L to obtain
impaction based capture efficiency E IS of a single fiber of
radius r f in the context of the definition (2.4.4a) of total nr p jz0 2E IS 1  nr p jz0
ln L)
efficiency E T of the depth filter: nr p jzL r f nr p jzL
 
2E IS 1 
2b exp L :
E IS : 6:3:42a r f
2r f
Integrating from rp,min to rp,max, we get
Here, if the fiber of radius23 r f is perpendicular to the gas
flow, then 2b is the width of the region of gas flow (see r p,max

Figure 6.3.9A) which is cleaned completely of any particles nr p jz0 dr p Nr p, min , r p, max z0
by the single fiber. An essentially identical definition may be r p, min
employed when the filter bed consists of granular particles  
2E IS 1 
of radius r f ; in that case, 2b will be the diameter of a Nr p, min , r p, max jzL exp L :
r f
cylindrical tube of contaminated gas, which will be cleaned
of dust particles by the spherical collector in the filter bed. 6:3:43
The value of b is obtained from the solution to the governing Therefore,
equation (6.3.40) and the gas velocity profile. The stream-
line corresponding to b is the limiting trajectory. r f Nr p, min , r p, max jz0
E IS ln : 6:3:44
Consider a depth filter containing a regular array of 21  L Nr p, min , r p, max jzL
cylindrical fibers (of diameter 2r f ) which occupy a volume
An expression of the overall filter efficiency due to inertial
fraction (1  ) of the fiber bed (Figure 6.3.9B). Generally this
impaction may also be developed as follows:
volume fraction is of the order of 0.1. Assume the number
concentration of particles of diameter 2r p to 2r p dr p at a  
Nr pmin , r pmax jzL 2E IS 1 
distance z from the inlet of a depth filter of length L to be ET 1  1  exp  L :
Nr pmin , r pmax jz0 r f
dNr p nr p dr p . Use the single-fiber capture efficiency
E IS defined by (6.3.42a) and focus on a bed length dz 6:3:45
(Figure 6.3.9B) in the gas flow direction. The number of fibers
We will very briefly illustrate how E IS may be estimated so
per unit width of the bed in this bed length is 1  dz=r 2f .
that E T may be calculated. The key to E IS is an estimate of
Since each fiber cleans up a region of width 2b of the gas flow
b, the distance of the limiting particle trajectory from the
(Figure 6.3.9B), the total width of the gas flow region cleared
centerline. An exact answer will require a solution of equa-
of particles is 2b1  dz=r 2f . The number of particles
tion (6.3.36) using appropriate initial conditions to deter-
of size r p removed as a result per unit time is the total width 
mine the particle trajectories. Write equation (6.3.36) in the
fluid approach velocity  particle number concentration at
manner of equations (6.2.47b) and (6.2.47c) in Cartesian
this location:
coordinates y and z, assuming no variation in the
2b1  dz x-direction and that U p is described by equation (6.2.46):
 v  nr p jz dr p : 6:3:42b
r 2f

d2 y dy
a vy  ; 6:3:46a
dt 2 dt
23
For other collector (fiber) geometries, instead of 2rf use the
mass flow rate of particles in the projected cross-sectional area of
the collector, whereas, for 2b, employ the actual mass rate of d2 z dz
a vz  : 6:3:46b
particles captured. dt 2 dt
6.3 Bulk flow parallel to force direction 389

 
Here a mp =6r p p 2r p 2 =18 ; vy and vz are the
y- and z-components of the gas velocity field (see Figure 1.0
6.3.9A). Particles which are far away from the fiber will
essentially follow the gas streamline, namely the vz -
component of the gas velocity ( v ) and the vy -compon- 0.1
ent of the gas velocity ( 0). To calculate the number of EIS e = 0.9
particles captured by the filter, we need to focus on that
particle trajectory coming from far away (where vz v 0.01 e = 0.99
and vy 0) whose y-value far away from the particle is
equal to b such that its trajectory is only at a distance of r p e = 0.999
from the fiber surface at z 0 (fiber centerline). Then all
0.001
other particle trajectories far away, whose y-values are less 0.01 0.1 1 10 100 1000
St
than b, will hit the fiber and be captured. So the initial
conditions of the particle trajectories far away from the Figure 6.3.9C. Capture efficiency for inertial impaction for a cylin-
fiber for the solution of equations (6.3.46a,b) are as follows: drical fiber of radius rf placed transverse to the gas flow as a
dx function of Stokes number for rp/rf = 0.01. (After Flagan and
at zt 0, U px v ; 6:3:46c Seinfeld (1988).)
dt
dx
at t 0, y b, U py 0: 6:3:46d
dt
particles automatically. Second, results in this figure suggest
To obtain a solution which provides a value of the
that, as the Stokes number increases, the particle capture
unknown b, one needs to know the gas velocity field, vy
efficiency becomes 1, regardless of how few fibers there are
and vz , as a function of y and z in the filter bed. The filter
in the bed. Further, the larger the fiber packing fraction in the
bed is full of fibers so the velocity field is complex. The
bed, the higher the capture efficiency for a given St.
usual approach is to adopt a cell model (Happel, 1959),
which reduces the problem to finding the gas flow field
around a single fiber in a specified volume, such that the 6.3.1.5 Electrostatic separation of fine particles in a
ratio of the void volume around the fiber to the total dry state
volume of the fiber plus the void volume (i.e. the specified
We consider here an example where two external forces are
volume) equals the void volume fraction of the fiber bed;
acting parallel to the direction of the bulk velocity. Mixtures
there are other conditions imposed on the boundaries of
of very fine particles to be separated are often in a dry state;
the specified volume which reflect the presence of the
this is especially true in the processing of various minerals
surrounding fibers in an appropriate way. Generally, a
(e.g. coals). When two particles come into contact, electric
complete numerical solution is needed for the set of equa-
charges develop via friction; the charge that remains on the
tions and the initial conditions. Using the cell model of
particles after separation of solid-to-solid contacts is called
Kuwabara (1959) for the velocity field around a fiber,
triboelectrification (Inculet, 1984). A fluidized bed of par-
Flagan and Seinfeld (1988) have provided a semianalytical
ticles is often a convenient method of achieving triboelec-
solution to estimate b. The basic approach (Flagan and
trification. One can also predict the polarity of the charge
Seinfeld, 1988) involves developing a solution of the par-
developed, but not necessarily its magnitude.
ticle trajectory coordinates y and z as a function of t and
Consider now a porous horizontal stainless steel plate
various parameters for an appropriate upstream distance
in a vertical vessel maintained at a high positive voltage (as
z where the fluid velocity vz v . Then, for the value of t
much as 30 kV) (Figure 6.3.10). Let air flow up through this
which brings the fluid streamline to z 0 (namely, the
porous plate and fluidize the bed of particles over this
fiber centerline), determine the value of y of the particle
porous charged plate. Let there also be a set of metallic
trajectory, where y r p r f so that particle capture
collection troughs at the top of the vertical vessel main-
occurs. The value of b is obtained from this expression,
tained at ground voltage. Thus there is an electrical field
y r p r f . Figure 6.3.9C illustrates how the value of E IS
directed vertically upwards. Any particle that develops a
(defined by equation (6.3.42a)) varies with the Stokes
positive charge via triboelectrification24 will be subject to a
number St for a given value of r p =r f , with the void
vertically upward electrostatic force; the particle will also
volume fraction as a parameter.
be subjected to gravitational force acting vertically
Two items are of importance regarding this analysis and
Figure 6.3.9C. First, smaller particles which may follow the
gas streamlines, and which therefore come within r p of the 24
There will be additional charge development via conductive
surface of the fiber, will be captured by the mechanism of induction when the particle contacts the stainless steel plate with
interception. The results shown in Figure 6.3.9C include these a positive charge.
390 Open separators: bulk flow parallel to force and CSTSs

Collection troughs

Conveyor
+ + Mineral positively
+ charged
+
Gangue
Gravitational Electrical
+
force field
+
Fluidized bed

+V Porous stainless
steel plate

Air flow

Figure 6.3.10. Separation of minerals from gangue material in an electrical field in the vertical direction after triboelectrification in an
air-fluidized bed. (After Inculet (1984).)

downwards, as well as to a drag force. The vertical direc- those bulk flow configurations where the bulk flows of the
tion (up or down) of the drag force exerted by the fluidizing vapor and/or the liquid are parallel to the direction of
air will be dependent on the relative velocity of the particle the chemical potential driving force between the vapor
with respect to the air. If the particles go up, they will be and the liquid. Two modes of feed liquid introduction are
collected in the trough; the electrical field strength may be studied: continuous liquid feed and batch liquid feed. The
as high as 200300 kV/m. The particle sizes treated may case of a continuous liquid feed is identified as flash vapor-
vary between 10 and 250 m. Examples include the separ- ization (Figure 6.3.11(a)). When the liquid is polymeric and
ation of Fe2O3 from particles of gangue, etc. the vapors generated are monomers and volatile solvents
used in the polymerization process, the device is called a
flash devolatilizer. A particular form is shown in Figure
6.3.2 Chemical potential gradient driven
6.3.11b. It is akin to vapor stripping from a liquid.
phase-equilibrium systems
In flash vaporization, the liquid feed mixture is heated
We consider here the role of bulk flow parallel to the under pressure as it flows through a heater into a pressure-
direction of the chemical potential gradient based force reducing valve, which opens into a flash drum (Figure
in phase-equilibrium based open two-phase systems. 6.3.11(a)). The vapor phase formed goes up the flash drum,
Vaporliquid systems of flash vaporization, flash devolati- and the remaining liquid jet and droplets go down into the
lization and batch distillation are considered first, followed liquid layer. The vapor phase is withdrawn at a molar flow
by a liquidliquid system for extraction. Solidliquid rate of Wtv, and the liquid phase is withdrawn at a molar
systems for zone melting and normal freezing are studied flow rate of Wtl from an incoming feed molar rate of Wtf. It
thereafter to explore how bulk flow parallel to the force is clear that the net velocity of the vapor phase formed is
direction is essential to considerable purification of solid up and that the net velocities of the bulk liquid and the
systems followed by solidvapor systems as in drying. liquid droplets are down. At the surface of the liquid layer
in the drum, the direction of the chemical potential driving
force is perpendicular to the horizontal liquid surface and
6.3.2.1 Vaporliquid systems: flash vaporization,
parallel to the direction of the net movement of the two
devolatilization and batch distillation
bulk phases, the vapor phase and the liquid jet and drop-
Distillation based separation of a liquid mixture exploiting lets. Although many of the droplets may have a complex
the inherent separation achieved in a closed vessel due to flow pattern, one could similarly argue that, as they fall
vaporliquid equilibrium (Section 4.1.2) is implemented in down, the multiple droplets essentially provide a liquid
an open separator vessel in a variety of ways. The vari- surface from which molecules escape to the vapor phase
ations are primarily due to the nature and origin of the vertically upwards.
bulk flows of the liquid and the vapor streams, and to the In reality, the design of the flash drum has to be such
manner of feed introduction. Here we will consider only that the upward vapor velocity is low enough not to entrain
6.3 Bulk flow parallel to force direction 391

(a) (b)
Vapor product Wtv, xiv Feed
Heater Vapor

Vapor flow
Steam heater
Wtf, xif Force Flash tank
direction
Feed liquid 4-30 Torr
Valve Flash drum
Liquid flow
230 C

230 C
Wtl, xil Polystyrene
Liquid product product

Figure 6.3.11. (a) Continuous flash vaporization. (b) Continuous flash tank devolatilization.

s
the liquid droplets (see equation (6.3.2)). Therefore, the 8gr p
upward vapor velocity vvz must be lower than the vertically K
3C D v
downward terminal velocity Upzt of any liquid droplet:
is empirically estimated. The units of K determine those of
vvz < U pzt : 6:3:47a
Upzt. Svrcek and Monnery (1993) provide estimates of K for
The terminal drop velocity may be obtained by balancing a variety of conditions. The radius of the flash drum, R,
the downward gravitational force on the drop of mass mp, should be somewhat larger than that based on the vapor
diameter 2rp and density p, (mpg(1  v/p)), against the volumetric flow rate Qv ( Wtv/v) and vvz:
drag force CD(rp2)(vUpzt2/2) exerted by the vapor of dens-  1=2
ity v moving up at the terminal velocity, Upzt: Qv
R : 6:3:50
! vvz
r 2p
mp g 1  v C D U2 ; 6:3:47b The above considerations highlight out the practical limits on
p 2 v pzt
the vapor-phase velocity in flash vaporization. Too high a
v
 vapor-phase velocity will considerably reduce separation via
u 
u8g  r p
t p v entrainment of drops. Assuming that there is no drop
U pzt : 6:3:48
3C D v entrainment, one would like to know the maximum separ-
ation possible in a flash separator. This will be achieved when
Here, we have used formula (3.1.61), where the drag force the vapor phase and the liquid phase exiting the separator are
expression used was given in (3.1.64); the value of the assumed to be in equilibrium. For molar flow rates of feed,
projected area Ap of the particle is (rp2) (Bird et al., vapor stream and the liquid stream being, respectively, Wtf,
1960); mp, the particle mass, is (4rp3/3)p; the drag coeffi- Wtv and Wtl, a total molar balance and an ith species balance
cient CD (see equation (6.3.4)) may be obtained from Bird (for a multicomponent system of n species) lead to
et al. (1960). In practice, the vertical vapor velocity in the
flash drum, vvz, is kept in the range 0.75Upzt < vvz < Upzt, W tf W tv W tl ; 6:3:51
where Upzt is the terminal drop velocity. Further, expres-
x if W tf x iv W tv x il W tl , i 1, 2, . . ., n: 6:3:52
sion (6.3.48) for the terminal drop velocity is written as
follows: Since the two product streams are in equilibrium,
v

u x iv
u 
t p v
Ki , i 1, 2, . . ., n: 6:3:53
x il
U pzt K , 6:3:49
v
Substituting for Wtv in terms of Wtf and Wtl and xiv in terms
where the value of Kixil, we get, from (6.3.52), an expression for xil:
392 Open separators: bulk flow parallel to force and CSTSs

x if x if K 1 1  K 2 K 2 K 1  1
x il   x 1v ; x 2v : 6:3:60
W tl
K i 1  W tf W tl 1  K i K1  K2 K1  K2
W tf
 
x if 1 W tv Further, from equations (6.3.55) or (6.3.54), we get
W tl
, i 1, . . ., n, 6:3:54
1 Ki W tv K 1 1  K 2 x 1f
W tl x 1v  : 6:3:61
K1  K2 W tv 1 W tv
where is the stage cut for this single-entry separator if j W tf K 1 1  W tf

1 v and j 2 l ( Wtv/Wtf). The corresponding


We can rearrange this relation to obtain (Lockhart and
expression for xiv in terms of xif is
McHenry, 1958)
x if x if  
x iv   1 W tv K 1  1 x 1f K 1  K 2  1 K 2
W tv
1 W tv
1  W tf Ki ,
W tf Ki W tf K1 K 1 1  K 2
 
W tl
x if 1 W tv which leads to
, i 1, 2, . . ., n: 6:3:55
1 KW tl
i W tv
W tv x 1f x 2f
 : 6:3:62
W tf 1  K2 K1  1
In a multicomponent system of n components (i 1,. . .,
k, . . ., n), to obtain the compositions of the vapor phase Thus, x1l, x1v and (Wtv/Wtf) can be explicitly determined,
and the liquid phase leaving the separator under equi- since, given P and T, K1 and K2 should be known.
librium, along with the total molar flow rates of the For multicomponent systems, an iterative procedure for
liquid product and the vapor product from the flash isothermal flash calculation proceeds along the following
drum for a given feed condition, will require the solution lines for given P and T. There are two possible cases: (a) Ki is
of the appropriate governing equations. For a system of essentially independent of the xivs and xils; (b) Ki depends on
n components, there are n equations (6.3.53) describing the xivs and xils. When Ki is independent of the xivs and xils at
vaporliquid equilibrium, n equations (6.3.54) or a given P and T, then, for given xif, the unknowns are the xils,
(6.3.55), describing xil in terms of xif or xiv in terms of the xivs and (Wtl/Wtf) or (Wtv/Wtf). The solution procedure is
xif, and one equation, as follows: determine the Ki values for the given P and T for
n n
each i using appropriate formulas, K-factor charts (Figures
X X
x iv x il 1, 6:3:56 4.1.5, 4.1.6), etc. Select a value of (Wtl/Wtf); calculate the xils
i1 i1 from (6.3.54) or the xivs from (6.3.55). Check whether
describing the relation in any phase between the mole n
X n
X
fractions of various components. Thus, there are in total x il 1 or x iv 1:
i1 i1
2n 1 equations and 2n 1 unknowns (n xivs, n xils and
(Wtv/Wtf), or (Wtl/Wtf)), and the solutions can be obtained If the sum is different from 1, adopt a new guess of (Wtl/
provided xif is given and P and T are specified. This in turn Wtf), calculate the liquid or vapor composition again and
will allow Ki to be determined as a function of xiv and xil. check whether the sum of the mole fractions in a given
The required calculations are called isothermal flash calcu- phase equals 1. If and when it does, check whether the sum
lations. In general, it will be an iterative process for a of mole fractions in the other phase also equals 1. If it does,
multicomponent system since Ki depends on xiv and xil; the guess was correct and the calculation is done.
when Ki is almost constant in relation to xiv and xil, calcu- An alternative, trial-and-error, procedure is to employ
lations become easier. n
X n
X
For a binary system of species 1 and 2, where P and T x il 1 and x iv 1
are specified, analytical solutions are possible provided Ki i1 i1

is independent of xiv and xil. Equation (6.3.53) may be in the manner of a function (see Rachford and Rice, 1952a, b),
written for i 1 and 2 as follows:   X n Xn
W tv
x 1v K 1 x 1l ; x 2v K 2 x 2l : 6:3:57 f x iv  x il 0, 6:3:63
W tf i1 i1

However, x1v x2v 1 and x1l x2l 1, leading to to determine the value of (Wtl/Wtf) or (Wtv/Wtf) which
x 1v K 1 x 1l ; 1  x 1v K 2 1  x 1l : 6:3:58 makes the function f(Wtv/Wtf) go to zero. The nature of
this function, for ease of calculation, is obtained as follows.
Adding these two equations and simplifying, we get From (6.3.55), obtain
1  K2 K1  1 K i x if
x 1l ; x 2l : 6:3:59 x iv W tv : 6:3:64a
K1  K2 K1  K2 K i  1 1
W tf

Correspondingly, From (6.3.54), obtain


6.3 Bulk flow parallel to force direction 393

x if x if Table 6.3.2.
x il W tv W tv : 6:3:64b
1W tv
W tf K i W tf W tf K i  1
K i x if  x if
W tv
K i  1 1
Relation (6.3.63) may now be written as Species xif Ki W tf

  X n 4:90:120:12
W tv K i x if  x if Ethane 0.12 4.90 03:91 0:5880:12
1 0:468
f W tv 0: 6:3:65
W tf i1 W tf K i  1 1 1:650:320:32
Propane 0.32 1.65 00:651 0:5280:32
1 0:208
When the value of the function f(Wtv/Wtf) is essentially zero 0:50:560:56
n-Butane 0.56 0.50 00:51 0:28
1 0:28
within a prescribed limit, the correct guess of (Wtv/Wtf) or
(Wtl/Wtf) has been made. A detailed basis for using such a
function of (Wtv/Wtf) has been discussed in King (1980, pp.
7187), which also provides different aspects of the calcu- From Table 6.3.2 we obtain
  X n
lation procedures for such problems. W tv K i x if  x if
One aspect of the calculation procedure needs to be f W tv 0:396:
W tf i1 W tf K i  1 1
considered at the very beginning of any calculation. The
vaporliquid equilibrium behavior in the flash vaporization Consider now (Wtv/Wtf) 1 (Table 6.3.3).
problem specification must be such that the system is
neither a subcooled liquid nor a superheated vapor. To Table 6.3.3.
check that the system has both phases present, calculate f
K i x if x if
(Wtv/Wtf) for two cases: (Wtv/Wtf) 0 and (Wtv/Wtf) 1. W tv
W tf K i 11
For a correctly specified system, f(Wtv/Wtf) will be positive Species xif Ki
in the first case and negative in the second case (King, 0:468
Ethane 0.12 4.90 13:91 0:468
4:9 0:095
1980, p. 75).
In flash vaporization characterized by the two product Propane 0.32 1.65 0:208
0:208
10:651 1:65 0:126
phases being in equilibrium, other calculation procedures,
0:28
if the problem specifications are different from the case of a n-Butane 0.56 0.50 10:51 0:28
0:5 0:56
specified P and T, should be considered. The following
additional types of problem specifications have been fre-
quently considered (King, 1980, pp. 7986): From Table 6.3.3 we obtain
(1) T and the ratio (xivWtv/xifWtf) specified for a particular i;   X n
W tv K i x if  x if
(2) P and the ratio (xivWtv/xifWtf) specified for a particular i; f W tv 0:339:
W tf i1 W tf K i  1 1
(3) P and the ratio (Wtv/Wtf) specified;
(4) isenthalpic flash and P specified. In this case, the sum These results indicate that both vapor and liquid are present
of the enthalpies of the vapor and the liquid product in the system under the specified conditions of the problem.
streams must equal the enthalpy of the feed stream. It is also clear that the correct value of (Wtv/Wtf) which will
make f(Wtv/Wtf) 0 will be somewhere between (Wtv/Wtf)
Illustrations of the solution procedures for a number of
0 and 1, since the function f appears to be almost equally
different cases have also been provided by King (1980,
positive and negative at the two limits of 0 and 1. Our first
pp. 7590). guess will be (Wtv/Wtf) 0.6. If it leads to too negative a value
of f(Wtv/Wtf), then we may use (Wtv/Wtf) 0.4 to find the
Example 6.3.1 A liquid mixture containing 12% ethane, 32% behavior of the function. See Table 6.3.4.
propane and 56% n-butane (see Example 4.1.1) is throttled
into a flash drum at 32  C and 700 kPa. The feed composition
provided is in mole %. Calculate the fraction of the feed Table 6.3.4.
stream which leaves the flash drum as a vapor if vapor
K i x if  x if
liquid equilibrium may be assumed. Determine the compos- W tv
K i  1 1
itions of the vapor stream and the liquid stream. Assume that W tf

the Ki values may be obtained from Figures 4.1.5 and 4.1.6.


(Wtv/Wtf) 0.6 (Wtv/Wtf) 0.4
Solution The governing equations are (6.3.65), (6.3.55) and
0:468 0:468
(6.3.54). First, one must make sure that the problem specifi- Ethane 0:63:91 0:140 0:43:91 0:1828
cations are such that the flash drum conditions are in the 0:208 0:208
Propane 0:149 0:165
two-phase region of vaporliquid equilibrium. We calculate 0:60:651 0:40:651

the value of f(Wtv/Wtf) from equation (6.3.65) for two values 0:28 0:28
n-Butane 0:60:51 0:4 0:40:51 0:35
of (Wtv/Wtf), namely 0 and 1. Consider (Wtv/Wtf) 0 first.
From Figure 4.1.6, we determine Ki for the three species (see f(Wtv/Wtf) 0.111 f(Wtv/Wtf) 0.0022
Table 6.3.2).
394 Open separators: bulk flow parallel to force and CSTSs

The correct guess is around (Wtv/Wtf) 0.4. Instead of same type of molecular weight density function as the vapor
trying to achieve higher accuracy by guessing (Wtv/Wtf) fraction or the liquid fraction. Since relation (6.3.68) has to be
0.41 or 0.42 (say), we will use 0.4 and determine the values of satisfied for all molecular weight species present in the
xiv and xil by using equations (6.3.55) and (6.3.54). system, and ff(M) may be given, the problem is quite
complex.
Table 6.3.5.
Approximate solution formalisms have been developed
x iv
x if
  x il
x if
  by Cotterman and Prausnitz (1985) and Kehlen et al. (1985)
W tv 1
W tf K i
W
1W tv
tf
W tl
W tf K i
W
1W tl
tf
based on the assumption that the same type of density
function may describe the molecular weight based distribu-
0:12 0:12 0:12
Ethane 0:40:2040:6 0:522 0:23 0:64:90:4 0:12
2:56 0:0468 tion in all three streams. Other approximate calculation
procedures have also been developed. One such procedure
0:32 0:32 0:32
Propane 0:40:6060:6 0:763 0:419 0:61:650:4 0:32
1:26 0:254 employs the first and second moments of each molecular
weight density function and develops appropriate relations
0:56
n-Butane 0:420:6 0:56
1:6 0:35
0:56
0:60:50:4 0:56
0:8 0:7 from (6.3.68) between the parameters of each density func-
tion. Suppose a distribution (see (4.1.33f)) describes the
xiv 0.999 xil 1.0008 behavior of all three streams. The mean and the standard
deviation for each stream are given, respectively, by
The results (Table 6.3.5) appear to be satisfactory. feed: f f f ; f 2f ;
The above example illustrates isothermal flash vaporiz- vapor: v v v ; v 2v ;
ation calculations for a multicomponent chemical mixture. liquid: l l l ; l 2l :
We now consider a continuous chemical mixture (treated
The shift parameter (where f(M) is zero) is assumed to be
earlier in Sections 4.1.2, 3.3.1, 3.3.7.1 and 2.4 (equations
the same for all streams:
(2.4.19) onwards).) A liquid, which is a continuous chemical
mixture, may also be subjected to an isothermal flash vapor- f v l : 6:3:69
ization process. The calculations for such a separation under
Taking the first moment of equation (6.3.68),
the condition of equilibrium have been implemented by,
among others, Cotterman and Prausnitz (1985) and Kehlen 1 1
Mof Mf f M dM Mov1 1  Mol , 6:3:70
et al. (1985), employing a gamma () distribution and a
Gaussian distribution, respectively, as the molecular weight f f f v v 1  l l v 1  l ;
density functions (see equations (4.1.33f, g)). The basic
governing relations are given below, employing three f f v v 1  l l : 6:3:71
molecular weight density functions: feed liquid, ff (M); vapor
fraction from the flash drum, fv(M); liquid fraction from the Taking the second moment of equation (6.3.68),

flash drum, fl(M). We focus on species in the molecular 2 2
Mof M 2 f f M dM Mov2 1  Mol , 6:3:72
weight range M to M dM.
Vaporliquid equilibrium in the flash chamber: where, for the distribution,

l M v M : 6:3:66 Mo2 2 2 :

Molar balance Therefore, from equation (6.3.72),


 2
W tf f f M dM W tv f v M dM W tl f l M dM: 6:3:67 f f f 2f v v 2 v 2v

We have seen earlier (e.g. equation (4.1.33c)) how the two 1  l l 2 1  l 2l , 6:3:73
density functions of the two phases (vapor and liquid) are
which, after simplification, yields
related to each other if there is vaporliquid equilibrium.
For example, if the liquid phase has a Gaussian distribution,

f f 1 2f v v 12v 1  l l 12l :
the vapor phase will also have a Gaussian distribution, with
6:3:74
different parameters. In a flash separator, such behavior is
still valid under equilibrium conditions. However, the Equations (6.3.74) and (6.3.71) are the final forms of the two
material balance relation (equation (6.3.67)), rewritten as moment based equations. Since feed properties are known,
f and f are known. Unknowns are v, v, l, l and .
f f M f v M 1  f l M 6:3:68
The phase-equilibrium relation (6.3.66) provides add-
(where is the fraction of the feed flow rate coming in that is itional relations between v, v and l, l since both phases
vaporized ( Wtv/Wtf)), shows that ff(M) need not have the satisfy a distribution (see (4.1.33c) for the reason why
6.3 Bulk flow parallel to force direction 395

both phases will have the same distribution). Cotterman To do this, we will first use the definition of the activity,
and Prausnitz (1985) have provided the following results: aij, of a species i in phase j (see (3.3.20b)):
 
aij ^f ij =f 0ij :
v l ; v l ; v l = 1  C 2 l , 6:3:75
For a volatile species i in a liquid exposed to an ideal gas
where C(2) is a function of temperature, pressure, etc., but
phase (under vacuum), f 0ij P sat sat sat
i i P i . Further,
not of M. Use of the last two results (since v l was already
assumed) in equations (6.3.71) and (6.3.74) leads to two under these conditions, ^f ^f p , so that
il iv iv
equations for two unknowns l, l or v, v for a given degree piv
of vaporization, , the stage cut. The results for such a ail sat : 6:3:76
Pi
calculation procedure have been illustrated in Cotterman
and Prausnitz (1985) for paraffins having a mean molecular The activity of a volatile species i in a polymeric solution
weight of 100 and a variance of 800; their calculation pro- containing volume fractions pl and il of the polymer and
cedure was more complicated than that presented above, species i, respectively, can be obtained from the Flory
since they considered a semicontinuous mixture containing Huggins theory as follows (Flory, 1953):
40% CO2, the rest being a continuous mixture of paraffins.
ln ail ln il ip 2pl pl
They have also considered multi-ensemble mixtures, where ail  
) exp pl ip pl 1 : 6:3:77
each ensemble is a continuous distribution of certain types il
of species: for example, they have considered three ensem-
bles, one for paraffinic compounds, one for napthenic com- This may be rewritten using (6.3.76) as
pounds and the third for aromatics.  
piv
We now briefly consider a flash devolatilizer (see Figure sat il exp pl ip pl 1 : 6:3:78
Pi
6.3.11(b)) (Biesenberger, 1983; Meister and Platt, 1989) to
remove residual monomers and solvents from a polymeric Here, ip is the FloryHuggins interaction parameter.
liquid. The polymeric liquid is, however, nonvolatile, Under devolatilization conditions, pl ~ 1, so that
whereas the other species are volatile and are to be removed
from the polymer. For example, a molten viscous feed from a piv P sat
i il exp ip 1 x iv P: 6:3:79
polystyrene reactor at around 170  C and containing 85%
If the density of the polymer and the volatile species i are
polymer, the rest being the monomer styrene,
p and i, respectively, the volume fraction il may be
ethylbenzene, cumene, styrene dimers and trimers, is
converted into a mass fraction uil to provide
pumped into a heater. The polymeric liquid and the associ-
ated species are heated up to 230  C and introduced into the i piv 1 x iv Pi
uil     , 6:3:80
flash tank maintained under vacuum (~430 Torr). As the p P sat
i exp ip 1 sat
P i p exp ip 1
foamy polymeric mass falls to the liquid level (just as in the
case of a hydrocarbon liquid in the earlier example), the where, for a very dilute solution of i in the polymer,
monomer and the volatile solvent species escape the poly-
uil =i uil =i uil p
mer phase into the gas phase and are withdrawn from the il , 6:3:81
uil =i upl =p upl =p i
tank at the top. The net direction of both phases flow is
perpendicular to the liquid surface; the force causing vola- since (uil/i) << (upl/p) and upl 1 in the very dilute
tilization is also perpendicular to the liquid surface. solution. Analogous to the vaporliquid equilibrium rela-
At its best, the volatilizer may be said to operate such that tion (4.1.31), where the vapor phase behaves ideally, if we
the vapor and liquid phases are in equilibrium (Meister and use mass fraction instead of mole fraction in the liquid
Platt, 1989), exactly as in Example 6.3.1. This may be achieved phase, we can rewrite (6.3.80) as
if there is enough time for diffusion of the monomer/solvent
through the polymer film. In reality, the rate of monomer/ x iv P 1
uil , 6:3:82
solvent diffusion through the foaming mass determines the P sat
i il , u

extent of residual monomer in the polymeric liquid at any where the infinite dilution activity coefficient on a mass
given vacuum level. But lack of knowledge of the interfacial fraction basis is given by
area in the foamy mass precludes an exact calculation; fur-  
ther, the values of the diffusion coefficients in such a system p exp ip 1
are in doubt. See Meister and Platt (1989) for preliminary il, u : 6:3:83
i
considerations on mass-transfer control. We will instead
briefly consider here the thermodynamic limit where there Based on the work by Vrentas et al. (1983) for polystyrene
is equilibrium between the vapor and the polymeric foamy ethylbenzene, where ip 0.35, and similar systems,
liquid for the volatile solvents and monomers. il, u  5, resulting in
396 Open separators: bulk flow parallel to force and CSTSs

Table 6.3.6.
Condensate
Component Feed composition (wt%)
Cooling
Polystyrene 84.27
water
Styrene 11.61
Ethylbenzene 2.68
Cumene 0.626
Others (styrene dimer, etc.) 0.814

Feed liquid

x iv P Steam
uil : 6:3:84
5P sat
i in

Assuming that essentially all of the volatile monomers and


solvents in the feed polymeric liquid are volatilized in the Figure 6.3.12A. Batch distillation with total condensation and
flash devolatilizer, the value of xiv can be determined from no reflux.
the feed composition introduced into the device.

Example 6.3.2 In a commercial flash tank devolatilizer weights of styrene, ethylbenzene and cumene are, respect-
(Meister and Platt, 1989), the output from a polystyrene ively, 104,106 and 120. Therefore
reactor having the composition (in wt%) shown in Table
6.3.6, is introduced at the rate of 5600 kg/hr at 230  C from 77:8=104
x 1v
a heater open to the flash tank. The polymer melt is removed 77:8 17:9 4:3

from the molten layer at 230  C at the bottom of the tank by a 104 106 120
gear pump. The vapor stream is withdrawn from the tank top
0:748 0:748
by a vacuum pump, which maintains the tank vacuum at 0:785;
5 Torr. Calculate the residual amount of ethylbenzene and 0:748 0:1689 0:0358 0:9527
styrene left in the withdrawn polymer melt in ppm (parts per 0:1689
million parts by weight). You are given the vapor pressure of x 2v 0:177; x 3v 0:038:
0:9527
the components at 230  C in Torr: styrene, 4644; ethylben-
zene, 5555; cumene, 4020. Correspondingly, for styrene,
Solution We will employ equation (6.3.84), 0:785  5
u1l 0:000169 169 ppm;
x iv P 5  4644
uil ,
5P sat
i
for ethylbenzene,
to calculate the mass fraction of both styrene and ethyl- 0:177  5
u2l 0:000031 31 ppm;
benzene, etc., left in the melt leaving the tank since we do 5  5555
not have data on il for styrene. Further properties of and for cumene,
others (styrene dimer, etc.) have not been provided. 0:038  5
Therefore we will use a material balance based only on u3l 0:0000094 9:4 ppm:
5  4020
the polymer, styrene, ethylbenzene and cumene (the
We now consider the case of a batch liquid feed introduced
dimer is supposed to have very low vapor pressure). For
into a distillation vessel, pot or flask (Figure 6.3.12A). The
the polymer-free basis,
liquid is heated and vapors continue to leave perpendi-
11:61 cularly from the top liquid surface. The escaping vapor
wt% styrene  100
11:61 2:68 0:626 flow is parallel to the chemical potential gradient based
11:61  100 driving force from the liquid to the vapor phase. Identify
77:8%; the vapor phase as j 1 (or v) and the liquid phase as
14:91
j = 2 (or l), since the more volatile component i 1 will be
2:68  100
wt% ethylbenzene 17:9% enriched in the vapor leaving the vessel top at a molar flow
14:91
rate Wt1. Usually there is a condenser at the top of the
wt% cumene 100  77:8  17:9 4:3%: batch distillation vessel; the total condensate flow rate
Define styrene as i 1, ethylbenzene as i 2 and cumene as
taken out as the product is also Wt1. The total number of
i = 3. Assume now that all of the styrene, ethylbenzene and molesXof liquid mixture charged into the vessel at time t
n
cumene are in the vapor phase. Therefore, their mole frac- 0 is m0 m0t2 . As time progresses, mt2 decreases, its
i1 i2
tions in the vapor phase, xiv, can be obtained from their mole composition xi2 changes; Wt1 also changes with time, just
fractions in the feed on a polymer-free basis. The molecular as xi1 changes with changing xi2. One would like to know
6.3 Bulk flow parallel to force direction 397

how the composition of the liquid changes with the   xil


mtl 1 dx il 1 x il
changing amount of the liquid in the heated vessel. (Usu- ln ln 6:3:89
m0tl Ki  1 x il K i  1 x 0il
ally, the total pressure P is held constant; as the liquid x oil
composition changes, the temperature will also change if
we assume vaporliquid equilibrium.) Alternatively, one and
would like to know the condensate composition for a given   xil  
mtl dx il 1 x il 1  x 0il
liquid composition in the vessel. ln 0 12 x il ln 0 12 ln :
mtl 112 1x il  x il 12  1 x il 1  x il
If in time dt, the change in the total liquid moles for x oil

species i is dmi2, and that in the total number of moles 6:3:90


(mi1) which have escaped as vapor is dmi1, then
An alternative form of equation (6.3.90) is
dmi2 dmi1 x i1 W t1 dt dx i2 mt2
  (   11   12 )
x i1 dmt1 x i2 dmt2  mt2 dx i2 : 6:3:85a mtl x il 12 1  x 0il 12 1
ln ln ln
m0tl x 0il 1  x il
By a total mass balance over any interval dt
  11   12
mtl x il 12 1  x 0il 12 1
dmt2 dmt1 : , : 6:3:91
m0tl x 0il 1  x il
Introducing the total mass balance into the ith component
Note that i 1 here for the more volatile species. This type
balance, we get
of batch distillation is also called differential distillation or
x i2 dmt2  mt2 dx i2 x i1 dmt2 Rayleigh distillation.
) mt2 dx i2 mtl dx il x i1  x i2 dmt2 x iv  x il dmtl : Example 6.3.3 A 50% benzene50% toluene liquid mixture
6:3:85b has been introduced as a feed into a batch distillation flask.
The distillation is carried out at 120 kPa total pressure. Con-
Rewrite this as sider the situation where 50% of the liquid mixture has been
dmtl dx il collected as the distillate. What is the composition of the
: 6:3:86 distillate mixture? An averaged separation factor 12 of 2.40
mtl x iv  x il
may be assumed (where i 1 is benzene) over the tempera-
Integrating this from m0tl m0t2 to mtl mt2, corresponding ture range under consideration.
to a change in the liquid-phase mole fraction from its Solution We employ equation (6.3.91) and a trial-and-error
initial value x 0il to the value xil at any time t, leads to
approach.
Here species i 1 is benzene. In this case,
m
tl   xil mtl =m0tl 0:5, x 01l 0:5, 12 2.40, x1l is unknown. Know-
dmtl mtl dx il ing mtl and x1l means that mtv is known. Therefore, the
ln : 6:3:87
mtl m0tl x iv  x il distillate composition (x1v)avg may be determined from the
motl x oil mass balance over species i 1 (benzene) as follows:
This relation is known as the Rayleigh equation (this type of m0tl x 01l mtl x 1l mtv x 1v avg ) x 01l 0:5 0:5x 1l 0:5x 1v avg :
relation between the composition of the two fractions,
In equation (6.3.91), x1l is unknown. We can solve it numer-
regions, etc., will be encountered later in other separation ically, or by straightforward trial-and-error based on an ini-
processes). The right-hand side of this equation may be tial guess (see Table 6.3.7).
integrated if the relation between xiv and xil is known. We introduce a value of x1l 0.35 into the overall mass
For a binary mixture (i 1,2), if it can be assumed that balance equation:
the vapor leaving the liquid having a composition of xiv is
0:5 0:5  0:35 0:5x 1v avg ) x 1v avg 0:5  0:175=0:5 0:65:
in equilibrium with the liquid of composition xil, then,
from (4.1.32a), It is clear from Example 6.3.3 for the benzenetoluene
x iv system (see Figures 4.1.1 and 1.4.1) that very little separ-
Ki : ation is achieved in a simple batch distillation. If each
x il

Alternatively,
12 x il Table 6.3.7.
x iv x i1 , 6:3:88
1 12  1x il
Guess   1   121

x 1l 1x 01l
if 12 is the separation factor in vaporliquid equilibrium for x1l x 01l
12 1
1x 1l
12
mtl =m0tl Comment
(VLE) defined by (4.1.21a) when i 1. Introduction of
these two relations between xiv and xil into the Rayleigh 0.3 0.695 0.563 0.391 too low
0.4 0.867 0.73 0.633 too high
equation (6.3.87) and the assumption of constant Ki and
0.35 0.778 0.636 0.4948 good
12 lead, respectively, to
398 Open separators: bulk flow parallel to force and CSTSs

individual drop of liquid in the feed liquid mixture were to present. One could add additional replacement solvent
undergo partial vaporization independently, then the com- next to reduce the original solvent to the level of a low-
position of the first vapor bubble in equilibrium with each level impurity (Chung, 1996; Gentilcore, 2002). These ref-
such drop can be calculated from equation (6.3.88): erences provide illustrative examples of solvent exchange
in batch distillation. Of course, the volatile replacement
12 x 1l 2:4  0:50 1:2
x 1v 0:706: solvent is also lost by distillation.
1 12  1x 1l 1 2:4  1  0:50 1 0:7
Gentilcore (2002) has illustrated a different mode of
As the benzene content of the remaining liquid decreases feed introduction for solvent exchange in batch distillation,
due to the removal of the first bubble, the benzene content and has showed that it minimizes the loss of the volatile
of the second bubble from the remaining liquid drop will replacement solvent. This has been named constant-level
be lower than that in the first bubble, and so on. Further, if batch distillation. This technique is implemented by
only part of the liquid lying on top of the liquid pool adding the replacement solvent continuously to the pot at
undergoes vaporization, the rest of the liquid will have a such a rate that the volume of the solvent mixture in the
lower benzene content. Thus, the highest amount of vapor pot remains constant, hence constant-level batch distilla-
having the highest benzene content may be obtained if the tion. We will now develop the relation corresponding to
whole batch liquid charged into the pot could somehow Rayleigh distillation for constant-level batch distillation,
exist as an infinite number of drops each independently where the replacement solvent added to the still is pure
undergoing a vaporization process, with only a small frac- species 2 and the original solvent (more volatile) is
tion being vaporized. species 1. We ignore the existence of other organic com-
The flash vaporization process of Figure 6.3.11(a) is a pounds in the pot from a distillation point of view.
step in that direction only if the extent of vaporization in Assume that the molar volumes of the two species are
each drop is quite small so that x1v is close to 0.706. identical. Then, over a small time interval, dt, the total
Otherwise, if there is going to be a substantial amount number of moles evaporated (vapor phase j 1, j v),
of vaporization in the flash process (as, for example, dmt1, must equal the total number of moles of pure
in Example 6.3.1), then the value of x1v in equilibrium exchange solvent feed added (j f), dmtf:
with x1l will be considerably lower. On the other hand,
dmt1 dmtf : 6:3:92a
the batch distillation process will start out with values of
x1v substantially larger than the x1v value from flash distil- Since the total number of moles of liquid, mt2, in the still
lation (in the equilibrium limit), since x1l at the beginning remains constant, by assumption (j 2, j l)
corresponds to the feed liquid in batch distillation.
On an overall basis, therefore, batch distillation can dmt2 0 dmtl 6:3:92b
end up with a higher vapor composition. Also, it is possible in the time interval dt. Carrying out a molar balance on the
that the last drop of liquid in the pot in batch distillation is more volatile species 1, since the exchange solvent feed is
pure heavy-component liquid (i.e. x2l 1); the amount pure species 2, we obtain
would be infinitesimally small. The flash process alone
cannot ever achieve a pure liquid drop or a vapor bubble dmt2 x 12 dmtl x 1l x 1l dmtl  mtl dx 1l
(King, 1980, p. 141). However, both of these distillation mtl dx 1l x 11 dmt1 : 6:3:92c
schematics are more productive than a closed separation
Rewriting this equality as
vessel since the total amount of vapor product in the
latter is quite small. The vessel where the vapor and dmt1 dx 1l dmtf
  , 6:3:92d
the liquid are in equilibrium is often identified as the mtl x 11 mtl
ideal stage. and integrating between the limits of initial liquid-phase
A frequent application of batch distillation in pharma- mole fraction x 01l and the final mole fraction x1l, when mtf
ceutical and fine chemical production processes involves moles of pure exchange solvent have been added since t
solvent exchange. The solvent employed in a particular 0, we get
reaction step needs to be replaced by another for the next
reaction step. Alternatively, the replacement solvent is mtf x1l x 01l x 01l

dmtf dx 1l mtf mtf dx 1l dx 1l
needed to carry out crystallization of the final product,   )
mtl x 11 mtl mtl x 11 x 1v
which is, in general, nonvolatile. If the final or intermediate 0 x o1l x 1l x 1l
product is thermally labile, then distillation is carried out at
6:3:93
a lower temperature under partial vacuum. The conven-
tional procedure is as follows. First, the batch is boiled In general, a graphical/numerical solution of the integral
down to remove a lot of the original solvent. Then the on the right-hand side is carried out on the basis of an
replacement solvent is added, and the resulting batch is available vaporliquid relation x1v f(x1l). If we can
distilled to reduce further the amount of original solvent assume that the relative volatility/separation factor 12
6.3 Bulk flow parallel to force direction 399

between the two solvents 1 and 2 is constant and employ From relations (6.3.85a,b) identifying the quantitative
relation (4.1.24) in the above integral, we get changes taking place in simple batch distillation over a
time interval dt, namely mtl dx il x il dmtl x iv dmtl , we
x 01l
obtain
mtf 1 x 1l 12  1
dx 1l , 6:3:94
mtl 12 x 1l dmtl x il =dt x iv dmtl =dt : 6:3:96
x 1l

which leads to (Gentilcore, 2002) This may be rearranged to obtain


 0
mtf 1 x 12  1 0 dmtl =dt x il  x iv mtl dx il =dt : 6:3:97
ln 1l x 1l  x 1l : 6:3:95
mtl 12 x 1l 12
Recognizing that dt mv dmt1 dmtl dmt2 , we get
Gentilcore (2002) has illustrated how the loss of the
dx il =dt dmtv =dt =mtl x il  x iv , 6:3:98
replacement solvent in the distillate can be considerably
reduced by continuously adding the replacement solvent which describes how the liquid-phase composition, xil,
under the condition of a constant-level control of the liquid changes with time. The solution will allow one to follow
in the pot (instead of a conventional batch distillation). the composition of the liquid mixture in the batch distilla-
This case illustrates how the mode of feed introduction tion vessel, the residue, with time.
influences the separation achieved, even though the Numerical integration of equation (6.3.98) (see Doherty
basic separation mechanism and the force vs. flow pat- and Malone (2001) for various approaches) is usually
tern remain unchanged. For a general and broad intro- carried out to plot the time series of xil, the liquid-phase
duction to batch distillation in all forms, consult Diwekar mole fraction of the more volatile species. Figure 6.3.12B
(1996) and Seader and Henley (1998, chap. 13). Section illustrates how xil changes with time for two different types
8.1.3 will focus on other flow vs. force configurations in of systems. For an ideal system (e.g. a benzenetoluene
batch distillation. system having a constant 12 2:5 at 90  C shown in Figure
4.1.1 with i 1 for benzene) of ethanolisopropanol, the
6.3.2.1.1 Residue curves in batch distillation The illus- solid lines represent how xil changes with time for a con-
trated forms of the Rayleigh equations (6.3.87) and (6.3.91) stant total pressure of 1 atm, for three different initial liquid-
allow one to relate the change in the total number of moles phase compositions x 01l . The time coordinate has been
in the residual liquid phase with the liquid-phase compos- appropriately modified (warped time coordinate of Doherty
ition change in simple batch distillation with no reflux and and Malone (2001)) in the plot to vary from 0 to 1. As time
total condensation. One would like to know how the liquid- progresses, the more volatile species (e.g. ethanol) will be
phase composition changes with time. Visualization of this depleted in the liquid solution (whose boiling point will
composition change pathway is going to be quite illustra- increase) until we are left with only isopropanol, which will
tive. For ternary systems, such visualizations are likely to ultimately be evaporated. Each line in the figure has an
be particularly useful. The topic of residue curve maps is arrow showing the direction of change of the liquid-phase
briefly introduced here to that end. composition of the more volatile species, i.e. ethanol.

Unstable 1.0 1.0 Unstable


node node
ethanol

0.8 0.8
xil (pentane)

0.6 0.6
Unstable
node
0.4 0.4
xil

0.2 0.2

Stable Stable
node 0 0 node
0 0.2 0.4 0.6 0.8 1.0
Modified time coordinate

Figure 6.3.12B. Liquid residue composition curves in simple batch distillation for two types of systems: solid lines illustrate residue curves
for the ideal system ethanolisopropanol at 1 atm; dashed lines show the residue curves for the minimum-boiling azeotropic system,
pentanedichloromethane system at 1 atm. (Figure developed from figs. 5.2 and 5.4 of Doherty and Malone (2001).)
400 Open separators: bulk flow parallel to force and CSTSs

The four dashed lines in Figure 6.3.12B correspond to a for an ideal ternary mixture of a light component (L),
minimum-boiling azeotrope (see Figure 4.1.3 for an intermediate component (I) and heavy component (H)
example, isopropyl ether-isopropyl alcohol system) of pen- (e.g. methanol, ethanol and n-propanol, respectively,
tane and dichloromethane at 1 atm total pressure with the (Doherty and Malone, 2001)) via three solid lines. There
azeotropic composition at ~0.52 mole fraction pentane is no time coordinate in the triangular diagram; the arrow
(Doherty and Malone, 2001). For x 01l values (for pentane) in each line shows the progression in time. Since the light
less than 0.52, where pentane is more volatile, the behavior component, L, is the most volatile of the three species
is similar to that of the ethanolisopropanol system (solid located at three vertices of this diagram, all curves start
lines) in that, as time passes, the residual liquid becomes form this L vertex. As time progresses, the curves first move
enriched in dichloromethane, the less volatile species. On toward increasing concentration of I, the intermediate
the other hand, for x 01l values higher than 0.52, the dotted boiling species (I is heavier than L), reflecting increasing
lines show a totally different trajectory, with the liquid concentration of I in the liquid residue. As time passes,
becoming enriched in pentane as time passes since the however, the concentration of H, the heavy component,
vaporliquid equilibrium behavior has been reversed keeps on increasing, and the curves turn toward the vertex
beyond the azeotropic composition; pentane has become H through a saddle point.
the less volatile species. The residue curve maps for ternary mixtures having
For both types of systems the trajectory in time is one binary minimum-boiling azeotrope are much more
illustrative. For the ethanolisopropanol system, the trajec- complicated. They are even more complex when there
tory (solid lines) moves toward x1l 0, which is identified are multiple azeotropes. See Doherty and Malone (2001)
as the stable node, whereas x1l 1 is the unstable node for illustrative figures. Siirola and Barnicki (1997) illus-
since the composition of the liquid phase is moving away trate residue curves for a variety of systems. Complex
from it since ethanol (i 1) is the more volatile species. properties of the residue maps are analyzed in detail
For the pentanedichloromethane system, we observe two in Doherty and Perkins (1978) and Van Dongen and
types of trajectories (dashed lines) for the two sides of the Doherty (1984).
azeotropic composition; correspondingly we have two
stable nodes and one unstable node.
6.3.2.2 Liquidliquid systems
The directional arrows in the solid and dashed lines of
Figure 6.3.12B provide the progression in liquid residue Bulk flow of one phase parallel to the direction of the force
composition with time for two different types of systems. may be visualized for liquidliquid systems by considering
This strategy becomes useful for visualizing the change in the two arrangements shown in Figure 6.3.13. In arrange-
liquid residue composition via a triangular diagram for a ment (a), the vessel is initially filled with an aqueous
ternary mixture in the batch distillation setup of Figure solution of a species i. To this solution is introduced, from
6.3.12A. Figure 6.3.12C illustrates the residue curve map the bottom of the vessel, an organic extraction solvent in
the form of drops. Since the organic solvent is likely to
I be lighter than the aqueous phase, it will rise to the top
(Intermediate
1.0 component) and form a layer of the organic extract, from which an
organic extract could be withdrawn continuously as shown
(Treybal, 1963, p. 220). Alternatively, consider a separating
0.8
funnel containing an organic-phase solution. If an aqueous
extracting phase is introduced from the top (as in arrange-
0.6 ment (b)) it is likely to be heavier, and therefore drop
through the organic phase as it extracts the solute and
0.4 forms a layer at the bottom of the funnel. A small fraction
of it may be continuously withdrawn as the extract from
0.2 the bottom of the separating funnel. The organic raffinate
is left in the separating funnel. In both cases, the effective
0 L (Light force for solute partitioning is parallel to the direction
H 0 0.2 0.4 0.6 0.8 1.0 component) of bulk flow of the phase introduced from outside;
(Heavy component) specifically, the force is perpendicular to the bulk interface
between the two phases. However, the transfer from the
Figure 6.3.12C. Illustrative residue curves of a ternary system drops to the sides during their motion introduces a
without any azeotrope: L represents the lightest component, deviation from this model.
I represents an intermediate boiling species, and H is the highest- This type of contacting two immiscible liquid phases
boiling component. Each represents the pure component at each for extracting solutes is termed differential extraction
vertex. (After Doherty and Malone (2001).) (Treybal, 1963, p. 220). There are a number of ways one
6.3 Bulk flow parallel to force direction 401

(a) (b)
Aqueous
extracting agent
Organic in
Organic extract extract out

Organic
raffinate
Aqueous raffinate Separating funnel
Aqueous
extract

Organic extracting
solvent in Aqueous extract out

Figure 6.3.13. Two arrangements for bulk flow of one phase in another stationary phase parallel to the direction of the chemical potential
driving force in immiscible liquidliquid systems.

can analyze the separation achieved in the arrangements If the concentrations of solute i in the two phases at
shown in Figures 6.3.13A and B as long as the rate at which equilibrium, namely Cio and Ciw, are related by a constant
the fresh organic or aqueous phase is introduced is small distribution coefficient,
compared to the volume of the system. First, one can
C io
assume that the two phases are completely immiscible io , 6:3:101a
C iw
and insoluble; the phase contact merely extracts the solute
i from, say, the aqueous phase to the organic phase. then we obtain
Second, we can assume that everywhere in the container,
C0iw
the two phases are in equilibrium via efficient mass trans-
Vw dC iw
port. (If this efficiency is achieved by a well-mixed liquid V to 
C0
: 6:3:101b
io C iw  ioio
phase, then the methodology of Section 6.4.1.2 ought to C iw
be used.)
Consider the arrangement in Figure 6.3.13A, where Integration leads to
an organic solvent having an initial solute i concentra-  0 
io V to C  C oio =io
tion of C 0io is introduced at a low rate into the container log iw o : 6:3:102a
Vw C iw  C io =io
containing the aqueous phase. Let the volumes of the
organic solvent and the aqueous phase in the vessel be Alternatively,
Vo and Vw, respectively. If the solute i concentrations in
 
the two phases are Cio and Ciw, respectively, then an C iw  C 0io =io io V to
0
0

exp  : 6:3:102b
introduction of dVo volume of the organic solvent will C iw  C io =io Vw
lead to a small decrease dCiw in the solute concentra-
Suppose Cioo is zero; the solvent added to the vessel is
tion in the aqueous phase:
solute-free. One can never reduce Ciw to zero unless an

V w dC iw C io  C 0io dV o : 6:3:99 infinite volume of this solute-free organic solvent is


passed through the system in the configuration of bulk
The total solvent volume added, V t0 , added to the system flow parallel to the force direction.
over the whole process is Bulk flow via direct mechanical conveying can also
Vto C0iw allow the achievement of bulk flow parallel to the direc-
dC iw tion of the force. Consider, for example, a liquid phase
V to dV o V w
, 6:3:100
C io  C 0io j 2 of volume V2 in a vessel containing species i 1,2.
0 C iw
Add a certain volume V1 of an immiscible liquid
where C 0iw is the initial solute i concentration in the aque- phase j = 1 on top of this liquid phase j 2. This liquid
ous phase in the vessel and Ciw is the final solute i phase will extract preferentially one of the species i.
concentration After equilibrium has been achieved, this top liquid
402 Open separators: bulk flow parallel to force and CSTSs

phase may be decanted away and fresh liquid phase mi1 2 mi1 2 mi2 1 mi1 2 mi2 2 mi2 1
Y i1 
j = 1 of volume V1 introduced again into the vessel. m0i2 mi2 1 m0i2 mi2 2 mi2 1 m0i2
After equilibration, this top liquid phase may also be 0 1
withdrawn, and fresh liquid phase j, having the same 0 @ mi2 2 mi2 1A 0
) Y i1 ki1 ki1 Y i2 2Y i2 1
volume, may be introduced again. Such discrete equilib- mi1 2 mi2 2 m0i2
rium contacts between the two phases may be carried 0
ki1 1 0
ki1
out many times. 0 : 6:3:104a
12
0
ki1 1 k 0 i1 1 ki1
In aqueousorganic systems, if the fresh liquid phase j
happens to be aqueous, then it will likely become the
Similarly,
denser and lower phase after introduction. If the vessel is
a separatory funnel, the removal of the water phase mi2 2 mi2 2 mi1 2 mi2 2
Y i2
through the bottom becomes straightforward. Thus, this m0i2 mi1 2 mi2 2 m0i2
introduction of new material and withdrawal can be easily 1
Y i2 2Y i2 1 0 : 6:3:104b
visualized. The direction of phase motion/withdrawal is ki1 12
parallel to the direction of force between the two immis- If we now carry on a third contact by bringing in fresh V1
cible phases. One is interested in finding out the amount to contact V2 after second contact (having a total number of
of separation achieved after, say, n such contacts between moles mi2(2) of species i), the molar distribution now consists
the two phases, where all of phase j 1 will be collected of mi1(3), mi2(3), where (mi1(3) mi2(3)) mi2(2). There-
after n contacts. We will utilize the extent of separation, , fore, based on these three contacts, the segregation fractions
to this end. are
After equilibration at the end of the first contact
between the two phases, we can relate the number of mi1 3 mi1 3 mi2 3 mi2 2 mi2 1
Y i1
moles of species i in the two phases by (equation (1.4.2)) m0i2 mi2 3 mi2 2 mi2 1 m0i2
0 k 0 i1
mi1 V1 ki1 Y i2 3Y i2 2Y i2 1 0 , 6:3:104c
k 0i1 i1 : k i1 13
mi2 V2
mi2 3 mi2 3 mi2 2 mi2 1 1
We assume that i1, V1 and V2, and therefore k0 i1, are Y i2 0 :
m0i2 mi2 2 mi2 1 m0i2 ki1 13
independent of the specific number of contacts. Assume
6:3:104d
the original phase (which becomes the raffinate after the
process) containing the solute i is j 2; the total number of By a process of induction, we may now conclude that, after
moles of solute i originally in this phase is m0i2 . As the first n contacts, with n volumes of fresh V1 contacting V2, the
volume V1 of the extractant is brought in, the segregation final values of Yi1 and Yi2 are
fractions after equilibration are given by
0
ki1 1
Y i1 and Y i2 ,
mi1 1 mi2 1 0
ki1 1n 0
ki1 1n
Y i1 1 , Y i2 1 ;
m0i2 m0i2 6:3:104e
mi1 1 k0 1 where Yi1 is the fraction of solute which is present in the
Y i1 1 0 i1 , Y i2 1 0 :
mi1 1 mi2 1 k i1 1 k i1 1 pooled extract volume nV1 and Yi2 is the fraction of solute
left in the raffinate of volume V2. The extent of separation
6:3:103a
between two solutes i 1,2 can be calculated after such
For the second contact, the total number of moles of i to a process as follows:
start with in phase 2 (raffinate) is mi2(1). The number of
1 1
moles of i in phases 1 and 2 after equilibration with fresh
absjY 22  Y 12 j 0  n : 6:3:105
k21 1n k11
0
1
V1 are mi1(2) and mi2(2), respectively, where (mi1(2)
mi2(2)) mi2(1). The segregation fractions are:
The number of such contacts, nopt, which yields an opti-
mi1 2 mi2 2 mum separation between the two solutes may be obtained
Y i1 2 , Y 2 ; by differentiating with respect to n (see Problem 1.5.1)
mi1 2 mi2 2 i2 mi1 2 mi2 2
(Treybal, 1963, p. 290; Rony, 1969):
k 0i1 1
Y i1 2 , Y 2 0 : 6:3:103b =n 0, 6:3:106
k 0i1 1 i2 k i1 1
which results in
If we define the segregation fraction after second contact  ,  
ln1 k 011 1 k 011
based on the original amount of solute m0i2 in phase 2 nopt ln ln : 6:3:107
ln1 k 021 1 k 021
(raffinate), then
6.3 Bulk flow parallel to force direction 403

One could employ a similar strategy to find out the volume decomposition and low vapor pressure, has been separ-
ratio (V1/V2) which yields the optimum extent of ated/purified on a large scale employing bulk flow parallel
separation: to the direction of the force. The force in this case is due to
2 3 the chemical potential difference of a species between the
6 1 7 melt and the solid/solid solution in contact with it. There
0 4 n 5 are many systems in which an impurity in the solid phase/
V 1 =V 2 V 1 =V 2 V1
1
21 V 2
solution prefers the melt. For example, ice (pure water)
2 3
separates when seawater is frozen. Table 4.1.7 illustrates
6 1 7 the values of the distribution coefficients, is, for a variety of
4 n 5:
V 1 =V 2 V1
1 impurities in silicon and gallium arsenide, two of the most
11 V 2
important semiconductor materials. For almost all such
The resulting value of the optimum ratio of (V1/V2) is impurities, is < 1. Thus, a melt in contact with the solid
will be enriched in the impurity. Conversely, if a melt is
11 =21 1=n1  1 frozen, the first solid frozen out will have an impurity
V 1 =V 2 opt , 6:3:108a
11  21 11 =21 1=n1 concentration Cis isCil < Cil, as shown in Figure 6.3.14
(a), which illustrates the temperatureconcentration dia-
which yields the highest extent of separation after n con-
gram, with the liquidus and solidus lines corresponding to
tacts. This result is also expressed as (Treybal, 1963, p. 290)
h i an impurity i whose is < 1.
11 =21 11 =21 1=n1  1 However, if a large volume of solid and liquid are
11 V 1 =V 2 opt : present together, the equilibrium distribution of an
11 =21  11 =21 1=n1 impurity between the two phases is almost impossible to
6:3:108b achieve since the diffusional rates in a solid are orders
It is important to note that Treybal (1963, p. 290) has of magnitude smaller. To circumvent this problem and
obtained such results for what are called cross current (or exploit the phenomena of is < 1 for large solid samples
cocurrent) multiple contacts. However, the results and and particular impurities, bulk motion is generally intro-
analysis are equally valid for the configuration of phase duced in the solidmelt system by two alternative
contact selected here. methods: zone melting and normal freezing. Both are
considered important components of a number of tech-
niques of crystallization from a melt, which are known
6.3.2.3 Solidliquid systems: zone melting and normal
as fractional solidification (Zief and Wilcox, 1967). We
freezing
consider zone melting first.
In Section 3.3.7.5 we identified three types of liquidsolid In the zone melting method introduced by Pfann
equilibrium: (1966), a small section (zone) of a long sample is melted
by a number of heating techniques at any instant of time;
(1) solid solution in equilibrium with a molten mixture;
the molten zone is slowly made to traverse the whole
(2) crystallization equilibrium between a crystal and a
length L of the sample (Figure 6.3.14(b)). As the molten
mother liquor which is a solution of the crystal
section refreezes, it rejects the impurity into the adjacent
material;
molten section; consequently, the impurities are pushed to
(3) leaching of a solute from a solid mixture by a solvent of
the furthest end of the long sample, which is subjected to
the solute.
freezing at the very end (Figure 1.4.3). To prevent contam-
In the first case, there is no external solvent. When ination by a container, the floating zone technique is used,
the molten mixture freezes, solid crystals are formed. The whereby the zone is held up by surface tension. The zone is
variety of techniques and processes employed to obtain kept molten by a radio frequency source (Gandhi, 1983),
purified crystals/solids or crystals/solids having the appro- often in a vertical position; electron beam, induction
priate amount of dopant/impurity are considered to be heating, etc., are also used.
examples of fractional solidification (Zief and Wilcox, In the zone melting process, at any given time the
1967). The system may be binary or may have three or whole sample/rod is solid except for the small zone, which
more components. On the other hand, crystallization pro- is molten. In the normal freezing process, the whole rod, to
cesses always involve a solvent, which has the solute(s) in start with, is molten. It is slowly pulled out of the heated
solution. When one solute crystallizes out of the solution, region, allowing the pulled-out section to freeze and reject
whereas the other(s) remain(s) in solution, the process is the impurity into the adjoining molten section (Figure
called fractional crystallization. 6.3.14(c)). The bulk motion of the rod as it is being pulled
Of the three general kinds of solidliquid systems out of the heater is clearly in a direction parallel to the
described in Section 3.3.7.5, a solid solution in equilib- direction of the chemical potential gradient at the solid
rium with a molten mixture or a solid that melts without melt interface.
404 Open separators: bulk flow parallel to force and CSTSs

(a)

(b)
Temperature

Heater
Cis Cil Direction of heater
motion
Liquidus
Molten
Solidus zone

Recrystallized Original
Composition Cij unmelted solid
solid Heater

(c)
Heater

Molten rod
Direction of
Recrystallized motion of rod
solid
Heater

Figure 6.3.14. (a)Temperatureconcentration diagram for an impurity whose is < 1: solidmelt system; (b) schematic of zone melting of
a solid rod/bar by traversing the molten zone along the rod length by moving the heating band around the rod at a controlled velocity;
(c) normal freezing technique for purifying a rod/bar by slowly pulling it out of the heated section.

At first sight, there is no bulk motion of the rod in the We will work with the mass fractions uil and uis of the
zone melting process (Figure 6.3.14(b)), where a heating impurity in the melt and the solid phase, respectively. The
band around the rod is moved at a given velocity. However, impurity distribution coefficient is0 is given by
as the heating band moves, so does the small molten zone uis C is l
in the rod along with it; correspondingly, the freezing is0 is l : 6:3:109a
uil C il s s
interface at the back of the molten zone advances in the
same direction at the same velocity. Thus, for the zone Due to the assumption of l s, we know that is0 is here.
melting process also, we have a case of bulk motion paral- Let the initial impurity mole fraction in the solid be uio. As the
lel to the direction of the chemical potential gradient molten zone advances (Figure 6.3.15(a)), section ABB0 A0 of
across the two solidmelt interfaces. length dz freezes and a corresponding section CDD0 C0 of
In the zone melting process, the molten zone slowly length dz melts. The total mass of impurity in the molten
advances along with the heating band along the rod length zone ABCD is uilllA, where A is the cross-sectional area of
(Figure 6.3.15(a)). As a small section of length dz from the the rod. The total amount of impurity in the solid zone
molten zone freezes at the trailing end of the zone, it CDC0 D0 (before melting) is uio sA dz. As the zone advances,
rejects the impurities into the nearby molten liquid. If the the total amount of impurity in the region ABC0 D0 ( uilllA
mixing in the molten zone (of length l) were effective, these uio sA dz) does not change, but is redistributed (there being
impurity molecules would be rapidly mixed with the bulk no impurity diffusion in the solids on the two sides) into the
of the melt, which will then have a uniform impurity con- molten zone A0 B0 C0 D0 ( {(uil duil) llA}) and the refrozen
centration all along the length l of the molten zone. Using zone ABB0 A0 ( {(is0 uil sA dz}):
such a basic assumption, it is possible to develop a theor-
uil l lA uio s Adz uil duil l lA is0 uil s Adz
etical estimate of the impurity concentration profile along
6:3:109b
the length of the rod after the heater has passed along the
rod once, provided the following assumptions are also duil
) uio  is0 uil s l l : 6:3:109c
made: the impurity distribution coefficient is constant; dz
the densities of the melt and the solid are essentially the
Rewrite this as
same; the diffusion coefficient of the impurity in the solid is
zero; there is no variation in impurity concentration along s dz dis0 uil
is0 0 , 6:3:109d
the cross section of the rod. l l is uil uio
6.3 Bulk flow parallel to force direction 405

Heater
Direction of
heater motion
l
dz dz

D D Molten zone A A
uil/o
Original rod Impurity Recrystallized
uio concentration solid
uis
profile uilb
dl B B
C C z Refrozen section

Heater
(a)

Arl (vfr rs / rl) ui Arl (vfr rs / rl) ui


z + z z

ui ui
Dil Arl Dil Arl
z z + z z z

z + z (b) z

Heater

Original rod vfr Recrystallized


solid
Molten zone

Heater
(c)

Figure 6.3.15. (a) Zone melting: impurity concentration profile in melt near the freezing interface; (b) a small element in the molten zone
illustrating various fluxes (coordinate system based on part (c)); (c) overall configuration in a coordinate system fixed in the heater.

and integrate to obtain is0 uil  uio z


ln is0 s : 6:3:109f
is0 uio  uio l l
z
is0 s lnis0 uil  uio constant: 6:3:109e
l l This may be rearranged as follows:
 
At z 0, when melting has just started, uil uio. uil z
is0 1  1  is0 exp is0 s : 6:3:109g
Therefore uio l l
406 Open separators: bulk flow parallel to force and CSTSs

However, uis is0 uil, so Dividing il by t to get the impurity mass fraction in the
  melt, uil, we obtain
uis z
1  1  is0 exp is0 s : 6:3:109h
uio l l
uil uil 2 uil
vz Dil 2 : 6:3:110b
As long as is0
is, this is reduced to the expression t z z
employed in Example 1.4.3, which illustrates also how the To determine the magnitude and direction of the velocity vz of
impurity mass fraction is distributed along the length of the melt, consider the following approach. Fix the xyz coordin-
the rod: for is0 < 1, the lowest impurity mass fraction is at ate system on the heating band; in this frame of reference,
the starting end and the highest is at the other end. materials flow from left to right in the negative z-direction
This derivation was based on the basic assumption that (Figure 6.3.15(c)). If the freezing rate (therefore the velocity
the liquid in the molten zone is well mixed. However, in of the freezing interface) is vfr (in cm/s), then the magnitude
almost all cases, the mixing effectiveness in the viscous of the velocity of the melt in the negative z-direction is vfr
melt is very poor. Essentially, molecular diffusion is the (s/l). This velocity is also utilized in Figure 6.3.15(b).
only mechanism available. The rejected impurity mol- Equation (6.3.110b) may therefore be rewritten as
ecules develop a concentration profile, which decreases
from a high of uil/o (Figure 6.3-15(a)) at the freezing inter- uil uil 2 uil
face to the lower value of uilb (<uil/o) in the bulk of the vf r s Dil 2 : 6:3:110c
t l z z
molten zone at a distance l from the freezing interface.
The l is identified as the boundary layer thickness (Burton This equation may also be developed by considering
et al., 1953). As a result, the impurity concentration in the the rates at which different methods of introduction and
refrozen zone of thickness dz is higher (than what it would withdrawal of impurity i take place across the boundaries
have been if the zone were completely mixed) if we assume of the differential molten element at z and z dz (Figure
it to be in equilibrium with the impurity concentration in 6.3.15(b)) inside the molten zone:

convective diffusive
z}|{ z}|{ rate of
2 3 2 3
introduction removal introduction removal accumulation
6 z}|{ z}|{ 7 6 z }| { z
}| {
6     7 6 ! 7 7 z}|{
6 7 6 u u 7
6A vf r s
uil

 A vf r s
uil 7 6 il il 7 uil Az :
6 7 6 Di A z  Di A z 7
t
6:3:111a
6 zdz 7
z5 6 z zdz 7
4 4 5

the melt at the freezing interface. The nature of this impur- Dividing this equation by Alz and taking the limit
ity concentration profile (Figure 6.3.15(a)) will depend on z ! 0, we obtain
the impurity partition coefficient, the impurity diffusion  
coefficient in the melt and the velocity of zone travel, uil 2 uil uil
vf r s Dil 2 : 6:3:111b
among other things. A solution of the impurity mass- l z z t
transfer problem in the molten zone will now be developed
The boundary conditions and the initial condition needed for
(Wilcox and Wilke, 1964; Wilcox, 1967).
the solution of equation (6.3.110c) will now be developed.
Consider now a differential element of the molten zone
Consider the mass-transfer rates at the freezing interface and
of thickness z at location z (Figure 6.3.15(b)). There are no
the melting interface at the two ends of the molten zone in
chemical reactions and/or external forces present in the
the rod in a coordinate system fixed on the heating band
molten zone. Assume a plane interface of area A at both z
(Figure 6.3.16). At z 0 (the freezing interface),
and z dz; further, assume that there are no convection and
concentration variations in the x- and y-directions at any z. If uil
vf r As uil vf r As uis  Dil Al
we assume that the mixture has a constant total mass density z
s uil
t (valid for a dilute mixture), we get (from Table 6.2.2, ) vf r uil  uis Dil : 6:3:112a
l z
equation (6.2.5p), Tables 3.1.3B and 3.1.3C, v v, Ui 0
and Ctl constant) Also, at z 0,
2 uis
il
vz il Dil 2il : 6:3:110a is0 , 6:3:112b
t z z uil
6.3 Bulk flow parallel to force direction 407

Heating band

uil vfr Ars uis


- Dil Arl
z
vfr Ars uis
vfr Ars uil Recrystallized
vfr Ars uil solid
Melting uil Freezing
- Dil Arl
interface z interface
z=l z=0

Heating band

Figure 6.3.16. Different mass fluxes of the impurity at the freezing interface and the melting interface in zone melting in a frame of
reference moving with the heating band.

where is0 is the impurity distribution coefficient. This u


il du
quantity is0 is related to that based on molar concentration, at 0, u
il  uis   il ; 6:3:116a
d
namely is , by

is0 l is : 6:3:112c lvf r s uis u du
s at , u
il   il  il : 6:3:116b
Dil l uio d
At z l (the melting interface),
uil
vf r As uil vf r As uis  Dil Al ) vf r s uil  uis We can conveniently assume that uis at z l is equal to uio,
z l
the initial impurity concentration. Therefore, the second
uil boundary condition (6.3.116b) is changed to
Dil : 6:3:112d
z
At t 0,
uil uio , 6:3:112e lvf r s u
il du
at , 1  u
il il : 6:3:116c
Dil l d
where uio is the initial impurity concentration in the rod.
An exact analytical solution of the above set of equa-
tions is available for the case of a constant distribution Since we have assumed a pseudo steady state, the initial
coefficient (Wilcox and Wilke, 1964). It is quite tedious. condition (6.3.112e) is no longer needed. A general solu-
For the sake of illustration, we will take a simplified tion for the ordinary differential equation (6.3.115) is
approach. If we assume a pseudo steady state condition, u
il c 1 c 2 exp: 6:3:116d
equation (6.3.110c) is reduced to
To determine the constants c1 and c2, employ the two
uil 2 uil boundary conditions at 0 and (lvfrs/Dill):
vf r s Dil 2 0: 6:3:113
l z z
(1) At 0, from equation (6.3.116a),
c 2 exp c2 c 1 c 2 exp0  is0 c1 c2 exp0
Following Wilcox and Wilke (1964), we define the following
nondimensional quantities: c1 c 2  is0 c1 c 2 : 6:3:116e
uil zvf r s
u
il ; : 6:3:114 (2) At (lvfrs/Dill) l, from equation (6.3.116c),
uio Dil l

In terms of these nondimensional quantities, equation 1  c 1  c 2 expl c 2 expl ) c 1 1: 6:3:116f


(6.3.113) is reduced to
Substituting this value of c1 into relation (6.3.116e), we get
u
il 2 u
il du d2 u
0 il il
: 6:3:115
2 d d2 c2 1 c2 1  is0 ) c 2 1  is0 =is0 : 6:3:116g

The boundary and initial conditions are changed as The required solution for the impurity mass fraction profile
follows: in the molten zone is then given by
408 Open separators: bulk flow parallel to force and CSTSs

 
uil 1  is0 zvf r s 10
u
il 1 exp  : 6:3:117 kis = 10
uio 0
is Dil l
5
2
1.5
Note that at z l (the melting interface), uil 6 uio; however,
1.0
if (lvfrs/Dill) < 0.1, then uil uio =is0 , corresponding to 1.0

Effective distribution coefficient, kis,eff


0.9
an almost completely mixed molten zone. In such a case, 0.5
the velocity of the interface movement due to the heater
motion is quite low compared to the rate of diffusive
dispersal/mixing of the impurities: a maximum amount 0.1
0.1

of purification takes place in the solid rod. When (lufrs/


Dill) > 5, uil is essentially equal to uio; no separation/
purification takes place. Thus, the above solution provides kis = 0.01

a perspective on the practical range of interface velocity/


0.01
heater velocity to achieve separation in zone melting/refin- Equilibrium distribution
coefficient,kis = 0.001
ing, given the zone length and the mixing rate as provided
by Dil. Although bulk motion parallel to the force direc-
tion is essential for purifying the solid rod, too rapid a
bulk motion compared to the rate at which impurities 0.001
0 1 2 3
rejected at the phase interface are transported away (dlvfr/Dil) (rs/rl)
from the phase interface is counterproductive. In fact,
at higher velocities, separation is lost. Figure 6.3.17. Effect of parameter (lvfr/Dil)(s/l) on effective dis-
In the zone melting processes, a more likely achieved tribution coefficient for partial liquid mixing in molten zone,
condition is illustrated via the impurity concentration profile according to Eq. (6.3.119a.) (After Wilcox (1967).)
in the molten zone in Figure 6.3.15(a): the impurity mass
fraction uil changes from the value uil jo at the freezing inter-
face (where it is assumed to be in equilibrium with the solid
molten zone near the freezing interface. Correlations for
mass fraction uisjo in the newly frozen section ABB0 A0
estimating the boundary layer thickness, l, are reviewed
via uisjo is0 uiljo ) to uilb, the impurity mass fraction in the
in Wilcox (1967). Typical values of parameters and transport
bulk of the melt at a distance l (usually << l), the boundary
and other coefficients are as follows: vfr ~ 2 mm/hr 2 mm/
layer thickness. Ideally, if the complete molten zone is well
min; Dil ~ 25  105 cm2/s; l ~ 0.02 0.1cm; l ~ 12 cm.
mixed, then
How the value of is0 ,eff differs from is0 as a function of the
uis jo is0 uilb : 6:3:118 parameter (lvfrs/Dill) is illustrated in Figure 6.3.17 for a
variety of values of is0 , the equilibrium impurity distribution
In the more common cases of partial liquid mixing with a coefficient. The dimensionless ratio vfr/(Dil/z), or vfr/(Dil/
boundary layer present near the freezing interface, an l), where (Dil/l) may be defined as a mass-transfer coeffi-
effective partition coefficient, is0 ,eff is defined as cient, appears to be a key parameter in zone melting, and,
we shall see elsewhere in this chapter, in other separation
uil jo techniques where the bulk flow is parallel to the direction of
is0
uis jo is0 uil jo uio the force acting on species i.
is0 ,eff uilb
uilb uilb In zone melting, if the partial liquid mixing model can be
uio employed, then the impurity mass fraction in the refrozen
  solid may be obtained from expression (6.3.109h) for the
1  is0 totally mixed case by replacing is0 by is0 ,eff :
is0 1
is0  
  uis z
1  is0 zj vf r s 1  1  is0 ,eff exp is0 ,eff s : 6:3:119b
1 exp  b uio l l
is0 Dil l
The amount of purification achieved in a single pass of
is0
) is0 ,eff  , 6:3:119a the heating band over the rod is insufficient in many applica-
l vf r s
is0 1  is0 exp  tions. The heating band is generally passed over the rod
Dil l length a number of times; each pass purifies the bulk of the
rod length substantially and concentrates the impurity at the
where we have used (6.3.117). This quantity is0 ,eff is always finishing end. Figure 6.3.18 illustrates how the impurity mass
larger than is0 , as long as is0 < 1. Therefore uis is higher in fraction, uis, normalized by the starting impurity mass frac-
the newly frozen solid due to inadequate mixing in the tion, uio, varies in the solid rod from the starting end (z 0) to
6.3 Bulk flow parallel to force direction 409

the finishing end (z L) for the case of is0 ,eff e 0:1 as the Results in Figure 6.3.18 highlight the extraordinary levels of
number of heating passes increases. After an infinite number purification achieved by multiple passes. In fact, ultrapurity
of heating passes, the ultimate distribution of impurity mass has often been a must in the semiconductor industry; frac-
fraction, uis , is achieved. This distribution cannot be tional solidification techniques are frequently employed
improved any further by additional heating passes: from a (Zief and Wilcox, 1967).
practical point of view, this implies the existence of the We will now briefly consider the normal freezing pro-
following relation between N and N 1 passes: cess; it is also sometimes called controlled progressive freez-
ing. As illustrated in Figure 6.3.14(c), the molten rod is
uN N 1
is uis . . . uis : 6:3:120
being pulled out from the heated zone; the section of the
An analysis of the ultimate distribution is available in rod which has been pulled out is frozen, while the rest
Wilcox (1967): remains in a molten state. The velocity of the freezing

surface may be assumed to be vfr. In a frame of reference
uis =uio A expBz : 6:3:121
fixed in the refrozen solid being pulled out, the freezing
The constants A and B are obtained from interface has a velocity vfr in the opposite direction, toward
is0 ,eff Bl=expBl  1; 6:3:122a the molten part of the rod. Let the total length of the rod be
L. If a length z of this rod is out of the heated zone and is
A BL=expBL  1: 6:3:122b solidified at any time t, then, for a rod of uniform cross-
sectional area, the weight fraction uz of the molten rod
102
which is solidified is uzL z; correspondingly, the weight
fraction 1uz of the rod still remains molten. Employing an
101
impurity mass balance in a section of length dz, which is
Original composition freezing (Figure 6.3.19A), we get, by equating this gain to
Relative concentration, uis / uio

1
First zone pass the solid phase the loss from the molten phase,
10-1 2nd
uis As dz duilb AL  zl ; 6:3:123a
3rd
-2
10
4th uis s dz l L  zduilb uilb l dz

10-3 5th , uilb l  uis s dz l L  z duilb ;


6:3:123b
10-4 6th
uis z uz
l duilb dz duz
-5 7th  ln1  uz :
10 uilb l  uis s L  z 1  u z
uis jz0 0 0
8th
10-6 6:3:123c
9th
Ultimate distribution
From studies in zone melting, it is known that models
10-7
0 0.2 0.4 0.6 0.8 1.0 of partial liquid mixing are more realistic, and the effective
Relative position, z / L partition coefficient approach of equation (6.3.119) is quite
useful. In this approach,
Figure 6.3.18. Calculated concentration profiles for multiple zone
uis
passes with is0 ,eff 0:1, l/L 0.2. (After Wilcox (1967).) uilb : 6:3:123d
is0 ,eff

Heater L

1 - uz Recrystallized solid

Molten Direction of
vfr
solid motion of rod
dz z

uz
Heater

Figure 6.3.19A. Schematic of the normal freezing system for impurity distribution analysis.
410 Open separators: bulk flow parallel to force and CSTSs

Further, the first solid to come out at z 0 will have an freeze must be removed before the sample can be subjected
impurity mass fraction to another pass, since, during the remelting of the sample,
e.g. rod, the impurities will be homogenized. Zone melting
uis jz0 is0 ,eff uio , 6:3:123e does not suffer from such a problem. Further, in normal
where uio is the impurity mass fraction in the original rod freezing, the molten state exists over a large length of the
(it is also the molten rod composition). Assuming further rod for a long time; therefore, evaporation, decomposition,
that s l , we get etc., can take place easily. However, normal freezing is often
the method of choice for bulk crystal growth, for example of
uis
duis  z silicon and gallium arsenide: the method is identified as the
ln 1  uz ln 1  Czochralski technique. A seed crystal is suspended into a melt
uis is0 ,eff  1 L
uis jz0 from a shaft, which is rotated as well as moved vertically
uis upward. The temperature is lowered, the crystal growth initi-
1  uz is,eff 1
0
)
uis jz0 ates and the shaft is moved upward. There is no impurity
uis separation achieved in this process. For an introduction to
is0 ,eff 1  uz is,eff 1 ;
0
) 6:3:124a
uio and brief analysis of how this technique distributes dopant
through the crystal, see Lee (1990).
uis  z is0 ,eff 1 The zone melting/normal freezing processes described
is0 ,eff 1  : 6:3:124b
uio L so far have been batch processes: one solid rod/ingot was
used in a device for a period of time needed to achieve the
An alternative approach of formulating the differential
requisite purification. To purify material at a large rate, con-
mass balance equation (6.3.123a) would be to use uz
tinuous zone melting may be undertaken, thus avoiding
instead of z as the independent variable:
either the necessity of a much larger size batch system or
uis duz duilb 1  uz : 6:3:125 the handling of many batches. The basic structure of a
continuous zone melting device is shown schematically in
This formulation has an advantage in that we do not have to Figure 6.3.19B. Molten feed is introduced near the middle of
assume a constant cross-sectional area (therefore, uzL 6 z). a column into, for example, a feed hopper, which is con-
The result (6.3.124a) is still obtained. Note that both results nected on one side to the enriching section and on the other
(6.3.124a) and (6.3.124b) are not valid as z ! L or uz ! 1. side to the stripping section. The feed is a source of the pure
Richman et al. (1967) have pointed out the following material for the enriching section, at the end of which the
advantages of normal freezing over the zone melting purified solid is obtained. The purified solid is withdrawn by
technique: melting at the end. There is a heating band which traverses
the enriching section it keeps on purifying the material
(1) larger-scale operation is easier;
as the band moves toward the feed section. At the feed
(2) mechanical stirring of the melt is easier;
section, the impurities from the enriching section are
(3) the purification achieved in one complete pass is
rejected. In practice, there may be a number of heating bands
higher than that in zone melting.
to increase the rate of purification; a particular heating band
Problems 6.3.14 and 6.3.15 illustrate that the purification may traverse only part of the length at the enriching section.
achieved in a single-pass zone melting process is less than The feed flow rate introduced at the feed hopper is split
that achieved by a normal freezing process. into two parts: the purified product flow rate at the purified
There is, however, one major disadvantage, namely that product exit and the waste flow rate at the waste exit end of
the impurities collected at the end of the rod that are last to the stripping section. The heating band over the stripping

Material flow
Additional
Impurity flow
heater to melt
Enriching section Stripping section
purified crystal
Heater Heater Additional heater
to melt waste
Product exit
Waste exit
Heater Heater
Feed

SOLID LIQUID Direction of travel of heaters

Figure 6.3.19B. Schematic of the principle of a continuous zone refining device.


6.3 Bulk flow parallel to force direction 411

section moves toward the waste exit and carries the impur- (1971), Keey (1972) and Porter et al. (1984); additional treat-
ities from the feed section toward that end. Thus, the impur- ments are available in textbooks on mass transfer and separ-
ities rejected from the enriching section are taken down the ation; see Treybal (1980), Belter et al. (1988) and Geankoplis
column into the stripping section. The actual mechanisms (2003).
employed to move the heating bands, materials and the Porous solids, whether they are food materials, non-
purification process, including those developed by W.G. food biological materials, pharmaceutical products, etc.,
Pfann, are illustrated in Moates and Kennedy (1967). often contain a significant amount of moisture. The
Fractional solidification processes have been employed removal of this moisture is necessary to reduce/eliminate
to separate/purify a variety of mixtures besides semicon- biological activity. Depending on the material, drying can
ductor materials. These include: be carried out at higher temperatures or at a low tempera-
ture under freeze-drying conditions. The process of freeze-
purification of water, as in desalination by freezing drying involves freezing the material first by exposure to a
(Orcutt, 1967), since it is well known that ice, which is
very cold air and then subjecting it to vacuum to sublimate
pure water, is obtained when saline water is frozen;
the moisture. Heating of the porous solid may be carried
purification of organics like naphthalene by the Proabd out (1) by circulating hot gas (e.g. air), (2) by conduction
Refiner (Molinari, 1967), which consists of controlled
from a hot metal plate on which the porous solid is kept,
cooling of the feedstock in a static apparatus, leading to
(3) by radiation from hot surfaces as well as from hot gases
progressive solidification on the cooling surface of the
or (4) by the combination of all three methods. The mois-
refiner followed by controlled heating which melts the
ture is removed from the porous solid by evaporation
impure fractions the earliest;
when drying takes place. Circulating hot gas removes the
ultrapurification of pharmaceuticals; moisture; alternatively, a vacuum removes the evaporated
purification of aluminum ingots on a large scale by moisture. We will focus here on vacuum driven processes
normal freezing.
only since the moisture bulk flow is parallel to the direction
Additional details are provided in Zief and Wilcox (1967). of the chemical potential gradient driven force, causing
evaporation of the moisture. Such a process is called
6.3.2.4 Solidvapor systems: drying
conduction drying or vacuum shelf drying.
We will focus here essentially on the removal of moisture Consider a wet porous solid bed on a tray heated from
from solids, porous solids, etc., therefore on the drying of the bottom by hot water, steam, etc. (Figure 6.3.20(a))
solids. Drying, however, has a broader connotation; the (Belter et al., 1988). From the top surface, where z L,
following are treated also as examples of drying: removal of the thickness of the solid bed, moisture is removed by
trace amounts of moisture from organic liquids (usually vacuum. At any time, at a particular value of z (<L) any
carried out by distillation in a column, Section 8.1.3; also by water present will be evaporated. Below this value of z
pervaporation, Section 6.3.3.4); removal of small amounts of (<L), the porous solid is wet; at larger values of z, the
moisture from gaseous stream via adsorption (Section porous solid is dry. As time progresses this z-value
7.1.1.2), membrane gas permeation (Sections 7.2.1.1 and becomes slowly smaller and smaller, and ultimately should
8.1.8). We restrict ourselves here to a brief treatment of become zero.
moisture removal from porous solids, only where there is To determine how z varies with time, and ultimately
bulk flow parallel to the force direction, namely vacuum the time tdrying needed to dry the whole bed on the tray,
drying. A general treatment of drying is available in King consider the rate of conductive heat transport per unit

(a) (b)
Vacuum 30
Equilibrium water content
kg H2O / 100 kg dry solid

z=L 24

z
} Dried porous solid 18
Bound water

z=0
qz
} Wet porous solid
12

6
Conductive heat flux
qz 0
Tray Steam or hot gas 0 100
Percent relative humidity

Figure 6.3.20. (a) Conduction drying of porous wet solids on a plate/tray heated from below and subjected to vacuum at the top surface.
(b) Typical equilibrium moisture content behavior vs. percent relative humidity.
412 Open separators: bulk flow parallel to force and CSTSs

cross-sectional area, namely the heat flux qz between the Kelvin effect, see equation (3.3.112e)); hydrated crystals, etc.
heated plate surface at temperature Tpan (where z 0) and Biological materials such as cells typically possess significant
the temperature at location z, Tz: bound water; wood and food items fall into this category also.
A quantitative description of the mass of water per unit
km

q z Ab Ab T pan  T z : 6:3:126 mass of dry solid is provided by plotting it against a quan-


z
tity called percent relative humidity. To understand this
Here km is an effective thermal conductivity of the material quantity, consider air in equilibrium with liquid water.
bed being dried and Ab is the cross-sectional area of the bed. The partial pressure of water vapor in the air, in equilib-
The rate at which the drywet interface location coordinate rium with liquid water at any temperature, is the vapor
z changes is very low. We may therefore assume a pseudo pressure of water, Psat sat
water . The saturation humidity H water is
steady state condition. This heat flux will evaporate water; defined as the mass of water vapor per unit mass of air
correspondingly, the drywet interface location will change present at a total pressure P:
and the volume of the wet bed, zAb, will decrease. The rate of
P sat 18
heat transfer from the plate bottom must equal the rate of H sat
water
water
 , 6:3:131
P  P sat
water 29
evaporation of the local moisture in the bed:
where 18 is the molecular weight of water and 29 is the
d
qz Ab H2 O,bed wb zAb : 6:3:127 molecular weight of air; the partial pressure ratio provides
dt
a ratio of the moles of water to moles of air. If the air is
Here wb is the mass of water per unit volume of the bed, drier than that at the saturation humidity, the humidity
H2 O,bed is the enthalpy of vaporization of water per unit Hwater is defined as
mass of water. Combine equations (6.3.126) and (6.3.127)
pwater 18
to obtain H water  , 6:3:132
P  pwater 29
dz km
1 dz 2
z T pan  T z : 6:3:128 where H water < H sat
water . The relative humidity is defined as
dt H2 O,bed wb 2 dt

H rel sat
water H water =H water , 6:3:133
The initial condition for this drying process corresponds to
the whole bed being wet, i.e. the location z L at t 0. with percent relative humidity being 100 H rel water .
Integrate and use this initial condition to obtain The equilibrium moisture content of many materials of

interest, such as various food items, wood, etc., are described
2 2 2k m T pan  T z as a function of the percent relative humidity, as illustrated in
z L  t: 6:3:129
H2 O,bed wb Figure 6.3.20(b) for a hypothetical system. The level of bound
water in this system at 100% relative humidity is 18 kg H2O/
The material in the pan becomes completely dry when
100 kg dry material. If the total level of water in this sample is
z 0 at t tend:
26 kg H2O/100 kg dry material, then the amount of free water
H2 O,bed wb

L2 : at 100% relative humidity is 8 kg H2O/100 kg dry material.
t end 6:3:130
2k m T pan  T z Similarly, one can develop estimates of bound water and free
water at other levels of relative humidity.
The above analysis ignores any sensible heat contribution as
the temperature profile changes in the wet section of the
6.3.3 Filtration and membrane separation processes
bed and any temperature profile in the dry region. However,
one of the more complex items in this analysis is the mass of We illustrate here those configurations of filtration and
water per unit volume of the bed (wb ). important membrane separation processes where the bulk
The mass of water per unit volume of the bed, or altern- flow of the liquid, solution/suspension or gas mixture is
atively the mass of water per unit mass of dry solid, consists parallel to the direction of the force driving the separation
basically of free water (free moisture) and bound water through the membrane/filter. The force consists primarily
(bound moisture). Free water is ordinary water existing in a of that due to the chemical potential gradient; almost
porous solid such that it has the same vapor pressure as that invariably there is a pressure gradient across the mem-
of water; therefore it can be evaporated. Further, it is mobile brane or the filter. Generally, separation of particles larger
in the porous solid structure, obeys capillarity (described in than 10 m by a filter cloth/medium is identified as
Section 6.1.4) and therefore could come up to the gassolid filtration. Particles/colloids of sizes less than 10 m but
(airsolid) surface. Bound water present in the porous solid, larger than 0.02 m when separated by a membrane are
however, has a vapor pressure lower than that of free water; it said to undergo microfiltration. These two processes
is not as mobile and has a higher heat of vaporization. Typical are considered first. Next we illustrate ultrafiltration,
examples are: moisture present within cells, biological or where solutes and macrosolutes having molecular weights
otherwise; water in very small-diameter capillaries (the between 1000 and 500 000 are separated by an ultrafilter;
6.3 Bulk flow parallel to force direction 413

Piston

Flow direction

Pf
Force direction

Slurry Particles

Pf - Pp = P
Cake
Membrane/filter/cloth
Bulk Flow
dir. Pp < Pf

Filtrate

Figure 6.3.21. Deadend filtration: bulk flow parallel to the direction of the force due to applied P Pf Pp.

colloids having similar dimensions are also separated by are carried out in the manner shown in Figure 6.3.21, as if a
ultrafiltration. The process of reverse osmosis separating piston is driving the particle suspension in the fluid toward
solutes of molecular weight25 up to 150200 is briefly con- the membrane/filter/cloth of a certain cross-sectional area in
sidered next. Separation of volatile solutes from a solution a vessel having the same cross-sectional area, we have bulk
through a dense membrane subjected to a vacuum on the flow of the particle-containing fluid parallel to the direction
other side of the membrane may be carried out by the of the applied force created by the pressure difference.
pervaporation process described subsequently. Finally, Let us focus first on cake filtration and microfiltration for
the separation of a gas mixture through a dense nonporous the case where the fluid is a liquid. In the configuration of
membrane is described. Figure 6.3.21, the technique is called deadend filtration. The
same configuration is routinely employed in laboratories
with a filter paper on, say, a Bchner funnel and a partial
6.3.3.1 Deadend cake filtration/microfiltration
vacuum on the side of the permeate/filtrate: a precipitate/
In the introduction to Section 6.3.1.4, we introduced the deposit builds up quickly on the filter paper as the slurry is
notion of sieving or surface filtration, wherein the fluid under filtered. As time passes, a particle based deposit continues to
pressure passes through the pores of a relatively thin mem- build up on the filter paper: it is called a cake. This cake
brane/filter/cloth whose pores/openings are smaller than provides an additional resistance to the flow of the filtrate
the particles. There is an applied pressure difference (P) through the membrane/filter/cloth in deadend filtration. As
between the two sides of the filter/membrane, the feed side time passes, deposition of the particles onto/in the cake
being at a higher pressure. Particles which cannot pass continues. Therefore the resistance to the flow of the filtrate
through the membrane/filter/cloth are deposited on it; the increases with time. If one wants to maintain a constant value
deposited material forms a cake of sorts, a porous bed, a of the filtrate flux, the applied pressure difference P has to
packed bed of particles. When the particle sizes range increase with time. Alternatively, for a constant applied pres-
between 0.02 and 10 m, the process is called microfiltration. sure difference, the flux of the filtrate will decrease with time
For larger particles, as well as particles in this size range, the (Figure 6.3.22).
process is generally called cake filtration. When the processes For both modes of filtration operation in Figure 6.3.22,
one would like to know the magnitude of the filtrate flux for
a given P, or vice versa for a given slurry-filter system or a
25
suspension-filter system. We will find that the nature of the
Solutes of molecular weight between around 150 and 1000 are
cake or deposit formed on the membrane/filter/cloth is
usually separated by nanofiltration; however, reverse osmosis
may be used at the low end of this range, while ultrafiltration often crucial to determining the flux level. Sometimes, as
may be used at the very high end. There are no sharp in the case of filtration of fermentation broths, the cake
boundaries. formed is highly compressible and filtration rates are quite
414 Open separators: bulk flow parallel to force and CSTSs

(a) (b)

Filtrate
Filtrate P
flux
P flux

Time Time

Figure 6.3.22. Two basic models of carrying out deadend filtration: (a) constant flux mode; (b) constant P mode.

Pf - Pp = P
Suspension
Amt
Filter
cake Pf V(t)

m Rc s
c Slope =
Membrane / m 2 (c - s) P
filter/cloth (mRm /P)
Pp

Filtrate (a) V(t)/Am (b)

Cake compression

log Rc
Time, t

Cake formation
Slope = s

Cake compression
Cake formation

log P (c) (Filtrate volume)2 , [V(t)]2 (d)

^ c by
Figure 6.3.23. (a) Cake filtration, bulk flow parallel to force, schematic with a cake of thickness c. (b) Determination of Rm and R
plotting (Amt/V(t)) against V(t)/Am for an incompressible cake. (c) Determination of s for a compressible cake described by equation
(6.3.138j). (d) Traditional cake filtration (solid line); nontraditional filtration behavior (dashed line).

low. To facilitate filtration, filter aids are often added to Qsm P P cm3
increase the filtration rate by developing a more porous/ vs volume flux N s V s :
m Rm cm2 -s
permeable cake. Filter aids are solid inert porous materials
of fine size, such as diatomaceous earths, perlites, asbestos,
Note: The quantity N s V s is also equal to the solvent vel-
etc. They increase the cake porosity and also make it much
ocity vs (cm/s) through the filter (compare equations
less compressible. They are used especially in the filtration
(3.4.60df) and (3.4.85)). At any instant of time, both the
of certain types of fermentation broths.
filter and the cake on top of it provide resistances to flow:
Consider Figure 6.3.23(a), which shows a membrane/
Rm for the filter and Rc for the cake. If the pressure drops
filter/cloth of thickness m, on the top surface of which a cake
through the membrane/filter and the cake are, respect-
of thickness c has been formed at time t (starting time t 0).
ively, Pm and Pc, then
We have observed in Section 3.4.2.3 that the volume flux
of the liquid through a porous medium may be described
by Darcys law (equation (3.4.88), repeated here for Qsm P m Qsc P c P c
vs N s V s , 6:3:134
convenience): m c Rc
6.3 Bulk flow parallel to force direction 415

where Qsm and Qsc and the permeabilities of the mem- Since the surface area of a particle in the cake per unit
brane/filter and the cake layer, respectively. Since P, the particle volume, avc , is related to dp by
applied pressure difference, is equal to P m P c ,
4 d 2p =4 6
where Pm is the pressure drop through the membrane avc
3 , 6:3:135j
and Pc is that through the cake, we get 4=3 dp =2 dp

vs
P
; 6:3:135a 21  c 2 a2vc c a1 1  c 2 a2vc c
Rm Rc  Rc , 6:3:135k
3c 3c
Rm m =Qsm , Rc c =Qsc : 6:3:135b a form quite similar to that of expression (6.3.135h) for Rm.
This representation is similar to that in Ohms law in From equation (6.1.4f) for flow in a packed bed of particles
electricity: (the BlakeKozeny equation), the constant 72 in equation
(6.3.135i) should be replaced by 150. Correspondingly, the
applied potential difference here P value of a1 ( 2) will be increased by a factor of (150/72).
current here velocity ,
resistance ^ c , the
Two other quantities are often used instead of Rc : R
where the cake resistance and the membrane (filter) resist- specific cake resistance, which is the cake resistance per
ance are in series. For a membrane/filter having a unit cake thickness, and R ^ cw , the specific cake resistance
cylindrical pore size distribution f(rp) and Poiseuille flow, per unit mass of the cake,
we obtain, from equation (3.4.86),
^ c Rc ;
R ^ cw Rc ,
R 6:3:135l
8 m c wc
Rm : 6:3:135c
m r 2p where wc is the mass of cake per unit area of the filter/
If the membrane/filter has Np pores, each of size rp, per membrane and is related to c for a mass density s of the
unit area, and the membrane porosity is m, then solids by

8 m wc s 1  c c : 6:3:135m
m N p r 2p , Rm : 6:3:135d
N p r 4p Consider now Figure 6.3.23(a). At any instant of time t, if Vp
Sometimes, such a cylindrical pore based representation is is the total volume of filtrate/permeate collected over the
avoided; instead membrane porosity, m, and pore surface time period 0 to t seconds, then, for a membrane/filter area
area/membrane volume, sm, are used. For cylindrical pores of Am, the filtration volume flux vs is
of constant size rp, 1 dV p P=
vs N s V s : 6:3:136a
N p 2 r p m Am dt R m R c
sm N p 2 r p : 6:3:135e
m Alternatively it may be written as
If either membrane pore volume based, smp , or the mem- 1 dV p P
brane solids volume based, sms , is used, then vs N s V s
: 6:3:136b
Am dt ^ c c
Rm R
N p 2 r p m 2
smp ; 6:3:135f To determine the filtration flux vs, we need to know c
N p r 2p m rp
how it changes with time as the cake layer builds up (Davis
N p 2 r p m 2 r p N p and Grant, 2001). To develop an equation for c with time,
sms : 6:3:135g we need to make a mass balance for the solids in the
m 1  m 1  m
slurry/suspension being filtered. Define the following
(Note: smp s in equation (3.3.89b).) In terms of sms and m, quantitites:
Rm may be expressed, from (6.3.135d), as
s volume fraction of solids in the suspensions being
2 1  m 2 s2ms m filtered;
Rm : 6:3:135h
m 3 c volume fraction of solids in the cake being formed.
In practice, the constant 2 in this equation may be replaced The rate of solid deposition in the growing cake per unit
by other empirically estimated values. membrane area is given by
The liquid flow resistance of the cake of deposited
particles of diameter dp on the membrane, Rc, may be dc solids volume change in cake volume
c  :
estimated from equation (6.1.4e) for a cake porosity of c as dt cake volume change in time  membrane area

The rate of solid deposition in the growing cake per unit


72 1  c 2 c
Rc : 6:3:135i membrane area from the production of the filtrate and the
d2p 3c
cake volume growth is given by
416 Open separators: bulk flow parallel to force and CSTSs

   
dc filtrate volume growing cake volume total suspension volume solids volume
vs s  :
dt membrane area  time suspension volume

Equating these two terms, we obtain (Davis and Grant, 2001) Integrate this equation for c to obtain
  2
dc dc ^ c c Rm c  s P
c vs s , R t 0, 6:3:138b
dt dt 2 c  s

which may be rearranged to yield with c 0 at t 0.


The solution of the quadratic equation provides an
dc vs s expression for c t :
, 6:3:136c
dt c  s q

Rm R2m 2R ^ c s Pt=c  s


where c 0 at t 0. Solution of equations (6.3.136b) and c t :
(6.3.136c) will provide an estimate of how vs changes with ^ c
R
time for a constant P and growing c , or how P changes
Take the positive root for a meaningful c t :
with time to maintain a constant vs. Integrating equation " #

(6.3.136c) with respect to time t will lead to the following
^ c s P t 1=2
2R
^
c t Rm =R c 1 1 :
relation between the volume of filtrate V p t collected in c  s R2m
time t through a filter of area Am:
6:3:138c
 
V p t =Am c  s =s  c t : 6:3:136d
Substitution into equation (6.3.136b) provides an expres-
Consider first the case of constant filtration flux in cake sion for the time-dependent filtration rate/volume flux for
filtration/microfiltration. Since the volume flux vs is constant P:
constant,  
P ^ c s P t 1=2
2R
vs 1 2 : 6:3:138d
1 dV p Rm c  s Rm
vs constant vso ) V p t vso Am t:
Am dt
Figure 6.3.22(a) shows that, for constant P, the filtration
6:3:137a
flux decreases. Substitution of expression (6.3.138c) for
From equation (6.3.136c), it follows that c t in expression (6.3.136d) yields
  "  #
d c vso s vso s c  s R m ^ c s P t 1=2
2R
) c t t, 6:3:137b V t=Am 1 1 ,
dt c  s c  s s ^ c
R c  s R2m

where we have assumed that c is constant, i.e. the cake is 6:3:138e


incompressible. From equation (6.3.136b), the pressure which illustrates how the filtrate volume V(t) changes with
drop P will now increase linearly with time for a constant time for constant P filtration.
filtration flux vs, as shown in Figure 6.3.22(a): The membrane resistance Rm and the specific cake
  resistance R ^ c are of interest here. They are not known in
P t vso Rm R^ c vs s t : 6:3:137c general. To determine them, consider the quadratic equa-
c  s
tion (6.3.138b) for c and the expression (6.3.136d) for
On the other hand, if the applied pressure drop P is held c t . Substitution of (6.3.136d) for c t into equation
constant with time, we have to recognize that the volume (6.3.138b) results in
flux, or filtration velocity vs, will decrease with time as the
cake builds up. Introducing expression (6.3.136b) for vs in ^ c
R V 2 t 2s V t s s P
Rm t:
(6.3.136c), we obtain 2 A m c  s 2
2 Am c  s c  s

dc s P

Multiplying both sides by
dt ^ c c
c  s Rm R
 

^ c c dc

s P c  s Am
Rm R : 6:3:138a
dt c  s P s V t
6.3 Bulk flow parallel to force direction 417

leads to Slide
Fixed Frame head
Filter
    head
Am R^ c s V t Rm media Plate
t : 6:3:138f
V t 2c  s P Am P Filtrate

In any particular case, if one plots (Am/V(t))t against (V(t)/


Am) (since V(t) at time t is known for a given filter area), the
intercept on the ordinate will yield Rm =P , and there-
fore the membrane resistance Rm; the slope will yield

^ c s =2c  s P, which, for known values of s, c,


R Feed
slurry
P and , will yield the specific cake resistance R ^ c (Figure Slide
bar
6.3.23(b)).
It is useful to consider some limiting forms of the Fixed Slide
head plate head
general flux decline and cake buildup behavior described plate
above. From the expression (6.3.138d) for filtration flux
decreasing with time, it appears that, as t becomes large,
Figure 6.3.24. Plate and frame filter press (http://www.mine-
vs / t 1=2 :
engineer.com).
 1=2
c  s P
vs : 6:3:138g
^ c s t
2R
(Figure 6.3.23(d), dashed line). This behavior is called
Correspondingly, from expression (6.3.138e) for the filtrate nontraditional filtration behavior under constant pressure
volume, V(t) varies with (t)1/2 as t becomes large, as does conditions. A review of the model and the mathematics of
the cake thickness c t (from expression (6.3.138c)): analysis of nontraditional filtration behavior is provided in
 1=2 Strickland et al. (2005), along with references to earlier
2c  s Pt model-building activities.
V t=Am ; 6:3:138h
^ c
s R Deadend filtration with larger particles has often been
carried out using the so-called plate and frame filter press
 1=2
2 s Pt shown in Figure 6.3.24. It consists, for example, of a set of
c t : 6:3:138i
^ c
c  s R plates with recessed space between the neighboring plates,
creating a chamber for the feed suspension to come in, and
The preceding development was based on the assumption a cake, formed on a cloth/canvas on each plate, acting as a
^ c , as well as Rm, were independent of P. It has
that R filter. The suspension enters each chamber through one
often been found, however, that R ^ c depends on the corner of the assembled plates; the filtrate passes through
applied P. This can be true for deposits of inorganic the canvas/filter and goes out of the device. Before filtra-
precipitates as well as cells and cell debris encountered tion, the plates are pressed together and sealed via gaskets
in microfiltration or filtration of fermentation broths when pressed by a hydraulic ram/screw. After filtration is
^ c is then represented as
and lysates. Conventionally, R over, the press is opened and the cake recovered from the
^ c s c R
^ cw ; canvas/filter. (Often before such a process, a suspension-
R
free liquid feed is introduced to remove soluble substances
^ c s c 0 P s :
R 6:3:138j from the cake.) Then the filter press is ready for another
cycle of operation.
A logarithmic plot of R ^ c against P in a log-log graph will Cake filtration processes are demanding because of the
provide s as the slope. Knowing s, one can calculate 0 from time-dependence of the filtration flux, vs, and the compress-
any data point (R^ c , P) for given s and c (Figure 6.3.23(c)). ibility of the cake formed. If one could operate under a steady
One can consider equations (6.3.138h) and (6.3.138j) state process in some fashion, it would be quite useful. This
to be representative of traditional constant pressure filtra- essentially means a controlled growth of the cake such that, at
tion behavior; the quadratic relation for cake formation any given location of the device, the filtration flux for a given
between time t and the filtration volume per unit mem- P will be constant with time. The deadend filtration config-
brane area, (V(t)/Am), exists for up to 85% of the total time, uration (see Figures 6.3.21 and 6.3.23(a)) of bulk flow of
followed by a short cake compression period (Figure 6.3.23(d), suspension parallel to the force direction has to be changed
solid line). However, in the filtration of many sludges con- to develop alternative configurations which limit the growth
taining large molecular weight biomacromolecules, some- of the cake. Two such conceptual possibilities are shown in
times called supercompactable, the cake formation period Figures 6.3.25(a) and (b). The first one involves physical
is rather short, whereas the compression phase is very long motion of the cake plus the filter cloth/membrane in a
418 Open separators: bulk flow parallel to force and CSTSs

(a) Consider now expression (6.3.138d) for vs t :


Direction of
Force direction  
motion of
P 2R^ c s P t 1=2
filter/membrane Cake vs t 1 2
containing Rm c  s Rm
the cake  2 2 
Filter/membrane Rm 2R^ c s t 1=2
) v s t :
P 2 c  s P
Filtrate
By definition,
(b)
Force Direction of P= P= P=
direction bulk flow of suspension vs e >> ,
Rm Rc Rc Rm
Cake
Filter/membrane ^ c c : Therefore,
where Rc R
 1=2
P 2
vs >> ,
Filtrate 2 R2m

Figure 6.3.25. Steady state filtration/microfiltration of a slurry via which suggests that
bulk flow perpendicular to the force direction. (a) Bulk motion of  1=2
2R^ c s t
filter/membrane containing the cake perpendicular to the force v s t :
c  s P
direction. (b) Bulk motion of the suspension/slurry perpendicular
to the force direction. Now consider expression (6.3.138e) for V(t):
" #
      R   
^ c s P t 1=2
2R
c s m
V t =Am 1 2 1 :
s R^ c c  s Rm
direction perpendicular to the force direction, and away from
the suspension, so that conditions at any location in the
Write this as
device with respect to the cake thickness remains unchanged 0 1 0 1 2( 3
)1=2
(Figure 6.3.25(a)). The cake thickness here will change in the   c  s A @Rm A 4 ^ c s P t
2R
V t =Am c @ 1  15
direction of its physical motion. The second one involves a s Rc c  s R2m
tangential motion of the suspension/slurry over the mem- 0 1 28 0 1 91=2 3
brane/filter perpendicular to the force direction (Figure c  s A 6<@R2m A 2R^ c s P t = Rm 7
@ 4  5:
6.3.25(b)); the shear rate will limit the growth of the cake as s : R2c c  s R2c ; Rc
the suspension/slurry becomes more concentrated. Both
configurations are examples of bulk flow perpendicular to Since Rm << Rc , we can write this as
the force direction and are treated in Sections 7.2.1.4 and 2( )1=2 3
     2 s P t
7.2.1.5. c s 4 5
V t =Am c
s ^ c 2c
c  s R
Example 6.3.4 Consider expressions (6.3.138c), (6.3.138d)
 1=2
and (6.3.138e) for c t, vs t and V t, respectively. For the 2c  s P t
limiting case where the cake resistance dominates over the ) V t Am :
^ c
s R
membrane resistance, determine the expressions for c t ,
vs t and V t .
Example 6.3.5 Calculate the membrane resistance Rm for
the following membranes/filters:
Solution Consider first expression (6.3.138c) for c t :
(1) m 0.6, rp 0.1 m, m 1 m;
0 1 2( 3 (2) m 0.6, rp 0.05 m, m 2 m.
)1=2
Rm A 4 ^ c s P t
2R Note: The membrane thickness m corresponds to that of a
c t @ 1  15
R^ c c  s R2m thin layer, having the required pore size and porosity, on top
2 3 of a thicker highly porous support layer having at least an
( )1=2
2 order of magnitude larger pore size; therefore the support
6 Rm 2 s P t Rm 7
) 1 4 2  5: layer resistance may be neglected.
^ 2 c  s R
R ^ c 2c ^ c c
R
c c
Solution Employ equations (6.3.135c,d) for a model based
^ c c , then
If Rm << R on uniform straight cylindrical pores:
 1=2
2 s P t 8 m
c t : Rm ; m N p r 2p :
^ c
c  s R m r 2p
6.3 Bulk flow parallel to force direction 419

Here r 2p r 2p for pores of the same size. g ^ c  0:01166


R^ c s 2  102 R
10 s-cm2 cm-s
Case (1): 2c  s P 2  0:65  550  1:33  103 g
8  104 cm cm-s2
Rm
0:6  0:1  104  0:1  104 cm2
10  0:65  5:5  1:33  1010
^ c
)R cm2 4:07  1010 cm2 :
4 11 11:66
8  10  10
1:5  107 cm1 :
6 (2) For large t, we obtain from equation (6.3.138i),
Case (2):
8  2  104 cm

Rm ^ c 1=2 :
c t 2 s P t=c  s R
0:6  0:05  104  0:05  104 cm2
At t 500 mins,
16  104  1013 16
 109 1:06  108 cm1 :
655 150 0
g
11=2
2  11:66  550  1:33  103  500  60 s
B cm-s 2 C
B C
c t B C
Example 6.3.6 Calculate the cake resistance Rc for the B 3
@ 10  0:65  2  102 g 10 1 C
A
 4:07  10
following cake properties: void fraction in the cake cm-s cm 2

0.35; the particles forming the cake are rigid and have 0 11=2
an effective diameter of 10 m; the cake thickness is 11:66  5:5  1:33  3  109 2A
@ cm
6 cm; a1 5. 0:65  4:07  1011
Solution Employ equation (6.3.135k): 0 11=2
0:1166  5:5  1:33  3
@ A cm;
dp 10  104 cm; 0:65  4:07

a1 1  c 2 a2vc c 6 c t 0:9661=2 0:983 cm


Rc cm1 ; c 0:35; avc ^ c c t 4:07  1010  0:983 cm1 ;
R c t R
3c dp
Rc t 4  1010 cm1 :
6
) avc cm1 ; c 6 cm
10  104
6.3.3.1.1 Membranes and separation characteristics in
5  0:65  0:65  36  6 microfiltration Whereas filter cloths/woven fabrics are
) Rc cm1 ;
0:35  0:35  0:35  106 useful for filtration of larger particles, microfiltration mem-
branes are widely used in deadend filtration mode to
5  0:4225  36  6  108
Rc 1:066  1010 cm1 : remove effectively suspended matter in the size range
4:28
0.1 to 10 m. The objective can be purification, clarific-
ation, sterilization, concentration and analysis. The sus-
Example 6.3.7 In the batch filtration of a particular aque- pended particle capture/retention efficiency ET may be
ous slurry having a density of 17.5 g of solid per liter of
described by
suspension, the vacuum based applied P is 55 cm Hg;
the solids volume fraction in the cake is 0.65. The suspen- 1  out
ET , 6:3:139a
sion viscosity is 2 cp. The mass density s of the solid in
in the suspension is 1.5 g/cm3. The batch filtration data
plotted according to Figure 6.3.23(b) yielded a slope of where in and out are the volume fractions of the particles
10 s/cm2. in the incoming feed stream and the treated stream going
^ c for an incompressible cake. through the filter. An alternative index used quite often is
(1) Determine R
the log reduction value (LRV) (see Problem 2.4.2(a)):
(2) Using the approximation for large t, determine the value
of Rc at t 500 minutes.
 
in
LRV log10 : 6:3:139b
Solution (1) The slope of the plot according to Figure out

^ c s =2c  s P : Here 2 cp 2 
6.3.23(b) is R
In many critical applications (see Goel et al. (2001)), for
102 g/cm-s; c 0.65;
example sterilization in the pharmaceutical industry
P 55 cm Hg 550 mm Hg 550  1:33  103 g=cm-s2 ;
involving water for injection, parenterals, serum and
plasma processing, aseptic processing, etc., or clarification
s solids volume fraction in suspension and biological sterilization in the beverage industry for
17:5 g solid=1:5 g=cm3 11:66 absolute microbial removal without heat or chemicals,

1000 cm3 1000 the microfilters have to be totally retentive of the particu-
(here we assume that the effective density of suspension is late contaminant/bacteria, etc. For example, sterilizing
essentially 1 g/cm3); s 0.01166. Therefore, the slope is grade filters are designed to be totally retentive of all
given by bacteria. This is determined by testing the membrane/
420 Open separators: bulk flow parallel to force and CSTSs

microfilter with the bacterium Pseudomonas diminuta, 6.3.3.2 Ultrafiltration


which has a diameter of 0.3 m but an aspect ratio of 1.5
Ultrafiltration (UF) is a pressure driven membrane separ-
to 1. Sterilizing grade filters which are rated for 0.22 m
ation technique involving solute(s) in a solution, where
have been shown to retain P. diminuta completely. The
the solute molecular weight can vary from around 1000
largest pore diameter in the membrane/filter must not
to 500 000. It is used primarily to (1) concentrate a solute in
allow such a bacteria to enter the membrane. The pore
a solution by removing the solvent through the membrane,
diameter based characterization of the membrane is
(2) purify the solution from smaller molecular weight
achieved by the so called bubble point technique.
impurities which pass through the membrane along with
In the bubble point technique, the membrane pores
the solvent and (3) fractionate solute mixtures in a solu-
are filled with a liquid that spontaneously wets the pores.
tion. Since the solutes have molecular weights larger than
A gas is brought on one side of the membrane at a pres-
1000, they are often identified as macrosolutes. Specifically,
sure. As the pressure is increased, at a certain pressure gas
the macrosolutes of relevance are proteins, macromolecules,
bubbles first appear on the other side of the membrane.
viruses, etc. Colloidal particles of appropriate dimensions
The bubble point pressure, PBP, corresponds to the lowest
may also be considered as macrosolutes. Comprehensive
gas pressure at which gas bubbles appear on the other
treatments of ultrafiltration are available in Zeman and
side. This value is obtained from the YoungLaplace
Zydney (1996), Cheryan (1998) and Kulkarni et al. (2001).
equation:
At the low end of the solute molecular weight range, this
4 cos technique overlaps the technique of nanofiltration/reverse
P BP , 6:3:140
dp osmosis; at the high end, it merges with microfiltration.
In ultrafiltration, the feed solution containing one or
were dp is the pore diameter, is the surface tension of the
more macrosolutes with or without small molecular weight
liquid and is the contact angle between the membrane
buffer solutes/impurities is imposed over an ultrafiltration
surface and the liquid. Often an experimentally determined
(UF) membrane at above atmospheric pressure (Figure
tortuosity factor, m, is used in the denominator of the
6.3.1(j)); the other side of the membrane is usually at
expression for PBP; m can vary in the neighborhood of 5
atmospheric pressure. The solvent, low molecular weight
to 3.33. The higher the value of PBP, the lower the size of
buffer solutes/impurities and the macrosolutes, whose
the largest pore. In practice, when the LRV is greater than
dimensions are smaller than the pore dimensions of the
9, i.e. in =out is greater than 109, the filter is considered
UF membrane, pass through the membrane; larger macro-
to be totally retentive (see Goel et al. (2001) for a detailed
solutes are retained by the membrane on the feed side. The
introduction).
rates at which the solvent, the microsolutes and/or the
Polymers commonly used to make such microfiltration
macrosolutes pass through the membrane depend on a
membranes are: polyvinylidene fluoride (PVDF), Nylon 66,
number of factors. These factors include: applied pressure
polytetrafluoroethylene (PTFE), polysulfone, cellulose,
difference across the membrane (P), the solution proper-
cellulose acetate/cellulose nitrate, polypropylene (PP),
ties, the solute dimensions, the membrane structure,
polyester, polycarbonate, etc. Ceramic microfiltration
the solutemembrane interaction, the bulk feed flow vs.
membranes are not uncommon. Polymeric membranes
force configuration and the macrosolute transport charac-
may have the following structures.
teristics of the feed flow field over the membrane. These
Homogeneous: porosity and pore size uniform along the last two conditions, as well as the nature and magnitude
membrane thickness. of the bulk flow with respect to the magnitude of the
Asymmetric: the membrane has a thin selective skin force, are of considerable importance in UF, as will be
region at the top facing the liquid/gas stream to be demonstrated.
filtered with a pore size much lower than that of the A quantitative analysis of separation in UF requires first
rest of the membrane. a knowledge of the transport rates of the solvent and
Composite: thin selective skin on top of the porous the macrosolutes through the UF membrane. When the
membrane is made of a different material. macrosolute molecular weights are not high (
1000),
the membrane pores may have dimensions in the range
The treatment so far has focused essentially on micro-
~12 nm; the osmotic pressure of a concentrated solution
filtration and cake filtration of a liquid feed. However,
of such macrosolutes will be significant with respect to the
microfiltration of process gas streams, as well as gas
applied pressure difference, P. The molar solvent flux
flowing through vents on tanks, containers, fermentors,
under ideal conditions will be described by the flux expres-
etc., are also quite important in the deadend mode. Gen-
sion (3.4.54) (Vilker et al., 1981):
erally these filters are made of hydrophobic materials like
PTFE, PVDF, PP, etc., so that water does not wet the pores P 
N sz AP  , 6:3:141a
and gases can easily flow through the filter pores. Rm Rc
6.3 Bulk flow parallel to force direction 421

 
where Rm is the resistance of the membrane, Rc is the C ip
Robs 1 , 6:3:141c
resistance of any layer deposited over the membrane, C if
is the osmotic pressure of a solution containing the
macrosolute and is equal to (fp). The subscripts is different from the true solute rejection, Rtrue, by the
f and p refer, respectively, to the feed side and the membrane,
permeate (product/filtrate) side. For such cases, the  
C ip
membrane is called a diffusive ultrafilter and the tech- Rtrue 1 0 : 6:3:141d
nique is sometimes identified as diffusive ultrafiltration. C il
However, instead of J sz , N sz is used in UF because of the Correspondingly, the observed solute transmission or
substantially enhanced role of solvent convection, which sieving coefficient, Sobs, will be different from the true value,
will be considered next. Strue, where
For larger membrane pores, and larger macrosolutes
whose solutions have much smaller osmotic pressures, the C ip C ip
Sobs ; Strue 6:3:141e
membrane is sometimes called a microporous ultrafilter; C if C 0il
the membrane pores are micropores/mesopores/large and
pores and the solvent volume flux N sz V s is described by
Darcys law, expression (3.4.88): Robs 1  Sobs ; Rtrue 1  Strue : 6:3:141f

Qsm P In the configuration of Figure 6.3.26(a), the rate at which


N sz V s : 6:3:141b the solvent passes through the membrane, and therefore
m
the solvent velocity through the membrane, is equal to the
The molar macrosolute flux, N iz , for macrosolute i may be magnitude of the velocity, vz, of the solution in the feed
described in general by equations (3.4.89a,b), which are side toward the membrane. It is as if there is a piston on
based on the combined diffusionviscous flow model. the feed solution side generating the feed pressure Pf: the
From the result (3.4.93b), we can obtain an estimate of piston velocity toward the membrane can only equal the
the macrosolute concentration, Cip, in the permeate as a velocity of the solvent/solution through the membrane for
function of a number of quantities, including the feed this incompressible liquid (Figure 6.3.26(b)).
macrosolute concentration, Cif, and the solvent flux, Consider a control volume (CV) extending from the
N sz V s , when there is no concentration polarization in membrane surface to a distance z into the feed liquid,
the feed solution. Knowing Cip and N sz V s , one may where z l, the thickness over which the concentration
calculate the extent of separation achieved in the UF changes (Figure 6.3.26(a)). Assume a pseudo steady state.26
device. Then a solute balance over the CV leads to the following:

0 1 0 1 0 1
solute flux into solute flux out solute flux out of the
@ the CV at z A  @ of the CV at z 0 membrane A @ CV at z into the bulk A
by convection by membrane transport liquid by diffusion

dC il
jvz jC il  jvz jC ip  Dil : 6:3:142a
dz

However, in practical UF processes, there is always Here, jvz j is the magnitude of the fluid velocity normal to
concentration polarization on the feed side. Figure 6.3.26 the membrane. We have assumed that it does not change
(a) illustrates the macrosolute concentration C il profile in over 0 z l; further, at the wall (z 0), jvz j N s V s , the
the feed solution in a batch cell; it increases from the value solvent volume flux through the membrane. Integrate

C ilb C if of the bulk solution to the concentration C 0il (6.3.142a) over the boundary layer thickness:
at the membranefeed solution interface where the macro-
solute is rejected by the membrane. This rejection may be

complete C ip 0 or incomplete 0 < C ip < C 0il . The 26


In the batch cell, the bulk macrosolute concentration changes
observed value, Robs, of the membranes solute rejection with time as the solvent and some of the macrosolutes pass
or solute retention, through the membrane.
422 Open separators: bulk flow parallel to force and CSTSs

(a)
Concentration polarization Gel polarization

Cil Batch Cil


cell
wall Pf
Pf > Pp Pf Cilb
Cilb Pf - Pp = P

z Cigel
z
dl Gel layer CV
CV C 0il
z=0
z=0
Membrane

Pp Cip Pp Cip

(b)
vz vz

Piston Piston driven operation

Membrane

vz

(c)
Pure
1 water

Solvent Very dilute


flux 2 solution

Pcrit
3 Cilf; kil3

Pcrit
4 Cilf; kil4 < kil3

Figure 6.3.26. Ultrafiltration. (a) UF in a batch cell: macrosolute concentration profile in feed side. (b) Piston driven UF in a batch cell: bulk
flow parallel to the force. (c) Observed behavior of solvent flux vs. P in macrosolute ultrafiltration. For an explanation of (1)(4), see the text.

Cilb
0 1 For the above result, we have described the solute mass-
l 0
dC il jvz j C  C ip
 dz ) ln@ il A transfer coefficient in the feed side above the membrane via
C il  C ip Dil C ilb  C ip
C 0il 0
k il Dil =l : 6:3:142c
jvz j N sV s
; The result illustrated by (6.3.142b) is singularly important
Dil =l k il
in UF. If in the feed solution, the mass-transfer coefficient
0 1 0 1
kil of the macrosolute i (which is rejected (partially or
C 0il  C ip jvz j A exp@jvz jA:
exp@ totally) at the membrane surface) from the interface to
C ilb  C ip Dil =l k il
the bulk liquid is not sufficiently large relative to the solv-
6:3:142b ent flux through the membrane, the wall concentration,
6.3 Bulk flow parallel to force direction 423

C 0il , builds up. For proteins, a gel layer (Michaels, 1968a) While Figure 6.3.26(c) illustrates how increasing polar-
may often be developed on the UF membrane surface ization affects the solvent flux, relation (6.3.142b) may be
when C 0il is high enough to be Cigel, leading to what is rearranged to quantify how the macrosolute retention/
called gel polarization (Figure 6.3.26(a)): rejection/transmission is affected simultaneously (if the
  macrosolute is not rejected completely). Rearrange this
C igel  C ip
N s V s jvz j k il ln : 6:3:143 equation as follows:
C ilb  C ip
0 1
Suppose C ip 0, i.e. the membrane effectively rejects/ @1  C ip A 0 1 0 1 0 10 1
retains the macrosolute i. Since Cigel is fixed for a particular C 0il
@ C 0il A @ jvz jA @1  Strue A@ Sobs A
macrosolute i in a given environment,27 one can increase 0 1 exp
C ilb k il 1  Sobs Strue
the solvent flux only by increasing kil. Although the solvent @1  C ip A
flux expressions (6.3.141a,b) suggest that an increase in P C ilb
0 1
will increase the solvent flux, no such increase takes place
jvz j Sobs
under the condition of gel polarization. ) 1  Sobs exp@ A  Sobs
k il Strue
This brings up an important characteristic of UF based
separation. If macrosolute molecules are being brought to
Sobs
an interface (in this case, the membranefeed solution ) Strue ! 6:3:144a
interface), for steady state operation, these macrosolute 1  Sobs exp jvz j
Sobs
k il
molecules also have to be removed/evacuated from this
interface. Removal of these rejected solute macromol-
Correspondingly,
ecules from this interface to the feed solution bulk by
diffusion is essential to continuing the separation at an 0 1 0 1
acceptable rate in steady state operation (compare the jv z j R true A 1  Robs ;
exp@ A @
general arguments provided in the two paragraphs around k il Robs 1  Rtrue
0 1 0 1 6:3:144b
equation (3.2.37)). Here one cannot arbitrarily keep on
1  R obs 1  Rtrue A jvz j :
increasing jvz j, i.e. the solvent flux, by increasing P; the ln@ A ln@
Robs Rtrue k il
bulk velocity parallel to the force direction (through the
membrane) cannot be increased arbitrarily unless the rate
of removal of the accumulated macromolecules from the Thus, knowing the solvent volume flux jvz j N sz V s
interface back to the bulk is simultaneously increased. The through the membrane and the macrosolute mass-transfer
latter rate is inherently low in the bulk flow parallel to coefficient, kil, in the feed solution, the true macrosolute
the force configuration. retention of the membrane, Rtrue, may be determined from
This basic phenomenon in UF has influenced its the observed retention values, Robs. A plot of ((1  Robs)/
development in many ways. High concentration of Robs) against the observed jvz j in a semilog plot will yield
macrosolutes at the interface coupled with a higher solv- the value of Rtrue from the intercept ((1  Rtrue)/Rtrue)).
ent flux through the membrane leads to a higher rate of From the slope of the line, one can also obtain an estimate
leakage of the macrosolute through the membrane (see of kil in the system.
equations (3.4.93a,b)); membrane fouling by the macro- From the solute flux expression (3.4.91c) in the
molecules is also increased. To avoid such conditions, combined diffusionviscous flow model, one can develop
one can operate UF at a lower P and carry on with a an expression for Strue or Rtrue. Consider relation (3.4.93b):
lower solvent flux. Figure 6.3.26(c) schematically illus-
C il Gi if expGi vz m m =Dip
trates the solvent flux behavior in UF as a function of ,
C 0il Gi ip  1 expGi vz m m =Dip
P for a number of conditions involving proteins as
macrosolutes: (1) no macrosolute, simple water flow
where the argument of the exponential function is the pore
through the membrane, water flux linearly proportional
Pclet number for solute i, Pem
i . Therefore
to P; (2) extremely dilute feed solution of macromol-
ecules, leading to a somewhat lower water flux than in C il C ip Gi if expPem
i
(1); (3) higher feed concentration solution of macromol- 0 0 S true 1  Rtrue :
C il C il Gi ip  1 expPem i
ecules at a certain value of kil, displaying gel polarization
6:3:145a
beyond a critical P; (4) solution concentration as in (3),
but operating at a lower kil, displaying gel polarization
It is also known that, when Pemi >> 1, employing relations
and a lower water flux level.
(3.4.92a) and (3.4.93a), we get

C il
27
There is considerable debate about what this Cigel is in Gi if Strue jPem ! S 1  R , 6:3:145b
C 0il i
equation (6.3.143).
424 Open separators: bulk flow parallel to force and CSTSs

where S or R are asymptotic values achieved at high (a)


solvent flux (Pem i >> 1). Substitution into relation
(6.3.145a), and the assumption that if ip , leads to
S expPem
i
Strue 1  Rtrue : 6:3:145c
S expPem
i 1

Such an analysis assumes that electrostatic and electroki-


netic interactions are not important in macrosolute trans-
port through the membrane. Pujar and Zydney (1994) have
(b)
considered such effects in protein ultrafiltration through 1
narrow pore membranes. Sharp

R = 1(Cip /Cif)
The above analysis/description of solvent flux and cut off Diffuse
macrosolute rejection/retention/transmission for an ultra- cut off
fillration membrane was carried out in the context of a
pseudo steady state analysis in a batch cell (Figure 6.3.26
(a)). Back diffusion of the macrosolute from the feed
solutionmembrane interface to the bulk solution takes 0

105

106
10 000
30 000
1000
place by simple diffusion against the small bulk flow paral-
lel to the force direction. The resulting mass-transfer coef-
ficients for macrosolutes will be quite small; the solvent Macrosolute molecular weight
flux levels achievable will be quite low. For practically
(c)
useful ultrafiltration rates, the mass-transfer coefficient is
increased via different flow configurations with respect to
the force.
In a small vessel like that shown in Figure 6.3.26(a),
a stirrer is incorporated to increase kil; it may become a
well-stirred vessel. A continuous buffer stream may also
enter such a well-stirred vessel containing an ultrafilter; Figure 6.3.27. (a) UF membrane as a bundle of size-distributed
however, the conditions in the vessel will change with tortuous capillaries. (b) Macrosolute retention behavior of two
types of UF membranes having a narrow or a broad pore size
time, unlike those for a CSTS at steady state (see Section
distribution. (c) Macrosolute retention via smaller membrane pore
6.4.2.1). These two configurations are used in laboratory-
size, pore mouth adsorption, pore blockage due to pore
scale applications. In larger-scale commercial/industrial
constriction.
applications, the bulk flow direction employed is perpen-
dicular to the force direction (see Section 7.2.1.3), a config-
uration commonly identified as crossflow. Such a flow
configuration can lead to high values of kil, which can then capillaries have the same diameter, and this diameter is
sustain high solvent flux; further steady state conditions constant along the membrane thickness. If we have spher-
can be maintained at any location in the flow channel ical macromolecules in the feed solution whose diameters
bounded by membranes. In the bulk flow parallel to the are significantly smaller than that of the capillaries, the
force configuration of Figures 6.3.26(a) and (b), steady pores, then these macromolecules will pass through the
state conditions cannot be maintained. In practice, slow membrane pores. However, if the macromolecules have
fouling of the membrane leads to a slow reduction in diameters larger than that of the pores, they cannot enter
membrane solvent flux. the membrane pores and are rejected.
The treatment discussed so far has not provided a In real life, most membranes have a distribution of capil-
physical picture of how the macrosolute rejection takes lary diameters, what is called a pore size distribution. For a
place in the UF membrane. Let us imagine this UF mem- given macromolecule, some of the pore diameters in a con-
brane (Figure 6.3.27(a)) to consist of a bunch of cylindrical ventional UF membrane will be larger, some smaller, than
capillaries traversing the thickness of the membrane in an the macromolecular dimensions. Therefore, the macromol-
otherwise impervious medium of membrane material ecules will pass through the larger pores and not through the
(usually polymeric but sometimes ceramic). The diameters smaller ones, resulting in a permeate solution concentration
of the capillaries may be uniform or variable. The capillar- of macromolecules less than that of the feed side, i.e. finite
ies may be straight or tortuous, with diameters changing solution rejection. A highly simplistic analysis of macrosolute
along the length; some of the capillaries may be intercon- rejection of a microporous ultrafiltration membrane will be
nected, others not. A real life UF membrane may have a carried out now for a membrane having a pore size density
variety of such features. Consider now a simple case: all function f(rp), where rmin < rp < rmax (see the corresponding
6.3 Bulk flow parallel to force direction 425

analysis for solvent flux in Section 3.4.2.3). The treatment macrosolute through a microporous ultrafilter depends
follows Michaels analysis (Michaels, 1968b). solely on the characteristics of the pore size distribution of
Suppose that the solvated macrosolute radius is ri. the membrane and the macrosolute dimension. If the mem-
Further, let the macrosolute concentration in the larger brane has a broad pore size distribution, macromolecules
pores (rp > ri) be equal to C 0il , i.e. the macrosolute concen- having a wide range of molecular weights and sizes will be
tration at the feed solutionmembrane interface. On the able to pass through the membrane. Such a membrane is
other hand, the macrosolute concentration in smaller said to have a diffuse cut off. On the other hand, if the
pores (rp ri) will be zero. The macrosolute flux through membrane has a narrow pore size distribution, the mem-
the membrane is therefore brane is assumed to have a sharp cut off; the membrane is
such that the size difference between the macrosolute which
N iz C 0il  volume flux of solvent through
is completely retained and the macrosolute which passes
" rmax
#
through with very little retention is quite small. The macro-
pores whose r p > r i C il dN sz V s , solute retention behaviors of these two types of membranes
ri are illustrated in the semilog plot of Figure 6.3.27(b).
6:3:146a It is now useful to explore the factors which control the
selectivity of the UF membrane for one macrosolute (i 1)
where dN sz V s is the contribution of the pores of radius rp
of solvated radius r1 over another macrosolute (i 2) of
to rp drp to the solvent volume flux:
solvated radius r2 (>r1). If we define the selectivity/separ-
 
m r 2p f r p dr p P ation factor of the membrane for species 1 over 2 via
dN sz V s : 6:3:146b
8 m m x 1p x 2f
12 , 6:3:147a
x 1f x 2p
The total solvent flux (volume flux) from (3.4.86) is
  then, for dilute solutions, we may rewrite this as
m r 2p P
N sz V s :  , 
8 m m C 1p C 2f C 1p C 2p
12 : 6:3:147b
C 1f C 2p C 1f C 2f
Since the macrosolute concentration, Cip, in the filtrate
(permeate) is given by From equation (6.3.146d) we obtain, in the absence of
N iz C ip N sz V s , 6:3:146c concentration polarization,
r max

we obtain
2 r 2p f r p dr p
r max
0 13
C 1p C 1p r1
6 m r 2p f r p P
C 0il 4 dr p @ A7
5 C 01l

C 1f
: 6:3:147c
8 m m r 2p
C ip N iz ri
0 1
C 0il C il N sz V s Similarly,
r2
0 m p @ P A
C il r max

8 m m
r 2p f r p dr p
r max
r max
C 2p C 2p r2
r 2p f r p dr p r 2p f r p dr p C 02l

C 2f
: 6:3:147d
r 2p
ri ri
r max
r 2p Therefore
r 2p f r p dr p
r max r2 r max
r min

r 2p f r p dr p r 2p f r p dr p r 2p f r p dr p
Strue 1  Rtrue: 6:3:146d
r1 r1 r2
12 r max r max

Therefore
r 2p f r p dr p r 2p f r p dr p
ri r2 r2

r 2p f r p dr p r2

r min r 2p f r p dr p
Rtrue : 6:3:146e
r 2p 1 r
1 r max : 6:3:147e

This interesting, but highly simplified, result suggests r 2p f r p dr p
r2
that the membrane rejection/retention/transmission of a
426 Open separators: bulk flow parallel to force and CSTSs

Suppose now that r2 is sufficiently small compared to rmax, A variety of UF membranes are commercially available.
i.e. the membrane allows a substantial amount of species 2 Table 6.3.8 provides an illustration of the properties and
to pass through. Therefore, unless r1 and r2 are far apart, performance characteristics of a series of polymeric flat
the integral in the numerator on the right-hand side of membranes that are used in a batch cell. The data on
(6.3.147e) is small compared to the denominator. The solute rejection were acquired in a stirred batch UF cell
selectivity of a diffuse cut off microporous ultrafilter is of the type to be considered in Section 6.4.
not very high unless the macrosolutes differ considerably
in the values of their solvated radii. Traditionally, therefore, Example 6.3.8 Saksena and Zydney (1994) have studied
fractionation of macrosolutes like proteins by ultrafiltration the protein transmission characteristics of a 100 000 MWCO
is limited to systems where the two macrosolutes differ in ultrafiltration membrane (OMEGA 100K) of polyethersul-
size by about seven to ten times (Cherkasov and Polotsky, fone using bovine serum albumin (BSA) as a model protein
1996). (mol. wt. 66 430) in a solution of pH 7.0 and ionic
strength of 0.15 M NaCl solution. The batch cell operation
Enhancement of the membrane selectivity for a given
characteristics and the protein transport properties are as
macrosolute has been achieved by considering a number of
follows:
other factors, including increasing/decreasing the effective
radius of a charged protein molecule. If the UF membrane k i 5:2  106 m=s; Pem
i S J v m =Dieff ;
has some charge on the surface, then macrosolutes having S 0:016; Dieff 1  1013 m2 =s; m 0:5 m;
the same charge will be effectively rejected/repulsed by the J v jvz j 106 m=s:
membrane. For proteins, this can be achieved by changing Determine the values of the observed and true transmission
the solution pH. If the solution pH is greater than the pI of coefficients, Sobs and Strue , respectively, of BSA under these
the protein, the protein will have a net negative charge (see conditions. (See Table 6.3.8 for a definition of MWCO.)
Figure 4.2.5(c)); if the membrane has a negative charge, this
protein will be excluded from the membrane pores. If the Solution We will first employ relation (6.3.145c) between
S expPem
ionic strength of the solution is, however, increased sub- Strue and S : Strue S exp Pemi 1.
i
stantially, then the extent of electrostatic shielding of the
Here
charged protein molecule will be substantially increased.
The negatively charged membrane, for example, will not S J v m
S 0:016; Pem
i
be able to reject the protein as much, resulting in decreased Dieff
selectivity in relation to another protein which may be m
uncharged (for example, bovine serum albumin, pI 4.7) 0:016  106  0:5  106 m
s
at the solution pH (say 4.7). (See the following references: ;
Saksena and Zydney (1994); van Eijndhoven et al. (1995); 1  1013 m2 =s
Nystrm et al., (1998).) By stacking three such membranes 0:016exp0:08
Pem
i 0:08 ) S true
one over the other, Feins and Sirkar (2004, 2005) were able 0:016 exp0:08  1
to obtain one pure protein in the permeate for a binary
0:016  1:0833
mixture of proteins whose molecular weight ratio was as ) Strue 0:1745:
0:0993
low as 1.03 to 2.05. This internal-staging concept has not
been treated further. Next we utilize relation (6.3.144a) between Sobs and Strue :
There are additional factors that influence membrane
selectivity or retention. If the macrosolute has tendencies Sobs
Strue   ;
of adsorption on the membrane surface, the adsorbed jvz j
1  Sobs exp Sobs
macromolecules may block the pore entrance or form a k i
layer on top of the membrane changing the membrane Sobs
selectivity (Figure 6.3.27(c)). If the pore diameter changes 0:1745  
106
along the membrane thickness, a macrosolute which could 1  Sobs exp Sobs
5:2  106
enter the pore at the feed side may plug the pore, render it
useless and change the pore size distribution. Due to lack Sobs

of convection, the top of this pore surface becomes a 1  Sobs exp0:192 Sobs
stagnant zone. A gel layer formed on top of the membrane ) 1:212  0:17451  Sobs 0:1745Sobs Sobs
from one macrosolute may substantially influence the
transport behavior of a smaller macrosolute. A globular 0:211
Sobs 1 0:211  0:1745 0:211 ) Sobs :
macrosolute/protein is likely to have a higher solute rejec- 1:0355
tion than a linear macromolecule of the same molecular
weight. For BSA, Sobs 0:204.
Table 6.3.8 Properties and performance characteristics of flat Amicon UF membranesa

UM05 UM2 UM10 YM10 YM30 PM10 PM30 XM50 XM100A XM300

Nominal MWCOb 500 1000 10 000 10 000 30 000 10 000 30 000 50 000 100 000 300 000
Average pore 2.1 2.4 3.0 4.0 3.8 4.7 6.6 11 48
diameter (nm)
Water fluxc 0.100.20 0.71.1 1.53.0 2.06.0 1.02.5 - 0.51.0
(ml/min/cm2)
Material polyelectrolyte Polyelectrolyte Polyelectrolyte Regenerated Regenerated Polysulfone Polysulfone PAN-co- PAN-co- PAN-co-
complex complex complex cellulose cellulose PVCd PVC PVC

Macrosolute (MW) Solute rejectione


pH5 pH10

D-Alanine (89) 15 80 0 0 0 0 0 0 0
DL-Phenylalanine 20 90 0 0 0 0 0 0 0
(165)
Tryptophan (204) 20 80 0 0 0 0 0 0 0
Sucrose (342) 70 80 50 25 0 0 0 0 0
Raffinose (594) 90 50 10 0 0 0 0 0
Inulin (5,000) 80 60 45 0 0 0 0
Dextran T10 (10 000) 90 90 5
Myoglobin (18 000) >95 >95 95 80 80 35 20
-Chymotrypsinogen >95 >98 >95 >80 >95 75 85 25 0
(24 500)
Albumin (67 000) >98 >98 >98 >90 >98 >98 >90 >90 45 10
Aldolase (142 000) >98 >98 >98 >98 >98 >95 50
IgG (160 000) >98 >98 >98 >98 >98 >98 >98 90 65
Apoferitin (480 000) >98 >98 >98 >98 >98 >98 >95 85
IgM (960 000) >98 >98 >98 >98 >98 >98 >98 >98 >98
a
From Amicon catalogs.
b
MWCO molecular weight cutoff; it means Ri 0.9 for a solute of the specified molecular weight.
c
Measured in a stirred cell after 5 minutes of pressure. All membranes at 55 psi, except XM300 at 10 psi.
d
Copolymer of acrylonitrile and vinyl chloride.
e
At 55 psi, except 10 psi for XM100A and XM300
428 Open separators: bulk flow parallel to force and CSTSs

6.3.3.3 Reverse osmosis have a batch cell or not, the salt concentration on the feed
side of the membrane will change with time (Nakano et al.,
In Section 3.4.2.1, the phenomenon of reverse osmosis
1967). Assume, however, a pseudo steady state for the sake
(RO) through a nonporous membrane was introduced. If
of the following analysis. At any instant of time, t, we can
the hydraulic pressure of a solution containing a micro-
assume (Nakano et al., 1967) that we have a bulk solute
solute, e.g. common salt, on one side of a nonporous
concentration, C ib , which increases to C 0i at the RO mem-
membrane exceeds that of another solution on the other
brane surface (Figure 6.3.28(a)). Therefore we have back
side of the same membrane by an amount more than
diffusion of the solute (i.e. salt) from the membrane sur-
the difference of the osmotic pressures of the same two
face to the bulk solution. From the pseudo steady state
solutions, then, according to the solution-diffusion model,
analysis of concentration polarization carried out for ultra-
the solvent will flow from the solution at higher pressure
filtration, resulting in equation (6.3.142b), and the corres-
to the one at a lower pressure (equation (3.4.54)) at the
ponding equation for RO, namely (3.4.65c), we have
following rate:
   
C sm Dsm V s vz C 0i  C ip jvz j jvz j
J sz P high  P low  high  low : exp exp : 6:3:150
RT m Vs C ib  C ip Di = k i
6:3:148a
If the value of ki is low and that of jvz j is high, one can
The solution at a higher pressure Phigh has a higher solute have a situation where the value of C 0i can become large
concentration, therefore a higher osmotic pressure high, enough so that, for a given P, P can become equal to
and is the feed solution; the corresponding quantities on (wall,feed  permeate); at this time, there will not be any
the other side of the membrane are Plow and low. For solvent flux, due to a zero driving force. The membrane is
reverse osmosis to take place, considered polarized. In practice, the k i values are suffi-
ciently high so that the extent of the concentration polar-
P P high  P low > high  low : 6:3:148b

ization modulus C 0i  C ip = C ib  C ip is low and not


The equation describing the solute transport according to too far from 1. However, in the mode of operation shown
the solution-diffusion model is (see equation (3.4.59)) in Figures 6.3.28(a) and (b), the value of k i is quite low; in
practice, therefore, different configurations of flow vs. force
Dim im
J iz C if  C ip : 6:3:149 are adopted. However, we will use this configuration and
m
pseudo steady state assumption to illustrate here the extent
Consider Figure 6.3.28(b) for a batch cell containing an RO and nature of separation achieved through the membrane
membrane and a feed solution; as the permeation is going in RO; specifically we will derive expressions for the solute
on, we can picture it as if the piston is driving the solution rejection, Ri, and the separation factor, si , between the
toward the membrane, i.e. the bulk flow of the feed solu- solvent s and the solute i in RO using the solution-diffusion
tion is parallel to the direction of the force driving the model. We will also point out the inadequacy of the
permeation velocity vz through the membrane. As the solution-diffusion model, especially, at high P values.
solvent permeates through the membrane, if the RO mem- Consider a dilute solution of a solute (specifically, a
brane is effective, the solute (e.g. salt) is rejected and the microsolute such as NaCl) i in a solvent s (say, water). By
salt concentration builds up on the feed side. Whether we definition,

(a)
Cil Batch
cell
Pf > Pp (b)
wall
Pf | vz | | vz |
Cilb Pf - Pp = P
z Piston Piston driven operation
dl
CV C 0il
z=0 RO RO membrane
membrane
| vz |

Cip Pp

Figure 6.3.28. (a) Reverse osmosis (RO): RO in a batch cell: solute concentration profile in feed side. (b) Piston-driven RO in a batch cell:
bulk flow parallel to the force.
6.3 Bulk flow parallel to force direction 429

x sp 1  x sf x x For membranes showing high rejection, i.e. C ip << C if , we


si
sp if , 6:3:151a obtain, from expression (6.3.154b),
1  x sp x sf x ip x sf

where x sp and x sf are the mole fractions of the solvent in Dim im RT


Ri 1  2 : 6:3:154d
the permeate and the feed stream, respectively. Corres- C sm Dsm V s P 

pondingly, 1  x sp and 1  x sf are the mole fractions


of the salt in the permeate and the feed, respectively, in this Correspondingly,
binary system. The definition for separation factor may be
2
reexpressed as follows: C sm Dsm V s P 
si 6:3:154e
Dim im RT
C sp C if
x sp x if C tp C tf C sp C if
si C ip C sf
, 6:3:151b for C ip << C if . Note that expression (6.3.154d) is merely
x ip x sf C ip C sf
C tp C tf the result of considering only the first term, i.e. replace
(1 x)1 by 1  x (valid when second-order terms, O(x2),
where Ctj is the total concentration in region j. For a dilute
are negligible).
solution, C sp C sf ; therefore,
These preceding two expressions for Ri and si indi-
C if cate that, as P  increases, both Ri and si increase
si : 6:3:151c
C ip for high rejections. In the limit of P becoming infinitely
large, Ri ! 1 and si ! . In reality, most practical mem-
On the other hand, by definition, the solute rejection Ri is
branes reach a limiting value, Ri , for solute species i.
(definition (2.2.1b)
Solute rejection at high P indicates that RO membranes
C ip are not perfect: there is some amount of salt transport, as
Ri 1  ; 6:3:152a
C if shown in Figure 6.3.29(a) (Lonsdale, 1966). This is a
major limitation of the solution-diffusion model of RO
consequently,
membranes with two parameters, A and (Dim im =m ).
Ri 1  1=si : 6:3:152b Models having three parameters can overcome this
limitation.
For the flow vs. force configuration of Figures 6.3.28(a) and (b) The solution-diffusion-imperfection model based on
three parameters and proposed by Sherwood et al. (1967)
J iz C ip
V s C ip 6:3:153a (illustrated in flux expressions (3.4.60a,b)) appears to be
J sz C sp
able to describe better the observed solute rejections vs.
for a dilute salt solution. Substitute here the expressions for solvent flux behavior in RO membranes (Applegate and
J iz and J sz from expressions (6.3.148a) and (6.3.149) Antonson, 1972). Rewrite the flux expressions (3.4.60a,b)
resulting from the solution-diffusion model: for a dilute solution as
Dim im
J iz m C if  C ip RT Dim im C if  C ip J vz K 1 P  K 3 P; 6:3:155a
V s C ip :
J sz C sm Dsm V s C sm Dsm V s P 
RT m P 

6:3:153b J vz C ip N iz K 2 C if  C ip K 3 P C if :
6:3:155b

Now, Then the expression for salt rejection, Ri 1  C ip =C if ,


is given as
C if V s C if C sm Dsm V s P 
si 6:3:154a (    
C ip Dim im RTC if  C ip
1 K3 K2
Ri C if P 1 f C if
2 f C if K1 K1
Therefore the solute rejection is given by "    

K3 K3
1 Dim im RT C if  C ip  C 2if P 2 1 2 C if P f 1
Ri 1  1 2 : 6:3:154b K1 K1
si C if C sm Dsm V s P 
   2 #1=2 )
This may be rearranged to yield K2 K2
 C if 2f C if 2
 4 f P C if ,
K1 K1
 
Dim im RT C ip
Ri 1  2 1  6:3:156
C sm Dsm V s P  C if
" #1 where f is the osmotic pressure of the feed solution. The
Dim im RT
) Ri 1 2 : 6:3:154c three parameters in this model, K1, K2 and K3, allow an
C sm Dsm V s P  excellent fit of the experimental data obtained from
430 Open separators: bulk flow parallel to force and CSTSs

(b)

(a)
100 Piston
Calculated from equation
(6.3.154c)

Water flux (mg/cm2-s)


0.20
Salt rejection (%)

90
Experimental data
Cif Feed saline
0.10 water
Section b
Section a
80

M
0
0 10 20 30 40 50 60 70 80 90
Applied pressure (atm)

Permeate

Figure 6.3.29. (a) Water flux and salt rejection vs. pressure for a 40% acetyl cellulose acetate membrane. Conditions: 0.1 M NaCl, 1 m
thick membrane (Lonsdale, 1966). Reprinted, with permission, from Desalination by Reverse Osmosis, U. Merten (ed)., MIT Press,
Cambridge, MA (1966), Figure 49, p. 117. (b) Flow parallel to force through a membrane which has defects (section b) having
an area fraction b and nondefective region (section a) of area fraction (1b); the corresponding characteristic permeabilities are
Lbp and Lap .

cellulose acetate and polyamide membranes (Applegate J vz Lp P  i ; 6:3:158a


and Antonson, 1972). For a detailed consideration of dif-
ferent models in RO, the review by Soltanieh and Gill J iz  1  i J vz C i : 6:3:158b
(1981) should be consulted.

The three parameters are: Lp QS I =m , P=2RT m


Another three-parameter model of reverse osmosis and i ; C i is an average of the feed and permeate concen-
membrane transport of some importance is the Spiegler trations. Note: Here the osmotic pressure is related to the
Kedem model (Spiegler and Kedem, 1966). In its differen- solute concentration Ci by Ci RT (van t Hoff equation
tial form (similar to equations (3.4.47) and (3.4.48)), the (3.3.86l)) for dilute solutions. For electrolytes, RTCi,
model proposes the following local flux expressions at any where is the number of ions dissociated from one mole of
location z in the membrane: electrolyte.
  One would like to know now what the relation is
dP d
volume flux J vz QSI  i ; 6:3:157a between the solute rejection, Ri, and these parameters in
dz dz
the SpieglerKedem model. Focus on the ratio J iz =J sz as
d Ci given by the relation (6.3.153a). If, instead, we consider
solute flux J iz P 1  i C i J vz : 6:3:157b
dz the ratio
(Note: Here Ci, and P correspond to infinitely thin solu-    
moles i cm3
tions in equilibrium with the local section of the mem- C ip  J vz  
J iz cm3 cm2 -s moles i
brane; therefore Ci is the molar concentration of solute i   C ip ,
J vz cm3 cm3
in a solution of osmotic pressure .) There are three par- J vz
cm2 -s
ameters here: QSI (the intrinsic hydraulic permeability), P
6:3:159
(the local solute permeability coefficient) and i (the local
solute reflection coefficient). When these two equations are then we can rewrite equation (6.3.157b) as
integrated across a membrane of thickness m , assuming
dC i
QSI , P and i to be essentially constant across the mem- P J vz f1  i C i  C ip g: 6:3:160a
dz
brane thickness, one obtains, for the whole membrane, two
equations for the SpieglerKedem model (based on the Integrate across the membrane of thickness m from Cif to
KedemKatchalsky model): Cip:
6.3 Bulk flow parallel to force direction 431

Cip m However, we know from equation (6.3.158a) that


dC i J vz
dz; 6:3:160b J vz Lp P  i :
1  i C i  C ip P
C if 0
  Therefore
1 1  i C ip  C ip J vz m
ln ; 6:3:160c Lp 1  b Lap b Lbp 6:3:165a
1  i 1  i C if  C ip P and
 
J vz m 1  i C ip  1  i C if
exp  Lap 1  b
P C ip i i , 6:3:165b
1  b Lap b Lbp
1  Ri C if  1  i C if
;
C ip i which indicates that i represents the fraction of the
6:3:160d volume flux due to the perfect section of the membrane.
  Example 6.3.9 Consider the physical picture (Figure 6.3.29(b))
J vz 1  i m i  Ri
exp  , 6:3:161 which is the basis for SpieglerKedem illustration, namely equa-
P i 1  Ri
tion (6.3.164a). We can rewrite it as
where we have used definition (6.3.152a) for Ri. Rearran- J vz K 1 P  f K 2 P: 6:3:166
ging this result leads to the following relation between
Here the nondefective membrane section does not allow
the solute rejection Ri and the solute reflection coeffi- any solute transport; the defective section allows feed
cient i: solution to pass through. Therefore the solute flux is
 n o given by
i 1  exp  J vz 1
P
i m

Ri n o : 6:3:162 N iz K 2 P C if 6:3:167
1  i exp  J vz 1
P
i m

Further, the permeate solutions from different sections of the


membrane are not mixed because flow is parallel to force
Consider the following limit:
and there is no lateral diffusion; therefore in equation
J vz ! , when P ! ; then Ri i : 6:3:163 (6.3.166), the first term has (P f) instead of (P ).
Determine the expression for Ri.
Thus, the SpieglerKedem model predicts correctly that,
Solution From relation (6.3.159), Niz Cip Jvz. By definition,
at very high P, Ri reaches a limiting value which is less
than 1 (i can have a maximum value of 1) for a given C ip N iz
Ri 1  1
solute. Simplistically, a value of i 1 means that the C if J vz C if
solute i is completely rejected by the membrane: it cannot K P C if
enter the membrane. ) Ri 1  2

C if K 1 P  f K 2 P
A physically clearer interpretation of i has been pro-
vided by Spiegler and Kedem (1966). Imagine the mem- K 2 P
1
brane to consist of regions (Figure 6.3.29(b)) which are K 1 K 2 P  K 1 f
perfectly semipermeable (no solute molecules can pass)
and other regions which are leaky, entirely nonselective, (Applegate and Antonson, 1972).
so that feed solution passes through. Let the hydraulic Example 6.3.10 Expressions (6.3.154c) and (6.3.154d)
permeability of such perfect regions be identified as Lap developed for Ri for a solution-diffusion RO membrane are
and its fractional cross-sectional area be (1 b) (note that not truly predictive of Ri since the right-hand side has a
i for this region is 1); further, let the hydraulic permeabil- term which contains Cip. Develop an expression for Ri
ity for the defective region be Lbp and its fractional cross- that contains only membrane transport parameters, feed
sectional area be b. Then the volume flux through the conditions and P; assume that the osmotic pressure
membrane is given by concentration relation is given by (Cij) bCij
(relation (3.4.61b)).
J vz 1  b Lap P  Lbp b P, 6:3:164a
Solution Assume the following:
since the flux through the defective fraction is governed by Dim im
Darcys law (equation (3.4.88)). We can rewrite this volume J iz C if  C ip K 2 C if  C ip ; 6:3:168
m
flux expression as
C sm Dsm V s
 
" # J sz N sz P  AP  ;
Lap 1  b RTm
J vz 1  b Lap b Lbp P  :
1  b Lap b Lbp 6:3:169

6:3:164b J sz AP  Ab C if  C ip ; 6:3:170


432 Open separators: bulk flow parallel to force and CSTSs


K 2 C if  C ip
J iz =J sz V s C ip
6:3:171
AP  Ab C if  C ip

) AV s PC ip  AbV s C if C ip AbV s C 2ip K 2 C if  K 2 C ip

) AbV s C 2ip K 2 AV s P  AbV s C if C ip  K 2 C if 0

q

2
AbV s C if  K 2  AV s P K 2 AV s P  AbV s C if 4AbV s K 2 C if
) C ip ;
2AbV s

q

2
C ip AV s bC if K 2 AV s P  K 2 AV s P  AV s bC if 4AV s K 2 bC if
Ri 1  :
C if 2AV s bC if

(We have used only the sign before the square root term
(Cili Cile)/2); this, however, will vary with the fractional
in the expression for Cip.)
water recovery, re, defined by (6.3.172a):
The treatment so far has been based on a particular feed
concentration, Cil, in the reverse osmosis cell (Figure 6.3.28 C ip
Ri, reqd 1  : 6:3:173
(a)). As time progresses, water from the feed solution will C ie
be removed as permeate; therefore the feed concentration
of species i, e.g. NaCl, will increase. If we require the Suppose the required permeate salt concentration, Cip,
process to yield a particular concentration of salt in corresponds to 500 ppm. A molar balance of the salt per-
the permeated water, then the salt rejection required of meation process is
the membrane, Ri,reqd, will have to increase. Further, since

the osmotic pressure of the feed solution increases with V f i C ili V f e C ile
V f i  V f e C ip ;

time, either the solvent flux will go down with time or the C ili V f e =V f i C ile 1  V f e =V f i C ip ; 6:3:174
driving pressure difference, P, has to go up. To these C ili 1  reC ile reC ip :
factors, one has to add the complication of concentration
polarization. To illustrate the effect of increasing feed salt

Use this result in the form C ie C ii  reC ip =1  re


concentration with time, we will ignore first the effect of in definition (6.3.173):
any concentration polarization and then focus on the con-
sequence of different values of fractional water recovery, re. C ip 1  re
For the reverse osmosis cell shown in Figure 6.3.28(a), it is Ri, reqd 1 
: 6:3:175
C ii  reC ip
defined as
We now calculate Ri, reqd for three values of re, namely 0,
volume of water permeated
rejbatch : 6:3:172a 0.75, 0.85, for C ili corresponding to 2000 ppm. Our calcula-
volume of initial water feed tions assume very dilute solutions so that ppm values may
For the flow cells/devices of Section 7.2.1.2s, the corres- be used directly without much error:
ponding definition is
500
re 0: Ri, reqd 1  0:75;
water permeation rate 2000
rejf low : 6:3:172b
water feed flow rate 500  0:25
re 0:75: Ri, reqd 1 
Consider Figure 6.3.28(a) and an initial volume Vfi of salt 2000  0:75  500
water of salt concentration Cili. Let the volume of salt 125
water remaining after the process is over be Vfe; let the 1 0:9231;
1625
corresponding salt concentration be Cile. Let the permeate
500  0:15
salt concentration required be Cip. The salt rejection re 0:85: Ri, reqd 1 
2000  0:85  500
required of the membrane, Ri,reqd, may be defined with
respect to the final feed salt concentration Cile (an alter- 75
1 0:9524:
native definition may be the average feed concentration, 1575
6.3 Bulk flow parallel to force direction 433

(a) (b)
Permeate Permeate
Vacuum
Vacuum

Low-pressure Low-pressure
vapor vapor
perm
aij
ij Membrane Membrane
Saturated vapor
Liquid feed aijevap
Liquid feed

|vz| |vz| |vz| |vz|

Figure 6.3.30. Vacuum driven pervaporation in a batch cell. (a) Conventional pervaporation with liquid feed imposed on membrane.
(b) Thermodynamically equivalent pervaporation configuration with evaporation followed by vapor permeation.

These results show how, as the fractional water recovery is vacuum on the other side. In this modified configur-
changes, the required salt rejection also changes ation, the feed imposed on the membrane is a vapor
considerably. mixture. Therefore, the transport equation through the
membrane for species i may be written as (following
equation (3.4.76))
6.3.3.4 Pervaporation
 
In Section 3.4.2.1.1, we were introduced to the rate J iz Qim =m pif  pip : 6:3:176
equations for the membrane separation technique of
pervaporation: a volatile liquid mixture on the feed side However, this hypothetical partial pressure, pif, of species i
of the nonporous membrane at around atmospheric in the feed side is related to the feed liquid phase mole
pressure; the other side of the membrane has a vapor/ fraction, xif, by the equilibrium relation (see relations
gaseous phase, usually at a lower pressure. The creation (3.3.61) and (3.3.62) for lower-pressure situations (so that
and maintenance of a vapor/gaseous phase on the per- sat
i 1))

meate side can be implemented by either passing a pig f^ig f^i x if if f 0i x if if f i


sweep gas/vapor on the permeate side or by pulling a
vacuum. In the configuration of bulk flow parallel to the sat
x if if P sat sat
i i x if if P i : 6:3:177
direction of the force across a membrane (see Figures 6.3
(i)(l)), vacuum will provide for permeated vapor flow One can therefore write the transport equation for the
parallel to the direction of force; sweep gas/vapor will alternative configuration (Figure 6.3.30(b)) for volatile
not (in general). species i in the feed as
To study separation in this flow vs. force configuration,  
it is useful to adopt a process model of pervaporation J iz Qim =m x if if P sat
i  pip : 6:3:178a
somewhat different from those employed in formulating
For volatile species j,
equations (3.4.67ah). Consider Figures 6.3.30(a) and (b).
  
In Figure 6.3.30(a), the volatile liquid feed mixture is J jz Qjm =m x j f jf P sat
j  pjp : 6:3:178b
imposed on the feed membrane: the permeation process
may be described by equations in Section 3.4.2.1.1. On In the bulk flow parallel to force configuration of Figure
the other hand, Figure 6.3.30(b) illustrates a thermo- 6.3.30(b),
dynamically equivalent configuration that is somewhat ! 
different (Wijmans and Baker, 1993): the liquid feed is J iz x ip Qim x if if P sat
i  P p x ip
 , 6:3:179
in equilibrium with a vapor phase; this vapor phase is in J jz x jp Qjm x jf P sat  P p x jp
jf j
contact with the feed side of the membrane while there
434 Open separators: bulk flow parallel to force and CSTSs

where we have assumed pip P p x ip , etc., for low levels of One is interested primarily in finding out the value of xip
absolute permeate pressure P p . Define the following sep- vis--vis xjp in the pervaporation process. A general
aration factors for the binary system of i and j: expression for xip can be derived in terms of xif for a
x ip x jf binary system. Rewrite relation (6.3.179) as follows:
ij ; perm
ij Qim =Qjm ;
x jp x if 

 x if if P sat
x ip perm i =P p  x ip
evap
ij if P sat
i P sat
j jf : 6:3:180 ij  ;
1  x ip P sat
j
1  x if jf  1  x ip
Here, ij is the separation factor for pervaporation, perm is Pp
ij
the ideal separation factor for vapor permeation through the 6:3:183
membrane in Figure 6.3.30(b) and evapij is the corresponding  
perm

perm
separation factor in the evaporative equilibrium between the ij x if if P sat
i =P p 1  x ip  x ip  x 2ip
ij
feed liquid and the hypothetical vapor imposed on the mem-
   
x ip 1  x if jf P sat 2
j =P p  x ip  x ip :
brane. Suppose now that P p , the permeate side absolute
pressure under vacuum, is such that, for both i and j,
Rearrange to obtain
x if if P sat
i >> P p x ip ;
2 0 1
then equation (6.3.179) may be rearranged to yield P sat
0 1 x 2ip perm
ij
4
 1  x ip ij perm perm @
 1 ij x if if i A
Pp
x ip sat
@
Qim A x if if P i ) x ip x jf ij #
x jp Qjm x jf jf P sat
j x jp x if perm P sat
1  x if jf P sat
j =P p ij x if if i 0:
Pp
010 1
Q if P sat
i 6:3:184
@ im A@ A perm evap evap perm :
ij ij ij ij
Qjm jf P sat
j

6:3:181 This is a quadratic in xip. The solution is

h i
perm
ij  1 perm
ij x if if P sat sat
i =P p 1  x if jf P j =P p
x ip
2perm
ij  1
perm perm 2 perm perm 1=2
fij  1 ij x if if P sat sat
i =P p 1  x if jf P j =P p   4ij  1ij x if if P sat
i =P p g
:
2ijperm1

6:3:185

Thus, the selectivity achieved in pervaporation is a product The only useful sign in the numerator is the  one.
of the selectivity achieved first by distillation/evaporation For a given binary system of i and j at a given
equilibrium of the feed liquid into vapor (evapij ) and then temperature, the parameters one can vary are the per-
the ideal selectivity of the vapor permeation through the meate side total pressure, Pp, and the properties of the
membrane perm
ij . One could write in general:28 membrane, and therefore the membrane selectivity,
pip x jf pip C jf perm
ij . Any considerations on the limits of selectivity
x ip x jf
ij achieved due to the variations of these two quantities
x jp x if pjp x if pjp C if
should also  take into account the vapor pressure ratio
0 10 1 0 1 P sat sat
i =P j , or alternatively evap ij given by (6.3.180)
pip pjf pif C jf perm pif x jf (Wijmans and Baker, 1993).
@  A@  A @ A
Consider first the perm
is very large com-
pjp pif pjf C if ij
pjf x if   case where ij
sat sat
pared to P i =P j . It is clear in such a case that, although
pip pif ,
perm
ij evap
ij ; ij evap
ij perm
ij 6:3:182
pip ! pif if P p > pif : 6:3:186a

28 Further,
All such discussions here assume pseudo-steady state since the
feed liquid composition changes with time in this configuration. pip P p if P p < pif 6:3:186b
6.3 Bulk flow parallel to force direction 435

(a) (b)
Batch
40
Cil
cell
Pf >Pp wall
Liquid Pf
Cilb
30
Feed aijevap
(cif,cjf) (pif,pjf) Permeate Pf Pp = P
Permeate
z
pressure
20 dl
( pi + pj ) aij
perm
(pip,pjp) CV C 0il
z=0
Vapor Membrane
10
aij

0 20 40 60 80 100 Pp, pip


Permeate concentration (wt%) Cip

Figure 6.3.31. Consecutive separation corresponding to Figure 6.3.30(b) for ethanolwater azeotropic system. (b) concentration polari-
zation in pervaporation. (After Wijmans and Baker (1993) and Wijmans et al. (1996).)

These results become clear if we assume an infinitely large since pip << pif, pjp << pjf, if the membrane selectivity
perm
value of ij . For example, for the second case (equation is low.
(6.3.186b)), as long as the permeate total pressure, Pp, is The relationship (6.3.182) between the separation factor
less than pif, one can have pure species i in the permeate ij for the pervaporation process and the separation factors
due to an infinitely selective vapor permeation membrane. evap
ij and perm
ij for the hypothetical thermodynamically
Thus the value of the pressure ratio (Pp/Pf) is important in equivalent process for Figure 6.3.30(b) can be conceptually
the case of very large values of permij . Similarly, consider illustrated via Figure 6.3.31(a). This figure is based on a
equation (6.3.186a) and the relation (6.3.182) in the similar one from Wijmans and Baker (1993) showing
following format for perm
ij ! : vaporliquid equilibrium of the azeotropic ethanolwater
! ! system at 60  C. The plot illustrates total permeate pressure
pip pjf evap pjf evap vs. alcohol concentration in the liquid phase (both feed
ij  ij ij : 6:3:186c
pjp pif pjp liquid and permeate vapor). Feed liquid having molar con-
centrations of C if and C jf (mole fractions xif, xjf) are in equi-
If i is a minor component in the feed,
librium with a hypothetical vapor phase (Figure 6.3.30(b))
 
Pf having the partial pressures of pif and pjf. This evaporation
pjf ! P f and pjp ! P p ) ij evap : 6:3:186d
P p ij process leads to a separation factor of evap
ij (here the perme-
ate pressure equals the feed pressure; therefore there is no
In both cases we see that the total pressure ratio (Pp/Pf)
perm membrane selectivity). Permeation of this hypothetical vapor
is quite important
 if ij is very high. On the other
sat sat perm through the membrane leads to a different composition in
hand, if P i =P j is large compared to ij , we the permeate vapor phase, where the total permeate pressure
obtain a different result. Consider relation (6.3.182) in
is pip pjp (< (pif pjf)). The separation factor achieved in
the following form:
this hypothetical process of vapor permeation is perm ij . It is
! ! clear how, in this case, the pervaporation process has created
pif C jf pip pjf
ij   ; 6:3:187a a highly alcohol-enriched permeate vapor phase from a
pjf C if pjp pif liquid feed of alcohol and water. The overall separation is
|{z}
built on two consecutive separations: evaporation followed
evap
ij
by vapor permeation.
! ! !  ! Pervaporation processes are carried out naturally with
J iz pjf Qim pif  pip pjf
ij evap
ij evap
ij
  ; volatile liquid mixtures. The liquid mixtures are of three
J jz pif Qjm pjf  pjp pif
types:
6:3:187b
(1) aqueous solution, containing volatile organic com-
ij evap
ij perm
ij , 6:3:188 pounds (VOCs);
436 Open separators: bulk flow parallel to force and CSTSs

(2) organic solution, containing water; liquidmembrane interface since the back diffusion pro-
(3) organic solution, containing organic species only. cess in the liquid film layer next to the membrane is not
In the liquid mixture of type (1), the VOCs may be present fast enough.
in small amounts from a few parts per million (ppm) to a To develop a quantitative estimate of this effect, we
few percent. Some examples are: wastewater streams con- follow the procedure followed earlier (see equations
taining small amounts of VOCs; process aqueous streams (6.3.142a) and (3.4.65b)) with due attention to the differ-
containing small amounts of VOCs (valuable or not); fer- ences here. Assume a pseudo steady state (for this batch
mentation broth containing volatile bioproducts, e.g. etha- cell in force parallel to bulk flow configuration) for the
nol, butanol, acetone, acetic acid, etc. control volume (CV):

     
solute flux into solute flux into solute flux out of
;
CV by convection CV by diffusion CV by membrane transport
d C il 6:3:190a
jvz j C il Dil jvz j C ip :
dz

In liquid mixtures of type (2), the solutions of primary Here jvz j is the magnitude of the liquid velocity generated
interest are: azeotropic and other mixtures containing variable in the film layer due to the permeation process in perva-
amounts of water in organics; dehydration of organic solvents poration. If we have only two species i and j in the feed
containing very small amounts of water. Removal of water liquid, then
from azeotropic mixtures of ethanolwater, isopropanol  
water, etc., is extensively practiced using polymeric mem- jvz j J iz J jz =C tl , 6:3:190b
branes (of crosslinked polyvinyl alcohol) that are highly polar
and selective for water. On the other hand, the membranes where C tl is the total molar density of the feed liquid. We
that are used to remove VOCs selectively from aqueous solu- may now rearrange equation (6.3.190a) as follows:
tions are usually highly nonpolar rubbery polymeric mem-

dC il
branes, e.g. dimethyl siloxane (silicone rubber). jvz j C il  C ip Dil ;
For liquid mixtures of type (3), there are a number of dz
Cilb
0 1
different types of mixtures: aromaticaliphatic separation; l
dC il C ilb  C ip
sulfur-containing organics from other petroleum constitu-
jvz j=Dil dz ) ln @ 0 A
C il  C ip C il  C ip
ents; olefins from paraffins, etc. C 0il 0

jvz j jvz j
6.3.3.4.1 Concentration polarization in pervaporation   ;
Dil =l k il
We have seen in Section 3.4.2.1.1 that the volatile species
permeating through the membrane in the pervaporation 0 1
process is driven by its concentration gradient in the non- C 0il  C ip jvz j C ip  C 0il
exp @ A : 6:3:190c
porous membrane. The concentration of this species in the C ilb  C ip k il C ip  C ilb
membrane at the feed liquidmembrane interface, C 0im , is
related to the feed liquid-phase concentration at this inter- This relation is identical to equation (6.3.142b); however,

face, C 0il , by a partition coefficient im : if the ratio C ip =C ilb is greater than 1, as is true here
(which is exactly the reverse of what happens in RO and
C 0im
im : 6:3:189 UF, where the solute is rejected by the membrane), we
C 0il
have severe concentration polarization when k il << jvz j.
For a steady state pervaporation process to take place, For example, suppose jvz j=k il e1, and the membrane is

molecules of species i have to diffuse from the bulk liquid such that C ip =C 0il is, say, 1000. Then, rewriting equation

C ilb to this interface C 0il < C ilb (see Figure 6.3.31(b)). (6.3.190c) as
Since C 0il is less than C ilb , C 0im is less than im C ilb . There-
fore, the species i permeation rate through the membrane C ip =C 0il  1


exp1 e
is reduced. This is exactly the opposite of what happens in C ip = C 0il  C ilb =C 0il
ultrafiltration (Figure 6.3.26(a)) or reverse osmosis (Figure
1000  1
6.3.28(a)), where the solute is being rejected by the mem-
e 2:718
1000  C ilb =C 0il
brane and therefore its concentration builds up at the feed
6.3 Bulk flow parallel to force direction 437

implies that and the concentration polarization relation (6.3.191e) as


follows:
0

2718  999

C ilb =C il 632:4:
1 C tl x ib  x 0i
2:718 J iz C ip jvz j k i C tl x ib  x 0i ) ;
k i C ip jvz j
Therefore the species i concentration in the liquid imposed 0 1
on the membrane surface is drastically reduced, leading to x 0i  x ip x 0i  x ib jvz j
a substantial reduction in the species i flux. Consequently, expjvz j=k i ) exp @ A  1
x ib  x ip x ib  x ip k i
concentration polarization can be severe in pervaporation 0

2 0 1 3
processes where the membrane enriches the species sub- x i =x ip  x ib =x ip jv z j
)
4exp @ A  15:
stantially in the permeate. An example is the selective x ib =x ip  1 k i
removal of VOCs from an aqueous solution through a
6:3:193a
rubbery pervaporation membrane (e.g. silicone polymer

based). For a description of such concentration polariza- Defining x ip =x ilb as Ei, the effective enrichment of the
tion effects in VOC removal from water, consult Wijmans species i by the pervaporation process, we get
et al. (1996).
    
We will now estimate the effective overall permeance x ib  x 0i jvz j 1

exp 1 1 : 6:3:193b
of species Qov x ip k i Ei
im =m due to the concentration polarization
effect since (Wijmans et al. (1996)) Employ these two results in relation (6.3.192) to replace

 
1=k il :
J iz Qovim =m x ib ib P sat
i  pip ; 6:3:191a
2 0 1 32 3

jv j 1
J iz k i C t x ib  x 0i ; 6:3:191b 0 1 0 1 ib P sat 4
i C t x ip exp
z
@ A 1541 5
  k i Ei
@ m A@ m A
J iz Qim =m x 0i 0i P sat
i  pip , 6:3:191c Qimov
Qim C t jvz jC ip
0 1 2 32 0 1 3
where
@ m A ib P sati 4 1 54 jvz j
1 exp @ A 15:
C ib C t x ib , C 0i C t x oi : 6:3:191d Qim jvz jC t Ei k i

Although il depends in general on the liquid-phase com- 6:3:194


position, ilb is not, in general, equal to 0il . However, we
This result is more general in the following sense. If a
will assume it to be valid here. Utilize now the concen-
species i does not undergo enrichment (Ei 1), there is
tration polarization relation (6.3.190c) in terms of mole
no concentration polarization effect. For components
fractions of species i:
which are depleted in the permeate (Ei < 1), the perme-

C t x ip  C t x 0il x ip  x 0i ance is increased over Qim =m , due to an increase in its


expjvz j=k il : 6:3:191e
C t x ip  C t x ib x ip  x ib concentration in the liquid film, and therefore in its trans-
port rate. Further, as the permeation rate increases, jvz j
Rearrange equations (6.3.191b) and (6.3.191c) as follows: increases; under such a condition, increasing m will lead
J iz ib P sat to a relative reduction in the liquid film resistance with
i
ib x ib P sat 0 sat
i  ib x i P i ; respect to the membrane resistance (Wijmans et al.,
k i C t
J iz 1996).
x 0i 0i P sat
i  pip :
Qim =m Example 6.3.11 Membranes made from crosslinked polyvi-
nyl alcohols are known to be highly selective for water over
Adding these two equations and assuming 0i ib , we get
alcohols, ethanol, propanol, etc. Schaetzel et al. (2001) have
  suggested that the diffusion coefficient of each component
P sat m
J iz ib i x ib ib P sat
i  pip , depends only on the weight fraction of the key component,
k i C t Qim

water, in such a membrane:
so that, from the definition (6.3.191a) of Qov im =m ,
J H2 O DH2 O, 0 exp H2 O uH2 O SH2 O
m m P sat  
ib i : 6:3:192  x H2 O, f H2 O, f P sat
H2 O  x H2 O, p P p ;
Qov
im Qim k i C t
 
J alc Dalc, 0 exp alc uH2 O Salc x alc, f alc, f P sat
alc  x alc, p P p :
This relation allows one to estimate the effective overall

permeance Qov im =m in the presence of concentration The values of the products DH2 O , 0 SH2 O for water and
polarization. Dalc, 0 Salc for i-propanol at 45  C are 8  104 and 7.4 
An alternative expression is often more useful. 107, respectively, in kg/hr-m2-mm Hg. If the permeate side
Rearrange the liquid film transport equation (6.3.191b) partial pressures may be neglected and H2 O alc ,
438 Open separators: bulk flow parallel to force and CSTSs

determine the separation factor at the azeotropic compos- Now,


ition of around 87% i-propanol by weight at 1 atm.
x VOC, p x H2 O, f J VOC, z x H2 O, f
VOCwater    ;
Solution At the azeotropic composition for any species in x H2 O, p x VOC, f J H2 O, z x VOC, f
vaporliquid equilibrium at pressure Paz,
Qov
VOC, m
x ig P az x if if P sat and x if x ig J VOC, z ib x if P sat
i  x ip P p i VOC;
i m
implies that Qov
VOC, m
J VOC, z ib x if P sat
i ;
P az if P sat
i H2 O, f P sat
H2 O alc, f P sat
alc : m
The separation factor Qov
water, m
J H2 O, z ib x if P sat
i  0 i water;
x H O, p x alc, f J H2 O
x alc, f m
H2 Oalc 2
x alc, p x H2 O, f J alc
x H2 O, f Qov
VOC, m =m VOCb x VOC, f P VOC
sat
x H O, f
  VOCwater ov sat  2
DH2 O, 0SH2 O exp H2 O uH2 O x H2 O, f H2 O, f P sat Qwater, m =m H2 Ob x H2 O, f P H2 O x VOC, f
H2 O x alc, f
 
H2 Ob 1
Dalc, 0 Salc exp alc uH2 O x alc, f alc, f P sat
alc x H2 O, f
! C t k VOC, m VOCb P sat
VOC
DH2 O, 0 SH2 O exp H2 O uH2 O x H2 O, f P az x alc, f
VOCb P sat
VOC Qwater, m P sat
H2 O
Dalc, 0 Salc exp H2 O uH2 O x alc, f P az x H2 O, f
! C t k VOC, m
:
DH2 O, 0 SH2 O 8  104 M alc Qwater, m P sat
H2 O

Dalc, 0 Salc 7:5  107 M H2 O
The VOCwater selectivity, defined as Qov ov
VOC =QH2 O , is given by
8  104  60
3555: C t k VOC, m
7:5  107  18
VOCb P sat
VOC Qwater, m
The separation factor defined in terms of weight fractions will
6.3.3.5 Membrane gas permeation
be 1066.
In Section 4.3.3, we studied the time-dependent potential
Example 6.3.12 Consider selective pervaporative transport of separation of a binary gas mixture by permeation
of a VOC species i through a silicone rubber membrane in through a nonporous membrane in a closed vessel.
preference to water. The separation conditions are such that A certain amount of a binary gas mixture was introduced
Ei is >> 1, of the order of 1000; further, the permeation rates
on one side of the membrane, the other side being empty
are quite low with respect to the feed side mass-transfer
at t 0 in the closed vessel. Initially there was some
coefficient (i.e. jvz j << k il ). In this pervaporation process, it
is known that the membrane resistance is quite low but the separation but all separation was lost ultimately as an
boundary layer resistance is quite high for the VOCs. On the equilibrium was reached between the gases on the two
other hand, for water, it may be safely assumed that the sides of the membrane. Since membrane gas separation
membrane resistance controls the water transport. Deter- relies on intrinsically different permeation rates of different
mine an expression for the separation factor, and find the species, we need an open system so that we can continu-
selectivity of the membrane for VOC over water when the ously remove the separated products in the permeate
permeate pressure is negligible. Assume: that the VOC con- stream.
centration in water is low for this pervaporation process. Consider now an open system with both feed gas and
Solution: Consider the general relation (6.3.194):
the permeated gas flowing parallel to the direction of the
        
m m P sat 1 jvz j force (Figure 6.3.32), toward the membrane for the feed
ilb i 1 exp 1 :
ov
Qim Qim jvz jC tl Ei k il and away from the membrane for the permeate; it is as if
there is a hypothetical piston driving the feed gas toward
For i VOC,
      the membrane. This is achieved by connecting a gas source
1 jvz j at constant pressure to the feed side of the membrane. The
1  1, exp  1 jvz j=k il for jvz j << k il :
Ei k il gas velocity toward the membrane is determined by the
Therefore rate at which gases permeate through the membrane and
    are withdrawn immediately from the permeate side. Note
m m P sat P sat
ov ilb i ilb i i VOC: that the cross-sectional area of the membrane through
QVOC m QVOC m C tl k il C tl k il which gas permeation takes place is the same as that of
For water, the corresponding result is the feed flow channel/piston and the permeate flow chan-
    nel. Any reduction or increase in the flow cross-sectional
m m area on either side of the membrane will introduce gas
:
Qov
water m Qwater m velocities in other directions. Such situations are of great
6.3 Bulk flow parallel to force direction 439

Pf xAf Flow Gas cylinder


direction Mixture of A and B

Force direction
Membrane
Pp xAp
xAf > xAp

Flow direction Pf > Pp

Figure 6.3.32. Gas permeation device, where the bulk gas flow direction is parallel to the direction of force across the membrane for
separating a gas mixture; the membrane is selective for species A.

practical use and are considered later in Section 6.4 and permeate side; under such conditions, the expression for
Chapters 7 and 8. the separation factor, AB, for the membrane between the
For a binary feed mixture of gases A and B at feed two gas species A and B is given above. The quantity

partial pressures of pAf and pBf and permeate partial pres- P p =P f is identified as , the pressure ratio, and (Qim/
sures of pAp and pBp respectively, the permeation rates per m), is the permeance of species i.
unit membrane area of the two species are, respectively When ! 0, i.e. the permeate side has a very low
(see equation (3.4.72)), pressure compared to that in the feed side, we find
QAm x Ap x Bf QAm =m
N Az J Az pAf  pAp ; AB ) AB , 6:3:198
m x Bp x Af QBm =m
Q
N Bz J Bz Bm pBf  pBp : 6:3:195
m where AB is called the ideal separation factor achieved in
the limit of zero pressure ratio. The condition 0 leads,
If the mole fractions of A and B in the feed gas mixture and
for the binary system, to a permeate gas composition x Ap
the gas mixture emerging from the membrane as the per-
given by
meate are, respectively, x Af , x Bf , x Ap , x Bp , then
  x Ap x Ap x x

AB Af AB Af
:
N Az J Az x Ap QAm =m pAf  pAp x Bp 1  x Ap x Bf 1  x Af
  : 6:3:196
N Bz J Bz x Bp QBm =m p p
Bf Bp
Therefore
Rearrange this equation as AB x Af

x Ap : 6:3:199
x Ap Q =m P f x Af  P p x Ap 1 x Af AB  1
Am
,
x Bp QBm =m P f x Bf  P p x Bp For nonzero values of , the separation factor relation
where Pf and Pp are the total gas pressures on the feed and (6.3.197) can now be rewritten as follows:
permeate side, respectively. Rearrange further to obtain

x Ap x Af  x Ap
 
AB

, 6:3:200
P p x Ap 1  x Ap 1  x Af  1  x Ap
1
x Ap x Bf Q =m P f P f x Af
AB Am  :
which leads to a quadratic in x Ap in terms of AB , and x Af :
x Bp x Af QBm =m P f P p x Bp
1
P f x Bf 


x 2Ap AB  1  x Ap x Af x AB  1 1 AB x Af 0:
6:3:197

At any time, let x Af , x Bf and P f represent the conditions on The solution for nonzero values of is (Huckins and
the feed side and let x Ap , x Bp , P p represent those on the Kammermeyer, 1953a,b)
440 Open separators: bulk flow parallel to force and CSTSs

n rn



x Af  o 
x
 o2   1 x Af
AB  1 1 1 1 Af AB  1 1  4 AB AB
x Ap 6:3:201
2 AB  1

 
In this expression for x Ap , only the negative root is mean- QAm
W tAm Am pAf  pAp ;
ingful in the numerator since 0 x Ap 1 for 0 x Af 1. m
Given AB , and x Af , one can now calculate  

QBm
x Ap 0 x Ap 1 . The permeances of species A and B, W tBm Am pBf  pBp : 6:3:203
m
QA =m and QB =m , are likely to be known for the mem-
brane; therefore AB is known. The pressure ratio, , is This mode of operation is such that the rate at which the
generally specified. For a given feed (known x Af ), the per- gas stream enters the device from the gas source equals
meate mole fraction, x Ap (which will vary between 0 and 1; the rate at which the gas stream permeates through the
x Ap > x Af for AB >1), is easily calculated for this mode of membrane ( stage cut 1). This ensures a fixed feed
operation. pressure Pf. (If the gas stream enters the device from the
Sometimes one needs to calculate the actual separ- source at a rate higher than the permeation rate, the feed
ation factor AB defined by (6.3.197). Write this expression gas pressure will keep on increasing since it is a compres-
as sible fluid, unlike that in the liquid-phase systems con-
 
sidered so far.) However, as permeation takes place, if
1  x Ap =x Af
AB AB : AB > 1, species A permeates more than species B; the
1  x Bp =x Bf gas mixture left on the feed side has more of species
B. Since the feed gas mixture having a constant compos-
Multiply the top and the bottom on the right-hand side by
ition xAf enters the feed side from a gas source, the actual
xBf/xBp:
0 1, 0 1 gas composition imposed on the membrane on the feed
side will keep on changing with time. This mode of oper-
 @ x Bf A @ x Bf
AB AB  AB  A ation cannot therefore maintain a steady state. The per-
x Bp x Bp
meate composition will also keep on changing with time
0 1
as the feed side composition has an increasing amount of
x Bf
) AB @  AB A AB x Bf =x Bp species B with time.
x Bp
The permeation rate of gases through nonporous
0 1 membranes is generally quite low. To achieve a high pro-
x Bf =x Bp duction rate in larger devices, a large membrane area is
) AB AB @
A,
x Bf =x Bp AB  1 needed. In this mode of operation (bulk flow parallel to
force), the feed channel cross-sectional area has to be
6:3:202a
equally large; similarly on the permeate side. This mode
which expresses AB in terms of AB , , x Bf and x Bp . A more of operation is therefore not practical for large-scale oper-
useful expression for AB eliminates x Bp from the right- ations (where the cross-sectional areas of the incoming
hand side to express it as a function of x Af , and AB (Stern feed gas stream and the outgoing permeate gas stream
and Walawender, 1969): are vanishingly small with respect to the membrane sur-
( face area in practical devices). One may use such a mode of



AB 1 AB  1 1 AB  1 2 operation only for a very small membrane area.
AB  
2 2 2 x Af 2 Some of the relations developed earlier, however, are
very basic to gas permeation processes: these include

   )1=2
AB  1  AB 2  1 AB  1 1 2 (6.3.197) and (6.3.201). For a given x Af and AB , as
:
2 x Af 2 x Af increases, the separation achieved decreases; as
6:3:202b decreases, the separation factor AB (definition (6.3.197))
increases, reaching AB in the limit ! 0. These relations
In the configuration shown in Figure 6.3.32, the feed gas are valid at any point in any gas permeator where x Af is the
composition and pressure imposed on the membrane feed mole fraction of A and x Ap is the corresponding mole
are uniform everywhere along the device, and likewise fraction in the permeated gas mixture as it emerges from
on the permeate side. Therefore, the total molar perme- the membrane (before mixing with any other gas mixture
ation rates of species A and B through the membrane in the permeate side of the device). Figure 6.3.33 illustrates
are: the value of the O2 mole fraction in the permeate side (xAp)
6.3 Bulk flow parallel to force direction 441

1 This is quite a high value. There are almost no polymeric


=0 membranes having such an ideal selectivity for O2 over N2.
There are, however, many different types of polymeric
0.8
Permeate mole fraction

0.2 membranes having a selectivity of O2 over N2 between 2


0.4 (silicone rubber) and 8 (polyestercarbonates) for produ-
0.6 cing oxygen-enriched air (OEA).
of O2, xAp

0.6
0.8 In Example 6.3.13, we were calculating/focusing on the
0.4 = 1.0 permeate composition, i.e the increase in O2 content over
that in the feed. Correspondingly, the N2 content of the
0.2 feed gas stream was increasing as O2 was permeating
selectively through the membrane. If a considerable
amount of membrane area is provided, the residual gas
0
left on the feed side will be highly concentrated in N2,
0 0.5 1 producing what is called nitrogen-enriched air (NEA); such
a gas mixture is quite useful for producing a relatively inert
Feed mole fraction of O2, xAf atmosphere, which is required in many applications, e.g.
inerting29 chemical storage tanks, on-board inert gas gen-
Figure 6.3.33. Effect of pressure ratio, , on the permeate mole eration in aircrafts, controlled atmosphere for fruits/vege-
fraction of oxygen through a cellulose acetate membrane (
AB 6) tables, etc. All such applications, however, utilize bulk flow
for a feed mixture of O2N2. of feed gas perpendicular to the force direction across the
membrane; see the appropriate treatments in Chapters 7
for different values of O2 mole fraction (xAf) on the feed and 8.
side of a cellulose acetate membrane exposed to various Example 6.3.13 and Figure 6.3.33 illustrate separ-
O2N2 gas mixtures; the different curves are for various ations involving * around 6. There are practical mem-
values of . The curve corresponding to 0 provides brane gas separation cases where * can be as high as
the highest O2 mole fraction in the permeate. This value 100 and more. Such membranes allow the achievement
is easily calculated from relation (6.3.197). All practical gas of a highly purified permeate. For example, there are
separation devices having a finite area will yield lower practical glassy polymeric membranes in use that yield
values since > 0, and the feed composition will change  H2 CH4 of around 100. In the separation of solvent
along the membrane, as we shall see in the relevant parts vapors (and volatile organic compounds (VOCs) in gen-
of Sections 7.2.1.1 and 8.1. eral) from N2/air, selectivities around 50500 can be
Example 6.3.13 Consider production of enriched O2 in a achieved using rubbery membranes, leading to a per-
permeate stream from atmospheric air. meate very substantially enriched in VOCs in the
permeate.
Case (1) You
have a
cellulose acetate membrane having Considerations on the extent of permeate enrich-
a value of QO2 =QN2 6:0. What will be the permeate
composition for a pressure ratio, 0? ment in the preferentially permeated component are
Case (2) You would like to produce tonnage oxygen (95% facilitated if we consider expression (6.3.201) for x Ap .
The value of x Ap depends on AB , and x Af . For a given
O2) from atmospheric

air. What is the minimum value of
QO2 =QN2 6:0 required under the condition 0? x Af , are there regions where AB is controlling or is
controlling? Consider Figure 6.3.34, where Baker and
Solution Case (1) We employ equation (6.3.198) valid for
Wijmans (1991) illustrate the VOC composition in the
0, where A O2, B N2. We have

permeate as VOCN2 and values of are varied for a feed


x Ap = 1  x Ap AB x Af = 1  x Af ; VOC concentration of x VOC, f 0.005. One notices that, in
x O2 f 0:21;  O2 N 2 6:0; the range 0.11, it essentially does not matter what

AB
x O2 p = 1  x O2 p 60:21=0:79 1:595; the ideal membrane selectivity (VOCN2 ) is; the permeate
1:595 composition is essentially controlled by the pressure
x O2 p 1:595  1:595 x O2 p ) x O2 p 0:614:
2:595 ratio, . On the other hand, in the range 0.010.1
and below 0.01, the permeate composition appears to be
Case (2) A O2, B N2, so determined essentially by VOCN 2 , with higher values

leading to very high values of x Ap . In this region of ,


x Ap = 1  x Ap AB x Af = 1  x Af ;
0:95 0:21
x Ap 0:95; x Af 0:21 ) AB
0:05 0:79
19
) AB 71:7:
0:265 29
Effort or steps taken to create an inert atmosphere.
442 Open separators: bulk flow parallel to force and CSTSs

value of the following ratio (note: the mass-transfer


Permeate vapor concentration
80
* coefficient for diffusion in the gas phase was obtained
= 500
Greater enrichment

60 from relation (3.1.132)):


* = 200 Qim =m
40

* = 100
(%)

0.5% Dij =RTg :


* = 50 Feed
20
* = 20 Assume, for illustration, that at 25  C we have a silicone
0 rubber membrane which is highly permeable to, say,
0.001 0.01 0.1 1 oxygen:

Pressure ratio (g = Pp /Pf ) Qim 5  108 cm3 STP  cm= cm2  s  cm Hg :

Let m 1 m 104 cm. From Table 3.A.1, the value


Higher vacuum
of DO2 N2 may be assumed to be ~0.18 cm2/s; assume:
g 1 cm; R 82.05 cm3-atm/gmol-K; T 298 K.
Figure 6.3.34. Permeate vapor concentration for a 0.5% VOC feed
as a function of VOCN2 and pressure ratio . (After Baker and
We have
Wijmans (1991).)

Qim 5  108 cm3 STPcm 1



m 104 cm2 scm Hg cm

Dij cm2
0:18
RTg s
cm3 atm cm Hg 1 gmol 1 liter
82:05  76    298 K  1 cm
gmolK atm 22:4 liter 103 cm3
cm3 STP
5  104
cm2 scm Hg
;
0:18 22:4  103 cm3

82:05 76  298  1 cm2 scm Hg
Q = 5  104  82:05  76  298 5  0:82  0:76  0:298
im m
0:23:
Dij =RTg 0:18  22:4  103 0:18  22:4

then, the permeate composition is controlled by the ideal Therefore, for a highly permeable membrane like silicone
membrane selectivity. One may conclude the following: rubber, if the membrane thickness is smaller than 1 m,
there will be a concentration polarization effect leading to a
high ! pressure ratio controls;
higher concentration of species B over the membrane
very low ! ideal membrane selectivity controls:
(Alpers et al., 1999). Practical membranes of glassy poly-
There is an important consideration in the separation mers used for many gas separations have lower thick-
of a gas mixture through a membrane, as shown in nesses, even though their Qim values are usually much
Figure 6.3.32. For a similar configuration, we have lower. For example, glassy polyimide H2-selective mem-
seen in Figure 6.3.26 for ultrafiltration that the concen- branes are reported to have a value of

tration of the rejected species builds up on the feed QH2 m =m 100  500  106 cm3 STP=cm2 -s-cm Hg
side of the membrane: the extent of the buildup is (Zolandz and Fleming, 2001), which is pretty close to the
dependent on the solvent flux through the membrane value we used for oxygen through a silicone membrane of
and the rate of back diffusion of the species, specific- 1 m thickness.
ally the ratio jvz j=k i . The lower the value of this
ratio, the lower the buildup of the rejected species 6.3.3.5.1 Composite membranes in gas permeation In
on the feed surface of the membrane. If we employ practical membrane based gas separation, the membranes
the same analogy here, then we should determine the as fabricated are often composite in nature: one membrane
6.3 Bulk flow parallel to force direction 443

(a)
pif
Membrane 1 d1
pii
Membrane 2 d2
pip

(b)
Defect

Coating A dA
Dense Skin B C B C dC B dB
asymmetric
membrane D
D dD
substrate Integral D
microporous
backing Not to scale

Figure 6.3.35. (a) Schematic of a two-layer composite membrane. (b) Schematic of a composite PRISM-type membrane.

layer of material 1, having a very small thickness 1, is 1


Qim = : 6:3:205a
supported on another membrane layer of material 2, of 1 =Qi1 2 =Qi2 
thickness 2. Let the species i permeance of the first layer of
An alternative representation,
membrane be Qi1 =1 and the species i permeance of the
second layer be Qi2 =2 . As shown in Figure 6.3.35(a), the
=Qim Rim 1 =Qi1 2 =Qi2  Ri1 Ri2 ,
feed gas mixture imposed on the membrane in the bulk
6:3:205b
flow vs. force configuration of flow parallel to force leads to
a hypothetical partial pressure profile of gas species i: pif to implies that if the membrane flux of species i, Niz, may be
pii to pip , where pii corresponds to that at the interface of represented in the manner of Ohms law, namely,
the first and second layers. Therefore, at steady state,
voltagepartial pressure driving force
currentflux
Q resistance
N iz J iz i1 pif  pii ; 6:3:204a
1 then the resistance of the two-layer composite membrane,
Q Rim, is merely the sum of the resistances of the individual
N iz J iz i2 pii  pip 6:3:204b layers in series (as in Ohms law) (Figure 6.3.35(a)). For n
2
  layers in series, relation (6.3.205b) may be generalized as
) pif  pii N iz =Qi1 =1 ; 6:3:204c follows:
  =Qim Rim 1 =Qi1 2 =Qi2    n =Qin :
pii  pip N iz =Qi2 =2 : 6:3:204d
6:3:205c

The ideal separation factor, ij ,


between two species i and j
Adding the last two equalities results in
being separated through a composite membrane having
  two layers, 1 and 2 (Figure 6.3.35(a)), is
pif  pii pii  pip pif  pip
2 3 Q = 1 =Qj1 2 =Qj2 
1 1 5 ij  im  : 6:3:206a
N iz 4 Qjm = 1 =Qi1 2 =Qi2 
Qi1 =1 Qi2 =2
0 1
pif  pip Q Following relation (6.3.197), the actual separation factor,
) N iz @ im A pif  pip : ij , for a composite membrane having two layers 1 and 2 is
1 =Qi1 2 =Qi2 
 
Therefore, for a composite membrane having two layers 1 1 =Qj1 2 =Qj2  1  x ip =x if
and 2, the overall effective membrane permeance Qim = ij   : 6:3:206b
1 =Qi1 2 =Qi2  1  x =x
is related to the permeances of the individual layers by jp jf
444 Open separators: bulk flow parallel to force and CSTSs

Many glassy membranes in commercial use have a struc- Therefore


ture more complicated than that shown in Figure 6.3.35(a).  
As shown in Figure 6.3.35(b), the composite membrane RiB RiC
=Qim Rim RiA RiD , 6:3:209
(called the PRISM membrane (Henis and Tripodi, RiB RiC
1981)) consists of a skin layer, B (of thickness B), of an
where
asymmetric glassy membrane which has some defects. On
top of the skin layer, a rubbery coating of material A, C B B
RiC ,
having thickness A, has been applied. However, this rub- C QiC C QiC C QiA
bery material has penetrated the defects of the glassy skin 0 1 0 1
region B; identify these defective regions filled by the B D
RiA A =QiA , RiB @ A, RiD @ A:
rubber material as region C of thickness C. Assume that QiB 1  B QiD
B is the fraction of the surface area covered by the defects
6:3:210
in the glassy skin. Further, let QiA and QiB be the permea-
bility coefficients of species i through the materials A and Often the resistance of the porous substrate region, RiD, is
B, respectively. Assume also that the porous substrate of neglected in comparison to the others:
the asymmetric glassy membrane B skin has a thickness D  
and a species i permeability coefficient QiD. RiB RiC
=Qim Rim RiA : 6:3:211a
It is now useful to postulate that, at distance A from RiB RiC
the top, a hypothetical partial pressure piA exists; simi-
larly, at distance (A B), the hypothetical partial pres- The ideal separation factor for two species i and j in the
sure of species i, piB, exists. Assume now that B C. The flow vs. force configuration of Figure 6.3.35(b) is given by
flux of species i, Niz, may be described as follows:  
RjB RjC
QiA RjA
N iz p  piA ; 6:3:207a Q = Rjm RjB RjC
A if ij  im   : 6:3:211b
Qjm = Rim RiB RiC
RiA
RiB RiC
QiB 1  B B QiA
N iz piA  piB piA  piB ;
B C
The utility of the structure of Figure 6.3.35(b) was acciden-
6:3:207b
tally discovered since it made the glassy polymer skin
QiD region B useful by increasing considerably the resistance
N iz p  pip : 6:3:207c
D iB of permeation through the defect region C by letting the
rubbery polymer coating penetrate the defects. Otherwise
Relation (6.3.207b) indicates that gas species i is diffusing
the high and nondiscriminating permeation through the
in parallel through region B and region C. Now
defects (region C) will drastically reduce the selectivity.
pif  piA N iz =QiA =A ; Normally the skin region was made up of polysulfone, a
N iz glassy polymer, whereas the highly permeable polydi-
piA  piB ( );
QiB 1  B QiA B methylsiloxane (silicone rubber) was used to coat the poly-
sulfone material.
B C

piB  pip N iz =QiD =D : 6:3:207d Example 6.3.14 Henis and Tripodi (1981) have described
the separation performance of a H2CO gas mixture through
Therefore a PRISM-type membrane using the following quantitative
information for a membrane of glassy polysulfone having a
pif  pip
2 3 coating of silicone rubber:

6 1 1 1 7 QH2A 5.2  108 scc-cm/cm2-s-cm Hg; QCOA 2.5  108


N iz 6
4Q =A Q 1  B Q B  Q 5
7
scc-cm/cm2-s-cm Hg; A 1  104 cm (~1 m); QH2B 1.2 
iB
iA
iA iD
109 scc-cm/cm2-s-cm Hg; QCOB 3  1011 scc-cm/cm2-s-
B B D
cm Hg; B 1  105 cm (0.1 m); 2  106.
since
(1) Calculate (QH2m/) and (QCOm/) for this composite
Qim membrane.
B C ) N iz pif  pip
(2) Calculate the ideal separation factor H2 CO .
pif  pip
    6:3:208 Solution (1) We will neglect the resistance of the porous
A B D substrate layer D in Figure 6.3.35(b). We now employ equations
:
QiA 1  B QiB B QiA QiD (6.3.210) and (6.3.211a) to calculate (QH2m/) and (QCOm/ ):
6.4 Continuous stirred tank separators 445


1
QH2 m =
A 1

QH2 A QH2 B 1  2  106 QH2 A 2  106

1  105 1  105
1
2 3
4
4 1  10 1  105 5

5:2  108 1:2  109 1  2  106 5:2  108  2  106
1 1
2 3 9:75  105 scc=cm2 -s-cm Hg:
5 1923 8333
41923 1  10 5
1:2  109 10:4  1014

Similarly,
1
QCOm = 2 3
4A =QCOA 1  105 5
QCOB 1  2  106 QCOA  2  106
1 1
2 3
5 4000 333 333
41  104 =2:5  108 1  10 5
3  1011 5  1014
2:9  106 scc=cm2 -s-cm Hg:

(2) The ideal separation factor, diffusional rates are considerably slower in liquid solu-

tions; therefore, in some separator vessels, stirring of the
H2 CO QH2 m = =QCOm =
liquid is commonly practiced. If the stirring is efficient, one
could imagine that the concentration, temperature, etc.,
9:75  105 =2:9  106 33:6:
are uniform throughout the vessel containing the liquid
Since the ideal separation factor for polysulfone is (1.2  making it a well-stirred vessel and a well-mixed separator.
109/3  1011) ~ 40, and that for silicone is 2.08, this compo- When separation requires contacting two immiscible
site membrane behaves more like the polysulfone membrane. phases, stirring is needed to disperse one phase into the
other to generate a high interfacial area and high mass-
6.4 Continuous stirred tank separators transfer rates. If such separators have essentially uniform
conditions everywhere in the separator, we can also have a
In this section, the flow vs. force configuration of a continu-
well-stirred/well-mixed separator; the term perfectly
ous stirred tank separator (CSTS) will be illustrated with a
mixed is also used. Note that, in some physical configur-
few examples. The examples cover crystallization, solvent
ations, it will be difficult to create such an environment, for
extraction, ultrafiltration and gas permeation.
example in highly viscous, high-temperature melts in
solidliquid systems.
6.4.1 Well-mixed separators CSTSs and batch If fluid streams with or without solid particles are
separators entering and other streams are exiting the well-stirred sep-
In some separators, the operating conditions may be such arator continuously, we have a continuous stirred tank sep-
that, throughout the separator, pressure, concentration, arator (CSTS), provided that its properties are uniform
temperature, etc., are uniform. In separation techniques throughout the separator. Figure 6.4.1(a) illustrates a CSTS
carried out in the absence of external forces, such a situ- which is a crystallizer. The conditions in such a separator
ation creates uniform conditions for separation throughout are time- and space-invariant. However, the intensity of
the separator. A small membrane cell exposed to a feed gas mixing conditions in the separator is such that the fresh
mixture (Figure 6.4.1) is such a separator since very high feed introduced into the separator is mixed in a time interval
diffusivity in the gas phase does not allow any concen- which is very short compared to the mean residence time of
tration gradient to develop due to selective gas permeation the fluid elements (and solid particles) in the separator.
through the relatively impermeable membrane. The Figure 6.4.1(b) illustrates a continuous well-stirred extractor
446 Open separators: bulk flow parallel to force and CSTSs

(a) (b)

Qf , rif , nf (rp)

Wtf, xif WtR, xiR

n1 (rp)
Wts, xis
Q1, ri1, n1 (rp)
Slurry/solution
volume, V WtE, xiE

(c) (d)

Pf t = 0, Vf 0, Ci 0 Concentrate/residue
Feed
t = t, VfR, CiR Pf xi 2 Wt2, xi2
Wtf, xif
Membrane
Pp xi 1 Permeate
Membrane Wt1, xi1

Cip,Pp<Pf

Figure 6.4.1. Well-stirred separations: (a) continuous well-stirred crystallizer (MSMPR); (b) continuous well-stirred solvent extraction
device; (c) continuous well-stirred ultrafiltration cell; (d) continuous well-stirred gas separation cell.

based on two immiscible phases. Figure 6.4.1(c) shows a crystallization as an example of a solidliquid system
common well-stirred laboratory format for ultrafiltration, where stirred tanks or well-mixed devices are used rou-
which was introduced in Section 6.3.3.2. Figure 6.4.1(d) tinely for larger-scale/industrial operations. We will focus
provides the schematic of a continuous well-stirred mem- primarily on a continuous stirred tank crystallizer with
brane gas separator. This section provides an introduction one feed stream coming in and one product stream
to these four separation techniques in a CSTS mode. A few going out from the crystallizer, which is well-mixed. We
other techniques may also be operated in this mode. will consider first various aspects of such a crystallizer
The presence of a stirrer in the separator is not a with respect to the population density function. Later we
prerequisite for realizing a CSTS. Severe backmixing, etc., will touch upon the progress of crystallization in relation
can easily create the de facto condition of a well-mixed to the cooling (for example) needed for crystallization.
separator. We will learn in Chapters 8 (and 7) that such Next, we will briefly mention a well-stirred batch/semi-
conditions are often detrimental to achieving high separ- batch crystallizer. We also briefly touch upon precipita-
ation. In CSTSs, there is no specific directional relationship tion in Section 6.4.1.1.4.
between the bulk flow and the force, unlike those dis- Consider a continuous crystallizer of volume V, as
cussed in the rest of Chapter 6 and especially in Chapters shown in Figure 6.4.2(a). A feed stream having a particle

7 and 8. Backmixing can destroy high composition changes (crystal) number density function nf r p (which is also the
achieved along the length of the separator. For phase population density function), volumetric flow rate Qf and
equilibrium based separation processes, a well-stirred species i mass concentration if enters the crystallizer
device can merely ensure at its best that the two exiting continuously. Product stream 1, having a particle (crystal)

phases are at equilibrium. number density function n1 r p , volumetric flow rate Q1


A batch mode of operation in the separation tech- and species i mass concentration i1 , leaves the crystallizer
niques discussed in this section also exists under well- continuously. The particle (crystal) number density func-

stirred conditions, and these are briefly touched upon. tion n r p in the well-mixed crystallizer is the same
throughout the crystallizer. The macroscopic population
balance equation for a stirred tank separator may be writ-
6.4.1.1 Solidliquid systems: solution crystallization,
ten using equations (6.2.60) and (6.2.61) as follows:
precipitation

n r p 1
In Section 3.3.7.5, we were introduced to liquidsolid G nr p B  De fQf nf r p  Q1 n1 r p g:
equilibria vis--vis crystallization, leaching and precipi- t r p V
tation. Here we are going to employ solution 6:4:1a
6.4 Continuous stirred tank separators 447

(a)

Qf, rif,nf (rp)

n1(rp) Q1, ri1, n1 (rp)

Slurry/solution
volume, V

(b)

ln n

b>0 1
slope =
ln n(rp) Gtres

b=0
b<0
Figure 6.4.2. MSMPR crystallizer. (a) Schematic of a well-mixed
crystallizer; (b) semilogarithmic plot of population density func-
rp tion n(rp) vs. crystal size rp.

If we assume that the processes of crystal breakage, to no crystals in the feed stream, or an unseeded30 system.
attritional change and agglomeration may be neglected, The corresponding solution for the crystal number density

then B 0 De. The governing equation for such a function, n r p , is particularly illustrative:
continuous stirred tank separator is
V d

G nr p nr p : 6:4:3a
n rp

1

Q1 dr p
G n r p fQf nf r p  Q1 n1 r p g,
t r p V
The quantity V =Q1 is essentially the mean residence
6:4:1b
time of the liquid/particles/crystals in the systems; it is
except, by definition, the crystal number density function also called the drawdown time and will be indicated here

n1 r p in product stream 1 is equal to that present by t res . A solution of this equation when the crystal

throughout the well-mixed crystallizer, i.e. n r p . This growth rate G (=dr p =dt) is independent of rp is quite

implies that there is no crystal size based separation (often useful, i.e. dG=dr p 0 (the so-called size-independent
called classification) taking place at the crystallizer exit. growth). This condition, encountered earlier as McCabes
We now consider a special case of steady state such L law of crystal growth (equation (3.4.30) leads to the
that the first term in the above equation is zero. Such a following form of the crystal population balance
crystallizer is often called a mixed suspension, mixed prod- equation:
uct removal (MSMPR) crystallizer (Randolph and Larson,

dn r p n rp
1988):  : 6:4:3b
dr p G t res
d 1
G nr p fQf nf r p  Q1 nr p g: 6:4:2
dr p V

A special case of this mode of crystallizer operation allows 30


In some crystallization processes, seed crystals of the same

a simplification of this equation: nf r p 0, corresponding material are introduced with the feed stream.
448 Open separators: bulk flow parallel to force and CSTSs

Define n0 as the crystal number density function value for (2.4.2c)), the total crystal mass per unit system volume, the
crystals of near-zero size, the embryos (see equation suspension density MT (equation (2.4.2f)), etc. The calcu-
(3.3.99)). Integration of this equation for particular values lation of such quantities appropriately normalized is facili-
of G and t res yields tated by recognizing that equation (6.4.4) may be
represented as (Randolph and Larson, 1988)
n r p
rp 0
1

dn r p dr p n rp rp y expx , 6:4:8

 ) ln@ 0 A  ;
n rp G t res n G t res
n 0 0 1 where

0 @ rp A y nr p =n0 and x r p =G t res : 6:4:9
n r p n exp  ;
G t res
Such calculations are illustrated in the examples below
6:4:4 (following Randolph and Larson (1988)).

ln n r p ln n0  r p =G t res : 6:4:5 Example 6.4.1 For the MSMPR crystal number density

function

(6.4.4), determine the expressions for
A semilogarithmic plot of n r p against the crystal size r p N t , F r p , r p1, 0 , AT and M T . Develop expressions for the
will yield an intercept of n0 and a slope of 1=G t res (if the cumulative crystal size distribution fraction and the cumula-
plot is for og10n(rp) vs. rp, the slope is (1/(2.303 Gtres))); tive surface area distribution fraction.
since t res V =Q1 is known, the slope will yield G, the
Solution We will employ the simplified representation of
linear crystal growth rate, (dr p =dt). A semilogarithmic plot

(6.4.8) whenever we can. To start with, as per equations
of n r p vs. r p in Figure 6.4.2(b) illustrates this behavior of (2.4.2c) and (6.4.4), the total number of crystals per unit
an MSMPR crystallizer for the case of the crystal-free feed volume, Nt, is

stream nf r p 0 and the L law of crystal growth.


Measurement of the crystal number density function then  

rp
yields two parameters, n0 and Gt res . Generally, the selected N t n r p dr p n0 exp  dr p
G t res
value of t res used to operate the MSMPR crystallizer will 0 0

influence the value of the growth rate, G, achieved in the 0 number of crystals
n G t res : 6:4:10
crystallizer. cm3
To understand the nature of the other parameter n0, Therefore the crystal size distribution function F(rp) (defined

consider the basic relation between n r p and N r p (rela- by (2.4.2d)) is:


tion (6.2.48a)) in the limit of r p ! 0:
rp x


dN r p Fr p nr p dr p =N t n0 =N t G t res expxdx
lim n r p n0 Lt : 6:4:6
r p !0 r p !0 dr p 0 0
x
We could express it also as the ratio in the limit n0 G t res =n0 G t res expxdx 1  exp x

0
dN r p =dt B B0 1  exp r p =G t res :
n0 lim Lt , 6:4:7
r p !0 dr p =dt r p !0 G Gjrp !0 6:4:11

where dN r p =dt in the limit of r p ! 0 is the rate at which This has a value of 1 when rp ! and a value of 0 when rp 0.
the smallest crystals, i.e. nuclei, are being born (i.e. the The number based mean crystal size, r p1, 0 , is obtained
value of B as r p ! 0, B0; see (6.2.50g), where the units are from definition (2.4.2g) as follows:

number of crystals number of crystals


, r p nr p dr p G t res 2 n0 x expxdx
volume  time m3 -s r p1, 0
Nt Nt
0 0
and dr p =dt in the same limit is the growth rate of the
smallest crystals (i.e. G as r p ! 0). The dynamics of the
n0 G t res 2
nucleation kinetics in a crystallizing system is captured in x expxdx G t res : 6:4:12
n0 G t res
this parameter n0, the nucleation population density par- 0

ameter, which will depend strongly on the supersaturation A number based cumulative crystal size distribution fraction
level (for B0, see (3.4.22b); for G, see (3.4.28)). may be defined by
Equation (6.4.4) provides an expression for the crystal

rp x
number density function n r p in an MSMPR crystallizer. nr p dr p
We now need information about the distribution functions rp Nt n0 G t res 2 x expx dx
of other properties of MSMPR crystallizers, e.g. the total 0
0
; 6:4:13
number of crystals per unit system volume, Nt (equation r p 1, 0 n0 G t res 2
6.4 Continuous stirred tank separators 449

The suspension density MT (defined by (2.4.2f)) is


1.0
0 1

0 r p
0.8 M T s v r 3p nr p dr p s v n r 3p exp@ A dr p
M MT G t res
0 0
0.6 M T 6 n0 s v G t res 4 g=cm3 :
6:4:18
0.4 rp+ f(x)
Example 6.4.2 Define a dominant crystal size vis--vis a
0.2 certain property/characteristic of the crystal as that crystal
mf (x)
size where the property has a maximum. Determine the
dominant crystal size with respect to the crystal mass for an
0 1 2 3 4 5 6 7 MSMPR crystallizer.
(rp /Gtres) = x
Solution The crystal mass density per unit liquid volume
between the crystal dimensions 0 and rp may be obtained
Figure 6.4.3. Mass fraction density function mf(x), mass distribu- from relation (2.4.2f) as
tion function (M/MT) and the density function of r p f x for an rp rp  
MSMPR crystallizer.
rp
M s v r 3p n r p dr p s s n0 r 3p exp  dr p :
G t res
0 0
rp A crystal mass density distribution function may be defined

r p n r p dr p x from relation (6.4.18) as


0
r
p x x expxdx rp
N t r p1, 0 0
s v r 3p nr p dr p
M o
1  1 x expx: 6:4:14a : 6:4:19
MT

s v r 3p nr p dr p
The corresponding density function, r
p f r p , in terms of x is
o

It has a value of 0 at rp 0 and 1 at rp (just like the crystal


r
p f x d r p =dx x exp x : 6:4:14b size distribution function F(rp) in Example 6.4.1). A density
function mf(rp) of the crystal mass distribution function
This is plotted in Figure 6.4.3. When
vis--vis the particle size rp is obtained over a differential
x ! , r
p ! 1; when x ! 0, r p ! 0.
size range of rp to rp drp:
The total crystal surface area per unit liquid volume, AT,
 rp 
may be defined using a crystal shape factor s based on the

surface area ap of a particle or a representative radius rp


dM=M T d s v r 3p n r p dr p MT
o
(definition (3.4.26)):

) dM=M T s v r 3p n r p dr p =M T ,



which implies that the mass fraction density function is
ap s r 2p ; AT ap n r p dr p s r 2p n r p dr p ;

0 0 dM=dr p s v r 3p n r p
mf r p : 6:4:20
6:4:15 MT MT
The dominant crystal size will make the maximum contribu-

AT s n0 r 2p exp r p =G t res dr p 2 n0 s G t res 3 : tion to mf(rp); therefore, at this dominant size rpd,
0

d mf r p =dr p 0:
6:4:16
For an MSMPR crystallizer,
A cumulative crystal surface area distribution fraction may be
defined as 0 1
3 0
0 1 d @s v r p n expr p =Gt res A
rp 0;
r p
dr p 6 n0 s v G t res 4
2
n0 s r p exp@ A dr p
rp
G t res d  3

0 r expr p =Gt res 0 ) 3 r 2p expr p =Gt res
A
p ap n r p dr p =AT dr p p
2 n0 s G t res 3
0 0 1
x expr p =Gt res
x 2 r 3p  0 ) r pd 3 G t res :
2
Ap x 1=2 x expxdx 1  @1 x A expx:
Gt res
2
0
6:4:21a
6:4:17
The crystal mass density distribution function (M/MT) for an
When x ! , A
p ! 1; when x ! 0, Ap ! 0: MSMPR crystallizer from relation (6.4.19) can be written as
450 Open separators: bulk flow parallel to force and CSTSs

 
x2 x3 crystal g 1 cm3
M=M T 1  expx 1 x , 6:4:21b 6  3  108  1:2 3

2 6 mm-liter cm 1000 mm3
where x r p =G t res . The crystal mass fraction, density func- 4
4
  8:3  104  120 mm
tion, mf(x), is plotted as 3
dM=M t =dx expxx 3 =6: 6:4:21c 18  1:2  4  0:984  104 g
283 g=liter:
Figure 6.4.3 illustrates mf(rp) as a function of x; it has a 3  1000 liter
maximum at x 3, i.e. rpd 3Gtres. This figure also illustrates
(M/MT). The density function of r A brief discussion about the experimental methods used to
p x , r p f x from Example
6.4.1 is also displayed in this figure. obtain the crystal number density function, n(rp), is relevant.
One common and classical method is based on sieving. The
Example 6.4.3 The crystals obtained from an MSMPR crys- crystal mass is sieved through a vertical stack of sieves of
tallizer were analyzed for their size distribution. A plot of ln increasingly smaller opening size, the largest being at the
((n(rp) crystals/mm-liter)) against rp (the abscissa) in mm top. The mass of crystals retained on a sieve of given size is
yielded a slope of 10 mm1. The intercept on the ordinate (ws), where ws is the mass of the crystal material of mass
yielded a value of 3  108 crystals/ mm-liter. The residence density s. The particle diameter difference between the two
time in the crystallizer was 120 minutes. Determine the sieves, the one at the top and the one on which (ws) was
growth rate and the nucleation rate. obtained, is identified as dp ( 2rp). We know from rela-
tion (2.4.2i) that
Solution The slope is given by (1/G tres). Since tres 120 

  3

minutes,
r p4, 3 r 4p n r p dr p r p n r p dr p 6:4:21d
1 1 1 1 0 0
G  ) G  
slope t res 10 mm1 120 min is an average particle radius based on the mass of the
1 mm mm particles. If, however, the limits in the integrals above are
8:3  104 : rmin and rmax, valid for the crystal fraction obtained for the
1200 min min
two sieves under consideration, instead of 0 and , then
The nucleation rate B as rp ! 0 is given by r p4, 3 will represent the crystal fraction under consideration.
crystal mm By definition (2.4.2f) for the mass density of a population,
B0 n0 G 3  108  8:3  104 we can write
mm-liter min
crystal

2:49  105 : ws s v r 3p n r p r p : 6:4:21e


liter-min
Therefore
Example 6.4.4 Determine the values of Nt, r p1, 0 , rpd and MT
for the MSMPR crystallizer product stream of Example 6.4.3. ws 2 ws
Assume spherical crystals of density 1.2 g/cm3. nr p , 6:4:21f
s v r p r p s v r 3p dp
3

Solution where r 3p is now r 3p4, 3 . Recall that v is defined via equation


(2.4.2e) based on radius (it is essentially the same as equa-
N t n0 G t res
tion (6.4.21e) defined for a finite drp), and will be different
crystal mm
3  108  8:3  104  120 min if equation (2.4.2e) is defined with respect to diameter dp.
mm-liter min Additional methods to measure crystal size distribution
crystal number based on particle mass employ light scattering/diffraction
38:3  120  104 2:98  107 : methods. Simultaneous measurements of particle size
liter liter
(based on particle volume) and their number are also carried
From expression (6.4.12), out by Coulter counters. There are a variety of other tech-
niques, based on centrifugal sedimentation, electroacoustic
r p1, 0 n0 G t res 2 =n0 G t res spectroscopy, microelectrophoresis, gravitational sedimenta-
tion, scanning electron microscopy, etc. An introduction to
mm these techniques is available in a NIST (National Institute of
G t res 8:3  104  120 min
min Standards and Technology) publication by Jillavenkatesa
et al. (2001).
9:96  102 mm 0:0996 mm:
Although equation (6.4.4) provides an expression for n(rp)
On the other hand, from relation (6.4.21a), in an MSMPR crystallizer under certain conditions, estimates
of additional quantities, such as the supersaturation, the
mm suspension density, the growth rate etc., are also of interest.
r pd 3G t res 3  8:3  104  120 min
min The supersaturation is of particular interest since the growth
rate, as we know from equations (3.4.28) and (3.4.29),
2:98  101 mm 0:298 mm;
depends on the fractional supersaturation. Further, the
M T 6 n0 s v G t res 4 extent of crystallization needs to be determined. These
6.4 Continuous stirred tank separators 451

ri
(ri risat) ri (tresAT)
tres
AT = 2n ys (Gtres)
0

n0 = f (ri risat) = B0/G G A1r (ri risat)

n0 G
MT

n = n0 exp (rp /Gtres)


n

n(rp)

Figure 6.4.4. Interaction between n(rp), G and B0 via n0. For an MSMPR crystallizer. (After Randolph and Larson (1988).)

calculations are facilitated by a mass balance on species i where we have employed definition (6.4.15) for AT. This
being crystallized over the MSMPR crystallizer shown in equation illustrates the inverse dependence of G on AT and
Figure 6.4.2(a): therefore on n(rp), which depends on the growth rate G and

Qf if  Q1 i1 Q1 M T

mass rate of inflow of species i mass rate of outflow of species i total mass rate of crystallization: 6:4:22

The total mass rate of crystallization, Q1MT, may be assumed


to be equal to the growth rate of the crystal surfaces:

Q1  M T G  AT  V  ip
flow rate of particle mass crystal dimension total crystal surface area
solution volume particle density
time time solution volume

6:4:23

Two assumptions were made in arriving at the expression on the nucleation rate B0 as r p ! 0 . Thus there is a feedback
the right-hand side. First, the mass rate of loss of i from the loop, as shown in Figure 6.4.4. To understand the role of the
solution due to the formation of nuclei only is negligible. inputs i M T and tres in this process, consider the
Second, since in our treatment the crystal growth rate G following treatment.
drp/dt, where rp is the characteristic radius (and not the We have observed in an earlier chapter (see equation
diameter), we do not have a factor (1/2) on the right-hand (3.4.28)) that the crystal growth rate G is strongly influenced
side (unlike that in Randolph and Larson (1988), where G by the level of supersaturation. If we express the growth rate
d(2rp)/dt is based on the growth rate of the diameter; there- as a linear function of the supesaturation in mass units, i.e.
fore, the value of the shape factor v in our equations (2.4.2f) i isat (instead of molar supersaturation Ci Cisat or frac-
and (3.4.26) is different from the shape factor kv in Randolph tional supersaturation s, see (3.3.98a, b)),
and Larsons (1988) eqs. (5.3-2/3)). The mass concentration
change i achieved in the crystallizer and defined by G A1 i  isat 6:4:27
 
i M T Qf =Q1 if  i1 g=cm3 6:4:24 then
i  isat extent of supersaturation
can now be utilized to develop an expression for the
i
growth rate G from relation (6.4.23):

: 6:4:28a
Q MT MT i A1 ip t res s n r p r 2p dr p
G 1 ; 6:4:25 0
AT V ip t res ip AT ip t res AT
Thus

G i , 6:4:26 i

i  isat : 6:4:28b
ip t res s r 2p n r p dr p A1 ip t res AT
0
452 Open separators: bulk flow parallel to force and CSTSs

From relation (3.4.22b) we know that, in homogeneous doubled, the supersaturation is reduced to half, and so is
nucleation, the nucleation rate the growth rate.
! On the other hand, for p > 1, relations (6.4.31b) and
0 1 (6.4.31c) indicate, respectively, that, for the same MT, both
B / exp    , 6:4:29a
ln s2 n0 and G increase with tres; therefore, B0 increases faster
(since B0 n0G) than when p 1, where n0 is independent
where s is the fractional supersaturation in units of molar of tres. A higher nucleation rate (B0) implies many more
concentration (easily converted to mass concentration). crystals; therefore a longer residence time may not necessar-
Therefore, since n0 from (6.4.7) is given by ily lead to larger crystals since supersaturation is being con-
n0  B0 =Gjr p !0 , 6:4:29b sumed by many more crystals growing.
We now briefly consider the case where tres is the same but
0
n has a stronger dependence on i  isat than G. the magma densities are different: M Ta 6 M Tb ; t res ja t res jb .
We will now follow an illustrative procedure from Ran- Obviously we are dealing with two different MSMPR crystal-
dolph and Larson (1988) and Wankat (1990, chap. 4) to find lizers. From expression (6.4.18) for MT for the two cases,
out first the role of the residence time, tres, in an MSMPR
crystallizer. For the sake of illustration, we will replace the M Ta 6 n0a s v Ga t res ja 4 ;
complex dependence of B0 on supersaturation indicated via
M Tb 6 n0b s v Gb t res jb 4 ;
(6.4.29a) by (for cases of primarily nucleation only)
M Ta =M Tb n0a G4a =n0b G4b : 6:4:32a
B0 A i  isat p , 6:4:30a
0
Substituting (6.4.30b) for n to obtain
where A is a rate constant and the value of p may vary
between 2 and 9 for aqueous systems. (The power p may M Ta =M Tb Ga 3p =Gb 3p ) Ga =Gb M Ta =M Tb 1=3p ;
be considered as the kinetic order in the expression for the 0 0

na =nb Ga =Gb p1 M Ta =M Tb p1=3p :


dependence of the nucleation rate on the supersaturation
6:4:32b
level.) This approach, illustrated in Randolph and Larson
(1988), is based on the Miers nucleation model (Miers and If p 1, the nucleation population density function n0 is
Isaac, 1906). Combining it with the linear dependence of independent of the magma densities, but growth rate G
the crystal growth rate G on supersaturation (6.4.27), we increases as magma density increases. If p > 1, then n0
get, from definition (6.4.7) for n0 , increases as MT increases; so does G. Similarly, from
(6.4.21), the dominant crystal size rpd also increases as MT

n0 A i  isat p =A1 i  isat A=A1 i  isat p1 increases for constant tres since G increases.


p1
A=A1 G=A1 : Example 6.4.5 The kinetic order p in the dependence of the
6:4:30b nucleation rate B0 on the supersaturation is an unknown
empirical parameter. Suppose you have an option of gener-
Let us now focus on an MSMPR crystallizer having a size- ating data from an MSMPR for a given system. You can vary
independent growth (McCabes L law). To determine the tres for a given MT. Indicate a procedure to determine p.
effect of tres, we assume two situations where the suspension
density MT is identical but tres, G and n0 are different. From Solution We know from our earlier analysis that, for a given
expression (6.4.18) for MT and for two situations (identified MT, if tres is varied, the growth rate G and the nucleation
by subscripts a and b) and n0 from above, population density function n0 will vary according to rela-
tions (6.4.31b) and (6.4.31c), respectively, for a given p.
M T 6 n0a s v Ga t res ja 4 6 n0b s v Gb t res jb 4 ; Therefore a procedure that could be followed is: carry out


p1

p1
A=A1 Ga =A1 Ga 4 t res j4a A=A1 Gb =A1 two different experiments for two values of tres at the same
MT. Obtain, from a plot of n(rp) vs. rp, the values of Ga, Gb, n0a
Gb 4 t res jb 4 :
and n0b . Assume different values of p and check whether the
6:4:31a same p can describe both of the following relations:
From this we get two results:
Ga =Gb t res jb =t res ja 4=3p , 6:4:33a
4=3p
Ga =Gb t res jb =t res ja ; 6:4:31b
n0a =n0b t res jb =t res ja 4p1=3p , 6:4:33b
0 0

na =nb t res jb =t res ja 4p1=3p : 6:4:31c for the two residence time values, t res ja and t res jb . The value of
p which satisfies both (6.4.33a) and (6.4.33b) is the desired
The exponent p in relation (6.4.30a) for B0 is clearly quite one.
important. If p 1, by (6.4.31b), the growth rate G depends
inversely on the residence time, tres; correspondingly, via The decrease in solute mass concentration i ( MT) along
relation (6.4.27), the supersaturation also depends inversely an MSMPR crystallizer depends on a number of quantities,
on the residence time. Further, n0 B0 =G is independent as one can see from relations (6.4.18) and (6.4.25). If the
of tres, i.e. n0a n0b . More importantly, as tres increases, extent of supersaturation in product stream 1 is not
the growth rate decreases. If t res jb 2 t res ja , Ga 2Gb . exhausted, the system is identified as a class I system with a
Also, i  isat a 2i  isat b : as the residence time is nonhigh yield. For class II systems, the supersaturation is
6.4 Continuous stirred tank separators 453

almost exhausted in the product stream, and the system is expression (6.4.18) for MT when tres is not changing but MT is.
considered to be a high-yield system. For two situations a and b, we already observed in results
The basic result (6.4.4) obtained for an MSMPR crystallizer (6.4.32a) that
was based on several assumptions.
M Ta =M Tb n0a G4a =n0b G4b ; Ga =Gb M Ta =M Tb 1=3p ;
0 0

(1) The crystal growth rate G ( drp/dt) is size- na =nb M Ta =M Tb p1=3p :


independent.
Correspondingly, the dominant crystal sizes r pd ja and r pd jb
(2) The product crystals are being withdrawn without any
for the two cases will be related via (6.4.21a) as follows:
classification, so that n1(rp) n(rp).
(3) The suspended solids have no effect on the crystal size r pd ja =r pd jb Ga =Gb M Ta =M Tb 1=3p : 6:4:38
distribution.
Therefore a larger suspension density will lead to larger
(4) There is no specific relation between the crystal growth crystal sizes. (Remember, it is based on an assumption
rate and the level of supersaturation. that G is linearly proportional to i  isat vis--vis assum-
(5) There is no secondary nucleation. ption (4).)
(6) There is no addition of seeds (i.e. nf 0). Third, the effect of secondary nucleation on the population
(7) There is no crystal breakage, attritional change or density is explored by considering its effect on the nucleation
agglomeration in the MSMPR crystallizer. rate B0 (relaxing assumption (5)):
(8) There is steady state operation. q
B0 A i  isat p M T : 6:4:39a
We now look very briefly at the consequences of relaxing q
Here we have assumed that MT
quantifies the effect of
some of these assumptions.
secondary nucleation on B0, whose rate expression used
Consider first the case where crystal growth rate depends
earlier was (6.4.30a). We can employ the linear dependence
on the crystal size (relaxing assumption (1)). Instead of the
of growth rate G on supersaturation given by (6.4.27) to
simplified equation (6.4.3b), we will have to work with equa-
rewrite the above relations as (Randolph and Larson, 1988)
tion (6.4.3a):
 
p q q
d Q nr p B0 A=A1 Gp M T A1p Gp M T : 6:4:39b
G nr p nr p 1  : 6:4:34
dr p V t res
Correspondingly, since the nuclei population density
If empirical relations are available for G(rp), it may be substi- n0 B0 =G, we get
tuted in the above relation. For example, Abegg et al. (1968)
q
have shown that n0 A1p Gp1 M T : 6:4:39c

b
Gr p G0 1 r p , b < 1, 6:4:35 For two situations a and b, we already know that

appears to fit most data; here G0, and b are experimen- M Ta =M Tb n0a G4a =nob G4b
tally determined constants. Using such a relation in equa- Therefore, using (6.4.39c), we get
tion (6.4.34) leads to  p1  
Ga M Ta q M Ta G4b
d
b nr p n0a =n0b :
G0 f 1 r p nr p g  : 6:4:36 Gb M Tb M Tb G4a
dr p t res
On rearrangement, we get
Integration of this equation has led to the following result for    
size-dependent crystal growth: 0 0
M Tb M Tb 4q1=p3 M Tb 4qp1=p3
nb =na :

b M Ta M Ta M Ta
nr p exp1=G0 t res 1  b n0 1 r p
( ) 6:4:39d
1 r p 1b 
 exp : 6:4:37 Also
G0 t res 1  b
Gb =Ga M Tb =M Ta 1q=p3 : 6:4:39e
When b 0 and 1, this relation is simplified to the
standard equation (6.4.4) for G(rp) constant. The dashed Often, q 1; we then get
lines in Figure 6.4.2(b) illustrate the nature of these crystal 0 0

Gb =Ga 1; nb =na M Tb =M Ta : 6:4:39f


number density functions. Figures 6.4.5(a) and (b) illustrate
schematically size-independent and size-dependent crystal In that case (q 1), the growth rate is unaffected by second-
growth rates. ary nucleation; the nuclei population density n0 changes
Second, we take into account the effect of suspended solids linearly with MT.
(relaxing assumption (3)). Large values of the suspension dens- Fourth, we consider how seeding affects the crystallizer
ity MT means the availability of a large surface area onto which performance (relaxing assumption (6)). Seeding can take
solutes may be deposited from the solution. Potentially this is place along with nucleation. Let us assume, for the sake of
equivalent to having a higher level of supersaturation, simplicity, that nucleation rates are substantially low and
i  isat ; alternatively, this means a lower supersaturation may be neglected. Therefore the number of crystals being
is needed to achieve a given MT. To study its effect, consider introduced into the MSMPR crystallizer per unit time will be
454 Open separators: bulk flow parallel to force and CSTSs

(a) (b)

G G

rp rp

(c) (d)

i=1 gr1
i=2 drp
rp gri = gr2
i=3 dt
gr3

t rp

(e) (f)

rp
G

t rp

Figure 6.4.5. Crystal growth rates in an MSMPR crystallizer. (a) Constant size-independent growth rate, G. (b) Size-dependent growth
rates. (c) Intrinsic growth dispersion: three crystals i 1, i 2 and i 3 have three fixed growth rates gr1, gr2 and gr3, as they grow at
different rates, as shown in part (d). (e) Random growth of one crystal with time. (f) Random growth rate of a growing crystal of part (e).

equal to the number of crystals leaving the crystallizer per The number of crystals, dNs, in the seed containing feed stream
unit time, except that the sizes of the product crystals will be in the size range rps to rps drp is obtained from (6.4.40b) as
larger than those in the feed. Let nf (rps) be the population dM se
density function of the seed crystals introduced into the feed; dN s nf r ps dr p : 6:4:40d
s v r 3ps
rps is the size of the seed crystals. Then the seed mass density
per unit liquid volume for crystals having dimensions This number remains unchanged for those specific crystals,
between rps rpsmin and rps rps, Mse, is obtained for crystal but their mass and size have increased in the product stream:
volume shape factor v from relations (2.4.2f) as
rps

dM T dM se

3 dN s ; 6:4:40e
M se s v r 3ps nf r ps dr p : 6:4:40a s v r ps r p s v r 3ps
r ps min

3
Correspondingly, the mass density for crystals between rps dM T s v r ps r p dN s ; 6:4:40f
and rps drp in the seed present in the feed is 3 3

dM se s v r ps dN s s v r ps nf r ps dr p ; 6:4:40g

dM se s v r 3ps nf r ps dr p : 6:4:40b MT N s

3
MT dM T s v r ps r p dN s
Let each seed crystal grow by a certain amount. If we assume 0 0
that the growth rate G is size-independent (assumption (1)),

then the net growth in size in the MSMPR crystallizer for all s v r ps r p 3 nf r ps dr p : 6:4:40h
0
crystals is rp Gtres cm for the given residence time. The
mass density per unit liquid volume for crystals in the prod- Therefore,
uct stream exiting the crystallizer (population density func- r 3p n1 r p r ps r p 3 nf r ps r p r p 3 nf r p ;
tion n1(rp)) is
n1 r p

M T s v r 3p n1 r p dr p : 6:4:40c r p r p 3 =r 3p : 6:4:40i
nf r p
0
6.4 Continuous stirred tank separators 455

The corresponding relation between MT and Mse is given Here Nit is the total number of crystals per unit volume of
by all sizes having the growth rate gri, and

3 0 1
r p r p
r 3p n1 r p dr p r 3p nf r p dr p
n0i r p A
rp3 N it  N i r p  exp@
0 0 1=gr i t res gr i t res
M T =M se ; rp
0 1
3 3
r p nf r p dr p r p nf r p dr p rp A
n0i gr i t res exp@
0 0 gr i t res


3

r p r p nf r p dr p B0i t res exp r p =gr i t res : 6:4:41c


0
: The population density n(rp) of crystals of size rp having all

r 3p nf r p dr p of the different growth rates is
0    

grX
i
rp r p
6:4:40j n rp n0i exp  n0i exp dgr :
gr i 0
gr i t res gr t res
These two relations allow the prediction of the population 0
density function n1(rp) and the suspension density MT of the 6:4:41d
product stream from the MSMPR crystallizer given the cor-
responding quantities of the seed crystals in the feed stream. We should recall now that Nt, the total number of crystals
per unit volume having all sizes and growth rates, has the
6.4.1.1.1 Growth rate dispersion in an MSMPR crystal- value n0G tres ( B0tres) for an MSMPR crystallizer (see
lizer The mechanisms of growth of crystals described very result (6.4.10)). This number includes contributions from
briefly in Section 3.4.1.3 are complex. Crystals may have a different crystal growth rates, gri, throughout the crystal
growth rate which is size-dependent. However, crystals of population, i.e. n0i gr i t res B0i t res , where one can write
the same size may also grow at different rates in an MSMPR
B0i t res B0 t res f g grdgr: 6:4:41e
crystallizer depending on or influenced by differing envir-
onments in the crystallizer or inherent structural difference Therefore, the number density of crystals having a size
between crystals. This is called growth rate dispersion. We greater than rp, (Nt N(rp)), may be obtained by summing
briefly identify first the results of one type of growth rate over all numbers related to each growth rate gri, namely
disperson, namely the intrinsic growth dispersion. Here, the (Nit Ni (rp)):
growth rate may vary from crystal to crystal, but an individ-

X

ual crystal has an inherent constant growth rate for the N t  N rp N it  N i r p


whole time, tres, in the crystallizer. Following Randolph io

and Larson (1988), we identify gri as the intrinsic growth

B0 t res exp r p =gr t res f g grdgr:


rate of the ith crystal (see Figures 6.4.5(c) and (d)). The 0
density function of these growth rates is illustrated by the 6:4:41f
probability density function fg(gri), such that the total
number of crystals per unit volume, Nt, is related to fg(gri) by The fraction of crystals having a size greater than rp is
given by
X
1 N t =N t f g gr i f g gr dgr, 6:4:41a

i0 0 N t  N rp N t  N rp
1  Fr p
Nt B0 t res
where gr is the growth rate variable of the continuous  
rp
growth rate density function fg(gr). exp  f grdgr , 6:4:41g
0 gr t res g
Let the population density of crystals of size rp in the
MSMPR crystallizer having a growth rate of gri be ni(rp); then where F(rp) is the fraction of crystals having a size between


0 and rp (definition (2.4.1c)). From the basic relation
ni r p n0i exp r p =gr i t res 6:4:41b
between F(rp) and f (rp) (definition (2.4.1c), namely
from equation (6.4.4). Correspondingly, the number dens- f (rp) (dF(rp)/drp)), we get
ity of crystals having a size greater than rp but possessing a 0 0 11


growth rate gri is dF r p d @ @ r p AA
  0 exp  f g grdgr;

r p dr p 0 dr p gr t res
N it  N i r p n0i exp dr p
0 gr i t res
1

f rp exp r p =grt res f g grdgr:


rp     0 gr t res
r p r p
 n0i exp dr p n0i exp dr p :
0 gr i t res rp gr i t res 6:4:42
456 Open separators: bulk flow parallel to force and CSTSs

Therefore, the crystal size density function f (rp) of an Therefore, from relation (6.4.43b) we get
MSMPR crystallizer can be related to the growth rate dens- j1
ity function fg(gr). j1 j Mogr
Mof r =Mof r r pji, j j 1t res j
: 6:4:44e
It is useful to develop some relations between the Mogr
moments of these two density functions. If we multiply
j Knowing r pj1, j from particle size distribution measure-
both sides by r p dr p and integrate between the limits of
j ments, one can now make guesses about fg(gr) and find
rp 0 and , we get Mof r , the jth moment of the crystal
out which one fits the above relation. Note further that the
size density function f (rp) (see (2.4.1g)):
suspension density MT, (2.4.2f), is
0 1
 

j f g gr dgr
r jp f r p dr p Mof r @ A M T s v r 3p nr p dr p s v N t r 3p f r p dr p
0 0 gr t res 0 0
s v B0 t res Mo3f r 6s v B0 t 4res Mo3gr :
0 1

rp A 6:4:44f
 r jp exp@ dr p ; 6:4:43a
0 gr t res
Consult Berglund (1993) and Berglund and de Jong (1990)
0 1 for further details of such an analysis. They observed that a

j f g gr dgr single growth rate (gri) density function fg(gr) is insufficient
Mof r @ A gr ji t j1 j
res p exppdp
0 gr t res 0
to describe the observed population density of crystal size.
0 1 However, two different growth rate distributions, when

f g gr dgr combined, could describe the performance of a pilot-scale
@ A gr ji t j1
res pj exppdp
0 gr t res 0 sugar crystallizer.
There is an additional phenomenon, namely random
j
t res j 1 gr j f g gr dgr fluctuations in growth rates of the different crystals around
0
j a mean value. For example, one crystal will display several
j! t res gr j f g gr dgr
0 growth rates during its growth in the crystallizer for the

j
j! t res
j
Mogr , time period tres; however, there may be a mean, as shown
in Figures 6.4.5(e) and (f). The governing macroscopic
6:4:43b
population balance equation for an MSMPR crystallizer,
j
where is the jth moment of the growth rate density
Mogr where nf(rp) 0, B De 0, may be written as
function fg(gr) (which may be a normal distribution
(4.1.33g) or a gamma distribution (4.1.33f), etc.). nr p Q n1 r p 2 nr p
G nr p  1 Dg r :
Before we use such a relation to determine the t rp V r 2p
nature of the growth rate density function fg(gr), we 6:4:45
should first recognize that the relation between f(rp)
Here we have employed equation (6.4.1a) and added the
and n(rp) is
growth dispersion term on the right-hand side using an

dNr p n r p dr p empirically determined growth diffusivity, Dgr. This was


f r p dr p : 6:4:44a suggested in the paragraph after equation (6.2.51c) (where
Nt Nt
instead of Dgr, Di was employed). See Randolph and Lar-
Therefore son (1988) for further analysis on this topic. Figures 6.4.5
(e) and (f) illustrate the nature of the random variation of a
r np nr p dr
Monfn
r np f r p dr p Monfr , crystal dimension with time, as well as the random vari-
0 Nt 0 Nt ation of the growth rate of this crystal with rp as it grows.
6:4:44b
6.4.1.1.2 Batch crystallization Crystallization from
where
smaller batches of solution is generally carried out in a

well-mixed crystallizer into which the batch is charged
Monfn r np nr p dr p : 6:4:44c
0 and either cooling or solvent evaporation or (less often)
antisolvent based precipitation is carried out. This method
Employ now the definition, (2.4.2h), of r pi1, i :
is employed for pharmaceuticals on the one hand and

inorganic salts on the other. The solution is kept well-
r i1
p nr p dr p r i1
p f r p dr p Moi1 Moi1
0
r pi1, i 0

fr fn mixed. However, in the cooling crystallizer, for example,
i :
i i Mof r Moif n the cooling heat exchanger is often externally located and
r p nr p dr p r p f r p dr p
0 0 the solution is rapidly circulated between the crystallizer
6:4:44d and the heat exchanger. Regardless of the method
6.4 Continuous stirred tank separators 457

adopted, the process is inherently unsteady. The equation of the crystal size distribution, specifically n
~ . Consider
from Section 6.2.3.2 to be used here now will have an equation (6.4.46c) along with (6.4.46d) for the zeroth
unsteady state term. Therefore n(rp) will depend both on moment for well-behaved functions:
crystal size rp and time. Further, in systems where the

batch volume is changing (for example, due to evaporation n
~ G n~

dr p dr p 0 n
~ dr p dG n
~ 0;
or semibatch operation), the solution volume may not be t r p t 0 0
0 0
assumed to be constant. Sometimes, crystal seeds are dN~t
introduced into the batch at time t 0; this has a strong n jr p !0 B0 V ,
G~ 6:4:46f
dt
influence on the final crystal size distribution obtained.
For an unseeded batch crystallizer, equation (6.2.65)
where
may be reduced, for the case where B De 0, to

V n rp , t
~t
N n
~ dr p : 6:4:46g
GV nr p , t 0: 6:4:46a 0
t rp
In the same fashion, one has to take moments up to order
Here, V may be a function of time. Therefore it is conveni-

three and then get an expression for wcrys in terms of other
ent to define a new variable, n ~ r p , t , instead of two vari-

moment based quantities. The numerical solution of these
ables, V(t) and n r p , t (Randolph and Larson, 1988):
equations with appropriate boundary conditions would

n
~ V t n r p , t : 6:4:46b lead to the desired solution. However, it is quite
complicated.
In terms of this new dependent variable, equation (6.4.46a) To avoid the complexities inherent in the above
is reduced to approach, an alternative approach is often adopted. Many
batch crystallization applications involve cooling crystal-
n
~ r p , t

Gn
~ r p , t 0: 6:4:46c lization wherein a hot solution, which is saturated, is
t r p
cooled at a certain rate. This creates the supersaturation
For the two independent variables, rp and t, boundary/ necessary for nucleation and growth of crystal (see Figure
initial conditions are needed to solve the above equation. 3.3.6B, line AB0 ). (Other strategies, namely solvent evapor-
For the boundary condition with respect to rp, say rp ! 0, ation, antisolvent addition, etc., may be similarly ana-
let B0 be the number of nuclei produced per unit volume; it lyzed.) In this approach, certain assumptions are made to
will, however, depend on time. From definition (6.4.7), allow the determination of the rate at which the solution
temperature is to be cooled. The approach based on earlier
B0 B0 V studies by Mullins group has been summarized by Belter
n0 ~ 0 n0 V
) n : 6:4:46d
Gjrp !0 Gjrp !0 et al. (1988), Wankat (1990) and Nvlt (1992). It is known as
determining the cooling curve of a batch crystallizer.
One also needs an estimate of crystal size distribution at
The cooling rate employed is influential in a batch
t 0, and to know how the operating conditions in the
crystallizer. Usually the heat transfer rate Q ( q Ahex,
batch crystallizer are changing due to evaporation from the
q is the heat flux and Ahex is the heat transfer surface area
solution or cooling of the solution. An appropriate mass
in the cooling crystallizer) is high at the beginning, since
balance on solute species i leads to
the heat transfer rate from the solution into the cooling
di V dwcrys fluid,
0, 6:4:46e
dt dt

Q qAhex U ov Ahex T solu  T cf , 6:4:47a


where i is the mass concentration of species i and wcrys
is the mass of crystals in suspension, since the sum total is directly proportional to the temperature difference
of the mass of solute i in two forms, iV (in solution) and between the solution due to undergo crystallization (Tsolu)
wcrys (in suspension), must be constant and invariant and the temperature of the cooling fluid (Tcf). As a result,
with time. In evaporative crystallization, as the solvent the solution becomes supersaturated quickly, resulting in
evaporates, V decreases and i increases, leading to crys- the production of a large number of crystal nuclei; conse-
tallization, i.e. wcrys increases. In cooling crystallization quently, further growth of these crystals becomes limited.
as temperature decreases, isat, the solubility, decreases Let
(see Figure 3.3.6B) and wcrys increases. The procedure for
pursuing such calculations is illustrated later in this ws0 mass of solution charged into the crystallizer;
section. cp0 specific heat of this solution;
Considerations of equations (6.4.46c) and (6.4.46e) wcrys mass of crystals formed at any time;
show that additional equations are needed to express wcrys Hcrys heat of crystallization;
in terms of n~ , etc. These are developed by taking moments Qsub heat transfer rate due to subcooling;
458 Open separators: bulk flow parallel to force and CSTSs

Qcrys heat transfer rate due to crystallization; Therefore


u0 mass fraction of solute in the solution charged;
uc mass fraction of solute in the crystallized solution dwcrys ws0 durc
 , 6:4:47k
on a solid-free basis; dT solu 1 ur0 dT solu
ur0 kilograms of solute per kilogram of solvent in the since uro is a constant.
solution charged, u0/(1u0); Substituting these results into relation (6.4.47h), we get
urc kilograms of solute per kilogram of solvent in the

crystallized solution, uc =1  uc : dT solu U ov Ahex T solu  T cf


(6.4.47b)  ,
dt ws0 cp0 H crys ws0 b 2cT solu =1 ur0

Following Nvlts (1992) approach, assume that the 6:4:48a


temperature dependence of the solubility of the solute is where we have employed the temperature dependence
given by (6.4.47c) of the solute solubility. The cooling rate
uc (dTsolu/dt) data may be employed to determine a number
urc jeq j a bT cT 2 , 6:4:47c of quantities. When there is no crystallization, the cooling
1  uc eq
rate expression,
where ucjeq is the mass fraction uc of solute in solution at  

dT solu U ov Ahex T solu  T cf


equilibrium with crystals at the solution temperature Tsolu.  , 6:4:48b
Now, any small change in temperature of the solution, dT, dt ws0 c p0
over time dt is due to a certain amount of heat transferred varies linearly with Tsolu; the slope yields Uov Ahex since ws0
due to solution subcooling, Qsub dt, as well as due to the is known and so is cp0. When crystallization begins, the
heat of crystallization, Qcrys dt, where slope of this plot will change. If the two slopes are identi-
fied as T_ 1 and T_ 2 (after crystallization), we obtain, after
Q Qsub Qcrys : 6:4:47d
some algebra, at the temperature, Tcrys, where crystalliza-
tion begins
But

f T_ 1 =T_ 2  1gcp0
Qsub dt ws0 c p0 dT solu 6:4:47e H crys
: 6:4:48c
b 2c T cryst =1 ur0
and
The total amount of heat transferred during the process of
Qcrys dt H crys dwcrys : 6:4:47f cooling crystallization from an initial solution temperature
of Tinitial to the final solution temperature of Tfinal spanning
Further, the overall heat transfer rate expression (6.4.47d) a time period of tfinal (in seconds) is given by
may be written as t fina l   
0 1 u0  ufinal
  Q dt ws0 c p0 T initial  T final H crys ,
Qsub dt Qcrys dt 1  ufinal
A dT solu ;
0
Q@ 6:4:47g 6:4:49
dT solu dt

where we have used the crystal mass balance expression
dT solu U ov Ahex T solu  T cf (6.4.47j). The crystal yield on the same basis is
0   1: 6:4:47h
dt Qsub dt Qcrys dt
@ A ur0  urfinal u0  ufinal
dT solu wcrys ws0 ws0 : 6:4:50
1 ur0 1  ufinal

From relations (6.4.47e,f), The process described above is somewhat uncontrolled,


0 1 leading to a high rate of nuclei generation at the beginning.
Qsub dt Qcrys dt   One can, however, introduce seeds into the batch to
@ A  ws0 c p0  H crys dwcrys :
dT solu dT solu minimize/eliminate nucleation. The rate of cooling in
cooling crystallization under such a condition is of interest.
6:4:47i The objective would be to carry out crystallization via
seeding in the metastable region (see Figure 3.3.6B) and
Now, from definitions (6.4.47b),
to prevent the uncontrolled rate of nucleation encountered
ur0 urc

in the supersaturated region. To do this, we focus on a


wcrys ws0  ws0 ws0 u0  ws0  wcrys uc
1 ur0 1 ur0 supersaturation balance (Belter et al., 1988; Wankat, 1990;
ur0  urc Nvlt, 1992), where the supersaturation (i isat) in mass
ws0 : 6:4:47j
1 ur0 units will be used. If we focus on a small time period t
6.4 Continuous stirred tank separators 459

when the temperature changed by T, then, per unit solu-


2
dT 3M se G r ps G t
tion volume, : 6:4:54d
dt disat =dT r 3ps
0 1 0 1 0 1
  change in change change
change in B saturation C Bdue to C Bdue to C
B C
@ concentration A B C
@ crystal A B
@ nuclei
C
A
super saturation
due to T growth generation

i  isat isat k gr AT i  isat p1 t k nu i  isat p t: 6:4:51

Here we have employed relation (6.4.30a) for the depend- Integration of this relation from the crystallization tem-
ence of the nucleation rate on the supersaturation level, with perature Tcrys to any temperature T provides (under the
knu being the rate constant; similarly, kgr is a rate constant for assumption r 3ps r 3ps )
growth, where AT is the crystal surface area per unit liquid   !
volume. In the limiting process of t ! 0, we get M se 3Gt G t G2 t 2
T T crys  1 2 :
disat =dT r ps r ps 3r ps
di  isat disat
k gr AT i  isat p1 k nu i  isat p : 6:4:55
dt dt
6:4:52
To avoid nucleation, equations (6.4.54c,d) suggest that the
If one can operate in the metastable region (Figure 3.3.6B) cooling rate should 
be slow at thebeginning; then, as the

2
where the supersaturation level (i isat) remains essen- crystal surface area r ps G t increases, the cooling
tially constant with time and the contribution of nucleation rate should increase proportionally.
is negligible, we obtain
Example 6.4.6 (Adapted from Nvlt (1992).) In a cooling
disat
p
crystallizer charged with a 1000 kg batch of a solution con-
k gr AT i  isat 1 : 6:4:53a
dt taining a salt at ur0 0.415 kg/kg of water and 60  C, the
However, we can write temperature of the solution being cooled was measured at
  various times; the measured temperatures and the cooling
disat disat dT rates are indicated in Table 6.4.1. The value of Cp0 3.25 kJ/
: 6:4:53b
dt dT dt kg-K; the cooling water in the heat exchanger is at 13.5  C.
Determine the crystallization temperature from the rate of
Further, the growth term from the overall crystal growth
cooling data. Determine the value of Uov Ahex from the data
coefficient Ko in expression (3.4.24) may be used to equate in the unsaturated region after determining the crystallizing
the growth term contribution in (6.4.52):

p
k gr AT i  isat 1 K o AT i  isat , 6:4:54a
where Table 6.4.1.

AT s r 2p n r p dr p s N t r 2p ; t (min) Tsolu ( C) dTsolu/dt (K/hr)


0

2 M se 6:4:54b
r 2p r ps Gt , Nt Ns , 0 60 
s v r 3ps
10 56.7 19.8
20 53.6 18.6
where Ns is the total number of seed crystals per unit 30 50.7 17.4
volume, Mse is the seed crystal suspension density of size 40 48.0 16.2
r ps (equation (6.4.40a)) and G is the growth rate. Therefore 50 45.6 14.4
! 60 43.3 13.8
dT K o AT i  isat K o s M se
    70 41.5 10.8
dt disat disat s v r 3ps 80 39.8 10.2
dT dT

2 90 38.2 9.6
 r ps Gt i  isat : 6:4:54c 100 36.7 9.0
110 35.3 8.4
From relation (3.4.27), 120 33.9 8.4
dr p K o s 130 32.7 7.2
G M i C iwb  M i C s 150 30.5 6.0
dt 3s v
190 26.8 5.1
dr p K o s 250 22.8 3.75
 isat ;
dt 3s v i 320 19.9 2.10
460 Open separators: bulk flow parallel to force and CSTSs

(a) (b) (c)


25

20 70
dTsolu 60
15 T 50
Cooling dt
fluid (C) 40
K 10 30
Cooling
hr 20
coil
5 10
0
0 0 2 4 6
0 20 40 60 t (hour)
Tsolu (C)

Figure 6.4.6. Batch cooling crystallizer: (a) schematic; (b) cooling rates vs. solution temperature in Example 6.4.6; (c) cooling curve for
Example 6.4.7.

temperature. Determine also the enthalpy of crystallization, Example 6.4.7 (Adapted from Nvlt (1992).) A batch cooling
given urc jeq 0.31 0.0017T 0.000012 T2, where T is in  C. crystallizer is to be used to crystallize CuSO4 in a period of 5
hours. The feed solution at 60  C is to be cooled at 30  C after
Solution Figure 6.4.6(a) illustrates a batch crystallizer with seeding the crystallizer with seed crystals of size
an internal heat exchanger. The calculated cooling rates r ps 0:075 mm; these crystals are expected to grow up to
given have been plotted in Figure 6.4.6(b) against the tem- 0.65 mm in 5 hours. The solubilities of CuSO4 at the two
perature T ( C). It appears that at higher batch solution temperatures are: 50  C ! 0.4 g/cm3; 20  C ! 0.24 g/cm3.
temperatures, we have a straight line having a particular The seed crystal suspension density is 244  106 g/cm3.
slope and then, at lower batch solution temperatures, we How should the crystallizer temperature be reduced to
have another line with a smaller slope. The last data point achieve the suggested crystal growth?
where the slope starts changing is 43  C; this must be the
temperature at which the crystallization begins. Solution We employ equation (6.4.55):
From the relation for the rate of cooling (6.4.48b), !
 
dT solu
M se 3Gt Gt G2 t 2
 U ov Ahex T solu  T cf =ws0 c p0 : T T crys  1 ;
dt disat =dT r ps r ps 3 r 2ps
Prior to crystallization, for two different solution tempera- Tcrys 60  C; Mse seed crystal suspension density 244 
tures T solu j1 and T solu j2 , 106 g/cm3; growth rate
80 1 0 1 9
ws0 c p0 <@ dT solu A @ dT solu A = 0:65  0:075 cm 0:575 cm
   T j  T solu j2 ; G  103
U ov Ahex : dt dt ; solu 1 10  5  60  60 s 5  36 s
1 2
80 1 0 1 9
< dT  57:5  105
@ soluA @ dT soluA = 3:2  106 cm=s;
U ov Ahex ws0 cp0    T solu j1  T solu j2 5  36
: dt dt ;
(1
)2 2 3
kJ 19:8  13:8 kJ
1000kg  3:25 hr1 1455 : 6 7
kg-K 56:7  43:3 hr-K 6 244  106 g=cm3 7 3  3:2  106 cm=s
T 60  C  6 7
60:4  0:24g=cm3 7 0:075  101 cm ts
4 5
From equation (6.4.48c), the enthalpy of crystallization is 60  30  C
given by 0 1

3:2  106 t 10:24  1012  t 2 A
f T_ 1 =T_ 2  1gcp0 @
 1
H crys
: 0:075  101 3  0:00750:0075
b 2c T crys =1 ur0
2 3
6
The values of T_ 1 and T_ 2 around the crystallization tem-   4244  10 5
60 C  30 C
perature 43  C are 13.8 and 10.8. The values of b and c are: 0:16
b 0.0017 and c 0.000012; Tcrys 43  C; ur0 0.415.
Therefore 1:28  103
 t 1 4:26  104 t 6:05  108 t 2
1
ff13:8=10:8g  1g 3:251 0:415
H crys 60  C  30  C  15:25  104  1:28  103
0:0017 2  0:000012  43  C
0:277  3:25  1:415 t1 4:26  104 t 6:05  108 t 2
466 kJ=kg:
0:0017 0:00103 60 C  5:85  105 t1 4:26  104 t 6:05  108 t 2 ,

6.4 Continuous stirred tank separators 461

where t is in seconds. The values of T at a few times are Stage 2


indicated in Table 6.4.2. The cooling curve has been illus-
d
1

trated in Figure 6.4.6(c). G2 n2 r p Q n1 r p  Q2 n2 r p ; 6:4:57b


dr p V2 1

6.4.1.1.3 Series cascade of MSMPR crystallizers In Stage 3


MSMPR crystallizers operated as cooling crystallizers, a
d
1

better control of temperature can be achieved by having a Gn nn r p Q nn1 r p  Qn nn r p :


dr p V n n1
number of MSMPR crystallizers operated in series, as shown
in Figure 6.4.7; this is an example of a series cascade of stages. 6:4:57c
Although we have identified the volumetric flow rates
If we assume that the growth rate Gi in the ith crystallizer is
from each stage as different from Qf (i.e. Q1, Q2, Q3, . . ., Qn independent of rp, then we can rewrite these equations in
from stages 1, 2, 3,. . ., n), generally we can assume
the general form for stage i as follows:
Qf Q1 Q2 Q3    Qn : 6:4:56
dni r p Qi1 ni1 r p Qi ni r p
 : 6:4:57d
One can also control the residence time in each stage by dr p Vi Gi V i Gi
varying the volumes of each stage. With greater control over Denoting the residence time in the ith crystallizer as tres, i,
temperature in each stage, due to the availability of larger where
cooling surface area (compared to that in one crystallizer)
and also the residence time in each crystallizer, crystal size t res, i V i =Qi , 6:4:57e
distribution control may be significantly improved. We will we can rewrite the general form of the ith-stage equation as
now identify the population balance equations for stages 1,
2 and n following equation (6.4.2) for MSMPR crystallizers, dni r p Qi1 ni1 r p 1 ni r p 1
 : 6:4:58
where the growth rate G depends on the stage number (i.e. dr p Qi Gi t res, i Gi t res, i
G1, G2, . . ., Gn) and the crystal number density function will
One should add constraints on the growth rate Gi as well as
obviously vary with each stage (i.e. n1(rp), n2(rp), . . ., nn(rp):
the nucleation and growth kinetics in each stage to the
Stage 1 above equations. See Randolph and Larson (1988) for a
detailed treatment. Nvlt (1992) has provided a general
d
1  
G1 n1 r p Qf nf r p  Q1 n1 r p ; 6:4:57a solution for equation (6.4.58).
dr p V1

6.4.1.1.4 Precipitation We have been introduced to the


Table 6.4.2. phenomenon of precipitation in Sections 3.3.7.5 and
4.1.9.4 via chemical reactions, addition of nonsolvents,
t (s) T ( C) salting out via addition of salts like ammonium sulfate
and the hydrophobic effect. Precipitation has considerable
3600 (1 hour) 59.31
7200 (2 hours) 56.99 similarities to crystallization in terms of the devices used
10 800 (3 hours) 52.06 and other concepts, except precipitates are amorphous
14 400 (4 hours) 43.54 solids, rather than crystals, which have specific geometrical
16 200 (4.5 hours) 38.63 shapes. Further, the particle in a precipitate is often an
18 000 (5 hours) 30.34 aggregate of smaller particles and is easily subject to break-
age under fluid shear. This is true of inorganic, organic as

Qf Q3
Q1 Q2 Qn
rif ri 3
ri 1 ri 2 rin
nf (rp) n3(rp)
V1 n1(rp) V2 n2(rp) V3 Vn nn(rp)

Stage Stage Stage Stage


1 2 3 n

Figure 6.4.7. Series cascade of MSMPR crystallizers.


462 Open separators: bulk flow parallel to force and CSTSs

well as biological (protein) precipitates. More often than If we have nf r p 0, then V =Q1 t res as in (6.4.3b),
not, precipitation is carried out in a batch vessel or in a and we get
CSTS (sometimes in tubular devices).  
Consider a CSTS being used as a precipitator (Figure dnr p k _ r 1
p

f =31 n f 1=3 r p  nr p
6.4.2(a)). The general equation (6.4.1a) is valid here also. dr p Ko

 
Further, the processes of agglomeration of smaller precipi- n rp 1
 1 : 6:4:66
tate particles into a larger aggregates and breakage of the rp t res K o
larger particles into smaller ones by shear are quite import-
ant (Randolph and Larson, 1988). For values of f 2 and 2.3 found to be reasonable,
The growth rate G of particle aggregates via collision Glatz et al. (1967) have solved this equation numerically for
with small primary particles is generally modeled as various values of the parameters k and K o , and compared

the results with those obtained from soy-protein precipita-
Gr p dr p =dt K o r p , 6:4:59 tion to select those values of k _ and K o which provided
agreement. Their model predicted that smaller particles
where K o A=4_ 1 , where A is a constant, _ is the shear
are obtained at larger _ ; and further that higher suspension
rate and 1 is the volume fraction of small primary particles
densities led to larger aggregate sizes.
in protein precipitation (Petenate and Glatz, 1983). The
Whereas both birth and death rates are of importance
death rate, De, or the rate of disappearance of larger aggre-
for protein aggregates, in the case of precipitation of spar-
gates into smaller aggregates due to shear in the agitated
ingly soluble salts, often of an inorganic nature, particle
vessel, has been modeled as
aggregate growth by agglomeration is of primary import-
Der p k _ r p nr p , 6:4:60 ance as if the aggregate growth resembles crystal growth.
Randolph and Larson (1988) have provided an analytical
where takes care of a higher breakage probability of solution for the population density function n(rp) in such a
larger size aggregates (Randolph and Larson, 1988). Spe- case. An additional factor to be considered in the case of
cifically, for protein aggregates, Petenate and Glatz (1983) protein precipitates is aging. After some time t in a field of
have described shear rate _ , the protein aggregates reach a constant value
Der p K _ 2 nr p r 3p : 6:4:61 at high values of the aging parameters _ t; this value is
usually slightly smaller than the starting precipitate particle
The birth rate, B(rp), is connected by mass balance to the size.
death rate: Additional analysis and details of MSMPR precipitators
  are available in Randolph and Larson (1988). Detailed
Br p f De f 1=3 r p , 6:4:62 considerations on the thermodynamics of precipitation
for protein-containing systems are available in Ladisch
where f is a measure of the number of particles obtained
(2001). Models describing precipitate aggregate growth
from the breakup of one particle. via diffusion and flow in bioseparations have been illus-
In the steady state form of equation (6.4.1a) represent- trated in Belter et al. (1988). An introductory treatment of
ing a MSMPR precipitator,
precipitation in bioseparations has been provided in Har-
d
1 rison et al. (2003, chap. 8). The subject of precipitation,
Gnr p B  De fQf nf r p  Q1 n1 r p g;
dr p V especially with reference to inorganic systems, has been
6:4:63 treated by Estrin (1993).

if we introduce these developments, we get


6.4.1.2 Solvent extraction: mixersettler
d

K o r p nr p f k _ f =3 r p nr p  k _ r p nr p For contacting two immiscible liquid phases on a labora-


dr p
tory scale, the separating funnel of Figure 6.3.13(b) is used.
1 To achieve rapid mass transfer in such a device with batch-
fQf nf r p  Q1 n1 r p g
V wise introductions of two immiscible phases (e.g. aqueous
and organic), one may employ a mechanical shaker or
dnr p
K o nr p K o r p 6:4:64 many rapid physical inversions. In larger-scale operation,
dr p
as well as in industrial practice, one uses a mixersettler
arrangement: in the mixer, there is usually a mechanical
dnr p nr p 1
)  fQf nf r p  Q1 n1 r p g stirrer to disperse one phase as drops into the other phase
dr p rp K o rp V
and to agitate the liquid phases vigorously; after the mass
k _ 1 =31  1=3  transfer/extraction is completed, the two liquids are
r ff n f r p  nr p g: 6:4:65
Ko p allowed to settle/separate in a separate settler, after which
6.4 Continuous stirred tank separators 463

(a) (b)
Impeller

Phase
Baffle interface

(c)
Equilibrium curve
C
x*iE
E
xeiE
B R(xiR , xiE)
xiE Operating line

xis F (xif , xis)


xiE

0 x*iR xeiR xiR xif


xiR

(d)

WtR ,xiR
Wtf ,xif

Wts ,xis

WtE , xiE

Figure 6.4.8. (a) Mixer a well-mixed vessel with one phase dispersed in the other. (b) Settlerthe two phases have settled into two
separate layers. (c) Equilibrium diagram, operating line and stage efficiencies in solvent extraction. (d) Well-mixed continuous mixer
settler.

the two separated phases are withdrawn from the settler. Green (1984) provide additional background on these
When the two immiscible liquid-phase feeds are intro- mixersettler devices.
duced in a batch fashion into a vessel for mixing, it is We consider briefly first the extent of batch extraction
possible to use the same vessel for the settling stage. How- achieved in a mixersettler operation carried out in a well-
ever, if the two feeds are introduced continuously, settling mixed extraction device (Figure 6.4.8(a)). We select a
is carried out in a separate vessel. A simple mixer and a system having the following properties:
gravity based settler are shown in Figures 6.4.8 (a) and (b).
(1) dilute solution of species i to be extracted;
Both liquid phases may be introduced through the vessel
(2) extracting solvent (phase j s) (which becomes the
bottom, and the dispersion is withdrawn from the top;
extract phase j E after extraction) and feed/raffinate
the latter is introduced into the settler. There are a number
phases (feed phase j f and raffinate phase j R,
of other techniques used in settlers besides gravity, e.g.
respectively) are immiscible.
electrostatic coalescence, centrifugal devices, porous
membranes, etc. Treybal (1963, chap. 10) provides a com- At time t 0, mtf moles of feed phase containing x if mole
prehensive introduction to mixersettlers in solvent extrac- fraction of species i are introduced into the mixer. Simul-
tion and a characterization of their performances. Our taneously, mts moles of extracting solvent are also intro-
treatment is based primarily on Treybal (1963). Perry and duced. We have a dispersion of one phase as drops in the
464 Open separators: bulk flow parallel to force and CSTSs

other phase. The dispersion is well-mixed and the com- According to this definition, the value of E ME is generally
position is assumed to be the same everywhere. As time less than 1.
progresses, the two phases edge closer to equilibrium, as One can look at the progress toward equilibrium based
illustrated in Figure 6.4.8(c). on the extract phase also. The extract-phase composition
At any time t, if one could separate the raffinate and the exiting the real device is x iE , corresponding to a raffinate
extract phases having the mole fraction based compos- composition of x iR . If, however, the extract phase of com-
itions of x iR and x iE , respectively, we can write the position, x iE , leaving the mixer were in equilibrium with
following molar balance equation for species i: the raffinate phase, the hypothetical raffinate composition
would have been x iR , corresponding to point B on the
mtf x if mts x is mtR x iR mtE x iE

equilibrium curve. An estimate of this difference is pro-
) mtf x if  x iR mtE x iE  x is , 6:4:67 vided by the Murphree raffinate-stage efficiency, E MR , which
is defined as
where mtf mtR and mts mtE . This progress of the com-

x if  x iR
positions of the feed (raffinate) phase x if and the solvent E MR : 6:4:71
(extract) phase x is toward the equilibrium composition of x if  x iR
x eiE and x eiR is shown in Figure 6.4.8(c). The compositional On the other hand, one can also define a stage efficiency E
trajectory of progress toward phase equilibrium is shown on the basis of the fractional approach to equilibrium. The
by the straight line FR, where point F has coordinates (x if , feed-phase composition change achieved is (x if  x iR ). If,
x is ) corresponding to the initial feed and the initial however, the phases leaving the mixer were at equilibrium
extracting solvent; point R represents the final two phase (point E, coordinates (x eiR , x eiE )), then the feed-phase com-
compositions of the raffinate (x iR ) and extract (x iE ). position difference achieved would have been x if  x eiR .
If one rewrites equation (6.4.67) as The stage efficiency E is defined as
mtf mtf x if  x iR
x iE  x iR x if x is , 6:4:68 E : 6:4:72
mtE mtE x if  x eiR
it is a representation of the extract-phase composition, x iE , On the basis of the extract phase, the corresponding stage
as a function of the raffinate-phase composition, x iR , and efficiency is given by
some other quantities defined earlier, namely x if and x is .

x iE  x is
This straight line, having a slope of mtf =mtE , is called E : 6:4:73
the operating line, since it charts the composition of the x eiE  x is
two phases as the extraction operation progresses toward The two efficiencies are equal, since line FR is part of line
phase equilibrium, represented by point E (x eiR , x eiE ), when FE, and the ratio of their intercepts on the x iR axis and x iE
the two phases are in equilibrium with each other. If, in axis is the same:
fact, phase equilibrium were achieved, then the total
number of moles of species i transferred would be given by length of FR
E : 6:4:74


length of FE
mi jtransferred mtf x if  x eiR mts x eiE  x is : 6:4:69
Consider the case where the equilibrium behavior is linear:
In practice, when the extraction process is completed in x eiE iE x eiR ; x iE iE x iR ;
the mixer and the two phases are separated in a settler, the 6:4:75
x iE iE x iR ,
two phases are often not at equilibrium with each other. In
such a case, the end of the process is located at a point and the ratio (mtE/mtR) is constant. It has been suggested
R (x iR , x iE ). The extent of approach to equilibrium achieved (Treybal, 1963, table 10.1) that the relations between E, the
by each phase is frequently estimated in the following way. stage efficiency, and E ME or E MR are as follows, when the
The extract-phase final composition achieved is x iE . How- distribution coefficient, iE , is constant and the solutions of
ever, if the raffinate phase leaving the mixer at a compos- species i in both of the two immiscible phases are dilute:
ition x iR were in equilibrium with this extract phase of    
mtE mtE
composition x iE , then this hypothetical extract compos- E ME 1 iE E MR 1 iE
mtf mtf
ition would have a higher value, x iE (corresponding to E mtE mtE : 6:4:76
1 iE E ME iE E MR
point C on the equilibrium curve in Figure 6.4.8(c)). An mtf mtf
estimate of this deficiency is called the Murphree extract-
stage efficiency, E ME : The progress of batch extraction in a well-mixed vessel
with time can be measured if the overall mass-transfer
x iE  x is composition change achieved in extract coefficient and the dispersed phase surface area per unit
E ME  :
x iE  x is ideal extract phase composition change volume of liquid are known. In Section 3.4.1.2, we were
6:4:70 introduced to the overall mass-transfer coefficient of
6.4 Continuous stirred tank separators 465

species A, KAw or KAo, depending on whether it was based separation is. A quantity called the number of transfer units
on aqueous phase (j w) or organic phase (j o). These is often employed to quantify this extent of difficulty in
Ks were based on concentration differences. Here we will separation. To illustrate, consider the well-mixed extractor
also use KxE and KxR, based on mole fractions and the operated continuously (Figure 6.4.8(d)). The extractor has
extract and the raffinate phases, respectively. Let the mole a liquid-phase volume of V and a liquid-phase height H or
fraction based overall mass-transfer coefficient of the species length L. In a small section of the extractor of length dL or
being extracted with respect to the extract phase be KxE in height dH or volume dV, the total rate of transfer of
the vessel, having a liquid volume of V and a drop surface species i from the feed liquid into the extract phase is
area per unit liquid volume of av. Then the molar rate of given by
transfer of the species being extracted, at any time, is

W tf dx iR K xR av x iR  x iR dV : 6:4:81
dfmts x is g dx is

mts K xE av V x iE  x iE , 6:4:77 The number of transfer units based on the raffinate phase,
dt dt
NtOR, is obtained from the above relation as
where x iE is the extract-phase mole fraction of species i
being extracted at any time and x iE is the hypothetical xiR V
dx iR K a K xR av V
extract-phase mole fraction of species i which would be N tOR
xR v dV ;
x iR  x iR W tf W tf
in equilibrium with x iR at any time. Further, we are dealing x if 0
with a dilute solution of i in both phases. Integrating this 6:4:82
equation over a time period of t seconds, we get
x if  x iR

N tOR : 6:4:83
mts x iE  x is K xE av V x iE  x iE avg t, 6:4:78 x iR  x iR

where In this example of a well-mixed continuous extractor, x iR


does not change along the extractor length; neither does
xiE


x iE . Further, x iR is the hypothetical composition of the
x iE  x iE dt x iE  x iE avg t, raffinate phase in equilibrium with the exiting extract-

x is phase composition x iE iE x iR . The composition change


and K xE av is assumed to be time-invariant. achieved in the raffinate phase, i.e. x if  x iR , is driven by
In well-mixed extractors operated continuously (Figure the mole fraction based concentration difference x iR  x iR .

6.4.8(d)), the two phases are well-mixed such that essen- Thus, NtOR measures the difficulty of separation in this
tially each liquid phase has the same composition through- case. If the change x if  x iR desired is large, but the driving
out the vessel; however, each such composition is different mole fraction based concentration difference, x iR  x iR , is
from that of the feed stream (or the solvent stream) small, then the value of NtOR is large. Correspondingly, for
entering the vessel. For example, the mole fractions of a given KxRav, V (therefore, H or L) has to be large, and the
the raffinate and the extract at all locations in the mixer separation is demanding. However, if the change
are, respectively, x iR and x iE . The molar rate of extraction of (x if  x iR ) desired is small and the driving mole fraction
species i per unit extractor volume is difference, (x iR  x iR ), is large, then the value of NtOR is


small, and the separation job is relatively easy; the
N i av K cE av C tE x iE  x iE : 6:4:79 extractor dimensions are also smaller. One can also dem-
onstrate easily the following relation between the Mur-
Here, CtE is the total molar concentration of the extract phree raffinate-stage efficiency, E MR , and NtOR:
phase having a species i mole fraction of xiE; also we have
x if  x iR
used the molar concentration based overall mass-transfer
coefficient, KcE, with respect to the extract phase. The total x if  x iR E MR x if  x iR
N tOR : 6:4:84
molar rate of extraction of species i in the vessel of volume x iR  x iR 1  E MR 1  x if  x iR
x if  x iR
V is given by

The volume V (or height H or length L) of an extractor
N i av V K cE av V C tE x iE  x iE : 6:4:80
needed for the separation may be expressed from equation
The dispersion leaving the extractor (Figure 6.4.8(d)) con- (6.4.82) as follows:
tains essentially two different streams, the extract and the  
W tf
raffinate. V Aex H Aex L N tOR , 6:4:85
K xR av
Equations (6.4.78) and (6.4.80) illustrate how mass-
transfer coefficients influence the total number of moles where Aex is the cross-sectional area of the extractor. The

transferred or the total molar rate of transfer taking place in term W tf =K xR av Aex is often called the height of a trans-
a well-mixed extractor operated batchwise or continuously. fer unit (HTU) or length of a transfer unit (LTU). This
One would also like to estimate how difficult this quantity identifies how quickly mass transfer is taking
466 Open separators: bulk flow parallel to force and CSTSs

place in a given extractor for a given flow rate. If it is very Substitute for xiR from (6.4.86d) and utilize (6.4.86e) in the
quick, then the value of HTU or LTU is small for the given integral (6.4.86c) to obtain
separation situation with respect to the composition xiE
change; therefore, for a given cross-sectional area, the dx iE
N tOE iE mtE
continuous extractor/separator is short. For a batch iE x if  x iE  x is  x iE
x is mtf
extractor, the equivalent situation would lead to a small
volume. xiE
Equation (6.4.84) provides a relation between NtOR and dx iE
0 1 0 1
one of the Murphree stage efficiencies for a continuous x is iE mtE A iE mtE A
@ e @
well-mixed extractor. In a well-mixed batch extractor, the  1 x iE iE x iR 1
mtf mtf
difficulty or ease of separation will be determined by the
xiE
time needed to achieve the desired change in composition. 1 d x iE
Thus, the number of transfer units NtOE based on the 0 1
x iE  x eiE
extract phase in this case may be obtained from equation @1 iE mtE A x is

(6.4.77) as mtf
0
1
xiE e
dx x  x is K a V t 1 x iE  x iE A
N tOE  is
 iE
xE v : 0 1 ln@ ; 6:4:86f
x iE  x iE x iE  x iE avg mts x is  x eiE
x is @1 iE m tE A
mtf
6:4:86a

A longer time here means a more difficult separation. One 1


N tOE    ln1  E , 6:4:87a
may also describe the change needed in terms of the iE mtE
1
raffinate phase: mtf
xiR
where for the stage efficiency, E, definition (6.4.73) was
dx iR x if  x iR K xR av V t
N tOR

: used. For a well-mixed batch extractor, the mass-transfer
x iR  x iR x iR  x iR avg mtf
x if information of relation (6.4.86a) will allow one to deter-
6:4:86b mine as a function of time the value of the stage efficiency
E achieved in the mixer. The corresponding results, in
For such an extraction process in a well-mixed batch terms of the raffinate streams, may also be derived in an
extractor, it is also useful to have a relation between the identical fashion for a well-mixed batch extractor:
stage efficiency E (definitions (6.4.72) and (6.4.73)) and  
mtE
NtOE or NtOR as defined above. Consider the integral in iE
equation (6.4.86a). Noting that dx is dx iE and K xR av V t mtf
N tOR   ln1  E :
x iE iE x iR , we rewrite it as mtf iE mtE
1
mtf
xiE xiE
dx is dx iE 6:4:87b
N tOE
6:4:86c
x iE  x iE iE x iR  x iE
x is x is The quantity (iE mtE =mtf ) encountered earlier in relation
(6.4.76) is called the extraction factor. For a continuous
However, from the basic mass balance equation (6.4.67), extractor, the corresponding expression for the extraction
we get factor is (iE W ts =W tf ) (see Problem 2.2.1).

In a mixer, which is frequently a cylindrical vessel with
x iR x if  mtE =mtf x iE  x is : 6:4:86d
an agitator (Figure 6.4.8(a)), there are several aspects of
If the two phases were at equilibrium at the end of extrac- importance in solvent extraction. The vessel usually has a

tion x eiR , x eiE , then relation (6.4.67) will become dished bottom, with vertical baffles placed along the length
  of the vessel/extractor and an impeller, propeller or turbine
mtE in the center of the vessel (sometimes at an angle to
mtR x if  x eiR mtE x eiE  x is ) x if x is
mtR prevent the creation or generation of a vortex). In
 
mtE aqueousorganic systems, one phase is continuous, with
x eiE x eiR :
mtR the other phase dispersed as drops; the continuous phase
must occupy at least 2530% of the total liquid volume: in
Since x eiE iE x eiR , we get general, it is higher. The phase to be dispersed is intro-
    duced into the continuous phase already present in the
mtE iE mtE vessel. A minimum impeller speed is needed to eliminate
x if x is x eiR 1 : 6:4:86e
mtR mtR separate layers of two immiscible liquid phases and to
6.4 Continuous stirred tank separators 467

ensure that one phase is completely dispersed in the other continuous phase/volume (there is factor missing)
as drops. whose fluctuations through velocity fluctuations lead to
The drop size achieved is important in estimating the pressure fluctuations, which create drops/drop breakup.
mass-transfer surface area developed in the dispersion. In Higher-pressure fluctuations leading to a higher Weber
certain situations, the surface area of the dispersion per number yield smaller size droplets. For an impeller-driven
unit volume of the total liquid phase, av, is available from system, with the impeller rotating at N revolutions per
measurements. The average or mean drop size appropri- minute (rpm), the impeller dimension is important.
ately defined can then be related to av. For example, the Therefore
Sauter mean diameter d32 of the drop size number density
 3

W e c v2c d imp = c N 2 d imp = : 6:4:92
distribution has been related to av via the following relation
and the dispersed phase volume fraction D:
The following correlations are useful for estimating av or
6 d32 in particular types of agitated devices.
d 32 D: 6:4:88
av
(1) For four-bladed flat-blade paddles and six-bladed
The Sauter mean diameter d32 is simply 2r p3, 2 , and may be turbines,
obtained from definition (2.4.2h) for r pii, i with i 2 for a
100 D W e0:6
drop size number density function n(rp) as av 6:4:93
C dimp


where dimp impeller diameter range 0.190.83 ft;
r 3p n r p dr p
organic liquid volume fraction dispersed in water,
0
r p3, 2 : 6:4:89 D 00.20; baffled-vessel diameter range 0.58


1.25 ft; C 1 3.75D for four-bladed paddles
r 2p n r p dr p
((dimp/dvessel) 2/3); C 1 9D for six-bladed tur-
0
bines ((dimp/dvessel) 1/3); quoted in Treybal (1963),
Since av is the total interfacial area of drops per unit based on Calderbanks work.
volume of the total liquid phase, and the drop phase (2)
volume fraction is D, we get
d 32 2 r p3, 2 0:052 d imp W e0:6 e4D for W e < 10 000;



6:4:94a
4r 2p n r p dr p 3D r 2p n r p dr p
0:6
0 0 3D 6D d 32 2 r p3, 2 0:39 dimp W e for W e > 10 000:
a v D :
r p3, 2 d32 6:4:94b
4 3

r n r p dr p r 3p n r p dr p
3 p These correlations were developed by Gnanasundaram
0 0
et al. (1979).
6:4:90
Additional physical properties and mass-transfer cor-
The mean drop size achieved, or the interfacial area relations are sometimes needed to determine the perform-
obtained, depends on the inertial forces due to dynamic ance of the well-mixed baffled device. The viscosity of the
pressure fluctuations breaking up the drop countered by two-phase dispersion, mix , and its density, mix , are to be
interfacial tension forces in the drop phase with respect to determined from (Treybal, 1963)
the continuous phase. This balance is the basis of Weber  
number, We: 1:5 D D
mix c 1 ; 6:4:95a
c D c
inertial force in the continuous phase
We mix c c D D , 6:4:95b
force due to interfacial tension
pressure fluctuations in the continuous phase where c and D are the viscosities of the continuous and the

countering pressure via interfacial tension dispersed phases, respectively; the corresponding phase
densities are c and D . For baffled vessels having an impeller,
c v2c
; the mass-transfer coefficient for the continuous phase, k c ,
=d p
may be obtained from the following correlation (Treybal,

W e c v2c dp = : 1963):
6:4:91 k c d vessel
0:833
0:052 Reimp Scc 0:5 : 6:4:96
Dic
Here, the subscript c refers to the continuous phase, is the
interfacial tension between the dispersed and the continu- Here d vessel is the diameter of the vessel, Dic is the diffusion
ous phases, and c v2c represents the kinetic energy of the coefficient of i in the continuous phase, Reimp is the
468 Open separators: bulk flow parallel to force and CSTSs

 
Reynolds number of the impeller d 2imp N imp c =c , overall mass-transfer coefficient and av for this well-mixed
N imp is the impeller speed in revolutions/time, and the extractor if the impeller rotates at 200 rpm. Calculate the value
Schmidt number of the continuous phase, Scc , is of the Murphree efficiency E MR . The temperature is 20 C.
c =c Dic . The correlations (3.1.167) and (3.1.168) for the You are given: density of aqueous solution 0.985 g/cm3;
density of toluene 0.866 g/cm3; viscosity of aqueous
dispersed-phase Sherwood number may be utilized to
solution 0.96 cp; viscosity of toluene 0.59 cp; diffusion
determine the value of the dispersed-phase mass-transfer
coefficient of organic compound in water 1.2  105 cm2/s;
coefficient k D . For an aqueousorganic system, if the diffusion coefficient in toluene 1.68  105 cm2/s; inter-
organic phase is assumed to be the extract phase as well facial tension 12 22 dyne/cm.
as the dispersed phase, we can follow relations (3.4.18) and
(3.4.19), and obtain, in terms of molar concentration dif- Solution To determine E MR in a continuous-flow system,
ferences, the following relations between the overall mass- we need NtOR (equation (6.4.84)). From equation (6.4.85),
KxR and av are needed to determine NtOR since Wtf and V
transfer coefficient based on a particular phase and the
are known. So our focus now will be to determine KxR and
individual phase mass-transfer coefficients:
av. Toluene flow rate into the vessel is 20% of the flow rate
1 1 1 of the aqueous solution. Therefore, toluene has to be the
; 6:4:97a dispersed phase: D (34/170) 0.20; c 0.80. To calcu-
K cR k cR iE k cE
late kcR, we employ equation (6.4.96) for the six-bladed
1 iE 1 turbine:
: 6:4:97b
K cE k cR k cE  0:833
k cR d vessel =Dic 0:052 d 2imp N imp c =c c =c Dic 0:5 ;
Here j w R and j o E for the phase identification.
There are a couple of additional factors to be kept in 0 10:833
mind. A certain amount of energy has to be supplied k cR  61 @ 612  200  0:985 A
0:052
through the impeller rotating at a speed beyond the min- 1:2  105 4  60  0:96  102
imum impeller speed. The power consumption, Pr, in such 0 10:5
agitated vessels may be obtained from graphical correl- 0:96  10 2
@ A ;
ations available between the power number, Po Pr= 0:985  1:21  105
N 3 d 5imp mix , and the impeller Reynolds number, Reimp
0:052  105
(Treybal, 1963). k cR 318 1580:833  8050:5
50:48
In continuous-flow mixers, either phase can be made
continuous: start the mixing by filling up the vessel with a 0:052  105  38 000  28:37

particular phase, then disperse the other. However, the 50:48
continuous-phase volume fraction in terms of the flow rate
ratio must be greater than 0.25. In batch mixing, the situ- 1:06  102 cm=s:
ation is more complex. The Weber number is
The length dimensions, or the height of the well-mixed 2 3
c v2c d imp c N d imp
vessel, having liquid extraction in a continuous fashion are We
12 12
quite important. Normally such a system will be analyzed
0:985  2002  30:54  932
under the condition of two different phases flowing parallel 14 156:
to each other, with the driving force perpendicular to both 602  22
phases: the phase concentrations will change along the Use equation (6.4.93),
length/height of the device. This is treated in Chapter 8. 100  D  W e0:6
An extreme case of such a configuration is encountered av ;
Cdimp
when there is so much back mixing in both phases that we
C 1 9 D for a six-bladed turbine (the (dimp/dvessel) ratio
have a well-mixed tank/vessel. This extreme situation is
needed for this correlation is 1/3), so
the only configuration of relevance in this section for the
case of continuous flow. 100  0:2  14 1560:6 20  309
av 72:55 cm1 ;
1 9  0:2  30:48 2:8  30:48
Example 6.4.8 An organic compound present in low con-
centrations in water is to be extracted into toluene in a well- 6D 6  0:2
dp 0:0165 cm:
mixed vessel. The baffled cylindrical vessel, having a six- av 72:55
bladed turbine impeller of 30.48 cm diameter, is 61 cm in
height and diameter. The aqueous solution flow rate into From equation (6.4.97a),
this vessel from the bottom is 170 liter/min; the toluene flow 1 1 1 1 1 1 1
rate (from the bottom also) is 34 liter/min. The value of the
K cR k cR iE k cE k cR 25 K cE k cR
distribution coefficient of this organic compound, iE , is 25.
The dispersion is withdrawn continuously from the top into a (assuming KcE is of the order of KcR). We may as well
settler. Determine the values of the continuous-phase based therefore consider
6.4 Continuous stirred tank separators 469

K cR 1:06  102 cm=s: or buffer solution or feed solution into this well-stirred
vessel at a constant pressure.
Further, k xR kcR C t ) k xR K cR C t . From equation
Consider Figure 6.4.9(a), where, at time t 0, a batch
(6.4.85), we observe that
solution of volume, Vf0, is introduced as feed to the vessel
K xR av V K cR C t av V 1 on top of the membrane; this well-mixed solution has a
N tOR ; C t gmol=cm3 ;
W tf W tf 18:5 molar concentration, Ci0, of macrosolute species i. If after
some time the well-mixed batch feed solution volume is
1:06  102  1  72:55  d vessel 2 reduced to VfR, the volume of the retentate, by means of
N tOR  61
170  103 ultrafiltration, what is the macrosolute concentration CiR in
18  4
60 the retentate? It may be assumed that the observed macro-
solute rejection Ri for the species i remains constant during
0:7255  1:06   61  61  61  60
this concentration process (assuming that the macrosolute
18  170  103  4
is substantially rejected). The extent of volume reduction in
2:38 units: the well-mixed feed solution is often identified as the
Vf0
From equation (6.4.84), volume concentration ratio VCR : 6:4:98
VfR
E MR
N tOR ) E MR 2:38  2:38 E MR If, in a differentially small interval of time, dt, a differen-
1  E MR
) E MR 2:38=3:38 0:70: tially small volume of magnitude jdV f R j permeates through
the membrane, and the concentration of the macrosolute
If the problem had involved a batch extractor, most of the
calculations would be similar, except, instead of equation in the ultrafiltrate is Cip, whereas the macrosolute concen-
(6.4.85), one should employ equations (6.4.86a,b) and tration in the retentate at this time is CiR, then a simple
(6.4.87a,b) to determine the progress of extraction as a func- macrosolute balance leads to
tion to time.

dV f R C iR  C ip V f R d C iR :
moles retained by moles contributing to increased
6.4.2 Well-mixed separators membrane based the membrane concentration in retentate 6:4:99
devices
By assumption, during this whole process the observed
In this section, we will briefly describe two membrane macrosolute rejection Ri is constant:
processes, ultrafiltration and membrane gas permeation,
C ip
using configurations where we can assume that the feed Ri 1  : 6:4:100
C iR
mixture region is well-mixed; its composition equals that of
the concentrate stream. We can now rearrange equation (6.4.99) to yield
dC iR dVfR
dV f R C iR Ri V f R dC iR ) Ri :
6.4.2.1 Ultrafiltration: well-stirred cell C iR VfR

We have already encountered general applications of Integrating from Vf0 to VfR and Ci0 to CiR, we obtain
ultrafiltration (UF) at the beginning of Section 6.3.3.2: 0 1Ri
C iR VfR @ Vf0A
(1) the concentration of a macrosolute by removing the log Ri log log ;
C i0 Vf0 VfR 6:4:101
solvent through the membrane,
Ri Ri
(2) the removal of smaller molecular weight impurities C iR C i0 V f 0 =V f R C i0 V CR :
along with the solvent through the membrane via fresh
Therefore, the retentate macrosolute concentration, CiR, at
solvent/buffer addition (called diafiltration), and
any time t may be estimated by knowing the reduction in
(3) the fractionation of a mixture of macrosolutes.
the feed volume, as long as the observed macrosolute
In all such applications, if a small volume of solution is to rejection is assumed to be constant (Cheryan, 1986,
be processed/treated in a laboratory, a well-stirred vessel 1998). Since in such operations Ri can be less than 1, some
with a membrane at one end is usually employed. Figure of the macrosolute is lost in the permeate. The yield of the
6.4.9(a) illustrates batch processing of a solution in a vessel macrosolute in this batch concentration process is
that is well-stirred. Flow in this system comes about due to
V f R C iR
the passage of the solvent and solutes/macrosolutes yield extent of recovery ; 6:4:102
V f 0 C i0
through the membrane and out of the system. The solution
is under pressure; the permeate emerges through the  Ri  Ri 1
VfR Vf0 Vf0
membrane into essentially atmospheric pressure. Figure yield Y iR , 6:4:103
Vf0 VfR VfR
6.4.9(b) illustrates the continuous introduction of solvent
470 Open separators: bulk flow parallel to force and CSTSs

(a)

Pf Vf 0
Ci 0
Pf

VfR CiR
Membrane Permeate

Volume = Vf0 - VfR


Cip Cip
Concentration = Cip
Pp<Pf Pp<Pf

(b)

Pf t=0,Vf 0, Ci0
Feed
solution or
buffer t=t, VfR, CiR
solution

Membrane

Cip, Pp<Pf

(c)
Stage 1 Stage 2
Stage 3
Add buffer Add buffer

(Vf0-VfR) (Vf0-VfR)

Pf Vf0 Pf Vf0 Pf Vf0


Vessel 1 t=0

t=0 t=0

Ci0|1 Ci0|2 Ci 0|3 Vessel 3


Membrane

Vessel 2
Cip|1 Cip|2 Cip|3

Vessel 1 Pf Pf t = t2 Pf

t = t1
VfR VfR t = t3
CiR|1 CiR|2
VfR CiR|1 Transfer VfR CiR|2 Transfer VfR CiR|3

Cip|1 Cip|2 Cip|3

Figure 6.4.9. Ultrafiltration in a well-stirred vessel: (a) batch ultrafiltration; (b) continuous diafiltration/ultrafiltration; (c) discontinuous
diafiltration using three stages.
6.4 Continuous stirred tank separators 471

where YiR is the extent of segregation of species i in the so its weight % in the permeate is 0.23%.
retentate phase (R-phase). For apoferitin:

Example 6.4.9 An aqueous buffer solution volume of 10 liter 1  0:0955 0:01


9 C ip 10 
contains by weight 1% inulin (mol. wt. 5000), 1% albumin Mapoferitin t Mapoferitin t
(mol. wt. 67 000) and 1% apoferitin (mol. wt. 480 000). Batch
ultrafiltration is carried out in a well-stirred UF cell having a 0:1  0:0955 0:0005
) C ip ,
membrane possessing a molecular weight cut-off value of 9  Mapoferitin t Mapoferitin t
30 000 till the solution volume is reduced to 1 liter. Obtain
the concentrations of the three proteins in the permeated so its weight % in the permeate is 0.05%.
volume in weight %.
We will now focus on the process illustrated in Figure 6.4.9
Solution First, select a membrane having a MWCO of 30 000. (b), wherein a well-stirred vessel having a volume of solu-
Table 6.3.8 indicates that membrane PM 30 may be one such tion V f 0 of a macrosolute i at concentration C i0 undergoes
membrane. The rejection values (Ri) for different proteins are ultrafiltration; however, unlike that in Figure 6.4.9(a), a
inulin 0%, albumin 90%, apoferitin > 98%: buffer solution having no solute species i is added continu-
In batch ultrafiltration, the volume concentration ratio (VCR)
ously to this vessel from an external source such that the
(definition (6.4.98)) is (10/1) 10. The retentate concen- solution volume remains constant at V f 0 in the vessel. This
tration of each protein can be determined using mode of operation is called continuous diafiltration. There
CiR Ci0(VCR)Ri . Here we assume: are two goals: if there is a lower molecular weight impurity
in the feed solution, it can be eliminated with the ultrafil-
(1) molar concentration of i,
trate, since its Ri value is likely to be low; the continuous
wt: fraction i addition of the solvent via the buffer also prevents an
Ci ;
mol: wt: i t increase in the viscosity of the solution of the macrosolute
(2) t is essentially constant for all solutions at t . as ultrafiltration goes on. In the analysis that immediately
follows, the subscript i for a solute is valid for the macro-
For inulin:
solute as well as for the low molecular weight impurity.
0:01 0:01 Consider a time interval dt over which a differential
C iR C i 0 VCRRi 100:00 ,
Minulin t Minulin t volume dV b of the buffer is added to the well-stirred
since Ri 0:0; therefore the weight % of inulin in the feed vessel containing a solution volume V f 0 ; to start
retentate is 1%. with, this solution has a solute concentration C i0 (the
For albumin: lower molecular weight impurity or the macrosolute of
0:01 0:0794 interest). Continuous diafiltration is carried out such
Ri 0:90 ) C iR 100:9 ; that the solution volume in the feed vessel remains
Malbumin t Malbumin t
constant at V f 0 . Therefore the volume of ultrafiltrate
therefore the weight % albumin in the retentate is 7.94%. produced in time dt is dV b : it has a species i concen-
For apoferitin:
tration of Cip at time t. A molar balance on species i
0:01 0:0955 leads to
Ri 0:98 ) C iR 100:98 ;
Mapoferitin t Mapoferitin t
dV b C ip V f 0 dC iR
therefore the weight % of apoferitin in the retentate is 9.55%.
If the permeate concentration of a species i is Cip, then moles of i lost decrease in moles
in permeate of i in retentate 6:4:104
9 literC ip 1 literC iR 10 literC i0 :
For inulin: It is further assumed here that, during the process, Ri is
constant, even though CiR and Cip are changing. Since
0:01 0:01
9 C ip 1 10  Ri 1  C ip =C iR , we can write C ip 1  Ri C iR . Introdu-
Minulin t Minulin t
0:01 cing this relation into equation (6.4.104), we get
) C ip ) 1:0 wt:%:
Minulin t
dV b 1  Ri C iR  V f 0 dC iR
Inulin, having 5000 mol. wt., with no rejection through the
CiR Vb
membrane, passes right through. So the permeate and the dC iR dV b 1  Ri
feed concentrations are identical. ) 1  Ri Vb
C iR Vf0 Vf0
For albumin: C i0 0

0:0794 0:01 C iR
9 C ip 1 10  ) ln 1  Ri V b =V f 0 ;
Malbumin t Malbumin t C i0
0:1  0:0794 0:0023
) C ip , C iR C i0 exp 1  Ri DF, 6:4:105
9  Malbumin t Malbumin t
472 Open separators: bulk flow parallel to force and CSTSs

where These calculations indicate that before a UF process starts in


vessel 2 (stage 2), the macrosolute (i protein) concen-
Vb
DF dilution factor tration is identical to Ci0, the original concentration we
Vf0
started with in vessel 1 (stage 1); however, the microsolute
volume of buffer added volume of permeate concentration has been reduced to one-tenth of the original

feed volume feed volume concentration in vessel 1. Therefore, before UF starts in
volume of permeate vessel 2 (stage 2), the protein solution is considerably purer
: 6:4:106 since the microsolute impurity concentration has been sub-
retentate volume
stantially reduced. If we carry this process further to a third
The fraction of solute (macrosolute) left in the feed/reten- well-stirred vessel (vessel 3, stage 3), we will obtain an
tate (R) liquid is the segregation fraction, given by impurity (species j) concentration which is one-hundredth
of the original concentration (Figure 6.4.9(c)). Multi-vessel
V f 0 C iR C iR discontinuous diafiltration may thus be a substitute for a
Y iR exp1  Ri DF : 6:4:107
V f 0 C i0 C i0 continuous diafiltration (Figure 6.4.9(b)). The strengths of
each approach have been compared by Cheryan (1986).
The purpose of continuous diafiltration in purification of a
Continuous diafiltration is preferred when retentate viscos-
protein solution from a small molecular weight impurity
ity increases greatly and therefore affects the flux of the
(e.g. NaCl) can become clear from this result. Suppose
solvent. Discontinuous diafiltration would be advantageous
the value of DF is 6, i.e. the volume of buffer solution
when a concentrated protein solution is required (amongst
added ( the volume of permeate) is six times that of
other advantages).
the feed (here also the retentate) solution. Let Ri for salt
A quantitative estimate of the concentration of any
(NaCl) 0.0 and Ri for protein 0.995. Then, for salt,
solute (microsolute or macrosolute) in the solution
1 1 remaining in the nth stage after discontinuous diafiltration
Y iR exp6 0:002;
exp6 403 may be developed as follows. Focus on Figure 6.4.9(c).
Consider vessel 1, containing a solute concentration C i0 j1
and, for protein,
in a solution volume V f 0 . If we carry out the batch concen-
1 1 tration for time t1 to reduce the solution volume to V f R
Y iR exp0:03 0:97:
exp0:03 1:0305 (<< V f 0 ), then the retentate concentration in solution
volume V f R is (by equation (6.4.101))
Therefore the impurity (salt) concentration is reduced to
0.2% of the original level at the cost of a loss of 3% of the
R
C iR j1 C i0 j1 V f 0 =V f R i , 6:4:108
protein. An ultrafiltration membrane that has a value of
Ri 1 for the protein will be better; however, the filtration provided Ri may be assumed constant during the process.
flux levels would be lower, prompting more membrane In the discontinuous diafiltration process, this solution is
area or a higher time for the continuous diafiltration transferred to vessel 2 (stage 2) and fresh buffer is added to
process. increase the volume back to V f 0 from V f R . Correspond-
An alternative strategy is often adopted: it is called ingly, the solute concentration in stage 2 will now be
discontinuous diafiltration. Consider Figure 6.4.9(c), which reduced to C i0 j2 :
shows three identical well-stirred vessels (vessels 1, 2 and
R 1
V f 0 C i0 j2 V f R C iR j1 ) C i0 j2 C i0 j1 V f 0 =V f R i :
3) having the same ultrafiltration membrane. At time t 0,
a volume V f 0 of the feed solution containing a macrosolute 6:4:109
i and a microsolute (say, salt) j is present in vessel 1. Such a solution in vessel 2 (stage 2) now undergoes batch
Assume for the time being that Ri 1 and Rj 0. Let the ultrafiltration for time t2 in a well-mixed vessel with a
value of the VCR in batch ultrafiltration be 10. Then constant Ri . The retentate concentration C iR j2 is
C iR j1 10 C i0 and C jR j2 C j0 (from equation (6.4.101)).

R
12Ri 1
Now let this small volume of concentrate (volume C iR j2 C i0 j2 V f 0 =V f R i C i0 j1 V f 0 =V f R :
V f 0 =10) be transferred to the next well-stirred vessel 6:4:110
(vessel 2), where an amount of fresh buffer solution is
added to bring the total solution volume to V f 0 . Therefore This solution of volume V f R is now transferred to vessel 3
the two solute concentrations in this vessel 2 are now: (stage 3) and fresh buffer is added to bring the solution

volume up to V f 0 . Correspondingly, the concentration
10 C i0 V f 0 =10 C i0 V f 0 C i0 j3 will be
C if C i0 ; and
Vf0 Vf0


2Ri 1
C j0 V f 0 =10
V f 0 C i0 j3 V f R C iR j2 ) C i0 j3 C i0 j1 V f 0 =V f R :
C jf ) C jf C j0 =10 :
Vf0 6:4:111
6.4 Continuous stirred tank separators 473

This solution of volume V f 0 now undergoes batch ultrafil- for the low molecular weight impurity,
tration in vessel 3 for time t3 to produce a retentate concen-
tration C iR j3 : C jR C j0 200 C j0 :
    Therefore
V f 0 Ri V f 0 3Ri 2
C iR j3 C i0 j3 C i0 j1 C iR C j0
VfR VfR ij  20  1 20:
  C i0 C jR
V f 0 13Ri 1
C i0 j1 : 6:4:112 Obviously batch ultrafiltration or one-stage discontinuous
VfR
diafiltration is a more efficient technique for purifying the
One can therefore conclude via a process of induction that protein of a low molecular weight impurity. However, one
if there are n vessels (or n stages), the retentate concen- must ensure that the required solvent flux level is achievable
tration in the nth stage will be in batch UF when the volume is reduced.

There are two other aspects of importance in practical


C iR jn C i0 j1 V f 0 =V f R 1nRi 1 : 6:4:113
ultrafiltration: processing time and the membrane area.
This result will allow one to calculate easily the concen- A brief treatment of processing time with a view to minim-
tration of a small molecular weight impurity after discon- izing it will be considered now. The processing time
tinuous diafiltration in n stages. Suppose Ri for the depends on a variety of factors: the total volume to be
impurity is 0 and V f 0 =V f R 10. Then we see that, for processed; the concentration of retained solids, especially
three stages, C iR j3 will be C i0 j1 =100, a considerable if it leads to gel polarized operation (equation (6.3.143));
degree of purification indeed. the mode of operation (continuous, batch, etc.) (Cheryan,
1986). One approach suggested by Ng et al. (1976) employs
Example 6.4.10 Compare the purification achieved for a
the gel-polarized condition and perfect rejection of the
protein (species i) in relation to a low molecular weight
protein i for continuous diafiltration operation:
impurity (species j) for the following two processes: one-
 
stage discontinuous diafiltration vs. continuous diafiltration. C igel
The initial volume of solution is 10 liter; the amount of N s V s jvz j k il ln : 6:4:115
C ilb
ultrafiltrate produced is 9.5 liter. The UF vessel is well-stirred.
The values of Ri and Rj for the membrane used are: Ri 1, In our notation used so far, C ilb C iR , where its value
Rj 0. Employ a separation factor, ij , to compare. changes with time from an initial C i0 to C iR . The processing
time, tp, required is
Solution The basis of comparison is ij :
C iR C j0 C iR C j0 permeate volume V b buffer volume added
ij : 6:4:114 tp :
C jR C i0 C i0 C jR solvent flux jvz j k il ln C igel =C iR

Calculate ij for both


6:4:116

processes: this means calculate
C iR =C i0 and C jR =C j0 for both processes. However, for a given amount of retained proteins, mi,
For continuous diafiltration: use equation (6.4.105), where
DF 9:5=10 0:95. For the protein, mi
V b DF V f 0 DF : 6:4:117
C iR
Ri 1 ) C iR C i0 exp1  1DF C i0 ;
Therefore
for the low molecular weight impurity, DF mi
t
: 6:4:118
C iR k i ln C igel =C iR
C jR C j0 exp1  0DF C j0 exp0:95

C j0 =2:58 ) C j0 =C jR 2:58: If C igel and k i remain unchanged during the continuous


diafiltration process, one can determine the optimum pro-
Therefore, for continuous diafiltration,
cessing time, topt, by determining C iR , which satisfies
C iR C j0 dt=dC iR 0:
ij  1  2:58 2:58:
C i0 C jR
dt DF mi DF mi
For one-stage discontinuous diafiltration (or batch ultra-  2
2
0
dC iR C iR k i ln C igel =C iR C iR k i ln C igel =C iR 2
filtration): use equation (6.4.101)

C igel
) ln C igel =C iR 1 ) C iRjoptimum ;
C iR C i0 VCRRi ; 0 1 e

10 DF mi
VCR V f 0 =V f R @ A 20: t opt
: 6:4:119
0:5 C igel =e k i lne

For the protein,


A few more aspects about the flux in UF should be remem-
Ri 1 : C iR C i0 201 ) C iR =C i0 20; bered. It depends on the temperature as well as other
474 Open separators: bulk flow parallel to force and CSTSs

Feed Concentrate/residue
Pf xi2
Wt2, xi2
Wtf , xif
Membrane

Pp xi1 Permeate
Wt1, xi1

Figure 6.4.10. Gas permeation device where both the feed side and the permeate side are completely mixed.

factors that influence k i besides C igel . Further, continuous may therefore be replaced by
UF systems that have a flow pattern over the membrane
 
surface different from that of Figures 6.3.26(b) and 6.4.9(a) QAm

x A1 W t1 Am P f x A2  P p x A1 : 6:4:122a
and (b) will require additional considerations. m

The corresponding equation for species B is


6.4.2.2 Membrane gas permeation in a CSTS
 
In a membrane gas permeator, both the feed side and the QBm

x B1 W t1 1  x A1 W t1 Am P f x B2  P p x B1
permeate side may be well-stirred. Sometimes this is m
identified as the complete mixing case. As shown in Figure  
6.4.10, a binary feed gas mixture of species A and B (mole QBm

Am P f 1  x A2  P p 1  x A1 : 6:4:122b
fraction of A, xAf) enters the feed side at a pressure Pf and m
molar flow rate Wtf. The feed side is assumed to be well-
mixed, therefore the gas composition of the exiting stream (Note: Using equation (6.4.122a) in mass balance
(j 2) xA2 (reject mole fraction of A) is present every- (6.4.120b) for species A, we obtain the equation corres-
where in the feed side of the permeator. The reject gas ponding to (6.2.44) for a continuous stirred tank separator
stream exits at a molar flow rate Wt2. The permeate side having one membrane surface.)
may be similarly well-mixed; the permeate molar flow rate In the equations identified above, the following vari-
is Wt1, and the mole fraction of the permeate stream ables appear: QAm =m , QBm =m , P f , P p , x Af , x A1 , x A2 ,
leaving the separator, xA1, is also present throughout the W t1 , W tf and Am . For the solution of any problem in such a
permeate side of the permeator of membrane area Am. system, a few variables are specified. The two common
Assume that the pressures on both sides of the permeator situations encountered are:
are uniform.
(1) QAm =m , QBm =m , P f , P p , x Af , x A2 and W tf are
An overall molar balance and a species A balance lead
specified; one needs to know x A1 , W t1 and Am .
to
(2) QAm =m , QBm =m , P f , P p , x Af , W tf and are speci-
W tf W t1 W t2 ; 6:4:120a fied; one needs to know x A1 , x A2 and Am .

W tf x Af W t1 x A1 W t2 x A2 ; 6:4:120b The first situation, identified with the Huckins and


x Af x A1 1  x A2 , 6:4:120c Kammermeyer (1953a,b) solution, is treated as follows.
Divide equation (6.4.122a) by equation (6.4.122b) to
where is the membrane stage cut (see equation (2.2.10a); obtain
(Wt1/Wtf)). In a well-stirred permeator, the gas com-
 
position and pressure on the feed side, xA2 and Pf, are x A1 QAm x A2  x A1
, 6:4:123
uniform everywhere along the length of the membrane; 1  x A1 QBm 1  x A2  1  x A1
similarly on the permeate side for xA1 and Pp. The local
gas permeation equation over a small membrane area where QAm =QBm AB , the ideal separation factor and

dAm, P p =P f , the pressure ratio. We have come across such


  an equation earlier (equation (6.3.200)). The solution of
QAm

x A1 dW t1 dAm P f x A2  P p x A1 , 6:4:121 the quadratic equation in the unknown quantity xA1 is


m
obtained as follows:
Problems 475

    s
  


x A2 1 x A2 1 2 4 AB AB  1 x A2
AB 1 1  1 
AB  1 

x A1 : 6:4:124
2AB  1

From equation (6.4.120c), can be easily determined;


therefore W t2 and W t1 are known, since W tf and are A molar balance on component i is given by
known (from the definition of ). The membrane area now W tf x if W t1 x i1 W t2 x i2 ,
becomes available from equation (6.4.122a) since all other
quantities are known. which leads to
The second case was treated by Weller and Steiner  
Qi
(1950a,b) and led to the following set of three algebraic W tf x if  W t2 x i2 W t1 x i1 Am P f x i2  P p x i1 ,
m
equations (after significant manipulations) for the three i 1, 2, . . ., n: 6:4:130
unknowns x A1 , x A2 and Am :
 
Adding all such equations for n species leads to
x A1 AB  P f  P p = = AB  1 ; 6:4:125   "
n n
#
Qrm X X
  W tf  W t2 W t1 Am P f iR x i2  iR x i1 ,


m
Pp Pp AB  Pf  P p = i1 i1
x A2 x A1 ; 6:4:131
Pf Pf AB  1
6:4:126 which can be rearranged to yield
 

"
# 1  W t2 =W tf W t1 =W tf
1  P p AB  Pf  P p =   " #
n n
x Af Qrm Am P f X X
Pf AB  1 iR x i2  iR x i1 ,
m W tf i1 i1
x A1 1  x A2 , 6:4:127
6:4:132
where
  where
W tf
QAm =m
, W t1 =W tf , AB : iR Qim =Qrm : 6:4:133
QAm =Am QBm =m
6:4:128 One can eliminate W t2 from equations (6.4.130) and
(6.4.131) to obtain (n  1) equations for i 1, 2. . ., (n  1):
From equation (6.4.127), can be obtained, since all "  #"
other quantities are available. From the definition (6.4.128) Qrm Am P f
x if  x i2 iR x i2  x i1
of , since W tf , and (QAm/) are known, a value of Am is m W tf
obtained. Substituting for into equation (6.4.125) yields ( )#
Xn Xn
xA1. Substituting xA1 now in equation (6.4.126) will yield x i2 iR x i2  iR x i1 : 6:4:134
xA2. i1 i1

For a multicomponent gas mixture containing species Further, x i1 is also obtained from these two equations,
i 1, 2,. . ., n, a numerical solution is necessary. Consider (6.4.130) and (6.4.131), as
first the development of the basic governing equations " ,( )#
Xn n
X
(Sengupta and Sirkar, 1995). Let the permeance of the
x i1 ir x i2  x i1 iR x i2  iR x i1 ,
species i through the membrane of thickness m and a i i
reference species r be, respectively, (Qim/ m) and (Qrm/
i 1, 2, . . ., n  1: 6:4:135
m). Usually, r is one of the species selected out of the
n species, i 1, 2, . . ., n. The ideal selectivity, ir , is Equations (6.4.132), (6.4.133) and (6.4.134) are (1 (n 1)
defined as (n 1) 2n 1) algebraic equations which have to be solved
simultaneously to obtain (n 1) xi1 variables, (n 1) xi2
ir Qim =Qrm : 6:4:129
variables and stage cut for known values of (n 1) xif
variables, different iR variables, Qrm =m Am P f =W tf  and .

Problems
6.1.1 Calculate the volumetric flow rate of air through a packed bed of spherical particles generated by a pressure drop
of 2000 Pa. The air has some other constituents; however, at the temperature of operation, 25  C and essentially
atmospheric pressure, the density and viscosity are as follows: 1.75  104 g/cm3 and 1.9  104 poise (g/cm-s).
476 Open separators: bulk flow parallel to force and CSTSs

The particles in the packed bed have a diameter of 2 mm; the packed bed is 1.5 m long. The fractional void volume
of the packed bed is 0.38. (Ans. 125.4 liter/min.)
6.1.2 Determine the magnitude of the electroosmotic velocity, vEOF, generated in a capillary (Figure 6.1.5) under the
following conditions: electrical force field E 1000 volt/m (newton/coulomb); zeta potential 0.1 volt; fluid
medium is water, whose dielectric constant d r (relative dielectric constant of water)  permittivity of
vacuum (0); 78.3 ( r)  8.854  1012 coulomb/volt-m (0); temperature 25  C.
(Ans. 7.8  103 cm/s.)

6.1.3 A thin film of water at room temperature is present in a shallow rectangular pan. If the two side walls 10 cm
apart differ in temperature by 10 K and the value of (d12/dT) for water is 0.15 (dyne/cm-K), develop
an estimate of the maximum velocity for a liquid height of 0.2 mm. Calculate the value of the Bond number
and indicate which force dominates. You are given: viscosity of water 1 centipoise; surface tension of water
12 72 dyne/cm.

6.1.4 To image the surface of a membrane used for separation, scanning electrochemical microscopy (SECM)
(Bard and Mirkin, 2001) may be used. The principle consists of moving a tip, which is a conductive disk
electrode of radius a, at a given rate over the substrate (here, the membrane) and measuring the variation
in the electrochemical response of the tip (the sensor) as it moves over the substrate. The tip is held at a
distance d above the substrate as it is moved at a rate of 10 m/s through the electrolytic solution between
the substrate and the tip. The SECM tip of radius a is embedded in an insulating glass sheath of radius
r gl 10a. Draw a diagram of the setup. What is the velocity profile in the gap between the tip and the
substrate if the tip velocity is v0 cm/s? Make an estimate of the fluid velocity at a distance d/2 from the
substrate for d 17 m and v0 10 m/s. Comment on deviations of the velocity profile due to the real-life
situation.
6.1.5 You have to develop a means of particle elutriation in microgravity or zero-gravity situations. In extended-
duration manned missions in space, there may be need for particle elutriation, and an external force is needed
to replace gravity (when there is zero gravity) or augment low gravity (e.g. on the surface of the moon (0.165g)
or on Mars (0.37g)). The particles do not/cannot have electrical charge; however, other techniques of particle
modification and other external forces may be utilized. Suggest a scheme by which particles may be fraction-
ated; your scheme must be easy to implement. Electrical power is available.
6.3.1 Consider capillary electrophoresis, where there is pressure driven flow of the eluent on top of the electro-
osmotic flow. This allows greater control of the flow, since controlling the extent of electroosmotic flow requires
either changing the capillary surface or the electrolytic solution or both.
(1) Identify the expression for a species velocity in such a configuration. Specify the exact expression for each
term. Assume that both bulk flows, Poiseuille flow and electroosmotic flow, are additive.
(2) Pressure driven flow is controllable. However, it will introduce another complication. Identify it and place
it in the context of other complexities in capillary electrophoresis.
(3) Write down the equation corresponding to equations (6.3.11a,b) for the present case. Speculate the effect
on separation if the pressure driven flow opposes the electroosmotic flow.
6.3.2 Consider two somewhat similar ionic species (e.g. dansyl amino acids, for example serine (1) and alanine (2)) to
be separated using capillary electrophoresis. Deliberate on the factors that may lead to increased resolution if
1  2 is small. Comment on the role of the electroosmotic flow in increasing the resolution Rs and affecting
the retention time.
6.3.3 Calculate the resolution that may be achieved between Na and K if they are subjected to an electrical field of
300 volt/cm in capillary electrophoresis, where the electroosmotic velocity may be neglected due to an
appropriate coating on the capillary surface. The capillary length is 100 cm.

6.3.4 What will be the observed solute mobility, B,eff , of a weak base B which will ionize partially in an aqueous
solution as
K d1
B H BH
in an applied electrical field in capillary electrophoresis? Develop two expressions, one in terms of C H and Kd1
and the other in terms of pH and pK1. You are given that the dissociation constant of the base is Kd1.
Problems 477

Air in
+
VOC

L/2
Z
VOC-rich Z=0 VOC-lean
air air

L/2 L/2

X-ray
generator

Figure 6.P.1.

6.3.5 Enantiomeric compounds R and S may be separated by capillary electrophoresis by complexing them with a
chiral selector compound B. Here the equilibrium binding constants for the R and S enantiomers are,
respectively, KR and KS (in a molar concentration based expression). Develop expressions for the effective
mobility of the R and S species, assuming that the complexes continue to have the same charge as the
uncomplexed compounds. Obtain an expression for the mobility difference between the effective mobilities
of the R and S species. What contributes to a nonzero value of this mobility difference?

6.3.6 In isoelectric focusing studied in Section 4.2.2.1 for protein separation in a closed vessel with no bulk flow, the
focusing point/location in the cell corresponded to no electrical force, since pH pI at that location. Suppose
you have a channel or a tube through which a buffer solution is flowing at velocity vz. There are two electrodes
at the two ends of this channel, creating an electrical field E(z) which varies with the distance z, since we have
created an electrical conductivity gradient along z by a selective and continuous decrease in the buffer ion
concentration (by some special means, say a dialysis membrane, which allows diffusion of buffer ions out of the
solution). If you now inject a sample of two proteins (BSA and hemoglobin, both negatively charged at
the buffer pH 8.7) along with the buffer near the anode to the left, as the buffer moves to the right toward
the cathode end, is it possible to focus the two proteins to two different locations in the tube? Write down the
flux expression Niz for a protein i at any location. What would be the value of the protein flux at the location
where it is focused? If the total moles of protein injected in the sample is given by mi, obtain a solution for Ci(z)
in terms of E(z) and other relevant quantities. Show why two proteins are focused at two different locations.
This technique is part of a family of gradient focusing techniques.

6.3.7 It has been observed that volatile organic compounds (VOCs), such as toluene, ethanol, etc., are selectively
ionized to positively charged ions when exposed to soft x-rays and x-rays in the presence of N2, O2, etc.,
constituting air. A VOC separator has been designed, as shown in Figure 6.P.1.
Air-containing VOCs coming in is split into two streams, one moving toward the negative electrode in the
gas flow duct and the other moving in the opposite direction, toward the positive electrode. Both streams have
the same velocity, jvz j. The first-order reaction rate constant based volumetric rates of formation and disappear-
ance of the VOC ion are, respectively, k 1 C i and k 2 C i , where Ci is the concentration of the VOC species and
C i is the concentration of the VOC ion. The electrical field strength is E volt/cm; the mobility of the VOC ion is
mi
. Write down the equations of change for the molar concentrations of the VOC ions and the neutral VOC
molecules in the z-coordinate for the gas stream flowing toward the negative electrode. Assume no radial or
circumferential gradients; the gas is very dilute in VOCs. Write down the boundary conditions.

6.3.8 Consider the separation configuration shown in Figure 6.P.2 for separating larger spherical microparticles
(~20 m) from smaller spherical microparticles (~10 m) flowing in a liquid suspension in a rectangular micro-
channel. As shown in part (a) of the figure, there are two inlet microchannels through which liquid enters, one
(the shaded stream) contains a mixture of particles, the other does not. The two microchannels, each of depth
around 50 m, converge into a straight pinched segment of length 100 m. Then the outlet from this segment
diverges (as shown in part (b)) quickly into a 1 mm broad segment. The widths of these two inlet channels, as
well as the converging segment, are 50 m. The larger particles and the smaller particles in the feed inlet
478 Open separators: bulk flow parallel to force and CSTSs

Liquid with particles Broadened Detection


segment line
Pinched
segment

1 mm
Liquid without
particles

(b)

(a)

Figure 6.P.2.

channel liquid will follow distinctly different trajectories in the broadened segment. Identify the trajectories of
particles of two different sizes. Identify the forces on the particles responsible for separation. Speculate on any
other factors influencing such separation.

6.3.9 In flash vaporization using a vertical drum, the terminal drop velocity is important in determining the diameter
of the column through which the vapor rises and leaves the column. In petroleum refining, often a three-phase
mixture enters a flash drum: a vapor phase, a hydrocarbon liquid and liquid water. The three-phase flash drum
separator should allow for the separation of the liquid water from the liquid hydrocarbon separated by an
interface, as well as the disengagement of drops from the vapor leaving at the top. The lighter liquid (viscosity
, density ) leaves via a nozzle below the feed introduction zone; the heavier liquid (viscosity h , density h )
leaves via a nozzle at the bottom of the column.
(1) Calculate the settling velocity (terminal velocity) for the drops in each liquid phase. Simplify the results,
assuming that drops behave like solid spherical particles in the Stokes regime.
(2) Determine the settling time for the drops in each phase (heights of each phase in the column are hh for the
heavy phase, h for the light phase).
(3) Given the volumetric flow rates of the vapor (Qv) and the two liquid phases (Qh and Q ), determine the
criteria for satisfactory separation for all phases. Draw a diagram of the vertical separator.
6.3.10 A liquid mixture containing 60 mole % propane, 10 mole % isobutane and 30 mole % n-butane is throttled into a
flash drum at 85  C and 2500 kPa. Calculate the fraction of the feed stream leaving as a vapor if vaporliquid
equilibrium is assumed. Determine the compositions of the vapor stream and the liquid stream. Obtain Ki
values from Figures 4.1.5 and 4.1.6.

6.3.11 Many hydrocarbon mixtures contain a small amount of water. In the processing of such mixtures, three phases
are frequently encountered: a vapor phase ( j v), a hydrocarbon liquid phase ( j h) and a water phase ( j
w). Consider the case of flash separation of such a hydrocarbonwater mixture. Define

vf W tv =W tf , hl W th =W t , W t W th W tw ,
Problems 479

where W t is the total molar flow rate of the liquid-phase mixture consisting of the molar flow rates of the
hydrocarbon stream (Wth) and the water stream (Wtw).
(1) Write down the molar balance equations for the total molar flow rates and that for the ith species flow rate.
For any species i, show that

x if hl 1  vf x ih 1  h 1  vf x iw vf x iv ,

where subscript f refers to the feed stream.


(2) Using the following Ki factors, Kih ( xiv/xih) and Kiw ( xiv/xiw), show that the following relation holds:




x ih x if = hl 1  vf f1  h 1  vf K ih =K iw g vf K ih :

(3) Identify other conditions to be satisfied in any solution procedure to be developed.


6.3.12 A batch distillation pot in a pharmaceutical laboratory can nominally hold up to 110 moles of liquid solvent. It is
useful to operate with a lower liquid level to prevent entrainment of drops, around 80 moles. This vessel
is agitated to maintain a reasonably uniform liquid composition. For this reason, the lowest liquid level allowed
is 20 moles.
(a) Consider the following process. A solvent containing some nonvolatile solute was distilled down to
20 moles from 80 moles solvent (species i 1). Next, another solvent (i 2) was introduced to bring
the total solvent level to 80 moles. Distillation was carried out until the final solvent level was reduced to
20 moles. Find out how many moles of solvent i 1 were removed in this last distillation step. Determine
how many moles of solvent i 2 were removed at the same time.
(b) Suppose you now follow the Gentilcore (2002) procedure of constant-level batch distillation, when the
number of solvent moles to start with in the distillation pot is mt m1 20 moles; what is the number of
moles of solvent i 2 that will be removed if the number of moles of solvent i 1 removed stays the same
as in (a)?
Given: 12 3.

6.3.13 Obtain an expression for the molten-phase impurity mass fraction distribution as a function of the molten-
phase impurity mass fraction in the bulk melt, employing the partial mixing model in single pass zone melting.
State your assumptions.
6.3.14 For the case of a completely mixed melt, compare the purification performance of zone melting of a solid rod
with that of normal freezing of the same rod at a distance of z 9 and , where is the zone length and the rod
length L 10. You are given: is0 0:1; s ; vfr 0.2 cm/hr; Di 105 cm2/s; 0:1 cm; 0.1 cm. Note:
This value of is0 is close to that of carbon in silicon (see Table 4.1.7).

6.3.15 For the case of a partially mixed melt, compare the purification performance of zone melting of a solid rod with
that of normal freezing of the same rod at a distance of z 8, where is the zone length and the rod length L
10. You are given: is0 0:1; s ; vfr 0.2 cm/hr; Di 105 cm2/s; 0:1 cm; 0.1 cm.

6.3.16 Figure 3.3.6A illustrates the temperature vs. composition phase diagram of the naphthalene-naphthol system.
In the composition range, where -naphthol is an impurity, the melt has a concentration of -naphthol lower
than that in the solid. The starting solid mixture contains 5% by weight of -naphthol. What is the velocity
of the freezing interface needed to produce a 2 cm zone composition of overall composition 3% by weight of
-naphthol? Neglect any convection in the molten zone (Wilcox and Wilke, 1964). You are given:
is0 1:85; Di 2.9  105 cm2/s; s 1:15 g=cm3 ; 0.97 g/cm3. (Ans. vfr 1.26  102 cm/hr.)

6.3.17 The permeability characteristics of a filter cake deposited on a highly permeable filter cloth were determined by
passing plain water at 20 C and an applied pressure difference of 0.95 atmosphere. The filter cloth was 5.08 cm
in diameter; the water flux was found to be 36 cm3/min. The filter cake thickness was measured to be 1.5 cm.
Determine the value of the filter-cake resistance, Rc, and the value of water permeability, Qwm, through the
cake. Neglect the filter cloth resistance. Ans: Rc 0:32 1010 cm1 ; Qwm 4:73 x 102 cm2 -cp=s-atm:

6.3.18 You have to sterilize an aqueous solution of a heat-labile pharmaceutical compound by a membrane process.
Pseudomonas diminuta (0.3 m in diameter for our purpose) is conveniently used as a bacterial model to test
the membrane for its integrity. However, you have an opportunity to test and determine only the bubble-point
pressure of the membrane with water. Such a test yielded a value of 55 psia. What is the value of the largest pore
480 Open separators: bulk flow parallel to force and CSTSs

+ Anode

Pf

Cake

Membrane

Pp Cathode

Figure 6.P.3.

diameter of this membrane? Is it okay for sterilization? You are given: surface tension of water 72 dyne/cm
(=g/s2); conversion factor for pressure, 1 g/cm-s2 1.4504  105 psia; m 4.0; 0*.

6.3.19 In deadend cake filtration, shown in Figure 6.3.21, suppose there is an arrangement (Figure 6.P.3) in which one
can put a cathode below the membrane and an anode up in the region of the piston. (In practice, a piston is not
needed; slurry under pressure is what is used.)
(a) Speculate what will happen to the filter cake if the particles being filtered have a net negative charge.
(b) Identify a z-coordinate normal to the membrane/filter cloth pointing away from the membrane toward the
anode. Write down the expressions for forces acting on any particle being deposited in the cake. Develop
the criterion for removal/no removal of particles from the cake. Deliberate on the various resistances to
the filtration flux in the presence of the electrical field.
(c) Would you maintain the voltage between the two electrodes on a steady basis, or apply pulses of voltage?
6.3.20 Equations (6.3.138f/h) in cake filtration indicate a particular type of dependence between the volume of the
filtrate, V(t), and the time for filtration, t. We would like to find out a corresponding relation in batch deadend
ultrafiltration without any stirring (Figures 6.3.26(a) and (b)). Assume for the time being that is not
important. Recognize that
1 dV p
N sz V s J v volume flux ,
Am dt
where Vp is the volume of the permeate collected from time t 0 to t t.
(1) Obtain the overall resistance in terms of the sum of the membrane resistance and a stagnant diffusion
layer resistance.
(2) Relate the diffusion layer thickness, , the bulk concentration of the solute, C ilb , the permeate volume,
V p t , and the observed solute rejection, Robs. Assume a uniform solute concentration in the diffusion
layer, Cigel.
(3) Incorporate the result from (2) and (1) above and obtain a dependence of t varying with the square of the

2
permeate volume, i.e. V p t . Assume that the value of the stagnant diffusion layer resistance, Rc, is
^ c.
proportional to the diffusion layer thickness, , and a specific diffusion layer resistance, R

6.3.21 Saksena and Zydney (1994) have studied the protein transmission characteristics of a 100 000 MWCO ultrafiltra-
tion membrane (OMEGA 100K) of polyethersulfone using a binary mixture of bovine serum albumin (BSA)
Problems 481

(mol.wt., 67 000) and immunoglobulin (IgG) (mol.wt., 155 000) in a solution of pH 7 and an ionic strength of
0.15 M NaCl solution. The batch cell operation characteristics and BSA transport properties have already been
identified in Example 6.3.8. The transport properties of IgG are as follows: S 0.0026; Dieff 4  1015 m2/s;
kil 3.3  106 m/s.
(1) Calculate the values of the observed and true transmission coefficients, Sobs and Strue, respectively, of IgG.
(2) Determine the selectivity (~ the separation factor) of BSA over IgG for this membrane.
Assume that the presence of one protein in a dilute concentration does not interfere with the ultrafiltration of
the other protein also present in a dilute concentration. (Ans. (1) Sobs 0.0119; Strue 0.009; (2) (SobsjBSA/
SobsjIgG) 17.14.)

6.3.22 In a stirred ultrafiltration cell using a flat UF membrane, an aqueous solution of the polymer Dextran 20 was
ultrafiltered. Data were gathered at different values of the water flux, and the solute rejection was measured.
Dextran 20 is a linear polymer, and, as the solvent flux was increased, the rejection observed for Dextran 20
decreased. A plot of the solvent flux against the quantity (1Robs)/Robs in a semilogarithmic plot (logarithmic on
the abscissa for (1Robs)/Robs) yielded a straight line with a positive slope and an intercept of 0.05 on the
abscissa.
(1) Provide an estimate of the true sieving coefficient for this Dextran 20 macromolecule of mean molecular
weight M w 20 000 and the membrane used. (Ans. Strue 0.0476.)
(2) For the same Dextran polymer and a stirred cell, water flux data were gathered at 1 atmospheric pressure
gauge and 25 C for different feed concentrations Cilb of Dextran; simultaneously, the permeate Dextran
concentrations, Cip, were also measured:

water
flux
gallons=ft2 -dayGFD 24 14:5 7:5
C ilb  C ip M w wt% 0:6 2:4 6:5

Plot these data appropriately and determine the value of Cigel corresponding to zero flux. Speculate about the
relation between Cigel and zero flux.

6.3.23 Consider ultrafiltration based separation of a protein in the configuration of Figures 6.3.26(a) and (b). Suppose
now another membrane, identical to the first membrane, is located below the first membrane at some distance
from it. Permeate generated from the first membrane having a protein concentration of C ip1 becomes the feed
to the second UF membrane. The protein concentration in the permeate from the second membrane is C ip2 .
The protein concentration in the feed to the first membrane is C if 1 . Assume a pseudosteady state; the
macrosolute observed rejection value R1 for the first membrane may be assumed to be valid for the second
membrane as well, in relation to its feed and permeate.
(a) Determine the value of the permeate concentration, C ip2 , as a function of R1 and C if 1 .
(b) What is the overall solute rejection, Rov, for the permeate C ip2 with respect to C if 1 ?
(c) If instead of two membranes, we have three membranes in an identical configuration, express Rov in terms
of R1. Assume R1 R2 R3.

6.3.24 Employ the principles illustrated in Problem 6.3.23 to calculate the value of the rejection of the protein
myoglobin (mol. wt., 17 566) at a pH of 4.35 through a regenerated cellulose membrane having a molecular
weight cut off of 30 000. The pI of myoglobin is 7.3. At a pH of 4.35, the myoglobin molecules are charged, have
larger hydrodynamic radius and therefore undergo considerable rejection in the regenerated cellulose mem-
brane (R 0.96) (Feins, 2004). Calculate the value of Rov, the overall myoglobin rejection, if two regenerated
cellulose membranes are stacked together. Obtain the corresponding value for a stack of three such mem-
branes. (Ans. two membranes ) Rov 0.9984; three membranes ) Rov 0.999936.)

6.3.25 For reverse osmosis desalination of seawater at a high P 102 atm using a cellulose acetate membrane, the
following information is available for a solutiondiffusion model based analysis (employ Example 6.3.10): A
8.03  107 gmol H2O/cm2-s-atm; (Cif) bCif 45.7 atm; (Dimim/m)salt K2 1.774  105 cm/s;
V s 18:05 cm3 =gmol. Determine the value of the salt rejection, Rsalt. (Ans. Rsalt 0.979.)
6.3.26 The physical picture of a reverse osmosis membrane illustrated in Figure 6.3.29b is sometimes identified as the
sieve transport model: one section of the membrane (fractional area b) has pores that are completely open, i.e.
nonrejecting, while the rest of the membrane (fractional area (1  b)) has complete rejection of the solute.
However, variations in this model arise depending on the extent of solute mixing downstream of the membrane
482 Open separators: bulk flow parallel to force and CSTSs

from different sections and the driving force for solvent transport. Consider the following case: the driving force
for solvent transport through both regions of membrane is (P  ). Employ the notation of Example 6.3.9.
Write down the expression for Jvz and Niz. Obtain an expression for the solute rejection Ri. What would be the
value of i in such a model?

6.3.27 To meet the requirement of a particular salt concentration in the permeate, the quantity Ri,reqd was defined by
(6.3.173) based on the final salt concentration, Cile. Define the Ri,reqd instead based on an average feed salt
concentration,

C ilavg C ili C ile =2, as Ri, reqd 1  C ip =C ilavg :

(1) Develop an expression for Ri,reqd (corresponding to (6.3.175)) in terms of the fractional water recovery, re.
(2) Calculate the values of Ri,reqd for a brackish water feed containing 2000 ppm salt for the following values of
re: 0, 0.75, 0.85, 0.95. The required Cip corresponds to 500 ppm.
(3) Calculate the values of Ri,reqd for a seawater feed containing 35 000 ppm salt for the following values of re:
0, 0.25, 0.3 and 0.5. The required Cip corresponds to 500 ppm salt. Comment on the requirement for
pressure as the fractional water recovery is increased.
(4) Calculate the final feed salt concentration, Cile, in ppm, corresponding to the recoveries specified in (3)
above.
6.3.28 (1) To remove CO2 from flue gases being released from a coal-burning power plant, a membrane process is
being explored. What should be the minimum selectivity of the polymeric membrane for CO2 over N2 if
the flue gas may be assumed to be 10% CO2, the rest being essentially N2? The permeate must have at least
90% CO2. (Ans. 81.)
(2) Consider an undesirable scenario where the atmosphere has 400 ppmv (parts per million by volume) of
CO2 and the rest may be assumed to be N2 from the perspective of membrane separation. How high
should the ideal membrane selectivity be to recover a 90% CO2 stream in the permeate for the purpose of
sequestering the CO2? (Ans. 22500.)

6.3.29 In a small cell of the type shown in Figure 6.3.32, feed gas mixture containing 50% helium and 50% methane
(mole %) is introduced at 17 atm. The helium permeability coefficient through the nonporous membrane has
the value 240 (units) and that for methane is 3 (units). The permeate pressure is quite low. Calculate the
composition of the permeate side gas emerging at the beginning of the process. (Ans. xHep 0.987.)

6.3.30 Consider the composite PRISM-type membrane illustrated in Example 6.3.14. For such a polysulfone
membrane having a thin coating of silicone rubber, the values of the permeability coefficients through layers
A and B for a H2CO mixture are:

QH2 A 5:2  108 scc-cm=cm2 -s-cm Hg;


QH2 B 1:2  109 scc-cm=cm2 -s-cm Hg;
QCOA 2:5  108 scc-cm-cm2 -s-cm Hg;
QCOB 3  1011 scc-cm=cm2 -s-cm Hg;
2  106 :

Calculate the values of QH 2 m = and H2 CO for the following cases:
(a) A 0.1 m; B 0.05 m;
(b) A 0.1 m; B 0.25 m;
(c) A 5 m; B 0.05 m;
(d) A 5 m; B 0.25 m.
Comment on the effect of the thickness, A, of the coating as well as the substrate skin thickness, B. (Ans. (a)
230  106; 38. (b) 48  106; 39. (c) 70  106; 13. (d) 33  106; 28.)

6.4.1 Define the crystal mass density distribution function per unit solution volume between the crystal dimensions


rp and to be M r p , s v r 3p n r p dr p .
rp

(a) Obtain an expression for M r p , =M T for an MSMPR crystallizer whose n(rp) is given by expression (6.4.5).

(b) Obtain an expression for the density function of M r p , =M T .


(c) Locate the value of rp where this density function has a maximum.
Problems 483

6.4.2 In Example 6.4.1, analytical expressions for the distribution functions r


p x and Ap x were derived for an

MSMPR crystallizer. Develop the density functions r p f x and Ap f x of these two distribution functions in
terms of x ( rp/G tres). Determine the values of x where these density functions may have a maximum.
Suggestion: Follow the approach of Example 6.4.2.
6.4.3 For a particular system, the dependence of the nucleation rate Bo on the supersaturation has a kinetic order p.
The dependence of the growth rate on the supersaturation has a kinetic order of growth of h (instead of 1).
From an MSMPR crystallizer, you have an option of generating data such that you can vary tres for a fixed MT.
Express the ratio of the two growth rates Ga and Gb for two values of t res ja and t res jb . Similarly, obtain a value of

the ratio noa =nob in terms of t res ja and t res jb . What would be the slope of a plot of log no vs. log G?

6.4.4 A batch cooling crystallizer is to be used to crystallize a salt. The feed solution at 60  C is to be cooled after seeding
the crystallizer with seed crystals of size r ps 0:075 mm; these crystals are expected to grow up to 0.65 mm. The
solubility change with respect to temperature in the temperature range of interest is 0.0053 g/cm3-K. The seed
crystal suspension density is 244  106 g/cm3. The solid density, s, is 3.6 g/cm3. The supersaturation is around
0.01 g/cm3. The diffusion-controlled crystal growth rate mass-transfer coefficient has a value of 1.15  103 cm/s
in the agitated crystallizer. Determine the expression for the cooling curve for this problem. The crystals may be
assumed spherical for the purpose of your calculations; in reality, they have a different shape.
6.4.5 Consider an unseeded batch crystallizer where all crystals are generated from nuclei via growth at a constant
growth rate, G. The governing equation is (6.4.51), where we assume that the supersaturation level is constant
with time. The size of the nuclei is r pn , the volume based shape factor for all nuclei/crystal size is v and Bo is
the nuclei generation rate per unit volume.
(1) Obtain the following expressions for AT:
t
k nu i  isat p s r 2p dt
AT ,
o v s r 3pn

(where r 2p r 2p , r 3pn r 3pn and s is the constant surface-area based shape factor). Show that this is
equivalent to
!
k nu i  isat p s r 3p
AT 1 :
3 v s G r 3pn

(2) Using the expression for growth in equation (6.4.51) for one crystal, show that

k gr s i  isat p1
G :
3 s v

(3) Employing these results in equation (6.4.51) for a constant supersaturation level, obtain the following
expression for the cooling curve:
"( !)4 #
k nu i  isat p r pn Gt
T 0  Tt 1 1 ,
4d isat =dT G r pn

where the temperature of the solution at t 0 is T0 and that at time t is T(t).


6.4.6 A mixer for solvent extraction is 61 cm in diameter and height. It has a six-bladed turbine impeller of 30.48 cm
diameter. The mixer is a regular cylindrical baffled vessel. Batch extraction of an organic pharmaceutical
compound is to be carried out from its dilute aqueous solution into p-xylene. The ratio of its molar concen-
tration in p-xylene to that in water is 20. The ratio of the organic-phase volume to the aqueous-phase volume in
the charge to the vessel is 0.2. The impeller rotates at 200 rpm. The temperature is 20  C.
(1) Calculate the time needed to achieve 90% of the equilibrium extraction.
(2) If the impeller was rotated at 100 rpm without changing the phase being dispersed, what would be the
time needed to achieve 90% extraction?
You are given: aqueous solution density 0.985 g/cm3; density of p-xylene 0.861 g/cm3; viscosity of
aqueous solution 0.96 cp; viscosity of p-xylene 0.60 cp; diffusion coefficient of organic compound in
484 Open separators: bulk flow parallel to force and CSTSs

Table 6.P.1.

Ri rejection %

Membrane Inulin (5000) Albumin (67000) Apoferitin (480000)

UM20 (MWCO 20000) 5 95 99


XM 100A (MWCO 100000) 1 35 98

water 1.2  105 cm2/s; diffusion coefficient of organic compound in p-xylene 1.6  105 cm2/s;
interfacial tension 12 21 dyne/cm. (Ans. (1) 0.96 s; (2) 3.96 s.)
6.4.7 Batch ultrafiltration is carried out in a well-stirred membrane cell using 10 liter of an aqueous buffer solution
containing 1 wt% inulin, 1 wt% albumin and 1 wt% apoferitin until the retentate volume is 1 liter. The mem-
brane is UM-20, having a MWCO of 20 000.
(1) Obtain the concentrations of the three proteins in the permeate and the retentate in wt%.
(2) Now add 9 liter of buffer to the retentate and carry out batch ultrafiltration using the UF membrane XM-
100A, having a MWCO of 100 000. The retentate volume obtained after UF is 1 liter. Obtain the concen-
tration of the three proteins in the retentate in wt%. (Ans. (1) retentate: inulin, 1.123%; albumin, 8.91%,
apoferitin, 9.97%; permeate: inulin 0.996%; albumin, 0.12%; apoferitin, 0.025%. (2) inulin, 0.115%; albu-
min, 1.09%; apoferitin, 9.32%.)
You are given the data in Table 6.P.1.
6.4.8 The problem involves purification of a monoclonal antibody species 1 present in 100 liter of a solution which
contains another protein species 2 as well as a small molecular weight impurity, 3. Species 1, a monoclonal
antibody, has a molecular weight of 450,000; protein 2 is albumin of molecular weight 67,000; impurity 3 has a
molecular weight of 250 dalton. The initial feed solution concentrations are: Protein 1, C10 g/liter; Protein 2,
C20 g/liter; low molecular weight impurity 3, C30 g/liter. To reduce the small molecular weight impurity level as
well as the level of albumin substantially from the solution of the monoclonal antibody, the following two-step
process has to be implemented:
(1) Continuous diafiltration where the buffer volume added is 500 liter.
Determine the concentrations of species 1 and species 2 and the low molecular weight impurity in the
retentate in terms of their original concentrations for PM 30 UF membrane (MWCO, 30,000). Given:
R1 1.0; R2 0.92; R3 0.0.
(2) We have to now reduce substantially the concentration of albumin from the solution resulting from step
(1) above with the monoclonal antibody being our product. Employ continuous diafiltration again using
XM100A membrane with the buffer volume added being 500 liter. Determine the concentrations of protein
1 and protein 2 and the low molecular weight impurity 3 in the final retentate in terms of their original
concentrations C10, C20. and C30 before step (1). Given: MWCO of XM100A membrane: 100,000; R1 0.96;
R2 0.45; R3 0.0.
7

Separation in bulk flow of feed-containing


phase perpendicular to the direction
of the force

Separations in which the feed phase or feed-containing of flow in a packed chromatographic column is studied in
phase has a bulk motion parallel to the direction of the Section 7.1.6 in what is called counteracting chromato-
force causing separation have been studied in Chapter 6. In graphic processes.
many separation techniques/processes/operations, the Many of the membrane separation processes con-
feed phase or feed-containing phase has a bulk motion sidered in the preceding six chapters produce a permeate
perpendicular to the direction of the driving force. In a phase or fraction from the feed stream. The permeate
number of situations, the feed is introduced in small phase/fraction is generally miscible with the feed phase/
amounts in a carrier fluid whose bulk motion is perpen- fraction from which it remains separated by the mem-
dicular to the force direction. Such separations will be brane; the residual fraction of the feed stream is either
studied in this chapter. called the reject or the concentrate. Such processes are
For separations based on distribution of species considered in Section 7.2.1 under conditions where the
between two phases in equilibrium, we study in Section permeate stream generated by the force present in the
7.1 primarily those two-phase systems where the second system flows parallel to the direction of the force, while
phase (e.g. adsorbent particles in a packed adsorbent the feed/concentrate/reject bulk flow direction is perpen-
bed) is stationary; the first phase, which is more often dicular to that of the force. Such a flow pattern is
the feed solution/mixture, moves perpendicular to the commonly termed crossflow. The particular membrane
direction of chemical potential driving force between techniques studied in order are: gas permeation (Figure
the two immiscible phases (Figure 7.0.1(a)). This first 7.0.1(e)); reverse osmosis (Figure 7.0.1(f)); ultrafiltration
phase (the mobile phase) bulk motion is generally in (Figure 7.0.1(g)); microfiltration (Figure 7.0.1(g)); rotary
one direction. The benefits of such bulk motion in terms vacuum filtration (7.0.1(h)). In the last example, the mem-
of extreme purification achievable in two-component brane/filter moves instead of the liquid feed. In Section
systems and multicomponent separation capability will 7.2.2, we consider briefly the process of granular filtration,
be illustrated. In the cyclic processes studied next, the in which bulk flow of the feed liquid takes place through a
direction of motion of the mobile phase is periodically loosely packed bed of granular filtration media, where
reversed; the direction of force is also reversed, except it most of the forces, except that causing inertial impaction
remains perpendicular to the bulk-phase flow direction (see Section 6.3.1.4), are operating perpendicular to the
(Figures 7.0.1(b) and (c)). The mobile phase is sometimes direction of the bulk flow and lead to particle capture by
generated by the separation operation, as in the case of the granular media.
the blowdown phase of pressure swing adsorption (PSA). Section 7.3 covers separation operations/processes/
The elution chromatographic process considered next techniques in which an external force field is applied
involves injection of a sample to be separated into a perpendicular to the direction of bulk flow of a single-
carrier fluid flowing perpendicular to the direction of phase solution or a dispersed/particulate multiphase
the force between the fluid and the stationary adsorbent mixture (Figures 7.0.1(i)(p)). The external force fields
phase (Figure 7.0.1(d)); this and other related chromato- considered are: electrical, centrifugal, gravitational and
graphic processes are also studied in Section 7.1.5. The magnetic. Specific processes treated include, among
imposition of an electrical force parallel to the direction others, electrophoresis (Figure 7.0.1(i)), dielectrophoresis,
486 Bulk flow perpendicular to the direction of force

Pressurization and
Direction high-pressure feed Blowdown Purge
for bulk Bulk flow
Stationary Bulk directions Liquid
fluid flow
adsorbent flow sample
phase direction injection

Column
Force
Force directions Chromatographic
direction column
from fluid to
adsorbent Force First half
direction of cycle

Rest of Time
Product Purge
the cycle

(a) Packed-bed adsorption (b) Pressure-swing adsorption (PSA) (c) Packed-bed adsorption (d) Chromatographic process
desorption

Filter cake

w
Wash nozzles
Rdrum

Feed flow
direction Brine flow Force Feed flow Dry Drum qw
Force direction direction direction Vacuum
direction Force direction Vacuum

Reject gas Reject Feed Reject Bulk


Filter cake Vacuum flow
Brine discharge q

Force
Feed gas Membrane Membrane Membrane qf Filter cake
mixture Filtrate
Filter cloth
Permeate water Permeate
Permeate gas Flow

Suspension

(e) Crossflow gas permeation (f) Reverse osmosis (g) Ultrafiltration/microfiltration (h) Rotary vacuum filtration

Feed Grounded electrode Sample inlet Negatively


Electrode
Corona wires Shealth
Electrode Gas enters charged
+ Jet formation inlet Laser collector
Force n z x and droplet beam for 235U +
rpf generator
Laser
y
Upz Trajectory beam
Bulk b Fluid jet with
flow Particle Upy periodic Uranium
entering disturbance Deflection Other vaporizer
precipitator Grounded electrode plates electrode
z=L Collection
vessel

(i) Electophoresis (j) Electrostatic precipitator (k) Cell sorting by flow cytometry (l) Atomic vapor laser isotope
separation (AVLIS)

Figure 7.0.1. Bulk flow of feed-containing phase/region perpendicular to the direction of force: examples.

electrostatic precipitation (Figure 7.0.1(j)), flow cytometry capabilities are achieved, as in field flow fractionation
(Figure 7.0.1(k)), laser isotope separation (Figure 7.0.1(l)), (Figure 7.0.1(o)). Column 7 in each of Tables 17 in the
particle separation in centrifuges/cyclones, etc. (Figure introductory chapter illustrates the large number of separ-
7.0.1(m)), gas separation by a separation nozzle process, ation operations/processes/techniques studied here via a
gravity settling (Figure 7.0.1(n)), high-gradient magnetic few distinguishing characteristics under three broad cat-
separation (7.0.1(p)). When any one of the external force egories: phase equilibrium based separations; membrane
fields is coupled with the velocity field of feed bulk flow based separations; separations in external and other force
perpendicular to the external force field, unique separation fields.
7.1 Force ri in phase equilibrium: fixed-bed processes 487

There is one basic phenomenon in the three types of 7.1 Chemical potential gradient based force in
separations identified above and described in detail in phase equilibrium: fixed-bed processes
this chapter. It arises from the force direction being
All separations which involve the distribution equilibrium
perpendicular to the direction of the bulk flow of the
of a solute or solutes between a fluid phase and a solid
feed-containing phase (phases in one stream in the
phase are implemented commonly with a fixed bed of
case of particle separation). Focus on Figures 7.0.1(i)
particles in a vessel. Although there are separation pro-
and 7.0.1(m). In a given period of time, a feed species
cesses where the solid particles are moved through the
or a protein molecule, or a cell or a particle, moves a
separation device, they have somewhat limited use due to
certain distance away from the axial line of its introduc-
the difficulty of moving solid particles. The mobile fluid
tion, as defined by its trajectory. The latter is shaped by
phase is either liquid or gaseous or supercritical. The solid
the magnitude of its species-specific/particle-specific vel-
phase may provide surface adsorption/desorption, ion
ocity perpendicular to the bulk flow and the magnitude
exchange, or partitioning (e.g. between a liquid-phase
of its velocity toward the separator outlet, determined
coating on solid particles and the mobile phase). In leach-
by the convective bulk fluid velocity. The higher the
ing processes, the solid particles are mixtures of different
species-specific/particle-specific velocity perpendicular
species, one of which is extracted by the leaching solvent.
to the bulk flow, the further the end-point of the
The mobile phase can be a feed liquid or gaseous stream of
species/particle trajectory from the point of introduction.
binary or multicomponent nature. Alternatively, it can be a
This is a successful recipe for multicomponent separ-
strip/sweep/eluent stream to strip/desorb/extract species
ation into fractions for both molecular/macromolecular
from the solid phase. The mobile phase can be obtained
species/ions as well as particles in external force based
also by stripping of the adsorbed species from the solid
systems.
surface during bed regeneration (in gaseous feed systems).
For crossflow membrane processes (Figure 7.0.1(e)
The driving force for species i transfer from the mobile
(g)), the species which encounters the least resistance from
phase to the surface of the solid phase or from the solid
the membrane (when subjected to its chemical potential
phase surface to the mobile phase in adsorption or leach-
based driving force across the membrane) will have the
ing processes is simply ri. Since the total pressures of
highest velocity of movement through the membrane.
both mobile and solid phases are equal (except in ion
Correspondingly, its rate of disappearance from the feed
exchange resins), only r0i and the concentration gradi-
stream will be highest, and its concentration in the bulk
ent component of ri are important. For a mobile gas
flow of the reject stream will be lowest. Current practice in
phase, the partial pressure gradient is the relevant quan-
membrane separation processes does not exploit the dif-
tity. In ion exchange systems, the driving force is rel
i .
ferences in individual species velocities through the mem-
The systems considered here have, in the carrier fluid,
brane for multicomponent separation.
at least one component, which is to be preferentially
In Figures 7.0.1(e)(g), 7.0.1(i)(l) and 7.0.1(m)(p),
adsorbed onto or desorbed from particles in simple fixed-
solutes/macromolecules/particles moving in the direc-
bed processes. The number of components in leaching is
tion of the force are removed continuously from the
at least three: leaching solvent and two components in
separator by removing the mobile fluid phase associated
solid phase, one of which is to be preferentially extracted.
with them. (In Figure 7.0.1(h), the membrane/filter
All ion exchange systems have more than two species if we
phase containing the particles is removed.) However,
identify each ion as a separate species. Finally, chromato-
in the fixed-bed processes/techniques of Figures 7.0.1
graphic processes are concerned with true multicompo-
(a)(d), when solute molecules move into a particle in a
nent systems to separate a number of species in the feed
packed bed, the stationary phase, these molecules cannot
fluid sample from one another in the product stream, the
be continuously moved out of the separator via a fluid
carrier fluid.
stream as in Figures 7.0.1(e)(p). The higher the driving
force on the solute molecules toward the adsorbent par-
ticle in the packed bed and the higher their affinity, the
7.1.1 Fixed-bed adsorption/desorption processes
greater is their extent of disappearance from the feed
stream. It is as if such molecules penetrate deeper into A simplified analysis of either adsorption onto or desorp-
the stationary phase of particles due to their higher tion from adsorbent particles in a fixed bed is carried out
species-specific migration velocity perpendicular to the here. The mobile phase may be liquid or gaseous. The
bulk fluid velocity. Correspondingly, when a desorption separation objective may be purification of the carrier fluid
process is initiated, the slower will be the emergence or recovery of the species (one or more) present in the
of these molecules from the stationary phase into the carrier fluid. The basis of separation is preferential adsorp-
moving fluid used to desorb. Multicomponent separation tion/desorption or partitioning between the carrier fluid
capability is an adjunct to such a phenomenon, as we and the adsorbent surface phase. (A list of common
will see in Section 7.1.5. adsorbents and their uses is given in Table 7.1.1.) The bulk
488 Bulk flow perpendicular to the direction of force

Table 7.1.1. Common adsorbents and their uses in fixed-bed processesa

Adsorbent Applications Comments

Activated alumina drying warm gases or air not very common


Activated carbon solvents or odors from air, indoor air pollutants, organics widely used
from vent streams, gasoline vapor recovery in
automobiles, decolorizing solutions, refining oils,
organics from water in water purification
Activated clays decolorizing petroleum products
Bauxite decolorizing petroleum products, drying gases
Carbon molecular sieve (CMS) (or production of N2 from air by faster O2 penetration relatively recent
molecular sieve carbon, MSC) through micropore
Silica gel drying of air and gases (desiccant), preferential common
adsorption of polar compounds over nonpolar ones
such as saturated hydrocarbons
Starch, cellulose, biomass solvents dehydration of ethanol and other organics relatively recent and limited use
Zeolites: a variety of crystalline dehydration of gases, dehydration of liquids (e.g. very widely used
silicates with pore diameters alcohol), n-paraffins from iso-paraffins, CO2 removal
between 0.3 and 1 nm from natural gas, desulfurization of natural gas and
other streams, fructose/glucose separation, sulfur
compounds from organics, N2/O2 separation
a
Yang (1987, 2003).

flow direction is perpendicular to the direction of the adsorbent particles suitable for adsorbing solute species i.
chemical potential driving force between the fluid and If the adsorption of the solute species imparts a color to the
the particle in the packed bed. adsorbent particles, then what we observe with time is as
follows. The color near the liquid entrance of the bed has
changed after some time. A color front is slowly moving
7.1.1.1 Fixed-bed adsorption: mobile feed liquid in
toward the liquid exit with time. As more time passes, the
axial flow
color of an increasing length of the bed changes. Finally,
The first separation operation considered in this mode is after an additional amount of time, the color of the whole
the adsorption separation of solutes between a mobile bed is changed. Suppose we had an opportunity to measure
feed liquid and a stationary bed of adsorbent particles. the concentration of species i in the effluent stream from the
The use of such an operation is widespread. Packed beds bed with time. We would find that for a long time from the
of powdered activated carbon (PAC) or granular acti- beginning there is no trace of solute species i in the effluent;
vated carbon (GAC) are commonly used to reduce the however, once the whole bed color has changed, there is
toxicity of effluents obtained from biological treatment suddenly quite a concentration of species i in the effluent.
processes for wastewater.1 Toxic organics are removed Obviously, at this time, the adsorbent bed is no longer
by adsorption on GAC or PAC. The packed bed of carbon useful. What is of interest from a process operation point
is then regenerated thermally (by steam, for example) or of view is the determination of the time at which the solute
by using a wet air oxidation process to oxidize the organ- species i broke through the adsorbent bed. Alternatively,
ics. Packed-bed adsorption from liquid feeds is also used one can determine the velocity of the color front in the bed;
on a large scale to decolorize aqueous sugar solutions knowing the bed length, we can then predict the time for
and petroleum fractions, to remove moisture from gas- breakthrough of the solute in the effluent.
oline and alcohol, and to fractionate aromatics from The solution of the problem of fixed-bed adsorption of
paraffinic hydrocarbons. A number of new adsorbents, a strongly adsorbed species i present in an inert mobile
e.g. carbon nanotubes, -complexation sorbents (such as feed liquid is at hand when the concentration C i2 of the
AgNO3/SiO2, AgNO3/clays), FeMnTi oxides, etc., are desired species in the liquid phase (moles of species i
being developed for challenging applications (Yang, per unit volume of the liquid phase) is obtained as a
2003). function of time t and spatial coordinates x, y, z. Complex-
Consider a solution of species i in a liquid feed C 0i2 ity of the fluid dynamics in a packed bed and the
entering a packed bed (Figure 7.1.1(a)) containing presence of convective dispersion require, however, con-
siderable reduction in this goal for practical purposes. In
1
the pseudo-continuum approach (Lee et al. (1977a);
Remember, these adsorbent particles are highly porous, and thus
see Section 6.2.1.1), only the axial (mean flow direction,
adsorption takes place primarily on the inside surface of the pores
(see Sections 3.3.7.6 and 3.4.1.4). z-coordinate) variation is retained, i.e. we look only for a
7.1 Force ri in phase equilibrium: fixed-bed processes 489

(a)
C 0i 2

Ci2

0
Liquid
exit
Screen

Screen
Liquid
entrance
t=t1 t2>t1 t3>t2 t4>t3
C 0i 2

Purified liquid Purified


(b) liquid

Force
direction
D C
z+z z+z
Stationary Mobile
L
z adsorbent z liquid
phase A B phase

z z
z=0 z=0
Feed Feed, C 0i 2

Packed-bed
schematic Packed-bed idealization
in pseudo-continuum
approach

Figure 7.1.1. (a) Movement of the color front in the packed adsorbent bed with time and the corresponding concentration of the coloring
species at the bed outlet; (b) packed-bed schematic and its idealization in pseudo-continuum approach for adsorption from a liquid.
(After Lightfoot et al., 1962.)

solution C i2 (z, t). This may be achieved by averaging over Here, is the void volume fraction of the packed bed2
the packed-bed cross section in the x- and y-directions. and the interstitial fluid velocity3 in z-direction is vz. The
However, to allow for species transport from the mobile effective diffusion coefficient of solute i in the liquid in
phase to the stationary (fixed-bed particles) phase the z-direction is Di, eff, z. The particle surface area per
and vice versa, it is necessary to consider each phase unit particle volume is av. The particles may be
separately without any x and y dependence, as shown in porous, with a porosity p. We will consider it later; for
Figure 7.1.1(b) (Lightfoot et al., 1962). This figure shows now, p 0. Referring to Figure 7.1.1(b), a species i mass
the cross section of only a small length, z, of the packed balance may be written in the absence of any chemical
bed. Note: C i1 z;t represents the species i concentration reaction as
per unit volume of the solid phase.
A mass balance on species i over the shell ABCD which
spans the whole cross section of the packed bed of length 2
In a capillary bundle model of the packed bed.
z may now be carried out. The total bed cross-sectional 3
From now on, we will use vz instead of vtz for the average fluid
area is Sc; the cross-sectional area for flow of fluid is Sc . velocity.
490 Bulk flow perpendicular to the direction of force

0 1
rate of accumulation in principle for appropriate initial and boundary conditions
@ of species i A imposed on the fixed bed of adsorbents. Extensive
in volume ABCD heat effects from adsorption will introduce additional
0 1 0 1
rate of inflow rate of outflow complexity.
@ of species i into A @ of species i out of A; 7:1:1 Species i is transferred from the mobile liquid phase to
volume ABCD volume ABCD an adsorbed state on the particle surface via a number of
2 3 steps: diffusion in the fluid phase around the particle,
C i2 C i1 5 diffusion across the fluidparticle interface, diffusion in
Sc z 4 1
t t liquid in the pores of the particle, adsorption on available
2 3 2 3 sites on the particle pore surface and surface diffusion, if
4 C i2 5 4 C i2 5 any (see Sections 3.1.3.2.3/4 and 3.4.2.3/4). A fundamental
Sc vz C i2 Di;eff;z Sc vz C i2 Di;eff;z :
z z approach would be to develop a species balance for such a
z zz
7:1:2 particle phase and couple it with equation (7.1.4) via an
additional mass-transfer relation for diffusion in the fluid
The mobile phase has been identified here by j 2 and the phase around the particle. Such an approach, with some
stationary phase by j 1. The assumptions are: (1) species simplifications by Rosen (1952, 1954), has been illustrated
i enters the volume ABCD by convection and diffusion at at the very end of this section.
location z; (2) species i leaves by similar mechanisms at There are a number of other solutions primarily based
location z z; (3) the particle phase (j 1) occupies a on various assumed mechanisms of mass transfer. One
volume fraction ( 1 ) of the packed-bed volume Scz. We such approach replaces the mass balance equation for
now implement a limiting process where, as z ! 0, the diffusion within the particle by simplifying assumptions.
differential equation In what is known as the linear driving force assumption,
Glueckauf (1955b) suggested that, for Dip t=r 2p > 0:1,
C i2 1 C i1 vz C i2
 
C i2
Di;eff;z 7:1:3
t t z z z
C i1 15Dip  
2 C i1 C i1 ;

7:1:5b
is obtained for C i2 . Note: C i1 is the molar concentration of t rp
species i based on particle volume only. Conventionally, 
where C i1 is in equilibrium with the bulk liquid concen-
for liquid-phase systems, vz is independent of z and so is
tration C i2 , and Dip is the effective diffusion coefficient of
Di, eff, z. Therefore
species i in the pores of particle of radius rp. A nonlinear
C i2 1 C i1 C i2 2 C i2 driving force approximation has been suggested by Ver-
vz Di;eff;z : 7:1:4
t t z z2 muelen (1953). In both models the liquid-phase resistance
around the particle is neglected.
This is the starting differential equation for almost all
Before focusing on some of the simpler solutions of
liquid-phase based fixed-bed processes for a single
such a system to develop an understanding of the single
adsorbable solute in an inert liquid under isothermal
solute separation capabilities of a fixed-bed adsorption
conditions with particles, where p 0. (Note: We could
process, we will provide a few more definitions for a
have obtained this equation directly from equation
packed adsorbent bed in general. The density of the solid
(6.2.35).) However, this equation does not take into account
material of the adsorbent particles is s . The bulk density of
the transfer of species i from liquid phase 2 to adsorbent
the adsorbent bed b is related to the void volume of the
particle phase 1; such a relation between C i1 and C i2 is
packed bed, solid material density s and the porosity of
needed before a solution of C i2 z;t may be obtained. In
the particle p via
terms of an overall mass-transfer coefficient K il based on
the liquid-phase concentration and the surface area of the b s 1 p 1 : 7:1:5c
particle (where a is the particle surface area per unit bed
volume and av is the specific surface area of the particle, i.e. This definition assumes the pores inside the particles to be
particle surface area per unit particle volume so that empty. If the bulk density is defined with respect to par-
a av 1 ), the mass-transfer rate per unit bed volume is ticles containing a fluid phase j 2 inside the pores, then

C i1 
b s 1 p 1 p 2 ; 7:1:5d
1 K il aC i2 C i2 ; 7:1:5a
t
where 2 is the fluid-phase density. Unless specified, we

where C i2 is a hypothetical mobile-phase concentration in will assume (7.1.5c) to be the valid relation for b .
equilibrium with C i1 . (We could have obtained this equa- We can now write down the differential equation for
tion also from equation (6.2.32) for the stationary phase.) If solute i in the manner of equation (7.1.3) for porous

the equilibrium relation is known, C i2 is easily expressed in adsorbent particles whose pores contain the external solu-
terms of C i1 . Thus, the solution for C i2 z;t can be obtained tion (therefore C i2 ):
7.1 Force ri in phase equilibrium: fixed-bed processes 491

C i2 C i2 C i1 vz C i2 where q 0i C i2 qi C i2 =C i2 . Note that relation (7.1.7)


p 1 1 serves the purpose of relating C i2 to C i1 , i.e. qi1 , and we
t t t z


C i2
 now have only one equation, (7.1.8), to solve to obtain
Di;eff;z : 7:1:5e C i2 z;t. This equation is sometimes called the De Vault
z z
equation (De Vault, 1943). An alternative form of equation
This equation is based on a number of assumptions, one (7.1.8) is more convenient:
being that partitioning of solute i between the external
solution and that in the pores of the adsorbent is such that C i2 vz C i2
 0: 7:1:9
b q 0i C i2 z

the partitioning coefficient is 1. This is true when the solute t
1
size is small in relation to the adsorbent pore size (by about
two orders of magnitude). An alternative form of this equa-
A key question in fixed-bed adsorption separation is: what
tion is as follows:
is the time needed for the fixed bed to become saturated if
vz C i2 it is fed continuously and steadily with a feed of constant
 
1 C i2 1 C i1
1 p concentration C 0i2 ? The feed initially introduced into the
t t z
  column (Figure 7.1.1(a)) displaces the liquid already pre-
C i2
Di;eff;z : 7:1:5f sent in the column. If the column particles did not have
z z
any species i to start with, the liquid displaced from the
When the solute size is larger (e.g. proteins) or the pore column and appearing at the outlet will be free of species i.
sizes are smaller, the partitioning of the solute between Meanwhile, species i from the feed liquid will be adsorbed
the mobile phase and the pore phase liquid (i.e. near the feed entry and, soon after, feed liquid free of
p
im C im =C i2 as in (3.3.89a)) has to be taken into species i will appear at the column outlet. As this process
account: continues, the particle surfaces will become saturated with
species i. Ultimately, all particles will lose their capacity of
vz C i2
 
1 C i2 1 C i1 adsorbing species i; the feed solution of concentration C 0i2
1 p im
t t z will appear at the column outlet. At this time, the adsorber
 
C i2 is taken off the feed line and subjected to regeneration
Di;eff;z : 7:1:5g
z z treatment so it may be used again for adsorption.
Obviously, the velocity with which the concentration
The simplest solution of a fixed-bed adsorption problem
C 0i2 moves down the column (the concentration wave vel-
is provided by isothermal equilibrium nondispersive
ocity) is less than that of the interstitial liquid. If one could
operation of the fixed bed. For single solute adsorption
ride with the wave having a concentration C 0i2 , one will
from the liquid phase to the adsorbent particles (where
only witness C 0i2 around oneself; therefore, for any wave
p 0) under isothermal conditions,
having a fixed value of specific concentration C i2 , dC i2 0.
(1) assume that the liquid-phase concentration Ci2 (z, t) in Now, C i2 is a function of the independent variables z and t;
the column everywhere is locally in equilibrium with therefore, for any small change in z and t, the change in C i2
the solid-phase concentration Ci1 (z, t), and is given by
(2) neglect the contribution of the axial diffusion and dis-    
C i2 C i2
persion term, Di;eff;z 2 C i2 =z 2 , in equation (7.1.4) dC i2 dt dz: 7:1:10
t z z t
(the plug flow assumption).
Define qi1 to be moles of species i in solid phase 1 per unit For any particular concentration C i2 , the concentration
mass of solid phase. Then C i1 may be replaced by qi1 using wave velocity must satisfy dC i2 0. Compare a reformu-
the following relation: lated (7.1.10) under such a condition,
   
C i1 q C i2 dz C i2
C i1 qi1 b =1 ; 1 b i1 ; 7:1:6 0; 7:1:11
t t t z dt z t

where b is the bulk density of the packed bed. Describe with equation (7.1.9) to obtain (De Vault, 1943)
the equilibrium relation of the first assumption now by
dz vz
vCi  : 7:1:12a
qi1 qi C i2 : dt b q 0i C i2
 
7:1:7
1

Using this and the second assumption in equation (7.1.4),
we get Here, vCi is the velocity with which a wave of concentration
C i2 travels along the column (the z-direction). For a given
q 0 C i2 C i2
 
C i2 C i2 , the above expression provides a unique relation
1 b i vz 0; 7:1:8
t z between the location z along the column and time t since
492 Bulk flow perpendicular to the direction of force

q 0i C i2 is fixed. The velocity vCi is variously called the The denominator (i.e. the coefficient determinant) in both
concentration wave velocity of species i, the migration rate expressions is zero under certain conditions. One such
of species i or the concentration front propagation velocity of condition is
species i (Sherwood et al., 1975). A more formal method of
vz dz vz
arriving at expression (7.1.12a) is given below. dz   dt )  :
b q 0i C i2 dt b q 0i C i2
This formal procedure is based on the method of char- 1 1

acteristics (Aris and Amundson, 1973). If a solution for C i2
were available, we may write any change in C i2 as 7:1:12e
    If the denominator is zero, for the derivatives to be finite
C i2 C i2
dC i2 dt dz dC i2 : 7:1:12b the numerator also has to be zero. From relation (7.1.12c),
t z z t
this implies
Now one can solve for the two derivatives of C i2 z;t using
equation (7.1.9): dC i2 0: 7:1:12f
    Result (7.1.12e) is valid if C i2 is constant along the charac-
C i2 C i2 vz
 0: teristic line traced by (7.1.12e), which is the same as
b q 0i C i2

t z z t
1 (7.1.12a).

It is clear from relation (7.1.12a) that the value of vCi for
The two expressions that result are a given C i2 will depend on the value of q 0i C i2 , i.e. on the
  nature of the equilibrium relation qi1 qi C i2 . Figure
 vz 
 0 2 3 7.1.2(a) shows two linear adsorption equilibrium isotherms
 0 
 q
41 b i i2 5  C  for two different species i 1 and i 2. As shown,

  species i 1 is more strongly adsorbed than species
 
   
C i2  dC i2 dz  i 2 and for C 12 C 22 :
 v z
 ; 7:1:12c
t z 1 2
 3  q0 1 C 12 > q0 2 C 22 : 7:1:13a
 q 0
C 

 41 b i i2 5 
  Therefore, by relation (7.1.12a)
 
  vC1 < vC2
dt dz 7:1:13b
 
  in equilibrium nondispersive operation; the species which
 0 1
    is more strongly adsorbed moves through the column
C i2  dC i2 dt 
 vz : 7:1:12d more slowly and will take longer to appear at the column
z t  1 2
 3 
 0  exit.

 41 b q i C i2 5  There is another way to visualize the migration velocity
 
  of a solute species down the column. When some extra
 
dt dz solute is added to the packed-bed section of length z

(a) (b)

le
i =1
o rab
Fav

q i1 ar
q i1 i =2 i ne
L
e
bl
ra
vo
fa
Un

Ci 2 Ci 2

Figure 7.1.2. Adsorption isotherms: (a) two linear isotherms for two species i 1, 2; (b) three isotherms, favorable, linear and
unfavorable.
7.1 Force ri in phase equilibrium: fixed-bed processes 493

which may already contain some solute (Figure 7.1.1(b)), as that in the mobile phase. For larger solute sizes, the
the following will happen. Due to a higher solute concen- partitioning of the solute between the mobile-phase and
tration in the mobile phase, more solute will be adsorbed the pore-phase liquid has to be taken into account. The
now on the adsorbent. Therefore the fraction of the extra corresponding result will be (see equation (7.1.5g))
solute added that remains in the mobile phase will be less vz
than 1, reducing the mobile-phase concentration. Suppose vCi : 7:1:13f
1 1
the small increase in mobile-phase concentration and 1 p im 1p s q0 i1 C i2

stationary-phase concentration of species i are, respect-
ively, C i2 and C i1 , then This result indicates that larger solutes (larger with respect
0 1 to the pore size) will have larger values of vCi .
fraction of solute The solute molecules of a particular species staying in
@ added that remains A Sc C i2
S C 1 Sc C i1 the mobile phase move down the column with the fluid
in the mobile phase c i2
velocity vz , whereas those that are adsorbed cannot move
1 down the column. Thus, a given species concentration
  :
1 C i1 moves down the column at an effective velocity vCi lower
1
C i2 than vz . For a given species concentration, the higher the
fraction of the species in the mobile phase, the higher its
From relation (7.1.6), this ratio is given by
speed through the column.
1 Figure 7.1.2(b) illustrates a few more adsorption equi-
C i ! 0 b 0 : 7:1:13c
lim 1 q i C i2 librium isotherms. For the isotherm identified as favorable,
it is obvious that if two concentrations C i2 j1 and C i2 j2 are
We now focus on beds with no dispersion and with a significantly apart and C i2 j2 > C i2 j1 , then
constant interstitial fluid velocity vz (used to derive equa-
q0 i2 C i2 j2 < q0 i2 C i2 j1 : 7:1:14a
tion (7.1.12a)). Due to adsorption, the probability of any
solute molecule being in the mobile phase after introduc- From (7.1.12a) and (7.1.13f),
tion into the control volume is reduced from 1 by the ratio
given above. Thus the velocity with which these molecules vC j2 > vC j1 : 7:1:14b
migrate down the column is reduced from the fluid vel- Therefore, a higher solute concentration exits the column
ocity vz by the same fraction: faster for a favorable isotherm in equilibrium nondisper-
vz sive operation. For the isotherm identified as unfavorable,
vCi : 7:1:13d
b q 0i C i2 it is easily shown that an exactly reverse behavior holds.
1 These features help one to visualize the movement of

trajectories of constant C i2 in the (z, t)-plane shown in
The mechanistic basis of the result (7.1.13b) is clearer from Figure 7.1.3(a). These lines (called characteristics) which
such an approach. start at z 0 represent different concentrations entering
The result (7.1.13d) was based on being the void the column (z 0) at different times. If a feed of constant
volume fraction occupied by the mobile phase in the concentration C 0i2 enters the column, then all lines starting
packed bed. If the adsorbent particles are also porous with at z 0 and t  0 will be parallel; the slope of each line is
a void volume fraction of p , and s is the actual density of equal to vCi for C i2 C 0i2 . The interstitial liquid at the
the solid adsorbent particle material, then, using argu- column inlet is represented by the trajectory AB, whose
ments as before, one can show that slope is vz , the liquid velocity. To the left of it lie the
vz trajectories of liquid originally present in the column at
vCi : 7:1:13e
1 1 different locations in the column (different z, t 0). On the
1 1 p s q i1 C i2
0
p
right side of line AB are characteristics of the feed liquid
The denominator in (7.1.13e) has (unlike that in expression which have C i2 0. The wave velocity of this zero concen-
(7.1.13c)) three sources that contribute: the fraction of total tration vCi (for C i2 0) should provide the slope of these
solutes that remain in the mobile phase, the fraction of characteristics. If the adsorption isotherm is favorable
total solutes present in the liquid in the pores of the (Figure 7.1.2(b)), vCi jC i2 0 < vCi jC i2 60 . These intersect the
adsorbent and the fraction of total solutes adsorbed on characteristics for C 0i2 along the line AS. At any point on
the surfaces of the pores of the adsorbent and other out- line AS, characteristics having two different values C i2 0
side surfaces. Note that s 1 p 1 is the bulk density, and C i2 C 0i2 simultaneously exist. Line AS is like a shock
b , of the bed used in earlier expressions. The above wave having two different concentrations on two sides
expression is valid when the solute size is small with of a line, a discontinuity (Sherwood et al., 1975). The point
respect to the adsorbent pore size so that the solute con- of intersection of this line of discontinuity AS with z L,
centration in the adsorbent pore liquid phase is the same the column length, is crucial. The value of time t
494 Bulk flow perpendicular to the direction of force

corresponding to the coordinates (L, t) of this point defines (a)


the time when the bed is completely saturated. As shown B S
in Figure 7.1.3(b), the exit concentration in the liquid
suddenly jumps from C i2 0 to C i2 C 0i2 at this time;
the feed solution breaks through the bed at this time, and
the adsorption operation must stop. The sudden jump part
z L
of the square concentration wave and its subsequent con-
stant value at C 0i2 is identified as the column breakthrough
curve.
If the isotherm is linear instead of favorable, the
characteristics for C i2 0 in between AB and AS in Figure z=0
A t
7.1.3(a) will be parallel to the characteristics for C 0i2 . Even t=0 t
then, when the characteristics for C 0i2 first hit z L, there
(b)
will be a concentration discontinuity from C i2 0 to
C i2 C 0i2 at the column outlet. For more details, see
Wankat (1986, vol. I, pp 1622).
To determine the value of time t t when the feed C 0i 2
solution breaks through a column of length L, a solute
mass balance is carried out over the column from time C i2 exit
t 0 to the time t of breakthrough. For the sake of gener-
ality, the mass balance is carried out in a column which
may have some solute present in the column at t 0, i.e.
C i2 z; 0 and C i1 z; 0 are nonzero. Further, p 6 0. Before t
the feed concentration breaks through the column end in 0
t
the liquid effluent, the liquid phase everywhere in the
column would have C 0i2 ; thus the total number of moles
Figure 7.1.3. (a) Characteristics for feed concentration C 0i2 for
of solute in the column at time t of breakthrough is
adsorption in the (z, t)-plane for the equilibrium nondispersive
given by
model for a favorable isotherm. (b) Breakthrough curve for equi-
LSc C 0i2 1 p C 0i2 im 1 C 0i1 librium nondispersive model with constant feed concentration in

an initially empty column.


LSc C 0i2 1 p C 0i2 im b qi1 C 0i2 :

L vz
This must equal the number of moles introduced into the 
t q C 0

1
column by the steady inflow of feed from time t 0 to time 1 p im b i 0 i2
C i2
t for breakthrough, i.e. vz Sc tC 0i2 plus the moles of solute
vz L
originally present in the column,   vCi jshock :
1 1 C 0i1 t
1 p im
L C 0i2
Sc C i2 z;0 1 p C i2 z;0 im 1 C i1 z;0 dz:

7:1:15c
0
This also happens to characterize the square wave front
The solute balance relation is therefore of concentration C 0i2 . For time t < t (breakthrough) t, the
L

vz Sc tC 0i2 Sc C i2 z;0 1 p C i2 z;0 im 1 C i1 z;0 dz

7:1:15a
0
LSc C 0i2 1 p C 0i2 z;0 im b qi C 0i2 :

location of the front will be inside the column where z < L;


For the special case studied so far, C i2 z; 0 0 C i1 z; 0;
for such times, relation (7.1.15c) should be expressed as
in such a case,
z=t instead of L=t. A more general expression for t or
L q C 0 vCi jshock may be derived for the case where C 0i2 z;0 C ii2
 
1
t 1 p im b i 0 i2 : 7:1:15b
vz C i2 and C 0i1 z;0 C ii1 (in equilibrium with C ii2 ). The relation
equivalent to (7.1.15a) is
Alternatively, the direct relation between the column
length and the time for breakthrough for a liquid feed of vz Sc tC 0i2 L Sc C ii2 1 p C ii2 im 1 C ii1

constant concentration C 0i2 fed at a constant interstitial


L Sc C 0i2 1 p C 0i2 im b qi C 0i2 : 7:1:15d

velocity vz is
7.1 Force ri in phase equilibrium: fixed-bed processes 495

On rearranging, one can obtain The volume V of feed solution needed to be passed
through the column of length L so as to saturate it
L L
vCi jshock with C0i2 under equilibrium nondispersive mode of oper-
t t
vz ation is:
  
Ci 1 C 0i1 C ii1
:
1
1 i2 V C 0i2 Sc L C 0i2 1 p C 0i2 im 1 C 0i1

1 p im
C 0i2 C 0i2
Sc L C 0i2 1 p C 0i2 im b qi C 0i2 ;

7:1:15e

Is this operation of fixed-bed adsorption with bulk liquid 


q C 0

phase flow perpendicular to the direction of the chem- V Sc L 1 p im b i 0 i2 : 7:1:16d
C i2
ical potential gradient force from the liquid to the solid
beneficial for separation when compared with just plain It will be found later that V is a useful quantity in the study
equilibration between the feed liquid and the adsorbent of the actual separation behavior of real columns which
without any bulk flow in a given direction? may not satisfy equilibrium nondispersive conditions. The
There are a number of ways of looking at this issue. First, value of t corresponding to V is t, and it is obtained from
the bulk liquid-phase flow perpendicular to force produces the expression (7.1.15c) for t.
a very substantial volume of absolutely solute-free feed The concentration wave front moving out of a column
liquid at the column exit. Simple equilibration in a batch in equilibrium nondispersive operation need not be a
process in a vessel will never yield solute-free liquid. For square wave front as shown in Figure 7.1.3(b). The square
example, feed solution of concentration C 0i2 will be reduced wave front resulted from an initially solute-free column
in plain batch equilibration to C i2 , determined by (for p 0) and a constant influent concentration C 0i2 . Suppose the
column has in its entry region a linear distribution of solute
Sc vz tC 0i2 Sc vz tC i2 1 Sc vz tC i1 ; 7:1:16a concentration from C 0i1 to 0, from column inlet to some
distance into the column. The liquid in immediate contact
which leads to
is in equilibrium, and its concentration then varies from
C 0i2 C 0i2 to 0, as shown in Figure 7.1.4. As the feed liquid enters
C i2 : 7:1:16b
1 the column, concentration waves having values from C 0i2 to
1 i1
0 will be moving down the column due to this initial
Here, i1 is the distribution coefficient of solute i between column loading. The concentration wave velocity of each
phases 1 and 2 at equilibrium, i.e. i1 C i1 =C i2 . The frac- concentration will depend on the value of q 0i C i2 for the
tion of the total solute i present in the liquid phase is particular C i2 , which in turn will depend on the nature of
obtained from the ratio R0 i , the adsorption isotherm.
Consider first the favorable isotherm of Figure 7.1.2(b).
Sc vz tC i2 C i2 2

R0 i This type of isotherm is characterized by d2 qi =dC i2 < 0
Sc vz tC i2 1 Sc vz tC i1 C i2 1 C i1 for all values of C i2 . For such an isotherm, if the values of C i2
1 1 1
: 7:1:16c at two locations z1 and z2 > z1 are such that C i2 jz1 > C i2 jz2 ,
1 C i1 1 mi1 =mi2 1 k0 i1
1 then q 0i C i2 jz1 < q 0i C i2 jz2 and vCi jz2 < vCi jz1 . Therefore,
C i2
the velocity vCi of the higher concentration nearer the
Thus, the purification capability of this pattern of bulk flow column inlet is higher than that of the lower concentration
vs. force is excellent for plain batch equilibration. further from the column inlet. After some time t (Figure
Second, one can compare the total number of moles 7.1.4), an entirely different concentration profile will be
transferred to the adsorbent phase in rival modes of oper- observed along the column. The concentration profile has
ation. If C 0i1 is the solid-phase concentration in equilibrium become quite sharp, almost like a square wave, since the
with the influent concentration C 0i2 , then the moles of species i higher concentrations have moved much faster and have
transferred to the solid adsorbent phase per unit volume of caught up with the much slower moving lower concentra-
adsorbent is C 0i1 0 for an initially solute-free adsorbent bed tions further down. This is identified as the self-sharpening
when the bulk flow is perpendicular to the force. A batch wave front. When C 0i2 is zero, that is pure solvent (called
operation without bulk flow will transfer according to relation sometimes eluent) comes in, the desorption behavior of the
(7.1.16a) only C i1 0 moles per unit volume of adsorbent, solute is often called elution.
where C i1 < C 0i1 . Thus, bulk flow of feed along the bed length We would now like to focus on calculating such elution
perpendicular to the force direction achieves a better utiliza- behavior from a preloaded column (Figure 7.1.4). The
tion of the intrinsic adsorption capacity of the adsorbents. nondispersive isothermal equilibrium operation of the
Third, the longer the bed, the larger the volume of feed packed adsorption bed described by equation (7.1.9) may
liquid that can be purified almost completely. A batch be written using the following new independent variable s
adsorption process lacks any such feature. (instead of time, t),
496 Bulk flow perpendicular to the direction of force

Favorable isotherm based


Linear isotherm based
C 0i 2
C 0i 2

Ci 2
Ci 2

0
z 0 z

Unfavorable isotherm based Figure 7.1.4. Movement of concentration profile along the column
C 0i 2 for three different types of isotherms when the initial column
section is loaded linearly with solute from C i2 C 0i2 at z 0 to
Ci 2 C i2 0 some distance down.

0 z

ds=dt vz 7:1:17a V V 0 Sc z z0 C i2 b q 0i C i2

dC i1
 
(where s is the volume of solution fed to the column per Sc z b q 0i C i2 Sc z 1

:
unit empty cross section of the bed (Lightfoot et al., 1962)) dC i2
as (for p 0) 7:1:17g

C i2
C i2 This expression allows us to calculate the eluent volume V
b q 0i C i2 0; 7:1:17b
z s that has to be passed corresponding to a given C i2 at the
t column outlet.
since s 0 vz dt vz t (for a constant feed flow rate).
If the isotherm is asimple linear one in Figure 7.1.2(b)
In general, we may write
2
(therefore d2 qi =dC i2 0 for all C i2 ), then all concen-
dC i2 C i2 =zdz C i2 =sds: 7:1:17c trations at all locations in the initial part of the column
(Figure 7.1.4), have the same wave velocity. As time
Following the procedure used in equations (7.1.12b) and progresses, the same linear concentration profile is trans-
(7.1.12e), we get ported down the column without any change in shape. On

s  the
other hand, if the isotherm is unfavorable
 b q 0i C i2 :

7:1:17d 2

z C i2 d2 qi =dC i2 > 0 in Figure 7.1.2(b)), lower concentra-
tions further down from the inlet of the column will have
(Note: Equation (7.1.12e) for vCi continues to be valid for higher vCi than higher concentrations near the beginning
any C i2 .) of the column inlet. This will lead to a more spread out
For a constant feed concentration, C i2 (0, t), we can profile along the column as time increases and concen-
integrate this equation to obtain trations move down the column. Such a condition is iden-
fs= b q 0i C i2 g z z0 C i2 ;

7:1:17e tified as a dispersive (or diffusive) wave front.


So far, the adsorption based purification of a liquid
where z0 C i2 is the distance corresponding to any C i2 at
 
feed flowing down a bed of adsorbent particles has been
the start of the operation. We may rewrite this relation as analyzed using the equilibrium nondispersive approxima-
s tion. For a liquid feed of constant concentration C 0i2 and a
  b q 0i C i2 :
 

7:1:17f bed initially free of any solute i, such an approximation


z z 0 C i2
suggests a square concentration wave exiting the column
This relation may be interpreted as follows. Consider a end at time t defined by result (7.1.15c). Alternatively, a
column of cross-sectional area Sc. The volume of solution volume V (given by relation (7.1.16d)) of feed liquid has
being passed, V, is given by sSc. Let the volume of solution to pass through the column before the feed solution
passed before elution is started be Vo. Then the elution concentration C 0i2 suddenly breaks through. In reality, the
solution volume passed is V Vo. If z0 C i2 0, we obtain
 
column effluent concentration has more of an S-shape
from relation (7.1.17f) (Figure 7.1.5(a)) than a square wave. This requires
7.1 Force ri in phase equilibrium: fixed-bed processes 497

(a) (c)
C 0i 2 Species
1, 2 and 3

concentration
Total solvent
Species

in effluent
C i2 2 and 3

Species
br
Ci 2 3

0 t tbr V
t3 t2 t1
V t t

(b) Figure 7.1.5. (a) Effluent concentration profile from a fixed bed
(breakthrough curve) for one solute i in feed. (b) Column concen-
C 0i 2 tration profile at three different times t1, t2 (>t1) and tbr for one
solute i in feed. (c) Breakthrough curve from a fixed bed for a feed
tbr
containing three solutes i 1, 2, 3: species 3 is least strongly
t1 < t 2 t2 < tbr
C i2 adsorbed; species 1 is most strongly adsorbed.

0
z L
LMTZ

termination of the adsorption operation at time tbr less longer going on, although some occurred at an earlier
than that t defined by (7.1.15c); the volume V br of puri- time; in the final section, with C i2 0, no mass transfer
fied liquid obtained would be less than V . The has started yet. Thus, mass transfer occurs only in the
breakthrough concentration of the solute C br i2 at t br is deter- intermediate region: this region of the bed is called the
mined by the nature of the operation. This value is usually adsorption zone or the mass-transfer zone (MTZ). As time
around 0.050.1 of C 0i2 ; when the liquid is to be purified, its increases, this adsorption zone travels through the bed;
value is much lower. Such a diffuse breakthrough of solute finally, it shows up in the bed effluent as concentration
instead of a sharp front may be caused by a lack of equi- increasing with time (Figure 7.1.5(a)). The length of this
librium between the two phases or by the presence of axial zone is identified as LMTZ.
diffusion and dispersion, or both. The lack of equilibrium A number of analyses have been developed to account
between the two phases may come about due to a finite for such features in real fixed-bed adsorption processes.
mass-transfer resistance in the liquid film surrounding the Two such analyses and their major results will be briefly
particle or due to diffusional resistances in the pores of the identified below. A linear equilibrium model with disper-
particle, or both. sion is considered first. Lapidus and Amundson (1952)
If one were to determine the solute concentration pro- solved equation (7.1.4) which included the axial dispersion
file in the liquid phase in the packed bed at any time prior term under the following assumptions:
to breakthrough, one will find a shape inverse to that
(1) local equilibrium exists everywhere between the liquid
shown in Figure 7.1.5(a). Figure 7.1.5(b) illustrates two
and solid phase and it is linear, e.g.
bed solute profiles at two instants of time t 1 and t 2 > t 1
that are far apart. In both profiles, the initial section of the C i1 i1 C i2 ; 7:1:18a
profile has a constant concentration C 0i2 corresponding to
the feed liquid coming in; the bed is saturated here and no where i1 is a constant;
longer has the capacity to adsorb any more solute. The
(2) the column is infinitely long.
final sections of the profiles in each case have a value of
C i2 0; they are without any solute and retain their intrin- The conditions used for the solution are: a steady
sic capacity for solute adsorption. In between, each profile liquid feed of constant inlet concentration C 0i2 , uniform
changes from C i2 C 0i2 to 0; this section of the bed has solute concentration of C ii2 and C ii1 (at equilibrium)
some solute adsorption capacity left. It is obvious that in throughout the bed for time t  0. Define a new dependent
the first section, where C i2 C 0i2 , solute transfer is no variable
498 Bulk flow perpendicular to the direction of force

i z;t C i2 z;t C ii2 =C 0i2 C ii2 : 7:1:18b concentration profile for a column of length z. The trun-
cated form4 (7.1.18h) of the solution (7.1.18e) represents
Under the assumption of local equilibrium, we can rewrite the complete solution accurately, provided Pez;eff is not too
equation (7.1.4) as large (Lapidus and Amundson, 1952). Sometimes it is

1 C 0i1 i z;t

i z;t 2 i z;t desirable to have the solution (7.1.18h) expressed in terms
1 0 vz Di;eff;z : of t and t (remember erf x erfx):
C i2 t z z2
( " #)
7:1:18c C i2 z;t C ii2 1 1 t=t
1 erf  1=2 : 7:1:18j
The boundary and initial conditions are as follows: C 0i2 C ii2 2 2 Di;eff;z t=vz zt

z 0; i 1 for t > 0; z ; i 0 for t > 0; Lapidus and Amundson (1952) have shown that, as the
value of vz increases or Di;eff;z decreases (increasing
t 0; i 0 for z>0: 7:1:18d
Pez;eff ), the S-shaped breakthrough curve approaches the
square wave form of nondispersive operation. On the other
The solution for equation (7.1.18c) under the boundary
hand, if we consider the extent of dispersion around the
and initial conditions is
8 9 center of the profile t and V , it varies as t 1=2 or V 1=2 : the
extent of dispersion increases with time or volume. We will
2 v s> 8 9
16 u v2z t vz z =
>
<u = <
i 41erf z exp employ equation (7.1.18h) sometimes to describe the
2 : 4Di;eff;z 4t Di;eff;z > Di;eff;z ;
t
breakthrough.
> ; :
8
v
9# There are a number of models which do not assume
u v2z t the existence of equilibrium between the liquid and the
> s>
<u =
 erfc t z ; 7:1:18e
: 4Di;eff;z
> 4t Di;eff;z > ; solid phases. However, they do not incorporate the effect of
axial dispersion. In such models of nonequilibrium non-
where dispersive operation of the column, the mass-transfer
rate between the liquid and the surface of the adsorbent
1 1 = C 0i1 =C 0i2 : 7:1:18f (primarily in the pores) is not infinitely fast; rather it is
finite. Further diffusion in the porous adsorbent particle is
The erfc quantity is a small number, and under conditions
quite important. One of the earlier models of this type is by
where vz z=Di;eff;z is not too large it may be neglected.
Rosen (1952, 1954).
Then we get
" ( Description of the mass-transfer rate between the
r)# liquid and the adsorbent particle is facilitated by a specifi-
1 vz t
i 1 erf p z ; 7:1:18g cation of the particle geometry. Relatively large particles
2 4Di;eff;z t 4t Di;eff;z
are often used in industrial adsorbents to reduce pressure
which may be rearranged to yield drops. Rosen considered spherical particles; the mass bal-
p
( " 1=2 #) ance equation for the concentration, C i2 , of species i in the
C i2 z;t C ii2 1 Pez;eff V V pore liquid of such an adsorbent particle (where the effect-
i 1 erf ;
C 0i2 C ii2
1=2
2 2 VV ive diffusion coefficient of species i is Dip ) is


7:1:18h p
C i2

1

C
p 
Dip 2 r i2 7:1:19a
where V vz Sc t is the volume of feed solution t r r r
which has passed through the column of length z from t 0 if spherical symmetry is assumed. (See Table 6.2.1; v 0;
to time t, J i J i 0.) In adsorption, the rate of transfer of species

C0
 i from the liquid to this particle can be described by means
V Sc z 1 i1  Sc z 1 i1
0 vz Sc t of a mass-transfer coefficient k c and a particle-average
C i2 p
concentration C i2 of species i for particles of radius r p :
7:1:18i
rp rp
is the volume of solution required to saturate a column 4 3 3
 
p p p p
C i2 r C i2 4r 2 dr ) C i2 3 C i2 r 2 dr;
of length z in the absence of dispersion, and 3 p rp
0 0
Pez;eff zvz =Di;eff;z is a Pclet number based on the
 
7:1:19b
effective axial diffusion coefficient.
Equation (7.1.18e) provides the liquid-phase solute
concentration in an infinitely long column at location z
from the inlet. Although the solution was developed
for an infinitely long column, the value of the concentra- 4
This solution is often the basis for what is called linear
tion profile at any z will also provide the output chromatography based on linear adsorption equilibrium (7.1.18a).
7.1 Force ri in phase equilibrium: fixed-bed processes 499

  p
4 3 C i2
p
 Due to the restriction of  0:01, this solution also repre-
r p k c 4r 2p C i2 C i2 jrrp
3 t sents negligible external mass-transfer film resistance if the
p
C 3k c p
 bed is long. Note that this solution is symmetrical around
) i2 C i2 C i2 jrr p : 7:1:19c
C i2 =C 0i2 1=2. Since these solutions are expressed in
 
t rp
dimensionless quantities, results from one particular
Rosen had assumed linear adsorption equilibrium between column could be used to predict the breakthrough curve
p
the liquid-phase concentration in the pore, C i2 , and the for other columns. The value of k c needed to make calcu-
p
solid-phase concentration in the pore, C i1 : lations in such systems may be obtained from (Dwivedi
p
C i1 i1 C i2 :
p and Upadhyay, 1977)

k c Sc 2=3 0:458 2r p G 0:407


!  
A particle-average concentration C i1 based on the particle
jD ; 7:1:20d
volume is then obtained simply from definitions (7.1.19b) as vz
rp
3 where G is the superficial mass velocity based on empty
p
C i1 i1 C i2 r 2 dr: 7:1:19d column cross-sectional area and the Reynolds number
r 3p
2r p G= > 10.
 
0

These four equations, along with the form of equation There are a number of other analytical solutions for
(7.1.4) without any axial dispersion, namely C i2 z;t available in the literature for linear isotherms.
These take into account axial dispersion, model par-
C i2 1 C i1 C i2 ticles using macropores and micropores, etc.; they have
vz 0; 7:1:19e
t t z been summarized by Ruthven (1984) in his Table 8.1.
Widespread use of powerful computers and sophisti-
have been solved by Rosen for the following conditions:
cated numerical methods have reduced the import-
p
at t 0; C i2 0 for z > 0 and 0  r  r p ; ance of such analytical solutions for breakthrough
for t  0; at z 0; C i2 C i2 :
0
7:1:19f curves.
For a constant liquid feed of concentration C 0i2 fed
The solution is obtained as an integral: into the bed, an immediate result of the diffuse break-
through of the feed solute through the column end is

C i2 1 2 d that the total solute adsorption capacity of the mass-
expf H 1 ;g sin 2 H 2 ;

;
C 0i2 2 transfer zone of length LMTZ cannot be fully utilized. At
0
the breakthrough time tbr for the breakthrough solute
7:1:20a concentration C br i2 at the column end z L, the solute
concentration profile in the solution in the mass-transfer
where zone is shown in Figure 7.1.5(b). This profile, assumed to
0 1 be symmetrical, shows that the hatched area represent-
3Dip i1 z @1 A Dip i1 2Dip t fz=vz g ing the integral
; ; ;
vz r 2p kC rp r 2p
LMTZ

H D1 H 2D1 H 2D2 S c b
 0
qi C i2 qi C i2 dL
 

7:1:21a
H 1 ; ;
1 H D1 2 H D2 2 0

H D2 represents the additional moles of solute i which could


H 2 ; ;
1 H D1 2 H D2 2 have been adsorbed if there were no diffuse breakthrough.
Due to the symmetric concentration profile, one can argue
sinh2 sin2 sinh2sin2
H D1 1; H D2 ; that essentially the total number of solute moles adsorbed
cosh2 cos2 cosh2cos2
in this region,
7:1:20b LMTZ

S c b qi C i2 dL;
 
and is the variable of integration. This exact solution of 7:1:21b
the solute breakthrough curve is numerically determined 0
due to the nature of the integral in solution (7.1.20a). If the
is approximately equal to the number of moles not
adsorbent bed length L is very large (specifically,  50
adsorbed,
and  0:01), an asymptotic solution of (7.1.20a) is
LMTZ
LMTZ

" !#  0
S c b qi C i2 dL Sc b qi C i2 qi C i2 dL;
   

C i2 1 3=2
1 erf : 7:1:20c
C 0i2 2 2=51=2 0 0
500 Bulk flow perpendicular to the direction of force

unless the sorption process is highly nonlinear. Therefore, are generally quite complex. It is useful, however, to
LMTZ LMTZ
consider an elementary (but highly inexact) analysis of

2 3
14 breakthrough in such a system containing more than one

 0 

S c b qi C i2 dL S c b qi C i2 dL5
 
2 solute. Assume that there are, say, three solutes i 1, 2, 3
0 0
present in the feed solution at concentrations of C 012 , C 022
LMTZ

 0 and C 032 , respectively. Assume further that each solute
Sc b qi C i2 qi C i2 dL;
 

7:1:21c adsorbs independently of the other two and that the


0 adsorption isotherm for each is linear in the following
where the second expression in brackets represents the fashion (Figure 7.1.2(a)):
total solute adsorption capacity of LMTZ at tbr for the q11 C 12
 
q21 C 22
 
q31 C 32
 
feed solution concentration C 0i2 . The total adsorption cap- > > : 7:1:22a
C 12 C 22 C 32
acity of the bed of length L for the same feed solution
concentration C 0i2 under the nondispersive equilibrium Therefore species 3 is least adsorbed, whereas species 1
condition is is most strongly adsorbed, with species 2 being in between.
In the context of a nondispersive equilibrium adsorp-
L
tion model, the breakthrough times t i (7.1.15c) for the
Sc b qi C 0i2 dL Sc b qi C 0i2 L:
   
7:1:21d
three species are related by
0
q C0 q C0
   
L L
 
Correspondingly, the lost adsorption capacity of the bed t1 1 b 1 0 12 > t 2 1 b 2 0 22 >
vz C 12 vz C 22
from (7.1.12c) is  0 
L q C

LMTZ LMTZ t3 1 b 3 0 32 : 7:1:22b
2 3
14 vz C 32

 0  0
S c b qi C i2 qi C i2 dL Sc b qi C i2 dL5
 

2
0 0
The least strongly adsorbed species 3 will come out of the
1
Sc b qi C 0i2 LMTZ : adsorber first (Figure 7.1.5(c)). Correspondingly there will
 
2
be a pure solution of species 3 at the adsorber outlet during
7:1:21e
the time period t 2 t 3 . At time t 2 , species 2 will break
An estimate of the fractional loss of the bed adsorption through and will be present along with solute 3; Figure
capacity due to the diffuse breakthrough is therefore 7.1.5(c) plots the total concentration of all solutes at the
given by column outlet as a function of time. We observe a staircase
1
 0 behavior, with all three solutes present at the outlet after t 1 .
2 Sc b qi C
 i2 
LMTZ 1 LMTZ
Figure 7.1.5(c) displays this behavior in the context of a
: 7:1:21f
Sc b qi C 0i2 L 2 L dispersive model. Such an outlet breakthrough profile
The quantity the length of the unused bed (LUB), is often development is called a frontal development.
used to describe this loss: Frontal development is employed in large-scale purifi-
cation applications, for example in decolorization of sugar,
1 LMTZ
 
LUB L: 7:1:21g corn syrup, for removal of oxidation products from waxes,
2 L used oils, etc. Adsorbents, such as activated charcoal, are
used to adsorb the strongly adsorbing color-causing
The smaller the length of the MTZ, the lower the loss of
impurity, etc. One pure species, the least adsorbing one,
bed adsorption capacity due to the diffuse breakthrough.
is obtained. However, the feed concentration of the least
Methods for calculation of LUB are available in Treybal
adsorbed species can be substantial, making this technique
(1980) and Wankat (1990, pp. 366375).
industrially useful.
It is useful to recall the basic liquid adsorption system
considered so far. There is one solute i to be adsorbed from an Example 7.1.1 Breakthrough calculations for equilibrium
inert solvent flowing through a column of adsorbent particles nondispersive operation of a fixed bed with a liquid feed.
under isothermal conditions. There can be many systems, (a) Lightfoot et al. (1962) have illustrated the breakthrough
however, with more than one adsorbable solute species in a behavior of a solution of lauric acid in petroleum ether
solvent which may or may not be inert. For an overview, the flowing through a packed bed of activated carbon adsorbent,
3 cm long, having 0.4 and a cross-sectional area of
reader is referred to Ruthven (1984); original studies of con-
1.32 cm2. The concentration of lauric acid is 0.035M. The
siderable importance in this area are by Glueckauf (1949),
equilibrium adsorption isotherm (Freundlich type, see
Helfferich and Klein (1970) and Rhee et al. (1970a). (3.3.112c)) is given as C i1 2:26 C 0:324 . Determine the break-
i2
The adsorption behavior and the breakthrough curve through volume of liquid when the solute feed concentration
for systems containing a number of adsorbable solutes in a will appear as a shock wave at the column end for a similar
solvent, as illustrated in the above-mentioned references, column 6 cm long.
7.1 Force ri in phase equilibrium: fixed-bed processes 501

(b) Baker and Pigford (1971) have studied the adsorption


2 3
100 4 1:037  3:025
of acetic acid on activated carbon from a dilute aqueous t 1:755
solution at 60  C. The sorption behavior of the solute was 11:6 0:10:59
found to be 2 3
3:13 5
gmol 8:62 1:755 8:62  13:925:
3:02 C 0:41
4
qi i2 ; 0:257
kg dry carbon
for C i2 in gmol/liter. The activated carbon bed characteristics Now, t 120 min. From equation (7.1.16d), the break-
are as follows: 0.43; p 0.57; s 1:82 g=cm3 ; L through volume
100 cm; Sc 3 cm2. The feed solution at a concentration of 
1 q C0

0.1 gmol/liter is introduced at 15 cm3/min. Determine the V Sc L 1 p im b i 0 i2
C i2
time for solute breakthrough and the volume of solution
that will have passed through the bed at the time of 3  100  0:43  13:925 1796 cm3 :
breakthrough.
(c)
(c) An enzyme present in a dilute solution is to be
C0
 
adsorbed on cellulosic adsorbent particles in a 100 cm long V Sc L 1 i1
0
packed bed 4 cm in diameter, having a bed porosity of 0.4. C i2
The enzyme feed concentration is 1 mg/liter; the linear 2
4  1000:4 0:6  40 cm3
adsorption isotherm is as follows: 4
 40024:4 30:66 liter:
C i1 mg=cm3 40 C i2 mg=cm3 :
Determine the breakthrough solution volume. Note: Use of equations (7.1.13d, e) is not recommended for
determining t L=vCi for adsorption examples with non-
Solutions (a) The breakthrough volume V of the solution to
linear equilibrium behavior since the characteristic lines for
be passed through the column for the feed concentration to
different concentrations can overlap, a physically impossible
break through in equilibrium nondispersive operation is
situation (Sherwood et al., 1975; Wankat, 1986).
given by relation (7.1.16d):

q C0
  Example 7.1.2 Elution calculation for equilibrium nondispersive
V Sc L b i 0 i2 : operation of a fixed bed with a liquid feed. In Example 7.1.1(a),
C i2
150 cm3 of the solution of lauric acid was passed. The bed was
Employing definition (7.1.6), we can rewrite this as completely saturated and the exiting solution had feed concen-
"  0:324 # tration 0.035 M. Now pure petroleum ether flow is initiated to elute
C0 2:26 C 0i2
 
V Sc L 1 i1 1:32  6 0:4 0:6 ; the adsorbed lauric acid. Calculate the volume of pure petroleum
C 0i2 C 0i2 ether passed corresponding to the following concentrations of
" # lauric acid at the bed outlet: 0.03 M, 0.02 M, 0.01 M and 0.005 M.
0 1
C i2 0:035M ) V 7:92 0:4 1:356 Solution From equation (7.1.17g), we can write
0:0350:676
V V 0 Sc z z0 C i2 1 dC i1 =dC i2 :

) V 7:92 0:4 13:1 7:92  13:5 107cm3 :


Here
(b) Since the adsorbent particle porosity has been pro- 0:676
vided, we will employ equation (7.1.15c) to determine the z0 C i2 0; dC i1 =dC i2 2:26  0:324  C i2 :
time t for breakthrough. For acetic acid, assume im 1. 0 3
Now, V 150 cm . Therefore
Here the interstitial fluid velocity h
0:676
i
V 150 1:32 z 0:4 0:6  2:26  0:324  1=C i2
volumetric flow rate 15cm3 =min h
0:676
i
vz 11:6cm=min: ) V 150 3:168 3:47=C i2
cross-sectional area 3  0:43cm2
Now (since z 6 cm for the outlet concentration). The volume of
pure petroleum ether passed is
L vz
   ; h
0:676
i
t 1 q C0 V 150 3:168 3:47=C i2
1 p im b i 0 i2
C i2
for the C i2 values of interest (see Table 7.1.2).
So
100 11:6 Table 7.1.2.

t 0:57 1 p 1 qi C 0i2
1  0:57  1 s Outlet concentration, C i2 (M) (V 150) cm3
0:43 C 0i2
11:6 0.03 ~34
;
1:82  0:43  0:57 3:02 0.02 ~52
1:755 0 0:59 0.01
0:43 C i2 ~82
0.005 ~120
502 Bulk flow perpendicular to the direction of force

(a) r0 (b)

rh

r dr

r dr

Figure 7.1.6 Radial flow fixed bed (adsorbents not shown): (a) flow direction radially outward; (b) flow direction radially inward.
Arrows show flow direction.


7.1.1.1.1 Fixed-bed adsorption radial flow Fixed- where V is the radial volume rate of flow of solution from
bed adsorption from a solution is sometimes carried the inside hole to the periphery of the packed bed. In the
out in a packed bed where the liquid flows radially limit r ! 0, one obtains
instead of axially along the column length. Typically 
C i2 C i1

C i2
there is a central duct or hole in the packed column. 2rL 1 V
t t r r
The feed solution may be added to the central hole and  
C i2
it flows radially outward (Lapidus and Amundson, 2LDi;eff;r r ; 7:1:23b
r r
1950); alternatively the feed solution may be forced to

enter the packed bed from the periphery, move through where V and Di;eff;r are assumed constants. Further,
the bed radially into the central duct of radius r h r < r < r r, and r varies from r h to r 0 . An alternative
(Huang et al., 1988a, b). These two configurations are form is given by (this could also be obtained from
shown in Figures 7.1.6(a) and 7.1.6(b), respectively. (6.2.32) in the manner of (6.2.35), with the z-coordinate
Note that in each case the radial bulk flow is perpen- replaced by r)
dicular to the force direction, which is vertically upward
V C i2 Di;eff;r
 
or downward depending on the location of the flow and C i2 C i1 C i2
1 r :
the particle. t t 2rL r r r r r
A differential equation for the concentration C i2 of 7:1:23c
species i in the mobile phase (averaged in the z- and
-directions) may be developed in the manner of equation If diffusion and dispersion in the radial direction are neg-
(7.1.2). Consider a cylindrical layer of adsorbent, of thick- lected, this equation is reduced to

ness r and bed length L, at radial location r for the flow C i2 C i1 V C i2
configuration of Figure 7.1.6(a) with a radially outward 1 0: 7:1:23d
t t 2rL r
flow. A species i balance, in the absence of any chemical
A solution of this nondispersive packed-bed adsorption
reaction, is
with radial flow is available (Lapidus and Amundson,
rate of accumulation rate of inflow at r 1950) under the condition of elution chromatography (we
rate of outflow at r r ;
8 9 0 1 0 1
< C C = C i2 A C i2 A
i2 i1
 
2rrL 1 V C i2 2L rDi;eff;r
@ V C i2 2L rDi;eff;r
@ ; 7:1:23a
: t t ; r r rr r
r r rr
7.1 Force ri in phase equilibrium: fixed-bed processes 503

will soon learn about this) for two cases: (1) equilibrium mass balance equation for species i (phase j 1, solid;
theory with linear adsorption isotherm; (2) nonequilibrium phase j 2, gas) is
theory based on a suitable rate equation. The governing
C i2 1 C i1 C i2 vz 2 C i2
equation analogous to (7.1.23b) for the radial flow packed- vz C i2 Di;eff;z ;
t t z z z2
bed adsorption where the effluent flows into the central
7:1:25
duct from the bed periphery is
  where C i2 is the gas-phase molar concentration and C i1 is
C i2 C i1 C i2
2rL 1 V the solid-phase molar concentration. Analyses of adsorber
t t r r
  performance for a gaseous feed often assume nondisper-
C i2 sive operation. Models of nondispersive operation either
2LDi;eff;r r : 7:1:24a
r r postulate equilibrium between the two phases or describe
If the diffusion term is neglected, this equation is simplified the mass-transfer rate of species in particular ways. Analy-
as follows: sis of the adsorber using an equilibrium nondispersive
mode of operation is considered next.

C i2 C i1 V C i2 It is customary to use the mole fraction x i2 of the gas
1 0: 7:1:24b
t t 2rL r phase instead of C i2 , the two being related by C i2 x i2 C t ,
where C t is the total molar concentration. Here, x i2 is the
The recovery of bioactive products from their dilute solu-
value of x i2 space-averaged over the x- and y-coordinates
tions is generally carried out using a fixed bed of adsorb-
in the same manner as C i2 is developed from C i2 . If the gas
ents. Since such solutions are dilute, often large solution
pressure drop along the packed bed is very small, C t may
volumes have to be processed; also, the solutions may be
be considered constant along the bed since C t is propor-
viscous. In a conventional fixed bed (Figure 7.1.1) of
tional to the total gas pressure. The mass balance equation
considerable length, this will lead to large pressure
(7.1.25) may now be written in terms of x i2 as (Ruthven,
drops. A radial flow fixed bed will, however, have a much
1984)
lower pressure drop. Results of the purification of an
enzyme and the removal of proteases from human x i2 1 C i1 x i2 vz
plasma using radial flow cartridges are available in vz x i2 0 7:1:26
t C t t z z
Huang et al. (1988b).
under nondispersive condition. The variation of the gas
velocity vz with the axial coordinate z is due to adsorption
7.1.1.2 Fixed-bed adsorption: mobile feed gas of species onto the adsorbent particles. For a binary gas
Preferential adsorption of a gas species or a vapor species mixture, either both species may be adsorbed or only one
from a flowing gas mixture by solid adsorbent particles in will. Assuming only one species is adsorbed, a mass bal-
a packed bed is frequently practiced in industry (see ance for species i being adsorbed is
Table 7.1.1). A large number of gas purifications (solvents
C i1 vz
or odors from air, sulfur compounds from natural gas, 1 Ct ; 7:1:27a
vent streams, etc., CO2 from natural gas, H2O from a t z
variety of gas streams) and bulk separations (O2N2, n-
where, following relations (7.1.6) and (7.1.7), we can write
paraffins from iso-paraffins, mixtures of aromatics, etc.)
are achieved by adsorption in a packed bed of adsorbent C i1 b qi1 b   C i2
such as activated carbon, zeolites, carbon molecular q0 C i2
t 1 t 1 i1 t
sieves, silica gel, etc. (Yang, 1987). When the adsorbent
q0 C i2 C t x i2
 
particles are saturated with adsorbates, the bed is regen-
b i1 : 7:1:27b
erated, by heating, by a purge gas or by lowering the 1 t
pressure. The latter process, known as pressure swing
adsorption (PSA), is quite important and is treated in Note that if the species adsorbed is in trace amounts, the
Section 7.1.2. change in vz along z due to adsorption would be negligible.
An analysis of fixed-bed adsorption with mobile feed Thus, for nontrace systems, using these two relations, rela-
gas (instead of feed liquid) should start with the mass tion (7.1.26) can be simplified to
balance equation (7.1.3) for species i. Unlike liquid feeds, 
q0 C i2
  
C i2 C i2
the gas velocity vz is a function of z unless some trace 1 b i1 1 x i2 vz 0: 7:1:28
t z
compounds are being removed. (For trace compound
removal, the volume V of feed gas needed to saturate the Following (7.1.9) and (7.1.11), we can obtain for the wave
column is given by (7.1.16d); the shock velocity is obtained velocity vCi of concentration C i2 for species i in a gaseous
from (7.1.15c)). Therefore, instead of (7.1.4), the general feed of one adsorbed species i (inert carrier)
504 Bulk flow perpendicular to the direction of force

dz vz
vCi  : 7:1:29
dt b q0 i1 C i2 v 0z vz
1 1 x i2

Note that vz changes with bed distance z and mole fraction C 0i 2


x i2 ; it would therefore be convenient to replace it by terms
containing x i2 . Assume the isotherm to be linear so that
Ci2
q0 i1 C i2 constant. Then, combining relations (7.1.27a)
 
C 0i 1
and (7.1.27l), the following expression is obtained: Ci 1

vz q0 x i2
b i1 : 7:1:30
z t T0

Using now relation (7.1.29) in the form of dz=dt in the


above equation, we get Zone
Zone I
Zone II III
vz q0 vz
b i1 h i: 7:1:31 L
x i2 1 b q0i1 1 x i2

Figure 7.1.7. Constant-pattern behavior in isothermal single-


This can be integrated between the limits of v0z , x 0i2 (at the
component adsorption for a gas mixture. (After Sircar and Kumar
adsorber inlet) and vz , x i2 anywhere else in the adsorber to
(1983).)
yield
q0 
vz 1 b i1 1 x 0i2

; 7:1:32 the favorable isotherm tends to sharpen the front, leading
1 b qi1 1 x i2
0
v0z
often to a constant-pattern behavior.
The nature of the constant-pattern behavior for iso-
h i
0 b q0i1  0
dz vz 1 1 x i2 thermal adsorption of a single gas species i in the presence
vCi h i2 : 7:1:33
dt
1 b qi1 1 x i2
0
of an inert gas is shown in Figure 7.1.7, where the profiles
of gas velocity vz , mobile-phase concentration C i2 of
This result implies that, as x i2 increases, vCi increases. In species i, adsorbent-phase concentration C i1 and tempera-
liquid systems with a linear isotherm, equation (7.1.12a), ture T are shown along the column. There are three zones
however, indicated no such concentration dependence of in the column: Zone I is saturated with species i, therefore
vCi . The effect of concentration on vCi in a nontrace gas- the values of vz , C i2 and C i1 are constant; Zone II is the
eous system is, then, similar to that observed with a favor- MTZ, where the values of these quantities vary from satur-
able isotherm (see Figure 7.1.4). A spread out profile along ation to values characteristic of the bed where no adsorp-
the column is compressed to a front sharper than would tion has taken place yet, the latter being Zone III. Using the
have been possible otherwise. constant-pattern model, Sircar and Kumar (1983) have
This type of behavior aids in creating what is known as provided analytical expressions relating the time difference
constant-pattern behavior. Consider the mass-transfer (t2 t1) in the breakthrough curve corresponding to two
zone (MTZ) in Figure 7.1.5(b). Near the entrance to the arbitrary composition levels C i2 j1 and C i2 j2 for a number of
column, such a profile is created spontaneously by the cases. They have also provided analytical expressions for
effects of axial dispersion and mass-transfer effects. Further the length of the MTZ. The cases studied include bulk
down the column, as the gas velocity decreases to vz from single-component adsorption satisfying Langmuir adsorp-
the inlet value v0z due to substantial adsorption, the con- tion isotherm under either gas film control or solid diffu-
centration front tends to become compressed: this com- sion control.
pression acts counter to the tendency of axial dispersion
and mass transfer to expand the front (as in an unfavorable
7.1.1.3 Fixed adsorbent bed regeneration
isotherm of Figure 7.1.4). Often these two opposing ten-
dencies balance each other, so that the MTZ (Figure 7.1.5 After the passage of a feed gas mixture or liquid solution
(b)) travels through a long column without any change. through a fixed adsorbent bed for some time, the adsorb-
Such constant-pattern behavior may be achieved also in ent particles are saturated with the solute. For reuse, the
systems where the adsorbed species is present in trace bed has to be regenerated; the particles in the bed acting
systems, provided the species displays a favorable iso- as mass-separating agents need to be restored to their
therm. The adsorption of moisture from air onto silica gel original state so that they may be useful again as mass-
adsorbent is a case in point. Although dispersion and separating agents. The process of desorption needed for
mass-transfer effects, if any, tend to broaden the front, the regeneration of adsorbents may be understood from
7.1 Force ri in phase equilibrium: fixed-bed processes 505

(a) (b) (c)


TAdsorption
A A T1 A

Adsorption
qi1A qi1A

qi 1 qi 1 qi 1

Adsorption
B
qi1|B qi1|B B
B

Desorption
B
TDesorption T1<T2
Desorption

p12 p12 p12

TAdsorption < TDesorption p12Adsorption > p12Desorption

Figure 7.1.8. Adsorption isotherms for a gas mixture and conditions for adsorptiondesorption: (a) thermal swing; (b) purge gas or
pressure swing; (c) combined thermal swing and purge gas.

the adsorption isotherms shown in Figure 7.1.8 for gas (see Section 7.1.2). Such processes are cyclic in nature;
separation, where pi2 is the partial pressure of species i in after desorption, the bed is ready for adsorption again.
the mobile gas phase; pi2 is related to C i2 for ideal gas An elementary basis for quantifying the movement
behavior by pi2 C i2 RT. of concentration of the adsorbed species in the column
If, at the end of adsorption, the adsorbate concentra- during desorption will now be developed. If we con-
tion on adsorbent particles is represented by point sider the adsorption/desorption of a trace amount of
A (Figure 7.1.8(a)) and the temperature is, say, TAdsorption, gaseous species from/into a carrier gas vis--vis an
for desorption the temperature is increased to TDesorption, adsorbent, then equation (7.1.25) is reduced to equa-
which is higher. The adsorption isotherm now is consider- tion (7.1.4), which is normally used for liquid-phase
ably lower; qi1 is reduced to a lower value, even if pi2 feeds. If now nondispersive operation is assumed, the
remains the same. The temperature of the bed is raised governing species balance equation in the column is
by supplying either a hot gas or steam. The hot gas may be (7.1.8). Thus, the relevant equation continues to be
obtained using the feed gas itself or some other gas, e.g. air. equation (7.1.9),
When the partial pressure of species i in the hot gas is
C i2 vz C
lower than that at point A, there is additional desorption   i2 0;
t b q 0i C i2 z
since qi1 jB < qi1 jB0 (Figure 7.1.8(c)). After desorption, the 1
bed is ready for adsorption again at a lower temperature, in
a cyclic fashion, in processes characterized as thermal- whereby the concentration wave velocity vCi is obtained as
swing adsorption (TSA). (equation (7.1.12a))
An alternative procedure to desorbing by a hot gas or
dz vz
vapor is to reduce the pressure of the bed. As shown in vCi  :
dt b q 0i C i2
Figure 7.1.8(b), although the temperatures of A and B are 1
the same (they are on the same isotherm), the partial
pressure of species i in gas phase at B is much lower; Let the fluid-phase concentration of a uniformly saturated
therefore, the adsorbate concentration is much lower. bed be C 0i2 . This fluid, of concentration C 0i2 , will be pushed
The pressure reduction in the bed is achieved either by out by the desorbing mobile phase entering the column at
lowering the pressure of the bed from the higher pressure, a velocity vz . The fluid at z 0 and t 0 (having a concen-
in a process usually called blowdown, or by pulling a tration C 0i2 ) will exit from the column of length z L at time
vacuum. Both strategies lower the value of pi2 , which can t L=vz . Fluid elements at z > 0 and t 0 having the
also be lowered by passing a purge gas. The process initial saturation concentration C 0i2 will exit earlier (Figure
whereby the absolute pressure level of the adsorber is 7.1.9(a)). Let the desorbing mobile-phase concentration be
reduced essentially at constant temperature, with or with- C i2 0 (it should be < C 0i2 ). The value of vCi correspond-
out a purge gas, is called pressure-swing adsorption (PSA) ing to any C i2 is obtained from the expression (7.1.12a)
506 Bulk flow perpendicular to the direction of force

(a) the column outlet concentration, C i2 (z L), changes


C 0i 2
with t (Figure 7.1.9(b)).
B C S To obtain such concentrations, i.e. C i2 (z L) as a
function of time, use the definition for vCi , namely
dz vz
 ; 7:1:35
dt b q0i C i2
z
L 1
Ci 2 = 0
for any given favorable isotherm, e.g. Langmuir isotherm
(equation (3.3.112b)),
A t
bi1 C i2 q
i i1 ; 7:1:36a
1 bi1 C i2 qSi1
(b)
where qSi1 is the moles of species i in solid phase 1 at
C 0i 2 saturation per unit solid-phase mass. Therefore

bi1 qSi1
Ci2 exit q 0i C i2 dqi1 =dC i2 : 7:1:36b
1 bi1 C i2 2

Substitute this into (7.1.35) using C i2 instead of C i2 and


0 integrate between z 0 and t 0 to z and t for any given
C i2 :
t
z t
vz vz t
Figure 7.1.9. (a) Characteristics for desorption of a bed saturated z dz   dt  :
b bi1 qSi1 b bi1 qSi1
with C 0i2 with an inert purge: equilibrium nondispersive model, 0 1 2 0 1 2
1bi1 C i2 1bi1 C i2
favorable isotherm. (b) Column exit concentration for desorption
of column in (a). 7:1:37a

Rearrange this as
given earlier. Consider any favorable adsorption isotherm ( 1=2 )
(Figure 7.1.2(b)). Obviously, C i2 1 zb bi1 qSi1
1 : 7:1:37b
C 0i2 bi C 0i2 vz t z
q0i1 C i2 jC i2 0 > q 0i1 C i2 jC 0i2 :
   
7:1:34a

Therefore For z L, the bed length, this expression (Walter, 1945)


provides the breakthrough curve for desorption at the
vCi jC i2 0 < vCi jC 0i2 < vz : 7:1:34b column outlet, a relation between C i2 jexit and t, as shown
in Figure 7.1.9(b). Further, the whole bed is completely
The characteristics lines for C i2 0 have been shown in
desorbed when C i2 0 at z L; this happens when
Figure 7.1.9(a). When such a line, AS, starting at t 0
intersects z L, the desorbing fluid breaks through the L vz
h b qS
i: 7:1:37c
column, i.e. the column desorption is complete. t 1 b i1 i1
Between this line AS and the line AB (whose slope is
that of the fluid velocity, vz) lie two regions. The first Recall that these two results are valid for nondispersive
region is between lines AB and AC, where the desorbing equilibrium desorption of a bed, initially saturated
fluid has arrived but its concentration is still C 0i2 ; such throughout at a level of C 0i2 by a mobile phase without
lines have origins on AB at different column locations any solute species, when Langmuir adsorption isotherm
and they exit the column with velocity vCi jC 0i2 . Such a line characterizes the adsorption equilibrium between the
originating at z 0 and t 0 is shown as AC. To the solute in the mobile phase and the adsorbent.
right of such a line, the mobile-phase column concen- The breakthrough curve developed above for desorp-
trations are changing. Further, the fluid exiting the tion was based on a pure mobile phase, i.e. C i2 0, sent in
column now has concentrations between C 0i2 and to desorb. Often such a phase is at a higher temperature,
C i2 0. The column exit concentrations as a function of e.g. a hot purge gas. In such a case, the species balance
time are shown in Figure 7.1.9(b). For a favorable iso- equation (e.g. (7.1.9)) is coupled with an overall heat
therm, this implies that the slopes of lines originally at balance equation. A solution of such a system is schemat-
t 0, z 0 now decrease. Ultimately, the line AS is ically illustrated in Figure 7.1.10(a) for nonequilibrium,
reached when the bed is completely desorbed. Note that nonisothermal desorption of a single adsorbed species
7.1 Force ri in phase equilibrium: fixed-bed processes 507

(a) Figure 7.1.10. (a) CO2 concentration profile ( C i2 ) and tem-


perature profile along the bed length at any time in desorp-
tion by a hot purge. (After Kumar and Dissinger (1986).)
Schematics of the (b) purity (% normal paraffins) and
(c) flow rate of the hydrocarbon stream leaving the sieve
bed during steady state cyclic operation of the Ensorb linear
T paraffins unit at Baytown Refinery (arbitrary units). The
lines represent the system behavior based on theoretical
Ci2
curves and plant data. (After Ruthven (1984).)

L
z=0 z

(b) (c)

Adsorption 3
15 Desorption
100 Cyclic operation
Hydrocarbon flow rate

One cycle: 27 min


Purity (%)

Purity (%)

2 Inlet
Flow rate

10 HC

NH3
50
1
5 0 14
Adsorption (13.5 min)
Desorption
(13.5 min)
0 0
0 5 10 15 20 25 30 0 5 10 15 20 25 30
Time (min) Time (min)

(e.g. CO2) from an adsorbent (0.5 nm molecular sieve) by a Dissinger (1986). A comprehensive introduction to bed
hot purge of N2 (Kumar and Dissinger, 1986). This figure regeneration is available in the treatment of cyclic batch
shows the temperature and CO2 concentration profile processes by Ruthven (1984).
along the bed length at any instant of time. The concen- There are additional methods for regenerating an
tration of CO2 in the saturated bed is shown near the adsorbent bed beside TSA and PSA. In the case of gas/
column end (near z L), where no desorption process vapor mixtures, the introduction of a displacement purge
has been initiated yet. In the initial sections of the column gas (as opposed to an inert purge gas) into the bed serves
near z 0, where desorption is complete, the very low first the same function as in PSA, namely the reduction of
value of C i2 reflects the purge concentration of CO2, if any, the partial pressure, pi2 , which facilitates desorption of the
just as the high purge gas temperature is present. Then adsorbed gas species i. Second, there is a separate and
there is a MTZ, where CO2 concentration in the bed rises more important function: the displacement purge gas
and the temperature falls. Finally, there is an additional adsorbs strongly and competes with the adsorbed feed
MTZ further down the column which pulls down the species to be separated. Further, if the temperature of the
higher temperature and gas-phase concentration to the displacement purge gas/vapor is higher, as in the case of
values existing in the saturated column prior to purge steam stripping of activated carbon to strip organic solv-
introduction. ents or volatile organic compounds, then we have the
There have been a number of investigations into the combined effects of TSA, the reduction of pi2 and desorp-
behavior of a column subjected to desorption. Although tion via displacement.
these include studies on isothermal desorption (Zwiebel A major requirement for a successful displacement
et al., 1972; Garg and Ruthven, 1973), nonisothermal purge gas/vapor, the displacer, however, is that it should
desorption based studies are often more realistic. The not be too strongly adsorbed compared to the feed species
reader interested in detailed understanding should refer to be desorbed. If it is, then it would be difficult to desorb it
to papers by Rhee at al. (1970b, 1972), Basmadjian et al. during the adsorption part of the next cycle. In fact, ideally,
(1975a,b), Kumar and Sircar (1984) and Kumar and it should have an almost identical affinity for adsorption on
508 Bulk flow perpendicular to the direction of force

the adsorbent as the feed species to be desorbed. Then, co-ion; the ion exchange resin beads in the column to start
during the beginning of the next cycle, the species in the with are in B form and the solution being continuously fed
feed mixture which is preferentially adsorbed will displace from the column top contains essentially AY. As the solu-
the displacer adsorbed during the regeneration part of the tion keeps coming down the column, the beads at the top
cycle. Even under optimal conditions, this displacer will of the column are converted to the A form. Just as we
contaminate the preferentially nonadsorbed (raffinate) observed in Figure 7.1.5(b) how the column concentration
species/stream, leaving the adsorber during the adsorption profile developed in an adsorbent column fed continuously
part of the cycle. It is desirable therefore to use a displacer with a solution of concentration C 0i2 , similarly here the top
which is easily separated from the raffinate stream. of the column will be converted to the A form. There will
An example of such a displacer is ammonia in the also be a mass-transfer zone (MTZ, as in Figure 7.1.5(b))
large-scale separation of linear paraffins of medium where, at one end, the column is in A form and, at the
molecular weight (C10C16) from branched chain and other end, it is in B form, with a continuous transition
cyclic isomers. The adsorbent is a 0.5 nm molecular sieve, going on in between (Figure 7.1.11(b)).
which, at 550600  F and slightly above atmospheric pres- What would be of interest here is to find out when the
sure, adsorbs the linear paraffins strongly from the feed feed ionic species A is going to break through from the
mixture (Figures 7.1.10(b) and (c)). Such a high feed tem- column end, as shown in Figure 7.1.11(c). As in Figure
perature precludes a TSA process for desorption lest it 7.1.5(a), the actual breakthrough curve is going to be dif-
should lead to cracking/coking. Ammonia is, therefore, fuse, shown by the dashed line in Figure 7.1.11(c). How-
used to desorb these paraffins. A higher ammonia flow rate ever, we will calculate the time corresponding to a step
is required since it is adsorbed somewhat less strongly than jump in the column outlet concentration of ionic species
the paraffins (Ruthven, 1984), but it is easily separated A (solid line in Figure 7.1.11(c)) in the manner of Figure
from the paraffins due to its high volatility after cooling 7.1.3(b). The governing equation for the concentrations of
the mixture and flashing it out via distillation. Another ionic species A in the two different regions, mobile phase
displacer for the system is n-hexane, which also possesses ( j 2) and the ion exchange resin phase ( j 1), namely
a high volatility with respect to the other species in the C A2 and C A1 , respectively, will continue to be equation
system. A brief introduction to these and related processes (7.1.4)):
has been provided by Ruthven (1984). An introduction to
the problems of modeling in such multicomponent C A2 1 C A1 C A2 2 C A2
vz DA;eff;z :
systems is provided in Wankat (1990, pp. 394400). t t z z 2
Displacer based desorption is practiced more often in An item to be noted here is that, even though the resin is
liquid-phase processes, particularly in ion-exchange resin swollen and highly porous (p 6 0), the ions inside the ion
based processes. Conventional adsorption based liquid- exchange resin beads are essentially next to the fixed
phase processes are not particularly suitable for displacer charges in the ion exchange resin beads. Therefore, the
based processes. Further, displacer based processes are concept employed to develop equation (7.1.13e) is not
invariably used in multicomponent separations (Rhee valid here; we therefore employ, de facto, p 0.
and Amundson, 1982) and are properly considered under The analysis carried out earlier to arrive at the
chromatographic processes (see Section 7.1.5) (Frenz and concentration wave velocity expression (7.1.12a) for vCi is
Horvath, 1985). also valid here. Therefore, for equilibrium nondispersive
operation of an ion exchange column, we can write, for
7.1.1.4 Ion exchange beds ionic species A,
vz
To remove a particular ion from an aqueous solution, a vCA h i: 7:1:38
b q0 A C A2
column containing ion exchange resin beads is often used, 1
especially if the ion is present at low concentrations. The
ion exchange bed/column may also be used to replace one What we will do now is to deliberate on the nature of
particular ion in solution by another ion. The resin beads the ion exchange equilibrium behavior of a given ionic
are typically introduced in a vertical cylindrical column species, say A. However, note what the quantity
and are supported from the bottom; the top section is free b qA C A2 =C A2 b q0 A C A2 used for adsorption of
so that, as the resins expand, there is room for expansion; solute A stands for here (see notation in equations
this is operationally important since resin beads swell in (3.3.121fn)):
water. Figure 7.1.11(a) illustrates a column filled with ver- qA C A2 qC A2 fR qC A2 g
tical resin beads. b R 1 1
C A2 C A2 C A2
Consider the following process scenario. The ion
C AR
exchange resin beads are to participate in an ion exchange 1 : 7:1:39a
C A2
process between two counterions A and B, with Y being the
7.1 Force ri in phase equilibrium: fixed-bed processes 509

(a) (b)
CA2
Feed
Feed

A A A A
A A A A Column
A A A A
A A A A
Exhausted
A A A A part of the
A A B A column

A B B A
B B B B
B B B B Mass-transfer
zone
B B B B
B B B B
B B B B

Unconverted
Treated Resin Treated part of the
solution beads solution column

(c) Diffuse

0
CA 2

Sharp
(shock wave)

0
t t br
V

Figure 7.1.11. Mass-transfer/concentration profiles in an ion exchange column: (a) sections of the column where it is in A form, B form or
in mixed AB form; (b) concentration profile of the A form of the resin in the column at any time; (c) breakthrough behavior at the column
bottom, sharp breakthrough and diffuse breakthrough.

However in view of relations (3.3.121g) for the resin phase Here we define the system in terms of the binary mixture
and (3.3.121h) for the external solution phase, we can of counterions A and B and ignore the presence of
express the resin phase and the external solution phase any co-ion Y on other ions. Therefore, the concentration
concentrations in terms of mole fractions x AR and x Aw in wave velocity vCA of ionic species A through the column is
the resin and water phase, respectively:5 given by (for a given concentration, say C Aw or mole frac-
tion x Aw )
C AR C AR C Aw C Aw
x AR ; x Aw : vz
C AR C BR C FC C Aw C Bw C tw vCA  : 7:1:40a
7:1:39b 1 C FC x AR
1
C tw x Aw

5
The corresponding expression for vCA jshock may be
The bar on top of any quantity refers to a quantity averaged over
the column cross section. obtained from a mass balance (as in equation (7.1.15c)):
510 Bulk flow perpendicular to the direction of force

vz Wankat (1990, pp. 465470) for removal of K from a solu-


vCA 
1 C FC x AR tion using an ion exchange bed in the form of Na. Langmuir

1 isotherm describes this ion exchange with KAB 1.54, where
C tw x Aw
vz A K and B Na. The mole fraction of potassium in the
 : 7:1:40b 0.2 N feed solution x Kw 0:7. Determine the concentration
1 C FC x AR jshock x AR jinitial
1 wave velocity of K and determine when this shock wave
C tw x Aw jshock x Aw jinitial
exits the column of length 30 cm and diameter 2 cm if the
For an initially empty column, x AR jinitial x Aw jinitial 0. volumetric flow rate of the solution is 20 cm3/min.
(Note: In determining dx AR =dx Aw , the overbar has no Next, regeneration of the bed is implemented by passing a
solution of 0.2N NaCl. Calculate the K concentration in the
relevance since the equilibrium relation is independent of
effluent as a function to time (eluent volume). You are given:
column cross section. However, there may be variations 0.4, C FC 3.96 equivalent/liter of resin.
along the cross section of x Aw , therefore x Aw is relevant.) Solution To employ equation (7.1.43c) for the shock wave
In such a context, the Langmuir ion exchange isotherm velocity vCA jshock , we need vz, the interstitial velocity:
(3.3.121n) for counterion A may be written as
cm3 2 20
qAR C AR K AB C Aw =C tw 20 d  vz   4cm2  vz  0:4 ) vz ;
x AR : min 4 4  0:4
qmax R C FC 1 C Aw =C tw K AB 1
15:9cm=min
7:1:41 vz 15:9cm=min ) vCA jshock 2 0 10 1 3:
41 @0:6A @3:96A x AR 5
Note that here the counterionic species under consider- 0:4 0:2 x Aw
ation along with counterion A is counterion B, so that
(compare (3.3.121g,h)) For the Langmuir isotherm, (7.1.41),

C AR C BR C FC ; C Aw C Bw C tw ; 7:1:42a K AB x Aw 1:54  0:7 1:078


x AR ) x AR 0:782:
1 x Aw K AB 1 10:70:54 1:378
x AR x BR 1 ; x Aw x Bw 1; 7:1:42b
Therefore
and the equilibrium constant K AB for the ion exchange
15:9 cm=min
process between counterions A and B is vCA jshock
1 1:5  18  0:782=0:7
C AR C Bw 15:9 cm=min
K AB : 7:1:42c 0:465 cm=min vCK jshock :
C Aw C BR 34:18
Since the column length is 20cm, this shock wave will appear
From isotherm (7.1.41), one can calculate x AR =x Aw for
at the column end at 43 20=vCK jshock minutes. In a
the Langmuir isotherm as
process to remove K from the solution, the feed solution
dx AR K AB flow will be stopped somewhat earlier so that the effluent
: 7:1:42d solution from the column is K-free.
dx Aw 1 x Aw K AB 12
We will now focus on the regeneration of the bed by the
Following the column mass balance based development of 0.2 N NaCl stream at t 43 minute. If we employ equation
(7.1.15c), one can determine the concentration wave vel- (7.1.17g) for z 0 C i2 0 for C K2 (at the column inlet), then
ocity of the shock wave at time t when the exit concentration
dC K1
 
1
jumps from C i2 0 to C 0i2 , the incoming concentration: V V 0 Sc z 1
dC K2
L vz
vCi vCi jshock  C FC dx AR
 
0 : 7:1:43a 1
t 1 C i1 Sc z 1 :
1 0
C tw dx Aw
C i2
We know that V 0 43 minute  20 cm3 =min 860 cm3 .
The corresponding quantities in the ion exchange process Now
for the counterion A are K AB
L vz dx AR =dx Aw
vCi jshock  7:1:43b 1 x Aw K AB 12
t

1 C AR
1 K KNa 1:54
C Aw :
1 x Kw K KNa 12 1 x Kw  0:542
for an initially empty column. Therefore
L vz The results in Table 7.1.3 illustrate the diffuse nature of the
vCi jshock  : 7:1:43c
t 1 C FC x AR K-containing stream exiting the column. The book by Wan-
1
C tw x Aw kat (1990, pp. 463494) should be read for greater details of
such equilibrium nondispersive calculation procedures for
Example 7.1.3 Using the equations developed above, we ion exchange columns. See Sherwood et al. (1975) for models
will now illustrate an ion exchange system worked out by involving mass-transfer resistance.
7.1 Force ri in phase equilibrium: fixed-bed processes 511

Table 7.1.3. Dry air out


 
1 C FC dx AR
Sc z 1 V V 0 cm3
x Kw dx AR =dx Aw C tw dx Aw

0.7 0.81 630


0.5 0.95 734
0.3 1.14 850
0.1 1.4 1045
0.0 1.54 1174 Adsorbent Adsorbent
bed B bed A

Wet air out


7.1.2 Pressure-swing adsorption process for
gas separation
The fixed-bed processes studied earlier use adsorbents
which are mass-separating agents. These mass-separating
agents have to be regenerated for reuse. An introduction to Solenoid valve
the process of fixed-bed adsorbent regeneration was pro-
vided in Section 7.1.1.3. The bed may be regenerated by a
temperature increase, a pressure decrease, a purge stream, Wet air in
a displacer stream or various combinations of these. In an
actual fixed-bed separation process, the adsorption step Figure 7.1.12. Continuous adsorption drying system using
and the desorption step are combined in a cyclic fash- pressure-swing adsorption. (After Skarstrom (1960, 1975).)
ion. The manner in which these cycles are operated has led
to a few distinct classes of fixed-bed processes. The nature
of the introduction of the feed (semicontinuous, pulse, throttled down to essentially atmospheric pressure and
temperature programmed, etc.) is integral to the develop- introduced into bed B from the opposite end (i.e. the exit
ment of such distinct processes. However, the direction of end of bed A). This low-pressure dried air acts as a purge to
the force between the fluid and the fixed bed in the desorb the moisture from bed B, which has already been
following processes always remains perpendicular to the saturated with H2O during a previous cycle. The resulting
bulk fluid flow direction: wet air is taken out through the front end of bed B and
discharged to the atmosphere.
(1) thermal-swing adsorption;
This process is continued for 3 minutes, by the end of
(2) pressure-swing adsorption (PSA);
which bed A is saturated with moisture and bed B is essen-
(3) inert purge;
tially dry and regenerated. Then the solenoid valve con-
(4) potential-swing adsorption;
nection is changed. Wet air is introduced at 40 psig into
(5) parametric pumping;
bed B by a connection in the solenoid valve, shown in
(6) cycling zone adsorption;
Figure 7.1.12 as dashed lines. This wet air leaves bed B at
(7) chromatographic processes.
the other end dry. Half of this exiting dried air is taken out
We focus first on pressure-swing adsorption processes. as dry product at 40 psig; the other half is throttled down to
Potential-swing adsorption is mentioned in passing. In near atmospheric pressure and sent to the previous prod-
the three sections that follow, we consider parametric uct end of bed A, whose pressure has now been quickly
pumping, cycling zone adsorption and chromatographic reduced. This dry air, acting as a purge, flows through bed
processes. Thermal-swing adsorption and inert purge have A in the opposite direction and exits at the other end as wet
already been briefly considered. air through the connection, indicated by the dashed lines
The basic mode of operation in pressure-swing adsorp- of the four-way solenoid valve. It is released to the atmos-
tion will be illustrated first using the Skarstrom cycle (Skar- phere. This process continues for another 3 minutes, when
strom, 1960, 1975) employed for air drying (Figure 7.1.12). bed B becomes saturated and bed A is regenerated. At this
The process uses two identical beds of a granular solid time, the cycle is complete, and wet air is sent to bed
adsorbent which preferentially adsorbs moisture from air. A ready for adsorption as before, while bed B has to be
Wet air at 40 psig enters bed A through the connection regenerated. Meanwhile, dry air at 40 psig is obtained con-
shown by a solid line in a four-way solenoid valve. The tinuously as the product.
air leaving the other end of bed A is essentially dry air at Such a process was originally known as heatless
40 psig. About half of the wet air entering is taken off as dry fractionation or heatless adsorption since no heat was
air product; the other half, also produced as dry air, is used to regenerate the adsorbents. Currently the term
512 Bulk flow perpendicular to the direction of force

pressure-swing adsorption (PSA) is used instead. Skarstrom by 1 and 2, respectively. We provide first the cycle descrip-
(1975) demonstrated that air containing 3800 volume ppm tion according to Chan et al. (1981).
of H2O (11 mm Hg H2O partial pressure) at 40 psig was dried A high-pressure feed mixture is continuously supplied
to 1 ppm H2O at the same pressure. However, it took some to the top of the bed 1 in step 1 and the adsorbable species
time (5 days) to achieve a steady state in the moisture are taken up by the bed. The effluent from the end of the
concentration in the dried air output. In modeling PSA bed is sufficiently purified and still at a high pressure.
processes, therefore, steady state achievement is usually A part of this effluent becomes the desired product; the
indicated by n ! , where n is the number of cycles. pressure of the remaining effluent is reduced without any
Since the original patent of Skarstrom was issued in 1960 change in mole fraction to the low-pressure level of bed 2,
(Skarstrom, 1960), there has been considerable progress in where it is introduced to the column top as a low-pressure
the research and development of PSA processes. A large purge to desorb species adsorbed by bed 2 during an
number of commercial plants based on PSA are used to purify earlier cycle. Figure 7.1.13(b) shows the essentially con-
or fractionate a variety of gas mixtures and vapor mixtures. stant bed 1 pressure and steady flow rates of high-pressure
These include: drying of air and other gases; air separation by feed and product and low-pressure purge during step 1
zeolites (N2 is more strongly adsorbed than O2 at equilibrium) (Weaver and Hamrin, 1974).
and by carbon molecular sieves (O2 more rapidly adsorbed, After some time, bed 1 approaches breakthrough con-
therefore the high-pressure product is a highly purified N2 ditions; step 2 is therefore initiated, wherein bed 1 is depres-
stream); H2 purification to very high levels (~99.9999%) from surized and there is removal of the adsorbed species in the
various refinery streams; adsorption removal of n-paraffins blowdown stream from the end of the bed where the high-
from mixtures containing branched-chain isomers and cyclic pressure feed was introduced earlier. Simultaneously, bed
hydrocarbons, etc. (vacuum desorption is used for n-paraffins 2, ready for adsorption, is pressurized by the high-pressure
with carbon numbers less than 10). The adsorbent particle feed gas mixture. Step 2 is of short duration, and is followed
diameter varies between 1/1600 pellets to as low as 60 mesh. by step 3, in which bed 2 is fed with high-pressure feed gas
The number of beds used simultaneously is often more than continuously. In step 3, bed 2 behaves as bed 1 in step 1, just
two, going all the way up to ten. The cycle time can go as low as bed 1 behaves in step 3 as bed 2 did in step 1. Similarly,
as a few seconds, especially if a single-bed based rapid PSA is in step 4, bed 1 is subjected to pressurization with
used (Keller, 1983). For feed gases slightly above atmospheric high-pressure feed (as bed 2 was in step 2); while bed 2
pressure, vacuum-swing adsorption has been used to desorb undergoes blowdown prior to complete regeneration by a
via vacuum (Sircar and Zondlo, 1977). An excellent treatment low-pressure product purge (as in step 1)
of PSA based processes is available in Yang (1987), which also A model developed by Chan et al. (1981) for the purifi-
provides a critical account of PSA models and experiments. cation of a binary gas mixture of species A and B by such
Before we consider PSA models, one should recognize a PSA cycle when A is present at a trace level will be
that separation by adsorption in PSA can take place in two presented below. Focus on bed 1s operation. The assump-
general ways: equilibrium separation and kinetic separation. tions employed in this model are:
In equilibrium separation, the adsorbent preferentially
(1) isothermal operation;
adsorbs one or more species in preference to others on an
(2) negligible pressure drop along the bed during high-
equilibrium basis. In zeolite adsorbents, as well as molecular
pressure feed flow or low-pressure purge flow;
sieves, molecules having an appropriately small dimension
(3) gassolid equilibrium exists always and everywhere;
can enter the zeolite pores and are adsorbed, whereas others
(4) linear adsorption isotherms are valid, with species
cannot enter the pores due to steric effects and are excluded.
A being preferentially adsorbed:
Such adsorption processes are considered under equilib-
rium separation processes (Yang, 2003). On the other hand, A1 > B1 : C A1 A1 C A2 7:1:44a
adsorption separation by kinetic effects involves adsorbents
through whose pores one of the species diffuses much more C B1 B1 C B2 ; 7:1:44b
readily than others (e.g. O2 diffuses 30 times faster than N2
(5) no axial dispersion;
through carbon molecular sieves having the right pore
(6) ideal gas law is valid;
opening dimensions). Such separations come under a kin-
(7) the interstitial gas velocity vz is constant during
etic separation mechanism in PSA processes.
constant-pressure steps 1 and 3.
Any model for PSA should provide ways of calculating the
7.1.2.1 An equilibrium nondispersive PSA model
fractional recovery of the less adsorbed species, its purity,
Consider the two columns or two beds used in Skarstroms the purity of the strongly adsorbed species dominated frac-
scheme for air drying. A simple PSA cycle for this tion, the minimum bed length, etc. We follow to these ends
process involves four steps, shown schematically in Figure the treatment by Chan et al. (1981) and focus on steady state
7.1.13(a), where two beds, bed 1 and bed 2, are identified operation obtained after initial transients. The governing
7.1 Force ri in phase equilibrium: fixed-bed processes 513

(a)
Step 1 Step 2 Step 3 Step 4
High-pressure feed Blowdown Low-pressure purge Pressurization with feed

1 1 1 1

Product Product

2 2 2 2

Low pressure Pressurization High pressure Blowdown


purge with feed feed

(b)
Bed 2 Bed 1
Step # 1 2 3 4 1 2 3 4

High

Pressure
Low
0 0

Feed
flow rate

0 0

Product
flow rate

0 0

Purge
flow rate

0 0

Figure 7.1.13. (a) Four steps in a PSA cycle. (b) Diagrams of pressure and flow rate changes for beds 1 and 2 during one cycle. (After
Weaver and Hamrin (1974).)
514 Bulk flow perpendicular to the direction of force

balance equations for species A and B are obtained from If the determinant in the denominator is zero, we get
equation (7.1.3) under nondispersive condition as
dz vz vz
h i: 7:1:51a
C A2 vz C A2 dt 1 A1  1 1 A1
  
C A1
1 0; 7:1:45a
t z t
For y=t to be finite, the numerator has to be zero,
C B2 vz C B2
  
C B1 which leads to
1 0: 7:1:45b
t z t
d ln y d ln P
Recall that C A2 and C B2 refer to (z, t)-dependent molar vz 1 A1 B1 :
dz dt
concentrations of species A and B in the gas phase, while
C A1 and C B1 refer to those in the solid adsorbent phase, as Using (7.1.51a), this may be rearranged to yield
before. Replace ig by yi here. Due to ideal gas behavior, d ln y 1 A1 B1 d ln P d ln y d ln P
 

) A 1 ;
dt 1 A1  dt dt B dt
C A2 pA =RT PyA =RT ; C B2 pB =RT PyB =RT ;
7:1:51b
7:1:46
where
where P is the total pressure and y A and yB are the local
(z, t)-dependent mole fractions of species A and B in the
A ; B ; A < B : 7:1:52a
gas phase. Since species A is present in very dilute concen- 1A1 1B1
tration, we may assume that y B 1 and represent y A
Note that, if the species are strongly adsorbed, i.e.
henceforth only by y. Substituting these into equations
1A1 >> and 1B1 >> , then the separation factor
(7.1.45a) and (7.1.45b) (where we now use the equilibrium
AB for species A and B is
relations (7.1.44a) and (7.1.44b)) leads to
A1 B 1
Py vz Py Py AB : 7:1:52b
 
1 A1 0; 7:1:47a B1 A
t z t
Since the interstitial velocity vz is constant during steps 1
P vz P P
 
1 B1 0: 7:1:47b and 3 by assumption (7), the characteristic velocity of any y
t z t
value, ( dz=dt), is also constant from (7.1.51a) since A1 is
Since P=z is zero by assumption in either bed/column constant. On the other hand, vz varies during steps 2 and 4;
at any time, we get the pressure also changes, just as mole fraction y also
    changes. The interstitial velocity during steps 1 and 3
P y y vz P y may be obtained by dividing the volumetric flow rate by
y P vz P Py 1 A1 y P 0;
t t z z t t SC , where SC is the column cross-sectional area.
7:1:48a We will now calculate the changes in the position of

P vz

P the characteristics for these two kinds of changes. For steps
P 1 B1 0: 7:1:48b 1 and 3, integrate (7.1.51a) and obtain, respectively,
t z t
zH A vzH t; 7:1:53a
Now multiply equation (7.1.48b) by y and subtract it from
equation (7.1.48a) to obtain
z L A vzL t: 7:1:53b
y y ln P
1 A1  vz 1 A1 B1 y : Here zH and zL , called the penetration distances, are the
t z t
7:1:49a net displacements of the concentration wave front in the
high-pressure feed step 1 (subscript H), having a steady
To use the method of characteristics (see equations interstitial gas velocity vzH , and the low-pressure purge
(7.1.12b) to (7.1.12f)), we can write any solution of y as step 3 (subscript L), having a steady interstitial gas velocity
 
y
 
y vzL , respectively.
dt dz dy 7:1:49b For the blowdown (step 2) and pressurization with feed
t z
(step 4), consider equation (7.1.51b) first. This allows the
to obtain gas mole fraction y to be related to pressure P in the
  column in two cases: as the high pressure, PH, is reduced
 dy dz 

d ln P very quickly to a low pressure, PL, during blowdown step 2,
 1 A1 B1 y
 
vz  y is increased from yH to yL; as the low pressure, PL, is
   dt
y 
increased to PH during the pressurization with feed in step
  7:1:50
t 
 dt dz 
 3, y is decreased from yL to yH. Integration of equation
 1 A1 vz  (7.1.51b) leads to estimates of such changes:
 
7.1 Force ri in phase equilibrium: fixed-bed processes 515

(a) (b)
Pressurize Feed Blowdown Purge Pressurize Feed Blowdown Purge

A G A
1.0 1.0
B zH D
0.8 0.8
F zL
0.6 D 0.6
z/L yH = y f z/L yH = yf
E E
0.4 C 0.4 B
yH = yH (z)
yH = yH (z)
0.2 0.2
C
0.0 0.0
t0 t1 t2 t3 t4 t0 t1 t2 t3 t4
t t

Figure 7.1.14. Characteristics movement during


h a PSA cycle
i for the adsorption of a binary
h mixture of
i A and B very dilute in A; G Gcrit,
L=zL 2, PH =PL 10. (a) 0:05, 1 PH =PL  L zH < 0; (b) 0:4, 1 PH =PL  L zH  0. Reprinted from Chem.
Eng. Sci., 36, (1981), p. 243, Y.N.I Chan, F.B. Hill, Y.W. Wong, Equilibrium theory of a pressure swing adsorption process, copyright
(1981), with permission from Elsevier.

y H =y L P H =P L 1 ; 7:1:54a Rearrange it as
d ln z d ln P
where : 7:1:55d
dt dt
A =B < 1: 7:1:54b Integrate to obtain
It is necessary to calculate how the characteristics position zH h . i
will change as the transition takes place rapidly from PH to ln ln P H PL ) zH =z L PH =P L ; 7:1:55e
zL
PL (blowdown) or from PL to PH (pressurization with feed).
For this purpose, we use equation (7.1.48b) in a rearranged where zH and zL are the z-coordinates of the characteristics
form, when the pressures are PH and PL, respectively. Obviously,
the absolute value of z at the end of any such process will
1 B1 ln P
 
vz
; depend on the z-coordinate at the beginning of the pro-
z t cess, which will be influenced by prior steps.
and integrate with respect to z (since the right-hand side is It is useful now to calculate the net displacement of a
not a function of z, remember P=z 0) to obtain concentration front during a complete cycle of four steps in
  PSA. Assume the column/bed length to be L and that step 1
1 ln P is as shown in Figure 7.1.13(a) with the high-pressure feed
vz z constant: 7:1:55a
B t continuously entering at the top end (z L). Prior to this,
If the coordinate system of the column is such that the step 4 occurred wherein the gas (mole fraction yf, feed
closed end of the column of length L (Figure 7.1.14) is mole fraction) entered the column (z L) at a low pres-
identified as z 0, then, during the blowdown or repres- sure, PL. Gas entering the column a short time later enters
surization steps, vz 0 at z 0. at a slightly higher column pressure, and the final gas in
That the constant in equation (7.1.55a) is zero implies this step enters at pressure PH. The characteristics location
that of the initial entering gas is obtained from (7.1.55e) as
  zH LP H =P L ; 7:1:56a
1 ln P
vz z: 7:1:55b
B t where zL is assumed to be L. Based on this assumption, the
Substitute this expression for vz into the basic equation zH location is now at B. The specific characteristic path is
(7.1.51a) for characteristics movement along the column difficult to plot for this step. This gas concentration front
to get (Figure 7.1.13(a)) (now at pressure PH) moves down the
  column by zH during step 1 (high-pressure feed flow) so
dz ln P zA that the lowest z-coordinate at the end of step 1 (point C in
: 7:1:55c
dt t B Figure 7.1.14(a)) has the value
516 Bulk flow perpendicular to the direction of force

z H zH LP H =P L A vzH t: 7:1:56b This expression provides the value of the gas mole fraction
of species A in the advancing front in the pressurization
The triangular region above AD has only a feed-gas com- step 4 (Figure 7.1.5(b) illustrates how the composition
position. The gas composition changes between C and D, it decreases in the front from the feed composition to the
being highest at D ( yf). The blowdown step that follows exiting composition). When P PH Pf, step 1 begins, and
(step 2) pushes the characteristics back. The zH and zL for the characteristics are pushed further into the column
this step are still related by (7.1.55e). Since zH in this toward z 0 by an amount zH . As long as zH P H from
relation is valid at the start of this step, it must be equal (7.1.58a) is larger than zH , the feed-gas composition
to that of the location described by (7.1.56b): never appears at z 0 and therefore never breaks through
into the high-pressure product leaving the column at z 0.
zL P H =P L LP H =P L A vzH t
This condition is identified by
) z L L A vzH t P H =P L ; 7:1:56c
z H P H LP L =P H  zH A vzH t: 7:1:58d
corresponding to point E, just as point F corresponds to
starting point D. Now comes the low-pressure purge step, If, in addition, vzL =vzH  crit , then Chan et al. (1981)
which pushes the characteristics still further toward z L have indicated that, at steady state, complete removal of
by z L (equation (7.1.53b)); thus, the final z-coordinate trace component A from the high-pressure product stream
(location G) at the end of step 3 is is obtained. The same result was theoretically demon-
strated earlier by Shendalman and Mitchell (1972) for the
L A vzH t P H =P L A vzL t: 7:1:56d same system, except that species B was inert and was not
adsorbed at all.
At the start of step 4, the characteristics front was at z L
A more general index to identify the onset of complete
(the feed gas entered the column at P PH). Therefore, in
purification is Gcrit, the value of the fraction of the feed
the whole cycle, the net displacement of the characteristics
introduced during steps 1 and 3, just required as purge in
front from z L has been
order to obtain complete purification:
zjoverall A vzH t P H =P L A vzL t
h i vzL P L amount of species B in purge
Gcrit : 7:1:58e
A t vzL vzH P H =P L : 7:1:57a vzH P H amount of species B in feed

Figure 7.1.14(a) is drawn such that zjoverall 0. On the basis of (7.1.57c),


The ratio of vzL , the velocity during the low-pressure Gcrit PL =P H 1 : 7:1:58f
purge step, and vzH , the velocity during the high-pressure
feed step, is an important parameter, , in PSA: A plot of Gcrit against P H =P L for various values of is
provided in Figure 7.1.15. The fraction of feed required as
vzL =vzH : 7:1:57b
purge decreases as (1) the ratio of the high feed pressure to
If zjoverall for a cycle is zero, then a critical value of the low purge pressure increases and (2) the value of
vzL =vzH is reached: decreases, i.e. the separation factor increases. However,
    too high a ratio of PH =P L does not reduce Gcrit by much.
vzL PH The fraction of the major component (species B) fed to
crit : 7:1:57c
vzH crit PL the process which is recovered in pure form in the high-
pressure product stream at steady state (identified by sub-
This definition is valid only when t for the low-pressure
script ) is
purge step 3 is equal to that for the high-pressure feed step 1.
Consider now step 4 and then step 1. Assuming that mB;feed mB;purge
f ; 7:1:59a
the characteristics start at zL L (top of the column where mB;feed mB;blowdown
fresh feed enters) with P PL and y yf in step 4, zH by
where mB represents the number of moles of species
equation (7.1.55e) for any P is
B introduced to or leaving the process per half-cycle.
z H P LP L =P ; PL  P  PH : 7:1:58a Assuming the ideal gas law to be valid, for any column/
bed having a total volume V, cross-sectional area Sc ( V/
By equation (7.1.54a), the y value at any P and zH(P) is then L) and a void volume fraction ,
related to yf by
mB;feed V =LvzH t P H =RT ; 7:1:59b
yH P P L =P 1 yf ; PL  P  PH 7:1:58b
mB;purge V =LvzL t P L =RT ; 7:1:59c
so that
since the feed gas occupies the void volume at pressure PH,
yH z y f z=L1= 1 : 7:1:58c whereas the purge occupies the void volume at pressure
7.1 Force ri in phase equilibrium: fixed-bed processes 517

based theory. The column/bed length was given. An alter-


1.0 native design oriented goal would be to calculate the
b = 1.0 shortest length of the column, Lmin, i.e. the least amount
of adsorbent required for complete removal of species
0.8 A. This is achieved when G Gcrit (equation (7.1.58f))
and L zL . These conditions ensure that component
0.6 A is just completely purged from a column at the end of
Gcrit 0.8 purge step 3; full adsorption capacity of all adsorbents in
the column is available in step 4 onwards. An expression
0.4 for Lmin is obtained by replacing vzH in (7.1.59b) using
(7.1.57c) and (7.1.53b):
0.4
0.2 A mB;feed P H =P L
Lmin : 7:1:60
Sc P H =RT
0.0
0.0 This equilibrium based analytical theory of a conventional
0 4 8 12 16 20
PSA process predicts that the mole fraction of species A in
PH /PL the high-pressure product gas, y, will continue to decrease
with time (Shendalman and Mitchell, 1972; Chan et al.,
Figure 7.1.15. Critical purge-to-feed ratio. (After Chan et al. 1981). This is contrary to experimental observations. More
(1981).) exact theories based on axial diffusion and mass-transfer
effects are needed to predict the observed behavior.
A number of other analyses for PSA systems have been
PL. The number of moles removed during blowdown carried out. Shendalman and Mitchell (1972) studied ana-
(when pressure is reduced from PH to PL) is lytically and experimentally the PSA separation of trace
amounts of CO2 from helium, assuming that helium is
V P H P L 
mB;blowdown 1 B1 V P H P L =RT ; completely inert and a linear isotherm is valid for CO2 with
B RT
instantaneous equilibrium. Fernandez and Kenney (1983)
7:1:59d
studied air separation in a single-column PSA using a linear
which includes the gas leaving the void volume of the bed equilibrium adsorption model without any axial diffusion.
over a pressure range (PHPL) and the adsorbed gas in the Knaebel and Hill (1985) studied two-column equilibrium
pressure range (PHPL). Therefore an expression for f is PSA for a general binary system using the same framework.
Raghavan et al. (1985) numerically simulated the CO2He
V =L RT
t
vzH P H vzL P L P H =PL 1 1 system of Shendalman and Mitchell (1972) using axial dis-
f h i ;
V =L RT
t
vzH P H V PHRTPL  P H =P L 1 PPHL 1 z
L persion and finite gas-to-solid mass-transfer resistance.
B L
Bulk separation of a ternary gas mixture, H2/CH4/CO2, has
7:1:59e
been experimentally and numerically studied by Doong and
where we have used the definition for crit from (7.1.57c) to Yang (1986). Variations in the simple PSA cycle, especially
replace vzH =vzL . in the pressurization and blowdown steps, have been stud-
If the preferentially adsorbed species A has to be ied. Suh and Wankat (1987) studied a combined cocurrent
recovered in a more enriched form, the blowdown and countercurrent blowdown cycle; Liow and Kenney (1990)
the purge stream would be the source; species A mole investigated the backfill part of the cycle involving pressur-
fraction yBDPG in this combined stream may be obtained ization of the adsorption bed with the product in a direction
from opposite to that of the feed mixture. The design of PSA
  1 systems has been treated by White and Barkley (1989).
L PH PH
y BDPG zL PL 1 PL mB;feed mB;blowdown Example 7.1.4 A refinery gas stream at 10.13  105 Pa
 :
yf m B;purge mB;blowdown contains primarily H2 besides some impurities. Consider this

L PH
zL1 1
PL
as a binary mixture where the impurity species A adsorbs at
7:1:59f the gas temperature onto the porous adsorbent used
according to qA1 (gmol/g adsorbent) 40CA2 (gmol/cm3).
This ratio is identified as the enrichment of species A by
This H2 stream has to be purified by a two-bed PSA process
Chan et al. (1981). When ! 0, this enrichment increases using the above adsorbent (s 2 g/cm3); the porosity of the
linearly with an increase in P H =P L . adsorbent is 0.3. The adsorbent bed porosity is 0.4. The
The above expressions provide theoretical estimates of bed diameter in a small-scale study to be conducted is 4 cm;
the productivity and process stream quality from a PSA the high-pressure feed-gas flow rate is 0.52 liter/s. The low
process for a trace component removal by an equilibrium pressure, PL, is 1.013  105 Pa. The durations of different
518 Bulk flow perpendicular to the direction of force

steps in the four-step (Figure 7.1.13(a)) process are: pressur- But


ization, 3 s; high-pressure feed flow, 80 s; blowdown, 3 s; low- 99:95
pressure purge, 80 s. Determine the length of the bed needed z L zH 0:027zL ) 99:95 0:027z L zL ) zL :
0:973
for essentially complete purification of H2. Determine
the values of crit and Gcrit. Calculate the enrichment of The required bed length
the impurity in relation to its feed concentration in the zL 102:7cm L:
blowdownpurge. Employ the equilibrium nondispersive
PSA model of Chan et al. (1981). The critical value of the (purge/feed) ratio, crit, is obtained
from (7.1.57c):
Solution The equilibrium model of Chan et al. was formu-
lated and solved using an equilibrium relation of the type for crit vzL =vzH crit P H =P L 100:0117
impurity A: C A1 A1 C A2 . We will now convert qA1 40C A2 ) crit 1:027:
to this form using
0 1 0 1
3
g adsorbent cm of adsorbent
qA1 gmols A=g adsorbent s @ 3 A 1 p @ A
cm of adsorbent material cm3 of porous adsorbent

C A1 mols A=cm3 of porous adsorbent


40  s 1 p C A2 40  2  0:7C A2 56 C A2 :
From result (7.1.58e),
We will adopt an approximate approach. If L is assumed to
be equal to zL, then, by equation (7.1.60), 1
Gcrit P H =P L 1 10 0:9883 0:102:
9:735
Lmin A mB;feed P H =P L =f Sc P H =RTg zL :
To determine the enrichment of the impurity species in the
But from (7.1.59b), blowdownpurge stream in relation to the feed stream, we
mB;feed V =L vzH tP H =RT: use (7.1.59f):
0 10 1 0 11
Therefore L P H PH
@ A@ 1A @ A
A V =LvzH tP H =RTP H =P L z L PL PL
Lmin yBDPG
0 10 1
Sc P H =RT yf
@ L A @P H 1A 1
) Lmin A vzH tP H =P L : z L PL
But (A/B) A since H2 is assumed not to be getting 0 1
adsorbed, and 0:01176  102:7
@ A 10 1 100:9883
0:4 0:4 99:95
A 0:01176:
1 A1 0:4 0:6  56 0:4 33:6
0:01176  102:7
10 1 1
Further, t 80 s, so 99:95
520 cm3 =s 520 cm 0:012  9 9:73
vzH ;
42  0:4 cm2 1:6 s 0:012  9 1
4
0:1087 9:73
520 y BDPG =y f 8:87:
) Lmin 0:01176   80  100:01176 1:1087
1:6
This analysis ignores the second term, ((1 )p/), in the
0:01176  520  80  1:027 denominator of relation (7.1.13e). To include this term,
) Lmin cm
1:6 terms in the analysis of Chan et al. (1981) have to change.
99:95 cm: Except for the study by Fernandez and Kenney (1983), all
We could have also obtained this length by employing equa- the studies mentioned above were based on multicolumn
tion (7.1.58d) for the high-pressure feed step. However, there arrangements. Turnock and Kadlec (1971) and Kowler and
is a pressurization step ahead of this step where, by equation Kadlec (1972) initiated studies of PSA processes using only
(7.1.55e), a single bed. There are basically two steps in such a pro-
zH =zL P H =P L ) zH =z L 10 0:0117 0:973: cess, adsorption and desorption; the adsorption step con-
sists of rapid pressurization and then high-pressure feed
To account for the short length of the bed (near z L) which
flow, while desorption is achieved by depressurization and
is consumed as the pressure rises from PL to PH in 3 s, we
employ the above result: adsorbed species removal from the feed introduction end.
Short cycle times (  20 seconds) are used, and product
99:95 cm zL z H zL : can be continuously withdrawn from the opposite end in a
7.1 Force ri in phase equilibrium: fixed-bed processes 519

variety of ways using a surge tank and pressure-reducing DCV


valve. Hill (1980) used a single-column PSA to recover a
weakly adsorbed impurity. A somewhat different single-
column PSA process for oxygen production has been
described by Keller (1983).
Reference Outlet streams
The PSA processes mentioned so far generally produce a enriched and
electrode
high-pressure product stream purified of the species strongly depleted in
adsorbed. Further, the models employed assume selectivity organic
between the different species generated by different equilib- component
rium relationships (see (7.1.52a) and (7.1.52b)). There are
PSA processes already developed or being developed that
are based on differing rates of diffusion of gases through Granular
molecular-sieve carbons (MSCs). For example, oxygen dif- activated
fuses very rapidly through MSCs compared to nitrogen, carbon
although their equilibrium uptakes are very similar. This
Direction of
property is being exploited in a few PSA processes; see Yang Ion current flow
(1987) for an introduction. Schork et al. (1993) have provided exchange
a model for such a process. membrane
We have in this section used porous solid adsorbents to Electrical
adsorb selectively specific gas species from the feed gas terminal
stream and then go through a cycle of desorption followed
by adsorption in PSA processes. Absorptiondesorption of
Organic-containing
specific gas species from a feed gas stream vis--vis an inlet stream
absorbent liquid has been carried out also using a porous
hollow-fiber membrane based rapid pressure-swing Figure 7.1.16. A suggested design of a single cell of two layers of
absorption (RAPSAB) process (Bhaumik et al., 1996). In this carbon particles separated by an ion exchange membrane for
cyclic separation process, a well-packed microporous potential-swing adsorption. Reprinted, with permission, from
hydrophobic hollow-fiber module was used to achieve non- Figure 5 of R.S. Eisinger and G.E. Keller, Electrosorption: a case
dispersive gas absorption (see Sections 3.4.3.1 and 8.1.2.2.1) study on removal of dilute organics from water, Env. Progr., 9(4),
from a high-pressure feed gas into a stationary absorbent Nov., 235 (1990). Copyright [1990] American Institute of
liquid on the shell side of the module during a certain part of Chemical Engineers (AIChE).
the cycle, followed by desorption of absorbed gases from the
liquid in the rest of the cycle. The total cycle time was varied
from 20 s upwards. Separation of mixtures of N2 and CO2 adsorption of ethylenediamine (EDA) on activated carbon
(around 10%), where CO2 is the impurity to be removed, from an aqueous alkaline brine at negative potentials of
was studied using an absorbent liquid such as pure water upto 1.0 volt; the potential was then changed to 0.0 volt or
and a 19.5% aqueous solution of diethanolamine (DEA). positive to desorb the EDA into a suitable aqueous solution
Three RAPSAB cycles studied differ in the absorption part. such that the solution will be highly concentrated in EDA.
Virtually pure N2 streams were obtained with DEA as An engineering assessment of this concept, also called
absorbent, demonstrating the capability of bulk separation electrosorption, was carried out by Eisinger and Keller
to very high levels of purification. (1990) using flat cells: a single cell consisted of two layers
of granular activated carbon separated by an ion exchange
membrane (Figure 7.1.16). Organic-containing inlet
7.1.3 Potential-swing adsorption
streams flow through each carbon layer. The carbon on
Sometimes, the amount of species adsorbed from a solu- one side of the membrane is held at the adsorbing poten-
tion onto an adsorbent, e.g. activated carbon, depends on tial (1.0 volt) while the other side is at the desorbing
the electrical potential applied to the adsorbent: the potential (0 volt). When the carbon on the adsorbing side
amount adsorbed at equilibrium depends on the potential approaches saturation with EDA, and therefore break-
applied. This equilibrium phenomenon of potential through is imminent, the potentials are switched: the
dependent adsorption could provide a basis for potential- adsorbing side becomes the desorbing side, and vice versa.
swing adsorption. Eisinger and Keller (1990) have utilized At the same time, the outlet stream switching valves would
such a phenomenon (see Zabasajja and Savinell (1989) for be activated. Note that the electrical potential in the carbon
electrosorption of alcohols on graphite surfaces and refer- particles is generated by a current flow transverse to the
ences to earlier studies) to develop a potential-swing carbon layer thickness via electrodes on the two sides. The
adsorption process. They have experimentally studied electrical field here merely generates the requisite
520 Bulk flow perpendicular to the direction of force

electrical potential on the adsorbents. The economic feed, and the bottom reservoir has a solute concentration
advantages and technical uncertainties of this concept less than that in the initial feed. More cycles lead to a
have been considered. Scale-up problems have also been higher solute enrichment in the top reservoir and greater
identified. Desalination of water by a similar technique, solute depletion in the bottom reservoir.
called capacitive deionization (CDI), is being studied A very high degree of separation can be achieved after
(Farmer, 1995; Farmer et al., 1996). Here two carbon elec- multiple cycles, limited only by axial dispersion and finite
trodes kept at 1.3 volts apart are used during ion collection. mass-transfer rate between the mobile and fixed phases.

7.1.4 Parametric pumping 7.1.4.1 A nondispersive equilibrium theory: batch


Parametric pumping is a cyclic fixed-bed process in which operation
the axial fluid flow direction and the driving force direction The earliest nondispersive linear equilibrium based theory
(perpendicular to the axial flow direction) are changed of parameteric pumping was developed by Pigford et al.
periodically and synchronously throughout the bed in (1969a). A number of others have since appeared (Aris,
the presence of reflux6 of the fluid at one or both ends of 1969; Gregory and Sweed, 1970; Rhee and Amundson,
the bed (Sweed, 1972). The most common driving force 1970; Chen and Hill, 1971). We illustrate here the model of
is ri created by a difference in temperature between parametric pumping developed by Pigford et al. (1969a) for
the mobile phase and the stationary phase. Other driving a batch system. The model has the following assumptions.
forces used include a pH difference or a partial pressure
difference. Some of the earliest descriptions of parametric (1) There is perfect local equilibrium between the solid
pumping appear in Wilhelm et al. (1966, 1968). and the fluid phases.
Consider Figures 7.1.17(a) and (b), illustrating the batch (2) A linear equilibrium relationship exists that depends
operation of a thermal parametric pump in the direct mode. on bed temperature.
To start with, the packed bed is filled with the feed liquid (3) No axial dispersion or diffusion occurs, and there is no
containing an adsorbable species i. The bottom reservoir is variation in the radial direction.
also filled with the same feed fluid, whereas the top reser- (4) There is an instantaneous change in bed temperature
voir is essentially empty. The cyclic process was initiated and velocity direction in a square-wave pattern, as
earlier by suddenly heating the fluid and the solid in the shown in Figure 7.1.17(c).
packed bed to Thot via the jacket and simultaneously forcing (5) During each half-cycle, the interstitial velocity and
the fluid to flow upward into the top reservoir in a synchron- temperature have the same value throughout the
ous fashion (Figure 7.1.17(c)). As this figure shows, the length of the bed.
velocity and temperature at any point in the bed remain Employing assumptions (3) and (5) in equation (7.1.3), the
constant for a time equal to half the cycle if a square wave is mass balance equation for species i is obtained as (j 1,
used (Pigford et al., 1969a). Generally, the amount of species i solid phase, j 2, mobile phase):
retained by the adsorbent is reduced as the temperature is
increased. Further, the bottom reservoir has the same con- C i2 C i2 C i1
vz 1 0: 7:1:61
centration as that in the packed bed. The fluid flowing into t z t
the top reservoir, however, is enriched or concentrated in If s is the density of the solid particles (mass/volume) and
species i, compared to conditions where the bed is colder. qi1 represents the moles of species i per unit mass of solid
During the next half of the square wave cycle, the fluid particles, then
from the top reservoir is pushed downward into the packed
bed, which is suddenly cooled to Tcold; the colder fluid is C i1 qi1 s : 7:1:62a
now pushed into the bottom reservoir with the same
If y i (instead of i2 for phase 2) represents the moles of
speed. Since the adsorbent particles are cold, their adsorp-
species i per mole of fluid, and f m stands for the number
tion capacity is much higher, thus purifying the fluid of
of moles per unit fluid volume, then
solute i. The fluid coming into the bottom reservoir is
substantially purified of species i. This cycle is ended when C i2 yi f m : 7:1:62b
the volume of fluid introduced into the bottom reservoir
during the second half of the cycle is equal to that removed The linear equilibrium assumption (2) is used to relate qi1
from the same reservoir during the first half of the cycle. to y i by
A new cycle is then initiated. Note that the top reservoir qi1 ai T yi ; 7:1:63
now has a solute concentration more than that in the initial
where ai T is the local temperature-dependent species i
equilibrium constant. Introducing definitions (7.1.62a) and
6
Reflux is considered in general in Chapter 8. (7.1.62b) into (7.1.4.1) leads to
7.1 Force ri in phase equilibrium: fixed-bed processes 521

(a)

Driven
piston (b)

Q Q Q Q

Packed
bed of
adsorbent Q Q Q Q
particles
Heating
and
cooling
jacket
Q Q Q Q

Heating Cooling
half-cycle half-cycle

Driving
piston

(c)
Tmean
Temperature

Thot

Tcold

v0
Velocity

0
v 0

0 0.2 0.4 0.6 0.8 1.0


Fraction of cycle

Figure 7.1.17. (a) Diagram of column for parametric pumping in direct mode. (b) Heat flow and fluid flow direction during two half-
cycles (Wilhelm et al., 1968). Reprinted, with permission, from I & EC Fund., 7(3), 337 (1968), Figure 3. Copyright (1968) American
Chemical Society. (c) Velocity and temperature at a point in the bed as a function of time.
522 Bulk flow perpendicular to the direction of force

y i y q temperature, qi1 decreases and y i increases because of


f m vz f m i 1 s i1 0: 7:1:64
t z t reduced adsorption capacity; hence Ai t is reduced from
A0i , the value at the mean temperature (Tmean), leading to a
Introduce now the equilibrium relation (7.1.63) into the
negative sign before ^a i sqt . Introduction of these
above equation. This results in
square-wave variations into the two governing equations
  
y y dAi T T (7.1.69) and (7.1.70) gives (here v0z v0 )
1 Ai T i vz i y ; 7:1:65
t z dT t i
dz v0z sqt v0 sqt
0 7:1:73
where dt 1 Ai ^a i sqt 1 bi sqt

1 s ai T and
A i T 7:1:66
f m
d ln y i d ln 1 Ai T d ln 1 bi sqt 
: 7:1:74
is a modified equilibrium constant. dt dt dt
To use the method of characteristics (equations Equation (7.1.73) provides the velocity of propagation of
(7.1.12b)(7.1.12f)), write any yi through the bed and represents the characteristics.
    During the half-cycle when the bed is hot (T Thot), the
y i y i
dt dz dyi 7:1:67 concentration wave velocity is given by
t z
 0 
to obtain dz v
T T hot ; : 7:1:75a
 dt 1 bi
 dy i dz 




   y T  dAi T v  During the half-cycle when the bed is colder (T Tcold),
 
yi i t dT z
  : 7:1:68 the concentration wave velocity is given by
t dt dz 

  0 
 1 A i T v z 
  dz v
T T cold ; : 7:1:75b
dt 1 bi
If the determinant in the denominator is zero, for y i =t
Focus now on Figure 7.1.18, which illustrates the charac-
to be finite the determinant in the numerator has to be
teristics lines for the process initiated by upflow of the
zero. These two determinants with a zero value lead,
feed fluid in the column to the top reservoir while the
respectively, to
bed temperature is instantaneously raised to Thot. The feed
dz vz fluid species concentration in equilibrium with the bed
7:1:69
dt 1 Ai T adsorbents at this temperature is C 0i2 and the correspond-
ing yi value is y 0i . Since this fluid is in equilibrium with the
and
adsorbents, as this concentration moves along the zt
d ln yi T dAi T characteristics, it remains unchanged during the half-cycle
 
vz : 7:1:70
dz t dT 0 < t < . Those characteristic lines which cross z L,
the column length, discharge a hot fluid of y i equal to y0i
Use of equation (7.1.69) in (7.1.70) yields into the top reservoir. Denoting by hy iT in the mean com-
d ln yi
 
T 1 dAi T d ln 1 Ai T position of the fluid entering the top reservoir from the
: column during the nth cycle, it would appear from
dt t 1 Ai T dT dt
7:1:71 the figure that only the characteristics in region (A) of
Figure 7.1.18 enter the top reservoir during the first hot
Equations (7.1.69) and (7.1.71) represent, respectively, the period (n 1) 0 < t < . All characteristics in the
characteristics and yi variation along the characteristics. column, however, have y i y0i . Therefore
The square-wave mode of variation of fluid velocity
and bed temperature (and therefore equilibrium constant hy iT i1 y0i : 7:1:76
Ai t ) may be represented (via assumption (4)), respect- Consider the next half-period < t < 2, when the bed
ively, by (Figure 7.1.17(c)) temperature and the fluid temperature have been lowered
vz vz t v0z sqt v0 sqt 7:1:72a to Tcold. The fluid entering the top of the column from the
top reservoir will now have the composition y 0i , and their
and characteristics with changed slopes lie in region (C) of
Ai t A0i ^a i sqt : 7:1:72b the zt diagram. These characteristics do not cross z 0,
the column bottom. The characteristics in region (D) do.
When the bed is heated, the fluid flows upward through These characteristics originated earlier from z 0 when
the bed into the top reservoir. Due to the higher T Thot during 0 < t < and therefore had a value of
7.1 Force ri in phase equilibrium: fixed-bed processes 523

yi T1 = yi0 yi T2 yi T 3

e a c d a c d
z=L

C C
A

B
B B
z b

D D
D D
0
Hot Cold Hot Cold Hot Cold
0 2 3 4 5 6
wt
yi0 yi B1 yi B2 yi B3

Figure 7.1.18. Characteristic lines used for solution by equilibrium theory: upflow and heating in phase (Pigford et al., 1969a). Reprinted,
with permission, from I & EC Fund., 8(1), 144 (1969), Figure 3. Copyright (1969) American Chemical Society.

y i 0;t y0i . This composition will be changed in region therefore enter the top reservoir. There are two regions in
(D) of < t < 2, where the characteristic slope is 2  t  3, region (B) and region (C), through which
changed to that of (7.1.75b) when t  . Further, from such characteristics pass. The characteristics in region (C)
the second governing equation (7.1.74), we get have originated from those in region (C) of  t  2,
with ultimate origin from the top reservoir having a com-
y i 1 bi sqt constant;
position of y 0i . This composition, remaining constant up to
i.e. t 2, is changed in the t  2 region since the tem-
perature is changed to Thot from Tcold:
fyi 1 bi sqtgcold fyi 1 bi sqtghot : 7:1:77a
   
1 bi 1 bi
Therefore region C; t  2 : y i jhot y i jcold y 0i :
1 bi 1 bi
yi jcold 1 bi
: 7:1:77b 7:1:80
yi jhot 1 bi
Since, for t  , the characteristics for y i jhot change to These characteristics thus yield a higher species i compos-
those for yi jcold , ition in the fluid entering the top reservoir. But the top
    reservoir also receives characteristics in region (B),
1 bi 1 bi
y i jcold yi jhot y0i : 7:1:78 2  t  3. These characteristics originated in region (B)
1 bi 1 bi
for  t  2, where they originated from region (B) of
The characteristics crossing z 0 will enter the bottom reser- 0  t  . The composition of these last characteristics is
voir. For the period  t  2, only the characteristics in simply y0i ; as these characteristics cross t , from Thot to
region (D) enter z 0. Therefore, the composition of this fluid Tcold, the composition is lowered to y0i 1 bi =1 bi . This
entering the bottom reservoir, represented by hy iB i1 , is last composition is increased to y0i as these characteristics
  cross t 2 and change from Tcold to Thot.
1 bi
hyiB i1 y0i : 7:1:79 We now know the compositions of the characteristics
1 bi
in regions (B) and (C) for 2  t  3, where the fluid in
In the second cycle, beginning 2  t  3, this very upflow enters the top reservoir at z L. To determine the
same fluid with hy iB i1 will enter the column bottom, which mean composition hy iT i2 of these characteristics entering
is now at T Thot from the bottom reservoir, and the the top reservoir via regions (B) and (C), the weighted
characteristic line slope is changed. However, we are more contribution of each region is needed since each region
interested in those characteristics which cross z L and occupies a different time interval:
524 Bulk flow perpendicular to the direction of force

    
1 bi ac cd This preceding relation leads to
 
hy iT i2 y 0i  y0i g; 7:1:81
1 bi ad ad
1 bi n
 
hyiB in y 0i : 7:1:86a
where ac, cd and ad are identified in Figure 7.1.18 for 1 bi
2  t  3. To determine ratios ac=ad and cd=ad ,
Use of both relations (7.1.85a) and (7.1.85b) yields
focus on the characteristics lines in both regions
 t  2 and 2  t  3. Specifically, in region (C) 1 bi n 2
( "   #)
2bi
of time period  t  2, characteristic line eb has a hy iT in y 0i 1 1 : 7:1:86b
1 bi 1 bi
slope of magnitude v0 =1 bi . Therefore, the distance
ab is v0 =1 bi (ideally the length of time t used The composition ratio between the top reservoir and the
should be =, but cancels out later anyway). From the bottom reservoir is
slope of the characteristics in region (C) for 2  t  3,
2bi 1 bi n 1 bi 2
    
hy iT in
ab
 0 
v 1 ab 1 bi 2 : 7:1:87
) ac 0 : hyiB in 1 bi 1 bi 1 bi
ac 1 bi v =1 bi 1 bi
These results indicate that, as n increases, hy iB in , the
Since ad is , we get bottom product composition, tends to zero (relation
ac=ad 1 bi =1 bi : 7:1:82a (7.1.86a)). Although as n increases hyiT in increases, relation
(7.1.85a) suggests that hyiT in hy iT in 1 . In a real system,
The ratio cd=ad is obtained as the ratio hy iT in =hy iB in does not tend to infinity, as suggested
by (7.1.87) for large n; instead, mass-transfer rate limita-
cd ac 2bi
1 : 7:1:82b tions between the phases and axial diffusion limit the pos-
ad ad 1 bi
sible enrichment (Pigford et al., 1969a). Figure 7.1.19
Thus compares the predictions from this equilibrium theory with
the experimental data on an n-heptanetoluene separation
1 bi 1 bi 2bi 2bi
    
hyiT i2 y0i y 0i 1 :
1 bi 1 bi 1 bi 1 bi
7:1:83 105

This result demonstrates that the hot stream entering the top
reservoir at the beginning of the second cycle 2  t  3
is considerably enriched beyond y0i (the value for hy iT i1 ).
Toluene composition ratio yiT n / yiB n

Although fluid of composition hy iT i2 enters the column 104


top from the top reservoir for the next half-cycle,
3  t  4, the bottom region of the column is more
important for determining hy iB i2 , the stream composition
entering the bottom reservoir. Characteristics in region (D) 103
here originated in region (D) for 2  t  3. The com-
position of the characteristics in this later region is hy iB i1 ;
this composition is changed when t  3. The changed
composition yi jcold is related to hyiB i1 yi jhot by (7.1.80); 102
thus, for 3  t  4 region (D),

1 bi 1 bi 2
   
hy iB i2 yi jcold y i jhot y 0i : 7:1:84
1 bi 1 bi 10
Thus the bottom reservoir solution is depleted in species i after
the second cycle, while the top reservoir is enriched in species i.
Pigford et al. (1969a) have provided general expres-
sions for hyiT in and hyiB in after n cycles (n > 0) in terms 1
0 10 20 30 40 50 60 70
of those immediately proceeding:
Number of cycles, n
1 bi 1 bi 2bi
    
hy iT in hyiT in 1 hy iB in 3 ;
1 bi 1 bi 1 bi Figure 7.1.19. Toluene composition ratio according to equilibrium
7:1:85a theory and the data of Wilhelm et al. (1968) for the system n-
heptanetoluene in a silica gel column (Pigford et al., 1969a).
1 bi
 
hyiB in hy iB in 1 : 7:1:85b Reprinted, with permission, from I & EC Fund., 8(1), 144,
1 bi (1969), Figure 6. Copyright (1969) American Chemical Society.
7.1 Force ri in phase equilibrium: fixed-bed processes 525

in a silica gel column (Wilhelm and Sweed, 1968; Wilhelm It is assumed that the integral on the right-hand side of
et al., 1968). (7.1.90a) exists. To calculate its value, assume specific
That separation achieved in batch parametric pumping forms for f c t a and vz t :
can be demonstrated more easily by what is known as the
vz t A1 cos t; f c t a A2 cos t a ; 7:1:90b
Tinkertoy model developed by Wilhelm et al. (1968). This
model is considered next. where A1 and A2 are arbitrary constants. Then

7.1.4.2 Tinkertoy model of parametric pumping C i2 A2


cos a ; 7:1:91
z A1
Consider Figure 7.0.1(c) from the very beginning of this
chapter. The hatched section represents the stationary phase a result indicating that the time-averaged local axial con-
or the packings in the manner of Figure 7.1.1, i.e. a pseudo- centration gradient in C i2 can be nonzero provided a is
continuum approach. Focus on a section of the column of anything but =2 or 3=2, i.e. there will be separation
differential length dz. As before, subscript j 1 represents provided the oscillations in velocity and concentration are
solid packing phase and j 2 represents the mobile fluid not out of phase by =2 or 3=2:
phase. The mass balance equation (7.1.61) for species i at any
axial column location z may be rearranged to yield 7.1.4.3 Continuous parametric pumping
C i2 1 C i1
 
C i2 Batch parametric pumping in a closed system was studied
vz t 7:1:88
z t t in the context of an equilibrium nondispersive model in
Section 7.1.4.1. Continuous open parametric pumps have
for nondispersive column operation. In parametric
been modeled by a number of investigators (Gregory
pumping, the flow velocity vz t is alternating in direction
and Sweed, 1970; Chen and Hill, 1971). A number of
(a particular form is illustrated by equation (7.1.72a)); the
experimental systems have been studied. These include
force direction, and therefore the mass flux direction, are
pH based operations (Sabadell and Sweed, 1970; Chen
simultaneously alternating in direction. During fluid
et al., 1979, 1980) and pressure based gas separation oper-
upflow, if the bed is heated, solute is released by adsorb-
ations (Keller and Kuo, 1982).
ents into the fluid (shown by the dashed lines in Figure
7.0.1(c)); during fluid downflow, the bed is cooled and the
solute is adsorbed by adsorbents from the fluid (shown by 7.1.4.4 Cycling zone adsorption
solid lines in Figure 7.0.1(c)).
It is useful to recapitulate briefly the basics of the two
It can now be argued (Wilhelm et al., 1968; Sweed,
separation techniques described earlier, namely PSA and
1971, pp. 175180) that if the force is oscillating with a
parametric pumping. Both are cyclic processes. In PSA,
frequency (the same as the velocity), the concentrations
one gaseous species is preferentially adsorbed from the
C i2 and C i1 will also be periodic in frequency . Therefore,
 feed gas flowing into the bed of adsorbents at a high
C i2 =t and C i1 =t will also be periodic in . Equa-
  
pressure, allowing the remaining gas species to be purified
tion (7.1.88) may be expressed now as
(for a binary mixture). The preferentially adsorbed species
C i2 is then desorbed from the adsorbent bed at a much lower
vz t fc t 7:1:89a
z pressure, usually with a low-pressure purge flowing in the
opposite direction (see Figure 7.1.13(a)). For liquid-phase
C i2 f t
) c : 7:1:89b adsorptiondesorption processes, pressure has a very
z vz t
limited effect; therefore, temperature swing is useful for
The change in mobile-phase composition C i2 over the liquid-phase adsorption. In parametric pumping, the
column of differential length dz is provided by the above liquid flow up or down the adsorbent bed is synchronized
relation at any instant of time t. We are more interested in with heating or cooling of the liquid and the bed, leading to
knowing what this change is over one cycle (t 2=), and the reservoirs at the top and bottom being enriched or
ultimately over many cycles. depleted in the solute (see Figures 7.1.177.1.19). The
Before determining a time average of C i2 =z over reversal of the direction of liquid flow through the adsorb-
 

one cycle, let it be recognized that, in general, f c t and ent bed (practiced in parametric pumping) is avoided in
vz t are displaced by a phase angle a . Therefore, an the technique called cycling zone adsorption, which was
averaged behavior over a time period 0  t  2= is proposed first by Pigford et al. (1969b).
obtained from Focus on Figure 7.1.20(a), which shows an adsorbent
column being heated or cooled in the direct mode (via a
2 2
 jacket). Unlike that in Figure 7.1.17(a), there are no pistons/
f c t a

C i2 C i2
dt dt: 7:1:90a reservoirs at the top and bottom of the column as such.
2 z z 2 vz t
0 0 Liquid flow in the bed is unidirectional; liquid feed is
526 Bulk flow perpendicular to the direction of force

(a) (b) Heating half-cycle Cooling half-cycle


Heating half-cycle Cooling half-cycle
Thot Cih Tcold Cic
Thot Cih Tcold Cic
Packed
adsorbent bed

Qin Qout

Packed
Tf , Cif adsorbent bed Tf , Cif Qin Qout

Tf , Cif Tf , Cif

(c) (d)
100

90 0.06

NaCl concentration (kmol/m3)


80
Temperature (C)

70
0.04
60 Feed
concentration
50
0.02
40 Behavior of
0.5 m Distance 0.0 m
experimental data
30 from feed
inlet
0.0
0 3 6 9 12 15 0 2.5 5 7.5 10 12.5 15
Time (min) Time (min)

Figure 7.1.20. Cycling zone adsorption. (a) Direct mode of operation with heat supplied/removed directly into the bed through the wall.
(b) Recuperative mode of operation with heat supplied or removed directly into the feed. (c) Temperature variation with time at two
distances from feed inlet. (After Knaebel and Pigford (1983).) (d) NaCl concentration of feed outlet as a function of time. (After Knaebel
and Pigford (1983).)

continuously flowing up the column of adsorbents (or down temperature, we are going through a cycling of the zone
the column of adsorbents!). However, the temperature of the adsorption behavior. (Note: Here we have assumed that
column is oscillating, for example, as in Figure 7.1.17(c). If the liquid feed temperature is essentially the same as that
the liquid feed is continuously flowing up the column, then, of the bed in the direct mode of operation.) One can have
during the first half of the cycle when the bed is heated up an alternative arrangement where the liquid feed will be
(the temperature is Thot), in an empty bed at the start, there heated or cooled outside in a heater/cooler and introduced
will be very little adsorption. However, in the next half-cycle, into the bed in the so-called recuperative mode (useful in
as the temperature is reduced to Tcold, the liquid feed under- parametric pumping as well) of operation (Figure 7.1.20
goes purification since solutes are adsorbed much more at (b)). Correspondingly, the outlet liquid in the hot part of
the low temperature. The liquid exiting the bed at the top has the cycle is enriched in the solute, whereas in the cold
much less solute and is purified. part of the cycle the outlet liquid is purified of the solute (in
At the beginning of the next cycle, the bed is heated up; both modes of operation).
the adsorbed solutes are desorbed and the feed liquid A variety of systems have been investigated using the
entering the bed becomes highly enriched in the solute cycling zone adsorption technique. See Wankat et al. (1975)
as it leaves the column at the top. This half of the cycle, in and Knaebel and Pigford (1983) for a list of useful refer-
succeeding cycles also, will keep on producing a solute- ences. Figures 7.1.20(c) and (d) illustrate schematically the
enriched liquid stream. Therefore, by cycling the zone recuperative mode of cycling of temperature in a bed
7.1 Force ri in phase equilibrium: fixed-bed processes 527

(a)
Eluting solvent Detector

Concentration
Liquid sample
injection

Time
Column

Mobile-phase Pump
reservoir

(b)

Concentration
Detector

Gaseous sample
injection

Time
Column

Carrier gas
cylinder

Figure 7.1.21. Schematics for a basic chromatographic system for (a) liquid sample or (b) a gaseous sample.

(Figure 7.1.20(b)) and the corresponding bed outlet liquid adsorbed in a small band of stationary adsorbent particles
concentration profile for purification of a brackish water in the region of the column top. (The nature of the adsorb-
(feed NaCl concentration, 0.030 kmol/m3) using thermally ent in relation to the nature of the feed fluid, gas or liquid
regenerable ion exchange resins Amberlite XD. Two basic will be discussed in detail in Section 7.1.5.1.2.) Next the
models of the cycling zone adsorption are available in Baker solvent used in the feed solution, another solvent or an inert
and Pigford (1971) and Gupta and Sweed (1971). carrier gas is introduced into the column top at a steady rate.
If one monitors7 the effluent concentration at the column
7.1.5 Chromatographic processes bottom or end, one observes, after some time, a series of
concentration pulses of different species appearing one
The fixed-bed processes studied so far are generally useful
after another (Figure 7.1.21). By collecting effluent volumes
for separating one solute or one ionic species from the
between appropriate times (see Section 2.5), it is possible to
solvent; in some cases, more than one solute or one ionic
have each particular solute species in the feed essentially
species are also separated from the solvent. Separation of
segregated into separate and different effluent volumes;
each individual species present in a multicomponent gas
thus, all species present in the multicomponent feed have
mixture or liquid solution can be achieved in a fixed bed of
been separated from one another. However, they are
adsorbent particles if the mode of operation, e.g. the
obtained as a solution in the mobile-phase solvent or carrier
method of feed introduction, is changed from that used so
gas. The solvent or carrier gas is identified as the eluent, and
far. A number of methods are commonly used to this end.
this process of multicomponent separation using a fixed bed
They are elution chromatography, displacement chroma-
of adsorbents is called elution chromatography.
tography and frontal chromatography (see Figure 7.1.5(c)).

7.1.5.1 Elution chromatography


7
Generally, two types of detectors are used to monitor gas
Consider first the process known as elution chromatography compositions: a thermal conductivity detector (TCD) for common
used to separate components from one another present in a permanent gases like O2, N2, CO2, He, Ar, etc., or a flame ionization
multicomponent gas mixture or liquid solution. In such a detector (FID) for common combustible hydrocarbon gases/vapors
such as CnH2n2, Cn H2n and all sorts of volatile organic compounds.
process, a small volume, V0, of the feed solution or feed gas Ultraviolet spectrophometric detectors, refractive index detectors
mixture is first introduced into the column at the column and conductivity monitors are used to monitor liquid-phase
top. Different species present in this feed sample are compositions.
528 Bulk flow perpendicular to the direction of force

The mechanism of separation may be described as C i2 vz C


2 3 i2
follows. As the solute-free solvent or carrier gas is intro- t   z
duced into the column, it desorbs the solutes adsorbed 41 1 p im 1 1 p s q0i C i2 5

on the column-top particles from the initial feed sample.
Those solutes that are adsorbed less strongly desorb more
Di;eff;z 2 C i2
easily and are quickly carried downstream by the eluent. 2 :
z 2
3
All solute species are desorbed and carried downstream, 41 1 p im 1 1 p s 0  5
qi C i2
where they are readsorbed on fresh adsorbents immedi-
ately downstream. As fresh eluent appears and contacts
7:1:93b
these particles, the solute molecules go through cycles of
repeated desorptionadsorption along the column length. Additionally, the total local molar concentration C it
This process continues till they appear in the effluent at of species i per unit volume (which incorporates both
column exit, with the least strongly adsorbed solute mobile- and stationary-phase solute contributions) is
appearing first and the most strongly adsorbed appearing given, for nonporous particles, by
last. C i1
C it C i2 1 C i1 C i2 1 C i2
C i2
C i1
 
7.1.5.1.1 Liquidsolid adsorption based elution chro- 1 C i2 1 i1  C i2 ; 7:1:94
matography First, we consider systems where the eluent C i2
is a liquid; further, the mechanism of solute partitioning where 1 i1  is now a constant. Equation (7.1.92) may
onto the solid particles is simply adsorption, e.g. adsorp- be reformulated now with C it z;t as the dependent variable:
tion on silica or alumina particles. We assume further that
C it vz C Di;eff;z 2 C
the eluent pressure drop along the bed does not influence   it   2it :
t 1 z 1 z
the adsorption process. There are a number of ways by 1 i1 1 i1

which elution chromatography in such a system may
be described mathematically. The differential equation 7:1:95a
describing the solute adsorptiondesorption process for a This is a linear equation with constant coefficients and is
dilute liquid-phase system in a fixed bed of adsorbent essentially equivalent to equation (3.2.10). Only the coeffi-
particles continues to be equation (7.1.4): cients are different. For porous adsorbent particles, we need
to change equation (7.1.94) and obtain instead the following:
C i2 1 C i1 C i2 2 C i2
vz Di;eff;z :
t t z z2 C it C i2 1 p im 1 i1 : 7:1:95b
We will develop a solution for elution chromatography Equation (7.1.93b) may now be reformulated with C it z;t
in two ways. In the first method, a direct solution of as the dependent variable:
this equation is developed for the initial and boundary
C it vz C
conditions appropriate for elution chromatography using   it
t 1 1 z
linear local equilibrium assumption everywhere. In the 1 p im i1

second method, the solution (7.1.18h) developed earlier
Di;eff;z 2 C
for a fixed-bed process with a continuous liquid feed   2it : 7:1:95c
1 1 z
stream is differentiated to approximate the conditions 1 p im i1

for elution chromatography. We consider the first
method now. Since the initial solute band on top of the column (very
Rewrite the governing equation using the local equilib- near z 0) is a very thin one, we will assume that all of the
rium assumption and relations (7.1.6) and (7.1.7) as solute species i (mi moles) introduced via the feed sample
are essentially located at z 0 at the start of solvent elution
C i2 vz C Di;eff;z 2 C i2
h i i2 h : operation (t 0); therefore, at t 0,
z2
i
t 1 b qi0 C i2 z 1 b qi0 C i2
   
C it mi =Sc z: 7:1:96
7:1:92
Although the length of the column is finite, we will assume that
For linear adsorption equilibrium, encountered in dilute the solution for z ! is usable for our purpose (exactly as in
systems, the boundary condition (3.2.14a)). For all t, then, at z ,
  1 C it
q0i C i2 i1 7:1:93a C it 0 and 0: 7:1:97
b z
is a reasonable assumption, with i1 constant. For porous The solution for C it from equation (7.1.95a) may be
adsorbent particles, employing equation (7.1.13f), we get obtained following equation (3.2.19) as the Gaussian con-
the following form of equation (7.1.92): centration profile,
7.1 Force ri in phase equilibrium: fixed-bed processes 529

92 3
t Ri L=R0i vz : 7:1:99d
2 8
> >
vz t
> >
= 7
6 <
6 z 7 If we can assume that the movement of each species (present in
1
 
6 > 7
1 the feed sample introduced in the column prior to elution with
>
6 > i1
> 7
mi 1
: ;
6 7
C it z;t v exp 6 8 9 7 7: the solvent) is independent of those of all other species (gener-
6
Sc u
u 4Di;eff;z t  6 7
ally valid for dilute systems), then it is obvious from relation
> >
Di;eff;z
6 >
< >
= 7
1
u
4 t
6 7
1 i1 (7.1.99b) that the migration velocity of each species i down
t
1
6   7
> 5
4
: 1
>
> i1 >
the column will be different as long as each i1 is different.
;

7:1:98a If the migration velocity of each species i down the
column is different from those of others, the concentration
Correspondingly, the solution for the profile C i2 of species i peaks monitored in the detector at the column end (Figure
in the mobile phase is the Gaussian concentration profile, 7.1.21) at different times will represent different species. Gen-
2 8
>
92 3
> erally, this is indicated by stating that each species i has a
vz t different retention time t Ri (see (2.5.3)) or retention volume
>
< >
= 7
6
6 z 7
V Ri after which it appears at the column exit. The retention
 
6 > 1 > 7
1
6 > 1 i1 >; 7
mi 1 i1 
:
volume, V Ri , is the volume of elution solvent that passes
6 7
C i2 z;t v exp 6
6 8 9 7 7:
Sc u 4Di;eff;z t
u 6 > > 7 through the column after which species i appears at the exit.
Di;eff;z
  6 >
< >
= 7
1 Since the mobile-phase velocity vz is constant, this volume
u
4 t
6 7
1 i1
t 6   7
1
V Ri is obtained from
4 5
: 1
> >
> i1 >
;

7:1:98b V Ri t Ri  volumetric flow rate of mobile phase
t Ri  Sc vz : 7:1:99e
This concentration profile may be written in a compact
If the column length is z, then, for any species i, t Ri and z
form as follows:
 ( ) are related by (7.1.99a). Substituting that relation into
 z R0i vz t2 (7.1.99e), we get
C iz z;t C iz z;t exp : 7:1:98c

max 2 2ti 
1

V Ri z Sc 1 i1 ; 7:1:99f

The quantities R0i and ti are defined below as we charac-
where, however,
terize this profile, with C iz z;tjmax being the pre-
z Sc V M 7:1:99g
exponential factor in (7.1.98b) as well as the value of
C i2 z;t when z R0i vz t. is the mobile-phase volume within the column of length z.
The center point of the profile of the total concentra- Rewriting (7.1.99f) as
tion C it of species i (also of C i2 ) at any time is located at 1
 
VRi VM VM i1 VM =R0 i V M V S i1 ;
vz t
z  R0i vz t zi0 : 7:1:99a
1 7:1:99h
1 i1
where VS is the stationary adsorbent-phase volume in the
Since the quantity column, the contributions of the quantities VM, VS and i1
, ! to V Ri become clear.
1
vz 1 i1 Therefore, as long as i1 for each species i between the

mobile fluid phase and the stationary adsorbent particles is
different, each V Ri will be different. Correspondingly, each
is a constant, the z-directional velocity of the center point
t Ri will be different. Further, both V Ri and t Ri are increased
of the C i2 z;t, as well as the C it z;t , profiles is
by an increase in VM, VS and i1 . Conversely, a reduction of
dz vz
 
 R0i vz ; 7:1:99b V Ri and t Ri is achieved for any given system by a reduction

dt center point of i 1 in VM and VS. The resolution, RS, between peaks of two
1 i1
neighboring species i and j is
h i
1 1
where R0 i , the fraction of the total solute i present in the t Ri t Rj L R0i R0j
mobile liquid phase, was defined earlier by relation RS
2 ti 2 tj 2vz ti tj
 
(7.1.16c). The standard deviation of this profile is given by
1 1
   
8 91=2 L 1 i1 1 j1
> >
2Di;eff;z t 91=2 3 :
>
< >
= 28 91=2 8
ti  : 7:1:99c
1

2Di;eff;z t Ri 2Dj;eff;z t Rj = 7
>
< >
= >
< >
: 1
> >
i1 >
6
2vz 6
> ;

7
:1 1 > :1 1 >
4> > 5
i1 j1
; ;
The retention time, t Ri , of any species i is obtained from
(7.1.99a) when z L, the column length: 7:1:99i
530 Bulk flow perpendicular to the direction of force

The resolution is proportional to t Ri t Rj and thus to Consider solution (7.1.18h) for a fixed-bed process
 

V Ri V Rj , i.e. to V S i1 j1 . Therefore, one often uses having a liquid feed of constant inlet concentration C 0i2 into
   

the net retention volume, V N i , for species i: an initially solute-free column (i.e. C ii2 0). The solution
may be expressed compactly as
VN i VRi VM VS i1 : 7:1:99j

Reflect now on what is being achieved in elution chro- C i2 z;t C 0i2 i z;V ; 7:1:101a
matography. By having the bulk flow of the mobile
phase (solvent) perpendicular to the direction of force
where i z;V is the solution (7.1.18h) for the linear dis-
(for adsorption from the mobile phase to the stationary
persive equilibrium condition. Suppose that a volume V 0
particle phase; for desorption from the stationary particle
of solution of concentration C 0i2 is passed through the
phase to the mobile phase), a multicomponent mixture of
column from time t 0 to t t0. Now, for time t > t0,
solutes is separated, as long as RS values are reasonable. If
suppose that the interstitial liquid velocity remains the
the feed mixtures were simply equilibrated with the
same, but that the incoming solution concentration is
adsorbent particle phase without any mobile-phase flow
changed to C 00 i2 . One is interested in knowing what
perpendicular to the force direction, no such separation
happens at t > t0, especially at very large times. Since
would have been achieved.
the system (including the governing equation, the bound-
Consider, in addition, the nature of the force being
ary and initial conditions) is linear, the sum of two solu-
used in separation. Since there is adsorption and/or
tions may be used to develop a solution for this changed
desorption of solutes between the mobile fluid and the
condition. For V  V 0 ,
stationary solid phases, the potential profile under consid-
eration for each solute is discontinuous, a simple step
function (see Figure 3.2.2); there are no external forces. C i2 C 0i2 i z;V V  V 0: 7:1:101b
According to Section 3.2, such a system in a closed vessel
without any flow does not have any multicomponent sep- For V > V 0 , one can assume that an inlet concentration of
aration capability. Multicomponent separation capability
 00
C i2 C 0i2 has been introduced into the column for a

is, however, achieved in elution chromatography by having volume flow equal to V V 0 ; however, the effect of C 0i2
bulk flow perpendicular to the direction of the discontinu- must continue. Therefore for V 0 < V < , the solution is
ous chemical potential profile. The velocity vz here func- additive:8
0
tions exactly like the quantity 1 in equation (3.2.37).
C i2 C 0i2 i z;V C 00 0
i2 C i2 i z;V V
0
V 0 < V < :
   
To illustrate this, ignore the diffusion coefficient based
term in (7.1.95a) to obtain 7:1:101c

C it vz C The second term reflects the contribution of the extra


  it : 7:1:100
1 solute concentration C 00 0
i2 C i2 as if the volume passed is
 
t z
1 i1
0 and the feed concentration is C 00 0 00
i2 C i2 . When C i2 0,


we find
In the trailing edge of the profile (along the column length z),
C it =z is positive and C it =t is negative. Thus, solute i
   
C i2 C 0i2 i z;V i z;V V 0 :
 
7:1:101d
is being picked up from the trailing edge region and carried
forward to increasing z to make room for the slower-moving If V 0 is small, as in elution chromatography, a limiting
species j to come and get adsorbed in the stationary adsorb- process can be initiated:
ent particles. Correspondingly, in the leading edge of the C it 0
C i2 0 i z;V i z;V V
 
profile, C it is decreasing with z, therefore C it is increasing V ; 7:1:101e
C 0i2 V0
with time due to solute i brought over from the trailing
edge region by mobile-phase convection. This is how C i2 i z;V
V0 : 7:1:101f
separate profiles of multiple species are developed along C 0i2 V
the column length, and ultimately the chromatographic
detector output shown in Figure 7.1.21 is obtained as a The approximation leading to the derivative is acceptable
function of time. only in the limit of V 0 ! 0. Using the solution (7.1.18h) for
The results of elution chromatography were illustrated i z;V ,
above using the solution of the equation for the local value
of the total solute concentration C it . In reality, a series of
8
profiles of mobile-phase concentration C i2 show up at the This is the principle of superposition. Since equation (7.1.18c) or
equation (7.1.95a) are linear equations due to constant i1, the
column end at different times in elution chromatography. superposition of two simpler solutions by simple addition leads to
Such a result may be obtained by using the second method the solution of the problem under consideration. This is the
mentioned earlier. advantage of linear chromatography models.
7.1 Force ri in phase equilibrium: fixed-bed processes 531

is the maximum value of C i2 achieved when V V (see


8 2 93
1=2 
Pez;eff V V =7
>
0
C i2 V 6
>
equation (2.5.6b)).
<
0 41 erf 1=2 >5
C i2 2 V : 2 VV

> ; The volume of eluent collected during the elution
8 9 of a solute band is ultimately the source of recovering
> >
the solute. The presence of multiple eluting solutes with
8 0 19 > >
1=2 < V V 2 >
> >
Pe 1 V
>
z;eff
< V = =
0
V p 1=2 :1 @ A exp : overlapping bands forces us to make a decision on the
2 VV 2 V ; >
> 4V V > > eluent volume acting as the cut point tc in time (see
> >
: Pez;eff ;
> >
Figure 2.5.1(b)). Correspondingly, one is interested in
7:1:101g knowing what is the fractional recovery of a particular
solute for a given amount of solute input moles
Since V V =2V is, in general, small compared to 1 in the mi V 0 C 0i2 m0i . For the species 1 type of profile
 

breakthrough region, we can rewrite the above as shown in Figure 2.5.1(b), the fractional recovery of solute
8 9 i from time t 0 (alternatively t ) to time t tc, the
1=2
>
> 2
>
> cut point, is given by
Pez;eff < V V >
>   =
 0 0
C i2 V C i2 p 1=2 exp > ; 7:1:102a mii
2 VV 4V V fractional solute recovery
m0i
> >
>
> >
Pez;eff
: ;
t tc
Qf C i2 t dt Sc vz C i2 t dt
where V 0
C 0i2
mi is the number of moles of solute i
: 7:1:102g
injected. This is a Gaussian profile around V , with a stand- m0i
Sc vz C i2 t dt
ard deviation V i given by


2V V
1=2 However, from equations (2.5.11)(2.5.13), we already
V i : 7:1:102b know that this ratio for Gaussian profiles is given by
Pez;eff
mii 1 1 t c t Ri 1 1 V c V Ri
   
For such a profile, the following approximations are valid erf p erf p ;
m0i 2 2 2 2



2 ti 2 V i
around9 V V V i :
7:1:102h
1=2 1=2
VV V and V i V 2=Pez;eff : 7:1:102c
 
where the final expression has been written by analogy,
since the volumes Vc (the volume at the cut point), VRi and
Employing the definition of V and Pez,eff in (7.1.18i),
V i in the elution process are linearly proportional to t c , t Ri
2S2C z2 1 i1 2 Di;eff;z 2Di;eff;z and ti Vc vz S t c ; etc:. Similarly, for the species 2 type
2V i / z; 7:1:102d of profile shown in Figure 2.5.1(b),
zvz vz
mii
where z is the column length. For illustrative examples of fractional solute recovery
m0i
the above procedure, consult Mayer and Tompkins (1947)
and Lightfoot et al. (1962). Expressions for similar profiles Qf C i2 t dt Sc vz C i2 t dt
tc
may be developed for other species present in the small t c : 7:1:102i
m0i
feed sample of volume V0 introduced initially at the top of Sc vz C i2 t dt
the column. To determine i1 and Pez;eff from V V , see

Problem 7.1.13, in which column information and flow From equations (2.5.14)(2.5.16), we already know that this
rates are provided. ratio for Gaussian profiles is given by
A simpler representation of the solute i elution profile
mii
(7.1.102a) is fractional solute recovery
m0i
V V 2
( )
1 1 tR tc 1 1 VR Vc
   
C i2 C i2 jmax exp ; 7:1:102e erf pi erf pi :
2 2V i 2 2 2 ti 2 2 2 V i
7:1:102j
where
h
1=2 p i
C i2 jmax V 0 C 0i2 Pez;eff =f2 V V 1=2 g 7:1:102f
Example 7.1.5 In the purification of an enzyme by an
appropriate column of adsorbents, the elution process pro-
vided the information given in Table 7.1.4 about the enzyme
9
V here refers to expression (7.1.16d) for solute i and therefore concentration (in suitable units) in the eluent leaving the
should more correctly be represented as V i , which is not to be column (volume 2 liter) and the total volume of eluent
confused with the partial molar volume of species i. passed.
532 Bulk flow perpendicular to the direction of force

Table 7.1.4. liquid, the stationary phase consisted of porous particles,


and the local mechanism of separation of various species
C i2 V (liter) between the mobile phase and the stationary phase was
simply adsorption. There can be a number of other types of
6.5 1.8
15.0 2.0a
elution chromatography depending on the combination of
mobile and stationary phases and the separation mechan-
a
It is known that the C i2 value of 15.0 was ism locally operative. The more well-known and frequently
the maximum; therefore V 2 liter.
used of these techniques are summarized in Table 7.1.5.
There are variations in a given technique identified in
Determine the value of V i (liter) and the fractional recovery
Table 7.1.5. For example, in liquidliquid chromatography
of the enzyme (assuming that the concentration profile is
Gaussian) when the eluent volume passed is 2.1 liters. (LLC), normally the stationary-phase liquid is polar and
the mobile phase is nonplolar, both being immiscible with
Solution Consider the Gaussian elution profile (7.1.102e): each other. In reversed-phase LLC, the stationary-phase
( )
V V 2 liquid is nonpolar, whereas the mobile phase is polar; the
C i2 C i2 jmax exp :
2 2V i polar phase can even be water. In many cases, the station-
ary liquid phase may be a monomolecular layer chemically
Apply the profile to the two data points provided and then
bonded to the surface groups of the porous support mater-
divide one by the other:
8 9 ial. This is often achieved with a nonpolar hydrocarbon
< V V 2 = liquid phase.
1
exp A very common column configuration in elution chro-
: 2 2V i ;
C i2 j1 0:65 matography is simply a tubular column packed with
8 9 )
C i2 j2 < V V 2 = 1:5 porous particles, the packings, with or without a bonded
2
exp 2 liquid phase on the particle surfaces. Other column
: 2 Vi ;
0 1 configurations include capillary columns or open tubular
2
1:8 2:0 columns, in which a thin liquid film of adsorbents has been
exp @ A
2 2V i applied (or bonded) to the internal surface of the capillar-
0 1 ) 0:43 ies. A potential variation of this is the microporous hollow
2
2 2 fiber membrane based column, wherein the stationary
exp @ A
2 2V i phase is held in the pores of fiber wall and the eluent is
0 1 0 1 passed through the bore of the fiber (Ding et al., 1989).
2
0:2 0:02 Analytical-scale column chromatography, used to iden-
exp @ A ) exp @ A 2:325
2 2V i 2 2V i tify the components in a sample or to determine the feed
sample composition, uses small columns; for example, in
) 0:02= 2V i 0:844 ) V i 0:154 liter:
gasliquid chromatography, a typical column may be 6 ft
Since the eluent volume passed, 2.1 liters, is larger than long. In preparative chromatography, column diameters of
V 2 liter, the cut point tc > t Ri . So the profile corresponding 1 to 2 cm are used, and the feed sample volumes are con-
to species 1 in Figure 2.5.1(b), is relevant. The formula for siderably larger. While this is used to prepare larger
fractional recovery is given by (7.1.102h):
amounts of purified materials or compounds for laboratory
 
1 1 Vc VR use, industrial-scale column chromatography, producing
fractional recovery erf p i
2 2 2 V i 100 metric ton/year, is being practiced (Chem. Eng., 1980,
 
1 1 2:1 2:0 1981). The same organization (Elf Aquitane) has demon-
erf 0:5 0:5 erf 0:459
2 2 1:414  0:154 strated a variety of separations in gas chromatography using
0:5 0:5  0:484 0:742: industrial-scale columns of diameter 40 cm, length 1.5 m,
containing 6080 mesh packing operating at 180  C for
Elution chromatography based on liquidsolid adsorption
injection times between 5 and 50 s and sample sizes
for small molecules typically employs adsorbents such as
between 10 and 50 g/s (Bonmati et al. 1980).
alumina, charcoal, silica, hydrophobic silica, etc. Surface
In column or capillary chromatography, the gas or
area, water content and the chemical nature of the adsorb-
liquid eluent is driven to flow by the application of a
ent (e.g. polar or nonpolar) distinguish one adsorbent from
pressure gradient along the column length. In paper or
another. For larger molecules/macromolecules/charged
planar chromatography, the adsorbent material is in a thin
species, other adsorbents needed are briefly touched upon
granular form (thin-layer chromatography) or in fibrous
in Sections 7.1.5.1.67.1.5.1.8.
form (as in paper chromatography); no such pressure gra-
7.1.5.1.2 Types of elution chromatography The type dient can be independently applied. Instead, the liquid
of elution chromatography described earlier was liquid eluent is driven along the layer by capillary action (i.e.
solid adsorption chromatography. The mobile phase was the capillary pressure, see Section 6.1.4). The rate of
7.1 Force ri in phase equilibrium: fixed-bed processes 533

Table 7.1.5. Phase combinations and local retention mechanisms in chromatography

Mobile
phase Stationary phase Local mechanism of separation Name

Liquid porous solid adsorbent adsorption liquidsolid adsorption


chromatography (LSC)
Liquid porous ion exchange resin particles ion exchange ion exchange chromatography
Liquid liquid stationary phase coated on a partitioning (as in solvent liquidliquid chromatography
porous solid support extraction between two (LLC)
immiscible liquid phases)
Liquid porous gels molecular size exclusion via gel gel permeation chromatography
pore size (GPC) or size exclusion
chromatography (SEC)
Liquid immobilized affinant on an insoluble reversible binding between solute affinity chromatography
support having strong affinity for and affinant
macromolecules
Gas liquid stationary phase with low differential solubility in the gasliquid chromatography (GLC)
vapor pressure coated on a stationary phase and differing
porous solid support solute vapor pressure
Gas porous solid adsorbent adsorption gassolid chromatography
Supercritical porous solid adsorbent or liquid adsorption or differential solubility supercritical fluid chromatography
fluid stationary phase coated on a and differing solute vapor
porous solid support pressures

spreading of the liquid cannot be precisely controlled. acting as a porous medium (see the solvent flux expres-
Further, the movement of the liquid is two-dimensional, sions (3.4.85), (3.4.88) and (6.1.4g)):
unlike in column chromatography, in which the movement
Qg dP
is in one dimension only, along the column length. vz : 7:1:103b
dz
7.1.5.1.3 Elution chromatography with mobile gas Remember: the Darcy permeability Qg is proportional to r 2p ,
phase Elution chromatography with a mobile gas phase where r p is the pore radius, and therefore to d 2p , where d p is
(gas chromatography, GC) can be carried out with either the packing diameter (since it is essentially proportional to
porous solid adsorbent or a liquid absorbent coated on the effective diameter of the interpacking opening).
porous solid support particles (see Table 7.1.5). The former Substituting for vz in terms of P from (7.1.103a) into the
is identified as gassolid chromatography (GSC), whereas above expression, then integrating and rearranging, we get
the latter is called gasliquid chromatography (GLC). )1=2
P in =P out 2
(
Sometimes in GLC, the liquid absorbent exists as a coating v z z
; 7:1:103c
on the inside surface of a capillary tube. The liquid absorb- vz;in P in =P out 2 P in =P out 2 1 Lz

ent in GLC must have low enough vapor pressure to be


considered essentially nonvolatile. where is assumed to be constant and independent of gas
The analysis of elution chromatography with a liquid pressure. The retention time t Ri may now be obtained from
eluent that was presented earlier was based on a constant relation (7.1.99b) if vz is considered to be a function of the
eluent velocity, vz. Unlike simple fixed-bed adsorption z-coordinate:
processes with low pressure drops, analytical-scale chro- t Ri
L
dz

matographic techniques for a mobile gas phase employ dt t Ri : 7:1:103d
long packed columns, where the gas undergoes a consider- R0i vz z
0 0
able pressure drop. As a result, the gas velocity changes
with column location. But the gas velocity is proportional Substituting into this an expression for vz(z) from (7.1.103c)
to the molar gas volume at every location. Therefore, by and integrating, one obtains
Boyles law, the gas pressure P and velocity vz at any L 1
location are related to those at the inlet (subscript, in) t Ri ; 7:1:103e
R0i vz;avg
and outlet (subscript, out) by
where
Pvz P in vz;in P out vz;out : 7:1:103a
vz;avg 3 P in =P out 2 1
" #
The gas velocity vz and the gas pressure gradient (dP/dz) at ; 7:1:103f
vz;out 2 P in =P out 3 1
any location are related by Darcys law for the packed bed
534 Bulk flow perpendicular to the direction of force

as demonstrated first by James and Martin (1952). This last representing the solvent species in the stationary coated
ratio allows the conversion of the known gas velocity at liquid phase. The net retention volume, by definition
column outlet, vz,out, to an average gas velocity, vz,avg, in (7.1.99), is
the column for calculating the retention time t Ri of species
RTmsl
i. Correspondingly, the retention volume of gas needs to be V N i V Sl i1 : 7:1:104c
il Psat
i
corrected due to this gas compressibility. In addition, if the
column temperature T is different from the exit tempera- Since the value of the quantity msl of the coating solvent is
ture Tout at which the gas flow rate is measured, an add- particular to a given column and packing, an alternative
itional correction has to be made. quantity, the specific retention volume V N i =wl , is some-
 

Given the flow conditions and column dimensions, the times used:
retention time or volume of a species is known, if i1 is V Ni RT
known. Since the variation in V Ri or t Ri between two sat ; 7:1:104d
wl il P i M Sl
neighboring peaks or species depends on values of i1 for
i 1, 2 (say), it is useful to enquire what properties of the where wl is the mass of the stationary liquid phase used as
gasstationary phase combination influence i1 . a coating and M sl is the molecular weight of this coating
For gasliquid chromatography (GLC) with liquid liquid. The specific retention volume of species i decreases
coating on packings, assume equilibrium to exist between as its vapor pressure increases; however, the coating liquid
the two phases for any species i. Then, from Section 3.3.7.1, also influences this value through il .
we get ^f ig ^f il , ^f ig x ig ig P and ^f il il x il f 0il . Since the A large number of organic liquids with maximum oper-
gas phase may be assumed to behave ideally at the low- ating temperatures varying between 100 and 350  C and
pressure characteristics of GLC, ig 1. For condensible varying degrees of polarity have been used as stationary
solute species (vapors) at low pressures, f 0il P sat i from phases in GLC. Nonstandard liquid phases used include:
equation (3.3.62). Further, the solute concentration in the liquid crystals; addition of nonvolatile complexing agents
stationary liquid phase on packing is very low, so that in the stationary liquid phase which preferentially complex
il il , corresponding to infinite dilution. Therefore with particular solutes (e.g. AgNO3 with olefins). Porous,
chemically inert solid supports often used to hold the
x il P x 1l x 2g 2l P sat
2
and 12 ; liquid phase include diatomaceous earth (called Chromo-
x ig il P sat
i x 1g x 2l 1l P sat
1 sorb), fluorine-containing polymers that are quite inert,
7:1:104a glass microbeads, etc.
where species 2 elutes earlier from the column than Whereas GLC is more often used to separate a variety
species 1. Generally, P sat sat of thermally stable volatile nonelectrolytic species with
2 > P 1 . However, the ratio of the
activity coefficients is quite important, especially in the molecular weight up to 300, gassolid chromatography
separation of two species of essentially equal vapor pres- (GSC) is primarily used to separate permanent gases, gas-
sure. The selectivity is obtained by different molecular eous hydrocarbons, etc. Porous adsorbents used in GSC
interactions between the solute species to be separated include graphitized thermal carbon black, silica, porous
and the stationary liquid phase. For a brief introduction glass, molecular sieves, porous polymer beads, etc. Gas
to the contribution of the activity ratio to the selectivity, see solid adsorption equilibrium provides the local separation
Karger et al. (1973). mechanism. Complications arise from chemisorption, deg-
The net retention volume, V N i (definition (7.1.99j)), for radation of samples due to higher column temperatures in
species i can also be expressed in terms of P sat and i . analytical separations and large retention volumes. Since
i
Consider the distribution coefficient i1 first and assume the surfaces of porous adsorbents are not well-defined,
ideal gas behavior: there are difficulties in developing effective models for
species retention volumes.
C i1 C i1 C i1 RT mil RT Gas chromatographic techniques are primarily used for
i1
C i2 C ig x ig P V Sl x ig P analytical separations. However, they have begun to be
      used in industrial-scale separations. A schematic of an
mil RT msl x il RT msl
; industrial-scale gas chromatographic unit (Chem. Eng.,
msl x ig P V Sl x ig P V Sl
1981) is illustrated in Figure 7.1.22. As described by Bonmati
where mil denotes the moles of solute i in a stationary liquid et al. (1980), liquid feed from a reservoir is fed by a pump to
phase of volume V Sl and solvent moles msl . Therefore, a vaporizer. The vaporizer is connected to the injector via a
  valve, whose opening and closing is controlled by an elec-
RT msl
il sat : 7:1:104b tronic programmer. In the injector, the vaporized feed is
il P i V Sl
mixed with the purified carrier gas, which is generally com-
To get this result, we have used relation (7.1.104a) for pressed hydrogen. This vaporized feed in the carrier gas is
x il =x ig and assumed mil << msl , the subscript s
 
introduced into the chromatographic column at particular
7.1 Force ri in phase equilibrium: fixed-bed processes 535

Electronic
programmer

Deoxygenator
Activated-
carbon beds

Compressor
GC
column

Condensers

Hydrogen

Carrier-gas
preheater Feed
valve

Feed
Feed storage surge
tank Injector
Recycle
products

Products

Figure 7.1.22. Basic flow diagram of a large-scale gas chromatographic process. (After Bonmati et al. (1980) and Chem. Eng. (1980).)

intervals. Typical injection times are between 5 and 50 s, theoretical plate (the ideal stage in Section 6.3.2.1). Simi-
and the sample size varies between 10 and 50 g/s. The larly, in elution chromatography, using a column packed
column diameter can be as much as 40 cm, and the column with adsorbent particles, one often encounters the notion
length is 1.5 m. It is uniformly packed with Chromosorb of of a number N of theoretical plates. On each plate, the
size 6080 mesh and coated with a suitable liquid. adsorbent particles are assumed to be always in equilib-
The end of the column is connected by a valving rium with the eluent vis--vis any solute species. The
sequence to a set of condensers and product collectors. eluent passes continuously and without mixing through
Different species move through the column at different these plates from one plate to the next (Figure 7.1.23).
speeds and exit at different times. A given fraction of the Different expressions have been developed to calculate
exiting stream from the column containing a particular the solute concentration along the column of N plates as
species is routed to a particular condenser and product a function of the eluent volume passed through the
collector via a particular valve opened at that time by the column (Martin and Synge, 1941; Mayer and Tompkins,
electronic programmer. The overhead of each condenser 1947; Said, 1956). The method of analysis followed here is
produces a stream of the carrier gas (e.g. hydrogen), which somewhat related to that of Said (1956). The earlier deriv-
is recycled to the injector after passing it through a carrier- ations by Martin and Synge (1941) and Mayer and Tomp-
gas cleaner, compressor, catalytic deoxygenator and a pre- kins (1947) are limiting cases.
heater. The bottoms from each condenser comprise the The following assumptions are made in the context of
particular product fraction. A portion of the liquid output is linear chromatography:
recycled to the feed storage or feed vaporizer. The resi-
(1) The chromatographic column length L essentially con-
dence time in the column is of the order of a minute or
sists of N theoretical plates, on each of which the
two; thus, thermal degradation is avoided at column tem-
adsorbent particles are always in equilibrium with the
peratures as high as 180  C.
eluent vis--vis any solute species. Further, each plate
contains VS/N cm3 of adsorbent, where VS is the total
7.1.5.1.4 Number of theoretical plates and plate height volume of adsorbent; similarly, each plate contains
in elution chromatography In an ideal equilibrium stage VM/N volume of mobile phase, where LSc VM. The
of distillation, the vapor and liquid mixtures are in equilib- distance between two consecutive plates is L/N,
rium, and each phase has the same composition every- where is the void volume in the column occupied
where. Such an equilibrium stage is often identified as a by the eluent.
536 Bulk flow perpendicular to the direction of force

Ci21 Ci 22 Ci23 Ci 2n Ci2N

Ci21 Ci22 Ci 23 Ci 2n Ci2N

Eluent
Mobile Ci11 Ci12 Ci13 Ci1n Ci1N
phase
1 2 3 n N

Figure 7.1.23. Theoretical plate model of a chromatographic column with N well-mixed plates; both phases in each plate are in
equilibrium, and the eluent passes continuously and without mixing through these plates.

 
(2) The eluent passes continuously and without mixing C i12 VS VM
C i21 dV dC i12 dC i22 :
through these plates. i1 N N
(3) The eluent phase concentration of species i on the nth
plate, C i2n , is linearly related to the adsorbent-phase Substituting for Ci21 from (7.1.105d) and rearranging,
concentration C i1n by we get
" #
dC i12 C i12 C 0i11 V
C i1n =C i2n i1 ; 7:1:105a ! exp V M VS :
V i1 i1 i1 N i1 N
d
where i1 is a constant. i1 VNS VNM

Figure 7.1.23 shows a schematic of the column with Its general solution is
N plates. At any time t during the elution process, the " !# " !#
adsorbent-phase concentrations on plates 1, 2, 3, n,, V C 0i11 V
C i12 exp V M VS VM V S constant2 :
N of species i are Ci11, Ci12, Ci13,, Ci1n,, Ci1N, respect- N i1 N N i1 N
ively. (At the beginning of the process (t 0), the values of
these quantities were C 0i11 ; C 0i12 ; C 0i13 ; ; C 0i1n ; C 0i1N .) The When V 0, C i12 C 0i12 . Therefore
corresponding mobile-phase concentrations on each plate !" !#
at any time t are Ci21, Ci22, Ci23,, Ci2n,, Ci2N. C 0i11 V 0 V
C i12 V M V S C i12 exp VM VS :
Consider plate 1 in Figure 7.1.23. If the total volume of N i1 N N i1 N
eluent that has crossed plate 1 at any time is V, then a 7:1:105e
differential material balance (for a short time interval dt)
on species 1 around plate 1 provides Similarly, a differential material balance around plate
3 leads to
VS VM
C i11 =i1 dV dC i11 dC i21 ; 7:1:105b  
N N C i13 VS VM
C i22 dV d C i13 dC i23 :
i1 N N
VS VM
C i11 =i1 dV dC i11 dC i11 ;
N Ni1 Employing the solution for Ci12 from (7.1.105e), we obtain
dC i11 dV V " !#
V S V M ) logC i11 V M V S constant1 : dC i13 C i13 1 V
C i11 i1 N N N i1 N ! exp V M
i1 i1 VS
V i1 N i1 N
When V 0, C i11 C 0i11 . Therefore d VM VS
" # N i1 N

0 V " #
C i11 =C i11 exp V M VS : 7:1:105c C0 V
N i1 N C 0i12 V M i11 V S  : 7:1:105f
N i1 N

Therefore, the eluent concentration leaving plate 1 and


A solution of this for the initial condition of C i13 C 0i13 ,
entering plate 2 is given by
when V 0, is
!
C i11 C0 V
" ! #"
C i21 i11 exp : 7:1:105d V C 0i11 V 2
i1 i1 VM
i1 VS C i13 exp V M VS 2
2 M i1 V S
V
N i1 N
N N
N N
C 0i12 V
A differential mass balance around plate 2 for species i in  C 0i13 : 7:1:105g
V VS 2

the manner of equation (7.1.105b) yields N
M
i1 N
7.1 Force ri in phase equilibrium: fixed-bed processes 537

For plate 4, the corresponding expression is where the nondimensional concentration wave velocity for
" !#" species i, v
Ci , is defined by
V C 0i11 V 3
C i14 exp V M 3 V

Sc vz t

i1 VNS 23 NM i1 VNS
V
N v
Ci L
N S c i1 1 
V M =N i1 V S =N
#
C0 V 2 C 0i13 V 0 vz t vCi t
 i12
2 V M
 VS
C i13 : 7:1:105h i : 7:1:105o
2 V M i1 V S N i1 N H
h
N N L=N 1 1 i1

By induction, the expression for plate n is Here H is the height of a theoretical plate if there are N
" ! #" plates in a column of length L:
V C 0i11 V n1
C i1n exp V M n1 L
i1 VNS n1! VNM i1 VNS

N H ; 7:1:105p
# N
C 0i12 V n2 0
n2 C i1n ; 7:1:105i and vCi is given by (7.1.15c) and (7.1.12a). The value of C i2j
n2! VNM i1 VNS

at the column outlet, where j N, will represent the peak
maximum when v Ci N:
which may be written in a compact fashion as follows:
v N2
" # !
n
X
0 Vnr C i2N C i2N jmax exp Ci ; 7:1:105q
C i1n C i1r n r 2N
n r! VNM i1 VNS

r1

where
" !#
V
exp V M VS : 7:1:105j
N i1 N mi 1
C i2N jmax p : 7:1:105r
V M =N i1 V S =N 2N
Special case: Only the first plate is loaded with solute
before the beginning of the elution process, i.e. Further, the value of the dimensionless standard deviation
C 0i12 C 0i13 C 0i1n 0; therefore
V i is given as
p p

Vi N L=H : 7:1:105s
" !#
0 V n 1 V
C i1n C i11 n1 exp V M
 VS
 ;
n 1! V M i1 V S N i1 N

N N Rewriting equation (7.1.105n) with v 
Ci replaced by vCi t=H
7:1:105k and j N, we get
2 1
which is a Poisson distribution function. If mi moles of
0
vCi

L
H t H
solute i were introduced in the first plate, then C i2N C i2N jmax exp @
B
A:
C
7:1:105t
2L=H
VS VM C0 VS VM
   
mi C 0i21 i1 i11 i1 ;
N N i1 N N
This expression may be written in two alternative forms
7:1:105l
to illustrate a Gaussian profile in terms of time t or
leading to distance z:
mi 1 Vn1
!
C i2n V M t t Ri 2
V S n 1! V M

VS n 1 C i2N C i2N jmax exp ; 7:1:105u
N i1 N

N i1 N 2H t 2Ri =L
" !#
V p
exp V M  : 7:1:105m ti t Ri = N ; 7:1:105v
N i1 VNS
z i0 L2
!
When the value of n is large, the Poisson distribution C i2N C i2N jmax exp ; 7:1:105w
becomes a Gaussian distribution. Giddings (1965) has indi- 2LH
cated that this discontinuous Poisson distribution may be p
approximated by a Gaussian distribution when the number zi LH : 7:1:105x
of plates exceeds 100. Two conclusions from these results are: zi increases with
The Gaussian distribution on plate j, where j is a large L1=2 ; ti decreases as N increases in the manner of N 1=2 .
number, may be described by Also, N can be determined from any data on t Ri and ti ;
" # therefore, H can be determined from (L/N).
mi 1 v j2
C i2j p exp Ci ; The strength of the plate model is that, for a discrete
V M =N i1 V S =N 2j 2j
number of stages, it can predict useful quantities, e.g.
7:1:105n which plate will have the highest solute concentration at
538 Bulk flow perpendicular to the direction of force

any time (volume of mobile phase) (Said, 1956; Giddings, Therefore, the plate number increases linearly with the
1965; King, 1980, pp. 379387; Wankat, 1990, pp. 313316). axial column Pclet number and the plate height decreases
It is also useful for comparing two chromatographic as N increases. Also N increases as Di;eff;z decreases, and as
columns in terms of the number N of plates/stages avail- vz and the column length increases.
able. If the column is of length L and contains N theoretical It is also useful to develop a relation in elution chro-
plates, then the two are related through a plate height H matography between the resolution RS between peaks of
(sometimes called the height of an equivalent theoretical two species i and j and the plate number in the column
plate, i.e. HETP) via (see the corresponding definition (among other quantities). We have already developed such
(6.3.24) in capillary electrophoresis) a relation (e.g. (6.3.25)) in capillary electrophoresis). Con-
L sider for this purpose the expression (7.1.99i) for resolution
N : 7:1:106a between species i and j:
H
For a fixed L, the larger the value of N, the more efficient t Ri t Rj t Ri t Rj
RS ; 7:1:106g
the column, and correspondingly, the smaller the value of 2 ti 2 tj 2 ti tj
the plate height. However, in the plate model of a chroma-
where the standard deviations are in time units. However, in
tographic column, the number of plates, N, is unknown;
relation (7.1.106d), i is in length units, zi . We may write
therefore H is unknown. In fact, H has to be predicted
independently via alternative models (Giddings, 1965). ti tj 2 t 2 z =vCi vCj 1=2 ; 7:1:106h
Giddings (1965) has defined the plate height H for a
uniform column free from concentration and velocity gra- where t is an averaged standard deviation in time units,
dients as z is an averaged standard deviation in length units and
H 2zi =L; 7:1:106b vCi vCj 1=2 is a geometrical average of the center-point
velocities of species i and j. Further, we have written
where zi is the standard deviation of the eluting concen- (dz/dt)center point of i and vCi in equation (7.1.99b). Now,
tration profile of species i. For a given column length L, the from relation (7.1.106d) in terms of length units, we may
extent of band broadening (equation (7.1.99c) as well as write
the text around equations (3.2.21a, b) and (3.2.23)) is dir- p L L
ectly related to H. As 2zi increases, H increases. Corres- N ; 7:1:106i
z z
pondingly, the number of plates, N, is reduced; therefore,
the peak capacity, nmax , is reduced. From expression where
(7.1.99c), since 2zi is proportional to Di,eff,z, H increases
z 1=2 zi zj 7:1:106j
linearly with Di,eff,z. A more general definition of H for
noununiform columns (Giddings, 1965) is is an averaged standard deviation of the two profiles in
length units. We note that, from (7.1.99d),
H d 2zi =dz; 7:1:106c
vCi R0 i vz 7:1:106k
i.e. the plate height is the increment in the variance 2i per
unit length of migration, the migration taking place in the and
z-direction.
t Ri L=R0i vz :
An expression for N in terms of L and zi ,
Therefore, from (7.1.106g) and (7.1.106h),
N L= zi 2 7:1:106d
0 1
(obtained from definitions (7.1.106a) and (7.1.106b)), indi- 1 1
@ A
cates that the smaller the bandwidth or dispersion and the R0i R0j LvCi vCj 1=2 R0 j R0 i
t Ri t Rj L
higher the column length, the larger the number of theor- RS
2 ti tj 4vz t 4vz z R0i R0j
etical plates. The i used in the definitions given above is in
2     1=2  
length units. On the other hand, the i used in definition L vz vCj vCi vCi vCj L vCj vCi
:
(7.1.102b) is in volume units, V i . The number of plates N 4 2  
vz z vCi vCj 4 z vCi vCj 1=2
is then defined by
 2 7:1:106l
V Ri
N ; 7:1:106e
V i Employing relation (7.1.106i) for N and defining vCi vCj as
where V i is used along with the retention volume V Ri for the geometrical average of vCi and vCj (i.e. vCi vCj 1=2 ), we get
species i. From relation (7.1.102c) for V i , observe that  
1 vCj vCi p
RS N: 7:1:106m
N Pez;eff =2 : 4 vCi vCj


7:1:106f
7.1 Force ri in phase equilibrium: fixed-bed processes 539

(a) (b)

B
Cnz
H nz H
5.0 m
3.5 m
A 1.8 m

nz nz

Figure 7.1.24. (a) Dependence of plate height H and its various components on vz from the Van Deemter equation (7.1.107a).
(b) Dependence of plate height H on vz for smaller particle sizes in HPLC. (After Majors, 2005.)

Therefore as the plate number increases, the resolution Therefore


between two species increases. We will see in Section p
7.1.5.1.5 that as the particle size is reduced, leading to a H min A 2 BC : 7:1:107c
larger N and lower H, the value of RS increases. The optimum velocity in gas chromatography is around
A more detailed description of the plate height H is 10 cm/s (Giddings, 1965) or 1030 cm/s (Mittlefehldt, 2002).
needed for a more exact description of chromatographic The optimum velocity in high-performance liquid chroma-
separation in a column of length L, which will allow also tography (HPLC) using small particles (~2 m) is around
a knowledge of N. Van Deemter et al. (1956) developed 0.1 cm/s (Majors, 2005) or 103 cm/s (Giddings, 1965). To the
such a description by comparing the Gaussian distribu- extent H may be represented by the dependence of 2zi on
tion equivalent of the concentration at the outlet of the Di;eff;z , values of H will be drastically different for gas chro-
column (plate N) given by (7.1.105m) with the concen- matography and liquid chromatography since diffusion coef-
tration profile for linear equilibrium, a long column and ficients for gaseous systems are larger than those for liquid
a differential solute feed pulse (equations (7.1.101f), systems by factors of 104105. For example, the values of H
(7.1.101g), V 0 ! 0). This concentration profile was a in HPLC vary between 5  104 to 30  104 cm (for ~2 m
simplified form of a solution developed by Lapidus and particles (Majors, 2005)). On the other hand, the values of H
Amundson (1952) based on mass-transfer effects as well in gas chromatography may vary, say, between 5  102 cm
as dispersion. The result of Van Deemter et al. (1956) is to 3  101 cm (Mittlefehldt, 2002). The range of variation
usually described by the following equation, called the observed in gas chromatography for the constants A, B and C
Van Deemter equation: in equation (7.1.107a) are: A 01 mm; B 10 mm2/s; C
B 0.001 0.1s; Hmin 0.5 2 mm (Moody, 1982). Note: Most
H A C vz : 7:1:107a
vz commercial chromatographic columns are run at a value of
vz larger than vzjmin . There is another reason for it. As the
Here, the A term is due to eddy dispersion and flow con-
particle sizes become smaller (5 m, 3.5 m, 1.8m), the Van
tribution to plate height and is independent of vz; it is a Deemter curve shown in Figure 7.1.24(b) flattens out; there-
function of the particle size and the packing efficiency. The fore, if operation is at vz > vzjmin , the increase in HETP is very
next term includes B, which depends on the molecular
little (Majors, 2005). This same reference provides an esti-
diffusion coefficient in the longitudinal direction, i.e. the mate of the number of plates in a 15 cm column as the
mobile-phase diffusivity. The third term includes C, which particle size is varied: 100 m, 200; 50 m, 1000; 10 m,
results from mass transfer between the mobile and station-
6000; 5 m, 12 000; 2.5 m, 25 000; 1.8 m, 32 500.
ary phases and has contributions from (1) diffusion in the
Additional developments have led to a different form of
film around the particle in the column, (2) diffusion in the the Van Deemter equation, where the plate height H may
liquid phase that is stagnant in the pores and (3) diffusion
be represented in the following fashion (Giddings, 1965,
in the liquid-phase coating on particles.
eq. (2.11.2), 1991; Horvath and Lin, 1978):
The dependences of each of the three terms on vz, the
mobile-phase interstitial velocity, are shown in Figure B X 1
H C vz : 7:1:107d
7.1.24(a). The nature of their variation with vz leads to an vz 1=A 1=C m vz
overall dependence of H with vz, which has a minimum at
The following alternative representation is also employed
a low value of vz. The minimum is obtained from
(Jennings, 1987; Karger et al., 1993):
dH=dvz 0 ) vzjmin B=C1=2 : 7:1:107b H H D H M H SM H S ; 7:1:107e
540 Bulk flow perpendicular to the direction of force

where Table 7.1.6.

HD is due to the longitudinal molecular diffusion, He flow rate Retention Peak base
HM is due to the mobile-phase contributions in a packed (cm3/min) time (cm) width (cm) H (cm)
bed,
HSM is due to the stagnant mobile phase in particle pores, 7.1 14 3.3 0.529
HS is due to the stationary-phase sorptiondesorption. 34.4 5.3 0.95 0.306
78.9 3.28 0.66 0.386
Expressions for the individual plate height contributions
are given below:
L 2 L=R0i vz 2
2
t Ri
    
2Dil 1 N
HD ; 7:1:107f zi zi =R0i vz zi =R0i vz
vz  2 2
t Ri t Ri t Ri 2
  
where Dil is the diffusion coefficient of species i in the 16 :
ti W bi =4 W bi
mobile-phase liquid and 1 is the obstruction factor
(<1.0) due to particles in a packed bed, which increases Example 7.1.7 One would like to determine the values of the
the diffusion path length; constants A, B and C in the Van Deemter equation (7.1.107a)
for plate height H in a gas chromatographic column by con-
1 ducting a few experiments. A minimum of three sets of data
HM i; 7:1:107g

h
1=2 d p Dil =3 d2p vz are needed to determine the three constants A, B and C. The
data in Table 7.1.6 were obtained by Moody (1982) from a
152.4 cm (5 ft) long 1/800 column operated at 82  C with a
where 2 and 3 are functions of the packing structure in
particular packing for a C4-compound, and helium as the
the packed bed;
carrier gas. Note: The retention time and the peak base width
1 m k il 2 d2p vz have been converted to length units. Determine the values of
H SM constant ; 7:1:107h A, B and C. What is the value of vzjmin ?
1 m 1 k il 2 p Dil
Solution Substitute the values of the helium flow rate and
where m denotes the fraction of mobile phase in particle plate height into the Van Deemter equation for the three
pores of tortuosity factor p and k i1 is the distribution ratio of cases as follows:
species i between the stationary and mobile phases;
B
0:529 A 7:1C;
4 k i1 d 2p vz 7:1
HS ; 7:1:107i B
1 i1 2 Dis 0:306 A 34:4C;
34:4
where 4 is a configuration factor dependent on the shape B
of the liquid dispersed on packing (as in LLC, see Table 0:386 A 78:9C:
78:9
7.1.5) and Dis is the diffusion coefficient of species i in the
Note: In the absence of the exact column cross section for
stationary phase. gas flow, we are using the gas flow rate instead of the actual
Example 7.1.6 In conventional gas chromatography, the gas velocity in the Van Deemter equation. Therefore the
typical quantity measured for species i is the retention time units of B and C will be different from those if vz were used
t Ri , under given conditions. To develop an estimate of the instead of the gas volumetric flow rate. Simultaneous solu-
number of plates N in the chromatographic column of length tions of these three equations yield the following values of
L, one also needs to determine the standard deviation i in the constants:
the column. Show that A 0:1329cm; B 2:665cm4 =min; C 0:00278 min=cm2 :
t Ri 2
 
N 16 If vz (cm/min, say) were used, the units of B would be cm2/
W bi min and those for C would be min. These units will become
will provide such an estimate for peaks that are assumed self evident if the conventional expressions for A, B and C in
Gaussian. Here W bi is the base width of the chromatographic terms of fundamental quantities of the chromatographic
peak. column and system are considered:
A dp ;
Solution The chromatographic peaks appear in the time
domain. The base width W bi of a Gaussian peak is essentially where is a dimensionless factor related to eddy
equal to 4 ti (see Section 2.5), where we have added a diffusion;
subscript t to indicate time units. In equation (7.1.106d),
B 2Di ;
N (L/zi)2, where zi is in length units. The effective migration
velocity of a peak of species i is R0i vz (from relation (7.1.99a)); where is a dimensionless factor related to the tortuosity of
this velocity also characterizes the time rate of migration of the flow path in the packed column; C has a number of
the base width of a chromatographic peak. Therefore contributions, as shown in equations (7.1.107gi).
7.1 Force ri in phase equilibrium: fixed-bed processes 541

7.1.5.1.5 Scale up or scale down packed columns for Q1


  2
adsorption/chromatography (elution) If one considers vz1 R21 Q 1 R2
: 7:1:108d
the plate height expressions provided in expressions vz2 Q2 Q2 R21
2
(7.1.107a) and (7.1.107fi), one observes that reducing the R2
particle diameter dp in the column in general leads to lower
Substituting this relation into the ratio (7.1.108c), we get
values of the plate height, and therefore a larger number of
   2   !
plates. Further, the peak capacity is increased due to a P 1 Q1 R2 L1 d2p2
reduction in i . This has been the driving force for a con- : 7:1:108e
P 2 Q2 R21 L2 d2p1
tinuous decrease in particle sizes in various chromato-
graphic techniques from values of 100 m in the 1950s to If the two packed beds employed two different particle
as low as 2 m in 2003 (Majors, 2005). It is useful to enquire sizes but had the same Q and R, then, to maintain the
about the effects of a change in the particle size in a packed same P, we must ensure
column for adsorption and chromatographic processes.
We have studied in earlier subsections how separation L1 =L2 d 2p1 =d 2p2 : 7:1:108f
takes place as a fluid containing various species to be
If d p2 is smaller than dp1 , L2 is going to be much smaller
separated moves through a packed bed of adsorbents or
than L1. Correspondingly, from relation (7.1.106c) for the
chromatographic medium. As the fluid flows, there is a
breakthrough time t, for a fixed vz (i.e. fixed Q and R), t will
significant pressure drop as a certain level of separation is
decrease with dp. Therefore, for the adsorptiondesorption
achieved. The question of interest here is: what happens if
cycle, the cycle time will have to be reduced considerably
the particle size is reduced? This is part of a broader
as the particle size is reduced:
question: how can one improve separation in a packed
column based process? L1 =t 1 L2 =t 2 ) t 1 =t 2 L1 =L2 d 2p1 =d2p2 :
The pressure drop experienced by a fluid as it flows through
7:1:108g
a packed bed of particles of diameter dp may be described by
the BlakeKozeny equation (6.1.4f), among others: Consequently, if d p2 < dp1 , t 2 << t 1 (Wankat, 1987).
1 2 150 Smaller particles will dictate a much faster breakthrough
P v0 L: 7:1:108a time and a much shorter cycle time unless the pressure
3 d 2p
drop is allowed to rise considerably in the following
Since v0, the superficial fluid velocity based on the empty fashion:
cross section, is related to the actual interstitial velocity vz
by v0 vz , we can rewrite the above relation as P 1 =P 2 d 2p2 =d 2p1 7:1:108h
!
vz 150 1 2 for constant Q, R and L. In fact, in high-performance liquid
P 2 L : 7:1:108b
dp 3 chromatography applications in practice, the particle sizes
can be as low as ~2 m, leading to very high values of
Generally, for rigid spherical particles, the bed porosity or pressure drops, of the order of 1001000 atm in columns
fractional void volume is independent of particle size. which are around 5 cm long (Majors, 2005). On the other
Unless the bed length and the bed diameter are vanish- hand, in larger-scale process applications, the adsorbent
ingly small (of the order of dp), is also independent of the particle sizes are ~1 mm. The pressure drops are much
bed length L and the bed diameter. Consider now two more moderate, the column diameter and length are
different packed beds: larger.
bed 1: length L1, particle diameter dp1, column radius R1, We have not discussed the nature of the separation
fluid volumetric flow rate Q1, velocity vz1, pressure being achieved as the scale of operation is changed from
drop P1; one column having one particle size to another with a
bed 2: length L2, particle diameter dp2, velocity vz2, smaller or larger particle size. One could wish to maintain
column radius R2, fluid volumetric flow rate Q2, pres- as constant the fractional column length lost in the adsorp-
sure drop P2. tion column, i.e.

The ratio of the pressure drops in the two beds is as follows LMTZ =L1 LMTZ =L2 ; 7:1:108i
(from relation (7.1.108b)):
! between the two scales of operation, where MTZ stands for
   
P 1 vz1 L1 d 2p2 mass-transfer zone.
: 7:1:108c Alternatively, from a chromatographic operation point
P 2 vz2 L2 d 2p1
of view, it may be desirable to maintain the same number
However, the ratio of the interstitial velocities is related to of theoretical plates, N, between the two scales of
the two volumetric flow rates and the two column radii by operation:
542 Bulk flow perpendicular to the direction of force

N 1 N 2: 7:1:108j Although all such chromatographic media may be used for


the separation of microsolute species, all three techniques
In practical chromatographic separations, on the other are used extensively for the separation of mixtures of pro-
hand, usually the driving force for smaller particles is teins/biomacromolecules. On the other hand, when the sta-
higher and higher N, since we have already pointed out tionary phase is a liquid coated on solid particles/beads, we
that smaller dp leads to lower H and therefore higher N for have liquidliquid chromatography (LLC); this technique is
a fixed L. used for smaller molecules. We will now provide an
From a quantitative point of view vis--vis the chroma- extremely brief introduction to each of these elution tech-
tographic operation, if the plate height is controlled by niques. A reasonably comprehensive introduction to
pore diffusion (in the stagnant mobile phase, equation analytical-scale chromatography using all four techniques is
(7.1.107h)) and /or stationary-phase sorptiondesorption, available in Karger et al. (1973). A comprehensive introduc-
the plate height H / d 2p vz. Since N (L/H), for a fixed L, tion to the first three techniques for bioseparations, including
L N 1 H 1 N 2 H 2: process-scale operations, is available in Ladisch (2001).
The general characteristics of liquidsolid adsorption
Therefore the ratio of the number of plates for the two based elution chromatography developed in Sections
columns of same length is 7.1.5.1.1 and 7.1.5.1.4 are also valid here. For example,
N 1 =N 2 H 2 =H 1 d2p2 =d 2p1 ; 7:1:108k the retention volume V Ri for species i in a column having
a stationary phase volume VS is related to the mobile-
assuming vz is constant. However, the P will be quite phase volume within the column, VM, and the solute parti-
different for the two columns by (7.1.108c). If, we cannot tion coefficient i1 by relation (7.1.99h):
assume pore diffusion control, but can represent the
V Ri V M V S i1 ;
HETP via
H a vnz d n1 where
p ; 7:1:108l

then Wankat (1987) has obtained the following result in a i1 C i1 =C i2 C iR =C iw ; 7:1:109a


straightforward fashion: with subscripts R and w representing the resin phase (the
stationary phase) and the mobile phase (aqueous phase
constant Q: N 1 =N 2 d p2 =dp1 n1 L1 =L2 R2 =R1 2n : j w), respectively. This partition coefficient has now to be
7:1:108m related to the concentration/activity of the salt (for
example) species in the eluent. We have seen earlier (in
Additional results for a number of ratios, (cycle time)2/
Section 4.1.9.4) that salts such as ammonium sulfate are
(cycle time)1, (adsorbent volume)2/(adsorbent volume)1,
used in hydrophobic interaction chromatography to dis-
etc., may be developed. (Note: n 1 represents pore
place proteins from ion exchange resins. In ion exchange
diffusion control.) Similar considerations may be
chromatography, NaCl plays a similar role in modifying the
employed for the estimation of the ratio (LMTZ/L) in rela-
ion exchange behavior of proteins. In the case of ion
tion (7.1.108i) provided an expression for LMTZ is employed
exchange chromatography of small ions, a variety of differ-
in terms of the mass-transfer coefficient, the particle sur-
ent species can act as the eluent species, e.g. KCl, HCl,
face area, etc. (Wankat, 1986, 1987). Wankat (1987) has
K3PO4, M2O3 (where M stands for a metal), etc. We identify
provided the following result:
such an eluent component/salt (a counterion) by s and
 n1    2n
L=LMTZ 1 d p2 L1 R1 assume that it is monovalent, s, i.e. Zs 1.
7:1:108n We first consider the separation of two charged/ionic
L=LMTZ 2 d p1 L2 R2
species A and B, where they are represented as AZA and
for the same volumetric flow rate through the two BZB . The counterion eluent component/salt s initially pre-
columns. sent in the ion exchange resin will exchange with each of
A and B via the corresponding ion exchange (see (3.3.121c)
7.1.5.1.6 Elution chromatography with a mobile liquid for guidance):
phase ion exchange chromatography The treatment
jZA j jZs j R
sZs jZs j AZA , jZs jjZA jR
AZA jZA jsZs ;
of elution chromatography with a mobile liquid phase con-
sidered so far involved liquidsolid adsorption chromatog- 7:1:109b
raphy (LSC) (see Sections 7.1.5.1.1, 7.1.5.1.2, 7.1.5.1.4 and jZB j jZs jR
sZs jZs j BZB , jZs j jZB jR
BZB jZB jsZs :
7.1.5.1.5). There are three other forms of stationary solid 7:1:109c
phase commonly used: ion exchange resin beads, porous
hydrogel beads (generally crosslinked) and affinity beads, The equilibrium constants KAs and KBs for these two ion
having functional groups with which the solute has an exchange reactions are (in activity units; see (3.3.121e)
extremely strong affinity to the exclusion of other solutes. and (3.3.122b) for activity and molar concentration units)
7.1 Force ri in phase equilibrium: fixed-bed processes 543

asw jZA j aAR jZs j asw jZB j aBR jZs j Consequently, plots of the capacity factor will have a
K As jZA j jZs j
; K Bs jZB j jZs j
: dependence on asw similar to that of i1 or V N i .
asR aAw asR aBw
The behavior of log V N i at low concentrations of the
7:1:109d
eluent ion, Csw (or asw) (Figure 7.1.25(a)) indicates that the
We need to recognize two realities in elution based ion difference between, say, V N A and V N B , is quite large.
exchange chromatography: (1) the concentrations of A and Therefore, the retention volumes for ions A and B will be
B are quite small, therefore the eluent counterion s, whose very far apart, i.e. the peaks will emerge from the column at
concentration is much larger, occupies virtually all of the very different times, leading to considerable wastage of the
resin sites; (2) the ion exchange equilibrium constant KAs, mobile phase. The conventional method of reducing this
or KBs, is essentially a constant vis--vis the concentration mobile-phase loss and causing the peaks to come out
levels. closer in time is gradient elution. This is achieved by
We are interested in i1 (see (7.1.109a)) so that we can changing the eluent ion concentration Csw (or asw) with
predict V Ri , i.e. V RA and V RB . If for species A we write time to a larger value so that V N B is much less compared to
    what it would have been at a lower Csw. (Constant eluent
C AR aAR
A1 ; 7:1:109e composition, temperature and flow rate is identified as an
C Aw aAw
isocratic mode of operation.)
then, if jZs j 1 (say, the salt is NaCl), We may explain this using the retention time t Ri for a
given species i. By definition (7.1.99d), t Ri L=R0i vz .
A1 K As asR jZA j =asw jZA j : 7:1:109f However, for a species which passes through without inter-
acting with the ion exchanger or the stationary-phase
Correspondingly,
material, R0i 1, since its vCi vz . Identify this as the
B1 K Bs asR jZB j =asw jZB j : 7:1:109g retention time for the mobile phase, t RM , the time taken
by the mobile phase introduced with the sample to come
Using definition (7.1.99j) of the net retention volume of to the column exit. Therefore,
species i, V N i , we get
t RM L=vz : 7:1:109m
log VN i log V Ri VM log VS i1 log VS log i1 ;
7:1:109h We may now write, for any species i which undergoes
adsorption, partitioning, ion exchange, etc., with the sta-
log V N A log V S log A1 tionary phase,
log V S log K As jZA j log asR log asw : 1
V Ri V M V M =R0i V M R0i 1
7:1:109i k 0 il
VM VM 1
Further, L L
R0i vz vz t Ri t RM
: 7:1:109n
log A1 log K As jZA j log asR log asw ; L=vz t RM

log i1 log K is jZi j log asR log asw : 7:1:109j From relation (7.1.109l), for two eluent ion concentrations
asw j1 < asw j2 , we get (see Figure 7.1.25(a))
Therefore plots of log V N A (or log V N i ) and log A1 (or log
i1 ) against log asw will be straight lines with negative slopes k 0 i1 j1 > k 0 i1 j2 and t Ri j1 > t Ri j2 : 7:1:109o
of jZi j, as shown in Figure 7.1.25(a). For two species A and
In the separation of a number of proteins by elution
B whose V N i differ because A1 and B1 are different, the
based ion exchange chromatography (see Ladisch (2001)
difference between V N A and V N B becomes substantial as
for much greater detail), first the sample is injected. Differ-
asw decreases; at low concentrations of the salt (say), the
ent proteins undergo ion exchange based binding with the
centers of the two elution peaks A and B are substantially far
resin (see Figure 4.1.16). Soon after the sample injection,
apart. On the other hand, if jZA j jZB j, the plots of log A1
the salt concentration in the mobile phase is increased
and log B1 vs. log asw will be parallel to each other.
with time, say, in a linear or stepwise fashion. Therefore,
In chromatographic literature, the quantity k 0i1 , the
the proteins which are bound more strongly, and therefore
distribution ratio or the capacity factor, is frequently used:
have a larger value of V Ri or t Ri at a given asw, will come
VS V Ri V M 1 out much earlier, since asw has been increased. Figures
k 0i1 i1 i1 ; 7:1:109k
VM VM 7.1.25(b) and (c) illustrate this behavior. Figure 7.1.25(b)
illustrates the elution profiles for proteins A and B, where
which leads to
the salt concentration in the eluent is constant at a low
1 value of asw j1 ; as a result, t RA and t RB are far apart. Figure
log k 0i1 log K is jZi j log asR log asw :
7.1.25(c) shows that gradient elution is started close to the
7:1:109l time when protein A elutes by a step increase in salt
544 Bulk flow perpendicular to the direction of force

(a)

D
4 4
D

logk i l
il
3 3
logVNi

log
B
B
2 2
A
A
1 1

1 0.8 0.6 0.4 0.2 1 0.8 0.6 0.4 0.2


log asw log asw

(b) (c)

Salt concentration
Protein concentration

Protein concentration
3 3 increased

2 2

1 1

0 0

0 tinj tRA tRB 0 tinj ts tRA tRB


t t

constant asw
asw
1 1 from t = 0 to t = ts
asw 2 > asw 1 from t ts

Figure 7.1.25. Ion exchange chromatography. (a) Variations of net retention volume V N i and partition coefficient i1 as well as the
capacity factor k0i1 with eluent ion activity. Gradient elution is depicted in (b) for constant eluent ion concentration and in (c) for
stepwise variation of eluent ion concentration.

concentration. The result is that the protein B peak comes same time (no separation). Beyond the intersection, at
out much earlier. Instead of a step change in salt concen- higher salt concentrations, species D will elute faster than
tration, the salt concentration in the eluent may be varied species B (or A if there is an intersection). Such behavior
continuously, for example in a linear fashion. The typical leads to complications; alternatively, it may be an oppor-
concentration variation of NaCl used as a result is from tunity for improved separation.
0.01 M to 0.1 M (up to 0.3 M).
The discussion of elution of two protein/ionic species 7.1.5.1.7 Elution chromatography with a mobile liquid
has so far employed species A and B identified in Figures phase size exclusion chromatography We will now
7.1.25(a)(c); both species had identical charges. Species briefly consider size exclusion chromatography (SEC),
D in Figure 7.1.25(a), however, has a different charge, in which is also called gel permeation chromatography
fact a higher charge jZD j > jZA j jZB j. Therefore, the (GPC), gel filtration or gel chromatography. In Section
negative slope of log V N D or log D1 vs. log asw is larger 3.3.7.4, we came across porous crosslinked polymers called
than that for species A or B. The difference in the two hydrogels and the partitioning behavior of macromol-
slopes leads to the following: at lower salt concentrations, ecules/proteins between their solutions and the hydrogels
D will elute much later; however, at the salt concentration (e.g. im given by (3.3.90i)). Generally, the lower the
where the two lines intersect, two species will elute at the molecular weight and the smaller the effective radius (e.g.
7.1 Force ri in phase equilibrium: fixed-bed processes 545

(a) (b) (c)


7 6 9

6 8
5

log [h]i Mi
log Mi

log Mi
5 7
4
4 6

3
3 5
0 1 1.5
im VRi VRi

Figure 7.1.26. Characteristics of size exclusion chromatography/gel permeation chromatography. (a) Partition coefficient of macromol-
ecular species i between solution and gel particle: Mi is the molecular weight of macromolecular species. (b) Elution volume of
macromolecular species i. (c) Universal calibration plot of many polymers in tetrahydrofuran (THF): log []i Mi vs. elution volume
V Ri . (After Grubisic et al. (1967).)

the radius of gyration, rg; hydrodynamic radius, rh), the


 
1
V Ri V M V M p im ; 7:1:110b
higher the value of im (Figure 7.1.26(a)). A larger pro-
tein/macromolecule has a lower im ; consequently, it will
V Ri V M V S p im V M V Sp im ; 7:1:110c
spend less time in the porous hydrogel phase and will be
eluted faster in a chromatographic column containing where VSp is the volume of the mobile phase present inside
hydrogel beads as the packing/sorbent phase. Let there be the porous hydrogel particle. Therefore the retention
no other type of interaction between the macromolecular volume of a species, V Ri , varies between the value of VM
solute and the porous hydrogel except geometrical parti- VSp for a very small molecule, such as salt, to VM for a
tioning between the pore liquid and the external liquid protein/macromolecule completely excluded from the
(e.g. (3.3.88a) and (3.3.89a)); this interaction is dependent pores of the gel particle. Employing one of the relations
on the pore size, solute size, pore shape and solute shape. (3.3.90i) (e.g. im a b log M i ), the following relation is
Here the value of im ! 1 when the solute size tends to zero. obtained between V Ri and the molecular weight Mi of the
In such a situation, there is no solute adsorption as such, so macromolecular species:
that q0i1 C i2 in equation (7.1.13f) is zero and
vz V Ri V M V S p a V S p b log M i ; 7:1:110d
vCi : 7:1:110a
1
1 p im which may be rearranged as follows:

V Ri p1 p2 log M i : 7:1:110e
Therefore, a large macromolecule/protein which cannot
enter the pores of the hydrogel beads will have a concen- As shown in Figure 7.1.26(b), the retention volume in size
tration velocity vCi vz , the interstitial fluid velocity. On exclusion chromatography/gel chromatography appears to
the other hand, a small solute such as salt will probably increase almost linearly with a decrease in Mi in a semi-
have im 1 and its vCi << vCi jprotein . logarithmic plot, except at the low end of Mi.
We have also obtained earlier, for the case of solute Important separation applications of this behavior
adsorption, the following relation for species i between the appear in desalting protein solutions and exchanging buffer
retention volume V Ri and the solute adsorption/partition solutions. Salts like ammonium sulfate (used in the precipi-
coefficient i1 (equation (7.1.99h)): tation of selected proteins (see Section 3.3.7.4 and Ladisch
  (2001)) or NaCl (used in ion exchange chromatography)
1
V Ri V M V S i1 V M V M i1 : may be easily removed from the solution of the protein/s

by passing the salt-containing solution through a gel
This was obtained from the differential equation (7.1.95a) particle filled column. Molecules used to develop buffer
for elution chromatography. If instead we worked with the solutions are usually much smaller than proteins; gel chro-
differential equation (7.1.95c), where i1 0, we would matography will also allow the purification of a protein
have obtained solution vis--vis the constituents of a buffer solution so
546 Bulk flow perpendicular to the direction of force

that other buffer constituents may be added to the purified log i M i log MjPS 7:1:110f
protein solution.
Such size exclusion based separations are carried out at the same elution volumes (Grubisic et al., 1967). Since
using hydrogel particles, which have a variety of the MarkHouwink equation relates the intrinsic viscosity
commercial names, e.g. Sephadex , Biogel , Fractogel ,
[] of a polymer fraction to its molecular weight via
Sepharose , Ultragel , etc. Sephadex , the trade name
i K i M i ai ; PS K PS M PS aPS ; 7:1:110g
for the earliest gel particle, was developed by the reaction
of dextran and epichlorohydrin (Porath and Flodin, 1959).
we obtain, for a given elution volume, the molecular
The range of molecular weights of molecules/biomole-
weight Mi of the polymer i fraction as
cules/proteins separated by the list of commercialized
gel particles identified above vary between 2  102 and 1 1 K PS 1 aPS
log M i log log M PS : 7:1:110h
 108. Although these gel particles do have a size distribu- 1 ai Ki 1 ai
tion, typical diameters are around 40165 m. Further,
these particles are somewhat soft and compressible; there- Therefore, knowing ai, aPS, KPS, Ki and MPS for the given
fore the columns are operated at quite low pressure drops. elution volume, Mi may be determined corresponding to
For example, typical column pressure drops allowed vary that elution volume.
between 10 and 300 cm H2O. For details on laboratory- One needs, however, the intrinsic viscosity relation
scale operation, see Scopes (1994). parameters for both PS as well as the polymer i under
Gel permeation chromatography has another very consideration for the given solvent, say THF, and the gel
important application in the separation of macromolecules particle under consideration. This procedure works
which are not biomacromolecules. Many organic polymers because of the following reason. We have observed the
are soluble in organic solvents. A number of gel particles following in relations (3.3.90eg):
are obtained from crosslinked styrenedivinylbenzene,
(1) the effective dimension of the polymer molecule in the
polyvinylacetate, porous silica, etc. (e.g. Styragel , Bio-
form of a polymer coil given by rg, the radius of gyr-
Beads , Porasil , etc.), which can be used with a variety
ation, is proportional to the hydrodynamic viscosity
of organic solvents, nonpolar as well as polar. A solution of
based radius, rh;
an organic polymer in an organic solvent will generally
(2) the product i M i / V h / r 3h , where Vh is the hydro-
have a molecular weight distribution (see Figure 2.4.8).
dynamic volume of the polymer fraction;
Such a sample solution when injected into an appropriate
(3) rh (or rg) controls the partitioning behavior of the
gel column will elute various monodisperse fractions (the
macromoleculer vis--vis the gel, and therefore the
species in such a fraction have molecular weights in a very
macromolecular separation behavior.
narrow band), whose average molecular weights will
decrease with increasing time. Therefore, GPC will allow There are two other important aspects of macromolecular
fractionation of a polymer sample into narrow molecular fractionation in organic solvents. For crosslinked polystyr-
weight fractions. If an appropriate calibration curve is ene based gel beads (e.g. Styragel), only certain solvents
available which correlates the retention volume of a spe- such as THF, benzene, xylene, etc., may be used so that the
cific eluted fraction with its molecular weight, then one can pores in the gel remain open, since PS is soluble in these
find out the molecular weight distribution of any sample solvents. Porous glass beads are therefore often used, since
injected into the column. a variety of solvents, including water and polar organic
This goal is achieved in the following fashion. Polystyr- solvents, are easily used without any pore closing prob-
ene (PS) is available commercially in narrow molecular lems. Pore diameters can vary between 50 and 10 000 nm.
weight fractions. One first determines the elution volumes Column pressures may go up to 10004000 psi (68
of individual PS fractions in a solvent, say tetrahydrofuran 272 atm). For an introduction to GPC using organic solv-
(THF). It has been found that a semilog plot of the product ents, see Allcock et al. (2003).
[](M) of different polystyrene fractions against the elution
Example 7.1.8 In a small laboratory column for SEC for
volumes of different PS fractions in THF provides a univer- proteins, it is known that the biomacromolecule blue dextran
sal calibration curve in GPC; here [] is the intrinsic viscos- (mol. wt. 2  106, having a blue dye attached to it), is
ity of the PS fraction of molecular weight M (see relation completely excluded. The total column volume is 20.5 milli-
(3.3.90f)) (for PS, standard samples of defined molecular liter (ml). An injected sample of blue dextran comes out with
weight are available commercially). Many different poly- the center of its peak around 7.2 ml. An injected sample of
mers have been found to obey the same behavior salt (NaCl) comes out with the center of its peak around
developed for PS fractions (Figure 7.1.26(c)). Therefore, if 15.1 ml. An introduced unknown protein sample has an elu-
such a universal calibration curve is available, the elution tion volume of 10 ml.
volumes of different polymer fractions of a sample of any (1) Determine the void volume of this gel column.
other polymer i coming out in solvent THF will have its (2) Calculate the fractional void volume of the gel particles.
7.1 Force ri in phase equilibrium: fixed-bed processes 547

(3) Estimate the value of im for the unknown protein. an antigen and an antibody. Further, this binding is revers-
(4) Estimate the radius of this unknown protein if the gel ible, using an appropriate pH or ionic strength, or a species
pores are assumed to be cylindrical with rp 10 nm. which acts as a displacer or modifier releasing the protein.
Solution However, the protein will have to be purified from the
displacer/modifier. Note: In chromatography, competing
(1) Since blue dextran is completely excluded, im 0 and species adsorb/bind to the beads/substrate to different
its V Ri V M 7:2 ml. Since the total volume of the extents. However, in affinity chromatography, under
column is 20.5 ml, the void volume of the column ideal/expected conditions, only the target protein is
7:2=20:5 0:351: adsorbed/bound. Therefore affinity adsorption would also
be an appropriate name.
(2) Since salt will most certainly have a value of im 1:0
A simplistic pictorial representation of such a process
in the gel particles, we have
is provided in Figure 7.1.27(a). An insoluble hydrophilic
V Ri V M V Sp im 7:2 V Sp  1:0 15:1 ml; stationary phase of polyacrylamide, polysaccharide, cross-
where V Sp is the volume of the mobile phase present linked dextran, etc., provides the matrix, or support gener-
inside the porous particles. Therefore ally, in the form of a bead. On the surface of the pores in
such a matrix, ligands are attached by specific chemistries.
V Sp 7:9 ml: In the figure, we show that protein A, has been attached to
However, the volume of the stationary phase V S in the the surface of a hydrogel bead pore. This protein (mol. wt.
column is (20.5 7.2), i.e. 13.3 ml. The fractional void 41 000) is obtained from the outer coat of the bacteria
volume of the gel particles, is given by Staphylococcus aureus. Antibodies of various kinds are
of great importance in modern therapeutic approaches.
V Sp
p The antibodies have the Y-shaped structure of immuno-
column volume mobile phase volume
globulins (IgGs). It is known that protein A interacts with a
V Sp 7:9 great deal of specificity with the so-called Fc region of the
) p 0:594:
stationary phase volume 13:3 IgG molecule. A variety of other proteins, impurities, etc.,
present in the feed/sample solution do not get attached to
(3) V Ri V M V Sp im ) 10 ) 7:2 7:9 im the surface-bound protein A. Once the affinity adsorption/
2:8 attachment process is over, elution is initiated by reducing
) im 0:35:
7:9 the pH/ionic strength. Generally, at a selected pH, the
attached IgG is desorbed and eluted out. For a comprehen-
(4) Employ equations (3.3.88a) and (3.3.89l): sive introduction to such separations, consult Boyle et al.
p   (1993), Scopes (1994) and Ladisch (2001).
C im ri 2
im 1 ; It is useful to note a few other characteristics of this
C il rp
separation technique.
0 12 0 1
r i r i (1) The pH used to elute the adsorbed IgG should be
0:35 @1 A ) 0:595 @1 A
10 nm 10 nm selected to ensure that the bonding of the affinity
ligand (sometimes called an affinant) to the matrix is
) r i 10  0:405 ) r i 4:05 nm:
unaffected.
(2) In spite of such steps, such affinity ligands are leached
7.1.5.1.8 Elution chromatography with a mobile liquid
out of the matrix at very low levels; they have to be
phase affinity chromatography, membrane chroma-
removed from the eluate.
tography Two chromatographic methods described
(3) Although the targeted biomacromolecule (e.g. IgG) is
earlier, size exclusion chromatography (SEC) and ion
adsorbed with high specificity, other proteins from the
exchange chromatography, are frequently used for the
feed/sample may also be adsorbed somewhat via ion
separation of biomacromolecules. However, both pro-
exchange/hydrophobic interactions, etc., between
cesses have limited selectivity. For example, in the process
these proteins and the substrate ligands.
of SEC for proteins, whose molecular weights/sizes are not
too far apart, one cannot achieve high selectivity for one This technique provides, in principle, a way of selectively
protein with respect to another (however, desalting and removing a target protein, enzyme, antibody, etc., from a
buffer exchange can be implemented with ease). Similarly, mixture via highly specific molecule-to-molecule inter-
in ion exchange chromatography, specificity between two actions. Such an activity requires highly selective ligands,
proteins is often quite limited. On the other hand, in which are currently being developed via specialized
affinity chromatography, specific ligands exist on the techniques.
adsorbent/bead surface which interact only with a specific These ligands are quite costly. It is desirable that the
protein. An example of such an interaction is that between ligands present inside a substrate matrix, e.g. crosslinked
548 Bulk flow perpendicular to the direction of force

(a) (b)
Solution/sample/eluent in
Bead
Stack of porous
membranes

Protein
Support IgG
A P1
surface Pore
One membrane Flow through
Pore membrane pore
surface
IgG P2 Ligand

Protein A

Figure 7.1.27. Affinity chromatography. (a) Protein A based affinity chromatographic removal of an immunoglobulin by a bead in a
packed bed. (b) Flat porous membrane based affinity chromatography.

agarose, etc., are utilized for binding to the fullest extent. Modeling of the performance of a stack of flat mem-
Although the beads are highly porous gels, diffusional rates branes, each having ligands in the pores (Figure 7.1.27(b))
of the large proteins into the pores are necessarily low. as a protein-containing solution flows through, has been
Therefore fractional ligand utilization is low, resulting in a carried out by Suen and Etzel (1992) and Gebauer et al.
costly inventory, especially if the mobile phase has a higher (1997). The governing equation for adsorption of a protein
velocity. These limitations are overcome by a technique from a solution as it flows through the membrane pore is
called membrane chromatography. still equation (7.1.4). However, with proteins, local equilib-
We have already observed in Figures 4.1.28(b) and (c) rium cannot be assumed; attention has to be focused on
how polymeric chains anchored on the surfaces of pores of the protein-binding kinetics. Suen and Etzel (1992) have
ka
a membrane can bind metals, etc., present in the solution employed P S kd
PS to describe the proteinligand inter-
flowing through the membrane pores. This concept essen- action (see equation (4.1.76)) kinetics. Their detailed
tially applied the principle of membrane chromatography model should be studied to determine the effects of axial
developed first with affinity adsorption of proteins to solve diffusion, flow velocity, proteinligand association kinetics,
the problem of removal of metals from a solution (Brandt membrane porosity and the variation in membrane
et al., 1988). Affinity ligands attached to the membrane thickness.
pore surfaces pick up proteins/antibodies/enzymes from A few details are, however, useful. The value of the
the solution in a highly selective fashion. The membranes ligand capacity in one membrane is quite low. Therefore,
may be in the form of flat films (or hollow fibers, see breakthrough can happen quickly unless the flow velocity
Section 7.2). However, the amount of pore surface area is kept low. When a number of membranes are stacked
available in one flat membrane (which is usually quite together (Figure 7.1.27(b)), the breakthrough can be sub-
thin) is not substantial. Therefore a stack of membranes stantially delayed; alternatively a higher flow velocity may
is used (Figure 7.1.27(b)). be utilized. Further, the breakthrough can be delayed if the
Usually highly porous microfiltration membranes ligand capacity on the membrane pore surfaces is large
(Sections 6.3.3.1.1 and 7.2.1.4) are employed. The convect- compared to the protein concentration in the feed solu-
ive velocity along the pore length brings the species to be tion. The pressure drop can, however, be controlled to a
adsorbed to each ligand very quickly. Ligand utilization is lower level compared to that in a packed bed of affinity
very high. As a result, the ligand inventory is quite low beads where intra-bead diffusion is quite slow and there-
compared to that in porous beads. fore ligand utilization will be low.
7.1 Force ri in phase equilibrium: fixed-bed processes 549

We will now very briefly describe another mobile- In normal LLC, the stationary-phase liquid is usually
liquid-phase chromatography technique having affinity more polar than the mobile phase. The liquid immiscibility
in the name: immobilized metal affinity chromatography criterion, however, requires 1 2  4 (Karger et al.,
(IMAC). It is also used in bioseparations, specifically in 1973). Karger et al. (1973) have demonstrated that, for
protein separations. In Section 5.4.2.1, we observed hydrocarbon compounds which are relatively nonpolar,
macromolecules having specific ligands bind metal ions 2 i cannot be made much less than 0, which with
in solution. In particular, polymers having iminoacetic 1 2  4 leads to Ki 0. This had led to reverse-phase
acid ligands are chelating agents for divalent cations such chromatography, where the stationary-phase liquid is non-
as Cu, Zn, Ni, etc. If an adsorbent is formed by polar and the mobile phase is polar. Nonpolar hydrocar-
attaching to its surface a molecule having, say, an iminoa- bon compounds then have high Ki values.
cetic acid chelator group hanging out, then such a group The problem with the coated liquid phase is that it may
will bind a divalent cation, e.g. Cu, present in solution be lost by solubilization with time in the mobile phase. To
in the mobile phase. Although this divalent cation is avoid it, the stationary-phase liquid may be chemically
tightly bound now to the adsorbent particle via chelation, bonded to the stationary-phase particle material. For a
not all of its coordination spheres (d-orbitals) are occu- review of the preparation of nonpolar bonded stationary
pied. It still has complexation ability. It has been found phases having hydrocarbon-like chains grafted onto the
that basic groups in proteins, histidine residues (for particle support surface, see Snyder and Kirkland (1979).
example), will bind with the partially complexed divalent
metal ion (which is part of the adsorbent particle). Other
7.1.5.2 Frontal chromatography and displacement
electron-donating side chains in proteins will also com-
chromatography
plex with the adsorbent-bound divalent metal. Trivalent
metal ions of iron and aluminum have also been used to We have already encountered frontal development in
this end. Figure 7.1.5(c). The breakthrough times for different fronts
In practice (Scopes, 1994), columns are filled with were provided in equation (7.1.22b) on the basis of the
the adsorbent particles having the ligands. Then a dilute nondispersive equilibrium adsorption model. Here the
salt solution of the metal, e.g. CuSO4, etc., is passed mobile phase moving through the column is the feed
through the column till it is saturated with the metal solution to be separated; a pure output is obtained only
ion. Excess metal ions are washed out using an appro- for the species that is least strongly adsorbed. Subsequent
priate weakly metal-complexing compound such as gly- components of the feed appear as a step contaminated by
cine to ensure that there are no free metal ions floating the species which have come out earlier.
around. Only then can proteins be separated by chro- Displacement chromatography involves initial feeding
matography, namely IMAC, through such a column. The of a large feed sample pulse into a column free of anything
actual operation is carried out at a high ionic strength, except pure mobile phase, say liquid flowing through the
e.g. 1 M NaCl. The technique was introduced by Porath column. Elution of the adsorbed species is next achieved
et al. (1975); an extensive earlier review is available by passing a displacer, generally a strongly adsorbed
(Porath, 1992). species, which displaces/replaces the species already
adsorbed from the sample. For a gaseous mobile phase,
7.1.5.1.9 Elution chromatography with a mobile liquid we have already seen an example of NH3 as a displacer for
phase: liquidliquid chromatography In traditional the linear paraffines of medium molecular weight
liquidliquid chromatography (LLC), the mobile liquid adsorbed in molecular sieve adsorbents (Figures 7.1.10
phase flows through a column packed with porous solid (b) and (c)). In general, controlled removal of the
beads/particles, whose surface has been coated with a adsorbed species from the adsorbent is achieved by a
liquid, which acts as the stationary phase. Partitioning of more strongly adsorbed substance, the displacer. The
the solute takes places between the mobile-phase liquid column has to be regenerated afterwards vis--vis the
(j 2) and the stationary-phase (j 1) liquid coating. displacer, for example by heating before the next feed
Partitioning of the solute i between the mobile- and pulse is introduced.
stationary-phase liquids may be determined, for example, For multiple solutes present in the feed sample pulse,
from a knowledge of the solubility parameters in the same there will be adsorption of each solute in the column.
manner as in liquidliquid extraction (e.g. equation Usually the solutes are present at a high concentration.
(4.1.34n)), repeated here for convenience: When the strongly adsorbed eluting solute displaces the
adsorbed solutes, each adsorbed solute goes down the
V i i 2 2 i 1 2

n K i : column and appears as a band at the column outlet; the


RT
multiple solutes appear one after another, with the bands
The equilibrium ratio Ki increases as i becomes closer overlapping. The displacer peak/band appears at the very
to 1 , the solubility parameter for the stationary phase. end.
550 Bulk flow perpendicular to the direction of force

(a) (b) (c) (d) (e) (f)


Flow
Regenerant
adapter Eluent flow
Cell/debris flow
Buffer out out with
solution

Buffer flow Feed solution Washing solution Eluent out


Packed Regeneration
adsorbent Fluidized (expanded bed) (expanded bed) Elution (packed adsorbent
bed (expanded bed) (packed adsorbent bed)
bed)

Figure 7.1.28. Different steps in a cyclic expanded-bed adsorption process for protein recovery/purification from a fermentation
broth/lysate.

7.1.6 Expanded bed adsorption (EBA) from a broth/ We will now describe the cyclic process followed in the
lysate aided by gravitational force expanded-bed adsorption process. The cyclic process is
implemented to start with using the buffer solution intro-
Products obtained via biotechnology are either extracellu-
duced from the bottom. Then the feed solution containing
lar or intracellular. Product recovery is implemented cor-
cells/cell debris is introduced into the expanded bed
respondingly from a fermentation broth with whole cells or
(Figure 7.1.28(c)). The cells/cell debris are carried through
from a lysate containing ruptured bacterial cells in a sus-
the space between the adsorbent particles, since the fluid
pension and product in solution. Conventionally, the cel-
velocity exceeds the terminal velocity of the cells and cell
lular material is often removed first via filtration/
debris. Due to the same reason, the density of the adsorb-
centrifugation; the product in the solution is then
ent particles is increased by having a quartz core, or even a
recovered/purified by a variety of steps including adsorp-
metallic alloy core, in the particles; these heavier adsorbent
tion/chromatography. (These processes are carried out in
particles should not be swept away, but should undergo
different devices generally with a holding tank in between.
mild/incipient fluidization. Further, often a size-
Xu et al. (2005) have developed one hollow fiber mem-
distributed adsorbent particle population (50400 m) is
brane device whose shell side is filled with chromato-
employed, with smaller particles moving around near the
graphic resin particles to achieve cyclical filtration,
top and the larger particles near the bottom of the
adsorption and elution of different protein peaks (see
expanded bed (see Figure 6.3.3(b)). Meanwhile, proteins/
Figure 11.1.3). Here we focus on an adsorbent bed where
biomacromolecules from the unclarified feed are adsorbed
the functions of filtration, adsorption and elution are
on the surface of these adsorbent particles.
carried out cyclically.)
Once the adsorption process is over, the flow of a
If such a feed is introduced from the bottom into
washing solution is initiated to wash away undesirable
a fixed adsorbent bed, the bed will immediately become
material (Figure 7.1.28(d)). Then the liquid flow is stopped
clogged by the whole cells or the lysate particulates/cell
and the particles settle down. Next, a flow of the buffer is
fragments, etc. (Figure 7.1.28(a)). The fixed bed will act as a
initiated in the opposite direction, leading to densification
granular filter (see Section 7.2.2). However, if the bed is
of the expanded bed into a packed bed on the bottom
expanded somehow, for example by fluidization, then the
support screen. Finally, the eluent is passed (Figure
cells and cellular debris can pass in between the adsorbent
7.1.28(e)) from the top to the bottom. Now the bed is
particles and out from the top of the bed (Figure 7.1.28(b)).
packed, so there is chromatographic elution, different
Obviously, before the start of the fluidization process, there
peaks of different protein species emerging at different
has to be a significant distance between a flow adapter
times in the eluent, which leaves through the bottom of
(also called a column adapter) at the top of the bed
the column. Usually, such a chromatographic elution pro-
and the actual top of the fixed bed (as shown in Figure
duces only partial purification of the proteins since the
7.1.28(a)).
7.1 Force ri in phase equilibrium: fixed-bed processes 551

whole bed is loaded with the protein in varying amounts. opposes the solute velocity in the chromatographic
Additional chromatographic purification steps are needed. column, R0 i vz vCiz . Therefore the net velocity of species
After elution through the bed, the bed is washed using a i down the column is
buffer solution flowing downward to remove undesirable
viz;net vCiz jU iz jnet ; 7:1:111
proteins and regenerate the bed to be used for the next
cycle (Figure 7.1.28(f)). where jU iz jnet is less than jU iz j due to the partitioning going
In this cyclic process of adsorption/elution, the forces on between the mobile and the stationary phases. Assume
causing adsorption/elution are perpendicular to the fluid now that the chromatographic/adsorbent column consists
flow direction (upward or downward). However, the force of two distinct regions j 1 and j 2 so that vCiz has two
of gravity acting on the particles is parallel to the direction different values in the two regions, vCi1 and vCi2 . Conse-
of bulk fluid flow. Two different bulk flow vs. force direc- quently, solute/macrosolute i will have a net z-direction
tions are utilized to achieve two different goals: (1) adsorp- velocity which will be different in the two regions:
tion/elution of proteins vis--vis adsorbent particles; (2)
expansion of the bed of adsorbent particles to allow cells/ viz;net j j1 vCi1 jU iz jnet;1 ; 7:1:112a
cell debris to pass through the particles undergoing incipi- viz;net j j2 vCi2 jU iz jnet;2 : 7:1:112b
ent/mild fluidization without being swept out of the
column. Consider the case where the electrophoretic velocities of
A few of the characteristics of the expanded-bed device solute i in the two zones are assumed to be identical in
and process are provided as follows: adsorbent particle magnitude (they have the same direction), i.e.
size, 50400 m; particle density ~1.21.3 g/cm3; ratio of
jU iz jnet;1 jU iz jnet;2 7:1:112c
maximum bed height to packed bed height, 23; flow
velocity range in expanded beds, 200500 cm/hr; cycle It is now possible to create a situation where the magni-
time, ~140 min. More details are provided on the website tude of viz;net j j1 and viz;net j j2 are identical but their direc-
(http://www.biotech.pharmacia.se). Recovery and purifi- tions are different:
cation of proteins from bacterial cultures, mammalian cell
cultures, yeasts, milk as well as blood have been studied jviz;net j j1 j jviz;net j j2 j; 7:1:112d
(Batt et al., 1995; Chang and Chase, 1996; Frej et al., 1997). viz;net j j1 viz;net j j2 0: 7:1:112e
The types of adsorbent resin beads used include cation
exchangers, anion exchangers, protein A for affinity Therefore at the interfacial region between the two regions
adsorption and chelating resins for immobilized metal j 1 and j 2 in the column, species i will be focused and
affinity chromatography. will concentrate.
On the other hand, other solute/macrosolute species/
proteins will move through this interfacial region since the
7.1.7 Counteracting chromatographic electrophoresis
magnitudes of this solutes viz;net jj1 and viz;net jj2 are not
and electrochromatography
identical. If, however, there is a third zone in the column,
We will study here the role of two different forces that are j 3, next to j 2 such that, for this solute,
active in a bulk flow field perpendicular to one of the forces
jviz;net j j2 j jviz;net j j3 j; 7:1:112f
but parallel to the other. In a fixed-bed/packed-bed
column, we have seen that a solute/macrosolute is con- viz;net j j2 viz;net j j3 0; 7:1:112g
vected down the column with a velocity ( vCi R0 i vz for
species i) given by expressions (7.1.99b) and (7.1.12a). This then this second solute species will accumulate in the
velocity is different from the liquid-phase velocity vz ( v0/ interfacial region between regions 2 and 3. This basic
, where v0 is the superficial velocity of the liquid based on principle is called counteracting chromatographic electro-
the empty column cross-sectional area), and is determined phoresis (CACE) and was first proposed by OFarrell (1985).
by the interactions between the species in the mobile- Additional treatments are available in McCoy (1986),
phase and the stationary-phase adsorbent as a result of Hunter (1988) and Ivory and Gobie (1990).
the chemical potential gradient based force acting perpen- We will now develop a quantitative illustration of the
dicular to the bulk liquid flow. Now, if an electrical field is results (7.1.112d,e) for any solute i and then provide an
imposed in the direction of the bulk liquid flow and the example from OFarrell (1985) in terms of protein separ-
solutes/macrosolutes are charged (e.g. proteins), we will ation. Later we will deal with electrochromatography in
have an additional z-directional imposed velocity of Uiz on general as applied to a chromatographic column having
the solute/macrosolute (Figure 7.1.29(a)) due to the elec- one stationary phase with an imposed electrical field in
trical field. the direction of the bulk liquid flow perpendicular to the
Assume now that the direction of this electrophoretic force direction between this mobile phase and the sta-
velocity is opposite to that of vz, and therefore that it tionary phase.
552 Bulk flow perpendicular to the direction of force

(a) (b)
Buffer in Buffer in

+ Electrode + Electrode
Column
section
filled Gel
with an |Uiz|net niz, net 1
excluding n Ciz
Focusing
z gel P-10
zone
Zone of solute solute I
focusing Gel
Column 2
section Focusing
filled zone
with an |Uiz|net niz, net solute 2
including n Ciz Gel
gel A-50 3
Electrode Electrode

Buffer out Buffer out

(c)
Buffer in Figure 7.1.29. (a) Solute focusing from a multicomponent batch
sample in a chromatographic column with counteracting elec-
trophoresis. (b) Focusing of two solutes in a three-gel column
+ Electrode
Excluding
from a batch sample in counteracting chromatographic electro-
gel 1 phoresis. (c) Continuous separation of two proteins in CACE.
Product 1 (After OFarrell (1985) and Ivory and Gobie (1990).)
Less excluding
gel 2
Feed in

Including
gel 3
Product 2
More including
gel 4
Electrode

Buffer out

Consider equation (7.1.2) for a charged solute species i vz U iz C i2


C i2 1 C i1
in a packed bed or a chromatographic column. If we have a
t t z
voltage applied in the z-direction, creating an electrophor-

C i2

etic velocity Uiz of charged species i, then equation (7.1.2) Di;eff;z : 7:1:113b
z z
may be reformulated as
Conventionally, vz and Uiz may be assumed to be inde-
2 3 2 3 pendent of the z-coordinate.10 Therefore
C i2 C i1 5 C i2 5
Sc Z 4 1 Sc 4vz U iz C i2 Di;eff;z
t t z C i2 1 C i1 C i2
z vz U iz
2 3 t t z
C i2 5 C
 
4
Sc vz U iz C i2 Di;eff;z : i2
z Di;eff;z : 7:1:113c
zz
z z
7:1:113a

Applying the limiting process of z ! 0 after dividing by 10


There may be variation of Uiz along the column due to
Scz, we get concentration effects.
7.1 Force ri in phase equilibrium: fixed-bed processes 553

If we assume isothermal equilibrium nondispersive oper- negatively charged at the pH employed; accumulation of
ation and adopt the methodology of equations (7.1.6) this protein in the intermediate region leads to a buildup
(7.1.8), we can rewrite this equation as of counterions, which increases the electrical conductivity
of this zone, and therefore decreases the voltage gradient
q 0 C i2 C i2 C i2
 
1 b i vz U iz 0: 7:1:113d E ( dV/dz) in the z-direction. This reduces the electro-
z z
phoretic velocity of the protein sufficiently so that the net
Using the methodology adopted to obtain the migration protein velocity becomes zero in this zone in between
rate of species i, vCi , via equations/relations (7.1.10), region 1 and 2. An advantage of this method is that
(7.1.11) and (7.1.12a), we can write, for any particular focusing occurs in the native electrolyte as opposed to
section of the column, the ampholytes needed in isolectric focusing (IEF; see
Section 4.2.2.1).
vz U iz It should be noted that all of the above treatment took
viz;net j j1 2 3
b q 0
i C i2 place in a batch solute introduction mode with a continuous
41
buffer flow. For example, the experimental arrangement
5

vz U iz used by OFarrell (1985) was as follows. A 0.7 cm diameter,
) viz;net j j1 2 3 2 3: 50 cm long glass column was packed with Biogel A-50 m
41 b q 0i C i2 5 41 b q 0
i C i2 5 beads upto a height of 25 cm from the bottom; the top of
the column was filled with BioGel P-10 beads, which
7:1:114a excludes all of the proteins under consideration. Two elec-
trode reservoirs (positively charged at the top and nega-
At the interface between column regions 1 and 2 (Figure tively charged at the bottom) were connected to the glass
7.1.29(a)), if the species i is to accumulate there, we should column through a large-bore tubing filled with polyacryla-
have mide gel. Another port connected to the column top
      allowed a flow from a peristaltic pump of the carrier solv-
ent (10 mM tris acetate, pH 7.4) to come in at 0.17 cm3/
     
    
 vz U iz    vz U iz  

0


 
0

: min. The voltage applied between the electrodes was
q q
     
b C i2 C i2
 1 i b i 600 volts. Ferritin was introduced in a batch sample and
 
1

   
  
 
column region1 was found to concentrate in a narrow band 2 mm wide
  
column region2

7:1:114b around the interface between the two sections of resins


(Figure 7.1.29(a)).
There are a number of assumptions implicit here since we By having successively different layers of gel matrices
are dealing with charged species; further, if we are dealing (beyond two), one can have different proteins focused in
with charged macrosolutes, there are additional assump- different intermediate regions from a batch sample. By
tions: we neglect the concentration polarization of proteins having three gel matrices, one can have two intermediate
on the surface of gel matrix in the presence of the electrical regions, each providing a focusing region for a separate
field; we have not considered the issue of intraparticle protein, thus separating a binary mixture (Figure 7.1.27(b)).
porosity; the nature of the partitioning equilibrium (7.1.6) By having four gel matrices, one can have three focusing
assumed has to be relevant; there is no electroosmotic regions for three different proteins (note, only for a batch
velocity. sample).
We will now briefly illustrate the example originally In the batch mode of sample introduction described
provided by OFarrell (1985) for CACE using beads above, both sample loading and product extraction are
employed in gel chromatography. Region 1, at the top of considered cumbersome. Ivory and Gobie (1990) have
the column, contains a gel that tends to exclude the demonstrated a continuous mode of CACE operation in a
protein (OFarrell employed the protein ferritin); therefore column, as illustrated in Figure 7.1.29(c). The buffer solu-
the value of vCiz is larger and very close to vz without the tion is introduced at the top. The feed solution mixture is
electrical field. The electrically induced velocity U iz in introduced into the middle of the column containing a
the opposite direction reduces this downward velocity in number of layers of different gel particles. Two concen-
region 1. But the net velocity in region 1 is downward. On trated product streams are withdrawn from an appropriate
the other hand, region 2 has a rather open gel with a intermediate position at the top half and bottom half of the
solute partition coefficient around 1. Therefore the down- column; this is done through a port, which, however, con-
ward value of vCiz in region 2 is much smaller and the tains no gel particles. For the exact configuration
electrically induced velocity in the opposite direction is employed, see Ivory and Gobie (1990). Figure 7.1.29(c)
larger. This leads to focusing of the protein into the inter- illustrates a column having five ports: one for feed sample
mediate region between the two regions having two dif- introduction, two for withdrawal of two products and two
ferent types of gel particles. The protein ferritin is for buffer introduction and withdrawal, all on a continuous
554 Bulk flow perpendicular to the direction of force

basis. There are four different gels stacked in the column driven velocity vz and a solute migration velocity Uiz due
to achieve continuous fractionation of two proteins. Multi- to the electrophoretic driving force, due to a constant
component separation is not possible in this configuration potential gradient E in the z-direction. Therefore the
on a continuous basis. effective velocity of an ionic species i is (see equation
This last point needs some elaboration. Consider in (6.3.10c))
Figure 7.1.29(c), any one of the two ports used to withdraw
hviz i hvz i m m
EOF E i E; 7:1:115
two different protein products continuously. If we have
three proteins present in the feed stream, then let the where m m
i is the ionic mobility of the charged species i, EOF
liquid coming to one of the ports be pure in one protein; is the electroosmotic mobility of electroosmotic flow (EOF)
it is clear that the liquid passing over the other port will (see (6.3.10c)) and hvz i is the cross-sectional averaged
have two other proteins: therefore the liquid stream with- value of the pressure driven capillary flow. We may now
drawn from this port cannot be pure in one protein. This is follow the standard procedure for conventional packed-
an inherent limitation of processes where the bulk flow is bed/chromatographic columns (see relations (7.1.99b)
parallel to the direction of the force. Here, even though we and (7.1.99h)) that the centerline of the peak of species i
have one force perpendicular to the bulk flow, in the moves down at a velocity
presence of the electrical force parallel to bulk flow, the
system in continuous operation is just like the counter- hviz i hviz i hviz i
viz;net     ;
current electrophoresis of Figure 6.3.4 and equations 1 q 0i C i2 V Ri =V M
1 il 1 b
(6.3.9a,b). However, in a batch mode, one can have multi-
component separation since each species will be focused 7:1:116
to its own zone and therefore can be withdrawn at a later hvz i m m
EOF E  i E
time. Multicomponent separation in electrochromatogra- viz;net  : 7:1:117
1
phy needs a different flow vs. force configuration, to be 1 il

discussed further in Section 8.2.
The selectivity ij between two solute species i may now be
defined in a number of ways. For example,
7.1.7.1 Electrochromatography
 
The CACE technique described above is a specialized off- 1
1 j1
shoot of the general technique of electrochromatography, viz;net hvz i m
EOF E m
i E
ij :
vjz;net m E m
j E

where in a column containing chromatographic beads as hvz i EOF 1
1 il
the buffer solution flows, an electrical field is simultan-
eously applied through two electrodes mounted at the 7:1:118
top and bottom of the column. The sample is generally
introduced in a batch fashion (an exception is shown in A major advantage of CEC over capillary electrophoresis
Figure 7.1.29(c)). Only one kind of chromatographic (CE) (Section 6.3.1.2) lies in its capacity to separate
medium is present in the column, unlike that in CACE; uncharged molecules. Whereas in CE uncharged mol-
the medium will depend on the species to be separated. It ecules will be convected at the bulk velocity, vEOF, of the
could be gel permeation beads for a mixture of proteins electroosmotic flow in an unselective manner, the incorp-
(Rudge and Ladisch, 1988), as studied first by Nerenberg oration of a chromatographic medium in CEC will allow
and Pogojeff (1969) for separation of blood plasma pro- the separation of uncharged compounds as well. For
teins; alternatively, for smaller molecules there is a vast charged molecules/macromolecules, the chromatographic
literature (Tsuda, 1995). One could also employ a capillary medium (for example, gel beads for proteins) provides
column, as in capillary electrophoresis (Section 6.3.1.2); additional selectivity to that achieved by the electrophor-
however, the capillary is packed with a suitable chromato- etic mobility differences (see (6.3.19)).
graphic medium. Alternatively, there may be a coating on A simplistic analysis, provided below, can allow the
the wall of an open capillary column. Electroosmotic flow quantification of which mechanism contributes to what
is possible with an appropriate silica-type tubular capillary extent to the overall selectivity in, say, CEC. To this end,
wall in the applied electrical field. When carried out in a we define a selectivity between species i and j via
capillary column, this combination of capillary chromatog-
t Ri t Rj
raphy with capillary electrophoresis is generally called ij jt R ; 7:1:119a
i t Ri
capillary electrochromatography (CEC).
We will now provide an elementary approach to cal- where the retention time for species i in the column of
culating selectivity or resolution in CEC for a batch sample. length L, t Ri , is obtained from
Suppose, in the capillary system under consideration,
there is an electroosmotic velocity vEOF;z , a pressure t Ri L=viz;net : 7:1:119b
7.2 Crossflow membrane separations, granular filtration 555

This yields when the permeate phase collected from different sections
h i h i of the membrane cannot be removed by simple crossflow
1 1 il 1 1 j1 and starts having a bulk motion parallel to the membrane

hvz i m m
EOF E i E hvz i m m
EOF E j E
on the permeate side, the nature of separation achieved
ij jt R h i ; may be affected; this configuration is treated in Sections
i 1
1 i1
8.1.9 and 8.2.4.2. Both the mode of feed introduction and
hvz i m m
EOF E i E the mode of product withdrawal vis--vis the force direc-
7:1:119c tion are important in separation devices.

leading to
 
1 
m m m
j i E hvz i EOF E i1 j1 1 E i1 m m

j j1 i
ij jt R h ih i : 7:1:120
i
1 1 i1 hvz i m m
EOF E j E

The three terms in the numerator identify three different


At the end of Section 7.2.1, the process of rotary
contributions to ij jt R : the first term, m m
j i E, is the
i vacuum filtration is described. Here the membrane con-
contribution by the differences in the electrophoretic
taining the particulate cake/deposit (see Figure 6.3.25(a))
mobility of the two species; the second term relates to the
is moved perpendicular to the pressure induced crossflow
difference in the partitioning characteristics and the cap-
causing filtration through the filter. If we imagine the
acity of the two phases for the two species i and j; the third
coordinate system to be fixed to the membrane, then the
term refers to the interaction between chromatographic
system configuration is similar to the crossflow microfiltra-
partitioning capacity and electrophoretic mobility (Deng
tion briefly considered earlier (Figure 6.3.25(b)) where the
et al., 1998).
liquid slurry/suspension moves parallel to the membrane
Columns in CEC have discontinuities of the electric
but perpendicular to the direction of the force.
field strength and flow velocity at the interface of the
In Section 7.2.2, we consider fixed-bed granular filtra-
packed and open sections of the column. Rathore and
tion for clarifying dilute suspensions of particles from a
Horvath (1998) have provided a methodology for treating
liquid. We will identify the similarities between this process
such a configuration.
and the fixed-bed adsorption/desorption processes studied
in Section 7.1.1: both processes employ a low velocity bulk
liquid flow through a bed of fixed particles and solute
7.2 Crossflow membrane separations, crossflow
molecules or particles are removed from the liquid by
filtration and granular filtration
forces acting perpendicular to the fixed particles. Both
This section will describe a number of membrane separ- processes achieve high levels of purification. From a
ation processes as well as filtration processes where the black-box type of perspective, granular filtration may also
force(s) causing separation is (are) acting perpendicular to look very similar to the dead-end filtration studied in
the direction of flow of the feed phase. In Section 7.2.1, we Section 6.3.3.1, but there are significant differences.
will treat the following membrane processes which rely on
crossflow: crossflow gas permeation, crossflow reverse
7.2.1 Crossflow membrane separations, crossflow
osmosis (RO), crossflow ultrafiltration and crossflow
filtration
microfiltration. We will encounter how the configuration
of bulk feed flow parallel to the membrane surface but The membrane/filtration processes have been considered
perpendicular to the force direction between the feed fluid in the following order: crossflow gas permeation, crossflow
and the membrane can lead to steady state operation (see reverse osmosis, crossflow ultrafiltration, crossflow micro-
Figures 7.0.1(e)(h); a relatively slow process of membrane filtration and rotary vacuum filtration.
fouling can introduce unsteadiness in an otherwise steady
process. We observed earlier, in Section 6.3.3, that, for all
7.2.1.1 Crossflow gas permeation
such membrane/filtration processes, the dead-end mode
of operation with bulk feed flow parallel to the direction of In Sections 6.3.3.5 and 6.4.2.2, we described gas separation
the force through the membrane led to unsteady state by permeation through a membrane under the conditions
operation. We also noted (e.g. in Section 6.3.3.5) that it is of bulk flow parallel to the force and having a well-mixed
often impractical to employ a large membrane surface area flow on both sides of the membrane. We learned that the
in the dead-end configuration. Membrane separation and configuration of bulk gas flow parallel to the force direction
filtration processes having crossflow and studied in this was not very useful: it could not be used in practice with
chapter do not suffer from these limitations. However, large membrane surface area, and it led to unsteady state
556 Bulk flow perpendicular to the direction of force

operation. Since the permeation rates of gases through At this time, it is useful to consider such an asymmetric
membranes are quite slow, large membrane surface areas or composite structure (a more complex structure is shown
are needed in industrial practice; also, steady state oper- in Figure 6.3.35(b)) in detail and explore what membrane
ation is highly desirable. The disadvantage of having a device configuration can allow the achievement of such
well-mixed flow is simply stated: it leads to lower permeate operating assumptions. The complex structure of, for
enrichment and lower retentate purification. If we consider example, an asymmetric membrane having a nonporous
the membrane as a concentration amplifier of the more skin may be modeled as a composite having a thin homo-
permeable species, then we achieve the highest permeate geneous skin on a porous backing. Conventionally, the
concentration at the permeator location where the feed has permeate emerging from the skin into the pores beneath
the highest concentration of the more permeable species. will have a composition indicated by x 0 ip and x 0 jp and a
In the well-mixed mode of operation, this feature is lost, total pressure of P 0 p > P p :
since both the permeate and the feed side are well-mixed.
N im Qim =m P f x if P 0p x 0ip
Further, in most practical gas permeation devices used at : 7:2:2b
N jm Qjm =m P f x jf P 0p x 0jp
larger scales, the feed is unlikely to be well-mixed; the
same is true of the permeate side. If there is essentially no pressure drop in the transport of
In this section, we therefore consider the crossflow these permeated gases through the porous substrate, then
configuration introduced in Figure 7.0.1(e). The gas flows P 0 p P p , which is the pressure in the bulk gas on the
on the feed side of the membrane under the following permeate side. Further, if this gas mixture does not mix
conditions: with any other gas stream emerging from other membrane
(1) there is no concentration polarization in the feed gas locations, then only x 0 ip x ip and x 0 jp x jp , leading to the
phase normal to the membrane surface; crossflow relation (7.2.2a).
(2) there is no longitudinal diffusion/dispersion along the These conditions may be achieved in a hollow fiber
feed flow direction11 (no-mixing case); permeator where the gas is fed through the hollow fiber
(3) the feed gas and the permeate gas streams undergo bore; the permeate is radially withdrawn from the shell
negligible flow pressure drops. side and there is a low pressure drop in the shell side in
the radial direction (Figure 7.2.1(c)). Two other commonly
The permeated gas is withdrawn in a direction perpen- used configurations, a shell-side fed hollow fiber module
dicular to the direction of flow of the feed gas and parallel and a spiral-wound device shown in Figures 7.2.1(d) and
to the direction of the force causing separation, the partial (e), will bring permeates emerging from different feed
pressure gradient component of the chemical potential concentrations together at every pore mouth. The hollow
gradient of the permeating species. Figure 7.2.1(a) illus- fibers in these membrane processes may have internal
trates this separation configuration schematically. diameters varying between 50 and 1000 m and wall thick-
An essential feature of the existing analyses of this nesses varying between 5 and 300 m. The analysis of such
crossflow configuration carried out by Weller and Steiner configurations is best considered in Chapter 8 since the
(1950a, b) is that the permeate gas stream emerging from compositions and pressure drops in both streams flowing
the membrane (at any axial location) does not mix with any perpendicular to the force direction are important. How-
other permeates emerging from the membrane at other ever, the crossflow analysis is still useful because the ana-
axial locations. Therefore the membrane permeation flux lytical results allow a quick estimation of the separation
Nim at any permeator location is given by performance.
  We will now develop the governing equations for
Qim
N im P f x if P p x ip ; 7:2:1 such a crossflow permeator. For the permeator configur-
m
ation of Figure 7.2.1(a) and the assumptions of (1) no
where the permeate side mole fraction xip of species i is concentration polarization and (2) no longitudinal diffu-
determined by the permeation fluxes of different species. sion/dispersion in the flow direction (z-coordinate), a
For a binary system of two species i and j, control volume analysis (in the manner of Figure 6.2.1
but without any x-coordinate dependence) leads to
N im Qim =m P f x if P f x ip x ip
: 7:2:2a (under conditions of no chemical reaction (Ri 0) and
N jm Qjm =m P f x jf P f x jp x jp
steady state ((Ci/t) 0))
An asymmetric or a composite membrane having a porous
substrate through which the permeate emerges satisfies 0 xy N iz jz N iz jzz xz N im : 7:2:3
this postulate (Figure 7.2.1(b)).
Here, the control volume dimensions x and y are along
the membrane flow channel of width W and height h,
11
Chen et al. (1986) have studied the role of axial diffusion in gas respectively. Further, the expression for Niz in view of
permeator analysis. assumption (2) is simply
7.2 Crossflow membrane separations, granular filtration 557

(a) z (b)
L Exit Feed gas flow
Nonporous
xA2 Wt 2L xif Pf
Feed skin
Wt2 xA1L
xAf Concentrate

Porous
Membrane substrate

Permeate xip,Pp x ip,P p

Permeate xA1L, Wt1L

(c)
Permeate

Reject

Hollow fiber
membrane

(d) Feed

Permeate Permeate

Reject
(e)
Module housing

Feed flow Residue flow


Collection pipe Permeate flow
Feed flow Residue flow
Feed flow
Spacer
Membrane

Permeate flow Spacer


after passing through
membrane

Figure 7.2.1. (a) Crossflow membrane module for gas permeation; (b) model for crossflow gas permeation in an asymmetric or composite
membrane; (c) tube-side feed crossflow hollow fiber module; (d) shell-side feed hollow fiber module; (e) spiral-wound module.

N iz C i2 v W t2 x i2 =W h; 7:2:4 of the permeator on the feed side. Rewriting equation


(7.2.3) as
where the flow stream on the feed side is j 2. For the
permeate j 1, p. At z 0, Niz (Wtf xif)/Wh, correspond- 0 W hN iz jz N iz jzz W z N im ;
ing to the feed location. We are interested in finding out
what would be the value of Wt2 and xi2 at z L, the end and taking the limit of z ! 0, we get
558 Bulk flow perpendicular to the direction of force

dN iz W h dW t2 x i2 x A1 1 x A2 x A1 x A2
W N im ) N im AB ; AB 1 ;
dz W dz 1 x A1 x A2 1 x A1 x A2
dW t2 x i2 Q AB x A2 1 x A2
) N im im P f x i2 P p x i1 ; x A1 ; 1 x A1 :
dAm m 1 x A2 AB 1 1 x A2 AB 1

7:2:5 7:2:11

where Integrating this relation between the limits of Wt2 and xA2
along the permeator length from z 0, Wt2 Wtf, xA2 xAf to
dAm W dz and Atm W L; 7:2:6
z L, Wt2 Wt2L, xA2 xA2L, Wt1L Wtp, xA1L xAp, we get
L being the permeator length in the z-direction for a total   x
A2L x
A2L
W t2L 1 dx A2 AB dx A2
membrane area Atm . This is the governing equation for the n
W tf AB 1 x A2 AB 1 1 x A2
feed side. The governing equation for the permeate side is x Af x Af
primarily dictated by the crossflow relation (7.2.2a). An   1   AB 1
x A2L AB 1 1 x Af AB
alternative form of this relation for a multicomponent n n ;
x Af 1 x A2L
system is
W t2L W tf W t1L
N im x ip 1
n n x ip : 7:2:7 W tf W tf
X X
N jm x jp   1   AB
j1 j1
x A2L AB 1 1 x Af AB 1
: 7:2:12
x Af 1 x A2L
However,
n
X n
X n
X Therefore, knowing AB and xAf, we have a relation
dW t2 x i2 dW t2 N im dAm N jm dAm : between the stage cut, , and the high-pressure outlet
i1 i1 j1
composition (concentrate/reject) xA2L. If either or xA2L
Therefore, is specified, the other quantity is obtained from the above
equation. Note that if the integration is not carried out
N im dAm dW t2 x i2
n x ip x i1 7:2:8 to the permeator end, this relates xA2 at any location to
X dW t2
N jm dAm the corresponding Wt2. It is also important to determine
j1 the value of xAp, the mole fraction of species A in the
permeate stream:
is the required relation for the permeate side compos-
x
ition. In addition, we have to satisfy the following two A2L

relations: x A1 dW t2
x Af
n
X n
X x Ap : 7:2:13
x
x i2 1; x i1 1: 7:2:9 A2L

i1 i1 dW t2
x Af
One can now follow the solution procedure of Pan and
Habgood (1978a) instead of the WellerSteiner (1950a, b) The integral in the numerator may be rearranged by using
approach, which is valid for a binary system only. expression (7.2.11) for xA1 in terms of xA2, expression
We will now develop an analytical solution for a (7.2.10) for dWt2 in terms of xA2 and expression (7.2.12)
binary feed gas mixture of species A and B by assuming for Wt2 in terms of xA2:
that the local separation factor AB is independent of
x
A2L 
the membrane separator length/location. This approach AB x A2 1x A2 AB 1
 
W t2 dx A2
(Stern and Walawender, 1969) is an adaptation of the 1x A2 AB 1 x A2 AB 11x A2
x Af
Naylor and Backer (1955) approach employed for gaseous x Ap
W tf W t2L
diffusion with porous barriers. Rewrite equation (7.2.8) for
i A as x
A2L   1 1   AB
AB 1 x A2 AB 1x Af AB 1
W tf dx A2
x A2 dW t2 W t2 dx A2 x A1 dW t2 ; AB 11x A2 x Af 1x A2
x Af

dW t2 dx A2 1 x A2 AB 1
 
dx A2 ; W tf
W t2 x A1 x A2 x A2 AB 1 1 x A2
AB x
A2L 1
7:2:10 AB 1 1x Af AB 1 x A2 AB 1
dx A2
AB 1 x Af AB1 1 1x A2
2AB 1
AB 1
where we have used definitions and relations such as x Af
7.2 Crossflow membrane separations, granular filtration 559

 q
12 1x 2 2 42 2 1x 2
2  3
12 1x 2
  AB1
1 1 AB
AB 1 x Af AB AB
41x A2L x A2L
AB 1
Y 2 ;
6 7
x A2L AB1 1 1x Af
5:
22
7:2:20
7:2:14
Y 2
This expression relates the permeate mole fraction xAp to S1Y 2 dY 2 ; 7:2:21
the residue composition xA2L as a function of xAf, and AB. Y 2;f

Since xAp, xA2L, xAf and are related via SQ1 =dP=Lf 1s; 7:2:22
x Af x Ap 1 x A2L ; 7:2:15
where S is the nondimensional membrane area, (Q1/d) is
we can rewrite expression (7.2.14) for xAp as the permeance ( permeability coefficient/membrane
thickness) of species 1, P is the feed side pressure, Lf is
2 ) AB1
the molar feed gas flow rate, s is the actual membrane area
(
1 1 AB 1x A2L x Ap AB

x Ap 1x A2L AB 1
( WL in our notation). For the special case of 2 >>1 (i.e.
4
x A2L AB1 1 1 1x A2L x Ap

2 1, b 1). These results are reduced to


#
AB
x A2L AB 1
: 7:2:16
1 Y 2 =Y 2;f a ; 7:2:23
This is an implicit expression for xAp in terms of xA2L, and a 
a Y2 a
 
AB. The actual separation factor, AB , is to be determined S 1 Y 2;f 1 Y 2 : 7:2:24
a1 Y 2;f a1
from the ideal separation factor AB , the pressure ratio
P p =P f and the exit composition for the concentrate
xA2L (since AB is assumed to be independent of the local Example 7.2.1 Consider the production of nitrogen-enriched
air from atmospheric air using an asymmetric cellulose acetate
variation of xA2 from xAf to xA2L at the exit) via relation
membrane having an ideal selectivity of QO2 m =QN2 m 6:0. If
(6.3.202b), where xA2 can be used instead of xAf.
the permeate side is maintained at a considerable vacuum, and
The total membrane area Atm ( WL) required for this if you can assume crossflow and no permeate side pressure
separation can be obtained by combining equations (7.2.5) drop in the module (spiral-wound or hollow fiber), then deter-
and (7.2.8): mine the stage cut and the permeate composition for the
following values of oxygen mole fraction in the concentrate:
dW t2 x A2 x A1 dW t2
dAm   ; xA2L 0.1, 0.05, 0.02, 0.01, where species A is oxygen. Plot the
Qim Qim
m P f x A2 P p x A1 m P f x A2 P p x A1 oxygen mole fraction in the permeate and concentrate as a
xAf function of the stage cut; include values for 0 and 1.
x A1 dW t2
Atm  : 7:2:17 Solution There is no specification of the permeate side
Qim
x A2L
m P f x A2 P p x A1 pressure, except we have considerable vacuum. Since the
pressure ratio (permeate pressure/feed pressure) is
Now employ equation (7.2.10) for dWt2, equation (7.2.12) very low, we may assume that AB AB ; therefore,
for Wt2 and equation (7.2.11) expressing xA1 in terms of xA2 AB O2 N2 6:0. Since AB is constant throughout the
and AB in the above integral for Am, and numerically module (due to negligible pressure drop along the permeate
evaluate it. side and very low ), we can use the result (7.2.12) based on
Pan and Habgood (1978a) have provided a solution to the NaylorBacker method to calculate from
this crossflow permeator problem without assuming a 1

AB

1 x A2L =x Af AB 1 1 x Af =1 x A2L AB 1
 
constant value of the selectivity AB for a binary mixture
characteristic of the NaylorBacker approach described given xAf 0.21, AB 6 and xA2L (different values; see
above. We use their notation here: species 1, base com- Table 7.2.1).
ponent (less permeable); xi, mole fraction of species i in
feed side; yi, mole fraction of species i in permeate side; i Table 7.2.1.
for i 6 1 is the selectivity of species i with respect to species
1, where 1 1; i 1 1=i ;Yi i yi; pressure 1

AB

xA2L x A2L =x Af AB 1 1 x Af =1x A2L AB 1


 
ratio ( permeate pressure/feed pressure); xAp

a 2 2 1=2 2 1 ; b 2 2 2 =2 2 1 : 0.1 (0.1/0.21)0.2 (0.79/0.9)1.2 0.73 0.27 0.507


7:2:18 0.05 (0.05/0.21)0.2 (0.79/0.95)1.2 0.600 0.40 0.45
0.02 (0.02/0.21)0.2 (0.79/0.98)1.2 0.481 0.519 0.386
Their results are as follows: 0.01 (0.01/0/21)0.2 (0.79/0.99)1.2 0.3297 0.6703 0.308
a  b  0.21 1 0 0.61a
Y2 2 Y 2 1Y2
 
1 ; 7:2:19 xAp is determined from (7.2.15): xAf xAp (1 ) xA2L.
Y 2;f 2 Y 2;f 1 Y 2;f a
In this case, xAp has to be determined from the relations (6.3.199) for
where Y 2;f is the value of Y2 at feed location; 0 and (6.3.201) for the nonzero .
560 Bulk flow perpendicular to the direction of force

(a) 1 1

0.8 0.8

xAp 0.6 0.6 xN2L = xBL


xO2p
0.4 0.4

0.2 0.2

0 0
0 0.2 0.4 0.6 0.8 1
q

(b)
Permeator 1 Permeator 2 Concentrate
Fresh retentate
feed reject

Recycle
Compressor or
vacuum pump Permeate
highly
enriched

(c)
1.0

0.8

Wt2L xN2L 0.6 a O2 N 2


Wtf xN f
2
0.4

0.2

0
0.05 0.04 0.03 0.02 0.01 0
xO2L

Figure 7.2.2. (a) Oxygen mole fraction in permeate vs. stage cut for a membrane having O2 N2 6:0. (b) Two permeators in series
operation with recycle of permeate from the second permeator to the feed. (c) Fractional nitrogen recovery vs. residual oxygen mole
fraction in NEA for different membrane selectivities. (part(c) after Baker (2004).)

Since 0 and AB AB here, For 1, xAp xAf 0.21, since all of the feed gas appears
as the permeate. If a value of permeate side pressure was
AB x Af 6  0:21 1:26 provided, then would be known; AB may then be deter-
x Ap 0:61; mined from (6.3.202b) knowing AB , and xAf.
1 x Af AB 1 1 0:21  5 2:05
What is clear from this example is that, as the value of
the maximum value achievable anywhere in the the stage cut, , increases, the permeate mole fraction of O2
permeator. The plot of xAp vs. the stage cut, , is illustrated in the oxygen-enriched air (OEA) decreases and the mole
in Figure 7.2.2(a). fraction of N2 in the retentate (the nitrogen-enriched air
7.2 Crossflow membrane separations, granular filtration 561

(NEA)) increases. However, the loss of nitrogen in the higher oxygen selectivity will lead to a lower loss of N2 in
permeate is also increased. If the goal is to obtain oxygen- the permeate and therefore a higher fractional N2 recovery
enriched air (OEA) as well as nitrogen-enriched air (NEA), in the NEA. For a more detailed introduction to different
the behavior of xAp vs. shown in Figure 7.2.2(a) provides membranes used in O2/N2 separation, see Zolandz and
guidance. One can take the permeate up to a stage cut of, Fleming (2001) and Baker (2004).
say, 0.10.2; the corresponding permeate oxygen mole frac- Other gas separation systems of interest are as follows.
tion will be around 0.55. On the other hand, the concen-
(1) Separation of H2 from a variety of gas streams: H2 from
trate mole fraction of nitrogen for a stage cut of 0.6703 will
N2 in purge streams in ammonia plants; recovery of H2
be 0.99, a reasonable quality nitrogen-enriched air. These
from various streams in petroleum refineries and
two product streams are shown in Figure 7.2.2(b). How-
petrochemical plants; H2/CO ratio adjustment in syn-
ever, in order to show what to do with the rest of the
thesis gas for oxo-alcohol plants. The membrane
permeate, Figure 7.2.2(b) shows that the gas permeator
materials used are polysulfone, cellulose acetate, poly-
has been split into two permeators: the first permeator
imides and polyaramides. Selectivities of H2 over N2,
produces OEA as the permeate; the concentrate from this
CH4, CO in these applications vary between 30 and
permeator goes to a second permeator having a high stage
200.
cut and producing the NEA as the concentrate. The com-
(2) Separation of CO2 from hydrocarbons: CO2 separated
position of the permeate from this second permeator is of
from light hydrocarbons in natural gas streams at high
interest.
pressures; CO2 separated from biogas (containing CO2
Let us make a rough calculation concerning the com-
and methane); CO2 separated from breathing gas
position of this second permeate, identified as x Apj2 (the
mixture.
first one being x Apj1 ) for xA2L leaving the permeators being
(3) Dehydration of air.
0.01 (99% N2). For 1 mole of feed air (21% O2) entering the
(4) Recovery of C3 condensable hydrocarbons from nat-
system per unit time, and W t1j1 and W t1j2 being the per-
ural gas using rubbery polymeric membranes.
meate flow rates from the two permeators,
(5) Recovery of helium from natural gas.
1  0:21 W t1 j1  x Ap j1 W t1 j2  x Ap j2 W t2 x A2L : A quantity of considerable practical utility is the membrane
area required to treat a gas stream of given flow rate to
Let the overall stage cut 0.67 ! Wt2 0.33; W t1j1 W t1j2
achieve a certain concentrate mole fraction or permeate
0.67. Let W t1j1 0.1, x Apj1 0.55, xA2L 0.01. Then
mole fraction under given conditions; these conditions
0:21 0:1  0:55 0:57x Apj2 0:33  0:01 ! x Apj2 0:266; involve specification of the value of (Qim/m) for one of
the species, namely species i, AB for a binary mixture of
a somewhat O2-enriched permeate. This stream, if recycled gases A and B, feed pressure and permeate pressure, alter-
to the fresh feed air, will increase the O2 concentration of the natively, one of the stream pressures and a pressure ratio.
net feed entering the permeator 1. Therefore the oxygen In general, there is a pressure drop in the feed side and the
concentration in the permeate from permeator 1 will be even permeate side. However, the analysis provided so far in
higher (as discussed in Section 2.2.2). On the other hand, if this section is based on zero pressure drops on either side.
we operate permeator 2 as a new permeator with a N2- Quite often, such an assumption does provide reasonable
enriched feed stream coming as the reject from permeator estimates unless the pressure drop is such that the value
1, then we may have a different situation if we operate it at a changes considerably along the permeator length.
high . The oxygen concentration in this feed may be around
0.15. If we operate the second permeator with a very high Example 7.2.2 We wish to estimate the membrane
stage cut (say 0.67), it is entirely likely that x Apj2 < 0.21. area required to produce nitrogen-enriched air having
x N2 2L 0:95 using a silicone rubber-coated hollow fiber
Such a stream, if recycled to the fresh feed air, will increase
membrane, with QN m =m at 25 C 4:6  10 10 gmol/s-
N2 concentration in the feed gas. This will lead to a better cm2-cm Hg and  2 2:1. The feed air is introduced at
O 2 N2
NEA from permeator 2, but a poorer OEA from permeator 1. 10 bar; the permeate is at 1 atm. In the microporous hollow
If the production of NEA is the goal, then the variation fiber membrane with a silicone coating on the fiber internal
of the production rate as a function of nitrogen concen- diameter, the feed flows through the fiber bore. The perme-
tration is also of interest. The higher the production rate of ate is withdrawn in crossflow on the shell side all along the
this NEA stream, the higher will be the fractional recovery module length (Figure 7.2.1(c)). Neglect any pressure drop

of N2 from the feed air in this product stream. However, as along the module length. The feed air flow rate at 25 C is
the purity of N increases in this product, the fractional N 900 kgmol/hr.
2 2
recovery W t2L x N2 2L =W tf x N2 f , will decrease. Figure 7.2.2 Solution Since there is no pressure drop in either stream
(c) illustrates this behavior qualitatively for membranes along the module length, and we have pure crossflow,
having different O2N2 selectivities (varying between 2 we can use the Pan and Habgood (1978a) analysis for non-
and 8) (after Baker (2004)). Obviously, membranes having constant AB , i.e. AB can change with composition.
562 Bulk flow perpendicular to the direction of force

Table 7.2.2. The configuration studied here is crossflow (see Figure


7.0.1(f)): feed solution (e.g. brine in desalination) flows along
x 2 x O2 2 Y2 the membrane module length parallel to the membrane
length direction while the permeate flows perpendicular
0.21 0.178 0
0.18 0.156 0.219
to the feed solution flow direction, i.e. in the direction of
0.14 0.124 0.47 the driving force. In many devices, the permeated liquid
0.10 0.092 0.66 collected along the membrane length flows in a particular
0.05 0.0476 0.858 direction. This second bulk flow (of the permeate) may have
a significant effect on separation; this case of two bulk flows,
The equations of relevance are (7.2.18) for O2 , O2 , a and b, where both are perpendicular to the direction of the force
(7.2.19) for , (7.2.20) for Y O2 ;f and Y O2 , (7.2.22) for the across the membrane, is studied in general in Chapter 8.
definition of nondimensional membrane area S and (7.2.21) Here we study those cases where the second bulk flow is of
for the calculation of this S to obtain the actual membrane no consequence in so far as separation is concerned. Thus
area s. In this approach crossflow as in Figure 7.0.1(f) is sufficient for analysis.
N2 1; O2 2:1; The physical configurations studied are: (1) spiral-
  wound module; (2) tubular module. Let us consider first
1
O2 1 0:524; 0:1; a highly simplified, but reasonably useful, model for a
O2
spiral-wound reverse osmosis module. The basic structure
a O2 O2 1= O2 O2 1

of a single-leaf spiral-wound membrane module is shown


0:1  2:1  0:524 1 in Figure 7.2.1(e). An actual module consists of a number
1:12; of leaves. A single-leaf module as shown consists of a
2:1  0:524  0:9
rectangular membrane packet (sandwich) spirally wound
b O2 O2 O2 = O2 O2 1

around a central collection pipe. Each membrane packet


0:1  2:1  0:524 2:1 (sandwich) is separated from the membrane in the next
2:01:
2:1  0:524  0:9 and nearest membrane packet by a spacer screen. There
To determine S from equation (7.2.21), we need to have are two membranes in each membrane packet, with a
the value of for a few values of Y2 and integrate. Determine product water side backing material in between (Figure
Y2,f 0.178 (from (7.2.20)) for x2 x2f x O2 f 0.21. 7.2.3(a)). Three ends of this packet are glued and sealed,
Now determine Y2 for different values of x 2 x O2 2 from and the fourth end empties into the collection pipe
(7.2.20) and then determine the value of the stage cut from
through a glued joint. As brine (or any other aqueous feed)
(7.2.19). The results of these calculations are summarized
flows along the module length over the spacers in between
in Table 7.2.2. Y 2
The value of the integral dY 2 in (7.2.21) is 0.0698. the membrane packets, permeation occurs and permeate
Therefore Y 2;f flows along the spirally wound permeate channel in the
membrane packet through the product water side backing
S 1 Y 2 0:0698 1 0:04760:858 0:0698 0:887:
toward the central collection pipe (see Figure 7.2.1 (e)). By
So, having a number of leaves in parallel, the length of the

Q N2 m P
  membrane permeate side is reduced; correspondingly, the
S 1 s permeate pressure drop is reduced.
d Lf
cm Hg If we open a spirally wound membrane channel and
4:6  10 10 gmol 10 atm  76 atm  3600 hrs
 0:9  s; lay it flat, then, as shown in Figure 7.2.3(b), brine flows at a
s-cm2 -cm Hg 900 kgmol 1000gmol
hr  kgmol high pressure in the mean flow direction (the residue flow
0:887  900  1000  1010 1 m2 direction in Figure 7.2.1(e)), designated here as the
s 3 cm2 4 z-coordinate; the membrane length in this direction is L.
4:6  10  76  3:6  10  0:9 10 cm2
The permeate flows inside the membrane packet in the
7:05  104 m2 : x-direction, the so-called permeate channel toward the
central collection tube; the width of the membrane packet
7.2.1.2 Reverse osmosis is W (in the x-direction).
Our basic assumptions in the following analysis are as
As we have seen in Section 6.3.3.3, the dead-end mode of
follows.
operation of reverse osmosis creates major problems:
steady state is not possible; it is difficult to generate a high (1) The pressure difference P changes very little along
value of the solute mass-transfer coefficient k i ; further, the z-direction from the value Pf at the feed entrance
as in Section 6.3.3.5 for gas permeation, it is often easier location (z 0).
to pack a lot of membrane surface area when the bulk (2) The permeate channel pressure drop along the
liquid flow is not parallel to the direction of the force. x-direction is negligible.
7.2 Crossflow membrane separations, granular filtration 563

(a) (b)
Membrane Membrane
Backing material

bf Permeate tube Feed flow


direction
z
Permeate
flow direction
x

y
bf Feed channel

Mesh Permeate channel


spacer
Glued end of
Glue line permeate channel
Membrane

(c) (d)
0.9
0.8 5
0.0
Boundary layer 0.7 =

0
0.2

=
0.6
Mesh wire =
re 0.5
0.4
0.3
.3
0.2 =0
Membrane 0.1
0
0.2 0.4 0.6 0.8 1.0 1.2
L+

Figure 7.2.3. (a) Details of a spiral-wound module. (After Ohya and Taniguchi (1975).) (b) Channels and flow configurations in an
unwrapped spiral-wound module. (c) How mesh wires in the brine side spacer screen disturb the brine flow and create new boundary
layers (schematically). (d) Fractional water recovery re as a function of normalized channel length L for various parametric values of ,
indicating the feed osmotic pressure level with respect to Pf.

(3) The membrane has a very high solute rejection (salt derived mass-transfer coefficient k i reflective of the flow
rejection 99%, i.e. Rsalt 0.99 ! 1.0). conditions (Schock and Miquel, 1987). One could repre-
(4) The solute mass-transfer coefficient k i in the mem- sent the flow in a simplistic way as a chopped laminar
brane feed channel has a high value and can be flow (Solan et al., 1971) with a boundary layer beginning
considered to be essentially constant along the at each cross wire in the spacer (illustrated schematically
z-coordinate. in Figure 7.2.3(c)). However, the mass-transfer coefficient
k i in the very thin boundary layer is high even if the
The species conservation equation for solute i (here, salt)
Reynolds number is quite low (essentially laminar flow
in the feed flow channnel may be written using the species
at this low Reynolds number, even though there is a lot of
conservation equation (6.2.5b) for steady state, no external
mixing). We will represent this velocity field as two-
force based species velocities (Ui 0) and no chemical
dimensional, vy (normal to the membrane) and vz (along
reactions as
the membrane length, z-direction). Equation (7.2.25) may
r C is v r J i r Dis r C is : 7:2:25 be represented in this context in the (y, z)-coordinate
system as
The flow conditions in the thin feed brine channel with
feed brine spacers in between is complex (see Schwinge    
et al. (2002, 2003) for the unsteady flow, vortex shedding C is vy C is vz C is C is
Dis Dis :
and mass-transfer enhancement in spacer-filled channels). y z y y z z
The complexity is captured by using an empirically 7:2:26
564 Bulk flow perpendicular to the direction of force

Since the axial concentration gradient of the solute, Cis,, is very If we use the solutiondiffusion model to describe the solv-
small,12 the axial diffusion term may be neglected, leading to ent flux, then, from equations (3.4.60cf), Nsy Nwy J wy :
 
C is vy C is vz C is jN wy j jJ wy j AP AP bC 0il bC ip ;
Dis ; 7:2:27
y z y y 7:2:35
 
C is vz C is where (see relation (3.4.64a)) C 0il is the wall concentration
C is vy Dis : 7:2:28
z y y of the solute and Cip is the solute concentration in the
Integrating this equation along the membrane feed chan- permeate. By assumption (3), we can neglect Cip here.
nel height bf (as if it is a flow between two flat plates with a Further, using the film model analysis and assumption
gap bf in between), we get (4) with jN wy j V w =k il << 1, we have already obtained
the following result (see (3.4.66a)):
bf  
d bf C is
C is vz dy d C is vy Dis : 7:2:29 AP b C isb
dz 0 0 y jJ wy j  jN wy j: 7:2:36
1 A b Ckisbil V w
For dilute solutions, vz
vz ; therefore, for one membrane
channel of width W, height bf and a brine flow rate of Q, Remember this Cis is really Cisb(z). We will next integrate
corresponding to an averaged axial brine velocity of vz,avg, equation (7.2.34) along the z-coordinate, the membrane
we get length direction:
" 
d 1 dC is 2 L dz

2L
C isbL
dC isb
vz;avg C isb C is vy Dis C isb jf
dz bf dy bf 2:
bf 0 vz;avg jf bf vz;avg jf C isb jf jN wy j V w C isb
  #
dC is 7:2:37
C is vy Dis ; 7:2:30
dy 0
The development of this equation did not involve any
where Cisb is the bulk solution concentration. Note that if assumption, other than an extremely low solute flux
we write the magnitude of the solute flux through either of through the membrane (assumption (3)). Therefore, any
the membranes on two sides of the feed brine channel as general expression for jN wy j jJ wy j, the membrane solv-
jN iy j, then the above expression is reduced to ent flux (which is the water flux, for example, in desalin-
ation), as a function of Cisb(z) may be used here with the
d 1 2 limitation of assumption (3), namely that Cip 0. An
vz;avg C isb

jN iy j jN iy j jN iy j:
dz bf bf option is to use the simplified water flux expression
7:2:31 (7.2.36) after defining a nondimensional membrane length
L as follows:
An exactly similar analysis may be carried out for the
solvent (water), yielding
h i
1 AbC isbkizV w dC isb
C isbL
2vywf L
L vywf C isb jf ;
d 2 vz;avgjf bf Af P f bC isb zC isb z2 V w
vz;avg C wsb jN wy j; 7:2:32 C isb jf
dz bf
7:2:38
where i w. By assumption (3), the solute flux jN iy j through
where
the membrane may be neglected for the time being:
vywf Af P f V w 7:2:39
d
vz;avg C isb 0 ) vz;avg jf C isb jf vz;avg C isb 7:2:33
dz is the volume flux or permeation velocity through the
at any axial location (any z). Now, if we are dealing with a membrane at the feed entrance location; further, by
very dilute solution, Cwsb(z) ~ constant 1=V w . Combine assumption (2), the transmembrane pressure drop P
now equations (7.2.31) and (7.2.33): everywhere is assumed to be constant at the value Pf
corresponding to the feed entrance location. In reality, P
d vz;avg jf C isb jf 2
 
jN wy j; as well as A will decrease somewhat along the membrane
dz C isb V w bf length; so will k i . The result of integration provides the
following analytical expression for L if we rewrite expres-
vz;avg jf C isb jf dC isb 2
jN wy j V w : 7:2:34 sion (7.2.38) as
C isb 2 dz bf
CisbL h A b Cisb z V w i
1 k i dC isb
L P f C isb jf 2 ; 7:2:40
C isb j P f bC isb zC isb z
12
The permeation rate of water through the membrane is quite low. f
Therefore the axial concentration change, (Cis/z), is small. No
wonder we need a lot of membrane surface area! leading to (Sirkar et al., 1982)
7.2 Crossflow membrane separations, granular filtration 565

  !
C isb jf b C isb jf Af b C isb jf V w produced. Earlier we employed the solutiondiffusion
L 1 model (e.g. equation (7.2.35)). The solute flux in the same
C isbL P f k il
" # model is given by equation (3.4.64b):
C isbL P f b C isb jf
 n : 7:2:41 Dim im 0
C isb jf P f b C isbL J iy C i C ip : 7:2:46
m
If we define the fractional water recovery as re (definition In the current problem, by assumption (3), C ip << C 0il .
(6.3.172b)), then, from the solute balance relation Further, the approximation leading to equation (7.2.36)
(7.2.33), valid for a membrane having a very high solute will also lead to (see equation (3.4.65d))
rejection,  
jN wy jV w
vz;avg jL C isb jf C 0il z C isb z 1 : 7:2:47
re 1 1 : 7:2:42 k i
vz;avg jf C isbL
Combining equation (7.2.36) with the above equation
Correspondingly, leads to
! 2 bC j  3
isb Dim im P

jN wy j

bC isb jf Af bC isb jf V w 1 1 Pf f J iy : 7:2:48

L re  n 4 5 : m b Ab
k il 1re 1 bC isbL

P f
Pf
The total molar solute permeation rate through the two
7:2:43
membranes lining one feed brine channel may now be
For feed solutions whose osmotic pressure is negligibly employed to determine C ip javg , the average concentration
small, i.e. bC isb jf << P f , this result is reduced to (Ohya of solute i in the permeate whose volumetric production
and Taniguchi, 1975) rate through the two membranes is Qpermeate:
L
vz;avg jf bf
 
L re ; L re : 7:2:44 Qpermeate C ip javg J iy 2W dz: 7:2:49
2 Af P f V w
0

The quantity (bf/2) needs to be elaborated on. Although bf is Introduce in the right-hand side integral expression
the gap between the membranes in the feed channel, one (7.2.48) for J iy and the expression for dz in terms of dCisb
can also define (2/bf) as the total active membrane surface from relation (7.2.34) to obtain
area per unit volume of the membrane feed channel:
W bf vz;avg jf C isb jf Dim im
 
2=bf 2W L=W Lbf : 7:2:45 C ip javg
Qpermeate m
An additional feature of a spiral-wound module is of interest; C isbL 
P jN wy j

dC isb
i.e. the number of feed brine channels in a module. There 2 : 7:2:50
C isb jf b Ab jN wy jV w C isb
can be as many as four feed brine channels around one
central collection tube; two membrane packets next to each
As before, along the module length P, A and k il are
other and separated by a feed side spacer create one mem-
assumed to be constants. On integration, we get (Sirkar
brane channel. The membrane surface area per unit device
et al., 1982)
" #
W bf vz;avg jf C isb jf Dim im 1 C isbL A PC isb jf 1 C isb jf
  
C ip javg 1 : 7:2:51
Qpermeate m Ab C isb jf jN wy jV w C isb 2 Ab V w C isbL

volume achieved in spiral-wound revere osmosis devices is To be noted here is that


around 800 m2/m3 (245 ft2/ft3) (Bhattacharyya et al., 1992). Qpermeate
This reference provides additional references and results for re : 7:2:52
vz;avg jf W bf
more detailed spiral-wound module design models
developed by Prasad and Sirkar (1985) and Evangelista Recognizing the result (7.2.42) for high rejection mem-
and Jonsson (1988). The mass-transfer coefficient ki in the branes as well as the expression for L from (7.2.37)
membrane channel may be determined from the (7.2.39), we obtain the following result:
Schock and Miquel (1987) correlation (3.1.170), provided
Dim im =m
in Table 3.1.8. C ip javg L re; 7:2:53
re A b V w
Our analysis has so far provided only an estimate of the
solvent recovery, re. Since the membrane is not perfect, it where no assumption has been made regarding jN wy j
is also necessary to determine the solute content (i.e. the except a very low solute permeation rate (leading to the
salt concentration in desalination) of the permeate solute mass balance result (7.2.33)).
566 Bulk flow perpendicular to the direction of force

We will briefly illustrate graphically the productivity Guess 1: re 0.14 leads right-hand side to be 0.15.
trend in a spiral-wound module. Figure 7.2.3(d) shows a Therefore our re guess is raised to 0.15.
plot of the behavioral trend exhibited by equation (7.2.43). Guess 2: re 0.15 leads right-hand side to be 0.161,
The objective is to predict the fractional water recovery re which is close to the left-hand side value of 0.16.
as a function of the nondimensional membrane channel
(Recall that in the limit of b C isb jf << P f ; L ! re (see
length L for various parametric values of b C isb jf
result (7.2.44)); this provides a basis for our guess of re, but
=P f for a given value of the ratio k il =Af P f V w .
it should also be remembered that re < L.)
Figure 7.2.3(d) does not provide exact values correspond-
ing to equation (7.2.43), but it provides the trends as illus- Example 7.2.4 If in Example 7.2.3 the value of Dim im =m
trated by Ohya and Taniguchi (1975). is known to be 2.4  105 cm/s, develop a rough estimate of
the salt concentration in the permeate.
Example 7.2.3 Brackish water containing NaCl at the level
of 2600 ppm is passed at 34 atm (gauge) through a spiral- Solution We employ equation (7.2.53) for a rough estimate
wound module. The gap between the membranes lining the of the average salt concentration in the permeate:
brine feed channel is 1.1 mm. The membrane length in the Dim im =m Dim im =m L re
brine flow direction is 70 cm. The length of the unwrapped C ip javg L re  bC j 
re A b V w

A P V isb f
membrane in the permeate flow direction is 150 cm. The re C isbf j w Pf f

pure water permeability constant for the membrane exposed


C ip javg
Dim im =m L re
to the feed brine at 20  C is 66.15  106 gmol/cm2-min-atm. ) bC j  :
The osmotic pressure of the brine solution is provided via C isb jf isb
re A P f V w Pf f
f 0.0115 (ppm)f, where f is in psi and the salt concen-
tration is in ppm. The average brine velocity in the brine we have Dim im =m 2:4  10 5 cm=s; L 0:16 and
channel is 320 cm/min. Estimate the fractional water recov- re 0:15 so
ery by assuming a highly rejecting membrane. You are given cm
that kil 0.51 cm/s. A P f V w 4:06  10 2
min
Solution We will employ equation (7.2.43) under the assump- 4:06  10 2 cm b C isb jf
; 0:0598;
tions of a highly rejecting membrane, constant P, kil, A, 60 s P f
corresponding to feed location and jN wy jV w =kil << 1 :
 
 2:4  10 5 0:16 0:15
b C isb jf b C isb jf 1 C ip javg =C isb jf 4:06  10 2
L re Af P f V w 0:15  60  0:0598
P f P f k il
b C isb jf 2:4  60  10 3
2  3
1 1 Pf ;
 n4 5: 0:15  4:06  5:98
1 re 1 b C isb jf  1 

P f 1 re
C ip javg C isb jf  0:0395 0:0395  2600 102 ppm:
The only unknown here is the fractional water recovery re,
which has to be determined by trial and error. Now The actual experimental value observed by Ohya and Tani-
vywf Af P f V w guchi (1975) was higher, at around 200 ppm. Note: Although
the units of C ip javg and C isb jf are mol/cm3, here we have
gmol 18:05cm3
66:15  10 6 34:0atm used ppm units.
cm2 -min-atm gmol
cm cm In the above examples, the fractional water recovery is
66:15  10 6  34:0  18:05 4:06  10 2 ; around 0.15. If one increases Pf, the fractional water recovery
min min
will increase, and so will the energy cost via the cost of
f osmotic pressure of feed solution 0:0115  2600psi: pumping. On the other hand, usually a higher Pf is used with
a higher feed salt concentration; therefore water recovery may
But
not increase. To increase the water recovery, a number of
b C isb jf 1:15  26 spiral-wound modules are connected in series (as shown in
f b C isb jf ) 0:0598:
P f 34  14:7 Figure 7.2.4(a)) inside the pressure vessel. There is a brine seal
between the module and the pressure vessel so that the brine is
2Lvywf 2  70cm  4:06  10 2 cm=min
L 0:16: forced to go through the channels of the module. The permeate
vz;avg jf bf 320cm=min  0:11cm
tubes are connected in series. Concentrated brine from one
Substitute these values into the expression for L above: module enters the next module as feed and so on. The frac-
tional water recovery is ultimately limited by the difference
0:0598  4:06  10 2
 
0:16 re 0:0598 between the concentrated feed pressure and the osmotic pres-
0:51 sure of the concentrate and the level of acceptable flux.
!
1 1 0:0598 To characterize the total performance of such a config-
 n  :
1 re 1 0:0598 uration of a number of modules in series, it is useful to

1 re
7.2 Crossflow membrane separations, granular filtration 567

(a)
Permeate tube Brine seal ring Module housing
Brine Concentrated
feed brine outlet
inlet
Module 1 Module 2 Module 3 Permeate exit

Brine seal ring Anti-telescoping support

(b)
Qf 1 Qf 2 QR2 Qfi QRi QRn
Module 1 Module 2 i th module n th module
Cif 1 Cif 2 CiR 2 Cifi CiRi CiRn

Qp1 Qp2 Qpi Qpn


Cip1 Cip2 Cipi Cipn n
Qpi
i1

Figure 7.2.4. (a) Schematic of a number of spiral-wound modules in series in a module housing which is a pressure vessel.
(b) Configuration for multiple modules in series or multi-tube system for reverse osmosis operation.

consider the configuration of a multi-tube system based 7.2.1.2.1 More detailed model for reverse osmosis in a
desalination, as shown in Figure 7.2.4(b). Although such module The simplified model already presented for
multi-tube based systems are rarely used at this time, the reverse osmosis desalination is quite useful, not just for
configuration is useful for spiral-wound modules of Figure illustrative reasons, but also for preliminary design pur-
7.2.4(a). Let the feed and permeate parameters for any poses. We repeat here our earlier assumptions (not all of
given ith module/tube be as follows: them explicitly).
feed: Qfi cm3/s, Cifi mol/cm3, Pfi; (1) Axial diffusion term in equation (7.2.26) neglected.
permeate: Qpi cm3/s, C ip javg mol/cm3. (2) Film theory employed for concentration polarization
estimate.
The retentate quantities needed are QRi cm3/s,
3 (3) Effect of wall permeation on the axial and normal
C iRi mol=cm and PRi. Note that, for the ith module/mem-
velocity profiles in the flow channels neglected.
brane tube,
(4) Constant physical properties.
P Ri P f i1 ; QRi Qf i1 ; C iRi C if i1 mol=cm3 : (5) Salt permeation for solvent flux determination neglected.
(6) Gravitational considerations neglected.
Further,
P f i P Ri P i ; The flow becomes highly complex in a spiral-wound
module containing a feed-side spacer screen. Numerical
the pressure drop in brine in the ith tube. The cumulative solutions of the governing equations incorporating most of
permeate volumetric flow rate entering the permeate side these complexities have been/are being implemented
X i
of the (i 1)th module is given by: Qpi , and therefore (Wiley and Fletcher, 2003) using computational fluid
i1 dynamics models (see Schwinge et al. (2003) for the com-
permeate concentration entering the permeate side plex flow patterns in a spacer-filled channel).
In these models, the equations governing the flow
i
X and species balance in the feed channel are standard and
Qpi C ipi
are coupled together via the variation of viscosity with
of i 1th module i1 i

: 7:2:54 concentration and wall permeation. On the other hand,
X
Qpi the wall permeation conditions of solvent and solute
i1
permeation need to be identified. If the membrane rejec-
Since in the analysis of the performance of a spiral-wound tion of the solute is identified via a salt rejection coeffi-
module fed with Qfi, Cifi and Pfi, we can obtain Qpi, C ip javg , cient, Ri, then the following conditions hold at the
and all other quantities in (7.2.54) may be obtained with- membrane wall (y 0) (Figure 7.2.3(b)) for the solute
out difficulty. transport:
568 Bulk flow perpendicular to the direction of force

d C is In this configuration (Figure 7.2.5(a)), the feed solution


feed channel side : Dil jvyw jC isw Ri ;
dy is convected along the membrane length at a pressure in
the range of 0.22 atm gauge; the permeate is withdrawn
dC ip
permeatechannelside : Dil jvyw jC ip jvyw jC isw 1Ri ; perpendicular to the membrane, and the permeate pres-
dy
sure is assumed to be the same all along the membrane
7:2:55 length. In practice, the permeate channel may be that for a
where hollow fiber module (as in Figure 7.2.1(c)) or a spiral-
  wound flat membrane module (as in Figure 7.2.1(e)). The
C ipw flow regime may be laminar or turbulent. Conditions at
Ri 1 : 7:2:56
C isw any location are essentially steady with respect to time,
except for any slow change due to membrane fouling.
The solvent permeation condition at the wall employs,
There are three general questions of importance here.
along with the no-slip condition (i.e. vz 0 at y 0), the
following type of boundary condition (Brian, 1965): (1) How does one predict the solvent flux in such a
      configuration?
C isw w
vyw vywf 1 1 vywf 1 1 ; (2) What would be the average solvent flux if there
C isf f
is considerable change in bulk concentration during
7:2:57
UF?
where is defined as (3) How does one treat the problem of purification of one
protein over another?
Ri f
7:2:58 The general species conservation equation (6.2.5b) for
P Ri f
macromolecular species i in solvent s may be written under
and we have assumed that w b C isw ; f b C isf . This the conditions of steady state, no external force based
may be written as follows: species velocities (Ui 0) and no chemical reactions as
  
vyw Ri f w P Ri w r C is v  r Ji r Dis r C is : 7:2:61
1 1 :
vywf P Ri f f P Ri f
7:2:59 For the two-dimensional flow conditions encountered
in the membrane device of Figure 7.2.5(a), the two coord-
This is a straightforward solutiondiffusion model, since inates relevant are y (normal to the membrane) and z

C
 (parallel to the length of the membrane) in the Cartesian
P 1 Cipw w
 
P Ri w isw P w p system. Therefore
;
P Ri f
 
C P f p
P 1 ipw f
C isf C is vy C is vz
   
C is C is
Dis Dis :
7:2:60 y z y y z z

where, for the feed solution driving force, Ri has been 7:2:62
suitably defined with respect to the feed concentration. Generally, the axial diffusion term (z-direction) is quite
small compared to the axial convection term. Therefore

C is vy C is vz
 
7.2.1.3 Ultrafiltration C is
Dis : 7:2:63
y z y y
In Sections 6.3.3.2 and 6.4.2.1, we studied ultrafiltration
(UF) in the configurations of a batch unstirred cell and in The solution of such an equation for an actual membrane
a well-stirred cell (and in a CSTS), respectively. In the device for ultrafiltration is difficult to obtain (see Zeman
batch unstirred cell, operation with bulk flow parallel to and Zydney (1996) for background information). One
the force direction led to the dead-end mode. The bulk therefore usually falls back on the stagnant film model for
protein concentration changed with time, leading to determining the relation between the solvent flux and the
unsteady state operation. The analyses of solvent flux and concentration profile (see result (6.3.142b)). To use this
solute rejection were carried out there, however, based on result, we need to estimate the mass-transfer coefficient
a pseudosteady state assumption. It was also pointed out k i Di = , for the protein/macromolecule. One can
(following equation (6.3.145c)) that bulk flow perpendicu- focus on the entrance region of the concentration bound-
lar to the direction of the force can lead to steady state ary layer, assume Dis to be constant for a dilute solution,
operation. In this section, one such common configur- v  v, v y 0 in the thin boundary layer, v z _ w y (where
ation, crossflow ultrafiltration, is briefly studied; it is also _ w is the wall shear rate of magnitude jdvz =dyj) and obtain
sometimes called high-performance tangential flow select- the result known as the Leveque solution at any location z
ive filtration. in terms of the Sherwood number:
7.2 Crossflow membrane separations, granular filtration 569

(a) Pp Cip Pp Pp (b) Cip = 0

y
b
Cilb nz Cisbo Cisb
Pf z z
Qo Q
y 0
b
Cil

Pp Cip Pp Pp Cip = 0
L

(c)
1.0

0.8
ns(z) avg 0.6
nso 0.4

0.2

0
1 2 3 4 5 6 7 8 9 10
+
C isbL, concentration factor

(d) Recycle
Initial Concentrate
solution
volume
Feed
tank
Membrane module

Permeate
(e) (f)
1.0
Buffer 3.0
Yield,Y1p

0.8 2.0
IncreasingY
1.0
0.6
Y=6 25 60 250 500
0.4 0.5
Membrane module
0.2 (DF)DS = 0.1

0
Permeate 1 10 100 1000
Purification factor P1

Figure 7.2.5. (a) Concentration profile of species i and axial velocity profile in a crossflow UF membrane channel. (b) Parallel-plate
crossflow UF membrane channel. (c) Reduction of averaged membrane module filtration flux with the extent of solute concentration.
(d) Process schematic for batch UF with a crossflow membrane module. (e) Process schematic for membrane diafiltration. (f) Variation
of the yield of purified product species 1 in the filtrate with its purification factor for different values of the parameters , (DF)S.
(After Van Reis and Saksena (1997).)
570 Bulk flow perpendicular to the direction of force

 1=3 for one wall of a symmetric channel having a gap 2b,


k i 2R 2R _ w
Shz ; 7:2:64a Figure 7.2.5(b))
Di 0:893 9 Di z
b b b  
where we have used a tube of diameter 2R as the mem- dC is vz dC is
dy dC is vy d Dis :
brane flow channel of interest. In practice, experimentally 0 dz 0 0 dy
obtained correlations for mass-transfer coefficients (illus- 7:2:65a
trated in Tables 3.1.8 and 3.1.5) are utilized in the
But
concentration polarization relation (6.3.142b)):13
b
 0  d 1 d
C i C ip C is vz dy QC isb ; 7:2:65b
N s V s jvy j k i n : 7:2:64b dz 0 2W dz
C ilb C ip
where Q is the volumetric flow rate of solution through the
The solvent flux expression is14
channel and C isb is the bulk concentration of the solute i in
N s A P : 7:2:64c solution; further, W is the width of channel, therefore 2W is
the membrane surface area per unit length. Equation
The solute rejection expression (from (6.3.145c)) is (7.2.65a) becomes
C ip S exp Pem  
i
Rtrue 1 0 1 : 7:2:64d 1 d dC is  dC is 
C i S exp Pem
i 1
 
QC isb C is vy jb C is vy j0 Dis b Dis y0 :
2W dz dy  dy 
There are three unknowns here: jvy j (the solvent volume 7:2:66
flux), C 0i (the wall solute concentration) and Cip for given
C ib , P, k i , S , Gi , m, m, Dip. One can solve the problem Due to symmetry, we can write vy jy0 0, dC is =
by simultaneous numerical solution of the three equations. dyjy0 0. We get
For systems having perfect rejection of the macrosolute,  !
d dC is 
Cip 0, Rtrue 1; only two equations ((7.2.64b) and Q C isb C is vy jb Dis b 2W : 7:2:67
(7.2.64c)) have to be solved. The complexities that have dz dy 
to be taken into account in this procedure include the
The magnitude of the right-hand side in parentheses is the
concentration dependences of the transport properties like
wall flux of species i, the flux through the membrane. Since
Di and .
the membrane rejects the macrosolute completely, we get
As ultrafiltration progresses along the module length,
the increased macrosolute concentration is likely to reduce d Q dC isb dQ
Q C isb 0 C isb : 7:2:68
the solvent flux through the membrane. We will therefore dz dz dz
consider now the problem of predicting the average solvent For the solvent, usually water in ultrafiltration of biological
flux obtained in a crossflow ultrafiltration membrane solutions, the equation corresponding to (7.2.63) is
module having a given membrane length or membrane
 
area when the following three conditions are satisfied at a C ws vy C ws vz C ws
Dws : 7:2:69
minimum: (1) membrane rejection of the macrosolute is y z y y
perfect; (2) gel polarization condition exists; (3) longitu-
Carrying out the integration in the same manner, we get
dinal diffusion is negligible compared to longitudinal con-

vective transport. A variety of module configurations exist. 1 d dC ws 
We may ignore these by assuming that the following char- QC wsb C ws vy jyb Dws yb N sy jyb ;
2W dz dy 
acterization may suffice: in the bulk flow direction of the z-
coordinate, the module length is L; the relevant transverse d  
Q C wsb N sy jyb 2W N sy jyb ams ; 7:2:70
dimension (e.g. the y-coordinate) is Rh, the hydraulic dz
radius, so that, for cylindrical tubes (or hollow fibers),
where ams is the surface
 area of membrane per unit channel
4Rh 2R, where R is the radius of the tube or hollow fiber,
length and N sy jyb is the solvent flux through the mem-
and for rectangular channels of gap 2b, 4Rh 2b. In
brane at the walls of the channel; recall that the channel walls
channels having a spacer, there are added complexities.
are identical and have the same membrane. Since, in most
For the macrosolute species i, consider equation
ultrafiltration applications, the solutions are quite dilute in
(7.2.63). For dilute solutions, we can replace vz by vz.
proteins15 or otherwise, we can rewrite this equation as
Integrate this equation across the membrane channel
transverse dimension y (from the center line y 0 to y b dQ 
N sy jyb ams =C wsb vs ams ; 7:2:71
dz

13
Note the change in the coordinate : we have jvy j here.
14 15
is to be replaced by i (see (6.3.158a)). Cwsb is essentially constant.
7.2 Crossflow membrane separations, granular filtration 571

where vs is the volume flux of the permeate (units of cm3/ L is obtained as a function of Q0 ; ams ; L and the extent of
cm2-s). Equations (7.2.68) and (7.2.71) are the basic increase in concentration from C isb0 to C isbL .
balance equations for determining how Q and Cisb Suppose the flux of the solvent is available for the inlet/
vary with distance z along the membrane module (Figure starting concentration, vs0 corresponding to C isb0 . One
7.2.5(b)). However, the solvent permeation flux vs N s V s would like to know how vs is changing with concentration
depends on, among other things, the bulk solute concen- C isb or z. Correspondingly, how does vs zjavg change vis--
tration Cisb (as well as the local P across the membrane). vis vs0 as C isb changes from C isb0 to C isbL . We will still
We have assumed earlier (assumption (2)) that a gel polar- employ the basic differential equation (7.2.77) as well as
ization condition exists. From result (6.3.143), we know (7.2.76). However, we will need to employ assumption (2),
that namely that a gel polarization condition exists, i.e. equa-
    tion (7.2.72) is valid. Therefore
C igel C ip C igel
vs k i n k i n ; 7:2:72
C ib C ip C isb
2 0 1, 0 13
C igel A C igel A5
vs z=vs0 k i z=k i0 n
4 @ n @
since C ip 0 by assumption and s. Therefore vs C isb C isb0
changes along the membrane length as Q and Cisb change. 2 3
Note that ki, the mass-transfer coefficient of the macro- nC isb =C isb0 5
k i z=k i0 41 : 7:2:79
solute species i, depends, among other things, on the nC igel =C isb0
solution velocity, the solution kinematic viscosity as well
as the solute diffusivity Dis. A common way of representing From relation (7.2.73), we get
these dependences is a correlation of the following type
k i z Qz a2 0 a2 z a3 Di 1 a3
         
(see Tables 3.1.5 and 3.1.8):
k i0 Q0 z 0 Di0
 a2   a3
k i 2b vz b
a1 : 7:2:73  1 a3  a3 a2
Di Di Di

a2 z
Q z : 7:2:80
Di0 0
Note that vz, and Di are all local quantities and vary with
z in the flow direction. We will now follow the procedure of The variation of Di with viscosity , concentration C isb and
Lopez-Leiva (1980) to develop an expression for the local other quantities may be obtained for dilute solutions of
permeate volume flux vs(z) as well as a length-averaged species i from relations (3.1.89) and (3.1.91a). We can
value vs zjavg . Define Cisb0, Q0, vs0, 0, Di0, ki0, to be the thereby obtain the following relation for (Di/Di0) if we
values of Cisb, Q, vs, , Di and ki at the inlet of the assume that the solution is ideal enough for
membrane channel (Figure 7.2.5(b)). Define further the d n =d n x i T;P 0 and the solution density does not
following nondimensional quantities: change much with the concentration change, i.e. (z)
C isb Q z ((z)/0), 0 (0/0):
C
isb ; Q ; z : 7:2:74
C isb0 Q0 L Di z=Di0 0 =z: 7:2:81
From equations (7.2.68) and (7.2.71), we get
This leads to
Q d C isb
vs ams : 7:2:75 
z
2a3 a2 1
C isb dz k i z=k i0 Q za2 : 7:2:82
0
Further, from equation (7.2.68),
Most often a3 1/3 (see Tables 3.1.5 and 3.1.8). Further,
QC isb constant Q0 C isb0 : 7:2:76 employing relation (7.2.76),
We can rewrite equation (7.2.75) as a2   a2 1=3
C isb0

z
k i z=k i0 : 7:2:83
Q0 C isb0 d C isb Q dC C isb 0
20 isb
vs ams : 7:2:77
C 2isb dz C isb L dz
Since one can obtain a relation for as a function of C isb ,
Integrating along the membrane length, we obtain we may rewrite this relation as
1 C  a2   a2 1=3
Q0 dC Q0 1 k i z 1 C
  
isb
isbL
isb
vs zjavg vs dz 1 ; k i C isb =k i0 :
0 ams L 1 C 2
isb ams L C isb L k i0 C
isb 0
7:2:78 7:2:84

where C is the value of


isb L C
at z 1. This expression is
isb We now go back to the basic differential equation (7.2.77),
essentially a solvent balance expression for the membrane introduce for vs(z) expression (7.2.79), wherein for
device of length L: the average solvent flux over the length k i z=k i0 , we introduce expression (7.2.84) given above:
572 Bulk flow perpendicular to the direction of force

1 a2 C (i 1) may be more selectively permeable than the other


" #  a2 1=3
Q0 dC nC
 
isb isb isb
2 v s0 1 : (i 2). Thus one would like to have a high yield of i 1 in
C dz nC igel C isb 0
isb ams L
 
the permeate; the yield of this solute in the permeate
7:2:85
(segregation fraction) is defined for a cumulative permeate
Introducing expression (7.2.78) for vs zjavg and integrating volume Vp as
from z 0 to 1 and C
isb 1 to C isbL , we get
V p
C ip dV p
V p C ip V p C 1p
vs zjavg

1

1 Y ip 0 Y 1p :
1 V f 0 C i0 V f 0 C i0 V f 0 C 10
C
vs0 C
isbL isbL dC isb   7:2:87
1
 2a2 hC isb i a2 1=3 nC
C isb 0 1 nC isb
igel Correspondingly, the other protein (i 2) should be
1 retained as much as possible in the retentate. Therefore
1 C
isbL
: 7:2:86 the yield of solute 2 in the retentate (in the continuous
I C
isbL diafiltration mode) may be defined as

Numerical integration to obtain the value of the integral C iR dV f
V f 0 C iR C 2R
IC
isbL for given dependences of (as well as Di ) on C isb Y iR Y 2R : 7:2:88
V f 0 C i0 V f 0 C i0 C 20
will yield values of the relative reduction in the solvent flux
with increase in concentration along the membrane under
We have observed earlier (in Section 6.4.2.1) that, for con-
the condition of gel polarization. Figure 7.2.5(c) illustrates
tinuous diafiltration (result (6.4.107)),
how the ratio vs zjavg =vs0 decreases with an increase in
the value of C isbL , which is sometimes called the Y iR Y 2R exp 1 R2 DF: 7:2:89
concentration factor (Lopez-Leiva, 1980).
In Figure 7.2.5(c), the values of the concentration Correspondingly, we may obtain
factor can be as high as 510. Although such an increase
in concentration can be achieved in long modules, a more V p C 1p V f 0 C 10 V f 0 C 1R V f 0 C 1R
Y ip Y 1p 1
likely scenario is crossflow ultrafiltration of a feed solution V f 0 C 10 V f 0 C 10 V f 0 C 10
from a batch vessel with the retentate recycled to the feed
tank (Figure 7.2.5(d)). At any given time, for the given feed 1 Y 1R 1 exp 1 R1 DF:
concentration entering the module, the module perform- 7:2:90
ance analysis may be implemented using equation (7.2.86).
Both results may be expressed in terms of the sieving
However, with time, this feed concentration of the rejected
coefficients S1 ( 1 R1) and S2 ( 1 R2) of the two
protein keeps on increasing substantially, resulting in high
solutes and the dilution factor DF (also called the number
concentration factors. The disadvantages of such a scheme
of diavolumes) as
are the potentially long time taken and the increase in
viscosity, leading to a significant reduction in flux and Y 1p 1 exp S1 DF; 7:2:91a
potential bacterial growth in many biological systems.
A common alternative used where possible is the Y 2R exp S2 DF; 7:2:91b
diafiltration mode with a crossflow UF membrane unit
where
and concentrate recycle (Figure 7.2.5(e)). Here the solution
concentration and viscosity are not allowed to increase due DF dilution factor V b =V f 0 V p =V f 0
to the continuous addition of buffer replacing the perme- number of diavolumes 7:2:91c
ate volume lost. Equations developed in Section 6.4.2.1 for
well-stirred UF cells having continuous diafiltration may be and Vb is the buffer volume added (Vp).
used here with appropriate care since we can treat the It is not enough to have a high yield of species 1 in the
crossflow UF device as a blackbox for the purpose of an permeate; we want the permeate to be as pure as possible
overall process mass balance and solute selectivity analy- in species 1. The selectivity of the UF process for one
sis. Similarly, the equations developed in Section 6.4.2.1 for protein (more permeable species 1) over the less perme-
a batch concentration process may be utilized here to deter- able protein (species 2) has been described by a number of
mine various quantities, such as the yield of macrosolute, indices (see Van Reis and Saksena (1997)):
retentate concentration, etc.
S1 1 R1 C 1p =C 1f
In many ultrafiltration applications, purification of 12 ; 7:2:92
S2 1 R2 C 2p =C 2f
one protein from another protein is often required from
a batch solution of volumeVf 0. Both proteins are perme- where 12 was defined in (6.3.147b) for dilute solutions.
able through the membrane; however, one of the solutes Another selectivity index employed is
7.2 Crossflow membrane separations, granular filtration 573

S S1 S2 : 7:2:93 S1 (DF)S2 (DF)S, of these parameters are of interest.


For example, if one combines the selectivity definition
To combine the properties of quantitative yield (Yip, YiR) (7.2.92) with expression (7.2.91a) for Y1p and expression
and the quality of the permeated product ( S1/S2), Van (7.2.97) for the purification factor P1 for the desired protein
Reis and Saksena (1997) have proposed an additional par- species 1 in the filtrate, we obtain the following relation
ameter called N S, which in our notation is between P1 and Y1p:
Vp
DF S S1 S2 7:2:94 1 exp S1 DF Y 1p
Vf0 P1 ;
1 exp S2 DF 1 1 Y 1p 1=
(N being the number of diavolumes, which is equal to our 7:2:100
dilution factor DF (6.4.106)). Noting that S ( Sobs) is Cip/
Cif , which is Cip/CiR, we see that since, from (7.2.91a),

V p C 1p V p C 2p V p C 1p V p C 2p 1 Y 1p exp S1 DF ) n1 Y 1p DFS2
DF S ; 1
V f 0 C 1f V f 0 C 2f V f 0 C 1R V f 0 C 2R ) nexp S2 DF n 1 Y 1p
 

7:2:95 ) exp S2 DF 1 Y 1p 1= :
a quantity similar to the extent of separation, , defined in The above expression relates the product purification
Chapters 1 and 2. It is exactly equal to at the initial stages factor P1 for species 1 in the filtrate to its yield in the filtrate
of UF when C1R C1f and C2R C2f . Y1p through the selectivity . The corresponding relation in
Van Reis and Saksena (1997) have identified the prod- terms of the parameter (DF)S is obtained from equations
uct yield and the product purification factor as the two (7.2.91a) and (7.2.97) as
dimensions of selective protein separation with, for
example, or (DF) S as parameters. The product purifi- Y 1p
P1 ; 7:2:101
cation factor P1 for solute 1 in the permeate was defined as 1 Y 1p 1 exp DF S
   since, from (7.2.91a),
V p C 1p V p C 2p hf
P1 12 : 7:2:96
V f 0 C 10 V f 0 C 20 Y 1p 1 exp S1 DF
Using expression (7.2.90) for the yield Y 1p and applying it and
to solute 2 as well (namely Y 2p ), we get
fexp S1 DFgexp DFS exp S2 DF:
Y 1p 1 exp S1 DF
P1 : 7:2:97 If the protein species 2 in the retentate is of interest,
Y 2p 1 exp S2 DF
the purification factor for species 2, P2, defined by (7.2.99)
For solute 2, the purification factor P2 in the retentate is is related to the yield of species 2 in the retentate Y2R by
defined as
 P 2 Y 2R 1 7:2:102
V f 0 C 2R V f 0 C 1R ft
P2 12 : 7:2:98 (from (7.2.99),
V f 0 C 20 V f 0 C 10

Using expression (7.2.91b) for the yield Y 2R , we get log P 2 DFS1 S2 S2 DF1
) P 2 exp S2 DF1 Y 2R 1 ;
Y 2R exp S2 DF
P2 exp DFS1 S2 :
Y 1R exp S1 DF with as the parameter. The corresponding relation
7:2:99 between P2 and Y2R with (DF)S as the parameter is

One can now plot the yield vs. the purification factor for P 2 exp DF S: 7:2:103
two cases: the desired protein is in the permeate; the desired
It is useful now to illustrate graphically the relation between,
protein is in the retentate. Commercialized UF processes
for example, P1 and Y1p with either or (DF)S as a param-
target both high product yield and high selectivity in bio-
eter. Figure 7.2.5(f) illustrates the behavior. There are two
pharmaceutical industries when it comes to biomacromo-
types of curves here (Van Reis and Saksena, 1997); one type
lecules/proteins: yields in the range of 75% 100% with ten-
of curve depends on the selectivity parameter , while the
fold or higher product purification factors (Van Reis and
other type is based on the parameter (DF)S ( (DF)(S1 S2)).
Saksena,1997). Before we illustrate such a plot, we need to
The key features of these two types of curves are as follows.
recognize the relation between Pi and Yi (Yip or YiR)
and their dependence on the important parameters for (1) When purification increases, yield decreases, regard-
this process DF, S1 and S2. We have indicated earlier that less of the parameter used. Yield is highest when the
often the following two combinations, (S1/S2) and (DF) purification is lowest and vice and versa.
574 Bulk flow perpendicular to the direction of force

(2) The highest value of the purification factor P1 for a


k il z
 
constant selectivity is : 0:41 :
k il 0
" # " #
Y 1p Y 1p We will now calculate the mass-transfer coefficient ratio
lim P 1 lim lim Y 1p :
Y 1p !0 Y 1p !0 1 1 Y 1p 1= Y 1p !0 1 1 using result (7.2.84):
7:2:104 
1 a2 C
   a2 1=3
isb
k il z=k il 0 ; a2 1=3;
(3) The lowest value of the yield Y1p for a given a value of C
isb 0
the parameter (DF)S is of interest; then, from 
15

2

(7.2.101), we get C
isb C isb z=C isb0 15=2 7:5:
100M i 100M i
P 1 1 exp SDF Assuming that the density change in the solution for the
Y 1p ; concentration levels is not substantial,
1 P 1 exp SDF
C
isb C
isb exp 0:00244solu z  1002
Which, in the limit of P 1 ! , is reduced to
0 0 exp 0:00244solu 0  1002
lim Y 1p 1 exp SDF exp SDF
P1 !

exp 0:00244 152 22

exp 0:54:
1 exp SDF: 7:2:105
Therefore
If one knows what the selectivity is during a particular
2=3
1

purification process, then one can move along a constant
k il z=k il0 1=7:51=3
and find out how Y1p will change with P1p from figures of exp 0:54
the type shown in Figure 7.2.5(f). 0:515  0:723 0:4145:

Example 7.2.5 This example will illustrate the level of solv- Therefore
ent flux reduction in a hollow fiber UF unit as a protein vs z=vs 0 0:41  0:4145 0:170;
solution is substantially concentrated. Consider a feed solu-
tion of BSA (bovine serum albumin) containing 2 g/100 cm3. indicating a very significant drop in flux with length or time
This is to be concentrated to 15 g/100 cm3. Kozinski and as the case may be.
Lightfoot (1972) obtained the following correlation for the
Example 7.2.6 Calculate the value of vs(0) in Example
viscosity of a BSA solution:
7.2.5 using the following additional information. Hollow
exp 0:00244solu  1002 ; fiber: I.D., 750 m; length, 50 cm. Feed solution of
BSA: density ~1 g/cm3; viscosity, 0.9 cp. Diffusivity of BSA:
where solu is the density of the BSA solution in g/cm3. It is Dil0 Dil 5:94  10 7 cm2 =s (Table 3.A.5). Velocity of
known that the value of a2 in the correlation (7.2.73) of the feed solution through fiber bore, 80 cm/s. For hollow fibers,
Sherwood number dependence on the Reynolds number is the Leveque solution (equation (3.1.145)) should be
1/3. It is further known from Kozinski and Lightfoot (1972) employed to determine the mass-transfer coefficient for the
that the gel concentration for the BSA solution is 58.5 g/ hollow fiber UF membrane.
100 cm3. Determine the fractional reduction in solvent flux
as the BSA solution is concentrated along the module length Solution Hollow fiber: I.D., di 750 m 750  104 cm
(z) (or time) from 2g/100 cm3 to 15 g/100 cm3. 0.075 cm; length, L 50 cm. Fluid velocity, vz 80 cm/s.
Fluid density 0 1 g=cm3 . Fluid viscosity, 0 0:9 cp
Solution In the gel polarization model, we know from results 0:009 g=cm-s. Leveque solution (equation (3.1.145)):
(6.3.143 and 7.2.92) that, between locations z 0 and z,
 1=3
2 0 1, 0 13 k il0 d i di
Shavg 1:615 ReSc1=3 ;
4 @ C igel A @ C igel A5 D il L
vs z=vs 0k il z=k il 0 n n
C isb z C isb 0 di vz 0:075  80  1
2 0 1, Re 667;
0:009
4 @ 58:5=M BSA 100A
) vs z=vs 0k il z=k il 0 n 0:009
15=M BSA 100 Sc 1:51  104 ;
Dil 1  5:94  10 7
0 1#
58:5=M BSA 100A 0:075 1=3
 
n @ Shavg 1:615   667  1:51  104 1=3
2=M BSA 100 50
k il0 di
"  ,  # 1:615  0:00151=3  1010  104 1=3 40 ;
58:5 58:5 Dil
) vs z=vs 0k il z=k il 0 n ln
15 2
40  5:94  10 7
k il0 3:17  10 4 cm=s:
) vs z=vs 0 log3:9=log29:25k il z=k il 0 0:075
7.2 Crossflow membrane separations, granular filtration 575

From the gel polarization model result: near the membrane a higher volume fraction of particles
vs0 k il0 nC igel =C isb0 ; than s. The higher particle concentration near the mem-
brane leads to diffusion of the particles to the bulk of the
vs0 3:17  10 4  2:303  log 58:5=2 suspension. There develops a boundary layer of higher
) vs0 1:07  10 3 cm=s: particle concentration and therefore particle volume frac-
tion from w at the membrane surface (y 0) to s (<w)
In industrial practice, a different unit, liter/m2-hr (lmh) is
employed; vs0 38.5 liter/m2-hr. at y > p, the boundary layer thickness. Note that the value
of this p is different from v, the thickness of the velocity
boundary layer. The particle concentration boundary layer
7.2.1.4 Crossflow microfiltration
thickness, p, depends on z and increases with z, as shown
In Figure 6.3.25(b), we illustrated how the unsteady behav- in Figure 7.2.6(a). However, p is very much influenced by
ior in dead-end cake filtration/microfiltration may be the velocity field, amongst other things.
avoided (among other methods) by having a bulk motion There are other important physical features in this flow
of the suspension/slurry perpendicular to the direction of configuration. At the inlet region of the membrane, z  0,
the force, causing, for example, microfiltration. At any axial the filtration flux vs is determined only by the resistance of
location, the time-dependent buildup of a cake layer is the membrane, Rm (from equation (6.3.136a)), P and :
avoided; instead, the thickness of the boundary layer of
vs0 P=0 Rm ; 7:2:106
particle suspension over the membrane keeps on increas-
ing in the bulk flow direction. This increase accommodates where we have specifically identified this flux value as vs0
the particles rejected at any axial location, as shown in (0 being the solvent viscosity). However, a short distance
Figure 7.2.6(a). However, there is a thin cake layer or an (zcr) downstream, a thin cake layer forms on the mem-
immobile layer of particles over essentially the whole brane, and it grows with the downstream distance coordin-
length of the microfiltration membrane unless the shear ate z. Correspondingly, the filtration flux is reduced from
rate due to the tangential flow is high enough to drag this vs0 to a vs which decreases as z increases beyond zcr:
layer and prevent the formation of a cake layer. Therefore,
in general, one can envisage a steady state operation ^ c c =Rm :
vs vs0 =1 R 7:2:107
delivering a filtrate/permeate free of particles at a steady
rate and a concentrated slurry at the exit of the separator. Here we have employed the filtration volume flux expres-
In reality, most systems display a decline in permeation sion (6.3.136b) and the expression (7.2.106) for vs0. For an
estimated value of the specific cake resistance R ^ c , vari-
flux with time, due to what is called fouling of the mem-
brane. Membrane fouling results from internal membrane ation of c with z will yield the dependence of vs on z; one
fouling and external cake fouling (Davis, 2001). Deposition can then integrate vs along z and obtain an expression for
or attachment of materials within the porous structure of the length-averaged filtration flux vs,avg for the membrane
the membrane by adsorption, adhesion, pore plugging length. This is a major goal for any model.
(Figure 6.3.27(c)) and precipitation leads to reduced solv- Solution of the particle concentration profile in the
ent permeability of the membrane: this is called internal particle concentration boundary layer from s in the feed
membrane fouling. External cake fouling results from the suspension liquid to the concentration w on top of the
formation of a stagnant cake layer on the membrane sur- cake (and equal to the concentration in the cake) requires
face aided by surface adsorption, pore mouth blockage, consideration of the particle transport equation in the
etc. (Figure 6.3.27(c)). We will describe later strategies boundary layer. We will proceed as follows. We will first
adopted to overcome such deficiencies. However, we will identify the basic governing differential equations and
first briefly describe a model that yields a realistic estimate appropriate boundary conditions (Davis and Sherwood,
of solvent flux in crossflow microfiltration, sometimes 1990) and then identify the required equations for an
identified as tangential-pass microfiltration (Romero and integral model and list the desired solutions from Romero
Davis, 1988; Davis and Sherwood, 1990) under the and Davis (1988). However, we will first simplify the
assumption of no fouling. population balance equation (6.2.51c) for particles under
conditions of steady state nr p =t 0 ; no birth and
 
As shown in Figure 7.2.6(a), consider a microfiltration
membrane with the tangential flow of the suspension over death processes (B 0 De), no particle growth
it in the direction of the membrane length coordinate z. U int
p 0 and no particle velocity due to external forces
The suspension pressure is likely to be 1 to 2 atmospheres U p 0, namely
above the pressure on the other side of the membrane. The r vnr p r Dp rnr p 7:2:108
coordinate normal to the membrane into the suspension is
y. A suspension having a solids volume fraction s contacts to an equation in terms of particle volume fraction in the
the membrane at z 0 and flows along z. As filtrate goes flowing liquid. Since n(rp)drp represents the number of
through the membrane, rejected solid particles build up particles between the size rp to rp drp per unit total
576 Bulk flow perpendicular to the direction of force

(a)
Suspension flow
nz (y)
f = fs

dp
dp fw Boundary layer of particles
y

Stagnant cake layer


Microfiltration membrane
ns0
Z
cr
Z
L

(b) (c)
A3

Particle ns0
boundary
dp layer ns
y A1 A2
L

z Dz zcr
z
p
Particle volume flux in through A1 =
0
nz f z dy
p
Particle volume flux out through A2 = + 0
nz f z+Dz dy

Particle volume flux in through A3 = ny fs Dz


p p
0
nz f z dy ny fsDz = 0
nz f z+Dz dy

(d) (e)
1.0
b=0
1.0
0.8
ns+ 0.8 d+
avg Increasing b 0.6 b=0
0.6
0.4
0.4
0.2
0.2 b = 10

0 0
1 2
103 1 10 102 103
10 10
z / zcr z / zcr

Figure 7.2.6. (a) Boundary layer of particles and a growing stagnant cake layer over a crossflow microfiltration membrane. (After
Romero and Davis (1988).) (b) Control volume balance for obtaining equation (7.2.121). (c) Local filtration flux profile along membrane
length. (d) Nondimensional length-averaged filtration flux for different . (After Davis (1992).) (e) Nondimensional cake thickness
profile. (After Davis (1992).)

 
volume, the volume fraction of such particles per unit total vy vz
Dp : 7:2:110
volume is n(rp) 4=3 r 3p dr p . If the particles are of only y z y y
one size, rp, then
  The equation needed along with it is the two-dimensional
4 equation of continuity for an incompressible liquid (6.2.5l):
nr p r 3p dr p ;
3
vy vz
where is the volume fraction of particles in the liquid, 0: 7:2:111
y z
and equation (7.2.108) becomes
While these two equations govern the particle concentra-
r v r Dp r: 7:2:109
tion boundary layer for a given liquid velocity field, their
The particle concentration boundary layer on top of the solution has to be carried out with the applicable boundary
membrane is essentially two-dimensional: equation conditions at y 0 (surface of the membrane for z  zcr
(7.2.109) is simplified to and surface of the cake for z  zcr) and y p(z) (the
7.2 Crossflow membrane separations, granular filtration 577

surface of the growing particle concentration boundary tubes : w 4 Q= R3 ;


 
7:2:119a
layer). If we consider the general expression for particle
channels : w 3 Q=2 b2 W :
 
number flux in a given coordinate direction (equation 7:2:119b
(3.1.70), the expression for the y-direction is
These two membrane channel configurations are often
N t encountered in crossflow microfiltration.
npy Dp U pyt N t ; 7:2:112
y We will now briefly illustrate how the governing equa-
tion for the integral model for crossflow microfiltration
where Nt is the total number of particles per unit volume of
developed by Romero and Davis (1988) is obtained. Then
the liquid. Since Upyt is equal to the liquid velocity vy (by
we will provide the results from such an analysis. Consider
our earlier assumption) and N t 4=3 r 3p , the particle
equation (7.2.110) and integrate it along the boundary
volume fraction in the liquid, the volume flux of particles in
layer thickness y 0 to p:
the y-direction is given by p p p  
  vz vy
4 3 dy dy Dp dy:
r p npy Dp vy : 7:2:113 0 z 0 y 0 y y
3 y 7:2:120a
At the surface of the membrane (z  zcr,) and on the surface Exchanging the integration and differentiation processes
of the cake on the membrane (z  zcr), the particle volume for the first term, we get
flux is zero, hits w and the liquid velocity magnitude in the
p
p p  
y-direction is equal to the local filtration volume flux vs:
vz dy d vy d Dp
z 0 0 0 y
 
at y 0; Dp vy 0; 7:2:114 
 
y Dp p Dp y 0 : 7:2:120b
y  y 
at y 0; vy vs ; vz 0; w ; 7:2:115
at y > p ; s : 7:2:116 Now, at y p, (/y) 0. Further, from the continuity
equation (7.2.111),
When we neglect the axial convection in equation (7.2.110) y y
vz 
and use boundary condition (7.2.114) in the integrated dvy dy vy vy 

vy vs : 7:2:120c
form of the particle concentration boundary layer equation 0 0 z y o

(7.2.110), we obtain Substituting these into equation (7.2.120b), we get


y y  
p
p  y
vz
 

dvy d Dp ; vy vy j0 vz dy d vs dy Dp  ;
0 0 y z 0 z y y 0
 0 0
 p y
Dp Dp ; vy Dp
 
7:2:117 vz  
y y  y vz dy vs dy s vs  Dp  ;

0 z 0 0 z y0 y y0
in general, with the restriction that it is not valid at y p. It
p   
 
vz s dy vs s vs  Dp  :

is obviously always valid at y 0. z 0 y0 y y 0
A completely general solution of the governing
7:2:120d
convective diffusion equation (7.2.110), and equation
(7.2.111), subject to the boundary conditions (7.2.114), The quantity in the parentheses on the right-hand side is
(7.2.115) and (7.2.116) is not available. There are two types the particle flux at y 0; by the boundary conditions
of solutions, similarity solutions and integral boundary (7.2.114) and (7.2.115), it is zero. We are left with
layer solutions (apart from complete numerical solutions).
p

Common to both of these solutions is the assumption that vz s dy vs s ; 7:2:121
the particle concentration boundary layer is very thin com- z 0
pared to the membrane channel dimension normal to the which is the governing equation for the integral model. If
axial flow; further, the shear stress due to the axial velocity we take a thin control volume spanning the boundary
gradient in the particle concentration boundary layer is layer thickness, having a thickness z in the axial direc-
equal to that at the wall, namely w: tion (Figure 7.2.6(b)), then we can derive the above equa-
tion easily by a balance on the particle flux: the left-hand
vz =y w 7:2:118
side identifies the net increase in particle flux across the
for all y  p. We should note that the values of w for boundary layer over thickness z, whereas the right-hand
laminar flow in tubes (of radius R) and channels (channel side identifies the particle flux coming in from the top
half height b, channel width W) are, respectively (Bird surface of length z (remember, there is no flux at the
et al., 2002), bottom surface of the control volume, where all particles
578 Bulk flow perpendicular to the direction of force

are rejected by the membranecake on top of it (Figure volume fraction ( s) (over that in the feed suspension)
7.2.6(b))). in the boundary layer being convected axially. In Romero
We can derive result (7.2.121) from considerations of and Daviss (1988) analysis, a nondimensional form of Qp
flux through these three surface areas of the control is utilized:
volume of length z in Figure 7.2.6(b):
Qp 30 v2s
Q
p ; 7:2:124
particle volume flux in through surface A1 of unit width 3w r 4p
 
perpendicular to the paper at z vz dy : where rp is the particle radius and w is the wall shear
0 z
stress.
7:2:122a
To develop an expression for Qp or Qp , Romero and
The corresponding particle volume flux in at surface A2 at Davis (1988) proceeded as follows. Using equation
  (7.2.117) between the limits of w and , and assuming
z z vz dy : vy vs,
0 zz

Particle volume flux in through top boundary layer w 0 0


Dp d
surface A3 is s vy jp z: 7:2:122b vy dy vs dy vs y: 7:2:125
y
y
At steady state,
However, the shear-induced particle diffusivity Dp() may
be described via relation (3.1.74) and a nondimensional
   
vz dy vz dy s vy jp z 0:
0 z 0 zz
particle diffusivity D
p as

7:2:122c Dp Dp
D
p 2 : 7:2:126
In the limit of z ! 0, we get, first by dividing by z r 2p _ r p jdvz =dyj
throughout and then taking the limit,
Further, from the shear stress relation (7.2.118),
p

w
vz dy s vy jp 0: 7:2:122d jdvz =dyj w = ;
z o 0

From the continuity equation and result (7.2.120c), where


y p
vz vz =0 7:2:127
v y vs dy ) vy jp vs dy:
0 z 0 z
is a nondimensional viscosity. Substituting these two rela-
7:2:122e
tions into (7.2.125), we obtain
Substituting this into equation (7.2.122d), we get   w !
Dp 2  dvz 
w
Dp d w 2
vs y rp   d r ;
p  dy  0 p


vz s dy vs s ; 7:2:122f
z o !
w w 0 0
vs 0 y D
p d D
p d
which is identical to equation (7.2.121). ; 7:2:128
r 2p w 0 0
Solution of equation (7.2.121) requires a knowledge of
the particle concentration profile (y) and the velocity an implicit equation for the particle volume fraction as a
profile vz(y). This is achieved by solving equations function of y, the distance from the wall. The value of w is,
(7.2.117) and (7.2.118) for vz(y) and (y) (Davis and however, unknown; its maximum value, max, is known,
Leighton, 1987). These profiles have been employed by and is equal to the maximum packing volume fraction of,
Romero and Davis (1988) to calculate a quantity called say, spheres, i.e. ~0.6 (Davis and Leighton, 1987). We need
the excess particle flux, also an expression for vz to develop an expression for Q p
p z z via (7.2.123) and (7.2.124). Use the wall shear stress rela-
Qp vz s dy vs s dz: 7:2:123 tion (7.2.118) to obtain
0 0
y
w w y dy
The quantity on the right-hand side identifies the rate at vz dy 7:2:129
0 0 0
which particles have been carried into the boundary layer
over the membrane length 0 to z through the top boundary (since vz 0 at y 0). Substitute this result, along with the
layer surface due to the filtration from the bulk suspension; differential form of (7.2.128) in terms of dy, into (7.2.123)
the left-hand side reflects this via the excess particle and (7.2.124):
7.2 Crossflow membrane separations, granular filtration 579

z
p
2

3
30 v3s0 s
3 v2 w 4 y 1 r 2p w D
p d5 Q
Q 30 4s p s Qp s jcr zcr : 7:2:134
p 3w r 4p
w rp 0
0 0 vs 0

r 2p w D If we now introduce the expression (7.2.107) for the filtra-


p
s d tion flux vs influenced by the developing cake layer into the
vs 0
right-hand side integral of expression (7.2.130) for Q p,
2
w 0
3
we get
w
Dp d 0 s D
p
4 5 d z
0 2 0
vs0 dz
s
30 v2s0 s
 z  3 2 0 1 ^
R c c =Rm
v Q
p Qp s jcr ;
vs s dz 30 4s : 7:2:130 3w r 4p 1 R^ c c =Rm 2
0 w rp
z
30 v3s0 s 1 dz
This is an implicit double integral based equation for the Q
p s jcr :
^ c c =Rm 2
3w r 4p 1 R ^ c c =Rm
0 1 R

unknown w. For given values of s and Q p , one can


determine the value of w at the wall via numerical inte- 7:2:135
gration (Romero and Davis, 1988), provided the functional
Although one would like to consider this expression and
dependences of Dp() and () on are known. These
determine the unknown c as a function of z, the growing
relations are:
cake layer thickness in the axial direction, so that vs may be
2
D
p 0:33 1 0:5 exp 8:8 7:2:131a determined from (7.2.107) as a function of z, we have to
consider the effect of a growing c on the shear stress w. If
(Leighton and Acrivos, 1987; Romero and Davis, 1988); we consider flow in a membrane tube (or hollow fiber) of
0:58 0:13=0:58 2 7:2:131b radius R, then, for a constant bulk liquid flow rate Q, we
may obtain wc, the wall shear stress in the cake region,
(Davis and Leighton, 1987). from (7.2.119a) as
Let us now focus on the right-hand side of equations
(7.2.130) or (7.2.123). At z 0, the particle boundary layer w R3 wc R c 3
) wc w R3 =R c 3
starts forming (Figure 7.2.6(a)). The filtration flux value or 4 4
the permeation velocity magnitude vs0 is constant for a
c 3
. 
certain length of the membrane. Therefore, on integration w 1 : 7:2:136a
R
we obtain

30 v3s0 s Similarly for a channel configuration, from (7.2.119b) we


Q
p z: 7:2:132 obtain
3w r 4p
b2 c 2
. 
However, the wall volume fraction of particles, w, keeps on wc w 1
2 w : 7:2:136b
increasing from s at z 0. When it reaches the maximum b c b
value max at zcr , a cake layer starts forming for z  zcr. The
The quantity w in equation (7.2.135) should now be
cake layer thickness c keeps on increasing beyond zcr:
replaced by wc. On substituting these two results into
3w r 4p equation (7.2.135), we get,
zcr Q
p jcr : 7:2:133
30 v3so s
membrane tube :
As a result, the filtration flux, vs, defined by (7.2.107) keeps 9 z
30 v3s0 s 1 Rc

on decreasing with increasing c, with R ^ c assumed con- dz
Q
p s jcr
2 ;
stant for an incompressible cake. The range of values of 3w r 4p 1 R

^ c c =Rm ^ c c =Rm
1 R
0
Q 7
p;cr in crossflow microfiltration can vary between 10 and 7:2:137a
3
10 (Davis and Leighton, 1987). These authors have also
suggested that, for very dilute suspensions, the value of Q p membrane channel :
should exceed 1  104 for z  zcr. However, Romero and 6 z
30 v3s0 s 1 bc

dz
Davis (1988) have suggested16 that, for z  zcr and a given Q
p j
s cr ;
3w r 4p 1 R ^ c c =Rm 2 1 R
^ c c =Rm

s, Q p is constant at the value given by (7.2.132), 0


7:2:137b

16 where w corresponds to the membrane tube/channel


Davis and Leighton (1987) have shown that, for a given s, the value
of the double integral representing Q without any cake layer. Define the following nondimen-
p s jcr becomes constant after
w ) max 0:6. sional quantities (Davis, 1992):
580 Bulk flow perpendicular to the direction of force

z z=z cr ; v L=zcr  1=3


s vs =vs0 ) jtube c =R ; 1 dz 2 3 1
v
s javg L 2=3 ;
^ c R=Rm ;
jtube R L=z cr 1=3 3 2 L
3
2z

0

c 7:2:144a
jchannel ; ^ c b =Rm :
jchannel R 7:2:138
b
where L L=zcr ;
Equation (7.2.137a) for a membrane tube is reduced to  2=3 
3 z cr 1=3
21
z
3 v
s javg ) vs javg 1:31 vso z1=3
cr =L
1=3

1 9 4 dz dz 5 2 L
1: 7:2:139 7:2:144b
1 2 1 1
0 1
!1=3
Equation (7.2.137b) for a membrane channel is similarly 3w r 4p Q
p s jcr
) vs javg 1:31 vso
reduced to L30 v3so s
21 z
3 !1=3
r 4p Q
1 6 4 dz dz 5 w p s jcr
1: 7:2:140 1:31 ;
1 2 1 1 0 s L
0 1
!1=3
For given values of the parameter , one can solve this
r 4p Q
p s jcr
vs javg 1:31 _ s ; 7:2:145
equation numerically for either a membrane tube or s L
a membrane channel and describe how varies
with z . Since a nondimensional expression for the where we have employed relations (7.2.133), (7.2.118) and
filtration flux may be developed from (7.2.107) and (7.2.127); _ is the value of the shear rate in the boundary
(7.2.138) as layer. The averaged filtration flux is proportional to the
shear rate in the boundary layer; it is independent of the
1
v
s ; 7:2:141 pressure drop applied. However, in the case of membrane
1
resistance control,
vs vs0 / P; 7:2:146
the length-averaged filtration flux, vs javg , is obtained as
z i.e. the averaged filtration flux is proportional to the
1 vs dz
v
s javg z : 7:2:142 applied P.
z 0 1
We will now illustrate graphically the behavior of the
Analytical expressions may be developed for the following length-averaged flux and the cake layer thickness with
limiting cases (Davis, 1992): distance along the membrane length. In Figure 7.2.6(c),
the filtration flux profile along the membrane length has
Membrane resistance controls
been shown for a given set of conditions. The flux is
constant till z zcr, after which it decreases steadily. In

1=9
v
s 1; j 1z ; j 1z 1=6 <<1;z  1 :
<< 1 tube channel Figure 7.2.6(d), the nondimensional filtration flux profile
along the nondimensional membrane length has been
7:2:143a
illustrated parametrically for as a parameter; it shows
Filtration flux is constant at vs0 along the whole length how the filtration flux decreases as the cake resistance
(Figure 7.2.6(d)). For large values of z (z  1), c is large increases, relative to the membrane resistance. This
and becomes a significant part of the membrane tube decrease is due to increasing cake thickness along the
radius or channel height (Figure 7.2.6(e)). membrane length for nonzero values of , i.e. R ^ c , as
shown in Figure 7.2.6(e).
Cake resistance controls (>> 1, z  1)
We observed earlier in dead-end cake filtration (equa-
" 1=3 #   1=3 tion (6.3.135k)) that R ^ c varies inversely with the square of
1 3z 1 3z 1
1 ; vs : the particle radius; therefore, in effect, the filtration flux vs
2 2 2 2
varies with the square of the particle radius for cake dom-
7:2:143b
inated filtration. The larger the particle radius, the higher

Filtration flux decreases significantly as z increases; c the filtration flux. As shown in equation (7.2.145), in cross-
increases with z, but slowly, and its magnitude remains flow microfiltration also the averaged filtration flux
small. The average flux over a length L of the membrane is increases with particle radius, here as (rp)4/3. Romero and
obtained from (7.2.142) by using (7.2.143a, b). For >> 1, Davis (1988) have shown via calculations based on their
and z (L/zcr) >> 1 with the cake resistance controlling, integral model that, for suspensions having particles larger
we can obtain than 4.3 m radius, the flux of solvent will correspond to
7.2 Crossflow membrane separations, granular filtration 581

that with zero cake resistance, i.e. the flux will correspond !1=3

r 4p Q
p s jcr
to that of a clean membrane. However, the shear rate is vs javg 1:31 _ s ;
s L
also important. Unless the shear rate is around 660 s1, the
flux for a suspension of particles of 5 m radius was shown w 10 g=cm-s2
by Romero and Davis (1988) to be less than that of a clean _ s 103 s 1 ;
0 10 2 g=cm-s
filter.17 An additional quantity to be taken into account is
the deformability of the particles, especially valid for cellu- r p 0:35  10 4 cm; s 0:005; L 40 cm: From equation
(7.2.147b),
lar suspensions, e.g. red blood cells in blood; the cake
resistance will be much higher and the cake will be more Q 5
p s jcr 9:79  10 1 4:38  0:005
compact. 9:79  10 5  0:979;
One would now like to determine the length-averaged
flux for a given membrane undergoing microfiltration from equation (7.2.131b),
under given conditions. Although we have the nondimen-  
0:58 0:13  0:005 2
sional expressions (7.2.143a) for the solvent flux for the s 1:01;
0:58 0:005
membrane dominated case and (7.2.143b) for the cake 0 11=3
resistance controlled situation, determination of the 3 3:54 1020 9:790:979105 A
vs javg 1:3110 1:01 @
length-averaged flux, as well as the local flux for the latter, 0:00540
is complicated due to two quantities, Q p s jcr and zcr.
0 11=3
Knowledge of one will yield the other from relation
@1509:581025 A
(7.2.133). The quantity Q p s jcr has to be obtained from 1310 11867108 0:00011cm=s:
0:2
the double integral (7.2.130) for the limit of w max for
appropriate dependences of D
p and on . Davis We will now calculate the value of vs javg by assuming the
(1992) and Romero and Davis (1988) have provided results particle radius to be 4.3 m. We should not assume either
(1) and (2), and (3), respectively: membrane control or cake resistance control. However, that
5
will require an extensive numerical effort. We will use the
1 lim Q
p s jcr 9:79  10 ; 7:2:147a same formula as used earlier to estimate the effect of a
s !0
particle size increase:
5
2 Q
p s jcr 9:79  10 1 4:38 s ; for s  0:1; 0 14=3
7:2:147b 4:3 A cm 1:18  10 4 12:284=3
vs javg 1:18  10 4 @
" # 0:35 s
Q
p s jcr
3 1 3:8 s ; for s  0:2: 12:28  2:308  1:18  10 4
lim s !0 Q
p s jcr

7:2:147c ) vs javg 3:344  10 3 cm=s;

One can now estimate the required quantities, at least for which is close to the value of the clean filter flux.
s  0.2, which is a high enough particle volume fraction The above example illustrates clearly how large a flux
for suspensions. reduction can take place in microfiltration compared to the
Example 7.2.7 In a ceramic tubular microfilter of diameter flux in a clean filter. In general, when crossflow microfiltra-
0.30 cm and length 40 cm, a suspension having particles of tion is initiated, one observes immediately a sharp flux
radius 0.35 m flows; the suspension particle volume fraction decline with time. This immediate flux decline is due to
is 0.005. The value of the viscosity, 0, is 10 2 g/cm-s (1 the increase in the filtration resistance due to the buildup
centipoise) and the wall shear stress is 10 g/cm-s2. The of the cake layer on the membrane; this buildup occurs
observed flux for the microfilter without any particles is 3.5 within minutes to an hour from startup and can easily
 103 cm/s for an applied pressure difference of 105 kPa.
reduce the value of vs javg to one-tenth of the clean filter
The conditions are such that one can assume a cake resist-
flux value, or an even greater reduction. A brief introduc-
ance controlled operation. Make an estimate of what the
average filtration flux will be at 25  C. Estimate the same if tion to modeling such a transient flux behavior on its way
the particle radius were 4.3 m. to steady state has been provided by Davis (1992). There is
an additional flux decay over a much longer period of time,
Solution Since we can assume cake resistance control, e.g. days, which takes place due to slow processes of mem-
we will employ equation (7.2.145) for the average filtration
brane fouling, compaction of the cake under the applied
flux:
P and membrane compaction.
Such losses of membrane solvent flux is avoided now a
days in practice by what is known as backflushing. The cake
17
As the particle size becomes larger, the phenomenon of inertial lift layer on the membrane is built up by the applied P over a
(Green and Belfort, 1980) becomes important. period of time as the liquid flows perpendicular to this
582 Bulk flow perpendicular to the direction of force

1.0 (c)
0.9
0.8
ns 0.7
ns 0 0.6
0.5
0.4
0.3
(b)
0.2
0.1 (a)

0
1000 2000 3000

Time (seconds)

Figure 7.2.7. Filtration flux decay with time after starting and the effect of backflushing: (a) no backflushing; (b) slow backflushing
introduced after some time (low frequency); (c) rapid backflushing with high frequency initiated immediately after starting. (After Bhave
(1991).)

force direction. If the force direction can be reversed, the membrane is unlikely to experience fouling. Thus the value
cake layer may be completely lifted off the membrane and of applied P should be reduced and operation continued
swept away by the tangential flow. The backflushing mode at a subcritical flux level to avoid membrane fouling (Field
is based on this strategy. Every so often, the permeate side et al., 1995; Howell, 1995).
pressure is suddenly increased to above the feed pressure.
A small amount of earlier permeated filtrate is forced back 7.2.1.4.1 crossflow microfiltrationoperational config-
into the feed side (and is therefore lost from the permeate): urations In Section 6.3, we observed in many examples
as it comes, it lifts off the cake and cleans the membrane that the magnitude of the bulk flow velocity which was
pores as well if there is some membrane fouling. The parallel to the direction of the force could not be arbitrarily
tangential flow on the feed side sweeps off the dislodged increased, otherwise the separation achieved could be
debris from the destruction of the cake layer. Figure 7.2.7 severely damaged or the separation process halted. Fur-
illustrates this behavior. Curve (a) illustrates the rapid ther, in many processes, there was unsteadiness due to this
flux decay during the first 30 minutes to 1 hour of conven- bulk flow vs. force configuration. Clearly, in crossflow
tional crossflow microfiltration. Curve (b) illustrates the microfiltration we have overcome this unsteadiness. Fur-
flux behavior for low values of backflush frequency started ther, in order to maintain a thin caker layer, we maintain a
after some time. Curve (c) illustrates the flux for a high high wall shear rate in the tangential flow field. In most
backflush frequency initiated after a very short interval, as crossflow microfiltration devices18 this means that the bulk
shown in the figure. Experience indicates that it is neces- flow velocity in the tangential flow direction (z) is quite
sary to introduce rapid backflushing if one wants to obtain high; on the other hand, the filtration velocity continues to
high flux, and further that it must be introduced immedi- be limited by the membrane, the cake or both. Thus, the
ately after the flow is introduced (Bhave, 1991). ratio (vz/vs) in crossflow microfiltration is high, unlike that
Recent research into the reduction of filtration flux that in dead-end microfiltration, where its value is 1. In add-
occurs as soon as the microfiltration process is started ition, in the latter, vs is likely to be significantly smaller than
(Figure 7.2.7(a)) indicates that it is very much dependent that in crossflow microfiltration due to a larger cake thick-
on the level of the flux and therefore the applied P. ness; also, in the dead-end mode, with time, the flux
Beyond a critical flux, vs,cr, deposits form on the membrane decreases at constant P.
since the forces dragging the colloidal particles in suspen-
sion toward the membrane are larger than the forces caus- 18
In those devices where the tangential shear rate can be generated
ing the particles to move away from the membrane. If this by means other than bulk motion, namely Dean vortices or Taylor
critical flux value is reached at the end of the membrane vortices, the correlation between shear rate and the bulk velocity may
channel, where the boundary layer is the thickest, then the be weak or zero.
7.2 Crossflow membrane separations, granular filtration 583

(a) (b)

(c)
fs z=L
Q bleedfs z=L

Qrecy
Qfiltrate

Qf fs z=0

fs Q tank
tank

Figure 7.2.8. Operational configurations of microfiltration: (a) batch concentration open system; (b) batch concentration closed
system; (c) feed and bleed operation in the continuous mode.

A result of this high bulk flow velocity and high shear V s tjt0 s tjt0 V s t s t; 7:2:149
rate in crossflow microfiltration is that the suspension
circulation rate is high when compared with the microfil- since all solid particles are assumed to be rejected by the
tration rate through the membrane. Either one provides a membrane.
very long membrane device or one keeps on circulating the One feature of the closed system described above is
solution from a reservoir/tank over a much smaller length a rapid increase in s in the recirculation loop. One
of the membrane for an extended length of time. The latter could avoid it by continually bleeding off a small portion
is the most common mode of crossflow microfilter oper- of the retentate/concentrate, as shown in Figure 7.2.8(c),
ation and is shown in Figure 7.2.8. This mode of batch in a method called feed and bleed. This bleed volume is
concentration can be carried out in two ways: open system small since the increase in concentration of s per pass
and closed system (Bhave, 1991). In an open system, the through the filter is also small. If we indicate by s jz0
concentrate from the module is sent back to the reservoir and s jzL the values of s at the inlet and outlet of the
for recirculation through a single pump; in a closed system, microfilter, respectively, then, for a totally rejecting
there are two pumps, and the energy loss associated with filter,
bringing the liquid suspension from the tank to full pres- Qf s jz0 Q2 s jzL ; 7:2:150
sure is avoided. Mass balance considerations for a suspen-
sion volume of Vs(t) having a particle volume fraction s(t) where Qf and Q2 are the volumetric suspension flow rates
being microfiltered through a membrane area Am leads to at the filter inlet and the concentrate end, respectively. The
flow rate ratio Qf/Q2 (>1) is close to 1 since the recircula-
t
tion rates are high; therefore s jzL = s jz0 is also close to 1
V s tjt0 V s tjt Am vs javg dt; 7:2:148
0 (but >1). However, this flow rate Qf into the filter consists
of the sum of the flow rate Qtank out of the tank and the
where vs javg for the membrane of area Am changes with
flow being recirculated Qrecy ( Q2), the latter being R
time since the suspension particle volume fraction s(t) is
times the bleed flow rate Qbleed:
time-dependent, as the suspension volume is reduced with
time. The corresponding increase in the volume fraction of Qf Qtank Qrecy Qtank R Qbleed ;
particles in suspension from s tjt0 to s t at any time t Qf s jz0 Qtank s jtank Qrecy s jzL :
is related to the suspension volume reduction by 7:2:151
584 Bulk flow perpendicular to the direction of force

From the second equation, one can obtain Filter cake

s jz0 Qtank =Qf s jtank R Qbleed =Qf s jzL : 7:2:152


w
However, to obtain steady state operation, the increase in Wash nozzles
Rdrum
solids volume flow rate due to filtration Qf Q2 s jz0
must not affect steady state operation:
Dry
qw
volume flow rate balance : Qf Qrecy Qbleed Qfiltrate ; Drum
Vacuum
Vacuum
7:2:153
Vacuum
The particle volume fraction balance around the filter is Filter cake
discharge q qf Filter cake
given by
Filter cloth
Qf s jz0 Qrecy s jz0 Qbleed s jz0 Qfiltrate s jz0 Filtrate

Qrecy s jzL Qbleed s jzL :


7:2:154
Suspension
Rearrange the last equation to obtain

Qfiltrate s jz0 Qf Q2 s jz0 Figure 7.2.9. Schematic of a rotary vacuum filter.

Qbleed Qrecy s jzL s jz0 ;

Qf Q2 j Q any valuable product left in the cake via filtration into the
s zL 1 filtrate ; filtrate inside the drum operating under partial vacuum.
Qbleed Qrecy s jz0 Q2
Only a section of the drum circumference is exposed to this
s jzL Q Qf cake washing. The next environment encountered by the
1 filtrate ; 7:2:155
s jz0 Q2 Q2 cake on the rotating drum surface (Figure 7.2.9) is essen-
tially dry air. The vacuum inside the drum continues to
where the ratio on the left-hand side is also called the
remove any remaining liquid in the cake on the filter
volume concentration factor (VCF). When the desired
medium. The fourth and final step in this cyclic process
VCF value is higher, then the bleed stream from the first
involves removal of the filter cake formed on the filter
stage could be used as if it were the feed stream from the
medium via a knife blade.
tank for the second stage in a multistage arrangement.
In conventional application, no coating is applied to
the filter medium. In the filtration of fermentation broths,
7.2.1.5 Rotary vacuum filtration
however, a thin but controlled layer of a filter aid, e.g.
Consider Figure 7.2.9 which illustrates schematically a diatomaceous earth, is applied onto the filter medium
drum rotating around its axis. The drum length (not before the drum enters the suspension as a precoat.
shown) is L. As the drum rotates at an angular velocity of The filtration of the fermentation broth leads to a buildup
radian/s, part of the outside of the drum remains sub- of the cake over it. When the knife blade is applied at the
merged in the suspension to be filtered. The drum surface end of one cycle, most often only a particular fraction of
has a filter medium exposed to the suspension at atmos- this precoat of filter aid is removed. The porous precoat
pheric pressure. Inside the porous drum surface, a partial facilitates extended treatment of the broth, which tends to
vacuum is maintained so that the filter medium has a P create a complex compressible cake. If the filtrate contains
across it from the suspension. As filtration takes place, the desired and valuable material, the cake discharged by
particles are deposited on the outside of the filter medium the knife blade is waste; filter aid material added as a
surface. As a particular section of the drum/filter medium precoat contaminates the cake and increases the volume
rotates through the suspension bath, there is continuing of this waste. It also changes the nature of the waste, which
filtration; so there is a cake buildup on the filter medium will now contain, in addition to cellular debris, cells, etc.,
as it goes into the suspension and then comes out of the siliceous material.
suspension after some time. During this movement We have already pointed out in the introduction to
through the suspension, the partial vacuum applied brings Section 7.2 that the separation configuration of this rotary
out filtrate into the inside of the drum. vacuum filtration technique corresponding to Figure
As this section of the drum comes out of the suspen- 6.3.25(a) may be conceived of as a crossflow filtration tech-
sion, nozzles on the outside spray clean liquid onto the nique (Figure 6.3.25(b)) if we fix the coordinate system to
drum filter medium surface and wash out any solution the filter medium/drum surface. In this configuration, the
remaining in the cake built up on the filter. This removes suspension/slurry will be moving with the linear velocity of
7.2 Crossflow membrane separations, granular filtration 585

the drum surface where the filter medium is located. How- ^ c s


 1=2
2R

ever, this velocity, and the corresponding tangential shear vs ; 7:2:160
c s P
stress magnitude, are rather low and are incapable of
affecting the cake thickness buildup (unlike that in crossflow f

microfiltration studied in Section 7.2.1.4) (Davis and Grant, Qfiltrate Rdrum L vs d


1992). Therefore, one should consider the cake buildup with 0
91=2 7:2:161
rotation (as angle in Figure 7.2.9 increases) as if we have
8
<2 P =
c s f
dead-end filtration (Section 6.3.3.1), where (/) may Rdrum L :
^ c s
R
replace time, t. Correspondingly, one can employ expres-
: ;
sions developed earlier in dead-end cake filtration for the
time-dependent cake thickness c(t) (6.3.138c) and the filtra- However, since the cakes developed in rotary vacuum
tion rate/volume flux for constant P, vs(t), (6.3.138d) and filtration are most often compressible, we employ expres-
^ c for a compressible cake to obtain
sion (6.3.138j) for R
write here (Davis and Grant, 1992)
)1=2
2 c s P1 s
(
^ c s P 1=2
" #
2 R

^ c 1
c Rm =R 1 ; 7:2:156 Qfiltrate Rdrum L f : 7:2:162
c s R2m s s c 0 f

^ c s P
 1=2 If tcycle is the time taken by the drum to go through one
2R

P
vs 1 : 7:2:157 rotation, then the fraction of the time the drum is in
Rm c s R2m
contact with the suspension is given by
Note that here Rm includes the resistance due to any pre-
t contact f f f
coat applied to the membrane. ; t contact t cycle : 7:2:163
t cycle 2 2
The question of interest is: what is the volumetric rate
of filtration in this rotating drum (of total surface area 2 Correspondingly, the drum surface area in contact with the
RdrumL) due to the submerged fraction of its surface? That suspension is Rdrum L f. Therefore, from (7.2.162), the
will yield the device filtration rate (unless the suspension filtrate production rate, Qfiltrate, is linearly proportional to
properties change with time they will for a batch Rdrum L f as well as to (tcontact) 1/2. Further, in expression
operation!) since, as long as the drum rotates, part of it is (7.2.162) for dilute suspensions (s << 1), {c s)/c} is ~1
always carrying out filtration (fractional surface area sub- and s s is the mass concentration of particles in suspen-
merged, f/2; total surface area submerged at any time, sion (g/cm3 or g/liter of suspension).
LRdrum f). This quantity may be obtained by integrating We will briefly consider now the step of cake washing.
expression (7.2.157) for the filtration flux over the range of This step is undertaken to recover the solutes left in the
angles 0 to f, the angle of contact with the suspension solution trapped in the cake mass. As the wash liquid
(over the time period the surface area of the drum is in displaces this trapped solution, the latter goes through
contact with the suspension): the filter medium and is recovered in the filtrate inside
f the drum. Therefore, washing enhances the recovery of
Qfiltrate Rdrum L vs d soluble fermentation products left in the liquid in the cake
0 mass in the case of a fermentation broth. Since the wash
f
2R ^ c s P 1=2 liquid is suspension-free as well as solute-free, the cake
 
P
Rdrum L 1 d;
0 Rm c s R2m filtration characteristics are unlikely to be altered. Hence,
7:2:158 the filtration rate during washing should correspond to the
filtration rate existing at the location where the drum came
(
^ c s P f  1=2 )
Rdrum Lc s Rm 2R out of the suspension:
Qfiltrate 1 1 :
^ c s
R c s R2m 
2R^ c s f  1=2
7:2:159 vs f : 7:2:164
c s P
If the cake resistance is much larger than the membrane
Correspondingly, the overall filtration rate during washing is
^ c >> Rm ; correspondingly,
( precoat) resistance, then R
we have already observed in Example 6.3.4 that 2 3 1=2
 1=2
w
2 ^ c s f
R
^ c s t
2R Qfiltrate=washing Rdrum L d

4 5
vs t : 0 c s P
c s P
2 3 1=2 7:2:165
We follow now the same procedure we followed earlier to 2 ^ c s f
R
obtain expression (7.2.159) for Qfiltrate from expression Rdrum Lw 4 5 :
c s P
(7.2.157) for vs():
586 Bulk flow perpendicular to the direction of force

Introducing the expression (6.3.138j) for R ^ c , we get 7.1.1.1, other conditions remaining constant, the purifica-
" #1=2 tion rate decreases with time. Further, the pressure drop
c s P1 s across the filter increases with time for a given filtration
Qfiltrate=washing Rdrum L w :
2 s c s 0 f rate. As in Section 7.1.1.1, the granular sand bed/media has
7:2:166 to be regenerated after some time to reverse filter clogging.
The regeneration is carried out by backwash: liquid/water
The volume of the wash liquid employed has been found
is brought in the bottom of the bed at a high rate and is
to be a factor in the extent of recovery of soluble materials.
made to flow up while the bed is expanded substantially.
The fraction of solute remaining after the wash has been
We have already been introduced to aerosol removal
related to the washing efficiency, wash , by Choudhury and
from gas/air streams via depth filtration in a fiber-filled bed
Dahlstrom (1957) via the following expression:
in Section 6.3.1.4 (see Figures 6.3.8, 6.3.9A and 6.3.9B). The
solute fraction remaining 1 wash vr ; 7:2:167 general technique is identified as fibrous filtration, where
where vr volume of wash liquid employed divided by the surface filtration is also possible. For cases where surface
volume of liquid left in the unwashed cake (i.e. a volume filtration does not exist in fibrous filtration, the process is
ratio), and wash washing efficiency. When vr 1, the very close to granular filtration of hydrosols. The only
solute fraction remaining is (1 washing efficiency); the differences are as follows: the fibrous bed is highly porous
value of vr in practice can vary up to 5. The value of wash compared to granular filtration beds; the bed pressure
has been found to go up to 0.86 by Choudhury and Dahl- drop is lower; the medium consists of fibers/cylinders as
strom (1957). A semilog plot of the solute fraction remaining opposed to granules/spheres used in granular filtration;
against vr yields a slope of log(1 wash ). Belter et al. (1988) the fibrous bed loaded with captured aerosol materials
have illustrated data for the extraction of the antibiotic cannot be regenerated easily (unlike that in granular filtra-
lincomycin from a cake of bacteria at two values of pH which tion) (Tien, 1989). We will focus primarily in this section on
appears to influence the extraction efficiency significantly. the granular filtration of hydrosols.
There exists a simple relation between Qfiltrate and We will briefly describe the filtration behavior of
Qfiltrate/washing and f and w (alternatively tcontact (7.2.163) hydrosols in a granular filter bed, but first, it is useful to
and the time for washing twash). Relations (7.2.162) and conceptualize the structure of the granular medium. The
(7.2.166) yield directly bed is assumed to consist of a large number of unit bed
elements (UBEs) in series (Tien, 1989). Each UBE has a
Qfiltrate f t contact
2 2 certain type of flow channel, and the granular medium
Qfiltrate=washing w t wash surface acts as a particle collector, collecting particles from
w Qfiltrate=washing V filtrate=washing the fluid flowing in the channel. The porous medium in
) 2 2
f Qfiltrate V filtrate each UBE may be represented in a number of ways.
w V filtrate=washing V retained t wash (1) The porous medium may be assumed to consist of a
) 2 ;
f V retained V filtrate t contact bundle of straight capillaries of equal size (capillaric
7:2:168 model). A particle follows a certain trajectory and is
captured when it hits the wall.
t wash
2vr V retained =V filtrate : 7:2:169 (2) If each granule is assumed to be a sphere, the porous
t contact
medium is a collection of spherical collectors collecting
For fixed tcontact, the variation of twash plotted against (vr) the particles as the liquid flows by (spherical collector
will yield a straight line whose slope is 2(Vretained/Vfiltrate). model).
(3) As the liquid flows through the granular medium, the
7.2.2 Granular filtration of hydrosols (and aerosols) flow may be described as the combination of flow
through the large void spaces followed by flow through
Sand filters are used extensively in large-scale water treat-
the constricted region in the area of contact between
ment and wastewater treatment. In such a filtration pro-
neighboring granules (constricted tube model). This
cess, the water to be treated flows very slowly down a filter
model appears to be capable of describing filter clog-
bed, which is generally 1 m deep and is made out of sand
ging more efficiently (Tien, 1989).
grains of diameter varying between 0.6 and 2 mm. The
pressure drop encountered by purified water leaving the Very detailed models of granular filtration have been
bottom of the bed varies between 2.5 and 10 meters of H2O. developed using such idealizations of the porous medium
Particulate contaminants/impurities are collected in the (Tien, 1989). A simple aspect of this filtration is that, as the
granular filtration media in the manner shown in Figure liquid flows down the porous medium, particles are trans-
6.3.8 for depth filtration. The word clarification is used for ported perpendicular to the flow by electrostatic force,
liquid feeds containing suspended materials/particulates, Brownian diffusion, various surface interaction forces and
namely hydrosols. Just like the adsorbent bed in Section interception. Gravitational force is also present: its
7.2 Crossflow membrane separations, granular filtration 587

(a) (b)
C ivp
Liquid flow
streamlines

Gravitational
Inertial impaction
settling z

Diffusion

L
Interception

R
Spherical
collector Electrostatic
deposition
Cvp z=L

Figure 7.2.10. Fixed-bed granular filtration: (a) various mechanisms of particle capture by a spherical collector granule (after Davis
(2001)); (b) granular packed-bed schematic for filtration.

direction is same as that of overall bulk flow; but, as the p 4=3r 3p dr p to arrive at equation (6.2.55), the De term
particle follows a curved streamline, it falls vertically down- will become Dep 4=3r 3p dr p . From the description of
ward and therefore at an angle to the local liquid flow. We the De-containing term in (6.2.50h), we conclude that
have already observed in Figures 6.3.9A and 6.3.9B how Dep 4=3r 3p dr p will represent the mass rate of disap-
inertial impaction can bring particles to be captured for the pearance of particles of size rp per unit filter volume; we
case of flow parallel to force in gas flow through a fibrous will represent it as (De)mv. Further, for incompressible
filter. In granular filtration, inertial impaction exists as flow, if we assume that all particles are of the same size
discussed in Section 6.3.1.4; but so do a number of other rp, then we can rewrite equation (6.2.55) after a control
forces/mechanisms just mentioned (see Figure 7.2.10(a)). volume analysis in the manner of equation (6.2.50i) as
follows (with Dp replaced by D reflecting dispersion
We will now provide a brief introduction to the model- p;eff
ing of the dynamic behavior of the fixed bed of granules as effects and where p;r p identifies the particle mass concen-
water containing a small concentration of particles (in the tration in the fluid as opposed to the total control volume
range of 100 parts per million (ppm) to 1% by weight) flows used earlier):
down the granular bed. Our simplified treatment will be
p;rp
guided by the comprehensive account to granular filtration v r p;rp r U p p;r p
t
of hydrosols and (aerosols) by Tien (1989).
D r2 p;rp Demv ; 7:2:170a
Consider a granular filter bed (of radius R, length L and p;eff

porosity ) through which a fluid (here, water) flows, as where is the porosity of the bed at any location. We need
shown in Figure 7.2.10(b). The superficial velocity of the to ignore the third term on the left-hand side since any
fluid (through the cross section of the empty bed) is v0. The particle-specific velocity leading to particle capture or
actual interstitial velocities of the fluid in the three coordin- otherwise is lumped into the (De)mv term. In addition, in
ate directions (r, , z) are vr, v and vz, respectively. Due to general for aerosols/hydrosols, particle velocity and fluid
radial symmetry, v 0. As in Section 7.1.1.1, we can velocity may be assumed to be identical for the overall
assume that there are no radial variations in this filter filtration behavior of interest here. (When an actual particle
bed where R >>> dgr, the average diameter of the grains trajectory model is considered, this assumption is invalid.)
in the bed. Although locally around a granule there may be We can now rewrite equation (7.2.170a) as
a vr component of velocity, it is not considered here in the
p;rp
lumped model of the overall bed performance. We will v r p;rp D r2 p;rp Demv :
t p;eff
now use the general dynamic equation (6.2.51c) in the
form of equation (6.2.55) for transport of particles in 7:2:170b
flowing fluidsconvective diffusion; however, we will retain If we ignore any dispersion effects and employ only the
the term De in equation (6.2.51c), while B 0. Here De will interstitial velocity in the z-direction, vz (vr, v not being
be used to indicate the disappearance of particles from the relevant), we get the time-dependent z-directional equa-
fluid. Since each term in (6.2.51c) is multiplied by tion for particle mass concentration, p;rp , in the liquid:
588 Bulk flow perpendicular to the direction of force

p;r p p;r p C vp C vp v
vz Demv 0; 7:2:171 vz 0: 7:2:178
t z z t t

where vz v0, the superficial velocity through the bed, The two independent variables here are z and t. If we
and p;r p is the particle mass concentration in the flowing introduce the transformation
liquid.
As depth filtration continues, particles are deposited on z
dz
the filter-bed granules. The filter-bed porosity decreases t 7:2:179a
vz
from the porosity 0 of the clean filter (the initial porosity at 0

time t 0); also, the pressure drop for fluid flow through the and change the independent variables to z and , then,
bed increases, due to a reduction in flow cross-sectional since
area as well as increased form drag from the changes in
C vp
   
the geometrical shape and dimensions of the granules. The C vp
dC vp dz d; 7:2:179b
pressure gradient in the axial direction increases from the z z
initial value (P/z)0 to (P/z). As time increases, keeps
we get
on decreasing and (P/z) keeps on increasing. Granular
bed filtration is therefore an inherently unsteady state oper- C vp C vp
      
C vp

ation. To facilitate the development of a solution of the z t z z z t
problem, some relations have to be developed between C vp 1
   
C vp
the time-dependent decrease of from 0, the time- : 7:2:179c
z z vz
dependent increase of (P/z) and the term (De)mv.
We identify v as the volume of particles deposited per Further,
unit filter volume, a specific volume based deposit. Then 
C vp
 
C vp
 
v
 
v

; :
t z z t z z
v
Demv p ; 7:2:172 7:2:179d
t
Therefore the balance equation (7.2.178) is reduced to
where p is the actual density of the particle material.
Further, the parameters and variables that influence this
       
C vp C vp C vp v
vz C vp 0;
may be described as z z z z
v 7:2:180
Demv p p f f ; p;rp ; v ; 7:2:173
t where, if we introduce expression (7.2.177) for , we get
where f is a parameter. Similarly, v will also influence the 
C vp
 
C vp

v
increase in the pressure gradient with time: vz 1 0: 7:2:181
z 1 d
P=z=P=z0 Gr f ; v ;

7:2:174
If Cvp << 1, this equation is reduced to
where f is a parameter. C vp v
Introducing the definition (7.2.172) into the particle vz 0: 7:2:182
z
mass concentration balance equation (7.2.171), we get
This equation, along with equation (7.2.177) for , relation
p;rp p;rp v (7.2.174) for the increase in pressure gradient and expres-
vz p 0: 7:2:175
z t t sion (7.2.173) for the time-dependent increase of v ,
govern the granular filtration process subject to the
We will now introduce a new dependent variable, Cvp, the
following initial and boundary conditions:
volume concentration of particles, i.e. the volume of par-
ticles per unit volume of the fluid: for  0; z  0; C vp 0; v 0; 7:2:183a
C vp p;rp =p : 7:2:176 for > 0; at z 0; C vp C ivp ; 7:2:183b

Further, if d is the porosity of the particle deposit on the where C ivp is the volume concentration of the particles in
filter media, then the porosity of the filter medium at any the liquid stream entering the filter.
time is related to v and the initial porosity 0 of the clean Solution of these equations requires knowledge of two
filter by unknown functions, f(f, p;rp , v), (relation (7.2.173)) and
0 v =1 d : 7:2:177 Gr(f, v) (relation (7.2.174)). A variety of models have been
developed; these have been summarized in Tien (1989).
We can now rewrite the balance equation (7.2.175) as The solution has to be carried out numerically (in general).
7.2 Crossflow membrane separations, granular filtration 589

However, since we have the v = term in equation f f ; v 1 a v ; 7:2:191


(7.2.182), which via postulate (7.2.173), is related to
f (f, p;r p , v) or f1(f, Cvp, v) (obtained via substitution of where a is a positive constant. Substituting this form of f
p;rp into f(f, p;rp , v) by Cvp via (7.2.176)), we need to have into the filtration rate expression (7.2.188) applied to loca-
this functional dependence. We will now briefly introduce tion z 0 leads to
a few expressions for f1(f, Cvp, v).  i 
d v = 1 a iv

vz 0 C ivp d : 7:2:192
Experimental observations during the initial filtration
period with a clean filter indicate that the volume con- On integration, using the value iv 0 at 0, we get
centration of particles in the liquid follow a logarithmic
law, i.e. the particle concentration decreases exponen- 1 a iv exp vz 0 C ivp a : 7:2:193
tially with z: From equation (7.2.190), we can now write

C vp =C ivp exp z; 7:2:184 d v = v 1 a v 0 dz:
which leads to Integrating and using the value of v at z 0, namely iv , we get
 
C vp =z C vp : 7:2:185
v 1 a iv
exp 0 z: 7:2:194
From equation (7.2.182), we get iv 1 a v

v = vz C vp ; 7:2:186 From the result (7.2.189) developed by Herzig et al. (1970),


we now obtain
which suggests that the time rate of filtration per unit filter
volume is proportional to the volume concentration of
!
C vp 1 a iv
particles in the liquid, a first-order behavior. Here , known i
 exp 0 z: 7:2:195
C vp 1 a i C vp =C i
as the filter coefficient, has the dimension of (length)1, as v vp

the dimensional consideration of equation (7.2.186)



We can now solve for C vp =C ivp from this equation:
indicates:
!

volume
 
length
 
volume
 
volume
 C vp exp 0 z 1
vz C vp :
v
volume time

volume volume C ivp 1 a iv a iv exp 0 z a iv 1 a iv e0 z
:
time 1 7:2:196

However, it is also known that the filter coefficient varies If we substitute into this relation expression (7.2.193) for
with time and may be expressed as a iv in terms of C ivp and , we get
0 f f ; v ; 7:2:187 C vp 1
:
C ivp 1expvz 0 C ivp a exp 0 zexpvz 0 C ivp a
where f f ; v 1 when v 0. Therefore the filtration
rate expression (7.2.186) becomes Rearranging,
v = vz 0 f f ; v C vp : 7:2:188 C vp exp vz 0 C ivp a
: 7:2:197
Models that have been developed employ different func- C ivp exp 0 z exp vz 0 C ivp a 1
tional dependences of f f ; v on v and other (adjust-
This expression of the volume concentration of particles in the
able) filter parameters. Herzig et al. (1970) have in
liquid phase as a function of time variable, , the location in
addition, developed the following results from an extended
the bed, z, and the incoming fluid-phase particle concentra-
analysis of particle conservation:
tion, C ivp , is a useful result illustrative of the modeling efforts.
C vp =C ivp v = iv ; 7:2:189 In general, numerical solutions are carried out. However, a
few parameters are needed; these parameters are estimated
v =z 0 f f ; v v ; 7:2:190 from experimental data by a complex procedure which has
been summarized in Tien (1989). The following details from
where iv is the value of v at z 0 (filter inlet) (it depends
an example illustrated in Tien (1989) are illustrative. It is
on , time); the corresponding value of Cvp is C ivp .
One can now postulate specific functional forms of based on the following assumed functional dependences of
f f ; v and develop an expression for Cvp, the particle f (defined by (7.2.187)) and Gr (defined by (7.2.174)):
volume concentration in the liquid phase. For example, 1
f ; 7:2:198a
if particle deposition results primarily in filter clogging, 1 a1 v 2
then Ornatski et al. (1955) (as quoted in Tien (1989))
postulate that Gr 1 b1 : 7:2:198b
590 Bulk flow perpendicular to the direction of force

1.0 Figure 7.2.11. (a) suspension concentration profile along the bed
as a function of time; (b) pressure drop behavior; (c) specific
C vp 0.8 particle deposit growth with time along the bed.
C ivp 0.6 Increasing t

0.4

0.2

0
Bed depth, z L
(a)
Pressure drop, P

Increasing t

Increasing t

sv
Clean
bed
t=0

Bed depth, z L Bed depth, z L

(b) (c)

The filter bed consists of granular activated carbon (GAC) wastewater treatment from Tien (1989) (we have identified
of the DARCO 20  40 brand with a mean grain diameter some of these at the beginning of this section); see
(dgr) of 0.594 mm. The filter bed length (L) is 1.3 m and the Table 7.2.3.
diameter is 7.6 cm. The clean filter void volume fraction
Example 7.2.8 Consider a filter bed operating under the
0 is 0.49. Suspended clay particles of diameter 440 m
conditions described in the example from Tien (1989) given
were introduced at a concentration of C ivp 88  previously. Specifically, the conditions are as follows:
10 6 cm3 =cm3 . The superficial velocity of water, C ivp 88  10 6 cm3 =cm3 ; 0 0.49; 0vz v0 0.138 cm/
v0 vz , is 1.38 mm/s. A parameter estimation method s; 0 1.02  102 mm1 1.02  101 cm1. However, the
led to the following values: 0 1.02  102 mm1; b1 function f may be described for lower levels of deposit via
464; a1 30.46. The value of (P/z)0 corresponding to the Ornatski expression f f ; v 1 a v , where the value
time t 0 is 3.36  102 atm/m. The suspension concen- of a has been found to be 42. Determine the value of
tration distribution in terms of Cvp as a function of the bed C vp =C ivp for t 2 hours at bed locations of z 20 cm and
depth was found to agree well with measurements for 100 cm. Compare with the values 0.24 and 0.05 obtained
experimentally.
time periods varying between 1 hr and 16 hrs. The behav-
ior is shown schematically in Figure 7.2.11(a). The Solution We will employ expression (7.2.197) for C vp =C ivp :
increase in pressure drop along the bed length (from the
C vp exp vz 0 C ivp a
bed inlet to a specific location in the bed) with time is i
:
illustrated in Figure 7.2.11(b), where the straight line at C vp exp 0 z exp vz 0 C ivp a
the bottom for t 0 represents the pressure drop through From definition (7.2.179a),
a clean filter. One can see that the pressure drop is much
z
larger in the initial sections of the filter, which collect dz 0
t t z;
much more of the particulate deposit. These types of vz v0
0
behavior have been experimentally verified (Tien, 1989).
Figure 7.2.11(c) shows the growth in v , the specific par- where we have assumed that, for the short time period,
ticle volume based deposit with time across the filter-bed vz v0 = may be approximated by v0 =0 . For time t 2
hours 7200 s, the values of corresponding to z 20 and
length, as obtained from the solution of the equations
100 cm are, respectively,
considered so far.
It is useful to summarize the common operating 0:49
jz20 7200  20cm 720071s 7129s;
conditions in granular filtration in drinking water and 0:138cm=s
7.2 Crossflow membrane separations, granular filtration 591

Table 7.2.3.

Variables under consideration Drinking water treatment Wastewater (tertiary) treatment

Filter-bed depth 1m 1m
Filter-grain diameter 0.06 cm 0.2 cm
Filtration rate 5 m/hr 7.5 m/hr
Concentration of particles in feed water to be treated 50 mg/liter 30 mg/liter
Treated water quality 0.1 mg/liter 5 mg/liter
Pressure drop 2.510 m of water 2.510 m of water
Running time 10100 hr 20 hr
Backwashing water requirement 3% of throughput 3% of throughput

A minimum rate of 15 gal/ft2/ min over a 15 minute period is required. A rate of 20 gal/ft2-min to provide 50% bed expansion is recommended.

0:49 (2) collection of spherical collectors (Figure 7.2.12(b))


jz100 7200  100 cm
0:138 cm=s (spherical collector model);
7200 355 s 6945 s: (3) constricted tube collectors (Figure 7.2.12(c))
For z 20 cm,
(constricted tube model).
0 1
cm 1
exp @0:138  1:02  10 1  88  10 6  42  7129 sA
s cm
C vp

C ivp exp 1:02  10 1  20 exp 0:138  1:02  88  10 7  42  7129
exp 0:37 1:447
0:158:
exp 2:04 exp 0:37 7:69 1:447 The granular filter bed contains n UBEs in series with n !
. For a filter bed of length L, the length l of a UBE is
For z 100 cm,
therefore
 
6945
exp 0:37  n L=: 7:2:199
C vp 7129
 
C ivp 6945 The overall filter efficiency ET (see definition (6.3.45))
exp 10:2 exp 0:37 
7129 
exp 0:36 1:433 E T 1 C vp jzL =C ivp ; 7:2:200a
0:
24 000 exp 0:36 24 000 1:433
where C vp jzL denotes the volume concentration of par-
In the original method of solution, f f ; v
ticles at the filter exit for an incoming concentration of C ivp .
1=1 a1 v 2 . Replacing this by the linear function of the
Consider now the ith UBE: the influent particle concen-
Ornatski expression, 1 a v , leads to an element of under-
prediction. Further, the value of a provided could be tration (from the (i 1)th UBE above it), C vp ji 1 , enters this
improved. ith UBE at a distance zi from the top; the effluent concen-
tration leaving this ith UBE is C vp ji . The particle collection
efficiency Ei of the ith UBE is given by
7.2.2.1 Brief overview of mechanistic models in granular
filtration E i 1 C vp ji =C vp ji 1 :
 
7:2:200b
The granular bed filtration model presented so far employed Therefore
expressions for f , Gr, etc. (7.2.198a,b), which are empirical
n
in nature. A more fundamental approach, based on the
Y
ET 1 1 E i ; 7:2:201
operative mechanism, is desirable. Refer to Tien (1989) for i1
a detailed treatment. Here we are going to provide a brief
relating the particle collection efficiency E i of UBEs to the
introduction to a more fundamental approach. As indicated
overall filter efficiency E T .
in the introduction to Section 7.2.2, the granular filter bed,
We know from an empirical understanding of clean
which is a porous medium, may be thought of as consisting
filter behavior (see (7.2.185)) that
of a large number of UBEs in series. Three types of UBEs
were identified earlier as alternative representations of the C vp
C vp ;
porous medium that is the granular filter bed: z
(1) bundle of straight capillaries as particle collectors where is the time-dependent filter coefficient. If we focus
(Figure 7.2.12(a)) (capillaric model); on this behavior (and equation), obtain a general solution
592 Bulk flow perpendicular to the direction of force

(a) Liquid flow

Capillary collectors

Limiting
particle
trajectory

l l
Capillary
collectors

(b) Liquid flow

Spherical collectors
Fluid
envelope
around
spherical
particle

Spherical
Limiting collectors
trajectory

(c)
Constricted
Liquid flow tube between
filter grains
Constricted tube collectors

Limiting
trajectory
l

Figure 7.2.12. Three models of granular media filtration mechanism. (a) UBEs, of capillary tube collectors and limiting particle trajectory
of a particle in a capillary. (b) UBEs of spherical collectors and limiting particle trajectory around a spherical collector. (c) UBEs of
constricted tube collectors and limiting particle trajectory in a constricted tube. (After Tien (1989).)

C vp C vp jz0 exp z 7:2:202 C vp ji 1 =C vp ji exp z i zi 1 ) n C vp ji 1 =C vp ji ;


7:2:204
and write it for the two concentrations of relevance around
the ith UBE, C vp ji and C vp ji 1 , where (zi zi1) . Therefore, using relation (7.2.200b),
C vp ji 1 C vp jz0 exp z i 1 ; n1=1 E i =; 7:2:205
C vp ji C vp jz0 exp zi ; 7:2:203
the required relation providing an estimate of in terms of Ei
we get of the UBE and . In the limit of small values of Ei (Ei << 1),
7.2 Crossflow membrane separations, granular filtration 593

E i =: 7:2:206 Particles in suspension flowing through the granular


filter media are collected by individual granules; the col-
We will now provide an estimate of in terms of the filter- lection efficiency of an individual collector/granule (to be
bed granule dimensions and the bed porosity . Assume identified by EMechS, where the subscript Mech refers to
that the filter-bed granules are uniformly spherical par- the particular mechanism and subscript s refers to the
ticles of diameter dgr. Assume further that the filter may single collector) has to be related to the UBE collection
be considered to be a cube, the dimension of each side efficiency Ei defined by (7.2.200b). The incoming liquid,
of which is n, where there are n UBEs in series, each of flowing with a superficial velocity v0 (into any unit cell of
length (note that n ! ). Therefore (1 ) (n33) Happels model, say) and particle mass concentration of
volume of granules in the filter. Assuming one granule p;rp for monosized particles (of radius rp), brings in par-
per one cubic UBE of side , there are n3 granules in total ticles at a rate of b2 vo p;rp over the projected cross-
in the bed, so that sectional area b2 of Happels unit cell of radius b. If only
3  1=3 a fraction EMechS of these particles is captured by the
4 d gr
1 n3 3 n3  ) d gr : granule, then the rates of particle capture by a single
3 8 61
granule and by nc collectors are, respectively,
7:2:207
b2 v0 p;rp E Mech S and nc b2 v0 p;r p E Mech S . The total rate of
In general, granule diameters show a significant variation; particle introduction into the UBE of unit cross-sectional
therefore an average grain diameter should be used. The area is v0 p;rp . Therefore the ith UBE collection efficiency,
only unknown that is now required to estimate , the filter Ei, is given by
coefficient, is Ei, the particle collection efficiency of the ith
nc b2 v0 p;rp E Mech S
UBE. Ei ; 7:2:209c
In the development of an expression for Ei, a more v0 p;rp
detailed model of the porous medium is needed. Of the 2 32=3
three types of models of a granular filter as a porous 2 61 hdgr i2 E Mech S
E i nc b E Mech S 4 5
medium, the capillaric model is not preferred. For the hdgr i2 41 2=3
sake of simplicity, we will consider one of the other two, 1=3
namely the spherical collector model. In this model, the 62=3 E Mech S 1:209 E Mech S :
4
filter grain is assumed to be a sphere. There are a
number of alternative approaches based on a spherical 7:2:209d
collector. We will illustrate the approach by Happel
We will now briefly illustrate how to calculate E Mech S for a
(1958). In Happels model, the granular porous medium
given capture mechanism.
is assumed to consist of a large collection of identical
As we have seen in Section 6.3.1.4 on the removal of
cells, where each cell consists of a spherical particle of
particles from air by a fibrous bed via the mechanism of
radius (hdgri/2) (i.e. half of the average grain diameter)
inertial deposition, if one can locate the limiting trajec-
surrounded by a liquid envelope of radius b, such that
tory (dimension b in Figure 6.3.9A), the particle capture
the void volume of this cell is identical to the void
efficiency can be determined (e.g. definition (6.3.42a)).
volume of the porous medium:
Determination of the limiting trajectory is achieved via
" #
3 particle trajectory analysis in the porous medium, i.e. the
4 hd gr i 4 3
b 1 : 7:2:208 granular filter medium. The governing equation for par-
3 8 3
ticle motion in the inter-particle space is equation
Consequently, (6.2.45):

hd gr i=2b 1 1=3 : 7:2:209a dU p


F iner
p mp F ext drag
tp F p ; 7:2:210
dt
Further, in Happels cell model, there is no interaction
between the contiguous cells through the cell surface where Up depends on the x-, y- and z- coordinates of the
boundaries. The number of spherical collectors in a UBE, particle.
nc, is obtained as follows: For hydrosols in granular filtration, the external force
F ext
tp consists of gravitational force, particlecollector sur-
 
4 hd gr i 3 face interaction forces, such as the unretarded London
1 nc ;
3 2 attraction force (defined in (3.1.16)) and electrokinetic
61

61
2=3 force (3.1.17) in the double layer, and electrostatic forces,
nc hd gr i 2 ; 7:2:209b if any, such as coulombic attraction/repulsion forces
hdgr i3
(3.1.15) (usually important in aerosol-removal processes
where we have employed expression (7.2.207) for . unless the collector particles are deliberately charged). In
594 Bulk flow perpendicular to the direction of force

addition, one has to consider inertial impaction and inter- 0 12 0 1 0 12


ception (see Section 6.3.1.4) if it is aerosol filtration, since @hd gr iA p @1 t Ag  @hdgr iA  p;r
particle velocities in hydrosol filtration are very low and 2 p 2 p
9
E GrS
inertial impaction (but not interception) may be neglected. 2
0 12
Further, one has to take into account the role, if any, of hdgr iA
 v0  @ 1 2=3  p;rp
Brownian diffusion. One next solves such an equation 2
12 0
((7.2.210)) to determine the trajectory of the particle in
0 1
hd gr i t
terms of the particle coordinates (x, y, z) as a function of 2@ A p @1 Ag
2 p
time in the geometrical configuration of the porous media
1 2=3 :
model adopted; it may be the capillaric model, the spher- 9v0
ical collector model or the constricted tube model. Tien
7:2:214
(1989) has provided detailed treatments for the latter two
models in view of their greater success in realistically If gravity were the only particle capture mechanism, then,
describing the particle deposition process in a granular for the ith UBE, the particle collection efficiency is given by
filter. However, these treatments are quite complex; we
will therefore avoid describing them here. Instead we will E i 1:209 E GrS : 7:2:215
provide illustrative estimates of E Mech S via simple analysis We consider next the particle collection efficiency due to
for a single force like gravity or Brownian motion (equation Brownian motion for very small particles (submicron dimen-
(3.1.43)). sions). Brownian motion or Brownian diffusion may be
Consider first Figure 7.2.10(a), where the liquid is incorporated by considering it simply as a diffusion process,
coming down vertically in the negative z-direction against where the diffusion coefficient for particles of radius rp is
the particle/grain of diameter hdgri. The particle of radius
rp and actual density p settling in the liquid of density t kB T
DBR Cc 7:2:216
will have a terminal velocity whose magnitude is given by 6 rp
(see equation (6.3.1))
(see (3.3.90c)); here Cc is the Cunningham correction
D E2  factor (the slip correction factor, (3.1.215)). The particle
d gr t
2 2 p 1 p g flux due to Brownian motion may be multiplied by the
jU pzt j : 7:2:211 surface area of the spherical filter grain, hdgri2, to deter-
9
mine the rate of particle capture:
Consider now Happels cell model (Happel, 1958) in the
rate of particle capture particle flux  hd gr i2 :
spherical collector model. The total rate at which the par-
ticles enter one cell in Happels model is given by the 7:2:217a
following expression: If the particle flux is represented via the traditional mass
superficial liquid velocity  cell incoming cross- flux expression based on a mass-transfer coefficient (here,
sectional area  particle mass concentration in kp), then

incoming liquid v0  b2  p;rp :



7:2:212a particle mass flux np k p p;rp p;rp jparticle surface :
The rate at which particles settle on the spherical collector 7:2:217b
of radius (hdgri/2) due to gravitational terminal velocity 
We can assume p;rp jparticle surface to be zero.
jU pzt j is given by
The mass-transfer coefficient, kp, may be estimated
from correlations of the Sherwood number, Sh, in a packed
hd gr i 2
 
jU pzt j   p:rp : 7:2:212b bed (where the diameter based definition is used):
2
k p hd gr i
Sh Shp : 7:2:218a
The particle collection efficiency due to gravitational force DBR
EGrS is
Therefore, the particle mass flux to the spherical collector,
2 np, is
hd gr i
jU pzt j  2  p;rp
E GrS : 7:2:213 Sh DBR Shp DBR
v0  b  p;rp 2 np : 7:2:218b
hd gr i p;r p hdgr i p;rp

If we employ result (7.2.209a) from Happels model, Since the rate at which particles enter the cell contain-
we get ing the spherical collector is given by (7.2.212a), the
7.2 Crossflow membrane separations, granular filtration 595

single-particle collector efficiency due to Brownian diffu- E IS E InterceptionS 1:5f 1=3 1 2=3 2r p =hdgr i2 :
sion, EBRS, is given by
7:2:224
rate of particle collection np  hd gr i2
E BRS ; Expressions for the efficiency of particle collection by
rate of particle entry v0  b2  p;rp electrostatic forces are available in Tien (1989) for four types
7:2:219 of electrical forces:
2 (1) a Coulombic force, when both particles and collectors
Shp DBR p;rp hdgr i
E BRS are charged;
hdgr iv0 b2 p;r p
7:2:220 (2) a charged collector inducing opposite charge on the
Shp hd gr iDBR
: particle;
v0 b2 (3) a charged particle inducing opposite charge on the
collector;
If we use Happels model, specifically result (7.2.209a), (4) particles having similar charges producing a repulsive
we get force among particles.
4 Shp 1 2=3 DBR Many of these mechanisms will be operative simultan-
E BRS : 7:2:221
v0 hd gr i eously in the granular filter, in which case the efficiencies
due to individual mechanisms have to be added up. For
Often, the mass-transfer correlation of Pfeffer (1964) based
example, if gravitational force, Brownian motion and inter-
on Happels (1958) cell model is used:
ception are operative simultaneously,
8 h i
91=3 X
< = v hd i1=3
2 115=3 E MechS E GrS E BRS E IS 7:2:225
0 gr
Shp 1=3 5=3 2
; Mech
: 231 31 21 ; DBR
and
7:2:222a X 
E i 1:209 E MechS 7:2:226
where the Pclet number Pep, defined by
from (7.2.209d) for Happels cell model for spherical col-
v0 hdgr i lectors representing the packed bed.
Pep ; 7:2:222b
DBR The same approach has been used successfully for
is employed to yield aerosol filtration in a fibrous filter bed. Rubow (1981) has
developed the following expression for the single-fiber
Shp f 1=3 Pe1=3
p : 7:2:222c efficiency for fibers of radius rf when Brownian diffusion
and interception operate simultaneously:
Here, f() represents the complicated expression in Shp X
dependent on . This leads to E MechS E BRS E IS ; 7:2:227a

E BRS 4 f 1=3 1 2=3 Pep 2=3 : 7:2:223


where
This estimate assumes that there are no surface interactive 1=3   2=3
1 v0 2r f

forces, e.g. repulsive or attractive double-layer forces between E BRS 2:86
the filter grain and the particle to be captured (equation fK1 DBR
" 1=3 !#
(3.1.17)). The double-layer force is attractive or repulsive v0 2r f

1
 1 0:389 ;
depending on whether the zeta potentials of the particle and fK 1 DBR
the collector grain are of opposite or the same sign. This
7:2:227b
happens because any particle or colloid (or a protein/macro-
2
molecule) in an aqueous environment has developed a 2
 
1 r p =r f
E IS 1 ; 7:2:227c
double layer and a zeta potential (see Figure 3.1.2D and f K 1 r p =r f r p =r f
equations (3.1.11ac)). For different expressions of EBRS,
where surface interaction forces are involved, see Tien where the Kuwabara hydrodynamic factor
(1989, sect. 4.5-2.3) for expressions developed by Spielman f K 0:75 0:5 n 0:252 frs 0:5 n 0:52 ;
and Friedlander, Chiang and Tien and Rajagopalan and Karis.
In the absence of any external forces, particles can also frs constant describing fraction of molecules reflected
be deposited on a collector granule by interception if diffusely from the surface 1.14 for air; v0 superficial
during its motion it comes to within the particle radius face velocity. The overall fibrous filter efficiency, ET,
from the spherical collectors surface. The result based on defined by (6.3.45) may be determined with this estimate
Happels model is of single-fiber efficiency E MechS :
P
596 Bulk flow perpendicular to the direction of force

7.3 External force field based separation: bulk molecule or drop. This is in contrast to the bulk flow of
flow perpendicular to force liquid parallel to the direction of the uniform electrical
force field employed in capillary electrophoresis (Section
This section covers separation by an electrical force field
6.3.1.2).
(Section 7.3.1), a centrifugal force field (Section 7.3.2), a
gravitational force field (Section 7.3.3) and a magnetic
force field (Section 7.3.5); Section 7.3.4 covers field flow 7.3.1.1 Electrophoresis
fractionation. A brief analysis of continuous free-flow electrophoresis
(CFE) of charged macromolecular species is provided
below. (For a brief introduction to electrophoresis, see
7.3.1 Electrical force field Biers chapter in Karger et al. (1973). For a more detailed
Charged bodies move when exposed to an electrical field. introduction, see Andrews (1981). For capillary electro-
The charged bodies may be molecules, macromolecules, phoresis, see Section 6.3.1.2; for isoelectric focusing, see
cells or particles in a liquid phase, or they may be ionized Section 4.2.2.1; for time-dependent separation of a batch
gas molecules or charged particles in the gas phase or in a sample by electrophoresis in a closed vessel, see Section
plasma. The charge on the body may be free or natural for 4.2.2.2.) The conditions are such that the effective bulk
the system to be separated, or it may be induced into the motion of the conducting aqueous buffer solution into
bodies by external sources including a nonuniform elec- which a feed mixture is introduced is perpendicular to
trical field. the direction of the electrical force field. A number of
Electrophoresis is used to separate naturally charged apparatus geometries have been studied. One of the most
macromolecules, particles and proteins in a liquid phase common devices employs thin-film continuous-flow elec-
via a uniform or nonuniform electrical field perpendicular trophoresis in which the buffer solution flows in a narrow
to the direction of the bulk liquid motion (Figure 7.0.1(i)). channel of width 2b between two flat plates with cooling
If a neutrally charged particle or cell becomes polarized in jackets; two oppositely charged electrode plates provide
a nonuniform electrical field and moves as a result in the the other two surfaces of this rectangular liquid flow duct
liquid phase partly in the direction of the applied force, the (Figure 7.3.1). Note: The feed mixture is introduced over a
phenomenon is called dielectrophoresis (see equation limited area of buffer flow.
(3.1.13) for an expression for the force). Neutral molecules The electrical force field between these two electrode
in a gas or vapor may be ionized by a laser beam via loss of plates is perpendicular to the bulk buffer flow direction.
electrons; the positively charged species are then attracted Positively charged species move toward the cathode in the
toward and collected by a negatively charged electrode, as positive x-direction. The axial buffer velocity (vz) profile in
in laser isotope separation. Electronegative impurities in a the laminar channel flow is parabolic. At the channel inlet
gas may have electrons attached to them if the gas is (z 0), the feed solution is introduced continuously in a
passed through a corona-discharge reactor. Such impurities narrow band at (y, x0), whereas the buffer solution flows
may then be collected by the positive electrode. into the channel through the entire channel cross section.
Particles in a gas having no charge or very slight charge One would like to know the locations (x, y) at z L, the
are allowed to acquire substantial negative charge in channel length, where each individual charged species
a corona generated by an intense electrical field around a present in the feed will appear. Note that only a uniform
wire; the particles then move in the electrical field to a electrical field is assumed to exist between the electrodes.
grounded collecting electrode in electrostatic precipitation The general governing equation for concentration C il
(see Figure 7.0.1(j)). When different particles (especially of of species i in a nonreactive liquid system is (equation
plastics) acquire different charges via triboelectrification (6.2.3b))
and are exposed to an electrical field, different particles C il
with different charge to mass ratios fall to different loca- r N i :
t
tions at the end of the electrostatic separator and are sep-
arated. In the process of flow cytometry used for cell sorting The flux N i of species i in a fixed coordinate frame of
(separation of different cells), droplets containing a par- reference for a dilute noninteracting multicomponent
ticular type of cell have different charges imparted to them system at constant temperature and pressure is (see equa-
and are collected in different bins (containers) when pass- tion (3.1.84a))
ing through two vertical (or inclined) electrodes applying a N i C il vt U i Dis rC il ;
 
voltage perpendicular to the vertically downward bulk
motion of the drops. In all such cases treated in this where U i is the migration velocity vector of solute species i
section, the bulk gas or liquid phase or a droplet flows defined by (3.1.84b). Incorporation of this expression of N i
perpendicular to the direction of the electrical force into the conservation equation for species i and the one-
exerted on the charged or polarized or ionized particle or dimensional (x-direction) migration of species i due to the
7.3 External force field based separation: bulk flow perpendicular to force 597

Buffer in
Feed sample
x introduction
Buffer velocity y
profile x0
z
x

z
y

Uix
+
nz
Electrode Electrode

2b 2b
Collection
xiL ports

Figure 7.3.1. Thin-film continuous-flow electrophoresis separator and idealized solute trajectory. (After Gobie et al. (1985).)

electrical field (Figure 7.3.1), leads to the following equa- Using the nondiffusive limit of assumption (1), one
tion in rectangular coordinates (see Table 6.2.1): finds at steady state from equation (7.3.1)
 2 
C il C il C il C il 2 C il 2 C il C il C il
vz U ix Dis ; vz U ix 0: 7:3:2
t z x x 2 y2 z2 z x
7:3:1 Therefore, a constant macromolecular solute concentra-
tion C il introduced at z 0 at the inlet end of the rect-
where, since the system is dilute, we have assumed
angular duct at a particular x x0 will have an (x, z)
vz vz . Further, vy and vx are assumed to be zero.
trajectory defined by
A highly simplified approach to the above problem is
adopted first by making the following assumptions: dx U ix
: 7:3:3
dz vz
(1) there is no solute diffusion in any of the three axial
directions; This relation is obtained by using the methodology
(2) the axial buffer moves in plug flow in the x-direction; employed earlier in deriving (7.1.12e). For constant U ix
(3) there is no electroosmotic flow velocity induced by the and vz , integration leads to
electrical field in the z-direction; U ix L
(4) the effect of Joule heating in the system due to the xL x0 : 7:3:4
vz
current is negligible;
Note that L=vz is the solute (as well as buffer) residence
(5) the system is of constant physical properties, e.g.
time in the rectangular device of length L. Thus, the
density, viscosity and diffusion coefficients are
straight-line trajectory of charged solute i arrives at z L
constant.
displaced a distance x L x 0 from its original x-location
Some of the complexities introduced by relaxation of these due to the electrical field. The y-coordinate of the solute is
assumptions are treated later. assumed to remain unaffected between x 0 and x. There is
598 Bulk flow perpendicular to the direction of force

neither convection nor diffusion in the y-direction (by solute species i are contained in a volume of dimensions
assumptions (1) and (2)). l z , l y and x (see Section 3.2). The solution for this is (see
The migration velocity U ix of charged species i in a equation (3.2.19))
uniform electrical field E in the x-direction is known to be
x U ix t2
!
mi 1
 
simply (from equations (6.3.8e) and (3.1.108m)) C il x;t exp : 7:3:9
lz ly 4Dis t 1=2 4Dis t
U ix Em
i ; 7:3:5
The concentration profile C il x; t may be calculated at
where mi is the electrophoretic mobility of the ith ionic t L=vz , corresponding to the device exit where various
species (in units of cm2/s-volt). Identify by x iL the x- fractions are to be collected:
coordinate location of species i at z L; then,
U ix L 2
2   3
x
x iL Em i =vz =L x 0 ; mi 1
  
7:3:6 6 vz 7
C il x;L=vz r exp 6 :
6 7
l z l y Dis L 4Dis L 7
suggesting that different species introduced into the device 2
4 5
vz vz
via the feed solution at z 0, x x 0 and y will appear at
different x-coordinate locations at z L as long as each m 7:3:10
i
is different. Since the buffer solution and the feed mixture This result, which is based on a number of assumptions,
are flowing in continuously, this allows continuous separ- including x 0 0, indicates a Gaussian concentration profile
ation of a multicomponent charged species mixture if we of species i around a center point of
collect separately and continuously the buffer solution at
U ix L Em L
each x iL location (Figure 7.3.1). The difference between the x i 7:3:11
vz vz
x iL values for species i and j is
 and a standard deviation ix of
x iL x jL E L=vz m m
i j : 7:3:7
ix 2Dis L=vz 1=2 : 7:3:12a
Obviously species having a particular charge (here posi-
tive) will have x iL values greater than x 0 ; others having the The width of the collection port for each species i at z L
opposite charge will arrive at z L with x iL values less should accommodate such a profile. The base width of
than x 0 . such a profile is 4 i (see Figure 2.5.2) to accommodate
There are a number of real-life features which result in 95% of species i introduced into the device. Therefore,
significantly lower separation in actual practice. Consider collection port width 42Dis L=vz 1=2 : 7:3:12b
the role of solute diffusion (assumption (1)). Diffusional
spreading cannot be prevented. Restricting diffusional Philpot (1940) obtained such a result in terms of t L=vz .
broadening for the time being to only the x-direction and The number of different charged species which can there-
ignoring the z-direction convection, the following equation fore be separated in a free-flow electrophoresis device is
is obtained from equation (7.3.1): limited by solute dispersion in the x-direction, amongst
other things. The peak-to-peak distance between two
C il C il 2 C il species may be estimated from equation (7.3.7). A more
U ix Dis 7:3:8a
t x x 2 exact solution, which includes the effect of dispersion in
instead of equation (7.3.2). The unsteady state term the z-direction, has been provided by Reis et al. (1974) for a
C il =t is needed here since, in the absence of a z- parabolic velocity profile vz y in the y-direction; effect-
direction convection term, the time t in effect defines the ively, the Philpot (1940) model underestimates x-
position along the x-axis where the C il dependence on z is directional band broadening substantially. The base width
to be determined (see Reis et al. (1974) for an exact pro- of the profile described by equation (7.3.9) depends on t; at
cedure, where a solution for C il is obtained as z values less than L, the base width will be smaller. Note,
C il C 1il z;y;t  C 2il x;t, where C 2il x;t satisfies (7.3.8a) however, the separation of Figure 7.3.1 is essentially at
and the requisite boundary conditions given below). For steady state. The time coordinate used here allows a spe-
simplicity, instead of a constant solute feed, we consider a cification of position along the z-axis.
pulse of solute at z 0 (i.e. t 0). Any solution for C il The feed solution is normally introduced into the
obtained from (7.3.8a) should then satisfy the following mobile buffer solution at z 0 of a continuous free-flow
conditions: electrophoresis (CFE) apparatus in a thin band of width 2
mi in the y-direction (Figure 7.3.2(a)). If assumption (2) of
at x
; C il 0; at t 0; C il x; 7:3:8b plug flow of buffer in the z-direction were truly valid, the
lz ly
preceding analyses (e.g. result ((7.3.6)) would hold also for
where we have conveniently assumed that x 0 0. The a solute band of width 2, i.e. the band will appear
undeformed but displaced by Em i L=vz in the x-direction.
 
second condition implies that originally mi moles of all
7.3 External force field based separation: bulk flow perpendicular to force 599

(a) (b)

2b 2d x
x

y y

Figure 7.3.2. (a) Solute band at inlet of device shown in (x,y)-plane; (b) solute band deformed into a crescent shape at the exit of the
device.

In conventional CFE equipment, the buffer flow, however, Sammons (1992) have designed a new type of CFE cham-
has a parabolic velocity profile: ber comprising a series of constrictions along the x-axis of
  separation to increase the peak-to-peak distance
y2
vz y vmax 1 2 : 7:3:13 substantially.
b The nonidealities considered so far fall into two cat-
Therefore, solute molecules at different y-coordinate loca- egories: those due to diffusion and those due to other
tions within the band of width 2 experience different effects, e.g. parabolic velocity profile, electroosmotic flow.
values of z-direction convection velocity vz . Consequently, A measure of the role of diffusion is provided by the axial
the residence time for solute species in the CFE device Pclet number ( Pez vz b=Dis ) obtained by nondimensio-
varies, depending on its y-location. For example, solute nalizing x, y, z and t in equation (7.3.1):
species in the band closer to the wall have a lower vz y
C il C il bU ix C il 2 C il 2 C il 2 C il
and therefore a longer residence time L=vz y; it will be Pez  2 2 2 ;
t  z Dis x  x y z
displaced further along the x-axis than those molecules
which are introduced near the channel center. Obviously 7:3:16
x iL x 0 , given by equation (7.3.6), will now depend on
where x  x=b, y  y=b, z z=b and
the y-location. Figure 7.3.2(b) illustrates the shape of the
t  tDis =b2 .
 
thin band in the (x, y)-plane.
In a typical CFE apparatus, Gobie et al. (1985) have
In reality, additional complications are introduced by
suggested that the dimensionless Pclet number, Pez , is of
three factors. First, diffusional spreading has to be super-
the order of 10 000 or greater. (An example would be
imposed on this band distortion. Second, there is an elec-
as follows: L 16 cm, b 0.187 cm, vz 0:7 cm=s,
troosmotic velocity vEOF in the x-direction in addition to
Dis 5  10 7 cm=s for a solute similar to albumin, so
the electrophoretic velocity U ix ; therefore the solute vel-
Pez 2:6  105 .) Therefore diffusional effects may be neg-
ocity in the x-direction is (Ivory, 1980):
lected. The dimensionless number bU ix =Dis is called the
 
3 y2 electrophoretic Pclet number; it is also expressed as
 
vtx vEOF vEOF 1 2 U ix : 7:3:14
bEm

i =Dis .

2 b

Displacement of solute molecules in the x-direction at z Example 7.3.1 Consider a CFE device such that the two flat
L compared to their x-coordinate at z 0 now depends on electrodes may be treated as if they were the walls of a flat slit
the y-coordinate in a much more complex fashion: of two infinite parallel plates spaced a distance b apart. The
axial velocity vz in such a configuration in laminar flow is
2
given19 by
n  o
vEOF 32 vEOF 1 by 2 U ix
x iL y x 0 y : 7:3:15
vz d z=d t 6vz;avg y=b y=b2 ;
  
2
vmax 1 by 2 =L
where y is the transverse distance to a point in the channel
Ivory (1980) has described the deformation of the rect- from one of these plates; y b at the other plate. For
angular solute band into a crescent shape. Figure 7.3.3 a uniform electrical potential profile between the two
illustrates the solute output behavior as a result of these electrodes, develop an equation for the trajectory of a
phenomena. To overcome such problems, Gobie et al.
(1985) have suggested employing recycling of the effluent 19
See Happel and Brenner (1965); equation (6.1.2b) is for a slit flow
fractions from the CFE apparatus and introducing them where the channel gap << width of the channel < length of the
into appropriate locations at the device inlet. Sharnez and channel in the mean flow direction (Bird et al., 2002).
600 Bulk flow perpendicular to the direction of force

x0 lateral
Carrier
fluid profiles y0 transverse

z0 axial
Solute input

Velocity
profile
g

Electrodes

Temperature
profile

Figure 7.3.3. Classical vertical CFE with buffer flowing cocurrent to gravity; observe the crescent shape in the solute output. Reprinted,
with permission, from W.A. Gobie, J.B. Beckwith, C.F. Ivory, Biotechnol. Progr., 1(1), 60 (1985). Copyright [1985] American Institute of
Chemical Engineers (AIChE).

charged protein relating its z-coordinate to its y-coordinate, 6vz;avg y2 y3


 
in the absence of any diffusive band broadening. z0 ;
m
i E 2b 3b2
Solution Let the y-coordinate be normal to one of the flat which relates z to y, assuming the starting point is z 0, y 0
electrodes; its y-value will be zero and that for the other (one of the electrode locations). (In reality, y will have a finite
electrode will be b. The velocity of any protein in the axial (z) value at z 0, say y0.) If one defines the following nondi-
direction will be that of the buffer: mensional variables:
vz dz=dt 6 vz;avg y=b y=b2 : z mi E y

z ; y ;
b vz;avg b
Its y-direction velocity will be
vy dy=dt m then
i E:
 2
y3
 2
y y3 y0
 
The trajectory equation is obtained following (7.3.3) as z z 6 6 0
;
0
2 3 2 3
dz=dy 6vz;avg y=b y=b2 =m i E

where the coordinates of solute injection are z0, y0,
(no diffusive effects). Integrating implies that correspondingly
7.3 External force field based separation: bulk flow perpendicular to force 601

z m
0 z 0 i E=bvz;avg ; y
0 y 0 =b:
CFE separator (Figure 7.3.4) is strictly nonuniform. The
electrical field varies inversely with the radius (see Problem
Therefore all solute trajectories may be described by the 3.1.5); the electrophoretic radial migration velocity there-
nondimensional result just derived. For a given solute of mi fore decreases as the radial coordinate of the charged
and z L, z will have a given value; correspondingly y will
solute molecules increases. Due to the very small thickness
have a particular value.
of the annular region in the rotationally stabilized CFE
An effect not yet touched upon is that covered by assump- separator, the electrical field is, however, almost uniform.
tions (4) and (5). Due to the passage of the electric current, Rolchigo and Graves (1988) have modeled continuous
a considerable amount of heat (identified in the literature electrophoresis processes in general when subjected to a
as Joule heating) is generated in the buffer solution, nonuniform electrical field.
leading to natural convection, which mixes different
Example 7.3.2 If one solute is injected at z 0 of the CFE
regions of the buffer and reduces separation. In smaller
separator of Figure 7.3.4 at z 0 (see Example 7.3.1 for the
devices that have a considerable amount of cooling surface notation as applied to Figure 7.3.4, where the annulus is very
area available for a given device volume (Hannig, 1961), thin, i.e. ((ro ri)/ro) << 1 so that the velocity profile of a thin
such an effect is reduced. The problem becomes magnified flat slit of Example 7.3.1 is applicable) and another solute is
for larger-scale operation. As a result, a scale up of continu- injected at the same location (z 0) such that the values of
ous FFE is problematic. Vermuelen et al. (1971) used small z
0 of the two differ by z 0 , how would the center lines of
spherical particles as a bed packing through which the these two solute trajectories differ from each other in the
buffer flowed and which ensured uniform flow. Mattock limit of z
0 ! 0? Assume y 0 0.
et al. (1980) developed an annular device in which the Solution In the limit of ((rori)/ro) << 1, the axial flow
outer cylinder is rotated and the inner cylinder is at rest. profile corresponds to that between infinite parallel plates.
An axial flow imposed on this rotational flow allows free- Therefore the trajectory equation developed in Example 7.3.1
flow electrophoresis to be carried out since the laminar for solute 1 is usable here. Therefore
axial flow is stabilized by the radial angular velocity gradi-  2 
y1 y3
ent in the buffer against any natural convection due to z z
0 6 1
2 3
Joule heating.
Figure 7.3.4 illustrates such a continuous free-flow Since y 0 0. Let the trajectory for a second solute,
electrophoretic separator. The feed sample to be separ- whose z
0 value is z 0 z 0 , be given by
ated (identified as the migrant mixture input) is intro-  2 
y2 y2
duced through a circumferential slit in the stationary z z
0 z 0 6 2 :
2 3
inner cylinder (the stator, 8 cm diameter) near the bottom
of the device, from where the different species flow up. (Note: y
1 y 1 =b ; y 2 y 2 =b.) Therefore
 2   2 
The buffer solution (identified as the carrier) is introduced y1 y 3
1 y2 y3
2
at the bottom of the stator, it flows up and between the z
0 6 :
2 3 2 3
stator and the outer rotating cylinder (the rotor, 9 cm
Let
diameter). Sections of each of the inner and outer cylin- 80 1 0
ders act as the two electrodes across which the voltage is < y 2 y 3 2
y
2 y
1 y 1 ) z 0 6
@ 1 1 A @y 1 y y
applied. Individual species migrate to different radial dis- 1 1
: 2 3 2
tances from the inner cylinder as they are moved up by !)
axial flow; the latter is divided into various fractions and y 2
1 1
y 3 2
3y 2 3

3y y y y
collected simultaneously through respective openings in 2 3 1 1 1 1 1 1

the stator near the top of the 1 m long annulus. As many n n 2 oo


6 y
 2 
y 1 y1 O y

1 1 :
as 20 fractions can be collected simultaneously by a col-
lection of stacked discs in the stator above the electrode Therefore
section; see Mattock et al. (1980) for details of electrode n 2 o
z
 2 
0 6 y1 y1 y
1 6O y

design and other aspects. A sample throughput of up to 1 :
20 ml/min can be achieved; the temperature rise is limited
Differentiating, we obtain
to 2025  C during the short residence time (3060
seconds) in the buffer flowing at a rate of 500 ml/min. dy
1 1
lim  2  :
This minimizes the loss of biological activity of proteins.
z0 !0 dz0 6 y1 y1
The extent of temperature rise in such a device has been This implies that, at the wall location, the gap between the
modeled by Noble (1985). center lines of the two solute trajectories will be very large
Continuous free-flow electrophoresis in a thin rect- (since y1 ! 1). Therefore it is desirable to withdraw the
angular channel of Figure 7.3.1 involved a uniform elec- product bands somewhere in between. These results were
trical field. The electrical field in the rotationally stabilized obtained by Beckwith and Ivory (1987).
602 Bulk flow perpendicular to the direction of force

(a) (b)

Carrier in

Migrant in Inner electrolyte in


Carrier input Migrant mixture input
Outer electrolyte in Inner electrolyte out
To collection manifold

Stator Migrant fractions outlets

Outer
electrolyte
out
Electrode section
(steel) + + Electrophoresis

Rotor Stator

Rotor

Figure 7.3.4. Diagrammatic representation of the continuous-flow electrophoretic separator. (a) Cutaway drawing of the system showing
the construction of the rotor, stator and main flow channels. (b) Separation of three components as they are carried up through the
annulus. From Figure 2, p. 7 of Velocity gradient stabilized continuous free flow electrophoresis. A review, P. Mattock, G.F. Aitchison,
A.R. Thomson, Separ. Purif. Meth., 9(1), 1 (1980); reprinted by permission of the publisher (Taylor and Francis Group, http://www.
informaworld.com).

7.3.1.1.1 Continuous dielectrophoresis In expression is proportional to the gradient of the square of the elec-
(3.1.13) for the dielectrophoretic force on an uncharged trical field strength. Whereas in electrophoresis, the sign of
particle in a nonuniform electrical field, the Clausius the charge (ve or ve) on a particle or a macromolecule
Mossoti function includes p and d, which are complex or a molecule and the direction of the electrical field are
quantities. However, only the real part is useful: quite important, here in dielectrophoresis the direction of
  the applied electrical field is of no consequence; therefore
p d
F 2 r 3p d Re rE2 : 7:3:17a dielectrophoresis can work with alternating current gener-
p 2 d
ating fields.
The sign of the quantity p d is important. If p is Continuous dielectrophoresis of whole cells using a
greater than d , then the direction of the force is toward stream-centered introduction of sample cell suspension
the region of high electrical field strength; it is called is illustrated in Figure 7.3.5 (Pohl, 1977). A stream of cells
positive dielectrophoresis. On the other hand, if p is smaller or other particles to be separated is fed into the center of a
than d , the direction of the force is reversed, and one has cocurrent liquid stream (the carrier liquid) very similar in
what is called negative dielectrophoresis: the particle is composition to that carrying the cells or particles. A sample
forced in the direction of lower field strength (Pohl and to carrier flow rate ratio of 1:5 to 1:100 is employed to keep
Kaler, 1979). (Of course, the particle may experience zero the sample stream as close to the center of the flow chan-
force as well.) Note further that the dielectrophoretic force nel (as far from the walls and the electrodes) as possible.
7.3 External force field based separation: bulk flow perpendicular to force 603

Sample cells in a stream Carrier liquid in We will now provide from Kralj et al. (2006) a brief
Curved electrode
example of continuous dielectrophoretic size based
particle sorting in laminar flow (in the z-direction) in a
rectangular parallel-plate type of microfluidic channel
(cross-sectional view of channel in Figures 7.3.6(a)
and (b), (x, y)-plane) containing an array of slanted planar
electrodes on a glass channel floor going from one side of
AC the channel to the other in one of the parallel plates
voltage
power (Figures 7.3.6(a) and (b), (x, z)-plane); here the main liquid
flow is in the z-direction. The particle mixture to be separ-
Flat electrode Cells showing negative
Cells showing positive dielectrophoresis ated is introduced through one inlet in one of the side
dielectrophoresis
walls, next to which there is another inlet used to introduce
a particle-free solution to get the particles away from the
Cells having no sidewall; the main channel inlet introduces the bulk liquid
deflection
flowing in the z-direction. Kralj et al. (2006) have approxi-
Carrier liquid out mated the electrical field due to this slanted array of elec-
trodes in the (x, z)-plane via
Figure 7.3.5. Schematic diagram of continuous dielectrophoresis  
2z 2x
using a stream-centered introduction of the sample cell suspen- jE 2 j AV 2 sin AV 2 ; 7:3:18
z x
sion. (After Pohl (1977).)
where A is the amplitude of the wave, V is the voltage
between the electrodes, z and x are the electrode
spacings in the z- and x-directions, respectively (deter-
Cells undergoing positive dielectrophoresis move in the
mined by the slant angle and the normal distance
radial direction of the merging of the curved rear electrode
between the electrodes). Since the dielectrophoretic
and the flat front electrode, whereas cells experiencing
force is proportional to E2, the two components of
negative dielectrophoresis move in the opposite direction.
interest are
Meanwhile, the bulk carrier flow perpendicular to this
 
electrical force carries the cells forward toward the carrier jE 2 j 2 2z 2x
exit. The cells or particles unaffected by the nonuniform AV 2 cos 7:3:19a
x x z x
electrical field continue to move along the line of original
introduction. Thus different cells trace out different trajec- and
tories and therefore can be withdrawn at different locations  
jE 2 j 2 2z 2x
from the stream near the liquid outlet. AV 2 cos : 7:3:19b
z z z x
Typical deflections experienced by cells in such a
device amount to several millimeters for axial flow path Figure 7.3.6(c) illustrates the variation of jE 2 j with the x-
lengths of 100 mm for an applied potential field of 48 volts coordinate at a small distance from the top plate.
rms (root mean square) in a medium of appropriate con- Kralj et al. (2006) studied the separation of polystyr-
ductivity (Pohl, 1977). The particular electrode configur- ene (PS) particles of diameters 26 m in a dilute aqueous
ation employed is called the isomotive electrode sucrose solution (having a density such that gravitational
configuration; it creates a constant dielectrophoretic force forces were zero since p t ). At the low particle Rey-
on a particle or cell over a wide region, i.e. over a large nolds numbers employed (<0.01), the drag force will be
range of radial locations (Pohl and Kaler, 1979). The mag- described by Stokes law. At steady state, the drag force
nitude of this force is will balance the dielectrophoretic force (equations
(6.2.47)):
F ar 0 ; 7:3:17b
 
p d
where a is a constant for a particular type of particle or cell 2r 3p d Re rE 2 6 r p v: 7:3:20
p 2d
and the zeroth power for r indicates r-independence of
F. This ensures that particles or cells of a certain type and From (3.1.14), we know that rE 2 rE E rjE 2 j.
size will experience the same external force independent Therefore, the two components of this force balance lead
of their radial location. This, along with the stream- to two components of the particle velocity:
centered introduction of sample stream containing  
cells, avoids the placement of living cells close to the wall, dx 2 r 3p d p d jE 2 j
particle velocityjx -dir Re ;
thus avoiding higher residence time and possible damage dt 6r p p 2d x
to the cells. 7:3:21a
604 Bulk flow perpendicular to the direction of force

(a) (b) (c) 11


Particle Particle-free 10
suspension solution inlet outlet 14
inlet E
2

Electrodes A Cross-sectional view (AA) 10


2
x y Large volts
Bulk Small 2
liquid z x particle particle m 6

A
2
Electrodes
outlet 0 1 2 3 4
x (m) 10

(d) (e)
1

x- coordinate position / 500 m


vz = 2 mm/s rp = 3 m
t 0.8
V = 10 volt
x= 2
2a vz
Displacement

0.6
rp = 2.5 m
x0
0.4

x (t ) rp = 2 m
0.2

rp = 1 m
0
Time 2 4 6 8 10 12 14
t (s)

Figure 7.3.6. Continuous dielectrophoretic separation of a particle mixture in microfluidic channel flow. (After Kralj et al. (2006); channel
height 28 m, channel width 500 m.) (a) Cutaway view of the channel in the (x,z)-plane showing the inlets of particle suspension,
particle-free solution, bulk liquid flow, two liquid outlets for two particle fractions and slanted electrodes. (b) Cross-sectional view at the
AA0 plane of (a), showing electrodes on the channel floor, larger and smaller particles separated in the x-direction; (c) Value of E2 of the
voltage variation in the periodic array of electrodes at a distance 3 m from the channel top. (d) Behavior of x(t) per equation (7.3.24)
and a plot of x against t per equation (7.3.27). (e) x-coordinate position of particles of different rp normalized with channel width of
500 m against time t.

dz a function of time t (which is linearly related to the axial


particle velocityjz-dir
dt displacement of the particle). Kralj et al. (2006) have pro-
 
2 r 3p d p d jE 2 j vided the following analytical solution of the above particle
vz Re ; 7:3:21b trajectory equation when z x for an electrode slant
6r p p 2d z
angle of 45:
where vz is the liquid medium velocity in the z-direction, p !!
a2 v2z 1 w1 a2 v2z 1
p
the direction of mean flow. Note: (1) The real part of the 2
xt vz t arc tan tan t ;
dielectric constant for the PS particle, p, is 2.5, much less w1 avz 1 2a
than that of the solution (~70); therefore we have negative 7:3:24
dielectrophoresis. (2) The magnitude of vz is much larger
than that of the second term on the right-hand side of where
equation (7.3.21b). Therefore  
p d 2
1=a 2 r 2p =3 x d Re A V 2; w1 :
dz=dt vz ) z vz t: 7:3:22 p 2d
7:3:25
We may now use this result in the particle trajectory equation
resulting from combining equations (7.3.19a) and (7.3.21a): This time-dependent oscillating x-directional displacement
2     of the particle (shown in Figure 7.3.6(d)) may be charac-
dx 2 rp p d 2 vz t 2x
d A V 2 Re cos : terized by means of an averaged x-directional displace-
dt 3 x p 2d z x
ment x at a time t defined by the half-period of the function
7:3:23
2N 1
Thus the extent of lateral particle displacement, x, due to t p ; N  0; 7:3:26
w1 v2z 1=a2
the negative dielectrophoretic force may be determined as
7.3 External force field based separation: bulk flow perpendicular to force 605

which is shown by the thick straight line in Figure 7.3.6(d). excitation is such that U235 is ionized to 235U by losing
The slope of the thick straight line is given by an electron. However, 238U is not ionized. The narrow-
  r   band laser source selectively excites only one isotope from
x 1 1 1 its ground state to the ionized state. The negatively biased
vz v2z 2 2
O 4 ; 7:3:27
t a 2a vz a electrode collects the positively charged 235U ions; the
which is acceptable since (1/a) is usually less than 0.02vz. electrical field between the two plates is perpendicular to
One can now calculate the x-directional displacement of the upward vapor flow, which consists essentially of
238
the particle at the end of the channel (where z L and t U (Grossman and Shepp, 1991), and the tails stream.
(L/vz)): In one such device, the 235U concentration is increased to
0 1 0 1 3.2%, which is enough for a reactor grade uranium. The
x performance achieved is somewhat reduced by charge-
xjzL @ At
1 @LA
t 2a2 vz vz transfer collisions in the neutral background vapor:
0 0 1 12
235 238 235 238
2 p d
7:3:28 U U , U U : 7:3:29
@ d Re @ AAA L 0 1
3x p 2d r4 V 4
@ p A: Removal of vapor-phase impurities from gas streams

2 v2z may be implemented by exposing the gas stream to a
corona-discharge reactor, where the low-energy electrons
This result shows that the displacement in the x-direction produced collide with gas molecules. Electronegative
is proportional to the fourth power of the particle radius impurities, such as sulfur compounds and halogen com-
and the voltage applied and inversely proportional to pounds, have a much higher probability of becoming nega-
the square of the flow velocity in the z-direction. Figure tive ions by electron attachment:
7.3.6(e) illustrates the x-direction displacement of particles
A e , B C;
of different radius normalized with respect to the channel
width of 500 m. This figure shows that the largest particle for example
(radius, 3 m) has a very large displacement. Kralj et al.
SF6 e , SF5 F: 7:3:30a
(2006) have experimentally verified the dependence of the
x-displacement vis--vis r 4p V 4 =v2z , as indicated by the Some of the other species which can be removed from air
result (7.3.28). via the mechanism of (7.3.30a) are SO2, CS2, H2S, COS, etc.
For the second type of reaction mechanism, illustrated by
(7.3.30b), the species which may be removed are I2, O2 as
7.3.1.2 Separation via ionization in gas/vapor phase: well as SF6:
laser excitation of isotopes, electron attachment in a
corona-discharge reactor A e , A: 7:3:30b

In liquid-phase separations, charged proteins, macromol- Tamon et al. (1995) have illustrated the devices and prin-
ecules, cells or particles have different mobilities in an ciples of gas purification using such ionization mechan-
applied electrical field. The charge may be free or excess isms. Consider a cylindrical reactor where a high voltage
charge, as in electrophoresis, or induced by polarization in a ( 315 kV) is applied between an inner piano wire acting
neutral particle, as in dielectrophoresis. In a gas or vapor as the cathode and an outer cylinder, which is the anode
phase of neutral species, if one of the species can be select- (Figure 7.3.7(b)). Electrons produced by the corona dis-
ively ionized in preference to the others, then the ionized charge drift toward the anode in the nonuniform electrical
species with the charge may be selectively removed by field. As they drift toward the anode, some of them collide
applying an electrical field perpendicular to the direction with the gas molecules and generate negative ions by
of the bulk flow of the gas or vapor. This principle has been electron attachment. The negatively charged ions will also
applied to the following types of systems: separation of drift toward the anode. Removal of the negatively charged
uranium isotopes in the atomic vapor laser isotope separ- ions at the anode may be carried out in two ways.
ation (AVLIS) technique; electron attachment to electro- In the deposition-type reactor, the negatively charged
negative impurities in a corona-discharge reactor. ions are attracted to the anode and are deposited there; the
In the AVLIS technique, raw uranium metal containing species adhere to the anode surface after they lose the
0.7% 235U and 99.3% 238U is vaporized at >4000  F. The electrons. This arrangement is not suitable if the electro-
vapor flows up; as it flows up, a laser beam (from a copper- negative impurities are not deposited by the negative
vapor laser) crosses this beam at a right angle (Figure 7.3.7 ions at the anode. An alternative arrangement uses the
(a)). The design is such that the laser beam traverses a sweep-out type reactor, where the anode is a sintered metal
maximum amount of the vapor. This vapor goes up based porous pipe (of brass, for example). The negatively
between two long plates acting as electrodes. The laser charged ions will come to the anode; many of the ions will
606 Bulk flow perpendicular to the direction of force

(a)

Laser beam

Negatively charged
collector for 235U+

Other
electrode

Uranium
vaporizer

(b)
Voltage source

Cathode (wire)

B
e Gas mixture, species A in inert gases
e

A
B
Anode Anode
A

e
B

Ground

Figure 7.3.7. (a) Exploded view of the AVLIS device for uranium isotope separation. (b) Electron attachment based gas purification in a
corona-discharge reactor (see equations (7.3.30a, b)). (After Tamon et al. (1995).)

lose their electrons and become neutral molecules. The electron attachment phenomenon as well as on gas-
enriched neutral molecules will be swept out through the discharge technology for ultrahigh purification. Bulk flow
porous anode (Tamon et al., 1995). Although there is con- of the gas perpendicular to the electrical field can be
siderable back diffusion of the enriched neutral molecules employed also to analyze/separate/capture different ions
toward the reactor center, an adequate sweep-out rate will in the gas produced from different chemicals. This is the
substantially reduce the back diffusion rate. Consult basis of ion mobility spectrometry, employed for explosives
Tamon et al. (1995) for earlier literature, especially on the detection; see Problem 7.3.6.
7.3 External force field based separation: bulk flow perpendicular to force 607

(a) Grounded electrode

Corona wires (large negative voltage) b

Gas enters vz z
rpf y y = 0 plane
Upz
Particle trajectory b
Particle Upy
entering
precipitator Grounded electrode
z=L

(b) Grounded
Corona wires
electrodes

Precipitator
shell
Precipitator
shell

Gas flow

Figure 7.3.8. (a) Particle trajectory in a rectangular precipitator duct showing the configuration of corona wires and the collecting
electrodes; (b) multi-duct arrangement in a precipitator shell. (After White (1963).)

7.3.1.3 Electrostatic precipitation of particles spray irrigation, film irrigation, intermittently flushed irri-
gation, self-irrigation, etc. (Jaasund, 1987).
Suspended particles are routinely separated from gas
Most particles in industrial gas streams have some
streams in large-scale industrial processes by electrostatic
electrical charge; however, the magnitude of this charge
precipitators, which were first introduced in 1906. The
is very small. In practice, suspended particles are imparted
process of separation consists essentially of three steps
substantial negative charge by passing the gas through a
(White, 1963).
device containing a fine wire at a large negative voltage and
(1) Suspended particles become electrically charged. a large particle-collecting electrode which is grounded
(2) The charged particles move in an electrical field and (Figure 7.3.8(a)). There exists a strong electrical field near
are collected at an electrode. the wire with a corona glow due to the large number of
(3) The electrode plates are cleaned by rapping and drop- ions generated. Although both positive and negative ions
ping by gravity of collected dust particles, which are are generated near the wire, the bulk of the gas space
removed from the precipitator (into a hopper). contains essentially negative ions generated by collisions
Step (3) is followed in dry electrostatic precipitator systems with electrons from the corona region; therefore most
only. In wet electrostatic precipitation systems, a liquid suspended particles acquire a negative charge (White,
(usually water) film is used to flush the collecting electrode 1963) and move toward the grounded large electrode,
surface of particles. This may be done in a variety of ways: which acts as the particle collector. (The general process
608 Bulk flow perpendicular to the direction of force

of particles acquiring charge via collision with ions gener- C c LE r min


 7:3:31c
ated in an electrical field is called field charging. Particles, 6bvz Qp;min
especially smaller ones, may also acquire charge via ion
particle collisions in the absence of an electrical field; this for a precipitator with a maximum dimension of b perpen-
is called diffusion charging. See Friedlander (1977) and dicular to the gas flow. (Here Qp, min is the charge on the
Flagan and Seinfeld (1988) for a comparison between the smaller particle size rmin.) The net vector of any particle
two processes.) path in such a case is linear (Figure 7.3.8(a)).
The movement of the negatively charged suspended Generally the charge on a particle of radius rp
particle toward the large grounded electrode takes place developed in an electrical field of strength Ec via the phe-
due to a large electrical field. This large electrical field may nomenon of field charging is given as (Flagan and Seinfeld,
be due to that between two large oppositely charged plates 1988)
through which the gas containing charged particles flows. 
3 p

Or the electrical field may simply be the corona between Qp 4 0 E c r 2p ; 7:3:32a
p 2
the negatively charged wire and the grounded electrode
(Figure 7.3.8(a)); this is identified as the single-stage or where 0 is the electrical permittivity of vacuum (8.854 
Cottrell precipitator. The former arrangement is usually 1014 coulomb/volt-cm (see text near equation (3.1.10c))
preceded by a first or corona stage to charge the particles; and p is the relative dielectric constant of the particle.
hence it is called a two-stage precipitator. The voltage used For a particle whose p >> 1 (conducting), the value of
in the two-stage precipitator varies between 10 and 15 kV, Qp is
whereas Cottrell precipitators employ voltages as high as
Qp 12 0 E c r 2p : 7:3:32b
30100 kV.
In an electrostatic precipitator, the path or trajectory of Flagan and Seinfeld (1988) have illustrated the charge of a
a suspended particle is determined by the nature of the particle of radius 0.5 m in an electrical field strength of
convective motion of the gas, the electrical force on the 500 kV/m to be 4.17  1017 coulombs:
particle and, in a limited way, by the particle size. Consider
first laminar flow of a gas containing charged particles in a coulomb
Qp 12   8:854  10 14
horizontal rectangular duct, as shown in Figure 7.3-8(a), volt-cm
which illustrates gas flowing at a constant velocity vz in the 500  103 volt
  0:52  10 8 cm2
z-direction parallel to the large electrodes. Assuming no 102 cm
slip between a particle and the gas, we observe that the 12   8:854  5  0:25  10 19 coulomb
migration velocity Upy of the particle toward the large
4:17  10 17 coulomb:
collecting electrode is perpendicular to the bulk velocity
Upz of the particle due to the axial gas motion. (Upz is also One can now write, from (7.3.31a),
essentially equal to gas velocity vz.) For a spherical particle
U py 2 0 E c Er p C c =: 7:3:32c
of radius rp and charge Qp in an electrical field of constant
strength E, the migration velocity Upy may be written, If Ec and E are provided in units of volt/meter, rp, the
according to equations (3.1.62) and (3.1.214a), as particle radius (in m) and , the gas viscosity (in kg/m-
hr), then Upy (in m/s) is provided by the following
Qp EC c Qp E expression:
U py d 7:3:31a
6r p fp
coulomb cm
U py 2  8:854  10 14  102
if Stokes law is assumed to be valid in the gas of viscosity volt-cm m
with the slip correction factor Cc included as submicron volt2 m 10 6 Cc s
 Ec E r p m 3600 :
particles are also involved. Such a particle will be captured m2 m kg hr
m-hr
by the large grounded electrode (of length L in the z-
direction) if it traverses a distance b (between the high- Knowing that 1 volt-coulomb 1 newton-meter and
voltage corona generating wires and the grounded collect- 1 newton 1 (kg-m)/s2, we get
ing electrode) in the y-direction in time t needed to tra-
6:37  10 14 r p m E c volt=m Evolt=m C c
verse axially the precipitator length L: U py m=s ;
kg=m-hr
 6 r p b 7:3:32d
t L=vz  b=U py

: 7:3:31b
Qp EC c
which is close to that recommended by Wark and Warner
For a gas stream with a particle size distribution (rmin (1976, chaps. 4, 5).
and rmax) and charge distribution, all particles will be Instead of a single set of corona wires in the
collected if duct between two collecting grounded electrodes
7.3 External force field based separation: bulk flow perpendicular to force 609

(Figure 7.3.8(a)), industrial practice often employs a pre- the suspension entering the device is pg pf . When 1,
cipitator shell containing a large number of parallel ducts, a particle arriving at the plate cannot reenter the gas phase.
with a set of corona wires in each duct and horizontal gas Soo (1989) had used the nondimensional variables
flow (Figure 7.3.8(b)). Optimization of the duct spacing in 
z z=b; y y=b; Pe bvz =Dp ; e Qp Eb=f dp Dp
 
such a configuration has been analyzed in White (1963).
Such optimization must consider the possibility of bridging 7:3:34d
of the collected dust layers within a gas flow channel. The
dust buildup on the wall near the bottom of vertical precipi- to obtain the following nondimensional form of equation
tators needs to be considered in any design of precipitators, (7.3.34a) and the boundary conditions:
especially in the multiduct design, to prevent short circuits.
pg pg 2 pg
In the analysis of dust precipitation in a duct, attention Pe 
e 2 : 7:3:35a
z y  y
has so far been paid to a single particle only. In a precipitator,
as the gas enters and flows horizontally, particles exist at all pg
At y 0; e pg ; 7:3:35b
locations of the cross section. One is then interested in y
finding out the density of particles as a function of two
pg
coordinates: z, in the direction of bulk gas flow, and y, normal at y 1; 1 e pg : 7:3:35c
y 
to it. To that end, for particles of size rp, a more general
analysis may be carried out to determine the fraction of An exact solution by separation of variables has been pro-
particles collected. Consider laminar gas flow using the par- vided by Soo and Rodgers (1971). The solution for small
ticle diffusion equation (6.2.55) in the gas in the presence of values of e and (Soo, 1989) is
an external force, no particle generation or agglomeration, !
dilute suspension and a quasi-stationary state: pg
1 e y exp e z =Pe: 7:3:36
mp ^   pf
v rpg d r pg F p r Dp rpg ; 7:3:33
fp The fraction jz of particles of size rp collected in an elec-
trostatic precipitator of length z is (it is also equal to the
where pg is the particle mass concentration in the gas. For
grade efficiency function Gr jz )
one-dimensional plug flow of the gas in the z-direction,
uniform electrical field of constant strength E in the y- 0b

1 01

1
direction, and a constant particle diffusivity Dp, the above @ pg vz dyA @ pg vz dy A
equation is simplified for particles of mass mp and radius rp 0 z 0 z
to (Soo and Rodgers, 1971) jz 1 0 b 1 1 01 1 G r jz :

@ pg vz dy A @ pg vz dy  A
2 pg
!
pg Qp E pg
vz d
Dp : 7:3:34a 0 z0 0 z  0
z fp y y 2
7:3:37
The boundary conditions for two locations, namely the Substituting the solution for pg , we get the following
wires (y 0) and the collector plates (y
b) are expression for z L (i.e. z L=b):
 
y0: Qp E=f dp pg Dp pg =y ; 7:3:34b
 
y
b : 1 Qp E=f dp pg Dp pg =y ; 7:3:34c

where is the sticking probability of the particle on


the collector plate. (The basis for the collector plate condi-
b b
tion is provided in the footnote.)20 The initial condition for
U pz pg dyjzz U pz pg dyjz
b

U pz pg dy lim 0 0
z z!0 z
20
Integrate equation (7.3.34a) from y 0 to y b (for example) to 0

obtain ve quantity rate of particle loss to wall


0 1
b
b BQp E C
!
Qp E n o
@ d Apg jyb vz pg dy:
vz pg dy pg jyb pg jy0 fp z
z f dp 0
0
Using boundary condition (7.3.34b) at y 0 and this result, we get
 
Dp pg =y jyb Dp pg =y jy0 :
 
Since vz U pz , we can make a total particle balance based on the 1 Qp E=f dp p jyb Dp pg =y jyb :
net rate of particle deposition at the wall:
610 Bulk flow perpendicular to the direction of force

81 9
< = g U py t: 7:3:40
 
1 e y dy exp e L=bPe
: ; The change in particle mass concentration, pg , as a result
jL 1 0 81 9 Gr jL ; of this capture from the value of pg may be described by
< =
 
1 e y dy pg Bp g Bp U py t
: ; ;
0 pg Ac Ac
Gr jL jL 1 exp e L=bPe: 7:3:38
where Bp is the perimeter of the duct since Bp g =Ac is really
 
Soo (1989) has provided the solutions for large values of the fractional cross section of the duct occupying the slow-
e as well as other types of conditions. Soo and Rodgers moving wall region of the gas. In this time interval t, the bulk
(1971) provide a general expression for jL as well as for gas moves an axial distance z, where z vz t. Therefore
pg y; z.
The previous analysis was based on laminar gas flow in pg Bp U py
z:
the precipitator ducts. In practice, turbulent flow exists. pg Ac vz
The complications introduced by turbulent flow for small
In the limits of pg ! 0 and z ! 0, we integrate to
dust particles may be appreciated if we consider first,
obtain
among others, the expression (7.3.32c) for the particle
migration velocity: Bp U py Bp U py
 
ln pg jpgpf L ) pg pf exp L :
Ac v z Ac v z
U py 2 o E c E r p C c =: 7:3:39
7:3:41
The magnitude of the particle migration velocity decreases
The fraction of particles collected in a precipitator of length
as the particle becomes smaller (see, however, the discus-
L can be obtained from above as (it is also equal to the total
sion following equation (7.3.42)). For smaller particles
efficiency ET)
( 1 m), Upy < 1 ft/s, whereas axial duct gas velocities vz
are generally of the order of 10 ft/s (White, 1963), between pf pg

Bp U py

1 and 20 ft/s (Wark and Warner, 1976). E T jL 1 exp L : 7:3:42
pf Ac vz
Secondly, the gas flow, and therefore the particle flow
or vector in turbulent flow, is highly irregular due to three- This result, known as the Deutsch equation, is identical to
dimesional flow fluctuations. Further, the magnitude of the equation (7.3.38) if the sticking probability 1 and
Ac =Bp b. Both equations point to certain characteristics
 
instantaneous velocity fluctuations in the y- and x-
directions are easily 1030% of the fluctuating axial gas of the precipitator behavior. Increases in particle migration
velocity vz. Such velocity fluctuations, especially in the velocity and precipitator length increase the extent of par-
y-direction mask the small y-directional migration vel- ticle capture, whereas an increase in gas velocity decreases
ocity due to the electrical field. Only if such turbulent- jL . Since Upy is proportional to the product of the electrical
flow fluctuations bring the small particle close to the col- fields EEc, an increase in the electrical field E and the par-
lector plates does the electrical field take over and the ticle charging field Ec will increase the particle charging
particle is captured, since the gas flow velocities are much efficiency. By the same argument, larger particles will have
smaller in the wall region. higher collection efficiency. Such a conclusion, however,
A simple model has been developed by Deutsch (1922) has to be tempered by the observation of a minimum in
to predict the extent of dust particle capture and conse- jL with respect to r p 0:5 m, possibly due to transition
quent gas cleanup in turbulent flow in a duct using a from diffusion to field charging (Friedlander, 1977). This
number of simplifying assumptions. value can be high, which is one reason why such devices
are quite useful for smaller particles unlike cyclones.
(1) Due to turbulent mixing, the particle concentration is
The quantity Bp L=Ac vz in the Deutsch equation
 
uniform across the duct cross section Ac at any axial
carries additional significance. It is essentially the ratio of
location.
the collecting electrode surface area over the gas flow rate.
(2) The gas moves in plug flow with a velocity vz along the
Thus, for identical values of Upy, similar particle collection
duct; however, close to the duct walls, the gas velocity
efficiencies will be obtained in two different size precipita-
is significantly reduced.
tors if the value of Bp L=Ac vz is identical. This provides a
 
(3) The particle migration velocity Upy due to the electrical
basis for precipitator scaleup. A survey of the volumes of
field in the region close to the duct wall is constant
electrostatic precipitators needed for required gas flow
independent of particle size, etc. All particles entering
rates is given in Figure 7.3.9 (Soo and Rodgers, 1971).
this wall region of thickness g will be captured.
The Deutsch equation is based on a highly simplified
Let the particle capture described in assumption (3) take situation where particle capture is dependent on its
place in time t. Therefore entering the wall region in a probabilistic sense. It does
7.3 External force field based separation: bulk flow perpendicular to force 611

not include effects of particle reentrainment, particle This allows the calculation of jL for any given f f r p . Such
agglomeration and other effects related to the corona. equations, however, have limited utility in actual design
Additional important effects are due to particle size distri- calculations.
bution, time-varying voltages applied to generate the elec- The fraction jL of particles collected is sometimes
trical field, displacements in corona wire locations, etc. To reported in terms of percent collection efficiency (after
the extent, the Deutsch equation describes a situation multiplication by 100). Values of such efficiencies in a few
where 1 (no particle reentrainment), to that extent it operating plants are indicated in Figure 7.3.9, which iden-
can describe wet precipitators, which are usually more tifies only a few major types of installations. The use of
efficient than dry precipitators (Jaasund, 1987). electrostatic precipitators is widespread; additional
We consider now briefly one of the effects not con- examples of where they are used are cement plants, in
sidered in the derivation of the Deutsch equation (7.3.42), the steel industry, in smelting operations, in the chemical
namely the presence of a particle size distribution. Let ff(rp) industry, the petroleum industry, in carbon black factories,
be the particle size density function and let Ff(rp) be the foundries, etc. In such industrial applications, industrial
corresponding distribution function of the dust cloud practice suggests techniques other than electrostatic pre-
entering the precipitator. Then cipitation for particles in the range 0.11 m since the
value of jL is <0.99 and decreases with a decrease in rp;
dF f r p f f r p dr p ; (Donovan, 1985; Eggerstedt et al., 1993).
where f f r p dr p represents the fraction of particles in the For turbulent gas flow in an electrostatic precipitator,
an alternative model may be developed by considering the
size range rp to rp drp entering the precipitator. Applying
flux of particles toward the wall due to eddy diffusion,
the form of the Deutsch equation (7.3.41) only to particles
Brownian diffusion and electrical migration:
of size rp having a migration velocity of U py r p , we may
 

write, from equation (7.3.41), 


p
np z DBr
p p U py p ; 7:3:47

Bp U py r p
 y
f 1 r p f f r p exp L 7:3:43
Ac v z where p is the time-averaged particle density at any (y, z)-
location.
for the fraction of particles between size rp and rp drp in
the exiting gas. Integrating over all particle sizes between 0
and rp, we get
7.3.1.4 Electrostatic separation of a particulate mixture of
rp rp
Bp U py r p different plastics/minerals
 
F 1 r p f 1 r p dr p f f r p exp L dr p ;
Ac vz In the electrostatic precipitator studied in Section 7.3.1.3,
0 0
7:3:44 the objective was to separate all particles present in the gas
stream; this is achieved by creating an electrical field per-
where F 1 r p is the fraction of particles in the exiting gas pendicular to the main gas flow and ensuring that the
having sizes between 0 and rp. When rp tends to , equa- particles become negatively charged for removal in the
tion (7.3.44) yields the fraction of all particles which electrical field. In electrostatic separation of a mixture of
entered the precipitator escaping the device of length L. plastics (or mineral) particles which are of relatively large
Therefore the fraction of particles collected in the precipi- dimensions, the particles first undergo triboelectric
tator of length L is (it is also equal to the total efficiency ET) charging in a charger (particle-charging device) where
the particles contact one another; as a result, electrons
are transferred from one particle to another in contact
Bp U py r p
 
E T jL 1 F 1 r p j0 1 f f r p exp L dr p : (a sort of charge exchange between different material par-
Ac vz
0 ticles). For example, triboelectric charging can change the
7:3:45 sign of the charge on polyethylene (PE) particles in a
More realistically, the limits of the integral are rmin and rmax mixture of particles of the plastics polyvinylchloride
for the minimum and maximum particle sizes. (PVC), PE and polystyrene (PS). When different particles
The integral expression (7.3.45) has an exponential having different charges are exposed to an electrical field
term containing U py r p . One could, in principle, employ via a high voltage (3060 kV), particles of different plastics
the expression (7.3.39) for Upy and rewrite jL as (or minerals) can have different trajectories; therefore dif-
2 3 ferent particles of different plastics arrive at different loca-
 
2B L E EC
  tions in the separator and can potentially be separated
p 0 c c
jL 1 4 f f r p exp r p dr p 5 E T : (similarly for minerals). The roles of the particle-charging
Ac vz
0 device (e.g. cyclones, rotating drum, fluidized bed, vibra-
7:3:46 tory feeder) and method, including humidity, pretreatment
612 Bulk flow perpendicular to the direction of force

107
Present installations (efficiencies in %)
Power plants 98%
Passage
Overall 98%

Incinerators 97%
Passage 92.5%
105 Overall 99.7%
Air conditioner 98.9%
Passage

Average for
Volume (ft3) 103
conventional
precipitators 95%

92%
101

101
101 103 105 107
Gas flow (cfm)

Figure 7.3.9. Survey of size of electrostatic precipitators vs. flow capacity. Reprinted from Powder Technol., 5, 43 (1971), S.L. Soo, L.W.
Rogers, Further studies on the electro-aerodynamic precipitator. Copyright (1971), with permission from Elsevier.

using surfactants, etc., are crucial in introducing an appro- the left side bin (for which y < b1,) the right side bin (for
priate feed mixture to the electrostatic separator. Such sep- which y > b2) and the middle bin (b1  y  b2). The left
arators are very useful in separating mixtures of different side vertical plate is at a voltage V volts; the right side
minerals (plastics for recycling), especially when their vertical plate voltage is at V volts. Therefore the electrical
densities are close. Refer to Wei and Realff (2003) for field strength (voltage gradient) in the y-direction for the
appropriate references in the area of plastics separation. parallel vertical plate configuration is
For mineral separation, see Inculet (1984).
V V 2V
There are different types of electrostatic separators, e.g. Ey ; 7:3:48
y c c
rotating drum, free-fall, etc. We will focus on the free-fall
electrostatic separator here. Figures 7.3.10(a) and (b) illus- which is a constant. However, Ey for the diverging vertical
trate, respectively, two different configurations: parallel plate configuration will vary with the y-coordinate. We will
vertical plates and diverging vertical plates (inclined at an focus first on particle trajectories in these two configur-
angle to the vertical direction). The plates have length L ations. We will then very briefly consider particle recoveries
and the gap between the plates at the feed location is c for at different locations of the separator exit influenced by
both the vertical plate as well as the diverging plate config- particle charge-to-mass distributions at the separator inlet.
uration. The origin of the (y, z)-coordinates is at the bottom Consider the configuration of parallel vertical plates
of the plates. There are three bins for collecting particles: to start with, where Ey is a constant. The properties that
7.3 External force field based separation: bulk flow perpendicular to force 613

(a) (b)

Feeder Feeder

a a a a
c c

+V V +V V

L
L
L
L

Plate Plate
q z q
z y
y

b1 b2 b1 b2
Left Right
bin bin

(c) (d) Feed


Tubes
+V

Brush
R1 +V V
R2
V

b1 b2

Figure 7.3.10. (a) Free-fall electrostatic separator with vertical plates; (b) free-fall electrostatic separator with inclined vertical plates.
(After Wei and Realff (2003).) (c) Drum-type electrostatic separator; (d) tubular free-fall separator. (After Beier and Stahl (1997).)

vary from particle to particle are charge, mass, shape and Let a particle start falling at t 0 from coordinates
size. It has been found that the effect of the drag force y y0 and L z 0 at the entrance of the separator. The
due to air through which the particles are falling is very two governing equations for particle trajectory coordin-
small and may be neglected. Therefore the particle shape ates (y, Lz) are (Figure 7.3.10(a)) (from equations
and size are unimportant. This, and a number of other (6.2.47c,b))
assumptions (described below), are integral to the par-
d2 L z
ticle trajectory analysis that follows. The Coulombic force mp mp g; 7:3:49a
dt 2
of interaction between two particles is neglected com-
pared to the electrostatic force on a particle. The effects d2 y
mp E y Qp ; 7:3:49b
of interparticle collisions are negligible. The walls of the dt 2
vertical plates are inelastic: any particle colliding with since drag forces are negligible. From the first equation, we
a vertical wall drops into the local bin. The plate get, after integration,
length/vertical particle fall height (L for parallel-plate
dL z gt 2
configuration and L cos for the diverging-plate config- gt a1 ; Lz a1 t a2 :
dt 2
uration) is much larger than the gap between the plates,
c. The electrical field Ey is unaffected by the charge Qp on At time t 0, L z 0 ) a2 0. At time t 0, the vertical
the particles. velocity of the particle is zero; therefore a1 0. We get
614 Bulk flow perpendicular to the direction of force

gt 2 One has to determine this quantity first. This is done, in


Lz : 7:3:50
2 general, by solving Poissons equation (Newman, 1973,
1991),
From the second equation, (7.3.49b), we get
   r d E r d r e ; 7:3:57
d2 y 2V
E y Qp =mp Qp =mp : where e is the electric charge density per unit volume and
dt 2 c
d is the dielectric constant of the medium. Since the
Integrating, we get dielectric constant of the medium is constant and, by our
  .  assumption, Ey is unaffected by the charge on the particles,
dy 2V the above equation is reduced to the Laplace equation in
Qp mp t a3 ;
dt c (y, z)-directions:
  .  2
2V t e
y Qp mp a3 t a4 : r r 0 ) r2 0;
c 2 d

At t 0, y y0, therefore21a4 y0. Further, at t 0, 2 2


2
0: 7:3:58
dy=dt 0, leading to a3 0. Therefore, we get y z2
  .  2
2V t Since the plate height (in the z-direction) is much larger
y y0 Qp mp : 7:3:51
c 2 than the gap between the plates, the second term may be
neglected. Therefore
It is of interest to know what is the particle trajectory,
i.e. the relation between y and (L z) as t increases. 2
From equation (7.3.50), we get t in terms of (L z); 0: 7:3:59a
y2
therefore,
  .  The boundary conditions are as follows:
2V L z
y y0 Qp mp : 7:3:52
c g c
at y L z sin : V;
2
If we nondimensionalize the two Cartesian coordinates 7:3:59b
c
(y, z) via at y L z sin : V:
2
y y=L and z z=L; 7:3:53

then Introducing the nondimensional variables


    . =V ; y y=L cos ; z z=L cos and
2V

y y
0 Qp mp g 1 z ; 7:3:54
c c c=L cos 7:3:59c

which is the desired trajectory equation relating y with (L into equations (7.3.59a,b), we get
z), the two coordinates characterizing the particle position
2
at any time t; the corresponding nondimensional coordin- 0: 7:3:60a
y2
ates are y and (1 z). At the bottom of the two plates
(z 0, z 0), At y c=2 1 z tan : 1;
7:3:60b
n o at y c=2 1 z tan : 1:
y y0 2V =cQp =mp g : 7:3:55
The solution of equation (7.3.60a) is
Consider now the second configuration (Figure 7.3.10(b)),
a5 y a6 :
where we have two diverging plates of length L at an angle
to the vertical. The first item to note here is that the
Applying the two boundary conditions, we get, at any z,
electrical field strength in the y-direction is no longer a
constant: 2
a6 0 and a5 :
c 21 z tan

Ey not a constant: 7:3:56
y Therefore

y
7:3:61
c =2 1 z tan
21
Here y0 is a random variable. A particle can be anywhere at t 0 and
along the y-coordinate and z L.
7.3 External force field based separation: bulk flow perpendicular to force 615

y at t 0; y y 0 ; y y 0 =L cos y
0;
Ey
y y y
also
1 1
V
c =2 1 z tan L cos at t 0; dy=dt 0; dy =dt 0: 7:3:68
V 1 The solution of equation (7.3.67) subject to these two
:
L cos c =2 1 z tan conditions (Wei and Realff, 2003) is
7:3:62 p p
y y a8 t arctan a8 t n1 a8 t 2 :

0 2a7 =a8

Since we have an expression for Ey, we can now substitute 7:3:69


it into equation (7.3.49b):
The particle trajectory coordinates y and z are therefore
d2 y V Qp =mp 1 related via
:
dt 2 L cos c =2 1 z tan hp p
y y 0 2a7 =a8 a8 1 z arc tan a8 1 z
Introducing a nondimensional time t via p i
n f 1 a8 1 z g : 7:3:70
t t=2 L cos =g1=2 ; 7:3:63
The y-coordinate of the particle at the bottom of the sep-
we get
arator is of greatest interest. For this location, z 0, z 0:
0 12 p p

p
d2 y @dt A V Qp =mp 1 y y 0 2a7 =a8 a8 arctan a8 n 1 a8 :
L cos ;
dt 2 dt L cos c =2 1 z tan 7:3:71

d2 y 2V Qp 2 2 a7 Note here that the constant a8 has only the geometric


; dimensions of the separator, but that the constant a7 has
dt 2 c mp g 2 L cos 1 a8 1 z
1 1 z tan
d dimension (Qp/mp), the particle charge per unit particle
mass. The particle charge-to-mass ratio is a random vari-
7:3:64 able; the initial particle coordinate at the start of the fall, y0,
where is also a random variable. The situation is identical in the
case of the separator with vertical plates; see equation
2 Lsin (7.3.55).
a7 f2V =cQp =mp gg; a8 :
c We will not develop the details of a particle recovery
We now need to solve the other equation, (7.3.49a). model here beyond introducing the basic approach (for
However, the particle trajectory coordinates here are (y, L details, see Wei and Realff (2003)). For the free-fall electro-
cos z) (from Figure 7.3.10(b)). Therefore, the corres- static separators shown in Figures 7.3.10(a) and (b), one
ponding equation is would like to determine the probability that the final par-
ticle position is either y < b1 (corresponding to the left bin)
d2 L cos z or y > b2 (corresponding to the right bin), since the prob-
mp mp g: 7:3:65a
dt 2 ability of going to the central bin is obtained by subtracting
The corresponding solution is the two above-mentioned probabilities from 1. For the left
bin (y < b1) at the separator exit, from result (7.3.71) we
L cos z gt 2 =2: 7:3:65b can write, for particles being treated in the separator of
Figure 7.3.10(b),
When nondimensionalized, this expression is reduced to
8 n o
1 z t 2 ; 7:3:66 < 2V Qp =mp g
b1 > y y 0
tan
a result which allows us to rewrite equation (7.3.64) as
:
)
p p p

d2 y 2a7 a8 arctan a8 n 1 a8 : 7:3:72


; 7:3:67
dt 2 1 a8 t 2

which is an ordinary differential equation of y in One needs to know the following probability to determine
terms of t. Since t is directly related to z via the fraction collected in the left bin:
(7.3.66), a solution of this equation will yield the
2V Qp p p

 
required relation between y and z, the particle trajec- p
Pr y0 a8 arctan a8 n 1 a8 < b1 :
tory coordinates. The two conditions needed to solve this mp g tan
equation are: 7:3:73
616 Bulk flow perpendicular to the direction of force

For the separator of Figure 7.3.10(a), one needs to know, location y0 and a particle charge-to-mass ratio (Qp/mp) will
correspondingly, the probability of (from result (7.3.55)) be the product of the two independent probabilities,
2V Qp L
 
Qp =mp Qp =mp 2
( )
Pr y 0 < b1 : 7:3:74 1 1
c mp g f y0 f Qm  p exp :
2a 2 Qm 2 2Qm
For both cases ( > 0, 0), one can express the prob- 7:3:81
ability (alternatively, the fractional recovery, re) as
n o The probability of finding all particles in the left bin (there-
Pr Qp =mp < b1 y 0 a9 ; 7:3:75 fore the fractional particle recovery, re) can be obtained by
integrating this joint probability density function over the
where, for > 0,
required limits:
g tan
a9 p

 
p p Qp =mp <b1 y 0 a9
2V a8 arctan a8 n 1 a8 rePr
b1 a<y 0 <b1 a
2 820 1 0 132 9 3
and, for 0, >
> Qp
>4@ A@ A5 >
Qp >
>
>
b1a
6
6b1 y0 a9
> 0 17
cg
> mp mp >
Qp 7
7
16 1
>
< >
=
a9 : 7:3:76
6
p exp d 7dy
7
mp 7 0
@ A
2V L 2a 6 2 Qm 2 2Qm
6
>
> >
>
b1 a 6
4
>
>
>
>
>
>
7
5
Now for the right bin (y > b2) for  0, we can similarly
>
: >
;

obtain the probability as b1a


2 0 1 0 1 3
1 4 0 @ Qp A b1 y0 a9 @ Qp A5

Id Id dy 0 ;
8 8 n o 2a mp 0 mp
< < 2V Qp =mp g b1 a
Pr y y 0 7:3:82
: : tan
) )
p p p
where I is the integrand describing the Gaussian distribu-
a8 arctan a8 n 1 a8 > b2 ;
tion. Using the variable transformation
7:3:77 Qp =mp Qp =mp
;
which is reduced to 2 Qm
p
n o
Pr Qp =mp > b2 y 0 a9 : 7:3:78 the integral between the limits and 0 could be changed
to the limits of 0 to (see equation (2.5.13) for erf(x)):
Now consider the case of the fractional recovery of the left
bin (y < b1). The probability of a particle coming into this b1a " b1 y0 a9  #
1 1 Qp
bin has been expressed via (7.3.75), where re Id dy0 :
2a 2 0 mp
b1 a
Qp =mp < b1 y 0 a9 :
This integral reduces to (Wei and Realff, 2003)
We have two density functions (or probability func-
tions) to deal with. First, the initial (starting coordinate) 1 g 2 g 1
re ; 7:3:83
particle coordinate y0, which can be anywhere between a 2 2g 2 g 1
and a; therefore the quantity (b1 y0) can vary between where
b1 a and b1 a. Further, this probability is uniform: any
starting location is as probable as any other starting loca- exp 2
erf p 7:3:84
tion. Consequently, the probability density function for the
initial particle location is
and
1 1
f y0 : 7:3:79 b1 aa9 Qp =mp b1 aa9 Qp =mp
b1 a b1 a 2a g1 p ; g2 p :
2 Qm 2 Qm
Second, the other important random variable is the particle 7:3:85
charge per unit particle mass, Qp/mp, which can be
assumed to vary as a Gaussian distribution: Related expressions for the fractional recovery in the right
and the middle bins are available in Wei and Realff (2003).
Qp =mp Qp =mp 2
( )
1 A few details about the process and the device (Wei
f Qm p exp ; 7:3:80
2 Qm 2 2Qm and Realff, 2003) are useful. The voltage V applied between
the plates can be as high as 5080 kV. The electrical field
where Qp =mp is the mean of the distribution and 2Qm is strength for the plate gaps used is of the order of 4  105
the variance. The joint probability of an initial particle volts/m. A typical estimate of Qp/mp is 3  106 coulomb/
7.3 External force field based separation: bulk flow perpendicular to force 617

kg. The plate angle varies between 0 and 15 . The plate relative humidity. The charged salt mixture is then separ-
lengths are in the range of 12 meters, while the plate ated by letting the mixture fall through a vertical charged
widths (perpendicular to the plane of the paper in Figure separator. Multiple stages are used to process various frac-
7.3.10(a) and (b)) are ~1 meter. The gap between the plates tions into increasingly pure fractions. The conditioning
is (L/3). agents role is quite important. Beier and Stahl (1997) have
An important issue is the charge developed in various shown that aliphatic monocarboxylic acids allow one to
components of a plastic mixture. Polyethylene (PE) and recover almost all of KCl in a mixture of KCl, NaCl and
polyvinylchloride (PVC) are very common plastics. In a MgSO4 H2O in the positive electrode, with very limited
copper-lined cyclone based tribocharging device, Yanar recoveries of NaCl and MgSO4 H2O. The addition of
and Kwetkus (1995) showed that PE particles develop posi- aromatic monocarboxylic acid or NH4 salts of aliphatic
tive charges and that PVC particles develop negative monocarboxylic acids leads to different patterns of
charges; they then demonstrated successful separation: recovery.
PE fraction contained >90% PE at a recovery of >60%. In the above examples, for the successful separation of
Wei and Realff (2003) have discussed two-stage operations minerals at the electrodes, a different arrangement is used.
with or without recycle to improve purity and considered A major problem is the adherence of dust/particles on the
several design options. They have also illustrated the electrodes in the configurations of Figures 7.3.10(a) and (b)
charging tendencies of different plastics via a triboelectric which reduces the field strength. Instead, a number of
series of plastics. vertical tubes standing side by side constitute the elec-
There is another useful geometrical configuration of an trode; however, these tubes slowly rotate around their axis,
electrostatic separator for separation of plastic particles: and any adhering dust/minerals on their surfaces are
the drum-type separator shown in Figure 7.3.10(c). There scraped away by brushes at the back (Figure 7.3.10(d)).
is a rotating drum charged with a high voltage (V) and an
outer plate also charged with a high voltage (V). As
7.3.1.5 Separation of cells using flow cytometry
shown, the outer plate is curved, but it can be straight as
well. Particles coming over the belt are subjected to three Flow cytometry is a successful technique for cytometry
forces as they approach the angular location of the outer (i.e. cell measurements) when a suspension of cells in a
plate: the electrostatic force due to the electrostatic poten- liquid stream flows through the device and the cells are
tial gradient between the two electrodes, a centrifugal force separated one by one. Cells of interest introduced via a
and the gravitational force. If the net outwardly radial force sample injected into the core liquid stream surrounded by
is positive for a particle, it will become detached from the a much thicker liquid flowing stream (called the sheath
belt. The location and force of detachment will determine flow) in laminar flow are analyzed via their optical signals,
the particle trajectory and therefore the collection bin usually by a laser beam (Figure 7.3.11). Prior to the sample
where it lands. injection, fluorescent dyes are introduced into the mixtures
The use of electrostatic separators for mineral particle of cells; particular cells are tagged with particular fluoro-
separation is common. It is primarily used for separating chromes. The laser beam interrogates individual cells as
mineral ores, ore tailing, pulverized coals containing clay they flow downward. The cells emitting the desired signals
particles (Inculet, 1984), coal ash, etc. For example, tribo- are identified extremely rapidly and then tagged individu-
electrification of coal ash from power plant exhaust leads ally in the following fashion. The liquid jet/stream beyond
to two fractions: carbon-rich material, which becomes the laser beam sensing zone is broken down into tiny
positively charged, and the rest of the ash, which is nega- droplets (Figure 7.3.11). Knowing which droplet contains
tively charged (Holusha, 1993). Similarly, high mineral a particular cell, the system introduces a charge on such a
matter containing coal will be separated via triboelectrifi- droplet; then, by electrostatic forces generated by charged
cation into a positively charged coal-rich fraction and a deflection plates (as in Section 7.3.1.4), the droplet con-
negatively charged mineral-rich fraction. taining a particular cell is directed to the right or left by a
Very-large-scale separation of raw salts by electrostatic certain distance and collected in several containers. The
separators is practiced at the level of 900 tons/hr (Beier and central container is for the liquid waste stream consisting
Stahl, 1997). In these processes, used for the separation of of droplets (without or with cells) having no charge on
the potash ore for example, the ore is first ground to free them since they are not of interest. For a comprehensive
the minerals of individual crystals, then the salt crystals introduction to flow cytometry, see Shapiro (1995). For
undergo a conditioning treatment, where they are coated shorter accounts, consult Hoffman and Houck (1998) and
with a reagent (or a combination of the reagents), followed Robinson (2004).
by warming and adjustment of the relative humidity, which A few more details about the technique will provide a
allows charge exchange to take place. The chemical condi- better prespective. The cell population has different sub-
tioning and humidity treatment are generally carried out in groups having (potentially, for example) different antigens
a fluidized bed using an air stream having the required on the cell surface. The sample to be analyzed is treated
618 Bulk flow perpendicular to the direction of force

Core liquid stream The presence of the desirable cell triggers droplet forma-
Sample introduced here tion, which can enclose an undesirable cell. Hoffman and
Sheath flow Houck (1998) have summarized the different modes of
Sheath flow
sorting, not only for droplet sorters, but also for other
enclosed sorting methods, e.g. catcher-tube sorting,
fluidic-switching sorting (here droplets are not generated
Cells from the sample stream; rather when there are desired
cells, a catcher tube catches it, etc.). These modes are:
(a) single-cell mode: reject if there is more than one cell
even though both may be desirable;
Core
Sheath (b) enrichment mode: collect as long as there is a desir-
able cell, even though there may also be an undesir-
Laser beam able cell;
(c) exclusion mode: reject when there is an undesirable
Charged cell; collect even if there are two desirable cells.
+
+ + + deflection
- It would be useful to illustrate quantitative measures of
plates
purity of the sorted cells with respect to the rate, ra, at
which cells appear in the sensing zone, the time period, T,
Collection bins Waste Collection bins
during which a cell can be sorted for selection or rejection
and subfr, the fraction of the subpopulation of desirable
Figure 7.3.11. Schematic view of a flow cytometer based on
cells to be sorted. The results, illustrated in Hoffman and
fluorescence-activated cell sorting via droplet sorting.
Hauk (1998), based on analyses available in literature are as
follows. The efficiency, E, of sorting cells is defined as the
(stained) with, say, a number of different antibodies (Abs) fraction of desired cells passing through the sensing zone
which bind with specific antigens (Ags) on the surface of that are captured/sorted since sometimes the desired cells
the cells. The antibodies are labeled with (e.g.) specific may not be sorted due to the presence of undesired cells.
fluorochromes, each of which provides a distinct color
(i) Single-cell mode:
and therefore a distinct identity to the laser probe, of which
there may be more than two in some systems. The analysis E exp raT : 7:3:86
of the spectral emission from the cell surface is carried out
(ii) Exclusion mode:
extremely rapidly, at rates as high as 10 00025 000 cells per
second. To ensure that the fluid flow regime is correct, E exp 1 sub fr raT : 7:3:87
laminar flow is maintained. The fluid jet diameter varies
between 50 and 400 m. The frequency of droplet forma- Illustration of the values of the parameters is useful. The
tion varies between 2000/s and 100 000/s. Vibration at time period, T, may vary from 25 to 200 s. As T increases,
frequencies of 10 000300 000 Hz via a piezoelectric crystal E decreases substantially since unwanted cells appear
oscillator is most often used to produce drops. Drops of more often. As the rate of arrival, ra, of cells to the sensing
interest are imparted positive or negative charges. The zone increases (ra varying between, say, 300 cells/s and 10
voltage applied between the two plates can be as high as 000 cells/s or more), the efficiency decreases. As the value
5000 volts to separate various drops. The technique is often of the subpopulation fraction, subfr, decreases, the effi-
identified in general as flow sorting via droplet sorters, ciency decreases, reaching a limit when subfr ! 0.01. The
sorting out one type of cell from another via different purity of the droplets sorted out depends on the probability
droplets. The overall technique is also known as that no other type of cell appears along with the sorted cell
fluorescence-activated cell sorting (FACS). in the droplet sorter or other sorters. Expressions (7.3.86)
The efficiency of cell sorting or cell purification has two and (7.3.87) provided above for the efficiencies are also
aspects: (1) How pure is the cell sorted in a particular used to estimate the purity of the cells in the sorted droplet
droplet? (2) What is the rate at which this sorting is taking population.
place? For the technique to be successful, the rate of cell
sorting has to be reasonably high; unduly slow techniques
7.3.2 Centrifugal force field
are not acceptable. High rates of cell sorting can lead to
situations where a cell that is undesirable appears in the Large-scale devices in industry employ centrifugal forces to
same drop with a cell that is desirable; this happens separate particles from a fluid or droplets from an immis-
because of the presence of an undesirable cell with a cible continuous phase. Usually, these large devices are
desirable cell in the liquid in the laser interrogation zone. single-entry separators: a multiphase feed stream enters
7.3 External force field based separation: bulk flow perpendicular to force 619

the separator and two multiphase product streams, an The feed liquid is assumed to be uniformly distributed
overflow and an underflow, leave the equipment. Centri- at the bottom of the centrifuge (i.e. z 0) from r rf to r0.
fuges of much smaller dimensions are used for preparative Therefore, particles of all sizes found in the feed suspen-
work or in analytical studies. The centrifugal force is sion are present at all radial locations (between r rf to
imparted to the multiphase fluid by rotating the whole r r0) at the centrifuge bottom (z 0). However, as soon
device around an axis or by imparting a swirl to the feed, as the particles are at various r-values at z 0, they are
introduced tangentially to a stationary cylindrical device, as immediately subjected to sedimentation. The particles are
in a cyclone or hydrocyclone separator. For a review, see thrown toward the bowl wall by the centrifugal force as
Svarovsky (1977, 1981) and Hsu (1981). they move vertically upward (z > 0) due to the axial vertical
In all such separators, the multiphase fluid has a tan- motion of the liquid. (The circumferential liquid velocity
gential motion as it rotates around a central axis. However, due to the rotation is not important in itself except for the
there exists an axial movement, which is superimposed on calculation of the centrifugal force on the particles.)
the primary circulatory motion, which generates the cen- Since each particle has a velocity in the z-direction due
trifugal force. This axial movement is essential to continu- to the z-directional liquid motion, as well as a centrifugally
ous removal through an outlet of the entering fluid from generated radially outward velocity, it follows a particular
the centrifugal separating equipment. More important, path. These paths are identified in Figure 7.3.12 as effective
from a separation point of view, is that this bulk motion particle trajectories. If the trajectory of a particle hits the
is perpendicular to the direction of the centrifugal force walls of the centrifuge (i.e. r r0) before z L, then the
generated by the rotation of the multiphase fluid around particle is separated from the liquid and is deposited along
the main axis. This enhances the multicomponent separ- the wall. When the deposit becomes thick, the centrifuge is
ation capacity considerably for larger-scale operations. stopped to remove the particle deposits, since a thick
However, this capability is generally unutilized since deposit reduces the cross-sectional area for liquid flow.
industrial processing focuses more on particle separation This particle deposit may be considered as the underflow
from a fluid or phase separation from a feed which is a in a continuous single-entry particle fractionators, even
multiphase dispersion. Note that, even without the axial though, on an extended-time basis, the operation is
bulk flow, the primary circulatory flow is perpendicular to intermittent.
the direction of the centrifugal force. A particle trajectory is a series of (r, z)-coordinates
In this section, particle separation from a liquid having occupied by a particle as it moves in the centrifuge.
one component of bulk motion along the axis of a tubular A particle trajectory, whose r value is less than r0 but
bowl centrifuge is studied first. Following the particle sep- z L, will not hit the centrifuge wall at r r0; this particle
aration, the separation of two immiscible liquids of differ- will leave the centrifuge with the liquid overflow and is not
ent densities is briefly considered. The performance of a captured by the device. A separation analysis should be
disk centrifuge is treated next; here the bulk liquid motion able to predict the particle size density function, f0(rp), in
toward the outlet is not perpendicular, but is at an angle to the liquid overflow for a given feed liquid particle size
the centrifugal force. Separation of particles from a gas in a density function ff (rp). Alternatively, the separation analy-
cyclone separator is then considered, followed by brief sis should provide an expression for the grade efficiency
treatments of the separation nozzle process for gas separ- function, Gr(rp) of the device.
ation and hydrocyclones. Separation analysis in a centrifuge for particles begins
with a particle trajectory analysis. This is conveniently
initiated by following the z-coordinate and the r-
7.3.2.1 Tubular bowl centrifuge
coordinate of the particle with time. Assuming that there
Consider first particle separation from a liquid suspension is no slip between the liquid and the particle, the z-
in a simple tubular bowl centrifuge; a schematic of this direction particle velocity (Upz) is equal to that of the
centrifuge is shown in Figure 7.3.12. It is essentially a liquid, vz(z, r). Therefore,
cylindrical bowl rotating around its axis at a high rpm
U pz dz=dt vz z;r: 7:3:88
(revolutions per minute). Smaller centrifuges may have
rpms as high as 15 000. A dilute feed suspension is intro- If Qf is the volumetric flow rate of the liquid, and the liquid
duced through a central nozzle at the bottom. The feed is assumed to have plug flow between radial locations r0
suspension may be assumed to attain the angular velocity and rf, then
of the bowl very quickly as it spreads out from the feed 
nozzle to the bottom of the bowl at z 0. However, the U pz dz=dt vz z;r vz Qf = r 20 r 2f : 7:3:89
feed liquid moves axially upward from z 0 to z L, the
length of the centrifuge. The radial location of the free For the r-direction motion of a particle of radius rp,
surface of this liquid, rf, is set essentially by the liquid outlet the general equation of particle motion (equation
weirs at the top of the bowl. (6.2.45)) is
620 Bulk flow perpendicular to the direction of force

Outlet weir
Outlet weir w

Free surface
of liquid
Liquid overflow Liquid overflow
with fine particles with fine particles

r0 r0

L
Effective particle Effective
rf rf
trajectories particle
trajectories
r
z r
Deposited solid
Deposited solid particles
particles
z=0

Dilute feed
suspension in

Figure 7.3.12. Schematic for particle separation from a liquid in a tubular bowl centrifuge.

dU pr d2 r dz 9Qf
mp mp 2 F ext drag
pr F pr : 7:3:90a : 7:3:92
dr 2 r 2 2 r r 2 r 2

dt dt 0 p t
p f
Here centrifugal force is the external force along with the
centrifugal buoyancy term (see equation (3.1.59)) and its Rearrange it and integrate between (r, z 0) and (r r0,
magnitude is given by mp r2 1 t =p . If the size of the
   z L) to obtain
particle (assumed spherical) is small enough for Stokes law to r0
 
dr r  2 r 2p 2 p t r 20 r 2f
be valid, the radial resistive drag force magnitude is 6 r p vr . ln
0
L: 7:3:93
If we further assume that the particles settle always with their r r 9Qf
r
terminal radial velocity (i.e. dU pr =dt d2 r=dt 2 0 is
   

achieved very quickly), we obtain Since r and z are the coordinates of a particle of radius
! rp and density p as it moves along its trajectory, equa-
t tion (7.3.93) defines the smallest value of the radial
F ext
pr m p r 2
1 6 r p U pr F drag
pr : 7:3:90b
p location r of the particle at z 0, where it must be
located if it is to hit the wall (r r0) and be captured
This leads to an expression for the radial particle terminal when z L. If the particle is located at a value of r
velocity: greater than that defined by equation (7.3.93) at z 0,
2 2 then it will surely hit the wall (r r0) before z L.
dr 2r p r 
U pr U prt p t : 7:3:91 However, if the particle at z 0 is located at an r smaller
dt 9
than that defined by equation (7.3.93), it will not hit
(Note: If gravity is the external force, Upz is obtained by the wall (r r0) by the time z L; then the particle
replacing 2 r by g; U pr jcentrifuge U pz jgravity 2 r=g, where escapes with the overflow. A critical radius r is thus
the ratio 2 r=g is called the centrifuge effect.) Combining identified with each particle size rp via equation
this equation with equation (7.3.89), one gets (7.3.93).
7.3 External force field based separation: bulk flow perpendicular to force 621

An alternative form of equation (7.3.93) is also quite


2 3 ,
r 20 r 2 r 2 r 2
useful. Suppose the particle starts out at r rf when z 0. G r r p Qf f p f f r p Qf f p f f r p 02 2 ;
 
4 5
r 0 r f

Then, if its r r0 when z L, r 20 r 2f

  2 r 2 2
  7:3:97
r0 max p t r 20 r 2f
ln L; 7:3:94 where r for any rp is to be obtained from the relation
rf 9Qf
(7.3.93) as
where rp is identified as rmax. This is the largest particle size 0   1
with any nonzero probability of not being captured. Any 22 p t r 20 r 2f L
2A
r r 0 exp r p : 7:3:98
particle with a smaller size (<rmax) and starting at r rf
@
9Qf
and z 0 will not be captured by the time z L, and
therefore must appear in the overflow stream. Any particle From equation (7.3.94), the value of Gr(rp) 1 when
of size r  r max must settle in the centrifuge. r p  r max . Using (7.3.98), we get, from relation (7.3.97)
There is another way of considering the general ques- for Gr(rp),
tion of particle settling in the centrifuge. Since the particle  
velocity in the z-direction is assumed to be equal to the z- r 20 r 2f Gr r max r 20 r 2f
direction liquid velocity, the maximum time available for 2 8   93
< 42 p t r 20 r 2f L
settling is equal to the liquid residence time
=
24 2
r 0 1 exp r max 5:
 : 9Qf ;
t res L=U pz L r 20 r 2f =Qf V eff =Qf ; 7:3:95
Therefore
where Veff is the liquid-filled volume of the centrifuge. 8  93 
Knowing the radial distance to be traversed by a particle
2
< 42 p t r 20 r 2f L
=
starting out at any r and z 0, the particle settles if tres is 41 exp 5 r 2p
: 9Q f ;
greater than or equal to the time required by the radial Gr r p 2
 
8   93 ;
particle terminal velocity to reach r r0. < 42 p t r 20 r 2f L =
2
We calculate now the grade efficiency function Gr(rp) 41 exp r max 5
: 9Qf ;
of the tubular bowl centrifuge. Since the z-directional
liquid motion is assumed to be of the plug flow type, the 7:3:99a
following equality holds:
4 2 p t r 20 r 2f L 2
" ( )#
r2

liquid flow rate through an annulus between r 0 and r
 Gr r p 2 0 2 1 exp rp
r 0 r f 9 Qf
liquid flow rate through an annulus between r 0 and r f
7:3:99b
r2 r2
 
0 : 7:3:96 for 0  r p  r max (Svarovsky, 1977, p.130).
r 20 r 2f
The above analysis was based on the assumption of
For a particle of size rp (uniformly present in the entering plug flow of liquid in the z-direction. Schachman (1948)
liquid at all radii between rf and r0) to settle by the time z analyzed the same problem using a velocity profile for
L, it must be at a radius larger than or equal to the r given annular axial flow of the liquid between rf and r0. The drag
by equation (7.3.93). Thus particles of size rp which are in force experienced by the spherical particle was obtained
the liquid flow cross-sectional area r 20 r 2 to start with
  above from Stokes law. For larger particle sizes or higher
(z 0) will settle. Let the total volume fraction of particles radial velocities, the resistive radial drag force may be
in the feed liquid be f and the particle size distribution in expressed by (see equation (3.1.64))
the feed liquid be ff (rp) (based on mass fraction). Then the t U 2pr
mass flow rate of particles of size rp entering the centrifuge F drag
pr C D Ap : 7:3:100
2
between r and r0 is
2  This changes the expression (7.3.91) for Uprt to
2 2
3
r 0 r  i91=2
Qf 4 5 f p f f r p :
8 h
2
r 20 r 2f dr <8r p r p =t 1 =
U prt ; 7:3:101
dt : 3C D ;
Remember these particles settle.
The corresponding mass flow rate of particles of size rp where, for a spherical particle, the value of Ap r 2p . One
between rf and r0 at z 0 is Qf f p f f r p , which is the total can now develop an expression for dz=dr for the particle
incoming rate of particles of size rp. Therefore, the value of trajectory similar to (7.3.92). Expressions for CD as a func-
Gr(rp) (by equation (2.4.4b)) is tion of the Reynolds number are provided by relations
622 Bulk flow perpendicular to the direction of force

(6.3.4) (see Bird et al. (1960, 2002), where the symbol f Table 7.3.1. Dimensions and/or density of some cells
for the friction factor is used instead of CD, the drag coeffi-
cient). Hsu (1981) has considered various flow regimes Density Dimensions
specifically for spherical particles in a centrifuge. Cell/particle (g/cm3) (m)
Although the grade efficiency function enables one to Acetobactor oxydans 0.5  1
determine easily the particle size distribution in the over- Candida mycoderma 3
flow and the underflow if the feed particle size distribution Chinese hamster ovary (CHO) cells 1.06
is known, a quantity, , called sigma, is frequently used in Chlorella 1.03 ~810
practice to compare different centrifuges (Ambler, 1952). It Coliform bacteria 0.5
is defined by the relation Escherichia coli (E. coli ) 1.09 1
Red blood cell 2.4  8.4
Qf 2U pz jgravity ; 7:3:102 Single-cell protein (Candida) 47
Yeast (Saccharomyces)
where U pz jgravity represents the gravitational settling vel- Bakers ~1.1 710
ocity of a particle which obeys Stokes law and has the Brewers ~1.1 58
S. bayanus 3
equiprobable size, i.e. rp,50 (see Section 2.4). It represents
the cross-sectional area of a gravity based separator which Data obtained from: Belter et al. (1988); Goel et al. (2001);
will process the same feed rate Qf with somewhat similar Harrison et al. (2003).
particle separation characteristics (via rp,50). The gravita-
tional settling velocity is defined as Tubular bowl centrifuges are routinely used in smaller-
 scale separations of suspended material from fermentation
2 r 2p;50 g p t
U pz jgravity : 7:3:103 beer. The objective may be to recover the cells which are to
9 be disrupted to recover intracellular bioproducts. Alterna-
tively, for extracellular bioproducts, the beer has to be
The equiprobable size, rp,50, is obtained by equating the
clarified (be free of cells, cellular debris, etc.). Table 7.3.1
expression for Gr(rp), (7.3.99b) to 0.50:
provides the dimensions and/or densities of a few types of
 
microorganisms/cells/particles. Two special features of the
2 8 93
< 42 p t r 20 r 2f L =
24 2
r 0 1 exp r p;50 5 separation of the suspensions in fermentation beer as feed
9Qf
are their very low density difference p t and smaller
: ;  
0:50  ;
r 20 r 2f particle sizes. In the separation of suspended materials
from the fluid in nonbiological systems, p t is consid-
 

which yields erably larger. Further, the particle dimensions have wide
0 1 " # variations.
Qf 9 2r 2
r 2p;50  @ Aln 2 0 2 : Example 7.3.3 This example illustrates the procedure by
r0 rf

2 r 20 r 2f L 22 p t which one can estimate the volumetric flow rate of a suspen-
sion which can be effectively treated by a tubular bowl cen-
7:3:104
trifuge having given dimensions. The centrifuge inside radius
An expression for sigma is now obtained for a tubular bowl
(r0) is 10 cm; the centrifuge design creates a free surface
centrifuge: radius rf of 4 cm. The bowl length is 90 cm. The centrifuge

2 2
 rotates at 5000 revolutions per minute (rpm). Determine the
Qf 2 r 0 r f L volumetric capacity of operation of the centrifuge for the

2U pz jgravity

g 2r 2
!: 7:3:105
following suspensions and the desired separations.
ln 2 0 2
r0 rf (a) E. coli cells of diameter 1 m, density 1.09 g/cm3, suspen-
sion viscosity 1.5 cp: complete recovery of the cells is
This expression for shows that it depends on the physical desired for subsequent intracellular product recovery by
dimensions of a centrifuge operating with a given liquid cell rupturing. The suspension is quite dilute.
and the thickness of the liquid in the bowl rotating at a (b) Solid particles of density 2 g/cm3, diameter 10 m, sus-
given condition. It has been observed in practice that pension viscosity 3 cp and density 1.3 g/cm3: complete
particle separation characteristics of centrifuges of different clarification of the liquid is desired along with recovery
sizes are quite similar if the following equality holds of the solids.
between different centrifuges: Solution (a) We will employ equation (7.3.94) to determine
   the value of Qf for the given rmax. Here the cell dimension
Qf  Qf  Qf  of E. coli will be taken as rmax. The value of Qf determined
centrifuge 1 centrifuge 2 centrifuge 3 : 7:3:106
   will be recommended as an upper limit. Here rmax 0.5
m 0.5  104 cm; rf 4 cm; r0 10 cm; p
Obviously such a relation is quite useful for scaleup. 1:09 g=cm3 ; t 1:00 g=cm3 (very dilute suspension);
7.3 External force field based separation: bulk flow perpendicular to force 623

5000  2 radian 5000  2 radian


h n oi
; Gr r p 1:19 1 exp 0:7404  10 r 2p ;
min 60 s
1:5 cp 1:5  0:01 g=cm-s; L 90 cm. Introducing where rp is m. For
these values into equation (7.3.94), we get r p 0:25 m ) Gr r p
  2 1:19 1 expf 0:7404  10  0:0625g;
10 20:5  10 4 50002  42  1:091:001001690
ln ;
4 9Qf 1:5  10 2 602
8 9
< 1 =
So (on rearranging) Gr r p 1:19 1 expf 0:462g 1:19 1
: exp0:462;
83  0:25  10 8  25  106  0:09  84  90 cm3
2 3
Qf 1 5
2:302 log 2:5  9  1:5  10 2  36  100 s 1:19 41 1:19f1 0:629g 0:4415:
1:588
8  31  0:25  25  0:09  84  90  60  10 3 liter

2:302  0:3979  13:5  36  100 min For

1:406 liter=min : r p 0:5 m ) Gr r p 1:19 1 exp f 0:7404  10  0:25g;


4
8 9
For Part (b), rmax 5 m 5  10 cm; r0 10 cm; p < 1 =
2g=cm3 ; t 1:3g=cm3 ; 3 cp 3  0:01g=cm-s; Gr r p 1:19 1 exp f 1:851g 1:19 1
: exp 1:851;
83  5  10 4 2  50002  21:31001690 cm3 2 3
Qf 1 5
2:302 log 2:5  9  3  10 2  602 s 1:19 41 1:19 1 0:157 1:000:
6:36
8  31  25  10 8  25  106  0:7  84  90  60 liter

2:302  0:3979  27  36  1000 min Therefore, cells of 0.5 m radius will be captured completely
in this device according to the theory. (Note: The theory is
552 liter=min:
not exact, so allow a safety factor and operate at a lower flow
Larger particle size and higher density enables a drastic rate than 1.4 liter/min.)
increase in the centrifuge processing capacity.
7.3.2.1.1 Continuous separation of two immiscible
Example 7.3.4 Determine the expression for the grade effi-
ciency function Gr(rp) for the centrifuge under the conditions liquids of different densities Consider a dispersion of
of Example 7.3.3(a). Calculate the value of the grade effi- one liquid phase as drops in another immiscible liquid
ciency function for cells of two sizes, rp 0.25 m and rp phase. If such a dispersion is allowed to stand in a vessel,
0.5 m. the heavier phase will settle to the bottom and the lighter
phase will rise to the top to form two distinct layers, unless
Solution Equation (7.3.99b) provides the expression for the
the dispersion is stabilized. However, the rate at which the
grade efficiency function Gr(rp) of the cylindrical centrifuge:
  phase separation will take place is generally quite slow
8 93
and often incomplete. If, however, centrifugal force is
2
2 2 2
r 2 < 4 p t r 0 r f L =
Gr r p 2 0 2 41 exp r 2p 5: employed, the phase separation can be implemented
r 0 r f 9Qf
much more rapidly than is possible with gravity.
: ;

Given that r0 10 cm; rf 4 cm; (p t) 0.09 g/cm ; 3 A tubular bowl centrifuge can be used to separate
1.5 cp 1.5  102 g/cm-s; L 90 cm; continuously two immiscible liquids having different dens-
5000  2 radian 1406 cm3 ities. Near the top of the rotating bowl in such a centrifuge
; Qf ; (Figure 7.3.13), two liquid outlets are provided at different
60 s 60 s
we have
2 8 93
>
> >
>
2 2 2>
> >7
100 4 5000 4  0:09  100 16  90 r
6 >
< =7
6 p
Gr r p 1 exp
1406
6 7
100 16 6 2 2
7
60  9   1:5  10
4 >
> >
>5
60
>
> >
>
: ;
2 8 93 2 8 93
100 4 < 163  25  106  8:1  84 r 2 = < 16  31  25  106  680:4 =
p 2
1 exp 5 1:19 41 exp r p 5:
84 : 540  14:06  1:5 ; : 11388 ;

So
h n oi radii (r1i, r2o) and different elevations via two weirs (see
Gr r p 1:19 1 exp 0:7404  109  r 2p ;
McCabe and Smith (1976, p. 966) Hsu (1981) and Perry
where rp is in cm, and and Green (1984, pp. 2165)). The feed liquid mixture
624 Bulk flow perpendicular to the direction of force

enters through a bowl opening in the bottom through a The two liquid outlet radii (r1i, r2o) are known (via mech-
stationary nozzle. Two layers of liquid are formed in the anical adjustments); the unknown radial interface location
bowl: the outer one is for the heavier liquid and the inner is obtained by equating the hydraulic pressure drops in the
one is for the lighter liquid. Both leave through separate two liquid layers reflecting mechanical equilibrium with
outlets at the top of the device. There is a liquidliquid atmospheric pressure being exerted at both r1i and r2o.
Therefore
interface of radius rint between the two liquid layers: rint >
 2   2 
r1i, rint > r2o. At rint, the radial location of the interface, the r r2 r r2
1 2 int 1i 2 2 int 2o ;
pressure, Pint is the same for both liquids. Equation (3.1.51) 2 2
for centrifugal equilibrium in any pure liquid integrated
which leads to
between two radial locations will yield for the light liquid   
( 1 ) and the heavy liquid ( 2 > 1 ), respectively,
r 2int r 22o 1 r 21i 1 1 : 7:3:108
   2   2  2 2
Ms r r2 r r2
P int P 1i 2 int 1i 1 2 int 1i ; When 1 ! 2 , the location of the interface becomes
Vs 1 2 2
   2   2  unstable. The above relation also shows that if r2o is
2
Ms r r r r2 increased, rint is also increased, pushing the liquidliquid
P int P 2o 2 int 2o 2 2 int 2o :
Vs 2 2 2 interface closer to the wall of the bowl. Meanwhile, if there
7:3:107 are any solid particles in the fluids, they are thrown to the
bowl wall and can be collected from the bottom. Such
w liquidliquid centrifuges have a narrower and taller bowl
than those used for particle separations (Figure 7.3.12).
Note that the bulk flow of each liquid phase is perpendicu-
Light liquid Light liquid lar to the direction of the external force causing the
Heavy liquid Heavy liquid separation.

r20 r20 7.3.2.2 Disk centrifuge


This liquidparticle separation device provides an example
in this chapter where the bulk liquid motion toward the
rli rli
device outlet is at an angle to the centrifugal force. The
rotating bowl in this device has a flat bottom and a conical
rint rint
top rotating at an angular speed rad/s around the vertical
axis (Figure 7.3.14). A stack of closely spaced conical disks
is set at an angle to the vertical axis between the conical
Centrifuge top and the bottom. The gap between the consecutive disks
wall
is 2b; the disks rotate with the bowl at the same angular
velocity. The feed suspension introduced at the top of the
device through a pipe flows through the bottom of the
device between the disk stack and the device flat bottom
Light liquid to the periphery of the bowl. From this region, the liquid
Heavy liquid suspension goes up, enters the gap between two consecu-
tive disks and flows toward the central device axis. Particles
in the suspension having a density p greater than the fluid
density t are radially thrown outward and hit the bottom
surface of the top disk in every channel. The collected
Solids particles slide outside and, at the end of each top disk
surface, get thrown outward to the bowl wall. The clarified
liquid leaves the device top through an annulus between
the end of the disk stack and the feed inlet pipe.
Figure 7.3.14 illustrates the trajectory of a particle in
w
Feed between two consecutive disks (dashed lines). Define two
coordinate axes: the positive z-axis parallel to the disks,
Figure 7.3.13. Schematic of a tubular centrifuge for separating two and in the direction of main liquid flow toward the main
immiscible liquids. vertical device axis, and the positive r-axis radially
7.3 External force field based separation: bulk flow perpendicular to force 625

Feed suspension

Overflow

rin
dr dz

y 2b Disks
Upy Upz
Upr

Particle
trajectories

rex
rin

Figure 7.3.14. Schematic of a disk centrifuge.

outward. A particle entering the channel between two be determined. Since the gap 2b between the two plates
consecutive disks moves along the positive z-axis carried is small compared to the value of r at any radial
by the main liquid flow at a velocity Upz; simultaneously, location, this flow area is 2r2b at any radial loca-
the centrifugal force imparts a radially outward velocity Upr tion r. So
( dr/dt) to the particle.
Qf
This radially outward particle velocity, Upr, can be U pz dz=dt vz ; 7:3:109
4 r b nc
broken down into two component velocities, Upy ( dy/
dt) normal to the disk wall, or the z-direction, and another where nc is the total number of channels through which
component in the negative z-direction reducing the posi- the liquid slurry having a volumetric flow rate Qf flows. The
tive z-direction velocity, Upz. The device operation will be total number of disks is therefore nc 1. Since the two
affected if Upz is negative or zero. The y-component of the coordinates of interest in the particle trajectory before the
radially outward velocity causes the particle to hit the particle hits the bottom of the top plate in any channel are
bottom surface of the top disk in every channel as it moves y and z, we need next Upy ( dy/dt). But
in the positive z-direction. Those particles which are small
enough not to hit the top disk by the time the channel end U py U pr cos dy=dt: 7:3:110
is reached escape with the liquid and are not captured by
the centrifuge. To determine Upr, we assume terminal settling velocity
We assume first that the positive z-component of conditions and applicability of Stokes law (equation
the particle velocity Upz ( dz/dt) is essentially equal (7.3.91)) (but ignore the contribution ( vz sin ):
to the positive z-component of the liquid velocity vz. We
2 2
further assume that this liquid is in plug flow. To calcu- dr 2r p r
U pr p t :
late this velocity vz, the flow cross-sectional area has to dt 9
626 Bulk flow perpendicular to the direction of force

Therefore leading to

dy 2r 2p 2 r Gr r p r 2p =r 2max for r p  r max : 7:3:119


p t cos : 7:3:111
dt 9
Further, the largest particle size with a nonzero probability
One can now obtain a relation between y and z for the of not being captured is obtained from equation (7.3.118)
particle trajectory, as (Ambler, 1952)
2 2 2 27Qf
dy 8 b nc r p r
p t cos ; 7:3:112 r 2max  : 7:3:120
4nc p t cot r 3in r 3ex 2

dz 9Qf

and integrate between the limits of y and z from the All particles of size rp > rmax will be captured in the device.
entrance to the flow channel between any pair of disks to Thus the grade efficiency function for a disk-stack centri-
the exit. It is, however, more useful to express the particle fuge does not depend on the channel height 2b but instead
location in terms of the radial coordinate r instead of the depends on the flow rate per channel. Also, the larger the
z-coordinate. Define rex and rin to be the radial coordi- value of , the smaller the value of rmax. Note that the
nates, respectively, for the inner location (where liquid particle capture efficiency E for particles of a particular size
exits the channel) and the outer location (where liquid is equal to the grade efficiency Gr(rp) for that particle size.
enters the channel). Any differential positive change dz in An important assumption in the derivation of equation
the z-coordinate of a particle along the channel involves a (7.3.120) was that the value of the particle z-velocity com-
differential change in r which is negative: ponent (dz/dt) was equal to the liquid velocity vz. By this
assumption, the component of the velocity in the negative
dr dz sin : 7:3:113 z-direction due to the centrifugal force was neglected. If we
Therefore do not neglect it, then the radial velocity of the particle is
(using equation (7.3.91))
dy 8 b nc p t 2 2 2
r p r cot : 7:3:114 2 2
dr 2r p r
dr 9Qf p T vz sin : 7:3:121
dt 9
Integration of this relation between the limits (rin, yin) and
Employ now expression (7.3.111) for (dy/dt) and expres-
(rex, yex) yields
sion (7.3.109) for vz to obtain
8 b nc p t cot 2 2  3
y ex yin r p r in r 3ex : 7:3:115 dy r 2 cos

27Qf 2 2 ; 7:3:122
dr r b1
For a particle to settle on the bottom surface of the top
where
plate at r rex, yex should equal 2b, the channel gap. At the
inlet end (r rin), the particle can enter the channel with !1=2
9Qf sin
the liquid at any value of y yin. Thus, for any particle of b1 : 7:3:123
8 b nc 2 r 2p p t
size rp entering at any yin to settle by the time the liquid
exits the channel,
Integration has to be carried out between the limits (rin,
8 b nc p t cot 2 2  3 yin) and (rex, yex), where yex 2b:
2b y in r p r in r 3ex :

7:3:116
27 Qf yex rex
r 2 dr
dy cos : 7:3:124
We can now assume, as we did for the tubular bowl centri- r 2 b21
yin r in
fuge, that particles of size rp are present uniformly at all
y-values at r rin. Then the grade efficiency Gr(rp) is This analysis was developed by Jury and Locke (1957); they
given by have also provided an analytical solution to the integral in
2b y in terms of the grade efficiency function:
Gr r p ; 7:3:117
2b 2b yin cos b1
 
b1 r in

Gr r p ln
since those particles of size rp which have y < yint at r rin 2b 2b 2 b1 r in
do not settle. Now, for rp  rmax, Gr(rp) 1 only if yin 0, b1 r ex
  
 r in r ex : 7:3:125
i.e. all particles entering the channel settle before r rex. b1 r ex
Therefore
Jury and Locke (1957) have provided, in addition, an
8bnc p t cot 2 expression for the thickness of the sludge film flowing
2b r max 2 r 3in r 3ex ;
 
7:3:118
27Qf down the cone wall.
7.3 External force field based separation: bulk flow perpendicular to force 627

Typical values and ranges for the operating conditions presented in the framework of a three-region model of
and dimensions of a disk centrifuge are provided now for cyclone by Dietz (1981) to introduce the inherent complex-
perspective: 35   50 ; nc 23; 2b 0:17 cm; bowl ities. The treatment begins with the Dietz (1981) model.
diameter 15 cm100 cm; rotational speeds up to 12 000 rpm The three regions of a cyclone particle separator in the
1257 rad=s; Qf up to 16 m3/min. The disk centrifuges model of Dietz (1981) consist of the entrance region
generally employed are larger than tubular bowl centrifuges (region 1), the downflow region (region 2) and the upflow
and can be run continuously. region (region 3) (see Figure 7.3.15(a)). Regions 2 and 3 are
sometimes called the annular region and the core region,
respectively. In these cyclone models, the cyclone of Figure
7.3.2.3 Cyclone dust separator
7.3.15(a) is replaced by a right circular cylinder of radius rc
To separate particles from gaseous streams, cyclones are and length L below the exit tube, equal to the length of the
frequently used in large-scale practices. In fact, cyclone actual cyclone below the exit pipe; the exit pipe is of radius
based dust collectors are one of the most widely used rt (Figures 7.3.15(b), (c)).
devices for removing larger-sized particles. Virtually all Regions 1 and 2 have the swirling gas flowing downward
cyclones used industrially are reverse-flow cyclones. There at a volumetric flow rate of Qv0 and Qv(z), respectively,
are two other types of cyclones: rotary-flow (Ciliberti and where Qv0 is the total volumetric gas flow rate entering the
Lancaster, 1976a, b) and uniflow (Ter Linden, 1949). Only cyclone. In Dietzs model, Qv(z) is related to Qv0 by
the reverse-flow cyclone will be considered here.
Qv z Qv0 f1 z=Lg: 7:3:126
A reverse-flow cyclone (Figure 7.3.15(a)) is a simple
hollow structure consisting of two parts: a cylindrical cyc- The linear decrease in Qv(z) with increasing z is due to the
lone barrel having an annular vortex finder or exit pipe; and radial gas velocity, vr(z), from the annular to the core
a truncated cone at the bottom joined at the top to the region (the upflow region):
cyclone barrel. The dusty gas is introduced tangentially
into the cyclone barrel through the inlet. The gas, having Qv0
jvr z j vr0 : 7:3:127
a swirling motion, goes around the cyclone as it moves 2r t L
down toward the dust exit; however, the swirling gas soon Dietz (1981) assumed (i) jvr z j to be a constant, vr0 , and
reverses its gross axial movement, rises up and exits (ii) that, due to turbulent mixing, the particle mass concen-
through the top of the vortex finder. The rotational motion tration profile in each region was radially uniform. How-
of the gas generates the radial centrifugal force on dust ever, each such profile,22 p1 z, p2 z and p3 z for
particles, which makes them hit the wall of the cyclone as regions 1, 2 and 3, respectively, changes with the z-
shown in the idealized flow pattern in an ideal cyclone coordinate due to particle deposition at the cyclone wall
(Flagan and Seinfeld, 1988) shown in Figure 7.3.15(b). and/or particle exchange between regions 2 and 3:
The centrifugal force on the particle may be as much as
52500 times larger than the gravitational force (Perry and d 
region 1 : p1 z Qv0 2 r c w z; 7:3:128
Green, 1984, p. 2083). If there is no reentrainment, the dz
dust particles settle and are removed through the dust exit d 
region 2 : p2 z Qv z 2 r c w z 2 r t v z;
at the bottom. Figure 7.3.15(b) shows only the circular gas dz
motion in an idealized flow. The gas and the particles have 7:3:129
an axial (z-directional) motion, which is not shown. An d 
analysis of the ideal flow cyclone for particle separation is region 3 : zQv z 2r c v z: 7:3:130
dz p3
available in Flagan and Seinfeld (1988).
The flow pattern of the vortex motion of the gas in Here the quantity w z is the particle mass flux at the wall
reverse-flow cyclone is quite complex. First, it is three- at axial coordinate z. For region 1, w z is the product of
dimensional; second, the flow is turbulent. An exact analy- the particle mass concentration, p1 z, and the radial par-
sis is therefore difficult. Soo (1989) has summarized a ticle velocity at the wall, Uprw:
fundamental analysis of velocity profiles and pressure region 1 : w z p1 zU prw : 7:3:131
drops in such a cyclone. He has also analyzed the
governing particle diffusion equation in the presence of For regions 2 and 3, p1 z has to be replaced by the
electrostatic, gravitational and centrifugal forces. He has corresponding particle mass concentrations, p2 z and
then provided an analytical expression for particle collec- p3 z, respectively. Further, Uprw is a function of the
tion efficiency under a number of limiting conditions. We z-coordinate:
will, however, opt here for a much simpler model of par-
ticle separation in a cyclone developed by Clift et al. (1991).
This approach is based on a modification of the original 22
Instead of Dietzs number density profiles, n1(z), n2(z) and n3(z),
model by Leith and Licht (1972). The model will be the particle density profiles, are used here.
628 Bulk flow perpendicular to the direction of force

(a)
Inlet
Vortex finder
(Exit pipe)
b

rc
a
Region 1
s
rt (b)
Exit pipe surface
rc
rt
Region 2 h y
q

H
rt
vr vq r
Region 3

Particle trajectory
Gas flow streamline
x
Particle hits outer wall
Truncated cone

Dust exit

(c) Figure 7.3.15. (a) Geometry of a conven-


tional reverse-flow cyclone; (b) particle tra-
jectory in a cyclone having an idealized
flow pattern. (After Flagan and Seinfield
(1988).) (c) Modified cyclone geometry for
a analysis. (After Dietz (1981).)
Region 1 rt s-
2
Q v0 Qv 0
z=0

vro Region 3 Gw (z)

Gw (z) rt Qv (z)

r
Qv (z)
Gv (z) L
Qv (z) rc

Gv (z)
Region 2
Region 2
z=L
7.3 External force field based separation: bulk flow perpendicular to force 629

Region 2 : w z p2 z U prw z: 7:3:132 v z p2 z jvr z j p3 z U prv z 7:3:137a

Note: In each region the radial particle flux due to the as a balance between the particle flux entering the core via
radial centrifugal force is perpendicular to the bulk flow the radial gas velocity jvr z j from outside to inside and the
of the fluid and the particles in the main flow (z) direction. particle flux going radially outward due to the centrifugal
We need to know what Uprw and Uprw (z) are; further, force on particles in the gas vortex in the core. The quantity
an expression for v z has to be found if the three equa- Uprv(z), i.e. the radial particle velocity at the region
tions (7.3.128) (7.3.130) are to be solved. We assume that 2/region 3 boundary, may be estimated in the manner of
equation (7.3.135)
Stokes law is valid;
the particle is hitting the wall at the terminal velocity U prv z 2 r 2p p v2tv =9r t : 7:3:137b
without any radial acceleration.
The tangential velocity vtv at r rt may be estimated from
From equation (7.3.90b), we obtain at the wall (r rc) equation (7.3.134) for a free vortex. An alternative, and
 more realistic, description of v z based on a particle
mp r c 2 1 t =p 6r p U prw : 7:3:133
diffusivity between regions 2 and 3 has been developed
However, r c 2 is simply U 2ptw =r c , where Uptw is the tan-
  by Mothes and Lffler (1984).
gential particle velocity U p at the wall U pw .
    By applying appropriate boundary conditions, Dietz
Now, if one assumes no slip between the particle and the (1981) obtained solutions for equations (7.3.128)(7.3.130).
gas in the tangential direction ( -direction), the tangential For example, region 1 extends from z s a/2 to z 0.
particle velocity at the wall, Uptw, is also equal to the Equation (7.3.128) may be written via equation (7.3.131) as
corresponding tangential fluid velocity, vtw, at the wall. At dp1 z 2 r c U prw
the wall, the fluid velocity vtw is zero. Note that, in the d ln p1 z dz: 7:3:138a
p1 z Qv0
boundary layer close to the wall, it is nonzero. Further, the
tangential gas velocity vt(r) in the vortex may be related to Integrating between s a=2 and z z, we get
the radial location by an equation valid for a free vortex in !
an ideal fluid:
p1 z 2 r c U prw a
ln z s ;
r m p0 Qv0 2
c
vt r r m constant; vt r vtw : 7:3:134
r 2 r c U prw a
 
p1 z p0 exp z s ; 7:3:138b
There is little error in replacing vt (r) in the wall region by Qv0 2
vtw for m 1 (ideal vortex); in general for cyclones, the where p0 is the particle density at the inlet, i.e.
vortex index m is between 0.5 and 1. We can therefore z s a=2.
write Using the solutions for p1 z, p2 z and p3 z for a
2 r 2p p v2tw given inlet condition, namely
U prw 7:3:135
9r c
a
p1 z s p0 ;
2
from equation (7.3.133) if mp 4 r 3p p =3 and t <<< p
for particles of density p . the particle collection efficiency ET of the cyclone is
Since Uprw and Uprw(z) are now replaced by vtw and obtained as Follows (Dietz, 1981):
vtw(z), we have to estimate the characteristic gas velocity in p3 z 0
the wall region, vtw, as a function of z. There are three regions ET 1
p0
in the cyclone. In region 1, vtw is constant since there is no
2r c U prw s a2
  
change in gas flow velocities; the gas flow rate and region 1=2 i
h
1 a0 a21 a2

exp ;
dimensions are constant throughout. A simple procedure Qv0
for estimating vtw in region 1 provided by Leith and Licht 7:3:139a
(1972) is to equate it to the average gas velocity at the cyclone
where
duct inlet of cross-sectional area ab (Figure 7.3.15(a)):
r c U prw r t vr0 U prv
vtw Qv0 =ab: 7:3:136 a0 7:3:139b
2r t U prv
In region 2 also, vtw may be assumed constant and equal to
r t U prv r t vr0 r c U prw
the above. Thus Uprw and Uprw (z) for regions 1 and 2 are a1 ; 7:3:139c
2r t U prv
known.
 2m
The only unknown now (except p1 ; p2 ; p3 ) is v z. r c U prw rt
Dietz (1981) described it by a2 : 7:3:139d
r t U prv rc
630 Bulk flow perpendicular to the direction of force

To obtain ET for a given cycloneparticle separation time taken by the gas to complete one complete turn from
system, a value of m needs to be assumed. Typical values the inlet:
assumed vary from 0.50.7 (Leith and Licht, 1972; Dietz,
1981). inlet region volume a r 2c r 2t
t turnjinlet :
We now consider the modification of the Leith and gas flow rate Qv0
Licht (1972) model by Clift et al. (1991); the result is a 7:3:146a
relatively simple procedure for estimating ET for a cyclone.
Then, if one knows empirically the number of turns, nt the
It assumes that the particle mass concentration profile, pg ,
gas takes in the cyclone before finding the exit pipe, the
is uniform radially, but varies axially; it is represented by
residence time tres of the gas may be estimated by
pg z for the whole cyclone. Consider now a length dz of
the cylindrical part of the cyclone. The rate at which par- nt a r 2c r 2t
ticle masses are being deposited from this volume to the t res nt t turnjinlet : 7:3:146b
Qv0
cyclone wall at r rc is 2r c dz pg zU prw . However, this
must equal the rate of change of particle mass in that Clift et al. (1991) have compared the particle collection
volume: efficiency predictions by various models plotted against
1=2
the quantity U prv =vr0

d 2  ; note that the latter is propor-
r c pg z dz 2 r c pg z U prw dz: 7:3:140 tional to the particle diameter 2rp. That particle collection
dt
efficiency by a reverse-flow cyclone rises very rapidly with
Simplifying 2rp is shown clearly in Figure 7.3.16. This figure is import-
 ant from a particle separation point of view. Cyclones are,
d ln pg z 2U prw
: 7:3:141 in general, quite efficient in removing coarse particles  5
dt rc 10 m. For finer particles, other separation techniques,
such as electrostatic precipitation (see Section 7.3.1.3),
Substituting for Uprw from equation (7.3.135), we get
 filters, etc., should be used. However, an electrocyclone
d ln pg z 4r 2p p v2tw concept has been developed wherein an electrode at a high
; 7:3:142 voltage is introduced into the cyclone through the center of
dt 9r 2c
the vortex finder where the cyclone wall is grounded. At
which, when integrated from t 0 to t tres, the mean low gas flow rates, it has a much higher efficiency than
residence time of the gas in the cyclone, yields conventional cyclones (Chen, 2001) for smaller particles.
For submicron particles, filters of different kinds are very
 4 r 2p p v2tw efficient (Eggerstedt et al., 1993).
ln pe =p0 t res ; 7:3:143
9r 2c The cyclone models illustrated above are meant merely
to introduce the reader to the complex subject of particle
where pe is the particle mass concentration of the gas
separation from gases by cyclones. More interested readers
exiting the cyclone. Note: The result is meant for particles
should pursue the original references, as well as the brief
of radius rp only.
overview on devices, in Perry and Green (1984, pp. 20-83
An alternative expression for particles of radius rp in
20-89). A model of particle separation in a down-exhaust
terms of the grade efficiency function or the particle col-
cyclone separator, of uniflow type, is available in Chen
lection efficiency for size rp, ET(rp),
et al. (1999b). These separators have a very low gas pres-
p 2r p vtw 2
" #
pe sure drop.
 
Gr r p E T r p 1 1 exp t res ;
p0 9 rc Example 7.3.5 A cyclone is used to remove as many par-
7:3:144 ticles as possible coming out of a cement kiln. The inlet gas
velocity entering the cyclone is 1524 cm/s. The particle dens-
directly illustrates the functional dependences of the col- ity and the gas viscosity are 2.9 g/cm3; and 2  104 g/cm-s,
lection efficiency on important quantities. Clift et al. (1991) respectively, The dimension b of the gas inlet to the cyclone
suggest that the residence time tres of the gas in the cyclone is 76 cm. The radius of the cyclone (rc) is 152 cm. The number
may be calculated as follows: of turns, nt, the gas goes through in the cyclone before
entering the exit pipe is 5 (Theodore, 2005). Determine the
cyclone volume value of the particle radius rp,50 which yields a value of Gr(rp)
t res ; 7:3:145 0.5 ET(rp) (this is the equiprobable size, or the cut diam-
gas flow rate Qv0
eter, (2rp,50)). Calculate also the value of ET(rp) for the
where the cyclone volume corresponds to that from the following values of rp: 0.5 m; 2.5 m; 10 m; 20 m; 30 m.
inlet to the vortex finder (or exit pipe). They have also Solution We will employ equation (7.3.144) to calculate
suggested that the volume of the inlet region is given by ET(rp) for a given rp. To employ ET(rp), we need to know,
a r 2c r 2t . One approximate procedure is to estimate the
 
amongst other things, tres. From equation (7.3.146b),
7.3 External force field based separation: bulk flow perpendicular to force 631

100 Exxon runs


51
48
80

Cyclone efficiency (%)


60

40

20

0
1 2 3 4 5 6 7 8 9 10
Particle diameter (m)

Figure 7.3.16. Comparison between theory and experiment for the second-stage cyclone at Exxons miniplant (assumed m 0.7).
Reprinted, with permission, from P.W. Dietz, Collection efficiency of cyclones, AIChE J., 27(6), 888 (1981). Copyright [1981] American
Institute of Chemical Engineers (AIChE).

a r 2c r 2t value of 2rp,50 calculated by Theodore (2005) was ~10 m.


 
t res nt : Theodore (2005) has recommended the following equation
Qv0
for calculating ET(rp):
Now
1
E T r p :
cm Qv0 1 2r p;50 =2r p 2

vtw 1524 :
s ab
The results are shown in Table 7.3.2.
Therefore
The analysis based on Clift et al. (1991) and equantions
Qv0 vtw ab 1524  a  76: (7.3.140)(7.3.144) led to result (7.3.144), based on a particu-
We do not know rt. We will neglect r 2t << r 2c : Therefore; lar size. If one is interested in the total particle collection
 
efficiency ET, we have to integrate over all sizes using the
5   1522 0  a incoming particle number density function nf(rp) as follows:
t res s
1524  a  76
r max  2 
5   152  152

2r v
s: p nf r p exp 9p prc tw t res dr p
1524  76
0
ET 1 r max : 7:3:147
Now calculate rp,50:

2 3 p nf r p dr p
p 4r 2p;50 v2tw 0
E T r p;50 0:5 Gr r p;50 1exp 4 t res 5
9r 2c
2
g cm2
3 7.3.2.3.1 Hydrocyclones The success of gassolid cyc-
2:9 3
 4  r 2p;50 cm2  1524  1524 2  s lones for dust collection has led to hydrocyclones for
6 cm s 7
solidliquid and liquidliquid separations. The latter
6 7
) exp 6
6 g
7 0:5
9  2  10 4  152  152
7
4
cm-s
5 involves oilwater emulsions. A number of extended treat-
2 3 ments are available on particle separation in hydrocy-
2:9  4  1524  1524  2 25 clones (Bradley, 1965; Svarovsky, 1982), which are
) 0:5 exp 4 r cm
18  152  10 4  152 p;50 extensively used in a variety of industries, including the
h i corn milling industry. As in cyclones used for gas
) 0:5 exp 203  104  10 8 r 2p;50 m2 cleaning, the vortex tube at the top produces an overflow,
) 0:0203r 2p;50 0:7 which is the clarified liquid containing fines, whereas
the underflow coming down the conical wall is concen-
) r 2p;50 34:48 ) r p;50 5:87m
trated slurry of larger particles. Frequently, multiple cyc-
) cut diameter 11:75m 2r p;50 : lones are used in series, with the overflow from one
We will now calculate the values of ET (rp) for different cyclone being the feed to the next one to improve the
values of rp. We will also list the values of ET from Theo- final clarification of the suspension. An introduction to
dore (2005) based on Lapples curve (Lapple, 1951). The various aspects of particle separation in hydrocyclones,
632 Bulk flow perpendicular to the direction of force

Table 7.3.2.
h i h i
rp (m) exp 203  10 4 r 2p m E T r p 1 exp 203  10 4 r 2p m ET (from Theodore, 2005)

0.5 exp [0.00507] ~0 0


2.5 exp [0.1268] ~0.12 0.20
5.0 exp [0.5075] ~0.40 0.50
10 exp [2.028] 0.869 0.80
20 exp [8.112] ~1 0.93
30 exp [18.252] ~1 0.98

including computational fluid dynamics (CFD) based fraction enriched in the lighter species and the fraction at a
modeling, has been provided by Salcudean et al. (2003). larger radius (r > rc) is the heavy fraction enriched in the
Liquidliquid dispersions/emulsions are also increas- heavier species. This enrichment is possible because the
ingly separated by hydrocyclones. Analyses based on drop- bulk gas is undergoing a rotational motion in the nozzle.
let trajectories are being developed (Wolbert et al., 1995). The magnitude of the centrifugal force is substantially
Rietema (1969) developed a hydrocyclone design analo- enhanced by the magnitude of the tangential velocity r,
gous to cyclones for gas separation: the continuous phase where is the angular velocity in the semicircular
leaves the cyclone at the top through the vortex finder as groove of radius r0: this gas velocity is in the range of
the overflow; the oil droplets in a concentrated emulsion 1001000 m/s. The process is carried out at subatmo-
appear in the underflow outlet at the bottom. Colman and spheric pressures for the separation of uranium isotopes
Thew (1988) have developed an alternative hydrocyclone U235F6 from U238F6. The feed pressure, for example, is
design wherein the clarified liquid leaves through the 290 Torr, whereas the product fractions are at a pressure
bottom outlet and the concentrated emulsion leaves of around 138 Torr and at a temperature Tp less than that of
through the top vortex-finder tube. A factor of importance the feed (Tf) (Benedict et al., 1981).
here is the density of the dispersed phase in relation to the We will now calculate, following Benedict et al. (1981),
continuous phase. The heavier dispersed phase will move the separation factor and other quantities for this process,
toward the wall; the lighter dispersed phase will move primarily explored for isotope separation. The fugacity
toward the hydrocyclone core. profile in the radial direction for a component i in a mix-
ture undergoing centrifugal rotation at an angular velocity
of radian/s is given by expression (4.2.3) for centrifugal
7.3.2.4 Separation nozzle process for gas separation equilibrium. We can rewrite this expression as a ratio of
So far, the description of centrifugal force based separation mass densities of the component i at the two locations, r
has been limited to the separation of particles from a fluid r1 and r2:
(gas or liquid) and the separation of a dispersion of two
^f i2 =^f i1 i2 =i1 expM i 2 r 22 r 21 =2RT p :
immiscible liquids. In each case, the bulk flow velocity was
perpendicular to the direction of the centrifugal force 7:3:148
created by a rotational bulk flow of the gasparticle/ If we define temporarily the mass density of species i at r
liquidparticle/liquidliquid in the device. This rotational 0 as i 0, knowing that the species mass density ij varies
bulk flow was developed either by rotation of the device with the radial location, we can also rewrite the above as
itself (as in a tubular bowl centrifuge, etc.) or by introdu-
cing the particle-loaded gas stream into a cyclone via a ij r i 0 exp M i 2 r 2 =2RT p : 7:3:149
swirling motion. Here we will focus on separating a gas
Define now the mass flow rate of species i between the
mixture via centrifugal force (see Section 4.2.1.1), where
radial locations r1 0 and r2 rc as wi1 for the product
the bulk motion direction itself is the source of the centri-
region j 1, the light fraction. Then
fugal force and is perpendicular to the centrifugal force. It
is called the separation nozzle process; for a comprehensive r2
r c r2
r c
introduction, see Benedict et al. (1981). wi1 rij rdr ri 0 exp M i 2 r 2 =2RT p dr
Consider Figure 7.3.17, which shows a convergent r 1 0 r 1 0
divergent slit opening into a curved nozzle of radius ro. RT p i 0=M i expM i 2 r 2c =2RT p 1 :

The gas mixture entering through the throat of dimension t 7:3:150


diverges, is turned around 180 by the curved nozzle and
comes up to a thin wall acting as a gas flow divider at a Correspondingly, the mass flow rate of species i between
radius rc. Two gas fractions are collected from two sides of the radial locations r1 rc and r2 r0 representing the
this wall; the fraction at a smaller radius (r < rc) is the light product region j 2 is given by
7.3 External force field based separation: bulk flow perpendicular to force 633

Gas flow divider


Light fraction
enriched in
lighter isotope
Heavy fraction
Feed enriched in
heavier isotope
Isotopic gas mixture
rc

Slit
r0

Figure 7.3.17. Cross section of the slit and curved nozzle in the separation nozzle process. (After Benedict et al. (1981).)

r2
r 0 r2
r 0
0 1
wi2 rij rdr ri 0expM i 2 r 2 =2RT p dr
6:353  0:25A
exp@ 1
r 1 r c r 1 r c
1:008596
12
RT p i 0=M i expM i 2 r 20 =2RT p expM i 2 r 2c =2RT p : exp6:353  0:25 1

7:3:151
exp6:353 exp6:353  0:25
 0 1
Define now a constant 6:353  0:25A
exp 6:353=1:008596exp @
A20 M 2 2 r 20 =2RT p : 7:3:152 1:008596

We may now define a separation factor 12 between two exp1:5747 1 exp6:353 exp1:588

species i 1 and 2 between the two product regions, exp1:588 1 exp 6:2988exp1:5747
region j 1 between r 0 and r rc and region j 2
between r rc, and r r0 via the following species mass 4:830 1 574 4:895 3:830  569:105
 1:037:
flow rate based definition (see definition (2.2.9a)): 4:895 1 544 4:830 3:895  539:17

exp M 1 2 r 2c =2RT p 1 exp M 2 2 r 20 =2RT p exp M 2 2 r 2c =2RT p



w11 w22
12
; 7:3:153a
w12 w21 exp M 2 2 r 2c =2RT p 1 exp M 1 2 r 20 =2RT p exp M 1 2 r 2c =2RT p

exp A20 M 1 =M 2 r 2c =r 20 1 expA20 exp A20 r 2c =r 20



 

12 
: 7:3:153b
exp A20 r 2c =r 20 1 exp A20 M 1 =M 2 expA20 M 1 =M 2 r 2c =r 20
 


One can now develop an estimate of the separation factor


12 between U235F6 (i 1) and U238F6 (i 2) at Tp 300 K,
when r0 0.1 mm, rc 0.05 mm and r0 300 m/s, based This value is considerably larger than values achieved in
on the value of A20 6.350 (Benedict et al., 1981, figs. 14.27 other isotope separation processes. However, the range of
and 14.22). Further, M1 349, M2 352, and R 8.31  values of 12 is 1.011.04, more commonly around 1.015.
107 g-cm2/s2-gmol-K. First, we check the following: One important characteristic of this process has not been
mentioned yet. The isotopic gas mixture is diluted using a
A20 M 2 2 r 20 =2RT p low molecular weight gas, e.g. H2. This increases the per-
352g=gmol  300  102 2 cm2 =s2 ipheral velocity r0 which is, at the sonic level, increasing
) A20 12 ; also, the diffusion coefficients of UF6 species are
2  107  8:3g-cm2 =s2 -gmol-K  300K
increased, which permits operation at a higher gas pres-
6:353;
sure and higher gas throughput without affecting separ-
this is, okay. So, for (rc/r0)2 0.25; (M2/M1) 1.008596, ation (Benedict et al., 1981). Note that a major difference
634 Bulk flow perpendicular to the direction of force

Cleaned gas
W ontlet

Dust-laden
feed gas inlet dz
y dg
x z
Dust
collection
hoppers

Figure 7.3.18. Gravity based dust settling chamber.

between this method and the first turn of the particle- Let the rectangular settling chamber (Figure 7.3.18)
loaded gas in a cyclone separator (Figures 7.3.15(a)(c)) is have the following dimensions: L (length in the main flow
the use of a convergentdivergent nozzle to generate sonic direction, z-coordinate)  2b (height perpendicular to
speeds and, correspondingly, a high centrifugal force. the main flow direction, y-coordinate; gravity is in the
negative y direction)  W (width of the flow cross section,
7.3.3 Gravitational force field x-coordinate). We adopt the following simplifying assump-
tions.
The gravitational force field as the sole driver for the sep- (1) Due to turbulent mixing, particle concentration is uni-
aration of particles shows up in two stages of removal of form across the duct cross-sectional area 2bW.
particles from a gaseous stream. At the end of all cleanup (2) The gas moves in plug flow with an axial velocity of vz
processes, the waste gas is released to the atmosphere along the duct length; the spherical particles have the
through a stack rising high above the ground. The particles same axial velocity. However, near the bottom surface
in this waste gas plume will be deposited on the ground as of the chamber, the wall region, the gas flow velocity is
the gas is dispersed into the atmosphere over a substantial significantly reduced. The wall region thickness is g .
length of the surrounding landscape. Gravitational force (3) The particles in the wall region have a gravitationally
based settling of dust particles from a gas stream is also induced vertically downward velocity corresponding to
employed as a preliminary cleanup step for removing the terminal velocity Upyt of a spherical Stokesian par-
larger particles prior to a more thorough cleanup. We will ticle (see expression (6.3.1)) of radius rp:
first consider this settling of larger particles in a gravity
based dust settling chamber (Figure 7.3.18). We will now 2
2 r p p g
briefly describe the settling of particles from a gaseous U pyt : 7:3:154
9
plume leaving a stack. Gravity is also utilized to settle
particles from a liquid stream. Inclined settlers are used Here we have neglected the density of air vis--vis the
to separate cells in a cell culture broth. Such devices will be density of the particle.
briefly described at the end of the section. Consider a control volume of length dz spanning the
In a gravity based dust settling chamber (Figure cross-sectional area 2bW of the rectangular duct. The
7.3.18), the gas stream is allowed to flow into a large number of particles in the size range rp to rp drp per unit
horizontal rectangular chamber which reduces the gas volume is indicated by n(rp)drp. Any change in the total
velocity. The particles, especially larger ones, settle toward number of particles in this size range entering and leaving
the bottom surface of the duct, which are essentially the this control volume per unit volume per unit time,
top of multiple dust-collecting hoppers. The gravitational
2bW vz nr p jz nr p jzdz dr p ;
 
7:3:155
force based settling velocity of particles is perpendicular to
the mean gas flow direction. The gas velocity inside the is due to the deposition of particles on the bottom chamber
device is considerably smaller than that in the duct at the wall. For a particle of radius rp to settle on the bottom wall
gas inlet. One could have laminar flow of the gas or turbu- with a terminal velocity Upyt, it must traverse vertically a
lent flow in the chamber. Turbulent flow is characterized distance g during the time dt needed to traverse axially a
by a high degree of mixing across the flow cross section. distance dz of the control volume:
The observed behavior of settling in such chambers sug-
gests a significant degree of vertical mixing. We will briefly dt dz=vz ; U pyt dt g ; g =U pyt dz=vz : 7:3:156
illustrate here the extent of particle removal under condi-
tions of turbulent mixing. We follow the treatment of Fla- Since only the particles within the wall layer g can settle,
gan and Seinfeld (1988). The model is very similar to that the fraction of particles entering the control volume that
of Deutsch (1922) for an electrostatic precipitator. can settle is g =2b. Since the total rate of particle entry into
7.3 External force field based separation: bulk flow perpendicular to force 635

the control volume is given by 2bW vz nr p jz dr p , we now Flagan and Seinfeld (1988) have described the particle
obtain, at steady state, trajectory in such a flow field and calculated the particle
0 1 collection efficiency of particles of radius rp as
g
2bW vz nr p jz nr p zdz dr p @ A2bW vz nr p jz dr p

LU pyt
2b Gr Er p : 7:3:162
0 1 vz;avg 2b
dz U pyt A2bW vz nr p j dr p ;
@ Since the gas residence time in the chamber is L=vz;avg ,
 
z
2bvz
one concludes that, as long as 2b=U pyt is smaller than or
 

7:3:157 equal to L=vz;avg for any particle, those particles will


 

settle.
nr p jzdz nr p jz U pyt


nr p  : It is useful also to calculate the total efficiency of par-
dz 2bv z z
ticle collection over all particle sizes. From equation
As dz ! 0, we obtain (7.3.159), we obtain the total number concentration of
particles of sizes from rp 0 to rp rp as
dnr p U pyt
nr p : 7:3:158 rp rp
dz 2bvz 
U pyt z

nr p jz dr p Nr p jz nf r p exp dr p :
If, at the chamber entrance (z 0), the value of nr p is 2bvz
0 0
nf r p , integration leads to, at location z,
7:3:163
U pyt z
 
nr p jz nf r p exp : 7:3:159 The total efficiency is given by
2bvz
r max
U pyt z
 
If we define the dust collection efficiency for particles of
nf r p exp dr p
radius rp as E(rp), then, for a chamber of length L, 2bvz
Nr pmax jL 0
ET 1 1 r max :
nr p jL

U pyt L
 N f r pmax
Er p Gr 1 1 exp ; 7:3:160 nf r p dr p
nf r p 2bvz
0

where Gr is the grade efficiency function. For a Stokesian 7:3:164


particle,

2r 2p p gL 2r 2p p gLW
! !
Example 7.3.6 Consider gravitational settling of dust par-
Gr Er p 1 exp 1exp ;
92bvz 9Qf ticles in a chamber 8 meters long and 1 meter high, where air
may be assumed to move in laminar flow at an average
7:3:161
velocity of 25 cm/s. The viscosity of air at the temperature
where Qf is the volumetric flow rate of the dust-laden air. of operation is 1.80  104 g/cm-s. The dust particle density
is 2 g/cm3. Develop an expression for the minimum size of
As r p ! , E(rp) ! 1. Obviously, a larger value of length L
particles which will settle in this chamber. What is the value
of the chamber, and a larger cross section to reduce the gas
of this minimum size for the conditions provided?
velocity vz, will increase the particle collection efficiency. In
practice, limitations in the physical dimensions of the Solution The particle that enters at the top of the chamber
gravitational settling chambers do not allow these devices has the greatest chance to escape. If we assume that the
to capture particles smaller than 50 m (Flagan and Sein- residence time of this particle corresponds to the average
feld, 1988). Correspondingly, particles having settling vel- gas velocity (a safer assumption than the local velocity near
the top), then the particle residence time at this axial velocity
ocities higher than 13 cm/s may be removed in a
is given by
gravitational settling chamber (Wark and Warner, 1976).
However, the observed gas cleaning behaviors of such L
t res : 7:3:165
settling chambers are approximately described by relation vz;avg
(7.3.161). The particle which enters at the center of the channel will
One can also have gravitational settling of dust par- have half of this residence time ( L/vz,max L/2vz,avg tres/
ticles when the dust-laden gas moves in laminar flow 2); however, we will see soon that is not a problem. Assuming
between the top and the bottom plates in the chamber. the particle settles with a settling velocity given by (7.3.154),
Unlike that in turbulent flow, the axial velocity has the then the time required for the particle entering at the top to
following parabolic profile (6.1.2b, c): settle is
2b
3 y 2
  
t settling : 7:3:166
vz y vz;avg 1 : U pyt
2 b
636 Bulk flow perpendicular to the direction of force

For this particle to settle and be captured in this device, compared with convection. If one can assume steady state,
t settling  t res ) assume t settling t res the equation describing the dispersion of a gaseous pollu-
tants species i may be written from equation (6.2.9) (as
) 2b=U pyt L=vz;avg 7:3:167 described in Table 6.2.4 for Cartesian coordinates)
2b vz;avg 2 2 p g  2 
) U pyt r C i C i C i Ci 2 C i 2 C i
L 9 p vx vy vz Dij
x y z x 2 y2 z2
(from relation (7.3.154) 7:3:168
!1=2
9b vz;avg
) rp :
p g L where we have assumed v  v. Since the z-direction is the
(Note: In (7.3.167), 2b multiplies vz,avg; if the particle is at center- mean direction of the gas (also wind) motion, we can
line (entrance point) b will multiply vz,max. The product will still neglect the convective terms in the x-and y-directions
be the same.) The minimum size of particle which will settle is and incorporate their fluctuating contributions through
!1=2 dispersion terms (see Section 6.2.1.1):
18 2b vz;avg
r p jmin ) r p jmin C i 2 C i 2 C i
2 p g L vz Di;eff;x D i;eff;y : 7:3:169
 z x 2 y2
4 g 1=2
9  100 cm  25 cm s  1:80  10 cm s
g cm ; (For an introduction to this approach, see Wark and
2 cm3  980 s2  800 cm
 1=2 Warner (1976), chap. 4.)
9  25  1:80  10 4 An approximate solution of this equation has been
rp j
min 16  980 obtained subject to an emission rate QeCif from the stack
as a point source (at ground level, i.e. x 0, y 0, z 0) as
16  10 4 cm 16 m:
well as additional boundary conditions:
We will now very briefly illustrate the procedure
followed to determine the rate of deposition of particles vz C i x;y;z dx dy Qe C if ; z > 0: 7:3:170
on the ground from stack emissions by gravity. Recognize 0

first that it is quite a complex problem. A stack or a chim- As z ! 0; C i ! point source; delta function;
ney rises a distance h above the ground (in the y-direction) 7:3:170b
(Figure 7.3.19). However, before such a gaseous plume
coming out of the stack turns around and follows the wind, C i ! 0; when x; y; z ! zero concentration; far away;
it (being usually hotter than the surroundings) rises fur- 7:3:170c
ther, then turns around at a height H (>h) above the C i
ground and follows the wind direction (z-coordinate). As as y ! 0; Di;eff;y ! 0 no diffusion to the ground surface:
y
it goes with the wind in the z-direction, the gaseous pollu- 7:3:170d
tants diffuse and disperse in all three directions, x, y and z
(the ultimate objective of stack emission of pollutants). Here, Qe is the volumetric flow of the gas releasing pollu-
Since the axial velocity in the z-direction is substantial, tants at a concentration Cif. The solution is (see Wark and
dispersion in this direction may be neglected when Warner (1976), pp. 1412)

Gas plume
Virtual point source of
plume and particles
Centerline of
gaseous plume
in the direction
of wind velocity
H

Stack Particle falling


h

on the ground

x z Ground level

Figure 7.3.19. Gaseous plume rising from a stack and dispersing in the atmosphere as particles fall to the ground by gravity. (After Wark
and Warner (1976).)
7.3 External force field based separation: bulk flow perpendicular to force 637

  2  
Qe C if x y2 vz We now need to calculate the rate at which particles
C i x;y;z exp :
2z Di;eff;x Di;eff;y 1=2 Di;eff;x Di;eff;y 4z are hitting the ground (y 0) at x 0, the centerline of
7:3:171 the plume. The corresponding particle concentration is
given by
If you consider dispersion in either the x-or y-direction and
H U pyt z=vz 2
( )
replace the z-coordinate by t z/vz, this problem and the Qe p0
p 0;0;z;H exp :
solution follows in a manner similar to equations (3.2.12) 2 vz x y 2 2y
and (3.2.15), except that the limits for x and y here are ( , 7:3:176
) and that for z is 0 to . In reality, the pollutant source
coordinates are x 0, y H and z 0. Therefore the The mass rate of transport of these particles into the
solution above will be modified to ground per unit area is the mass flux of particles at location
( " z, ground;rp z (mass/time-area):
yH2
!# )
Qe C if x2 vz
C i x;y;z;H exp :
2zDi;eff;x Di;eff;y 1=2 Di;eff;x Di;eff;y 4z ground;r p z U pyt p 0;0;z;H; 7:3:177
7:3:172
H zU pyt =vz 2
( )
Qe p0 U pyt
ground;r p z exp :
This solution has the form of a Gaussian distribution if it is 2 vz x y 2 2y
written as follows:
7:3:178
y H2
( " ! #)
Qe C if x2 Since Upyt depends on the particle size and density, the
C i x;y;z;H exp ;
2 vz x y 2 2x 2 2y above result is valid for particular values of Upyt, i.e. par-
ticles of a particular size and density having a particular
if
Upyt. To determine the total particle deposition rate, we will
2 Di;eff;x z 2 Di;eff;y z have to sum it over all particle sizes/densities, or alterna-
2x ; 2y : 7:3:173
vz vz tively all particle terminal velocities.
Unlike separation devices generally encountered in the
Note that (z/vz) t; therefore, as time increases, the stand-
laboratory or used for industrial operations, in this case
ard deviation of the profile from the centerline (x 0,
nature and natural phenomena provide the environment
y H, z) increases (Figure 7.3.19) in both the x- and
for the separation of dust particles from the gaseous plume
y-directions.
released into the atmosphere from the stack. The key
We are interested in the rate at which particles come
unknowns in the equation (7.3.178) for ground;rp z are x
out of this gaseous plume and hit the ground via gravity.
and y , although there is some empiricism involved in
For particles of a given size/density having a particle set-
estimating Upyt also. Turner (1969) provides charts to esti-
tling velocity Upyt, the particle having the same coordinates
mate values of x (his y ), y (his z ) as a function of the
x, z as the pollutants will, however, have fallen toward the
axial distance z (his x) from the source for six types of
ground by a distance Upyt z/vz, where (z/vz) is the time t
atmospheric conditions.
over which the plume has traveled an axial distance z. If
Gravity is also the primary force for separation of
the particle mass concentration at the source is p0, we can
droplets from a gas/vapor phase into a liquid layer in
rewrite the solution for particle mass concentration p (x, y,
what are called knockout drums in many chemical and
z, H) from equation (7.3.173) in an analogous fashion as
petrochemical industries. Pressure relieving equipment
y H2 generally contains a large cylindrical drum into which
( " #)
Qe p0 x2
p x;y;z;H exp : the vapor/gas and liquid mixture are introduced at one
2 vz x y 2 2x 2 2y
end of the drum at the highest point. The droplets formed
7:3:174
are subjected to downward motion by gravity, an upward
At this time, we can introduce a correction for the vertical buoyancy force and a drag force by the gas/vapor escap-
drop by the particle from y H to y H (Upty z/vz) in time t: ing through the top exit at the other end of the drum.
(      2 ) Droplets follow a downward trajectory, ultimately falling
Qe p0 x2 y H U pyt z=vz into the liquid layer occupying about half the height of the
p x;y;z;H exp :
2vz x y 2 2x 2 2y drum. Those that do not will escape with the gas/vapor.
7:3:175 Knockout drum design therefore often incorporates con-
ditions such that droplets 100 m and larger are captured
Therefore the centerline of the particle plume slopes by coming down and touching the liquid; to ensure this,
downward (toward y 0) as z and t increase (Wark and the vapor velocity should be low enough to provide
Warner, 1976). enough residence time.
638 Bulk flow perpendicular to the direction of force

2b
Overflow, Q1, f1(Upzl), fs1

L
Underflow, Q2, f2 (Upzt), fs2

Feed, Qf, ff (Upzt), fsf

Figure 7.3.20. Inclined lamella settler with continuous flow for particle classification at steady state.

7.3.3.1 Inclined settlers total vol: flow rate : Qf Q1 Q2 ; 7:3:179


We have seen in Section 4.2.3.4 that inclined settlers pro- total solids vol:flow rate : Qf sf Q1 s1 Q2 s2 : 7:3:180
vide a much larger vessel bottom surface area for particles
to settle by gravity than simple vertical vessels; therefore, The total solids volumetric flow rate for particles having
inclined setters can clarify a much larger volume of liquid Upzt is given by
by simple gravity. If such an inclined settler, in the form of Qf sf f f U pzt Q1 s1 f 1 U pzt Q2 s2 f 2 U pzt : 7:3:181
a lamella settler (Figure 4.2.7) has, the suspension flow up
the channel slowly, the particles will slowly settle onto the Earlier theories (e.g. PNK theory), briefly identified in
bottom channel plate, slide down the channel wall and Davis et al. (1989), indicate that if one can assume that all
form an underflow stream. Larger particles will settle particles settle with the same settling (terminal) velocity
quickly and are going to show up in the underflow stream. U pzt , then the volumetric rate of production of the clarified
The overflow stream leaving at the top of the channel will liquid (Qc) will be
contain the finer particles (Figure 7.3.20). In this configur-
ation, only a component of the external gravitational force, Qc U pzt W L sin 2b cos ; 7:3:182
namely, g sin is perpendicular to the bulk flow direction
since we have already identified in Section 4.2.3.4 the
(similar to that in a disk centrifuge, Figure 7.3.14).
surface area for settling to be W(L sin 2b cos ) for a
When a suspension is introduced into the inclined
lamella settler having a channel plate width of W, channel
lamella settler, the feed suspension may be characterized
plate length L and channel plate gap of 2b. Now, this
by means of its solids volume fraction sf and its particle
clarified liquid flow rate, Qc, will combine with the unset-
size density function ff(rp). The corresponding quantities
tled feed suspension flow rate, namely (Q1 Qc), to make
for the overflow and underflow streams are: s1 , f1(rp); s2 ,
up the overflow volume flow rate Q1. For a balance on
f2(rp). Often such problems are analyzed instead using the
particle volume fraction, this implies
solids volume fraction s and the particle settling (ter-
minal) velocity density function f(Upzt), where the particle
Q1 s1 Q1 Qc sf : 7:3:183
settling velocity Upzt in the Stokes law range is related to
the particle radius rp by relation (6.3.1):
(This analysis assumes that the particle concentration in
U pzt 2=9 r 2p p t g=: the unsettled suspension entering the overflow is the same
as that in the feed suspension (Davis et al., 1989).) If there
Using such a formalism (Davis et al., 1989), we can obtain is a distribution of particle settling velocities, then, for
the following mass balance relations for the total volumet- particles having a particular settling velocity Upzt, the cor-
ric flow rate, total solids flow rate and the flow rate of solids responding balance is
having a particular settling velocity Upzt (subscript j 1,
overflow; subscript j 2, underflow): Q1 s1 f 1 U pzt Q1 Qc U pzt sf f f U pzt : 7:3:184
7.3 External force field based separation: bulk flow perpendicular to force 639

If the clarified liquid production rate for particles having a Q1 Qc U pzt f f U pzt
settling velocity Upzt is Qc(Upzt) and all particles have the f 1 U pzt U pztc ; 7:3:189
same Upzt, then Q1 Qc U pzt f f U pzt dU pzt
0

Qc U pzt  Q1 and f 1 U pzt 0: 7:3:185 where

However, when there is a distribution in Upzt, the analy- U pzt < U pztc 7:3:190
sis is somewhat complicated. Let Qc(Upzt) in relation
and
(7.3.184) now denote the volumetric rate at which the
suspension containing particles with settling velocities f 1 U pzt 0 for U pzt  U pztc : 7:3:191
less than Upzt is produced due to settling of particles
having settling velocities greater than Upzt. One can Correspondingly, the probability density function f2(Upzt)
now integrate relation (7.3.184) over all particle settling of the underflow stream may be obtained by substituting
for s1, s2 and f1(Upzt) in equation (7.3.181):
U pztc
Qf sf f f U pzt sf f 1 U pzt Q1 Qc U pzt f f U pzt dU pzt
f 2 U pzt  U pztc  7:3:192
Qf fQ1 Qc U pzt g f f U pzt dU pzt
0
Qf Q1 sf
Qf Q1
 U pztc 
Qf f f U pzt f 1 U pzt Q1 Qc U pzt f f U pzt dU pzt
0
) f 2 U pzt  U pztc  ; 7:3:193
Qf Q1 Qc U pzt f f U pzt dU pzt
0

where we have employed equations (7.3.179) and


velocities such that the overflow production rate Q1 is
(7.3.187). Substituting for f1(Upzt) from equation (7.3.189),
now defined as
we get
Q1 Qc U pztc ; 7:3:186
Qf Q1 Qc U pzt f f U pzt
f 2 U pzt :
where Upztc is the cutoff settling (terminal) velocity such
 U pztc
that particles having Upzt > Upztc settle to the bottom plate Qf Q1 Qc U pzt f f U pzt dU pzt
0
before reaching the overflow: 7:3:194

Q1 s1 f 1 U pzt dU pzt Q1 s1 sf Q1 Qc U pzt f f U pzt dU pzt
0
0
U pztc
Q1 Qc U pzt Q1 Qc U pzt
) s1 sf f f U pzt dU pzt f f U pzt dU pzt :
0 Q1 U pztc Q1

The second integral on the right-hand side is zero from


definition (7.3.186), leading to
U pztc Note: We have generally employed particle size density
Q1 Qc U pzt functions ff (rp), f1(rp) and f2(rp), where the probability
s1 sf f f U pzt dU pzt : 7:3:187
0 Q1 density function depends on the random variable rp, the
The particle volume fraction in the underflow, s2 , is now particle radius. In the analysis considered here for inclined
obtained from the total particle balance (7.3.180) as settlers, we are dealing with density functions ff(Upzt), f1
 U pztc  (Upzt) and f2(Upzt). Since, by relation (6.3.1), the relation
between Upzt and rp is (if Stokes law is valid)
sf Qf fQ1 Qc U pzt gf f U pzt dU pzt
0
s2 : U pzt 2=9r 2p p t g=;
Qf Q1
7:3:188 we have
One can obtain the probability density function f1(Upzt) of f U pzt f r p =jdU pzt =d r p j; 7:3:195a
the overflow stream by substituting (7.3.187) in to relation
(7.3.184): f U pzt 9 f r p =4 p t g r p : 7:3:195b
640 Bulk flow perpendicular to the direction of force

7.3.4 Field-flow fractionation for colloids, may be created by a variety of fields (to be discussed soon).
macromolecules and particles At steady state, there should be no net flux of species i in
the y-direction as the flux due to the field is counteracted
This technique is primarily utilized in a relatively small
by back diffusion of species i away from the wall:
scale of operation. It involves a somewhat different inter-
action of the bulk flow with the force operating perpen- dC is
N iy 0 C is U iy Dis ; 7:3:199a
dicular to the bulk flow direction compared to what we dy
have seen so far. So far, whenever we have employed the
dC is
bulk flow, limited attention was paid to the fact that there Dis C is U iy : 7:3:199b
dy
is a velocity profile in the bulk flow taking place in the
channel/device. Such velocity profiles can often be dam- A solution of this equation is obtained from
aging to the separation achievable (for example, see, the
dC is U iy U iy U iy
 
comments in the paragraph preceding equation (6.3.6)) or dy ) nC is y a ) C is C i0 exp y ;
C is Dis Dis Dis
they can introduce complications/distortions in the sep-
aration achieved (see Figures 7.3.2(b) and 7.3-3 in thin- 7:3:200
film continuous-flow electrophoresis). In the field-flow where Ci0 is the concentration of species i at the wall, y 0.
fractionation (FFF) technique, however, the velocity pro- The direction of the force field applied is such that Uiy is a
file of bulk flow in the channel is crucial to the separation negative quantity. We can therefore write the above
achieved. Before we discuss this feature, let us describe concentration profile as
the effect of a force field applied perpendicular to the bulk  
flow direction in a channel where a suspension of col- jU iy j
C is C i0 exp y ; 7:3:201
loids, macromolecules or particles is flowing. The force Dis
field can be electrical, centrifugal, gravitational, thermal
where jU iy j is the magnitude of the force field induced
diffusion or crossflow from a pressure gradient across the
velocity of species in the y-direction. One could define a
channel wall.
characteristic24 thickness i of this concentration profile by
Let the dimensions of such materials in suspension
defining
vary between the wide range of 1 nm and around 100 m
(Giddings, 1993). As shown in Figure 7.3.21(a) , a force Dis
i : 7:3:202
field applied in the negative y-direction in the channel jU iy j
creates a flux of macromolecules/colloids/particles toward
Rewrite the concentration profile (7.3.201) now as
the wall. For macromolecular species i, we may write the
following expression23 (See 3.1.88): y
 
C is C i0 exp : 7:3:203
i
dC is
N iy C is vty U iy Dis : 7:3:196
dy If there are two macromolecular species a and b, the
characteristic thicknesses of their profiles extending from
If the flow in the channel is laminar and fully developed,
the wall out are a and b , respectively. Species subjected
then vty 0 and
to a lower jU iy j and possessing a higher Dis will have a
dC is larger i. Therefore the larger species, whose diffusion
N iy C is U iy Dis ; 7:3:197
dy coefficient at infinite dilution D0is may be described by the
StokesEinstein equation (3.3.90c)
where Uiy is the force field induced migration velocity of
species i in the y-direction. For particles, an expression for kB T
the particle number flux npy in the y-direction in terms of D0is ; 7:3:204
6 r i
the total number density Nt of particles may be written
from (3.1.70) as will have a smaller D0is , and therefore a smaller Dis. For two
macromolecular species a and b having similar Uiy (say), if
dN t ra > rb, Day < Dby; therefore a < b. Thus the smaller
npy N t U py Dp : 7:3:198
dy macromolecular species will have a profile whose average
This field induced macromolecular migration velocity Uiy extends out further from the wall (Figure 7.3.21(a)).
normal to the channel wall and toward the wall at y 0

24
The characteristic thickness here corresponds to a distance where
23
There is also an axial flux Niz since Cis(y,z) is a function of y and z. the value of Cis has been reduced to 36% of its value at the wall;
Here we assume that Cis(y,z) may be represented as Cis(y) f(z) and therefore the bulk of the molecules are contained in the region y 0
focus on Cis(y). to y i.
7.3 External force field based separation: bulk flow perpendicular to force 641

(a)
Parabolic
velocity Force field Channel
profile wall

Detector
response

b
y
b a
z a b
Channel wall da db time

(b)
1.0
Ri (c)
Liquid in crossflow
6li
0.8 Porous
wall
0.6 Equation
Ri (7.3.214)
a b Membrane
0.4 (10 000
MWCO)
Porous
0.2 wall
Crossflow out

0
0 0.1 0.2 0.3 0.4 0.5 0.6
li = (di / b)
(e)

(d)
Flow inlet Flow to
(sample injection) detector Flow b
Flow
b
Spin

Exploded a
Flow
W

view

Figure 7.3.21. Field-flow fractionation (FFF). (a) Basic configuration of field-flow fractionation device and the detector response to a
sample. (b) Retention ratio of species i vs. retention parameter i. (c) Schematic for flow field-flow fractionation with a crossflow.
(d) Physical configuration/dimensions of a FFF channel with inlet/outlet. (e) Configuration of the channel in sedimentation FFF. (After
Giddings (1993).)

Field-flow fractionation exploits this difference in the wall, whereas the smaller molecules reside primarily in
distance of the mean of the species profile from the chan- higher axial velocity zones further away from the wall.
nel wall by coupling it with the velocity profile in the Therefore the smaller molecules will show up (via a
channel. In laminar channel flow with a parabolic velocity detector) at the channel exit faster than the larger mol-
profile (equation (6.1.2b)), the larger molecules are then ecules if there is an injection of a sample containing differ-
concentrated in the slower axial velocity zones closer to the ent macromolecular species upstream in the channel
642 Bulk flow perpendicular to the direction of force

(f) Hot wall (g)


Electrode
Membrane
Force field

Membrane
Cold wall a b
Electrode

(h)
Force field

Detector
y response
Channel wall
r3 r2 r1
z
Parabolic r1 r2 r3 > r2 > r1 Time
velocity profile

(i)
Sample liquid flow Particle product fraction a
Channel wall Force field

Inlet
splitter Transport laminae
Outlet
splitter

Channel wall
Carrier liquid flow Particle product fraction b

Figure 7.3.21. (cont.) (f) Thermal FFF. (g) Electrical FFF. (h) Steric FFF and the detector response to a sample injection. (i) SPLITT
fractionation.

(Figure 7.3.21(a)). It is as if fluid lamellae (laminae) of each given by25 (Happel and Brenner, 1965; Grushka et al.,
species located at different distances from the wall (y 0) 1973; Giddings, 1991)
travel at different velocities; therefore they arrive at different  
y y2
times at the channel end. (For a comparison, in elution vz y 6vz;avg 2 ; 7:3:205
b b
chromatography (Section 7.1.5.1) different species also arrive
at different times at the end of the column; however, each where
species exists throughout the column cross section as the
species peak travels. Similarly, in capillary electrophoresis Pb2
vz;avg ; 7:3:206
(Section 6.3.1.2) species arriving at the end of the capillary 12 L
at different times exist throughout the capillary cross section.) L is the channel length and P is the axial liquid pressure
This technique was first proposed by Giddings (1966). drop. If a macrosolute which is not affected by the force
Quantitative analysis of the retention time, t Ri , of a field is injected into the liquid upstream, then t R0 , its
macrosolute species in the channel after sample injection
may be carried out as follows (Giddings, 1991). The axial
velocity profile, vz y, of a liquid of viscosity in a thin 25
Equation (6.1.2b) provided earlier is for a slit flow where the
rectangular channel formed between two infinite parallel channel gap << width of the channel plates < length of the
plates (Figure 7.3.21(a)) spaced a distance b apart is channel in the mean flow direction (Bird et al., 2002).
7.3 External force field based separation: bulk flow perpendicular to force 643

retention time (i.e. the time when it shows up at the exit) 6 Dis
lim Ri 6i =b 6i ; 7:3:215
will be essentially determined by the average liquid vel- i ! 0 jU iy jb
ocity, vz;avg :
lim Ri 6 RT=F ext
tiy b; 7:3:216
t R0 L=vz;avg : 7:3:207 i ! 0

The retention time t Ri of any macrosolute species i affected


where we have employed definition (3.1.82) for the migra-
by the force field can be determined if we have the know-
tion velocity Uiy of species i in the y-direction and
ledge of an average velocity of the liquid zone carrying this
Dis D0is RT=f di (relation (3.1.86)). To be able to detect
species i, vci :
two different species A and B at the outlet of the channel,
b
we need to have a reasonable value of the difference,
C is y vz ydy
t Ra t Rb t R ; which is given by
vci o b : 7:3:208
1 1 t R0 b ext
 
C is ydy

o
t R t R0 F tay F ext
tby : 7:3:217
Ra Rb 6RT
Substituting expressions (7.3.203) and (7.3.205) into the
To that end t R should be such that the ratio, t R =t R0 ,
above relation, we get
should be of order 1:
b  
y y2 
6vz;avg exp y=i  2 dy

b F ext ext 
tay F tby  b F ext F ext

o b b t R gmole tay tby 
macromolecule
vci b : 7:3:209 ;
t R0 6RT 6k B T
exp y=i dy
o 7:3:218

Defining t Ri as the retention time of species i in the chro- since R kB N~ , where kB is Boltzmanns constant. The
matographic column (here the channel) as channel gap in FFF techniques is around 100300 m
(typical length, 30 cm, 2 cm width). Assume b 300 m.
t Ri L=vci ; 7:3:210
For T 298 K, if t R =t R0 is going to be around 1, then the
one can determine a retention ratio for species i, Ri, force difference F ext
t jmolecule required is
defined as  6k B T
vci tR F ext ext ext
t jmacromolecule F tay F tby jmacromolecule
Ri 0: 7:3:211 b
vz;avg t Ri
6  1:3807  10 23 newton-m  298 K

From expression (7.3.209) for vci ,
we get 300  10 6 m K
b b 8:22  10 17 newton 10 16 newton;
6 6
y exp y=i dy 2 y 2 exp y=i dy
b o b o which is not much (Giddings, 1993).
Ri b
exp y=i dy
o

6i exp i 1 i 1 i 6 i exp i 1 2i exp i 1 i 1 22i



; 7:3:212
i exp i 1 1

Most of the above analysis was carried out for macro-


where i is a retention parameter for species i defined as
molecules. One could also carry out an almost identical
i i =b: 7:3:213 analysis for particles using the particle flux expression
(7.3.198), as long as the particle dimensions are less than
Hovingh et al. (1970) and Giddings (1991) have shown that
0.1 m. The following selectivity definitions have been
this result for Ri yields, after rearrangement,
used in FFF techniques for macromolecules (molecular
1
   
Ri 6 i coth 2i : 7:3:214 weight M) and particles (particle diameter dp):
2i  
 d log t 
R
macromolecules : SM  ; 7:3:219a

Figure 7.3.21(b) (after Grushka et al. (1973)) schematically  d log M 
illustrates the retention ratio Ri as a function of the
 
 d log t 
R
retention parameter i. As the characteristic thickness particles : Sdp  : 7:3:219b

 d log d p 
of the concentration profile i decreases, the retention
ratio Ri decreases linearly. In the limit as (i/b) ! 0 Alternative definitions of selectivity, employing the reten-
(i.e. i ! 0) tion ratio Ri instead of t Ri , are:
644 Bulk flow perpendicular to the direction of force

 
 d log R  similarly for particles (<1 m) of size down to 1 nm (Gid-
 i
SM  ; 7:3:220a dings, 1993). Consider a channel (Figure 7.3.21(d)) where
 d log M 
the channel volume is Vchannel. Let the volumetric flow rate
 
 d log R  of the crossflow be Qcrossflow. Then, from relation (7.3.222),
 i
Sd p  : 7:3:220b
 d log d p  i Dis Dis Dis W Lb
i 2
b jU iy jb Qcrossflow b b Qcrossflow
It is useful now to explore the force fields typically WL
employed and the corresponding FFF techniques. These 7:3:223
Dis V channel
are: 2 ;
b Qcrossflow
(1) flow FFF (viscous drag force in crossflow);
(2) sedimentation FFF (centrifugal force); where W is the width of the channel, L is its length and b is
(3) thermal FFF (thermal diffusion due to a temperature the gap between the channel plates. Experimentally, one can
gradient); determine Ri from (7.3.211) from measurements of t Ri and
(4) electrical FFF (electrical potential gradient). t R0 . If one then employs the result (7.3.214), one can deter-
In flow FFF (Giddings et al., 1976), the channel walls are mine i . Substitution of this i into the result (7.3.223) derived
porous (e.g. frits are used), as shown in Figure 7.3.21(c). above will yield Dis for that fraction/species/the particle for
Liquid crossflow is imposed at the porous top wall and it known experimental values, of b, Vchannel and Qcrossflow. Then
goes out through the porous bottom wall of the channel. the StokesEinstein relation (3.3.90c) will yield ri for the
This flow in the negative y-direction is superimposed on macromolecules. If relations like (3.3.90d) are available, the
the main channel flow in the z-direction; this crossflow molecular weight will be known for the fraction. Thus, a
drags macromolecules/particles to the bottom porous wall prediction of which macromolecule will appear at what time
(frit). To prevent macromolecules/particles from leaving t Ri is possible in this technique. It is a similar case for
through the porous channel wall, a membrane that retains particles; however, when particles are larger than a few
all macromolecules/particles to be separated is employed hundred nanometers, there are wall effects due to the
on top of the bottom porous wall; the liquid goes through particlewall interactions, for example, by geometrical exclu-
the membrane and the porous channel wall at a velocity sion (see relations (3.3.88a,b)). This will be considered later.
jU y j. The macromolecules (or particles) experience a drag For sedimentation FFF, consider the thin channel going
force in the negative y-direction due to this negative y- around a centrifuge basket like a thin belt (Figure 7.3.21(e))
directional fluid motion, whose magnitude is given by (if with appropriate arrangements for incoming flow and
we assume Stokes law) outgoing flow. The radial force component on a particle
of density p, volume Vp, radius rp in a liquid of density t
k B TjU y j rotating at an angular velocity at radial location r is
jF tiy jmolecule f di jmolecule jU y j 3 d i jU y j :
Dis obtained, from expression (3.1.59), as
7:3:221
4 3 
F ext
tpr r 1 t r 2 : 7:3:224
(Note: 3 p p p

RT kB N~T kB T This may be rearranged in terms of the magnitude of the


f di jgmol Nf
~ di jmolecule ; Dis ;
f di jgmol d
N f i jmolecule 3di
~ force jF ext
tpr j as follows:

see relations (3.1.86), (3.1.91d) and (3.3.90c)). One can d 3p


clearly see now from definition (7.3.202) that, for a given jF ext
tpr j jp t j r 2 : 7:3:225
6
crossflow velocity Uy,
Similarly, the magnitude of the radial force component on
Dis kB T one gmol of species i may be obtained from (3.1.52) as
i ; 7:3:222
jU iy j 3 d i jU y j
Vi Ms
    
leading to different values of i for macromolecules experi- jF ext
tir jgmol M i 1 r2 : 7:3:226
Mi Vs
encing different drag forces due to their different sizes.
Larger molecules will have smaller i closer to the wall. Correspondingly, the force on one macromolecule will be
The same analysis is also valid for microsized particles.
Mi Vi Ms
    
This technique, often identified as FLFFF (flow field- jF ext
tir jmolecule 1 r2 : 7:3:227
~
N Mi Vs
flow fractionation), is generally the most useful of the
different FFF techniques (Kirkland and Dilks, 1992). It has Now, the radius of this thin channel inside the centrifuge is
been used to separate polymers when the molecular large; so the radial variation at different radial locations in
weights (MWs) of polymers are in the range 104107 and the channel for a given radial direction may be neglected.
7.3 External force field based separation: bulk flow perpendicular to force 645

Therefore, regardless of the radial location where the where DTis is the thermal diffusion coefficient for species i
solute/particle band is (in Figure 7.3.21(e)), the centrifugal whose mole fraction in the dilute solution is x is (therefore
acceleration, r2, may be assumed to be a constant, (1 xis) ~ 1).
equal to Gc. Due to the very high temperature gradient, the solution
We may now estimate i for a given macromolecular density, and therefore Ct, will vary with distance y. Since
species i from definition (7.3.202) as the macrosolute concentration Cis Ct xis, we may write
Dis Dis RT RT dC is dx is dC t
i ext ext h  i : Ct x is ; 7:3:230
jU ir j jF tir jgmol =f di jF tir jgmol M i 1 V i M s Gc dy dy dy
Mi Vs
dC is dx is dC t dT
  
Correspondingly, Ct x is : 7:3:231
dy dy dT dy
RT
i h  i : 7:3:228 Therefore, at steady state,
Vi Ms
Mi 1 M i V
Gc b
s
dC is DTis dT C is dC t dT
     
J iy jtotal 0 Dis C is ;
The equivalent result for particles of radius rp, diameter dp dy Dis dy Ct dT dy
and density p is 7:3:232

Dp Dp f dp Dp f dp 1 dC is dnC is DTis
1 dC t dT
  
i r p : 7:3:233
jU pr j jF ext
tpr j d 3p C is dy dy Dis C t dT dy
jp t jGc
6 The quantity C t1 dC t =dT is a coefficient of thermal
B B
7:3:229
k T 6 r p 6k T expansion analogous to (see equation (6.1.10)) and will
3 ;
d 3p dp jp t jGc be indicated here as c . Therefore
6 r p jp t jGc
6 dn C is
 T
Dis

dT
c : 7:3:234
dy Dis dy
where we have used the StokesEinstein equation (3.3.90c)
for the particle diffusivity Dp and f dp 6 r p from Stokes As the temperature decreases in the negative y-direction (T
law. One can now determine the retention ratio Ri from increasing with y), Cis, however, decreases with increasing
equation (7.3.214) for any macromolecular species or par- y from the bottom wall. From the form of equation
ticle. For particle separation, it is clear from expression (7.3.199b), we see that
(7.3.229) that i(rp) will differ significantly from particle to  T
Dis dT
 
particle due to the dp 3 dependency; a small difference in rp dn C is c dy:
will be magnified substantially. However, to separate Dis dy
smaller particles and macromolecules, high values of Therefore,
are required, which leads to problems of sealing liquid    
inflow/outflow. Therefore sedimentation FFF is applied to  U   DT 
dT  1
 iy   is
 c  : 7:3:235
particles of size larger than 1030 nm and to polymers  Dis   Dis dy  i

having a molecular weight >106107 dalton.
In thermal FFF, one channel wall (Figure 7.3.21(f)) is Since c is negligible under the conditions employed in
kept hotter than the other to create a temperature gradient thermal FFF, we get
(dT/dy) of sufficient magnitude; in practice, values ~104 Dis
K/cm are maintained. For a channel gap b of, say, 300 m, i : 7:3:236
DTis dT=dy
a temperature difference T of 300  104 cm  104 K/cm
300 K is required. Polymers and particles are supposed For a linear temperature gradient,
to be driven toward the colder wall (as in thermal diffu-
sion), and this motion is opposed by diffusion. (See rela- b Dis
i : 7:3:237
tion (4.2.63) for gas species; the approach to be used here DTis T
is similar.) Although there are many complications,
Therefore i =b should be linearly proportional to T1;
Grushka et al. (1973) have employed the following expres-
this behavior has been verified experimentally. See, for
sion for the net molar flux for a polymer species (which
example, the behavior of i =b for the macromolecule
becomes zero at steady state):
polystyrene of molecular weight 51 000 in the solvent ethyl-
dx is dT benzene in Grushka et al. (1973). If we can assume a
J iy jtotal Dis C t DTis C t x is 1 x is 0;
dy dy certain type of dependence of Dis on the molecular weight
of the polymer (e.g. r i / M i (see relations 3.3.90df);
diffusive fluxJ D T
iy flux due to thermal diffusion J iy therefore, from the StokesEinstein relation (3.3.90c)
646 Bulk flow perpendicular to the direction of force

D0is / M i ), it is clear from relations (7.3.211), (7.3.214) At this time, it is necessary to point out two very
and (7.3.215) that, as i ! 0, Ri 6i and important characteristics of the separation techniques
being studied. First: So far, the technique is a chromato-
1 tR 1 1 DTis T
i / DTis T M i ; 7:3:238 graphic technique (Section 7.1.5) with samples
Ri t R0 6i 6 Dis
being injected upstream and different macromolecular
allowing one to fractionate the polymers of different species/particles/proteins detected/separated at different
molecular weights, provided the variation of DTis with Mi retention times. Second: No wall effects were considered
does not interfere. The basis of variations of DTis is, how- (see discussion after equation (7.3.223)). This second issue
ever, quite complex. is quite important, and it ends up influencing the first issue
In electrical FFF, the channel walls are bounded by semi- as well.
permeable membranes (having a molecular weight cutoff of When the particles are larger than a few hundred
10 000 daltons), with electrodes located on the outside of the nanometers, a few other factors not considered so far
channel walls. As the electrical field is applied perpendicular become important. First: The variation of particle diffusion
to the channel flow, small ions in the buffer solution move coefficient is no longer important. Second: The magnitudes
toward the electrodes through the semipermeable mem- of the forces become much larger. Third: Most importantly,
branes. However, charged macromolecules/proteins/col- a region of the wall up to a thickness of ri from the wall is
loids cannot go through the membranes and are pressed to no longer accessible to the center of the particle due to
the channel walls. The migration velocity of species i toward geometrical exclusion or steric exclusion (Figure 3.3.5A).
the wall, Uiy, in an electrical field of magnitude Ey, is m Since in normal laminar flow there is almost no back
i Ey
(see relation (6.3.8f)). So diffusion for larger particles, the force acting on the
particle pushes it to the bottom wall (y 0) and the particle
Dis
i ; 7:3:239 essentially stays there. Thus, the centerline of the particle,
m
i Ey which is at y ri, is also equal to i, a characteristic
where m thickness of the layer of particles of radius ri. These par-
i is the ionic mobility of species i. As long as the
ratio Dis =m ticles will move with the fluid velocity characteristic of the
i of the macromolecular species i varies signifi-
cantly with i under otherwise constant conditions, one can region y ri. Larger particles will elute much more quickly
separate different charged macromolecules/proteins/ (and have smaller retention times) since their centers are
charged colloids by electrical FFF (Grushka et al., 1973). at a larger y and are exposed to a higher fluid velocity
Caldwell and Gao (1993) have studied this subject in (Figure 7.3.21(h)). This is called the Steric elution order.
some detail. For colloidal particles, they have pointed out These techniques (depending on the force field) are useful
that, for smaller -potentials (see equation (3.1.11a)), the for larger particles (0.5200 m) and are generically identi-
electrophoretic mobility m fied as steric FFF. On the other hand, all the other tech-
i can be described by
niques studied earlier (for smaller particles and
B
m
i 2 d k T=3ef r i =; 7:3:240 macromolecules), where geometrical exclusion effects are
negligible and Dis is important, are called normal-mode
where d is the dielectric constant of the medium, kB is FFF, and they display normal elution order, where smaller
Botlzmanns constant, is the liquid viscosity, is the particles elute faster/earlier.
Debye length and f (ri/), the Henry function, increases From Figure 7.3.21(h), it is clear that the value of the
monotonically from 1 to 1.5 as ri/ ranges from zero to retention parameter i of particles of radius ri should be
infinity. Therefore m
i is relatively insensitive to particle/ equal to (ri/b) and that for i ! 0, the retention ratio for
colloid/macromolecule size, varying by a maximum factor particle i, Ri 6i from equations (7.3.213) and (7.3.214). In
of 1.5 at a given ionic strength in a given liquid. Conse- general, however, there exist hydrodynamic effects which
quently, the variation of i appears primarily due to vari- tend to increase i beyond ri. This is accommodated via the
ation in Dis, which varies essentially inversely with the ri of following expression for Ri (Ratanathanawongs and Gid-
the particle/colloid/charged macromolecule. dings, 1992):
Caldwell and Gao (1993) applied voltages 2 volts
Ri 6i r i =b; 7:3:241
across the channel walls (the channel thickness ~178 m)
and were able to separate a variety of polystyrene latex where i is a dimensionless steric correction factor. In steric
particles with sizes ranging from 60 nm to much larger FFF, i 1. However, there often exist hydrodynamic lift
values. Grushka et al. (1973) have illustrated the separation forces which raise the particle band higher up from the
of protein mixtures such as albumin, hemoglobin and wall, and i  2. Such operating modes are called lift
-globulin. In both of these, and other related, studies, hyperlayer FFF. Note: The shear-induced diffusion coeffi-
the voltage gradient applied was orders of magnitude cient of particles is proportional to r 2p , therefore r 2i (here);
smaller than that in capillary electrophoresis (see Section so larger particles have higher shear-induced diffusivity.
6.3.1.2). Thermal effects were found to be minimal. However, regardless of the general mode of operation used
7.3 External force field based separation: bulk flow perpendicular to force 647

in FFF, normal or steric, it is desirable that i < 0.2b due to a higher flow rate, particles tend to get lifted off, and
(Ratanathathanawongs and Giddings, 1992). the shear-induced particle diffusivity becomes substantial
One of the important characteristics of steric FFF for (see, e.g., relations (3.1.74) and (7.2.131a)). Larger particles
particle fractionation is that it can take place continu- will then have their centers located at a y larger than (dp/2).
ously; i.e. the feed particle mixture can be fed continuously The existence of diffusion becomes useful in another
at the channel inlet, and one can obtain continuously separation context. If a sample stream comes in through
several product fractions at the channel outlet (Figure the feed inlet splitter (Figure 7.3.21(i)) and it has larger and
7.3.21(i)). Usually, the outlet flow is split into two fractions smaller particles/species, what can happen in the absence
by an outlet splitter (Fuh et al., 1992), although multisplit of any force field is as follows. The smaller particles/mol-
outlets have also been realized (Giddings, 1985) to produce ecules can diffuse rapidly into the lower stream generated
several different fractions simultaneously. In both of these by the carrier liquid. Therefore the liquid leaving through
configurations, there is also an inlet splitter, with the the upper outlet stream a is now substantially purified of
sample being introduced from the top and the carrier smaller molecules/particles (Williams et al., 1992). The
liquid from the bottom. However, unlike normal-mode only force that exists here perpendicular to the feed flow
FFF, which can develop a complete particle size distribu- direction is that due to diffusion (which we do not identify
tion or molecular weight distribution via the elution mode as a force in the sense of other forces; see the explanation
of operation, steric FFF produces a finite number of frac- following equation (3.2.22b)).
tions regardless of the mode of operation, elution or In all the FFF techniques considered so far, the flow
continuous. took place in the environment of a channel between two
In steric FFF with an inlet splitter and an outlet splitter wide flat plates. The channel plate widths are orders of
(the technique is called SPLITT fractionation (SF) (Gid- magnitude larger than the gap between the plates (e.g.
dings, 1985; Fuh et al., 1992), the sample-containing liquid 2 cm  200300 m). However, there is always an edge
enters through the top while the sample-free carrier liquid effect at the two ends of the width of the plate. Much more
is introduced from the bottom, with the inlet splitter in important, however, is the requirement in flow FFF and
between (Figure 7.3.21(i)). Usually the carrier liquid flow electrical FFF that there be a membrane lining the channel
rate exceeds that of the sample-containing stream and to allow crossflow permeation in flow FFF and buffer ion
compresses it to a thin laminae. The y-location of the outlet transport in electrical FFF.
splitter is at a lower level; the in-between region is called If the flow channel is provided by the lumen of a porous-
the transport laminae. A particle must cross the thickness wall hollow fiber membrane, then both aspects are taken
of this transport laminae to appear in the outlet splitter care of. Edge effects are eliminated due to the cylindrical
stream b. geometry of the hollow fiber. Flow entering the hollow fiber
If the velocity Upy may be described for a sedimenta- at one end is automatically divided into two parts: one part
tion process (gravity and/or centrifugal forces, see equa- that permeates through the wall creating the needed cross-
tion (4.2.16b) for example) via flow, and the other part that flows straight through with
different particles/macromolecules having different reten-
U py sp 2 r sp Gc ; 7:3:242
tion times (Lee et al., 1974). In the case of electrical FFF, the
then a particle having a diameter larger than a critical hollow fiber is placed in the wide gap between two planar
diameter dp,cr will appear in the outlet splitter stream b electrodes (Lightfoot et al., 1981). This technique has been
(bottom), whereas those that are smaller will appear in the called electropolarization chromatography.
upper outlet splitter stream a. The value of dp,cr is given by Example 7.3.7 Determine the dependence of i on the
(Fuh et al., 1992) radius rp or ri of the particle or macromolecule in (1) normal
( )1=2 FFF techniques: sedimentation FFF; flow FFF and (2) steric
18V_ a V_ a0 FFF. Comment on how the selectivity Sdp will vary with rp.
d p;cr ; 7:3:243
W L 2 rp t You may exchange ri for rp or dp for di anywhere. (3) Are
there any implications vis--vis the separation time?
where V_ a is the volume flow rate through the upper
Solution (1) Consider sedimentation FFF. From relation
outlet splitter stream a, V_ a0 is that coming in through
(7.3.229),
the feed inlet splitter, W is the width of the channel of
length L, 2r (or g for a gravitational field) is the centrifugal 1 1
i r p / / 3:
field strength applied normal to the flow and sp is the d 3p rp
sedimentation coefficient from (4.2.16b) defined here as
From the definition of selectivity (7.3.220b),
p t d2p =18. Thus larger and heavier particles
emerge through the bottom stream at the outlet.  
 d log R 
 i
There is a complication in steric FFF. As we have seen Sdp  :
 d log d p 
earlier, when exposed to a high shear field near the wall
648 Bulk flow perpendicular to the direction of force

Assume now that i ! 0; then Ri 6i 6i/b. Since, from where


(7.3.229), erg
    R 8:317 joule=gmol-K 8:317  107
6k B T  d log d 3    gmol-K
 p   
Ri 3 ) Sd p    3 3: g-cm2
dp jp t jGc b  d log d p    8:317  107
s2 -gmol-K
Next consider flow FFF. From (7.3.222), Ri 6i 6i/b; so
    and T 298 K. Consider first the T2 virus particle of Mi 49
6k B T  d log d 1 

 
   106g=gmol. Also we have that
i 
Ri ) Sdp    1 1:
3di jU y jb  d log d i    
2000 rpm rad 2

Gc r2 15 cm  2
(2) Consider steric FFF. We have 60s=min rev
2
200 cm

   
 d log R 
 i
 d log r 
 i
15 ; b 300  10 4 cm;
Ri 6i r i =b ) Sdp     1 3 s2
 d log d p   d log dp 
so we have
g-cm2
8:317  107  298
s2 -gmol
i 0 1
6 g 15  4  104  2 cm 4 1
49  10   300  10 cm  @1 A
gmol 9 s2 1:57
8:317  107  298  9  104
0:0705:
49  106  60  104  2  300  0:363

(assuming i is independent of ri). We can safely assume that for such low i, Ri 6i 0.423.
(3) From relation (7.3.211), we obtain for two particles/ For the T7 virus particle of Mi 240  106, and having the
species a and b (ra > rb), same density 1.57 as before,
t Ra Rb 6b 6b 49
: i 0:0705  0:0144; Ri 6i 0:0863;
t Rb Ra 6a 6a 240
Now, in sedimentation FFF from (7.3.229), other things a substantially different retention parameter and ratio
being equal, (almost inversely proportional to Mi).
t Ra b r3
3a :
t Rb a rb 7.3.5 Magnetic force field
So if ra 10rb, t Ra 1000 t Rb . Correspondingly, in flow FFF
A magnetic force field has been employed to separate par-
t Ra b r a ticles having differing magnetic properties, primarily from a
) t Ra 10t Rb :
t Rb a r b liquid, via the technique of high-gradient magnetic separ-
In steric FFF, from (7.3.241) (assuming a b), ation (HGMS). This technique requires the presence of
ferromagnetic cylinders (steel wools, etc.), to develop strong
t Ra Rb rb
; t Rb 10t Ra : variations in the magnetic force field on the particles in the
t Rb Ra ra
local regions around the ferromagnetic cylinders, leading to
These calculations indicate that the time needed to achieve the capture of particles on those cylinders. The technique of
resolution between the largest and smallest particles (if they free-flow magnetophoresis, on the other hand, is simpler: it
vary by a ratio of 10) can be very high in sedimentation FFF, does not employ additional ferromagnetic cylinders or
whereas that in flow FFF/steric FFF is much less. spheres, and is somewhat similar to free-flow electrophor-
Example 7.3.8 Grushka et al. (1973) report obtaining good esis (Section 7.3.1). We will describe initially the free-flow
values of the retention ratio for the two virus particles T2 (Mi magnetophoresis based particle separation. Then we will
49  106, i 1.57) and T7 (Mi 240  106, i 1.57) when provide a brief treatment of HGMS based particle separ-
using sedimentation FFF at a moderate angular velocity of ation from a fluid. However, a few basic features of particle
2000 rpm. Assume that the centrifuge radius is 15 cm and the motion in a magnetic field will be considered first.
channel gap is 300 m at 25  C. Calculate the values of the As pointed out in Section 3.1.2.4, there are three
retention parameter and retention ratio for both virus particles. classes of magnetic materials: ferromagnetic materials
Solution: We will calculate first the value of i. From the (Fe, Co, Ni), which are strongly magnetic; paramagnetic
result (7.3.228), materials (which are far less magnetized compared to
RT ferromagnetic materials); and diamagnetic materials.
i   ; Expression (3.1.24) for the magnetic force on a nonferro-
Vi Ms
Mi 1 M Gc b
i V s magnetic spherical particle is
7.3 External force field based separation: bulk flow perpendicular to force 649

!
m m
p s mag 2 2 p s
F mag 2 m 3
s rp rH m
0 :
2
7:3:244 U pzt momag
p Smag ; momag
p r ;
p
m m
p 2s
9 p
d B20
 
If the magnetic permeabilities m m Smag m : 7:3:251
p and s of the particle dz 20
and solution may be described, respectively, via
7.3.5.1 Free-flow magnetophoresis
m m
p 0 1 p ; m m
s 0 1 s ; 7:3:245
We now briefly illustrate free-flow magnetophoresis in ana-
where m 0 is the magnetic permeability of free space and p logy to free-flow electrophoresis (Section 7.3.1.1). Consider
and s are, respectively, the magnetic susceptibilities of the Figure 7.3.22, in which a rectangular flat separation cham-
particle and the solution, then ber is shown. Here the vertical coordinate y is normal to
!   the (x,z)-plane. There are a number of inlet and corres-
mag m 3
p s B0 2 ponding outlet channels at the two ends of the flat separ-
F p 2 s r p r m : 7:3:246
p 2 s 3 0 ation chamber. Buffer liquid comes in through all inlet
channels except one at one end, where a particle mixture
Here the magnetic field strength H m is related to the
0 is introduced along with the liquid flow. A nonuniform
magnetic flux density B0 by B0 =m

0 . This relation may
magnetic field is applied perpendicular to this laminar
be rewritten as follows:
0 1 liquid flow. Those particles which are nonmagnetic, e.g.
2 polystyrene microspheres (Pamme and Manz, 2004), go
3V p p s A rB0
F mag
p 2 m
0 1 s
@
2 straight through without any deflection in the direction of
4 p 2 s 3 m
0
the magnetic field gradient to the corresponding outlet
channels in the z-direction. Paramagnetic particles, how-
0 1
2
p s
31 s V p @ A rB0 : 7:3:247 ever, follow a trajectory deflected in the direction of the
p 2 s 3 2m
0 magnetic field (x-coordinate), the extent of deflection
depending on the magnetic susceptibility p and the par-
When p and s are much smaller than unity, we obtain ticle radius rp since
V p p s
F mag
p rB0 2 ; 7:3:248a mag
U pxt / r 2p p s : 7:3:252
2m0

which is essentially expression (3.1.22), sometimes The particle trajectory equations are (see equations
written as (7.3.88) and (7.3.91)):
dx 2 p s d B20
 
F mag
p V p p s H m
0 rB0 : 7:3:248b mag
U pxt r 2p ; 7:3:253a
dt 9 dx 2m
0
In one-dimensional form (say the z-direction), this force
dz
expression is given by U pzt vz ; 7:3:253b
dt
d B20 dB0
 
F mag
pz V p p s V p p s H m
0 : where vz is the fluid velocity in the mean flow direction
dz m
0 dz
(z-direction). Typical magnitudes of these flow velocities in
7:3:248c microfluidic environments employed by Pamme and Manz
For paramagnetic particles, (p s) is positive; for diamag- (2004) were 0.10.4 mm/s. As shown in Figure 7.3.22, para-
netic particles, it is negative. magnetic particles are deflected in the x-direction; different
A nonferromagnetic particle moving in the direction of particles follow different trajectories, arriving at different
the nonuniform magnetic force field would encounter a outlet channels, and are therefore separated from one
drag force from the fluid and very soon achieve a terminal another. The larger the particle volume (or size, rp) and
velocity, usually identified as the magnetic migration vel- the higher the magnetic susceptibility p of the particle
mag
ocity U pzt , where, if Stokes law is valid, material, the higher the deflection in the x-direction since
mag
 2 U pxt is higher. The particle trajectory is obtained from
mag d B0
F drag
pz 6r p U pzt V p p s r ; 7:3:249 equations (7.3.253a,b) as
dz 2m
0
 2
 2 dx 2 2 p s d B0
2 p s d B0 rp ; 7:3:254a
 
mag
U pzt r 2p : 7:3:250 dz 9vz dx 2m 0
9 dz 2m 0
mag dx 2m p s m d H m
This magnetic migration velocity U pzt (Watson, 1973), also 0
r 2p H0 0
: 7:3:254b
dz 9vz dx
called the magnetic velocity, may be expressed in terms of a
mag
magnetophoretic mobility mop and the magnetic field To identify the exact extent of deflection of the particle
force strength Smag (Moore et al., 2004) by in the x-direction, one has to know the manner in
650 Bulk flow perpendicular to the direction of force

Rectangular flat laminar flow chamber


S
N

Paramagnetic Liquid (buffer)


particles of Flow channels
increasing size
and susceptibility
Nonmagnetic x Particle mixture sample
particles z y

Figure 7.3.22. Free-flow magnetophoresis in a horizontal channel. (After Pamme and Manz (2004).)

Mag. bead with


Method 1 Mag, + Mag, coated Mab
bead bead
targeting cell
(Mab) Cell

Method 2 +
Cell (Mab) Mab-coated target cell
Mag, + Mag,
bead bead

Bead coated with


anti-IgG antibody

Figure 7.3.23. Two methods of cell separation using magnetic beads and antibodyantigen binding. (After Gee (1998).)

which the nonuniform magnetic field is varying in the x- The manner in which such free-flow magnetophoresis
direction. (FFM) is applied in practice is related to the needs of life
Pamme and Manz (2004) have provided estimates of sciences vis--vis separation of native biological particles.
the magnetic force being exerted on small superparamag- These native biological particles by themselves do not lend
netic particles.26 Particles of diameter 4.5 m were found to to practical separation by FFM. However, they can be fixed
traverse a distance 2 mm in the x-direction when the value to magnetic or magnetizable microspheres having specific
of the distance tranversed in the z-direction was 6 mm; the antibodies (Abs) on their surfaces via antigen (Ag)Ab
axial fluid velocity was 0.3 mm/s. From equation (7.3.249), bridges from antigens on the surfaces of the biological
we obtain for a liquid consisting essentially of water (vis- particles. Two examples of this strategy are illustrated in
cosity 1 cp) Figure 7.3.23 (Gee, 1998). A common form of the magnetic
or magnetizable microspheres consists of superparamag-
mag 2 mm
F mag
px 6r p U pxt 6  2:25 m  1cp   0:3 netic, polystyrene based particles 2.04.5 m in diameter
6 s
having iron oxide homogeneously distributed throughout
kg 10 3 m the particle (Hausmann et al., 1998); these are called Dyna-
6  2:25  10 6 m  1  10 3  0:1 
m-s s beads (as mentioned earlier). For the separation of cells
kg-m and organelles, the FFM method has been identified as
6  2:25  0:1  10 12
s2 Continuous Immunogenic Sorting.

kg-m
4:24  10 12 4:24 pN piconewton: 7.3.5.2 High-gradient magnetic separation
s2
A high-gradient magnetic separator contains a loosely
The force on the smaller particles of similar p will be packed bed of ferromagnetic wool of stainless steel; there
correspondingly smaller by the volume ratio (equation is one inlet at the bottom for feed fluid to come in and
(7.3.248c)) of the particles. one outlet at the top for treated feed fluid to go out
(Figure 7.3.24). A liquid feed containing paramagnetic par-
ticles to be removed enters the device from the bottom.
26
Dynabeads, Dynal Biotech, Oslo, Norway. A uniform magnetic field is applied in the same direction
7.3 External force field based separation: bulk flow perpendicular to force 651

Cleaned stream containing nonmagnetics dv


mp F mag F gr F drag :
dt
Water stream for 7:3:255
inertial magnetic gravitational drag
backflushing
force force force force
Electromagnetic
coils The strands in the stainless steel wool may be modeled as a
Iron enclosure
Stainless thin cylinder of radius rw. The random configurations of the
steel wool wires are ignored at this time; they may be considered as the
Magnetics sum total of interactions involving three arrangements: lon-
obtained during gitudinal (uniform magnetic field H m 0 perpendicular to the
backflushing wire axis and particle velocity parallel but opposed); trans-
Feed stream
verse ( H m0 perpendicular to the wire axis and perpendicular
to the particle velocity, which is perpendicular to the wire
Figure 7.3.24. Schematic of a HGMS unit for cyclic operation.
axis); axial ( H m
0 perpendicular to the wire axis as in the
longitudinal arrangement, but particle velocity parallel to
as that of the fluid flow. Around each strand of the stainless the wire axis). One could represent a general configuration
steel wool, the uniform magnetic field becomes highly of the particle-to-wire interaction as shown in Figure 7.3.25
distorted, creating a nonuniform magnetic field whose (a) (Oak, 1977; Birss and Parker, 1981; Liu and Oak, 1983).
gradient can be of the order of 105 tesla/m. Particles in (The physical configuration is similar to that of a particle
the size range of microns are forced to the stainless steel flowing with air around a solid fiber of radius rf in depth
strands and retained there. The nonmagnetic particles pass filtration; See Figure 6.3.9A.) Based on such a configuration
through with the fluid. and the analyses of Birss and Parker (1981) and Oak (1977),
The matrix of stainless steel wool will soon be covered we can provide the following expressions for the r-
with paramagnetic particulates captured from the fluid, at component and the -component of the individual force
which time the liquid feed flow is stopped. A wash water terms on a spherical particle of radius rp and volume Vp
stream is passed. Then the magnet is turned off and a in equation (7.3.255), after neglecting the gravitational
water stream is passed in the direction opposite to that of and inertial force components vis--vis the other forces.
the feed stream; this stream carries away the paramag- In the following expressions, Ms stands for the saturation
netic particles, which fall off the strands of stainless steel magnetization of the wire, i.e. the value of Mw (for example,
wool. After these particles are removed, the magnet is in equation (3.1.23) when it is saturated). For the:
turned on and the feed liquid starts flowing in. The pro- r-component:
cess thus operates cyclically; its operation is very similar   
1 m 4M s r 3w M s r 2w

m
to the operation of the packed-bed adsorber in pressure- F mag V p H cos 2 ;
pr
2 0 p s 2r w r 3 0
2r 2
swing adsorption (Figure 7.1.13(a)). The only difference
7:3:256a
here is that, instead of the adsorption process, we have
magnetic force based particle capture in a direction per- dr
  
pendicular to the main flow direction. Shutting off the F drag
pr 6r p v cos 1 r w =r2 : 7:3:256b
dt
magnetic force field leads to release, as in desorption
processes. As long as (p s) is positive, the radial magnetic force
This HGMS technique is widely employed in industry F mag
pr on the particle is negative; i.e. the particle is attracted
to the ferromagnetic wire as long as H m

for the separation of paramagnetic particles from a 0 cos 2
stream with or without nonmagnetic particles. This tech- M s r 2w =2r 2 > 0; the force magnitude is largest for 0
nique can also be employed to remove paramagnetic (for 0). The configuration becomes that of flow parallel
particles from a gas stream (Gooding and Felder, 1981). to force (Section 6.3.1.4). There are regions around the
The stainless steel wool matrix is very open, with a wire where the particle is repulsed by the wire (Birss and
porosity around 0.95. However, it is important that the Parker, 1981). For diamagnetic particles ((p s) is nega-
matrix openings are not very large and that the wire tive), the attractive force is largest for /2.
dimensions are quite small since the magnetic field vari- For the -component:
ation around a wire is strongest over a range of distances
1 4M s r 3w H m
 
very close to the wire (Birss and Parker, 1981) and of the mag
F p m 0
sin 2 ; 7:3:257a
0 p s V p 3
order of the particle diameter. We will now provide a 2 2r w r
very constricted glimpse into the modeling of such a r  2   
d
  
drag w
separation device. F p 6 r p r v sin 1 :
The particle trajectory equation (see equation (6.2.45) dt r
in such a device is as follows: 7:3:257b
652 Bulk flow perpendicular to the direction of force

(a) y
Particle
accumulation
H0

rp Particle

r Free
q stream
S, steel
rw v velocity
wire g
z

Ms

Uniform
magnetic H0
field

(b) Particle

Particle
2 trajectory

(y/rw)
1 (ylim/rw) = 2.39

mag
(Upz /v) = 10

1 2 3 4 5 6

(z/rw)

S, steel wire

Figure 7.3.25. (a) Configuration of the paramagnetic particle, magnetized stainless steel wire and a particle; coordinate system for
analysis (Birss and Parker, 1981; Liu and Oak, 1983). (b) The trajectory of a particle being captured. (After Watson (1973).)

Therefore the governing equations of particle motion Similarly for the -component:
in the r-direction and the -direction are as follows. For
1 4M s r 3w H m
 
the r-component: m0 p s V p
0
sin 2
2 2r w r 3
  
1 4 M s r 3w M s r 2w
  
d
 r 2 
m V p H m
cos 2 6 rp r v sin 1
w
0;
2 0 p s 2 rw r3 0
2 r2 dt r
dr
  
6 rp v cos 1 r w =r2 0: 7:3:259a
dt
leading to
7:3:258a
d
 r  2 
This leads to the following for the r-component equation r v sin 1
w
from equation (7.3.255): dt r

dr rw 2 m 4M s r 3w H m
  
0 p s
 
v cos 1 Vp 0
sin 2: 7:3:259b
dt r 12 r p 2r w r 3
m0 p s V p 4M s r 3w M s r 2w
  
3
Hm
0 cos 2 2
:
12 r p 2r w r 2r Dividing equation (7.3.258b) by equation (7.3.259b), we get
7:3:258b the particle trajectory equation:
7.3 External force field based separation: bulk flow perpendicular to force 653

   m V
2 4M s r 3w M s r 2w
 
p s p
1 dr cos 1 rrw 012 r v 2r w r 3 Hm 0 cos 2 2 r 2
p
 2  m0 p s 4M s r 3w H m
; 7:3:260a
r d

sin 1 rrw 12 rp v V p sin 2
 0
2r w r 3

r w 2  2m p s r 2p M s H m
0
 r w 3 Ms
 r w 2 
1 dr 1 r cos 0 9 r v r cos 2 2H m
r
w 0
2 M Hm  : 7:3:260b
r d
 r 2  2m p s r p s 0 rw 3
1 rw sin 0 9 r v

r sin 2
w

From definition (7.3.260) for the magnetic migration vel- angle between 0 and 90 . When 0 and 0, it may
mag
ocity U pzt , we have appear as a case of bulk flow parallel to force (as in Section
2
! 6.3.1.4). However, if 6 0, there is always a component of
mag 2 2 p s d H m m
0 0 force perpendicular to the bulk flow (as long as 0; it is
U pzt r
9 p dz 2 true even for 6 0 as long as 6 ). But the overall
2 2 m Hm dH m configuration continues to be particles taken out of the
r 0 0 p s 0
; 7:3:261a
9 p dz bulk flow in the perpendicular direction and deposited in
m 2 m m
the woolen medium.
mag 2 0 r p p s H 0 M s 2m 2
0 r p p s H 0 M s
U pzt :
9 rw 9 rw 7.3.5.2.1 Performance of a HGMS filter We will
7:3:261b develop now an expression for the fractional capture of
the paramagnetic particles as the liquid flows through the
Using this definition, we can rewrite the expression
HGMS filter bed of ferromagnetic wool. We assume that
(7.3.260b) as follows:
the trajectory model has yielded a value of the y-coordinate
mag  
rw 2 U pzt r w 3 M s rw 2
   
1 cos cos 2 of the limiting particle trajectory, y im , equal to b (see
 
1 dr r v r 2H m r
o
mag   : Figures 7.3.25(b) and 6.3-9A). If the void volume fraction
r d 2 U 3
  
1 rrw sin vpzt rrw sin 2 of the bed is , then (1 ) is the bed volume fraction
7:3:262a occupied by strands of the ferromagnetic wool. For a
length dz in the feed flow direction of the filter bed of unit
Defining a nondimensional radial coordinate r (r/rw),
cross-sectional area perpendicular to fluid flow, the length
we get

1 3
  
mag
1 1=r 2 cos U pzt =v cos 2 2HMm sr2
 
1 dr r 0
: 7:3:262b
r d 1 3
 
mag
1 1=r 2 sin U pzt =v
 
r sin 2

The solution of this trajectory equation can be developed of the ferromagnetic wire in the wool is 1 dz= r 2w . As
numerically. What is of interest, however, is the y- in Figures 7.3.25(b), and 6.3.9A and 6.3.9B, a total width of
coordinate of the limiting trajectory of a particle which will 2y im 2b in the y-direction and unit length of the wire
be captured: if the particle starts out at a y-value in
provides a fluid flow cross-sectional area of 2y im per unit
the range y lim =r w  y=r w  y im =r w , where the
wire length: all particles coming in through this capture
magnitude of the y-value of the limiting trajectory is
cross section are ultimately captured by the stainless steel
jyim j at z , then its trajectory will surely hit the stain-
less steel wire (r rw) at differing values of . Luborsky and wires. Therefore the total capture cross section for the filter
Drummond (1975), as well as Liu and Oak (1983), have bed of thickness dz is 21 y im dz= r 2w (assuming that
provided a few simplified expressions for yim : there are no overlaps between the capture cross sections
p mag mag
for different location of the wire).
yim =r w 3 3=4 U pzt =v 1=3 if U pzt =v > 1; For a velocity v of the fluid entering the bed and a
7:3:263a feed fluid particle number concentration of n(rp)drp (the
mag mag
p
total number of particles of size rp to rp drp per unit fluid
yim =r w U pzt =2v if U pzt =v  2: 7:3:263b volume), the number of particles removed per unit time
Watson (1973) also provided the last result. This limiting per unit flow cross-sectional area is given by
trajectory coordinate y im far away from the stainless steel v  nr p jz dr p  21 y im dz
wire is similar to the coordinate b in Figure 6.3.9A except v dnr p dr p jz ;
r 2w
the particle trajectory does not follow the streamline at all 7:3:264a
near the wire (Figure 7.3.25(b)).
A few general features are to be noted here. In Figure where the right-hand side provides the corresponding total
7.3.25(a), the particle may come in and hit the wire at any change in particle flow rate over the distance dz (see the
654 Bulk flow perpendicular to the direction of force

p
development for equation (6.3.43)). We can rearrange this 100400). For values of the parameter less than 2, we
mag
relation for particles of size rp as follows: know that yim =r w 0:5U pzt =v (equation (7.3.263b)).
For higher values of the parameter, yim =r w keeps on
dnr p =nr p jz 2 y im 1 = r 2w dz: 7:3:264b

mag
increasing and can be as high as 7 for U pzt =v 100.
mag
Integrate from z 0 to z L, the bed length, to obtain Let us, for the sake of illustration, take U pzt =v to be 1.
Then
n nr p jz0 =nr p jzL 2y im 1 = r 2w L;

Nr p jzL =Nr p jz0


 
7:3:264c 8 9
< 2  0:5  1  r  0:5  51=
nr p jz0 =nr p jzL expf 2yim 1 = r 2w Lg: w

exp
: 0:95   r 2w ;
7:3:264d
8 9
When all particles have the same size rp,
< 0:05  51 =
exp 4
r max
2r
max
3 : 0:95   45  10 ;

nr p jz0 dr p 4 nr p jzL dr p 5 expf 2y im 1=r 2w Lg

8 9
< 5  51  102 =
r min r min exp exp f 189g:
: 0:95   45 ;
7:3:264e

) Nr p jz0 Nr p jzL expf 2y im 1=r 2w Lg


This result shows that the exiting fluid stream will have
7:3:264f very few of the particles left; almost all of them will be
captured by the bed. Performances of HGMS units have
) Nr p jzL Nr p jz0 expf 2yim 1=r 2w Lg;

generally been found to support such conclusions. The


7:3:265 influence of the captured particle buildup on the stainless
steel wires is important; see Liu and Oak (1983) for a
where N(rp) represents the total concentration of particles,
detailed analysis.
all of which have the same size, rp. If there is a particle size
distribution, then we will obtain, instead of (7.3.264e),
r max
7.3.6 Radiation pressure optical force
Nr min ; r max jzL nr p jzL dr p It is known (equation (3.1.47)) that radiation pressure
r min from continuous wavelength (cw) visible light is known
r max
to apply a force on and accelerate freely suspended par-
nr p jz0 expf 2yim 1 = r 2w Lgdr p ; 7:3:266

ticles in the direction of the light. This optical force (radi-


r min ation pressure force) induced migration of particles,
especially colloidal particles, is called photophoresis (PP).
since y im depends on the particle size rp.
Such forces have very little impact on larger particles.
Some comments are needed to qualify the results
However, micron and submicron particles can encounter
developed above. As shown in Figure 7.3.25(a), the mag-
substantial acceleration. Different refractive indices of
netic field H0 is perpendicular to the stainless steel wire.
transparent particles vis--vis the surrounding liquid, as
However, about one-third of the strand length is parallel to
well as light absorption, are the bases of this force. Helm-
H0. Therefore the argument of the exponential in equation
brecht et al. (2007) have demonstrated that a simple
(7.3.265) should be multiplied by 2/3 (Watson, 1973). Fur-
crossflow setup, in which a laser beam illuminated per-
ther, any change in the bed void volume fraction due to
pendicular to the mean flow direction of the fluid contain-
particle buildup on the wires is neglected. Ignoring add-
ing suspended microparticles causes a lateral shift in
itional assumptions inherent in the above analysis, let us
different particles. Different particles follow different tra-
make an estimate of the particle concentration reduction
jectories, as in other cases of bulk flow perpendicular to
Nr p jzL Nr p jz0 the direction of the force.
 
To describe such trajectories, consider the net photo-
achieved in a bed of length L 51 cm (say) following Liu phoretic force F rad on a particle of radius rp by a light beam
and Oak (1983). The bed void volume fraction is around (whose waist size wa is much larger, i.e. rp << wa); here
0.95; therefore (1 ) ~ 0.05. An average wire radius is the refractive index of the particle (npar) is larger than that
around 45 m ( rw). The value of yim depends very of the surrounding medium (nmed) so that the particle acts
mag
strongly on the ratio U pzt =v ; the value of this last like a spherical lens (Helmbrecht et al., 2007). The net
parameter in practice is generally greater than 0.5 (the force F rad resulting from the refraction of incident light
minimum value needed for particle buildup (Liu and (a laser beam) which transfers momentum to the particle
Oak, 1983)) and can go to quite high values (as much as has been described by Helmbrecht et al. (2007) as
Problems 655

nmed Pr r p 2 where
F rad 2 ; 7:3:267
c wa 1 nmed Pr rp
U pxt : 7:3:270
where Pr is the total power of the laser light, c is the 3 c wa2
velocity of light, is a measure of the conversion of inci-
The magnitudes of such terminal velocities identified as
dent momentum into particle movement (ranging from 0,
photophoretic velocities are in the range 1 to 80 m/s for
no conversion, to 2 for total reflection). This expression is
particles having diameters in the range of 1 to 10 m.
more detailed than (3.1.47).
However, for particles of comparable diameter, the velocity
Any such particle motion will immediately encounter a
of silica particles (which were transparent to the laser light
drag force, which, from Stokes law, may be written as
used) was six times less than that of melamine particles.
F drag 6 r p U px ; 7:3:268 Therefore, separation of particles of the same size but
having different optical properties appears feasible (Helm-
where the direction coordinate of the photophoretic
brecht et al., 2007).
motion is x, whereas the main bulk fluid is in the z-
There are unresolved complexities in such a continuous
direction (see, for example, Figure 7.3.22). Soon, a terminal
separation technique for particles due to the uncertainties in
migration velocity Upxt is achieved:
determining the effective power ( 2Pr (rp/wa)2). Also, the
2 nmed Pr r p 2 incident light has two components, an axial and a radial
F drag 6 r p U pxt F rad ;
c wa component, each leading to a separate PP velocity. See Helm-
7:3:269 brecht et al. (2007) for additional references on the subject.

Problems
7.1.1 You have an adsorber packed with an adsorbent such that 0.40. However, the adsorbent particles are also
porous, with a void volume fraction of p 0.30, and s ( 2.0 g/cm3) is the actual density of the material of the
solid adsorbent particles.
(1) If the interstitial velocity vz of the liquid in the packed bed is 53 cm/min, calculate the migration velocity
(concentration wave velocity) of species 1 whose equilibrium relation between the solid-phase concen-
tration of solute C11 and the mobile/liquid-phase concentration C12 is given by C11 4C12. Compare it
with the situation where p 0. (Ans. 7.11 cm/min; 7.57 cm/min.)
(2) Neglect now any p ( 0). The adsorber is 12 cm long. If it takes 20 cm3 of the mobile phase to saturate the
column completely, determine the diameter of the column. (Ans. diameter 0.867 cm.)
(3) Calculate the breakthrough time for solute 1 if the bulk liquid flow rate is 12 cm3/min under the conditions
of part (2). (Ans. 1.65 min.)
(4) Calculate the moles of species i adsorbed from a feed solution of concentration Cif from 20 cm3 of mobile
phase needed to saturate the column as in (2). Calculate now the volume of adsorbents needed to remove
99% of the amount adsorbed above in a closed vessel with no flow of mobile phase. Assume p 0, Ci1
4Ci2 and Ci2 0.01Cif ; the units of Cif are gmol/liter. Compare it with the adsorbent volume in part (2).

7.1.2 (a) An adsorber containing nonporous adsorbent particles is removing a C18 compound from a feed solution
of heptane. The adsorber is 15 cm long. The interstitial velocity is 30 cm/min. The C18-compound concen-
tration in the feed solution is 0.001 gmol/liter. The corresponding equilibrium concentration in the
adsorbent phase is 0.009 gmol/liter. Calculate the breakthrough time for the C18 compound assuming
nondispersive equilibrium adsorption behavior. If the feed solution comes in at a higher temperature and
the equilibrium adsorbent phase concentration at that temperature is 0.006 gmol/liter, calculate the
breakthrough time. You are given that the bed void volume fraction 0.45.
(b) The adsorbent column diameter in (a) is 1 cm. Calculate the volume of feed solution which will completely
saturate the column in the absence of any dispersion at the lower temperature of operation.
(c) Determine the solute migration velocity through such a column if the adsorbent particles are

Das könnte Ihnen auch gefallen